Surgery Essence [8 ed.] 9389776554, 9789389776553

This eighth edition is replete with new trends in the field of surgery and recent advances. In most of the PG entrance e

365 26 106MB

English Pages 1234 Year 2020

Report DMCA / Copyright

DOWNLOAD PDF FILE

Table of contents :
Cover
Title
Acknowledgments
Contents
Recent Questions
Image Based Questions
Annexures
Ch-1-Press Section 1
Ch-2-Press
Ch-3-Press
Ch-4-Press Section 2
Ch-5-Press
Ch-6-Press
Ch-7-Press
Ch-8-Press
Ch-9-Press Section 3
Ch-10-Press
Ch-11-Press
Ch-12-Press
Ch-13-Press
Ch-14-Press
Ch-15-Press
Ch-16-Press
Ch-17-Press
Ch-18-Press
Ch-19-Press
Ch-20-Press
Ch-21-Press Section 4
Ch-22-Press
Ch-23-Press
Ch-24-Press
Ch-25-Press
Ch-26-Press Section 5
Ch-27-Press
Ch-28-Press
Ch-29
Ch-30-Press Section 6
Ch-31-Press
Ch-32-Press
Ch-33-Press Section 7
Ch-34-Press
Ch-35-Press Section 8
Ch-36-Press
Ch-37-Press
Ch-38-Press
Ch-39-Press Section 9
Ch-40-Press
Ch-41-Press Section 10
Ch-42-Press
Ch-43-Press
Ch-44-Press
Ch-45-Press
Ch-46-Press
Ch-47-Press
Ch-48-Press
Ch-49-Press
Ch-50-Press
Ch-51-Press
Recommend Papers

Surgery Essence [8 ed.]
 9389776554, 9789389776553

  • 0 0 0
  • Like this paper and download? You can publish your own PDF file online for free in a few minutes! Sign Up
File loading please wait...
Citation preview

Surgery Essence

Surgery Essence Eighth Edition

Pritesh Kumar Singh

MBBS (MAMC), MS (Surgery), FMAS, FIAGES Author of Surgery Essence, AIIMS Essence, NEET Essence

Director, Chief Advisor of Editorial Board—PGMEE, Jaypee Brothers Medical Publishers Guest Faculty of Surgery, Ningbo University, China; Stavropol State Medical University, Russia Ex. Senior Resident, Lady Hardinge Medical College and Associated Sucheta Kriplani Hospital Kalawati Saran Children's Hospital and Ram Manohar Lohia Hospital New Delhi, India

JAYPEE BROTHERS MEDICAL PUBLISHERS The Health Sciences Publisher New Delhi | London

Jaypee Brothers Medical Publishers (P) Ltd Headquarters Jaypee Brothers Medical Publishers (P) Ltd 4838/24, Ansari Road, Daryaganj New Delhi 110 002, India Phone: +91-11-43574357 Fax: +91-11-43574314 Email: [email protected] Overseas Office J.P. Medical Ltd 83 Victoria Street, London SW1H 0HW (UK) Phone: +44 20 3170 8910 Fax: +44 (0)20 3008 6180 Email: [email protected] Website: www.jaypeebrothers.com Website: www.jaypeedigital.com © 2020, Jaypee Brothers Medical Publishers The views and opinions expressed in this book are solely those of the original contributor(s)/author(s) and do not necessarily represent those of editor(s) of the book. All rights reserved. No part of this publication may be reproduced, stored or transmitted in any form or by any means, electronic, mechanical, photocopying, recording or otherwise, without the prior permission in writing of the publishers. All brand names and product names used in this book are trade names, service marks, trademarks or registered trademarks of their respective owners. The publisher is not associated with any product or vendor mentioned in this book. Medical knowledge and practice change constantly. This book is designed to provide accurate, authoritative information about the subject matter in question. However, readers are advised to check the most current information available on procedures included and check information from the manufacturer of each product to be administered, to verify the recommended dose, formula, method and duration of administration, adverse effects and contraindications. It is the responsibility of the practitioner to take all appropriate safety precautions. Neither the publisher nor the author(s)/editor(s) assume any liability for any injury and/or damage to persons or property arising from or related to use of material in this book. This book is sold on the understanding that the publisher is not engaged in providing professional medical services. If such advice or services are required, the services of a competent medical professional should be sought. Every effort has been made where necessary to contact holders of copyright to obtain permission to reproduce copyright material. If any have been inadvertently overlooked, the publisher will be pleased to make the necessary arrangements at the first opportunity. The CD/DVD-ROM (if any) provided in the sealed envelope with this book is complimentary and free of cost. Not meant for sale. Inquiries for bulk sales may be solicited at: [email protected] Surgery Essence Third Edition: 2015 Fourth Edition: 2016 Fifth Edition: 2017 Sixth Edition: 2018 Seventh Edition: 2019 Eighth Edition: 2020 ISBN: 978-93-89776-55-3

Dedicated to My Parents and Uncle, Dr CP Singh

Editors ENDOCRINE SURGERY • Dr Ashish Jakhetiya (MCh, Surgical Oncology, AIIMS)

• Dr Subham Jain (MCh, Surgical Oncology, TATA)

• Dr Subham Garg (MCh, Surgical Oncology, TATA)

• Dr Niket Harsh (MCh, GI Surgery, AIIMS)

HEPATOBILIARY PANCREATIC SURGERY • Dr Swati Agarwal (DNB, Surgical Oncology)

• Dr Vaibhav Varshney (MCh, GI Surgery, GB Pant Hospital)

• Dr Harsh Shah (MCh, GI Surgery, GB Pant Hospital)

• Dr Amit Jain (MCh, GI Surgery, GB Pant Hospital)

GASTROINTESTINAL SURGERY • Dr Vaibhav Varshney (MCh, GI Surgery, GB Pant Hospital)

• Dr Niket Harsh (MCh, GI Surgery, AIIMS)

• Dr Harsh Shah (MCh, GI Surgery, GB Pant Hospital)

• Dr Amit Jain (MCh, GI Surgery, GB Pant Hospital) UROLOGY

• Dr Gaurav Kochar (MCh, Urology)

• Dr Shiva Navariya (MCh, Urology)

• Dr Manoj Kumar Das (MCh, Urology)

• Dr Animesh Singh (MCh, Urology, AIIMS)

CARDIOTHORACIC VASCULAR SURGERY • Dr Vivek Wadhva (MCh, CTVS, PGI Chandigarh)

• Dr Tarun Raina (MCh, CTVS, GB Pant Hospital)

PLASTIC SURGERY • Dr Alok Tiwari (MCh, Plastic Surgery)

• Dr Ritesh Anand (MCh, Plastic Surgery)

NEUROSURGERY • Dr Amit Kumar Singh (MCh, Neurosurgery, RML Hospital)

• Dr Shivender Sobti (MCh, Neurosurgery, RML Hospital)

• Dr Ishu Bishnoi (MCh, Neurosurgery, GB Pant Hospital)

• Dr Ugan Singh (MCh, Neurosurgery)

HEAD AND NECK • Dr Ashish Jakhetiya (MCh, Surgical Oncology, AIIMS)

• Dr Subham Jain (MCh, Surgical Oncology, TATA)

• Dr Subham Garg (MCh, Surgical Oncology, TATA)

• Dr Niket Harsh (MCh, GI Surgery, AIIMS)

SURGICAL ONCOLOGY • Dr Ashish Jakhetiya (MCh, Surgical Oncology, AIIMS)

• Dr Subham Jain (MCh, Surgical Oncology, TATA)

• Dr Subham Garg (MCh, Surgical Oncology, TATA)

• Dr Niket Harsh (MCh, GI Surgery, AIIMS)

GENERAL SURGERY • Dr Harindra Sandhu (MS, Surgery)

• Dr Mohit Garg (MS, Surgery)

• Dr Mayank Agarwal (MS, Surgery)

• Dr Gunjan Desai (MS, Surgery)

Preface to the Eighth Edition First of all I would like to thank all my students, including the super speciality aspirants, for their constant feedback regarding improvement of the book (Surgery Essence) and making the book as the bestseller. I can proudly say that all my students have contributed a lot to get me to this place, where I am today. They have helped me in becoming a better teacher, a better author and, most importantly, a better human being. I take this opportunity to thank all of you. The happiness you all give me keeps me telling always to work harder, to bring a positive change in the life of my students. This will be reflected in the pages of this book. I always strive to provide a winning edge to my students. This book has become the bestseller chiefly because of the suggestions, love and even critics that I come across and am informed of. Higher education has become necessary, as graduation alone is found inadequate in this highly competitive and dynamic world. Trends in the way, the questions being asked are changing continuously. The NEET exam is now the standard exam for both postgraduation and super specialty. As an author, I closely follow the kind of questions being asked and the change of pattern of questions in the NEET exam. I am pleased to present the 8th edition of Surgery Essence replete with new trends in the field of surgery and recent advances. In most of the PG entrance exams, lots of image-based questions are being asked, especially in AIIMS and NEET. To provide to an edge, image-based questions are given in the beginning. Triads, signs, investigation of choices and topics based on “most common” type of questions are included in the annexures to save your precious time and to help you in revision at the most crucial hours. Synopsis of the chapter is given in the beginning to develop concepts about the topics. The recent questions and their concepts are highlighted and have been written in the way that will help the students to remember and reproduce them in the examination hall. The information provided is cogent but concise to save the precious time, as we all know the clock is ticking. Time is one thing that can never be recovered once gone. Be careful! I am passionate about excellence. Excellence in the field of education, and in my efforts to groom my students to make them confident enough that they lose the fear of failure. Being the Director of Dr Pritesh Institute, I follow the same principle in this institute so that the students should be benefited most with an extra edge. I believe that all my students should know the importance of challenges. Challenges are what make life interesting and overcoming them is what makes life meaningful. For the time being, the only challenge that you should be facing is to secure a good rank in the entrance exam. One of the most important keys to success is having the discipline to do what you know you should do, even when you do not feel like doing it. Nobody ever wrote down a plan to be broke, lazy or stupid. These things happen when you do not have a plan. PG entrance examination has made the medical world very competitive and has made it imperative for the students to acquire all the skills and competencies to deliver results. My aim, as an author and as a teacher, is to provide students with a learning experience which when amalgamated with perseverance and commitment helps them in achieving goals. I am still not sure about one thing that who is more happy when a student achieves something, the student or the teacher, but I am very sure that the teacher is more satisfied when he sees his students achieving what they deserve and desire. I am working day and night to get that satisfaction and you have to work equally hard so that you do not let me down. I always tell my students to dream big but not while sleeping. But, these dreams should always be accompanied with intelligence and hard work. To guide your work intelligently, this book and the author, both are there, with you throughout the year. But, the hard work is totally in your hands. Accept responsibility for your life. Know that it is you who will get you where you want to go, no one else. Extensive revisions have been made to minimize the chances of error but still some mistakes might be there which should be brought to the notice of the authors through e-mail address or in writing.

x

Surgery Essence It is a pleasure now to give outlet to the overflowing appreciation and thanks to all my colleagues, friends, teachers and family because this book is the result of encouragement and guidance from all of them. I am pleased to acknowledge the overwhelming love I have received from my students, who are my ultimate source of inspiration. Wishing you all the best and looking forward for your feedback and suggestions…

Pritesh Kumar Singh

[email protected] drpriteshsingh drpriteshsingh drpriteshsingh drpriteshsingh /Dr Pritesh Institute drpriteshsingh drpriteshsingh

www.drpriteshsurgeryclasses.com

MBBS (MAMC), MS (Surgery) FMAS, FIAGES Author of Surgery Essence, AIIMS Essence, NEET Essence, Director, Chief Advisor of Editorial Board PGMEE, Jaypee Brothers Medical Publishers Guest Faculty of Surgery, Ningbo University, China Stavropol State Medical University, Russia www.drpriteshinstitute.com

Acknowledgments I would like to express my greatest gratitude to the people who have helped and supported me throughout my project. I wish to thank my parents for their undivided support and interest, who inspired me and encouraged me to go my own way, without whom I would be unable to complete my project. First of all I would like to thank my beloved wife Dr Usica Singh, for her constant support and motivation. She helped me in updating the book from the latest editions of standard textbooks. She helped me throughout this project by giving her valuable advises and feedbacks regarding improvement of the book. I express my sincere thanks to my friends Dr Niket Harsh (MCh, GI Surgery, AIIMS) and Dr Saurabh Rai (MS, Orthopedics). They provided me the explanations of difficult and controversial questions. I am very thankful to Dr SK Tudu, Ex. HOD of Surgery, Maulana Azad Medical College, New Delhi, for the valuable help. He was always there to show us the right track when we needed his help. It is with the help of his valuable suggestions, guidance and encouragement, that I was able to complete this project. I am grateful to Dr MP Arora, for the continuous support for the project, from initial advice and contacts in the early stages of conceptual inception and through ongoing advice and encouragement to this day. I sincerely thank my uncle Dr SD Maurya (President SELSI and Ex. Professor of Surgery, SNMC, Agra), for his valuable advice and knowledge regarding the surgery subject and surgical skills, which helped me a lot in preparation of certain topics of surgery given in this book. I wish to express my sincere thanks to Dr OP Pathania and Dr S Thomas. I wish to express my sincere thanks to Dr Manoj Andley, Professor of Surgery, LHMC, New Delhi, for helping me throughout this project. His caring and fatherly attitude for the unit as well as towards his residents needs a mention. His excellent way of teaching and presentation helped me a lot in making various explanations in the book. His hard working and caring attitude towards patients is source of inspiration for me and surgery residents. I am very thankful to Dr Ashok Kumar, Professor of Surgery, LHMC, New Delhi, for his valuable and indispensable help. His unique ideas regarding presentation of explanations helped me a lot in this project. It is with the help of his valuable suggestions, guidance and encouragement, that I was able to complete this project. I wish to express my sincere thanks to Dr Lalit Aggarwal, Dr Gyan Saurabh, Dr Sudipta Saha, Dr P Rahul, Assistant Professor of Surgery, Lady Hardinge Medical College, for guiding me to complete general surgery topics. I wish to express my sincere thanks to Dr Pawan Kumar, Dr Priya Hazrah, Dr Nikhil Talwar, Dr Ezaz Siddiqui, Dr Ashish Arsia, Dr Sadan Ali, Dr Jitender and Dr Kusum Meena, Assistant Professors of Surgery, Lady Hardinge Medical College, for their indispensable contribution. I would like to thank Dr UC Garga, Professor of Radiology, Dr RML Hospital, New Delhi, for his special guidance for radiology and valuable advices for improvement of the book and boosting my morale to bring this project. I express my extreme gratitude for immense inspiration from my family members specially: • Dr Avinash Kumar Singh (Urologist)



Dr Charu Singh (Dermatologist)

• Mr Abhay Kumar Singh (MBA, IMT, Ghaziabad)



Mrs Deepasha Singh (MBA, IMT, Ghaziabad)

• Mr Ritesh Kumar Singh (B Tech, MBA, Symbiosis, Pune)



Ms Pratibha Singh (M Tech, Computer Science)

• Ms Monika Singh (B Tech, Computer Science)



Ms Khushboo Singh (B Tech, Computer Science)

• Mr Rohit Kumar Singh (B Tech, Computer Science)



Dr Anita Singh (MD, Pediatrics, KGMC, Lucknow)

• Dr Kundan Kumar Patel (DMRD)



Dr Akanksha Singh (DGO, KGMC)

• Dr Jigyasa Singh (MS, Gynae IMS, BHU)



Mr Abhishek Kumar Singh (B Tech, IIT Kharagpur)

• Dr Ambuj Kumar Singh (MD Dermatology)



Mr Rahul Kumar Singh (B Tech)

xii

Surgery Essence I would like to specially thank my friends for their invaluable help and advice from time to time specially: • Dr Niket Harsh



Dr Suarabh Rai

I feel pleasure in conveying my sincere thanks to my friends and colleagues specially: • Dr Shipra Goel (MD, Microbiology)



Dr Mayank Agarwal (MS, Surgery)

A special thank of mine goes to Dr Parul Gautam (MD, Pathology, MAMC), who helped me in completing the project and exchanged her interesting ideas, thoughts which made this project easy and accurate. Her help for topics related to tumor and pathology is indispensable. I am equally grateful to my friend Dr Sushant Bhanja (MD, Pediatrics), who gave me moral support and guided me in different matters regarding the topics related to Pediatric Surgery. He has been very kind and patient, whilst suggesting me the outlines of this project and correcting my doubts. I would be failing in my duty if I do not express my thanks to all my friends who have really inspired me to write this book specially: • Dr Vivek Kumar (MD, Medicine)



Dr Neha Chaudhary (MD, Pediatrics)

• Dr Harwinder (MS, Orthopedics)



Dr Nitasha (MS, Ophthalmology)

• Dr Ugan Singh (MCh, Neurosurgery)



Dr Pragati Meena (MS, Gynae, SMS, Jaipur)



Dr Aniket Malhotra (MD, Pediatrics)



Dr Anant Shukla (MD, Anesthesia)



Dr Bhamini Agal (MS, Gynae, SMS, Jaipur)

• Dr Anant Pachisia (MD, Anesthesia)

I would like to express my sincere thanks to my colleagues at Dr RML Hospital, especially Dr Amit Kumar Singh (MCh, Neurosurgery), Dr Shivender Sobti (MCh, Neurosurgery), Dr Humam (SR, Neurosurgery), Dr Wazid (DNB, Neurosurgery), Dr Uzair (DNB, Neurosurgery), Dr Azaz (DNB, Neurosurgery) and Dr Neeraj (DNB, Neurosurgery). I would like to express my sincere thanks to my colleagues at Lady Hardinge Medical College and Associated Dr RML Hospital, Dr Sushma Kataria, Dr Gyan Ranjan, Dr Kamal Yadav, Dr Priyank Yadav, Dr Vineet, Dr Munish, Dr Nivedita, Dr Tarun Raina, Dr Sumit Saini and Dr Abhinav Veerwal. I would like to express my sincere thanks to my colleagues at Lady Hardinge Medical College and Associated Dr RML Hospital, Dr Meenakshi, Dr Ankur, Dr Prashant, Dr Rigved, Dr Munish Raj, Dr Diwakar Pandey, Dr Vikram Deswal, Dr Gunjan Desai, Dr Vikas, Dr Nikunj Jain, Dr Hari Singh, Dr Vimlesh, Dr Mannu, Dr Anshul, Dr Vikas and Dr Abhijeet Jha, Dr Mayank Aggarwal, Dr Vipul Dogra, Dr Abhishek, Dr Kunjan, Dr Sumit, Dr Kartikey, Dr Rao Bhupender. I would like to express my sincere thanks to my colleagues at Lady Hardinge Medical College and Associated Dr RML Hospital, for their valuable advice, specially: • Dr Divish Saxena (Asst. Prof of Surgery, LMMC, Nagpur)

• Dr Ravindra Gupta (Ex. SR, RML Hospital, MS, Gynae, PMCH)

• Dr Prasad Bhukebag (SR, RML Hospital)

• Dr Ritesh Pathak (SR, RML Hospital)

• Dr Anil Gulwani (MCh, Urology)

• Dr Nitin Sardana (Ex-SR, LHMC)

• Dr Arvinda PS (MCh, GI Surgery)

• Dr Rahul Rai (Ex-SR, LHMC)

• Dr Yogender (SR, LHMC)

• Dr Anand Yadav (Ex-SR, LHMC)

• Dr Shiv Navariya (MCh, Urology)

• Dr Nihar (MCh-Hepatobiliary Surgery)

• Dr Zuber Khan (FNB, Minimal Invasive Surgery, LHMC) I would like to express my sincere thanks to my colleagues at Maulana Azad Medical College and Associated LNJP Hospital, for their valuable advice, specially: • Dr Mohit Garg (MS, Surgery)



Dr Kamal Kishore Gautam (MS, Surgery)

• Dr Anurag Mishra (Assistant Professor, MAMC)



Dr Ashish Airen (MS, Surgery)

I would also like to thank Mr Varish Sharma and Mr Anurag Sharma of MAMC Bookshop for their encouragement for writing this book. I would like to thenk Mr Sahil Mahajan (Manager, Dr Pritesh Institute) and Mr Rajesh Jha (Assistant Marketing Manager, Dr Pritesh Institute) for their contribution.

Acknowledgments I would like to thank Dr Ashish Jakhetiya and Dr Inderjeet Yadav, who helped me a lot in gathering different information, collecting data and guiding me from time to time in completing this project. Despite their busy schedules, they gave me different ideas to help make this project unique. Last but not the least I want to thank all my students who appreciated me for my work and motivated me and finally to God who made all the things possible. I convey my sincere thanks to Jaypee Brothers Medical Publishers (P) Ltd, New Delhi, India, for their efforts and suggestions, especially Shri Jitendar P Vij (Group Chairman), for helping me through my idea.

xiii

Contents Recent Questions with Explanations

p1–p32

Image Based Questions Annexures

i–xxvii

SECTION 1: ENDOCRINE SURGERY 1. Breast 3 2. Thyroid 49

3.

Parathyroid and Adrenal Glands 87

SECTION 2: HEPATOBILIARY PANCREATIC SURGERY 4. Liver 107

5.

Portal Hypertension 143

6. Gallbladder 165

7.

Bile Duct 194

8. Pancreas 226

SECTION 3: GASTROINTESTINAL SURGERY 9. Esophagus 275 10. Stomach and Duodenum 314 11. Peritoneum 365 12. Intestinal Obstruction 381 13. Small Intestine 409 14. Large Intestine 435 15. Ileostomy and Colostomy 468 16. Inflammatory Bowel Disease 473 17. Vermiform Appendix 488 18. Rectum and Anal Canal 500 19. Hernia and Abdominal Wall 525 20. Spleen 552

SECTION 4: UROLOGY 21.

Kidney and Ureter 567

22.

Urinary Bladder 626

23.

Prostate and Seminal Vesicles 646

24.

Urethra and Penis 662

25.

Testis and Scrotum 684

SECTION 5: CARDIOTHORACIC VASCULAR SURGERY 26.

Arterial Disorders 711

27.

Venous Disorders 745

28.

Lymphatic System 761

29.

Thorax and Lung 770

SECTION 6: PLASTIC SURGERY 30. Burns 815 31.

Plastic Surgery and Skin Lesions 828

32.

Wound Healing, Tissue Repair and Scar 851

xvi

Surgery Essence SECTION 7: NEUROSURGERY 33.

Cerebrovascular Diseases 863

34. CNS Tumors 886

SECTION 8: HEAD AND NECK 35.

Oral Cavity 907

36.

Salivary Glands 922

37. Neck 939 38.

Facial Injuries and Abnormalities 952

SECTION 9: ONCOLOGY 39. Oncology 963 40. Sarcoma 984

SECTION 10: OTHERS 41.

Pediatric Surgery 993

42. Trauma 996 43. Transplantation 1035 44.

Anesthesia and Perioperative Complications 1052

45.

Robotics, Laparoscopy and Bariatric Surgery 1055

46.

Sutures and Anastomoses 1065

47.

Sterilization and Infection 1077

48.

Fluid, Electrolyte and Nutrition 1097

49.

Blood Transfusion 1108

50. Shock 1114 51. Miscellaneous 1123

 Most

Important Important Important

 Very 

Recent Questions NEET PG (RECENT QUESTIONS) 2020 1. A 40 years old women presented to the clinic with a 4 cm mass in the upper outer quadrant. Biopsy from the mass showed densely packed cells with bland nuclei and mucin infiltrating the stroma. Most probable diagnosis is: a. Colloid carcinoma b. Tubular carcinoma c. Papillary carcinoma d. Medullary carcinoma 2. Which of the following statements is true regarding retrosternal goitres? a. Operated only if patient is symptomatic b. Sternal incision is always required c. Majority of the goitres derive their blood supply from mediastinal vessels d. Majority of the retrosternal goitres can be removed by a neck incision 3. A 25 years old male presented with a 2 cm nodule. Thyroidectomy was done. The histology picture is given below. What is the most likely diagnosis?

a. b. c. d.

a. CP class A c. CP class C

b. CP class B d. CP class D

7. Ligation of common hepatic artery will impair blood supply in: a. Right gastric and right gastroepiploic artery b. Right gastric and left gastric artery c. Right gastroepiploic artery and short gastric vessels d. Right gastric and short gastric vessels 8. An 80 years old female presented with colicky pain and jaundice. Serum bilirubin and GGT was raised. MRCP image shows gallstones and CBD stones with dilated biliary radicles. Next step in the management: a. Cholecystectomy b. ERCP c. CECT d. PET scan 9. A 35 years old male presented with recurrent episodes of abdominal pain, abdominal tenderness, fever and jaundice. MRCP image of the patient is given below. Most likely diagnosis is:

Papillary thyroid carcinoma Medullary thyroid carcinoma Toxic nodular goitre Follicular thyroid carcinoma

4. A 30 years old female presents with a diffuse thyroid swelling. On investigations, TSH levels were elevated. Postoperative HPE reports showed intense lymphocytic infiltration and Hürthle cells. Which of the following is the most likely diagnosis? a. Grave’s disease b. Hashimoto’s thyroiditis c. Follicular carcinoma d. Medullary thyroid carcinoma 5. In a patient with parathyroid adenoma, how do we confirm the removal of the correct gland after surgery? a. 50% reduction in PTH within 10 mins of gland removal b. 50% reduction in PTH within 5 mins of gland removal c. 25% reduction in PTH within 10 mins of gland removal d. 25% reduction in PTH within 5 mins of gland removal 6. What is the Child-Pugh class for patient who has a serum bilirubin of 2.5 mg/dL, serum albumin of 3 g/dL, INR of 2 along with mild ascites but no encephalopathy?

a. b. c. d.

Primary sclerosing cholangitis Primary biliary cirrhosis Oriental cholangiohepatitis Caroli’s disease

10. Tumor of the uncinate process of pancreas affects which of the following? a. Superior mesenteric artery b. Portal vein c. Common hepatic artery d. Inferior mesenteric artery 11. Most common pancreatic endocrine neoplasm: a. Insulinoma b. Gastrinoma c. VIPoma d. Glucagonoma 12. What is the most common site of gastrinoma in MEN 1 syndrome? a. Jejunum b. Ileum c. Duodenum d. Stomach

p2

Surgery Essence 13. Patient presents with halitosis and a swelling in the lateral aspect of the neck, which on pressing gives rise to a regurgitant sound. Barium swallow image is shown. What is the diagnosis?

a. Laryngocele c. Pharyngeal pouch

b. Achalasia cardia d. Schatzki’s ring

14. On esophageal manometry, abnormal spastic contractions in esophagus >450 mm Hg·s·cm in the body is suggestive of: a. Type I achalasia b. Type II achalasia c. Type III achalasia d. Jackhammer esophagus 15. What is the diagnosis based on the given barium swallow image?

a. Achalasia cardia c. Esophageal stricture

b. Carcinoma esophagus d. GERD

16. A middle aged man complains of upper abdominal pain after a heavy meal. There is tenderness in the upper abdomen and on X-ray, widening of the mediastinum is seen with pneumo-mediastinum. What is the diagnosis? a. Spontaneous perforation of the esophagus b. Perforated peptic ulcer c. Foreign body in esophagus d. Rupture of emphysematous bulla 17. A young man met with a motor bike accident and had injury to ileum and jejunum. Therefore the entire ileum and part of jejunum was resected. Which of the following would the patient suffer from? a. Vitamin B12 deficiency b. Constipation c. Gastric ulcer d. Hypogastrinemia

18. A patient presented with abdominal pain, blood in stool and a palpable mass on the abdominal examination. The following barium image was obtained from the patient. What is the most probable diagnosis?

a. Meckel’s diverticulum b. Intussusception c. Volvulus d. Diverticulitis 19. Patient presents with peritonitis and during surgery a diverticular perforation is seen with fecal peritonitis. What is the Hinchey’s stage? a. 1 b. 2 c. 3 d. 4 20. A 5 years old child presented with history of blood in stools. On examination, there was a polypoidal mass in the rectum, biopsy of which is shown as below. Most probably diagnosis is:

a. Villous adenoma c. Vascular malformation

b. Juvenile polyp d. Serrated adenoma

21. What is the diagnosis based on the given image?

a. Omphalocele c. Umbilical hernia

b. Gastroschisis d. Spigelian hernia

Recent Questions 22. Management of RCC less than 4 cm in size: a. Radical nephrectomy b. Partial nephrectomy c. Chemotherapy d. Surgery followed by chemotherapy

30. Patient with clinical signs of DVT had tachycardia and history of bladder cancer. According to modified Well’s scoring, the probability of pulmonary embolism would be: a. Low b. Moderate c. High d. Severe

23. A man is brought to the emergency after he fell into a man hole and injured his perineum. He feels the urge to micturate but is unable to pass urine and there is blood at the tip of the meatus with extensive swelling of the penis and scrotum. What is the location of the injury? a. Bulbar urethra b. Prostatic urethra c. Bladder d. Membranous urethra

31. A child was brought to the emergency with multiple injuries which included a femur fracture as well. He was confused and dyspneic. There was presence of rash all over the body. What is the most probable diagnosis? a. Fat embolism b. Air embolism c. Pulmonary embolism d. DIC

24. Identify the investigation being performed and possible diagnosis:

32. What is the most probable cause of this condition?

a. b. c. d. a. b. c. d.

MCU with membranous urethral stricture MCU with bulbar urethral stricture RGU with penile urethral stricture RGU with prostatic urethral stricture

25. True statement about intermittent claudication: a. Felt at rest b. Most common site is the calf c. Claudication distance can vary from day to day d. Relieved after getting out of bed and walking 26. A 30 years old man presents with cramping gluteal pain after walking 500 meters. Which is the vessel involved? a. Arterial disease with aorto-iliac involvement b. Arterial disease with femoral artery involvement c. Femoral venous insufficiency d. Saphenous venous insufficiency

Chronic lymphedema Chronic arterial blockage Chronic venous blockage Chronic venous insufficiency

33. Elderly man with a long standing mole over the face which is increasing in size and showing irregular borders. Diagnosis: a. Superficial spreading melanoma b. Lentigo maligna c. Acral melanoma d. Nodular melanoma 34. For frost-bite rewarming, the temperature should be: a. 20oC b. 25oC c. 37oC d. 42oC 35. CT scan image showing cystic lesion with suprasellar calcification is suggestive of:

27. Male patient presents to the hospital with abdominal pain and is incidentally detected with an abdominal aortic aneurysm. What is the appropriate management of this patient? a. Immediate surgery b. USG monitoring till size of the aneurysm reaches 70 mm c. Monitor till size reaches 40 mm d. Monitor till size reaches 55 mm 28. Which one of the following is not a component of THORACOSCORE? a. Performance status b. Complication of surgery c. ASA grading d. Priority of surgery 29. What is the T stage of a 2.5 cm lung carcinoma, which is not involving the pleura? a. T1a b. T1b c. T1c d. T2

a. Craniopharyngioma c. Astrocytoma

b. Meningioma d. Oligodendroglioma

36. Which of the following is not a surgical landmark for facial nerve identification in parotid surgery? a. Inferior belly of omohyoid b. Tragal pointer c. Posterior belly of digastric d. Dissecting from peripheral branches

p3

p4

Surgery Essence 37. A middle aged man presented with a swelling over the neck since childhood. Neck swelling has a bag of worm appearance. Most probable diagnosis is: a. Plexiform neurofibroma b. Toxic nodular goitre c. Vasculitis d. Lymphangioma 38. Primary survey of ATLS does not include: a. CT scan b. FAST c. Chest X-ray d. Pelvic X-ray 39. A 20 years old boy is brought to the emergency following a RTA with respiratory distress and hypotension. He has subcutaneous emphysema and no air entry on the right side. What is the next best step in the management? a. Start IV fluids after insertion of wide bore IV line b. Needle decompression in the 5th intercostal space c. Shift to ICU and intubate d. Positive pressure ventilation

46. Graft taken from identical twin is called as: a. Autograft b. Allograft c. Isograft d. Xenograft

AIIMS NOVEMBER 2019 47. Which of the following structure is not removed in radical neck dissection? (AIIMS November 2019) a. SCM b. Digastric muscle c. Spinal accessory nerve d. Level 1-5 lymph nodes 48. Identify this instrument:

(AIIMS November 2019)

40. Patient with stab injury to the lower chest presented with low pulses and BP improved after giving IV fluids. Chest X-ray showed clear lung fields. Next step in the management: a. Chest tube insertion b. CECT abdomen c. CECT chest d. E-fast 41. The given below image in a patient with road traffic accident shows:

a. Monopolar cautery c. Hyfrecators

b. Bipolar cautery d. LigaSure

49. Which of the following cannula is used in the patient with dehydration and diarrhea?  (AIIMS November 2019, November 2018) a. Blue b. Green c. Grey d. Pink 50. Rearrange the sequence of hand washing:  (AIIMS November 2019)

a. Battle sign c. Citelli’s abscess

b. Bezold abscess d. Griesinger sign

42. A 5 years old child is having acute liver failure. Which one of the following criteria is not included in the King’s college criteria? a. Age 6.5 c. Bilirubin >300 μmol/L d. Jaundice 12000/mm3 or 380C c. Pulse rate >90/minute d. Respiratory rate >24/minute and PaCO2 95%) of retrosternal goitres can be removed transcervically. “Retrosternal goitre tends to arise from the slow growth of a multinodular gland down in to the mediastinum. As the gland enlarges within the thoracic inlet, pressure may lead to dysphagia, tracheal compression and eventually airway symptoms. The vast majority of patients have minimal symptoms. Patient should be considered for surgery if there is significant airway compression, if symptoms are present or in young patients in whom symptoms are likely to develop.”-Bailey 27/e p810 “If a decision is made to proceed to surgery, assessment of the extent of disease is critical. The vast majority (>95%) of retrosternal goitres can be removed transcervically. Those at most risk of requiring conversion to an open sternotomy approach include malignant or revision cases, those which extend into the posterior mediastinum and those in which the diameter of the goitre exceeds that of the thoracic inlet. In such cases a joint case with thoracic surgery should be planned.”-Bailey 27/e p811 3. Ans. a. Papillary thyroid carcinoma (Ref: Schwartz 11/e p1647; Sabiston 20/e p898) The given HPE image shows optically clear nuclei (Orphan Annie eye nuclei), which is highly suggestive of papillary carcinoma thyroid. “Papillary Carcinoma Thyroid: The diagnosis is established by characteristic nuclear cellular features. Cells are cuboidal with pale, abundant cytoplasm, crowded nuclei that may demonstrate “grooving,” and intranuclear cytoplasmic inclusions (leading to the designation of Orphan Annie nuclei, which allow diagnosis by FNAB. Psammoma bodies, which are microscopic, calcified deposits representing clumps of sloughed cells, also may be present.”-Schwartz 11/e p1647 4. Ans. b. Hashimoto’s thyroiditis (Ref: Bailey 27/e p821; Schwartz 11/e p1640; Sabiston 20/e p891) History of diffuse thyroid swelling in a female, who is hypothyroid (elevated TSH levels) and HPE reports showing intense lymphocytic infiltration with Hürthle cells is highly suggestive of Hashimoto’s thyroiditis. “Hashimoto’s thyroiditis is also more common in women (male-to-female ratio is 1:10 to 20) between the ages of 30 and 50 years old. The most common presentation is that of a minimally or moderately enlarged firm granular gland discovered on routine physical examination or the awareness of a painless anterior neck mass, although 20% of patients present with hypothyroidism, and 5% present with hyperthyroidism (Hashitoxicosis).”-Schwartz 11/e p1640 “On microscopic examination, the gland is diffusely infiltrated by small lymphocytes and plasma cells and occasionally shows well-developed germinal centers. Thyroid follicles are smaller than normal with reduced amounts of colloid and increased interstitial connective tissue. The follicles are lined by Hürthle or Askanazy cells, which are characterized by abundant eosinophilic, granular cytoplasm.”-Schwartz 11/e p1640

Recent Questions 5. Ans. a. 50% reduction in PTH within 10 mins of gland removal (Ref: Bailey 27/e p830; Sabiston 20/e p934) “The Miami criteria were developed to determine the extent of resection. A drop in the PTH into the normal range and to less than half the maximum preoperative PTH at 10 minutes appears to accurately predict single-gland disease.”-Bailey 27/e p830 “Various protocols exist that use intraoperative PTH measurements. The “Miami” criteria developed by Irvin and colleagues describe biochemical cure as a 50% decrease in PTH levels from baseline 10 to 15 minutes after resection of the targeted parathyroid gland.” -Sabiston 20/e p934 6. Ans. b. CP class B (Ref: Bailey 27/e p1157; Schwartz 11/e p1365; Sabiston 20/e p216) The Child-Pugh class for patient who has a serum bilirubin of 2.5 mg/dL (2 points), serum albumin of 3 g/dL (2 points), INR of 2 (2 points) along with Mild ascites (2 points) but no encephalopathy (1 point) is class B (total points = 9). Child-Turcotte-Pugh (CTP) Scoring System • CTP score is a measure to assess hepatic functionQ in many liver diseases. • It was initially devised to classify patients into risk groups prior to undergoing porto-systemic shunt surgeriesQ. • It is used to assess prognosis in cirrhosisQ and many liver diseases. Child-Turcotte-Pugh (CTP) Score Variable

1 Point

2 Points

3 Points

Serum albumin (g/dL)

>3.5

2.8–3.5

450 mm Hg·s·cm in the body is suggestive of Type III achalasia (spastic achalasia). Type of Achalasia

Criteria

Type I achalasia (classic achalasia)

• Elevated median IRP (>15 mm Hg), 100% failed peristalsis (DCI 15 mm Hg), no normal peristalsis, premature (spastic) contractions with DCI >450 mm Hg·s·cm with ≥20% of swallowsQ • May be mixed with panoesophageal pressurisation

DCI, distal contractile integral (mm Hg·s·cm); EGJ, esophaogastric junction; IRP, integrated relaxation pressure (mm Hg); LES, lower esophageal sphincter. 15. Ans. b. Carcinoma esophagus (Ref: Bailey 27/e p1086; Schwartz 11/e p1070; Sabiston 20/e p1028) The given barium swallow shows advanced carcinoma esophagus with abrupt, irregular narrowing in the distal esophagus, with more proximal dilation along with shouldered margin and apple core appearance. “Barium esophagram may demonstrate irregular narrowing or ulceration. The classic “apple-core” filling defect is seen only if there is symmetrical, circumferential narrowing. Instead, there is often an asymmetrical bulge seen with an infiltrative appearance.” -Sabiston 20/e p1028 16. Ans. a. Spontaneous perforation of the esophagus (Ref: Bailey 27/e p1072, 1073; Schwartz 11/e p1083; Sabiston 20/e p1025) History of upper abdominal pain after a heavy meal along with tenderness in the upper abdomen and on X-ray, widening of the mediastinum with pneumo-mediastinum is highly suggestive of Spontaneous perforation of the esophagus (Boerhaave’s syndrome). “Barotrauma (spontaneous perforation, Boerhaave’s syndrome): This occurs classically when a person vomits against a closed glottis. The pressure in the oesophagus increases rapidly, and the oesophagus bursts at its weakest point in the lower third, sending a stream of material into the mediastinum and often the pleural cavity as well.”-Bailey 27/e p1072

p11

p12

Surgery Essence “The diagnosis can usually be suspected from the history and associated clinical features. A chest radiograph is often confirmatory with air in the mediastinum, pleura or peritoneum. Pleural effusion occurs rapidly either as a result of free communication with the pleural space or as a reaction to adjacent inflammation in the mediastinum. A contrast swallow or CT is nearly always required to guide management.”-Bailey 27/e p1072, 1073 17. Ans. a. Vitamin B12 deficiency (Ref: Bailey 27/e p1256; Schwartz 11/e p1254; Sabiston 20/e p1291) “Short Bowel Syndrome: Length alone, however, is not the only determining factor of complications related to small bowel resection. For example, if the distal two thirds of the ileum, including the ileocecal valve, is resected, significant abnormalities of absorption of bile salts and vitamin B12 may occur, resulting in diarrhea and anemia, although only 25% of the total length of the small bowel has been removed.”-Sabiston 20/e p1291 “Short Bowel Syndrome: Patients with as little as 100–200 cm of jejunum anastomosed to an intact colon may therefore be able to maintain satisfactory macronutrient, fluid and electrolyte status, although they will, of course, be at risk of fat-soluble and B12 vitamin deficiencies and will also generally need oral nutritional supplements of trace elements, vitamins and minerals.” -Bailey 27/e p1256 18. Ans. b. Intussusception (Ref: Bailey 27/e p129; Schwartz 11/e p1253; Sabiston 20/e p1879; Nelson 20/e p1813) History of abdominal pain, blood in stools and a palpable mass on the abdominal examination along with barium enema showing coiled spring sign is highly suggestive of Intussusception. “Intussusception: Classically, a previously healthy infant presents with colicky pain and vomiting (milk then bile). Between episodes, the child initially appears well. Later, they may pass a ‘redcurrant jelly’ stool. Clinical signs include dehydration, abdominal distension and a palpable sausage-shaped mass in the right upper quadrant. Rectal examination may reveal blood or rarely the apex of the intussusceptum.”-Bailey 27/e p129 “Contrast enemas demonstrate a filling defect or cupping in the head of the contrast media where its advance is obstructed by the intussusceptum. A central linear column of contrast media may be visible in the compressed lumen of the intussusceptum, and a thin rim of contrast may be seen trapped around the invaginating intestine in the folds of mucosa within the intussuscipiens (coiledspring sign), especially after evacuation.”-Nelson 20/e p1813 19. Ans. d. 4 (Ref: Bailey 27/e p1274; Schwartz 11/e p1287; Sabiston 20/e p1333) Hinchey Classification of Complicated Diverticulitis Grade I

Mesenteric or pericolic abscessQ

Grade II

Pelvic abscessQ

Grade III

Purulent peritonitisQ

Grade IV

Faecal peritonitisQ

20. Ans. b. Juvenile polyp (Ref: Bailey 27/e p1327; Schwartz 11/e p1291; Sabiston 20/e p1372) History of bleeding PR in a 5 years old child with presence of a polypoidal mass in the rectum and biopsy showing dilated crypts and inflamed stroma is highly suggestive of juvenile polyp. “Juvenile polyp: This is a bright red, glistening pedunculated sphere (‘cherry tumour’), which is found in infants and children. Occasionally, it persists into adult life. It can cause bleeding, or pain if it prolapses during defecation. It often separates itself, but can be removed easily with forceps or a snare. A solitary juvenile polyp has virtually no tendency to malignant change, but should be treated if it is causing symptoms. It has a unique histological structure with large mucus-filled spaces covered by a smooth surface of thin rectal cuboidal epithelium.”-Bailey 27/e p1327 21. Ans. b. Gastroschisis (Ref: Bailey 27/e p135; Schwartz 11/e p1554; Sabiston 20/e p1071) In this newborn image, the bowel is exposed and located on the right side of umbilical cord is highly suggestive of gastroschisis. “Gastroschisis is another congenital defect of the abdominal wall in which the umbilical membrane has ruptured in utero, allowing the intestine to herniate outside the abdominal cavity. The defect is almost always to the right of the umbilical cord, and the intestine is not covered with skin or amnion.”-Sabiston 20/e p1071

Recent Questions 22. Ans. b. Partial nephrectomy (Ref: Bailey 27/e p1420; Sabiston 20/e p2098) “The trend in extirpative surgery is to perform nephron sparing or partial nephrectomy for most T1 tumors (Tumor 7 cm or less in greatest dimension, limited to the kidney). Partial nephrectomy is equivalent to radical nephrectomy in this tumor stage and should be considered for all patients with a T1a tumor (Tumor 4 cm or less) and most with T1b (Tumor more than 4 cm but not more than 7 cm) tumors.”-Sabiston 20/e p2098 23. Ans. a. Bulbar urethra (Ref: Bailey 27/e p1479 “Rupture of the bulbar urethra: There is a history of a blow to the perineum, usually due to a fall astride injury. The bulbar urethra is crushed upwards onto the pubic bone, typically with significant bruising. In the days of sailing ships, the common cause was falling astride a spar and the modern equivalent is seen among workers losing their footing on scaffolding. Cycling accidents, loose manhole covers and gymnasium accidents astride the beam account for a number of cases.”-Bailey 27/e p1479 24. Ans. b. MCU with bulbar urethral stricture (Ref: Schwartz 11/e p2093) The given image is of MCU showing bulbar urethral stricture. 25. Ans. b. Most common site is the calf (Ref: Bailey 27/e p943; Schwartz 11/e p953; Sabiston 20/e p1780) “Intermittent claudication: Intermittent claudication occurs as a result of anaerobic muscle metabolism and is classically described as debilitating cramp-like pain felt in the muscles that is: reliably brought on by walking; not present on taking the first step (unlike osteoarthritis); reliably relieved by rest both in the standing and sitting positions; usually within 5 minutes. The distance that a patient is able to walk without stopping varies (claudication distance) only slightly from day to day. It is decreased by increasing the work demands and hence oxygen requirements of the muscles affected, e.g. walking up hill, increasing the speed of walking and/ or carrying heavy weights, and secondly by general health conditions that reduce the oxygen delivery capacity of the arterial system, e.g. anaemia or heart failure. The muscle group affected by claudication is classically one anatomical level below the level of arterial disease and is usually felt in the calf because the superficial femoral artery is the most commonly affected artery (70% of cases). Aorto-iliac disease (30% of cases) may cause thigh or buttock claudication. Buttock claudication in association with sexual impotence resulting from arterial insufficiency is eponymously called Leriche’s syndrome. It is very rare.”-Bailey 27/e p943 26. Ans. a. Arterial disease with aorto-iliac involvement (Ref: Bailey 27/e p943; Schwartz 11/e p941; Sabiston 20/e p1738) “Aorto-iliac disease (30% of cases) may cause thigh or buttock claudication. Buttock claudication in association with sexual impotence resulting from arterial insufficiency is eponymously called Leriche’s syndrome. It is very rare.”-Bailey 27/e p943 “Aorto-iliac disease: Symptoms typically consist of bilateral thigh or buttock claudication and fatigue. Men report diminished penile tumescence and may have complete loss of erectile function. These symptoms in the absence of femoral pulses constitute Leriche’s syndrome.”-Schwartz 11/e p941 27. Ans. d. Monitor till size reaches 55 mm (Ref: Bailey 27/e p961; Schwartz 11/e p920; Sabiston 20/e p1725) “An asymptomatic abdominal aortic aneurysm in an otherwise fit patient should be considered for repair if >55 mm in diameter (measured by ultrasonography). The annual incidence of rupture rises from 1% or less in aneurysms that are 55 mm, provided there is no major comorbidity. Regular ultrasonographic assessment is indicated for asymptomatic aneurysms 2 cm but ≤3 cm in greatest dimension. 8th AJCC TNM Classification of Lung Cancer Tis: Carcinoma in situ T1a: Tumor ≤1 cm in greatest dimensionQ T1b: Tumor >1 cm but ≤2 cm in greatest dimensionQ T1c: Tumor >2 cm but ≤3 cm in greatest dimensionQ T2: Tumor >3 cm but ≤5 cm or tumor with any of the following features: Involves main bronchus, regardless of distance to the carina but without involvement of carinaQ • Invades visceral pleuraQ • Associated with atelectasis or obstructive pneumonitis that extends to the hilar region either involving part of or the entire lungQ • T2a: Tumor >3 cm but ≤4 cm in greatest dimensionQ • T2b: Tumor >4 cm but ≤5 cm in greatest dimensionQ T3: Tumor >5 cm but ≤7 cm in greatest dimension or one that directly invades any of the following: parietal pleura, chest wall (including superior sulcus tumors), phrenic nerve, parietal pericardium; or separate tumor nodule(s) in the same lobe as the primaryQ T4: Tumor >7 cm or of any size that invades any of the following: diaphragm, mediastinum, heart, great vessels, trachea, recurrent laryngeal nerve, esophagus, vertebral body, carina; or Separate tumor nodule(s) in a different ipsilateral lobe to that of primaryQ N1: Metastasis in ipsilateral peribronchial and/or ipsilateral hilar lymph nodes and intrapulmonary nodes, including involvement by direct extensionQ N2: Metastasis in ipsilateral mediastinal and/or subcarinal lymph node(s)Q N3: Metastasis in contralateral mediastinal, contralateral hilar, ipsilateral or contralateral scalene, or supraclavicular lymph node(s)Q M1a: Separate tumor nodule(s) in a contralateral lobe; tumor with pleural or pericardial nodules or malignant pleural (or pericardial) effusionQ M1b: Single extra-thoracic metastasis in a single or multiple organsQ M1c: Multiple extra-thoracic metastasis in single or multiple organsQ 30. Ans. b. Moderate (Ref: Bailey 27/e p989)

The patient with clinical signs of DVT (score 3) has tachycardia (score 1.5) and history of bladder cancer (score 1). According to modified Well’s scoring, there would be moderate probability (Score 5.5) of pulmonary embolism. Modified Wells Criteria for Predicting Pulmonary Embolism (PE) Variable

Score

Clinical signs & symptoms DVT (minimum of leg swelling & pain on palpation of deep veins)

3

Alternative diagnosis less likely than PEQ

3

Q

1.5

Heart rate >100 bpmQ Immobilisation >3 days or surgery within past 4 weeks

1.5

Previous DVT or PEQ

1.5

HaemoptysisQ

1

MalignancyQ (treatment or palliation within past 6 months)

1

Q

Low risk: 6 31. Ans. a. Fat embolism (Ref: Bailey 27/e p956; Sabiston 20/e p501) “Fat embolism may follow major bony fractures.”-Bailey 27/e p956 “Fat emboli syndrome (FES) is a condition characterized by respiratory distress, altered mental status, and skin petechiae.” -Sabiston 20/e p501 32. Ans. a. Chronic lymphedema (Ref: Bailey 27/e p998; Sabiston 20/e p1850) In the given image, there is swelling of dorsum of foot and buffalo hump appearance of foot, which is seen in chronic lymphedema.

Recent Questions “Unlike other types of oedema, lymphoedema characteristically involves the foot. The contour of the ankle is lost through infilling of the submalleolar depressions, a ‘buffalo hump’ forms on the dorsum of the foot, the toes appear ‘square’ because of confinement of footwear and the skin on the dorsum of the toes cannot be pinched because of subcutaneous fibrosis (Stemmer’s sign).” -Bailey 27/e p998 “In most patients with second- or third-stage lymphedema, the characteristic findings on physical examination can usually establish the diagnosis. The edematous limb has a firm and hardened consistency. There is loss of the normal perimalleolar shape, resulting in a “tree trunk” pattern. The dorsum of the foot is characteristically swollen, resulting in the appearance of the “buffalo hump,” and the toes become thick and squared.”-Sabiston 20/e p1850 33. Ans. b. Lentigo maligna (Ref: Bailey 27/e p609; Schwartz 11/e p530; Sabiston 20/e p729) “Lentigo maligna melanoma occurs most commonly on the sun-exposed areas of older individuals and is manifested as a flat, dark, variably pigmented lesion, with irregular borders and a history of slow development.”-Sabiston 20/e p729 “Lentigo maligna represents 10% of melanoma cases and is a less aggressive subtype of melanoma in situ that typically arises on sun-exposed areas of the head and neck.”-Schwartz 11/e p530 34. Ans. d. 42°C (Ref: Bailey 27/e p632; Schwartz 11/e p523; Harrison 20/e p1930) “Frostbite usually affects the distal aspects of the extremities or exposed parts of the face, such as the ears, nose, chin, and cheeks.”-Harrison 20/e p1930 “Initial treatment is rewarming, performed in an environment where reexposure to freezing conditions will not occur. Rewarming is accomplished by immersion of the affected part in a water bath at temperatures of 40°–44°C (104°–111°F). Massage, application of ice water, and extreme heat are contraindicated. The injured area should be cleansed with soap or antiseptic, and sterile dressings should be applied. Analgesics are often required during rewarming. Antibiotics are used if there is evidence of infection.”-Harrison 20/e p1930 “Frostbite injuries affect the peripheries in cold climates. The initial treatment is with rapid rewarming in a bath at 42°C. The cold injury produces delayed microvascular dam- age similar to that of cardiac reperfusion injury. The level of damage is difficult to assess, and surgery usually does not play a role in its management, which is conservative, until there is absolute demarcation of the level of injury.”-Bailey 27/e p632 “Even so, the standard treatment of frostbite injury begins with rapid rewarming to 40°C to 42°C.”-Schwartz 11/e p523 35. Ans. a. Craniopharyngioma (Ref: Schwartz 11/e p1858; Sabiston 20/e p1915; Harrison 20/e p648) “Craniopharyngiomas are rare, usually suprasellar, partially calcified, solid, or mixed solid-cystic benign tumors that arise from remnants of Rathke’s pouch. They have a bimodal distribution, occurring pre-dominantly in children but also between the ages of 55 and 65 years. They present with headaches, visual impairment, and impaired growth in children and hypopituitarism in adults. Treatment involves surgery, RT, or a combination of the two.”-Harrison 20/e p648 36. Ans. a. Inferior belly of omohyoid (Ref: Bailey 27/e p790) “LOCATION OF THE FACIAL NERVE TRUNK: The main methods of facial nerve trunk localisation can be divided into antegrade and retrograde. The former utilises anatomical landmarks to identify the nerve trunk after its exit from the stylomastoid foramen, which is then traced distally. Landmarks commonly used are: 1. The inferior portion of the cartilaginous canal. This is termed Conley’s pointer (tragal pointer) and indicates the position of the facial nerve, which lies 1 cm deep and inferior to its tip; 2. The upper border of the posterior belly of the digastric muscle. Identification of this muscle not only helps to mobilise the parotid gland, but also exposes an area immediately superior, in which the facial nerve is usually located; 3. The squamotympanic fissure; 4. The styloid process (the nerve is superficial to it); 5. The mastoid process can be drilled and the nerve identified more proximally.”-Bailey 27/e p790 37. Ans. a. Plexiform neurofibroma (Ref: Harrison 20/e p655; Neena Khanna 4/e p34) “Plexiform neurofibroma: A special type of neurofibroma is the plexiform variant, which involves an entire large nerve and its branches, forming a mass of tangled, ropelike structures that feel like a “bag of worms” on palpation and that can reach an enormous size and be associated with massive soft tissue overgrowth leading to significant functional impairment. A plexiform neurofibroma is considered to be pathognomonic of NF1.”-Fitzpatrick 6/e p1159

p15

p16

Surgery Essence

Fig. Plexiform neurofibromatosis “Plexiform neurofibromas: Diffuse plaques that feel knotty or wormy on palpation.”-Neena Khanna 4/e p34 38. Ans. a. CT scan (Ref: Sabiston 20/e p413, 414, 416) Primary survey of ATLS does not include CT scan. “Following a defined order of assessment, life-threatening conditions are immediately addressed at the time of identification. This initial assessment, also termed the primary survey, follows the mnemonic ABCDE: Airway and cervical spine protection; Breathing; Circulation; Disability or neurologic condition; Exposure and environmental control”-Sabiston 20/e p413 “Circulation: A chest radiograph can then evaluate for thoracic blood loss, and a pelvic radiograph will identify a pelvic fracture. To evaluate the abdomen, the focused abdominal sonography in trauma (FAST) scan is a rapidly obtainable ultrasound examination that assesses for intraperitoneal fluid.”-Sabiston 20/e p416 39. Ans. b. Needle decompression in the 5th intercostal space (Ref: Bailey 27/e p367; Schwartz 11/e p186; Sabiston 20/e p415) “Tension pneumothorax: The clinical presentation is dramatic. The patient is increasingly restless with tachypnoea, dyspnoea and distended neck veins (similar to pericardial tamponade). Clinical examination may reveal tracheal deviation; this is a late finding and is not necessary to clinically confirm diagnosis. There will also be hyper-resonance and decreased or absent breath sounds over the affected hemithorax. Tension pneumothorax is a clinical diagnosis and treatment should never be delayed by waiting for radiological confirmation. Treatment consists of immediate decompression, initially by rapid insertion of a large-bore cannula into the second intercostal space in the mid-clavicular line of the affected side, then followed by insertion of a chest tube through the fifth intercostal space in the anterior axillary line.”-Bailey 27/e p367 40. Ans. d. E-fast (Ref: Bailey 27/e p366; Schwartz 11/e p188) “Thoracic Injury: Ultrasound can be used to differentiate between contusion and the actual presence of blood. Extended focused assessment with sonar for trauma (eFAST) is becoming the most common investigation. The technique uses sonar assessment in the chest, looking for a cardiac tamponade or free blood and air in the hemithorax on each side, and assessment for blood in the abdominal cavity, in the paracolic gutters, subdiaphragmatic spaces and pelvis.”-Bailey 27/e p366 41. Ans. a. Battle sign (Ref: Bailey 27/e p332; Schwartz 11/e p1835) “Battle’s sign: A skull base fracture may be associated with bruising over the mastoid process.”-Bailey 27/e p332 “Extravasation of blood results in ecchymosis behind the ear, known as Battle’s sign.”-Schwartz 11/e p1835 42. Ans. d. Jaundice 7 days (not 6.5 or Three of the following five criteria: • Patient age 40 yearsQ • Serum bilirubin >300 μmol/LQ • Time from onset of jaundice to development of coma >7 daysQ • INR >3.5Q • Drug toxicity, regardless of whether it caused ALF Q

Patients with Acetaminophen-Induced ALF Arterial pH 6.5Q • Serum creatinine >300 μmol/LQ • Presence of encephalopathyQ (grade III or IV)

43. Ans. b. CMV (Ref: Bailey 27/e p1541; Schwartz 11/e p363) “The risk of viral infection is highest during the first 6 months after transplantation and the most common problem is CMV infection. CMV disease may arise because of either reactivation of latent infection or primary infection that can be transmitted by an organ from a CMV-positive donor.”-Bailey 27/e p1541 44. Ans. b. 48 hours (Ref: Sabiston 20/e p291) “Nosocomial infections can be defined as those occurring within 48 hours of hospital admission, 3 days of discharge or 30 days of an operation.”-Practical Healthcare Epidemiology 3/e p124 45. Ans. c. Induration (Ref: Bailey 27/e p48) Induration is not the part of the ASEPSIS wound score. The ASEPSIS wound score Criterion

Points

Additional treatmentQ • Antibiotics for wound infection • Drainage of pus under local anaesthesia • Debridement of wound under general anaesthesia

0 10 5 10

Serous dischargeQ Erythema

Daily 0–5 Daily 0–5

Q

Purulent exudateQ

Daily 0–10

Separation of deep tissuesQ

Daily 0–10

Isolation of bacteria from woundQ Stay as inpatient prolonged over 14 days as result of wound infection

10 Q

5

46. Ans. c. Isograft (Ref: Sabiston 20/e p602) “Isografts are organs transplanted between identical twins and are immunologically indistinguishable and thus are not rejected.” -Sabiston 20/e p602

AIIMS NOVEMBER 2019 47. Ans. b. Digastric muscle (Ref: Bailey 27/e p758; Schwartz 11/e p647; Sabiston 20/e p794) Digastric muscle is not removed in radical neck dissection. “Classical Radical Neck Dissection (Crile): The classic operation involves resection of the cervical lymphatics and lymph nodes and those structures closely associated: the internal jugular vein, the accessory nerve, the submandibular gland and the sternocleidomastoid muscle. These structures are all removed en bloc and in continuity with the primary disease if possible. The main disability that follows the operation is weakness and drooping of the shoulder due to paralysis of the trapezius muscle as a consequence of excision of the accessory nerve.”-Bailey 27/e p758 48. Ans. a. Monopolar cautery (Ref: Sabiston 20/e p235-236) The name of given instrument is monopolar cautery.

p17

p18

Surgery Essence Monopolar Electrocautery

Bipolar Electrocautery

LigaSure

Hyfrecators

49. Ans. c. Grey (Ref: Bailey 25/e p29; ATLS 10th edition 2018 Chapter 3 http://bulletin.facs.org/2018/06/atls-10th-edition-offers-new-insights-intomanaging-trauma-patients/) Grey cannula has the large bore (16 G) with flow rate of 180 mL/min and is the preferred option for rapid fluid resuscitation in patients with rehydration. 
 Color code

Gauge

External Diameter (mm)

Length (mm)

Flow Rate (mL/min)

Orange

Q

14G

2.1

Grey

16GQ

Green

Indications

45

240

Q

Trauma, surgical proceduresQ

1.8

45

180Q

Trauma, surgical proceduresQ

18GQ

1.3

32/45

90Q

Trauma, quick blood transfusionQ

Pink

20GQ

1.1

32

60Q

Normal IV or blood transfusionQ

Blue

22GQ

0.9

25

36Q

Children, older adultsQ

Yellow

24G

0.7

19

20

Neonates, children, elderly

Violet

26G

0.6

19

13

NeonatesQ

50. Ans. a → c → d → b (Ref: https://www.who.int/gpsc/5may/Hand_Hygiene.pdf) Sequence of hand washing: a → c → d → b

Recent Questions

Step 1 Step 2 Step 3 Step 4 Step 5 Step 6 Step 7

• • • • • • •

Steps of Handwashing Rub palms together Rub the back of hands Interlace fingers and rub hands together Interlock fingers and rub the back of fingers of both hands Rub thumb in a rotating manner followed by the area between index finger and thumb for both hands Rub fingertips on palm for both hands Rub both wrists in a rotating manner; Rinse and dry thoroughly

51. Ans. b. Negative pressure wound therapy (Ref: Bailey 27/e p30; Schwartz 11/e p; Sabiston 20/e p) The name of given treatment modality is vacuum assisted closure or negative pressure wound therapy (NPWT). “Vacuum-assisted closure: This is now more correctly known as negative pressure wound closure. Applying intermittent negative pressure of approximately −125 mm Hg appears to hasten debridement and the formation of granulation tissue in chronic wounds and ulcers. A foam dressing is cut to size to fit the wound. A perforated wound drain is placed over the foam, and the wound is sealed with a transparent adhesive film. A vacuum is then applied to the drain. Negative pressure may act by decreasing oedema, by removing interstitial fluid and by increasing blood flow. As a result, bacterial counts decrease and cell proliferation increases, thereby creating a suitable bed for graft or flap cover.”-Bailey 27/e p30 52. Ans. a. Lahey’s method (Ref: Hamilton Bailey’s Demonstration of Physical Signs in Clinical Surgery 19/e p409, 410) In this method of thyroid palpation, thyroid gland is pushed to left side and being palpated with fingers of left hand. This is Lahey’s method of thyroid palpation. “The hallmark of a thyroid swelling is that the swelling moves on deglutition. Examination of the thyroid is best performed both from behind and in front of the patient with their neck in slight extension. The gland can be made more prominent by Pizillo’s manoeuvre in which the patient’s hands are placed behind their occiput and the head is pushed backwards against the clasped hands.”-Hamilton Bailey’s Demonstration of Physical Signs in Clinical Surgery 19/e p409 “The lobes may be better examined by Lahey’s method in which, for examination of the left lobe, the thyroid gland is pushed to the left from the right side by the examiner’s left hand, and similarly to the right by the right hand. Smaller nodules within the substance of the gland are better appreciated by palpating the gland using the thumb while the patient swallows (Crile’s method).”-Hamilton Bailey’s Demonstration of Physical Signs in Clinical Surgery 19/e p410 Thyroid Palpation

Pizillo’s Method

Lahey’s method

Crile’s method

• Examination of thyroid • Thyroid gland is made more prominent • Lobes may be better • For palpation of is best performed in short necked & obese patients. examined by Lahey’s method smaller nodules both from behind & in • Patient’s hands are placed behind within gland substance • Thyroid gland is pushed front of the patient with Palpating the gland their occiput & head is pushed to one side by examiner’s their neck in slight using the thumb while backwards against the clasped hand & lobe is palpated with extension. the patient swallows hands. fingers of other hand.

53. Ans. b. E. multilocularis (Ref: Harrison 20/e p1644, 1645) Slow growing alveolar like tumour in liver is caused by Echinococcus multilocularis. Echinococcus multilocularis causes malignant hydatid disease. “E. multilocularis, which causes multilocular alveolar lesions that are locally invasive, is found in Alpine, sub-Arctic, or Arctic regions, including central and northern Europe; western China and central Asia; and isolated areas in North America.” -Harrison 20/e p1644 “The larval forms of E. multilocularis characteristically present as a slowly growing hepatic tumor, with progressive destruction of the liver and extension into vital structures. Patients commonly report upper-quadrant and epigastric pain. Liver enlargement and obstructive jaundice may be apparent. The lesions may infiltrate adjoining organs (e.g., diaphragm, kidneys, or lungs) or may metastasize to the spleen, lungs, or brain.”-Harrison 20/e p1645

p19

p20

Surgery Essence 54. Ans. d. Platelet transfusion (Ref: Bailey 27/e p1163; Schwartz 11/e p1367; Sabiston 20/e p1149) In patients with acute variceal bleeding with hypotension fluid resuscitation and packed cell transfusion is indicated. Octreotide infusion is given to control the ongoing bleeding. After stabilization of the patient, upper GI endoscopy is done to confirm the diagnosis and for endoscopic variceal ligation or sclerotherapy. In variceal bleeding patients, associated thrombocytopenia is usually secondary to hypersplenism due to cirrhosis and is treated if the platelet count falls below 50 × 109/L. So, platelet transfusion is usually not given in the patients of acute variceal bleeding. “Varices usually present with the acute onset of a large-volume haematemesis, the lower oesophagus being the most common site for variceal bleeding. The diagnosis may be suspected if the patient is known to have liver cirrhosis, but it needs to be confirmed following initial resuscitation of the patient. Variceal haemorrhage is a medical emergency. Patients with massive hemorrhage should be admitted to the intensive care unit. Venous access should be obtained through two large-bore peripheral cannulae. Resuscitation should be commenced, ideally with blood. Liver function tests will reveal underlying liver disease, and a coagulation profile will reveal any underlying coagulopathy. Hypervolaemia should be avoided since this may increase portal pressure and exacerbate the bleeding. Vitamin K is administered (10 mg intravenously (IV)) as well as tranexamic acid (1 g i.v.), but correction of a coagulopathy will require the administration of fresh frozen plasma (FFP). A major transfusion protocol should be activated. An associated thrombocytopenia is usually secondary to hypersplenism due to cirrhosis and is treated if the platelet count falls below 50 × 109/L. Treatment with a splanchnic vasoconstrictor (such as terlipressin) should be started. Administration of a prophylactic antibiotic is recommended to prevent or treat associated bacterial infection. As soon as the patient is haemodynamically stabilized, an upper gastrointestinal endoscopy should be performed to establish the diagnosis because 50% of patients with portal hypertension will have a non-variceal source of bleeding.”-Bailey 27/e p1163 55. Ans. a → b → d → c (Ref: Campbell 11/e p1149) Renal injury in order from lower to higher grade: a. Subcapsular hematoma → b. Hematoma confined to Gerota’s fascia → d. Avulsion of renovascular pedicle → c. Shattered kidney

Fig. Renal injury classification Grade I II

Type

Description

Contusion

• Microscopic (>3 RBCs/HPF) or gross hematuria, urological studies normal

Hematoma

• SubcapsularQ, nonexpanding without parenchymal laceration.

Hematoma

• Nonexpanding perirenalQ hematoma, confined to renal retroperitoneum

Laceration

• 1 cm parenchymal depth of renal cortex without collection system rupture or urinary extravasationQ

IV

Laceration

• Parenchymal laceration extending through collecting systemQ

Vascular

• Main renal artery or vein injury with contained hemorrhage

Laceration

• Completely “Shattered kidney”Q

Vascular

• Avulsion of renal hilum, devascularising the kidney.

V

Note: Advance one grade for bilateral injuries up to grade III. 56. Ans. d. Stage 4 (Ref: Bailey 27/e p29, 30, 615, 616; Schwartz 11/e p290, 291, 523, 524) The given pressure sore is stage 4 as there is full thickness skin loss through fascia with extensive tissue destruction. Staging of Pressure Sore Stage

Description

1

Non-blanchable erythema without a breach in epidermis (Early superficial ulcerQ)

2

Partial thickness skin loss involving epidermis & dermis (Late superficial ulcerQ)

3

Full thickness skin loss extending into subcutaneous tissue but not through underlying fascia (Early deep ulcerQ)

4

Full thickness skin loss through fascia with extensive tissue destruction, maybe involving muscle, bone, tendon or joint (Late deep ulcerQ)

57. Ans. c. Immediate exploration of the affected side and operate opposite side even if asymptomatic (Ref: Bailey 27/e p1501; Campbell 11/e p3390) In cases of testicular torsion, immediate exploration of the affected side and operation of opposite side even if asymptomatic should be done because the anatomical predisposition is likely to be bilateral. “Surgical exploration of the testis through a hemiscrotal transverse (dartos pouch) or midline raphe incision should first address the affected side. The testis is delivered and the tunica vaginalis opened to note the color of the testis, the number of rotations, and the anatomy of the tunica vaginalis. The testis is untwisted, wrapped in warm soaked gauze, and observed for improvement in color while the contralateral testis is fixed with nonabsorbable suture to reduce the risk of metachronous torsion. The affected testis is re-examined for potential viability, and the largely subjective decision for orchidopexy or orchiectomy is made. A Doppler flow probe or incision of the tunica albuginea with assessment of bleeding may document intratesticular flow after detorsion; however, the reliability of these assessments lacks validation. If the testis is to be retained, it is fixed either via dartos pouch or directly to the dartos with nonabsorbable suture.”-Campbell 11/e p3390 58. Ans. a. Jackson-Pratt suction drain (Ref: Schwartz 11/e p1383; Sabiston 20/e p1302) The given drain is Jackson-Pratt suction drain, used as a postoperative drain for collecting bodily fluids from surgical sites.

“A Jackson-Pratt drain (also called a JP drain) is a closed-suction medical device that is commonly used as a postoperative drain for collecting bodily fluids from surgical sites. The device consists of an internal drain connected to a grenade-shaped bulb via plastic tubing.” 59. Ans. a. Roos test (Ref: Sabiston 20/e p2007) The given clinical picture suggests the diagnosis of thoracic outlet compression syndrome (TOS). In the test, patient is instructed to draw shoulders backwards, raise arms to horizontal position with elbows flexed to 90° and Exercise the hands. This test is known as Roos test (Arm claudication test). “A Roos test is performed by asking the patient to hold both arms overhead in a surrender position while opening and closing the fists. This reproduces symptoms within 1 minute and, if continued, the arm collapses at the side.”-Sabiston 20/e p2007

p21

p22

Surgery Essence 60. Ans. a. Malignant melanoma (Ref: Schwartz 11/e p530; Sabiston 20/e p726) ABCDE mnemonic is used for malignant melanoma. ABCDE rule for Malignant melanoma: Asymmetry (one half of the mole doesn’t match the other), Border irregularity, Color that is not uniform, Diameter greater than 6 mm (about the size of a pencil eraser), and Evolving size, shape or color. “Melanoma commonly is manifested as an irregular pigmented skin lesion that has grown or changed over time. The ABCDEs of melanoma are used to guide diagnosis and the decision to perform a biopsy: asymmetry, irregular borders, color changes, diameter greater than 6 mm, and evolution or change over time.”-Sabiston 20/e p726 “Melanoma most commonly manifests as cutaneous disease, and clinical characteristics include an Asymmetric outline, changing irregular Borders, Color variations, Diameter greater than 6 mm, and Elevation (ABCDE). Other key clinical characteristics include a pigmented lesion that has enlarged, ulcerated, or bled.”-Schwartz 11/e p530

A

B

C

D

E

61. Ans. b. To create pneumoperitoneum (Ref: Schwartz 11/e p1412, 459-460; Sabiston 20/e p1051, 2058) The given instrument is Veress needle, used to create pneumoperitoneum in laparoscopic surgeries. “Pneumoperitoneum is created using a Veress needle or insufflation via a Hasson port.”-Schwartz 11/e p1699

“The pneumoperitoneum is created with carbon dioxide gas, either with an open technique or by closed needle technique. Initially, a small incision is made in the upper edge of the umbilicus. With the closed technique, a special hollow insufflation needle (Veress needle) that is spring-loaded with a retractable cutting outer sheath is inserted into the peritoneal cavity and used for insufflation.” -Schwartz 11/e p1412 62. Ans. a. Allen test (Ref: Bailey 27/e p444; Schwartz 11/e p1412, 459-460; Sabiston 20/e p1051, 2058) The test being done is Allen’s test, which is done before performing the ABG analysis. Allen’s test examines the patency of the interconnection between the superficial and deep palmar arches, and determines whether either artery is capable of maintaining the arterial supply to the hand in isolation.

Recent Questions “Sites for arterial cannulation: At the wrist, the radial artery is commonly used for arterial cannulation for arterial blood sampling and blood pressure monitoring. Thrombosis or vasospasm of the radial artery at this site should not normally jeopardize the circulation to the hand since the interconnecting palmar arterial arches provide a collateral circulation to the hand. Allen’s test should be performed prior to any such procedure.”-Gray’s 41/e p791

A

B Fig. Allen’s test

“Allen’s test examines the patency of the interconnection between the superficial and deep palmar arches, and determines whether either artery is capable of maintaining the arterial supply to the hand in isolation. To perform the test, the radial and ulnar arteries of one hand are compressed by the examiner’s thumbs. The forearm is then elevated and the hand exsanguinated by clenching the fist. The hand is then opened while arterial compression is continued. The palm of the hand appears pale, unless there is an anomalous arterial supply, e.g. from a persistent median artery. The rapid return of a normal pink colour to the palmar skin following removal of digital pressure from one of the arteries suggests that artery is capable of perfusing the hand adequately in isolation. If the palmar skin remains pale, it must be assumed either that there is a distal obstruction to the artery or that the artery cannot adequately perfuse the hand in isolation. The test is then repeated with release of digital pressure on the other artery.”-Gray’s 41/e p791 63. Ans. d. 600–800 mL/day (Ref: Bailey 27/e p280; Schwartz 11/e p85) Daily insensible fluid loss is 600–800 mL/day. “The healthy person consumes an average of 2000 mL of water per day, approximately 75% from oral intake and the rest extracted from solid foods. Daily water losses include 800 to 1200 mL in urine, 250 mL in stool, and 600 mL in insensible losses. Insensible losses of water occur through both the skin (75%) and lungs (25%) and can be increased by such factors as fever, hypermetabolism, and hyperventilation. Sensible water losses such as sweating or pathologic loss of gastrointestinal (GI) fluids vary widely, but these include the loss of electrolytes as well as water. To clear the products of metabolism, the kidneys must excrete a minimum of 500 to 800 mL of urine per day, regardless of the amount of oral intake.”-Schwartz 11/e p85 Water exchange (60- to 80-kg man) Routes

Avg Daily Volume (mL)

Minimal (mL)

Maximal (mL)

H2O gain: Sensible: Oral fluids

800–1500Q

0

1500/h

Solid foods

500–700

0

1500

Q

Insensible: Water of oxidation

250Q

125

800

Water of solution

0

0

500 Contd…

p23

p24

Surgery Essence Contd… Water exchange (60- to 80-kg man) Routes

Avg Daily Volume (mL)

Minimal (mL)

Maximal (mL)

H2O loss: Sensible: Urine

800–1500Q

300

1400/h

Intestinal

0–250

0

2500/h

Sweat

0

0

4000/h

600Q

600

1500

Q

Insensible: Lungs and skin

64. Ans. d. 1-A; 2-B; 3-D; 4-C (Ref: Bailey 27/e p322, 323, 326; Schwartz 11/e p183, 195; Sabiston 20/e p413, 417) 1. Primary survey: A. Identify what can cause death; 2. Secondary survey: B. Identify other injuries; 3. Definitive care: D. Make a management plan; 4. Resuscitation: C. Treat what can cause death “Following a defined order of assessment, life-threatening conditions are immediately addressed at the time of identification. This initial assessment, also termed the primary survey, follows the mnemonic ABCDE: Airway and cervical spine protection; Breathing; Circulation; Disability or neurologic condition; Exposure and environmental control.”-Sabiston 20/e p413 “Secondary Survey: A thorough head to toe examination is required to assist in identifying all potential injuries. This is often performed immediately after the primary survey in patients who are stable and not requiring emergent intervention. Findings identified during the secondary survey often prompt further evaluation with imaging or other diagnostic modalities.”-Sabiston 20/e p417 “The primary survey aims to identify and manage the most immediately life-threatening pathologies first.”-Bailey 27/e p323 “Secondary Survey: All severely injured patients require a detailed top to toe examination after life-threatening injuries have been identified and managed during the primary survey.”-Bailey 27/e p326 Primary Survey • Aimed at detecting & simultaneously treating immediately life threatening injuriesQ • Identified by the mnemonic ‘ABCDE’ • A: Airway maintenance with cervical spine protectionQ • B: Breathing (ventilation & oxygenation)Q • C: Circulation with hemorrhage controlQ • D: Disability (Brief neurological examination)Q • E: Exposure /Environmental controlQ

Secondary Survey • Consists of head to toe systematic assessment of abdominal, pelvic & thoracic areasQ • Complete inspection of body surface to find all injuries & neurological examinationQ • Patients & surrogates should be queried to obtain an AMPLE history • A: AllergiesQ • M: MedicationQ • P: Past illness or pregnancyQ • L: Last mealQ • E: Events related to injuryQ

Tertiary Survey • Comprehensive patient evaluation after initial resuscitation periodQ • Usually performed about 24 hours after admissionQ • Include a thorough physical examination combined with targeted radiographic imaging (X-ray usage or CT) based on examination findingQ • Decreases the delay in diagnosis of potentially lifethreatening injuriesQ

65. Ans. b. CMV (Ref: Harrison 20/e p1035; Robbin’s 9/e p360; Bailey 27/e p1541; Schwartz 11/e p363) Most common cause of post solid organ transplant infection among the given options is CMV. “The risk of viral infection is highest during the first 6 months after transplantation and the most common problem is CMV infection. CMV disease may arise because of either reactivation of latent infection or primary infection that can be transmitted by an organ from a CMV-positive donor. Recipients at most risk from CMV infection are those who are CMV seronegative (i.e. those who have not been infected previously with CMV) and receive an organ from a CMV-seropositive donor (about half of all UK donors are CMV seropositive).”-Bailey 27/e p1541

Recent Questions AIIMS MAY 2019 66. Ans. a. Neurogenic tumor (Ref: Schwartz 11/e p727; Sabiston 20/e p1608) Most common mediastinal tumor is neurogenic tumor. “Mediastinal pathology varies significantly by patient age. In children, neurogenic tumors of the posterior mediastinum are most common, followed by lymphoma, which is usually located in the anterior or middle compartment. Thymoma in childhood is rare. In adults, the most common tumors include neurogenic tumors of the posterior compartment, benign cysts occurring in any compartment, and thymomas of the anterior mediastinum.”-Schwartz 11/e p727 Mediastinal Tumors in Adults Percentage of Total

Tumor Type

Location

Neurogenic tumors

21

Posterior

CystsQ

20

All

Thymomas

19

Anterior

LymphomasQ

13

Anterior/middle

Germ cell tumors

11

Anterior

Mesenchymal tumors

7

All

Endocrine tumors

6

Anterior/middle

Q

Q

Mediastinal Tumors in Children Percentage of Total

Tumor Type

Location

Neurogenic tumorsQ

40

Posterior

LymphomasQ

18

Anterior/middle

CystsQ

18

All

Germ cell tumors

11

Anterior

Mesenchymal tumors

9

All

Thymomas

Rare

Anterior

Q

Locations of the Common Mediastinal Masses Anterior Mediastinum • ThymomaQ (MC in anterior mediastinum) • LymphomaQ • Germ cell tumorsQ • Thyroid & parathyroid massesQ • Bronchogenic cystQ • AneurysmQ

Middle Mediastinum • Cysts (MC in middle mediastinum): −− Pericardial (MC)Q −− BronchogenicQ −− Enterogenous −− NeuroentericQ • Vascular masses (aneurysmQ) • LN enlargement & lymphomaQ • Mesenchymal tumors • Pheochromocytoma

Posterior Mediastinum • • • • • • • •

Neurogenic tumors (MC overallQ) MeningocelesQ Mesenchymal tumors Pheochromocytoma LymphomaQ Bochdalek herniaQ Bronchogenic cystQ Enterogenous cystQ

Mediastinal Masses • • • • •

MC anterior MM: ThymomaQ MC middle MM: CystQ (Pericardial cyst is MCQ) MC posterior MM: Neurogenic tumorsQ MC MM (overall): Neurogenic tumorsQ MM seen in all three compartments of mediastinum: Lymphoma, bronchogenic cyst & mesenchymal tumorsQ • IOC for diagnosis of MM (except neurogenic tumors): CTQ • IOC for diagnosis of neurogenic tumors: MRIQ

p25

p26

Surgery Essence 67. Ans. b. Inadequate preoperative preparation (Ref: Bailey 27/e p815; Harrison 20/e p2707; Schwartz 11/e p1662, 10/e p1534; Sabiston 20/e p304) Thyrotoxic crisis (storm) is an acute exacerbation of hyperthyroidism. It occurs if a thyrotoxic patient has been inadequately prepared for thyroidectomy. “Thyrotoxic crisis (storm): This is an acute exacerbation of hyperthyroidism. It occurs if a thyrotoxic patient has been inadequately prepared for thyroidectomy and is now extremely rare. Very rarely, a thyrotoxic patient presents in a crisis and this may follow an unrelated operation. Symptomatic and supportive treatment is for dehydration, hyperpyrexia and restlessness. This requires the administration of intravenous fluids, cooling the patient with ice packs, administration of oxygen, diuretics for cardiac failure, digoxin for uncontrolled atrial fibrillation, sedation and intravenous hydrocortisone. Specific treatment is by carbimazole 10–20 mg 6-hourly, Lugol’s iodine 10 drops 8-hourly by mouth or sodium iodide 1g IV. Propranolol intravenously (1–2 mg) or orally (40 mg 6-hourly) will block β-adrenergic effects.”-Bailey 27/e p815 68. Ans. b. PCNL (Ref: Campbell 11/e p1281, 1286; 10/e p1364, 1374, 1380-1381, 1399-1405; Bailey 27/e p1409) Treatment of choice in a patient with a staghorn calculus with minimal hydronephrosis is PCNL. “The management of patients with staghorn stones by a combined approach must be viewed as primarily percutaneous in nature, with SWL being used only as an adjunct to minimize the number of access points required. Improved PNL techniques, incorporating the increasing use of flexible nephroscopy and providing complete or nearly complete clearance of stone material at the time of the primary procedure, may have decreased or eliminated the need for additional SWL treatment.”-Campbell 11/e p1281 “The procedure of choice for patients with staghorn calculi is PNL. When they are left untreated, staghorn calculi are associated with the loss of renal function and increased mortality.”-Campbell 10/e p1374 “PNL followed by either SWL or repeated PNL, should be used for most patients with struvite staghorn calculi, with PNL being the initial element of the combination therapy.”-Campbell 11/e p1286 Renal Stones Treatment decisions by stone burden Stone size/ Composition

Treatment of Choice

Stone ≤2 cm

• ESWLQ • Unless factors of stone composition, location, location or renal anatomy shift the balance towards more invasive modalities (PCNL/URS).

Stone >2 cm

• PCNLQ

Staghorn calculi

• PCNL + ESWL is TOCQ • Initial approach is PCNL, followed by ESWL, as an adjunct to minimize the number of repeat PCNL accesses.

69. Ans. d. Wide bore needle insertion in 2nd intercostal space (Ref: Sabiston 20/e p428; Schwartz 11/e p186, 10/e p163; Bailey 27/e p367) History of chest trauma with respiratory rate >40/minute, BP 90/60 mm Hg and hyper-resonant note on involved side is highly suggestive of tension pneumothorax. Immediate decompression with large bore needle in mid clavicular line in 2nd intercostal space is the urgent treatment for this patient. “Tension pneumothorax and simple pneumothorax have similar signs, symptoms, and examination findings, but hypotension qualifies the pneumothorax as a tension pneumothorax. Although immediate needle thoracostomy decompression with a 14-gauge angiocatheter in the second intercostal space in the midclavicular line may be indicated in the field, tube thoracostomy should be performed immediately in the ED before a chest radiograph is obtained. Recent studies suggest the preferred location for needle decompression may be the 5th intercostal space in the anterior axillary line due to body habitus.”-Schwartz 11/e p186 “Tension pneumothorax: Treatment consists of immediate decompression, initially by rapid insertion of a large-bore cannula into the second intercostal space in the mid-clavicular line of the affected side, then followed by insertion of a chest tube through the fifth intercostal space in the anterior axillary line.”-Bailey 27/e p367 70. Ans. d. Greater auricular nerve (Ref: Bailey 27/e p792) After parotidectomy, numbness on the face during shaving is due to damage of greater auricular nerve. “Complications of parotid gland surgery include: haematoma formation; infection; deformity: unsightly scar and retromandibular hollowing; temporary facial nerve weakness; transection of the facial nerve and permanent facial weakness; sialocele; facial numbness; permanent numbness of the ear lobe associated with great auricular nerve transection; Frey’s syndrome”-Bailey 27/e p792

Recent Questions Complications of Parotidectomy • Hematoma formation • Infection • Deformity: unsightly scar & retromandibular hollowing

• Temporary facial nerve weaknessQ • Transection of facial nerve & permanent facial weaknessQ • Sialocele

• Facial numbnessQ • Permanent numbness of ear lobe associated with great auricular nerve transectionQ • Frey’s syndromeQ

71. Ans. b. Serum lipase (Ref: Bailey 27/e p1223; Schwartz 11/e p1444; Sabiston 20/e p1526; Harrison 20/e p2439, 2440) Severe pain in epigastrium and multiple episodes of vomiting with guarding in epigastrium in chronic alcoholic patient is highly suggestive of acute pancreatitis. The cornerstone of the diagnosis of acute pancreatitis is the clinical findings plus an elevation of pancreatic enzyme levels in the plasma. So the next best step is serum lipase, which helps in making the diagnosis. “Abdominal pain is the major symptom of acute pancreatitis. Pain may vary from a mild discomfort to severe, constant, and incapacitating distress. Characteristically, the pain, which is steady and boring in character, is located in the epigastrium and periumbilical region, and may radiate to the back, chest, flanks, and lower abdomen. Nausea, vomiting, and abdominal distention due to gastric and intestinal hypomotility and chemical peritonitis are also frequent complaints.”-Harrison 20/e p2439 “Acute pancreatitis: Abdominal tenderness and muscle rigidity are present to a variable degree, but compared with the intense pain, these signs may be less impressive.”-Harrison 20/e p2439 “Any severe acute pain in the abdomen or back should suggest the possibility of acute pancreatitis. The diagnosis is established by two of the following three criteria: (1) typical abdominal pain in the epigastrium that may radiate to the back, (2) threefold or greater elevation in serum lipase and/or amylase, and (3) confirmatory findings of acute pancreatitis on cross-sectional abdominal imaging. Patients also have associated nausea, emesis, fever, tachycardia, and abnormal findings on abdominal examination. Laboratory studies may reveal leukocytosis, hypocalcemia, and hyperglycemia.”-Harrison 20/e p2440 “The cornerstone of the diagnosis of AP is the clinical findings plus an elevation of pancreatic enzyme levels in the plasma. A threefold or higher elevation of amylase and lipase levels confirms the diagnosis.”-Sabiston 20/e p1526 72. Ans. b. Gastrinoma (Ref: Bailey 27/e p849; Schwartz 11/e p1480; Sabiston 20/e p952, 954) Most common functional neuroendocrine tumor of pancreas is insulinoma. In the question, insulinoma was not given in the option, so the correct answer would be gastrinoma. “Insulinomas are the most common functional pancreatic endocrine neoplasms and present with a typical clinical syndrome known as Whipple’s triad.”-Schwartz 11/e p1480 “Insulinoma is the most common functioning PNET, with an incidence of 1 to 2 per 1 million population annually in the United States.”-Sabiston 20/e p952 “Gastrin-secreting PNET (gastrinoma) is the second most common functional pancreatic endocrine tumor, with an incidence of 1 per 2.5 million population, and was first described in 1955 by Zollinger and Ellison.”-Sabiston 20/e p954 Neuroendocrine Tumors of Pancreas • • • •

MC NET of Pancreas: Non-functional (Mostly malignant)Q MC functional NET of Pancreas: InsulinomaQ MC benign NET of Pancreas: InsulinomaQ 
 MC malignant functional NET of Pancreas: GastrinomaQ 
 


73. Ans. b. Nothing is to be done (Ref: Campbell 11/e p3440, 10/e p3565; Bailey 27/e p125) Blind-ending testicular vessels during diagnostic laparoscopy is most consistent with an abdominal vanishing testis. So, nothing is to be done in this situation. “Diagnostic laparoscopy followed by laparoscopic orchidopexy if the testis is abdominal has become the preferred approach of many clinicians. Important observations include size and position of the spermatic vessels and vas; testicular size, quality, and position; and patency of the internal inguinal ring. The combination of a closed internal ring and a blind-ending spermatic artery and vas is most consistent with an abdominal vanishing testis, whereas a hernia is frequently but not always associated with a viable abdominal or distal testis.”-Campbell 10/e p3565

p27

p28

Surgery Essence

74. Ans. b. 4 cm (Ref: Schwartz 11/e p576-578) Tumor >4 cm is not classified as locally advanced breast cancer. Early invasive breast cancer includes stage I, IIA, IIB; Locally advanced breast cancer includes stage IIIA, IIIB, IIIC & Distant metastases includes stage IV. Locally advanced breast cancer includes T4 (T4a: Extension to chest wall, not including pectoralis muscle; T4d: inflammatory breast cancer), N2 (N2a: Metastasis in ipsilateral level I, II axillary LNs fixed or matted) & N3. 8th AJCC (2017) TNM Staging for Breast Cancer T: Primary tumor

N: Regional lymph nodes

T1: Tumor ≤2 cm

N1: Metastasis to movable ipsilateral level I, II axillary LNs

T2: Tumor >2 cm & ≤5 cm

N2a: Metastasis in ipsilateral level I, II axillary LNs fixed or matted

T3: Tumor >5 cm

N2b: Metastasis only in clinically apparent ipsilateral internal T4a: Extension to chest wall, not including pectoralis mammary LNs and in the absence of clinically evident axillary LNs metastasis muscle (Note: Clinically apparent is defined as detected by imaging studies T4b: Edema (including Peau d’orange) or ulceration of (excluding lymphoscintigraphy) or by clinical examination or grossly skin, or satellite skin nodules confined to the same breast visible pathologically.) T4c: Both T4a & T4b


N3a: Metastasis in ipsilateral infraclavicular LNs
 N3b: Metastasis in ipsilateral internal mammary LNs & axillary LNs

T4d: Inflammatory carcinoma Note • Invasion of dermis alone does not qualify as T4. • Chest wall includes ribs, intercostal muscles & serratus anterior but not the pectoral muscles. • Dimpling of the skin, nipple retraction, or other skin changes, except those in T4b & T4d, may 
occur in T1, T2 or T3 without affecting the classification.

N3c: Metastasis in ipsilateral supraclavicular LNs M: Distant metastases M0: No distant metastasis M1: Distant metastasis

Stage I

Stage IIA

Stage IIB

Stage IIIA

T1 N0M0

T0N1 M0 T1N1 M0 T2 N0M0

T2N1 M0 T3 N0M0

T0 N2 M0 T1-2 N2 M0 T3 N1-2 M0

Stage IIIB T4 N0-2 M0

Stage IIIC

Stage IV

AnyT N3 M0

AnyT anyN M1

Recent Questions 76. Ans. d. 4 → 3 → 1 → 2 (Ref: https://www.cdc.gov/hai/pdfs/ppe/ppe-sequence.pdf) The correct order of removing PPE (Personal Protective Equipment): Gloves → Face shield → Gown → Mask. “The sequence for removing PPE is intended to limit opportunities for self-contamination. The gloves are consider the most contaminated pieces of PPE and are therefore removed first. The face shield or goggles are next because they are more cumbersome and would interfere with removal of the other PPEs. The gown is third in the sequence, followed by the mask or respirator.” Sequence of Putting on PPE 1. GownQ 2. Mask or respiratorQ 3. Goggles or face shieldQ 4. GlovesQ

Sequence of Removing PPE 1. GlovesQ 2. Goggles or face shieldQ 3. GownQ 4. Mask or respiratorQ

77. Ans. d. Unequally dilated pupils (Ref: Schwartz 11/e p161) Unequally dilated pupils are not the component of qSOFA. “The qSOFA suggests potentially life-threatening sepsis when at least two of the following parameters are met: altered mental status, systolic blood pressure of 100 mm Hg or less, and respiratory rate greater than 22 breaths/minute.”-Schwartz 11/e p161 Quick Sequential Organ Failure Assessment (SOFA) Score qSOFA (Quick SOFA) Criteria Points • Respiratory rate ≥22/minQ 1 • Change in mental statusQ 1 • Systolic BP ≤100 mm HgQ 1 Interpretation Score Mortality 0 3 days) IV: PusQ • IVa: At one point only (2 cm) V: Deep or severe wound infection with or without tissue breakdown; Hematoma requiring aspirationQ 80. Ans. d. Respiratory rate >24/minute and PaCO2 20 breaths/min (not >24/minute) for patients spontaneously ventilating or a PaCO2 StruviteQ

Prostate

Calcium phosphateQ

Salivary gland (Submandibular)

Calcium carbonateQ

Annexure 17 IDEAL TIME FOR TREATMENT Ideal time for Treatment Undescended testis

6 monthsQ

Hypospadias

6–12 monthsQ

Umbilical hernia

5 yearsQ

Cleft lip

3–6 monthsQ

Cleft palate

6–18 monthsQ

Congenital hydrocele

2 yearsQ

Annexure 18 IMPORTANT NAMES IN THE FIELD OF SURGERY Father of surgery Father of Indian surgery

• SushrutaQ

Father of modern surgery

• Joseph ListerQ

Father of modern neurosurgery

• Harvey CushingQ

Father of modern plastic surgery

• Harold GilliesQ

Father of vascular surgery

• Rudolph MatasQ

Father of pediatric surgery

• William Edward LaddQ

Father of modern urology

• Hugh Hampton YoungQ

Annexures

xiii

Annexure 19 INVESTIGATION OF CHOICE

Ultrasound

GallstonesQ

MRCP

CBD stone

MRI

Endoscopy with biopsy Colonoscopy with biopsy Sigmoidoscopy with biopsy Proctoscopy with biopsy Cystoscopy with biopsy FNAC Biopsy

Acute cholecystitisQ

Q

Chronic cholecystitisQ

PSC

Choledochal cystQ

Pancreas divisum

Biliary stricturesQ

Chronic pancreatitisQ

Investigation of Choice Acute mesenteric ischemia

• AngiographyQ

Mesenteric venous thrombosis

• CECTQ

Chronic mesenteric ischemia

• AortographyQ Investigation of Choice

ADPKD

CT scanQ

Retroperitoneal Fibrosis Medullary Sponge Kidney

IVPQ

VUR

MCUQ

Retrocaval ureter

MRIQ

PUJ Obstruction

DTPA scanQ

Renal structure or surface

DMSA scanQ

Annexures

Manometry 24-hours pH monitoring Somatostatin receptor scintigraphy (IOC for localization)

Investigation of Choice Hiatus herniaQ Zenker’s diverticulaQ LeiomyomaQ Gastric diverticulaQ Small bowel diverticulaQ Crohn’s diseaseQ Colonic diverticulaQ DiverticulitisQ Hepatocellular carcinomaQ (Triple phase CT) Q Mesenteric cyst Renal cell carcinomaQ GI tuberculosisQ Retroperitoneal fibrosisQ Acute pancreatitisQ Retroperitoneal sarcomaQ Carcinoma pancreasQ Renal tuberculosisQ Q Pancreatic pseudocyst ADPKDQ Carcinoma gall bladderQ GISTQ Brain tumorsQ Spinal cord tumorsQ Pancoast tumorQ Soft tissue sarcomaQ Staging of carcinoma penisQ Barrett’s esophagusQ Carcinoma esophagusQ Carcinoma stomachQ Q Carcinoma colon Carcinoma rectumQ Carcinoma anal canalQ Carcinoma bladderQ Parotid tumorsQ Thyroid malignanciesQ Carcinoma breast Skin malignanciesQ Q Carcinoma penis Oral cavity malignanciesQ Q Achalasia cardia Diffuse esophageal spasmQ Nutcrackers esophagusQ GERDQ All neuroendocrine tumors of pancreas except insulinomaQ Carcinoid tumorsQ

Barium swallow Barium meal Barium meal follow-through Enteroclysis Barium enema CECT

xiv

Surgery Essence

Annexure 20 TUMOR MARKERS

Annexures

Markers • Human chorionic gonadotropin • Calcitonin • Catecholamines



• Alpha-Fetoprotein



• CEA



• •

Associated Cancers Hormones Trophoblastic tumorsQ, nonseminomatous testicular tumors Medullary carcinomaQ of thyroid PheochromocytomaQ Oncofetal Antigens LiverQ cell cancer, nonseminomatousQ germ cell tumor of testis, lungQ cancer Adenocarcinoma of the colonQ, pancreasQ, lungQ, breastQ, ovaryQ, prostateQ

Isoenzymes • Prostatic acid phosphatase • Prostate cancer • Neuron-specific enolase • Small cell cancer of lungQ, NeuroblastomaQ • Lactate dehydrogenase • Lymphoma, Ewing sarcoma

• Immunoglobulins • PSA and prostate specific membrane antigen

Specific proteins • Multiple myelomaQ and other gammopathies • Prostate cancerQ

Non-neoplastic Conditions • Pregnancy

• Cirrhosis, hepatitis • Pancreatitis, hepatitis, inflammatory bowel disease, smoking • Prostatitis, prostatic hypertrophy • Hepatitis, hemolytic anemia, many others • Infection, MGUS • Prostatitis, prostatic hypertrophyQ

Mucins and other Glycoproteins • Cancer of ovaryQ, fallopian tube, endometriumQ, • PregnancyQ, endometriosisQ, PIDQ, cervix, breastQ, lungQ, pancreasQ and colonQ uterine fibroidsQ • ColonQ cancer, pancreaticQ cancer • Pancreatitis, Ulcerative colitis • Hodgkin’s diseaseQ, anaplastic large cell lymphoma • Hairy cell leukemia, adult T cell leukemia/lymphomaQ

• CA-125 • CA-19-9 • CD30 • CD25

Annexure 21 MOST COMMON • • • • • • • • • • • • •

Indications of Liver Transplantation MC indication for LT: Cirrhosis from Hepatitis C (HCV) Q 2nd MC indication for LT: Alcoholic liver diseaseQ MC indication for LT in children: Biliary atresiaQ MC metabolic disorder requiring LT: Alpha-1 antitrypsin deficiencyQ MC indication for LT following acute liver failure: Acetaminophen toxicityQ

Pediatric Tumors MC malignant tumor of infancy NeuroblastomaQ MC extracranial solid tumor in children MC abdominal malignancy in children MC primary malignant renal tumor of childhood Wilms’ tumorQ MC renal tumor of infancy Congenital mesoblastic nephromaQ MC soft tissue tumor in infants and children RhabdomyosarcomaQ MC solid tumor of childhood Brain tumorQ MC cancer of childhood LeukemiaQ (30%) >Brain tumorsQ (22%) MC cancer in males (PLC): Prostate >Lung >ColorectalQ MC cancer in females (BLC): Breast >Lung >ColorectalQ Cancer deaths in males (LPC): Lung >Prostate >ColorectalQ Cancer deaths in females (LBC): Lung >Breast >ColorectalQ

Annexures

xv

Annexure 22 MISCELLANEOUS • Widest portion of colon: CecumQ • Narrowest portion of colon: SigmoidQ • MC site of colonic rupture caused by distal obstruction: CecumQ • Colon absorbs water, NaClQ; secretes K+, HCO3 and mucusQ • MC site of ischemic colitis: Splenic flexure Sarcomas with Lymph Node Metastasis (MARCES) • Malignant fibrous histiocytomaQ • AngiosarcomaQ • RhabdomyosarcomaQ

• Clear cell sarcomaQ • Epithelial sarcomaQ • Synovial sarcomaQ

Tumors with Spontaneous Regression (NCR MR) • NeuroblastomaQ • ChoriocarcinomaQ • Renal cell carcinomaQ

• Malignant melanomaQ • RetinoblastomaQ

Malignancies associated with Migratory Thrombophlebitis • CA pancreas (MC)Q • CA lungQ • GI malignanciesQ

• Prostate cancerQ • Ovarian cancerQ • LymphomaQ

• Trousseau’s syndrome: Migratory thrombophlebitisQ • Trousseau’s sign: Carpopedal spasm in hypocalcemiaQ • Troisier’s sign: Palpable left supraclavicular LN (Virchow’s node)Q Perineural Spread is seen in

• • • • •

3. CholangiocarcinomaQ 4. Ductal adenocarcinoma of pancreasQ

Small Round Blue Cell Tumors (WEL PNR) Wilms’ tumor • Primitive neuroectodermal tumor Ewing’s sarcoma • Neuroblastoma Lymphoma • Rhabdomyosarcoma Medulloblastoma • Askin tumor Small cell variant of osteosarcoma • Desmoplastic small cell tumor Causes of Postoperative Fever Day

Cause

2–5 days

• Atelectasis of the lungQ

3–5 days

• Superficial and deep wound infectionQ

5 days

• Chest infection including viral respiratory tract infection, UTI and thrombophlebitisQ

> 5 days

• Wound infection, anastomotic leakage, intracavitary collections and abscessesQ Increased Cancer Risk in Obese Patients (PEEL CP GO KBC)

• • • •

ProstateQ EndometrialQ EsophagusQ LiverQ

• • • •

CervixQ PancreasQ GallbladderQ OvarianQ

• • • •

KidneyQ Bile ductQ BreastQ Colon and rectumQ

Annexures

1. Adenoid cystic carcinoma 2. CA GBQ

Q

xvi

Surgery Essence Psammoma Bodies (PSM)

Annexures

1. Papillary carcinoma thyroidQ

2.  Papillary carcinoma (RCC) Q

3.  Serous cystadenomaQ

Proctoscope

10–12 cmQ

Rigid sigmoidoscope

25 cmQ

Flexible sigmoidoscope

60 cmQ

Colonoscope

160 cmQ

• Most radiosensitive ovarian tumor

• DysgerminomaQ

• Most radiosensitive brain tumor

• MedulloblastomaQ

• Most radiosensitive testicular tumor

• SeminomaQ

• Most radiosensitive lung tumor

• Small cell CAQ

• Most radiosensitive kidney tumor

• Wilms tumorQ

• Most radiosensitive bone tumor

• Ewing’s SarcomaQ and Multiple myelomaQ

Condition

Seen in

• Necrolytic erythema migrans

• Glucagonoma

• Erythema chronicum migrans

• Lyme’s disease

• Erythema infectiosum (fifth disease)

• Parvovirus B19

• Erythema marginatum

• Acute rheumatic fever

Screening Immunohistochemistry • Epithelial Markers: Cytokeratin (positive in carcinomas)Q • Lymphoid Markers: CD-45 (positive in lymphoma)Q • Melanocytic Markers: S-100 (positive in melanoma)Q • Mesenchymal Markers: Vimentin (positive in sarcoma)Q • Neuroendocrine Markers: Chromagranin and neuron-specific enolaseQ

Annexure 23 FIRST ORGAN TRANSPLANTATION First kidney transplantation (in identical twins)

• MurrayQ (1954)

First liver transplantation

• StarzlQ (1963)

First pancreas transplantation

• Kelly & LilleheiQ (1966)

First heart transplantation

• Christian BarnardQ (1967)

First lung transplantation

• Fritz DeromQ (1968)

First pancreatic islet cell transplantation

• SutherlandQ (1974)

First heart & lung transplantation

• Reitz & ShumwayQ (1981)

First successful intestinal transplantation

• Deltz (1988)

4.  MeningiomaQ

Annexures

xvii

Annexure 24 SURGICAL POSITIONS Supine position

• MC surgical position, patient lies with back flat on operating room bed

Trendelenburg position

• Same as supine position but the upper torso is loweredQ.

Reverse Trendelenburg position • Same as supine but upper torso is raised & legs are loweredQ. Fracture Table Position

• For hip fracture surgery • Upper torso is in supine position with unaffected leg raised. Affected leg is extended with no lower support. The leg is strapped at the ankle and there is padding in the groin to keep pressure on the leg and hip.

Lithotomy position

• Used for gynecological, anal & urological proceduresQ. • Upper torso is placed in the supine position, legs are raised and secured, arms are extendedQ.

Fowler’s position

• Begins with patient in supine position. Upper torso is slowly raised to a 90o positionQ.

Semi-Fowler's position

• Lower torso is in supine position & upper torso is bent at a nearly 85o position. The patient’s head is secured by a restraint.

Prone position

• Patient lies with stomach on the bed. Abdomen can be raised off the bed.

Jackknife position

• Also called the Kraske positionQ • Patient’s abdomen lies flat on the bed. The bed is scissored so the hip is lifted & legs & head are lowQ.

Knee-chest position

• Similar to the jackknife except the legs are bent at the knee at a 90o angle.

Lateral position

• Also called the side-lying position, it is like the jackknife except the patient is on his or her side. Other similar positions are Lateral chest & Lateral kidney.

Lloyd-Davies position

• Common position for surgical procedures involving the pelvis & lower abdomen. • Majority of colorectal & pelvic surgery is conducted in the Lloyd-Davis positionQ.

Kidney position

• Patient’s abdomen is placed over a lift in the operating table that bends the body to allow access to the retroperitoneal space. • A kidney rest is placed under the patient at the location of the lift.

Sims’ position

• Variation of the left lateral position • Patient will roll to his or her left side. Keeping the left leg straight, the patient will slide the left hip back and bend the right leg. This position allows access to the anusQ. Abdominal Examination Signs Description

Diagnosis

Aaron sign

Pain or pressure in epigastrium or anterior chest with Acute appendicitisQ persistent firm pressure applied to McBurney’s pointQ

Bassler sign

Sharp pain created by compressing appendix between abdominal wall and iliacus

Chronic appendicitis

Blumberg’s sign

Transient abdominal wall rebound tendernessQ

Peritoneal inflammation

Carnett’s sign

Loss of abdominal tenderness when abdominal wall muscles are contracted

Intra-abdominal source of abdominal pain

Chandelier sign

Extreme lower abdominal and pelvic pain with movement of cervix

Pelvic inflammatory disease

Claybrook sign

Accentuation of breath and cardiac sounds through abdominal wall

Ruptured abdominal viscus

Courvoisier’s sign

Palpable gallbladder in presence of painless jaundiceQ Periampullary tumorQ

Cruveilhier sign

Varicose veins at umbilicus (caput medusae)Q

Portal hypertensionQ

Danforth sign

Shoulder pain on inspiration

Hemoperitoneum

Fothergill’s sign

Abdominal wall mass that does not cross midline and remains palpable when rectus contracted

Rectus muscle hematomas

Mannkopf’s sign

Increased pulse when painful abdomen palpated

Absent if malingering

Ransohoff sign

Yellow discoloration of umbilical region

Ten Horn sign

Pain caused by gentle traction of right testicle

Ruptured CBDQ Q

Acute appendicitisQ

Annexures

Sign

xviii

Surgery Essence

Annexure 25

Annexures

IMPORTANT POINTS ABOUT TUMORS Breast

• • • • • • • •

MC type of breast cancer: AdenocarcinomaQ MC subtype of breast cancer: Invasive ductal cancerQ Least common type of breast cancer: PapillaryQ Most malignant type of breast cancer: Inflammatory breast cancerQ Breast cancer associated with best prognosis: TubularQ MC site of breast cancer: Upper outer quadrantQ Least common site of breast cancer: Lower inner quadrantQ MC site of metastasis: Bone (Lumbar vertebra >Femur >Thoracic vertebra)Q

Thyroid

• • • • • •

MC type of thyroid cancer: Papillary > Follicular >Medullary > AnaplasticQ MC site of metastasis from papillary carcinoma: LungsQ MC site of metastasis from follicular carcinoma: BonesQ (Osteolytic secondaries) MC site of metastasis from Medullary carcinoma: LiverQ MC site of metastasis from anaplastic carcinoma: LungsQ MC primary responsible for metastasis to thyroid: CA breast >CA lungQ

Adrenal

• MC adrenal tumor: Non-functioning adenomaQ

Liver

• • • •

Spleen

• MC neoplasm of spleen: Lymphoma (Non-Hodgkin’s lymphoma)Q • MC primary tumor of spleen: HemangiomaQ • MC primary malignant tumor of spleen: AngiosarcomaQ

Gallbladder

• MC site of CA gallbladder: Fundus (60%) > Body (30%) >Neck (10%)Q • Maximum incidence of CA gallbladder: India > PakistanQ • MC histological type of CA gallbladder: Diffuse infiltrative or sclerosingQ

Bile Duct

• MC site of cholangiocarcinoma: Hilum (65%) >Distal (25%) >Intrahepatic (10%)Q • MC histological type of cholangiocarcinoma: Diffuse infiltrative or sclerosingQ

Pancreas

• • • • • •

MC site of carcinoma pancreas: HeadQ MC site of gastrinoma: Duodenum (1st part) > PancreasQ MC site of insulinoma: Equally distributed in head, body & tailQ MC site of glucagonoma & mucinous cystadenoma: Body & tailQ MC site of somatostatinoma, PPoma, serous cystadenoma & IPMN: HeadQ MC site of VIPoma: TailQ

Esophagus

• • • • •

MC type of carcinoma esophagus: SCCQ MC type of carcinoma esophagus in western population: AdenocarcinomaQ MC site of SCC esophagus: Middle 1/3rdQ MC site of Adenocarcinoma esophagus: Lower 1/3rdQ MC site of carcinoma esophagus: Middle 1/3rdQ

Stomach

• MC site of carcinoma stomach, gastric lymphoma: AntrumQ • MC site of carcinoma stomach in pernicious anemia: FundusQ • MC site of diffuse variety of carcinoma stomach: FundusQ

Small intestine

• • • • •

MC malignancy of liver: MetastasisQ MC primary malignancy of liver: HCCQ MC primary malignancy of liver in children: HepatoblastomaQ MC benign tumor of liver: HemangiomaQ

MC tumor of small bowel: Stromal tumor >AdenomaQ MC tumor of small bowel in children: LymphomaQ MC malignant tumor of small bowel: AdenocarcinomaQ >Carcinoid MC site of carcinoid, adenoma, lipoma, lymphoma, leiomyoma: IleumQ MC site of adenocarcinoma: DuodenumQ

Annexures Colon-Rectum

• MC site of colorectal cancer: RectumQ • Least common site of colorectal cancer: Hepatic flexureQ • MC site of colon cancer: SigmoidQ

Appendix

• MC neoplasm of appendix: Carcinoid tumorQ • MC malignant neoplasm of appendix: Mucinous adenocarcinoma > Adenocarcinoma >Carcinoid tumorQ

Anal canal

• MC type of carcinoma anal canal: SCC>BCC>MelanomaQ

Kidney

• MC type of RCC: Clear cell carcinomaQ • MC type of RCC seen in dialysis associated disease: Papillary carcinomaQ • Type of RCC with best prognosis: Chromophobe carcinomaQ

Urinary bladder

• • • •

Prostate

• MC type of carcinoma prostate: Adenocarcinoma >TCCQ • MC site of carcinoma prostate: Peripheral zone (75%) >Transition zone (15%) >Central zone (10%)Q

Penis & Urethra

• • • • •

MC type of carcinoma penis: SCCQ MC site of carcinoma penis: Glans >Prepuce >Shaft (GPS)Q MC site of carcinoma male urethra: Bulbomembranous urethraQ MC type of carcinoma prostatic urethra: TCC >SCCQ MC type of carcinoma penile urethra: SCC >TCCQ

Testis

• • • • • • • • • • •

MC histological type of testicular tumour: SeminomaQ (MixedQ if given in the option) MC bilateral primary testicular tumour: SeminomaQ Most radiosensitive testicular tumor: SeminomaQ MC testicular tumor in infant & children up to 3 years: Yolk sac tumourQ Testicular tumour with best prognosis: Yolk sac tumourQ MC testicular tumor in pre-pubertal children: TeratomaQ MC testicular tumor in patients >60 years: LymphomaQ MC bilateral testicular tumour: LymphomaQ MC secondary testicular tumour: LymphomaQ MC histologic type of testicular lymphoma: DLBLQ Testicular tumour with worst prognosis: Hurricane tumour (Type of choriocarcinoma)Q

Scrotum

• MC benign lesion of scrotum: Sebaceous cystQ • MC malignant tumor of scrotum: SCCQ

MC type of carcinoma bladder: TCC >SCC >AdenocarcinomaQ MC benign mesenchymal tumor of urinary bladder: LeiomyomaQ MC malignant mesenchymal tumor of urinary bladder: LeiomyosarcomaQ MC malignant mesenchymal tumor of urinary bladder in children: RhabdomyosarcomaQ

Adenocarcinoma

• • • •

MC histological typeQ MC in non-smokers, young patients, femalesQ Located peripherallyQ Slow growth & propensity to metastasize to opposite lungQ • Metastasize more frequently to CNSQ • Most cells contain mucinQ • Noguchi classificationQ is used for adenocarcinoma

Squamous Cell Carcinoma

• MC in smokersQ; MC type in IndiaQ • MC variety associated with hypercalcemia (produces PTH-rp)Q • CentralQ in distribution • Prone to undergo central necrosis & cavitationQ • Pancoast tumor is histologically SCCQ • Associated with best prognosisQ

Annexures

Lung

xix

xx

Surgery Essence

Annexures

Lung

Small Cell Carcinoma

• Most malignant, centralQ in distribution, strongly related to smokingQ • Associated with massive hilar or mediastinal lymphadenopathy, mediastinal invasion & perihilar massQ • MC variety associated with paraneoplastic syndrome, hypokalemia & SVC syndromeQ • Most responsive to chemotherapy (cisplatin + etoposide) • Shows response to radiotherapyQ • Hormones produced by small cell carcinoma: ACTH, AVP (vasopressin), calcitonin, ANF, gastrin releasing peptideQ

Large Cell Carcinoma

• Highly undifferentiated with cavitating natureQ • Metastasize early with poor prognosisQ

Heart

• MC cardiac tumor: MetastasisQ • MC primary cardiac tumor: MyxomaQ • MC primary cardiac tumor in infants & children: RhabdomyomaQ

Brain

• • • • • • • •

Oral cavity

• • • •

Salivary gland

• • • • •

Sarcoma

• MC site of GIST: Stomach >Small bowel >Colorectum & esophagusQ • MC soft tissue sarcoma in adults: Liposarcoma >Leiomyosarcoma >Malignant fibrous histiocytomaQ • MC soft tissue sarcoma of extremities: Malignant fibrous histiocytoma > LiposarcomaQ • MC soft tissue sarcoma of retroperitoneum: LiposarcomaQ • MC pediatric soft tissue sarcoma: RhabdomyosarcomaQ

Bone

• MC site of primary for bone metastasis: CA Breast > CA Prostate >RCC >CA Lung > CA Thyroid > CA Bladder (BP increased by RL in TB)Q • MC site of bone metastasis: Thoracic vertebraQ • MC cause of osteoblastic secondaries in males: CA ProstateQ • MC cause of osteolytic secondaries in males: RCCQ • Lytic expansile metastasis is seen in: RCC & follicular carcinoma thyroidQ • MC cause of osteoblastic & osteolytic secondaries in females: CA BreastQ • MC tumor metastasize to bone in females: CA BreastQ

MC brain tumor: MetastasisQ MC primary brain tumor: Meningioma (35%) > Glial tumors (30%)Q MC malignant brain tumor of children: MedulloblastomaQ Most radiosensitive brain tumor: MedulloblastomaQ MC astrocytoma in children: Pilocytic astrocytomaQ MC astrocytoma in adults: Glioblastoma multiformeQ Astrocytoma is supratentorial in adults & infratentorial in childrenQ Maximum incidence of calcification in brain tumor: Craniopharyngioma (most) >Oligodendroglioma (90%) >Meningioma (25%)Q • MC pituitary tumor: AdenomaQ (arising from anterior lobe) MC site of CA oral cavity: Tongue >LipQ MC histological type of CA oral cavity: Squamous cell carcinomaQ MC type of cancer in India: CA oral cavityQ MC site of CA oral cavity in India: Buccal mucosaQ (38%) > Anterior tongue (16%) >Lower alveolus (15.7%) • LN metastasis is most common in: CA tongueQ >Floor of mouth >Lower alveolus >Buccal mucosa >Upper alveolus >Hard palate >LipQ. • Bilateral lymphatic spread is common in: Lower lipQ, supraglottisQ & soft palateQ. MC neoplasm of salivary gland: Pleomorphic adenomaQ MC malignant tumor of salivary gland: Mucoepidermoid carcinomaQ MC neoplasm of salivary gland in children: HemangiomaQ MC malignant tumor of salivary gland in children: Mucoepidermoid carcinomaQ MC malignant tumor of minor salivary glands: Adenoid cystic carcinomaQ

Annexures

xxi

Annexure 26 NAMED OPERATIONS Named Operations

Done for

• Hadfield’s operationQ

Duct ectasiaQ

• Sistrunk operationQ

Thyroglossal cystQ

• Hartley-Dunhill procedureQ

Subtotal thyroidectomyQ (10-12 gm of thyroid remnant is left in the same lobe)

• • • •

Puestow procedureQ Duval procedureQ Beger’s procedureQ Frey’s procedureQ

Chronic PancreatitisQ

• Whipple’s procedureQ • Traverso-Longmire procedureQ

Periampullary carcinomaQ

• Kasai procedureQ

Extrahepatic biliary atresiaQ

• Nissen’s fundoplication

GERDQ

Q

• Heller’s cardiomyotomyQ

Achalasia cardiaQ

• Ivor-Lewis operationQ • Orringer transhiatal esophagectomyQ • Mckeon en-bloc esophagectomyQ

Carcinoma esophagusQ

• • • •

Shoemaker procedureQ Pouchet procedureQ Kelling-Madlener procedureQ Csendes procedureQ

Type IV gastric ulcerQ

• Ramstedt-Fredet pyloromyotomyQ

Infantile hypertrophic pyloric stenosisQ

• Bishop-Koop operationQ

Meconium ileusQ

• Bianchi procedureQ

Short bowel syndromeQ

• Ladd’s operation

Malrotation of gutQ

Q

Hirschsprung’s diseaseQ

• Hartmann’s procedureQ

Carcinoma sigmoid colonQ

• Kocher’s maneuverQ

Mobilization of duodenumQ

• Extended Kocher’s maneuverQ

Right sided medial visceral rotationQ

• Mattox maneuverQ

Left sided medial visceral rotationQ

• Cattell-Braasch maneuver • Mitrofanoff’s procedure • Malone procedure • • • •

Q

Q

Q

Milligan-Morgan open hemorrhoidectomyQ Ferguson closed hemorrhoidectomyQ Whitefield submucosal hemorrhoidectomyQ Longo’s stapler hemorrhoidectomyQ

For extensive retroperitoneal exposureQ AppendicovesicostomyQ AppendicolostomyQ HemorrhoidectomyQ

• Well’s procedureQ • Ripstein procedureQ • Frykman & Goldberg procedureQ

Abdominal rectopexyQ

• Delorme’s mucosectomyQ • Thiersch anal encirclementQ • Altmier’s rectosigmoidectomyQ

Perineal rectopexyQ

• Lord’s procedureQ • Notara’s lateral sphincterostomyQ

Fissure in anoQ

Annexures

• Swenson operation • Duhamel operationQ • Soave operationQ

Q

xxii

Surgery Essence • Bascom procedureQ • Karydakis procedureQ

Pilonidal sinusQ

• Bassini repair, Shouldice repairQ • McVay repairQ • Lichtenstein repairQ

Inguinal herniaQ

• • • •

Lockwood operationQ Lothiessen operationQ McEvedy operationQ Henry procedureQ

Femoral herniaQ

• Dowd’s operationQ

Annexures

• Mayo’s repair

Lumbar herniaQ Umbilical herniaQ

Q

• Rovsing’s operation

Deroofing of cyst in ADPKDQ (Rovsing’s sign: Pain in right lower quadrant during palpation of left lower quadrant in acute appendicitis) (Rovsing’s syndrome: Abdominal pain, nausea & vomiting on hyperextension of spine in horse shoe kidney)

Q

• Anderson Hynes dismembered pyeloplastyQ

PUJ obstructionQ

• Lich-Gregoir technique • Leadbetter-Politano techniqueQ

Methods of ureteric implantation in VURQ

• Boari’s operationQ

Lower ureteric reconstruction with a strip of bladder wall

• Frayer’s suprapubic prostatectomyQ • Millin’s retropubic prostatectomyQ • Young’s perineal prostatectomyQ

Open prostatectomy for BPHQ

Q

• • • • •

Dennis-Brown techniqueQ MAGPIQ Mathiew procedureQ Asopa or Duckett techniqueQ Thiersch-Duplay or Bracka techniqueQ

HypospadiasQ

• • • • •

Winter shuntQ Al-Ghorab shuntQ Quackel or Sacher shuntQ Grayhack shuntQ Barry shuntQ

Surgical management of ischemic priapismQ

• Nesbitt operationQ • • • •

Peyronie’s diseaseQ

Fowler-Stephens orchiopexy Ladd & Gross orchiopexyQ Ombridann’s orchiopexyQ Keetley-Torek orchiopexyQ

Q

• Lord’s plication of sacQ

Small hydroceleQ

• Jaboulay’s eversion of sac • Palomo’s operation

Undescended TestisQ

Q

Medium sized hydroceleQ VaricoceleQ

Q

• Trendelenberg operation

Varicose veinQ

• Gilles, Neibulowitz & Kinmonth procedureQ

Reconstructive operation for lymphedemaQ

• Kontoleons, Homans, Thompson & Charles procedureQ

Excisional operation for lymphedemaQ

• Moh’s micrographic surgeryQ

BCC or SCC involving vital areas, cosmetic areas or recurrent tumorsQ

• COMMANDO’s operationQ

COMbined Mandibulectomy And Neck Dissection Operation for carcinoma tongue fixed to mandible with infiltration of floor of mouthQ.

• Newman & Seabrocks’ operationQ

Parotid fistulaQ

• Millard rotation advancement techniqueQ • Thompson, Le Musurier & Tennison-Randall operationQ

Cleft lip repairQ

Q

Annexures

xxiii

Annexure 27 DIFFUSE LARGE B CELL LYMPHOMA (DLBL) IS MC TYPE OF LYMPHOMA IN • Primary CNS lymphomaQ (in immunocompetent patients) • Orbital lymphomaQ • Thyroid lymphomaQ • Breast lymphomaQ

• • • • •

Gastric lymphomaQ Small intestinal lymphomaQ Appendicular lymphomaQ Colorectal lymphomaQ Testicular lymphomaQ

Annexure 28 TRIANGLES IN SURGERY • Superiorly: Cystic arteryQ • Medially: Common hepatic ductQ • Laterally: Cystic ductQ

Hepatocystic triangle

• Superiorly: Inferior surface of liverQ • Medially: Common hepatic ductQ • Laterally: Cystic ductQ

Femoral triangle

• Superiorly: Inguinal ligamentQ • Medially: Medial border of adductor longus muscleQ • Laterally: Medial border of sartorius muscleQ (Femoral artery pulsations are felt at this siteQ)

Inferior triangle of Petit

• Inferiorly: Iliac crestQ • Medially: Latissimus dorsi muscleQ • Laterally: External oblique muscleQ

Superior triangle of Grynfelt

• Superiorly: 12th ribQ • Medially: Paraspinal musclesQ • Laterally: Internal oblique muscleQ

Scalene triangle

• Anteriorly: Scalenus anticusQ • Posteriorly: Scalenus mediusQ • Inferiorly: First ribQ (Trunk of Brachial plexus & subclavian vessels are compressed at Scalene triangle causing thoracic outlet syndromeQ)

Sherren’s triangle

• Bounded by lines joining anterior superior iliac spine, pubic tubercle & umbilicusQ • Area of skin hyperaesthesia in acute appendicitisQ

Simon’s triangle

• Superiorly: Inferior thyroid arteryQ • Laterally: Common carotid arteryQ • Medially: EsophagusQ (Simon’s triangle aids in identification of recurrent laryngeal nerveQ)

Joll’s triangle

• Laterally: Upper pole of thyroid gland & superior thyroid vesselsQ • Superiorly: Attachment of strap muscles & deep investing layer of fascia to hyoidQ • Medially: MidlineQ • Floor: Cricothyroid muscleQ (External branch of superior laryngeal nerve lies in Joll’s triangleQ)

Annexures

Calot’s triangle

Annexures

xxiv

Surgery Essence Triangle of Auscultation

• Superiorly & medially: Inferior portion of trapezius • Inferiorly: Upper border of latissimus dorsi • Laterally: Medial border of scapula (Only part of back not covered by muscles, respiratory sounds are better heard)

Triangle of Doom

• Bounded laterally by the gonadal vesselsQ • Medially by the vas deferensQ • Apex oriented superiorly at the internal ringQ (Contain external iliac vesselsQ, deep circumflex iliac vein, the femoral nerve & genital branch of the genitofemoral nerve)

Triangle of Hesselbach

• Laterally: Epigastric arteryQ • Medially: Lateral border of rectus abdominisQ where it is attached to pubic crest • Inferiorly: Inguinal ligamentQ

Triangle of Pain

• • • •

Medially: Gonadal vesselsQ Superiorly: iliopubic tractQ Laterally: PeritoneumQ This triangle contains from lateral to medial: −− Lateral femoral cutaneous nerveQ (MC injured nerveQ) −− Anterior femoral cutaneousQ −− Femoral branch of the genitofemoral nerveQ −− Femoral nerveQ

Annexure 29 PREOPERATIVE & PREPROCEDURE MEDICATION INSTRUCTION GUIDELINES (PPAC FORM 5) Preoperative & Preprocedure Medication Instruction Guidelines (PPAC Form 5) Instruct patients to take the following medications with a small sip of water EVEN IF OTHERWISE NPO: Antihypertensive medications

• Continue on the day of the operation or procedureQ

Diuretics

• Continue on the day of the operation or procedureQ

Cardiac medications (digoxin)

• Continue on the day of the procedureQ

Antidepressant, antianxiety & psychiatric medications

• Continue on the day of the operation or procedureQ

Thyroid medications

• Continue on the day of the operation or procedureQ

Birth control pills

• Continue on the day of the operation or procedureQ

Eye drops

• Continue on the day of the operation or procedureQ

Heartburn or reflux medications (Prilosec, Zantac)

• Continue on the day of the operation or procedureQ

Narcotic pain medications

• Continue on the day of the operation or procedureQ

Antiseizure medications

• Continue on the day of the operation or procedureQ

Asthma medications

• Continue on the day of the operation or procedureQ

Steroids (oral & inhaled)

• Continue on the day of the operation or procedureQ

Statins (e.g., Zocor, Lipitor)

• Continue on the day of the operation or procedureQ

Aspirin

• Usually continue; discontinue 7 days before plastic surgery & surgery on retinaQ

COX-2 inhibitors

• Continue on the day of the operation or procedure unless the surgeon specifiesQ (usually concerned about bone healing)

NSAIDs

• Usually continue; discontinue 48 hours before plastic surgery & surgery on retinaQ

Vitamins, iron, Premarin

• Discontinue on the day of the operation or procedureQ

Annexures Topical medications (creams & ointments)

• Discontinue on the day of the operation or procedureQ

Oral hypoglycemic drugs

• Discontinue on the day of the operation or procedureQ

Insulin

• For all patients, discontinue all regular or combination (70/30 preparations) insulin on the day of the operation or procedure.Q • Type 2 diabetics should discontinue all insulins of any type.Q • Type 1 diabetics should take a small amount (usually ⅓) of their usual AM longacting insulin (e.g., Lente or NPH) on the day of the operation or procedure.Q • Type 1 diabetics should not take any short-acting insulin such as regular insulin on the day of the procedure.Q • Patients with an insulin pump should continue their basal rate only.Q

Viagra, Levitra, Cialis or similar drugs

• Discontinue 36 hours before surgeryQ

Warfarin (Coumadin)

• Discontinue 4 days before surgery except for patients undergoing cataract surgery without a bulbar blockQ

Plavix (clopidogrel)

• Discontinue 7 days before surgery except for vascular patients or those undergoing cataract surgeryQ

Herbals & nonvitamin supplements

• Discontinue 7 days before surgeryQ

MAOIs

• Patients taking these antidepressant medications need an anesthesia consultation before surgery (preferably 3 weeks before surgery)Q

xxv

COX, cyclooxygenase; MAOIs, monoamine oxidase inhibitors; NPO, nothing by mouth; NSAIDs, nonsteroidal anti-inflammatory drugs.

Annexure 30 Condition

Most Commonly occurs on

Parathyroid insufficiency

2 –5th dayQ

Duodenal stump blowout

4th–7th dayQ

Bowel anastomotic leak

7th dayQ

Wound dehiscence

5th–8th dayQ

T-tube cholangiogram

7th–10th dayQ

Perforation in typhoid ulcer

3rd weekQ

nd

Location

Hemorrhoids

3, 7 & 11’O clock positionQ

Vascular supply of bile duct

Co-axial 3’O clock & 9’O clock positionQ

TUIP (Transurethral incision of prostate)

Incision at 5 & 7’O clock positionQ

Fissure-in-ano

6’O clock positionQ

Endoscopic sphincterotomy

11’O clock positionQ

Optical internal urethrotomy

12’O clock positionQ

MC position of appendix

12’O clock positionQ (Retrocecal)

Annexures

Condition

xxvi

Surgery Essence

Annexure 31

Annexures

Condition

MC Organism Responsible

Breast abscess Splenic abscess Acute pyelonephritis (hematogenous spread) Carbuncle Hand infections

• Staphylococcus aureusQ

Acute suppurative thyroiditis

• Staphylococcus aureusQ > StreptococcusQ

Pyogenic liver abscess

• In Western countries: E. coliQ • In Asian countries: Klebsiella pneumoniaeQ • In children with chronic granulomatous disease: Staphylococcus aureusQ

Amoebic liver abscess

• Entamoeba histolyticaQ

Hydatid cyst

• Echinococcus granulosusQ

Emphysematous cholecystitis

• Clostridium welchiiQ (anaerobe) > E. coliQ (aerobe)

Emphysematous pyelonephritis Acute pyelonephritis (ascending form) Chronic pyelonephritis Perinephric abscess UTI Acute & chronic bacterial prostatitis Prostatic abscess Infected pancreatic necrosis

• E. coliQ

Xanthogranulomatous pyelonephritis

• ProteusQ

Cholangitis

• E. coliQ > KlebsiellaQ

Peptic ulcer MALT lymphoma

• H. pyloriQ

Spontaneous bacterial peritonitis

• In adults: E. coliQ • In children: Group ‘A’ StreptococciQ

Secondary bacterial peritonitis

• BacteroidesQ (anaerobe) > E. coliQ (aerobe)

Peritonitis in CAPD

• Staphylococcus epidermidisQ

Acute mesenteric lymphadenitis

• Yersinia enterocoliticaQ

Gastrointestinal tuberculosis Genitourinary tuberculosis

• Mycobacterium tuberculosisQ

Appendicular perforation

• BacteroidesQ (anaerobe) > E. coliQ (aerobe)

Anorectal abscess

• E. coliQ > BacteroidesQ

OPSI (Overwhelming post-splenectomy infection)

• Streptococcus pneumoniaeQ

Struvite stone (Staghorn calculi)

• ProteusQ

Schistosomiasis

• Schistosoma hematobiumQ

Acute epididymo-orchitis

• Sexually active male SalmonellaQ

Lymphedema

• Wuchereria bancroftiQ

Burn sepsis

• PseudomonasQ

Cellulitis

• Streptococcus pyogenesQ

Erysipelas

• Beta-hemolytic group ‘A’ StreptococciQ

Gas gangrene

• Clostridium perfringensQ

Chronic burrowing ulcer (Meleney gangrene)

• Microaerophilic non-hemolytic StreptococciQ

Annexures

xxvii

Annexure 32 Etiology

Type of Renal Calculus

• Laxative abuse

Ammonium urateQ

• Thiazide • Ileostomy

Uric acidQ

• • • •

Primary hyperparathyroidism Crohn’s disease Short bowel syndrome Excess intake of spinach, rhubarb, tea, chocolate & pepper

Calcium oxalateQ

• Ethylene glycol

Oxalate stoneQ

• Inflammatory bowel disease

Calcium oxalate > Uric acidQ

• Allopurinol

Xanthine stoneQ

• Renal tubular acidosis

Calcium phosphateQ

Annexures

SECTION

1

Endocrine Surgery CH A PT ERS ˆˆ ˆˆ ˆˆ

Breast Thyroid Parathyroid and Adrenal Glands

CHAPTER

1

Breast

NIPPLE DISCHARGE Nipple Discharge • Unilateral, spontaneous, serous or serosanguinous discharge from a single duct is usually caused by an intraductal papillomaQ, or rarely by an intraductal cancer. • Mostly the underlying cause is a duct papilloma or duct ectasiaQ, but since the chances of malignancy are high, it must be investigated further. • Risk of malignancy increases if an underlying massQ is present.Q Causes of Nipple Discharge Colour

Cause

Blood-stained

• Duct papillomaQ (MC)

•  Intraductal carcinomaQ

•  Duct ectasiaQ

Serous

• Fibrocystic diseaseQ

•  Duct ectasiaQ

• CarcinomaQ

Black, green, paste like or grumous discharge

• Duct ectasia

Q

Investigations • Mammography: Can show underlying suspicious lesions • Cytological examination: (may identify malignant cells, but a negative finding does not rule out cancer) Ductography • • • • •

Primary indication: Nipple discharge (particularly when the fluid contains blood) Radiopaque contrast media is injected into one or more of the major ducts and mammography is performed Intraductal papillomas: Small filling defectsQ surrounded by contrast media Cancers: Irregular masses or as multiple intraluminal filling defectsQ Duct ectasia: Dilated cystic structureQ Q

• Ultrasound: May show presence of an underlying mass or duct ectasia Final Diagnosis • Final diagnosis is made by excising the involved duct (Microdochectomy)Q and any underlying mass if present and subjecting then for a histopathological diagnosis. • Radical duct excision (removal of all lactiferous ducts) is not doneQ. Treatment • Firstly exclude a carcinoma by occult blood test and cytology. • Simple reassurance may then be sufficient but, if the discharge is proving intolerable, an operation to remove the affected duct or ducts can be performed (microdochectomy).

CARCINOMA BREAST: RISK FACTORS Risk Factors for Breast Cancer 1. Age: Incidence increases with ageQ 2. Country of birth: More common in western countriesQ 3. Family history and genetic risk factors (BRCA)Q 4. Hyperestrogenemia: −− Early menarcheQ, late menopauseQ −− NulliparityQ −− ObesityQ 5. Late first full term pregnancyQ

 6. Alcohol and high fat dietQ  7. Personal history of malignancy: −− Contralateral breast cancerQ −− Ovarian and endometrial cancerQ   8. Previous benign breast diseaseQ  9. High socioeconomic statusQ 10. Radiation exposureQ 11. Hormone replacement therapyQ

4

Surgery Essence • Combined (estrogen + progesterone) HRT is associated with increased risk of CA breast.Q • Only estrogen HRT is not associated with increased risk of CA breastQ.

Endocrine Surgery

• SmokingQ and OCPsQ does not appear to increase risk of breast cancer • Longer duration of breast feeding has a protective effectQ BRCA-1 • Chromosome: 17Q • BRCA-1 associated breast cancers: −− Invasive ductal carcinomas −− Poorly differentiatedQ −− Hormone-receptor negativeQ −− Early age of onset −− Bilateral • Associated ovarian, colon and prostate cancersQ.

BRCA-2 • Chromosome: 13Q • BRCA-2 associated cancers: −− Invasive ductal carcinomas −− Well differentiatedQ −− Hormone-receptor positiveQ. −− Early age of onset −− Bilateral • Associated ovarian, colon, prostate, pancreas, gallbladder, stomach cancers and melanomaQ.

Carcinoma Breast Risk Assessment Models

Gail ModelQ • • •

Claus ModelQ

Most frequently used model Incorporates: 1. Age at menarche 2. Number of breast biopsies 3. Age at first live birth 4. Number of first-degree relatives with breast cancer Predicts the cumulative risk of breast cancer according to decade of life

• Based on assumptions about the prevalence of high-penetrance breast cancer susceptibility genes. • Incorporates more information about family history but excludes other risk factors. • Estimates of breast cancer risk according to: decade of life based on presence of 1st and 2nddegree relatives with breast cancer and their age at diagnosis.

• Risk factors that are less consistently associated with breast cancer (diet, use of OCPs, lactation) or are rare in the general population (radiation exposure) are not included in either the Gail or Claus risk assessment model • None of these models accounts for the risk associated with mutations in BRCA-1 & BRCA-2

CARCINOMA IN SITU Ductal Carcinoma in Situ • Although DCIS is predominantly seen in the female breast, it accounts for 5% of male breast cancers. • DCIS carries a high risk for progression to an invasive cancerQ. • DCIS is classified on the basis of nuclear grade & presence of necrosisQ Pathology • Proliferation of epithelium that lines the minor ducts, resulting in papillary growths within the duct lumina. • Papillary growths (papillary growth pattern) eventually coalesce & fill the duct lumina so that only scattered, rounded spaces remain between the clumps of atypical cancer cells, which show hyperchromasia and loss of polarity (cribriform growth pattern). • Eventually pleomorphic cancer cells with frequent mitotic figures obliterate the lumina & distend the ducts (solid growth pattern). • With continued growth, these cells outstrip their blood supply and become necrotic (comedo growth pattern).

Section 1

Histological Types of DCIS • Low Grade: Cribriform, Papillary & MicropapillaryQ

•  High Grade: Solid & ComedocarcinomaQ

Diagnosis • Calcium deposition occurs in the areas of necrosis and is a common feature seen on mammographyQ. • DCIS most frequently presents as mammographic calcificationsQ. Treatment • Non-palpable DCIS: Section by needle localisation technique with specimen mammography to ensure that all visible evidence of cancer is excised • Low grade DCIS (cribriform or papillary subtype 4 cm in diameter or disease in >1 quadrant): MastectomyQ

Breast

Age (years) Incidence Clinical signs Mammographic signs Premenopausal Incidence of synchronous invasive carcinoma Multicentricity Bilaterality Axillary metastasis Subsequent carcinomas: Incidence Laterality Interval to diagnosis Histologic type

• • • • • • • • •

LCIS 44–47 (Early) 2–5% (Less common) None None 2/3Q 5% 60–90%Q 50–70%Q 1%

• • • • • • • • •

DCIS 54–58 (Late)Q 5–10% (More common)Q Mass, pain, nipple discharge MicrocalcificationsQ 1/3 2–46%Q 40–80% 10–20% 1–2%Q

• • • •

25–35% BilateralQ 15–20 yearsQ Ductal

• • • •

25–70%Q Ipsilateral 5–10 years Ductal

Section 1

• • • • • • • • • • • • • •

5

Lobular Carcinoma • LCIS originates from the terminal duct lobular units and develops only in the female breastQ. • LCIS is mostly multicentric & bilateral • Increased risk of invasive carcinoma is in the both breastsQ. Histopathology • Characterized by distention & distortion of terminal duct lobular unitsQ by cancer cells • Cytoplasmic mucoid globulesQ are a distinctive cellular feature. • Histologic hallmark of invasive lobular carcinoma is tendency of tumor cells to invade in linear strands (Indian file pattern)Q Clinical Characteristics Presenting symptom in most cases is breast mass with ill-defined margins Usually presents as an incidental findingQ, on breast biopsy performed for other indication. Average age at diagnosis: 44-47 years; more common in white womenQ Invasive breast cancer develops in 25-35% of women with LCIS, in either breast, regardless of which breast harbored the initial focus of LCIS, and is detected synchronously with LCIS in 5% of cases. • In women with LCIS, up to 65% of subsequent invasive cancers are ductal, not lobular, in origin. • Marker of increased risk for invasive breast cancerQ rather than as an anatomic precursor. • Invasive lobular carcinoma: Different pattern of metastases, propensity to involve peritoneal surface & meningesQ, less likely to metastasize to lungs or bone. Diagnosis • • • •

• Observation/Chemoprevention/Prophylactic bilateral mastectomyQ

CARCINOMA BREAST Breast Cancer • • • •

MC cancer in women in the world and MC cancer in urbanQ women in India MC type is adenocarcinomaQ and most carcinoma arises from terminal duct lobular unitQ MC type of CA breast: Invasive ductal (schirrous) carcinomaQ Least common type of CA breast: PapillaryQ Q

• • • • •

Most malignant type of CA breast: Inflammatory breast cancerQ Best prognosis is seen in: TubularQ MC site of CA breast: Upper outer quadrantQ (leftQ breast >right) Least common site of CA breast: Lower inner quadrantQ MC site of metastasis is BoneQ (Osteolytic deposits in Lumbar vertebra >Femur >Thoracic vertebra >Rib >Skull)

Endocrine Surgery

• Calcifications associated with LCIS typically occur in adjacent tissues (neighborhood calcification)Q • Neighborhood calcification is a unique feature of LCISQ and contributes to its diagnosis. Treatment

6

Surgery Essence • Metastatic disease (Malignant pleural effusionQ) is the principal cause of death from breast cancer. • 2nd MC cause of cancer related death in womenQ (MC is CA lung in both males and females)Q • Pathway of metastasis in breast cancer: Cancer cells from breast → Posterior intercostal veins → Batson veretebral venous plexus → intracranial dural venous sinus → Brain Clinical Features

Endocrine Surgery

• Early breast cancer may be asymptomaticQ Symptoms indicating possibility of breast cancer • Change in size or shape of breastQ •  Single duct discharge, particularly blood stainedQ Q • Skin dimpling, nipple retraction •  Axillary node enlargementQ • Peau-d-orange: Due to obstruction of subdermal lymphatics (lymphatic permeation by tumor cellsQ) leading to cutaneous lymphatic edemaQ • Symptoms indicating possibility of Metastasis: − Breathing difficulty, bone pain, symptoms of hypercalcemiaQ, abdominal distention, jaundice • • • • •

Multifocality: Second cancer in the same quadrant (within 4 cm) Multicentricity: Second cancer outside the quadrant of primary cancer (away at least 4 cm) Dimpling: Small depression over skin of breast due to infiltration of ligament of Cooper by carcinomaQ Puckering: Small fold or wrinkle of skin over the breast due to infiltration of ligament of Cooper by carcinomaQ Cancer en-cuirasse: Infiltration of breast skin & chest wall with multiple nodules & ulceration by the carcinomaQ

Evaluation Triple Assessment • Clinical examinationQ  •  Imaging (USG or mammography)Q  •  Tissue sampling (FNAC or true cut biopsy)Q • Confident diagnosis by triple assessment in 99.9% • First investigation for suspected case of breast cancer: MammographyQ • • • •

•  Best and diagnostic investigation: BiopsyQ

MC cancer in women in India: CA cervixQ MC cancer in urban women in India: CA breastQ MC cancer in women in the world: CA breastQ MC cause of cancer related death in men and women: CA lungQ > CA breast

CARCINOMA BREAST INVESTIGATIONS Investigations in CA Breast FNAC

True-cut (core-cut) Biopsy

• FNA is easily performed, but requires a trained cytopathologistQ for accurate specimen interpretation.

• Core cutting needle biopsy provides a histologic specimen suitable for interpretation by any pathologistQ.

• False-negative resultsQ are most common in fibrotic or well-differentiated tumors.

• ER, PR status and presence of HER-2 overexpression can be routinely determinedQ from core biopsy specimens,

Section 1

• FNA does not reliably distinguish invasive cancer from • Diagnostic technique of choice for patients who will receive DCISQ, potentially leading to the overtreatment of gross DCIS. preoperative systemic therapyQ. Biopsy Techniques for Breast Lesions Technique Advantages Disadvantages FNAC • Rapid, painless, inexpensive. • Does not distinguish invasive from in situ cancerQ. • No incision prior to selection of local therapy • Markers (ER, PR, HER-2) not routinely availableQ. • Requires experienced cytopathologistQ. • False negatives and insufficient specimens occur. True-cut • Rapid, relatively painless, inexpensive. No incision. • False-negative results, incomplete lesion characterization can (core-cut) • Can be read by any pathologistQ, markers occur. Biopsy (ER, PR, HER-2) routinely availableQ. Excisional • False-negative results rare. • Expensive, more painful. biopsy • Complete histology before treatment decisions. • Creates an incision to be incorporated into definitive surgery. • May serve as definitive lumpectomy. • Unnecessary surgery with potential for cosmetic deformity in patients with benign abnormalities.

Breast

7

Ultrasonography in Breast Disease

• Breast cysts • Benign breast masses • Breast cancer

• Smooth margins and echo-free center • Smooth contours, round or oval shapes, weak internal echoes Well-defined anterior and posterior marginsQ. • Irregular wallsQ but may have smooth margins with acoustic enhancement

Section 1

• Initial investigation for palpable lesions in women 2-≤10% 4B: >10-≤50% 4C: >50-≤95% >95% Not applicable

MRI • Screening with MRI is superior to mammography in detecting invasive breast cancer in younger womenQ, where the sensitivity of mammography is low due to presence of mammographically dense breast parenchymaQ Indications for Breast MRI 1. Lobular carcinoma : Difficult to detect and measure by conventional method because of multifocal and infiltrating 2. Staging of primary breast cancerQ 3. Occult primary tumour with malignant axillary lymphadenopathy and normal mammogram and breast USGQ 4. Screen younger women with high familial risk of breast cancerQ 5. Assessing the integrity of breast implantQ Q

Pattern of Calcification in Breast Diseases Carcinoma Microcalcification, punctate, branchingQ Fibroadenoma PopcornQ (coarse, granular, crushed stone) Fibrocystic disease Powdery Fat necrosis Curvilinear

CLASSIFICATION OF BREAST CANCER In-situ Carcinoma • Ductal carcinoma in–situ • Lobular carcinoma in–situ

WHO Classification of Breast Cancer Invasive Carcinoma (MC) • Ductal carcinoma (MC) • Lobular carcinoma • Tubular (Cribiform) carcinoma • Mucinous (Colloid) carcinoma • Medullary carcinoma • Papillary carcinoma • Metaplastic carcinoma • Inflammatory carcinoma

Paget’s Disease of Nipple

Section 1

Invasive Breast Carcinoma Invasive Ductal Carcinoma • Invasive ductal carcinoma of the breast with productive fibrosis (scirrhousQ, simplex, NST) accounts for 80% of breast cancers • Presents with macroscopic or microscopic axillary LN metastases in up to 60%Q of cases. • Usually occurs in perimenopausal or postmenopausal women in the 5th to 6th decades of life • Presents as a solitary, firm massQ. Medullary Carcinoma • Special-type breast cancer, accounts for 4% of all invasive breast cancers • Frequent phenotype of BRCA-1 hereditary breast cancer. • Grossly, the cancer is soft & hemorrhagic. • A rapid increase in size may occur secondary to necrosis and hemorrhage. • On physical examination, it is bulky and often positioned deep within the breast.

Breast

• Better 5-year survival rate than those with NST or invasive lobular carcinoma. Mucinous Carcinoma (Colloid Carcinoma) • Special-type breast cancer, accounts for 2% of all invasive breast cancers • Typically presents in the elderly population as a bulky tumor. • Characterized by extracellular pools of mucin • Analysis of multiple sections is essential to confirm the diagnosis • LN metastases occur in 33% of cases, and 5- and 10-year survival rates are 73 and 59%, respectively.

Section 1

Medullary Carcinoma is Characterized Microscopically by • Dense lymphoreticular infiltrate of lymphocytes and plasma cell • Large pleomorphic nuclei • Sheet-like growth pattern

9

Papillary Carcinoma • Special-type cancer of the breast that accounts for 2% of all invasive breast cancers. • Presents in the 7th decade of life and occurs in a disproportionate number of nonwhite women. • Typically small. • Defined by papillae with fibrovascular stalks and multilayered epithelium. • Low frequency of axillary LN metastases and had 5- and 10-year survival rates similar to those for mucinous and tubular carcinoma. Tubular Carcinoma • Special-type breast cancer and accounts for 2% of all invasive breast cancers. • Usually diagnosed in the perimenopausal or early menopausal periods. • Distant metastases are rare in tubular carcinoma with long-term survival approaches 100%.Q Invasive Lobular Carcinoma • Accounts for 10% of breast cancers. • Invasive lobular carcinoma is frequently multifocal, multicentric, and bilateral.Q

CARCINOMA BREAST STAGING 8th AJCC (2017) TNM Staging for Breast Cancer

T2: Tumor >2 cm & ≤5 cmQ T3: Tumor >5 cmQ T4a: Extension to chest wall, not including pectoralis muscleQ T4b: Edema (including peau d’orange) or ulceration of skin, or satellite skin nodules confined to the same breastQ T4c: Both T4a & T4bQ T4d: Inflammatory carcinomaQ Note • Invasion of dermis alone does not qualify as T4Q. • Chest wall includes ribs, intercostal muscles & serratus anterior but not the pectoral musclesQ. • Dimpling of the skin, nipple retraction, or other skin changes, except those in T4b & T4d, may occur in T1, T2 or T3 without affecting the classificationQ. N: Regional lymph nodes N1: Metastasis to movable ipsilateral level I, II axillary LNsQ N2a: Metastasis in ipsilateral level I, II axillary LNs fixed or mattedQ N2b: Metastasis only in clinically apparent ipsilateral internal mammary LNs and in the absence of clinically evident axillary LNs metastasisQ N3a: Metastasis in ipsilateral infraclavicular LNsQ N3b: Metastasis in ipsilateral internal mammary LNs & axillary LNsQ N3c: Metastasis in ipsilateral supraclavicular LNsQ M: Distant metastases M0: No distant metastasis; M1: Distant metastasis

Endocrine Surgery

T: Primary tumor T1: Tumor ≤2 cmQ

10

Surgery Essence

[Note: Clinically apparent is defined as detected by imaging studies (excluding lymphoscintigraphy) or by clinical examination or grossly visible pathologicallyQ.] Stage I T1 N0M0

Stage IIA T0N1 M0 T1N1 M0 T2 N0M0

Stage IIB T2N1 M0 T3 N0M0

Stage IIIA T0 N2 M0 T1-2 N2 M0 T3 N1-2 M0

Stage IIIB T4 N0-2 M0

Stage IIIC AnyT N3 M0

Stage IV AnyT anyN M1

Endocrine Surgery

Special Conditions in Staging • Positive LN in opposite axilla

• Metastasis

• Two mass in same breast

• Staging according to big mass

• Mass in both breasts

• Separate staging for both breasts

CARCINOMA BREAST MANAGEMENT Breast Cancer Treatment A.  Early Invasive Breast Cancer (Stage I, IIA, IIB): • Mastectomy + Axillary LN status assessmentQ or • BCT + Axillary LN status assessment + RTQ • If sentinel LN can not be identified or found to harbor metastatic disease, axillary LN dissection (Level I+II) should be done Indications of Adjuvant Chemotherapy 1. LN positiveQ 2. Tumor >1 cmQ 3. LN negative, >0.5 cm with adverse prognostic factors: −− Blood vessel or lymph vessel invasionQ −− High nuclear or histologic gradeQ −− Her-2-neu over expressionQ −− Negative hormone receptor statusQ • Tamoxifen should be given for hormone receptor positive, cancer >1 cmQ • Trastazumab should be given for Her-2-neu positive cancerQ B.  Locally Advanced Breast Cancer (Stage IIIA, IIIB, IIIC)Q: • Neoadjuvant chemotherapy + MRM + Adjuvant RTQ • BCT for IIIA with N1 with patients who achieve good response to neoadjuvant chemotherapyQ • Systemic chemotherapy + radiotherapy are indicated in treatment of grossly involved internal mammary nodes (N3b) C.  Distant Metastases (Stage IV): • Prolong survival and improve quality of lifeQ • Hormonal therapies are preferred to cytotoxic therapy as it is associated with minimal toxicityQ

Section 1

Indications of Hormonal Therapy

Indications of Systemic Chemotherapy

1. Hormone receptor positive (ER/PR positive)Q

1. Hormone receptor negativeQ

2. Bone or soft tissue metastases onlyQ

2. Hormone refractoryQ (after 3 endocrine regimens)

3. Limited or asymptomatic visceral metastases

3. Symptomatic visceral metastasesQ

Local Regional Recurrence • Who had mastectomy: Resection of local regional recurrence with reconstruction + chemotherapy + hormonal therapy + RT (if not received RT previously)Q • Who had lumpectomy: Mastectomy with reconstruction + chemotherapy + hormonal therapyQ

Indications of Radiotherapy in Carcinoma Breast • Locally Advanced Breast CancerQ (to decrease recurrence rate) • Margin is positive after mastectomyQ

• After breast conservation surgeryQ • Metastases to 4 or more lymph nodesQ

Breast

11

CHEMOTHERAPY IN CARCINOMA BREAST

• First-generation regimen such as a 6-monthly cycle of cyclophosphamide, methotrexate and 5-fluorouracil (CMF)Q will achieve a 25% reduction in the risk of relapse over a 10- to 15-year periodQ. • CMF is no longer considered adequate adjuvant chemotherapyQ • Modern regimens include an anthracycline (doxorubicin or epirubicin) and taxanes. • Effect of combining hormone & chemotherapy is additive although hormone therapy is started after completion of chemotherapy to reduce side-effects. • Most popular combinations were CMF & CAFQ (Cyclophosphamide, Adriamycin [doxorubicin], and 5-fluorouracil. • In the United States, a combination of Adriamycin (doxorubicin) and cyclophosphamide (AC) or AC plus a taxane (docetaxel, paclitaxel) are likely to be used as polychemotherapyQ. • For HER-2–positive breast cancer, adding trastuzumabQ to polychemotherapy is approved for use as a surgical adjuvant. • Anthracycline-containing combinations are significantly better than no treatment, single-agent treatment, or CMFQ.

Section 1

Chemotherapy in CA Breast

New Drugs in CA Breast Ixabepilone

• Used for antracycline & taxane resistant breast cancerQ

Lapatinib

• Inhibitor of Her-2-neu & EGFR tyrosine kinase • Second line Her-2-neu therapyQ

Sunitinib

• Approved for advanced renal cancer & refractory metastatic breast cancerQ

HORMONE THERAPY IN CARCINOMA BREAST Hormonal Therapy in Carcinoma Breast 1. Ovarian suppression or ablation: −− Bilateral oophorectomyQ −− Medically by LHRH agonist (Goserelin, Leuperolide)Q 2. SERM: Tamoxifen and RaloxifeneQ

4. Anti-estrogens: FulvestrantQ 5. Progestins: Megesterol & Medroxyprogesterone acetate Aromatase Inhibitors • No increased risk of endometrial carcinomaQ • Decreases bone mineral density & increases risk of fractureQ • Used in postmenopausal patientsQ Hormonal Therapy in Carcinoma Breast • Tamoxifen is DOC in premenopausal patientsQ • Aromatase inhibitors are DOC in postmenopausal patients

Tamoxifen • Tamoxifen is a standard hormonal treatment of breast cancer in both premenopausal & postmenopausal womenQ • Tamoxifen is effective in Estrogen Receptor (ER) positive breast carcinoma but some ER negative tumors also respond to tamoxifenQ. • Tamoxifen is approved for primary prophylaxis of breast cancer in high risk womenQ • It reduces the recurrence rate of breast cancer in ipsilateral as well contralateral breastQ • Tamoxifen is associated with reduced risk of cancer in the contralateral breastQ. • Dose: 10 mg BD × 5-yearsQ • While tamoxifen blocks estrogen receptors on the breast, it stimulates these receptors in the uterus (because tamoxifen is a partial against of ER), may lead to endometrial hyperplasia & endometrial cancerQ

Endocrine Surgery

3. Aromatase lnhibitors: −− Non-steroidal: Letrozole and AnastrazoleQ −− Steroidal: ExmestaneQ

12

Surgery Essence Tamoxifen • Potent antagonist in breast carcinoma cells, blood vessels and at some peripheral sitesQ • Partial agonist in the pituitary, bone, uterus and liver (Pit Bul) • Tamoxifen causes retinal deposits, decreased visual acuity & cataracts in occasional patientsQ • Tamoxifen increases the risk of thromboembolic eventsQ Adverse Effects of Tamoxifen • Hot flushes, nausea & vomiting (MC) • Menstrual irregularitiesQ, vaginal bleeding, discharge, pruritus vulvae & dermatitis • Endometrial cancerQ

Section 1

Endocrine Surgery

Q

• ThromboembolismQ • CataractQ • Retinal deposits & decreased visual acuity

LYMPH NODE METASTASIS IN CARCINOMA BREAST Lymphatic Metastasis in CA Breast • Lymphatic spread in CA breast occurs through subareolar lymphatic plexus of Sappey’s lymphatic plexus, cutaneous lymphatics & inflammatory lymphaticsQ. • Lymphatic metastasis occurs primarily to the axillary (75%)Q & internal mammary lymph nodesQ. • Tumors in the posterior one third of breast are more likely to drain to the internal mammary nodesQ. • • • •

Involvement of LNs has both biological & chronological significance. It represents not only an evolutional event in the spread of the carcinoma but is also a marker for the metastatic potentialQ of that tumour. Involvement of supraclavicular nodes and of any contralateral lymph nodes represents advanced diseaseQ. LN metastasis is treated by surgical dissection or radiotherapy

Axillary LN Levels in relation with Pectoralis minor Level

Relation with Pectoralis minor

Axillary LNs Included

I

Below or lateral

Anterior, posterior, lateralQ

II

Posterior (behind)

Central, InterpectoralQ (Rotter’s nodes)

III

Medial or above

ApicalQ

Breast

13

MASTECTOMY

Removal of breast tissue, nipple-areola complex, & skinQ.

Extended simple mastectomy

Simple mastectomy + removal of level I axillary LNs.

Modified radical mastectomy

Removes all breast tissue, nipple-areola complex, skin and level I & II axillary LNsQ.

Halsted’s radical mastectomy

Removes all breast tissue and skin, nipple areola complex, pectoralis major and minor muscles and the level I, II & III axillary LNsQ.

Extended radical mastectomy

Radical mastectomy + Removal of internal mammary LNs

Super radical mastectomy

Radical mastectomy + Removal of internal mammary, mediastinal & supraclavicular LNs

Section 1

Types of Mastectomy Simple or total mastectomy

Variants of MRM Auschincloss Procedure

Removes all breast tissue, nipple-areola complex, skin and level I & II axillary LNsQ.

Patey’s Procedure

Pectoralis minor is removed to allow complete dissection of level III axillary LNsQ

Scanlon’s Modification of Patey’s Procedure

Pectoralis minor is divided instead of removingQ. Division of pectoralis minor allows complete removal of level III axillary LNsQ

COMPLICATIONS OF MASTECTOMY Complications of Mastectomy • Seroma −− MC complicationQ, beneath skin flaps & axilla, occurs in 30% cases −− Catheter is retained until drainage is Axillary LN statusQ • In case of metastasis, the prognosis no more depends upon the lymph node statusQ • In breast carcinoma metastasis, prognosis best depends upon estrogen & progesterone receptor statusQ (ER & PR status)

Breast

15

• Prognostic markers like PCNA, Ki-67, bcl-2, bax:bcl-2, VEGF, HER 2/neu, EGFr expressions are associated with poor prognosisQ.

Section 1

• • • • • • • • •

Prognostic and Predictive Factors for Invasive Breast Cancer Tumor Factors Host Factors Nodal status • Age Tumor size • Menopausal status Histologic/nuclear grade • Family history Lymphatic/vascular invasion • Previous breast cancer Pathologic stage • Immunosuppression Hormone receptor status • Nutrition DNA content (ploidy, S-phase fraction) • Prior chemotherapy Extent of intraductal component • Prior radiation therapy HER-2/neu expression 

Oncotype DX • Oncotype DX is a genomic test that predicts the likelihood of a cancer recurrence, likelihood of benefit from chemotherapy, & likelihood of survival in patients with newly diagnosed breast cancer that has not spread to LNs (node negative) and is hormone receptor-positive. • Oncotype DX evaluates the activity of 21 genes from a sample of the patient’ s cancer to determine the patient’s Recurrence Score. • Recurrence Score ranges from 0 to 100, with a higher score indicating a greater risk of recurrence. • Oncotype DX diagnostic tests help individualize treatment planning for breast, colon and prostate cancer patients. Oncotype DX may be used to guide chemotherapy decisions among certain women with: • Node-negativeQ

• Hormone receptor-positiveQ

• HER2-negative breast cancerQ

Name of Test

Brief Description

Scoring/Measurement

Tissue Needed

Oncotype DX

A genomic test that uses a 21-gene assay to provide an Recurrence Score, a number individual, quantitative assessment of the likelihood of disease between 0 and 100 that correlates recurrence. to a specific likelihood of breast cancer recurrence within 10 years of initial diagnosis

Fixed-tissue blocks

Mamma Print

A unique 70-gene assay that has the ability to identify which early-stage breast cancer patients are at risk of distant recurrence following surgery, independent of Estrogen Receptor status and any prior treatment.

Low risk or high risk

Paraffin embedded or fresh tissue

PAM50

A 50-gene test in development that is designed to be performed in local routine hospital pathology laboratories and has been optimized to separate intrinsic disease subtypes that are used to generate a ROR score.

Risk of Recurrence (ROR) score

Fixed-tissue blocks

GENE EXPRESSION PROFILING IN BREAST CANCER • “Gene expression profiling, which can measure the relative quantities of mRNA for essentially every gene, has identified five major patterns of gene expression in the NST group: luminal A, luminal B, normal, basal-like, and HER2 positive. These molecular classes correlate with prognosis and response to therapy, and thus have taken on clinical importance.”

Endocrine Surgery

For Breast Cancer For Colon Cancer For Prostate Cancer • It helps physicians and patients • For Colon Cancer, it quantifies • For prostate Cancer, it provides to decide on the best course of recurrence risk in stage II and stage a more precise & accurate treatment. III colon cancer, beyond traditional assessment of risk based on qualitative measures. individual tumor biology. • For invasive breast cancer, it predicts chemotherapy benefit • This enables an individualized approach • Provides additional, clinically and likelihood of distant breast to treatment planning. relevant insight into underlying cancer recurrence. prostate tumor biology, enabling • It measures a group of cancer genes physicians & their patients to make • For DCIS patients, it predict the in the tumor, providing a quantitative treatment decisions with greater risk of local recurrence. Recurrence Score so physicians & confidence. patients can have a more complete discussion of recurrence risk.

16

Surgery Essence Luminal Criteria Type

Properties

Luminal A (MC)

ER & PR +ve, Her-2-neu –ve (MC type ; Best prognosisQ)

Luminal B

ER, PR & Her-2-neu +ve (Triple positive)Q

Q

Q

Endocrine Surgery

Normal breast-like Well-differentiated, ER-positive Basal cell type

Triple negative, positive for myoepithelial markersQ (basal keratins, P-cadherin, p63, or laminin), CK-5, 6 & 17, EGFR

Her-2 type

Her-2-neu +ve, ER & PR –veQ (worst prognosis)

Immunohistochemistry in CA Breast • Most widely used test for ER & PR receptor statusQ • Immunohistochemistry analysis of heat-treated paraffin sections (of tumor tissue) has largely superseded ELISA ligand-binding assay. • ER and PR positive status (>10 fmol on ELISA; >15 H-score on Immunohistochemistry)Q predict improved response to endocrine treatment, time to relapse and overall survivalQ. Heat Map • Portrayal of global gene expressionQ is called heat map • This illustration provides an unbiased look at breast cancer according to gene expressionQ

TRIPLE-NEGATIVE BREAST CANCER (TNBC) Triple-negative Breast Cancer (TNBC) • TNBC: Breast cancer that does not express the genes for ER, PR & Her-2-neuQ • Accounts for 15–25% of breast cancer cases. • More common in premenopausal womenQ Pathology • Germline mutations of BRCA1 & BRCA2 genesQ are the causative factor • Also known as basal-likeQ (75% of basal-type breast cancers are triple negative) • Some TNBC overexpresses EGFR & transmembrane glycoprotein NMBQ (GPNMB). Treatment • Standard treatment: Surgery (Mastectomy/BCS) + Adjuvant chemotherapy + RadiotherapyQ • Didox (synthetic antioxidant) in addition to chemotherapy reduces drug resistanceQ. • Didox inhibits ribonucleotide reductase M2 (RRM2)Q • RRM2 contributes to cells resistance of the chemotherapy resulting in relapseQ. • TNBCs are very susceptible to chemotherapyQ. • BRCA1-related TNBC is particularly susceptible to platinum-based agents & taxanesQ. Prognosis • High risk of recurrenceQ after treatment

BREAST RECONSTRUCTION

Section 1

Breast Reconstruction • • • • • •

Autogenous

Alloplastic

TRAM flap (MC) Lattisimus dorsi flapQ Gluteal flap Ruben’s flapQ Thoracoepigastric flap Lateral thigh flap

• Silicone gel implant • Silicone implant with saline refillQ

Q

Combined Q

• Lattisimus dorsi flap with implantQ • TRAM flap with implantQ

• Placement of implant in a submuscular plane beneath pectoralis majorQ, superior portion of rectus abdominis, & serratus anterior muscles provides better protection against implant extrusion, as well as decreased risk for capsular contracture & implant displacementQ

Breast

Tissue expander

Latissimus dorsi flap TRAM flap

Common Reconstructive Options after Mastectomy Advantages Disadvantages One stage procedure, minimal prolongation, Poor symmetryQ if skin removed or in large ptotic breasts. hospitalization, or recovery, low cost Capsular contracture, leakage, ruptureQ possible. Short operative time, hospitalization, recovery not Multiple physician visits post-op. prolonged, low cost Poor symmetry large or ptotic breasts. Capsular contracture, leakage ruptureQ possible. Short operative time, hospitalization, recovery not Donor site scarQ prolonged, low cost Usually requires an implantQ Moderate prolongation hospitalization and recovery. Natural contour. Good match for large or ptotic breasts. Donor site scarQ Abdominoplasty. Fat necrosis, flap loss possible. Abdominal wall weakness and herniaQ. Significant prolongation hospitalization plus recovery.

Section 1

Type Implant

17

• MC method of breast reconstruction: Implants (silicon implants)Q • Surgical breast reconstruction should never done prior to RTQ. • Best flap for breast reconstruction: DIEP flapQ (Deep inferior epigastric perforator flap) > TRAM FLAPQ

INFLAMMATORY BREAST CARCINOMA (MASTITIS CARCINOMATOSA) • IBC (stage IIIB) accounts for 33% of skin of breastQ. Pathology • Permeation of the dermal lymph vessels by cancer cells is seen in skin biopsy specimensQ. • There may be an associated breast massQ.

Diagnosis • IOC for diagnosis is skin biopsyQ. Treatment • Multimodal approach (NACT + Mastectomy + RT ± Hormonal therapy) Q • Chest wall, supraclavicular, internal mammary and axillary lymph node basins receive adjuvant radiation therapy. • This multimodal approach results in 5-year survival rates that approach 30%Q. • Both inflammatory breast cancer & Paget’s disease may or may not be associated with breast mass.

BREAST CANCER DURING PREGNANCY Breast Cancer during Pregnancy • Occurs in 1 of every 3000 pregnant women • MC non-gynecologic malignancy associated with pregnancyQ. • Ductal carcinoma is MC type, accounting for 75–90%Q of breast cancer in pregnancy. Q

Clinical Features • Presents as painless palpable massQ with or without nipple discharge • Axillary LN metastases in up to 75% patients • Approx. 97%) patients with Paget’s disease have an underlying ductal carcinomaQ (in situ or invasive) • Paget’s disease may (54%) or may not (46%) be accompanied by a massQ • Invasive breast cancer coexists with Paget’s disease in 93% of patients with mass and in 38% of patients without massQ Diagnosis • Complete mammography and biopsy is required to rule out occult multicentric disease • Biopsy showing Paget cell is diagnosticQ Treatment • Most commonly utilized procedure is simple mastectomyQ • Wide excision of nipple and areola to achieve clear ,margins + Radiotherapy + Axillary staging • Lumpectomy + Radiotherapy + Axillary LN dissection

CARCINOMA OF MALE BREAST Carcinoma of Male Breast • Peak in 6th decadeQ of life, accounts for less than 1% of all cases of breast cancer. • MC Type: Infiltrating ductal carcinomaQ.

Section 1

• Male breast cancer is preceded by gynecomastia in 20% of menQ. • Hormone receptor positive: 80%; Her-2-neu positive: 35% • Lobular carcinoma (both in-situ and invasive) is rarely seen due to absence of lobules in males. Predisposing Factors • Excess endogenous or exogenous estrogen (Testicular disease, infertility, obesity, cirrhosis)Q • Radiation therapy, Klinefelter’s syndrome and testicular feminizing syndromesQ. • BRCA2 mutationsQ • Gynecomastia is not a risk factor for carcinoma male breastQ. Clinical Features • MC presentation is lumpQ. • Local pain, axillary adenopathy, nipple retraction, ulceration, bleeding, & discharge.

Breast

19

• Breast cancer in men more commonly involves the pectoralis major muscleQ due to scanty breast tissue.

Treatment • Treatment of male breast cancer is surgical (Most common procedure: MRM) Q • Adjuvant radiation therapy is in high risk cases for local-regional recurrence. • Eighty per cent of male breast cancers are hormone receptor positive, and adjuvant tamoxifen is considered. Prognosis • Stage >Lymph node status is the best prognostic indicatorQ as in female breast carcinoma. • Stage by stage prognosis is same as female CA breastQ

Section 1

Diagnosis • Evaluation includes breast imaging studies and diagnostic needle or surgical biopsy.

CYSTOSARCOMA PHYLLODES Phyllodes Tumors • • • •

Tumors of mixed connective tissue & epithelium (biphasic proliferation of stroma & mammary epithelium) Q Also known as serocystic disease of BrodieQ Classified as benign, borderline, or malignant. Borderline tumors have a greater potential for local recurrence.

Pathology • • • • •

Sharply demarcated from surrounding breast tissue, which is compressed & distorted. Connective tissue composes the bulk of these tumors, which have mixed gelatinous, solid & cystic areas. Cystic areas represent sites of infarction & necrosis. Gross cut tumor surface: Classical leaf-like (phyllodes) appearanceQ. Stroma of a phyllodes tumor has greater cellular activity than fibroadenoma. • Stromal cells of fibroadenomas • Either polyclonal or monoclonalQ • Stromal cells of phyllodes tumors • Always monoclonalQ

Clinical Features Smooth, rounded, usually painless multinodular lesions Average age: 4th decade. Large, mostly massive size but always mobile over chest wallQ Bosselated surface with pressure necrosis of overlying skinQ Diagnosis is suggested by larger size, a history of rapid growthQ, and occurrence in older patients. • Differentiated from carcinoma by: No fixity to skin and pectoralis, no nipple retraction, no LN involvementQ • • • • •

Diagnosis • Mammographic evidence of calcifications & morphologic evidence of necrosis do not distinguish between benign, borderline, and malignant phyllodes tumors. • Ultrasound: Discrete structure with cystic spaces • Diagnosis is best made by biopsyQ Treatment • Small phyllodes tumors • Large phyllodes tumors • Phyllodes tumor with suspicious malignant elements

• Wide local excisionQ • MastectomyQ • Re-excision of biopsy site to ensure complete excision of tumor with a 1 cm margin

• Axillary dissection is not recommended because axillary LN metastases rarely occurQ. • Metastases from malignant phyllodes tumors occur via hematogenous spread, with common sites including lung, bone, abdominal viscera and mediastinum.

Endocrine Surgery

• Most malignant phyllodes tumors contain liposarcomatous or rhabdomyosarcomatous elements rather than fibrosarcomatous elementsQ. • Evaluation of number of mitoses & presence or absence of invasive foci at the tumor margins may help to identify a malignant tumorQ.

20

Surgery Essence BREAST LEIOMYOSARCOMA

Endocrine Surgery

Breast Leiomyosarcoma • Leiomyosarcomas are malignant tumorsQ composed of cells showing smooth muscle features • Locally aggressive tumor and hematogenous metastasisQ • Palpable axillary lymphadenopathy is uncommonQ and when encountered usually represents reactive LNs uninvolved by metastatic diseaseQ • A clinically negative axilla in presence of large tumor may be indicative of breast sarcoma • Tumor is well-circumscribed or encapsulatedQ Treatment • Simple mastectomy is gold standardQ of treatment • Axillary lymph node dissection is not indicatedQ • Postoperative radiation may be used in patient undergoing mastectomy because chest wall recurrence are observed

BENIGN DISORDERS OF BREAST Aberrations of Normal Development and Involution • ANDI classification encompasses all aspects of breast condition, including pathogenesis and degree of abnormality. Early Reproductive Years • Fibroadenomas in younger women aged 15–25 years • Nipple inversion is a disorder of development of the major ducts, which prevents normal protrusion of the nipple. • Mammary duct fistulas arise when nipple inversion predisposes to major duct obstruction, leading to recurrent subareolar abscess and mammary duct fistula. Later Reproductive Years • Cyclical mastalgia and nodularity usually are associated with premenstrual enlargement of the breast and are regarded as normal. • In epithelial hyperplasia of pregnancy, papillary projections sometimes give rise to bilateral bloody nipple discharge. Involution Macrocysts are common, are often subclinical, and do not require specific treatment. Sclerosing adenosis is considered a disorder of both the proliferative and the involutional phases of the breast cycle. Duct ectasia (dilated ducts) and periductal mastitis are other important components. Sixty per cent of women 70 years of age exhibit some degree of epithelial hyperplasia. Atypical proliferative diseases include ductal and lobular hyperplasia, both of which display some features of carcinoma in situ. • Women with atypical ductal or lobular hyperplasia have a fourfold increase in breast cancer risk. • • • • •

Section 1

ANDI Classification of Benign Breast Disorders Normal

Disorder

Disease

Early reproductive years (age 15–25 years) 

Lobular development Stromal development Nipple eversion

FibroadenomaQ Adolescent hypertrophy Nipple inversion

Giant fibroadenoma Gigantomastia Subareolar abscess Mammary duct fistula

Later reproductive years (age 25–40 years) 

Cyclical changes of menstruation Epithelial hyperplasia of pregnancy

Cyclical mastalgiaQ Nodularity Bloody nipple discharge

Incapacitating mastalgiaQ

Involution (age 35–55 years)

Lobular involution Duct involution Dilatation Sclerosis Epithelial turnover

Macrocysts Sclerosing lesions Duct ectasia Nipple retraction Epithelial hyperplasia

Periductal mastitis Epithelial hyperplasia with atypiaQ

Breast

21

Fibroadenoma

Pathology • Encapsulated spherical lesion, composed of fibrous and glandular tissueQ • Arise from interlobular stroma, stromal cells can be monoclonal or polyclonal Types • Pericanalicular (Hard): Due to proliferation of connective tissue inside the elastic lamina • Intracanalicular (Soft): Due to proliferation of connective tissue outside the elastic lamina Clinical Feature • Painless, slowly growing solitary mobile lump in the breast (Breast mouse) Q

Section 1

• MC benign tumor of female breastQ; •  MC age group: 15–30 yearsQ; •  Known as breast mouseQ • Etiology: Increased sensitivity of focal areas of breast tissue to estrogenQ

Diagnosis • IOC is FNACQ (Antler horn configurationQ of ductal epidermal cells) • Characteristic popcorn calcificationQ on mammography Treatment • No treatmentQ is necessary when diagnosis is confirmed. • Excision biopsy is the treatment of choice for suspicious lesionQ and for cosmetic indications.

Breast Cyst • Occur most commonly in the last decade of reproductive lifeQ as a result of a non-integrated involution of stroma and epithelium. Clinical Features • Often multiple, may be bilateralQ and can mimic malignancy. • Typically present suddenly and cause great alarm; prompt diagnosis and drainage provides immediate reliefQ. Diagnosis • Diagnosis can be confirmed by aspiration and/or ultrasoundQ. Treatment • Aspiration for Solitary cyst: If they resolve completely, and if the fluid is not blood-stained, no further treatment is required (30% will recur and require reaspiration) Q • Core biopsy or local excisionQ: If there is a residual lump or if the fluid is blood-stained, for histological diagnosis (exclude cystadenocarcinoma, which is more common in elderly women)

• Secretion of milk looking discharge from one or both breasts unrelated to pregnancyQ is called galactorrhea. • Physiological galactorrhea is the continued production of milk after lactation has ceased and menses resumed and is often caused by continued mechanical stimulation of the nipple. • In both men and womenQ, galactorrhea may vary in colour and consistency. • Galactorrhea is commonly associated with prolactinomaQ. Treatment • Treatment is aimed at normalizing prolactin levelQ. • BromocriptineQ (dopamine agonist) is the drug of choice. • Surgery is considered when there is failure of medical therapyQ (Trans-nasal trans-sphenoidal excision of pituitary adenoma is doneQ)

Amazia • Congenital absence of the breastQ may occur on one or both sides. • It is sometimes associated with absence of the sternal portion of the pectoralis major (Poland’s syndrome) Q. • It is more common in malesQ.

Mondor’s Disease • A variant of thrombophlebitis involving the superficial veins of the anterior chest wallQ and breast. • Also known as “string phlebitis,” a thrombosed vein presenting as a tender, cord-like structureQ. • Frequently involved veins: Lateral thoracic vein, thoracoepigastric vein, superficial epigastric vein. • Benign, self-limited disorder. Clinical Features • Acute pain in the lateral aspect of the breast or the anterior chest wallQ. • A tender, firm cord is found to follow the distribution of one of the major superficial veins.

Endocrine Surgery

Galactorrhea

22

Surgery Essence Diagnosis • When the diagnosis is uncertain, or when a mass is present near the tender cord, biopsy is indicated. Treatment • NSAIDs and application of warm compressesQ along the symptomatic vein with restriction of motion and brassiere support of the breast • Usually resolves within 4–6 weeks. • When symptoms persist or are refractory to therapy, excision of the involved vein segment

Endocrine Surgery

Duct Papilloma • Intraductal pailloma are true polyps of epithelium lined breast ducts. • BenignQ lesions (not pre-cancerous) • Mostly solitaryQ, located under the areola (within 4-5 cm of nipple orifice) • Generally 1 year) Q 2. Continued growthQ 3. Psychological or cosmetic problemQ

4. TendernessQ 5. Suspected malignancyQ

• Surgical Procedures: Mastectomy, subtotal mastectomy, subcutaneous mastectomy, reduction mammoplastyQ. Suction Assisted Lipectomy • Removes only adipose tissue , if performed as a sole method Q

• Performed as an adjunct surgical procedureQ • Use is limited in cases that are severe or in fibrous breasts • Reduces overall breast size and may result improved appearance, but it does not remove the glandular tissue (does not correct gynecomastia) Q • Most cases of gynecomastia are amenable to simple liposuction with minimal glandular resection through periareolar incision as necessaryQ.

Multiple Choice Questions NIPPLE DISCHARGE



1. Blood stained nipple discharge is seen in:  (Recent Question 2017, DNB 2013, 2011, Orissa 2011, PGI June 2009, UPPG 2010, AIIMS Nov 2003, All India 2005)

a. Breast abscess c. Ductal papilloma



b. Fibroadenoma d. Fat necrosis of breast

2. Green discharge is most commonly seen with: (Recent Question 2016, Kerala PG 2015, WBPG 2015, AIIMS Nov 98) a. Duct papilloma b. Duct ectasia c. Retention cyst d. Fibroadenosis



3. A 25 years old female complains of discharge of blood from a single duct in her breast. The most appropriate treatment is: (All India 2008) a. Radical excision b. Microdochectomy c. Radical mastectomy d. Biopsy to rule out carcinoma



4. True statement (s) about nipple discharge is/are: a. Mammography (PGI June 2004) b. Cone excision done in single intraductal tumour c. Mammography done when duct papilloma is liver >brain. Diagnosis • Diagnosis is established by FNAC of the thyroid mass or lymph nodeQ. • Once thyroid cancer is diagnosed on FNAC, a complete neck ultrasound to evaluate the contralateral lobe and for LN metastases in the central & lateral neck compartments. Treatment • Total or near-total thyroidectomyQ • During thyroidectomy, enlarged central neck nodes should be removedQ. • Biopsy-proven lymph node metastases detected clinically or by imaging in the lateral neck in patients with papillary carcinoma are managed with modified radical neck dissection. Prognosis • PTC have an excellent prognosis with a >95% 10-year survival rateQ.

FOLLICULAR CARCINOMA THYROID Follicular Carcinoma of Thyroid

Section 1

• • • •

FTC account for 10% of thyroid cancers Occurs more commonly in iodine-deficient areasQ. More common in women with mean age of 50 years Genes implicated in FCT: p53Q, PTENQ, RasQ , PAX8/PPAR1

Pathology • Usually solitary lesion surrounded by capsuleQ. • Histologically, follicles are present, but the lumen may be devoid of colloidQ. • Malignancy is defined by the presence of capsular and vascular invasionQ. • Tumor infiltration and invasion, as well as tumor thrombus within the middle thyroid or jugular veins, may be apparent at operation. Clinical Features • Usually present as solitary thyroid nodules, occasionally with a history of rapid size increase, and long-standing goiterQ. • Pain is uncommon, unless hemorrhage into the nodule has occurred. • Cervical lymphadenopathy is uncommon at initial presentation (about 5%)

Thyroid

• MC site of metastasis is bone (Osteolytic metastasis with pulsating secondaries in flat bones)Q • MC site of metastasis: VertebraQ >Ribs >Pelvis Bones >Skull Diagnosis • FNAC is unable to distinguish benign follicular lesions from follicular carcinomasQ. • Intraoperative frozen-section examination usually is not helpful, but should be performed when there is evidence of capsular or vascular invasion, or when adjacent lymphadenopathy is present. Treatment • • • •

Follicular lesion: HemithyroidectomyQ (80% of these patients will have benign adenomas) Thyroid cancer: Total thyroidectomyQ Total thyroidectomy in older patients with follicular lesions >4 cm because of the higher risk of cancer in this setting (50%)Q. Prophylactic nodal dissection is unwarrantedQ because nodal involvement is infrequent.

Section 1

• Preoperative clinical diagnosis of cancer is difficult unless distant metastases are present. • Large follicular tumors (>4 cm) in older men are more likely to be malignantQ.

61

Prognosis • Cumulative mortality: 15% at 10 years and 30% at 20 years. • Most important prognostic factor: Age and distant metastasis. Poor Long-term Prognosis • Age >50 years • Tumor size >4 cmQ • Higher tumor gradeQ

• Marked vascular invasionQ • Extrathyroidal invasionQ • Distant metastasesQ

Q

HÜRTHLE CELL CARCINOMA Hürthle Cell Carcinoma • Account for 3% of all thyroid malignancies • Considered to be a subtype of follicular thyroid cancerQ • More often multifocal and bilateral (about 30%)Q • Usually do not take up RAI (about 5%)Q • More likely to metastasize to local nodes (25%) and distant sitesQ • Associated with a higher mortality rateQ (about 20% at 10 years) Pathology • Characterized by vascular or capsular invasion and can’t be diagnosed by FNACQ. • Tumors contain sheets of eosinophilic cells packed with mitochondria, which are derived from the oxyphilic cells of the thyroid gland. Treatment • Unilateral Hürthle cell adenomas: HemithyroidectomyQ • Invasive Hürthle cell neoplasms: Total thyroidectomy + Routine central neck node removalQ (MRND when lateral neck nodes are palpable) • Retinoic acid, PPAR agonists have shown some utility in treating these tumors in vitro

MEDULLARY CARCINOMA THYROID Medullary Carcinoma Thyroid • • • • • •

Neuroendocrine carcinoma arising from parafollicular ‘C’ cellsQ of thyroid Parafollicular ‘C’ cells are derived from the ultimobranchial bodiesQ & secrete calcitoninQ ‘C’ Cells are concentrated superolaterally in thyroid lobes, from where MTC usually develops Most MTCs (75–80%) arise sporadicallyQ Spread is both lymphatic & hematogenousQ MC site of metastasis: LiverQ

Endocrine Surgery

Hürthle Cell Tumors Differ from Follicular Carcinomas in

62

Surgery Essence Medullary Carcinoma Thyroid Familial: 20%Q (Non-MEN setting/ MEN-2A/MEN-2B) • Originate in one lobeQ • Multicentric and bilateralQ • Seen in 6th decade • Occur in younger ageQ Q • RET protoncogene mutation • Associted with C-cell hyperplasiaQ • RET protoncogeneQ mutation Sporadic: 80%Q

Endocrine Surgery

Clinical Features Medullary Carcinoma Should Be Suspected • High level of serum CalcitoninQ & CEAQ • Cervical lymph nodes at time of presentation (LN involvement, thyroid and blood borne metastases occurs early)Q • DiarrheaQ at the time of presentation. • Amyloid Q in stroma histologically. • MEN setting: Evidence of Pheochromocytoma/Hyperparathyroidism/Thyroid cancer in family. • (Discovery of medullary carcinoma thyroid makes family surveillance advisable)Q Diagnosis • • • •

Diagnosed by FNACQ I131 scan is of no use as MTC is TSH independentQ. Tumor marker: Calcitonin is raised in almost all cases of MTC Calcitonin excess in MTC is not associated with hypocalcemia

Treatment • Total thyroidectomy + Central LN dissection ± Ipsilateral MRND if tumor >1 cmQ • If nodes are positive on ipsilateral side: Bilateral MRND • Vandetanib (EGFR inhibitor) is the only drug approved by US FDA for treatment of advanced & progressive MTC Follow-up • Level of Calcitonin falls after resection and is raises again in cases of recurrence, used for follow upQ. Prognosis • MTC is associated with poor prognosisQ.

ANAPLASTIC CARCINOMA Anaplastic Carcinoma • • • •

Accounts for 1% of all thyroid malignancies Mainly affect women in 7th and 8th decadeQ The typical patient has a long-standing neck mass, which rapidly enlarges and may be painfulQ. Most aggressive form of thyroid cancerQ

Pathology • Grossly: Firm & whitish in appearance. • Microscopically, sheets of cells with marked heterogeneity & characteristic giant & multinucleated cellsQ.

Section 1

Clinical Features • • • •

Typical manifestation: An older patient with dysphagia, cervical tenderness & a painful, rapidly enlarging neck massQ. Superior vena cava syndrome can also be part of the findings. The clinical situation deteriorates rapidly into tracheal obstruction & rapid local invasionQ of surrounding structures. Associated symptoms: Dysphonia, dysphagia & dyspnea • Lymph nodes usually are palpable at presentation. • Evidence of metastatic spread also may be present. • MC site of metastasis: LungsQ

Diagnosis • Confirmed by FNAC revealing characteristic giant & multinucleated cellsQ. • Incisional biopsy occasionally is needed to confirm the diagnosis

Thyroid

63

Treatment

• Most aggressive thyroid malignanciesQ, with CA Lung • If thyroid metastases is detected pre-mortem, MC site of primary: RCCQ > CA Breast > CA Lung

THYROID LYMPHOMA Thyroid Lymphoma

• PainlessQ and associated with feverQ • Patients with thyroid lymphoma virtually never have hyperthyroidism but frequently have hypothyroidismQ. • Hypothyroid patients have evidence of autoimmune thyroiditis or Hashimoto’s thyroiditisQ. Diagnosis • Diagnosis is confirmed by core-needle biopsyQ. Treatment: External Beam Radiotherapy + ChemotherapyQ • Patients with thyroid lymphoma respond rapidly to chemotherapy (CHOP—cyclophosphamide, doxorubicin, vincristine, and prednisone) and associated with improved survival. • Combined treatment with radiotherapy & chemotherapy often is recommended. • To alleviate pressure symptoms, surgical resection (Thyroidectomy and nodal dissection) is recommended.

THYROIDECTOMY Steps of Thyroidectomy • A Kocher transverse collar incision, typically 4 to 5 cm in length, is placed in or parallel to a natural skin crease 1 cm below cricoid cartilage. • Subcutaneous tissues & platysma are incised sharply and subplatysmal flaps are raised superiorly to the level of thyroid cartilage and inferiorly to the suprasternal notchQ • Strap muscles are divided in the midline along the entire length of mobilized flaps, & thyroid gland is exposed. • Middle thyroid veins are ligated and dividedQ.

Endocrine Surgery

• MC type is NHL B cellQ type, of intermediate grade. • Majority of patients have thyroid disease plus cervical or mediastinal lymph nodesQ. • More common in females. • Most thyroid lymphomas develop in patients with Chronic Lymphocytic ThyroiditisQ Clinical Features • Lymphomas are rapidly growing tumours, present with rapidly enlarging neck mass which is often painless. • Patients may present with acute respiratory distress & dysphagia • About 10–30% present with symptoms relating to local invasion, including hoarseness, dyspnoea with stridor, or dysphagia.

64

Surgery Essence • Dissection plane is kept as close to the thyroid as possible & superior pole vessels are individually identified, skeletonized, ligated, & divided low on the thyroid gland to avoid injury to external branch of superior laryngeal nerveQ. • RLNs can be most consistently identified at the level of cricoid cartilage. • Parathyroids usually can be identified within 1 cm of the crossing of the inferior thyroid artery and the RLNQ

Endocrine Surgery

• Inferior thyroid vessels are dissected, skeletonized, ligated, and divided as close to the surface of thyroid gland as possible to minimize devascularization of the parathyroids (extracapsularQ dissection) or injury to the RLNQ. • RLN is most vulnerable to injury in the vicinity of the ligament of Berry. Any bleeding in this area should be controlled with gentle pressure before carefully identifying the vessel & ligating it. Use of the electrocautery should be avoided in proximity to the RLNQ. • Once the ligament is divided, the thyroid can be separated from the underlying trachea by sharp dissection. • Parathyroid glands that have been inadvertently removed during the thyroidectomy should be resected, confirmed as parathyroid tissue by frozen section, divided into 1-mm fragments, and reimplanted into individual pockets in the sternocleidomastoidQ muscle. The sites should be marked with silk sutures and a clipQ.

COMPLICATIONS OF THYROIDECTOMY Complications of Thyroidectomy • Hemorrhage −− Due to slipping of ligature on the superior thyroid arteryQ, bleeding from muscular artery −− Hematomas may cause airway compromise and must be evacuated immediatelyQ. −− Hematomas may occur immediately or later on. −− An immediate bleed occurs after or shortly before extubation when the patient lightens from anaesthesia and may begin to cough, causing a vessel to open. • Respiratory obstruction: Causes include −− Tension hematomaQ −− Laryngeal edema (by anesthetic intubation): MC cause of respiratory obstructionQ −− Bilateral recurrent laryngeal nerve paralysisQ • Recurrent laryngeal nerve paralysis −− May be unilateral or bilateral, transient or permanent. −− Bilateral paralysis causes respiratory obstruction - Dyspnea, stridor. • Injury to other nerves −− External branches of superior laryngeal nerveQ (MC injured nerve during thyroid surgery: External laryngeal nerveQ) −− Cervical sympathetic trunk - may cause Horner’s syndrome. • Parathyroid insufficiency −− Due to removal of the parathyroid glands or infarction due to vascular injuryQ. −− Vascular injuryQ is more important. −− Cases usually present 2–5 days after operationQ with symptoms of hypocalcemia (circumoral and fingertip numbness and tingling tetany, carpopedal spasm and laryngeal stridor)Q −− Treatment with oral calcium & vitamin D supplementsQ −− IV calcium gluconateQ may be required in severe cases.

Section 1

• Thyroid insufficiency • Thyrotoxic crisis −− Occurs if the thyrotoxic patient has been inadequately prepared for thyroidectomyQ.

Multiple Choice Questions PAPILLARY CARCINOMA 1.

A 35 years old female presented with a swelling in the neck for the past 2 months, she had the treatment for Hodgkin’s lymphoma when she was 22 years with irradiation. On, examination, her vitals were normal, there was a single, firm, irregular nodule, moving with deglutition in the left side of midline. Clinical examination also revealed a single node in the left side of the neck. The most likely clinical diagnosis of this condition is: (COMEDK 2011) a. Recurrence of lymphoma b. Malignant goiter c. Benign multinodular goiter d. Toxic nodular goiter

2.

Most probable pathological diagnosis would be:  (COMEDK 2011) a. Anaplastic carcinoma b. Follicular carcinoma c. Medullary carcinoma d. Papillary carcinoma

3.

The FNAC of the lesion should reveal: a. ‘Orphan-Annie eye’ nucleus cells b. Amyloid deposits c. Epitheloid cells and giant cells d. Follicular cells

4.

The ideal treatment of the above condition would be: a. Total thyroidectomy with lymph nodal dissection of the same side (COMEDK 2011) b. Radiotherapy c. Lobectomy d. Lobectomy with isthmusectomy

5.

About papillary carcinoma what is/are true? (PGI Dec 2008) a. Often encapsulated b. Prognosis is bad c. Lymph node metastases is common d. Can metastasize to lung e. Multiple foci of tumour is seen

6.

Variant of papillary carcinoma thyroid: a. Medullary b. Warthin c. Columnar d. Insular e. Diffuse sclerosing

7.

Which of the following would be the best treatment for a 2 cm thyroid nodule in a 50-year-old man with FNAC revealing it to be a papillary carcinoma? (AIIMS May 2011) a. Hemithyroidectomy b. Total thyroidectomy with left sided modified neck dissection c. Near total thyroidectomy with radiotherapy d. Hemithyroidectomy with modified neck dissection

8.

9.

10.

About papillary carcinoma true statement is/are: a. Radiation is a risk factor (PGI Nov 2010) b. Multifocal c. Hematogenous spread is common d. Distant metastasis is seen

11.

Most common thyroid malignancy is: (DNB 2012, MHPGMCET 2002) a. Anaplastic carcinoma b. Follicular carcinoma c. Medullary carcinoma d. Papillary carcinoma

12.

Which thyroid malignancy is common after radiation exposure?  (Recent Question 2016, MHSSMCET 2005) a. Follicular b. Papillary c. Medullary d. Anaplastic

13.

A 10 years old boy presented with cervical lymph adenopathy. Needle biopsy from the nodes revealed secondaries from papillary carcinoma of thyroid. The child under went complete removal of tumor near total thyroidectomy and radical neck dissection. What should be the immediate next line of management? (All India 2012) a. Start thyroxine suppression therapy b. I-131 whole body scan to assess the extent of disease c. Bone scan to evaluate secondaries d. CECT scan to assess any residual disease

14.

Orphan Annie-eye nuclei seen in: a. Papillary carcinoma of thyroid b. Medullary carcinoma of thyroid c. Anaplastic carcinoma of thyroid d. Follicular carcinoma of thyroid

15.

Psammoma bodies are seen in following except: (PGI 2002) a. Serous cystadenoma of ovary b. Mucinous cystadenoma of ovary c. Meningioma d. Papillary carcinoma of thyroid

16.

Which of the following would be the best treatment for a 2 cm thyroid nodule in a 50 years old man with FNAC revealing it to be a papillary carcinoma? (All India 2009) a. Hemithyroidectomy (Recent Question 2015) b. Subtotal thyroidectomy with modified neck dissection c. Near total thyroidectomy with modified neck dissection d. Hemithyroidectomy with modified neck dissection

17.

True regarding papillary carcinoma of thyroid: a. Undifferentiated carcinoma  (MCI March 2006) b. Blood-borne metastasis is commoner c. Excellent prognosis d. Capsulated

18.

Which type of thyroid carcinoma has the best prognosis? (DNB 2010, All India 96) a. Papillary carcinoma b. Anaplastic carcinoma c. Follicular carcinoma d. Medullary carcinoma

19.

Compared to follicular carcinoma, papillary carcinoma of thyroid have: (PGI Dec 2007, June 2005, Dec 2006) a. More male preponderance b. Bilaterality c. Local recurrence common d. Increased lymph node metastasis e. Increased mortality

(COMEDK 2011)

(PGI June 2007)

Psammoma bodies may be seen in all of the following, except: (Recent Question 2016, All India 2011) a. Follicular carcinoma of thyroid b. Papillary carcinoma of thyroid c. Meningioma d. Serous cystadenocarcinoma of ovary Features of papillary carcinoma includes: (PGI May 2011) a. FNAC easy b. Almost always unifocal c. Psammoma body d. Spread to cervical LN e. Bad prognosis

(Orissa 2011)

66

Surgery Essence 20.

21.

Endocrine Surgery

22.

23.

24.

25.

26.

Occult thyroid malignancy with nodal metastasis is: (DNB 2005, 2001, AIIMS Sept 96) a. Medullary carcinoma b. Follicular carcinoma c. Papillary carcinoma d. Anaplastic carcinoma Least malignant thyroid cancer is: (AIIMS Nov 2003) a. Papillary carcinoma b. Follicular carcinoma c. Anaplastic carcinoma d. Medullary carcinoma Lateral aberrant thyroid refers to: (AIIMS June 2002) a. Congenital thyroid abnormality b. Metastatic foci from primary in thyroid c. Struma ovarii d. Lingual thyroid Which of the following is used in the treatment of differentiated thyroid cancer? (All India 2006) a. I-131 b. 99mTc c. P-32 d. I-131 MIBG In treatment of papillary carcinomas thyroid, radioiodine destroys the neoplastic cells predominantly by:  (AIIMS Nov 2005) a. X-rays b. Beta rays c. Gamma rays d. Alpha particles A 21 years old woman has 3 cm node in the lower deep cervical chain on the left. The biopsy is interpreted as revealing normal thyroid tissue in a lymph node. The most likely diagnosis is:  (DNB 2012, DPG 2009 Feb) a. Subacute thyroiditis b. Metastatic carcinoma thyroid c. Hashimoto’s disease d. Lateral aberrant thyroid All of the following regarding papillary carcinoma thyroid is true except: (All India 90) a. Multicentric origin b. Secondaries to lymph nodes c. Slowing growing d. Bony metastasis in early stage

27. Most common type of carcinoma thyroid having least chances of hematogenous spread: (Recent Question 2015) a. Follicular b. Papillary c. Anaplastic d. Medullary 28. Chance of metastasis to lymph nodes in PTC:  (Recent Question 2017) a. 60% 29.

Section 1

30.

31.

Orphan-Annie eye nuclei is seen in: (Recent Question 2017) a. Papillary carcinoma thyroid b. Follicular carcinoma thyroid c. Medullary carcinoma thyroid d. Anaplastic carcinoma thyroid Psammoma bodies are seen in: (Recent Question 2017) a. Papillary carcinoma thyroid b. Follicular carcinoma thyroid c. Medullary carcinoma thyroid d. Thyroid lymphoma A 27-year-old lady with 20 weeks pregnancy presented with a thyroid nodule on right side. FNAC from the nodule was suggestive of papillary carcinoma. Which of the following is contraindicated in her management? (AIIMS May 2017) a. Total thyroidectomy plus neck node dissection b. Right lobectomy c. Radioactive iodine ablation
 d. Total thyroidectomy


FOLLICULAR CARCINOMA 32.

33.

34.

35.

All of the following are true for follicular carcinoma of thyroid except: (COMEDK 2006) a. Lymph node involvement rare b. Vascular involvement common c. Younger patients have good prognosis d. Diagnosis by FNAC Thyroid carcinoma with pulsating vascular skeletal metastasis is: (COMEDK 2007, All India 95) a. Follicular b. Anaplastic c. Medullary d. Papillary Follicular carcinoma of thyroid is due to mutation of: a. RAS b. HGF (JIPMER 2010) c. RET d. ABL A well differentiated follicular carcinoma of thyroid can be best differentiated from a follicular adenoma by: a. Hurthle cell change (All India 2011, 2009) b. Lining of tall columnar and cuboidal cells c. Vascular invasion d. Nuclear features

36.

FNAC is useful in all the following types of thyroid carcinoma except: (UPPG 2010, MCI March 2005, All India 95) a. Papillary b. Follicular c. Anaplastic d. Medullary

37.

Most probable malignancy that develops in a case of longstanding goiter is: (MCI June 2018, Recent Question 2015,  Kerala PG 2015, AIIMS Feb 97, Nov 2001) a. Follicular carcinoma b. Anaplastic carcinoma c. Papillary carcinoma d. Medullary carcinoma

38.

Bone metastasis is common in which thyroid tumor: a. Follicular b. Papillary(AIIMS Nov 99) c. Hurthle cell tumour d. Anaplastic

39.

Thyroid nodule of 4 cm size, mobile but causing compressive symptoms. All are true except: (DNB 2011) a. FNAC is investigation of choice b. FNAC cannot distinguish follicular adenoma from carcinoma c. Managed by sub-total thyroidectomy d. Cold nodules are diagnostic of malignancy

40.

In case of adenomatoid goiter which carcinoma is commonest to occur: (AIIMS Nov 98) a. Medullary carcinoma b. Follicular carcinoma c. Papillary carcinoma d. Anaplastic carcinoma

41.

Carcinoma thyroid with blood borne metastasis is: a. Follicular b. Papillary(AIIMS Feb 97) c. Medullary d. Anaplastic

42.

Lymph node metastasis is least commonly seen with: a. Papillary CA Thyroid  (All India 94) b. Medullary CA Thyroid c. Follicular CA Thyroid d. Anaplastic CA Thyroid

43.

A 20 years old female patient presented with a thyroid swelling. Most probably, the fine needle aspiration cytology will not diagnose: (AIIMS Nov 97) a. Papillary carcinoma of thyroid b. Medullary carcinoma of thyroid c. Non-Hodgkin’s lymphoma of thyroid d. Follicular carcinoma of thyroid

Thyroid 44.

(WBPG 2012, DPG 2007)

45.

Metastasis from follicular carcinoma should be treated by: a. Radioiodine b. Surgery (MCI Sept 2006) c. Thyroxine d. Observation

46.

True regarding follicular carcinoma of thyroid: a. Hematogemous spread (JIPMER 2014, 2013) b. Commonly multifocal c. Readily diagnosed by face d. Most commonly carcinoma of thyroid

47. The microscopic feature that differentiates a follicular carcinoma from a follicular adenoma: (COMEDK 2014) a. Nuclear pleomorphism b. Hurthle cell change c. Capsular invasion d. Absence of colloid 48.

FNAC cannot detect which of the following?  (AIIMS November 2014) a. Follicular carcinoma b. Papillary carcinoma c. Colloid goiter d. Hashimoto’s thyroiditis

MEDULLARY CARCINOMA 49.

50.

Screening method of medullary carcinoma thyroid is: a. Serum calcitonin (All India 97, AIIMS Nov 95) b. Serum calcium c. Serum alkaline phosphate d. Serum acid phosphatase

51.

False statement about feature of MTC: (PGI Nov 2011) a. Familial MTC may presents in 2nd decade b. It has characteristic amyloid stroma c. Secrete serotonin d. Take up radioiodine e. Secrete calcitonin

52.

Thyroid radioiodine ablation therapy is useful in all except: a. Recurrent papillary carcinoma (PGI May 2011) b. Residual papillary carcinoma c. Anaplastic carcinoma d. Follicular carcinoma e. Medullary carcinoma

53.

Age for prophylactic thyroidectomy in MEN IIB syndrome?  (MHSSMCET 2009) a. 1 month b. 2 months c. 4 months d. 6 months

54. Recommended age for prophylactic thyroidectomy for MEN-2 is:  (Recent Question 2015) a. 5 years b. Before 1 year c. At the time of diagnosis d. Any time 55.

Thyroid carcinoma associated with hypocalcemia is:  (AIIMS Dec 94) a. Follicular carcinoma b. Medullary carcinoma c. Anaplastic carcinoma d. Papillary carcinoma

Medullary carcinoma thyroid arises from: (AIIMS Nov 93) a. Parafollicular cells b. Cells lining the acini c. Capsule of thyroid d. Stroma of the gland

57.

A biopsy from a mass in front of the neck revealed parafollicular cells. How do you follow up?   (JIPMER November 2017) a. Calcitonin b. T4 c. Thyroxine d. Thyroglobulin

58.

Treatment of medullary carcinoma thyroid: a. Surgery and radiotherapy (AIIMS Nov 2008) b. Radiotherapy and chemotherapy c. Surgery only d. Radioiodine ablation

59.

Needle biopsy of solitary thyroid nodule in a young woman with palpable cervical lymph nodes on the same sides demonstrates amyloid in stroma of lesion. Likely diagnosis is: a. Medullary carcinoma thyroid  (All India 2002) b. Follicular carcinoma thyroid c. Thyroid adenoma d. Multinodular goiter

60.

In medullary carcinoma thyroid tumour marker is:  (WBPG 2014, AIIMS June 98) a. TSH b. Calcitonin c. T3, T4 and TSH d. Alpha Fetoprotein

61.

After thyroidectomy for medullary carcinoma of thyroid, which is important for determining recurrence of tumour? a. Thyroglobulin b. TSH (MCI Sept 2009) c. CEA d. Thyroxine levels

62.

The expression of the following oncogene is associated with a high incidence of medullary carcinoma of thyroid:  (AIIMS Nov 2005) a. p53 b. Her-2-neu c. Ret proto-oncogene d. Rb gene

63.

A 26 years old women presents with a palpable thyroid nodule, and needle biopsy demonstrates amyloid in the stroma of the lesion. A cervical lymph node is palpable on the same side as the lesion. The preferred treatment should be: a. Removal of the involved node, the isthmus, a portion of the opposite bone and he enlarged lymph node b. Removal of the involved lobe, the isthmus, a portion of the opposite lobe, and he enlarged lymph node c. Total thyroidectomy and modified neck dissection on the side of the enlarged lymph node d. Total thyroidectomy and irradiation of the cervical lymph nodes (BIHAR PG 2014, All India 2002)

64.

Amyloid stroma is seen in which carcinoma thyroid: a. Papillary carcinoma (AIIMS June 2000) b. Medullary carcinoma c. Anaplastic carcinoma d. Follicular carcinoma

65.

A patient has pituitary tumour and pheochromocytoma and a thyroid nodule. Which carcinoma is most likely to occur?  (AIIMS Nov 2000) a. Follicular carcinoma b. Medullary carcinoma c. Papillary carcinoma d. Anaplastic carcinoma

66.

A 52 years old female patient presents with symptoms of pheochromocytoma. She also has a thyroid carcinoma. Her thyroid carcinoma is of which type: (AIIMS June 99) a. Anaplastic b. Medullary c. Follicular d. Papillary

Endocrine Surgery

Treatment of medullary carcinoma thyroid: a. Surgery and Radiotherapy  (AIIMS May 2011) b. Radiotherapy and Chemotherapy c. Surgery only d. Radioiodine ablation

56.

Section 1

Hurthle cells tumour is: a. Papillary carcinoma thyroid b. Follicular carcinoma thyroid c. Medullary carcinoma thyroid d. Anaplastic carcinoma

67

68

Surgery Essence

Endocrine Surgery

67.

68.

Serum calcitonin is a marker for: (DNB 2003, All India 94) a. Anaplastic carcinoma b. Papillary carcinoma c. Medullary carcinoma d. Follicular carcinoma

69.

Treatment of choice for medullary carcinoma of thyroid is: (AIIMS May 2005) a. Total thyroidectomy b. Partial thyroidectomy c. I-131 ablation d. Hemithyroidectomy

70.

71.

72.

All of the following are helpful for diagnosis of medullary carcinoma thyroid except: (PGI 2000) a. Spindle cell stroma with few follicles b. Amyloid deposition c. Calcitonin in stroma d. Histological mitochondria is essential for diagnosis Which of the following gene defects is associated with development of medullary carcinoma of thyroid? a. Ret proto-oncogene b. FAP gene (All India 2004) c. Rb gene d. BRCA-1 gene Commonest presenting complaints of medullary carcinoma thyroid: (PGI 84) a. Diarrhea b. Dysphagia c. Hoarseness d. Flushing

73.

Which of the following is true about medullary carcinoma? a. Calcitonin is not a marker (DPG 2008) b. Arises from parafollicular C cells c. Produces PTH d. Take up radioiodine

74.

True about medullary carcinoma thyroid: (DPG 2007) a. Good prognosis b. Associated with MEN-1 c. Increased calcitonin is not associated with hypocalcemia d. Treated by near total thyroidectomy

75.

Section 1

MEN-2 is seen with the following type of thyroid carcinoma: a. Papillary b. Medullary (All India 97) c. Anaplastic d. Follicular

All are true regarding medullary carcinoma of thyroid except: (JIPMER 2014, 2013) a. It arises from ‘C’ cells b. Secrete high levels of calcitonin c. It is dependent on TSH d. Most cases are familial

76.

Which of the following does not take radioactive iodine?  (Recent Question 2017) a. Medullary carcinoma thyroid b. Papillary carcinoma thyroid c. Follicular carcinoma thyroid d. Hürthle cell carcinoma thyroid

77.

Calcitonin is the marker for: a. Papillary carcinoma thyroid b. Follicular carcinoma thyroid c. Medullary carcinoma thyroid d. Anaplastic carcinoma thyroid

78.

Medullary carcinoma thyroid is associated with which mutation?  (Recent Question 2017) a. RET b. MET c. p53 d. PTEN

79.

RET proto-oncogene is associated with development of: a. Medullary carcinoma thyroid (Recent Question 2018) b. Astrocytoma c. Paraganglioma d. Hürthle cell tumor thyroid

(Recent Question 2017)

ANAPLASTIC CARCINOMA 80.

Not true about anaplastic thyroid carcinoma: a. Local infiltration common  (PGI May 2011) b. Spread by lymphatic route c. Long term survival in patient undergoing surgery d. Surgery is of limited value e. Highly chemosensitive

81. A patient with long standing multinodular goitre develops hoarseness of voice and swelling undergoes sudden increase in size. Likely diagnosis is:  (Recent Question 2014, All India 2001) a. Follicular carcinoma b. Papillary carcinoma c. Medullary carcinoma d. Anaplastic carcinoma 82.

The treatment of choice for anaplastic carcinoma of thyroid infiltrating trachea and sternum will be:  (AIIMS Nov 2005) a. Radical excision b. Chemotherapy c. Radiotherapy d. Palliative/Symptomatic treatment

83. Least common thyroid malignancy is: (Recent Question 2015) a. Papillary b. Follicular c. Medullary d. Anaplastic

THYROID METASTASIS 84.

Metastasis in thyroid gland come most commonly from carcinoma of: (PGI June 98) a. Testis b. Prostate c. Breast d. Lungs

THYROID LYMPHOMA 85.

All of the following are true about lymphoma of the thyroid except: (All India 2007) a. More common in females b. Slow growing c. Clinically confused with undifferentiated tumors d. May present with respiratory distress and dysphagia

CARCINOMA THYROID 86.

False statement regarding thyroid carcinoma:(PGI Nov 2011) a. Medullary thyroid carcinoma is associated with MEN-2A b. Follicular carcinoma -Most common type of carcinoma c. Papillary carcinoma -Multifocal d. Thyroid lymphoma is often associated with Hashimoto thyroiditis e. Anaplastic carcinoma occur in old age women

87.

True about thyroid carcinoma: (PGI Dec 2006) a. Follicular carcinoma have worse prognosis than papillary carcinoma b. Papillary carcinoma spreads by hematogenous route more frequently than follicular carcinoma c. Papillary carcinoma have increased mortality than follicular carcinoma d. Follicular carcinoma are more bilateral than papillary carcinoma e. Follicular carcinoma have more male incidence than papillary carcinoma

88.

Low risk in carcinoma thyroid: (PGI Dec 2006) a. Men 1 cm in diameter

RETROSTERNAL GOITER 122. Most common symptom of retrosternal goiter: a. Dysphagia b. Stridor (Punjab 2010, PGI June 97) c. Dyspnea d. Superior vena cava syndrome

Section 1

123. Retrosternal goiter is characterized by: (DPG 2005) a. Stridor b. Always malignant c. Bilateral d. None of the above 124. Most commonly used approach for retrosternal goitre: a. Transthoracic via second intercostal space b. Transthoracic via fourth intercostal space c. Trans-sternal through anterior mediastinum d. Transcervical (Recent Question 2019)

THYROTOXICOSIS 125. Thyroid storm after operation is due to: (COMEDK 2007) a. Inadequate control of hyperthyroidism b. Massive bleeding c. Recurrent laryngeal nerve injury d. Postoperative infection

126. Which of the following is the agent of choice for treating thyrotoxicosis during pregnancy? (COMEDK 2010) a. Carbimazole b. Propylthiouracil c. Methimazole d. Radioactive I-131 127. All of the following are features of thyrotoxicosis, except: a. Diastolic murmur (Recent Question 2016) b. Soft non-ejection systolic murmur c. Irregularly, irregular pulse d. Scratching sound in systole 128. Dancing carotid is seen in: (AIIMS Dec 98) a. Thyrotoxicosis b. Hypothyroidism c. AV Fistula d. Blow out carotid 129. The best marker to diagnose thyroid related disorder is: a. T3 b. T4 c. TSH d. Thyroglobulin 130. The occurrence of hypothyroidism following administration of supplemental iodine to subjects with endemic iodine to subjects with endemic iodine deficiency goiter is known as: a. Jod- Basedow effect  (All India 2004) b. Wolff-Chaikoff effect c. Thyrotoxicosis factitia d. De Quervain’s thyroiditis 131. In thyrotoxicosis, β-blockers do not control: (All India 94) a. Anxiety b. Tremors c. Tachycardia d. Oxygen consumption 132. All of the following are associated with thyroid storm, except:  (All India 2002) a. Surgery for thyroiditis b. Surgery for thyrotoxicosis c. Stressful illness in thyrotoxicosis d. I-131 therapy for thyrotoxicosis 133. Cardiovascular findings in an elderly thyrotoxicosis patient are all, except: (All India 2000) a. Early diastolic murmur b. Systolic ejection murmur c. Scratch in left 2nd intercostal space d. Irregularly irregular pulse 134. Treatment of thyroid storm includes all, except:  (AIIMS Nov 2003) a. Propranolol b. Radioactive iodine c. Hydrocortisone d. Lugol’s iodine 135. Difference between thyrotoxicosis and malignant hyperthermia is: (AIIMS June 2001) a. Hyperthermia b. Tachycardia c. Muscle rigidity d. Elevated serum CPK level 136. In thyrotoxicosis, which of the following is seen? a. Pretibial myxedema b. Glycosuria c. Unilateral exophthalmos d. All 137. Toxic adenoma on scanning appear as: (JIPMER 98) a. Hot nodule b. Cold nodule c. Warm nodule d. Neutral 138. Thyroid storm after operation is due to: (COMEDK 2007) a. Inadequate control of hyperthyroidism b. Massive bleeding c. Recurrent laryngeal nerve injury d. Postoperative injection 139. A 48-year-old woman underwent subtotal thyroidectomy. She has vague family history of malignant hyperthermia. She develops agitation, restlessness, fever, tremor, shivering, and tachypnea. Thyrotoxic crises can be best distinguished from malignant hyperthermia by estimating: (Kerala 2004) a. Temperature variation b. Increased CPK levels c. LDH d. Muscular rigidity

Thyroid

143. Which of the following is a symptom of hypothyroidism?  (JIPMER 2014, 2007) a. Hyperactivity b. Palpitation c. Diarrhoea d. Hair loss 144. Reduction of size and vascularity prior to thyroidectomy is done by: (Recent Question 2015) a. Iodides b. Propylthiouracil c. Radioiodine d. Propranolol 145. A patient underwent thyroidectomy for Hyperthyroidism. Two days later he presented with features of thyroid storm. What is the most likely cause? (AIIMS November 2015) a. Poor antibiotic coverage b. Rough handling during surgery c. Removal of parathyroid d. Inadequate preoperative preparation

GRAVE’S DISEASE 147. All of the following are features of Grave’s disease except: (MCI Sept 2005) a. More common in males b. Tremor c. Pretibial myxoedema d. Intolerance to heat 148. Complications of therapy with radioactive iodine includes: a. Thyroid malignancy b. Hypothyroidism c. Leukemia d. All of the above 149. Which of the following conditions is most common complication of radioiodine treatment of Grave’s disease?  (COMEDK 2005, 2004) a. Thyroid storm b. Subacute thyroiditis c. Thyroid cancer d. Hypothyroidism 150. In which of the following conditions radioactive iodine (Irradiation) can be used in Grave’s disease: (PGI Nov 2010) a. Recurrence b. Age >40 years c. Elderly d. Pregnant e. Presence of associated co-morbidities 151. Pretibial myxedema is seen in: (MHPGMET 2005) a. Thyrotoxicosis b. Hypothyroidism c. Hyperparathyroidism d. All

153. Which of the following statements about Grave’s disease is false? (Recent Question 2018) a. Results in hyperthyroidism b. Autoimmune disorder c. Common in male d. Referred as diffuse toxic goiter 154. All of the following are true about Graves disease except: a. Cardiac failure is common (JIPMER 2013) b. Hypertrophy and hyperplasia or thyroid gland is due to TSH-Rab c. Remissions and exacerbations are not infrequent d. It is highly vascular with audible bruit

HYPOTHYROIDISM 155. In case of hypothyroidism which investigation is most informative and most commonly used: (AIIMS June 98) a. Serum TSH level b. Serum T3, T4 level c. Serum calcitonin assay d. Serum TRH assay 156. Hypothyroidism with increased TSH level is seen in all except: (PGI 90) a. Sheehan’s syndrome b. Lithium carbonate therapy c. Post radioiodine ablation d. Endemic goitre

POST THYROIDECTOMY COMPLICATIONS 157. During thyroidectomy, inferior thyroid artery is ligated at: a. Maximally away from the gland (MCI Sept 2005) b. Close to the gland c. Half way from the gland d. None of the above 158. Complications of total thyroidectomy include all except:  (AIIMS May 2005) a. Hoarseness b. Airway obstruction c. Hemorrhage d. Hypercalcemia 159. Two hours after subtotal thyroidectomy for thyrotoxicosis, young woman rapidly becomes agitated and complains of increasing difficulty in breathing. Her pulse rate rises and central cyanosis is noticed on examination, her neck is found to be tensely swollen beneath the stitches. The most appropriate management in this case would be: (DPG 2011) a. Intranasal oxygen b. Passing an endotracheal tube in the ward c. Removing sutures from all layers in the ward and evacuation of hematoma d. Immediate transfer of the patient to the operation theatre for tracheostomy 160. Horner’s syndrome, all are true except: (AIIMS May 2011) a. Miosis b. Anhydrosis c. Hyperchromatic iris d. Apparent exophthalmos 161. Horner’s syndrome is seen in all except:  (AIIMS Nov 2010) a. Carotid artery aneurysm  b. Medial medullary syndrome c. Can occur following surgery for Raynaud’s syndrome d. Multiple sclerosis

Endocrine Surgery

146. Treatment of choice for recurrent thyrotoxicosis after surgery is: (MCI June 2018) a. Further surgery b. Radioiodine followed by surgery c. Radioiodine d. Observation & follow-up

152. Therapy of choice for diffuse toxic goiter in a patient over 45 years: a. Surgery b. Antithyroid drugs c. Radioiodine d. Antithyroid drugs first followed by surgery

Section 1

140. A 55-year-old male patient underwent cholecystectomy for Gall stone calculus. During surgery the patient’s pulse was irregularly irregular, 160/min, BP = 80/50 mm of Hg, temp. 40°C. On examination a swelling in the neck was found. Most likely diagnosis is: (MAHE 2007) a. Thyroid storm b. Myocardial infarction c. Arrythmias d. Stridor 141. All of the following conditions are associated with hyperthyroidism, except: (All India 2011) a. Hashimoto’s thyroiditis b. Grave’s disease c. Toxic multinodular goiter d. Struma ovary 142. Hyperthyroidism occurs in: (PGI Nov 2011) a. Hashimoto thyroiditis b. Graves’ disease c. Medullary thyroid carcinoma d. Plummer’s disease e. Struma ovarii

71

Section 1

Endocrine Surgery

72

Surgery Essence 162. Horner’s syndrome does not include: (COMEDK 2004) a. Ptosis b. Anhydrosis c. Flushing d. Mydriasis 163. During thyroidectomy, inferior thyroid artery ligation is done at what level? (MHPGMCET 2003) a. As close to the thyroid gland as possible b. As far away as possible from the thyroid gland c. In the trachea-esophageal groove d. Any of the above 164. About thyroid surgery all are true except: (Punjab 2008) a. Superior thyroid artery is ligated near the gland b. Capsule is removed c. Inferior thyroid artery is ligated away from gland d. Capsule is kept intact 165. Hypoparathyroidism following thyroid surgery occurs within: (AIIMS Nov 2004, Nov 2003) a. 24 hours b. 2–5 days c. 7–14 days d. 2–3 weeks 166. Complications of hemithyroidectomy include all of the following except? (All India 2008) a. Hypocalcemia b. Wound hematoma c. Recurrent laryngeal nerve palsy d. External branch of superior laryngeal nerve palsy 167. A patient undergoes thyroid surgery, following which he develops perioral tingling. Blood calcium is 8.9 meq/L. Next step is: (All India 2001) a. Vitamin D orally b. Oral calcium and vitamin D c. Intravenous calcium gluconate and serial monitoring d. Wait for calcium to decrease to < 7.0 before talking further action 168. Patient after thyroid surgery presents with perioral paresthesia. Serum calcium level is 7 mg/dl. What will be the best management: (AIIMS Nov 2000) a. Oral vitamin D3 b. Oral vitamin D3 with calcium c. IV calcium gluconate d. Oral calcium 169. A post-thyroidectomy patient develops signs and symptoms of tetany. The management is: (All India 2000) a. IV calcium gluconate b. Bicarbonate c. Calcitonin d. Vitamin D 170. In postoperative room after thyroid surgery patient developed sudden respiratory distress, dressing was removed and it was found to be slightly blood stained and wound was bulging. What will be first thing to be done? a. Tracheostomy (AIIMS June 2000, Nov 2000) b. Cricothyroidotomy c. Laryngoscopy and intubation d. Remove the stitch and take the patient to O.T. 171. A patient presents with swelling in the neck following a thyroidectomy; what is the most likely resulting complication? a. Respiratory obstruction  (All India 2001) b. Recurrent laryngeal nerve palsy c. Hypovolemia d. Hypocalcemia 172. Patient presents with neck swelling and respiratory distress few hours after a thyroidectomy surgery. Next management would be: (All India 2001) a. Open immediately b. Trecheostomy c. Wait and watch d. Oxygen by mask

173. Most dangerous complication in a patient who had undergone thyroid surgery and develop hematoma at the operative site: (AIIMS Nov 99) a. Respiratory obstruction b. Recurrent laryngeal nerve palsy c. Dysphagia d. Shock 174. After thyroidectomy, patient developed stridor within 2 hours. All are likely cause of stridor except: a. Hypocalcemia (AIIMS Nov 2001, Nov 2000) b. Recurrent laryngeal nerve palsy c. Laryngomalacia d. Wound hematoma 175. A 50-year-old male is suffering from severe dyspnea after thyroid surgery, treatment of choice is:  (AIIMS June 97) a. Tracheostomy b. Open the operative site c. Wait and watch d. Cricothyroidotomy 176. A patient operated for thyroid surgery for a thyroid swelling, later in the evening developed difficulty in breathing. There was swelling in the neck. The immediate management would be: (AIIMS June 2002) a. Epinephrine injection b. Tracheostomy c. IV calcium gluconate d. Open the wound sutures in the ward 177. Replacement dose of thyroxine is: (All India 93) a. 0.1–0.2 mg b. 0.3–0.4 mg c. 1–2 mg d. 3–4 mg 178. All of the following are early life threatening complications of thyroid operation except: (DPG 2010, SGPGI 2005) a. Tracheomalacia and collapse of the larynx b. Wound hematoma with compression of the trachea c. Hypocalcemia d. Thyroid storm 179. Vocal cord palsy in thyroid surgery is due to injury to: a. Superficial laryngeal nerve  (COMEDK 2008, UPSC 2008) b. Recurrent laryngeal nerve c. Ansa cervicalis d. Vagus nerve 180. Hemorrhage after thyroidectomy is due to:  (Recent Question 2014) a. External carotid artery b. Internal carotid artery c. Superior thyroid artery d. Inferior thyroid artery

THYROIDITIS 181. Thyroid biopsy of a patient showed the presence of Hurthle cells. Antibodies found in this condition are:  (Punjab 2011) a. Anti-TPO b. Anti-mitochondrial c. Anti-RNP d. Anti-dsDNA 182. A 40-year-old female presents with fever, fatigue, diffuse painful swelling in the midline of the neck, FNAC of the same reveals epitheloid cells and giant cells, the likely diagnosis is: a. Acute thyroiditis b. Subacute thyroiditis c. Tubercular lymphadenitis d. Hashimotos thyroiditis 183. A patient with autoimmune thyroiditis present with hypothyroidism. Which of the following is true? a. Thyroid peroxidase antibodies (JIPMER 2011) b. Painless enlargement of thyroid c. Common in men d. No malignant risk

Thyroid

185. Hashimoto’s thyroiditis, all are true except: (AIIMS May 2011) a. Follicular destruction b. Increase in lymphocytes c. Oncocytic metaplasia d. Orphan Annie eye nuclei 186. The laboratory investigation of a patient shows ↓T4, and ↑TSH. Which of the following is the most likely diagnosis:  (All India 2011) a. Grave’s disease b. Hashimoto’s disease c. Pituitary failure d. Hypothalamic failure 187. Which of the following conditions is associated with hypothyroidism? (All India 2011) a. Hashimoto’s thyroiditis b. Grave’s disease c. Toxic multinodular goiter d. Struma ovary

198. All are true abut Hashimoto’s thyroiditis except: (Kerala 95) a. Antithyroid microsomal antibodies b. Antithyroid nuclear antibodies c. Anti-TSH receptor antibodies d. Increased level of thyroid hormones 199. The thyroiditis also known as “Painless Thyroiditis”: a. Subacute lymphocytic thyroiditis (MAHE 2007) b. de-Quervain’s thyroiditis c. Hashimoto’s thyroiditis d. Riedel’s thyroiditis 200. A patient present with bilateral proptosis, heat intolerance and palpitations; most unlikely diagnosis here would be: (All India 2001, 2000) a. Hashimoto’s thyroiditis b. Thyroid adenoma c. Diffuse thyroid goiter d. Reidel’s thyroiditis 201. DeQvervain’s thyroiditis is characterised by: a. Mononuclear cell infiltration (COMEDK 2014) b. Histiocyte reaction c. Giant cell infiltration d. Eosinophilia

THYROGLOSSAL CYST AND FISTULA 202. Hyoid bone is closely associated with: (Orissa 2011) a. Bronchogenic cyst b. Cystic hygroma c. Branchial cyst d. Thyroglossal cyst/fistula 203. True about thyroglossal cyst is all except: a. Does not move with deglutition (MHPGMCET 2003) b. Move with protrusion of tongue c. Sistrunk’s operation is treatment of choice d. Most common site is subhyoid region 204. A 10-year-old child presented with midline swelling in anterior position of neck. Most probable diagnosis is: a. Thyroglossal cyst b. Thyroglossal fistula c. Cold abscess d. Acute lymphadenitis

193. A patient presents with bilateral proptosis, heat intolerance and palpitations. Most unlikely diagnosis here would be:  (All India 2001) a. Hashimoto’s thyroiditis b. Thyroid adenoma c. Diffuse thyroid goitre d. Reidel’s thyroiditis 194. A 25-years-old male presents with ophthalmologic signs of thyrotoxicosis. All are possibilities, except: (All India 2002) a. Diffused thyroid goitre b. Hashimoto’s thyroiditis c. Riedel’s thyroiditis d. Adenomatous goitre 195. A young patient has a midline, occurring after an attack of sore  a. Acute thyroiditis b. c. Subacute thyroiditis d.

tender swelling in neck throat. The diagnosis is: (AIIMS Nov 93) Thyroglossal cyst Toxic goiter

205. Sistrunk’s operation is done in:  (WBPG 2015, MHPGMCET 2008, 2006) a. Parotid tumor b. Thyroglossal fistula c. Thyroglossal cyst d. Branchial fistula

196. In Hashimoto’s disease serum antibodies are mainly against:  (Recent Question 2016) a. Thyroid follicles b. Thyroxine c. Thyroglobulins d. Iodine

206. Most common site of thyroglossal cyst: (Recent Question 2013, DNB 2009, 2007, 2005, 2003, MHSSMCET 2005, AIIMS June 97) a. Suprahyoid b. Hyoid c. Subhyoid d. Intra-thyroid

Endocrine Surgery

188. Most common cause of thyroiditis is:  (All India 2000) a. Reidel’s thyroiditis b. Subacute thyroiditis c. Hashimoto’s thyroiditis d. Viral thyroiditis 189. All of the following are true of de-Quervain’s thyroiditis except: (All India 1996) a. Pain b. Increased ESR c. Increased radioactive iodine uptake d. Fever 190. Not a feature of de-Quervain’s disease: (All India 2002) a. Autoimmune in etiology b. ↑ ESR c. Tends to regress spontaneously d. Painful and associated with enlargement of thyroid 191. Which of the following is wrong about subacute thyroiditis? a. Usually presents with painful enlargement of thyroid gland (Orissa 2011) b. There may be features of hyperthyroidism or hypothyroidism c. In the thyrotoxic phase radioiodine uptake is increased d. High ESR 192. ‘Hurthle cells ‘ are seen in: (All India 95) a. Agranulomatous thyroiditis b. Hashimoto’s thyroiditis c. Papillary carcinoma of the thyroid d. Thyroglossal cyst

197. The only reason for operating in case thyroiditis: a. To prevent cancerous degeneration b. For relief of pain in neck and ear c. To overcome pressure on trachea or esophagus d. To cure the toxic reaction e. If there is auto immune reaction

Section 1

184. A person has fever and pain in thyroid gland. True statement(s) is/are: (PGI June 2009) a. T3 and T4 level normal b. ↑ESR c. ↑TSH d. It is due to TB e. Radioactive iodine uptake is ↑ed

73

74

Surgery Essence

Section 1

Endocrine Surgery

207. The following statements about thyroglossal cyst are true, except: (All India 2006) a. Frequent cause of anterior midline neck masses in the first decade of life b. The cyst is located within 2 cm of the midline c. Incision and drainage is the treatment of choice d. The swelling moves upwards on protrusion of tongue 208. Thyroglossal cyst may occasionally give rise to carcinoma: a. Papillary b. Medullary c. Anaplastic c. Follicular 209. Sistrunk’s operation consists of: (DPG 2009 March) a. Excision of hyoid bone and cone of tongue muscle b. Excision of hyoid bone and the cyst c. Excision of central part of hyoid bone and cone of tongue muscles upto foramen caecum d. Excision of cyst only 210. A central midline neck swelling is noted in a 4 years old girl posted for tonsillectomy. The swelling is, painless, mobile, and cystic, just below the hyoid bone of size 2x1.1x1cm. USG showed a thick walled cystic lesion. Management would include: (AIIMS Nov 97) a. Surgical removal b. Antibiotics c. Percutaneous aspiration d. Chest X-ray

THYROID ANATOMY AND PHYSIOLOGY 211. The occurence of hyperthyroidism following administration of supplemental iodine to subject with endemic iodine deficiency goitre is known as:  (All India 2012) a. Jod-Basedow effect b. Wolff-Chaikoff effect c. Thyrotoxicosis factitia d. De-Quervains thyroiditis 212. Which of the following most closely represents the lowest detection limit for third generation TSH assays? a. 0.4 mIU/L b. 0.04 mIU/L (All India 2012) c. 0.004 mIU/L d. 0.0004 mIU/L 213. Recurrent laryngeal nerve is in close association with:  (AIIMS Nov 93) a. Superior thyroid artery b. Inferior thyroid artery c. Middle thyroid vein d. Superior thyroid vein 214. Normal thyroid weight varies …….. with dietary iodine content: a. Directly proportional b. Inversely c. Inverse cubically d. Not fixed 215. Average weight of thyroid gland where diet is rich in iodine is: a. 10–12 gm b. 14–16 gm c. 18–20 gm d. 28–30 gm 216. Protein bound iodine measures secretary function of thyroid in all of the following circumstances except: (All India 90) a. Nephrotic syndrome b. Following hemithyroidectomy c. During ampicillin therapy d. Asthamatics on ephedrine 217. Isthmus of thyroid gland overlies the: (DPG 97) a. 1st tracheal cartilage b. 1st and 2nd tracheal cartilage c. 2nd and 3rd and 4th tracheal cartilage d. 3rd and 4th tracheal cartilage

MISCELLANEOUS 218. Scabard trachea is seen in: a. Thyroid cancer c. Goitre

(Karnataka 99) b. Thyroiditis d. All of the above

219. A new born with a goiter large enough to cause dyspnoea is best treated with: a. Sulfonamides b. Tracheostomy c. T3 d. Iodides 220. Which of the following factors contribute to the development of duodenal ulcer? (PGI 2001) a. I-131 b. I-125 c. Tc-99 d. P-32 221. In pregnancy: a. Radioiodine contraindicated b. Thiouracil is contraindicated c. Surgery is contraindicated d. None

(APPG 2004)

222. Pendred’s syndrome is due to a defect in: (MCI June 2018, COMEDK 2007, 2008) a. Chromosome 7p b. Chromosome 7q c. Chromosome 8p d. Chromosome 8q 223. Reddish swelling in the region of foramen caecum: (DPG 2007) a. Lingual thyroid b. Lingual tonsil c. Ranula d. Thyroglossal cyst 224. True about Struma Ovarii: a. Ectopic thyroid b. Ectopic ovary c. Malignancy d. Benign lesion e. Included in teratoma

(PGI June 2007)

225. Absent parathyroid, thymic aplasia with immuno-deficiency and cardiac defects are features of: (MHPGMCET 2009) a. Autoimmune polyglandular syndrome b. Pendred syndrome c. Di George syndrome d. Lesch-Nyhan syndrome 226. This picture depicts …….. of examination of the thyroid gland. Choose the correct answer: (APPG 2016) a. b. c. d.

Kocher’s method Lahey’s method Crile’s method Pizzillo’s method

Explanations PAPILLARY CARCINOMA

1. Ans. b. Malignant goiter (Ref: Schwartz 10/e p1542-1544; Sabiston 20/e p902; Bailey 27/e p818; Harrison 20/e p2715, 19/e p2305)



2. Ans. d. Papillary carcinoma



3. Ans. a. ‘Orphan-Annie eye’ nucleus cells



4. Ans. a. Total thyroidectomy with lymph nodal dissection of the same side



5. Ans. c. Lymph node metastases is common; d. Can metastasize to lung; e. Multiple foci of tumour is seen Papillary carcinoma are rarely encapsulatedQ



6. Ans. c. Columnar d. Insular; e. Diffuse sclerosing (Ref: Schwartz 10/e p1542, 9/e p1361-1363; Sabiston 20/e p902, 19/e p906-909; Bailey 27/e p818, 26/e p765, 25/e p793-796) Types of Papillary Carcinoma Associated with Poor Prognosis 1. Tall cellQ 2. InsularQ 3. ColumnarQ 4. Diffuse sclerosingQ



5. Clear cellQ 6. TrabecularQ 7. Poorly differentiated type

7. Ans. b. Total thyroidectomy with left sided modified neck dissection 8. Ans. a. Follicular carcinoma of thyroid (Ref: Schwartz 10/e p1542; Sabiston 20/e p902; Bailey 27/e p818) Psammoma Bodies (PSM) 1. Papillary carcinoma thyroidQ 3. Serous cystadenomaQ



2. Papillary carcinoma (RCC) Q 4. MeningiomaQ

9. Ans. a. FNAC easy; c. Psammoma body; d. Spread to cervical LN



10. Ans. a. Radiation is a risk factor; b. Multifocal; d. Distant metastasis is seen



11. Ans. d. Papillary carcinoma



13. Ans. a. Start thyroxine suppression therapy (Ref: Schwartz 10/e p1549; Sabiston 20/e p906; Bailey 27/e p819)

12. Ans. b. Papillary

Well Differentiated Thyroid Cancer 1. Papillary carcinoma of thyroid 2. Follicular carcinoma of thyroidQ Q 3. Follicular variant of papillary carcinoma thyroid 4. Hurthle cell carcinoma (variant of follicular carcinoma thyroid) Q Q

Postoperative Management of Well-differentiated Thyroid Cancer Radioiodine Therapy • Postoperative RAI therapy reduces recurrence and provides a small improvement in survival, even in low-risk patientsQ. • Metastatic differentiated thyroid carcinoma can be detected and treated by 131I in about 75% of patientsQ. • RAI effectively treats >70% of lung micrometastasesQ that are detected by RAI scan in the presence of a normal chest x-ray, whereas the success rates drop to CA Lung • If thyroid metastases is detected pre-mortem, MC site of primary: RCCQ >CA Breast > CA Lung

CARCINOMA THYROID

86. Ans. b. Follicular carcinoma—Most common type of carcinoma (Ref: Schwartz 9/e p1361; Sabiston 20/e p900; Bailey 27/e p818)



87. Ans. a. Follicular carcinoma have worse prognosis than papillary carcinoma; e. Follicular carcinoma have more male incidence than papillary carcinoma



88. Ans. b. Women 40 years; c. Elderly; e. Presence of associated co-morbidities (Ref: Schwartz 10/e p1532-1533; Sabiston 20/e p892-893; Bailey 27/e p812; Harrison 20/e p2706, 19/e p2296) 151. Ans. a. Thyrotoxicosis

152. Ans. c. Radioiodine

153. Ans. c. Common in male (Ref: Schwartz 10/e p; Sabiston 20/e p; Bailey 27/e p) 154. Ans. c. Remissions and exacerbations are not infrequent

HYPOTHYROIDISM 155. Ans. a. Serum TSH level (Ref: Schwartz 10/e p1529; Sabiston 20/e p895; Bailey 27/e p802; Harrison 20/e p2701, 19/e p2288) • The ultrasensitive TSH assay has become the most sensitive and specific test for the diagnosis of hyper- and hypothyroidism and for optimizing T4 therapyQ. • The enhanced sensitivity and specificity of TSH assays have greatly improved laboratory assessment of thyroid functionQ. 156. Ans. a. Sheehan’s syndrome (Harrison 20/e p2734, 19/e p2257) Sheehan’s syndrome causes hypopituitarism leading to decreased TSHQ.

POST THYROIDECTOMY COMPLICATIONS 157. Ans. b. Close to the gland (Schwartz 10/e p1551-1554; Sabiston 20/e p913; Bailey 27/e p813) • Both superior and inferior thyroid vessels should be ligated close to the thyroidQ. • Superiorly, to avoid injury to the external branch of the superior laryngeal nerve. • Inferiorly, to minimize devascularization of the parathyroids (extracapsularQ dissection) or injury to the RLNQ.

Thyroidectomy: Schwartz 10/e p1553 • The dissection plane is kept as close to the thyroid as possible and the superior pole vessels are individually identified, skeletonized, ligated, and divided low on the thyroid gland to avoid injury to the external branch of the superior laryngeal nerveQ • The inferior thyroid vessels are dissected, skeletonized, ligated, and divided as close to the surface of the thyroid gland as possible to minimize devascularization of the parathyroids (extracapsular dissection) or injury to the RLNQ. 158. Ans. d. Hypercalcemia (Ref: Schwartz 10/e p1556; Sabiston 20/e p920; Bailey 27/e p815) 159. Ans. c. Removing sutures from all layers in the ward and evacuation of hematoma 160. Ans. d. Apparent exophthalmos (Ref: Harrison 20/e p3173, 19/e p196, 208) Clinical Features • • • • •

PtosisQ Miosis (constricted pupil)Q AnhydrosisQ EnopthalmosQ Loss of ciliospinal reflexesQ

Horner’s Syndrome Less Common Features • Hyperactive accommodation • Hypochromic heterochromia • Hypotony • Hyperaemia

Endocrine Surgery



84

Surgery Essence 161. Ans. b. Medial medullary syndrome (Ref: Harrison 20/e p3173, 19/e p196, 208) 162. Ans. d. Mydriasis 163. Ans. a. As close to the thyroid gland as possible (Ref: Schwartz 10/e p1553; Sabiston 20/e p913) 164. Ans. c. Inferior thyroid artery is ligated away from gland; d. Capsule is kept intact (Ref: Schwartz 10/e p1553)

Endocrine Surgery

• The inferior thyroid vessels are dissected, skeletonized, ligated, and divided as close to the surface of the thyroid gland as possible to minimize devascularization of the parathyroids (extracapsular dissection) or injury to the RLNQ. • Dissection is extracapsular (Capsule is removed) Q. 165. Ans. b. 2–5 days

166. Ans. a. Hypocalcemia

167. b. Oral calcium and vitamin D (Ref: Sabiston 20/e p919, 19/e p921; Bailey 27/e p815) Management of Post-operative Hypocalcemia Transient Hypocalcemia

Prolonged or Permanent Hypocalcemia

• Asymptomatic with calcium level >8 mg/dL: No treatmentQ • Mild symptoms or calcium level 90% of body’s iodine contentQ Daily iodine requirement: 100–150 µgQ Father of thyroid surgery: Theodor KocherQ Gland weight varies inversely with iodine intakeQ. Isthmus that is located just inferior to the cricoid cartilage, usually anterior to 2nd & 3rd (mainly) and 4th tracheal ringsQ. A pyramidal lobe is present in about 50% of patients. Thyroid capsule is condensed into the posterior suspensory or Berry’s ligamentQ near the cricoid cartilage & upper tracheal rings. • Thyroid gland has a thin capsule of connective tissue, which extends into glandular parenchyma & divides each lobe into irregularly shaped and sized lobules. • External laryngeal nerve runs close to the superior thyroid artery and the recurrent laryngeal nerve runs close to the inferior thyroid arteryQ.

• • • • • • • •

86

Surgery Essence 220. Ans. a. I-131

221. Ans. a. Radioiodine is contraindicated

222. Ans. b. Chromosome 7q (Ref: Harrison 20/e p2693, 19/e p2284; Schwartz 10/e p1534)

Pendred’s Syndrome • Consists of congenital sensorineural hearing loss + goitreQ • Due to defect in sulfate transport protein (chromosome 7qQ) to the thyroid gland and cochlea

Endocrine Surgery

Rafetoff Syndrome

End organ resistance to T4Q

223. Ans. a. Lingual thyroid (Ref: Schwartz 10/e p1522) Reddish swelling in the region of foramen caecum is Lingual thyroid.

Lingual Thyroid Forms a rounded swelling at the back of tongue at the foramen caecumQ It may represent the only thyroid tissue presentQ May cause dysphasia, impairment of speech, respiratory obstruction or hemorrhage Medical treatment options include administration of exogenous thyroid hormone to suppress TSH and RAI ablation followed by hormone replacement. • Surgical excision is rarely needed but, if required, should be preceded by an evaluation of normal thyroid tissue in the neck to avoid inadvertently rendering the patient hypothyroid. • • • •

224. Ans. d. Benign lesion; e. Included in Teratoma (Ref: Shaws Gynecology 14/e p336-337)

Struma Ovarii • • • • •

Highly specialized variety of teratoma A benign ovarian tumor containing thyroid tissueQ The tumor is solid and consisting entirely of thyroid tissue Some cases develop thyrotoxicosisQ Most of the tumor is innocent, but malignant transformation have been recorded

225. Ans. c. Di George syndrome (Ref: Schwartz 10/e p1574)

Di-George Syndrome Characterized by • Congenital cardiac defects, particularly those involving great vesselsQ • Hypocalcemic tetany due to failure of parathyroid developmentQ • Absence of normal thymus, T-cell immunodeficiencyQ

Section 1

226. Ans. d. Pizzillo’s method.

CHAPTER

3

Parathyroid and Adrenal Glands

MULTIPLE ENDOCRINE NEOPLASIA MEN-2 (MEN-2A or Sipple syndrome)

MEN-1

MEN-3 (MEN-2B)

MEN-4 (MEN-X)

Components

• Parathyroid hyperplasia or adenomaQ • Pancreatic NETQ • Pituitary adenomaQ • Bronchial & thymic carcinoidsQ • Adrenocortical tumors • Subcutaneous or visceral lipomasQ • Facial cutaneous angiofibromasQ • CollagenomasQ

• Medullary carcinoma thyroidQ • PheochromocytomaQ • Parathyroid hyperplasia or adenomaQ • Hirschsprung’s diseaseQ • Cutaneous lichen amyloidosisQ

• Medullary carcinoma thyroidQ • PheochromocytomaQ • Intestinal ganglioneuromaQ • Mucosal neuromasQ • MegacolonQ • Marfanoid featuresQ

• • • •

Gene/Defect

MEN1 geneQ

RET oncogene (cysteineQ codon)

RET oncogene (tyrosine kinaseQ domain)

Cyclin dependent kinase inhibitor (CDNKIB) gene

10Q

12

Autosomal dominantQ

Autosomal dominantQ

Chromosome 11

10Q

Transmission Autosomal dominant

Q

Autosomal dominant

Q

Hyperparathyroidism Pituitary adenoma Pancreatic NET Gonadal, adrenal, renal & thyroid tumors • (MEN-1 not having mutation of MEN1 gene is known as MEN-4)

MEN-1 (Wermer’s Syndrome) • Autosomal dominantQ • Defect: MEN1 geneQ on chromosome 11Q (encodes tumour suppressor protein, meninQ) Characteristic Features Common Manifestations • Parathyroid hyperplasia or adenomaQ • Pancreatic NETQ • Pituitary adenomaQ

• • • • •

Less Common Manifestations Bronchial and thymic carcinoidsQ Adrenocortical tumors Subcutaneous or visceral lipomasQ Facial cutaneous angiofibromasQ CollagenomasQ

Parathyroid Gland • • • •

MC endocrine abnormality (>98% of affected individuals) in MEN-1 is multiglandular parathyroid tumorsQ. Hyperparathyroidism is MC manifestation (cardinal sign of MEN-1 is parathyroid adenoma of multicentricity) Q. Parathyroid hyperplasia is the MC cause of hyperparathyroidism in MEN-1. Hypercalcemia is first biochemical abnormalityQ detected in MEN 1 and may precede the clinical onset of a pancreatic NET or pituitary neoplasm by several years.

Pancreatic Neuro-endocrine Tumors • • • •

Pancreatic NET is 2nd MC manifestationQ. Nonfunctioning or that secrete pancreatic polypeptide are MC pancreatic NET in MEN-1Q. MC functional NET in patients with MEN-1 is gastrinomaQ followed by insulinoma. MC increased pancreatic hormone: Pancreatic polypeptides >Gastrin >InsulinQ. (PGI)

Pituitary Adenoma • MC tumor: Prolactinoma > Somatotrophinoma > Corticotrophinoma (PSC)

88

Surgery Essence PRIMARY HYPERPARATHYROIDISM Primary Hyperparathyroidism • PHPT arises from increased PTH productionQ from abnormal parathyroid glands and results from a disturbance of normal feedback control exerted by serum calcium. • More common in womenQ

Endocrine Surgery

• Solitary adenomaQ is the MC cause (in 80%) • Parathyroid adenomas are most commonly located in inferior parathyroid glands. • Increased PTH production leads to hypercalcemia via: − Increased GI absorption of calcium − Increased production of vitamin D3 − Reduced renal calcium clearance Etiology • • • • •

Exposure to low-dose therapeutic ionizing radiation and familial predispositionQ Renal leak of calcium Declining renal function with age Alteration in the sensitivity of parathyroid glands to suppression by calcium Lithium therapy

Genetics • Most cases of PHPT are sporadic • Also associated with MEN1, MEN2A, isolated familial HPT, and familial HPT with jaw-tumor syndrome. Clinical Features • Patients with PHPT formerly presented with the “classic” pentad of symptoms: − Kidney stonesQ − Painful bonesQ − Abdominal groansQ Q Q − Psychic moans − Fatigue overtones • Alteration in the “typical” patient with PHPT due to widespread use of automated blood analyzers. • Patients are more likely to be minimally symptomatic or asymptomatic. • Currently, most patients present with weakness, fatigue, polydipsia, polyuria, nocturia, bone and joint pain, constipationQ, decreased appetite, nausea, heartburn, pruritus, depression, and memory loss. • Renal calculi are typically composed of calcium phosphate or oxalateQ. Osteitis Fibrosa Cystica in Advanced PHPT Pathognomonic radiologic findings on X-rays of hands, characterized by: • Subperiosteal resorptionQ (most apparent on the radial aspectQ of middle phalanxQ of 2nd and 3rd fingers) • Bone cystsQ • Tufting of distal phalangesQ Diagnosis • Elevated serum calcium & intact PTH or two-site PTH levels, without hypocalciuria establishes the diagnosis of PHPT with virtual certaintyQ. • Decreased serum phosphate (50%) & elevated 24-hour urinary calcium (60%) in PHPTQ Localization •

99m

Tc-labeled sestamibi: Most widely used & accurate modalityQ (sensitivity >80% for detection of parathyroid adenomas)

Section 1

Treatment • Parathyroidectomy for patients having “classic” symptoms of PHPT or 1 mg/dL above the upper limits of normal 2. Life-threatening hypercalcemic episode 3. Creatine clearance reduced by 30% 4. Kidney stones on abdominal X-rays 5. Markedly elevated 24-h urinary calcium excretion (≥400 mg/d)

6. Substantially decreased bone mineral density at the lumbar spine, hip, or distal radius 7. Age 1 year after a successful transplantQ. Treatment

PARATHYROID CARCINOMA Parathyroid Carcinoma • Accounts for approximately 1% of PHPT cases. Clinical Features Parathyroid Carcinoma is suspected preoperatively by • Presence of severe symptomsQ •  Serum calcium levels > 14Q mg/dL • Significantly elevated PTHQ levels (5 × normal) • Palpable parathyroidQ gland −− Local invasionQ is most common; LN metastases in 15% & distant metastases in 33% at presentation. −− Intraoperatively: Presence of a large, gray-white to gray-brown parathyroid tumor adherent to or invasive into surrounding tissuesQ and enlarged LN. Diagnosis • Accurate diagnosis necessitates histologic examination that reveals local tissue invasion, vascular or capsular invasionQ, trabecular or fibrous stroma, and frequent mitoses. Treatments • Parathyroid cancer: Bilateral neck exploration + En-bloc excision of tumor and ipsilateral thyroid lobe ± MRND in presence of LN metastasesQ • Reoperation for locally recurrent or metastatic disease to control hypercalcemia. • CinacalcetQ (reduce PTH levels by directly binding to CASR cells on parathyroid) is useful in controlling hypercalcemia in refractory parathyroid carcinoma.

Endocrine Surgery

• Subtotal or total parathyroidectomy with Autotransplantation + Upper thymectomyQ.

90

Surgery Essence INCIDENTALOMA Incidentaloma

Endocrine Surgery

• • • •

Incidentally discovered adrenal masses through imaging performed for unrelated/nonadrenal disease. Differential diagnosis includes both secreting & nonsecreting neoplasms. In patients with a history of malignancy, metastatic disease is the most likely cause of adrenal masses, particularly when bilateralQ. In those without a clear history of malignancy, at least 80% of incidentalomas will turn out to be nonfunctioning cortical adenomas or other benign lesions that do not require surgical managementQ.

Clinical Evaluation • Diagnostic work-up of an incidentaloma is aimed at identifying patients that would benefit from adrenalectomy • Workup for adrenal incidentaloma integrates hormonal evaluation with size criteriaQ. • Evaluation begins with history taking, with a focus on previous malignancy, hypertension, and symptoms of glucocorticoid or sex steroid excess. • Biochemical investigations for hormonally active tumors are followed by consideration of size criteria. • Tumors >6 cm carry a >25% risk for malignancyQ. • CT-guided FNAC is rarely helpfulQ in the evaluation of adrenal masses and may be hazardous. • The diagnosis of primary adrenal malignancy cannot be reliably based on cytologic criteria aloneQ. • Use of FNAC is generally confined to patients with a history of extra-adrenal malignancyQ in whom the clinician seeks to establish the diagnosis of metastatic diseaseQ. • Pheochromocytoma must be excludedQ before attempting such a procedure to avoid precipitating potentially fatal hypertensive crisis. Treatments • Surgery for hormonally active tumors and masses carrying significant risk for malignancyQ. • Most incidentalomas can be removed laparoscopicallyQ, except for those displaying obvious malignant features on imaging. • Remove all incidentalomas measuring >5 cm and to strongly consider removal of those measuring 3–5 cm, follow up with CT, every 6 months for lymph nodes. • Treatment: Resection followed by chemotherapyQ (cyclophosphamide + vincristine + dacarbazine)

Section 1

premature

MDH: Malignant pheochromocytoma secrete Dopamine and HVAQ

NEUROBLASTOMA Neuroblastoma • • • •

Arise from neural crestQ and may originate anywhere along the distribution of sympathetic chain MC tumor diagnosed in infants Paravertebral reteroperitoneumQ (28%) > Posterior mediastinum Q (15%) > Pelvis (5%) > Cervical areaQ

93

Clinical Features • MC presentation: Fixed, lobular mass extending from the flank toward the midlineQ of the abdomen • Most (80%) cases present before 4 years and peak incidence is 2 yearsQ of age • Metastasis is present in 60–70% of patients at the time of diagnosisQ • Orbital metastasis commonly present with periorbital ecchymoses and proptosis called as Raccoon eyesQ. • Infants with stage 4S may display cutaneous metastasis called as blueberry muffin lesionsQ. • Chronic watery diarrheaQ (due to secretion of VIP) and opsoclonus-myoclonusQ (dancing eyes, dancing feetQ) are unusual paraneoplstic manifestations. • MC site of metastasis in older children are bonesQ (Long bones–MC, facial bones, skull particularly sphenoid), bone marrow and LN. • In infants metastasis is confined to liver or subcutaneous tissueQ. • Lung metastasis are rareQ in neuroblastoma Diagnosis • Anemia, thrombocytopenia or thrombocytosis (more common) • Increased LDH, ferritin, urinary catecholamines and neuron specific enolase • X-ray or CT: Stippled calcificationQ (MC abdominal tumor to demonstrate calcification prior to chemotherapy) • Drooping Lily sign: Neuroblastoma displaces kidney inferolaterallyQ • MRIQ is superior to CT in assessing vessel encasement, vessel patency, spinal cord compression and bone marrow involvement.

Treatment • Localized neuroblastoma: ExcisionQ • Unresectable tumor: Biopsy, initially treated by chemotherapy and radiotherapy followed by surgical resection of residual tumorQ • Disseminated disease: ChemotherapyQ (Cyclophosphamide, vincristine, dacarbazine, doxorubicin, Cisplatin) Prognosis • Shimada classificationQ describes prognosis based on the degree of differentiation, mitosis-karyorrhexis index, presence or absence of Schwannian stromaQ. International Neuroblastoma Staging System Stage

Definition

1

Localized tumor with complete gross excision, with or without microscopic residual disease; representative ipsilateral LNs negative for tumor microscopically (nodes attached to and removed with the primary tumor may be positive)

2A

Localized tumor with incomplete gross excision; representative ipsilateral nonadherent LNs negative for tumor microscopically

2B

Localized tumor with or without complete gross excision, with ipsilateral nonadherent LNs

3

Unresectable unilateral tumor with contralateral regional LN involvement; or midline tumor with bilateral extension by infiltration (unresectable) or by LN involvement

4

Any primary tumor with dissemination to distant LNs, bone, bone marrow, liver, skin, and/or other organs (except as defined for stage 4S)

4S

Localized primary tumor (as defined for stage 1, 2A, or 2B), with dissemination limited to skin, liver, and/or bone marrow (limited to infant 14 mg/day, associated with: (PGI Dec 2002) a. Medullary carcinoma thyroid b. Von-Hippel Lindau disease c. Sturge-Weber syndrome d. Grave’s disease e. Neurofibromatosis





39. Palpation on the costovertebral angle produces pain and tenderness in acute adrenal insufficiency. This is: a. Rotch’s sign b. Rossolimo’s sign c. Rogoff’s sign d. Osler’s sign



40. Commonest cause of Cushing syndrome is:  a. Adrenal adenoma b. Carcinoma c. Hyperplasia d. Atrophy

30. Which of the following is true about parathyroid? a. Post parathyroid glands are within junction of inferior thyroid artery and RLN b. Most common location of ectopic parthyroid glands is paraesophageal c. Lower parathyroid is anterior to RLN d. All of the above

(Kerala 95)

Parathyroid and Adrenal Glands





45. True about adrenocortical carcinoma: (Recent Question 2016) a. Rare tumor b. More than half are functional c. Most commonly associated with Cushing syndrome d. All of the above 46. A 50-year-old male presents with severe refractory hypertension, weakness, muscle cramps and hypokalemia, the most likely diagnosis is: (COMEDK 2011) a. Hypoaldosteronism b. Hyperaldosteronism c. Cushing syndrome d. Pheochromocytoma



47. Which one of the following is not a CT feature of adrenal adenoma? (AIIMS Nov 2010) a. Low attenuation b. Homogeneous density and well defined borders c. Enhances rapidly, contrast stays in it for relatively longer time and washes out late d. Calcification is rare



48. After bilateral adrenalectomy, patient developed gradual loss of vision, with hyperpigmentation of skin, and headache. Likely cause is: (MHSSMCET 2006) a. Addison’s disease b. Nelson’s syndrome c. Cushing’s disease d. Hypopituitarism



49. Nonfunctional adrenal tumors are operated at what size: a. >3 cm b. >5 cm (MHSSMCET 2009) c. >6 cm d. >10 cm



50. In renal agenesis, the adrenal gland is: a. Absent b. Present on contralateral side c. Ectopic in the iliac fossa d. Present at the usual location



51. Most prevalent incidentacoma is: (MHCET 2016) a. Cushing’s adenoma b. Pheochromocytoma c. Adrenocortical carcinoma d. Non-functioning adenoma



54. The investigation of choice for extra adrenal pheochromocytoma: (PGI Dec 2007) a. MIBG scan b. MRI c. CT d. X-ray e. USG



55. All are true about pheochromocytoma except: a. 90% are malignant (All India 2011) b. 95% occur in the abdomen c. They secrete catecholamines d. They arise from sympathetic ganglions

56. Investigation useful for detecting extra-adrenal pheochromocytoma: (PGI May 2011) a. USG b. CT c. T2-weighted MRI with gadolinium contrast d. MIBG

52. Which one of the following clinical features is not seen in pheochromocytoma? (COMEDK 2011) a. Hypertension b. Episodic palpitations c. Weight loss d. Diarrhea

57. True about pheochromocytoma is: (MHPGMCET 2002) a. Arises from chromaffin cells of adrenal medulla b. Bilateral in 20% of all cases c. Hypotension rules out pheochromocytoma d. Almost always a malignant tumor

58. Pheochromocytoma with malignant potential exclusively secretes: (MHPGMCET 2008) a. Dopamine b. Epinephrine c. Metanephrine d. Norepinephrine

59. False regarding pheochromocytoma: a. 10% of nonfamilial adrenal pheochromocytomas are bilateral (MHSSMCET 2006) b. Only 10% of hypertensive patients have an underlying pheochromocytoma c. 10% of adrenal pheochromocytomas arise in childhood d. FNAC is must for diagnosis



60. Commonest symptom of pheochromocytoma is:  (Recent Question 2015) a. Palpitation b. Headache c. Sweating d. Dyspnea

61. In pheochromocytoma, the urine will contain: a. VMA b. HIAA c. Both d. None

(UPPG 96)



62. False statement about pheochromocytoma: a. 10% are bilateral (All India 97) b. Arises from chromaffin cells c. Extra adrenal tumor - increased nor adrenaline levels d. Increased VMA levels in urine



63. Radionuclide used in pheochromocytoma is: a. Radioactive iodine I-131 (COMEDK 2005) b. Technitium pertechnate c. Radiolabelled chromium d. I123 Metaiodobenzylguanidine (MIBG)

(MHSSMCET 2006)

PHEOCHROMOCYTOMA

53. Episodic hypertension is a feature of: (JIPMER 2010) a. Carcinoid tumor b. Insulinoma c. Pheochromocytoma d. Zollinger-Ellison syndrome

64. The most common site of ectopic pheochromo-cytoma is:  (COMEDK 2008) a. Organ of Zuckerkandl b. Bladder c. Filum terminale d. Celiac plexus

Endocrine Surgery





Section 1



41. Most common cause of Addison’s disease in India:  (AIIMS Nov 2011) a. Tuberculosis b. Post-partum c. Autoimmune d. HIV 42. Indication for surgery in a case of adrenal incidentaloma: a. Size >5 cm (MAHE 2008, 2007) b. Bilateral adrenal metastasis c. Functional tumor d. All of the above 43. Incidental finding in CT scan, a 3 cm adrenal mass, which of the following is not done? (UPPG 2008) a. Adrenalectomy b. Dexamethasone suppression test c. Measurement of catecholamines d. Midnight plasma cortisol 44. Accidental finding of incidentaloma (Adrenal mass) on USG is detected. Following is/are to be ruled out: (PGI Dec 2006) a. Cushing’s disease b. Metastasis c. Adrenal adenoma d. Carcinoma e. Adrenal hyperplasia

97

98

Surgery Essence

Section 1

Endocrine Surgery



65. A patient presented with headache and flushing. He has a family history of his relative having died of a thyroid tumour. The investigation that would be required for this patient would be: (AIIMS June 99) a. Chest X-ray b. Measurement of 5 HIAA c. Measurement of catecholamine d. Intravenous pyelography

66. Investigation of choice in case of a patient with episodic hypertension, headache and thyroid nodule: (AIIMS Nov 97) a. Urinary HIAA b. Urinary catecholamine and aspiration of nodule c. Thyroid function test only d. Urinary basic amino acid metabolite

67. Best way to localize extra-adrenal pheochromocytoma: (MCI June 2018) a. X-ray b. Nucleotide scan c. VMA excretion d. Clinical examination

NEUROBLASTOMA

68. Neuroblastomas: Good prognostic factor is: (PGI June 2000) a. N-myc amplification b. RAS oncogene c. Hyperdiploidy d. Translocations



69. Opsoclonus-Myoclonus is a phenomenon seen in: (PGI 97) a. Wilm’s tumor b. Neuroblastoma c. Meningioma d. Cortical tuberculoma



70. All of the following are correct about neuroblastoma except: a. Arises from adrenal cortex  (Recent Question 2017) b. Can cause paraplegia c. May cause hypertension d. Secretes hormones



71. Not seen in neuroblastoma is: (UPPG 96) a. Diarrhea b. Proptosis c. Splenomegaly d. Bone involvement

72. True about neuroblastoma: (PGI Dec 2006) a. Seen in adrenal glands b. ↑ VMA/HVA c. Lymphatic metastasis more common than blood metastasis d. Presents with abdominal mass e. Old age presentation implies good prognosis 73. Which of the following statements about neuro-blastoma is not true? (All India 2009) a. Most common extra cranial solid tumor in childhood b. >50% present with metastasis at time of diagnosis c. Lung metastasis are common d. Often encase aorta and its branches at time diagnosis 74. Mrs. Neena noted an abdominal mass in left side of her 6 months old child, which showed calcification near the left kidney. What will be the cause? (AIIMS Nov 2000) a. Leukemia b. Neuroblastoma c. RCC d. Lymphoma 75. Tumor arising from olfactory nasal mucosa is: a. Nasal glioma (All India 2012) b. Adenoid cystic carcinoma c. Nasopharyngeal carcinoma d. Esthesioneuroblastoma

Explanations MULTIPLE ENDOCRINE NEOPLASIA

1. Ans. d. MEN-2B: (Ref: Schwartz 10/e p289; Sabiston 20/e p993, 1003, 1006; Bailey 27/e p856-857; Harrison 20/e p2752, 19/e p2335) MEN-1 (WerMer’s Syndrome)

Parathyroid Adenoma

Pancreatic NET (PGI)

Pituitary Adenoma

• MC endocrine abnormality (>98% of affected individuals) in MEN-1 is multiglandular parathyroid tumorsQ. • Hyperparathyroidism is MC manifestation (cardinal sign of MEN-1 is parathyroid adenoma of multicentricity) Q. • Hypercalcemia is first biochemical abnormalityQ detected in MEN-1 and may precede the clinical onset of a pancreatic NET or pituitary neoplasm by several years.

• Pancreatic NET is 2nd MC manifestationQ. • Nonfunctioning or that secrete pancreatic polypeptideQ are MC pancreatic NET in MEN-1. • MC functional NET in patients with MEN-1 is gastrinomaQ followed by insulinoma. • MC increased pancreatic hormone: Pancreatic polypeptides >Gastrin >InsulinQ (PGI)

• Prolactinoma is most commonQ. • Diagnosed by increased prolactin (>200 µg/l) and MRI.

MEN-2A (Sipple syndrome MEN) Medullary carcinoma thyroidQ PheochromocytomaQ Parathyroid hyperplasia or adenomaQ Hirschprung’s diseaseQ Cutaneous lichen amyloidosisQ

MEN-2B (MEN-3)

Components

• • • • •

• • • • • •

Medullary carcinoma thyroidQ PheochromocytomaQ Intestinal ganglioneuromasQ Mucosal neuromasQ MegacolonQ Marfanoid featuresQ

Defect

• RET oncogene (cysteineQ codon) • Chromosome: 10Q

• RET oncogene (tyrosine kinaseQ domain) • Chromosome: 10Q

Transmission

• Autosomal dominantQ

• Autosomal dominantQ



2. Ans. a. ↑ VMA in urine, c. Hypergastrinemia, d. Hyperprolactinemia (Ref: Schwartz 10/e p289; Sabiston 20/e p993; Bailey 27/e p856-857; Harrison 20/e p2747, 19/e p2335)



3. Ans. a. Parathyroid (Ref: Schwartz 10/e p289; Sabiston 20/e p993; Bailey 27/e p856-857; Harrison 20/e p2747, 19/e p2335)



4. Ans. a. Pheochromocytoma, b. Hyperparathyroidism, d. Medullary carcinoma of thyroid



5. Ans. c. MEN-2B



6. Ans. a. Prophylactic surgery (Ref: Schwartz 10/e p1550; Sabiston 20/e p1008) Prophylactic Thyroidectomy in RET Mutation Carriers MEN-2A

Before 5 yearsQ

MEN-2B

Before 1 yearQ



7. Ans. a. Hyperparathyroidism



8. Ans. a. Gastrinoma (Ref: Sabiston 20/e p1000; Schwartz 10/e p1071; Bailey 27/e p857)



9. Ans. c. Medullary carcinoma (Ref: Sabiston 20/e p998; Schwartz 10/e p289; Bailey 27/e p857)



10. Ans. a. 1 year (Ref: Sabiston 20/e p1008; Schwartz 10/e p1550; Bailey 27/e p858)

100

Surgery Essence PARATHYROID GLAND

11. Ans. a. Parathyroid hyperplasia, b. Adenosis, c. MEN-1 (Ref: Schwartz 10/e p1559-1563; Sabiston 20/e p927; Bailey 27/e p825; Harrison 20/e p2925, 19/e p2470-2475)



12. Ans. c. Removal of 3½ glands (Ref: Harrison 20/e p2928, 19/e p2473-2475; Schwartz 10/e p156; Sabiston 20/e p931)

Treatment of Primary Hyperparathyroidism

Endocrine Surgery

• Initial correction of hypercalcemia (Rapid IV Normal saline with furosemide)Q • Neck exploration is done and treatment is done accordingly A single parathyroid adenoma (85%)Q

• ResectionQ

Two adenomas (5%)

• ResectionQ

Hyperplasia of all four glands (10-15%)

• Resection of 3½ glandsQ • Resection of all four glands with autotransplantation of a parathyroid gland in the forearm (brachioradialis) or SCM muscleQ

Parathyroid Autotransplantation • Whenever multiple parathyroids are resected, it is preferable to cryopreserve tissue, so that it may be autotransplanted should the patient become hypoparathyroidQ. • Approx. 12–14 pieces of 1 mm are transplanted into the nondominant forearm in belly of brachioradialisQ muscle

13. Ans. c. Inferior parathyroid lobe (Ref: Harrison 20/e p2925, 19/e p2470) • Parathyroid adenomas are most commonly located in inferiorQ parathyroid glands.



14. Ans. d. Identifying hyperplasia of all 4 glands at surgery in parathyroid hyperplasia (Ref: Harrison 20/e p2928, 19/e p2470)

• Parathyroid adenoma can be differentiated from hyperplasia only at the time of surgeryQ. • In case of adenoma, only one gland is found to be enlargedQ, the other three are normal. • In hyperplasia, all four glands are enlargedQ.

15. Ans. d. All of the above (Ref: Schwartz 10/e p1560; Sabiston 20/e p937; Bailey 27/e p835)



16. Ans. b. Removal of adenoma



17. Ans. d. Ultrasound-guided alcohol injection of the mass (Ref: Harrison 20/e p2929, 19/e p2470)

• Patient is a case of recurrent hyperparathyroidism, as she was operated previously for parathyroid adenoma. • In the setting of recent myocardial infarction, CHF and atrial fibrillation, any operation carries a high risk. • Ultrasound-guided alcohol injection in the mass is preferred in this setting.

18. Ans. a. Single adenoma



19. Ans. c. Carcinoma breast (Ref: Harrison 20/e p2925, 19/e p2470) • Parathyroid adenoma is the MC cause of hypercalcemiaQ. • Malignant tumors are the MC cause of hypercalcemic crisis, of which CA breastQ is the common cause.



20. Ans. b. Following surgery (Ref: Schwartz 10/e p1574; Sabiston 20/e p936; Bailey 27/e p825)

Section 1

Parathyroid Insufficiency or Hypoparathyroidism • Mostly due to removal of the parathyroid glands or infarction due to vascular injuryQ. • Vascular injuryQ is more important. • Cases usually present 2–5 days after operationQ with symptoms of hypocalcemia (circumoral and fingertip numbness and tingling tetany, carpopedal spasm and laryngeal stridor) Q • Treatment with oral calcium and vitamin D supplementsQ • IV calcium gluconateQ may be required in severe cases.

21. Ans. a. After thyroid surgery, b. DiGeorge syndrome, c. Radical resection of head & neck cancer (Ref: Harrison 20/e p2935; Schwartz 10/e p1574; Sabiston 20/e p925)

Parathyroid and Adrenal Glands

101

22. Ans. b. Increased uptake by bones (Ref: Schwartz 10/e p73)



• Hypocalcemia in immediate postoperative period following excision of parathyroid adenoma is due to increased uptake by bonesQ. • It is known as Hungry Bone Syndrome 23. Ans. b. Parathyroidectomy for removal of adenoma, c. Thyrocalcitonin (Ref: Harrison 20/e p2935, 19/e p2478)



Hypercalcemic Crisis • Patients with PHPT may occasionally present acutely with nausea, vomiting, fatigue, muscle weakness, confusionQ, and a decreased level of consciousness; a complex referred to as hypercalcemic crisis.

Section 1

Hungry Bone Syndrome

• These symptoms result from severe hypercalcemia from uncontrolled PTH secretion, worsened by polyuria, dehydration, and reduced kidney functionQ and may occur with other conditions causing hypercalcemia. • Calcium levels are markedly elevated and may be as high as 16 to 20 mg/dLQ. • Parathyroid glands tend to be large or multiple, and the tumor may be palpable. • Patients with parathyroid cancer or familial HPT are more likelyQ to present with hypercalcemic crisis. Treatment • Treatment consists of therapies to lower serum calcium levels followed by surgery to correct HPT. • Mainstay of therapy: Rehydration with a 0.9% saline and diuresis with furosemideQ • Other drugs used to lower serum calcium levels: − Bisphosphonates, CalcitoninQ    − Mithramycin (plicamycin), Gallium nitrateQ − Glucocorticoids (Hydrocortisone)Q

24. Ans. d. Osteosclerosis

25. Ans. b. Thyroidectomy



27. Ans. c. Hyperparathyroidism

28. Ans. d. Total thyroidectomy (Ref: Harrison 20/e p2937, 19/e p314)

26. Ans. d. All of the above

Chronic renal failure, Hypoparathyroidism, Pseudo hypoparathyroidism are causes of hypocalcemia.

29. Ans. d. All of the above (Ref: Schwartz 10/e p1572-1573; Sabiston 20/e p927-928; Bailey 27/e p833; Harrison 20/e p2933, 19/e p2478-2479)



30. Ans. d. All of the above (Ref: Schwartz 10/e p1568; Sabiston 20/e p929-930; Bailey 27/e p829)

Identification of Parathyroids • Approximately 85% of the parathyroid glands are found within 1 cm of the junction of the inferior thyroid artery and RLNsQ.

Lower parathyroid glands

• • • •

Superior to junction of inferior thyroid artery and RLNsQ Dorsal (posterior) to RLNQ Inferior to junction of inferior thyroid artery and RLNsQ Ventral (anterior) to RLNQ

• The thin fascia overlying a “suspicious” fat lobule should be incised using a sharp curved hemostat and scalpel. This maneuver often causes the parathyroid gland to “pop” out. • Alternatively, gentle, blunt peanut sponge dissection between the carotid sheath and the thyroid gland often reveals a “float” sign, suggesting the site of the abnormal parathyroid gland. • Normal parathyroids are light beige and only slightly darker or brown compared to adjacent fat. • MC location of ectopic parathyroid gland: ParaesophagealQ >Mediastinal >Intrathymic

31. Ans. b. Heterotopic



32. Ans. a. Brachioradialis (Ref: Schwartz 10/e p1569; Sabiston 20/e p936; Bailey 27/e p830)

“Total parathyroidectomy requires complete resection of all glands combined with immediate heterotopic transplantation of several 1- to 3-mm slices of fresh parathyroid tissue into individual pockets created in the brachioradialis muscle of the nondominant forearm.”Sabiston 20/e p936



33. Ans. b. Sestamibi scan (Ref: Sabiston 20/e p930; Schwartz 10/e p1565; Bailey 27/e p827)



34. Ans. d. MRI (Ref: Sabiston 20/e p930; Schwartz 10/e p1565; Bailey 27/e p828)

ADRENAL GLANDS

35. Ans. d. Excision of both adrenal glands (Ref: Schwartz 10/e p1590-1590) • Patients undergoing surgical treatment of endogenous hypercortisolism require glucocorticoid replacement. • Steroids are not given pre-operatively because these patients are already hypercortisolemic. • Instead hydrocortisone 100 mg IV is given after the removal of second hyperplastic adrenal gland.

Endocrine Surgery

Upper parathyroid glands

102

Surgery Essence

36. Ans. a. Hyperparathyroidism (Ref: Harrison 20/e p2923; Sabiston 20/e p925) 37. Ans. c. Estimation of urinary metanephrines, VMA and catecholamines

The combination of symptoms suggests MEN-2A (Sipple syndrome), so the patient should be investigated for pheochromocytoma by estimation of urinary metanephrines, VMA and catecholamines. MEN-2A (Sipple Syndrome)

Endocrine Surgery

• Medullary carcinoma thyroidQ • Hirschprung’s diseaseQ

• PheochromocytomaQ • Cutaneous lichen amyloidosisQ

• Parathyroid hyperplasia or adenomaQ

38. Ans. a. Medullary carcinoma thyroid, b. Von-Hippel Lindau disease, c. Sturge-Weber syndrome, e. Neurofibromatosis (Ref: Schwartz 9/e p1399-1400; Sabiston 20/e p985; Bailey 26/e p784-785) Syndromes Associated with Pheochromocytoma (MVVS) • MEN-2A and MEN-2BQ • VHL syndromeQ



• Von-Recklinghausen syndromeQ • Sturge-Weber syndromeQ

39. Ans. c. Rogoff’s sign (Ref: www.medhelp.org)

Rogoff’s Sign • Costovertebral angle pain and tenderness in acute adrenal insufficiency is known as Rogoff’s signQ.

40. Ans. c. Hyperplasia (Ref: Harrison 20/e p2724, 19/e p2314) • MC cause of Cushing’s syndrome is iatrogenic exogenous administration of steroidsQ. • MC endogenous cause of Cushing’s syndrome is bilateral adrenal hyperplasiaQ secondary to hypersecretion of ACTH from pituitary or from an ectopic non-pituitary source. Causes of Cushing’s Syndrome Endogenous ADRENAL HYPERPLASIA • Pituitary ACTH overproduction: −− MicroadenomaQ >Macroadenoma • Ectopic ACTH overproduction: • Small cell carcinoma lungQ − Thymus carcinoid −− Carcinoma pancreas −− Bronchial adenoma ADRENAL NEOPLASIA • AdenomaQ



Exogenous • Iatrogenic exogenous administration of steroids (MC cause) Q

• CarcinomaQ

41. Ans. a. Tuberculosis (Ref: ASI 7/e p1073)

Section 1

• Most common cause of adrenal insufficiency (Addison’s disease) in developing countries is TuberculosisQ followed by autoimmune disorders.

42. Ans. d. All of the above (Ref: Schwartz 10/e p1589; Sabiston 20/e p985-987; Bailey 27/e p839; Harrison 20/e p2731, 19/e p2321)



43. Ans. a. Adrenalectomy



44. Ans. All (Ref: CSDT 11/e p811-812; Bailey 27/e p839; Schwartz 10/e p1589; Sabiston 20/e p986; Harrison 20/e p2731, 19/e p2321)



45. Ans. d. All of the above (Ref: Schwartz 10/e p1588; Sabiston 20/e p979-980; Bailey 27/e p843; Harrison 20/e p2732, 19/e p2322)



46. Ans. b. Hyperaldosteronism (Ref: Harrison 20/e p2729, 19/e p1618-1619) Manifestation of Conn’s Syndrome (Primary Hyperaldosteronism) Clinical Laboratory • Muscle weakness and fatigue (due to • Hypokalemia hypokalemia) • Metabolic alkalosis • Hypertension without edema • Hypernatremia • Polyuria and polydipsia • Increased Aldosterone • Low rennin



47. Ans. c. Enhances rapidly, contrast stays in it for a relatively longer time and washes out late (Ref: Grainger Radiology 4/e p1388; Dahnert Wolfgang Radiology Review Manual 6/e p919) • Adrenal adenoma on contrast enhanced CT/MRI show rapid uptake and relatively rapid washout of contrastQ material than do non adenomas.

Parathyroid and Adrenal Glands

103

CT Features of Adrenal Adenoma



Well defined /sharply definedQ 50% of patientsQ RareQ CharacteristicQ Common (Stippled calcificationQ)

Wilm’s Tumor MC renal tumor in children Generally confined to the kidney Common Uncommon Rare



74. Ans. b. Neuroblastoma



75. Ans. d. Esthesioneuroblastoma (Ref: Dhingra 5/e p217-218; Washington Manual of Surgical Pathology 2/e p45; Harrison 20/e p656, 19/e p199-e4)

SECTION

2

Hepatobiliary Pancreatic Surgery CH A PT ERS ˆˆ ˆˆ ˆˆ ˆˆ ˆˆ

Liver Portal Hypertension Gallbladder Bile Duct Pancreas

CHAPTER

4

Liver

PYOGENIC LIVER ABSCESS Pyogenic Liver Abscess • • • • •

Liver is MC site of abdominal visceral abscess PLA accounts for majority of hepatic abscessesQ Higher incidence of cryptogenic abscess occur in Asian seriesQ No significant gender, ethnic or geographic differences in disease frequencyQ Associated comorbid conditions: Cirrhosis, CRF, history of malignancy Q

E. coli MC in western countriesQ Klebsiella pneumoniae MC in Asian countriesQ Staphylococcus MC in children, suffering from chronic granulomatous diseaseQ • Multiple abscesses occur in patients with a biliary originQ • Solitary abscesses are more likely than multiple abscesses to be polymicrobial. Routes of Infection in PLA • Biliary tract (MC):Q −− CBD stones leading to cholangitis (in Asia)Q −− Hilar cholangiocarcinoma in western countriesQ −− CBD strictures • Portal vein (2nd MC) Q • Hepatic artery: Q −− Hematogenous spread, usually monomicrobial, staphylococcus or streptococcus • Direct extension: −− From subdiaphragmatic abscess −  From suppurative cholecystitis −− From empyema in chest −  From perinephric abscess • Penetrating or blunt trauma • Cryptogenic Clinical Features • MC presenting symptom is feverQ. • MC LFT abnormality is an elevation of ALPQ. • Classic presentation: Fever, jaundice (25%)Q, and right upper quadrant pain & tenderness • Fever, chills, & abdominal pain are the most common presenting symptoms • Usually single, involve right lobeQ • Malignancy, jaundice, deranged LFT and sepsis are associated with poor prognosis. • • • •

Endogenous Endophthalmitis in PLA A rare complication specific to KlebsiellaQ hepatic abscesses Occurring in 3% of cases. More common in diabetic patientsQ. Early diagnosis and treatment represent the best chance to preserve visual functionQ.

Diagnosis • USG & CT are the main diagnostic modalitiesQ • Diagnosis is confirmed by aspiration & cultureQ • CXR: Elevated hemidiaphragm, right sided pleural effusion or atelectasis Treatment • Percutaneous catheter drainage + IV antibiotics has become the treatment of choiceQ. • After 2 weeks of parenteral antibiotics, oral agents should be used for further 4 weeks.

108

Surgery Essence

Hepatobiliary Pancreatic Surgery

AMOEBIC LIVER ABSCESS Amoebic Liver Abscess • Caused by Entamoeba histolytica whose cysts are acquired through the feco-oral routeQ • Trophozoites reach the liver through portal venous systemQ. • Solitary and more common in right lobe of liverQ. • Low incidence of invasive amoebiasis in menstruating womenQ • Majority of patients are young men (may be due to heavy alcohol consumption) Pathogenesis • MC form of invasive disease is colitis, frequently affects the cecum & ascending colonQ • In colon: Flask-shaped ulcersQ (MC site: Cecum & ascending colon) Q • Synchronous hepatic abscess is found in one third of patients with active amebic colitis. Clinical Features • MC symptom is abdominal painQ • Typical clinical picture: Patient of 20-40 yrs of age, with history of travel to endemic area, presents with fever, chills, anorexia, right upper quadrant painQ. • Results from an obligatory colonic infection, a recent history of diarrhea are uncommonQ. • Active colitis and amoebic liver abscess rarely occur simultaneously, as a rule colonic lesions are silentQ • Jaundice is rareQ • Raised PT is MC LFT abnormalityQ. Diagnosis • USG and CT are the main diagnostic modalitiesQ • Diagnosis is confirmed by serological testsQ (ELISA) for antiamoebic antibodies. • Cultures of amoebic abscess are usually sterile or negativeQ. • CXR: Elevated hemidiaphragm, right sided pleural effusion or atelectasis • ALA: Reddish-brown anchovy pasteQ; more reliable characteristic than color is the odour of the fluid. Treatment • Metronidazole (750 mg orally TDS × 10-14 days) is the mainstay of treatment and is curative in over 90% of patientsQ, clinical improvement is seen within 3 days. • Luminal agents include iodoquinol, paromomycin & diloxanide furoateQ. • Average time to radiologic resolution of abscess is 3-9 months Indications of Aspiration in ALA 1. Diagnostic uncertaintyQ 4. High risk of rupture (size >5 cm, left lobe abscess)Q Q 2. Failure to respond to therapy in 3-5 days 5. PregnancyQ (Therapeutic trial with high dose Metronidazole is deemed inappropriate) 3. Pyogenic superinfectionQ Complications

Section 2

• Most frequent complications: Rupture into the peritoneum (MC)Q, pleural cavity, or pericardium. • Size of abscess appears to be the most important risk factor for rupture • Laparotomy is indicated in cases of doubtful diagnosis, hollow viscus perforation, fistulization resulting in hemorrhage or sepsis, and failure of conservative therapy. • Treatment of rupture into the pleural space: ThoracentesisQ • Rupture into bronchi is self-limited with postural drainage & bronchodilatorsQ.

HYDATID DISEASE Hydatid Disease • Hydatid disease is a zoonosis, occurs primarily in sheep-grazing areasQ of the world • Endemic in Mediterranean countries, Middle East, Far East, South America, Australia, New Zealand, & East Africa. • • • •

Humans contract the disease from dogs, and there is no human-to-human transmissionQ. Hydatid cyst is caused by Echinococcus granulosusQ. Other species affecting human beings: E. multilocularis, E. vogelli, E. oligarthusQ Malignant hydatidosis is caused by E. multilocularisQ

Liver

109

Life-cycle

• Sheep: Usual intermediate hostQ • Human: Accidental dead end intermediate hostQ without human to human transmission • In the human duodenum, parasitic embryo releases an oncosphere, that penetrate mucosa, allowing access to bloodstreamQ. • In the blood, oncosphere reaches liver (MC)Q or lungs, develops its larval stage, hydatid cyst. • Organs most commonly involved are: Liver >Lungs >Spleen >Kidney >Brain >BoneQ. Hydatid Cyst • Three weeks after infection, a visible hydatid cyst develops • The cyst wall has two layers: − Ectocyst: outer gelatinous membraneQ −  Endocyst: inner germinal membraneQ • Pericyst: Fibrous capsule derived from host tissues, develops around the hydatid cyst.

Section 2

• Dogs are the definitive hostQ of E. granulosus • Eggs are passed (up to thousands of ova daily) and deposited with the dog’s feces.

• Scoleces develop into an adult tapeworm in definitive hostQ • Scoleces differentiate into a new hydatid cyst in intermediate hostQ • Hydatid sand: Freed brood capsules and scoleces in the hydatid fluid Clinical Features • Equally common in males & females, age of 45 years. • Most (75%) are singular, located in right liver (VII & VIII)Q. • Mostly asymptomaticQ until complications occur • MC presenting symptoms: Abdominal pain, dyspepsia & vomiting. • MC sign: HepatomegalyQ

Diagnosis • USG and CT are the main diagnostic modalitiesQ −− Daughter cyst within the large cyst (rosette appearance) & calcification of wall are highly suggestive of hydatid cystQ. • Diagnosis is confirmed by serological testsQ (ELISA, Immunoblot, Arc-5, IHA) for antibodies. • Ring like calcification on CECT is seen in hydatid cystQ • In cases of suspected biliary involvement (Jaundice), ERCP (gold standard) Q or PTC is necessary. Treatment • Most cysts are treated surgicallyQ • Conservative management is appropriate in elderly patients with small, asymptomatic, densely calcified cystsQ. • Treatment options: PAIR, pericystectomy, marsupialization, leaving the cyst open, drainage of the cyst, omentoplasty, or partial hepatectomy to encompass the cyst. • Currently, PAIR is the preferred method of treatmentQ for anatomically & surgically appropriate lesions Surgery remains the treatment of choice for cysts where • PAIR is not possible or cysts are refractory to PAIRQ • For complicated cysts (communicating with biliary tract)Q • Radical (resection) and conservative (drainage & evacuation) surgical approaches appear to be equally effective at controlling disease. • Pericystectomy is the preferred surgical approachQ (complete cyst with surrounding fibrous tissue are removed) • If surgical cystectomy is not technically feasible, then formal liver resection can be done.

Chemotherapy in Hydatid Disease • Chemotherapy with albendazole or mebendazole is effective at shrinking the cysts • Cyst disappearance occurs in fewer than 50% of patientsQ. • Preoperative treatment may decrease the risk for spillageQ and is a reasonable and safe practice. • Chemotherapy without definitive resection or drainage is only considered for: − Widely disseminated diseaseQ      −  Patients with poor surgical riskQ

Hepatobiliary Pancreatic Surgery

• Complications: Rupture of the cyst into the biliary tree (MC) Q or bronchial tree, or free rupture into peritoneal, pleural, or pericardial cavities. • Intrabiliary rupture is MC complication of hydatid liver cystsQ • Free ruptures can result in disseminated echinococcosis and a potentially fatal anaphylactic reaction.

110

Surgery Essence

Hepatobiliary Pancreatic Surgery

Treatment of E. multilocularis • E. multilocularis cyst is always multiloculatedQ.

•  Treatment is surgical resectionQ.

PAIR PAIR (Puncture, Aspiration of cyst content, Injection of scolicidal agent, and Reaspiration) • Currently, PAIR is preferred method of treatmentQ for anatomically & surgically appropriate lesions • The efficacy of PAIR in managing hydatid cysts is >75%. • During PAIR, patient is given prophylactic coverage of albendazole Scolicidal Agents • • • •

Hypertonic (20%) salineQ: 100% scolicidal with contact time of 6 minutes 0.5% cetrimide with 0.05% chlorhexidineQ Absolute alcoholQ 10% povidone iodineQ

Contraindications of PAIR • • • • • •

Superficially located cystsQ Inaccessible or hazardous locationQ of cyst Cysts with multiple internal septalQ divisions (honeycombing pattern) Dead or inactive cystsQ Cysts communicating with biliary treeQ Lung or brain cystsQ Most Common Symptom

Sign

Pyogenic abscess

FeverQ

RUQ tendernessQ

Raised ALPQ

Amebic abscess

Abdominal pain

Hepatomegaly

Raised PTQ

Hydatid cysts

No symptom or Abdominal painQ

RUQ massQ

Q

Laboratory abnormality Q

Eosinophilia

ACTINOMYCOSIS OF LIVER Actinomycosis of Liver • Most commonly, Actinomyces reaches liver through portal veinQ. • Liver is gradually replaced by multiple abscesses, typical honey comb liverQ Diagnosis • Needle aspiration: Actinomyces in pusQ Treatment • Antibiotic (Penicillin) is the DOCQ.

Section 2

HEPATIC CYST Simple Hepatic Cyst • Contain serous fluid, do not communicate with the biliary tree, and do not have septationsQ • Single in 50% casesQ Treatment • Nonsurgical treatment: Aspiration and injection of a sclerosing agent (most frequently ethanol) Q. • Surgical therapy: Fenestration or unroofing the extrahepatic portion of cystQ. Complication • MC complication: Intracystic bleedingQ.

Liver

111

Polycystic Liver Disease

• Renal function and hepatic cyst load are correlatedQ. • Associated condition: Berry aneurysm and mitral valve prolapseQ. Types • Type I: Few large cysts (>7-10 cm) • Type II: Multiple medium cysts (5-7 cm) • Type III: Diffuse small to medium cysts (hypoglycemiaQ, erythrocytosis, hypercalcemia • Vascular bruit (25%), GI bleed (10%), tumor rupture (2-5%), jaundice due to biliary obstruction (10%), paraneoplastic syndrome (70% of patients with HCC. • Monoclonal antibody HepPar-1 (hepatocyte paraffin-1) identifies a unique antigen on hepatocyte mitochondria and is used to identify hepatocytes or HCC. Diagnosis Non-invasive Diagnostic Criteria for HCC • Focal lesion 1-2 cm: Two imaging techniques with arterial hypervascularization & venous washoutQ. • Focal lesion >2 cm: One imaging technique with arterial hypervascularization & venous washoutQ. • Techniques to be considered: Dynamic CT & MRI. • Screening is based on regular ultrasound scanning in high risk populationQ • Biopsy proof of HCC is not requiredQ Treatment • Complete excisionQ by partial hepatectomy or by total hepatectomy and transplant is the only treatment modality with curative potentialQ. Remember: Only 15-20% of HCC is resectable, because rest of the tumors have • Multicentricity, Bilobar involvementQ       • Portal vein invasion, Lymphatic metastasisQ

Section 2

Okuda Staging System for HCC (BATA)Q Clinical parameter

Cut-off value

Points

Serum Bilirubin

3

0 1

Serum Albumin

>3 5 cm • Lymph node-positive primary lesionQ Q

•  Disease free interval 200 ng/mLQ

• Synchronous liver metastasis is associated with poor prognosis

•  More than one tumorQ

Hepatobiliary Pancreatic Surgery

• Occurs in young adults without underlying cirrhosisQ • Non-encapsulated but well, circumscribed, so high resectability rateQ • Grows slowly and & has better prognosisQ Pathology

116

Surgery Essence Local Ablative therapy for liver secondaries

Hepatobiliary Pancreatic Surgery

Mechanism

Technique

Zone of Necrosis

Advantages

Disadvantages

Freezing

Cryotherapy

3-5 cm

Large zone of necrosis Easily followed by ultrasound

Requires laparotomy Large size probe size

Hyperthermic coagulative necrosis

Radiofrequency or microwave ablation

2 cm

Percutaneous technique

Small size of necrosis

Local injection therapy

Ethanol, acetic acid, chemotherapy, hot saline

3 cm

Simple, inexpensive

Inhomogenous distribution

HEPATOBLASTOMA Hepatoblastoma • • • • •

MC primary hepatic tumor of childhood, more common in malesQ. Low birth weight may represent a risk factorQ. Most cases are sporadic, also associated with Beckwith-Wiedemann syndrome & FAPQ No evidence of association with HBV or HCV infection or any other chronic viral hepatitisQ. These patients usually do not have cirrhosis or inborn errors of metabolismQ.

Pathology • Five histologic subtypes: Fetal, embryonal, mixed mesenchymal, macrotubular, and anaplastic or small cell. Clinical Features • Median age of presentation is 18 months, and almost all cases occur before 3 yearsQ. • MC presenting sign is an asymptomatic abdominal massQ. • Mild anemia & thrombocytosisQ are commonly found at presentation. • Serum AFP levels are elevated in 85-90% of patients and can serve as a useful marker for therapeutic responseQ. Diagnosis • CT scan reveals a vascular mass that is often (50%) speckled with calcificationQ. • To confirm the diagnosis, an initial biopsyQ is required. Treatment • For unresectable tumors, the initial surgical procedure should include a diagnostic biopsy and placement of a vascular access device for chemotherapyQ. • A second laparotomy is performed after four cycles of chemotherapy, if imaging studies show a good response, and the tumor appears resectable. • Neoadjuvant chemotherapy (cisplatin, 5-fluorouracil, vincristine) followed by resectionQ • 50% of patients with pulmonary metastases can be cured with resectionQ of the hepatic tumor and chemotherapy or resection of the pulmonary metastasesQ. Prognosis

Section 2

• Survival appears to be dependent on complete resectionQ. • Long-term survival rates of 60-70% can be expected with complete resectionQ.

EPITHELIOID HEMANGIOENDOTHELIOMA Epithelioid Hemangioendothelioma • • • •

Malignant soft tissue tumor of endothelial originQ. Infantile variety is benign, adult one is malignant & highly aggressiveQ. Female predominance is related to vinyl chloride exposure & OCPsQ. In approximately one half of cases, cutaneous hemangiomas are also present (45%)Q.

Pathology • Factor VIII stainingQ differentiates it from other nonvascular tumors. • Liver parenchymal architecture is preservedQ.

Liver

117

Diagnosis

Treatment • Total hepatectomy and liver transplantationQ (disease is diffuse & multifocal).

BILIARY CYSTADENOMA

Section 2

• Percutaneous biopsyQ is performed for diagnosis. • CT scan: Irregular hypodense lesions that may have hypervascular enhancementQ in the periphery following injection of intravenous contrast.

Cystadenoma • Mostly intrahepaticQ (83%), may occur within the extrahepatic bile ducts (13%) or GB (0.02%). • Located in the right lobe (50%)Q • Cystadenoma with mesenchymal stroma occurs exclusively in young and middle-aged womenQ Pathology • Cyst has a globular external surface with multiple protruding cysts and locules of various sizes. • Fluid content is mucinous. • Lined with columnar epithelium and have papillary infoldings. Clinical Features

Diagnosis • Ultrasound: Cystic structure with varying wall thickness, nodularity, septations, and fluid-filled locules. • CECT: Enhancement of the cyst wall and septaQ • ERCP: Displacement of the intrahepatic bile ducts by the tumor and no communication between the biliary tree and cystadenoma. Treatment • Enucleation for neoplastic cysts with no signs of malignancyQ • Formal hepatic resection for neoplastic cysts with signs of malignancyQ

BILIARY CYSTADENOCARCINOMA Biliary Cystadenocarcinoma • Rare malignancy, typically intrahepaticQ in location. • May arise from preexisting biliary cystadenomas. • FemaleQ to male ratio is 2 : 1; CA 19-9 is raisedQ Pathology • The presence of an associated ovarian-like stromaQ in female patients appears to signify a favorable prognosis • Tends to be multilocular, fluid from the cyst can be blood stained, clear, or bile tinged. • Preoperative cyst aspiration is not recommended because there is a risk for peritoneal tumor seedingQ Clinical Features • Cystadenocarcinoma tends to be larger than cystadenoma, but clinical features are similar Diagnosis • Radiologic findings: Thick or irregular wall, peripheral enhancement, associated mass, or papillary tumor projectionsQ into the cyst cavity. Treatment • The only potentially curative treatment is complete removal, by a major liver resection with clear marginsQ. • Survival rates for this disease have been reported in the range of 25% to 100% at 5 years.

Hepatobiliary Pancreatic Surgery

• Usually presents as a large cystic massQ (10-20 cm). • Mainly affects womenQ older than 40 years. • Majority of patients present with a history of abdominal pain or mass

118

Surgery Essence

Hepatobiliary Pancreatic Surgery

WILSON’S DISEASE Wilson’s Disease • • • •

Wilson’s disease is an autosomal recessiveQ inherited disorder of copper metabolism. Characterized by excessive deposition of copper in the liver, brain, and other tissuesQ. Major physiologic aberration: Excessive absorption of copper from the small intestine and decreased excretion of copper by the liver Q. Defect on chromosome 13q (ATP7B gene)Q

Pathology • Kayser-Fleischer rings consist of electron-dense granules rich in copper and sulfur. • Rings form bilaterally, initially appearing at superior poleQ of cornea, then inferior pole, &, ultimately, circumferentially. Clinical Features • Patients with Wilson’s disease usually present with liver disease during first decade of life or with neuropsychiatric illness during third decadeQ. • Any young patient with unexplained chronic or fulminant liver disease should be investigated for Wilson’s disease Q. • Kayser-Fleischer rings are formed by deposition of copper in Descemet’s membrane in limbus of cornea. The color may range from greenish gold to brownQ • When well developed, rings may be readily visible to the naked eye or with an ophthalmoscope • When not visible to the unaided eye, rings may be identified using slit-lamp examination or gonioscopy. • Kayser-Fleischer rings are observed in up to 90% of individuals with symptomatic Wilson’s disease and are almost invariably present in those with neurologic manifestationsQ. Diagnosis • Approximately 90% of patients have ceruloplasmin levels of 100 mg/dQ (reference range, 250 µg/g of dry weightQ even in asymptomatic patients. Treatment • ZincQ is the treatment of choice for maintenance therapy in Wilson’s disease.

COURVOISIER’S LAW Courvoisier’s Law • In obstruction of the common bile duct due to a stone, distention of gallbladder seldom occurs; the organ usually is shriveledQ. • If there is no disease in the gallbladder and the obstruction is due to cancer of ampulla, pancreas or bile duct, then gallbladder will be distendedQ. Exceptions to Courvoisier’s Law • Double impaction of stonesQ (one in cystic duct & other in CBD). • Oriental cholangiohepatitisQ

•  Pancreatic calculus obstructing the ampulla of VaterQ •  Mucocele due to stone in the cystic ductQ

Section 2

LIVER ANATOMY Functional Anatomical Divisions of Liver • Functional anatomy of the liver is based on Couinaud’s division of liver into eight (subsequently nine) functional segments, based upon the distribution of portal venous branches & location of hepatic veins in the parenchyma (Couinaud 1957). • Segment IX is a recent subdivision of segment I, and describes that part of segment that lies posterior to segment VIIIQ. • Liver is divided into four portal sectors by four main branches of portal vein. These are right lateral, right medial, left medial & left lateral (sometimes the term posterior is used in place of lateral & anterior in place of medial). • Three main hepatic veins lie between these sectors as intersectorial veins. These intersectoral planes are also called portal fissures (scissures). Fissures containing portal pedicles are called hepatic fissures. • Each sector is sub-divided into segments (usually two) based on their supply by tertiary divisions of vascular biliary sheaths.

Liver

Sectors and segments of the liver: • Sectors of liver are made up of between one & three segments: right lateral sector = segments VI & VII; right medial sector = segments V & VIII; left medial sector = segments III & IV (and part of I); left lateral sector = segment II. • Segments are numbered in an ante-clockwise spiral centered on the portal vein with the liver viewed from beneath, starting with segment I up to segment VI, and then back clockwise for the most cranial two segments VII & VIII.

I II III IV

Segmental Nomenclature Caudate lobeQ V Right anterior inferior segment Left lateral superior segment VI Right posterior inferior segment Left lateral inferior segment VII Right posterior superior segment Left medial segment or Quadrate lobeQ VIII Right anterior superior segment

B

A

C

D

Segments of the liver (after Couinaud). (A) superior view; (B) posterior view; (C) anterior view; (D) inferior view.

Hepatobiliary Pancreatic Surgery

The fissures and sectors of the liver. (Right lateral = right posterior; right medial = right anterior)

Section 2

Fissures of the liver: • Three major fissures, not visible on the surface, run through the liver parenchyma and harbor the three main hepatic veins (main, left & right portal fissures)Q. • Three minor fissures are visible as physical clefts of the liver surface (umbilical, venous & fissure of Gans)Q.

119

Multiple Choice Questions LIVER ABSCESS 1.

Most common cause of liver abscess in chronic granulomatous disease: (ILBS 2012) a. Klebsiella b. Staph. aureus c. Peptostreptococcus d. E. coli

2.

Most common cause of liver abscess: (AIIMS GIS May 2011) a. E. coli b. Proteus c. Klebsiella d. Staphylococcus

3. 

4.

Which of the following is the most common cause of pyogenic liver abscess? (All India 2012, AIIMS GIS Dec 2009) (Recent Question 2014) a. Trauma b. Biliary tract infection c. Colonic Diverticulitis d. Appendicitis All are true about pyogenic liver abscess except:  (JIPMER GIS 2011) a. Most common route of infection is biliary tree b. Most common site is right lobe c. Klebsiella is most common in gas forming abscess d. Percutaneous drainage is least cured

12.

True statement regarding pyogenic liver abscess is/are:  (PGI May 2018) a. More common on left side of liver b. Surgical drainage is the treatment of choice c. Most common organism responsible is E. coli d. X-rays are diagnostic e. Diagnosis is confirmed by aspiration and culture

13.

Indications for needle aspiration in liver abscess are: a. Recurrent (PGI June 2006) b. Left lobe c. Refractory to treatment after 48–72 hours d. >10 cm size e. Multiple

14.

True treatment regarding hepatic amoebiasis:  a. More common in females b. Multiple lesions c. Mostly treated conservatively d. Jaundice is common

15.

Which one of the following is not corrected regarding amoebic liver abscess? (ICS 2005) a. Its usual occurrence is in the right lobe b. Patient is toxic c. Surgical drainage is always indicated d. Extension of abscess from liver to pericardium is the most dreaded complication All are used in treatment of amoebic liver abscess except: a. Diloxanide furoate  (MAHE 2007) b. Chloroquine c. Metronidazole d. Emetine

(PGI 96)

5.

All are true about amoebic liver abscess except: a. Metronidazole is mainstay of treatment (PGI Nov 2010) b. Multifocal abscess can not be treated by aspiration c. More common in left side d. More common in female

6.

Liver abscess ruptures most commonly in: (AIIMS GIS 2003) a. Pleural cavity b. Peritoneal cavity c. Pericardial cavity d. Bronchus

7.

True about amebic liver abscess: (AIIMS GIS May 2008) a. Male:female >10:1 b. Not predisposed by alcohol c. More common in diabetics d. E. histolytica is isolated in >50% from blood culture

17.

Not true about amebic liver abscess is:  a. Adult forms are seen b. Conservative treatment is generally seen c. Larvae are seen d. USG can diagnose it

8.

Not an indication for percutaneous aspiration in amebic liver abscess: (AIIMS GIS May 2008) a. Radiographically unresolved lesion after 6 months b. Suspected diagnosis c. Left lobe liver abscess d. Compression or outflow obstruction of hepatic or portal vein

18.

9.

Commonest cause of pyogenic liver abscess: (AIIMS Sept 96) a. Aspiration b. Hematogenous spread from a distant site c. Direct contact d. Lymphatic spread

A patient with 8 cm × 8 cm abscess in right lobe of liver was treated with aspiration multiple times (3 times) and with systemic amebicide. Now cavity is remaining in right lobe of liver but there is nothing in the cavity. Seven days course of luminal amebicides is given. How will you follow-up? a. Stool examination only (AIIMS Nov 2012) b. USG weekly for 1 month followed by monthly USG till 1 year c. USG weekly for 3 months followed by CT scan at 3 months d. USG or CT scan monthly and stool examination weekly

19.

A young patient presents to the emergency department with fever and right upper quadrant pain. Clinical examination reveals obvious hepatomegaly but there is no jaundice. Ultrasound reveals a solitary, homogeneous, hypoechoic lesion in the right lobe measuring 5 cm × 5 cm × 4 cm. Tests for hydatid disease were negative. Which of the following is the best recommendation for initial treatment? (All India 2011) a. Multiple aspirations and antiamoebics/antibiotics b. Catheter drainage and antiamoebics/antibiotics c. Antiamoebics/antibiotics alone d. Hepatectomy followed by antiamoebics/antibiotics

10.

Anchovy sauce pus is a feature of: (All India 99) a. Amebic liver abscess b. Lung abscess c. Splenic abscess d. Pancreatic abscess

11.

In pyogenic liver abscess commonest route of spread is:  (AIIMS Nov 98, AIIMS Nov 95) a. Hematogenous through portal vein b. Ascending infection through biliary tract c. Hepatic artery d. Local spread

16.

(DPG 2006)

Liver 20.

HYDATID CYST 21.

Water lily appearance in a chest radiograph suggests:   (COMEDK 2005, 2004) a. Metastasis b. Cavitating metastasis c. Aspergilloma d. Ruptured hydatid cyst



22.

Which of the following is not a sign of pulmonary hydatidosis? (COMEDK 2010) a. Water lily sign b. Rising sun sign c. Meniscus sign d. Drooping lily sign

23.

Ultrasound was performed in a patient of hydatid cyst. What is the name of this sign? a. Water lily sign b. Honeycomb sign c. Spoke wheel sign d. Triradiate sign

False about hydatid cyst of liver: (PGI Nov 2010) a. Hepatic resection is never done b. Laparoscopic aspiration of cyst is performed c. Most commonly located in the right liver d. Mostly asymptomatic e. Most common causative organism is Echinococcus granulosus

25.

Not an indication for PAIR treatment in hydatid cyst:   (PGI Nov 2010) a. Size >5 cm b. Multiloculated c. Cyst in lung d. Recurrence after surgery e. Perforated cyst

26.

Capitonnage is used in treatment of: (MHSSMCET 2008) a. Choledochal cyst b. Dermoid cyst c. Hydatid cyst d. Renal cyst

27.

A 40-year-old female presented with abdominal discomfort, dyspepsia and palpable abdominal mass. USG and CECT was performed. What is the diagnosis based on this ultrasound image? a. Hydatid cyst b. Multiple HCC c. Liver secondaries d. Polycystic liver disease

In the treatment of hydatid cyst, PAIR is contraindicated in: a. Lung cyst (PGI Dec 2006) b. Size > 5 cm c. Not amenable to treatment with albendazole d. Multiple e. Inaccessible location

29.

A 40-year old male presents with a painless cystic liver enlargement of four years duration without fever or jaundice. The most likely diagnosis is: (UPSC 96) a. Amoebic liver abscess b. Hepatoma c. Hydatid cyst of liver d. Choledochal cyst

30.

All are complications of hydatid cyst in the liver except: a. Jaundice b. Suppuration (APPG 97) c. Cirrhosis d. Rupture

31.

During surgical exploration for hydatid cyst of the liver, any of the following agents can be used as scolicidal agent except: (UPSC 2004) a. Hypertonic sodium chloride b. Formalin c. Cetrimide d. Povidone iodine

32.

Ring like calcification on CECT is seen in:  (Recent Question 2016) a. HC b. FNH c. Liver metastasis d. Hydatid cyst

33.

Which of the following is true about hydatid cyst of liver?  (Recent Question 2014, DPG 2007) a. Surgical management is done always b. Conservative treatment is effective c. Aspiration is safe d. E. multilocularis is the most common cause

34.

Treatment of hydatid cyst: (APPG 2008) a. Excision of cyst b. Percutaneous drainage c. Conservative management d. None

Hepatobiliary Pancreatic Surgery

24.

28.

Section 2

Flask shaped ulcers in colon is caused by:  (Recent Question 2019) a. Giardia lamblia b. Entamoeba histolytica c. H. pylori d. Enterobius vermicularis

121

Surgery Essence

Hepatobiliary Pancreatic Surgery

122

35.

Investigation of choice for hydatid disease is: a. CT scan b. ELISA (MCI Sept 2009) c. Biopsy d. USG

36.

Medical management of hydatid disease is indicated in a. Pregnancy b. Infected hydatid cyst c. Moribund patients d. Multiple peritoneal cyst

37.

The sensitivity of Casoni’s test is: (Recent Question 2016) a. 50% b. 60% c. 75% d. 90%

38.

Malignant hydatidosis is caused by: a. Echinococcus granulosus b. Echinococcus multilocularis c. Echinococcus vogelli d. Echinococcus oligarthus

39.

40.

45.

46.

47.

(Recent Question 2016)

FOCAL NODULAR HYPERPLASIA 48.

Malignant hydatid disease is caused by:  (Recent Question 2017) a. E. granulosus b. E. oligarthus c. E. vogelli d. E. multilocularis

49.

What is the name of this sign seen on CT chest?  (Recent Question 2017) a. Rising sun sign b. Water lily sign c. Meniscus sign d. Serpent sign



Section 2

HEPATIC ADENOMA



(ILBS 2012)

41.

False about hepatic adenoma: a. Chances of malignancy in 10% cases b. Rupture in 20% cases c. Hepatic architecture is maintained d. Most patients are symptomatic

42.

All are true about liver adenoma except: (AIIMS GIS May 2011) a. Normal liver architecture b. Increased fat c. Increased glycogen d. Cells arranged in cords

43.

About hepatic adenoma, all are true except: a. Increased glycogen and fat in hepatocytes b. Normal liver architecture  (AIIMS GIS Dec 2009) c. Bile ductules are not seen d. Tumor markers are normal

44.

Most common liver tumor in those on OCPs: (MHSSMCET 2007) a. HCC b. Liver cell adenoma c. Bile duct adenoma d. Focal nodular hyperplasia All are true about hepatic adenoma except: a. Usually multiple (JIPMER GIS 2011) b. OCP is a predisposing factor c. Has cords of benign hepatocytes d. 50-75% are symptomatic Which of the following liver tumors always merit surgery? (DPG 2009 March) a. Hemangioma b. Hepatic adenoma c. Focal nodular hyperplasia d. Peliosis hepatis

Most common cause of non-traumatic hemo-peritoneum: a. Hepatic adenoma b. FNH (AIIMS GIS 2003) c. HCC d. Hemangioma

50.

51.

52.

53.

Central stellate scar is seen in: (AIIMS GIS May 2011) a. FNH b. Hemangioma c. Hepatic adenoma d. HCC Radiographic image of a benign tumor is given below. What is the most probable diagnosis? a. Hemangioma b. Focal nodular hyperplasia c. Hepatic adenoma d. Peliosis hepatis

All of the following are true regarding FNH except: (AIIMS GIS Dec 2010) a. Not frequently associated with OCPs b. Surgical resection is required due to risk of malignancy c. Stellate scar is diagnostic d. Typical hepatic vascularity is not seen with spoke wheel pattern True statement regarding focal nodular hyperplasia: a. More common in young women (PGI Nov 2011) b. Associated with OCP use c. May present with abdominal pain d. Excision biopsy may aid in diagnosis e. Progress to cirrhosis Similarity between FNH and hepatic adenoma are all except: (AIIMS GIS 2003) a. Hemoperitoneum is common b. Biliary abnormalities are seen c. More common in females d. Associated with OCPs Which one of the following hepatic lesions can be diagnosed with high accuracy by using nuclear imaging? (AIIMS Nov 2004) a. Hepatocellular carcinoma b. Hepatic adenoma c. Focal nodular hyperplasia d. Hemangioma

Liver HEMANGIOMA



55.

56. 57. 58.

59.

All are true about liver hemangioma except: a. CHF is very common (AIIMS GIS Dec 2006) b. Incidental detection c. Consumptive coagulopathy can occur d. Spontaneous regression is seen Most common benign tumor of liver is: (DNB 2005, 2000, JIPMER GIS 2011 a. Hemangioma b. Hepatic adenoma c. Hepatoma d. Hamartoma CECT with nodular enhancement is suggestive of: a. Hepatic adenoma b. FNH (AIIMS GIS Dec 2006) c. Hemangioma d. HCC Most common nodule found in the liver is: a. Hepatoma b. Hamartoma c. Hemangioma d. Cholangiodenoma Which is the commonest incidentaloma detected in the liver? (Karnataka 94) a. Focal nodular hyperplasia b. Hemangioma c. Hepatocellular adenoma d. Hydatid cyst Most common liver tumor in children:(Recent Question 2017) a. Hemangioma b. Non-parasitic cyst c. Adenoma d. Focal nodular hyperplasia

HEPATOCELLULAR CARCINOMA RISK FACTORS 65. 66.

67.

HEPATOCELLULAR CARCINOMA 68. 69. 70.

HEPATIC CYST

61.

62.

63.

Solitary Hypoechoic lesion of the liver without septa or debris is most likely to be: (AIIMS Nov 2005) a. Hydatid cyst b. Caroli’s disease c. Liver abscess d. Simple cyst Simple hepatic cyst, all are true except: ( AIIMS GIS Dec 2006) a. Asymptomatic b. Lined by columnar epithelium c. Intracystic bleeding is common and deroofing is mandatory d. Congenital What is the diagnosis based on this ultrasound image? a. Simple hepatic cyst b. HCC c. Liver secondaries d. Intrahepatic cholangiocarcinoma

72. 73.

Treatment of choice for simple cyst of liver:  (JIPMER November 2017) a. Percutaneous drainage b. Cysto-enterostomy c. Deroofing d. Aspiration

POLYCYSTIC LIVER DISEASE 64.

71.

Tumor marker of HCC: (AIIMS GIS 2003) a. AFP b. Alpha fucosidases c. DCGP d. Carbohydrate antigen In high risk population, HCC is best detected by: a. USG b. CT (AIIMS GIS 2003) c. MRI d. PET scan All are true about AFP except: (AIIMS GIS 2003) a. Not return to normal after hepatic resection b. Levels >400 ng/mL with typical radiological findings is diagnostic of HCC c. Can be raised in other benign conditions d. Fibrolamellar HCC has normal levels AFP is elevated in: (PGI May 2011) a. HCC b. Hepatoblastoma c. Infant hemangioendothelioma d. Amoebic liver abscess e. Embryonic sarcoma Okuda staging contains all except: (ILBS 2012) a. Bilirubin b. Tumor size c. Ascites d. AFP What is the most probable diagnosis based on this triplephase CT? a. Hemangioma b. Focal nodular hyperplasia c. Hepatic adenoma d. Hepatocellular carcinoma

Treatment of symptomatic polycystic liver disease is:  (DPG 2008) a. Deroofing of the cyst b. Injection of sclerosant c. Hepatic resection d. Liver transplantation

74.

In HCC: (AIIMS GIS 2003) a. Arterial bruit is present in 80% cases b. Two third patients present with signs of liver disease c. Hemoperitoneum in 7% patients d. Percutaneous biopsy is mandatory for diagnosis

Hepatobiliary Pancreatic Surgery

60.

All are risk factors for HCC except: (AIIMS GIS Dec 2006) a. HBV b. HCV c. Alcohol d. IBS Which of the following most significantly increases the risk of HCC? (AIIMS May 2012) a. HBV b. HAV c. CMV d. EBV True regarding HCC: (JIPMER 2010) a. Non alcoholic steatohepatitis is a risk factor b. OCP’s are a cause c. Focal nodular hyperplasia may turn malignant d. Chromosomal abnormalities are common

Section 2

54.

123

124

Surgery Essence

Hepatobiliary Pancreatic Surgery

75.

76.

77.

78.

79.

80.

81.

82.

83.

Section 2

84.

85.

86.

Most common paraneoplastic syndrome of HCC: (AIIMS GIS 2003) a. Hypoglycemia b. Hypertension c. Hypercalcemia d. Erythrocytosis Oncological outcome in HCC is described by all except: (AIIMS GIS Dec 2010) a. MELD b. BCLC c. CLIP d. OKUDA The following are true about HCC except: (AIIMS Nov 2003) a. It has a high incidence in East Africa and South East Asia b. Its worldwide incidence parallels the prevalence of hepatitis B c. Over 80% of tumors are surgically resectable d. Liver transplantation offers the only chance of cure in those with unresectable disease The most unlikely clinical feature of hepatocellular carci­ noma is: (All India 94) a. Hepatomegaly b. Raised AFP levels c. Raised alkaline phosphatase d. Jaundice True about Hepatocellular carcinoma: (PGI Dec 2002) a. Most common tumor of liver b. Resectable only in 1% cases c. AFP increased in 70% cases d. USG guided aspiration biopsy is used for diagnosis Spontaneous rupture of the liver occurs in: a. Hepatoma b. Portal hypertension c. Spherocytosis d. Secondary deposits Tumor marker for primary hepatocellular carcinoma are all except: (AIIMS May 2007) a. Alpha-feto protein b. Alpha-2 macroglobulin c. PIVKA-2 d. Neurotensin All of the following are modalities of therapy for hepatocellular carcinoma except: (AIIMS Nov 2005) a. Radiofrequency ablation b. Transarterial catheter embolization c. Percutaneous acetic acid d. Nd-YAG laser ablation Which of the following liver tumour has a propensity to invade the portal or hepatic vein? (AIIMS June 2004) a. Cavernous hemangioma b. Hepatocellular carcinoma c. Focal nodular hyperplasia d. Hepatic adenoma Ramu, 40-year old male, chronic alcoholic, diagnosed as cirrhosis, presents with a lump in the right lobe of liver. Serum AFP level is normal. Most probable diagnosis is: a. Fibrohyperplasia  (AIIMS June 2001) b. Hepatocellular carcinoma c. Secondaries d. Hepatocellular adenoma Least common presentation of HCC: (AIIMS GIS 2003) a. PUO b. Mass c. Jaundice d. Weakness Hypercalcemia is seen in: (AIIMS GIS May 2011) a. Pancreatic cancer b. HCC c. CA stomach d. CA GB

87.

All are tumor markers of HCC except: (PGI SS Dec 2010) a. Neurotensin b. AFP c. CA 19-9 d. PIVKA-2

88.

New drug in HCC: a. Sorafenib c. Sunitinib

(GB Pant 2010) b. Bevacizumab d. Cetuximab

89. Consider the following features - Asian Male, alcoholic cirrhosis, hypervascular lesion during arterial phase of CT & Portal vein thrombosis. The above features are mostly suggestive of: (APPG 2016) a. Cholangiocarcinoma b. Hepatocellular carcinoma c. Metastatic colorectal carcinoma d. Neuroendocrine tumors 90.

91.

92.

The screening for HCC in chronic liver disease is:  (MHCET 2016)) a. Serial USG + AFP b. Serial LFT + AFP c. Serial LFT + CT scan d. Serial USG + Serial LFT Which of the following is not true about Milan’s criteria?  (Recent Question 2017) a. Single tumor 1000 pg/ml d. Occur in younger individuals

97.

Which of the following is having better prognosis? a. HCC b. Cholangiocarcinoma 
 c. Fibrolamellar variant of HCC d. Angiosarcoma

LIVER SECONDARIES 98.

All of the following modalities can be used for in situ ablation of liver secondaries, except: (All India 2006) a. Ultrasonic waves b. Cryotherapy c. Alcohol d. Radiofrequency

99.

Multiple liver secondaries are most common in the following cancers: a. Head of pancreas b. Stomach c. Gallbladder d. Periampullary

Liver

101. Calcific hepatic metastases are seen in: (COMEDK 2009) a. Adenocarcinoma of the colon b. Carcinoid tumours c. Renal cell carcinoma d. Lymphoma 102. Which of the following liver metastasis appear hypoechoic on ultrasound? (All India 2012) a. Breast cancer b. Colon cancer c. RCC d. Mucinous adenocarcinoma

HEPATOBLASTOMA 103. All are true about hepatoblastoma except: a. Associated with FAP (AIIMS GIS May 2008, Dec 2006) b. Most cases 40 years and asymptomatic (AIIMS GIS Dec 2006) b. Malignant predisposition c. Surgical resection is required d. Large projection with thickened wall is suggestive of malignancy

111. Most common indication for liver transplantation in children is: (JIPMER GIS 2011) a. Biliary atresia b. Indian childhood cirrhosis c. HCC d. Hepatitis C infection 112. Place of first liver transplant: (ILBS 2011) a. Pittsburgh b. Boston c. Colarado d. Cambridge 113. Auxiliary orthotopic liver transplant is indicated for: a. Metabolic liver disease (AIIMS May 2008) b. As a standby procedure until finding a suitable donor c. Drug induced hepatic failure d. Acute fulminant liver failure for any cause 114. In orthotropic liver transplantation, which is the best way to get bile drainage in donor liver? a. Donor bile duct with recipient bile duct or Roux-en-Y choledochojejunostomy b. Donor bile duct with duodenum of recipien c. Donor bile duct with jejunum of recipient d. External drainage for few days followed by choledo­ chojejunostomy 115. Reduced liver transplants: (GB Pant 2011) a. Given to two recipients after dividing into two parts b. Left lateral lobe divided and given to child c. Left lateral segment divided from segment 2 and given to child d. Part of liver segment transplanted into recipient depend­ ing upon requirement 116. Liver after transplantation enlarges by: (Recent Question 2015) a. Increase in size of cell b. Increase in number of cells c. Both of the above d. None of the above 117. All are indications of liver transplantation except: (Recent Question 2014) a. Cholangiocarcinoma b. Cirrhosis c. Biliary atresia d. Fulminant hepatitis

HEPATIC RESECTION 118. Contraindications to major hepatic resection for metastatic disease includes all of the following except:  (COMEDK 2006, Karnataka 2006) a. Total hepatic involvement b. Advanced cirrhosis c. Extrahepatic tumour involvement d. Jaundice from extrinsic ductal obstruction 119. The minimum amount of normal perfused liver parenchyma to be left intact when a hepatic resection is planned is: a. 10% b. 20% (COMEDK 2008, 2007) c. 50% d. 75% 120. Regarding hepatic artery ligation which statement is false? a. The best results are obtained in case of hemobilia b. Not useful in primary hepatoma (Karnataka 96) c. Can cure secondary carcinoma d. Must be covered by massive antibiotic administration 121. Vascular inflow occlusion of the liver is by: a. Clamping the hepatic artery b. Occluding the portal vein c. Clamping the hepatic veins d. The Pringle maneuver

(DNB 2012)

122. Pringle maneuver may be required for treatment of: a. Injury to tail of pancreas b. Mesenteric ischemia c. Bleeding esophageal varices d. Liver laceration (APPG 2015, Recent Question 2014)

Hepatobiliary Pancreatic Surgery

105. AFP is raised in: (KGMC 2011) a. 100% of hepatoblastoma b. 90% of hepatoblastoma c. 100% of HCC d. 90% of HCC

LIVER TRANSPLANTATION

Section 2

100. All are considered to be poor prognostic factors in liver metastasis except: (JIPMER GIS 2011) a. Synchronous lesions b. Metachronous lesions c. Extra-hepatic metastasis d. CEA >200 ng/ml

125

Hepatobiliary Pancreatic Surgery

126

Surgery Essence 123. Left trisegmentectomy involves removal of: (GB PANT 2010) a. Segment II III IV V VIII b. Segment II III IV c. Segment IV V VI VII VIII d. Segment V VI VII VIII

HEPATIC REGENERATION 124. Following resection of 2/3rd of the liver, regeneration is complete within: a. 2-3 months b. 8-10 weeks c. 4-6 months d. 4-5 weeks

LIVER TRAUMA 125. A 17-years old boy is admitted to the hospital after a road traffic accident. Per abdomen examination is normal. After adequate resuscitation, his pulse rate is 80/min and BP is 110/70 mmHg. Abdominal CT reveals 1 cm deep laceration in the left lobe of the liver extending from the done more than half way through the parenchyma. Appropriate management at this time would be: (DPG 2011, UPSC 2005) a. Conservative management b. Abdominal exploration and packing of hepatic wounds c. Abdominal exploration and ligation of left hepatic artery d. Left hepatectomy 126. ‘Beer-Claw’ appearance on CECT abdomen is seen in:  (MHCET 2016) a. Hepatic laceration b. Pancreatic laceration c. HCC d. RCC

LIVER ANATOMY 127. Left posterior sector of liver consists of: (JIPMER GIS 2011) a. Segment II and III b. Segment II, III and IV c. Segment II only d. Segment I only 128. All of the following are true about caudate lobe except: (AIIMS GIS Dec 2010) a. Blood supply from both right and left hepatic artery b. Ductal drainage from both right and left duct c. Venous drainage is mainly by left and middle hepatic vein d. Supply by both branches of portal vein

Section 2

129. Which is not true regarding the basis of functional divisions of liver? (AIIMS May 2015) a. Based on portal vein and hepatic vein b. Divided into 8 segments c. There are three major and three minor fissures d. 4 sectors 130. Left medial sector contains segment: a. III, IV b. II, III c. I, II d. I, IV

(GB Pant 2010)

131. Portal triad is not formed by: (Punjab 2008) a. Hepatic artery b. Portal Vein c. Bile duct d. Hepatic vein 132. Function of hepatic Kupffer cells is: (COMEDK 2004) a. Formation of sinusoids b. Vitamin A storage c. Increase blood perfusion d. Phagocytosis 133. Which of the following is false about portal vein? a. Formed behind the neck of pancreas (JIPMER 2011) b. Bile duct lies anterior and right to it c. Gastro duodenal artery lies to the left and anterior to it d. Ascends behind the 2nd part of duodenum

134. Surgeon excises a portion of liver to the left of the attachment of the falciform ligament. The segments that have been resected are: (All India 2011, 2008) a. Segment 1a and 4 b. Segment 1 and 4b c. Segment 2 and 3 d. Segment 1 and 3 135. False about hepatic duct: (AIIMS May 2011, 2009) a. Left hepatic duct formed in umbilical fissure b. Caudate lobe drains only left hepatic duct c. Right hepatic duct formed by V and VIII segments d. Left hepatic duct crosses IV segment 136. Surgical lobes of liver are divided on the basis of: a. Hepatic artery b. Hepatic vein c. Bile ducts d. Portal vein (PGI June 2002) e. Central veins 137. In Couinaud’s classification, segment IV of liver is:  (AIIMS Nov 2007) a. Caudate lobe b. Quadrate lobe c. Right lobe d. Left lobe 138. Line of surgical division of the lobes of the liver is: a. Falciform ligament to the diaphragm (AIIMS 85, 87) b. Gall bladder bed to IVC c. Gall bladder bed to the left crus of diaphragm d. One inch to the left of falciform ligament to the IVC 139. With Couinaud’s nomenclature, which one of the following segments of liver has an independent vascularization? a. Segment I b. Segment II (UPSC 2002) c. Segment IV d. Segment VIII 140. The Couinaud’s segmental nomenclature is based on the position of the: (All India 2004) a. Hepatic veins and portal vein b. Hepatic veins and biliary ducts c. Portal vein and biliary ducts d. Portal vein and hepatic artery 141. Quadrate lobe of liver is present between: (DPG 97) a. Groove for ligamentum teres and gallbladder b. Inferior vena cava and fissure for ligamentum venosum c. Groove for inferior vena cava and fissure for ligamentum venosum d. Porta hepatis and falciform ligament 142. Boundary of Morrison’s pouch is formed by: (DPG 2008) a. Kidney b. Falciform ligament of liver c. Spleen d. Pancreas 143. The right lobe of liver consists of which of the following segments: (AIIMS 2004) a. V, VI, VII and VIII b. IV, V, Vi, VII and VIII c. I, V, VI, VII and VIII d. I, IV, V, VI, VII and VIII 144. Liver is divided in 2 halves by all except: (AIIMS 2004) a. Right hepatic vein b. Portal vein c. Hepatic artery d. Common bile duct 145. Which of the following is not a capsular plate? (AIIMS Nov 2011) a. Portal plate b. Hilar plate c. Umbilical plate d. Cystic plate 146. Right hepatic duct drains all, except: (AIIMS May 2009) a. Segment I b. Segment III c. Segment V d. Segment VI

LIVER FUNCTION TESTS AND JAUNDICE 147. A patient has a surgical cause of obstructive jaundice. USG can tell all of the following except: (AIIMS Nov 2012) a. Biliary tree obstruction b. Peritoneal deposits c. Gall bladder stones d. Ascites

Liver

149. Which is wrong about Crigler-Najjar syndrome Type-I? a. Very high level of unconjugated bilirubin occurs in neonatal period (Orissa 2011) b. Kernicterus is usual c. It responds well to phenobarbitone d. Hepatic histology is normal 150. Conjugated hyperbilirubinemia is seen in: a. Dubin-Johnson syndrome b. Criggler-Najjar syndrome c. Criggler-Najjar syndrome- II d. Gilbert syndrome (COMEDK 2011) 151. Most common surgical cause of obstructive jaundice: a. Periampullary carcinoma (NEET 2013, Punjab 2007) b. Carcinoma gallbladder c. Carcinoma head of pancreas d. CBD stones (PGI Nov 2011)

153. True about obstructive jaundice: (PGI May 2011) a. Unconjugated bilirubin b. Positive indirect Vanden Bergh test c. Pruritus d. Pale stools e. Icterus 154. In non hemolytic jaundice, urobilinogen is seen in: (PGI 99) a. Obstructive jaundice b. Hepatic fibrosis c. Fatty liver d. Infective hepatitis 155. Courvoisier’s law is related to: (Recent Question 2016) a. Jaundice b. Ureteric calculi c. Portal hypertension d. The length of skin flap in skin grafting 156. Which of the following is an exception of Courvoisier’s law? (Recent Question 2015) a. Double impaction b. Portal lymphadenopathy c. Periampullary Carcinoma d. None of above 157. Which is not elevated in a child presenting with jaundice, icterus, pruritus and clay colored stools?  (AIIMS Nov 2011, Nov 2006) a. Gamma glutamyl transpeptidase b. Alkaline phosphatase c. 5’-nucleotidase d. Glutamate dehydrogenase 158. One is not the feature of obstructive jaundice: a. Pruritus (AIIMS Nov 95) b. Elevated level of serum bilirubin c. Raised alkaline phosphatase d. Raised urinary urobilinogen

160. A 50 years old patient presented with progressive jaundice. Liver function test was done in which conjugated serum bilirubin-4.8% and total bilirubin-6.7%, alkaline phosphatase-550 IU, SGOT-50, SGPT-65. Most probable diagnosis is: (AIIMS Nov 2013) a. Jaundice due to choledocholithiasis b. Dubin-Johnson syndrome c. Viral hepatitis d. Malignant obstructive jaundice 161. In a patient with obstructive jaundice, what is the possible explanation for a bilirubin level of 40 mg/dL? a. Malignant obstruction 
 (AIIMS November 2016) b. Complete obstruction of common bile duct 
 c. Renal failure d. Liver failure 


MISCELLANEOUS 162. Liver biopsy is done through 8th ICS midaxillary line to avoid: (All India 97) a. Lung b. Pleural cavity c. Subdiaphragmatic space d. Gallbladder 163. “Crumbled egg appearance” in liver is seen in: (Recent Question 2016) a. Hepatic adenoma b. Chronic amoebic liver abscess c. Hydatid liver disease d. Hemangioma 164. “Honey-comb liver is seen in: a. Micronodular cirrhosis (Recent Question 2016) b. Dubin Johnson’s syndrome c. Actinomycosis d. Hydatidosis 165. Primary sinusoidal dilatation of liver is also known as:  (COMEDK 2010) a. Hepar lobatum b. Peliosis hepatic c. Von-Meyerburg complex d. Caroli’s disease 166. Obstruction of IVC leads to: (Punjab 2008) a. Dilatation of thoracoepigastric veins b. Caput medusae c. Hemorrhoids d. Esophageal varices 167. Middle aged man presents with complaints of weakness, fatigue and hyperpigmentation. On examination hepatomegaly and hypoglycemia are present. Diagnosis:   (JIPMER 2011) a. Addison’s disease b. Hemochromatosis c. IDDM d. Cushing’s syndrome 168. A 20 years old male presents with extrapyramidal symptoms and liver damage. Diagnosis: a. Wilson’s disease b. Huntington’s disease c. Parkinson’s disease d. Hemochromatosis 169. Risk factor for angiosarcoma of liver: (MHSSMCET 2008) a. OCPs b. Phenacetin c. Vinyl chloride d. All of the above 170. Focal lesion of liver is best detected by: (AIIMS GIS 2003) a. MRI b. CT c. USG d. PET scan

Hepatobiliary Pancreatic Surgery

152. True about Criggler-Najjar syndrome II: a. Autosomal dominant b. Kernicterus is frequently present c. Child may alive to adolescence d. Cause unconjugated hyperbilirubinemia e. Phenobarbitone therapy is ineffective

159. Vitamin to be corrected in obstructive jaundice: (DNB 2009) a. Vitamin K b. Vitamin C c. Vitamin D d. Vitamine B12

Section 2

148. Cholecystocaval line: (Recent Question 2019, 2018) a. Separate right and left hepatic lobes b. Separate gallbladder from portal vein c. Separate right anterior and right posterior sectors d. Separate left medial and left lateral sectors

127

Explanations LIVER ABSCESS

1. Ans. b. Staph. aureus (Ref: Sabiston 20/e p1445-1449; Schwartz 10/e p1284-1285; Bailey 27/e p1168; Blumgart 6/e p1074, 5/e p1006-1115; Shackelford 8/e p1434, 7/e p1464-1471)

Pyogenic Liver Abscess in Children • In children, Staphylococcus PLAQ is most common • Occurs in the setting of chronic granulomatous diseaseQ, disorder of granulocyte function and hematologic malignancies. • In chronic granulomatous conditions, abscess are dense and thick, early excision and treatment with antibiotics against Staphylococcus aureus is recommendedQ.

2. Ans. a. E. coli



4. Ans. d. Percutaneous drainage is least cured

3. Ans. b. Biliary tract infection



5. Ans. c. More common in left side, d. More common in female (Ref: Sabiston 20/e p1449-1452; Schwartz 10/e p1285; Bailey 27/e p1168; Blumgart 6e/ p1074-1076, 5/e p1016-1024; Shackelford 8/e p1431, 7/e p1471-1478)



6. Ans. b. Peritoneal cavity



8. Ans. a. Radiographically unresolved lesion after 6 months (Ref: Sabiston 20/e p1452)

7. Ans. a. Male:female >10:1

Amoebic Liver Abscess • Although clinical improvement after adequate treatment with antiamebic agents is the rule, radiologic resolution of the abscess cavity is usually delayed. • The average time to radiologic resolution is 3 to 9 months and can take as long as years in some patients. • Studies have shown that more than 90% of the visible lesions disappear radiologically, but a small percentage of patients are left with a clinically irrelevant residual lesion.

9. Ans. b. Hematogenous spread from a distant site

Hematogenous spread is most common among the given options.

10. Ans. a. Amebic liver abscess

11. Ans. b. Ascending infection through biliary tract



12. Ans. c. Most common organism responsible is E. coli, e. Diagnosis is confirmed by aspiration and culture (Ref: Schwartz 10/e p12841285; Sabiston 20/e p1445-1449; Bailey 27/e p1168)



13. Ans. b. Left lobe, d. >10 cm size



15. Ans. c. Surgical drainage is always indicated



16. Ans. a. Diloxanide furoate (Ref: Goodman Gilman 12/e p1420)

14. Ans. c. Mostly treated conservatively

Diloxanide Furoate • Diloxanide furoate is highly effective luminal amebicideQ but has no systemic anti-amebic activityQ, because furoate ester is hydrolysed in the intestine and the released diloxanide is absorbed. • Diloxanide is a weaker amebicide than its furoate ester and no systemic antiamebic activity is seen despite its absorption.

17. Ans. c. Larvae are seen (Ref: Bailey 27/e p1169; Schwartz 10/e p1285)

Larvae are not seen in amoebic liver abscess.

18. Ans. b. USG weekly for 1 month followed by monthly USG till 1 year (Ref: Sabiston 20/e p1451; Schwartz 10/e p1284; Blumgart 6/e p 1087, 5/e p1016-1024; Shackelford 8/e p1440, 7/e p1471-1478)

In uncomplicated cases of amebic liver abscess, follow-up is done with ultrasound.

Liver

19. Ans. c. Antiamebics/antibiotics alone

20. Ans. b. Entamoeba histolytica (Ref: Schwartz 10/e p1285; Bailey 27/e p58)

HYDATID CYST 21. Ans. d. Ruptured hydatid cyst (Ref: Sabiston 20/e p1452-1455; Schwartz 10/e p1285-1286; Bailey 27/e p1169; Blumgart 6/e p1108, 5/e p10351048; Shackelford 8/e p1425, 7/e p1459-1462)

Section 2

• Presence of a solitary homogeneous, hypoechoic lesion in the right lobe of the liver in a young patient with fever and right upper quadrant pain suggests a diagnosis of amebic liver abscess. • The initial treatment of choice for amebic liver abscess is metronidazole alone. • Multiple aspirations and/ or catheter drainage or hepatectomy have no role in the “initial” management of amebic liver abscess.

129

22. Ans. d. Drooping lily sign (Ref: Wolfgang 2/e p309) • Drooping lily sign is seen in neuroblastoma and duplication of ureterQ. • Duplication of ureter: Drooping lily signQ on IVP (Nonvisualized upper pole of a duplex system displaces the lower pole down, looking like a drooped down lily flower on IVP) Characteristic Signs of Pulmonary Hydatidosis • Meniscus sign • Double arc signQ • Moon signQ

• Water lily signQ • Crescent signQ

Q

• Only completely calcified cyst (eggshell appearance) is accepted as a dead cyst. • CL, CE 1, and CE 2 are active fertile cysts; CE 3 is a transitional cyst with degeneration started; CE 4 is a degenerated cyst; CE 5 is a calcified cyst; CE 4 and CE 5 is inactive cyst. Gharbi Classification of Hydatid Cyst



Type I

Pure (clear) fluid collectionQ

Type II

Fluid collection with a split wall (Floating membrane)Q

Type III

Fluid collection with septa (Honeycomb image)Q

Type IV

Heterogeneous complex massQ

Type V

Calcified mass (eggshell)Q

23. Ans. a. Water lily sign (Ref: Bailey 27/e p66)

24. Ans. a. Hepatic resection is never done

25. Ans. b. Multiloculated, c. Cyst in lung, d. Recurrence after surgery, e. Perforated cyst

26. Ans. c. Hydatid cyst: (Ref: Blumgart 6/e p1114, 5/e p1045) Methods of Management of the Residual Cavity after Cyst Evacuation • • • •



External tube drainage Capsulorrhaphy CapitonnageQ Myoplasty

27. Ans. a. Hydatid cyst


• • • •

OmentoplastyQ Internal collapseQ IntroflexionQ MarsupializationQ

• Introflexion plus omentoplasty • Cystojejunostomy or Cystogastrostomy

Hepatobiliary Pancreatic Surgery

WHO Classification of Hydatid Cyst CL Type • Well-circumscribed liquid image with a clearly defined wall CE 1 • Concentric hyperechogenic halo around the cyst which may contain free-floating hyperechogenic foci called hydatid sandQ CE 2 • Multivesicular cyst with the daughter and grand-daughter cysts identified by honeycomb, rosette, spoke wheel or cluster imagesQ CE 3 • Partial or total detachment of the laminated layer with floating undulated hyperechogenic membranes showing the dual wall, water lilly and water snake signsQ. CE 4 • Cystic and solid componentsQ together without visible daughter cysts. CE 5 • Matrix or amorphous mass with a solid or semisolid appearance; limited amount of calcification; least common type.

130

Surgery Essence

Hepatobiliary Pancreatic Surgery

28. Ans. a. Lung cyst, e. Inaccessible location (Ref: Sabiston 20/e p1454; Bailey 27/e p1169; Blumgart 6/e p1117, 5/e p1047-1048; Shackelford 8/e p1425, 7/e p1461)

29. Ans. c. Hydatid cyst of liver

30. Ans. c. Cirrhosis

31. Ans. b. Formalin

32. Ans. d. Hydatid cyst (Ref: Sabiston 20/e p1452) • Ring like calcification on CECT is seen in hydatid cyst.

33. Ans. c. Aspiration is safe (Ref: Bailey 27/e p1169) Aspiration is safe in Hydatid cyst of liver. 34. Ans. b. Percutaneous drainage 35. Ans. b. ELISA

36. Ans. d. Multiple peritoneal cyst, c. Moribund patients (Ref: Sabiston 20/e p1453)



37. Ans. b. 60% • The sensitivity of Casoni’s test varies from 55 to 65%Q.



38. Ans. b. Echinococcus multilocularis 39. Ans. d. E. multilocularis (Ref: Sabiston 20/e 1452p; Schwartz 10/e p1286)



40. Ans. b. Water lily sign (Ref: Wolfgang 2/e p309) “The water-lily sign is seen in hydatid cyst when there is detachment of the endocyst membrane which results in floating membranes within the pericyst that mimic the appearance of a water lily. It is classically described on plain radiographs (mainly chest X-ray) when the collapsed membranes are calcified but may be seen on ultrasound and CT.” Water Lily Sign

HEPATIC ADENOMA 41. Ans. c. Hepatic architecture is maintained (Ref: Sabiston 20/e p1455; Schwartz 10/e p1290-1291; Bailey 26/e p1083; Blumgart 6/e p1284, 5/e p1258-1262; Shackelford 8/e p1537, 7/e p1564-1565) 42. Ans. a. Normal liver architecture

43. Ans. b. Normal liver architecture



44. Ans. a. Hepatic adenoma

45. Ans. b. Liver cell adenoma



46. Ans. a. Usually multiple



Section 2

47. Ans. b. Hepatic adenoma (Ref: Bailey 27/e p1172; Taber’s Medical Dictionary 19/e p1600) Hepatic adenoma always merits surgery. As no characteristic radiological features to differentiate these lesions from malignant tumor. These tumors are thought to have malignant potential and resection is therefore the treatment of choiceQ.

FOCAL NODULAR HYPERPLASIA 48. Ans. a. FNH (Ref: Sabiston 20/e p1456; Schwartz 10/e p1291; Bailey 27/e p1172; Blumgart 6/e p1306, 5/e p1255-1258; Shackelford 8/e p1536, 7/e p15631564)

49. Ans. b. Focal nodular hyperplasia (Ref: Sabiston 20/e p1456; Schwartz 10/e p1291; Bailey 27/e p1172) The given image shows central stellate scar, which is seen in focal nodular hyperplasia.



50. Ans. b. Surgical resection is required due to risk of malignancy

Liver

131

51. Ans. a. More common in young women, b. Associated with OCP use, c. May present with abdominal pain, d. Excision biopsy may aid in diagnosis

HEMANGIOMA 54. Ans. a. CHF is very common (Ref: Sabiston 20/e p1456-1457; Schwartz 10/e p1289-1290; Bailey 27/e p1172; Blumgart 6/e p1300, 5/e p1250-1255; Shackelford 8/e p1532, 7/e p1560-1563)

55. Ans. a. Hemangioma



58. Ans. b. Hemangioma (Ref: Schwartz 10/e p1288-1289; Bailey 27/e p1172; Blumgart 6/e p1300, 5/e p1250-1262)

57. Ans. c. Hemangioma

Hemangioma

Hepatic Adenoma

Focal Nodular Hyperplasia

Peliosis Hepatis

• MC hepatic neoplasmQ • Benign in nature • CT scan showing characteristic appearance of slow contrast enhancement due to small vessel uptakeQ in the hemangioma • Little malignant potential, hemangioma is not an indication for surgeryQ

• Rare benign liver tumors • No characteristic radiological features to differentiate these lesions from malignant tumorsQ • These tumors are thought to have malignant potential and resection is the treatment of choiceQ

• Benign conditionQ of unknown etiology • Focal overgrowthQ of functioning liver tissue supported by fibrous stroma • Patients are middle aged femalesQ, no association with underlying liver disease • Sulphur colloid scan of liver is useful for diagnosis. FNH contain both hepatocytes and Kupffer cellsQ. The latter take up the colloid, differentiating FNH from either a benign adenoma or a primary or metastatic cancer, none of which contains a significant number of Kupffer cellsQ.

• Multiple cystic blood filled spacesQ in the liver associated with dilatation of the sinusoidsQ. • Leads to hepatic enlargement and painQ • Associated with use of OCPs, anabolic steroids and BartonellaQ • If due to infection, treatment requires parenteral doxycyclineQ (surgery is not required in every case)

59. Ans. a. Hemangioma (Ref: Sabiston 20/e p1456; Schwartz 10/e p1289; Bailey 27/e p1171)

HEPATIC CYST

60. Ans. d. Simple cyst: (Ref: Sabiston 20/e p1469; Schwartz 10/e p1288-1289; Blumgart 6/e p530, 5/e p1052-1054; Shackelford 8/e p1421, 7/e p14531457)



61. Ans. c. Intracystic bleeding is common and deroofing is mandatory



62. Ans. a. Simple hepatic cyst (Ref: Sabiston 20/e p1469; Schwartz 10/e p1288; Blumgart 6/e p530, 5/e p1052-1054; Shackelford 8/e p1421, 7/e p1453-1457) Simple Hepatic Cyst • Contain serous fluid, do not communicate with the biliary tree, and do not have septationsQ • Diagnosis: USG (Posterior acoustic enhancement)



63. Ans. c. Deroofing (Ref: Schwartz 10/e p1289; Sabiston 20/e p1469)

POLYCYSTIC LIVER DISEASE

64. Ans. a. Deroofing of the cyst (Ref: Sabiston 20/e p1469; Schwartz 10/e p1288-1289; Blumgart 6/e p1140, 5/e p1054-1062; Shackelford 8/e p1424, 7/e p1457-1458; Maingot 11/e p776)

Treatment of symptomatic polycystic liver disease is deroofing of the cyst.

HEPATOCELLULAR CARCINOMA: RISK FACTORS 65. Ans. d. IBS (Ref: Sabiston 20/e p1458; Schwartz 10/e p1291, 1294-1296; Bailey 27/e p1173; Blumgart 6/e p1334, 5/e p1284; Shackelford 8/e p1542, 7/e p1565-1568)

66. Ans. a. HBV

67. Ans. a. Non alcoholic steatohepatitis is a risk factor (Ref: Shackelford 8/e p1542, 7/e p1567) • Cirrhosis due to NAFLD (Non-alcoholic fatty liver disease) mainly caused by obesity is a risk factor for HCC.

HEPATOCELLULAR CARCINOMA

68. Ans. a. AFP (Ref: Sabiston 20/e p1458-1463; Schwartz 10/e p1294-1296; Bailey 27/e p1175; Blumgart 6/e p1335, 5/e p1283-1289; Shackelford 8/e p1544, 7/e p1567-1576)

Hepatobiliary Pancreatic Surgery



56. Ans. c. Hemangioma

Section 2

52. Ans. a. Hemoperitoneum is common 53. Ans. c. Focal Nodular Hyperplasia

Hepatobiliary Pancreatic Surgery

132

Surgery Essence

69. Ans. a. USG



70. Ans. a. Not return to normal after hepatic resection



72. Ans. d. AFP (Ref: Sabiston 20/e p1460; Blumgart 6/e p1335, 5/e p1286-1287; Shackelford 8/e p1544, 7/e p1567-1576)



73. Ans. d. Hepatocellular carcinoma (Ref: Sabiston 20/e p1459; Schwartz 10/e p1291; Bailey 27/e p1173-1174)

71. Ans. a. HCC, b. Hepatoblastoma

In the given triple-phase CT, there is arterial hyper vascularization with venous washout, diagnostic of HCC. 74. Ans. c. Hemoperitoneum in 7% patients (Ref: Sabiston 20/e p1459)



Clinical Features of HCC • Vascular bruit (25%) , GI bleed (10%), tumor rupture (2-5%)Q, jaundice due to biliary obstruction (10%), paraneoplastic syndrome (hypoglycemiaQ, erythrocytosis, hypercalcemia Q



76. Ans. a. MELD

77. Ans. c. Over 80% of tumors are surgically resectable



78. Ans. d. Jaundice

79. Ans. c. AFP is increased in 70% cases



80. Ans. a. Hepatoma 81. Ans. b. Alpha-2 macroglobulin



82. Ans. d. Nd-YAG laser ablation (Ref: Sabiston 20/e p1461; Schwartz 10/e p1291, 1294-1296; Bailey 27/e p1174; Blumgart 6/e p1338, 5/e p12871288; Shackelford 8/e p1547, 7/e p1571-1577)

Treatment of HCC • Hepatic resection as the first-line treatment for cirrhotic patients with small HCC and preserved liver function and reserve salvage transplantation for recurrence or deterioration of liver function after hepatic resectionQ • Only patients with normal bilirubin concentration and absence of portal hypertension should be considered for resectionQ. • If not candidate for surgery offer percutaneous ablation; patients with more advanced disease (large or multifocal HCC) without portal vein invasion are candidates for transarterial chemoembolization if liver function is preserved (the sole palliative approach that has been shown to have a positive impact in survival is transarterial chemoembolization)Q. • For patients without cirrhosis who develop HCC, resection is the treatment of choice Q. • For those patients with Child’s class A cirrhosis with preserved liver function and no portal hypertension, resection also is considered Q. • If resection is not possible because of poor liver function and the HCC meets the Milan criteria (one nodule 40 years and asymptomatic Majority of patients of biliary cystadenoma present with a history of abdominal pain or massQ.

LIVER TRANSPLANTATION

Section 2

111. Ans. a. Biliary atresia (Ref: Sabiston 20/e p637-638; Schwartz 10/e p1277; Bailey 27/e p1554; Blumgart 6/e p661, 5/e p1662-1663; Shackelford 8/e p1365, 7/e p1519-1520) 112. Ans. c. Colorado (Ref: Sabiston 20/e p637, 19/e p655-664; Schwartz 10/e p1277, 9/e p295; Bailey 27/e p1554, 26/e p1427, 25/e p1408; Blumgart 6/e p1848, 5/e p1662) 113. Ans. a. > d. Metabolic liver disease >Acute fulminant liver failure for any cause (Ref: Blumgart 6/e p 1779, 5/e p1689-1693) APOLT is used in acute fulminant liver failure mainly caused by metabolic liver diseases, not in liver failure from any cause.

Indications of Auxiliary Partial Orthotopic Liver Transplantation (APOLT) 1. Reversible fulminant hepatic failureQ 3. Non-cirrhotic metabolic liver diseaseQ

2. Small-for-size graftsQ 4. ABO-incompatibilityQ

Liver

135

Auxiliary Partial Orthotopic Liver Transplantation (APOLT)

• APOLT can compensate for enzyme deficiency in non-cirrhotic metabolic liver disease (most commonly Criggler-Najjar syndrome) without complete removal of native liver. • Transplants of ABO-incompatible grafts are often unavoidable due to limited number of potential donor candidates. A high incidence of early graft failure with a high rate of biliary and vascular complications in ABO-incompatible liver transplantation is reported. The remnant liver could sustain a patient’s life if the anticipated graft failure occurred in an ABO-incompatible case. • In small-for-size graft, the remnant liver is expected to support the function of implanted graft during the early post-op period. The graft liver expands its function in proportion to volume growth. After the graft liver has grown sufficiently, it can be expected to meet the hepatic functional demands of the recipient.

Section 2

• In fulminant hepatic failure, APOLT provides temporary support until the native liver recovers and then immunosuppression can be withdrawn.

114. Ans. a. Donor bile with recipient bile duct or Roux-en-Y choledochojejunostomy (Ref: Blumgart 6/e p1807, 5/e p1727-1728) The preferred method biliary drainage during orthotropic liver transplantation is direct end-to-end anastomosis between donor common bile duct and recipient common bile duct. Bile duct anastomosis in orthotropic liver transplantation

1. Suprahepatic IVC 4. Hepatic artery

Choledochojejunostomy (CDJ) • This is an Roux-en-Y configuration anastomosis between the end of the common bile duct and the side of a loop of jejunum • This is an alternative bile duct anastomosis and is performed when CDCD anastomosis is not feasible. This may be the case when the recipient extrahepatic bile duct is diseased or small or when there is significant recipient donor duct size mismatch.

Five sequential anastomosis of OLT (SIPH-B) 2. Infrahepatic IVC 3. Portal vein 5. Bile duct

115. Ans. d. Part of liver segment transplanted into recipient depending upon requirement 116. Ans. b. Increase in number of cells

117. Ans. a. Cholangiocarcinoma

HEPATIC RESECTION 118. Ans. d. Jaundice from extrinsic ductal obstruction (Ref: Sabiston 20/e p1470; Blumgart 5/e p1462) Jaundice from extrinsic ductal obstruction is not a contraindication to major hepatic resection. 119. Ans. b. 20% (Ref: Blumgart 5/e p1462; Shackelford 7/e p1494) • Up to 70% to 75% of the hepatic volume may be resected with good recovery in patients with relatively normal hepatic parenchymaQ (without active hepatitis, cirrhosis, or metabolic defects), as long as the remnant liver has adequate portal venous and hepatic arterial inflow, adequate hepatic venous outflow, and adequate biliary drainage. 120. Ans. b. Not useful in primary hepatoma, c. Can cure secondary carcinoma (Ref: Sabiston 20/e p1470; Blumgart 6/e p1925, 5/e p1344)

Hepatic Artery Ligation • Used in management of hemobilia, hepatic artery aneurysm and pseudoaneurysm • Hepatic artery ligation or embolization is a method of ablation for HCC and liver secondaries, as these tumors are exclusively supplied by hepatic artery but it does not cure the malignancyQ. • In each case massive antibiotic therapy should be given post-operativelyQ. 121. Ans. d. The Pringle maneuver (Ref: Sabiston 20/e p439; Blumgart 6/e p1581, 5/e p1547; Shackelford 8/e p1417, 7/e p1450-1451)

Hepatobiliary Pancreatic Surgery

Choledochocholedochostomy (CDCD) • This is end-to-end anastomosis between donor common bile duct and recipient common bile duct • This is preferred bile duct anastomosis and is used when the recipient bile duct not diseased.

136

Surgery Essence

Hepatobiliary Pancreatic Surgery

Pringle Maneuver (Total Inflow Occlusion) • Total clamping of the hepatic pedicle, by placing an atraumatic clamp across the foramen of WinslowQ. • Appropriate-sized vascular clamp or loop snare easily controls hemorrhage from either the portal vein or the hepatic arteriesQ. • Inflow occlusion durations of up to 30 minutes can be tolerated safely in cirrhotic livers and possibly up to 60 minutes in early disease. • If prolonged occlusion is required, intermittent clamping can be used with repeated clampings of 10-20 minutes duration, each followed by 5 minutes declamping.  122. Ans. d. Liver Laceration 123. Ans. a. Segment II III IV V VIII (Ref: Blumgart 6/e p1548, 5/e p1512-1513; Shackelford 8/e p1466, 7/e p1501-1503)

Trisegmentectomy • Right trisegmentectomy or extended right hepatectomy: Complete resection of segment IV with the right  liver (removal of segment IV, V, VI, VII, VIII)Q. • Left trisegmentectomy or extended left hepatectomy: Complete resection of segments V and VIII with the left liver (removal of segment II, III, IV, V, VIII)Q.

HEPATIC REGENERATION 124. Ans. c. 4-6 months

Liver Regeneration • Following resection of 2/3rd of the liver, regeneration is complete within 5-6 months.

LIVER TRAUMA 125. Ans. a. Conservative management (Ref: Sabiston 20/e p437-439; Schwartz 10/e p173-174, 1642; Bailey 27/e p1161; Blumgart 6/e p1890-1891, 5/e p1806-1814; Shackelford 8/e p1448, 7/e p1479-1487) In stable patients, conservative management is preferred option. 126. Ans. a. Hepatic laceration

LIVER ANATOMY

Section 2

127. Ans. c. Segment II only (Ref: Sabiston 20/e p1421-1422; Schwartz 10/e p1264-1269; Bailey 27/e p161)



Liver

137

Lobar Anatomy of the Liver

• Cantlie’s line describes a 750 angle with a horizontal planeQ • It extends from the gallbladder fossa to the left side of the IVCQ. • • • •

Right & left halves of the liver is delineated by a plane through the MHV and IVCQ. Right portal fissure divides the right lobe into an anteromedial and posterolateral sector. RHV courses along this fissureQ. Right portal fissure describe an angle of 400 with the transverse planeQ. Left portal fissure divides the left lobe into an anterior and posterior sector, LHV courses along this fissureQ.

Section 2

• Liver is divided into two lobes by the main portal fissure (scissura), known as Cantlie’s lineQ. • Physiological right & left lobe are equally divided by an imaginary line (cholecysto-vena caval lineQ) running from GB fossa to groove for IVCQ.

• In the right lobe: −− Anteromedial sector: Segment V anteriorly & segment VIII posteriorlyQ −− Posterolateral sector: Segment VI anteriorly & segment VII posteriorlyQ • In the left lobe: −− Anterior sector is divided by the umbilical fissure into segment IV and segment IIIQ −− Posterior sector is comprised of only one segment, segment II Q • Umbilical fissure is not a scissuraQ, does not contain a hepatic vein, but contains the left portal triad. • Left scissura runs posterior to the ligamentum teres and contains the LHV; the left liver is split into an anterior (segments III and IV)Q and posterior (segment II- the only sector composed of a single segment)Q sector by the left scissura.



128. Ans. c. Venous drainage is mainly by left and middle hepatic vein (Ref: Sabiston 20/e p1422; Schwartz 10/e p1267; Bailey 27/e p1154; Blumgart 6/e p37, 5/e p36; Shackelford 8/e p1250, 7/e p1431)

Caudate lobe Caudate lobe is unique • It receives blood supply from both right & left portal pediclesQ • Bile drain into both right & left hepatic ductQ • Venous drainage is directly into IVCQ • Caudate lobe is anatomically divided into three parts, 1. Spigel lobe (Segment I) 2. Paracaval portion (segment 9)

3. Caudate process

129. Ans. b. Divided into 8 segments (Ref: Gray’s 40/e p1165, 1166,1178; Sabiston 20/e p1421-1423; Blumgart 6/e p37, 5/e p31-37; Shackelford 8/e p1250, 7/e p1426-1430) All of the given options are true. If we have to choose one answer, most preferred option is ‘liver is divided into 8 segments’, because sometimes segment IX is described. Segment IX is a recent subdivision of segment I, and describes that part of the segment that lies posterior to segment VIII. “Current understanding of the functional anatomy of the liver is based on Couinaud’s division of the liver into eight (subsequently nine) functional segments, based upon the distribution of portal venous branches and the location of the hepatic veins in the parenchyma (Couinaud 1957).”-Gray’s 40/e p1165 “Segment IX is a recent subdivision of segment I, and describes that part of the segment that lies posterior to segment VIII.”-Gray’s 40/e p1166 130. Ans. a. III, IV

Hepatobiliary Pancreatic Surgery

• Caudate lobe (segment I), lies between left portal vein & IVC and extends to hepatic venous confluenceQ.

138

Surgery Essence

Hepatobiliary Pancreatic Surgery

• Left medial sector contains segment III & IV

•  Left lateral sector contains only one segment, II.

131. Ans. d. Hepatic vein

Portal Triad Relations • CBD laterallyQ

• Portal vein posteriorlyQ

• Hepatic artery mediallyQ

Mickey Mouse View: Ultrasound image of hepatic artery, bile duct & portal vein is in a configuration, referred as Mickey Mouse ViewQ.

132. Ans. d. Phagocytosis (Ref: Sabiston 20/e p1430)

Kupffer Cells • • • •

Kupffer cells, derived from the macrophage-monocyte systemQ Are irregular stellate-shaped cells that also line the sinusoids, insinuating between endothelial cells. Phagocytic, play a major role in the trapping of foreign substancesQ and initiating an inflammatory response. MHC-II antigens are expressed on Kupffer cells but do not confer efficient antigen presentation compared with macrophages elsewhere in the body.

Hepatic Stellate Cells • Hepatic stellate cells (also known as Ito cells or lipocytes) are cells high in lipid content (accounting for their phenotypic identification)Q • Found in the space of DisseQ • Have dendritic processes that contact hepatocyte microvilli and also wrap around endothelial cells. • Major function: Vitamin A storage and synthesis of extracellular collagenQ. 133. Ans. d. Ascends behind the 2nd part of duodenum (Ref: Gray’s 40/e p1170) Portal vein ascends behind the 1st part, not the 2nd part of duodenum. Portal Vein Part Infraduodenal Part Retroduodenal Part

Supraduodenal Part

Position Anterior Posterior Anterior Posterior

Anterior Posterior

Structure Neck of pancreas IVC Duodenum (first part)Q Common Bile ductQ Gastroduodenal arteryQ IVC Hepatic arteryQ Bile ductQ IVCQ (separated by epiploic foramen)

134. Ans. c. Segment 2 and 3 (Ref: Sabiston 20/e p1421; Schwartz 10/e p1264-1265; Bailey 27/e p1154)

Falciform Ligament

Section 2

• Falciform ligament is the most obvious external landmark on the liver surface. • Plane passing through the falciform ligament passes through the left lobe • It divides the left lobe into a medial segment (segment IV) and lateral segment (segment II and III)Q 135. Ans. b. Caudate lobe drains only left hepatic duct (Ref: Blumgart 6/e p37, 5/e p36)

Caudate Lobe • In approx. 80% of the individuals, caudate lobe drains into both the right & left hepatic ductsQ • In 15%, caudate lobe drains only into left hepatic duct • In 5%, caudate is drained exclusively by right hepatic duct. 136. Ans. b. Hepatic vein, d. Portal vein 137. Ans. b. Quadrate lobe 138. Ans. b. Gallbladder bed to IVC

Liver

139

Liver

Hepatic Vascular Supply Hepatic artery

Section 2

Weight: 1800 gm in men and 1400 gm in women Total blood flow: 1.5 L/minQ Free pressure in a hepatic vein: 1-2 mm HgQ Liver can store up to maximum of 65 gm of glycogen/kg of liver tissueQ. Account for 4% of body weight, consumes about 28% of total body blood flow and 20% of the total oxygen consumed by the bodyQ. • Expends 20% of the total kilocalories used by the whole body. • • • • •

Q

Portal vein

• Hepatic artery supplies about 30 mL/min per 100 gm of liver tissueQ • Approx. 25% of the total blood flow to the liverQ • Provide 30-50% of the oxygen requirement Q. • The intrahepatic bile ducts are exclusively perfused by the hepatic arterial blood via the peribiliary plexusQ.

• • • •

Portal vein carries 90 mL/min per 100 gm liver tissueQ It carries about 75% of the total blood flow to the liverQ It may provide 50% to 70% of the oxygen requirementQ. Normal portal pressure is 5-10 mm HgQ.

Relationship between Hepatic Artery and Portal vein Blood Flow • There is an increase in hepatic arterial blood flow after portal flow reduction but the converse is not observedQ. 139. Ans. a. Segment I

141. Ans. a. Groove for ligamentum teres and gallbladder Quadrate lobe of liver is present between groove for ligamentum teres and gallbladder. 142. Ans. a. Kidney (Ref: Gray’s 40/e p1101, 1108) Boundary of Morrison’s pouch is formed by kidney. Boundaries of Morrison’s pouch (Hepatorenal pouch) Anteriorly

Posteriorly

• Inferior surface of right lobe of the liver

• Right suprarenal gland

• Gall bladder

• 2nd part of duodenum

• Upper part of right kidney

Superiorly • Inferior layer of coronary ligament

Inferiorly • Opens into general peritoneal cavity

• Hepatic flexure of colon • Transverse mesocolon • Part of head of pancreas 143. Ans. a. V, VI, VII and VIII

Couinaud’s Classification • Caudate lobe: IQ



•  Left lobe: II, III, IVQ



•  Right lobe: V, VI, VII, VIIIQ

144. Ans. a. Right hepatic vein 145. Ans. a. Portal plate (Ref: Sabiston 20/e p1423; Bailey 27/e p1154; Blumgart 6/e p574, 5/e p41; Shackelford 8/e p2077, 7/e p1430)

Fascial Plates of Liver Hilus • Fascial plates of liver hilus, represents a fusion of endoabdominal fascia around the portal structures • Fascial plate is formed by: cystic, hilar & umbilical plate Q • Hepatic veins lack endoabdominal fascial investment Q.

Hepatobiliary Pancreatic Surgery

140. Ans. a. Hepatic veins and portal vein

Hepatobiliary Pancreatic Surgery

140

Surgery Essence

Fascial plates of liver hilus 146. Ans. b. Segment III

LIVER FUNCTION TESTS AND JAUNDICE 147. Ans. b. Peritoneal deposits (Ref: Sabiston 20/e p1081; Schwartz 9/e p1140) • Peritoneal deposits are not detected by ultrasound. Even CECT can miss the peritoneal deposits. • Best investigation for diagnosis of peritoneal deposits is Diagnostic laparoscopy. 148. Ans. a. Separate right and left hepatic lobes 149. Ans. c. It responds well to phenobarbitone (Ref: Harrison 20/e p2344, 19/e p2001) Feature Total serum bilirubin (mg/dL) Routine liver tests Response to Phenobarbital Kernicterus Hepatic histology Bile characteristics Color Bilirubin fractions Bilirubin UDP-glucuronosyl transferase activity Inheritance (all autosomal)

Crigler-Najjar Syndromes Type I 18-45 (usually >20)Q NormalQ NoneQ UsualQ NormalQ

Type II 6-25 (usually 20Q) Normal Decreases bilirubin by >25%Q RareQ NormalQ

Pale or colorless >90% unconjugatedQ Typically absentQ; traces in some patients

Pigmented Largest fraction (mean: 57%) monoconjugatesQ Markedly reduced: 0-10% of normal

RecessiveQ

Predominantly recessive (AR>AD)Q

150. Ans. a. Dubin-Johnson syndrome (Ref: Harrison 20/e p2345, 19/e p2003) Causes of Hyperbilirubinemia

Section 2

Indirect (unconjugated) Hyperbilirubinemia Hemolytic disorders • • • •

Inherited SpherocytosisQ Elliptocytosis G-6-PD and pyruvate kinase deficienciesQ Sickle cell anemiaQ

Acquired • Microangiopathic hemolytic anemia • PNHQ • Immune hemolysis

• Ineffective erythropoesis in cases of vitamin B and iron deficiencies • Drugs: Rifampicin, ribavarinQ, probenicid Inherited conditions: • Criggler-Najjar and Gilbert’s syndromeQ

Direct (conjugated) Hyperbilirubinemia Inherited Conditions: 1. Dubin-Johnson syndromeQ 2. Rotor’s syndromeQ

Liver

141

151. Ans. d. CBD Stones 153. Ans. c. Pruritus, d. Pale stools, e. Icterus (Ref: Harrison 20/e p2343-2345, 19/e p1998)

Obstructive Jaundice • • • •

MC surgical cause of obstructive jaundice is CBD stonesQ Characterized by dark urine, clay colored stools, icterus and pruritusQ. Presence of urobilinogen in urine rules out obstructive jaundiceQ. USG is the best test to differentiate medical from surgical jaundice. Parameter Blood Hemoglobin (12-18 gm/dl) Unconjugated bilirubin (0.2-0.7 mg/dl) Conjugated bilirubin (0.1-0.3 mg/dl) ALP (3-13 KAU; 30-120 IU/L) Aminotransferases Cholesterol Stool Colour of stool Fecal urobilinogen Urine Bilirubin Urobilinogen

Hemolytic

Hepatocellular

Obstructive

Decreased ↑­ N N N N

N N or ­ ­ N or ­ ↑­ N

N N ­ ↑­ N ↑

N Increased

N Decreased

Pale Trace to absent

Nil ↑

­ ↑

­ ↓

155. Ans. a. Jaundice (Ref: Saiston 20/e p1544)

156. Ans. a. Double impaction

157. Ans. d. Glutamate dehydrogenase (Ref: Harrison 19/e p1997, 18/e p2529)

Enzymes Elevated in Cholestasis

• ALPQ

• 5’-NucleotidaseQ • Gamma glutamyl transpeptidaseQ

158. Ans. d. Raised urinary urobilinogen 159. Ans. a. Vitamin K 160. Ans. a. Jaundice due to choledocholithiasis (Ref: Sabiston 20/e p1495; Schwartz 9/e p1113; Blumgart 5/e p1129-1146; Schackelford 7/e p15991604; Harrison 20/e p2431, 19/e p1998) 161. Ans. c. Renal failure (Ref: Bailey 27/e p213; Textbook of hepatology 6/e p206) All causes of cholestatic jaundice, i.e. malignant obstruction, complete CBD obstruction can cause high jaundice but presence of concomitant renal failure leads to increase in bilirubin beyond 30 mg/dL.
 This is because conjugated bilirubin is soluble and can be excreted in urine. But in renal failure, this mechanism is absent and the serum bilirubin can rise up to very high levels. Usually even in the presence of total absence of bile flow the bilirubin levels reach a plateau of around 25-30 mg/dl as there is continuous excretion of conjugated bilirubin the bile. 
When there is such high bilirubin (>30/40 mg/dl) in obstructive jaundice, then suspect associated renal failure 
or ongoing hemolysis (Sepsis with DIC which may be the case in ascending cholangitis, sickle cell anemia, etc.) 


MISCELLANEOUS 162. Ans. a. Lung (Ref: Shackelford 7/e p1491-1492) • Liver biopsy is done through 8th ICS in midaxillary line to avoid Lung.

Liver Biopsy • Needle biopsy has proved to be most usefulQ technique to obtain representative liver tissue for analysis. • For accurate and reliable grading and staging of chronic viral hepatitis; a biopsy specimen of 2 cm Q in length or longer containing

at least 11 complete portal tractsQ is needed. • Liver biopsy is done through 8th ICS in midaxillary line to avoid LungQ.

Hepatobiliary Pancreatic Surgery

154. Ans. d. Infective hepatitis

Section 2

152. Ans. a. Autosomal dominant, c. Child may alive to adolescence, d. Cause unconjugated hyperbilirubinemia

142

Surgery Essence 163. Ans. c. Hydatid liver disease

Section 2

Hepatobiliary Pancreatic Surgery

• Crumbled egg appearance in liver is seen in hydatid diseaseQ. 164. Ans. c. Actinomycosis (Ref: Harrison 20/e p1221, 19/e p1089) 165. Ans. b. Peliosis hepatic (Ref: Shackelford 7/e p1556-1557) 166. Ans. a. Dilatation of thoracoepigastric veins • IVC obstruction can lead to dilatation of thoracoepigastric veins. 167. Ans. b. Hemochromatosis (Ref: Harrison 20/e p2980, 19/e p2516)

Manifestation of Hemochromatosis • Liver: Hepatomegaly, cirrhosis, HCCQ • Pancreas: Diabetes mellitusQ • Heart: CHF, cardiomyopathyQ

•  Skin: HyperpigmentationQ (bronzing of skin) •  Joints: ArthropathyQ •  HypogonadismQ



168. Ans. a. Wilson’s disease (Ref: Harrison 20/e p2982, 19/e p2519) 169. Ans. c. Vinyl chloride • Vinyl chloride is a risk factor for hepatic angiosarcoma. 170. Ans. a. MRI (Ref: Shackelford 8/e p1545, 7/e p1560, 1571)

MRI in Liver Lesions • MRI gas emerged as the best imaging test for liver lesion detection and characterizationQ • MRI provides high lesion-to-liver contrast and does not use radiationQ. • Liver-specific contrast media, such as mangofodipir trisodiumQ (taken up by hepatocytes) and ferrumoxides (taken up by Kupffer cells)Q demonstrate selective uptake in the liver and are primarily used for lesion detection. • These two contrast agents are also useful in characterizing specific liver tumors, such as FNH, hepatic adenoma and HCCQ.

CHAPTER

5

Portal Hypertension

ETIOLOGY OF PORTAL HYPERTENSION Etiology of Portal Hypertension Presinusoidal Extrahepatic or sinistral: • Splenic vein thrombosisQ • SplenomegalyQ • Splenic arteriovenous fistulaQ

Sinusoidal Intrahepatic: Cirrhosis due to• HBV, HCV, AlcoholQ • Metabolic abnormality • Autoimmune hepatitis • Primary biliary cirrhosis & PSC

Intrahepatic: • SchistosomiasisQ • Congenital hepatic fibrosis • Nodular regenerative hyperplasia • Idiopathic portal fibrosis • Myeloproliferative disorderQ • Sarcoid and GVHD

Postsinusoidal Intrahepatic: • Veno-occlusive diseaseQ

Posthepatic • Budd-Chiari syndromeQ • Congestive heart failure • IVC webQ • Constrictive pericarditisQ

• MC cause of intrahepatic presinusoidal portal hypertension: SchistosomiasisQ • MC cause of sinusoidal portal hypertension: CirrhosisQ

PORTAL HYPERTENSION Portal Pressure • Normal portal vein pressure: 5–10 mm HgQ • Normal portal vein pressure: 10–15 cm salineQ • Variceal formation occurs when portal pressure is >10 mm HgQ. • Variceal bleeding occurs when portal pressure is >12 mm HgQ.

Portal Hypertension • • • •

Definition: Portal pressure >10 mm HgQ MC cause of portal hypertension in United States: CirrhosisQ. Consequence of both increased portal vascular resistance & increased portal flowQ. Portal hypertension results in splenomegaly with enlarged, tortuous, & even aneurysmal splenic vessels. • Cruveilhier-Baumgarten murmurQ: Audible venous hum in caput medusa

• • • •

Hyperdynamic portal venous circulation seems to be related to severity of liver failureQ. Upper GI bleeding is caused by portal hypertension in about 90% of instances. Most bleeding episodes occur during the first 1 to 2 years after identification of varicesQ. Colour Doppler is investigation of choice for evaluation of PHT. • About one third of deaths in patients with known esophageal varices are due to upper GI bleedQ • A larger proportion dies as a result of liver failureQ.

• MC causes of death in cirrhosis patients: Hepatic failureQ • 2nd MC causes of death in cirrhosis patients: variceal hemorrhageQ

144

Surgery Essence

Hepatobiliary Pancreatic Surgery

Laboratory Abnormalities in Cirrhosis • • • • •

Cirrhosis is often accompanied by anemia, leukopenia, & thrombocytopeniaQ. Degree of thrombocytopenia has been found to be a quite accurate predictor of presence of esophageal varicesQ. Hypoalbuminemia and a prolonged INR are reliable indices of chronic rather than acute liver diseaseQ. ALT/AST >2 is highly suggestive of alcohol as the cause of liver diseaseQ. Common serum electrolyte abnormalities in cirrhosis are hyponatremia, hypokalemia & metabolic alkalosisQ.

HEPATOPULMONARY SYNDROME Hepatopulmonary Syndrome • Patients with long-standing cirrhosis and portal hypertension are prone to develop HPS. • It is defined as a triad of signs: Liver diseaseQ + increased alveolar-arterial gradients (hypoxemiaQ) + evidence of intrapulmonary vascular resistance (intra-pulmonary vascular dilatation)Q. • Severity depends upon liver disease, oxygenation defect and pulmonary vascular dilatationQ Clinical Features • Clinical features are orthodeoxia, platypneaQ, and insidious & slow progression of dyspnea, clubbing, distal cyanosis and spider angiomas. Diagnosis • Contrast echocardiography is the study of choice to diagnose HPS by demonstrating the presence of intra-pulmonary vascular dilatationsQ. Treatment • Liver transplant reverses HPS in most of the patientsQ.

PORTOPULMONARY HYPERTENSION Portopulmonary Hypertension • PPH is pulmonary artery hypertension in portal hypertensionQ. • Portopulmonary Hypertension (PPH) occurs when there is pulmonary vasoconstriction and increased pulmonary artery pressureQ. Clinical Features • Asymptomatic PPH predisposes to intra-operative cardiac arrhythmias and arrestQ. Diagnosis • Patients with pulmonary artery pressure gradient (PAPG) >25 mm Hg should undergo right heart catheterization for further assessmentQ. • With right heart catheterization, a mean pulmonary artery pressure of >25 mm Hg with a capillary wedge pressure 25 mm Hg, pulmonary capillary wedge pressure 240 dynes S. cm–5 occurring in setting of portal hypertension is diagnostic of PPHQ.

Section 2

Treatment • Best treatment: Combination of medical therapy and liver transplantationQ. • Moderate to severe PPH (PAP > 50 mm Hg) is a contraindication for liver transplantation; first consider prostanid therapy (epoprostenol) to reduce pulmonary hypertensionQ.

CHILD-TURCOTTE-PUGH (CTP) SCORING SYSTEM Child-Turcotte-Pugh (CTP) Scoring System • CTP score is a measure to assess hepatic functionQ in many liver diseases. • It was initially devised to classify patients into risk groups prior to undergoing porto-systemic shunt surgeriesQ. • It is used to assess prognosis in cirrhosisQ and many liver diseases.

Portal Hypertension

Class A

5–6 pointsQ

Class B

7–9 pointsQ

Class C

10–15 pointsQ

3 Points 3 >6 >2.3 Uncontrolled Uncontrolled

Section 2

Child-Turcotte-Pugh (CTP) Score Variable 1 Point 2 Points Serum albumin (g/dL) >3.5 2.8–3.5Q Bilirubin (mg/dL) 4 mg/dL), ALP, transaminases, & amylase may be present. Diagnosis • USG: IOC for diagnosing acute cholecystitisQ (sensitivity 85%, specificity 95%). • HIDA scan: Gold standardQ for diagnosing acute cholecystitis • USG findings: Presence of gallstones, thickening of GB wall (> 4 mm), pericholecystic fluid, GB distention, impacted stone, and a sonographic Murphy’s signQ (focal tenderness directly over the GB). • HIDA scan: No filling of GB with the radiotracer (99mTc-HIDA) after 4 hours indicates an obstructed cystic ductQ

• A normal HIDA scan excludes acute cholecystitisQ. Treatment • IV fluids, antibiotics, and analgesia should be initiatedQ. • Cholecystectomy is the definitive treatmentQ for patients with acute cholecystitis. • Early cholecystectomy performed within 2 to 3 days (within 72 hours) Q of presentation is preferred over interval or delayed cholecystectomy that is performed 6 to 10 weeks after initial medical therapy. • Laparoscopic cholecystectomy is the preferred approach to patients with acute cholecystitisQ. • Morbidity rate, hospital stay, and time to return to work are lower in patients undergoing laparoscopic cholecystectomy than open cholecystectomy. • Early laparoscopic cholecystectomy, due to a reduced length of hospital stay and readmissions, is a more costeffective approach than open cholecystectomy for acute cholecystitis.

MEDICAL THERAPY FOR GALLSTONES Medical Therapy for Gallstones • Medical therapy for gallstones utilizes bile acids: Chenodeoxycholic acid (CDCA) & Ursodeoxycholic acid (UDCA)Q Mechanism of Action • They inhibit HMG-CoA reductaseQ, rate limiting enzyme for cholesterol synthesis, thus decreases cholesterol saturation of bileQ • They cause dispersion of cholesterol from the stones by physicochemical meansQ

• • • •

Prerequisites for Medical Treatment

Drawbacks of Medical Treatment

Radioluscent (cholesterol) stonesQ Stones 70 years)Q • Accounts for 1% of all cases of small bowel obstruction and occur in fewer than 1% of patients with gallstones. Bouveret’s Syndrome • Duodenal obstruction due to gallstones, usually in the bulb is known as Bouveret’s syndromeQ. • It is treated by duodenostomy or pyloroplastyQ. • Nausea, vomiting, and abdominal pain, signs & symptoms of intestinal obstruction • A history of gallstone-related symptoms may be present in only 50% of patients. • Pain may be episodic and recurrent as the impacted stone temporarily obstructs the bowel lumen and then dislodges and moves distally, known as tumbling obstructionQ. • MC site of obstruction is ileumQ (60%); jejunum (15%); stomach (15%) colon (5% > sigmoid colon); & duodenum (5%). Diagnosis • Abdominal X-ray: Evidence of an intestinal obstruction with pneumobilia or a calcified stoneQ distant from the gallbladder. • MC site of obstruction is the terminal ileumQ because of its narrow lumen. Treatment • It is a surgical emergencyQ without a period of waiting in the hope that stone will pass • In case of obstruction in the ileum calculus can be manipulated proximally to a healthy jejunumQ where a safe enterotomy and stone removal may be executed. • Stable patients: Takedown of the biliary-enteric fistula and cholecystectomyQ during the same procedure is warranted because recurrent cholecystitis and cholangitis are common. • Unstable patients or a significant inflammation in RUQ: Unstable to withstand a prolonged operative procedure, the fistula can be addressed at a second laparotomyQ.

MUCOCELE Mucocele (Hydrops) • Hydrops or mucocele result from prolonged obstruction of the cystic duct, usually by a large solitary calculusQ. • Obstructed GB lumen is progressively distended by mucus (mucocele) or by a clear transudate (hydrops) produced by mucosal epithelial cellsQ. Clinical Features • A visible, easily palpable, nontender gallbladderQ sometimes extending from the RUQ into the right iliac fossa may be found on physical examination. • The patient with hydrops of the gallbladder frequently remains asymptomaticQ, although chronic RUQ pain may also occur.

Section 2

Treatment • Early cholecystectomyQ, because empyema, perforation, or gangrene may complicate the condition.

Gallbladder Empyema • GB empyema results from progression of acute cholecystitis with persistent cystic duct obstruction to superinfectionQ of the stagnant bile with a pus-forming bacterial organism. Clinical Features • Clinical picture resembles that of cholangitis with high fever; severe RUQ pain; marked leukocytosis; and often, prostrationQ. • Empyema carries a high risk of gram-negative sepsis and/or perforationQ. Treatment • Emergency surgical intervention with antibiotic coverage is required as soon as the diagnosis is suspected.

Gallbladder

169

PROPHYLACTIC CHOLECYSTECTOMY Indications of Prophylactic Cholecystectomy Cardiac transplant recipientsQ • Family history of GB cancer and asymptomatic stonesQ Lung transplant recipientsQ • Cholelithiasis encountered during elective abdominal proceduresQ Chronic TPN requirementQ • Nonfunctioning GBQ Recipients of biliopancreatic diversionQ (bariatric patient) Q Children with hemoglobinopathyQ (sickle cell, thalassemia and spherocytosis) • Typhoid carrier with positive bile culture Q Q • Trauma to GB Asymptomatic gallstone ≥3 cm • Porcelain GB Stone associated with the polyp

• Many heart/lung transplant recipients use cyclosporine as maintenance immunotherapy; chronic cyclosporine useQ (>2 years) has been associated with the prevalence of gallstones.

Section 2

• • • • • • •

ACALCULOUS CHOLECYSTITIS Acalculous Cholecystitis • Acute inflammation of gallbladder without stones • Accounts for 5-10% of all patients of acute cholecystitis • More fulminant courseQ than the acute calculous cholecystitis. • More commonly progresses to gangrene, empyema, or perforationQ. • Visceral ischemia is common in acute acalculous cholecystitis and may explain the high incidence of GB gangrene. Predisposing Factors Q

Elderly and critically ill patients after traumaQ BurnsQ Long-term TPNQ Major operations (abdominal aneurysm repair & cardiopulmonary bypassQ)

Etiopathogenesis • Exact etiology is not clear • GB stasis & ischemiaQ have been implicated as causative factors. • Stasis (disturbed micro-circulation) is common in critically ill patients not being fed enterally and may lead to colonization of the GB with bacteriaQ. • Decreased arteriolar and capillary filling is present in contrast with the dilatation of these vessels in acute calculous cholecystitis. Clinical Features • Similar to acute calculous cholecystitisQ. • Patients may present with only unexplained fever, leucocytosis and hyperamylasemia and right upper quadrant tendernessQ. • If untreated, rapid progression to gangrene and perforationQ may occur. Diagnosis • Ultrasonography is the diagnostic test of choiceQ, especially because it can be done at the bed side. • Cholescintigraphy demonstrates absent gallbladder filling Treatment • Emergency cholecystectomyQ for stableQ patients • Because of high incidence of gangrene, perforation and empyema, open cholecystectomyQ is often the preferred approach. Percutaneous Cholecystostomy • If patients are unfit for surgery, percutaneous, ultrasound guided, or CT guided cholecystostomy is the treatment of choiceQ. • About 90% patients improve with percutaneous cholecystostomy.

XANTHOGRANULOMATOUS CHOLECYSTITIS Xanthogranulomatous Cholecystitis • XGC: Inflammatory disease characterized by a focal or diffuse destructive inflammatory process with lipid-laden macrophagesQ

Hepatobiliary Pancreatic Surgery

• • • •

Hepatobiliary Pancreatic Surgery

170

Surgery Essence Pathology • Inflammatory response to extravasated bile, possibly from ruptured Rokitansky-Aschoff sinusesQ. • Presence of hypoechoic nodules or bands in thickened GB wallQ together with calculi (cholesterol or mixed gallstones) in patient of chronic disease. • There is extension of yellow tissue into adjacent organs, fistulas from GB to skin or duodenum may develop, may be mistaken for cancerQ. Clinical Features • Similar to acute cholecystitis Diagnosis • Thickening of GB wall is most common radiological finding, sometimes presence of hypoattenuated bandsQ. Treatment • Surgical treatment (Laparoscopic cholecystectomy) remains the most effective and feasible option for XGC

EMPHYSEMATOUS CHOLECYSTITIS Emphysematous Cholecystitis • Thought to begin with acute cholecystitis (calculous or acalculous), followed by ischemia or gangrene of GB wall and infection by gas producing organismsQ. • Occur most frequently in elderly men and patients with diabetes mellitusQ. • Gallstones are observed in 28–80% of patients with emphysematous cholecystitis. • Emphysematous cholecystitis in the presence of acalculous cholecystitis is well established. Causative organisms of Emphysematous Cholecystitis • Anaerobes: Cl. welchii or Cl. perfringens (MC) Q • Aerobes: E. coliQ

Clinical Features • Clinical manifestations are essentially indistinguishable from those of nongaseous cholecystitis. Diagnosis • Diagnosis is usually made on abdominal X-rayQ • Abdominal X-ray findings: Gas within the GB lumen, dissecting within GB wall to form a gaseous ring or in pericholecystic tissuesQ. • IOC for diagnosis: CT scanQ Treatment • Morbidity & mortality rates with emphysematous cholecystitis are considerable. • Prompt surgical drainage coupled with appropriate antibiotics is mandatoryQ. • Cholecystectomy is best treatment of complicated acute cholecystitisQ. • Unstable patients: Percutaneous cholecystostomy under LAQ can be performed to drain GB.

Section 2

MIRIZZI’S SYNDROME (FUNCTIONAL HEPATIC SYNDROME)

Mirizzi’s Syndrome (Functional Hepatic Syndrome) • It is defined as obstruction of the common hepatic duct or CBD by external compression or by erosion of stone in the Hartmann pouch or cystic ductQ. • External compression has been classified as type 1 whereas erosion as type 2 Mirizzi syndrome by McSherryQ.

Gallbladder

171

• Csendes subclassified type 2 into three categories based on the percentage of the wall of CBD eroded by offending calculus. Csendes Classification of Mirizzi’s Syndrome Obstruction of common duct by external compression only (no erosion)Q Erosion of less than one-third circumference of common ductQ Erosion of up to two-third circumference of common ductQ Total/near total circumferential destruction of common ductQ Erosion of GB in common duct with cholecystoenteric fistula

Treatment Type I

Partial cholecystectomyQ

Type II & Type III

Partial cholecystectomy leaving behind a cuff of gallbladder for reconstruction of bile duct (choledochoplasty) with T-tube drainageQ

Type IV & V

Bilioenteric anastomosisQ

Section 2

Type I Type II Type III Type IV Type V

• Stenosis of the biliary tree often resolves spontaneously in the postoperative period, and choledochotomy is seldom indicatedQ.

STRAWBERRY GALLBLADDER (CHOLESTEROLOSIS) Strawberry Gallbladder (Cholesterolosis) • It is an acquiredQ histologic abnormality of gallbladder epithelium that results in an excessive accumulation of lipid (cholesterols esters & triglyceride)Q within epithelial macrophage of GB wall. • Cholesterol stones are found in half of the casesQ. Pathology

Treatment • Cholecystectomy is indicated in symptomatic cholesterolosis or when cholelithiasis is presentQ.

GALLBLADDER POLYPS Polypoid Lesions of the Gallbladder Cholesterol polyps

Adenomatous polyp

• Cholesterol polyps are the most commonQ • Usually 60 yearsQ        −  Coexistence of                  gallstonesQ − Documented increase in sizeQ −  Size > 10 mmQ

GALLBLADDER ADENOMYOMATOSIS Gallbladder Adenomyomatosis • Adenomyomatosis is a benign condition characterized by hyperplastic changesQ of unknown etiology involving the GB wall. • It causes overgrowth of mucosa, thickening of muscular wall, and formation of intramural diverticula or sinus tracts termed as AschoffRokitansky sinusesQ. • These sinuses may contain cholesterol crystalsQ. • The disease can be focal or diffuse. • It has no malignant potentialQ.

Hepatobiliary Pancreatic Surgery

• Gross Appearance: Mucosa has pale, yellow streaks running longitudinally giving rise to the term strawberry gallbladderQ (although the mucosa is usually bile stained rather than red). • Diffuse form (“strawberry gallbladder”): GB mucosa is brick red and speckled with bright yellow flecks of lipidQ. • Localized form: Solitary or multiple “cholesterol polyps” studding the gallbladder wallQ.

172

Surgery Essence

Hepatobiliary Pancreatic Surgery

Diagnosis • USG: The presence of cholesterol crystals in these sinuses can result in “diamond ring sign”Q, “V-shaped”Q, or “comet-tail” artifactsQ on USG. • Adenomyomatosis appears as a sessile polyp with characteristic microcysts on USG and is often >10 mmQ. • Cholecystography is more specific for diagnosis. Treatment • Cholecystectomy is indicated in symptomatic adenomyomatosis or when cholelithiasis is presentQ.

RISK FACTORS FOR CARCINOMA GALLBLADDER • • • •

Risk Factors for Carcinoma Gallbladder Gallstones >3 cmQ • Adenomatous polypsQ Q Porcelain gallbladder • Primary sclerosing cholangitisQ Anomalous pancreaticobiliary junctionQ • ObesityQ Choledochal cystsQ • Salmonella typhi infectionQ

PORCELAIN GALLBLADDER Porcelain Gallbladder • Porcelain GB is characterized by extensive calcium encrustation of GB wallQ. • The term porcelain gallbladder has been used to emphasize the blue discoloration & brittle consistency of GB wall at surgeryQ. Pathology • Calcium salt deposition within the wall of a chronically inflamed gallbladderQ Clinical Features • Most porcelain GB (90%) are associated with gallstonesQ. • Mean age of patients is 54 years (38–70 years). • Patients are usually asymptomatic and the condition is usually found incidentally on plain abdominal radiographs, sonograms or CT images. • Incidence of CA GB: 6% • Surgery should not be delayed even if the patient is asymptomatic, because the occurrence of carcinoma is remarkably high. Diagnosis • In porcelain GB, plain radiographic findings are usually straight forwardQ. • CT scan is diagnostic in cases of doubt Treatment • Cholecystectomy in all patientsQ with porcelain GB (high incidence of development carcinoma GB).

CARCINOMA GALLBLADDER

Section 2

Carcinoma Gallbladder • • • • • •

Highest incidence of CA GB in India and Pakistan More common in women of 6th & 7th decadeQ Cholelithiasis is seen in 75–98% of all patients with CA GBQ. Incidence of CA GB in a population of patients with gallstones is from 0.3–3%Q. CA GB is an aggressive malignancy with poor prognosisQ Nevin classificationQ is used for CA GB staging. Q

• Calcified GB is associated with cancer in 10Q of cases. • Helicobacter pylori & H. bilis demonstrated to increase the risk of CA GB by 6 foldQ. • Increased risk of CA GB in FAP & HNPCCQ. • MC gene mutation in CA GB: p53> K-ras>BRAFQ • MC mode of CA GB spread: Direct invasionQ into the adjacent organs. Pathology • MC Site: Fundus (60%)Q >Body (30%) >Neck (10%)

Gallbladder

• • • •

Section 2

• Histological types: −− Diffuse Infiltrative: MC typeQ −− Nodular or mass forming −− Papillary: Exhibits polypoid or cauliflower appearance and have best prognosisQ. • AdenocarcinomaQ is the MC histologic subtype of CA GB. • In CA GB: Direct hepatic invasion in 59%Q , LN metastasis in 45%, perineural invasion in 42% cases. Clinical Features

173

Most commonly presents with RUQ pain often mimicking cholecystitis and cholelithiasisQ. Weight loss, jaundice, and an abdominal mass are less common presenting symptoms. Chronic cholecystitis with a recent change in quality or frequency of the painful episodes in 40% patientsQ Malignant biliary obstruction with jaundice, weight loss, and RUQ pain.

Diagnosis • USG is first diagnostic modalityQ used in evaluation of patients with RUQ pain. • USG: A heterogeneous mass replacing the GB lumen and an irregular gallbladder wallQ • CT scan: Mass replacing the gallbladder (MC finding); focal or diffuse gallbladder wall thickening; and an intraluminal polypoidal mass.Q • Typical cholangiographic finding: Long stricture of the common hepatic ductQ. • Triple phase CT is used to identify hepatic arterial or portal venous involvement. • Unresectable or incurable CA GB: Percutaneous biopsy or FNAC for confirmatory tissue diagnosis.Q Tumor Markers • Best tumor marker for CA GB is CA19-9Q (CA19-9 >20 U/ml-75% of sensitivity and specificity). • CEA >4 ng/mL is associated with 93% specificity but 50% sensitivity.

8th AJCC (2017) TNM Classification of Carcinoma Gallbladder

Stage IA T1a N0M0

T1a

Lamina propria invasionQ

T1b

Muscular invasionQ

T2

Invade the perimuscular connective tissueQ T2a: Invade the perimuscular connective tissue on the peritoneal side with no extension to serosaQ T2b: Invade the perimuscular connective tissue on the hepatic side with no extension into the liverQ

T3

Serosal perforation and/or direct invasion of the liver (regardless of extent) and/or invasion of any other single extrahepatic organQ.

T4

Tumor invades the main portal vein, hepatic artery or two or more extrahepatic organQ

N1

Metastasis to 1-3 regional nodesQ

N2

Metastasis to 4 or more regional nodesQ

M1

Distant metastasis

Stage IB T1b N0M0

Stage IIA T2a N0M0

Stage IIB T2b N0M0

Stage IIIA T3 N0M0

Stage IIIB T1-3 N1M0

Stage IVA T4 N0-1M0

TREATMENT OF CARCINOMA GALLBLADDER Treatment of Carcinoma Gallbladder • Gallbladder cancer: Incidental pathological finding after Laparoscopic cholecystectomy • T1a with negative cystic duct margin: No further therapyQ • T1a with positive cystic duct margin: Reresection of cystic duct or CBD to negative marginQ • T1b, T2, T3 tumor with no evidence of metastasis: Reresection, extended cholecystectomy (possible CBD or extended hepatic resection)Q

Stage IVB Tany N2M0 Tany Nany M1

Hepatobiliary Pancreatic Surgery

TNM CLASSIFICATION OF CARCINOMA GALLBLADDER

Hepatobiliary Pancreatic Surgery

174

Surgery Essence • T4: Extended cholecystectomy with extended right hepatectomyQ • N2 or M1 disease: Clinical trial (chemoradiation or chemotherapy) in good performance statusQ • Laparoscopic trocar site scars are excised for staging purpose to identify M1 disease than for any potential therapeutic benefitQ. Preoperatively diagnosed CA GB • • • •

T2, T3: Extended cholecystectomyQ T4 N0: Extended cholecystectomy with extended right hepatectomyQ N1 or hilar invasion: Extended cholecystectomy with CBD resectionQ N2 or M1: Clinical trial (chemoradiation or chemotherapy) in good performance status, palliative care in poor performance statusQ

Surgical Technique • For patients suspected of having resectable gallbladder cancer, begin surgical exploration with laparoscopy, in the absence of disseminated disease, proceed with open laparotomy. • Extended cholecystectomy consists of cholecystectomy with en bloc resection of segments IVB and V; including lymphadenectomy of the cystic duct, pericholedochal, periportal, and posterior pancreaticoduodenal and local interaortocaval lymph nodes. • During a standard cholecystectomy the serosa of the gallbladder is typically opened and the avascular subserosal layer is used as the surgical plane of dissection. • In case of suspected carcinoma the plane of dissection is along the cystic plate of the liver to avoid violation of the gallbladder subserosa. • Only 15% of patients develop loco-regional recurrence while most (85%) had recurrence involving a distant site. Gallbladder Cancer with Obstructive Jaundice • Percutaneous external biliary drainage offers a minimally invasive and effective means of palliation.

Indications for repeat operative intervention in CA GB diagnosed incidentally after laparoscopic cholecystectomy • Pathologic analysis identifies T2 or greater degree invasionQ • Cystic duct margins are positiveQ • Presence of intra-operative bile spillageQ

Unresectable Carcinoma Gallbladder • Median survival for patients presenting with unresectable disease is 2-4 months with 1-year survival is less than 5%Q. • Goal of palliation is to relieve jaundice, pain, bowel obstruction and prolongation of life. • Percutaneous stents are effective for relieving jaundice and should be used, as expected survival does not usually warrants a surgical bypassQ. • Gemcitabine plus cisplatin (reference regimen) is used for palliation of unresectable diseaseQ.

GALLBLADDER: ANATOMY AND PHYSIOLOGY Gallbladder: Anatomy and Physiology • It is lined by a single, highly-folded, tall columnar epithelium • The mucus originates in tubuloalveolar glandsQ found in mucosa lining the infundibulum & neck, but are absent from the body & fundus. • It is covered by the serosa except where it is embedded in the liver

Section 2

• GB lacks a muscularis mucosa & submucosaQ. • Normal capacity of the gallbladder is 30–50 mLQ. • Mucosa contain crypt of LuschkaQ. Cystic Duct • Cystic duct measures 2–4 cm in length & contains prominent concentric folds known as spiral valves of HeisterQ. • Cystic duct frequently exhibits a tortuous or serpentine courseQ. • Diameter of the cystic duct ranges from 1–5 mm. • Mucosa of the cystic duct is arranged in spiral folds known as valves of HeisterQ surrounded by a sphincteric structure called sphincter of LutkansQ.

Gallbladder

Functions of Gallbladder • • • •

Reservoir of bile Concentration of bile 5-10 timesQ Secretion of mucus, 20 mL/ day by tubuloalveolar glands Acidification of bile

Section 2

• Cystic artery is nearly found within the Hepatocystic triangleQ, the area bound by the cystic duct, common hepatic duct and the liver margin. • GB mucosa has the greatest absorptive capacityQ per unit of any structure in the body. • Hartmann’s pouch is an acquired diverticulumQ of the infundibulum or neck of the gallbladder.

175

Limey Bile • Calcium salts in the lumen of the GB in sufficient concentration may produce calcium precipitation and diffuse, hazy opacification of bile or a layering effect on plain abdominal X-rayQ. • Filled with a mixture of calcium carbonate and calcium phosphate usually, the consistency of toothpaste.Q • Caused by gradual obstruction of the cystic duct or CBD by chronic pancreatitis or carcinoma pancreasQ. • Organisms are rarely grown from emulsion. Diagnosis • Best revealed on plain radiographyQ Treatment • Limey bile, or milk of calcium bile, is usually clinically innocuousQ • Cholecystectomy is recommended when it occurs in a hydropic gallbladderQ. Composition of Bile Hepatic Bile 160.0 5 90 45 4 2

GB Bile 270.0 10 15 10 25 —

Bilirubin Protein Bile acids

1.5 150 50

15 — 150

Phospholipids Cholesterol

8 4

40 18

Total solids pH

— 7.8

125 7.2

• The inorganic solute has a concentration in bile that is similar to plasma and account for the bile osmolality of 300 mOsm/kg.

Courvoisier’s Sign • A palpable, nontender gallbladderQ • Usually results from a distal common duct obstruction secondary to a peripancreatic malignancyQ

Phrygian Cap • This anomaly is most common of gallbladderQ • Created by an infolding of a septum between body & fundusQ • GB functions normally, and this anomaly is not an indication for cholecystectomyQ.

Moynihan’s Hump (Caterpillar’s Turn) • Most dangerous anomaly (for cholecystectomy)



•  Right hepatic artery takes a tortuous turn.

Hepatobiliary Pancreatic Surgery

Characteristic Na+ K+ ClHCO3– Ca2+ Mg2+

Multiple Choice Questions GALLSTONES: PATHOGENESIS





1. All of the following are essential for formation of gallstones except: (MHSSMCET 2008) a. Bile stasis b. Nucleation c. Crystallization d. Lithogenic bile 2. Cholesterol gallstones are made up of:(Recent Question 2019) a. Crystalline cholesterol monohydrate b. Crystalline cholesterol dihydrate c. Amorphous cholesterol monohydrate d. Amorphous cholesterol dihydrate

3. All are true about pigmented stones except:  (AIIMS GIS Dec 2006) a. Seen in cholangiohepatitis b. Secondary CBD stones c. Primary CBD stones d. More common in Asians 4. False about brown pigmented stones: (AIIMS GIS May 2008) a. Associated with disorders of biliary motility and associated bacterial infection b. More common in Caucasians c. Soft and earthy in texture d. High content of cholesterol and calcium palmitate 5. Lithogenic bile has the following properties: (All India 96) a. ↑ Bile and cholesterol ratio b. ↓ Bile and cholesterol ratio c. Equal bile and cholesterol ratio d. ↓ Cholesterol only 6. Stone formation in gallbladder is enhanced by all except: (All India 96) a. Clofibrate therapy b. Ileal resection c. Cholestyramine therapy d. Vagal stimulation 7. Gallbladder stone formation is influenced by all except: a. Clofibrate therapy  (All India 98) b. Hyper alimentation c. Primary biliary cirrhosis d. Hypercholesterolemia 8. Incidence of gallstone is high in: (AIIMS Nov 93) a. Partial hepatectomy b. Ileal resection c. Jejunal resection d. Subtotal gastrectomy 9. True statement about gallstones are all except:  (AIIMS Nov 99) a. Lithogenic bile is required for stone formation b. May be associated with carcinoma gallbladder c. Associated with diabetes mellitus d. More common in males between 30–40 years of age 10. Which among the following does not lead to pigment gallstones? (PGI June 99) a. TPN b. Clonorchis sinensis c. Hemolytic anemia d. Alcoholic cirrhosis 11. True about gallstones: (PGI Dec 2002) a. More common in females b. Gallstones, hiatus hernia, CBD stones form Saints triad c. Limey bile precipitated d. Lithotripsy always done 12. All are component of Saint’s triad except: (AIIMS Nov 95) a. Renal stones b. Hiatus hernia c. Diverticulosis of colon d. Gallstones





13. Commonest type of gallstone is: (DNB 2011) a. Cholesterol stone b. Pigment c. Mixed d. All are equally common 14. Percentage of gallstones which are radiopaque:  (NEET 2013, JIPMER 86) a. 10% b. 20% c. 30% d. 40% 15. Which of the following statement is correct about gallstones? a. 1-Cholesterol, 2-Black, 3-Brown b. 1-Cholesterol, 2- Brown, 3- Black c. 1- Brown, 2-Black, 3- Cholesterol d. 1- Black, 2- Brown, 3- Cholesterol

Gallbladder

28. Mercedes Benz sign or Seagull sign is seen in:  (Recent Quetion 2015 MHSSMCET 2006) a. Gallstones b. Renal stones c. CBD stones d. Hydatid cyst



17. Gallstones do not contain: a. Oxalate c. Phosphate

(Recent Question 2014) b. Cholesterol d. Carbonate





18. Cholesterol gallstones are due to:  a. Decreased motility of gallbladder b. Hyposecretion of bile salts c. Hypercholesterolemia d. All of the above

(JIPMER 95)

29. Which is not required for visualization of gallbladder in oral cholecystograph? (AIIMS Nov 95, All India 97) a. Functioning liver b. Motor mechanisms of gallbladder c. Patency of cystic duct d. Ability to absorb water



30. What is the diagnosis based on the given HIDA-scan?



19. True color of cholesterol stone is: a. Black b. Brown c. Dark yellow d. Pale yellow

(DNB 2012)



20. The predominant constituent of the pale yellow gallstones in the gallbladder is: (COMEDK 2007) a. Mucin glycoprotein b. Calcium carbonate c. Cholesterol d. Calcium phosphate



21. Most common type of gallstone in India is:(MCI March 2009) a. Cholesterol b. Pigment c. Mixed d. Both A and C



22. Calculous cholecystitis is associated with all of the following except: (MCI March 2005) a. Oral contraceptives b. Estrogen c. Obesity d. Diabetes 23. Which is true about gallstones? (Punjab 2010) a. Pigment stones are most common b. Bacterial nidus of infection may be seen c. Even if asymptomatic gallbladder should be removed d. They are mostly solitary 24. By definition pigment stone contain how much % of cholesterol? (MHSSMCET 2005) a. 1 cm b. With stone c. >3 in number d. Locally invasive 96. Risk factors for malignant change in an asymptomatic patient with a gallbladder polyp on ultrasound include all of the following, except: (AIIMS May 2011, All India 2009) a. Age > 60 years b. Rapid increase in size of polyp c. Size of polyp > 5 mm d. Associated gallstones 97. Identify the diagnosis of the given gross specimen:  (AIIMS November 2017)

a. Cancer gallbladder c. Strawberry gallbladder

b. Cholesterolosis d. Polyps in gallbladder

CARCINOMA GALLBLADDER PREDISPOSING FACTORS 102. True about CA GB and gallstones: (GB Pant 2011) a. 3% association b. 30% association c. 50% association d. 90% association 103. Precancerous lesions of GB are all except:  (AIIMS GIS Dec 2010) a. Porcelain GB b. Typhoid carrier c. ABPDJ d. Biliary ascariasis 104. Organism associated with fish consumption and also causes carcinoma gallbladder: (AIIMS Nov 2012, AIIMS Nov 2010) a. Gnathostoma b. Anglostrongyloidosis cantonensis c. Clonorchis sinensis d. H. dimunata 105. Risk factor for carcinoma gallbladder: (PGI Nov 2011) a. Female sex b. Choledochal cysts c. Xanthogranulomatous cholecystitis d. Calcification of gallbladder e. Gallstone 106. All are risk factors for CA GB except: (GB Pant 2011) a. Adenomyosis b. ABPDJ c. Gallstones d. Adenomatous polyps

Hepatobiliary Pancreatic Surgery

GALLBLADDER POLYP AND ADENOMYOMATOSIS

Section 2





181

Hepatobiliary Pancreatic Surgery

182

Surgery Essence 107. All of the following are risk factors for carcinoma gallbladder, except: (AIIMS June 2004) a. Typhoid carriers b. Adenomatous gallbladder polyps c. Choledochal cyst d. Oral contraceptives 108. Precancerous lesion of gallbladder is: (AIIMS June 98) a. Porcelain gallbladder b. Mirizzi’s syndrome c. Cholesterosis d. Acalculous Cholecystitis 109. All of the following are the risk factors for carcinoma gallbladder except: (Recent Question 2017) a. Primary sclerosing cholangitis b. Porcelain gallbladder c. Multiple 2 cm gallstones d. Choledochal cyst 110. All of the following are true about porcelain gallbladder except: (Kerala PG 2015) a. May be seen on plain X-ray b. More commonly diagnosed on CT c. It is an indication for cholecystectomy d. Always denotes benign etiology 111. Incidental finding in a female patient of age 56 years who underwent the CECT abdomen is suggestive of: a. Carcinoma gallbladder b. Gallbladder polyp c. Porcelain gallbladder d. Gallstone

118. All are true about CA GB except: (GB Pant 2011) a. Redo surgery is radical or extended cholecystectomy increases significant survival advantage b. Inter-aortocaval node involvement potentially rule out cure c. 2 years of age: Abdominal painQ • In children, the major clinical symptoms are recurrent abdominal pain (81.8%), nausea & vomiting (65.5%), mild jaundice (43.6%), an abdominal mass (29.0%), and fever (29.0%). • In adults, abdominal pain (87%) and jaundice (42%) are present frequently. Less common clinical findings include nausea (29%), cholangitis (26%), pancreatitis (23%), and an abdominal mass (13%). Diagnosis

Type I Type II Type III Type IVA Type IVB Type V

Treatment of Choledochal Cyst Roux-en-Y hepaticojejunostomyQ Excision with T-tube repairQ Roux-en-Y hepaticojejunostomyQ Endoscopic sphincterotomy and cyst unroofingQ Hepatic resection for localized diseaseQ Liver transplantation for diffuse diseaseQ Transduodenal sphincteroplasty and Roux-en-Y hepaticojejunostomyQ • Hepatic resection for localized diseaseQ • Liver transplantation for diffuse diseaseQ • • • • • • •

Complications of Choledochal Cyst • Recurrent cholangitisQ • PancreatitisQ • GallstonesQ

• Cirrhosis with portal hypertension • Portal vein thrombosis • MalignancyQ

Lilly technique • Lilly technique: Serosal surface of the duct is left adhering to portal vein, while the mucosa of cyst wall is obliterated by curettage or cautery, when cyst is densely adhered to the portal vein secondary to long-standing inflammatory reactionQ. • In this situation, a complete, full-thickness excision of the cyst may not be possible.

CHOLEDOCHOLITHIASIS Choledocholithiasis • CBD stones are classified by point of origin • Found in 6–12% of patients with GB stonesQ

Hepatobiliary Pancreatic Surgery

• IOC for choledochal cyst: MRCP (non-invasive) • ERCP: More useful in defining the distal ductal anatomy and the presence of APBDJ • PTC: Useful in defining the proximal ductal anatomy and the presence of intrahepatic disease.

196

Surgery Essence

Hepatobiliary Pancreatic Surgery

• Retained stones discovered within 2 years of cholecystectomyQ • Recurrent stones detected >2 years following cholecystectomyQ Primary CBD stone • • • •

Secondary CBD stone

Formed within the biliary tractQ Associated with biliary Stasis and infectionQ More commonly seen in AsianQ populations Soft, friable, light-brown stones or sludge in the common ductQ

• • • •

Formed initially in the GBQ Migrate through the cystic duct into CBDQ Most common bile duct stones in Western countriesQ Usually cholesterol stonesQ

Clinical Features • CBD stones may be silent and are often discovered incidentallyQ. In these patients, biliary obstruction is transient, and laboratory tests may be normal. • Clinical features suspicious for biliary obstruction due to CBD stones include biliary colic, jaundice, clay colored stools, and darkening of the urineQ. • Fever and chills may be present in patients with choledocholithiasis and cholangitis. • Serum bilirubin (>3.0 mg/dL), aminotransferases, and ALP are commonly elevated in patients with biliary obstruction but are neither sensitive nor specific for the presence of common duct stones. • Among these, serum bilirubin has the highest positive predictive valueQ (28%–50%) for the presence of choledocholithiasis. • Laboratory values may be normal in one thirdQ of patients with choledocholithiasis. Diagnosis • USG: First test, can document GB stones and estimate the CBD diameterQ • A dilated bile duct (>8 mm in diameter) in a patient with gallstones, jaundice and biliary pain is highly suggestive of choledocholithiasis. • MRCP: Provides excellent anatomic detail, with sensitivity and specificity of 95% and 98%, respectively, for CBD stonesQ. • ERCP: Diagnostic and therapeutic test of choice for patients with suspected CBD stonesQ. Treatment • Treatment options are ERCP, laparoscopic or open CBD Exploration.

CHOLANGITIS Cholangitis • Ascending bacterial infectionQ of the biliary ductal system with obstruction • MC cause of acute cholangitis is choledocholithiasisQ • MC organisms present in the bile in patients with cholangitis: E. coliQ, Klebsiella pneumoniaeQ, Streptococcus faecalis, & Bacteroides fragilis. Etiology • Choledocholithiasis (MC)Q • Biliary-enteric anastomotic strictures

• •

Benign strictures Cholangiocarcinoma and periampullary cancer

Clinical Features • Characterized by Charcot’s triadQ: Abdominal pain + jaundice + fever • Cholangitis may be either self-limited or toxic with severe illness, including jaundice, fever, abdominal pain, mental status changes, and hypotension (Reynold’s pentad)Q.

Section 2

• Fever & chills are the MC presentation (due to cholangiovenous & cholangiolymphatic reflux)Q • Fever is the most consistent sign, generally intermittent, spiking and associated with shaking chillsQ. Diagnosis • Leukocytosis, hyperbilirubinemia, and raised ALP and transaminasesQ • Positive blood culture is more common in partial obstructionQ than with complete obstruction • Cholangiography (If ERCP is not available, PTC should be performed) is mandatory as a diagnostic and potentially therapeutic interventionQ. Treatment • Initial treatment: IV antibiotics & aggressive hydrationQ

Bile Duct Septic shock with toxic cholangitis: ICU monitoring and vasopressors to support blood pressure. Most patients will respond to these measures alone. Urgent biliary decompression will be necessary in 15% casesQ. Biliary decompression may be performed endoscopically or by a percutaneous transhepatic route based on the level of the obstruction.

Methods of Biliary Decompression ERCP with Sphincterotomy and Stone Extraction

Percutaneous Transhepatic Cholangiography (PTC)

Surgical Decompression

• Procedure of choiceQ • Early endoscopy is diagnostic & therapeuticQ • Permits biliary decompression by sphincterotomy & stone extractionQ • If stone can’t be removed, a nasobiliary catheter or stent is inserted to decompress biliary tractQ

PTC is performed if: • ERCP has failed or not availableQ • Proximal or hilar obstructionQ • Stricture of biliary enteric anastomosisQ

• Surgical decompression is indicated when neither ERCP nor PTC is possibleQ. • Consists of CBD decompression with a T-tubeQ

Section 2

• • • •

197

Elective Definitive Treatment in Stabilized Patients Cholecystectomy with choledochotomy and CBD explorationQ T-tube is left in place for cholangiography and removal of any retained stoneQ T-tube cholangiogram is done on 7th-10th day post-operativelyQ Remove the T-tube if cholangiogram is normalQ If residual stone is discovered on the post-operative cholangiogram, T-tube should be left in place for 4-6 weeks for the tract to matureQ. • The stones are removed percutaneously through the matured tract by Burhenne’s techniqueQ. • • • • •

ERCP

• Endoscopic clearance of CBD stones can avoid the need for an open operationQ if expertise in laparoscopic common bile duct exploration is not available. Indications of preoperative ERCP • Patients with worsening cholangitisQ • Ampullary stone impactionQ

• Biliary pancreatitisQ • CirrhosisQ

• If clearance is not possible because of multiple stones, intrahepatic stones, impacted stones, difficulty with cannulation, duodenal diverticula, or biliary stricture, this information is known before surgery. • Endoscopic sphincterotomy with stone extraction is well tolerated in most patients, with a 5–8% complication rateQ. • Complete clearance is achieved in 71–75% of patients at the first procedure and in 84–95% of patients after multiple endoscopic proceduresQ. • Prompt cholecystectomy after endoscopic clearance of the CBD should be performed during the hospital admission if the patient is fit for surgeryQ. • Patients > 70 years should have their ductal stones cleared endoscopically as their sole therapy; only about 15% become symptomatic from their Gallstones in their remaining lifetime, which can then be treated as symptoms ariseQ. Contraindications of Endoscopic sphincterotomy • • • •

CBD diameter >2 cmQ Long suprasphincteric stricture, >15 mmQ Peri-vaterian diverticulumQ Duodenal wall and head of the pancreas severely inflamedQ

BILE DUCT INJURY AND LIGATION Bile Duct Injury and Ligation • • • •

Most benign strictures follow iatrogenic bile duct injuryQ Most commonly during laparoscopic cholecystectomyQ Incidence of bile duct injury during open cholecystectomy is 0.1–0.2%Q Incidence of bile duct injury during laparoscopic cholecystectomy is 0.3–0.85%Q

Hepatobiliary Pancreatic Surgery

ERCP

198

Surgery Essence

Hepatobiliary Pancreatic Surgery

Pathogenesis Risk Factors for Bile Duct Injury • Acute or chronic inflammationQ • ObesityQ

• Anatomic variationQ • BleedingQ

• Bile duct injury rate increased in acute cholecystitis, pancreatitis, cholangitis, & obstructive jaundiceQ. • As surgeon experience increases beyond 20 cases, bile duct injury rate decreasesQ. • Errors leading to laparoscopic bile duct injuries stem from ‘misperception’Q, not erorrs of skill, knowledge or judgment. • Primary cause of error in 97% cases was a ‘visual perceptual illusion’Q, whereas only 3% injuries were due to faults of technical skillsQ.

• Surgical technique with inadequate exposure and failure to identify structures before ligating or dividing them are the most common cause of significant biliary injuryQ. • Routine use of intra-operative cholangiography may limit the extent of bile duct injury, but does not seem to prevent itQ. • If a bile duct injury is suspected during cholecystectomy, a cholangiogram must be obtained to identify the anatomyQ. • Classic Laparoscopic Injury: A long length of the common duct is excised up to the proximal common hepatic duct, which is either occluded or left to drain bile into the peritoneal cavity. Clinical Features • About 25% of major ductal injuries are recognized intraoperativelyQ because of bile leakage, an abnormal cholangiogram, or late recognition of the anatomy. • MC presentation of a complete occlusion of the common hepatic or bile duct is jaundice with or without abdominal painQ. • Patients may also present months or years after prior surgery with cholangitis or cirrhosis secondary to a biliary tract injury. Diagnosis • USG or CT should be performed in patients with signs of abdominal pain or peritonitis, sepsis, or any other clinical suspicion of bilomaQ. • Patients must be stabilized with immediate parenteral antibiotics and image-guided percutaneous drainage of any fluid collectionsQ. • Cholangiography should be performed to establish the presence of ductal stricture, identify the level of the stricture, and identify the nature of the injury when necessaryQ. • PTC is the imaging method of choice for most postoperative biliary stricturesQ • ERCP may be easier to obtain in a patient with a biliary stricture and cholangitis who requires urgent cholangiography and biliary decompression. • However, this is only useful in patients with bile duct continuity. • Cystic duct leaks or small tangential injuries can be treated with endoscopic stenting. • In situations in which the biliary stricture is too tight to pass with ERCP, PTC may be performed for proximal biliary decompression. • CT arteriography should be considered in the preoperative evaluation of patients with benign biliary strictures. • Unrecognized injury to the hepatic artery or a portal vein branch occurs with a frequency of 12–47% concomitant with a bile duct injury. • In patients presenting with late strictures with evidence of liver dysfunction, a CT arteriogram should be performed to evaluate for evidence of portal hypertension.

BISMUTH CLASSIFICATION OF BILE DUCT STRICTURES

Section 2

Bismuth Classification of Bile Duct Strictures Type 1

Low common hepatic duct stricture; hepatic duct stump >2 cmQ

Type 2

Mid common hepatic duct stump < 2 cmQ

Type 3

High stricture (hilar), no hepatic duct stump; confluence intactQ

Type 4

Destruction of the hilar confluence; right and left hepatic ducts separatedQ

Type 5

Involvement of aberrant right sectoral hepatic duct alone with or without a concomitant hepatic duct strictureQ

• The shortcomings in Bismuths classification were that firstly it did not stipulate the length of the stricture, and secondly, and more importantly it did not take into account the presence of biliary leaks, which are more common after laparoscopic cholecystectomy. • This, latter fact has been incorporated in Strasberg’s classification; also patients with limited strictures, isolated right hepatic duct strictures or cystic duct leaks cannot be classified.

Bile Duct

199

STRASBERG CLASSIFICATION OF LAPAROSCOPIC BILIARY INJURIES

Type B Type C Type D Type E

• • • • • • •

Strasberg Classification of Laparoscopic Biliary Injuries Bile leaks from minor ducts still in continuity with the CBDQ Includes leakage from cystic duct stump and from a subvesical duct of LuschkaQ MC causes of biliary leaks seen after laparoscopic cholecystectomyQ Occlusion of a part of the biliary tree, almost always an aberrant right sectoral ductQ Transection without ligation of an aberrant right sectoral ductQ A lateral injury to an extrahepatic ductQ Includes biliary strictures, divided into E1 to E5 as classified by Bismuth

Section 2

Type A

Strasberg classficiation of postoperative bile duct strictures

• • • • •

Amsterdam Academic Medical Center’s classification Neuhaus’ classification Csendes’ classification Stewart-Way’s classification of laparoscopic bile duct injuries Chinese University of Hong Kong (CUHK) classification

MANAGEMENT OF THE BILE DUCT INJURY RECOGNIZED AFTER CHOLECYSTECTOMY Management of the Bile Duct Injury Recognized after Cholecystectomy • Patients with a bile leak will present earlyQ • Patients with postoperative biliary strictures alone often present with jaundice or cholangitis months to years after the initial injuryQ. Intraoperative Considerations • Management of postoperative biliary strictures following ductal injury depends on the degree of injury, the presence of stricture-induced complications, and the operative risk of the patient. • • • •

Planning of the following specific goals Control the infection (abscess or cholangitis)Q Drain the bilomaQ Complete the cholangiographyQ Provide definitive therapy with controlled reconstruction or stentingQ

• Independent predictors of stricture recurrence after an initial operative repair: − Cholangitis before the initial repair            − Incomplete cholangiography   −− Primary repair within 3 weeks of the bile duct injury • If immediate repair is to be attempted, consultation with more experienced surgeons should be made. • In those cases in which expertise is not available at the time of a recognized injury, or difficult circumstances preclude elaborate reconstructive attempts, external biliary drains will allow patient recovery and transfer to a center of excellenceQ. Successful repair of biliary strictures requires adherence to specific surgical principles • Use of proximal bile duct with minimal inflammationQ • Creation of a tension-free anastomosis with the use of a Roux-en-Y jejunal limbQ • Direct mucosa-to-mucosa anastomosisQ

Hepatobiliary Pancreatic Surgery

Other Classification Systems for Bile Duct Injuries

Hepatobiliary Pancreatic Surgery

200

Surgery Essence • Direct operative biliary-enteric bypass is the gold standard procedureQ for the long-term treatment of biliary strictures. These procedures have low operative mortality and acceptable morbidity. • Hepp-Couinaud approach to bile duct reconstruction is the best optionQ in most circumstances. This technique requires dissection of the hilar plate to expose the left hepatic duct and allow for a side-to-side anastomosis of the left hepatic duct to the Roux-en-Y jejunal limb. Successful repair of biliary strictures requires adherence to specific surgical principles • Use of proximal bile duct with minimal inflammationQ • Creation of a tension-free anastomosis with the use of a Roux-en-Y jejunal limbQ • Direct mucosa-to-mucosa anastomosisQ • Direct operative biliary-enteric bypass is the gold standard procedureQ for the long-term treatment of biliary strictures. These procedures have low operative mortality and acceptable morbidity. Interventional Radiologic and Endoscopic Techniques • These techniques allow: −− Percutaneous drainage of abdominal fluid collections −− Preoperative identification of the ductal anatomy through PTC −− Stricture dilation with or without placement of palliative stents for bile drainage in the patient whose overall physiologic status precludes a major operation. • Percutaneous transhepatic dilation can be employed in patients with intrahepatic ductal disease and in patients in whom ERCP is not possible. • Success rate of percutaneous transhepatic dilation is 50–70%Q. • Patients with anastomotic strictures (including biliary-enteric anastomotic strictures) have the highest success ratesQ. • Treatment of biliary strictures with interventional radiologic methods, requires multiple sessions of dilations, and nonischemic strictures (anastomotic strictures) respond bestQ. • Endoscopic stenting and drainage is a successful treatment option for cystic duct leak or small common bile duct leaks following laparoscopic cholecystectomy. • Ischemic biliary strictures will not respond permanently to dilation. Early retreatment (through repeat dilation or biliary-enteric reconstruction) of postdilation recurrent strictures is essential to prevent secondary biliary cirrhosisQ.

Goals of Therapy in Iatrogenic Bile Duct Injury • Control of infection limiting inflammation: − Parenteral antibioticsQ − Percutaneous drainage of periportal fluid collectionQ • Clear and thorough delineation of entire biliary anatomy: −− MRCP/PTCQ − ERCP (especially if cystic duct stump leak suspected)Q • Re-establishment of biliary enteric continuity: −− Tension-free, mucosa-to-mucosa anastomosisQ − Roux-en-Y hepaticojejunostomyQ −− Long-term transanastomotic stents if involving bifurcation or higherQ

BILIARY FISTULA Biliary Fistula

Section 2

• Internal fistulas are spontaneous, rare, and occur without a significant collection of bile. • External fistulas are more common and are often caused by iatrogenic injury after operations, invasive procedures, or trauma involving the biliary tract.

External Biliary Fistula • External fistulas are more common and are often caused by iatrogenic injury after operations, invasive procedures, or trauma involving the biliary tract. Etiology • Bile leakage from the cystic duct remnantQ • Central hepatectomy and caudate resectionQ

•  Difficult cases of open cholecystectomyQ •  Hepatic cryotherapy or harmonic scalpel useQ

Treatment • Patients with leaks from the cystic duct, duct of Luschka, and T-tube tract are optimal candidates for endoscopic treatmentQ. • Patients treated with stents alone experience equally good outcomes as patients treated with a combination of stents and sphincterotomyQ

Bile Duct

201

Internal Biliary Fistula

• Calculous biliary tract disease (90%)Q • Duodenal ulcer (6%) • Neoplasm, trauma, parasitic infestation, and congenital anomalies (4%) Diagnosis

Section 2

• Internal fistulas are spontaneous, rare, and occur without a significant collection of bile. • Cholecystoduodenal fistulas (72–80%) are MC biliary-enteric fistulas followed by cholecystocolic fistulas (8-12%Q) • Most cholecystoduodenal fistula are asymptomaticQ. • The site of fistula most commonly located in the vaterian segment of the CBD in case of the choledochoduodenal fistulaQ. Etiology

• Only one-third of biliary-enteric fistulas will present with air in the biliary treeQ. • A negative upper gastrointestinal series in the presence of pneumobilia is an indication for a barium enema, which discloses greater than 95% of cholecystocolic fistulaeQ. Management • In the absence of obstruction, residual stones, or symptoms, except for cholecystogastric and cholecystocolic fistulas, no operation should be performed because most fistulas close spontaneouslyQ. • For cholecystocolic fistula, choledochotomy is recommended as a first step, followed by cholecystectomy, and finally takedown and repair of the fistula to reduce bacterial contaminationQ. • In other cases, the usual approach is repair of the fistula, then cholecystectomy and closure of the bowel.

SPHINCTER OF ODDI DYSFUNCTION Sphincter of Oddi Dysfunction

• Sphincter of Oddi Stenosis: −− Also known as papillitisQ −  Benign intrinsic obstruction of the CBD outletQ • Sphincter of Oddi Dyskinesis: −− It is an intermittent functional blockage of the high-pressure zone of the sphincterQ −− Basal pressure is elevated, but administration of smooth muscle relaxants (nitrates) causes decrease of the basal sphincter pressure in functional dyskinesisQ Clinical Features −− Pain similar to biliary colic with normal LFT and episodes of acute pancreatitisQ Diagnosis • A dilated CBD (>12 mm diameter) or increase in CBD diameter in response to CCK is a typical ultrasound findingQ. • Endoscopic manometry is considered gold standard Q for diagnosis • Nardi TestQ: The most widely used pharmacologic test to assess sphincter function is the morphine-prostigmine provocation testQ Treatment • Treatment of choice: Transduodenal sphincteroplasty with transampullary septectomyQ

BILIARY ATRESIA Biliary Atresia • Characterized by progressive obliteration of the extrahepatic and intrahepatic bile ductsQ. • Etiology is unknown; incidence 1 in 12,000 live birthsQ. • Presently, there is no medical therapy to reverse the obliterative processQ • Patients who are not offered surgical treatment uniformly develop biliary cirrhosis, portal hypertension, and death by 2 years of ageQ. • MC indication for pediatric liver transplantationQ

Hepatobiliary Pancreatic Surgery

• Pain similar to biliary colic with normal LFT and episodes of acute pancreatitis have been attributed to a poorly defined syndrome known as Sphincter of Oddi dysfunctionQ. • The pathogenesis is unclear • Postulated theories include gallstone migration inducing fibrosis of the sphincter, trauma, pancreatitis, and congenital anomalies. • Modified Milwaukee classification is used for Biliary Sphincter of Oddi dysfunctionQ Types

202

Surgery Essence

Hepatobiliary Pancreatic Surgery

Pathology • Bile duct proliferation, severe cholestasis with plugging, and inflammatory cell infiltrate are the pathologic hallmarks of this diseaseQ. • Over time, these changes progress to fibrosis with end-stage cirrhosisQ. • Positive for neural cell adhesion molecule (CD56) staining Classification

• • • •

Type I Type IIa Type IIb Type III

• • • •

Kasai Classification of Biliary Atresia CBD is obliterated, proximal bile ducts are patentQ Atresia of common hepatic duct (CHD) with patent cystic duct & CBDQ Obliteration of CHD, cystic duct & CBDQ Atresia of CBD, cystic duct & hepatic ducts upto porta hepatisQ (MC type, responsible for > 90% casesQ)

Clinical Features • Infants with biliary atresia present with jaundice at birth or shortly thereafterQ. • Infants with biliary atresia characteristically have acholic, pale gray stools, secondary to obstructed bile flow. • With passage of time, progressive failure to thrive and, if untreated, develop stigmata of liver failure and portal hypertension (splenomegaly and esophageal varices) • Associated malformations in 25%: Polysplenia, malrotation, preduodenal portal vein, and intrahepatic vena cavaQ. Diagnosis • USG of the liver and GB is important in the evaluation of the infant with cholestasis. • USG: GB is shrunken and CBD is not visibleQ. A triangle cord signQ found on ultrasound has a predictive accuracy of 95%, the gallbladder ghost triadQ in which the gallbladder is short (12 mmQ • Dilated cystic duct



46. Ans. a. ERCP



49. Ans. a. Meniscus sign (Ref: Sutton 6/e p971)

47. Ans. d. CBD stones 48. Ans. b. ERCP

• MR cholangiography: Typical “meniscus sign” is seen when CBD stoneQ is wedged at the level of the papilla. 50. Ans. a. Distended gallbladder

51. Ans. a. E. coli



52. Ans. c. Common bile duct stone (Ref: Bailey 27/e p1234) • In obstruction of the CBD due to a stone, distention of gallbladder seldom occurs; the organ usually is shriveledQ.



53. Ans. a. Stone in common bile duct

54. Ans. b. Endoscopic choledocholithotomy



55. Ans. d. Markedly elevated transaminases

56. Ans. c. Endoscopic papillotomy



57. Ans. c. Common bile duct (Ref: American Journal of Gastroenterology; Dec 1963, Vol. 41 Issue 6, p620)

Leucine Aminopeptidase • Increased Leucine aminopeptidase (LAP) activity is seen in: −− Carcinoma of the pancreas, choledocholithiasis, acute pancreatitisQ −− Viral hepatitis, cirrhosis, carcinoma with liver metastasesQ • In common bile duct obstruction, whether due to carcinoma pancreas or choledocholithiasis, the elevated serum LAP levels returned to normal following relief of the obstruction. This is in agreement with the hypothesis that the increased serum LAP activity in these conditions is the result of bile duct obstruction.

58. Ans. b. Distended and palpable gallbladder



59. Ans. a. Observation, c. Medical dissolution of stone (Ref: Bailey 27/e p1200; CSDT 11/e p1163) • Medical dissolution is used for Gallstones, not the CBD stones. • CBD stones are rarely asymptomatic, often present with complications. CBD stones should be treated, if diagnosed. Treatment options are ERCP, laparoscopic or open CBD exploration.

CHOLEDOCHOTOMY AND CBD EXPLORATION

60. Ans. d. Convert to open cholecystectomy and CBD stone removal (Ref: Shackelford 8/e p1290-1291, 7/e p1326-1331) Management of CBD Stones Associated with GB Stones Pre-operatively Detected Stones Unsuspected stones found at the time of Cholecystectomy Experienced Laparoscopic Surgeon Experienced Laparoscopic Surgeon • Cholecystectomy and choledochotomy in same • Laparoscopic CBD exploration and stone retrieval through the cystic sittingQ ductQ • Laparoscopic choledochotomy and stone extractionQ

Hepatobiliary Pancreatic Surgery



Hepatobiliary Pancreatic Surgery

220

Surgery Essence Inexperienced Laparoscopic Surgeon • Pre-op ERCP with stone removal and laparoscopic cholecystectomy laterQ.

Inexperienced Laparoscopic Surgeon • Convert to open procedure and remove CBD stoneQ • Complete the cholecystectomy and refer the patient for ERCPQ • Conversion to an open procedure is preferred over ERCPQ, because the success rate of ERCP is not 100%Q.



61. Ans. d. Abnormal gamma glutamyl transferase



62. Ans. a. Synthetic absorbable (Ref: Shackelford 8/e p1290-1291, 7/e p2222-2224) • Synthetic absorbable suture such as Vicryl is preferred for CBDQ. • Vicryl sutures are used in general soft tissue approximation and vessel ligationQ. • Non absorbable sutures ordinarily remain where they are buried within the tissues. This can cause late complications such as the development of Gallstones around non-absorbable sutures in the common bile duct or bladder stones in the urinary bladder. In these situations it is best to use absorbable materialsQ.



63. Ans. c. 12th postoperative day

64. Ans. c. T-tube cholangiogram (Ref: Sabiston 20/e p1501; Schwartz 10/e p1322)

65. Ans. b. 5–9 days



67. Ans. c. Endoscopic sphincterotomy and stone extraction



68. Ans. a. Retained stone (Ref: Sabiston 20/e p1496)

66. Ans. a. Supraduodenal choledochotomy

Most common complication of common bile duct exploration retained stone. • Clearance of all common bile duct stones is achieved in 75–95% of patients with laparoscopic CBD explorationQ. • The rate of retained CBD stone is 50% cases (Ref: Sabiston 20/e p1507-1508; Blumgart 6/e p727, 5/e p680-697) GB stones are present in b. Mirizzi’s syndrome 149. Ans. a. 7 cm (Ref: Grays 39/e p1228) 150. Ans. c. Use of heavily T2-weighted image without contrast to create the three dimensional image of the biliary tree using MIP algorithm (Ref: Blumgart 6/e p359-361, 5/e p315-315) MRI cholangiography and MRI cholangiopancreatography (MRCP) are imaging techniques used to evaluate the biliary system. Heavily T2-weighted images are used to provide an overview of the biliary system and pancreatic duct. Excellent diagnostic-quality images are obtainable, with high sensitivity and specificity for evaluation of biliary duct dilation, strictures, and intraductal abnormalities.

Magnetic Resonance Cholangiopancreatography

MIP (Maximum Intensity Projection) Algorithm • CE FAST (Fourier-acquired steady state) or FSE (Fat spinal echo) require image processing with a maximum intensity projection (MIP) algorithm, allowing rotation of summed image and display of the cholangiogram to best advantage. 151. Ans. a. Percutaneous transhepatic cholangiogram (Ref: Schwartz 10/e p1315; Sabiston 20/e p1489; Bailey 27/e p1193)

Hepatobiliary Pancreatic Surgery

• The basic principle of MRCP is to use T2-weighted images, in which stationary or slowly moving fluid, including bile, is high in signal intensity; all the surrounding tissues, including retroperitoneal fat and the solid visceral organs, are lower in signal. • MR-specific techniques for obtaining cholangiographic images include two-dimensional and three-dimensional sequences, breathhold or non–breath-hold techniques, and respiratory gated techniques. • MRCP plays an important role in imaging benign disorders of the biliary and pancreatic system, and it is part of a comprehensive imaging evaluation of malignancies of the biliary system. • MRCP is noninvasive, eliminating the morbidity associated with ERCP or PTC. • An additional advantage of MRCP includes visualization of the extrabiliary anatomy, allowing for exclusion or inclusion of alternative diagnoses. • Surgical clips may create an artifact known as susceptibility, which may obscure the region of interest by producing areas of signal void. This artifact may mimic a stone, so caution must be used in evaluating MRCP images in postoperative patients to avoid a false-positive diagnosis.

CHAPTER

8

Pancreas

PANCREAS DIVISUM Pancreas Divisum • MC congenital anomaly of the pancreasQ • It occurs when the ductal systems of the dorsal & ventral pancreatic duct fail to fuseQ during the second month of gestation. Types • Type 1 has two completely separate pancreatic ducts. • Type 2 has only a dorsal duct, with no evidence of a ventral duct of Wirsung. • Type 3 has a dominant dorsal duct with only a small, narrow filamentous connection between the dorsal and ventral ducts. −− Common to all variants of pancreas divisum is that all or most pancreatic secretion flows through the accessory papillaQ. Diagnosis • IOC for diagnosis of pancreas divisum: MRCPQ • Gold standard investigation for diagnosis: ERCPQ Treatment • Operative dorsal duct sphincterotomy, with or without sphincteroplastyQ, is the preferred surgical treatment. • Patients with pancreas divisum and acute recurrent pancreatitis are good candidates for endoscopic therapyQ whereas patients with chronic pancreatitis or chronic pain alone (or both) do not appear to do as well.

ANNULAR PANCREAS Annular Pancreas • Circumferential or near-circumferential band of pancreas tissue surrounding the 2nd part of duodenumQ • It is of ventral pancreas origin and is usually proximal to ampullaQ. • Duodenal stenosis or atresia is present at the site of annulus in 40%Q • Down’s syndromeQ (trisomy 21) is present in 15-25%. • Intestinal malrotation, tracheoesophageal fistula, & congenital heart defects, Meckel’s diverticulum and imperforate anus are also not uncommon. Diagnosis • Definitive diagnosis is made by ERCP. Treatment • Treatment of choice: DuodenoduodenostomyQ >Duodenojejunostomy. • Duodenoduodenostomy has replaced duodenojejunostomy as the treatment of choice because it has a lower incidence of postoperative complications, particularly obstruction & blind-loop syndromesQ.

PANCREATIC CYST Pancreatic Cyst • • • • •

Solitary (congenital, duplication, or dermoid) cysts of the pancreas are rareQ. Multiple pancreatic cysts, lined with cuboidal epithelium, are more commonQ. They are frequently associated with polycystic disease of the liver or kidneyQ. Can be seen in up to half of patients with von Hippel-Lindau diseaseQ. Pancreatic cysts only rarely become symptomatic, and no treatment is indicatedQ.

Pancreas

227

NESIDIOBLASTOSIS

• Also known as persistent hyperinsulinemic hypoglycemia of infancyQ • Characterized by diffuse hyperfunction of pancreatic beta cells with enlargement of their nucleiQ • Neither the beta cell proliferation rate nor the overall beta cell mass is increasedQ. Clinical Features • Early recognition of congenital hyperinsulinism is critical because, if untreated, profound hypoglycemia may lead to brain damage. • Babies may be described as jittery, floppy, or lethargic; seizures are commonQ Diagnosis • Inappropriately elevated insulin in the setting of hypoglycemiaQ, along with the need for continuous glucose infusion (60 mmol/L • Urinary trypsinogen activation peptide

• Ranson/Glasgow score ≥ 3Q • CRP ≥130Q mg/mL

• A CRP level ≥130 mg/mLQ defines severe pancreatitis.

At Admission • • • • •

During Initial 48 Hours

Age >55 yearsQ WBC >16,000Q cells/mm3 Blood glucose >200Q mg/dL Serum LDH >350Q IU/L AST >250Q U/L

• • • • • •

Hematocrit fall >10Q percentage points BUN elevation >5Q mg/ dL Serum calcium fall to 6Q Litres

Ranson’s Prognostic Criteria for Gallstone Pancreatitis At Admission • • • • •

During Initial 48 Hours

Age >70 years WBC >18,000 cells/mm Glucose >220 mg/ dL Serum LDH >400 IU/L AST >250 U/L

• • • • • •

Hematocrit fall >10 percentage points BUN elevation >2 mg/ dL Serum calcium fall to 5 mEq/L Arterial PO2 4 litres

• Patients with one or two criteria have a predicted mortality of less than 1%, with three criteria (10%) or four criteria (15%); with more than seven criteria 50%

Modified Glasgow Criteria • This system comprises eight factors. The presence of any 3 or more within 48 hours of admission defines the patient as having severe disease. Criteria during Initial 48 Hours • • • •

Age >55 years WBC count >15,000 cell/mm Blood urea nitrogen >45 mg/dL Arterial PO2 180 mg/dL Serum LDH >600 IU/L Serum calcium 25 mg/dL  •  Impaired mental status  •  SIRS ≥ 2 of 4 present  •  Age > 60 years  •  Pleural effusion` (Score 0–2: Mortality < 2%; Score 3–5: Mortality > 15%)

COMPLICATIONS OF ACUTE PANCREATITIS Complications of Acute Pancreatitis • • • •

Sterile and infected peripancreatic fluid collections Pancreatic necrosis and infected necrosis Pancreatic pseudocysts Pancreatic ascites

• Pancreaticopleural fistulas • Vascular complications • Pancreatocutaneous fistula

Vascular Complications of Acute Pancreatitis

Section 2

• Acute pancreatitis is rarely associated with arterial vascular complications. • MC vessel affected: Splenic arteryQ • Other vessels: Superior mesenteric, cystic, and gastroduodenal arteriesQ • • • • • • •

Vascular Thrombosis Pancreatic inflammation can produce vascular thrombosis MC affected vessel: Splenic veinQ in severe cases, it can extend into the portal venous system Imaging demonstrates splenomegaly, gastric varices, and splenic vein occlusion Thrombolytics have been described in the acute early phase Most patients can be managed with conservative treatment Recurrent episodes of upper gastrointestinal bleeding caused by venous hypertension should be treated with splenectomyQ

Pathogenesis • It has been proposed that pancreatic elastase damages the vessels, leading to pseudoaneurysm formation.

Pancreas

231

Clinical Features

• If possible, arterial embolization should be attempted to control the bleedingQ. • Refractory cases require ligation of the affected vessel. • The mortality ranges from 28-56%. • MC affected vessel in acute pancreatitis: Splenic artery (pseudoaneusysm formation)Q • MC affected vessel leading to vascular thrombosis caused by acute pancreatitis: Splenic veinQ

Section 2

• Spontaneous ruptureQ results in massive bleeding. • Clinical manifestations include sudden onset of abdominal pain, tachycardia, and hypotension. Treatment

Pancreatic Fistula • Output via an intra-operatively placed drain (or percutaneous drain) of any measurable volume or drain fluid on or after post-operative day 3, amylase >3 times of normal serum value. • Pancreatic fistula classified into low output (200 mL/day). Factors Known to Influence the Outcome of a Pancreatic Anastomosis • Texture of the pancreatic remnantQ

• Caliber of the main pancreatic ductQ

• Exocrine pancreatic juice output

• Surgical technique appliedQ

Q

• Texture of Pancreatic Remnant: Performing an anastomosis on a soft pancreas is more difficult than on a firm or hard pancreas. Thus, a pancreatic anastomosis in chronic pancreatitis has a lower risk of anastomotic failure compared to pancreatic cancerQ. • Pancreatic Juice Output: Decreased juice output is associated with a lowered risk of anastomotic failureQ. • Surgical Technique: An end-to-side pancreaticojejunostomy irrespective of the caliber of the main pancreatic duct and the texture of the pancreatic parenchyma. • Most cases resolve spontaneously by conservative treatmentQ • Benefit of prophylactic octreotide in the prevention of pancreatic fistulaQ • Role of octreotide, once a fistula is established remains unclearQ.

PSEUDOPANCREATIC CYST Pseudopancreatic cyst • A chronic collection of pancreatic fluid surrounded by a nonepithelialized wall of granulation tissue and fibrosisQ • Pseudocysts account 75% of cystic lesionsQ of the pancreas. • MC complication of chronic pancreatitisQ • • • •

Located anywhere from the mediastinum to the scrotumQ MC site of pseudocyst: Lesser sacQ Traumatic pseudocysts tend to occur anterior to the bodyQ of the gland Chronic pancreatitis pseudocysts are commonly located within the substance of the gland

• Incidence of Pseudocysts: − Acute pancreatitis: 10-20% of patientsQ          − Chronic pancreatitis: 20-40% of patients • Multiple in 17%Q cases • Alcohol is MC cause of pancreatitis related pseudocystsQ. Pathophysiology • Pancreatic duct leak with extravasation of pancreatic juice results in a pancreatic fluid collection (PFC). • Acute pseudocyst: Over a period of 3 to 4 weeksQ, the PFC is sealed by an inflammatory reaction that leads to development of a wall of acute granulation tissue without much fibrosis. • Acute pseudocysts may resolve spontaneously in up to 50% of cases, over a course of 6 weeks or longerQ. • Pseudocysts >6 cm resolve less frequently than smaller ones but may regress over a period of weeks to monthsQ. Clinical Features • Pseudocysts usually cause symptoms of pain, fullness, or early satiety. • Abdominal pain is MC symptomQ, occurs in up to 90% of patients • Other common symptoms include early satiety, nausea and vomiting (50% to 70%), weight loss (20% to 50%), jaundice (10%), and low-grade fever (10%)Q

Hepatobiliary Pancreatic Surgery

Management

Hepatobiliary Pancreatic Surgery

232

Surgery Essence • Physical examination: Upper abdominal tenderness in the majority of patients, and 25-45% will have a palpable abdominal massQ. • Symptoms of early satiety, nausea, and vomiting may be secondary to gastroduodenal obstruction caused by a mass effect of the pseudocyst. Pseudocyst Complications (Shackelford 7th/1159) • Infection (MC)Q: 14%

• Duodenal obstruction

• Pain due to expansion

• Rupture

• Hemorrhage: up to 10%

• Abscess

Diagnosis • • • •

No definitive laboratory findings are available to establish a diagnosis of pancreatic pseudocyst. Elevated serum amylase & lipaseQ concentrations may occur in half of these patients. Persistently elevated amylase after resolution of atitis should prompt investigation for a pseudocyst. CECT abdomen is investigation of choiceQ for diagnosis of a pancreatic pseudocyst.

Treatment • Pseudocyst 5 cm in diameter and 5 cm diameter is an indications for drainageQ • If infection is suspected, the pseudocyst should be aspiratedQ (not drained) by CT- or US-guided FNA, and the contents examined for organisms by Gram’s stain and culture • If infection is present, and the contents resemble pus, external drainage is employed, using either surgical or percutaneous techniques • Pseudocysts communicate with the pancreatic ductal system in up to 80% of cases, so external drainage creates a pathway for pancreatic duct leakage to and through the catheter exit site. • Methods of Internal drainage: −− Percutaneous catheter-based methods (transgastric puncture and stent placement to create a cystogastrostomy) −− Endoscopic methods (transgastric or transduodenal puncture and multiple stent placements, with or without a nasocystic irrigation catheter) −− Surgical methods (a true cystoenterostomy, biopsy of cyst wall, and evacuation of all debris and contents) • Surgical options: Cystogastrostomy, Roux-en-Y cystojejunostomy, cystoduodenostomy. • Preferred modality: Internal drainage of cyst by cystojejunostomyQ

• Cystojejunostomy has a slightly lower recurrence rate, but it is associated with significantly more blood loss and operative time The D’EGIDIO Classification of Pancreatic Pseudocyst Context

Pancreatic Duct

Duct-pseudocyst

Treatment

Communication Type I

Acute post-necrotic pancreatitis

Normal

No

Percutaneous drainage

Type II

Acute-on Chronic pancreatitis

Abnormal (no stricture)

50:50

Internal drainage or resection

Type III

Chronic pancreatitis

Abnormal (stricture)

Yes

Internal drainage with duct decompression

HEREDITARY PANCREATITIS

Section 2

Hereditary Pancreatitis • Autosomal dominantQ disease • Due to a missense mutation on cationic trypsinogen, or PRSS1Q (Protease, Serine 1) results in premature, intrapancreatic activation of trypsinogen. • The incidence is equal in both sexes. Clinical Features • Characterized by recurrent episodes of acute pancreatitis or familial aggregation of chronic pancreatitisQ • Typically, patients first present in childhood or adolescence with abdominal pain and are found to have chronic calcific pancreatitis on imaging studies. • Progressive pancreatic dysfunction is common, and many patients present with symptoms due to pancreatic duct obstructionQ. • The risk of subsequent carcinoma formationQ is upto 40%, age of onset for carcinoma is >50 years old.

Pancreas

233

Clinical Pancreatic Syndromes and Associated Genetic Mutations • PRSS1Q (Cationic trypsinogen) gene

• Idiopathic chronic Pancreatitis

• CFTRQ

• Tropical calcific Pancreatitis

• SPINK1 (PTSI)Q

TROPICAL (NUTRITIONAL) PANCREATITIS Tropical (Nutritional) Pancreatitis • Tropical chronic pancreatitis is highly prevalent among adolescents and young adults raised in Indonesia, southern India, and tropical AfricaQ. • Associated with mutations of PSTI or SPINK1 geneQ • It is subdivided into: - Tropical calcific pancreatitis, characterized by severe, recurrent, chronic abdominal pain and extensive pancreatic calcificationsQ. - Fibrocalculous pancreatic diabetes, which is characterized by significant pancreatic endocrine insufficiencyQ

Section 2

• Hereditary Pancreatitis

Etiopathogenesis • Protein-caloric malnutrition and toxic products of some indigenous foodstuffsQ may contribute to the disease. • Cassava root contains toxic glycosides, increases susceptibility to free radical injury of the pancreasQ Clinical Features

CHRONIC PANCREATITIS Chronic Pancreatitis • Characterized by the persistent inflammation & irreversible fibrosis associated with atrophy of pancreatic parenchymaQ. • Associated with chronic pain and endocrine & exocrine insufficiencyQ • In most cases of chronic pancreatitis, exocrine insufficiency precedes endocrine insufficiency by many yearsQ • Exocrine insufficiency is more closely related with morphologic changesQ. • Approximately 90% of beta cell mass must be lost before clinical diabetes develops. • Classification of various causes of chronic pancreatitis based on the TIGAR-O systemQ (TIGAR-O consist of toxic-metabolic, idiopathic, genetic, autoimmune, recurrent severe, obstructive). Etiology • Heavy alcohol consumption is MC cause of CP (70-80%)Q • Smoking increases the riskQ of alcohol-induced CP. • Other causes: Chronic duct obstruction, trauma, pancreas divisum, cystic dystrophy of the duodenal wall, hyperparathyroidism, hypertriglyceridemia, autoimmune pancreatitis, tropical pancreatitis, & hereditary pancreatitis (account for 7 mm) require a decompressing procedure and patients with normal pancreatic duct require a resectional procedureQ

Stage Normal Equivocal Mild Moderate Severe

Cambridge Criteria for Chronic Pancreatitis on ERCP Typical Changes Normal appearance of side branches and main pancreatic duct Dilatation/obstruction of < 3 side branches; normal main pancreatic duct Dilatation/obstruction of > 3 side branches; normal main pancreatic duct Additional stenosis and dilatation of main pancreatic duct Additional obstructions, cysts, stenosis of main pancreatic duct; calculi

RESECTION PROCEDURES IN CHRONIC PANCREATITIS Resection Procedures in Chronic Pancreatitis • It is believed that inflammatory process in the pancreatic head controls both the severity of symptoms and further progression of disease in remainder of the gland • Pancreatic head is pacemaker of chronic pancreatitisQ. • Because of this resection of pancreatic head has been shown to completely relieve the pain of chronic pancreatitis in 70-80%Q patients. • Distal pancreatectomy is the ideal procedure for patients whose chronic pancreatitis is confined to pancreatitis tailQ. • Usually, distal pancreatectomy is combined with splenectomy for technical reasons, but spleen can be preserved if its vascular supply is secure.

Section 2

Surgical Procedures in Chronic Pancreatitis Drainage Procedure • Puestow ProcedureQ (Longitudinal Pancreaticojejunostomy): Resection of the tail followed by a longitudinal pancreaticojejunostomyQ • Partington and Rochelle Modification of Puestow Procedure: Elimination of the resection of the pancreatic tailQ. • Duval Procedure: Distal pancreatectomy with Rouxen-Y pancreaticojejunostomy (caudal PJ)Q

Resection Procedure • Beger’s Procedure (Duodenal Preserving Pancreatic Head Resection DPPHRQ) • Warren’s modification of Beger’s procedure • Frey’s Procedure (Local Resection of the Head of the Pancreas Combined with Longitudinal Pancreaticojejunostomy LR-LPJ)Q • Berne modification: (Combines some aspects of Beger’s and Frey’s) • Hamburg modification of Frey’s

Pancreas

235

Surgical Procedures in Chronic Pancreatitis Ideal procedure: DPPHRQ (Beger’s) In presence of portal vein thrombosis: Frey’sQ Small duct disease: V-shaped excisionQ Disease recurrence in body and tail (after DPPHR, Whipples or Longmire-Traverso procedure): V-shaped drainageQ Disease limited to tail: Spleen-preserving distal pancreatectomyQ

SEROUS CYSTADENOMA OF PANCREAS

Section 2

• • • • •

Serous Cystadenoma of Pancreas • SCNs usually occur in the head of the pancreas; Affect women almost exclusivelyQ • Most are benign and have no malignant potentialQ Q

Pathology • SCNs are largeQ, well-circumscribed masses (microcystic) • Microscopic examination reveals multiloculated, glycogen-rich small cystsQ • Typically have small cystQ filled with clear fluid with spongelike or honeycomb appearanceQ. Clinical Features • Most are asymptomaticQ • Patients commonly present with vague abdominal painQ and less frequently with weight loss & obstructive jaundice. Diagnosis • Central calcification gives rise to a characteristic central sunburstQ, radial, or stellate scar pattern on CT scanQ (10-20%) Treatment • SCN is benign, resection is indicated when diagnosis is in doubt or when they become symptomaticQ, size > 4 cm

MUCINOUS CYSTADENOMA OF PANCREAS Mucinous Cystadenoma of Pancreas • MCN: MC cystic neoplasmQ of the pancreas • Frequently seen in young women, mean age 5th decadeQ; More common in the body & tail of the pancreas Pathology • MCNs contain mucin-producing epitheliumQ, macrocystic • Histology: Presence of mucin-rich cells and ovarian-like stromaQ • Estrogen & progesterone staining are positive in most casesQ Clinical Features • Up to 50% of patients present with vague abdominal pain. • A history of pancreatitis may be found in up to 20% of patients, which explains the common misdiagnosis of pseudocyst. Diagnosis • CT scan: Presence of a solitary cyst with fine septations & rim of calcificationQ • Cross-sectional imaging may not be able to distinguish between benign and malignant MCNs • Presence of eggshell calcification, larger tumor size, or a mural nodule on cross-sectional imaging is suggestive of malignancyQ • Cyst fluid analyses: Mucin-rich aspirate, high CEA & low amylase levelsQ Treatment • Surgical excision is indicated for all mucinous cystic neoplamsQ • Pancreatic resection is the standard treatment for MCNsQ.

Hepatobiliary Pancreatic Surgery

• Aspiration from cyst yields non-viscous fluid with low CEA & low amylase levelsQ.

236

Surgery Essence

Hepatobiliary Pancreatic Surgery

INTRADUCTAL PAPILLARY MUCINOUS NEOPLASM (IPMN) Intraductal Papillary Mucinous Neoplasm • IPMN is also known as mucin-secreting carcinomaQ, villous adenoma of the duct of Wirsung • Seen in 6th to 7th decade of life; Equal sex distributionQ • More common in head & uncinate processQ of the pancreas Types of IPMN −− Side branch IPMN: Involves dilation of the pancreatic duct side branches −− Main duct IPMN: Abnormal cystic dilation of the main pancreatic duct −− Mixed-type IPMN: Side branch IPMN that has extended to involve the main pancreatic duct Pathology • Ductal epithelium forms a papillary projection into the duct • Mucin production causes intraluminal cystic dilationQ of the pancreatic ducts • Careful histologic examination of the entire specimen (invasive component in 35-40%)Q Clinical Features • Present with abdominal pain or recurrent pancreatitis, (caused by obstruction of pancreatic duct by thick mucinQ). • Some patients (5-10%) have steatorrhea, diabetes, & weight loss secondary to pancreatic insufficiency. • Predictors of malignancy: Jaundice, elevated serum ALP, mural nodules, diabetes, & main pancreatic duct diameter ≥ 7 mmQ Diagnosis • Endoscopy: Mucus extruding through a large, fish-mouth like papillary orifice is virtually diagnostic of IPMNQ • CT scans: Dilated main pancreatic duct, cysts of varying sizes, and possibly mural nodulesQ • Aspirated fluid: Mucinous content with elevated CEA & amylase levelQ Pre-operative localization is problematic in IPMN • Over production of mucous, dilation can occur proximal & distal to the tumorQ • Propensity of tumor to spread microscopically along the ductQ Treatment • Partial pancreatectomy: For main duct, symptomatic, and large branch-type IPMNs (>3 cm), or IPMNs with an invasive componentQ • Observation: For asymptomatic small (< 3 cm) branch duct IPMNs without associated nodularity.

RISK FACTORS FOR PANCREATIC CARCINOMA Risk Factors for Pancreatic Carcinoma • There is association between risk of pancreatic cancer, H. pylori colonization, and ABO blood groupsQ. • Older age, African American race, low socioeconomic status, Ashkenazic jewish heritage are associated with increased risk of pancreatic cancer. • Host etiologic factors associated with increased risk of pancreatic cancer include history of diabetes mellitus, chronic cirrhosis and pancreatitis, a high fat or cholesterol diet, and prior cholecystectomyQ.

Section 2

Familial Pancreatic Cancer • Predisposing Conditions: Hereditary pancreatitis, HNPCC, Hereditary Breast Cancer associated with the BRCA2 mutation, Ataxia Telangiectasia, FAMMM, and Peutz-Jegher syndrome. (H3-AFP)Q • BRCA-2 mutation is the MC germline mutation in patients with hereditary pancreatic cancerQ • K-ras is the MC somatic mutation in patients with carcinoma pancreasQ • Peutz-Jegher syndrome carries the highest relative risk of pancreatic cancerQ • Patients with pancreatic cancer with DNA mismatch repair mutations have a better prognosis • K-ras mutations & HER2/neu over expression are the earliest changes to occur

• Established • Associated • Possible

Risk Factors for Pancreatic Carcinoma • TobaccoQ •  Inherited susceptibilityQ Q   • Chronic pancreatitis • Diabetes mellitus type 2Q • Physical inactivityQ •  Certain pesticides

•  ObesityQ •  High carbohydrate/sugar intake

Pancreas

• DPC4 • BRCA2

• 55 • 7

Predisposing Conditions for Familial Pancreatic Cancer (H3-AFP) • Hereditary pancreatitisQ • HNPCCQ • Hereditary Breast Cancer associated with the BRCA2 mutationQ

Section 2

Genetic Mutation in Pancreatic Cancer (KRAP-16: K-ras >p16)) Gene Pancreatic Cancer % • p16Q • 82 • K-rasQ • 95-100 (MC) • p53Q • 75

237

• Ataxia TelangiectasiaQ • FAMMM (Familial atypical multiple mole melanoma) syndromeQ • Peutz-Jegher syndromeQ (Highest risk)

CARCINOMA PANCREAS Carcinoma Pancreas • MC type is pancreatic ductal adenocarcinoma (PDAC)Q; MC site: Head (75%) > Body (15%) >Tail (10%)Q • More common in Men, African Americans, mean age at diagnosis is 72 yearsQ • Association between risk of pancreatic cancer, H. pylori colonization, and ABO blood groups • Established risk factors: Smoking (Tobacco) and Inherited susceptibilityQ

• K-ras2 oncogene is activated (by point mutation) in >95% of pancreatic cancers (MC gene mutation)Q Pathology • Macroscopically, ductal adenocarcinoma is a scirrhous (scar forming)Q type of carcinoma • It is associated with abundant desmoplastic stromaQ, in which the neoplastic glands are widely scatteredQ Clinical Features MC symptom for patients with PDACs in the periampullary region is jaundiceQ. Pain typically arising in epigastrium & radiating to the back. Weight loss affecting more than 50% of individuals. For tumors of body & tail of pancreas, pain & weight loss become more common at presentation. A palpable distended gallbladder in 1/3rd of patients with periampullary PDAC (Courvoisier Law)Q With widespread disease, a left supraclavicular node (Virchow’s node)Q may be palpable. Periumbilical lymphadenopathy may be palpable (Sister Mary Joseph’s node)Q. • In cases of peritoneal dissemination, perirectal tumor involvement may be palpable via DRE, referred to as Blumer’s shelfQ. • • • • • •

Presenting Symptoms of Periampullary Tumors • Jaundice (75%)Q

• Pruritus (11%)

• Weight loss (51%)Q

• Fever (3%)

• Abdominal pain (39%)

• Gastrointestinal bleeding (1%)

• Nausea/vomiting (13%) Diagnosis • Tumor markers: CA19-9 (most sensitive)Q & CEA. • Individuals with blood Lewis antigen–negative status (10-15%) do not develop elevation of the CA19-9 • MDCT is investigation of choice for the evaluation of lesions arising in the pancreasQ. • ERCP: Reserved for cases requiring therapeutic or palliative interventionQ • Double duct sign on ERCP is highly suggestive of pancreatic head cancerQ • EUS: For identifying lesions 3 cm in diameterQ 2. Body or tail tumorsQ

3. Equivocal findings of metastasis on CTQ 4. CA19-9 levels >100 U/mLQ

TNM CLASSIFICATION OF PANCREATIC CANCER Tis T1

8th AJCC (2017) TNM Classification of Pancreatic Cancer Carcinoma in situ Tumor limited to pancreas upto 2 cm in greatest dimension T1a: Tumor ≤0.5 cm in greatest dimension

Section 2

T1b: Tumor >0.5 cm but ≤1 cm in greatest dimension T1c: Tumor >1 cm but ≤2 cm in greatest dimension T2

Tumor limited to pancreas >2-4 cm in greatest dimension

T3 T4

Tumor >4 cm in greatest dimension Tumor involves celiac axis, superior mesenteric artery and/or common hepatic artery

N1

Metastasis in 1-3 regional LN

N2 M1

Metastasis in 4 or more regional LN Distant metastasis

Stage 0 Tis N0 M0

Stage IA T1 N0 M0

Stage IB T2 N0 M0

Stage IIA T3 N0 M0

Stage IIB

Stage III

T1-T3 N1 M0

T1-T3 N2 M0 T4 AnyN M0

Stage IV Any T AnyN M1

Pancreas

239

CARCINOMA PANCREAS: TREATMENT

Section 2



A Whipple’s procedure

B Longmire-Traverso procedure

Whipple’s Procedure (Pancreaticoduodenectomy) • It consists of complete removal of the pancreatic and hepatoduodenal ligament lymph nodes, the duodenum with a short segment of the proximal jejunum and the distal half to two-thirds of the stomach with the right half of the greater omentum. Whipple’s Procedure Involves Resection of Distal stomachQ • DuodenumQ GallbladderQ • Proximal jejunumQ CBDQ • Regional lymphatics Head of pancreasQ

Restoration of GI Continuity Requires • PancreaticojejunostomyQ •  HepaticojejunostomyQ •  GastrojejunostomyQ • Pylorus Preserving Pancreaticoduodenectomy (PPPD) or Longmire-Traverso procedure is the preferred surgery for carcinoma head of pancreasQ. • The Whipple procedure is now reserved for situations in which the entire duodenum has to be removed (e.g. in FAP) or where the tumour encroaches on the 1st part of the duodenum or the distal stomach and a PPPD would not achieve a clear resection marginQ. Pylorus Preserving Pancreaticoduodenectomy or Longmire-Traverso Procedure • To retain a functioning pylorus, the entire stomach and 2cm of the first part of duodenum and their neurovascular supply are preservedQ. • Pylorus Preserving Pancreaticoduodenectomy (PPPD) or Longmire-Traverso procedure is the preferred surgery for carcinoma head of pancreasQ.

SOLID PSEUDOPAPILLARY TUMOR (PAPILLARY CYSTIC CANCER OR FRANTZ TUMOR) Solid Pseudopapillary Tumor (Papillary Cystic Cancer or Frantz Tumor) • Approximately 90% of solid pseudopapillary tumor harbor APC/beta-catenin mutationQ. • Regarded as low grade malignant potentialQ tumor. Pathology • Lesions may be large, encapsulated, evenly distributedQ throughout the pancreas. • Strongly positive for beta-catenin, progesterone receptors, vimentin, CD-10 and CD-56Q. • It can mimic the histologic appearance of NET but lacks the nuclear features of NET and lacks the neuroendocerine markers such as chromagranin and synaptophysin. Clinical Features • Non-specific signs and symptoms related to size of tumor at the time of presentation • Typically occur in women in the second or third decadeQ of life Diagnosis • CT scan: Hypodense areas representing hemorrhage or necrosis • Diagnosis is based on presence of typical histological characteristics such as foamy macrophages, cholesterol clefts, nuclear grooves, and aggregate of hyaline globulesQ. Treatment • Complete resection is associated with long-term survivalQ even in presence of metastatic disease. • Incomplete removal result in local recurrence.

Hepatobiliary Pancreatic Surgery

• • • •

240

Surgery Essence

Hepatobiliary Pancreatic Surgery

NEURO ENDOCRINE TUMORS (NET) OF PANCREAS NET of Pancreas • MC NET of Pancreas: Non-functional (Mostly malignant) >InsulinomaQ • MC benign NET of Pancreas: InsulinomaQ • MC malignant functional NET of Pancreas: GastrinomaQ

Localization of NET of Pancreas • Somatostatin receptors are present in >90% of gastrinomas; in contrast, pancreatic adenocarcinomas do not possess somatostatin receptors. They are also present in a significant portion of glucagonomas and nonfunctioning endocrine tumorsQ. • The sensitivity for SRS is over 80% for all pancreatic NET excluding insulinomasQ • SRS has an overall sensitivity of 80% to 100% and specificity >90% for gastrinomasQ. • SRS is also useful for detecting hepatic metastases from noninsulinoma endocrine tumors

INSULINOMA Insulinoma • Insulinoma is MC functioning tumor of the endocrine pancreasQ • The average age at diagnosis is 45 years. Location of Insulinoma • 97% in pancreas (equal distribution in the head, body, and tail)Q • 3% in duodenum, splenic hilum, or gastrocolic ligamentQ • Typically small, with an average size of 1.0 to 1.5 cm. • Diagnostic hallmark is Whipple’s triad: Symptoms of hypoglycemia + Low blood glucose levels (40-50 mg/dL) + Relief of symptoms after the administration of glucoseQ. Clinical Features • Diagnostic hallmark is Whipple’s triadQ: Fasting-induced neuroglyopenic symptoms of hypoglycemia (diaphoresis, shaking, mental confusion, obtundation, and seizures), low blood glucose levels (40 to 50 mg/dL), and relief of symptoms after the administration of glucose. • Sympathetic overactivityQ in response to hypoglycemia: Fatigue, weakness, fearfulness, hunger, tremor, diaphoresis, and tachycardia. • CNS disturbance: Apathy, irritability, anxiety, confusion, excitement, loss of orientation, blurred vision, delirium, stupor, coma, and/or seizures. • Significant weight gain: Patients eat frequently to prevent hypoglycemia. • It is a painless condition. Diagnosis • Gold standard test for the diagnosis of insulinoma is the 72-hour fastingQ test • An insulin-to-glucose ratio > 0.4 is consistent with insulinomaQ • Provocative testing with tolbutamide, glucagon; or intravenous calcium is rarely required. • CECT or MRI: Hyperattenuating as compared with surrounding pancreatic tissue because of rich vascular supplyQ Localization

Section 2

• Angiography will detect approximately 70% of insulinomas >5 mm, showing a characteristic vascular blushQ • Portal venous sampling for insulin with or without arterial stimulation with calcium is the best pre-operative method of localizationQ • EUS with intra-operative palpation is best localization technique for InsulinomaQ Treatment • Diazoxide decreases beta cell release of insulin, used to prevent or attenuate symptoms of hypoglycemia prior to surgical intervention once the diagnosis is madeQ. • Insulinomas are well suited for laparoscopic resection or enucleationsQ. • Insulinoma of head of pancreas

• Enucleation is TOCQ

• Insulinoma of body or tail of pancreas

• Distal pancreatectomy is TOCQ

• Streptozotocin, with or without 5-fluorouracil, is associated with improved survival in metastatic pancreatic endocrine tumorsQ.

241

Pancreas GASTRINOMA/ZOLLINGER-ELLISON SYNDROME

• • • •

Section 2

Zollinger-Ellison Syndrome Gastrinoma is MC functioning malignantQ pancreatic endocrine tumor. More common in men, mean age 50 yearsQ ZES occur in two forms: Sporadic (75%) & MEN-1 association (25%)Q Those associated with MEN-1 are almost always multiple, early onset, more common in duodenumQ • MC site of Gastrinoma: Duodenum > PancreasQ • Duodenal primary tends to spread to local lymph nodesQ • Pancreatic primary tends to spread to the liver • All gastrinomas also produce chromagranin AQ • Gastrinoma → increased gastrin secretion → marked gastric acid hypersecretion → Peptic ulcer Location • MC site is duodenum (50-70%) followed by Pancreas (20-40%)Q • In Duodenum, MC in 1st partQ (71%) >2nd part (21%) >3rd part (8%) • About 70-90% of gastrinomas are located within the Passaro’s triangleQ. Boundaries of Passaro’s Triangle • Junction of cystic duct and CBDQ • Junction of 2nd & 3rd part of duodenumQ • Junction of neck & body of pancreas

Hepatobiliary Pancreatic Surgery

Clinical Features • Gastric acid hypersecretion causes peptic ulcer disease often refractory severe diarrhea • MC presenting symptoms are abdominal painQ (70-100%), diarrhea (50-70%) & GERD (30-35%). • Causes of Diarrhea: Inability of the intestinal tract to absorb the large volumes of fluid secreted by the stomach, acid-induced injury to the mucosa of the upper small intestine, precipitation of bile salts, inactivation of pancreatic lipase by low pH • Unique characteristic of acid-induced diarrhea: Halted by nasogastric aspiration of gastric secretionsQ • Most patients have peptic ulcers (MC are duodenal ulcers)Q • ZES must be excluded in all patients with intractable peptic ulcers, severe esophagitis, or persistent secretory diarrhea. • MC cause of death in ZES: Liver metastasisQ Diagnosis • Hypergastrinemia with increased secretion of gastric acid confirms the diagnosis • Elevated gastrin alone is not sufficient to diagnose ZES (Basal gastric acid hypersecretion must be demonstrated). Zollinger-Ellison Syndrome • 100% patients will have a fasting serum gastrin level >100 pg/mL • BAO >15 mEq/hr in most patients and >5 mEq/hr in patients with prior surgery to decrease gastric acid secretion • Levels >1000 pg/mL are diagnosticQ • Elevated serum gastrin level with a pH 200 pg/mL in the gastrin value after administration of secretin is diagnosticQ



242

Surgery Essence

Hepatobiliary Pancreatic Surgery

Localization • Somatostatin receptor scintigraphy (SRS) is imaging test of choice for localizing both primary and metastatic gastrinomasQ. Treatment • Acid secretion is controlled by PPIsQ • Pharmacologic control of acid secretion with PPIs has rendered total gastrectomy and other surgical acid-reducing procedures unnecessaryQ. • ZES in MEN-1: Hyperparathyroidism should be treated first because it can complicate the management of their gastrinoma, neck exploration should be performed before resection of gastrinomaQ. • Distal pancreatectomy: Gastrinoma involving body or tailQ of pancreas • Pancreaticoduodenectomy: Gastrinoma involving headQ of pancreas Prognosis • Best predictor of survival for patients with gastrinoma is the presence of liver metastasesQ • LN metastases are not predictiveQ • Resection of all gross disease and metastases may provide palliation of symptoms and may prolong survivalQ.

GLUCAGONOMA Glucagonoma (Hyperglycemic Cutaneous Syndrome) • Compared with other pancreatic NET, they tend to be largerQ, averaging 5-10 cmQ in size at the time of diagnosis. • More common in females; 70% are malignantQ. • MC site: Body and tailQ of pancreas. • Tumors are usually >3 cm in diameter and are highly vascular (tumor blush)Q. • The diabetes is mild and not associated with diabetic ketoacidosis because insulin secretion by beta cell is preservedQ. Clinical Features • Classic presentation of the 4Ds: Diabetes, dermatitis, DVT, and depressionQ. • Necrolytic erythema migrans are MC manifestations of the disease, seen in 2/3rd of patientsQ. • Necrolytic erythema migrans: The characteristic rash occur in areas of friction; rash is migratory, red, and scaling, associated with intense pruritusQ • Parenteral administration of amino acids was found to result in the disappearance of the skin lesionsQ • Deep vein thrombosis occurs in 30%Q of patients. • Neurologic manifestations include ataxia, dementia, scotomata, and proximal muscle weakness. Diagnosis • Fasting glucagon level >50 pmol/L is considered diagnosticQ. • VIPomas, glucagonomas, and somatostatinomas are usually larger and easily localized by CTQ. • SRS can be performed if CT is not informative. Treatment • Resection is the treatment of choice for VIPomas, glucagonomas, somatostatinomas, and nonfunctional pancreatic NET and remains the only curative optionQ. • Dacarbazine is uniquely effective against glucagonomaQ as compared with other pancreatic NET and complete remission has been reported in several cases.

PPOMAS/NON-FUNCTIONING PANCREATIC ENDOCRINE TUMOR

Section 2

PPomas/Non-functioning Pancreatic Endocrine Tumor • Approximately 10-25% of all neuroendocrine pancreatic tumors are nonfunctionalQ. • Tumors are large (>5 cm) and almost all (80%) are malignant and metastatic; • Usually solitary, mostly in the headQ Clinical Feature • Plasma elevation of PP is not associated with specific symptom. Diagnosis • Elevated plasma levels of chromagranin A and B are found in 69-100%Q of patients; neuron specific enolase in 31%; PP in 50-75%; alphaHCG in 40%; beta-HCG in 20%. • Atropine Suppression Test: Atropine (1 mg intramuscularly) does not suppress the levels of plasma PP with PETs but did suppress the level by 50% or more in all patients without tumors. • Adenocarcinoma can be distinguished from NETs by immunohistochemical staining with chromagranin AQ.

Pancreas

243

Treatment • Dopamine agonists decrease circulating levels of PP and chromogranin A in patients with large, unresectable islet cell tumorsQ.

VIPOMAS (VERNER-MORRISON SYNDROME) VIPomas (Verner-Morrison Syndrome) • Also known as WDHA syndrome (watery diarrhea, hypokalemia, achlorhydria) or pancreatic choleraQ

Section 2

• Mostly located in the head of the pancreas and require a pancreaticoduodenectomy.

• Usually solitary; MC site is tail of pancreasQ • Two-thirds are malignantQ. Clinical Features • Diagnostic triad: Secretary diarrhea + High levels of circulating VIP + Pancreatic tumorQ. • Profuse, watery, iso-osmotic secretary diarrhea is MC presenting symptom and may exceed a volume of 3 to 5 liters/day. • The diarrhea persists despite fasting, which qualifies it as a secretory diarrhea and, despite nasogastric aspiration, which differentiates it from the diarrhea of ZES. • Characterized by: Hypokalemia, hypercalcemia, hypochlorhydria & hyperglycemiaQ. • The tetany has been attributed to hypomagnesemia from chronic diarrhea. Diagnosis • Constant features are diarrhea, hypovolemia, hypokalemia and acidosis, variable features are achlorhydria or hypochlorhydria, hyperglycemia and flushing with rashQ. Treatment • Aggressive preoperative hydration and correction of electrolyte abnormalities & acid-base disturbancesQ. • Octreotide is commonly used preperatively to reduce diarrhea volume and facilitate fluid & electrolyte replacement. • Resection is the treatment of choice for VIPomas, glucagonomas, somatostatinomas, and nonfunctional pancreatic NET and remains the only curative optionQ.

SOMATOSTATINOMA Somatostatinoma • MC site is head of the pancreasQ. • Most somatostatinomas are solitary and located within the pancreatic head (MC)Q or duodenum • Equally common in males and females, of about 50 years. Clinical Features • Somatostatin excess causes steatorrhea (30-68%), mild diabetes (60%), cholelithiasis (70%), and hypochlorhydria (86%)Q. Diagnosis • VIPomas, glucagonomas, and somatostatinomas are usually larger and easily localized by CTQ. Treatment • Resection is the treatment of choice for VIPomas, glucagonomas, somatostatinomas, and nonfunctional pancreatic NET and remains the only curative optionQ. • In 75% cases, somatostatinomas are metastatic and >5 cm at the time of diagnosis. • Whipple’s procedure in localized tumor is curative. • In unresectable disease, octreotide and interferon alfa may improve symptoms.

Neurotensinoma • Neurotensinoma cause hypokalemia, weight loss, hypotension, cyanosis, flushing and diabetesQ. • They are usually malignantQ.

Hepatobiliary Pancreatic Surgery

• VIPomas, glucagonomas, and somatostatinomas are usually larger & easily localized by CTQ.

Hepatobiliary Pancreatic Surgery

244

Surgery Essence

• • • • •

GRFoma It occurs most often in the lung (bronchus)Q, then pancreas, jejunum, adrenal glands and retroperitoneum. Patient presents with acromegaly and a pancreatic massQ. Pancreatic GRFomas are large (>6 cm). One third will have metastasized at the time of diagnosis. Approximately 50% of patients with GRFomas also have ZES and 33% have MEN-1.

PANCREATIC TRAUMA Pancreatic Trauma • • • • •

Pancreatic injuries are uncommon. Penetrating injuries into the abdomen are the MC injuries seen in adultsQ. Isolated pancreatic injuries are not common. Up to 90% of patients present with associated hepatic, gastric, splenic, renal, colonic, or vascular lesions. MC associated injury is to a hollow viscus (38%)Q; followed by the liver (19%); and spleen (11%). Pancreatic Trauma in Children • MC mechanism in children is abdominal blunt traumaQ. • Direct compression of the epigastrium against the vertebral column and a blunt object (handlebar) is typically seen after bicycle injuriesQ. • MC segment of the pancreas affected is the bodyQ.

I II III IV V

Grade Hematoma Laceration Hematoma Laceration Laceration Laceration Laceration

Pancreatic Organ Injury Scale Type of Injury Minor contusion without duct injury Superficial laceration without duct injury Major Contusion without duct injury or tissue loss Major Laceration without duct injury or tissue loss Distal transaction or parenchymal injury with duct injury Proximal transaction or parenchymal injury involving ampulla Massive disruption of pancreatic head

Diagnosis • CT scan: Investigation of choice to evaluate patients with abdominal traumaQ. • CT findings: Peripancreatic hematomas, free fluid in the lesser sack, or abnormal thickening of Gerota’s fascia suggest pancreatic injury. • ERCP: Most reliable test to demonstrate pancreatic duct integrityQ • Isolated pancreatic amylase level measurement is not recommended because up to 40% of patients with pancreatic duct transected have normal serum amylase levels. Serial quantification levelsQ increase the sensitivity of the assay. Treatment • Definitive treatment is based on surgical findings. • Major pancreatic resections in stable patients with isolated pancreatic injuryQ. • Damage control surgery is indicated for complex injuries or unstable patientsQ. • Most (up to 75%) of deaths occur within the 48 to 72 hours after trauma, and most are related to hypovolemic shockQ. Complication • A persistent drain output or pancreatic fistula is the MC complication after pancreatic traumaQ.

Section 2

Grading

Treatment of Pancreatic Injuries

• Grade I

• Observation aloneQ

• Grade II

• Debridement, drainage, possible repairQ

• Grade III

• Distal resection, possible Roux-en-Y drainageQ

• Grades IV and V

• • • • • •

Damage controlQ, hemostasis/drainage Resection and possible Roux-en-Y drainageQ Triple-tube decompressionQ Pyloric exclusion technique Duodenal diverticularizationQ Pancreaticoduodenectomy

Multiple Choice Questions ACUTE PANCREATITIS: ETIOLOGY AND RISK FACTORS

1. Which of the following is the most common non-alcoholic cause of acute pancreatitis? (COMEDK 2008, 2007) a. Thiazides b. Hypercalcemia c. Hyperlipidemia d. Gallstones

2. The commonest cause of acute pancreatitis is: (COMEDK 2008) a. Biliary calculi b. Alcohol abuse c. Infective d. Idiopathic

3. Pancreatitis may be produced by following drug: (PGI Dec 96) a. Colchicine b. L-Asperaginase c. Ciprofloxacin d. Nalidixic acid



4. Most common complication after ERCP is: (AIIMS May 2007) a. Acute pancreatitis b. Acute cholangitis c. Acute cholecystitis d. Duodenal perforation



5. Post-operative pancreatitis is seen in which type of surgery? (MHSSMCET 2005) a. Billroth type I b. Splenectomy c. Nephrectomy d. Cardiopulmonary bypass



6. Which of the following is not an etiological factor for pancreatitis? (AIIMS May 2014) a. Abdominal trauma b. Hyperlipidemia c. Islet cell hyperplasia d. Germline mutations in the cationic trypsinogen gene



7. Which of the following drug causes acute pancreatitis?  (Recent Question 2017) a. L-Asparaginase b. Metronidazole c. Ciprofloxacin d. Penicillin

ACUTE PANCREATITIS: CLINICAL FEATURES, DIAGNOSIS AND TREATMENT

8. Which of the following does not cause an increase in serum amylase? (COMEDK 2008) a. Pancreatitis b. Carcinoma lung c. Renal failure d. Cardiac failure



9. Poor prognostic factor in a patient with acute pancreatitis: a. Leucocytosis >20,000/µL  (NEET Pattern, JIPMER 2011) b. ↓ serum amylase c. ↓ serum lipase d. Diastolic BP >90 mm Hg



10. A lady presents with three day history of epigastric pain radiating to back serum amylase levels were observed to be normal while USG abdomen reveals gallbladder stones and an enlarged pancreas. CT scan was done which clinched the diagnosis. Which of the following is the most likely diagnosis? (All India 2011) a. Acute cholecystitis b. Acute pancreatitis c. Acute appendicitis d. Acute peritonitis



11. Medical treatment of acute pancreatitis includes: a. Calcium b. Glucagon (PGI Nov 2011) c. Aprotinin d. Cholestyramine e. Antibiotics



12. Which of the following criteria is/are not included in Ranson’s scoring? (PGI Nov 2011) a. WBC >16,000/µl b. Serum amylase >350 IU c. Age > 55 years d. Serum LDH >700 IU e. Serum AST >250 U/dL



13. Hyperamylasemia is seen in all except: a. Peritonitis b. Acute pancreatitis c. Carcinoma esophagus d. Ruptured ectopic pregnancy e. Perforated peptic ulcer



14. Ranson’s scoring for acute pancreatitis includes: (PGI May 2011) a. Age >55 years b. WBC >16,000/µL c. Sequestration of fluid >6L d. BUN >10 mg/dl e. LDH >700 IU



15. Which of the following is most diagnostic investigation for acute pancreatitis? (MHPGMCET 2003) a. Serum amylase b. Serum lipase c. Serum P-isoamylase d. Serum LDH



16. Which of the following is not a feature of acute pancreatitis? (DNB 2011, Orissa 2011) a. Hyperbilirubinemia b. Hypercalcemia c. Hyperglycemia d. Increased serum LDH level

(PGI May 2011)

17. Which of the following types of pancreatitis has the best prognosis? (APPG 2005, All India 2004) a. Alcoholic pancreatitis b. Gallstone induced pancreatitis c. Post operative pancreatitis d. Idiopathic pancreatitis

18. Which one is not the bad prognostic sign for pancreatitis? a. TLC >16,000/µL (AIIMS June 2000) b. Calcium 200 mg% d. Prothrombin >2 times the control



19. Which one is not poor prognostic factor for acute pancreatitis?  (AIIMS Nov 99) a. Hyperglycemia b. Hypocalcemia c. Raised LDH level in blood d. Hyperamylasemia 20. Poor prognostic factor in acute pancreatitis is: (PGI Dec 96) a. Increased serum amylase b. Decreased calcium c. Decreased blood sugar d. Decreased PaO2





21. Destruction of fat in acute pancreatitis is due to: a. Lipase and trypsin (MHCET 2016) b. Secretin c. Lipase and elastase d. Cholecystokinin and trypsin

Hepatobiliary Pancreatic Surgery

246

Surgery Essence 22. All the following can be used to predict severe acute pancreatitis except: a. Glasgow score > 3 b. APACHE II score > 8 c. CT severity score > 6 d. C-reactive protein 350 IU, d. Serum LDH >700 IU

13. Ans. None



14. Ans. a. Age >55 years, b. WBC >16000/µL, c. Sequestration of fluid >6L

15. Ans. b. Serum lipase



16. Ans. b. Hypercalcemia

Pancreas

259

17. Ans. b. Gallstone induced pancreatitis (Ref: Oxford textbook of surgery 2/e p1766)



18. Ans. d. Prothrombin >2 times the control

19. Ans. d. Hyperamylasemia



20. Ans. b. Decreased calcium, d. Decreased PaO2

21. Ans. a. Lipase and trypsin



22. Ans. d. C-reactive protein 60 mmol/L

At 48 Hours

• Polymorphonuclear elastase • Urinary trypsinogen 2 • Urinary trypsinogen activation peptide



23. Ans. a. Low serum lipase



25. Ans. a. Acute fluid collection (Ref: Sabiston 20/e p1528-1529)

Section 2

• The prognosis is best in patients where pancreatitis is caused by a remediable cause such as cholelithiasisQ. • Eradication of gallstones prevents further attacks of pancreatitisQ.

• Ranson/Glasgow score ≥ 3Q • CRP ≥130Q mg/mL

24. Ans. d. Acute pancreatitis

Indications for Operative Treatment in Acute Pancreatitis • • • • • •

26. Ans. b. Hemorrhagic pancreatitis (Ref: Bailey 25/e p1144; Sabiston 20/e p1528; Maingot’s 11/e p786,977) • All of the following patients presenting with abdominal pain and shock need immediate laparotomy except hemorrhagic pancreatitis. Bailey says “In peritonitis due to pancreatitis or salpingitis, or in cases of primary peritonitis of streptococcal or pneumococcal origin, non-operative treatment is preferredQ (if diagnosis can be made with certainity).” Sabiston says “Ruptured ectopic is a surgical emergencyQ.” Definitive treatment of ruptured aortic aneurysm is operation, not monitoring and rescuscitationQ Maingot’s says regarding liver hemangioma “Intra-abdominal hemorrhage is extremely uncommon, but when does it occur, it should be considered as a life threatening emergency and treated with combination of angiography with embolization and surgeryQ.”



27. Ans. a. Steatorrhoea



28. Ans. c. Acute pancreatitis



30. Ans. a. Acute pancreatitis (Ref: Sabiston 20/e p1527; Schwartz 10/e p1351-1360; Bailey 26/e p1128, 1135, 1138)

• Steatorrhoea is seen in chronic pancreatitis, not in acute pancreatitis. 29. Ans. b. Acute pancreatitis

Radiological Appearance Acute Pancreatitis • Renal halo signQ • Gasless abdomenQ • Ground glass appearanceQ • Colon cut off signQ • Sentinel loopQ

Chronic Pancreatitis • • • •

Chain of lakes appearanceQ String of pearl appearanceQ Beaded appearanceQ Numerous irregular calcificationsQ are pathognomonic (on X-ray)

CA Pancreas Double contour of medial border of duodenal C loop Double duct signQ Dilated / widening of duodenal C loopQ Mucosal irregularityQ Scrambled egg appearance Inverted / reverse 3 sign of FrostbergQ: Seening CA head of pancreas • Rose thorning of medial wall of 2nd part of duodenumQ • • • • • •



31. Ans. c. Serum calcium

32. Ans. c. Gallbladder stone



33. Ans. b. Infection

34. Ans. b. Serum amylase



35. Ans. d. Severe acute pancreatitis (Ref: Sabiston 19/1522-1523; Bailey 27/e p1223)

The CT image showing extensive pancreatic necrosis in combination with the acute clinical presentation is typical of severe acute pancreatitis.

Hepatobiliary Pancreatic Surgery



Failure of non-operative management with at least 48 hours of maximal ICU supportQ Infected necrosisQ Extravisceral airQ Hemorrhage uncontrolled by interventional techniqueQ Colonic complicationsQ Operation is indicated for documented infection or for sterile pancreatic necrosisQ with persistent systemic illness.

Hepatobiliary Pancreatic Surgery

260

Surgery Essence

36. Ans. d. Hypoglycemia



39. Ans. a. Isotonic crystalloid by IV line (Ref: Sabiston 20/e p1528; Schwartz 10/e p1358; Bailey 27/e p1225)

37. Ans. b. Pancreatitis

38. Ans. a. Acute pancreatitis

“While there are proponents for aggressive fluid therapy and for specific resuscitation goals, it is probably best to resuscitate with a balanced crystalloid and to restore normal blood volume, blood pressure, and urine output. On the basis of recent data it appears that lactated Ringer’s solution may be superior to normal saline in reducing the systemic inflammatory response.” —Schwartz 10/e p1358

40. Ans. b. Colon cut off sign

ACUTE PANCREATITIS: COMPLICATIONS

41. Ans. c. Acute pancreatitis



42. Ans. b. Severe acute pancreatitis (Ref: Sabiston 20/e p1526; Schwartz 10/e p1356; Bailey 27/e p1019) Signs seen in Retroperitoneal bleeding associated with Severe Pancreatitis Grey Turner Sign Ecchymosis in flank



Cullen’s sign Periumbilical ecchymosis

Fox sign Ecchymosis in inguinal region

43. Ans. a. Needle aspiration, c. External drainage (Ref: Sabiston 20/e p1529; Schwartz 10/e p150-151; Bailey 27/e p1227; Blumgart 6/e p902 5/e p858; Shackelford 8/e p1079 7/e p1127-1128)

Sterile and Infected Peripancreatic Fluid Collections (Pancreatic Abscess) • The presence of acute abdominal fluid during an episode of AP has been described in 30-57% of patientsQ. • In contrast to pseudocysts and cystic neoplasias of the pancreas, fluid collections are not surrounded or encased by epithelium or fibrotic capsuleQ. • Treatment is supportive because most fluid collections will be spontaneously reabsorbedQ by the peritoneum. • Pancreatic abscess: The presence of fever, elevated WBC count, and abdominal pain suggest infection of this fluid and percutaneous aspiration is confirmatoryQ • Percutaneous drainage and IV administration of antibiotics should be instituted if infection (pancreatic abscess) Q is present

44. Ans. a. Low protein (Ref: Schwartz 9/e p1204; Bailey 27/e p1228; Shackelford 7/e p1128)

Section 2

Pancreatic Ascites: Management • Pancreatic ascites occurs from a pancreatic duct disruption or from a leaking pseudocyst. • High amylase levels are found in the ascitic fluidQ. • Initial treatment: Non-operative (elimination of enteral feeding, institution of nasogastric drainage, and administration of somatostatin) Q • Repeat paracentesis may also be helpful. • Roughly 50% to 60% of patients can be expected to respond to this treatment with resolution of pancreatic ascites within 2 to 3 weeksQ. Persistent or recurrent ascites: endoscopic or surgical treatment • Endoscopic pancreatic sphincterotomy with or without placement of a transpapillary pancreatic duct stentQ. • Resection (for leaks in the pancreatic tail) or internal Roux-en-Y drainage (for leaks in the head and neck region)Q

Pancreas 45. Ans. d. Leak from the stented duct (Ref: Schwartz 10/e p1378; Sabiston 20/e p1531; Bailey 27/e p1228)



46. Ans. b. Bluish discoloration around umbilicus



47. Ans. d. Middle colic artery thrombosis (Ref: Sabiston 20/e p1531; Schwartz 10/e p1351-1360; Bailey 27/e p1228)



48. Ans. c. Acute hemorrhagic pancreatitis



50. Ans. b. Hypocalcemia



51. Ans. a. Grey Turner sign

49. Ans. c. Commonly associated with hemosuccus pancreaticus

• Most common metabolic complication of acute pancreatitis is hypocalcemiaQ. 52. Ans. b. Laparotomy and surgical debridement

CHRONIC PANCREATITIS: ETIOLOGY, CLINICAL FEATURES AND DIAGNOSIS

53. Ans. a. Cationic trypsinogen or PRSS1 (Ref: Sabiston 20/e p1532; Schwartz 10/e p1362; Bailey 27/e p1230; Blumgart 6/e p915 5/e p861; Shackelford 8/e p1087, 7/e p1134)



54. Ans. d. Serum amylase is always raised (Ref: Sabiston 20/e p1531-1536; Schwartz 10/e p1360-1390; Bailey 27/e p1231)



• Amylase and lipase levels are not always elevated in chronic pancreatitis, making the diagnosis often clinical. 55. Ans. a. Can present with steatorrhea, b. Present with mid epigastric pain radiating to back, d. Predisposes to carinoma, e. Complete pancreatectomy relieves pain in majority of patients (Ref: Schwartz 10/e p1371-1382; Sabiston 20/e p1531-1535; Bailey 27/e p1230-1233)



56. Ans. a. Abdominal pain • Abdominal pain is the primary manifestation and MC symptom of chronic pancreatitisQ.



57. Ans. c. Hypercalcemia (Ref: Sabiston 20/e p1532; Schwartz 10/e p1360, 1366-1367; Bailey 27/e p1230; Harrison 20/e p2445, 19/e p2098)



58. Ans. b. Cassava ingestion



59. Ans. b. Chronic pancreatitis (Ref: Sabiston 20/e p1535, 19/e p1521; Schwartz 10/e p1383; Bailey 27/e p1231) • Chain of lakes appearanceQ or String of pearl appearanceQ or Beaded appearanceQ is characteristic of chronic pancreatitis.



60. Ans. b. Chronic pancreatitis



62. Ans. None (Ref: Harrison 20/e p2445, 19/e p2098)



• Chronic pancreatitis is seen in all the given conditions 63. Ans. a. Chronic pancreatitis (Ref: Sabiston 20/e p1533; Schwartz 10/e p1369; Bailey 27/e p1231)

61. Ans. a. Hypercalcemia

64. Ans. b. Calcification, c. Duct penetrating sign, e. Dilation of bile and pancreatic duct (Ref: Bailey 25/e p1157; Sabiston 20/e p1533) • Irregularity of the pancreatic duct, intraductal or parenchymal calcifications, diffuse pancreatic involvement, and normal or smoothly stenotic pancreatic duct penetrating through the mass (duct penetrating sign) Q favor the diagnosis of chronic pancreatitis over cancer. 66. Ans. b. Alcohol (Ref: Sabiston 20/e p1531; Schwartz 10/e p1362; Bailey 27/e p1230)



65. Ans. b. Chronic pancreatitis



67. Ans. d. Treatment is mainly surgical (Ref: Sabiston 20/e p1532; Schwartz 10/e p1366; Bailey 27/e p1230)



68. Ans. b. ERCP



69. Ans. c. Autosomal recessive inheritance (Ref: Schwartz 10/e p1352; Sabiston 20/e p1882; Bailey 27/e p1230)

CHRONIC PANCREATITIS: TREATMENT AND COMPLICATIONS

70. Ans. b. Somatostatin is effective in fistula closure (Ref: Blumgart 6/e p1010, 5/e p965; Shackelford 8/e p894, 7/e p1278-1280) • Trials have demonstrated the benefit of prophylactic octreotide in the prevention of pancreatic fistula; however the role of octreotide, once fistula is established remains unclearQ. In such situations, the octreotide administration is continued for upto 2 weeks, while in those who do not seem to respond, it is discontinued.



71. Ans. a. DPPHR (Ref: Sabiston 20/e p1535; Schwartz 10/e p1360-1390; Blumgart 6/e p934, 5/e p875-881; Shackelford 8/e p1198 7/e p1138-1142)

Resection Procedures in Chronic Pancreatitis • It is believed that inflammatory process in the pancreatic head controls both the severity of symptoms and further progression of disease in remainder of the gland • Pancreatic head is pacemaker of chronic pancreatitisQ. • Because of this resection of pancreatic head has been shown to completely relieve the pain of chronic pancreatitis in 70-80%Q patients. • Distal pancreatectomy is the ideal procedure for patients whose chronic pancreatitis is confined to pancreatitis tailQ. • Usually, distal pancreatectomy is combined with splenectomy for technical reasons, but spleen can be preserved if its vascular supply is secure.

Hepatobiliary Pancreatic Surgery

In this X-ray, calcification is seen in epigastric region mainly towards left, along the course of pancreatic duct, seen in chronic pancreatitis.

Section 2



261

Hepatobiliary Pancreatic Surgery

262

Surgery Essence

72. Ans. a. Distal resection of tail of pancreas with end to end pancreaticojejunostomy



73. Ans. d. Longitudinal Pancreaticojejunostomy • Drainage procedure in the form of longitudinal pancreaticojejunostomy is the surgical treatment of choice for chronic pancreatitis with dilated ductsQ.



74. Ans. d. Medical management • The patient has non-disabling pain that has responded to analgesia. Such patients should be best managed by non-operative means. Medical management is the best option. • In the patients having severe pain, not relieved by analgesics with similar situation should be best managed with longitudinal pancreaticojejunostomy.



75. Ans. c. Side to side pancreaticojejunostomy



76. Ans. a. Celiac ganglia (Ref: Sabiston 20/e p1534; Schwartz 10/e p1380; Bailey 27/e p1231)

Chronic Pancreatitis • Pain from the pancreas is carried in sympathetic fibers that traverse the celiac ganglia, reach the sympathetic chain through the splanchnic nerves, and then ascend to the cortex • Celiac plexus nerve blocksQ performed either percutaneously or endoscopically have been employed to abolish this pain with inconsistent results

77. Ans. d. Distal pancreatectomy

78. Ans. d. None



79. Ans. a. Renal artery stenosis

80. Ans. d. Gallbladder stone is the most common cause



81. Ans. c. Pseudocyst (Ref: Sabiston 20/e p1530)

PSEUDOPANCREATIC CYST

82. Ans. b. Endoscopic treatment may be curative (Ref: Sabiston 20/e p1530; Schwartz 10/e p1377; Bailey 27/e p1231; Shackelford 8/e p1100, 7/e p1144-1148)



83. Ans. b. Pseudocyst



85. Ans. d. Hemorrhage (Ref: CSDT 11/e p638)

84. Ans. a. Infection

• Serious post-op hemorrhage from cyst occurs from cystogastrostomyQ.

86. Ans. c. Pancreatitis



87. Ans. d. Splenic artery (Ref: Maingot 11/e p977)

Artery Involved in Pseudoaneurysm associated with Pancreatic Pseudocyst • Splenic artery (30-50%): Most commonQ • Gastroduodenal artery (10-15%) • Inferior and superior pancreaticoduodenal artery (10%)

88. Ans. b. Post inflammatory cyst



89. Ans. d. Traumatic pancreatitis • Trauma is the most common cause of acute pancreatitis in children.

Section 2

Pancreatic Trauma in Children • MC mechanism in children is abdominal blunt traumaQ. • Direct compression of the epigastrium against the vertebral column and a blunt object (handlebar) is typically seen after bicycle injuriesQ. • MC segment of the pancreas affected is the bodyQ.

90. Ans. d. USG and follow up

91. Ans. d. Percutaneous aspiration is the treatment



92. Ans. a. Percutaneous aspiration is treatment of choice

93. Ans. d. Most common site is in head of pancreas

• Most common site of pancreatic pseudocyst is lesser sacQ

94. Ans. b. Conservative

95. Ans. d. Carcinomatous change

Pancreas

96. Ans. d. Treatment of choice is percutaneous aspiration 98. Ans. d. Infection

263

97. Ans. c. Calcification in the cyst wall 99. Ans. c. Pseudopancreatic cyst

100. Ans. a. IPMN (Ref: Sabiston 20/e p1538-1539; Schwartz 9/e p1234; Blumgart 6/e p960, 5/e p903-905; Shackelford 8/e p1162 7/e p1220-1222) 101. Ans. a. Microcystic adenoma (Ref: Sabiston 20/e p1537; Schwartz 10/e p 1410-1413; Blumgart 5/e p902-903; Shackelford 7/e p1218-1220) • Mucinous cystadenoma is macrocystic, not the microcystic adenoma. 102. Ans. c. Intraductal papillary mucinous neoplasm

Section 2

CYSTIC NEOPLASMS OF PANCREAS

103. Ans. a. Treatment is enucleation 104. Ans. b. Enucleation is performed (Ref: Sabiston 20/e p1537; Schwartz 10/e p1410; Bailey 27/e p1234) 105. Ans. a. 30% are associated with malignancy (Ref: Sabiston 20/e p1538; Schwartz 10/e p 1409-1410) 106. Ans. b. Serous cystadenoma (Ref: Sabiston 20/e p1538; Schwartz 10/e p1409) 107. Ans. a. Always involves main pancreatic duct (Ref: Sabiston 20/e p1538) Cystic Neoplasm of Pancreas SCN

Characteristics

MCN

IPMN

F >> MQ (4 : 1)

F >>> MQ (10 : 1)

F = MQ

Age (years)

60-70

50-60

60-70

MC site

Head

Body and tail

HeadQ

Pathology

Multiple small cysts (microcyst) separated by internal septations

Thick-walled, septated macrocystQ with smooth contour; ± solid component

Poorly demarcated, lobulated, polycystic mass with dilation of main or branch ductsQ

Radiology/Investigations

Central sunburst calcificationsQ Egg-shell calcificationsQ on on CECT CECT

Mucin protruding from fish mouth opening on endoscopy

Cytology

Scant glycogen-rich cells, with positive Periodic Acid Schiff stainQ

Sheets and clusters of columnar, mucin-containing cells

Tall, columnar mucin-containing cells

Communication with ducts

No

No

YesQ

Amylase level

Low

Low

HighQ

CEA level

Low

HighQ

Mucin stain

Negative

Positive

Q

Q

HighQ Q

PositiveQ

CARCINOMA PANCREAS: ETIOLOGY AND RISK FACTORS 108. Ans. a. Her-2-neu (Ref: Sabiston 20/e p1541-1542; Schwartz 10/e p1395; Bailey 27/e p1234) • K-ras mutations and HER2/neu over expression are the earliest changes to occur in pancreatic carcinomaQ 109. Ans. a. Acute pancreatitis (Ref: Sabiston 20/e p1541-1542; Schwartz 10/e p1395; Bailey 27/e p1234) 110. Ans. a. K-ras (Ref: Sabiston 20/e p1542; Schwartz 10/e p1395; Bailey 27/e p1234) 111. Ans. d. Cronkhite-Canada syndrome

112. Ans. d. FAP

113. Ans. b. K-ras

114. Ans. d. History of partial gastrectomy

115. Ans. a. Urgent weight reduction, b. Strict vegetarian diet, c. Stop alcohol, e. Stop cigarette smoking 116. Ans. c. BRCA activated

117. Ans. c. Peutz-Jegher’s syndrome (Ref: Sabiston 20/e p1542)

118. Ans. a. K-ras (Ref: Sabiston 20/e p1543; Schwartz 10/e p1395; Bailey 27/e p1234) 119. Ans. a. K-ras

120. Ans. d. Ataxia-Telangiectasia

CA PANCREAS: CLINICAL FEATURES AND DIAGNOSIS 121. Ans. c. Jaundice (Ref: Sabiston 20/e p1544; Schwartz 10/e p 1394; Bailey 27/e p1234; Blumgart 6/e p979, 5/e p919-925; Shackelford 8/e p1138, 7/e p1190-1196)

Hepatobiliary Pancreatic Surgery

Sex

264

Surgery Essence 122. Ans. a. External compression of duodenum (Ref: Schwartz 10/e p1395; Sabiston 20/e p1544; Bailey 27/e p1234)

Hepatobiliary Pancreatic Surgery

123. Ans. a. MDCT • Diagnostic investigation in carcinoma pancreas is MDCTQ • MDCT is investigation of choice for the evaluation of lesions arising in the pancreasQ • IOC for diagnosis, staging and follow-up in CA pancreas: MDCTQ 124. Ans. a. T1

125. Ans. a. Most common site is body and tail

126. Ans. d. EUS guided transgastric biopsy (Ref: Schwartz 10/e p1397-1399; Sabiston 20/e p1544; Bailey 27/e p1235) • “EUS is useful if CT fails to demonstrate a tumour, if tissue diagnosis is required prior to surgery (e.g. a mass has developed on a background of chronic pancreatitis and a distinction needs to be made between inflammation and neoplasia), if vascular invasion needs to be confirmed, or in separating cystic tumours from pseudocysts. Transduodenal or transgastric FNA or Trucut biopsy performed under EUS guidance avoids spillage of tumour cells into the peritoneal cavity. Percutaneous transperitoneal biopsy of potentially resectable pancreatic tumours should be avoided as far as possible. Histological confirmation of malignancy is desirable but not essential, particularly if the imaging clearly demonstrates a resectable tumour. The lack of a tissue diagnosis should not delay appropriate surgical therapy. In patients judged to have unresectable disease, tissue diagnosis should be obtained prior to starting palliative therapy.” —Bailey 27/e p1235 127. Ans. c. Pancreatic carcinoma (Ref: Grainger 4/e p1356-64)

128. Ans. a. Periampullary carcinoma

129. Ans. a. Pancreatic cancer, c. Stomach cancer (Ref: Robbins 9/e p332, 126) Malignancies Associated with Migratory Thrombophlebitis • CA pancreas (MC)Q

• Prostate cancerQ

• CA lung

• Ovarian cancerQ

• GI malignanciesQ

• LymphomaQ

Q

• Trousseau’s syndrome: Migratory thrombophlebitisQ • Trousseau’s sign: Carpopedal spasm in hypocalcemiaQ • Troisier’s sign: Palpable left supraclavicular LN (Virchow’s node)Q 130. Ans. a. Head is the most common site, c. Obstruction of bile and pancreatic secretion is common 131. Ans. a. Hypotonic duodenogram, c. USG, d. Endoscopy, e. CT Scan 132. Ans. d. Periampullary carcinoma

133. Ans. c. CA head of pancreas

134. Ans. d. Carcinoma head of pancreas (Ref: radiopaedia.org/articles/frostburg-inverted-3-sign-1) • Frostburg inverted 3 sign is a sign seen on a barium examination where there is effacement and distortion of the mucosal pattern on the medial wall of the second part of the duodenum due to focal mass and local edema. It is most commonly associated with carcinoma of the head of the pancreas. 135. Ans. d. Ductal adenocarcinoma

136. Ans. a. Ductal adenocarcinoma

137. Ans. d. CT scan (Ref: CSDT 11/e p645; Schwartz 10/e p1398)

Section 2

• Investigation of choice for carcinoma pancreas: MDCTQ • Currently CT is probably the single most versatile and cost effective tool for diagnosis of pancreatic cancer 138. Ans. b. Syndrome of inappropriate secretion of ADH (Ref: Harrison 20/e p2690, 19/e p2280) Causes of Syndrome of Inappropriate Antidiuresis (SIADH) Neoplasms • • • • • • •

LungQ DuodenumQ, PancreasQ OvaryQ, BladderQ, ureterQ ThymomaQ, MesotheliomaQ Bronchial adenomaQ Carcinoid Gangliocytoma, Ewing’s sarcoma

Infections • • • • • •

PneumoniaQ, bacterial or viral AbscessQ, lung or brain Cavitation (aspergillosis) Tuberculosis, lung or brain Meningitis, bacterial or viral EncephalitisQ, AIDSQ

Neurologic • • • • • •

Guillain-Barré syndromeQ Multiple sclerosisQ Delirium tremensQ Amyotrophic lateral sclerosisQ Hydrocephalus, Psychosis Peripheral neuropathyQ

Pancreas Metabolic

Vascular

Pulmonary • Asthma, PneumothoraxQ • Positive-pressure respiration Head trauma

• Cerebrovascular occlusions, hemorrhage • Cavernous sinus thrombosisQ Congenital malformations • Agenesis corpus callosum • Cleft lip/palateQ • Other midline defects

• Closed and penetratingQ

Drugs • • • • • • • • •

VasopressinQ or desmopressin ChlorpropamideQ Oxytocin, high doseQ Vincristine, CarbamazepineQ Nicotine, PhenothiazinesQ CyclophosphamideQ Tricyclic antidepressantsQ Monoamine oxidase inhibitorsQ Serotonin reuptake inhibitorsQ

139. Ans. b. Carcinoma head of pancreas

Section 2

• Acute intermittent porphyria

265

140. Ans. a. EUS guided transgastric biopsy (Ref: Sabiston 20/e p1544; Schwartz 10/e p1399; Bailey 27/e p1235) 141. Ans. c. Carcinoma pancreas (Ref: Sabiston 20/e p1544; Schwartz 10/e p1397; Bailey 27/e p1235) • Double duct sign on ERCP is highly suggestive of pancreatic head cancerQ. 142. Ans. a. Periampullary carcinoma (Ref: Shackelford 7/e p1183)

143. Ans. a. Head

144. Ans. b Periampullary carcinoma (Ref: Sabiston 20/e p1544; Schwartz 10/e p1395; Bailey 27/e p1234) • “The waxing and waning nature of jaundice is due to sloughing of ampullary cancer, resulting in transient resolution of the jaundice.” 145. Ans. d. Carcinoma head of pancreas

CARCINOMA PANCREAS: TREATMENT AND PROGNOSIS 147. Ans. a. Lung (Ref: Sabiston 20/e p1544) Metastatic Tumors to Pancreas • MC site of primary: RCCQ > Malignannt melanoma • On autopsy, MC site of primary: CA lungQ 148. Ans. a. Delayed gastric emptying (Ref: Sabiston 20/e p1548; Schwartz 10/e p1406; Blumgart 6/e p464, 5/e p819-825; Shackelford 8/e p 1144, 7/e p1200) • • • •

Morbidity Following Pancreaticoduodenectomy Delayed gastric emptying (18%)Q • Cardiac events (3%) Pancreas fistula (12%) • Bile leak (2%) Wound infection (7%) • Overall reoperation (3%) Intra-abdominal abscess (6%)

149. Ans. b. Pain is early in the course of the disease (Ref: Sabiston 20/e p1544) Carcinoma Pancreas • Symptoms include unexplained episodes of pancreatitisQ, painless jaundice, nausea, vomiting, steatorrhea, and unexplained weight loss • With further spread beyond the pancreas, these patients may note upper abdominal or back pain when peripancreatic nerve plexuses are involved and ascites when peritoneal carcinomatosis or portal vein occlusion developsQ 150. Ans. c. Gemcitabine (Ref: Bailey 27/e p1237; Blumgart 6/e p 1040, 5/e p925; Shackelford 8/e p 1146, 7/e p1202) • GemcitabineQ is currently the standard of care for patients with metastatic pancreatic cancer. 151. Ans. b. Tumors of head pancreas (Ref: Blumgart 6/e p 1020, 5/e p959-960)

Middle or Central or Segmental Pancreatectomy • Middle pancreatectomy is a safe, effective procedure for treatment of benign and low grade malignant neoplasms of the mid pancreasQ Advantages of Middle or Segmental Pancreatectomy • Preserves pancreatic parenchyma • Reduces the risk of exocrine and endocrine insufficiency • Consists of a limited resection of the mid portion of the pancreas

Hepatobiliary Pancreatic Surgery

146. Ans. a. R0 resection (Ref: Sabiston 20/e p1546; Schwartz 10/e p1394; Blumgart 6/e p 983, 5/e p819-825; Shackelford 8/e p1144, 7/e p1192-1202)

Hepatobiliary Pancreatic Surgery

266

Surgery Essence • Can be performed in selected patients affected by tumors of the pancreatic neckQ • In experienced hands it is associated with no mortality but with high morbidty, even if the rate of “clinical” panareatic fistula is about 20% • Middle pancreatectomy is avoided in patients affected by main duct IPMN. Indications of Middle or Segmental Pancreatectomy • Benign or low grade malignant tumor (Neuroendocrine tumors, serous cystadenoma, branch duct IPMNs) Q • Location in the neckQ or its contiguous portion • A distal pancreas stump of at least 5cm in lengthQ 152. Ans. c. Median survival in locally advanced (stage III) disease is 3-6 months (Ref: Sabiston 20/e p1547; Blumgart 6/e p 963, 5/e p922; Shackelford 8/e p 1144, 7/e p1195-1196) 153. Ans. b. Duodenojejunostomy (Ref: Sabiston 20/e p1546; Bailey 27/e p1237; Shackelford 8/e p1185, 7/e p1196) • Duodenojejunostomy is done in PPPD, not in Whipple’s procedure. • Gastrojejunostomy is done in Whipple’s procedure. 154. Ans. c. Portal vein

155. Ans. d. Choledochojejunostomy (Ref: Devita 9/e p979)

Palliative Surgery in Advanced, Non-resectable Pancreatic Adenocarcinoma • Surgery for advanced, non-resectable pancreatic adenocarcinoma can palliate obstruction of CBD or duodenum, as well as control visceral painQ • Hepaticojejunostomy, choledochojejunostomy or choledochoduodenostomy offers durable drainage of an obstructed bile duct.Q • Cholecystojejunostomy is less reliable but can be employed when tumor bulk precludes common duct procedure • Gastrojejunostomy (antecolic anastomosis) palliates gastric outlet obstructionQ • Antecolic anastomosisQ is done to avoid complications from an expanding lesser sac tumor 156. Ans. d. Periampullary (Ref: Shackelford 8/e p 1138, 7/e p1187-1206) Periampullary Carcinoma 1. Adenocarcinoma of head of the pancreas (40-60%)Q 2. Adenocarcinoma of ampulla of vater (10-20%) 3. Distal bile duct adenocarcinoma (10%) 4. Duodenal adenocarcinoma (5-10%) • Patients with pancreas adenocarcinoma involving the body or tail of the gland are more likely to have weight loss and abdominal pain as their initial complaints. • These lesions can grow to a larger size before producing symptoms and are often diagnosed at a later stage with a poorer prognosis. • Most body and tail cancers have already metastasized to distant sites or extended locally to involve nodes, nerves, or major vessels by the time of diagnosis. • Best prognosis: Duodenal adenocarcinoma >Ampullary carcinoma >Distal Bile duct adenocarcinoma >Head of pancreas >Body and tail of Pancreas (DAD Head Body and Tail)Q 157. Ans. a. Disruption of pancreatic anastomosis

158. Ans. c. Pancreaticoduodenectomy

159. Ans. b. Mobilization of 2nd part of duodenum

Section 2

• Kocher maneuver is a technique for mobilization of the duodenumQ. 160. Ans. b. Pancreatic carcinoma

161. Ans. c. CA pancreas

162. Ans. c. Good prognosis 163. Ans. d. Acute pancreatitis never occurs in CA pancreas (Ref: Sabiston 20/e p1544)

Clinical Features of Carcinoma Pancreas • MC site is head and uncinate process • Patients with lesions that occur near the bile duct, such as those near the ampulla, head of the pancreas, and uncinate process, are much more likely to have obstructive jaundiceQ. • Those with lesions in the body or tail of the pancreas are more likely to complain of pain. Q

Pancreas Pain suggests unresectabilityQ in carcinoma pancreas Two third (65%) patients present with diabetes in carcinoma pancreasQ Patients may also have acute pancreatitis secondary to obstruction of the pancreatic ductQ. Elderly patients with acute pancreatitis but without a history of alcohol use or gallbladder stones should be screened for a neoplasmQ.

164. Ans. a. CA 19-9

165. Ans. b. CA duodenum (Ref: Shackelford 8/e p1138, 7/e p1201)

• Best prognosis: Duodenal adenocarcinoma >Ampullary carcinoma >Distal Bile duct adenocarcinoma >Head of pancreas >Body and tail of Pancreas (DAD Head Body and Tail) 166. Ans. a. Retroperitoneal margin (Ref: Shackelford 7/e p1196-1200)

Section 2

• • • •

267

167. Ans. c. Cardiopulmonary complications

168. Ans. b. Distal pancreatectomy with splenectomy (Ref: Sabiston 20/e p1547; Schwartz 9/e p1226; Bailey 27/e p1236; Shackelford 8/e p1186, 7/e p1198-199)

Resectional Surgery for Pancreatic Body and Tail Tumors • Most body and tail cancers have already metastasized to distant sites or extended locally to involve nodes, nerves, or major vessels by the time of diagnosis. • Splenic vein involvement or occlusion is not a sign of nonresectabilityQ. • Involvement of the splenic and SMV confluence generally precludes resection. • Resection involves a distal pancreatectomy either with or without concomitant splenectomyQ. • Splenectomy is usually performed with distal pancreatectomy in patients suspected of having carcinoma to obtain better margins, to remove the lymph nodes at the tip of the pancreas and the hilum of the spleen, and to avoid tedious dissection of the splenic artery and vein. 170. Ans. a. Pyloric antrum

171. Ans. a. Delayed gastric emptying (Ref: Sabiston 20/e p1548; Schwartz 10/e p1406; Bailey 27/e p1236) 172. Ans. b. 22 months (Ref: Sabiston 20/e p1547) “After surgical resection and adjuvant therapy for pancreatic cancer, the median survival is approximately 22 months, with 5-year survival of 15% to 20%. Most patients experience relapse of disease in the form of metastatic disease (85%) and, less commonly, local recurrence (40%). In the absence of surgical resection, those with locally advanced disease who receive palliative chemotherapy may survive 10 to 12 months, whereas those with metastases rarely survive beyond 6 months.”-Sabiston 20/e p1547 173. Ans. d. Pancreaticojejunostomy, hepaticojejunostomy, gastrojejunostomy

PSEUDOPAPILLARY TUMOR 174. Ans. c. These are small tumors (Ref: Blumgart 6/e p968, 5/e p908-909; Shackelford 8/e p1173, 7/e p1268) 175. Ans. d. Chromagranin is positive

INSULINOMA 176. Ans. a. EUS with intra-operative palpation (Ref: Sabiston 20/e p947, 952; Schwartz 10/e p1391; Bailey 27/e p849; Blumgart 6/e p 1001, 5/e p935-937; Shackelford 8/e p1150, 7/e p1206-1210)

Localization of Insulinoma • A small portion of insulinomas remain unlocalizable despite extensive studies and are therefore considered occult. • When the diagnosis is certain based on the result of a 72-hour fast, surgical exploration with careful inspection, palpation, and intraoperative ultrasound (IOUS) is indicatedQ. • Studies have shown that the combination of surgical exploration with IOUS identifies almost all insulinomasQ.

177. Ans. d. Selective arteriography • Portal venous sampling for insulin with or without arterial stimulation with calcium is the best pre-operative method of localizationQ. • EUS with intra-operative palpation is best localization technique for InsulinomaQ.

Hepatobiliary Pancreatic Surgery

169. Ans. b. Ascites

Hepatobiliary Pancreatic Surgery

268

Surgery Essence 178. Ans. a. Insulinoma

179. Ans. a. Usually asymptomatic and need no treatment

180. Ans. c. Weight loss is important feature

181. Ans. a. 72-hours fasting test

182. Ans. b. Xylose test

183. Ans. a. Intra-operative USG

184. Ans. b. Enucleation

185. Ans. a. Insulinoma

NET of Pancreas • MC NET of Pancreas: Non-functional (Mostly malignant) >InsulinomaQ • MC benign NET of Pancreas: InsulinomaQ • MC malignant functional NET of Pancreas: GastrinomaQ 186. Ans. d. Equally distributed

187. Ans. a. Insulinoma

188. Ans. d. Equally distributed in head, body and tail (Ref: Sabiston 20/e p952; Schwartz 10/e p1391; Bailey 27/e p849) 189. Ans. b. Weight loss

ZOLLINGER-ELLISON SYNDROME 190. Ans. a. Duodenum (Ref: Sabiston 20/e p954; Schwartz 10/e p1071-1073; Bailey 27/e p850; Blumgart 6/e p999, 5/e p937-941; Shackelford 8/e p702, 7/e p749-756) 191. Ans. c. Decreased BAO and MAO • Basal acid output is increased in GastrinomaQ. 192. Ans. d. SRS

193. Ans. c. Most common site is pancreas

194. Ans. d. 4th part of duodenum

195. Ans. c. Decreased BAO/MAO

196. Ans. a. 50% are associated with adrenal malignancy

197. Ans. c. Beta cell tumors of the pancreas

198. Ans. d. H2 receptor antagonist

199. Ans. a. Endoscopic ultrasound

200. Ans. a. Gastrinoma triangle (Ref: Schwartz 10/e p1392; Sabiston 20/e p954; Bailey 27/e p1141) 201. Ans. b. Junction of 3rd and 4th part of duodenum

202. Ans. a. Duodenum

203. Ans. d. Serum gastrin value of 200 pg/ml with secretin stimulation 204. Ans. d. Massive HCl in response to histamine injection 205. Ans. a. Gastrin levels >1000 pg/mL, b. BAO (Basal acid output) >15 meq/hr, c. Somatostatin is inhibitor of HCL secretion, d. Omeprazole is helpful, e. Secretin increases gastrin secretion in Zollinger-Ellison syndrome 206. Ans. b. Exocrine tumor

207. Ans. b. Reduced BAO: MAO ratio

208. Ans. b. Zollinger-Ellison syndrome 209. Ans. a. Pylorus (Ref: Sabiston 20/e p1210; Schwartz 10/e p1392; Bailey 27/e p851) 210. Ans. c. Zollinger-Ellison syndrome

ENDOCRINE TUMORS OF PANCREAS 211. Ans. a. SRS (Ref: Sabiston 20/e p957; Schwartz 10/e p1393; Bailey 27/e p849; Blumgart 6/e p1001, 5/e p940-941; Shackelford 8/e p1150, 7/e p1211-1212) 212. Ans. a. Insulinoma 213. Ans. d. SRS (Ref: Sabiston 20/e p947) 214. Ans. b. Glucagonoma 215. Ans. a. Most PPomas are benign (Ref: Sabiston 20/e p958; Schwartz 10/e p1393; Bailey 27/e p852; Shackelford 8/e p1155, 7/e p1213-1214)

Section 2

216. Ans. d. Increased acid secretion (Ref: Sabiston 19/e p954, 956, 960; Schwartz 10/e p1392-1393; Bailey 27/e p850) 217. Ans. c. Somatostatinoma (Ref: Sabiston 20/e p957; Schwartz 10/e p1393; Bailey 27/e p849) 218. Ans. b. Glucagonoma (Ref: Sabiston 20/e p957; Schwartz 10/e p1393; Bailey 27/e p849) Necrolytic erythema migrans is the most common symptom of glucagonoma. 219. Ans. a. Glucagonoma

220. Ans. b. Somaststationomas

221. Ans. a. Cyanosis (Ref: Sabiston 20/e p956)

222. Ans. a. SRS

223. Ans. a. VIPoma (Ref: Sabiston 20/e p965; Schwartz 10/e p1392; Bailey 27/e p849) 224. Ans. a. Glucagonoma (Ref: Sabiston 20/e p957; Schwartz 10/e p1393; Bailey 27/e p849) 225. Ans. a. Insulinoma is MC, b. VIP cause diarrhea, d. Somatostatinoma cause gallstone formation, e. Gastrinoma has high chance of malignancy

Pancreas

269

NET of Pancreas • Gastrinoma

MC site • Duodenum (1st part) >PancreasQ

• Insulinoma

• Equally distributedQ in head, body and tail

• Glucagonoma

• Body and TailQ

• Somatostatinoma and PPoma

• HeadQ

• VIPoma

• TailQ

All pancreatic cancers and neoplastic cysts are most common in pancreatic head except: • Mucinous cystic neoplasm and Glucagonoma: MC in body and tailQ

Section 2

Tumor

• VIPoma: MC in TailQ • Insulinoma: Equally distributedQ in head, body and tail

PANCREAS DIVISUM 226. Ans. a. Most common congenital anomaly (Ref: Sabiston 20/e p1522; Schwartz 10/e p1365-1366; Bailey 27/e p1219; Blumgart 6/e p861, 5/e p818-819; Shackelford 8/e p1127, 7/e p1134-1135) 227. Ans. b. Pancreas divisum (Ref: Sabiston 20/e p1522; Schwartz 10/e p1366; Bailey 27/e p1219) In this ERCP, major part of pancreas is drained by accessory duct of Santorini via minor papilla and head and uncinate process is drained by duct of Wirsung via ampulla suggestive of pancreas divisum. Pancreas Divisum

228. Ans. e. Treatment is division of ring (Ref: Sabiston 20/e p1522; Bailey 27/e p1219; Blumgart 6/e p870, 5/e p823-826; Shackelford 8/e p1215, 7/e p1179-1180) 229. Ans. d. Duodenoduodenostomy (Ref: Blumgart 6/e p860, 5/e p823-826) • Read the text from the latest edition of Blumgart, surgery of liver, biliary tract and pancreas. It clears that duodenoduodenostomy is the treatment of choice for annular pancreas, not the duodenojejunostomy given in Harrison or Maingot. • Duodenoduodenostomy has replaced duodenojejunostomy as the treatment of choiceQ because it has a lower incidence of postoperative complications, particularly obstruction and blind-loop syndromesQ.

Treatment of Annular Pancreas • A review of the existing literature published in 1980 concluded that “while there is no single operative procedure of choice, experience militates against any direct attack on the offending annulus”. This conclusion stands, and any attempt to divide the annulus itself risks the formation of a pancreatic fistula. • Early pediatric series established duodenal bypass as the treatment of choice, although mortality rates remained high, likely related to the presence of other congenital malformations and the lack of supportive care. • Duodenoduodenostomy has replaced duodenojejunostomy as the treatment of choice because it has a lower incidence of postoperative complications, particularly obstruction and blind-loop syndromesQ. 230. Ans. b. Duodenoduodenostomy

Hepatobiliary Pancreatic Surgery

ANNULAR PANCREAS

270

Surgery Essence PANCREATIC TRAUMA

Hepatobiliary Pancreatic Surgery

231. Ans. a. Hyperamylasemia is not specific (Ref: Sabiston 20/e p440, 1552; Schwartz 9/e p179) 232. Ans. d. HRCT is investigation of choice 233. Ans. b. Fracture is common at the junction of head and body, d. Peritoneal lavage is good for making the diagnosis 234. Ans. d. Amylase increases in 90% cases, e. HRCT is investigation of choice

PANCREATIC TRANSPLANTATION 235. Ans. b. Amylase levels (Ref: Sabiston 20/e p644-665; Schwartz 10/e p340-344; Bailey 27/e p1553; Blumgart 6/e p1881, 5/e p1796-1805; Shackelford 8/e p1230, 7/e p1251-1260) • Bailey 25/e p1425: ‘Urinary drainage of the pancreas has the advantage that urinary amylase levels can be used to monitor graft rejection’Q

MISCELLANEOUS 236. Ans. d. Observe and medical treatment (Ref: Blumgart 6/e p873, 5/e p827; Bailey 27/e p120) 237. Ans. b. Ileum • Terminal ileum is filled with meconium in mucoviscidosis. 238. Ans. a. Juvenile diabetes • Type 1 diabetes is a T cell-mediated autoimmune disease, characterized by lymphocytic infiltration of the pancreatic isletsQ. 239. Ans. c. Mesentery (Ref: Sabiston 20/e p1522; Bailey 27/e p1219; Blumgart 6/e p873, 5/e p827)

Ectopic Pancreas • • • • •

The most common sites are in the walls of the stomachQ, duodenumQ, or ileum, in a Meckels diverticulumQ, or at the umbilicusQ Less common sites include the colon, appendix, gallbladder, omentum, and mesentery. Most ectopic pancreatic tissue is functionalQ. Islet tissue is frequently present when ectopic pancreas is located in the stomach and duodenum. Ectopic pancreatic tissue is a submucosal, irregular nodule of firm, yellow tissue that has a central umbilicationQ; pancreatic secretions often exit through this umbilication.

240. Ans. b. Superior mesenteric vein (Ref: BDC 4/e pvol-II/284-285) • The posterior surface of neck of pancreas is related to the termination of superior mesenteric vein and beginning of portal veinQ. 241. Ans. d. All

242. Ans. c. Insulinoma

243. Ans. b. Occurs in adults more than children 244. Ans. c. Causes severe hypoglycemia (Ref: Goodman Gilman 12/ep1271)

Diazoxide

Section 2

• K+ channel opener, causes arteriolar dilatationQ • Diazoxide produces hyperglycemia by decreasing insulinQ Used in insulinomaQ • When used as intravenous antihypertensive agentQ, it causes excessive hypotension 245. Ans. b. 11 O’ clock position (Ref: Sabiston 20/e p1396; Schwartz 10/e p1327)

Transduodenal Sphincteroplasty • This cut is made superiorly (at the 11’ O clock position) Q for 4 to 5 mm. • The sphincter is incised at the 11’ O clock position to avoid injury to the pancreatic ductQ. 246. Ans. a. Stomach 247. Ans. c. Surgical intervention is required in most because of features of chronic pancreatitis 248. Ans. b. Beta cells (Ref: Sabiston 20/e p943)

Pancreas Islet Cells

271

Content • Glucagon, glicentinQ, pancreastatinQ

• Beta cells

• Insulin, amylinQ, pancreastatinQ

• D cells

• Somatostatin

• D2 cells

• VIP

• G cells

• Gastrin

• PP cells

• Pancreatic polypeptide

249. Ans. b. CT scan (Ref: Sutton 7/e p796)

Section 2

• Alpha cells

• CT is the mainstay of pancreatic imaging, able to demonstrate focal masses within the gland calcifications, duct dilatation, cysts, abscesses and associated abnormalities in upper abdominal organs (hepatic metastases), lymph nodes and peri-pancreatic vascular structures. • CT is a useful tool for guiding percutaneous pancreatic biopsy and cyst aspiration or drainage. 250. Ans. b. Cystic duct (Ref: Sabiston 20/e p1482)

Hepatobiliary Pancreatic Surgery

SECTION

3

Gastrointestinal Surgery CH A PT ERS ˆˆ ˆˆ ˆˆ ˆˆ ˆˆ ˆˆ ˆˆ ˆˆ ˆˆ ˆˆ ˆˆ ˆˆ

Esophagus Stomach and Duodenum Peritoneum Intestinal Obstruction Small Intestine Large Intestine Ileostomy and Colostomy Inflammatory Bowel Disease Vermiform Appendix Rectum and Anal Canal Hernia and Abdominal Wall Spleen

CHAPTER

9

Esophagus

CONGENITAL DIAPHRAGMATIC HERNIA (BOCHDALEK HERNIA OR POSTEROLATERAL HERNIA) Congenital Diaphragmatic Hernia (Bochdalek Hernia or Posterolateral Hernia) • CDH term is used for Bochdalek herniaQ • Incidence 1 in 2000 to 5000Q live births. • Most CDH defects are on the left side (80%); up to 20% on right sideQ. • Rarely bilateral. • Survival rate for CDH 70–90%Q. • Bag & Mask ventilation is contraindicated in CDHQ. Pathogenesis • The cause is thought to result from failure of normal closure of the pleuroperitoneal canalQ in the developing embryo. • As a result, abdominal contents herniate through the resultant defect in the posterolateral diaphragm and compress the ipsilateral developing lungQ. • Compression of the lung results in pulmonary hypoplasia involving both lungs, with the ipsilateral lung being the most affectedQ. • Pulmonary vasculature is distinctly abnormal in that the medial muscular thickness of the arterioles is excessive and extremely sensitive to the multiple local and systemic factors known to trigger vasospasmQ. • Main factors affecting morbidity and mortality: Pulmonary hypoplasia & pulmonary hypertensionQ. Clinical Features • Classic triad: Respiratory distress + Dextrocardia + Scaphoid abdomenQ • MC presentation is respiratory distress due to severe hypoxemia. • Infant appears dyspneic, tachypneic, & cyanotic, with severe retractions. • Anteroposterior diameter of the chest may be large, & abdomen may be scaphoidQ. Diagnosis • Diagnosis is made at the time of a prenatal ultrasound during pregnancy. • Postnatal diagnosis by a plain chest radiograph demonstrates the gastric air bubble or loops of bowel within the chestQ. • There may also be a mediastinal shift away from the side of hernia or polyhydramniosQ from the obstructed stomach. • Pneumothorax always occurs on contralateral to the side of CDHQ. Treatment • Physiologic stress associated with early repair probably adds more insult and that survival is not improvedQ when compared with delayed repair. • A variable period of time (24–72 hours) to allow for stabilization before surgical repairQ. • The viscera are reduced into the abdominal cavity, & posterolateral defect in diaphragm is closed using interrupted, nonabsorbable suturesQ. • In most cases (80%–90%), a hernia sac is not present. If identified, it is excised at the time of repairQ. • Advantage of a prosthetic patch is that a tension-free repair can be frequently obtained in large defectsQ.

276

Surgery Essence MORGAGNI HERNIAS (RETROSTERNAL HERNIAS OR LARREY’S HERNIAQ) Morgagni hernias (Retrosternal hernias or Larrey’s herniaQ)

Gastrointestinal Surgery

• Congenital hernia of anteromedial, retrosternal diaphragm • Occur in the triangular space between the muscle fibers that make up the diaphragm • They extend from the xiphisternum and costal margin to the central tendon of diaphragm. • Ninety percent are right sidedQ because the pericardium itself prevents left-sided hernias • Superior epigastric vessels may pass through Morgagni spaceQ • Most commonly involved viscus is transverse colonQ Clinical Feature • Patients are usually asymptomaticQ Diagnosis • Anterior mediastinal masses are found incidentally on chest radiographsQ. Treatment • Prompt surgical repair after diagnosis is prudent to avoid incarceration or strangulation of abdominal organs. • A transabdominal routeQ is the preferred choice. • Prosthetic mesh is generally required to repair the defectQ.

HIATUS HERNIA Types of Hiatal hernia Type I

Sliding hiatal hernia (MC)Q

Type II

True paraesophageal herniaQ

Type III

Mixed paraesophageal hernia (I and II)Q

Type IV

Paraesophageal hernia containing other intra abdominal organsQ Types of Hiatal Hernia

Pathophysiology of Hiatus Hernia

Section 3

Type I Hernia or Sliding HH • Characterized by upward displacement of the GE junction into the posterior mediastinumQ. • The stomach remains in its usual longitudinal alignmentQ. • A higher incidence of HH has also been found in people with inguinal herniasQ. • Majority of patients with HH are asymptomaticQ • The prevalence and size of the sliding HH correlate with increasing severity of reflux diseaseQ. Type II Hernia • True PEH: Defined by a normally positioned intra abdominal GE junction with upward herniation of the stomachQ alongside it. • A PEH develops when there is a defect, possibly congenital, in the hiatus anterior to the esophagus. • Persistent posterior fixation of the GE junction is the essential difference between a PEH and a sliding HH. Type III Hernia • Mixed hernia: Characterized by displacement of both the GE junction and a large portion of the stomach cephalad into the posterior mediastinumQ. • Starts as a sliding HH, and over time as the hiatus enlarges, and more of fundus and body of the stomach herniate into the chest.

Esophagus

277

Type IV Hernia

FACTORS AFFECTING LES PRESSURE Lower Esophageal Sphincter (LES) Pressure Decreased by • • • • • • • •

Increased by

Prostaglandin E1 and E2, Progesterone Morphine and MeperidineQ TheophyllineQ Barbiturates, DiazepamQ, Dopamine CCB, Atropine, NitratesQ Chocolate, CoffeeQ Alcohol, PippermintQ Smoking, FatQ

Q

• • • • • • • •

Section 3

• Esophageal hiatus has dilated to such an extent that the hernia sac also contains other organs such as the spleen, colon, or small bowelQ. • Bowel obstruction and complicationsQ due to altered anatomy.

Bombesin, Angiotensin IIQ PP, Substance P, MotilinQ GastrinQ AntacidsQ CholinergicsQ Domeperidone MetoclopramideQ Prostaglandin F2α

• PMT BD CAN decrease LES pressure: Prostaglandin E1 and E2, Progesterone, Morphine and Meperidine, Theophylline, Barbiturates, Diazepam, Dopamine, CCB, Atropine, Nitrates • CAPS Fat decrease LES pressure: Chocolate, Coffee, Alcohol, Pippermint, Smoking, Fat. • PSM BAG increase LES pressure: PP, Substance P, Motilin, Bombesin, Angiotensin II, Gastrin

GASTROESOPHAGEAL REFLUX DISEASE (GERD) Gastroesophageal Reflux Disease (GERD)

Pathophysiology • LES has the primary role of preventing reflux into the esophagus. Factors Contributing to the High-pressure Zone in the Lower Esophagus • Intrinsic musculatureQ of the distal esophagus which are in a state of tonic contraction • Sling fibers of the cardiaQ which are at the same anatomic depth of the circular muscle fibers of the esophagus but are oriented in a different direction • DiaphragmQ: during inspiration the anteroposterior diameter of the crural opening is decreased, compressing the esophagus and increasing the measured pressure at the LES • Transmitted pressureQ of the abdominal cavity • GERD is often associated with a hiatal hernia (MC type is type I or sliding herniaQ). • A hiatal hernia is neither necessary nor sufficient to make the diagnosis of GERD, and the presence of such a hernia does not constitute an indication for operative correctionQ. • Many patients with hiatal hernias do not have symptoms and do not require treatmentQ. Clinical Features • Classical triad of symptoms is retrosternal burning pain, epigastric pain and regurgitation. • MC presentation of GERD: Long-standing history of heartburn and a shorter history of regurgitationQ. • Symptoms of GERD: Heartburn (80%)Q, Regurgitation (54%), Abdominal pain (29%), Cough (27%), Dysphagia for solids (23%), Belching (15%), Bloating (15%), Aspiration (14%), Wheezing (7%). Diagnosis • Endoscopy: Exclude other diseases, especially a tumor, and to document the presence of peptic esophageal injury. An essential step in the evaluation of GERD, who are being considered for operative interventionQ. • Manometry: For information about the function of the esophageal body and LESQ

Gastrointestinal Surgery

• Classical triad of symptoms is retrosternal burning pain, epigastric pain & regurgitationQ. • GERD is associated with complications such as esophageal ulcerations (5%), peptic strictures (4–20%), & Barrett’s esophagus (8–20%).

278

Surgery Essence

Gastrointestinal Surgery

24-hour pH Monitoring • Gold standard for diagnosing and quantifying acid reflux is the 24-hour pH testQ. • Information from the study: Total number of reflux episodes (pH < 4), longest episode of reflux, number of episodes lasting >5 minutes, extent of reflux in the upright and supine position. • DeMeester scoreQ: An overall score is obtained with the use of a formula that assigns a weight to each item according to its capacity to cause esophageal injury. • DeMeester score needs to be 10 years of life expectancy and are in need of lifelong therapy due to a mechanically defective sphincter, surgical therapy may be considered the treatment of choiceQ. • The principles of modern Nissen fundoplication include secure crural closure and creation of a short (≤2 cm), 360-degree “floppy” fundoplication designed to most closely replicate the normal physiology of the gastroesophageal flap valveQ.

TYPES OF FUNDOPLICATION Type of Fundoplication Watson Dor Toupet Belsey Mark IV Nissen

• • • • •

Degree of Wrap 90-degree anterior fundoplicationQ 180-degree anterior fundoplicationQ 180-degree posterior fundoplication subsequently modified to a 270-degree wrapQ 270-degree anterior fundoplicationQ 360-degree fundoplicationQ

ACHALASIA CARDIA Achalasia Cardia MC motility disorder of esophagusQ         • MC hypomotility disorder of esophagusQ Achalasia means “failure to relaxQ” (sphincter remains in a constant state of tone with periods of relaxation) Both the muscle of esophagus & LES are affectedQ. Prevailing theory: Destruction of the nerves to LES is primary pathology & degeneration of neuromuscular function of body of esophagus is secondaryQ. • Premalignant condition leading to squamous cell carcinoma

• • • •

Section 3

• Triple A-syndrome or Allgrove diseaseQ: Achalasia, Alacrima and ACTHresistant Adrenal insufficiency. Pathogenesis • Progressive inflammation & selective loss of inhibitory myenteric neurons in Auerbach’s plexus of esophagus that normally secrete VIP & nitric oxideQ. • This results in failure of relaxation of LES and aperistalsis of esophageal body with subsequent functional obstruction at the level of GE junction & gradual dilatation of esophagusQ. Clinical Features • Classic triad of symptoms consists of dysphagia, regurgitation, and weight loss. • Heartburn, postprandial choking, & nocturnal coughing are seen commonly. • Men & women are equally affected, with no ethnic predisposition to the diseaseQ.

Esophagus

Diagnosis • Barium swallow: • Dilated esophagus with a distal narrowingQ • “Bird’s beak”, “Pencil-tip” or “Rat’s tail” appearanceQ • Hurst phenomenon: Thin stream of barium flows beyond bird beak due to increased esophageal pressureQ • Lack of a gastric air bubbleQ on the upright portion is a result of the tight LES not allowing air to pass easily into the stomach. • Massive esophageal dilation, tortuosity, and a sigmoidal esophagus (megaesophagus) in advanced stageQ

Section 3

• Regurgitation of undigested, foul-smelling foods is common, and with progressive disease, aspiration can become life-threateningQ. • Pneumonia, lung abscess, & bronchiectasis often result from long-standing achalasia. • Dysphagia progresses slowly over yearsQ.

279

• Mecholyl test is positive in AchalasiaQ • CCK test is positive in AchalasiaQ • Manometry is gold standard test for diagnosis. −− Absence of body peristalsis & poor LES relaxation is mandatoryQ for diagnosis. Manometry Findings of Typical Achalasia Abnormalities of LES • Incomplete or absent LES relaxationQ • Elevated LES pressureQ

Abnormalities of Esophageal Body • Elevated intraesophageal pressureQ (pressurization of the esophagus) from incomplete air evacuation • Simultaneous mirrored contractions with no evidence of progressive peristalsisQ • Low-amplitude waveformsQ indicating a lack of muscular tone

Treatment

• Laparoscopic Heller myotomy is now the operation of choiceQ. • Extent of Heller’s myotomy: 2 cm above GE junction to 1 cm belowQ, over stomach. • Partial antireflux procedure (Toupet or Dor fundoplication)Q will restore a barrier to reflux and decrease postoperative symptoms. • Esophagectomy is considered megaesophagus, sigmoid esophagus, failure of more than one myotomy, or an undilatable reflux strictureQ.

DIFFUSE ESOPHAGEAL SPASM Diffuse Esophageal Spasm • Esophageal contractions are repetitive, simultaneous, and of high amplitudeQ. • Basic pathology is related to a motor abnormality of the esophageal body that is most notable in the lower two thirds of the esophagus. • More common in women and is often found in patients with multiple complaintsQ. Clinical Features • Clinical presentation: Chest pain and dysphagiaQ (may be related to eating or exertion and may mimic angina) • Complain of a squeezing pressure in the chest that may radiate to the jaw, arms, and upper back. • The symptoms are often pronounced during times of heightened emotional stressQ. Diagnosis • Barium swallow: −− Corkscrew or rosary-bead esophagus, segmental spasm or pseudodiverticulosis appearanceQ −− Due to presence of tertiary contractionsQ −− Indicative of advanced diseaseQ

Gastrointestinal Surgery

• Early stage: Sublingual nitroglycerin, nitrates, or calcium channel blockersQ may offer hours of relief of chest pressure before or after a meal. • Bougie dilationQ up to 54 French may offer several months of relief but requires repeated dilations to be sustainable. • Botulinum toxin: −− Injection of botulinum toxin (Botox) directly into the LES blocks acetylcholine release, preventing smooth muscle contraction, and effectively relaxes the LESQ. −− With repeated treatments, Botox may offer symptomatic relief for years −− Symptoms recur more than 50% of the time within 6 monthsQ.

280

Surgery Essence • Manometry is gold standard test for diagnosisQ. −− Classic manometry findings: Simultaneous, multipeaked contractions of high amplitude (>120 mm Hg) or long duration (>2.5 sec).

Gastrointestinal Surgery

Treatment • Mainstay of treatment for DES is nonsurgical, and pharmacologic (Nitrates, calcium channel blockers) or endoscopic intervention (Bougie dilation) is preferredQ. • Indications of surgery (long esophagomyotomy): −− Incapacitating chest pain or dysphagia who have failed medical and endoscopic therapyQ −− Presence of a pulsion diverticulum of the thoracic esophagus

NUTCRACKER ESOPHAGUS Nutcracker Esophagus • • • •

Hypermotility disorder also known as supersqueeze esophagusQ. Esophagus with hypertensive peristalsis or high-amplitude peristaltic contractions. Most common and most painful esophageal hypermotility disorderQ. Associated with hypertrophic musculature resulting in high-amplitude contractions of the esophagus

Clinical Feature • Chest pain and dysphagia are typical symptoms. Diagnosis • Gold standard of diagnosis is the subjective complaint of chest pain with simultaneous objective evidence of peristaltic esophageal contractions 2 standard deviations above the normalQ values on manometric tracings. • On manometry, amplitude >180 mm Hg and duration of contraction >6 seconds Treatment • The treatment of nutcracker esophagus is medical (Calcium channel blockers, nitrates, and antispasmodics) Q

HYPERTENSIVE LES Hypertensive LES • The LES pressure is above normal, motility of esophageal body may be hyperperistaltic or normal. Clinical Feature • Patients with hypertensive LES present with chest pain or dysphagiaQ. Diagnosis • Diagnosis is made by manometry. −− Elevated LES pressure (>26 mm Hg) and normal relaxation of the LES. −− Esophageal body may be hyperperistaltic or normal. Treatment

Section 3

• Botox injections alleviate symptoms temporarily, and hydrostatic balloon dilation may provide long-term symptomatic relief. • Surgery in patients who fail interventional treatments and those with significant symptoms. • A laparoscopic modified Heller esophagomyotomy is the operation of choice.

ZENKER’S OR PHARYNGOESOPHAGEAL DIVERTICULA Zenker’s or Pharyngoesophageal Diverticula • Mucosal outpouching (pulsion diverticulum) occurring through the triangular bare area (Killian’s triangle) Q, between the upper oblique fibers (thyropharyngeus muscle) and lower horizontal fibers (cricopharyngeus muscle) of the inferior constrictor muscleQ • Increased intraluminal pressures (secondary to abnormal esophageal motility)Q pushes mucosa and submucosa through a muscular defect in the wall of the esophagus creating a pulsion diverticulumQ • It is a pseudodiverticulaQ • It arises posteriorly in the midline of the neck, mouth is in midline but sac projects laterallyQ (usually left laterally)

Esophagus

Section 3

• It is not a true esophageal diverticulaQ, as it arises above the upper esophageal sphincter (the cricopharyngeus sphincter) • MC esophageal diverticulaQ Pathology • Neuromuscular incordinationQ in this region • May be due to different nerve supply of the two parts of inferior constrictor muscleQ −− The thyropharyngeus (oblique fibers) supplied by the pharyngeal plexus −− Cricopharyngeus (horizontal fibers) by recurrent laryngeal nerve Clinical Features • Usually seen in patients over 50 yearsQ • MC symptom is dysphagiaQ • Undigested food is regurgitated into the mouth, especially when the patient is in recumbent positionQ • Swelling of the neck, gurgling noise after eating, halitosis, and a sour metallic taste in the mouth are common symptoms • Cervical webs are seen associated in 50%Q of patients with Zenker’s diverticula, can cause dysphagia post-operatively if not treated. Diagnosis • Barium swallow is diagnostic Complications • Pneumonia and lung abscess due to aspirationQ (MC) • Perforation, Bleeding •  Carcinoma Management • Surgical therapy (Cricopharyngeal myotomy + Diverticulopexy) is treatment of choiceQ

281

Treatment Options for Zenker’s Diverticula Cricopharyngeal myotomy- a myotomy alone is sufficient for small diverticula Myotomy with excision of sac- done for large (>4 cm) diverticula Diverticulopexy Diverticulo-esophagostomy using a linear cutting staple gun −− The septum between the esophagus and the diverticula is divided −− Also known as Dohlman procedureQ

MIDESOPHAGEAL OR TRACTION DIVERTICULA Midesophageal or Traction Diverticula • Inflammation of the lymph nodes exerts traction on the wall of the esophagus leading to the formation of a true diverticulum in the midesophagus. • Caused by inflamed mediastinal lymph nodes from tuberculosis, histoplasmosisQ and resultant fibrosing mediastinitis. • Typically present on the right owing to the overabundance of structures in the midthoracic region of the left chest and wide mouthed. Clinical Features • Most patients are asymptomatic, incidentally found during a workup for some other complaint. • Dysphagia, chest pain, and regurgitation can be present and are usually indicative of an underlying primary motility disorder. Diagnosis • Investigation of choice is barium swallowQ (lateral views to determine side) • CT scan is helpful to identify any mediastinal lymphadenopathy and may help to lateralize the sac. • Manometry in all patients, symptomatic or not, to identify a primary motor disorder. Treatment • In asymptomatic patients who have inflamed mediastinal lymph nodes from tuberculosis or histoplasmosis: ATT or antifungal agentsQ • Diverticulopexy for symptomatic or 2 cm or larger diverticulum • Esophagomyotomy in severe chest pain or dysphagia and a documented motor abnormalityQ

Gastrointestinal Surgery

• • • •

282

Surgery Essence EPIPHRENIC DIVERTICULA

Gastrointestinal Surgery

Epiphrenic Diverticula • Epiphrenic diverticula are found adjacent to diaphragm in the distal third of the esophagus, within 10 cm of the GEJ. • Most often related to thickened distal esophageal musculature or increased intraluminal pressure. • Pulsion or false diverticula, often associated with DES, achalasia, and most commonly NEM (non-specific esophageal motility) disorders. • In patients in whom a motility abnormality cannot be identified, a congenital (Ehlers-Danlos syndrome) or traumatic cause is considered. • More common on the right side and tend to be wide-mouthedQ. Clinical Features • Most patients are asymptomatic. • Dysphagia or chest pain indicative of a motility disturbance. • The diagnosis is often made during the workup for a motility disorder, and the diverticulum is found incidentally. Diagnosis • Investigation of choice is barium swallow (lateral views to determine side) • Manometry to identify a primary motor disorder. Treatment • Diverticulopexy • Long esophagomyotomy in severe chest pain, dysphagia, or a documented motor abnormality Killian-Jamieson Diverticula • Lateral cervical esophageal diverticula • Located just below cricopharyngeus • Mostly asymptomatic

SCLERODERMA Scleroderma • Systemic disease accompanied by esophageal abnormalities in 80% of patients. • In most, the disease follows a prolonged courseQ. • Renal involvement occurs in a small percentage of patients and signals a poor prognosis. • The onset of the disease is usually in the third or fourth decade of life • Occurring twice as frequently in womenQ as in men. Pathophysiology • In the GI tract, the predominant feature is smooth muscle atrophyQ.

Section 3

Smooth muscle atrophy in lower two-thirds of esophagus →Incompetent LES →GERD →StrictureQ • Normal peristalsis in the proximal striated esophagus, with absent peristalsis in the distal smooth muscle portionQ. • The LES pressure is progressively weakened as the disease advances. • Gastroesophageal reflux commonly occurs in patients with scleroderma, because they have both hypotensive sphincters and poor esophageal clearance. • This combined defect can lead to severe esophagitis and stricture formationQ. Clinical Features • Dysphagia and GERD • Postural dysphagia for liquids: Dysphagia for liquids in recumbent position, not in upright position • Dysphagia for solids is unrelated to posture Diagnosis • Manometry: −− Normal peristalsis in the proximal striated esophagus, with absent peristalsis in the distal smooth muscle portion −− LES pressure is decreased but sphincter relaxation to deglutition is normal

Treatment

• Barium Swallow: Dilated, barium-filled esophagus, stomach, and duodenum, or a hiatal hernia with distal esophageal stricture and proximal dilatationQ.

• PPIs, antacids, elevation of the head of the bed, and multiple dilations for strictures for reflux • Esophageal shortening may require a Collis gastroplasty in combination with a partial fundoplicationQ. • Surgery reduces esophageal acid exposure, but does not return it to normal because of the poor clearance function of the body of the esophagusQ.

Esophagus

283

BARRETT’S ESOPHAGUS

Section 3

Barrett’s Esophagus • Distal squamous mucosa is replaced by metaplastic specialized (intestinalized columnar) epithelium, e.g. goblet cells, as a response to chronic injury; may regress after treatmentQ • Also called columnar lined esophagusQ • Metaplasia of esophageal squamous epithelium into columnar in distalQ esophagus • MC type of columnar epithelium is intestinal epithelium (Intestinal metaplasiaQ) Etiology • Usually due to chronic GERD Q • Columnar epithelium of Barrett’s may be more resistant to acid, pepsin & bileQ • Often associated with sliding hiatal herniaQ Clinical Features • Higher incidence in whites, males & obese • Symptoms: long history of heartburn & other reflux symptoms; more massive reflux with more numerous and longer episodes than most reflux patients • Major risk factor for esophageal adenocarcinomaQ Diagnosis • Characteristic endoscopic appearance plus characteristic histologic findingsQ • 8 random biopsies recommended • Report should include type of epithelium present and presence/absence of dysplasia, grade of dysplasia, & extent of dysplasiaQ • Barrett’s esophagus requires both endoscopically visible segment of columnar lining of distal esophagus and intestinal metaplasia showing goblet cells on biopsyQ

PREDISPOSING FACTORS FOR CARCINOMA ESOPHAGUS Predisposing Factors for Carcinoma Esophagus Squamous Cell Carcinoma • Alcohol • SmokingQ • Ingested carcinogens: −− NitratesQ, nitrites, nitrosaminesQ −− Smoked opiatesQ −− Fungal toxins in pickled vegetables • Mucosal damage: −− Chronic AchalasiaQ −− Lye (caustic) ingestion −− Long term ingestion of hot liquids −− Radiation induced stricturesQ • Plummer-vinson syndromeQ • Tylosis palmaris et plantarisQ: Congenital HyperkeratosisQ and pitting of palms and soles Q • Human papilloma virusQ • Esophageal diverticulaQ • BulimiaQ • Deficiency: Vitamin A, zinc, molybdenum Q

Adenocarcinoma • GERD (leading to Barrett’s esophagusQ) • ObesityQ • SclerodermaQ Scleroderma: Smooth muscle atrophy in lower 2/3rd of esophagus → Incompetent LES → GERD → Stricture • Diet deficient in fruits and vegetables • Diet high in animal protein and cholesterolQ

Gastrointestinal Surgery

Endoscopy • Red velvety GI type mucosa between pale squamous mucosa of lower esophagus & lush pink gastric mucosaQ • May have tongues extending up from GE junction or a broad band displacing GE junction proximally Positive Stains • Goblet cells contain acid mucin, usually sialomucinQ (Alcian blue+ at pH 2.5, although stain generally not needed or recommended), columnar cells contain neutral mucins (PAS+); intestinal metaplastic cells are often CK7+/CK20-; also CDX2+. • Guanylyl cyclase C+, Hep+ • In routine practice, only H&E is used for diagnosis Treatment • Antireflux therapyQ • Endoscopy every 1-2 years to detect dysplasia or early adenocarcinoma with 4 quadrant biopsies using jumbo forceps at intervals of 2 cm or less throughout the length of the Barrett’s segment plus any suspicious lesionsQ.

284

Surgery Essence CARCINOMA ESOPHAGUS

Gastrointestinal Surgery

Carcinoma Esophagus • • • • •

MC esophageal cancer worldwide: Squamous cell carcinomaQ MC esophageal cancer in United States (Western countries): AdenocarcinomaQ More common in malesQ MC site of CA esophagus: Middle 1/3rd (Overall)Q Chemotherapy regimen: Epirubicin + CisplatinQ + 5-FU (ECF) Squamous Cell Carcinoma

Adenocarcinoma

• Rarely seen before the age of 30 years • Highest mortality rates seen in men between 60-70 yearsQ of age. • Predominantly affects African American menQ • MC site: Middle 1/3rdQ • Obesity is protective • H. pylori CAG-A strain is a risk factorQ • Usually appears as an exophytic lesion with a large fungating massQ • More sensitive to chemoradiotherapyQ • Treated aggressively with nonsurgical therapyQ

• • • • • • • • • •

Seen infrequently before the age of 40 years Increases in incidence with ageQ Disease affecting white menQ Barrett’s esophagus: 40-foldQ increased risk for adenocarcinoma MC site: Lower 1/3rdQ Obesity is a risk factor H. pylori CAG-A strain is a protective Polypoid (5-10%), flat (10-15%), fungating (20-25%), or infiltrative (40-50%)Q Not as sensitive to chemoradiotherapy Treated by a more aggressive surgical approachQ.

Pathology • Esophageal cancer asserts aggressive biologic behavior. • With only two layers to the esophageal wall, tumors rapidly infiltrate through the muscular wall into surrounding structuresQ. • The rich vascular and lymphatic supply facilitates spread to regional lymph nodesQ. Clinical Features • Early-stage cancers: Asymptomatic or mimic symptoms of GERD. • MC symptom: Dysphagia >Weight lossQ • • • • •

Most patients with esophageal cancer present with dysphagia and weight loss, symptoms that usually indicate advanced disease. Choking, coughing, and aspiration from a tracheoesophageal fistula (In advanced cases)Q Hoarseness and vocal cord paralysis from direct invasion into the recurrent laryngeal nerve (In advanced cases)Q MC site of metastasis: LiverQ >lung >bone Paraneoplastic manifestation associated with adenocarcinoma: Motor NeuropathyQ

Diagnosis • Barium swallow: First investigation doneQ in suspected case of CA esophagus (classic finding of an apple core lesionQ) • Endoscopy with biopsy: Investigation of choice for diagnosis of CA esophagusQ. • Endoscopic Ultrasound: Investigation of choice for staging of CA esophagus, best for T staging and LN metastasisQ. • CECT (abdomen and chest): Assess the length of the tumor, thickness of the esophagus and stomach, regional LN status and metastasis to liver and lungsQ.

Section 3

Treatment of CA Esophagus High-grade dysplasia (Tis) or T1a

• Endoscopic Mucosal ResectionQ

Localized Esophageal Cancer

• T1: Vagal sparing or transhiatal or minimal invasive esophagectomy with limited LN dissectionQ • T2 and T3: Neo-adjuvant chemoradiation + SurgeryQ • Cervical SCC or Non-ideal candidate for resection: Definitive chemoradiationQ

Locally Advanced Cancer

• ChemoradiationQ (± Surgical resection in T4a)

Metastatic Disease

• Definitive chemoradiationQ (for involved distant LN or metastatic disease)

Malignant TEF

• Coated SEMSQ (self-expanding metallic stents)

• Postoperative chemoradiation is reserved for GE junction tumorsQ • Extent of Resection: An in-situ margin of 10 cmQ should be the goal

Esophagus

285

Prognosis • Depth of tumor invasion (T) •  Number of involved lymph nodes (N)Q •  LocationQ of the tumor in the esophagus • Prognosis is better for tumors of the cervical esophagus and tumors located at GE junctionQ, in comparison to tumors located in the thoracic esophagus.

TNM CLASSIFICATION OF CARCINOMA OF THE ESOPHAGUS 8th AJCC (2017) TNM Classification of Carcinoma of the Esophagus Tis: Carcinoma-in-situ/ High-grade dysplasia

Section 3

Long-term Survival Following Esophagectomy Depends on Q

N1: Metastasis in 1-2 regional LNsQ

T1a: Tumor invades lamina propria or muscularis mucosa T1b: Tumor invades submucosaQ

N2: Metastasis in 3-6 regional LNsQ

T2: Tumor invades muscularis propriaQ

N3: Metastasis in 7 or more regional LNsQ

T3: Tumor invades adventitiaQ

M1: Distant metastasisQ

Q

T4a: Tumor invades pleura, pericardium, azygous vein, diaphragm or peritoneumQ T4b: Tumor invades other adjacent structures such as aorta, vertebral body or tracheaQ

Stage 0

IIA

IIB

IIIA IIIB

IVA IVB

N0 N0 N0 N0 N0 N0 N0 N0 N0 N0 N1 N2 N1 N2 N1-2 N0-1 N2 Any Any

M0 M0 M0 M0 M0 M0 M0 M0 M0 M0 M0 M0 M0 M0 M0 M0 M0 M0 M1

1, X 2-3 Any 1 2-3, X Any 1 2-3 Any X Any Any Any Any Any Any Any Any Any

Stage Grouping Location Any Any Any Any Any Any Lower Upper, middle Upper, middle X Any Any Any Any Any Any Any Any Any Any

Adenocarcinoma Stage

T

N

M

0

Tis (HGD)

N0

M0

N/A

IA

T1a

N0

M0

1, X

IB

T1a

N0

M0

2

T1b

N0

M0

1, 2

IC

T1a, T1b

N0

M0

3

T2

N0

M0

1, 2

IIA

T2

N0

M0

3, X

IIB

T3

N0

M0

Any

T1

N1

M0

Any

T1

N2

M0

Any

T2

N1

M0

Any

T3

N0

M0

Any

T2

N2

M0

Any

IIIA

IIIB

IVA

IVB

T3

N1-2

M0

Any

T4a

N0-1

M0

Any

T4a

N2

M0

Any

T4b

Any

M0

Any

Any

N3

M0

Any

Any

Any

M1

Any

TYPES OF ESOPHAGECTOMY Ivor-Lewis

Orringer

Types of Esophagectomy • Transthoracic esophagectomyQ • Double incision: Midline laparotomy followed by right sided thoracotomyQ • Done for tumors of middle 1/3rd of esophagusQ • Transhiatal esophagectomyQ • Double incision: Midline laparotomy followed by Cervical incisionQ • MC procedure done for carcinoma esophagusQ

Grade

Gastrointestinal Surgery

IA IB

T Tis (HGD) T1a T1a T1b T2 T2 T3 T3 T3 T3 T3 T1 T1 T2 T2 T3 T4a T4a T4b Any

Stage Grouping Squamous Cell Carcinoma N M Grade N0 M0 1, X

286

Surgery Essence McKeon

• En-bloc esophagectomyQ • Three incisions: Right sided thoracotomy, followed by midlineQ laparotomy, followed by cervical incisionQ • Associated with maximum morbidity and mortalityQ

Gastrointestinal Surgery

Lymphadenectomy in CA Esophagus One-field lymphadenectomy

Removal of the intra-abdominal nodes (draining the proximal stomach and distal esophagus)Q

Two-field lymphadenectomy

Removal of the intra-abdominal + Intrathoracic nodesQ

Three-field lymphadenectomy

Removal of the intra-abdominal + Intrathoracic + Cervical LNQ

REPLACEMENT CONDUITS AFTER ESOPHAGECTOMY Replacement Conduits after Esophagectomy • • • •

Best conduit after esophagectomy (overall): StomachQ Conduit of choice after esophagectomy in CA esophagus: StomachQ Conduit of choice after esophagectomy in benign disorders (caustic injuries, acid-peptic disease), unhealthy stomach: ColonQ Conduit of choice for short segment replacement: JejunumQ • • • •

Routes of Replacement of Esophagus Posterior mediastinumQ through the bed of the resected esophagus Anterior mediastinal in the retrosternalQ position Lateral traspleural placement behind the lung rootQ Antethoracic or presternal subcutaneous routeQ

• Gastric conduit is based on right gastric and right gastroepiploic vesselsQ • Left colon is based on left colic artery (Branch of IMA), placed in isoperistaltic direction. • Posterior mediastinal route is preferred (shortest route) Q

LEIOMYOMA Leiomyoma • Leiomyoma is MC benign esophageal tumorQ. • The average age 4th-5th decades, more common in malesQ. • Originate in smooth muscle, 90% are located in lower two thirds of the esophagus. • Usually solitary and typically oval.Q • They remain intramural, having the bulk of their mass protruding toward the outer wall • The overlying mucosa is freely movable and normal in appearanceQ. Clinical Features • Many leiomyomas are asymptomatic. Dysphagia and pain are MC symptomsQ. • Location and size tend not to correlate consistently with symptoms Diagnosis • Barium swallow is IOC for leiomyoma (classical, smooth, contoured, punched-out defect)Q

Section 3

• Endoscopy: Freely movable mass, which bulges into the lumen, • Should not be biopsied because of an increased chance of mucosal perforation at the time of surgical enucleationQ. Treatment • Enucleation is TOC for leiomyomaQ.

DYSPHAGIA LUSORIA Dysphagia Lusoria • It is a disorder of swallowing caused due to vascular anomaliesQ (congenital abnormalities)Q • Vascular rings and pulmonary slings occur as a result of developmental abnormalities of the great vessels that cause compression of the esophagusQ. • The MC anomaly is right subclavian artery arising from the descending aorta and travels behind the esophagus to complete its course to the right upper extremity, may cause significant posterior compressionQ of the esophagus.

Esophagus

Section 3

• Anomalous right aortic arch with a left ligamentum arteriosum and a resultant retroesophageal left subclavian artery will form a complete ring that will also cause posterior esophageal compressionQ. • Pulmonary artery sling (left pulmonary artery arises from the right pulmonary artery instead of from the main pulmonary artery trunk)Q Clinical Features • Both vascular rings and pulmonary artery slings cause dysphagia. • Recurrent respiratory infections and difficulty in breathing are also common symptoms. Diagnosis • Barium swallow: Extrinsic anterior or posterior compression of the esophagus. • Angiography or HRCT: Identify the anomalous anatomyQ. Treatment • In symptomatic patients, both vascular rings and pulmonary artery slings are repaired. • Results: Dysphagia resolves nearly 100% of the timeQ.

287

Kommerell’s Diverticulum • An aberrant right subclavian artery originating from the proximal descending thoracic aorta can be associated with aneurysmal change at the origin of the artery, known as Kommerell’s diverticulumQ.

BOERHAAVE SYNDROME Boerhaave Syndrome

Clinical Features • Mackler’s triadQ of thoracic pain, vomiting, and cervical subcutaneous emphysema is seen in spontaneous esophageal perforation. • Thoracic perforations cause substernal and epigastric painQ. • Mediastinal emphysema & pleural effusions are common, but early cervical subcutaneous emphysema is noted in only 20% or less of patients. • Fever & sepsis develop with increasing contamination and inflammation of the mediastinum and pleural cavities. • Patients with an abdominal perforation have epigastric abdominal pain that is also often referred to the back and left shoulderQ. Diagnosis • Chest X-ray: HydropneumothoraxQ • The diagnosis is confirmed with a contrast esophagogramQ. This technique will demonstrate extravasation in 90% of patients. • Gastrografin (water soluble) is preferredQ to prevent extravasation of barium into the mediastinum or pleura. If no leak is seen, a barium study should follow. • Chest CT: Mediastinal air and fluid at the site of perforation. • Endoscopy: If the esophagogram is negative or if operative intervention is planned. Treatment • Appropriate resuscitation, secured airway, IV fluids and broad-spectrum antibiotics are started immediately, and the patient is monitored in an ICU. Within 24 Hours After 24 Hours • Golden period for primary closure of an esophageal perforation • Debridement of devitalized tissue + Esophageal is within the first 24 hoursQ. diversion or resection + Creation of an esophagostomy + Wide drainage + feeding • Within 24 hours of perforation, inflammation is generally minimal, jejunostomyQ and primary surgical repair is recommendedQ. • Mortality rate: >50%Q • Mortality rate: 8-20%

Gastrointestinal Surgery

• Spontaneous rupture usually occurs on left posterolateral side of distal esophagus into left pleural cavity or just above the gastroesophageal junction. • These patients are typically male (85%), 40 to 60 years of age, who have a history of recent emesisQ.

288

Surgery Essence TRACHEO-ESOPHAGEAL FISTULA

Gastrointestinal Surgery

Tracheoesophageal Fistula • • • •

TEF is an abnormal communication (fistula) between the esophagus and tracheaQ. TEF is usually associated with esophageal atresia, however it may also exist without atresiaQ. Prevalence of TEF is 2.6-3 per 10,000 births, with a slight male predominanceQ. MC anomaly associated with TEF is CVS (VSD) Q.

A

B

C

D

E

Classification of TEF • • • • •

Type A: Atresia only (6%) Type B: Atresia with proximal TEF (2%) Type C: Atresia with distal TEF (85%): Most commonQ Type D: Atresia with both proximal and distal TEF (rare) Type E: TEF only (1%)

Clinical Features • • • • •

The diagnosis of EA is entertained in an infant with excessive salivation along with coughing or choking during the first oral feeding. A maternal history of polyhydramnios is often present. The newborn baby with atresia regurgitates all of its first and subsequent feedsQ. Saliva pours continuously from its mouthQ. Repeated episodes of coughing, choking and cyanosisQ occur on feeding in TEF. Associated Anomalies

Section 3

• MC anomaly associated with TEF is CVS (VSD). • Esophageal atresia may occur as part of VACTERLQ group of anomalies: −− V: Vertebral body segmentation defectsQ −− A: Anal atresiaQ −− C: Cardiovascular (PDA, VSD)Q −− TE: Tracheoesophageal fistulaQ −− R: Renal (unilateral renal agenesis)Q −− L: Limb anomalies (radial ray hypoplasia)Q Diagnosis • • • •

The inability to pass a NG into the stomach is a cardinal feature for the diagnosis of EA. If gas is present in the GIT below the diaphragm, an associated TEF is confirmed Inability to pass a NG tube with absent radiographic evidence for gastrointestinal gas is virtually diagnostic of an isolated EA without TEF. For H-type: Tracheobronchoscopy + Endoscopy should be performed

Treatment • Surgical repair with tension free esophageal anastomosis Outcome • The mortality in TEF is directly related to the associated anomalies, particularly cardiac defects and chromosomal abnormalitiesQ.

Esophagus

289

SCHATZKI’S RINGS

• Consists of a concentric symmetric narrowing representing an area of restricted distensibility of the lower esophagusQ. • Lying precisely at the squamocolumnar mucosal GEJ, involves mucosa and submucosaQ • It consists of esophageal mucosa above and gastric mucosa below. • It does not have a component of true esophageal muscle, nor is it associated with esophagitis. • It is often accompanied by a small hiatal hernia Clinical Features • Most patients with Schatzki’s rings present with dysphagia. • The dysphagia is usually to solid foods only and comes on abruptly with nearly complete obstruction.

Section 3

Schatzki’s Rings

• Episodic aphagia: Intermittent obstruction of the nondistensible ring by large pieces of meatQ. Diagnosis • Diagnosis of a Schatzki’s ring is made with a barium swallowQ. • Schtazki ring: Type ‘B’ ring is located at GE junctionQ Treatment • Asymptomatic patients incidentally found to have a Schatzki’s ring require no treatment. • Best form of treatment of a symptomatic Schatzki’s ring without reflux: Esophageal dilation for relief of the obstructive symptoms. • Ring with proven reflux and mechanically defective sphincter: Antireflux procedure • Surgical excision is not indicated, can cause devastating esophageal strictures.

ANATOMY AND PHYSIOLOGY OF ESOPHAGUS Anatomy and Physiology of Esophagus

• Only Auerbach plexus is present in esophagus (Meissner’s plexus is absent)Q. • Lymphatic channels in the lamina propria are the anatomic features unique to the esophagusQ. • The dense submucosal lymphatic plexus facilitates early dissemination of esophageal malignancies. In submucosa elastic fibers & collagen combine to make this the strongest esophageal layer. Submucosal glands of mixed type are the characteristic of the esophagusQ. • Lacks a serosal layerQ; Strongest Cayer: submucosaQ • Lining: Lined by stratified, non-keratinized squamous epitheliumQ

• Muscles: −− Upper cervical region- Stratified muscleQ −− Middle- Gradual transition from stratified to smooth muscleQ −− Lower- Smooth muscleQ Upper Esophageal Sphincter

Lower Esophageal Sphincter

• Length: 4-5 cmQ • Pressure: 60 mm HgQ • Comprises three skeletal muscle groups: Distal portion of inferior pharyngeal constrictor, cricopharyngeus and circular muscle of proximal esophagusQ

• Length: 5 cmQ • Abdominal length: 2 cmQ • Pressure: 6-26 mm HgQ

Types of Esophageal Contractions Primary Contractions • • • •

ProgressiveQ contractions Triggered by voluntary swallowingQ Speed: 2-4 cm/sec Reach LES about 9 sec

Secondary Contractions • ProgressiveQ contractions • Triggered by distension or irritationQ of esophagus (but not by voluntary swallowing)

Tertiary Contractions • Non progressive, non-peristaltic contractionsQ • Occur spontaneously and simultaneously between swallowsQ

Gastrointestinal Surgery

• Narrowest tube of GIT; Narrowest region of esophagus: Cricopharynx (15 mm diameter) • Extent: C6-T11Q; Length: 25-30 cmQ • Cervical esophagus begins as a midline structure that deviates slightly to the leftQ of the trachea as it passes through the neck into the thoracic inlet. • At the level of the carina, it deviates to the right to accommodate the arch of the aorta. • It then winds its way back under the left main-stem bronchus and remains slightly deviated to the left as it enters the diaphragm through the esophageal hiatusQ. • Immediately before entering the abdomen, esophagus is pushed anteriorly by descending thoracic aortaQ.

Multiple Choice Questions CONGENITAL DIAPHRAGMATIC HERNIA



1. Which of the following is the most important determinant of prognosis in neonatal congenital diaphragmatic hernia (CDH)? (All India 2011) a. Pulmonary hypertension b. Delay in surgery c. Size of defect d. Gestational age at diagnosis 2. Which of the following is the least important prognostic factor in congenital diaphragmatic hernia? (All India 2011) a. Pulmonary Hypertension b. Delay in emergent surgery c. Size of defect d. Gestational age at diagnosis



3. All are true about Bochdalek hernia except: (GB Pant 2011) a. Posterolateral b. Left side c. Present in second decade d. Congenital



4. Most common organ that herniates in Morgagni’s hernia: a. Spleen b. Liver (MHSSMCET 2009) c. Stomach d. Transverse colon



5. Which of the following is contraindication for Bag and mask ventilation? (AIIMS June 2000) a. Septicemia b. Tracheoesophageal fistula c. Meconium aspiration d. Diaphragmatic hernia



6. False regarding Bochdalek hernia is: a. Spleen and kidney can herniate b. Occurs posterolaterally c. Always occurs on right side d. Hernia may or may not have sac



7. Not true about Bochdalek hernia: a. Seen on right side (Recent Question 2015, AIIMS Nov 97) b. Associated with hypoplasia of lung c. Associated with hiatus hernia d. Pericardial cyst is a differential diagnosis



8. The diagnostic feature of congenital diaphragmatic hernia on prenatal ultrasonography is: (AIIMS June 2001) a. A cyst behind the left atrium b. Mediastinal shift with normal heart axis c. Peristalsis in the thoracic cavity d. Absence of gas bubble under the diaphragm







(AIIMS Nov 93)

9. Most common site of Morgagni hernia: (AIIMS Nov 2006) a. Right anterior b. Right posterior c. Left anterior d. Left posterior 10. In congenital diaphragmatic hernia all are seen except: a. Common on left side (JIPMER 99) b. Abdominal distension c. Can be detected antenatally d. Heart beat shifted to right 11. Morgagni hernia: (APPG 98) a. Hernia between the costal and sternal part of the diaphragm b. Hernia through the pleuriperitoneal canal c. Hernia through the lumbar triangle d. Hernia through inguinal canal



12. A neonate with a scaphoid abdomen and respiratory distress has: (All India 94) a. Congenital pyloric stenosis b. Diaphragmatic hernia c. Volvulus d. Wilm’s tumor



13. Most common content of Morgagni foramen:  (Recent Question 2017) a. Stomach b. Small intestine c. Transverse colon d. Spleen

14. Most common diaphragmatic hernia in a newborn infant:  (Recent Question 2017) a. Bochdalek b. Morgagni c. Paraesophageal type I d. Paraesophageal type III

HIATUS HERNIA

15. True about hiatus hernia: (PGI June 2008) a. Surgery is indicated in all symptomatic cases b. Para-esophageal type is more complicated c. Para-esophageal type is common type d. Common in infants



16. Select the TRUE statement regarding the picture depicted here: (APPG 2016)

a. b. c. d.

A is known as sliding hernia Part of upper stomach enters into thorax in both A and B A is the more common form A causes symptoms of gastroesophageal reflux



17. The most common complication seen in hiatus hernia is: a. Esophagitis (DNB, 2011, All India 2005) b. Aspiration pneumonitis c. Volvulus d. Esophageal stricture



18. A 55-year-old male had retrosternal discomfort unrelated to physical exertion. Pain gets worse after lying down there is partial relief with antacids. The most likely diagnosis is: (UPSC 96) a. Ischemic heart disease b. Carcinoma esophagus c. Achalasia cardia d. Hiatus hernia

Esophagus





19. Most useful investigation in sliding hernia in female:

20. For hiatal hernia, investigation of choice is: a. Barium meal follow through (DPG 2006, DPG 2005) b. Barium meal upper GI c. Barium meal upper GI in Trendelenburg position d. Barium meal double contrast

28. The gold standard for diagnosis of gastroesophageal reflux disease (GERD) is: (Recent Question 2017, MHCET 2016,  JIPMER 2014, Orissa 2011, COMEDK 2008, 2007,  PGI June 1998) a. Barium swallow b. Endoscopy c. 24-hours pH monitoring d. Esophageal manometry



21. Rossetti modification of Nissen’s fundoplication means:  (MHSSMCET 2006) a. Excludes the stomach wall in the wrap b. Include only the posterior stomach wall in the wrap c. Include only the anterior stomach wall in the wrap d. Include both the anterior and the posterior stomach wall in the wrap

29. A 35-year-old lady presented with dysphagia, nocturnal asthma and weight loss for 6 years. The most probable diagnosis is: (COMEDK 2010) a. Achalasia cardia b. Lye stricture of esophagus c. Gastroesophageal reflux disease d. Cancer esophagus



30. Which of the following is the earliest indicator of pathological gastroesophageal reflux in infants (GERD)? (All India 2011) a. Respiratory symptoms b. Postprandial regurgitation c. Upper GI bleed d. Stricture esophagus



31. Best test to diagnose gastroesophageal reflux disease and quantify acid output is: (AIIMS May 2011, Nov 2008) a. Esophagogram b. Endoscopy c. Manometry d. 24-hours pH monitoring 32. Most important pathophysiological cause of GERD is: a. Hiatus hernia (AIIMS May 2012) b. Transient LES relaxation c. LES hypotension d. Inadequate esophageal clearance 33. In GERD, what demonstrates the best anatomical picture? (MHSSMCET 2007) a. Barium swallow b. 24-hours pH-monitoring c. Endoscopy d. Manometry

a. Fluoroscopy c. Palpation method

b. Barium meal d. Ultrasound



22. Fundoplication is used in treatment of: (DNB 2012, MHPGMCET 2002) a. Hiatus hernia b. Achalasia cardia c. CHPS d. CA esophagus



23. In Nissen’s Fundoplication wrapping is done: a. 1/3 b. 1/4 (MHSSMCET 2009) c. 3/4 d. 1/2



24. Retrocardiac lucency with air fluid level is seen in: a. Hiatus hernia (Recent Question 2013) b. Distal end esophageal obstruction c. Eventration of diaphragm d. None





25. What is the most probable diagnosis based on the given barium study image? (Recent Question 2016) a. Type I hiatus hernia b. Bochdalek hernia c. Morgagni hernia d. Type II hiatus hernia





34. Most common complication after Nissen’s fundoplication: (AIIMS GIS Dec 2011, Dec 2006) a. Esophageal injuries b. Stomach injuries c. Liver injuries d. Pneumothorax



35. All are true about antireflux surgeries except: a. Nissen’s is 360 degree complete wrap (JIPMER GIS 2011) b. Watson is 90 degree posterior c. Toupet is 270 degree posterior d. Dor is a partial fundoplication



36. Reflux esophagitis is prevented by: a. Long intra-abdominal esophagus b. Increased intra-abdominal pressure c. Right crus of diaphragm d. Increased intra-thoracic pressure



37. Which of the following mechanism can not prevent gastroesophageal reflux? (AIIMS Nov 98) a. Looping fibers of right crus of diaphragm b. Mucosal folds at gastroesophageal junction c. Circular muscle fibres of GE sphincter d. Angle made by the esophagus with stomach



38. Which one of the following drugs exacerbate reflux esophagitis? (COMEDK 2004) a. Chlorpropamide b. Metoclopramide c. Theophylline d. Cisapride

REFLUX ESOPHAGITIS



26. LES pressure is decreased by all except: (GB Pant 2011) a. Alcohol b. Protein c. Fat d. Peppermint 27. The lower esophageal sphincter tone (pressure) is increased by: (COMEDK 2005) a. Glucagon b. Gastrin c. Emptying of the stomach d. Chocolate

(PGI Dec 2001)

Gastrointestinal Surgery



(UPPG 2008)

Section 3



291

292

Surgery Essence

Gastrointestinal Surgery





39. The aim of preventing reflux esophagitis by repairing hiatus hernia is achieved by: (AIIMS 79, Rohtak 87) a. Bringing the stomach inferior to diaphragm b. Reconstitution of the angle of hill c. Repair of defect in diaphragm d. All of the above 40. Peptic esophagitis: (UPSC 2000) a. Is effectively demonstrated by barium swallow b. Is always associated with hiatus hernia c. Can be readily confirmed by esophagoscopy d. Is associated with the production of higher than normal amounts of gastric acid



41. Complications of reflux esophagitis: (MAHE 2005) a. Stricture b. Schatzki’s ring c. Barrett’s esophagus d. All of the above



42. Most common cause of esophagitis is: (AIIMS May 2009) a. Alcohol b. Smoking c. Spicy and hot food d. Esophageal reflux



43. What is the name of this fundoplication?

a. Nissen’s fundoplication c. Dor fundoplication



48. Amyl nitrite inhalation test is used to detect: (COMEDK 2005) a. CA esophagus b. Achalasia cardia c. Esophageal diverticulum d. Tracheoesophageal fistula



49. Increasing difficulty in swallowing both for solids and liquids in a woman with bird’s beak appearance in X-ray seen in: (PGI June 2008) a. Achalasia cardia b. Carcinoma c. Reflux esophagitis d. Barrett’s esophagus e. Esophagitis



50. A 30-year-old women comes with dysphagia for both solid and liquids and barium swallow shows parrot beak appearance. On esophageal manometry, LES pressure is increased. Management includes: (PGI June 2008) a. Nitrates b. Ca2+ channel blockers c. Botulinum toxin d. Myotomy



51. Hellers operation is done for:  (Recent Question 2018, Recent Question 2016, 2014,  DNB 2013, 2012) a. Achalasia cardia b. Hiatus hernia c. Diaphragmatic d. Reflux esophagitis



52. The defect in achalasia cardia is present in: a. Myenteric plexus of Auerbach b. Meissner’s plexus c. Kesselbach’s plexus d. Mesenteric plexus



53. What is the diagnosis of barium esophagogram? (APPG 2015) a. Achalasia cardia b. Hiatus hernia c. Diffuse esophageal spasm d. Reflux esophagitis



54. In achalasia cardia true is: (PGI June 2000) a. Pressure at distal end increased with no peristalsis b. Low pressure at LES with no peristalsis c. Pressure >50mm Hg with peristalsis d. Pressure at the distal end increased with normal relaxation 55. A young patient presents with history of dysphagia more to liquid than solids. The first investigation you will do is: (AIIMS June 2003) a. Barium swallow b. Esophagoscopy c. Ultrasound of the chest d. CT scan of the chest

(MAHE 2008)

b. Watson fundoplication d. Toupet fundoplication

Section 3

ACHALASIA CARDIA

44. All are true about achalasia except: (GB Pant 2011, JIPMER GIS 2011) a. It predisposes to malignancy b. Body peristalsis is normal c. LES pressure is increased d. Dilatation of proximal segment



45. All  a. b. c. d.





of the following are true about achalasia cardia except: (Recent Question 2017) Achalasia means absence of relaxation Heller’s myotomy is the treatment choice It is premalignant condition Caused by selective loss of stimulatory myenteric neurons

46. All are manometric features of achalasia except: a. High LES pressure  (JIPMER GIS 2011) b. Decreased LES relaxation c. Segmental body peristalsis d. Manometry helps in diagnosis 47. A female patient has dysphagia, intermittent epigastric pain. On endoscopy, esophagus was dilated above and narrow at the bottom. Treatment is: (AIIMS May 2012) a. PPI b. Esophagectomy c. Dilatation d. Heller’s cardiomyotomy





56. Treatment for achalasia associated with high rate of recurrence: (All India 2002) a. Pneumatic dilatation b. Laparoscopic myotomy c. Open surgical myotomy d. Botulinum toxin

Esophagus 57. About achalasia cardia all are correct except: a. Mostly in women (APPG 84, Kerala 86, JIPMER 98) b. Dilated esophagus narrowing to a point c. Heller’s operation treatment of choice d. Not a premalignant condition



58. All are true of achalasia cardia except: (JIPMER 90) a. Dysphagia b. Aspiration pneumonitis c. Mecholyl test is hyposensitive d. X-ray finding of dilated esophagus with a narrow end



65. This characteristic appearance is seen on barium swallow in:  (Recent Question 2016) a. Achalasia cardia b. Nutcrackers esophagus c. Diffuse esophageal spasm d. Hypertensive LES



66. Motility in the upper third of the esophagus is decreased in all except: a. Pseudo-bulbar palsy b. Chagas disease c. Myasthenia gravis d. Scleroderma e. Cricopharyngeal carcinoma



67. Most common motility disorder leading to dysphagia: (JIPMER 2010) a. Nut cracker esophagus b. Esophageal web c. Diffuse esophageal spasm d. Achalasia cardia

59. Following are radiological evidence of Achalasia cardia except: (Karnataka 98) a. Smooth narrowing of esophagus b. Dilated tortuous esophagus c. Absence of air in the fundus d. Exaggerated peristalsis



60. Bird’s beak appearance is seen in:  (Recent Question 2017, Recent Question 2015,  J and K 2001) a. Volvulus b. Intussusception c. Achalasia d. Ulcerative colitis 61. This characteristic appearance is seen on barium swallow in: (Recent Question 2016) a. Carcinoma esophagus b. Achalasia cardia c. Nutcrackers esophagus d. Diffuse esophageal spasm



62. Achalasia cardia is characterized by all except: (UPPG 2009) a. Most common in women b. Dysphagia is most common symptom c. Premalignant condition d. Parrot beak’s appearance



63. True about achalasia cardia is all except: (MHSSMCET 2009) a. On repeated Botox injection, recurrence rate increase b. More common in young women c. Surgery is treatment of choice d. Not a premalignant condition



68. Best investigation for Zenker’s diverticulum is: (Recent Question 2014, AIIMS Nov 2011, AIIMS GIS Dec 2010) a. Barium swallow b. Endoscopy c. CECT d. EUS



69. An elderly male present with history of dysphagia, regurgitation, foul breath and cough. Bilateral lung creps are noted on examination. The most likely diagnosis is:  (All India 2012, AIIMS GIS Dec 2010) a. Schatzki’s ring b. Zenker’s diverticulum c. Corkscrew esophagus d. Plummer-Vinson syndrome



70. True statement about Zenker’s diverticulum: a. Congenital (NEET Pattern, GB Pant 2011) b. Feeling of obstruction in esophagus c. Traction diverticulum d. Not present with recurrent aspiration pneumonitis



71. All of the following statements about Zenker’s diverticulum are true except: (All India 2009) a. Acquired diverticulum b. Lateral X-rays on barium swallow are often diagnostic c. False diverticulum d. Out pouching of the anterior pharyngeal wall just above the cricopharyngeus muscle



72. A 50-year-old male Raju, presents with occasional dysphagia for solids, regurgitation of food and foul smelling breath. Probable diagnosis is:  (Recent Question 2016, AIIMS June 99) a. Achalasia cardia b. Zenker’s diverticulum c. CA esophagus d. Diabetic gastroparesis

ESOPHAGEAL MOTILITY DISORDERS

64. Corkscrew esophagus is seen in which of the following conditions? (Recent Question 2017, Recent Question 2015,  Bihar PG 2014, NEET 2013, DNB 2008, 2005, 2001) a. Carcinoma esophagus b. Scleroderma c. Achalasia cardia d. Diffuse esophageal spasm

Gastrointestinal Surgery

ZENKER’S DIVERTICULUM

Section 3



293

294

Surgery Essence

Gastrointestinal Surgery



73. A 60-year-old diabetic male patient presented to the OPD with halitosis and dysphagia. Barium swallow was done. What is the diagnosis based on the given findings? a. Achalasia cardia b. Carcinoma esophagus c. Zenker’s diverticulum d. Leiomyoma

82. Which part of esophagus is mainly affected in scleroderma? (MHPGMCET 2003) a. Upper third b. Middle third c. Lower third d. All the above



83. Connective tissue disorder which is associated with gastroesophageal reflux is: (PGI Dec 99) a. SLE b. Scleroderma c. Behcet’s syndrome d. Dermatomyositis

PLUMMER–VINSON SYNDROME (AIIMS 97)



84. Not true about Plummer-Vinson syndrome is: a. Occurs in elderly males b. Postcricoid webs c. Predispose to hypopharynx malignancy d. Koilonychia

74. Which of the following is true about Zenker’s diverticulum? a. It is asymptomatic (PGI Dec 2007) b. Occurs in the mid-esophagus c. Treatment is simple excision d. It occurs in children

85. In Plummer–Vinson syndrome, obstruction is due to: (DPG 2006) a. Esophageal dysmotility b. Esophageal stenosis c. Postcricoid webs d. None of the above





75. Commonest complication of Zenker’s diverticulum is: (AIIMS Nov 96) a. Dysphonia b. Gastroesophageal reflux c. Lung abscess d. Perforation

86. All are features of Plummer-Vinson syndrome except: (COMEDK 2008) a. Esophageal web b. Iron deficiency c. Achalasia cardia d. Dysphagia





76. The pharyngeal diverticulum is a protrusion of mucosa between: (UPSC 2000) a. The two parts of inferior constrictor muscle of the pharynx b. The two parts of middle constrictor muscle of the pharynx c. The two parts of the superior constrictor muscle of the pharynx d. Cricopharyngeal and posterior part of suprahyoid membrane

87. Not a feature of Plummer-Vinson syndrome: (Punjab 2008) a. Web present in lower part of esophagus b. Koilonychia c. Premalignant d. Common in edentulous females



77. Dohlman’s procedure is used in: (Recent Question 2013) a. Rectal prolapsed b. Esophageal achalasia c. CA esophagus d. Zenker’s diverticulum



78. Dohlman surgery in Zenker’s diverticulum is:  (Recent Question 2019) a. Endoscopic stapling of septum b. Endoscopic suturing of pouch c. Resection of pouch d. Laser excision

ESOPHAGEAL DIVERTICULA

Section 3



BARRETT’S ESOPHAGUS (PGI Dec 2007)



88. True about Barrett’s esophagus:  a. Long esophageal segment involved b. Metaplasia c. Peptic ulcer d. Para esophageal hernia e. Leads to adenocarcinoma



89. All of the following are correct about Barrett’s esophagus except: (Recent Question 2019) a. Predisposes to adenocarcinoma b. Columnar to squamous metaplasia c. Associated with GERD d. Acquired condition

90. A 55 years old patient presented with dysphagia. Identify the diagnosis from upper GI biopsy of esophagus showed in the following picture:  (AIIMS May 2017)

79. Which of the following is a true diverticulum of esophagus?  (AIIMS November 2016) a. Zenker’s diverticulum b. Meckel’s diverticulum
 c. Epiphrenic diverticulum d. Parabronchial diverticulum

SCLERODERMA

80. Not a component of POEMS syndrome: (JIPMER 2010) a. Polyneuropathy b. Esophageal atresia c. Endocrinopathy d. Multiple myeloma



81. Acronym ‘POEMS’ stand for: (PGI May 2011) a. Esophageal dysmotility b. Polyneuropathy c. Endocrinopathy d. M-protein e. Scleroderma

a. Squamous cell carcinoma b. Eosinophilic esophagitis c. Barrett’s esophagus d. Adenocarcinoma

Esophagus



92. Barrett’s esophagus is: (All India 2002, AIIMS June 93) a. Lower esophagus lined by columnar epithelium b. Upper esophagus lined by columnar epithelium c. Lower esophagus lined by ciliated epithelium d. Lower esophagus lined by pseudostratified epithelium



93. Barrett’s esophagus is diagnosed by: (AIIMS May 2012, Nov 2007, DNB 2008) a. Squamous metaplasia b. Intestinal metaplasia c. Squamous dysplasia d. Intestinal dysplasia



94. Barrett’s esophagus can lead to: (AIIMS June 98) a. Stricture b. Reflux esophagitis c. Peptic ulcer d. Achalasia



95. True regarding Barrett’s esophagus is: (AIIMS Nov 95) a. Benign course b. Premalignant condition c. Squamous metaplasia of lower esophagus d. Medical treatment is not useful



99. Premalignant lesion of carcinoma esophagus includes: (PGI Nov 2011, June 2007) a. Tylosis b. Plummer-Vinson syndrome c. Barrett’s esophagus d. Achalasia cardia e. Scleroderma

100. Squamous cell carcinoma of esophagus is caused by: (MHPGMCET 2009) a. Tobacco/Alcohol b. Alkalies c. GERD d. All of the above 101. Which is not a predisposing factor for carcinoma esophagus? a. Esophageal diverticula (Orissa 2011) b. Plummer-Vinson syndrome c. Mediastinal fibrosis d. Caustic ingestion 102. The adenocarcinoma of esophagus develops in:  (Recent Question 2017, Bihar PG 2014,  COMEDK 2014, All India 2002, 98) a. Barrett’s esophagus b. Long standing achalasia c. Corrosive stricture d. Alcohol abuse 103. Esophageal carcinoma is not predisposed by: (PGI June 99, PGI Dec 95) a. Achalasia b. Scleroderma c. Corrosive intake d. Barrett’s esophagus 104. Not a predisposing factor for carcinoma esophagus: (AIIMS May 2009) a. Diverticula b. Human papilloma virus c. Mediastinal fibrosis d. Caustic ingestion 105. Risk factor for adenocarcinoma of esophagus: (KGMC 2011) a. Barrett’s esophagus b. Corrosive injury c. Achalasia cardia d. All of the above

CA ESOPHAGUS CLINICAL FEATURES, DIAGNOSIS AND TREATMENT 106. Which of the following is best indicator of survival in CA esophagus? (AIIMS GIS May 2008) a. TNM stage b. Resection margin c. Histology and location d. Size of tumor 107. Best prognosis in CA esophagus: (AIIMS GIS Dec 2010) a. Polypoidal b. Fungating c. Ulcerative d. Infiltrative

CARCINOMA ESOPHAGUS PREDISPOSING FACTORS

108. Sievert classification is for: a. Esophageal cancer b. Stomach cancer c. GE junction tumors d. CA pancreas

(KGMC 2011)



97. All are the predisposing factors for carcinoma esophagus except: (COMEDK 2011) a. Achalasia b. Paterson Brown Kelly Syndrome c. Zenker’s diverticulum d. Ectodermal dysplasia

109. Characteristic features of LN involvement on EUS in CA esophagus are all except: (AIIMS GIS Dec 2011, May 2008) a. Round contour b. Sharp border c. Hyperechogenic d. Size >1 cm



98. All are risk factors for epidermoid carcinoma except:  (ILBS 2011) a. Achalasia b. Barrett’s esophagus c. Nitrosamines d. Corrosives

110. Most commonly used chemotherapy regimen used in CA esophagus: (AIIMS GIS May 2008) a. 5-FU + Cisplatin b. Cisplatin + Vinblastine c. Cisplatin + Paclitaxel d. Cisplatin + Epirubicin

Gastrointestinal Surgery

96. What is the most probable diagnosis on the basis of given endoscopy image? (Recent Question 2016) a. GAVE b. Schatzki ring c. Barrett’s esophagus d. Esophageal varices



Section 3

91. A chronic alcoholic presents with regurgitation and retrosternmal pain. Endoscopic biopsy confirms Barretti’s esophagus. What is the most appropriate management in this case? (JIPMER 2013) a. Endoscopic biopsy every 2 years b. PPI c. H. pylori treatment d. Balloon dilatation

295

296

Surgery Essence

Gastrointestinal Surgery

111. Which is not used in palliation in CA esophagus? (AIIMS GIS May 2008) a. EMR b. Photodynamic therapy c. Laser therapy d. Self-expanding stents 112. T-staging of esophagus is best done by: (AIIMS GIS Dec 2011, DNB 2002) a. EUS b. CT c. MRI d. PET 113. Commonest site of esophagus carcinoma is: (UPPG 2008) a. Upper 2/3rd b. Middle 1/3rd c. Lower 1/3rd d. Crico-esophageal junction

121. Esophageal carcinoma is adequately assessed by: a. Barium swallow  (PGI SS Dec 2009) b. Barium swallow + endoscopy c. Endoscopy d. USG 122. Most common complication of placing stent in CA esophagus: (AIIMS GIS May 2011) a. Migration b. Chest pain c. Perforation d. Bleeding 123. What is the diagnosis based on the given findings?

114. Barium esophagogram findings in carcinoma esophagus are all except: (UPPG 2009) a. Rat-tail deformity b. Pencil tip appearance c. Apple-core appearance d. Filling defect 115. A 60-year-old chronic smoker presented with progressive dysphagia. Barium swallow was done, what is the name of sign? (Recent Question 2017) a. Bird beak appearance b. Pencil tip appearance c. Apple core appearance d. Rat tail appearance

a. Mallory Weiss syndrome b. Carcinoma esophagus c. Carcinoma stomach d. Esophageal varices 124. Lymph node metastasis in CA esophagus is best detected by: (JIPMER SS 2016, AIIMS GIS May 2011) a. PET b. EUS c. CT d. Thoracoscopy + laparoscopy

Section 3

125. Best palliation in CA esophagus: a. Surgical bypass b. Surgical placement of MB tube c. Endoprosthesis d. Feeding jejunostomy

(GB Pant 2011)

116. MC site of CA esophagus is:  (MCI Nov 2017, Recent Question 2015, AIIMS Feb 2007) a. Middle 1/3rd b. Upper 1/3rd c. Lower 1/3rd d. Lower end of esophagus

126. Most common site for squamous cell CA esophagus is: (All India 2001) a. Upper third b. Middle third c. Lower third d. Gastro-esophageal junction

117. The commonest site of carcinoma esophagus in India is: (AIIMS Nov 2003) a. Upper 1/3rd b. Middle 1/3rd c. Lower 1/3rd d. GE junction

127. True about CA esophagus: (PGI Dec 2003) a. MC in middle 1/3rd b. Adenocarcinoma is common variety c. Carcinoma develops at the achalasia segment d. Smoking is a risk factor e. Endoscopy is the investigation of choice

118. A patient presents with dysphagia of 4 weeks duration. Now he is able to swallow liquid food only. Which of the following is the one investigation to be done? (DPG 2009 Feb) a. Barium studies b. Upper GI endoscopy c. CT Scan d. Esophageal manometry 119. Treatment of choice for CA esophagus: (PGI SS Dec 2009) a. Esophagectomy b. External radiotherapy c. Internal radiotherapy d. Chemotherapy 120. Stage of CA esophagus is best decided by: (PGI SS June 2009) a. Depth of tumor b. Size of tumor c. Histopathological grade d. Age of the patient

128. Treatment of advanced esophageal cancer is: (DNB 2006) a. Chemoradiation only b. Curative en-bloc resection c. Chemoradiation followed by curative enbloc resection d. Chemoradiation followed by palliative enbloc resection 129. Regarding esophagus malignancy-operation all are true except: (MHSSMCET 2009) a. Thoracoscopic assisted surgery b. Radiotherapy c. Chemotherapy + surgery d. Transhiatal esophagectomy

Esophagus

131. True about carcinoma esophagus is: (Kerala 94) a. Most common site is lower end b. Both adeno and squamous cell carcinoma occur c. Commonest histology is adenocarcinoma d. More common in females 132. Which is the most reliable diagnostic method for staging the esophageal cancer? (UPSC 2006) a. MRI b. Endoscopic ultrasound c. CT scan d. Thoracoscopy 133. Early stage of carcinoma esophagus is diagnosed by: a. Barium meal  (UPPG 2008) b. Transesophageal USG c. MRI d. Endoscopy 134. False statements about carcinoma esophagus are all of the following except: (MCI March 2009) a. Most common in lower third b. Histologically, adenocarcinoma only c. Unrelated to tobacco chewing d. It is more common in females

136. Treatment of SCC of esophagus: (GB PANT 2011) a. Cisplatin b. Etoposide c. Adriamycin d. Bleomycin

142. Ivor Lewis operation is the treatment of choice for cancer involving .............. esophagus: (MHSSMCET 2009) a. Upper 1/3rd b. Middle 1/3rd c. Lower 1/3rd d. Entire esophagus 143. Which of the following surgical approach was first described by Orringer for the management of carcinoma esophagus?  (J amd K 2005) a. Transhiatal b. Thoracoscopic c. Left thoracoabdominal d. Right thoracoabdominal 144. Commonest cause for mortality in Ivor Lewis operations: a. Pulmonary atelectasis (AIIMS Nov 98) b. Anastomotic leak c. Thoracic duct fistula d. Sub diaphragmatic collection 145. Which of the following operations was first described by Orringer? (COMEDK 2006) a. Enbloc esophagectomy b. Transhiatal esophagectomy c. Thoracoscopic esophagectomy d. Transthoracic esophagectomy 146. The ideal replacement for the esophagus after esophagectomy is: (COMEDK 2010) a. Stomach b. Jejunum c. Colon d. Synthetic stent 147. After esophagectomy, stomach tube is based on supply form: (MHSSMCET 2006) a. Right gastroepiploic artery b. Right gastric artery c. Left gastroepiploic artery d. Left gastric artery

LEIOMYOMA

137. In Esophageal cancer prognosis is best determined by: (AIIMS May 2015) a. Cellular differentiation b. Age of patient c. T stage d. Length of involvement

148. Commonest benign tumor of the esophagus: (JIPMER 2014, DPG 2009 Feb) a. Leiomyoma b. Papilloma c. Adenoma d. Hemangioma

138. True regarding esophageal squamous cell carcinoma is/are:  (PGI May 2018) a. Barrett’s esophagus is a risk factor b. Common in middle third of esophagus c. Stomach, jejunum or colon can be used for replacement after surgical removal d. Chemoradiation has little role in inoperable patients e. Staging is done by CECT

149. This characteristic appearance is seen on barium swallow in:  (Recent Question 2017) a. Achalasia cardia b. Carcinoma esophagus c. Leiomyoma d. Diffuse esophageal spasm

ESOPHAGECTOMY 139. Conduit in gastric pull up is based on: (AIIMS GIS Dec 2011, May 2008) a. Right gastric and right gastroepiploic artery b. Right gastric and left gastroepiploic artery c. Left gastric and right gastroepiploic artery d. Left gastric and left gastroepiploic artery 140. Which is the best substitute for esophagus? (MHSSMCET 2005, MP 99, All India 96, PGI Dec 95) a. Stomach b. Jejunum c. Left sided colon d. Right sided colon

Gastrointestinal Surgery

135. Which of the following should never be carried out as a treatment for carcinoma esophagus? (MHPGMCET 2008) a. Gastrostomy for palliation b. Radiotherapy alone c. Radical esophagectomy d. Radiotherapy and chemotherapy

141. First successful esophagectomy was done by: (AIIMS GIS Dec 2011) a. Miculikz b. Kaplan c. Torek d. Orringer

Section 3

130. Transhiatal esophagectomy was planned for adenocarcinoma of lower end of esophagus. The approach would be in the following order: (AIIMS Nov 2007) a. Abdomen-Neck b. Abdomen-Thorax-Neck c. Neck-Thorax-Abdomen d. Abdomen-Thorax

297

298

Surgery Essence

Gastrointestinal Surgery

150. Endoscopic treatment of leiomyoma of esophagus is contraindicated due to: (JIPMER GIS 2011) a. Infection b. Chances of dissemination c. Perforation d. Perforation and dissemination

DYSPHAGIA 151. Intermittent dysphagia is caused by: (PGI June 2004) a. Stricture b. Reflux esophagitis c. Achalasia cardia d. Pharyngeal diverticulum e. Diffuse esophageal spasm 152. A 60-year-old patient presenting with dysphagia of 6 weeks duration with solid foods, now can swallow only liquids. Investigation done for diagnosis: (PGI Dec 2003) a. CXR b. Ba swallow c. Endoscopy d. USG e. C.T. Scan 153. Investigation of choice for dysphagia for solids: a. Barium swallow b. Endoscopy (PGI Dec 2000) c. X-ray chest d. C.T. Scan 154. Odynophagia occurs in: a. Achalasia c. Monilial esophagitis

(AIIMS GIS Dec 2011, PGI June 96) b. Herpes esophagitis d. Barrett’s esophagus

155. A 40-year-old female patient presented with dysphagia to both liquids and solids and regurgitation for 3 months. The dysphagia was non-progressive. What is the most likely diagnosis? (AIIMS May 2006) a. Carcinoma of the esophagus b. Lower esophageal mucosal ring c. Achalasia cardia d. Reflux esophagitis with esophageal stricture

DYSPHAGIA LUSORIA 156. Dysphagia lusoria is caused by: (Karnataka 2013, PGI SS June 2001) a. Abnormal elongation of arch of aorta b. Aneurysm of arch of aorta c. Esophageal web d. Esophageal diverticula

Section 3

157. Dysphagia lusoria is due to:  (Recent Question 2016, 2014, AIIMS Nov 2003) a. Esophageal diverticulum b. Aneurysm of aorta c. Esophageal web d. Compression by aberrant blood vessel 158. All are true about dysphagia lusoria except: (DPG 2008) a. Right aortic arch b. Vascular ring c. Due to aberrant subclavian artery causing pressure on esophagus d. Acquired in later life

ESOPHAGEAL PERFORATION AND INJURY 159. Mackler’s triad includes: a. Vomiting c. Lower thoracic pain e. Pleural effusion

(PGI Nov 2009) b. Subcutaneous emphysema d. Peripheral cyanosis

160. True about Boerhaave’s syndrome: a. MC at lower 1/3rd b. Hematemesis is early symptoms c. Acute chest pain d. Surgically treated e. Operation is done after 24 hours

(PGI Nov 2009)

161. True about esophageal injury: (PGI June 2009) a. Barium swallow is diagnostic b. Treatment is primary repair c. MC after penetrating injury d. Mortality increased if repair after 24 hours 162. Boerhaave’s syndrome, true is: a. Iatrogenic b. Silent manifestation c. Present with acute chest pain d. Treatment is surgical

(PGI May 2005)

163. Which is most common site for iatrogenic esophageal perforation? (Recent Question 2017, Recent Question 2014,  DNB 2013, 2012, AIIMS Nov 97) a. Abdominal portion b. Cervical portion c. Above arch of aorta d. Below arch of aorta 164. Commonest cause of esophageal perforation is: a. Acid ingestion b. Hyperemesis c. Instrumentation d. Carcinoma infiltrating 165. A 40-year-old female presents with perforation of distal third of esophagus. Best form of management is: a. Antibiotics and drainage (JIPMER GIS 2011) b. Tubal drainage c. Primary resection and anastomosis d. Esophagectomy with bringing proximal end as fistula in neck 166. In majority of patients with esophageal leaks in thoracic cavity of less than 12 hours duration, the treatment of choice is: (UPSC 97) a. Primary closure, drainage and antibiotics b. Early esophagogastrostomy c. Exclusion and diversion of continuity d. Total esophagectomy and gastric pull up 167. Best flap for esophagus repair: a. Colon b. Stomach c. Jejunum d. Latissimus dorsi 168. In esophageal perforation all are seen except: (UPPG 2000) a. Pain b. Bradycardia c. Fever d. Hypotension 169. Boerhaave’s syndrome is due to: a. b. c. d.

(Recent Question 2016, DNB 2012, UPPG 2007) Drug induced esophagus perforation Corrosive injury Spontaneous perforation Gastroesophageal reflux disease

170. When a ‘spontaneous perforation’ of the esophagus occurs as a result of severe barotrauma while a person vomits against a closed glottis, the condition is known as: (UPSC 2008) a. Mallory-Weiss syndrome b. Plummer-Vinson syndrome c. Kartagener syndrome d. Boerhaave’s syndrome

Esophagus 180. What is the type of tracheoesophageal fistula (TEF) seen on barium swallow?

Section 3

171. Longitudinal tear of esophagus is seen in: (MHPGMCET 2002) a. Boerhaave’s syndrome b. Mallory-Weiss syndrome c. Nutcracker’s esophagus d. Lye stricture

299

172. In Boerhaave’s syndrome, perforation of esophagus is seen at? (MHSSMCET 2009) a. Upper anterior b. Lower posterior c. Upper posterior d. Lower anterior 173. Most common site a spontaneous rupture of esophagus is: a. Cricopharyngeal junction (Recent Question 2017, DNB 2009) b. Cardioesophageal junction c. Mid esophagus d. After the crossing of arch of aorta 174. Investigation of choice for esophageal rupture is? (DNB 2014) a. Dynamic MRI b. Rigid esophagoscopy c. Barium contrast swallow d. Water soluble low molecular weight contrast swallow 175. Which of the following is true about Boerhaave’s syndrome?  (Recent Question 2017) a. May present with peritonitis b. Forceful vomiting against open glottis c. Upper third esophagus location d. Most patients are managed by conservative management

177. True about esophageal perforation: (PGI November 2017) a. Morbidity is mainly due to mediastinal infection b. Partial tear involving mucosa should be managed conservatively c. Simple exploration and end to end anastomosis is done as soon as possible in all cases d. Spontaneous perforation most commonly occur in mid thoracic esophagus e. Serum amylase levels may be high as a result of absorption of amylase from the pleural cavity

TRACHEOESOPHAGEAL FISTULA 178. Esophageal atresia is most commonly associated with: (KGMC 2011) a. Respiratory anomalies b. Anorectal malformations c. Genitourinary d. CVS 179. The most common type of tracheoesophageal fistula is:  (Recent Question 2017, Recent Question 2014,  MAHE 2008, 2007) a. Esophageal atresia without tracheoesophageal fistula b. Esophageal atresia with proximal tracheoesophageal fistula c. Esophageal atresia with distal tracheoesophageal fistula d. Esophageal atresia with proximal and distal tracheoesophageal fistula

Atresia with proximal TEF Atresia with distal TEF Atresia with both proximal and distal TEF TEF without esophageal atresia

181. Malignant tracheoesophageal fistula best treated with: (PGI May 2005) a. Radiotherapy b. Chemotherapy c. Stenting d. Tube e. Surgical correction 182. Treatment of malignant tracheoesophageal fistula includes: (PGI June 2007) a. Expandable metal stent b. Surgery with graft c. Gastrostomy tube d. Radiotherapy e. Chemotherapy 183. Which one of the following life - threatening congenital anomalies in the newborn presents with polyhydramnios, aspiration pneumonia, excessive salivation and difficulty in passing a nasogastric tube? (APPG 2016) a. Choanal atresia b. Tracheoesophageal fistula c. Diaphragmatic hernia d. Gastroschisis 184. Which of the following is associated with poor prognosis in tracheo-esophageal fistula in children? (Recent Question 2016) a. Weight Tumor ingrowth overgrowth (10%)> Stent migration (9%)> Severe reflux (8%) • The use of stents coated with silicone or polyurethane may prevent or delay tumor ingrowth and subsequent esophageal obstruction. • Coated stents have been used with good success (>90%) for the treatment of tracheoesophageal fistulaQ. • • • •

Tumor ingrowth may be addressed by insertion of another stent or by tumor ablation. Placement of stents through proximally located tumors, especially those near the cricopharyngeus, is often not well tolerated. Stents placed across GE junction have a greater tendency to migrate and may result in symptomatic acid refluxQ. Fixed diameter plastic endoluminal prostheses, associated with significant morbidity and mortality and low rate of dysphagia relief, have largely been abandonedQ.

123. Ans. b. Carcinoma esophagus (Ref: Sabiston 20/e p1028; Schwartz 10/e p1005; Bailey 27/e p1088) 124. Ans. b. EUS (Ref: Sabiston 20/e p1028, 1031; Schwartz 10/e p1003-1014; Bailey 27/e p1090; Shackelford 7/e p420-421)

Gastrointestinal Surgery

• A barium swallow is recommended for any patient presenting with dysphagia. • The esophagram gives an overview of anatomy and functionQ. • It is able to differentiate intraluminal from intramural lesions and to discriminate between intrinsic (from a mass protruding into the lumen) and extrinsic (from compression of a structures outside the esophagus) compression. • The classic finding of an apple-core lesion in patients with esophageal cancer is recognized easilyQ

308

Surgery Essence

Gastrointestinal Surgery

Carcinoma Esophagus • EUS is superior to CT or PET for assessment of both T and N stage. • Endoscopic ultrasound (EUS) is the best diagnostic tool available to assess the locoregional extent of disease (T and N Staging)Q • The depth of tumor penetration of the esophageal wall and the presence of lymph node involvement can be assessed with an ultrasound probe attached to the tip of a flexible endoscopeQ. • CT is generally considered to be less accurate in determining lymph node involvement than other modalities such as EUSQ. 125. Ans. c. Endoprosthesis 126. Ans. b. Middle third 127. Ans. a. MC in middle 1/3rd, b. Adenocarcinoma is common variety, c. Carcinoma develops at the achalasia segment, d. Smoking is a risk factor, e. Endoscopy is the investigation of choice 128. Ans. d. Chemoradiation followed by palliative enbloc resection 129. Ans. b. Radiotherapy 130. Ans. a. Abdomen-Neck (Ref: Sabiston 20/e p1035; Schwartz 10/e p1009-1012, 1011-1012; Shackelford 7/e p429-430)

Orringer Transhiatal Esophagectomy • Double incision: Midline laparotomy followed by cervical incisionQ • Cervical anastomosis is doneQ • MC procedure done for carcinoma esophagusQ 131. Ans. b. Both adeno and squamous cell carcinoma occur

132. Ans. b. Endoscopic ultrasound

133. Ans. b. Transesophageal USG

134. Ans. None

135. Ans. a. Gastrostomy for palliation • A gastric pull-up in the posterior mediastinal position has the best functional result, and every effort is made to preserve and use this successful combination. 136. Ans. a. Cisplatin

137. Ans. c. T stage

138. Ans. b. Common in middle third of esophagus, c. Stomach, jejunum or colon can be used for replacement after surgical removal, d. Chemoradiation has little role in inoperable patients

ESOPHAGECTOMY 139. Ans. a. Right gastric and right gastroepiploic artery (Ref: Sabiston 20/e p1035-1036; Schwartz 10/e p1009; Bailey 27/e p1092; Shackelford 7/e p518-520) 140. Ans. a. Stomach 141. Ans. c. Torek (Ref: Shackelford 7/e p416) • First successful esophagectomy was done by Torek. 142. Ans. b. Middle 1/3rd (Ref: Sabiston 20/e p1035-1036; Schwartz 10/e p1009; Bailey 27/e p1092; Shackelford 7/e p427-430)

Section 3

Tumor Margin for Curative Excision • In GI malignancies (stomachQ, small intestineQ, colonQ and proximal rectumQ), tumor margin for curative excision is 5cmQ except: − Esophagus: 10 cmQ − Distal rectum: 2 cmQ 143. Ans. a. Transhiatal 144. Ans. b. Anastomotic leak (Ref: Sabiston 20/e p1035-1036; Schwartz 10/e p1009; Shackelford 7/e p538-545) Anastomotic leak following Ivor Lewis esophagectomy is a feared complication that in the past was associated with a 50% mortality rate. The anastomosis is intrathoracic, leak cause severe mediastinitis. Complications of Esophagectomy • Anastomotic Leak (MC) • Anastomotic stricture • Pulmonary complications Q

• Recurrent laryngeal nerve palsy • Chylothorax

Esophagus

309

Anastomotic Leak after Esophagectomy

145. Ans. b. Transhiatal esophagectomy

Section 3

• Incidence of anastomotic leak is higher following cervical anastomosis (10-15%) than intrathoracic anastomosis (5-10%). • Although leak is more common following cervical anastomosis, it is rarely life-threateningQ. • Anastomotic leak following Ivor Lewis esophagectomy is a feared complication that in the past was associated with a 50% mortality rateQ. • Confirmation is usually possible by Gastrografin swallowQ or instillation of contrast through the nasogastric tube. • Immediate intervention is required, and attempts at direct repair with muscle flap reinforcement and wide drainage are often successfulQ. • Patients who are unstable or severely ill should be diverted with a spit fistula, and either excluded at the hiatus, or have the conduit closed and returned to the abdomenQ. • Pulmonary complications are the MC cause of postoperative morbidity and mortality in transhiatal esophagectomyQ 146. Ans. a. Stomach

147. Ans. a. Right gastroepiploic artery, b. Right gastric artery

LEIOMYOMA 148. Ans. a. Leiomyoma (Ref: Sabiston 20/e p1032; Schwartz 10/e p1216; Bailey 27/e p1084; Shackelford 7/e p465-469) 149. Ans. c. Leiomyoma (Ref: Sabiston 20/e p1032; Schwartz 10/e p1017; Bailey 27/e p1085) • Barium swallow is IOC for leiomyoma (classical, smooth, contoured, punched-out defect)Q 150. Ans. c. Perforation

DYSPHAGIA 151. Ans. b. Reflux esophagitis, d. Pharyngeal diverticulum, e. Diffuse esophageal spasm (Ref: Sabiston 20/e p1049; Schwartz 10/e p625) Dysphagia Progressive Achalasia SclerodermaQ Peptic strictureQ CarcinomaQ

• Lower esophageal ring (Schatzki’s ring)Q • Diffuse esophageal spasmQ • Zenker’s diverticulumQ

152. Ans. a. CXR, b. Ba swallow, c. Endoscopy, d. USG, e. C.T. Scan The clinical history is suggestive of carcinoma esophagus. All of the given investigations are used for the complete work-up. 153. Ans. b. Endoscopy 154. Ans. b. Herpes esophagitis, c. Monilial esophagitis (Ref: Harrison 19/e p1901, 1909; Bailey 27/e p1103) If odynophagia is present, candidial (monilial) or herpes esophagitis or pill induced esophagitis should be suspected.

Odynophagia • Odynophagia means painful swallowing seen in inflammatory lesions of food passage (i.e. oral cavity, pharynx and esophagus) • Causes: − Candidial (monilial) esophagitisQ − Herpes esophagitisQ − Pill induced esophagitisQ 155. Ans. c. Achalasia cardia Presence of dysphagia to both solids and liquids suggests the diagnosis of achalasia.

DYSPHAGIA LUSORIA 156. Ans. a. Abnormal elongation of arch of aorta (Ref: Sabiston 20/e p1610; Bailey 27/e p1105) Causes of Dysphagia Lusoria • • • •

Abnormal right subclavian arteryQ (MC) Right aortic archQ Double aortic archQ Abnormal innominate arteryQ

• Vascular ring (constriction) formed by a PDA or a ligamentum arteriosum and the pulmonary artery or aortic archQ.

157. Ans. d. Compression by aberrant blood vessel (Ref: Sabiston 20/e p1581; Bailey 27/e p1105)

Gastrointestinal Surgery

• • • •

Intermittent

Q

310

Surgery Essence 158. Ans. d. Acquired in later life (Ref: Sabiston 20/e p1581) Dysphagia lusoria is a disorder of swallowing caused due to vascular anomaliesQ (developmental or congenital abnormalities).

Gastrointestinal Surgery

ESOPHAGEAL PERFORATION AND INJURY 159. Ans. a. Vomiting, b. Subcutaneous emphysema, c. Lower thoracic pain (Ref: Sabiston 20/e p1025-1026; Schwartz 10/e p1018; Bailey 27/e p1073; Shackelford 7/e p478-484) 160. Ans. a. MC at lower 1/3rd, c. Acute chest pain, d. Surgically treated 161. Ans. a. Barium swallow is diagnostic, b. Treatment is primary repair, c. MC after penetrating injury, d. Mortality increased if repaired after 24 hours 162. Ans. c. Present with acute chest pain, d. Treatment is surgical 163. Ans. b. Cervical portion (Ref: Sabiston 20/e p1025-1026; Schwartz 10/e p1018; Bailey 27/e p1073; Shackelford 7/e p478-479) Esophageal Perforation Iatrogenic

Spontaneous

• Most common typeQ • Caused by endoscopyQ • MC site is cervical esophagus (cricopharyngeal area)Q

• Esophageal rupture after vomiting • MC site: left posterolateral side of the distal esophagusQ

164. Ans. c. Instrumentation 165. Ans. d. Esophagectomy with bringing proximal end as fistula in neck 166. Ans. a. Primary closure, drainage and antibiotics

167. Ans. b. Stomach (Ref: Shackelford 7/e p483)

Operative Repair of Esophageal Perforation • The principles of repair comprise a clear exposure of the perforation and debridement of devitalized tissue, followed by a primary closure. • Following debridement of devitalized tissue, primary mucosal repair should then be performed with interrupted, absorbable suture, taking care to minimize esophageal stricturing while obtaining adequate suture purchase on vital tissue. • The muscular layer is then reapproximated with an interrupted or running sutureQ. • Subsequent coverage with a vascular pedicle, such as an intercostal muscle flap, and pleural, pericardial, or omental pedicle allows further buttressing of a repair and is recommended whenever feasibleQ.  • Gastric fundus may also be suitable tissue reinforcement especially for the distal perforationQ. • Depending on the site of perforation, any of the antireflux procedures (Belsey Mark IV, Nissen, Dor, Toupet) may be used to buttress the repair. 168. Ans. b. Bradycardia

169. Ans. c. Spontaneous perforation

170. Ans. d. Boerhaave’s syndrome

171. Ans. a. Boerhaave’s syndrome

172. Ans. b. Lower posterior

173. Ans. b. Cardioesophageal junction

174. Ans. d. Water soluble low molecular weight contrast swallow 175. Ans. a. May present with peritonitis (Ref: Sabiston 20/e p1025; Schwartz 10/e p1018; Bailey 27/e p1072)

Section 3

176. Ans. d. Hypaque (Ref: Diseases of the Esophagus By G. Vantrappen, J. Hellemans (2012)/p135) “Iodinated Contrast Materials Water-soluble iodinated contrast media can be used to visualize the upper gastrointestinal tract. These contrast materials are considered safer than barium sulfate in patients with esophageal perforation or communication between esophagus and airways and when there is a risk of aspiration into the bronchi. They are also used for the examination of the esophagus in the early postoperative period following esophageal surgery or correction of hiatal hernia. The demonstartion of leak is a contraindication for the withdrawal of the postoperative gastric tube. The most commonly used iodinated water-soluble contrast media are Gastrograffin and Hypaque.”- Diseases of the Esophagus By G. Vantrappen, J. Hellemans (2012)/p135 177. Ans. a. Morbidity is mainly due to mediastinal infection, b. Partial tear involving mucosa should be managed conservatively, e. Serum amylase levels may be high as a result of absorption of amylase from the pleural cavity

TRACHEOESOPHAGEAL FISTULA 178. Ans. d. CVS (Ref: Sabiston 20/e p1866-1868; Schwartz 10/e p608-609; Bailey 27/e p133; Shackelford 7/e p509-515) 179. Ans. c. Esophageal atresia with distal tracheoesophageal fistula

Esophagus

311

180. Ans. d. TEF without esophageal atresia (Ref: Sabiston 20/e p1867; Schwartz 10/e p1609; Bailey 27/e p133) 181. Ans. c. Stenting 182. Ans. a. Expandable metal stent, b. Surgery with graft, c. Gastrostomy tube (Ref: CSDT 11/e p1314; Bailey 27/e p1095)

Malignant Tracheoesophageal Fistula • • • •

It is a sign of incurable esophageal carcinomaQ Some have advocated surgical bypass and esophageal exclusion, but this is a major procedure Self expanding metallic stent is the best treatmentQ Semi-rigid prosthetic tubings may be used (Gastrostomy tube)Q

Section 3

Type of tracheoesophageal fistula (TEF) seen on barium swallow is H type (TEF without esophageal atresia).

183. Ans. b. Tracheoesophageal fistula 184. Ans. b. Major cardiac anomaly

ESOPHAGEAL RINGS AND WEBS 185. Ans. a. Mucosal ring at squamocolumnar junction, c. Dysphagia is the symptom (Ref: Sabiston 20/e p1026-1027; Schwartz 10/e p984; Bailey 27/e p1102; Shackelford 7/e p89-90) 186. Ans. c. Schatzki ring (Ref: Sabiston 20/e p1027; Schwartz 10/e p984; Bailey 27/e p1102) 187. Ans. b. Lower esophagus 188. Ans. c. Involves mucosa, submucosa and muscularis 189. Ans. b. Located at lower esophagus, c. Causes dysphagia 190. Ans. b. Occurs at GE junction (Ref: Gastroenterol Hepatol (NY); 2010 November 6 (11): 701-704) Types of Esophageal Ring on Barium Examination Located few cm proximal to GE junction

Type B

Schatzki’s ring: MC esophageal ring found on esophagogram, at GE junctionQ

Type C

Located at most distal portion of esophagus, formed by diaphragmatic crural pressure

191. Ans. b. Schatzki ring (Ref: Sabiston 20/e p1026; Schwartz 10/e p984; Bailey 27/e p1102; Shackelford 7/e p89-90)

FOREIGN BODY 192. Ans. d. Above the cricopharynx (Ref: Schwartz 9/e p2378; Dhingra 4/e p64) The first constriction where the esophagus commences is at the cricopharyngeal sphincter: This is the narrowest portion of the esophagus and is the most common site of foreign body. 193. Ans. a. Esophagus “Coins in the esophagus are round in appearance on the frontal view whereas coins in the trachea are usually seen on end and are linear in shape.” Foreign Body • Coins account for 70% of pediatric ingested foreign bodies • Coins will typically become ‘stuck’ at the level of the cricopharyngeus muscle. Coin

Chest X-ray Anteroposterior (AP) View

In trachea

• Visualized in the sagittal planeQ (acquired while entering through vocal cords)

In esophagus

• Visualized in coronal orientationQ

ESOPHAGUS: ANATOMY AND PHYSIOLOGY 194. Ans. d. Lower esophagus before entering the diaphragm is anterior and left sided (Ref: Sabiston 20/e p1067; Schwartz 10/e p941-947; Shackelford 7/e p18-24)

Gastrointestinal Surgery

Type A

312

Surgery Essence 195. Ans. c. Tertiary (Ref: Sabiston 20/e p1014-1015)

Gastrointestinal Surgery

Tertiary Contractions of Esophagus • Tertiary contractions are simultaneous, non progressive, non peristaltic waves that can occur throughout the esophagusQ • Tertiary contractions represent uncoordinated contractions of the smooth muscles that are responsible for the ‘Cork Screw’ appearance of esophageal spasm on Barium swallow • Tertiary contractions do not have a physiological function and may be observed in the elderly and in patients with esophageal motility disordersQ 196. Ans. c. 40 cm Esophageal Constrictions No.

Distance from incisor teeth

Bony level

Anatomical Landmark

1

15 cm

Q

C6

Pharyngoesophageal junction (At beginningQ)

2

25 cm

Q

3

40 cmQ

Aortic arch, left bronchusQ T10

Pierces diaphragmQ

BALD: Beginning, Aortic arch, Left Bronchus, DiaphragmQ 197. Ans. b. Where it pierces the diaphragm

198. Ans. a. Left phrenic nerve

199. Ans. d. Left main stem bronchus 200. Ans. b. Bleeding from varices occurs from midesophagus • Bleeding from esophageal varices most commonly occur in the distal esophagusQ. 201. Ans. a. 6–26 mm Hg (Ref: Sabiston 20/e p1016; Bailey 27/e p1068) “The normal LOS is 3–4 cm long and has a pressure of 10–25 mm Hg.”-Bailey 27/e p1068 202. Ans. c. Transmural (Ref: Schwartz 10/e p1004)

DIAPHRAGM 203. Ans. b. Diaphragmatic palsy • Fluoroscopy is used for diagnosis of diaphragmatic palsyQ. 204. Ans. a. Laparoscopy (Ref: Sabiston 20/e p432; Schwartz 10/e p202-203; Bailey 27/e p365) 205. Ans. d. Dorsal mesocardium (Ref: Langman 11/e p161)

Development of Diaphragm • Septum transversum → Central tendon • Mesentery of esophagus → CruraQ • Cervical myotomes (muscular input)Q Q

•  Pleuroperitoneal membranes → Small intermediate muscular portionQ •  Body wall → Peripheral muscular diaphragmQ

206. Ans. b. Circumferential (Ref: Sabiston and Spencer’s Surgery of Chest 8/e p chapter 7) Circumferential incision is generally taken for diaphragmatic surgery.

Section 3

Diaphragmatic Incisions • Diaphragmatic incisions can be divided into three groups: circumferential, central tendon, and radial. 1. Circumferential incisions: −− Circumferential incisions in the periphery result in little loss of function. −− Must be at least 5 cm lateral to the edge of the central tendon to avoid the posterolateral and anterolateral branches of the phrenic nerve. −− Difficult to correctly realign after a long operation. −− Placement of surgical clips on each side of the muscular incision can greatly facilitate the correct spatial orientation on closing. 2. Incisions in the central tendon: −− Do not interrupt any major branch of the nerve itself. −− Provide excellent visualization of the abdomen from the thorax, and vice versa. −− Easy to open and to close.

Esophagus

MISCELLANEOUS 207. Ans. b. 5

208. Ans. b. Achalasia cardia

209. Ans. b. Achalasia cardia

210. Ans. a. Pharyngeal pouch • Second swallow is due to regurgitation of barium from pharyngeal pouch

Section 3

3. Transverse radial incision: −− Made from the midaxillary line centrally −− Relatively safe −− May result in segmental diaphragmatic paralysis if the incision transects the crural or posterolateral branches of the phrenic nerve.

313

211. Ans. d. Esophageal perforation (Ref: Harrison 19/e p1720)

Hamman’s Sign or Hamman’s Crunch • A crunching, rasping sound, synchronous with heartbeat, heard over the precordium and sometimes at a distance from the chest in spontaneous mediastinal emphysemaQ. • Hamman’s sign may be present in acute Mediastinitis (as in esophageal perforation) Q. 212. Ans. c. 50 (Ref: Shackelford 7/e p545) • To restore normal swallowing, stricture should be dilated to at least 16 mm diameter or 50 French. Q • Guidewire directed dilatation of esophageal strictures is normal practice in most unitsQ. 214. Ans. c. 50 cm (Ref: Bailey 27/e p1136)

213. Ans. b. Guide wire directed stent

• The Roux loop should be at least 50 cm long to avoid bile reflux esophagitisQ 215. Ans. a. Eosinophilic esophagitis (Ref: Schwartz 10/e p986)

216. Ans. b. Dual probe pH monitoring (Ref: Pediatric Otolaryngology (2014)/p1511) 217. Ans. a. CMV esophagitis (Ref: Harrison 20/e p2218) “CMV esophagitis occurs primarily in immunocompromised patients, particularly organ transplant recipients. CMV is usually activated from a latent stage. Endoscopically, CMV lesions appear as serpiginous ulcers in an otherwise normal mucosa, particularly in the distal esophagus.”-Harrison 20/e p2218 Causes of esophagitis Severe reflux esophagitis with mucosal ulceration & friability

Cytomegalovirus esophagitis

Herpes simplex virus esophagitis with target-type shallow ulcerations

Candida esophagitis with white plaques adherent to esophageal mucosa

218. Ans. d. Herpes simplex esophagitis (Ref: Harrison 20/e p2218) “Herpes simplex virus type 1 or 2 may cause esophagitis. Vesicles on the nose and lips may coexist and are suggestive of a herpetic etiology. Varicella-zoster virus can also cause esophagitis in children with chickenpox or adults with zoster. The characteristic endoscopic findings are vesicles and small, punched-out ulcerations.”-Harrison 20/e p2218

Gastrointestinal Surgery

“Eosinophilic esophagitis (EE): A barium swallow should be the first test obtained in the patient with dysphagia. EE has a characteristic finding often called the “ringed esophagus” or the “feline esophagus,” as the esophageal rings are felt to look like the stripes on a housecat. The endoscopic appearance of EE is also characteristic, and also appears as a series of rings.”-Schwartz 10/e p986

CHAPTER

10

Stomach and Duodenum

HELICOBACTER PYLORI Helicobacter Pylori • First successful culture of organism was done by Marshall and WarrenQ, who named it Campylobacter pyloridis. • Around 90% of duodenal ulcers and 75% of gastric ulcer are associated with H. pylori infectionQ. • Gastric antrum is MC site of colonizationQ. • It can live only in gastric epithelium, because only gastric epithelium expresses specific adherence receptors in vivo that can be recognized by organism. • Also found in heterotopic gastric mucosa in proximal esophagus, Barrett’s esophagus, gastric metaplasia in the duodenum, Meckel’s diverticulum, and heterotopic gastric mucosa in the rectumQ. • After the person is infected, usually in childhood, it is probably for life because spontaneous remission is rareQ. • There tends to be inverse relationship between infection and socioeconomic statusQ. • Mechanisms by which H. pylori promote ulcer formation include stimulation of gastrin release, inhibition of somatostatin release, interruption of inhibitory vagal reflexes and inhibition of gastroduodenal bicarbonate secretionQ. • After eradication of the organism, ulcer recurrence is rareQ. Characteristic Features • Spiral shaped, gram (-)ve rod, motile with lophotrichous flagellaQ • The sole source is human gastric mucosa • Biochemical reactions: Catalase, oxidase and urease positiveQ • Grows well when incubated at 37°C in microaerophilic conditionsQ. • Media used include Skirrow’s medium, chocolate mediumQ. Pathogenesis • Grows optimally at pH 6.0-7.0Q and would be killed at pH within the gastric lumen. • But it survives as it is found deep in mucus layer near epithelial surface, without invading mucosa where physiologic pH is present. It produces potent urease, which provides ammonia to buffer acidQ. • Major diseases associated H. pylori virulence factors are vacuolating cytotoxin (Vac A)Q, and group of genes called CagPalQ. • H. pylori colonization decreases somatostatin producing cells →↑ Gastrin →↑Acid → Gastric metaplasia in duodenum → UlcerationQ. Clinical Manifestations • 90% of duodenal ulcer and 75% of gastric ulcerQ are related to H. pylori • Increase risk of gastric adenocarcinoma, gastric MALT lymphomaQ. • Extra-gastrointestinal pathologies that are linked include ischemic heart disease and cerebrovascular diseaseQ. • CAG-A positive strain is protective for adenocarcinoma esophagus but can lead to SCC of esophagusQ. Diagnosis • Histologic visualization of H. pylori is the gold standard of diagnostic testQ (special stains used are silver, Giemsa, Genta or Warthin starry stain)Q. • The method of choice to diagnose if endoscope is employed is rapid urease testQ. • Serology is the test of choice for initial diagnosis when endoscopy is not required. • After treatment, Urea breath test is the method of choice but should be performed after 4 weeks of therapyQ. Accuracy of Diagnostic Methods • Chronic inflammation on a gastric mucosal biopsy specimen is 100% sensitive testQ. • Rapid Urease test on a gastric mucosal biopsy specimen is 100% specific testQ.

Stomach and Duodenum

315

PEPTIC ULCER Duodenal Ulcer

• MC Site: −− Lesser curvature along the incisura angularis (Type 1)Q

• MC site: −− 1st part of duodenum (overall MC site for peptic ulcer)Q

• Clinical features: −− Equal in both sexes −− Pain in the epigastrium after taking foodQ; relieved by vomiting −− Pain is uncommon during night −− Hematemesis common −− Appetite good, but hesitant to eat as eating leads to pain that results in loss of weightQ

• Clinical features: −− More common in malesQ −− Pain in early morning, decreases after food (hunger painQ) −− Pain common during nightQ −− Melena common −− Appetite good, eats more frequently and there is weight gainQ

• Features on Barium meal: −− Niche on lesser curve with notch on greater curveQ −− Regular/round margin of ulcer create spoke wheel patternQ −− Overhanging mucosa at the margins of a benign gastric ulcer projects inwards, towards the ulcer: Hampton’s lineQ −− Converging mucosal folds at the base of the ulcerQ

• Features on Barium meal: −− Deformed or absent duodenal cap (because of spasm)Q −− Appearance of trifoliate duodenum due to secondary duodenal diverticulaQ

• Complications: −− Perforation: MC complication of gastric ulcer (Into lesser sac)Q −− Hour glass contracture: Exclusively in women due to cicatricial contracture of lesser curve ulcerQ −− Tea pot stomach (hand bag stomach): Cicatrisation and shortening of the lesser curvatureQ −− Malignant transformationQ

• Complications: −− Bleeding: MC complication, on posterior wall, gastroduodenal arteryQ is most commonly involved −− Perforation: More on anterior wall, if posterior, into pancreas −− Gastric outlet obstruction due to pyloric stenosis- least commonQ −− Duodenal ulcers are benign (No malignant transformation)Q

TYPES OF GASTRIC ULCER

Modified Johnson’s Classification of Gastric Ulcer Type

Location

Acid Secretion

I

Lesser curvature, near incisura angularis (MC)

II

Body of the stomach and duodenumQ

HighQ

III

Prepyloric (within 2-3 cm of the pylorus)

HighQ

IV

High on the lesser curve, near GE junctionQ

Low

V

Anywhere, induced by medication (NSAIDs)

Low

Q

Q

Low

Gastrointestinal Surgery

• Etiology: −− Stress, anxiety: ‘hurry, worry, curry’Q −− H. pylori (90%)Q −− NSAIDs, steroidsQ −− Blood group O+veQ −− Endocrine: Zollinger-Ellison syndrome, MEN-1, Cushing’s syndrome, hyperparathyroidismQ −− Alcohol, smoking, vitamin deficiencyQ −− Chronic pancreatitis, CirrhosisQ

Section 3

Gastric Ulcer • Etiology: −− Atrophic gastritis −− H. pylori (75%)Q −− Smoking, AlcoholQ −− Lower socioeconomic statusQ −− Altered mucosal barrier function (NSAIDs)Q −− There is either normochlorhydria or achlorhydriaQ −− CirrhosisQ

316

Surgery Essence

Gastrointestinal Surgery

Gastric Ulcer Benign Ulcer

Malignant Ulcer

• Generally at lesser curvatureQ • Smooth radiating foldsQ with Hampton line and collarQ • Overhanging marginsQ showing regeneration • Mucosal rugae projects outwards from the margins of ulcerQ • Penetrating sign (ulcer crater projects into stomach wall rather than into a mass in a stomach wall). • Huge baseQ • Preserved peristalsisQ • Heals within 8-10 weeksQ

• At greater curvatureQ • Interrupted nodular, clubbed folds with Lasman Kirklin complexQ (malignant ulcer with no mass) • Eccentric with heaped up and everted marginsQ • Mucosal rugae stop far of the ulcerQ • Carman’s meniscus sign (meniscoid appearance of trapped barium in ulcer bed) • Necrotic baseQ • No peristalsisQ • No healingQ

Gastric Ulcer on Double Contrast upper GI series Benign Ulcer: • In benign gastric ulcer, mucosal folds radiate from crater in sponge like mannerQ. • Typically benign ulcer craters extend beyond the luminal margin of the stomach and have radiating gastric folds, ulcer mound due to mucosal edema, ulcer collarQ (a lucent ring that separates ulcer crater from gastric mucosa), Hampton’s line and Penetrating signQ (ulcer crater projects into stomach wall rather than into a mass in a stomach wall). Malignant Ulcer: • In malignant ulcers, gastric folds are amputated or clubbed and do not reach the edge of ulcer crater have parallel gastric folds. • Malignant ulcers are characterized by Carman’s meniscus signQ (meniscoid appearance of trapped barium in ulcer bed), intraluminal craterQ (crater erodes into the mass within the gastric cavity) and Kirklin complexQ (heaped up margins touching bed causes lucent rim around ulcer on barium meal).

COMPLICATIONS OF PEPTIC ULCER Complications of Peptic Ulcer • Intractability (Non-healing) • Bleeding: MC complication of peptic ulcerQ • Perforation: MC complication of gastric ulcerQ • Gastric outlet obstruction (Rare) Duodenal Ulcer

Section 3

Complications

Treatment

Intractable

Highly selective vagotomyQ

Bleeding

Truncal vagotomy with pyloroplastyQ and oversewing of bleeding vessel

Perforated

Omental patch repair (with Truncal vagotomy in stable patientsQ)

Obstruction

(Rule out malignancy) • Truncal vagotomy with antrectomy is ideal procedureQ • Truncal vagotomy with gastrojejunostomy in cases of inflammation and scarring of duodenal bulbQ

GASTRIC OUTLET OBSTRUCTION Gastric Outlet Obstruction • MC cause of gastric outlet obstruction: CA stomachQ • Site of stenosis or obstruction in peptic ulcer disease: 1st part of the duodenumQ • More common with duodenal ulcer and type III gastric ulcer and requires that malignancy should be ruled out.

Stomach and Duodenum

317

Clinical Features:

• A saline load test is helpful, it is performed by emptying the stomach with a nasogastric tube and instilling 750 mL of saline, the patient is placed in sitting position, and 30 minutes later the nasogastric tube is aspirated, normally < 400 mL should remain in the stomach, and 90% of subjects have a residue of less than 200 mL. • The finding of > 400 mL residual saline is consistent with a diagnosis of gastric outlet obstruction. Treatment: • Surgery is generally indicated if the obstruction fails to resolve despite 48–72 hours of adequate IV fluid replenishment, antisecretary therapy and nasogastric decompressionQ.

Section 3

• Symptoms of gastric retention, including early satiety, bloating, indigestion, anorexia, nausea, vomiting, epigastric pain, and weight loss. • Patients are frequently malnourished and dehydrated and have a metabolic alkalosis, factors that increase operative risk. Diagnosis:

• Truncal vagotomy and antrectomyQ is the ideal procedure. • The inflammation and scarring at duodenal bulb or previous proximal duodenal surgeries prevents safe performance of an antrectomy, in this setting truncal vagotomy with drainage (Gastrojejunostomy) is the preferred approachQ.

TYPES OF VAGOTOMY Types of Vagotomy Highly Selective Vagotomy

Vagotomy and Drainage

• Procedure of choice for chronic or intractable duodenal ulcersQ • Nerves of Latarjet supplying the antrum are preservedQ (and hence gastric motility) • Drainage procedure is not requiredQ • Lowest mortality rate and side effectsQ • Minimal chances of dumping syndrome and gastric atonyQ • Relatively high recurrenceQ

• TV is performed by division of left and right vagus nerves above the hepatic and celiac branchesQ just above the GE junction. • MC operation performed for duodenal ulcerQ • Intermediate morbidity and recurrence rateQ

Vagotomy and Antrectomy • Procedure of choice for recurrent duodenal ulcersQ • Lowest recurrence rateQ • High mortality and morbidityQ

Operations for Gastric Ulcer Billroth-I Gastrectomy • Gastroduodenostomy • Performed when there is a sufficient portion of upper duodenum remainingQ • Remaining portion of stomach is reattached to the duodenumQ Q

Billroth I (Gastroduodenal) anastomosis

Billroth-II Gastrectomy • Loop gastrojejunostomyQ • Performed if the stomach cannot be reattached to duodenumQ • Remaining portion of duodenum is sealed off, a hole is cut into the jejunum & stomach is reattachedQ at this hole.

Billroth II Gastrectomy (Loop Gastrojejunostomy)

Roux-en-Y Gastrojejunostomy

Gastrointestinal Surgery

OPERATIONS FOR GASTRIC ULCER

318

Surgery Essence Polya Gastrectomy • A type of posterior gastroenterostomy which is a modification of Billroth II operation. • Resection of 2/3 of the stomach with blind closure of the duodenal stump and retrocolic anastomosis of the full circumference of the open stomach to jejunumQ. Elective Gastric Ulcer Operations

Gastrointestinal Surgery

Type

Procedure

Type I

• Distal gastrectomy with Billroth I or II reconstructionQ

Type II and III

• Truncal vagotomy plus antrectomyQ

Type IV

• • • •

Schoemaker procedureQ Pouchet procedureQ Kelling-Madlener procedure (For unstable patientsQ) Csendes procedure (For stable patientsQ)

Remember: • Benign BPH: (Benign ulcer- Penetrating sign and Hampton’s hump) • Malignant CIK: (Carman’s meniscus sign, Intraluminal crater, Kirklin complex)

COMPLICATIONS AFTER GASTRECTOMY Postgastrectomy/Vagotomy Syndrome Secondary to Gastric Resection • Dumping syndrome • Metabolic disturbancesQ

Section 3

Q

Secondary to Gastric Reconstruction • • • •

Afferent loop syndrome Efferent loop obstructionQ Alkaline reflux gastritisQ Retained antrumQ syndrome Q

Postvagotomy Syndrome • Postvagotomy diarrheaQ • Postvagotomy gastric atonyQ • Incomplete vagal transectionQ

Metabolic Complications after Gastrectomy • • • •

Metabolic complications are more common and serious after partial gastrectomyQ than after vagotomy. More common in Billroth IIQ >Billroth I procedure Severity is directly related to the extent of gastric resectionQ. MC metabolic defect appearing after gastrectomy: AnemiaQ

Stomach and Duodenum

319

Anemia

Iron deficiency anemia: • More than 30% of patients undergoing gastrectomy suffer from IDAQ • Cause: Combination of decreased iron intake, impaired iron absorption, and chronic blood lossQ • Addition of iron supplements to the patient’s diet corrects IDAQ. Megaloblastic anemia:

Section 3

• MC metabolic defect appearing after gastrectomy: AnemiaQ • Iron deficiency anemia (IDA) is more common than vitamin B12 deficiency anemiaQ.

• Megaloblastic anemia can occur when >50%Q of the stomach is removed such as occurs during subtotal gastrectomyQ. • Vitamin B12 deficiency occur secondary to poor absorption of the substance owing to lack of intrinsic factor secretion in the gastric juice. • Treatment: Intramuscular injections of cyanocobalaminQ every 3 to 4 months indefinitely because its administration orally is not a reliable route. Folate deficiency: • Folate deficiency is rare after gastric resection • May coexist with either an iron or vitamin B12 deficiency. • Usually be corrected by dietary supplementation. Impaired Absorption • Common metabolic disturbance after gastric resections is impaired absorption of fat. • Steatorrhea may occur as a result of inadequate mixing of bile salts and pancreatic lipase with ingested fat because of the duodenal bypassQ. • Deficiency in uptake of fat-soluble vitamins may also occurQ. • Osteoporosis and osteomalacia have also bee`n observed after gastric resection and appear to be caused by deficiencies in calciumQ. • If fat absorption is also present, the calcium malabsorption is further aggravated because fatty acids bind calciumQ. • Treatment: Calcium supplements with vitamin DQ

Dumping Syndrome • Dumping syndrome refers to a constellation of post-prandial symptoms occurring due to accelerated emptying (dumping) of hyperosmolar stomach contents into the small bowel. • It is usually seen in operation which destroys the pyloric sphincter (i.e gastrectomy, antrectomy and drainage procedures) Q. • It also affects a small percentage of patients with highly selective vagotomy due to loss of receptive relaxation of stomach. Dumping Syndrome Early Dumping • It occurs immediately after meals (after 15-30 minutes) Q • Dumping of hyperosmolar contents into small bowelQ results in rapid fluid influx from the circulation into gastrointestinal tract. This leads to acute intestinal distention and peripheral & splanchnic vasodilatationQ. • This gives rise to vasomotor & abdominal symptoms: Epigastric fullness, sweating, light headedness, tachycardia, diarrhea. • Symptoms can be ameliorated by lying down & saline infusionQ.

Late Dumping • It is seen 2-3 hrs after mealQ • Occurs due to reactive hypoglycemiaQ. • Carbohydrate load in small bowel → Increased plasma glucose → Increased insulin secretion → Hypoglycemia • Symptoms are relieved by administration of sugarQ.

Management • Dietary management: Diet therapy is done to reduce jejunal osmolality. −− Multiple small meals, food low in carbohydrate and rich in fat and proteins are takenQ. −− Liquids during meals should be avoidedQ. • Somatostatin analogues (octreotide): −− Diet therapy is usually successful but if it fails, the patient is started on octreotide.

Gastrointestinal Surgery

DUMPING SYNDROME

320

Surgery Essence

Gastrointestinal Surgery

• Surgery: −− Surgery is rarely requiredQ as most of the patients improve with time, dietary management and Octreotide. −− Surgical procedures used to treat dumping are: • Use of an antiperistaltic loop of jejunum between the residual gastric pouch and intestine • Conversion of Billroth II to Billroth I anastomosis • Conversion to Roux-en-Y-anastomosis.

GASTROJEJUNOCOLIC FISTULA Gastrojejunocolic Fistula • Gastrojejunocolic fistula may form between transverse colon & upper jejunum after a Billroth II surgical procedureQ. (The Billroth procedure attaches the jejunum to the remainder of the stomach). Clinical Features: • The symptoms of GJF are diarrhea, epigastric pain and discomfort, gastrointestinal bleeding, feculent eructation, fecal vomiting, weight loss and weaknessQ. • Anemia, leukocytosis, electrolyte disturbances and hypoalbuminemia are common laboratory findingsQ. Diagnosis: • Barium enema is diagnostic of GJC fistulaQ. • Investigations used: Barium enema and endoscopyQ • Success rate of barium enema in correctly diagnosing the fistula is 95-100%Q Treatment: • The historical approach was 2-3-staged operations even involving a preliminary diversion colostomy in order to ameliorate the nutritional status of the patient and to decrease mortalityQ • Three staged operation: (1) colostomy (2) resection of the fistula (3) colostomy closure • Today, because of the parenteral & enteral support treatments and the developments in intensive care conditions, one-stage resection can be appliedQ

UPPER GI BLEED Risk Stratification Systems for Upper GI Bleeds • Help to identify the patients at higher risk of major bleeding or death (facilitate patient triage) • Commonly used scoring systems: −− Rockall score (takes account of endoscopic findings; most usefulQ) −− Blatchford score (used during initial assessment, does not require endoscopic dataQ) Commonly Used Risk Stratification Systems for Upper GI Bleeds Blatchford Score (PUSH + Melena/Syncope + Cardiac/Hepatic Dysfunction)

Section 3

• • • • •

PulseQ Blood Urea nitrogenQ Systolic BPQ HemoglobinQ Presence of MelenaQ, SyncopeQ, HepaticQ or Cardiac dysfunctionQ

Rockall & Baylor Score (CASDE) Comorbid diseaseQ (cardiac, hepatic, renal, or disseminated cancer) AgeQ (80 years) ShockQ (systolic BP 100 beats/min) Diagnosis at the time of endoscopyQ (Mallory-Weiss tears, nonmalignant lesions, or malignant lesions) • Endoscopic stigmata of recent bleedQ
 • • • •

Bleed Risk Classification • To predict risk of rebleeding & mortality in upper GI bleed • Predicts risk on initial presentation based on five criteria: 1. Ongoing BleedingQ 2. Low systolic BPQ (1.2 times the control value) 4. Erratic mental statusQ 5. Unstable comorbid DiseaseQ

Stomach and Duodenum

321

THE FORREST CLASSIFICATION

Grade Ia Ib IIa IIb IIc III • •High rebleeding risk: I & IIaQ



•  Intermediate risk: IIbQ



Section 3

The Forrest Classification (For Endoscopic Findings and Rebleeding Risk) Description Rebleeding Risk Active, pulsatile bleedingQ High OozingQ, non-pulsatile bleeding High Non-bleeding visible vesselQ High Adherent clotQ Intermediate Black dotQ Low Clean baseQ Low •  Low risk: IIc & IIIQ

MALLORY WEISS SYNDROME Mallory Weiss Syndrome • Mallory-Weiss tears are related to forceful vomiting, retching, coughing, or strainingQ • Forceful contraction of abdominal wall against an unrelaxed cardia, resulting in mucosal laceration of proximal cardiaQ as a result of the increase in intragastric pressure. • Results in disruption of gastric mucosa high on the lesser curve at cardia (just below GE junction)Q • Tear is partial thickness, extending through the mucosa and submucosaQ Clinical Features: • Classically, seen in alcoholic patientsQ after a period of intense retching and vomiting after binge drinking. • Cause of up to 15% of all severe upper GI bleedsQ • Arterial bleeding (usually from left gastric arteryQ), usually painless and are rarelyQ associated with massive bleeding.

Diagnosis: • Usually diagnosed by history • Endoscopy is used to confirm the diagnosisQ. Treatment: • Supportive therapy is often all that is necessary because 90% of bleeding episodes are self-limited, and the mucosa often heals within 72 hoursQ. Persistent Bleeding in Mallory Weiss Syndrome is managed by • Endoscopic electrocoagulationQ or endoscopic therapy with injection • Angiographic embolizationQ • Surgery consists of laparotomy and high gastrotomy with oversewing of the linear tearQ, if above maneuvers fails. • Recurrent bleeding from a Mallory-Weiss tear is uncommonQ. Remember: A Sengstaken-Blakemore tube will not stop bleeding in Mallory-Weiss syndrome, as the bleeding is arterial and the pressure in the balloon is not sufficient to overcome the arterial pressure and is contraindicatedQ.

DIEULAFOY’S GASTRIC LESION Dieulafoy’s Gastric Lesion • Caused by an abnormally large (1–3 mm)Q, tortuous artery coursing through the submucosa • Occurs 6–10 cm from the GE junction, generally in ‘fundus’ near the cardia along lesser curvatureQ. • Erosion of superficial mucosa overlying the artery occur secondary to pulsations of large submucosal vesselQ arteriole • Artery is exposed to gastric contents, leading to further erosion & bleeding occursQ. • Mucosal defect is 2–5 mm in size

Gastrointestinal Surgery

• The overall mortality rate is 3-4%, with the greatest risk for massive hemorrhage in alcoholic patients with preexisting portal hypertensionQ.

322

Surgery Essence Clinical Features: • More common in men (2:1) with peak incidence in 5th decadeQ. • Associated with sudden onset of massive, painless, recurrent hematemesis with hypotension • Recurrent bleeding with spontaneous cessation is commonQ. Diagnosis:

Gastrointestinal Surgery

• Endoscopy is the diagnostic modality of choice, correctly identifying the lesion in 80% of patients. • Repeated endoscopies may be needed to correctly identify the lesion because of intermittent nature of the bleedingQ • Angiography showing a tortous ectatic artery in the distribution of the left gastric artery with accompanied contrast extravasation in the setting of acute bleeding. Treatment: • • • •

Initial attempts at endoscopic control are often successfulQ. Application of thermal or sclerosant therapy is effective in 80–100% of cases.Q In cases that fail endoscopic therapy, angiographic coil embolization can be successfulQ. Gastric wedge resection to include the offending vessel is reserved when other modalities have failedQ.

WATERMELON STOMACH (GASTRIC ANTRAL VASCULAR ECTASIA) Watermelon Stomach (Gastric Antral Vascular Ectasia) • A rare entity characterized by presence of both inflammatory and vascular components in mucosaQ. Pathology: • Dilated mucosal blood vessels in the lamina propria, often containing thrombi, with no evidence of vascular malformation on angiographic and morphologic examinationQ. • Mucosal fibromuscular hyperplasia and hyalinization are often presentQ. • Predominantly affects the distal portion (Antrum)Q of the stomach Clinical Features: • Patients are generally elderly women with chronic bleedingQ. • Most have an associated autoimmune connective tissue disorders, and at least 25% have chronic liver diseaseQ. • Patients typically have iron deficiency anemia & chronic blood loss requiring transfusionsQ. Diagnosis: • Diagnosis is based on typical endoscopic and biopsy appearance of mucosaQ. • Gross endoscopic examination reveals prominent longitudinal folds with parallel striking red stripes atop the mucosal folds of the distal stomach, much like the rind of a watermelonQ. Treatment: • Lesions are treated by endoscopic cauteryQ. • In patients with portal hypertension, TIPS should be considered firstQ.

RISK FACTORS FOR CARCINOMA STOMACH

Section 3

Factors Associated with Increased Risk of Developing Stomach Cancer Nutritional • • • •

Low fat or protein consumptionQ Salted meatQ High nitrate consumptionQ High complex-carbohydrate consumptionQ

Social • Low social classQ

Medical • • • • • •

Prior gastric surgeryQ H. pyloriQ infection Epstein-Barr virusQ Gastric atrophy and gastritisQ Adenomatous polypsQ Male genderQ Occupational

• Rubber workersQ • Coal workersQ

Stomach and Duodenum Environmental Poor food preparation (smoked, salted)Q Lack of refrigerationQ Poor drinking water (well water)Q SmokingQ

• • • • •

Genetic factors Blood group ‘A’Q Pernicious anemiaQ Family history Hereditary nonpolyposis colon cancerQ Li-Fraumeni syndromeQ

• Decreased risk of carcinoma stomach: Aspirin, Diet (high fresh fruit and vegetable intake), Vitamin A and C, calcium, selenium, zinc and ironQ. • Alcohol is not a risk factor for CA stomachQ. COX-2 Inhibitors are protective in • Barrett’s esophagusQ • Carcinoma stomachQ • SCC and adenocarcinoma of • Desmoid tumorsQ esophagusQ • Small duodenal and rectal polyps in FAPQ

Section 3

• • • •

323

MÉNÉTRIER’S DISEASE (HYPOPROTEINEMIC HYPERTROPHIC GASTROPATHY) Ménétrier’s disease (Hypoproteinemic Hypertrophic Gastropathy) • A rare, acquiredQ, premalignantQ disease, of unknown cause • Characterized by massive gastric folds in the fundusQ & corpus of the stomach, giving the mucosa a cobblestone or cerebriform appearanceQ. • Associated with CMV infection in childrenQ & H. pylori infection in adultsQ. Pathology: • Foveolar hyperplasia (expansion of surface mucous cells) with absent parietal cellsQ. • Increased TGF-alpha has been noted in the gastric mucosaQ • Associated with protein loss from the stomach, excessive mucus production, & hypochlorhydria or achlorhydriaQ. Clinical Features: • Epigastric pain, vomiting, weight loss, anorexia, & peripheral edemaQ.

Treatment: • Medical treatment is limited to albumin replacement and maintenance of adequate nutrition, acid suppression, octreotide & H. pylori eradicationQ. • Total gastrectomy for bleeding, severe hypoproteinemia or cancerQ.

CLASSIFICATION SYSTEM FOR CARCINOMA STOMACH Lauren Classification System for Carcinoma Stomach

• • • • • • • • • • •

Intestinal EnvironmentalQ Gastric atrophy, intestinal metaplasia MenQ> Women Increasing incidence with age Gland formationQ HematogenousQ spread Microsatellite instabilityQ APC gene mutationsQ p53, p16 inactivationQ EpidemicQ Distal part of the stomachQ

• • • • • • • • • •

Diffuse FamilialQ Blood group ‘A’Q WomenQ > Men Younger age groupQ Poorly differentiated signet ring cellsQ Transmural/Lymphatic spreadQ Decreased E-cadherinQ p53, p16 inactivationQ EndemicQ Proximal part of the stomachQ

Gastrointestinal Surgery

Diagnosis: • Typical gastric mucosal changes can be detected by radiographic or endoscopic examination. • Biopsy should be performed to rule out gastric carcinoma or lymphomaQ. • Twenty-four-hour pH monitoring reveals hypochlorhydria or achlorhydriaQ • Chromium-labeled albumin test reveals increased GI protein lossQ.

324

Surgery Essence

Gastrointestinal Surgery

Type I Type II Type III Type IV Type V

Bormann Classification (Based on macroscopic appearance) Polypoid or fungatingQ cancers Fungating & ulcerated with surrounded by elevated bordersQ Ulcerated lesions infiltrating the gastric wallQ Infiltrates diffusely (Linnitis plastica)Q Unable to be classified

CARCINOMA STOMACH Carcinoma Stomach • Incidence of gastric cancer, especially distal cancer is decreasingQ; Distribution: Distal (40%) > Middle (30%) = Proximal (30%) • Incidence of GE junction tumors are increasingQ • Approximately 90% of all tumors of the stomach are malignant, the vast majority of which are gastric adenocarcinomaQ. • MC genetic abnormalities: p53 and COX-2 geneQ (SPC: Stomach, p53 and COX-2 gene) • Tumor Markers: CEA, CA 19-9, CA-125, CA 72-4 & beta-HCG Clinical Features • MC Symptoms: Abdominal pain (62-91%)Q > weight loss (22-61%). • Typically, pain is constant, nonradiating, & unrelieved by food ingestion. • Proximal tumors involving the gastroesophageal junction often present with dysphagia, whereas distal antral tumors may present as gastric outlet obstructionQ. • Diffuse mural involvement by tumor, as occurs in linnitis plastica, leads to decreased distensibility of the stomach and complaints of early satietyQ. • Ascitis, jaundice or palpable mass indicate incurable diseaseQ. • Transverse colon is a potential site for malignant fistulization & obstruction from gastric primary tumor. • Patients may present with a palpable abdominal mass, a palpable supraclavicular (Virchow’s) or periumbilical (Sister Mary Joseph’s) lymph node, left axilla (Irish nodes) peritoneal metastasis palpable by rectal examination (Blummer’s shelf), or a palpable ovarian mass (Krukenberg’s tumor)Q via retrograde lymphaticsQ most commonly. • Paraneoplastic syndromes include thrombophlebitis (Trousseau’s syndrome), neuropathies, nephrotic syndrome, & DIC. Lymph Node Metastasis • Relative risk of nodal metastases at a specific nodal location depends on both the site of origin of the primary tumor and width & depthQ of invasion of gastric wall. • Proximal stomach & GE junction tumors: Higher propensity of spread to nodes in the mediastinum & pericardial regionQ. • Tumors in the body of stomach: Highest likelihood of spreading to nodes along the greater & lesser curvature, near the location of primary tumor massQ. • Tumors in the distal stomach: High likelihood of spread to the periduodenal, peripancreatic, & porta hepatis nodesQ. Diagnosis • Endoscopy with biopsy is the best methodQ to diagnose gastric cancer • IOC for staging of early gastric cancer: EUS • Best investigation for preoperative staging: CECTQ

Section 3

Treatment of Carcinoma Stomach according to site Proximal-third

Extended gastrectomy, including the distal esophagusQ

Middle-third

Total gastrectomy and D2 LN dissectionQ.

Distal-third

Intestinal-type: Subtotal gastrectomy with D2 LN dissectionQ Diffuse-type: Total gastrectomy with D2 LN dissectionQ

Recurrence: • Most recurrences occur within the first 3 yearsQ. • Loco-regional failure rate is highest at the anastomosis or stump (25%)Q > stomach bed (21%) > regional nodes. • MC site of metastasis: LiverQ > lung > bone.

Stomach and Duodenum

325

Surveillance

Prognosis: • Prognostic factors for CA stomach: DepthQ of invasion and LN statusQ

Lymph Node Stations 1. Right cardiac; 2. Left cardiac; 3. Lesser curvature; 4. Greater curvature; 5. Suprapyloric; 6. Infrapyloric 7. Left gastric; 8. Common hepatic; 9. Celiac; 10. Splenic hilus; 11. Splenic artery; 12. Hepatoduodenal ligament; 13. Retropancreatic; 14. Mesenteric root; 15. Transverse mesocolon; 16. Paraaortic

Section 3

• Follow-up should include a complete history and physical examination every 4 months for 1 year, then every 6 months for 2 years, and then annually thereafter.

Sister Mary Joseph nodule • Sister Mary Joseph nodule or node, also called Sister Mary Joseph sign, refers to a palpable nodule bulging into the umbilicus as a result of metastasis of a malignant cancer in the pelvis or abdomenQ. • Gastrointestinal malignancies account for half of underlying sources (most commonly gastric cancer, colonic cancer or pancreatic cancer, mostly of the tail and body of the pancreas), and men are more likely to have an underlying cancer of the gastrointestinal tractQ. • Gynecological cancers account for about 1 in 4 cases (primarily ovarian cancer and also uterine cancer)Q. Proposed mechanisms for the spread of cancer cells to the umbilicus • Direct transperitoneal spreadQ • Via lymphaticsQ which run alongside the obliterated umbilical vein • Hematogenous spreadQ

• Via remnant structures: −− Falciform ligamentQ  −− Median umbilical ligamentQ −− Remnant of the vitelline ductQ

Prognosis: • Sister Mary Joseph nodule is associated with multiple peritoneal metastases and poor prognosisQ.

8th AJCC (2017) TNM Classification of Carcinoma of the Stomach Tis: Carcinoma in situ: intraepithelial tumor without invasion of the lamina propria, high grade dysplasia

N1: Metastasis in 1-2 regional LNsQ

T1a: Tumor invades lamina propria or muscularis mucosaQ

N2: Metastasis in 3-6 regional LNsQ

T1b: Tumor invades submucosaQ

N3a: Metastasis in 7-15 regional LNsQ

T2: Tumor invades muscularis propria

N3b: Metastasis in 16 or more regional LNsQ

Q

T3: Tumor penetrates subserosal connective tissue without invasion of visceral peritoneum or adjacent structuresQ T4a: Tumor invades serosa (visceral peritoneum)Q T4b: Tumor invades adjacent structures

M1: Distant metastasis

Q

Stage Grouping Stage

IA

IB

T1N0

T1N1 T2N0

IIA T1N2 T2N1 T3N0

IIB T1N3a T2N2 T3N1 T4aN0

IIIA T2N3a T3N2 T4aN1-2 T4bN0

IIIB T1-2N3b T3-4aN3a T4bN1-2

IIIC

IV

T3-4aN3b T4bN3a-3b

Any T, Any N, M1

GASTRIC STUMP CARCINOMA Gastric stump carcinoma • It is defined as carcinoma arising in the gastric remnant > 5 yearsQ following pervious gastric resection for benign diseases or > 15 yearsQ after curative surgery for malignant disease.

Gastrointestinal Surgery

TNM CLASSIFICATION OF CARCINOMA OF THE STOMACH

326

Surgery Essence Etiology • Thought to be related to changes in the gastric mucosa which arise as a result of the change in the anatomical relationship between the stomach and the small intestine following surgery. • Enterogastric refluxQ plays a very significant role in the pathogenesis.

Gastrointestinal Surgery

• Other important etiological factors include: Bacterial proliferation and hypochlorhydriaQ which increases the mucosal susceptibility to carcinogenesis by N-nitrosamines. Pathology • Most of the stump carcinomas are often near the stoma but many of these tumors are quite large at presentation. • Equally divided between intestinal and diffuse subtypesQ. • Histologically they are classified as AdenocarcinomaQ, Adeno-squamous, Squamous cell carcinoma, Small cell and undifferentiated carcinoma. Clinical Features • Usually present late and the disease is advanced. Treatment • Surgical resection remains the only effective modality of treatment. • Overall average survival is 4–6 monthsQ. • Surgery whether palliative, has survival benefit.

LEATHER BOTTLE STOMACH (LINITIS PLASTICA) Leather Bottle Stomach (Linitis plastica) • Pyloric antrum is MC site affected in localized varietyQ • Stomach is massively thickened (feels like leather)Q Pathology: • Caused by proliferation of fibrous tissue mainly in submucosaQ • Characterized by Mother of pearl appearance • Mucosa appears normal Clinical Features: • Early satiety due to reduced stomach capacityQ • LN metastasis is commonQ Treatment: • Treated by radical gastrectomyQ Prognosis: • Associated with poor prognosisQ

GASTROINTESTINAL STROMAL TUMOR Gastrointestinal Stromal Tumor • GISTs: MC mesenchymal tumor of the GI tract • MC primary site for GIST: Stomach (60–70%) > small bowel (20–25%) > colorectum & esophagus (5% each)Q. Q

Section 3

• Most GISTs are positive for CD-117 (95%), BCL-2 (80%), CD-34 (70%)Q. • Types: Spindle cell (70%) and Epitheloid (30%)Q Pathology: • Arise from the muscularis propria and most likely originate from the cells of CajalQ • Expression of the receptor tyrosine kinase KIT (CD 117), 5% express platelet derived growth factor receptor alpha (PDGFRA)Q. • PDGFRA mutations in GIST appear to confer a very favorable prognosis with low risk of recurrenceQ. • New tumor markers of GIST: DOG-1 (discovered on GIST-1) & protein kinase C-theta Clinical features: • Patients usually present after the fourth decade, with the mean age of 60 years at diagnosis. • MC presentations of gastric GISTs: GI bleeding and pain or dyspepsia. Carney triad Association of extra-adrenal paragangliomas, pulmonary chondromas & multifocal GISTQ.

Stomach and Duodenum

327

Diagnosis:

• Percutaneous or endoscopic biopsy should only be performed if the results would obviate the need for surgeryQ. Treatment: • Bleeding manifestation is the MC indication for surgeryQ. • GIST should be treated with segmental resectionQ (margins of 1cm) • LN metastasis are uncommon, regional lymphadenectomy is not recommendedQ • Intraoperative incisional biopsy prior to resection should be avoided, because it risks tumor spillageQ

Section 3

• CT: IOC for evaluation of primary tumor & accurate stagingQ • PET-CT: Gold standard for recurrent GISTQ

• ImatinibQ (selective inhibitor of type 3 tyrosine kinase KIT), is approved for use in CD117-positive unresectable and metastatic GISTs. • Functional imaging of GIST with 18FDG-PET scanning represents a useful diagnostic modality for early-response assessment with imatinib therapyQ. • SunitinibQ is used in imatinib-refractory disease. • RegorafenibQ: Third line therapy in the patients failing imatinib & sunitinib therapyQ. Prognosis: • Tumor size is a predominant factor for survival in surgical series for primary GIST. • MC sites of disease failure after complete resection: LiverQ, omentum or peritoneal cavity. • Half to two third primaries will have disease failure within the liverQ and nearly 40% will have liver as the only site of failure. Generally hepatic involvement is multifocal. • Median time to recurrence after resection of primary GIST is 2 yearsQ.

ABDOMINAL LEIOMYOSARCOMA Abdominal Leiomyosarcoma

Clinical Features: Asymptomatic in initial stageQ Earliest symptom: Noticeable lump or swelling within the abdomen. The sign most often cited is bleedingQ. These tumors sometimes necrose and bleed into the bowel. Leiomyosarcomas that occur in the digestive tract can also cause gastrointestinal blockage or bleedingQ, which can manifest as blood in the stool. • Uterine leiomyosarcomas can cause vaginal bleedingQ • • • •

Diagnosis: • Endoscopy can be used to visualize the tumorsQ. • A definitive diagnosis requires a biopsyQ Treatment: • First-line treatment: Surgical removal of the cancerous tissue. • Uterine leiomyosarcoma: Total hysterectomyQ • Gastric leiomyosarcoma: Total gastrectomyQ

GASTRIC LYMPHOMA Gastric Lymphoma • Stomach is MC site for lymphomaQ in the GIT • MC site of gastric lymphoma: AntrumQ • MC gastric lymphoma is diffuse large B-cell lymphomaQ (55%) > extranodal marginal cell lymphoma (MALT) (40%) > Burkitts lymphoma (3%) > mantle cell and follicular lymphomas. • DLBL is MC type of NHL, extranodal lymphoma and GI lymphoma.

Gastrointestinal Surgery

• Leiomyosarcoma describes a type of soft-tissue cancer that occurs within smooth muscle cells. • This kind of cancer can be difficult to detect as it often causes no organ dysfunction until the tumor has gotten to be large in sizeQ. • Most common locations: Uterus & stomachQ.

328

Surgery Essence • Stomach, which is devoid of organized lymphoid tissue, is the MC site of MALT lymphomaQ. Clinical Features:

Gastrointestinal Surgery

• • • • •

Lymphomas occur in older patients (sixth and seventh decades) More common in men Vague symptoms, namely epigastric painQ, early satiety, and fatigue. Constitutional B symptoms are very rare. Although overt bleeding is uncommon, more than half of patients present with anemiaQ.

Diagnosis: • Endoscopy: Nonspecific gastritis or gastric ulcerations, with mass lesions being unusual. • Evidence of distant disease should be sought through upper airway examination, bone marrow biopsy, and CT of the chest and abdomen to detect lymphadenopathy. • Any enlarged lymph nodes should undergo biopsy. • H. pylori testing should be performed by histology and, if negative, confirmed by serology. Staging Systems for Primary Gastrointestinal Non-Hodgkin’s Lymphoma Ann Arbor

Description

Relative incidence (%)

IE

Tumor confined to gastrointestinal tract

26

IIE

Tumor with spread to regional lymph nodes

26

IIE

Tumor with nodal involvement beyond regional lymph nodes (para-aortic, iliac)

17

IIIE-IV

Tumor with spread to other intra-abdominal organs (liver, spleen) or beyond 31 abdomen (chest, bone marrow) Treatment of Gastric Lymphoma Low-grade MALT

High-grade (aggressive)

• Confined to gastric wall and no t(11:18) translocation: H.pylori eradication • Stage I, II, III: Chemotherapy + RT therapy and re-evaluate at 12 months • Lymph node involvement and t(11:18) translocation: H.pylori eradication therapy and re-evaluate at 3-6 months; if lymphoma persists: • Stage I: XRT • Stage II: Chemotherapy + RT

• Stage IV: Chemotherapy + RT • Residual disease: Further chemotherapy or Surgery

• Stage III or IV: H.pylori eradication therapy and Chemotherapy +/- RT • External beam radiotherapy: 30 Gy with 10 Gy boost • Chemotherapy regimens cyclophosphamide, doxorobucin, vincristine, prednisone (CHOP) +/- rituximab • Rituximab is chimeric monoclonal antibody against CD-20, preferred for high grade MALT or DLBL.

Dawson’s Criteria

Section 3

(Requirements for the diagnosis of Primary GI lymphoma) • • • • • •

Absence of palpable lymphadenopathy Normal bone marrow biopsy and peripheral blood smearQ Absence of mediastinal lymphadenopathyQ on chest radiographs Disease grossly confined to the affected viscusQ Regional lymphadenopathy onlyQ Absence of hepatic or splenic involvementQ unless via direct extension of the primary tumor. Q

PRIMARY GASTRIC HODGKIN’S DISEASE Primary Gastric Hodgkin’s disease • Primary gastric Hodgkin’s disease is extremely rare ( 35% of BSAQ

Gastrointestinal Surgery

• Enzymatic therapy to attempt dissolution of the bezoar. Papain and cellulase have been used with some success. • Generally, enzymatic débridement is followed by aggressive Ewald tube lavage or endoscopic fragmentationQ. Failure of these therapies would necessitate surgical removal.

330

Surgery Essence • Increased acid secretion in Cushing’s ulcer but not in Curling’s ulcerQ Pathophysiology: • Multifactorial etiology

Gastrointestinal Surgery

• Impaired mucosal defense mechanisms against luminal acid such as a reduction in blood flow, mucus, and bicarbonate secretion by mucosal cells, or a reduction in endogenous prostaglandinsQ. • In stress (hypoxia, sepsis, or organ failure), mucosal ischemia is the main factor responsible for the breakdown of these normal defense mechanismsQ. Risk factors or Predisposing clinical conditions • ARDSQ • Multiple traumaQ • Major burn > 35% of BSAQ

• Hepatic dysfunctionQ • Oliguric renal failureQ • Large transfusionQ requirements

• HypotensionQ • Prolonged surgical proceduresQ • SepsisQ

Clinical Features: • More than 50% of patients develop their stress gastritis within 1–2 days after a traumatic event. • Only clinical sign may be painless upper GI bleeding that may be delayed at onset. • Bleeding is usually slow and intermittentQ Diagnosis: • Endoscopy is required to confirm the diagnosisQ and to differentiate stress gastritis from other sources of GI hemorrhage. Treatment: • Definitive fluid resuscitation with correction of any coagulation abnormalities and treatment of the underlying sepsisQ • Intraluminal gastric pH should be maintained >5.0 with antisecretory agents. • Most of the superficial erosions are not actively bleeding do not require ligature unless a blood vessel is seen at its baseQ. • Operation is completed by closing the anterior gastrotomy and performing a truncal vagotomy & pyloroplastyQ to reduce acid secretion. Prophylaxis: • Complete neutralization of luminal acid or antisecretory therapy precludes the development of experimental stress gastritisQ. • Sepsis control, ventilatory support, adequate nutrition and correction of dyselectrolytemiaQ • Drugs used are: Antacids, H2-receptor antagonists and sucralfate

GASTRIC VOLVULUS Gastric Volvulus • Organoaxial (two thirds): Torsion occurs along the stomach’s longitudinal axisQ • Mesenteroaxial (one third): Torsion occurs along the vertical axisQ Primary Gastric Volvulus • Seen in association with congenital asplenia & wandering spleenQ • Usually mesenteroaxialQ • Partial (13 mm − Channel length of at least 17 mmQ • Barium Meal: −− String signQ: indicating a narrowed elongated pyloric canal that does not relax is seen (most specific sign) −− Shoulder signQ: caused by hypertrophied muscle indenting the antrum −− Double-track signQ: caused by redundant mucosa

Gastrointestinal Surgery

• • • • •

332

Surgery Essence Treatment: • Pyloric stenosis is never a surgical emergency although dehydration and electrolyte abnormalities may present a medical emergencyQ • Fluid resuscitation and correction of electrolyte abnormalitiesQ and metabolic alkalosis is essential before surgery. −− It is important that the underlying metabolic alkalosis is slowly corrected with normal saline. −− Treatment of HPS is by a Ramstedt-Fredet pyloromyotomy (cutting across the abnormal pyloric musculatureQ while preserving the underlying mucosa).

Gastrointestinal Surgery

Postoperative care: • • • •

Postoperatively, infants are usually allowed to resume enteral feedings. Vomiting after surgery occurs frequently but is usually self-limitedQ. Complications: Incomplete myotomy, mucosal perforation (usually at the duodenal end), and wound infection. If the mucosa is inadvertently opened then feeding is delayed for 48 hoursQ.

DUODENAL ATRESIA Duodenal Atresia • Occurs as a result of failure of vacuolization of the duodenum from its solid cord stage. Anatomic variants of Duodenal Atresia Type I → Mucosal web with intact muscular wall (windsock deformity) Type II → Two ends separated by a fibrous cord Type III → Complete separation with a gap within the duodenum. Associated Anomalies • Prematurity, Down syndrome, polyhydramniosQ • Malrotation, annular pancreas, biliary atresiaQ • Cardiac, renal, esophageal, and anorectal anomalies Clinical Features: • In most cases, the duodenal obstruction is distal to the ampulla of Vater, and infants present with bilious emesis in the neonatal periodQ. Diagnosis • X-ray abdomen: Double-bubble sign (air-filled stomach and duodenal bulbQ). • Diagnosis is confirmed, if there is no distal airQ. • If distal air is present, an upper GI contrast study is performed rapidly, not only to confirm the diagnosis of duodenal atresia but also to exclude midgut volvulusQ. Treatment: • Diamond-shaped duodenoduodenostomy is the treatment of choiceQ. • 5 ‘D’s of duodenal atresia: DA, Down’s syndrome, Distal to ampulla, Double bubble sign, Duodenoduodenostomy

DUODENAL ADENOCARCINOMA

Section 3

Duodenal Adenocarcinoma • Duodenum is MC site of small intestinal adenocarcinomaQ • 15% in the proximal; 40% in the middle and 45% in the distal duodenum (MC)Q • Resectability and prognosis are better than other upper GI cancers. Clinical Features • Most often presents as obstruction, with nausea, vomiting, and abdominal pain as the most frequent symptoms. • Anemia from bleeding and biliary or pancreatic obstructive symptoms can also be seen on initial presentation if the tumor is located within the periampullary regionQ. Diagnosis • Endoscopy is the diagnostic test of choice for duodenal tumorsQ, which allows for direct visualization and biopsy as needed. Treatment • For 1st or 2nd portion: Whipple procedureQ • For 3rd or 4th portion: Segmental duodenal resectionQ

Multiple Choice Questions H. PYLORI

1. H. pylori has been implicated in all, except: (Orissa 2011) a. Gastric ulcer b. Gastric carcinoma c. Gastric lymphoma d. Gastric leiomyoma



2. Which of the following is a common gastric lesion associated with H. pylori and undergoes regression following eradication of the infection? (MHPGMCET 2009) a. Inflammatory polyp b. Metaplastic polyp c. Fundic gland d. True adenomas



3. Eradication of helicobacter pylori has been proved to be beneficial in which of the following disorders of the stomach? (COMEDK 2006) a. Low grade MALT lymphoma b. Erosive gastritis c. Carcinoma stomach d. Gastroesophageal disease 4. A patient of peptic ulcer disease. When investigated endoscopically showed chronic antral gastritis. Which of the following dye will be able to stain the specimen? (AIIMS Nov 2000) a. PAS b. Zeihl-Neilson stain c. Gramstain d. Warthin-Starry stain





5. H. pylori causes: a. Type A Gastritis c. Autoimmune

(TN 2001) b. Type B gastritis d. Allergic Gastritis



6. H. pylori infection causes carcinoma by which mechanism: a. Production of nitrosamines (TN 2003) b. Gastric metaplasia c. Increasing acid secretion d. Causing mutation



7. Eradication of H. pylori has been proved to be beneficial in the following condition except: (ICS 2005) a. Duodenal ulcer b. Gastric ulcer c. Low grade MALT lymphoma d. Hypertrophic gastritis



8. H. pylori infection is associated with development of which malignancy: (DPG 2011) a. MALTomas b. Atherosclerosis c. Sarcoma d. Gastrointestinal stromal tumor (GIST)



9. Urease breath test is used to diagnose in which bacteria?  (Recent Question 2018) a. Streptococci b. H. pylori c. C. jejuni d. Bacteroides



11. Which one is not associated with peptic ulcer? a. Smoking cigarette  (AIIMS Nov 95) b. Zollinger-Ellison’s syndrome c. Plummer-Vinson syndrome d. Cirrhosis



12. Which of the following factors contribute to the development of duodenal ulcers? (PGI June 2001) a. Lysolecithin b. Gastric acid c. Alcohol abuse d. Prostaglandins e. Smoking



13. Commonest site of peptic ulcer is: a. 1st part of duodenum b. 2nd part of duodenum c. Distal 1/3rd of stomach d. Pylorus of the stomach



14. The most common site of a benign (peptic) gastric ulcer is: a. Upper third of lesser curvature (AIIMS June 2004) b. Greater curvature c. Pyloric antrum d. Lesser curvature near incisura angularis



15. What is correct about duodenal ulcer? (Kerala 2003) a. 25% will occur if H. pylori is not eradicated b. Magnesium containing drugs may cause constipation c. Bismuth is not used for long terms d. None of the above

16. With reference to duodenal ulcers, consider the following statements: (UPSC 2007) 1. They occur most often in the second part of duodenum 2. Infection with H. pylori and NSAID - induces injury account for majority of duodenal ulcer 3. Malignant duodenal ulcers are extremely rare 4. Eradication of H. pylori has greatly reduced the recurrence rates in duodenal ulcers Which of the statements given above are correct? a. 1, 2 and 3 only b. 2, 3 and 4 only c. 1 and 4 only d. 1, 2, 3 and 4

17. Burning epigastric pain is due to: a. Vomiting b. Reflux esophagitis c. Duodenal ulcer d. Gastric ulcer



18. Increased gastric acid secretion occurs in: (AIIMS GIS May 2011) a. Type I gastric ulcer b. Type III gastric ulcer c. Type IV gastric ulcer d. All of the above



19. Prepyloric or channel ulcer in the stomach is termed as: (Recent Question 2016, COMEDK 2008) a. Type 1 b. Type 2 c. Type 3 d. Type 4



20. Gastric ulcer type III is located at: (Recent Question 2017) a. Lesser curvature b. Body c. Prepyloric region d. GE junction



21. Most common location for chronic gastric ulcer:  (Recent Question 2018) a. Antrum b. Fundus c. Greater curve d. Lesser curve

PEPTIC ULCER ETIOLOGY AND CLINICAL FEATURES

10. Peptic ulcer is associated with all except one: (AIIMS Feb 97) a. Cirrhosis b. Zollinger Ellison syndrome c. Primary hyperparathyroidism d. Pernicious anemia

(All India 99)

Gastrointestinal Surgery

334

Surgery Essence 22. A 30-year-old male presents to the emergency department with symptoms of epigastric pain radiating to back that wakes him up at night and is relieved by consuming food. He gives history of similar pain in the past which was diagnosed as perforated duodenal ulcer and treated with omental patch surgery on two occasions. Pain before and after surgery has been controlled with proton pump inhibitors and analgesics. The likely diagnosis on this occasion is: (All India 2011) a. Duodenal Ulcer b. Gastric Ulcer c. Atrophic Gastritis d. Chronic Pancreatitis

PEPTIC ULCER COMPLICATIONS







Section 3





23. A 30-year-old male presented with massive hematemesis. A 2 × 2 cm ulcer was visualized on upper GI endoscopy on the posterior aspect of first part of duodenum. The bleeding vessel was visualized but bleeding could not be controlled endoscopically. Blood transfusion was done and patient was planned for surgery. His BP was 90/70 and PR = 110/min with Hb = 9 gm% at the time of surgery. Which of the following would be best surgical management?  (All India 2012) a. Antrectomy with ligation of left gastric artery b. Duodenotomy with ligation of bleeding vessels with postoperative PPI c. Duodenotomy with ligation of bleeding vessels, truncal vagotomy and pyloroplasty d. Duodenotomy with ligation of bleeding vessels, highly selective vagotomy 24. Which of the following vessel is most commonly involved in hemorrhage from duodenal ulcer? a. IVC (Recent Question 2016, 2014, All India 2012) b. Gastroduodenal artery c. SMA d. Inferior pancreatico duodenal artery 25. Most common complication of chronic gastric ulcer is: a. Tea pot stomach (AIIMS June 93) b. Scirrhous carcinoma (Adenocarcinoma) c. Perforation d. Massive hematemesis 26. In gastric outlet obstruction in a peptic ulcer patient, the site of obstruction is most likely to be:  (All India 2002, AIIMS June 93) a. Antrum b. Duodenum c. Pylorus d. Pyloric canal 27. A Posteriorly perforating ulcer in the pyloric antrum of the stomach is most likely to produce initial localized peritonitis or abscess formation in the following: a. Omental bursa (lesser sac) (PGI June 2009, All India 2003) b. Greater sac c. Right subphrenic space d. Hepatorenal space (pouch of Morison) 28. A 60-year-old male had a sudden fall in toilet. His BP was 90/50 mm Hg and PR = 100/min. His relatives reported that he is a known case of hypertension and CAD and was regularly taking aspirin, atenolol and sorbitrate. The most likely diagnosis: (AIIMS May 2012) a. Gastric ulcer with bleeding b. Acute MI with cardiogenic shock c. Acute CVA d. Pulmonary embolism



29. Treatment of perforated peptic ulcer includes: (PGI Dec 2001) a. IV fluids b. Drainage of paracolic gutter c. Immediate surgery d. Antacids e. IV pantoprazole

30. Percentage of patients with perforated peptic ulcer who show free gas under the diaphragm: (UPPG 2009) a. 100% b. 75% c. 50% d. 90% 31. Commonest cause of death in peptic ulcer patients is:  (All India 90) a. Perforation b. Hemorrhage c. Pyloric stenosis d. Malignancy 32. Investigation of choice in peptic ulcer perforation is:  (KERALA 94) a. USG b. X-ray abdomen c. Paracentesis d. CT scan

33. About 6–8 hours after peptic ulcer perforation the disappearance of abdominal wall rigidity is due to: (UPSC 95) a. Cessation of acid secretion in the stomach b. Revival from initial shock c. Dilution of acid in the peritoneal cavity d. Fatigue of reflex arc



34. Posterior perforation of peptic ulcer drain into: (DNB 2009) a. Omental bursa b. Greater sac c. Foramen of Winslow d. Paracolic gutter



35. Prognosis in a case of duodenal perforation is determined by all except: (PGI 96) a. Age of the patient b. Duration of history c. Basal pneumonia d. Peritonitis



36. The vessel which needs to be ligated in a patient with a bleeding peptic ulcer is: (APPG 2015) a. Gastroduodenal artery b. Superior pancreatico-duodenal artery c. Left gastric artery d. Left gastroepiploic artery



37. In last decade, duodenal ulcer and its morbidity is reduced due to:  (Recent Question 2015) a. Lifestyle modification b. Eradication of H. pylori c. Proton pump inhibitors d. None of the above

PEPTIC ULCER DIAGNOSIS AND TREATMENT

38. PPI’s for peptic ulcer disease should be taken: (JIPMER 2011) a. Before breakfast b. After breakfast c. After lunch d. After dinner



39. Which of the following acid reducing surgery doesn’t require drainage procedure? (PGI Dec 2007) a. Highly selective vagotomy b. Billroth-I operation c. Antrectomy d. Gastric resection e. Truncal vagotomy

40. Highly selective vagotomy preserves: (MHPGMCET 2002) a. Nerves of Latarjet b. Nerve of Kuntz c. Nerve of Mayo d. All of the above

41. Stump of stomach and duodenum is present in:   (MHSSMCET 2006) a. Billroth-I operation b. Billroth-II operation c. Whipple’s operation d. Truncal vagotomy

Stomach and Duodenum 42. Pyloroplasty of choice when the DU is fibrosed and contracted? (MHSSMCET 2009) a. Finney’s pyloroplasty b. Billroth type I surgery c. Billroth type II surgery d. Ramsted’s operation



43. H. pylori is associated with _____% of gastric ulcers: a. 5 b. 20 (JIPMER 2011) c. 40–60 d. 80



44. Barium meal characteristic feature of malignant gastric ulcer is: (JIPMER November 2017) a. Hampton line b. Carman’s meniscus sign c. Ulcer cap d. Ulcer crater



45. A patient who has undergone partial gastrectomy presents with neurological symptoms. Most probable diagnosis:   (JIPMER 2011) a. Folic acid deficiency b. Thiamine deficiency c. Vitamin B12 deficiency d. Iron deficiency





46. Treatment of high lying ulcer near gastroesophageal junction is/are: (PGI Nov 2009) a. Pouchet procedure b. Kelling-Madlener operation c. Csendes procedure d. Total gastrectomy e. Vagotomy and pyloroplasty

47. In a highly selective vagotomy, the vagal supply is severed to: (COMEDK 2008) a. Proximal two-thirds of stomach b. Antrum c. Pylorus d. Whole of stomach (PGI Nov 2009)

48. Incorrect about gastric ulcer: a. Most common on lesser curvature b. 70% H. pylori related c. Type IV ulcer most common type d. Treatment is primarily medical e. 30% GU are associated with malignancy



49. True statement(s) regarding peptic ulcer disease: a. Anterior ulcer bleeds more commonly (PGI June 2009) b. Posteriorly perforated ulcer is always management conservatively c. Anti-H. pylori drugs must be included in the treatment regime d. H. pylori is known to increase incidence of gastric malignancies e. Increase acid production in prerequisite for gastric ulcer



50. All of the following drugs are used in the management of peptic ulcer except: (COMEDK 2005) a. Alginic acid b. Sucralfate d. Misoprostol d. Ipratropium



51. Surgery of choice for chronic duodenal ulcer is:  (Recent Question 2014, AIIMS June 93) a. Vagotomy + antrectomy b. Total gastrectomy c. Truncal vagotomy + pyloroplasty d. Highly selective vagotomy



52. Patient presents with recurrent duodenal ulcer of 2.5 cm size, procedure of choice: (All India 2001) a. Truncal vagotomy and antrectomy b. Truncal vagotomy and gastrojejunostomy c. Highly selective vagotomy d. Laparoscopic vagotomy and gastrojejunostomy

53. Lowest recurrence rate in duodenal ulcer treatment is seen with: (MHCET 2016, Recent Question 2014, AIIMS Nov 94, a. Highly selective vagotomy All India 2002) b. Truncal vagotomy c. Truncal vagotomy and antrectomy d. Truncal vagotomy and pyloroplasty



54. Endoscopy is useful in diagnosis of peptic ulcer in following situations except: a. Post bulbar ulcer b. Stomal ulcers c. Giant duodenal ulcer d. Duodenal erosion



55. All of the following are indications for surgery in a case of duodenal ulcer except: (UPCS 96) a. Acute perforation of ulcer b. Pyloric stenosis c. Massive hemorrhage d. Typical periodicity



56. Maximal reduction in gastric acidity is achieved by:  (Recent Question 2014, UPCS 97) a. Truncal vagotomy and pyloroplasty b. Truncal vagotomy and antrectomy c. Partial gastrectomy d. Highly selective vagotomy



57. Perforated peptic ulcer is treated by: a. Vagotomy + Pyloroplasty b. Vagotomy + Antrectomy c. Vagotomy + repair of perforation d. Graham’s repair



58. Lesser curvature anterior seromyotomy is indicated in: a. Gastric ulcer b. Gastric CA  (MAHE 2005) c. Duodenal blowout d. Duodenal ulcer



59. The most commonly practiced operative procedure for a perforated duodenal ulcer is:  (Recent Question 2014, Karnataka 2005) a. Vagotomy and pyloroplasty b. Vagotomy and antrectomy c. Vagotomy and perforation closure d. Graham’s omentum patch repair



60. In a highly selective vagotomy, the vagal supply is severed to: (COMEDK 2007) a. Proximal two-thirds of stomach b. Antrum c. Pylorus d. Whole of stomach



61. Treatment of choice in type III gastric ulcer is: (UPPG 2008) a. Vagotomy only b. Vagotomy and antrectomy c. Vagotomy and pyloroplasty d. Highly selective vagotomy



62. HSV is done in: (AIIMS GIS Dec 2006) a. Menetrier’s disease b. Giant gastric ulcer c. Gastric mucosal erosions d. Megaesophagus treatment by esophageal mucosal resection

(SGPGI 2005)

GASTRECTOMY AND COMPLICATIONS

63. Most common metabolic complication of gastrectomy: (AIIMS GIS Dec 2011) a. Iron deficiency anemia b. Megaloblastic anemia c. Hypocalcemia d. Osteoporosis

Gastrointestinal Surgery





Section 3



335

336

Surgery Essence 64. Most common abnormality after gastric resection and Billroth-II: (AIIMS GIS Dec 2009) a. Vitamin B12 deficiency b. Steatorrhea c. Calcium deficiency d. Vitamin D deficiency

Gastrointestinal Surgery





65. A patient of partial gastrectomy presents with neurological symptoms. Most probable diagnosis is: (JIPMER 2011) a. Folic acid deficiency b. Thiamine deficiency c. Vitamin B12 deficiency d. Iron deficiency

77. The operation where in the stump of the stomach is directly anastomosed to the stump of the duodenum is called: (Karnataka 96) a. Polya gastrectomy b. Hoffmeister gastrectomy c. Billroth-I gastrectomy d. Billroth-II gastrectomy



66. Gastric atony occurs in all except: (AIIMS GIS Dec 2006) a. Billroth-I b. Billroth-II c. HSV d. Posterior selective vagotomy with anterior seromyotomy

78. Gastrojejunostomy is an example of: a. Clean contaminated wound (DNB 2001, JIPMER 2008) b. Clean uncontaminated wound c. Unclean uncontaminated wound d. Unclean contaminated wound



79. Which is a clean surgery: a. Hernia surgery c. Cholecystectomy



80. Duodenal blowout is: a. Perforation of duodenal ulcer b. Iatrogenic c. Complication of partial gastrectomy d. Due to trauma



81. Postvagotomy diarrhea can be effectively managed by: (UPSC 2002) a. Steroids b. Thyroxin c. Somatostatin analogue d. Parathormone



82. The first gastrectomy was performed in 1881 by: a. Miculikz b. Wolfer (Bihar PG 2016) c. Billroth d. Moynihan



83. All are true about gastric resection except: a. Decreased protein absorption b. Increased intestinal secretion c. Calcium deficiency d. Increased intestinal motility



67. Long-term effects of gastrectomy includes: (PGI SS Dec 2009) a. Renal calculi b. Vitamin C deficiency c. Hypothyroidism d. Osteomalacia



68. Dumping syndrome is due to: (Recent Question 2017,  Recent Question 2015, All India 99) a. Diarrhea b. Presence of hypertonic content in small intestine c. Vagotomy d. Reduced gastric capacity









Section 3



69. Which is not true about dumping syndrome? (DNB 2014) a. Post vagotomy b. Small frequent meals is beneficial c. Starch is beneficial d. Clinical features include diarrhea and bloating 70. All are true regarding early post-cibal syndrome, except: a. Distension of abdomen  (All India 2000) b. Managed conservatively c. Hypermotility of intestine is common d. Surgery is usually indicated 71. Duodenal blow out following Billroth gastrectomy most commonly occurs on which day:  (Recent Question 2015, AIIMS June 93) a. 2nd day b. 4th day c. 6th day d. 12th day 72. In gastrectomy following occurs except: a. Calcium deficiency b. Steatorrhea c. Fe2+ deficiency d. Fluid loss

73. The earliest manifestation seen after gastrectomy:  (PGI Dec 2005) a. Incidence of infection b. Loss of storage capacity c. Loss of HCI d. Loss of intrinsic factor



74. The commonest earliest complication of TV and GJ is: a. Stomal obstruction b. Paralytic ileus (AIIMS 91) c. Gastric leak d. Anastomotic hemorrhage

75. Anemia is greater in which of the following gastric resection: (PGI 81, AMU 85) a. Billroth-II b. Billroth-I c. Both of the above are equal d. Neither of the above

76. A person who had undergone gastrojejunostomy suddenly develops severe diarrhea. Which should be suspected? (TN 95) a. Gastric carcinoma b. Tuberculosis of abdomen c. Gastrojejunocolic fistula d. Gastric amoebiasis

(APPG 97)

(DPG 2008)

UPPER GI BLEED

84. BLEED risk criteria include all except: (AIIMS GIS Dec 2009) a. Ongoing bleeding b. Low urine output c. BP < 100 mm Hg d. Altered mental status



85. In Forrest classification, high-risk of bleeding is associated with all except: (KGMC 2011) a. Visible vessel b. Visible pulsatile bleeding c. Adherent clot d. Visible oozing from vessel

(PGI Dec 97)



(Recent Question 2013) b. Gastric surgery d. Rectal surgery

86. In case of UGI bleeding, all are true about endoscopy except: (AIIMS GIS Dec 2009) a. Decreases transfusion requirement b. Leads to early discharge of the patient c. Can detect causes in all cases d. Best tool for localization of bleeding

87. Investigation of choice for UGI bleed:  (WBPG 2012, PGI SS 2004, June 97) a. Endoscopy b. Angiography c. CT d. Barium studies



88. Which of the following is the incorrect statement regarding GI bleeding? (AIIMS Nov 2004) a. The sensitivity of angiography for detecting GI bleeding in about 10–20% as compared to nuclear imaging b. Angiography can image bleeding at a rate of 0.05–0.1 ml/ min or less c. 99m Tc-RBC scan will image bleeding at rates as low as 0.05–0.1 ml/min d. Angiography will detect bleeding only, if extravasation is occurring during the injection of contrast

Stomach and Duodenum 89. True about upper GI bleeding: (PGI DEC 2003) a. Melena is the only symptom b. Bleeding occurs beyond the ampula of vater c. Endoscopy can best diagnose it d. Peptic ulcer is the MC cause e. ↑ed BUN





90. A male executive, 50 years of age is seen in seen in casualty with hypotension and hematemesis. There is previous history suggestive of alcohol intake of 100 ml daily. The blood loss is around 2 litres. Most probable diagnosis is: (AIIMS June 2001) a. Gastritis b. Duodenal ulcer c. Mallory-Weiss tear d. Esophageal varices

100. Most common cause of upper gastrointestinal tract bleeding is: (Recent Question 2015, 2014, 2013) a. Esophageal varices b. Peptic ulcer c. Gastritis d. Mallory weiss tear



91. A 42-year-old company executive presents with sudden upper GI bleed (5 litres) of bright red blood, with no significant previous history. The diagnosis is: (All India 2000) a. Esophageal varices b. Duodenal ulcer c. Gastritis d. Gastric erosion

92. Which of the following management procedures of acute upper gastrointestinal bleed should possibly be avoided? (AIIMS Nov 2003) a. Intravenous vasopressin b. Intravenous beta-blockers c. Endoscopic sclerotherapy d. Balloon tamponade 93. Following resuscitation, a patient with bleeding esophageal varices should be treated initially with: (AIIMS Nov 2004) a. Sclerotherapy b. Sengstaken Blackmore tube c. Propranolol d. Surgery



94. During sclerotherapy (by endoscopy), following are complications except: (PGI June 98) a. Hepatic encephalopathy b. Perforation c. Stenosis d. Fibrosis



95. A patient presented to emergency ward with massive upper gastrointestinal bleed. On examination, he has mild splenomegaly. In the absence of any other information available, which of the following is the most appropriate therapeutic modality? (AIIMS Nov 2005) a. Intravenous propranolol b. Intravenous vasopressin c. Intravenous pantoprazole d. Intravenous somatostatin

96. A patient comes with hematemesis and melena. On the upper GI endoscopy there was no significant finding. 2 days later the patient rebleeds. Next line of investigation is: a. Emergency angiography (AIIMS May 2007) b. Repeat upper GI endoscopy c. Enteroscopy d. Laparotomy



97. The most sensitive test to detect GI bleeding is: a. Selective angiography (Recent Question 2016) b. Radiolabelled erythrocyte scanning c. I-131 fibrinogen studies d. Stool for occult blood 98. In the Forrest classification for bleeding peptic ulcer with a visible vessel of pigmented protuberance is classified as: (Recent Question 2016, COMEDK 2006) a. FI b. FII a c. FII b d. FII c

101. Among the following, the least common cause of acute upper GI bleeding is: (APPG 2015) a. Vascular ectasia b. Mallory Weiss tear c. Ulcer d. Varices 102. Regarding Upper GI bleed, true statement is:  (Recent Question 2018) a. Most common cause is variceal bleeding b. It is bleeding upto ampulla of Vater c. Most commonly performed management is endoscopic banding d. Rockall scoring is used for risk stratification

MALLORY-WEISS TEAR 103. False about Mallory-Weiss syndrome: (ILBS 2011) a. Massive hemorrhage is MC manifestation b. Alcohol is an associated etiology c. Conservative treatment is effective in most of the cases d. Anti-reflux procedure doesn’t have added advantage 104. An old man presenting to the emergency following a bout of prolonged vomiting with excessive hematemesis following alcohol ingestion is likely to suffer from: (MCI June 2018) a. Mallory-Weiss syndrome b. Esophageal varices c. Gastric cancer d. Bleeding disorder 105. Mallory-Weiss syndrome is partial thickness rupture occurs at:  (Recent Question 2017, Recent Question 2016,  WBPG 2014, PGI Dec 97) a. Gastric cardia b. Esophagus mucosa c. Gastroesophageal junction d. Gastroduodenal junction 106. Violent vomiting after forceful retching present with sudden severe hematemesis diagnosis is: (Recent Question 2018) a. Haemangioma b. Carcinoma oesophagus c. Mallory-Weiss syndrome d. Esophageal varices 107. Bleeding from a Mallory Weiss tear occurs usually from:  (AIIMS May 2015) a. Phrenic vein b. Left gastric artery c. Short gastric arteries d. Coronary vein

DIEULAFOY’S LESION 108. All are true about Dieulafoy’s lesion except:  (AIIMS GIS Dec 2011) a. Angiographic embolization is the preferred treatment b. Endoscopic treatment can be given c. Pulsation of artery causes ulceration d. Submucosal artery

Gastrointestinal Surgery



99. A 45-year-old executive suddenly develops hematemesis at home. He is brought to the hospital 4 hours later, there he again has a bout of hematemesis. Total blood loss would be around 2 liters. Most likely diagnosis is: (AIIMS 2001) a. Gastritis b. Esophagitis c. Esophageal varices d. Duodenal ulcer

Section 3



337

338

Surgery Essence

Gastrointestinal Surgery

109. Dieulafoy’s lesion: a. Within 6 cm of GE junction b. In esophagus c. In ileum d. In rectum

(PGI SS June 2005)

115. A 50 years old male presented with the history of epigastric pain, anorexia, weight loss and pedal edema. On laboratory examination, total protein and albumin was low. Endoscopy was performed and the image is given below. What is the most probable diagnosis?

110. Dieulafoy’s lesion is:  (Recent Question 2016, MHSSMCET 2006) a. Prolapse gastropathy b. Gastric antral vascular ectasia c. Gastric hemorrhagic telengectasias d. Aberrant vessel in the mucosa that bleeds form a mucosal defect

GASTRIC ANTRAL VASCULAR ECTASIA 111. All are true about GAVE except: (JIPMER GIS 2011) a. Dilated submucosal venous plexus b. Bleeding is the most common presentation c. Pain is most common clinical symptom d. Argon laser treatment is established one 112. ‘Watermelon stomach’ is: (MHSSMCET 2008) a. Prolapse gastropathy b. Gastric antral vascular ectasia c. Gastric hemorrhagic telengectasias d. Aberrant vessel in the mucosa that bleeds form a mucosal defect 113. What is the most probable diagnosis on the basis of given endoscopy image? (Recent Question 2016) a. Dieulafoy’s lesion b. Gastric antral vascular ectasia c. Menetrier’s disease d. Mallory-Weiss syndrome

a. Gastric varices c. Hamartomatous polyp

b. Carcinoma stomach d. Menetrier’s disease

116. All are true about Menetrier’s disease except: a. Protein loss (AIIMS GIS Dec 2006) b. Hyperchlorhydria c. Cobblestone appearance of mucosa d. Associated with CMV and H. pylori 117. Thickened gastric folds are found in: (PGI June 2003) a. Lymphoma b. Menetrier’s disease c. Carcinoma d. Eosinophilic gastritis e. Giardiasis 118. Menetrier’s disease is characterized by all of the following except: (COMEDK 2006) a. Giant folds in the pyloric antrum b. Foveolar hyperplasia c. Hypoalbuminaemia d. Hypochlorhydria

GASTRIC POLYPS 119. Most common benign tumour of the stomach is: a. Adenoma b. Lipoma c. Hamartoma d. Leiomyoma

Section 3

120. The commonest gastric polyp is: (COMEDK 2008) a. Hyperplastic polyp b. Inflammatory polyp c. Adenomatous polyp d. Part of familial polyposis

MENETRIER’S DISEASE 114. Menetrier’s disease is characterized by all except:  (AIIMS GIS Dec 2011) a. Excessive protein loss b. Excessive mucus production c. Diarrhea d. Hyperchlorhydria

121. True regarding hyperplastic gastric polyp is:(PGI May 2018) a. Most common gastric polyp b. Pre-malignant c. Mostly non-Hodgkin’s lymphoma d. More common in young adults e. Surgery is done if they are symptomatic

CARCINOMA STOMACH PREDISPOSING FACTORS 122. All of the following increases the risk of CA stomach except: (PGI SS 2004) a. Benign ulcer b. Atrophic gastritis c. Previous gastric surgery d. Blood group A

Stomach and Duodenum

124. Risk factors for gastric cancer include: (PGI November 2017) a. Chronic atrophic gastritis b. Obesity c. Smoking d. H. pylori infection e. Menetrier’s disease 125. Gastric carcinoma is associated with blood group: a. A b. B (PGI SS June 2001) c. AB d. O 126. E-cadherin is more often mutated in: a. Diffuse type of gastric cancer b. Intestinal type of gastric cancer c. Malignant ulcer of stomach d. Erosive gastritis

(COMEDK 2010)

127. Which of the following anemia is a risk factor for the development of gastric carcinoma?  (DNB 2011, COMEDK 2007) a. Pernicious anemia b. Megaloblastic anemia c. Aplastic anemia d. Hemolytic anemia 128. Risk factor for development of gastric CA: (All India 2002) a. Blood group ‘O’ b. Duodenal ulcer c. Intestinal hyperplasia d. Intestinal metaplasia type III

130. Predisposing factors for gastric carcinoma are: (PGI June 2002) a. Atrophic gastritis b. Hyperplastic polyp c. Adenomatous polyp d. Achlorhydria e. Animal fat consumption 131. Which of the following is the most significant risk factor for development of gastric carcinoma? (All India 2006) a. Paneth cell metaplasia b. Pyloric metaplasia c. Intestinal metaplasia d. Ciliated metaplasia 132. All of the following predispose to gastric carcinoma except: (All India 1990) a. Achlorhydria b. ‘O’ blood group c. Pernicious anaemia d. Postgastrectomy 133. Precancerous condition of the stomach is: (Kerala 91) a. Lipoma b. Linnitis plastic c. Atrophic gastritis d. Hyperacidity 134. Malignant transformation is commonly seen in: (AIIMS 91) a. Stomal ulcer b. Gastric ulcer c. Chronic duodenal ulcer d. Postbulbar ulcer 135. All are true about gastric carcinoma except: a. More in low socioeconomic group b. Most common at fundus c. H. pylori infection increases risks d. Vitamin C protects

(DNB 2006)

136. Gastric malignancy is predisposed with: (Kerala 2004) a. Blood group O b. Intestinal metaplasia c. Gastric hyperplasia d. Duodenal ulcer

CA STOMACH CLINICAL FEATURES AND TREATMENT 138. All are true about diffuse carcinoma of stomach except: a. Occurs at distal end (PGI SS June 2001) b. Occurs at cardia c. Genetic rather than environmental cause d. Occurs in younger patients 139. According to Borrman’s classification, Linnitis plastica is:  (AIIMS GIS May 2011) a. Type I b. Type II c. Type III d. Type IV 140. All of the following are true about diffuse gastric cancer according to Lauren’s classification except: a. Familial (Recent Question 2017) b. More common in males c. Undifferentiated d. More common in the proximal part 141. Hereditary diffuse gastric carcinoma is associated with:  (Recent Question 2017) a. Ductal carcinoma NOS subtype b. Lobular carcinoma c. Ductal carcinoma in-situ d. Metaplastic carcinoma 142. Type 1 gastric cancer according to Bormann’s classification:  (Recent Question 2017) a. Protruding b. Ulcerated c. Flat d. Excavated 143. GE junction tumor is: a. Siewert type I c. Siewert type III

(Recent Question 2017) b. Siewert type II d. Siewert type IV

144. Not seen in intestinal type of gastric cancer:  (AIIMS GIS Dec 2011, Dec 2009) a. Decreased E-cadherin b. APC c. Microsatellite instability d. p53 145. Not true in case of diffuse carcinoma stomach: a. More common (AIIMS GIS Dec 2009) b. Poorly differentiated with signet ring cells c. Transmural or lymphatic spread d. Decreased E-cadherin 146. Not seen in intestinal type of gastric cancer:  (AIIMS GIS Dec 2009) a. Decreased E-cadherin b. APC c. p16 d. p53 147. All are true about diffuse gastric carcinoma (Lauren’s) except: (JIPMER GIS 2011) a. Blood group ‘A’ b. E-cadherin positive c. Hematogenous spread d. Common in females 148. Carcinoma stomach which perforates serosa but doesn’t involve nearby structures, the stage is: (KGMC 2011) a. I b. II c. IIIA d. IIIB 149. Diffuse and intestinal variant of CA stomach both have:  (AIIMS GIS Dec 2006) a. E-cadherin b. APC c. p53 d. Microsatellite instability

Gastrointestinal Surgery

129. Predisposing factor for CA stomach are all except: a. Chronic gastric atrophy  (PGI Dec 97) b. Hyperplastic polyp c. Metaplasia grade III intestine d. Pernicious anemia

137. AKT-1 amplification is seen in: (Recent Question 2016) a. CA bladder b. CA colon c. Breast cancer d. Gastric cancer

Section 3

123. Due to popularity of refrigeration reducing the need to preserve food, which cancer’s incidence has dramatically declined? (AIIMS May 2013) a. Esophagus b. Stomach c. Colon d. Oropharyngeal malignancies

339

340

Surgery Essence

Gastrointestinal Surgery

150. Gastric lymph node station no 5: (AIIMS GIS Dec 2009) a. Suprapyloric b. Splenic hilum c. Lessar curvature d. Greater curvature 151. Level 9 lymph node includes: a. Celiac nodes b. Splenic hilum c. Splenic artery d. Hepatoduodenal ligament

(Recent Question 2016, AIIMS GIS Dec 2010)

152. Sister Joseph’s nodule may indicated cancer of all the following except: (COMEDK 2004) a. Somach b. Large bowel c. Rectum d. Ovary 153. Which of the following statements about gastric carcinoma are true? (All India 2011) a. Squamous cell carcinoma is the most common histological subtype b. Often associated with hypochlorhydria/achlorhydria c. Occult blood in stool is not seen d. Highly radiosensitive tumor 154. Most common site of carcinoma of stomach is: (JIPMER 2010) a. Proximal stomach b. Gastric antrum c. Lesser curvature d. Greater curvature 155. An ulcero-proliferative lesion in the antrum of the stomach 6 cm in diameter, invading the serosa, with 10 enlarged lymph nodes around and pylorus with no distant metastasis, the TNM staging is: (COMEDK 2011) a. T2N1M0 b. T3N2M0 c. T4N1M0 d. T1N3M0 156. True about gastric stump carcinoma: a. Enterogastric reflex is the cause b. Prognosis good after surgery c. It is always adenocarcinoma in nature d. Diffuse type is only variety

(PGI Nov 2009)

157. All are true about gastric CA except: (PGI June 2009) a. H. pylori association is present b. D2 gastrectomy include total gastrectomy c. Surgical non curative lesion should not be resected d. Patient under total gastrectomy should be given vitamin B12 e. Hematemesis present in majority of patients 158. Troisier’s sign is: (MHPGMCET 2008, 2006, APPG 96) a. Metastatic left supraclavicular lymphadenopathy b. Carpopedal spasm in hypocalcemia c. Migratory thrombophlebitis d. Any of the above

Section 3

159. Independent risk factor for carcinoma stomach: a. H. pylori (MHPGMCET 2008) b. Gastrectomy with drainage procedure c. Vagotomy with drainage procedure d. All of the above 160. Irish node is most commonly seen in: (WB PG 2015) a. Ca stomach b. Ca lung c. Ca larynx d. CA endometrium 161. True about carcinoma stomach include all the following except: (MHSSMCET 2007) a. Smoky diet is risk factor b. Incidence is now decreasing in Japan and China c. Duodenal ulcers are not associated with gastric cancer d. Gastric adenocarcinoma is a radioresistant tumor 162. When carcinoma of stomach develops secondarily to pernicious anemia, it is usually situated in the:

a. Pre-pyloric region c. Body

b. Pylorus (All India 2006) d. Fundus

163. The best prognosis in carcinoma stomach is with:  (Bihar PG 2014, UPSC 2008, All India 95) a. Superficial spreading type b. Ulcerative type c. Linnitis plastica type d. Polypoidal type 164. All the following indicates early gastric cancer except: (Recent Question 2015, DNB 2006, All India 2002, AIIMS Feb 97) a. Involvement of mucosa b. Involvement of mucosa and submucosa c. Involvement of mucosa, submucosa and muscularis d. Involvement of mucosa, submucosa and adjacent lymph nodes 165. For early diagnosis of CA stomach, which method is used? a. Endoscopy (AIIMS Feb 97) b. Staining with endoscopic biopsy c. Physical examination d. Ultrasound abdomen 166. An adult presented with hematemesis and upper abdominal pain. Endoscopy revealed a growth at the pyloric antrum of the stomach. CT scan showed growth involving the pyloric antrum without infiltration or invasion into surrounding structures and no evidence of distant metastasis. At laparotomy neoplastic growth was observed to involve the posterior wall of stomach and the pancreas extending 6 cm up to tail of pancreas. What will be the most appropriate surgical management? (All India 2010) a. Closure of the abdomen b. Antrectomy and vagotomy c. Partial gastrectomy + distal pancreatectomy d. Partial gastrectomy + distal pancreatectomy + splenectomy 167. Kally, a 60 years old male diagnosed to have carcinoma stomach. CT scan of abdomen showed a mass measuring 4 × 4 cm in the antrum with involvement of celiac nodes and right gastric nodes. Management of choice is:  (AIIMS June 2001) a. Total gastrectomy b. Subtotal gastrectomy c. Palliative d. Chemotherapy 168. True about early gastric carcinoma: (PGI Dec 2002) a. Invasion of mucosa and submucosa with neighboring lymph node b. Invasion of mucosa and submucosa irrespective to L.N. spread c. Endoscopic removal of lesions d. Conservative gastrectomy 169. Operability in carcinoma stomach is indicated by all except: a. Involvement of omental nodes b. Involvement of lymph nodes at the celiac axis c. Lymph node at porta hepatis d. Solitary metastatic nodule in the liver e. Krukenberg tumor 170. Linnitis plastica is commonly seen in:  (Recent Question 2014, DNB 2005, 2001, 2000, All India 91) a. Carcinoma stomach b. Sarcoidosis c. Lymphoma d. Leiomyosarcoma 171. All of the following may be features of a silent carcinoma of the body of the stomach except: a. Obstructive jaundice b. Ascites c. Dysphagia d. Krukenberg tumours

Stomach and Duodenum

173. Gastric carcinoma involving the antrum with lymph node involvements. The pancreas, liver and peritoneal cavity are normal. Most appropriate surgery is: a. Total radical gastrectomy b. Palliative gastrectomy c. Gastrojejunostomy d. None of the above 174. Lymphatic drainage of CA stomach is mostly to: (DPG 2005) a. Left gastric b. Pyloric c. Celiac d. None of the above 175. Sister Mary Joseph nodule is most commonly seen with: (AIIMS May 2009) a. Ovarian cancer b. Stomach cancer c. Colon cancer d. Pancreatic cancer 176. Which of the following is not associated with this finding given in the image? a. Carcinoma pancreas b. Carcinoma stomach c. Testicular tumors d. Ovarian cancer

184. PDGFRA mutation is seen in: (Recent Question 2016) a. Aplastic anemia b. ITP c. GIST d. GI lymphoma 185. Imatinib used in treatment of:  (Recent Question 2017, Recent Question 2016) a. GIST b. GI lymphoma c. CA esophagus d. CA colon 186. Most common site of GIST: (Recent Question 2016) a Esophagus b. Stomach c. Small intestine d. Colon 187. False about GIST: a. Stomach is most common site b. Can present with bleeding c. Commonly metastasize to lymph nodes d. Can present with peritoneal metastasis 188. Sunitinib is used in: a. GIST c. Colonic carcinoma

Section 3

172. Presenting symptom of carcinoma stomach is: a. Bleeding b. Obstruction c. Perforation d. Weight loss

341

(KGMC 2011)

(KGMC 2011) b. Rectal cancer d. Pancreatic carcinoma

189. Tyrosine kinase inhibitor imatinib is used for the treatment of:  (JIPMER 2011) a. Fibrosarcoma phylloides b. GIST c. MALT d. Seminoma 190. A 50 years old male presents with obstructive symptoms. Biopsy of stomach reveals gastrointestinal stromal tumor (GIST). Most appropriate market for GIST is: (Recent Question 2017, Recent Question 2016, AIIMS May 2011) a. CD-34 b. CD-117 c. CD-30 d. CD-10 191. Gold standard investigation for recurrent gastrointestinal stromal tumor is: (AIIMS May 2011) a. MRI b. MIBG c. USG d. PET-CT

178. Locally invasive gastric carcinoma. Investigation of choice to know depth of cancer invasion: (Recent Question 2013) a. CECT b. MRI c. Barium d. EUS

192. True about GIST all except:  (Recent Question 2015, AIIMS Nov 2010) a. Most common in duodenum b. Necrosis and ulceration present c. PET is used to assess response to therapy d. Cell circumscribed

179. Most common cause of Krukenberg’s tumor is: (DNB 2014) a. Ovary b. Liver c. Stomach d. Kidney

GASTROINTESTINAL STROMAL TUMOR 180. Cell of origin in GIST: a. Mesenchymal c. Smooth muscle

b. Argentaffin d. Epithelial

(GB Pant 2011)

181. Which of the following is false about GIST? a. More common in female (Recent Question 2017) b. >5 cm in size is high-risk c. Mesodermal origin d. Treatment of choice is segmental resection 182. For high-risk cases of GIST of size >10 cm, imatinib therapy is given for: (Recent Question 2017) a. 1 year b. 2 years c. 3 years d. 5 years 183. Treatment of choice for localized GIST:  (Recent Question 2017) a. Segmental resection b. Total gastrectomy c. Distal gastrectomy d. Imatinib mesylate

193. Carney triad consists of: (PGI May 2011) a. Gastric carcinoma b. Paraganglioma c. Pulmonary chordoma d. Carcinoma bronchus e. Chondromatosis 194. GIST (Stromal tumors of GI tract) arise form: (MHSSMCET 2008) a. Paneth cells b. Stave cells c. Enterocytes d. Interstitial cells of Cajal 195. Commonest stomach tumour which bleeds: a. Adenocarcinoma b. Squamous carcinoma c. Leiomyosarcoma d. Fibrosarcoma 196. Most common type of gastric sarcoma: (MCI June 2018) a. Lipoma b. Glomus tumour c. Leiomyosarcoma d. Leioblastoma 197. Bleeding is seen maximally in which gastric tumors? (Punjab 2008) a. Adenocarcinoma b. Squamous cell carcinoma c. Leiomyosarcoma d. GIST

Gastrointestinal Surgery

177. Peritoneal dissemination of gastric cancer is best detected by: (COMEDK 2014) a. USG b. Laparoscopy c. CT d. MRI

342

Surgery Essence

Gastrointestinal Surgery

198. Which of the following is not true about Gastrointestinal Stromal Tumor (GIST)?  (AIIMS November 2014) a. Originates from interstitial cells of Cajal b. Most common mesenchymal tumour of gastrointestinal tract c. Prognosis depends on size d. ALK gene mutation is seen in most of the cases

GASTRIC LYMPHOMA 199. All are true about stomach lymphoma except: a. Most common type is NHL (GB Pant 2011) b. Large B cell type c. Chemosensitive d. Most common site is fundus 200. The commonest site of lymphoma in the gastrointestinal system is: (COMEDK 2007) a. Small bowel b. Stomach c. Large intestine d. Oesophagus 201. Treatment of gastric lymphoma includes: (PGI May 2011) a. Chemotherapy b. Radiotherapy c. Surgery d. Anti-H. pylori treatment e. Endoscopic resection 202. Indication of surgery in gastric lymphoma are all except: a. Bleeding (PGI May 2011, AIIMS Nov 2005, Nov 2002) b. Perforation c. Residual disease after chemotherapy d. Intractable pain 203. False about gastric lymphoma is: (AIIMS May 2008) a. Stomach is the most common site b. Associated with H. pylori infection c. Total gastrectomy with adjuvant chemotherapy is treatment of choice d. 5 years survival rate after treatment is 60% 204. The treatment of Hodgkin’s disease of stomach is: a. Gastric resection (Karnataka 89) b. Gastric resection and chemotherapy c. Purely medical d. None of the above 205. The following are true regarding primary gastric lymphoma except: (APPG 2016) a. More amenable to treatment than gastric adenocarcinoma b. H. pylori is implicated especially in MALT type c. Mostly of B cell origin d. Clinically can be easily differentiated from gastric adenocarcinoma by the presence of early satiety and prominent lymph node metastases

DUODENAL ATRESIA

Section 3

209. A newborn baby was brought with the history multiple episodes of bilious projectile vomiting. X-ray abdomen was done. What is the diagnosis? (Recent Question 2016) a. Duodenal atresia b. Jejunal atresia c. Ileal atresia d. Hypertrophic pyloric stenosis

206. Anomaly associated with duodenal atresia is: (DNB 2010) a. Down’s syndrome b. Duodenal adenomas c. Limb defects d. Autoimmune disorders 207. Antenatal double bubble appearance on ultrasound is due to: (Bihar PG 2014, PGI June 97) a. Diaphragmatic hernia b. Duodenal atresia c. Gastric volvulus d. Intussuception 208. Double Bubble sign is seen with: (Recent Question 2016,  PGI Dec 2006, DNB 2007, 2003, AIIMS May 2009) a. Pyloric stenosis b. Duodenal atresia c. Ileal atresia d. Esophageal atresia

210. Which is the treatment of choice for duodenal atresia?  (DNB, 2011, 2002 MHSSMCET 2005) a. Duodenoduodenostomy b. Duodenojejunostomy c. Bishop-Koop Procedure d. Gastroduodenostomy 211. Duodenal atresia is associated with: (Recent Question 2017) a. Down’s syndrome b. Patau’s syndrome c. Turner’s syndrome d. Edward’s syndrome

HYPERTROPHIC PYLORIC STENOSIS 212. Congenital hypertrophic pyloric stenosis is associated with: a. Hypokalemia (PGI May 2018, May 2011) b. Hypochloremic metabolic alkalosis c. Hypochloremic metabolic acidosis d. Hyperchloremic metabolic acidosis e. Hyperchloremic metabolic alkalosis 213. A 3 weeks old patient presenting with vomiting and failure to thrive is found to have pyloric stenosis. What should be the next step of management? (AIIMS May 2011) a. Its emergency so do pyloromyotomy immediately b. Fluid resuscitation may be delayed c. Correction of electrolyte disturbances d. Cardiopulmonary resuscitation 214. Investigation of choice to diagnose hypertrophic pyloric stenosis in infants is: (Recent Question 2015, 2014, COMEDK 2011) a. Contrast radiology b. Gastroscopy c. Ultrasound abdomen d. CT abdomen 215. All are seen in hypertrophic pyloric stenosis except:  (NEET Pattern, AIIMS GIS 2003) a. Hyponatremia b. Hypokalemia c. Metabolic acidosis d. Metabolic alkalosis 216. Hypertrophic pyloric stenosis presents as: a. Mass in epigastriumv (Recent Question 2014, GB Pant 2011) b. More common in girls c. Congenital d. Present at birth with bilious vomiting 217. Olive shaped mass on feeding is pathognomonic of: a. Hypertrophic pyloric stenosis (DNB 2005) b. Duodenal atresia c. Jejunal atresia d. Ileal stenosis 218. An infant after four weeks of birth was brought with the history multiple episodes of nonbilious projectile vomiting. X-ray abdomen was done. What is the diagnosis?

Stomach and Duodenum 227. For a patient of gastric outlet obstruction, the OPD fluid management is: (PGI June 2003) a. Normal saline b. Hypertonic saline c. Na+ bicarbonate to counteract aciduria d. Hypotonic saline without potassium e. Normal saline with potassium 228. True about hypertrophic pyloric stenosis is all except: a. Present at 4 weeks (DNB 2007) b. First born male is commonly affected c. Ramstedt operation is done d. Visible peristalsis is always seen

219. Ramsted’s operation is performed for: a. Hirschsprung’s disease b. CHPS (Bihar PG 2014, MHSSMCET 2005, Kerala 94) c. Duodenal atresia d. Anorectal malformation 220. What is the most characteristic of congenital hypertrophic pyloric stenosis? (All India 2003) a. Affects the first born female child b. The pyloric tumour is best felt during feeding c. The patient is commonly marasmic d. Loss of appetite occurs early

222. Metabolic abnormalities associated with infantile pyloric stenosis in early phase include all except: (DNB 2012) a. Hypokalemia b. Aciduria c. Hypochloremia d. None of the above 223. Make the diagnosis of a 26-day-old infant presenting with recurrent non-bilious vomiting with constipation and loss of weight: (AIIMS June 99) a. Esophageal atresia b. Choledochal cyst c. Ileal atresia d. Pyloric stenosis 224. If the mucosa was accidentally opened at operation (Ramstedt) it is wise not to feed the child orally for: (AMU 90) a. 12 Hours b. 24 Hours c. 48 Hours d. 1 week 225. Which of the following is not a feature of ultrasound in CHPS? (AIIMS Nov 2011) a. 95% sensitivity by ultrasound b. Thickness of pylorus > 4 mm c. Channel length > 16 mm d. High gastric residue 226. Hypochloremia, hypokalemia and alkalosis are seen in:  (DNB 2012, AIIMS June 2003) a. Congenital hypertrophic pylori stenosis b. Hirschsprung’s disease c. Esophageal atresia d. Jejunal atresia

229. String sign on barium meal is seen in: a. Duodenal atresia (Recent Question 2015) b. Intestinal obstruction c. Duodenal ulcer d. Congenital hypertrophic pyloric stenosis 230. The abdominal mass is palpable in …. region in hypertrophic pyloric stenosis.: (Recent Question 2018) a. Umbilical b. Right hypochondrium c. Epigastrium d. Right iliac fossa

GASTRIC OUTLET OBSTRUCTION 231. Persistent vomiting in G.O.O. causes: (PGI Dec 2002) a. Hyponatremic hyperchloremia occur b. Hypernatremia without hypochloremic alkalosis c. Hypokalemic metabolic alkalosis d. Paradoxical aciduria 232. When peptic ulcer leads to gastric outlet obstruction, the most likely site of obstruction is? (Orissa 2011) a. Antrum b. Pylorus c. Lesser curvature d. First part of duodenum 233. A patient complains of occasional vomiting of food particles eaten a few days age. His wife reports that his breath smells foul. The most likely diagnosis is: a. Pyloric obstruction b. Carcinoma stomach c. Carcinoma esophagus d. Achalasia cardia 234. The most common cause of gastric outlet obstruction in India is: (All India 2006) a. Tuberculosis b. Cancer of stomach c. Duodenal lymphoma d. Peptic ulcer disease 235. Regarding acute dilation of stomach which is incorrect: a. Occurs with fracture femur b. Occurs with plaster c. Resolves spontaneously without treatment d. Hypophosphatemia is to be avoided 236. Commonest operation done for peptic ulcer with gastric outlet obstruction is: a. Truncal vagotomy with pyloroplastry b. Highly selective vagotomy with pyloroplastry c. Truncal vagotomy with gastrojejunostomy d. Gastrojejunostomy 237. A 35 years old male who had chronic duodenal ulcer for the last six years presents with worsening of symptoms loss of periodicity of symptoms, pain on rising in the morning sense of epigastric bloating and postprandial vomiting. The most likely cause of the worsening of his symptoms is the development of: (DPG 2010, UPCS 96) a. Posterior penetration c. Carcinoma

b. Gastric outlet obstruction d. Pancreatitis

Gastrointestinal Surgery

221. In a case of hypertrophic pyloric stenosis, the metabolic disturbance is: (Recent Question 2017, Recent Question 2016,  Bihar PG 2014, JIPMER 2013, All India 2002) a. Respiratory alkalosis b. Metabolic acidosis c. Metabolic alkalosis with paradoxical aciduria d. Metabolic alkalosis with alkaline urine

Section 3

a. Duodenal atresia b. Jejunal atresia c. Ileal atresia d. Hypertrophic pyloric stenosis

343

344

Surgery Essence 238. A patient with antral carcinoma repeatedly vomits. Not seen is: (AIIMS 98) a. Acidosis b. Hypokalemia c. Hyponatremia d. Hypochloremia

Gastrointestinal Surgery

239. Most severe degree of alkalosis occurs in obstruction of: a. Cardiac end b. Pylorus (MAHE 98) c. Ileocaecal region d. Colon 240. All of the following are seen in chronic pyloric obstruction except: (MCI March 2010) a. Alkaline urine b. Acidic urine c. Hypochloremia d. Hypokalemia 241. Acute dilatation of stomach is not managed by: a. N/G tube aspiration (AIIMS June 97) b. Stop oral feeds c. Surgery d. Fluid and electrolyte balance 242. Which of the following is true about acute dilatation of stomach? (PGI Dec 2005) a. Dilatation of stomach seen on X-ray b. Presents with vomiting c. Aspiration d. Immediately open the abdomen e. Atony of stomach

BEZOARS 243. Bezoar in the stomach present as: (Punjab 2009) a. Melena b. Perforation c. GI obstruction d. Diarrhea 244. The following are the complications of trichobezoars except: a. Hematemesis b. Perforation and peritonitis c. Obstruction d. Malignancy 245. A female in her twenties presents with complaints of pain abdomen, abdominal distention and vomiting. On examination, she was found to have alopecia and a crepitus in the epigastrium. What is your diagnosis? a. Trichobezoar (AIIMS November 2014) b. Carcinoma pyloric antrum c. Intestinal tuberculosis d. Rectus sheath hematoma

Section 3

246. True about trichobezoars are all except: (MAHE 2006) a. It is caused by Trichuris b. It is a psychiatric manifestation c. Ball of hairs in the stomach d. Pulling the hair and sucking of hair is usually seen 247. True about gastric bezoar: (PGI November 2017) a. After gastrectomy, it does not occur b. Caused by ingestion of fruits, vegetable, fibers and hairs c. In Rapunzel syndrome, bezoar has a tail-like extension into the small bowel d. Surgery is always indicated e. It increases chance of gastric ulceration

STRESS GASTRITIS 248. In case of upper GI bleed associated with stress gastritis all are true except: (AIIMS GIS Dec 2009) a. Surgery should be done if transfusion requirements is > 6 units b. Vagotomy may be added

c. Surgery involves anterior gastrotomy with ligation of bleeding ulcers and superficial erosions d. Total gastrectomy is rarely indicated 249. Cushing ulcers are: (Recent Question 2019) a. Stress ulcers in burns b. Stress ulcers in head injury c. Stress ulcers in hiatus hernia d. Stress ulcers in analgesic drug abuse 250. In case of upper GI bleeding due to stress gastritis, all of the following decreases bleeding risk except: a. Treatment of sepsis (AIIMS GIS Dec 2009) b. Improvement of BP c. Elective ventilation d. Correction of coagulopathy 251. All are true about stress gastritis except: a. Backflow H+ uptake by mucosal cells b. Decreased HCO3 secretion c. Decreased mucosal blood flow d. Increased H. pylori infection

(GB Pant 2011)

252. Stress-induced ulcers are most commonly found in the: a. Fundus of stomach (COMEDK 2010) b. Antrum of stomach c. Pyloric channel d. First part of duodenum 253. Common sites of for Cushing ulcers include all of the following except: (All India 99) a. Esophagus b. Stomach c. 1st part of duodenum d. Distal duodenum 254. Most common site of Curling’s ulcer:  (Recent Question 2014, AIIMS Nov 2008) a. Ileum b. Stomach c. Duodenum d. Esophagus 255. Erosive gastritis commonly occurs at: a. Body b. Fundus c. Lesser curvature d. Antrum

(JIPMER 93)

256. Curling’s ulcer is seen in:  (NEET 2013, DNB 2008, All India 88) a. Burn patients b. Patients with head injuries c. Zollinger Ellison syndrome d. Analgesic drug abuse 257. Which of the following is not true of Curling’s ulcer? a. Seen in burn patients (Karnataka 96) b. Are solitary penetrating ulcer c. Are shallow multiple erosions d. Has also been described in children after head injury or craniotomy 258. Stress ulcers is caused by all of the following except?  (APPG 2006) a. Burns b. Cortisol therapy c. Penicillin therapy d. Pulmonary insufficiency 259. In a burn patient, the doctor is looking for curling ulcer. Which part should be examined? (AIIMS Nov 2013) a. 1st part of duodenum b. 2nd part of duodenum c. 3rd part of duodenum d. Junction between 2nd and 3rd part of duodenum

Stomach and Duodenum GASTRIC VOLVULUS

MISCELLANEOUS

261. Borchardt’s triad of acute epigastric pain violent retching and inability to pass a nasogastric tube is seen in patients with: (J & K 2005) a. Achalasia cardia b. Acute gastric volvulus c. Jejunogastric intussusceptions d. Hiatus hernia 262. Borchardt’s triad is characterized by the following except: a. Inability to pass Nasogastric tube (MHSSMCET 2008) b. Vomiting c. Shock d. Diarrhea

STOMACH ANATOMY AND PHYSIOLOGY 263. Criminal nerve of Grassi: a. Anterior branch of vagus at pylorus b. Anterior branch of vagus at cardia c. Proximal branch of posterior vagus d. Distal branch of posterior vagus

(ILBS 2011)

264. Electrical pacemaker of stomach is situated in:  (Karnataka 2001) a. Fundus b. Body c. Incisura Angularis d. Gastroesophageal junction

266. G-cells are present mostly in: a. Fundus b. Cardia c. Pyloric antrum d. Body

(TN 91)

267. In which of the following organs submucosal glands are present? (COMEDK 2004) a. Colon b. Anal canal c. Duodenum d. Stomach (COMEDK 2008) b. Suppression of appetite d. Suppression of sleep

269. Posterior gastric artery is a branch of: (JIPMER 2011) a. Left gastric artery b. Right gastric artery c. Splenic artery d. Hepatic artery

GASTRIC DIVERTICULUM 270. Deverticulum of the stomach: a. Pain is the main symptom b. Usually at cardiac end c. Usually on posterior surface d. Inversion is the satisfactory treatment e. All of the above

(PGI Nov 2014)

271. The most frequent symptom of gastric diverticulum is:  (Recent Question 2016) a. Epigastric lump b. Hematemesis c. Vomiting d. Pain

273. Deficiency of the abdominal muscle is associated with: (PGI June 99) a. Eagle-Barrett syndrome b. Christopher syndrome c. Megacystitis d. Megaureter 274. Raised gastrin level without associated increase in acid secretion is seen in: (AIIMS Nov 94) a. Carcinoma stomach b. Gastrinoma c. Pernicious anemia d. G-cell hyperplasia 275. Which is the most common cause of hypergastrinemia?  (MCI November 2017) a. Post-vagotomy b. After intake of PPI c. Resection of small intestine d. Atrophic gastritis 276. Hour glass stomach is seen in: (AIIMS 92) a. Gastric carcinoma b. Gastric ulcer c. Gastric lymphoma d. Corrosive strictures 277. Highest pickup for gastrojuejunocolic fistula is by:  (J and K 2001) a. Ba swallow b. Ba meal c. Ba enema d. Ba meal follow through 278. Duodenal adenocarcinoma: (PGI 2004) a. Most common small bowel carcinoma b. Type of periampullary carcinoma c. Jaundice and anemia - most common symptom d. Local resection - curative 279. For gastric lavage in an adult the stomach tube should be passed up to: (COMEDK 2004) a. Up to 20 cm b. Up to 30 cm c. Up to 40 cm d. Up to 50 cm 280. Thickened gastric folds on barium meal are seen in: (COMEDK 2010) a. Leiomyosarcoma b. Scleroderma c. Amyloidosis d. Linnitis plastica 281. Duodenal diverticularization is indicated in: (PGI Nov 2011) a. Duodenal fistula b. Duodenal polyp c. Duodenal ulcer d. Pancreatic injury e. Duodenal injury 282. Characteristic features of type A gastritis are all except: (MHPGMCET 2007) a. Gastric antrum is predominantly affected b. Chronic hypergastrinemia occurs c. Hypochlorhydria occurs d. Hypertrophy of gastric enterochromaffin like cells 283. Finney’s stricturoplasty is done when the length of bowel stricture is: (MHSSMCET 2006) a. > 1 cm b. > 10 cm c. < 10 cm d. < 2.5 cm 284. In Hunt Lawrence pouch reconstruction, the length of pouch is: (MHSSMCET 2008) a. 10 cm b. 15 cm c. 20 cm d. 25 cm 285. Most common epithelial tumor of stomach:  (Recent Question 2018) a. Carcinoid b. GIST c. Granulosa cell tumor d. Sarcoma

Gastrointestinal Surgery

265. Function of thick gastric mucosa is: (All India 97) a. Protects epithelium b. Neutralizes HCL c. Traps foreign particles d. None of the above

272. Duodenal stricture is caused by: (PGI Dec 2002) a. Amebiasis b. T.B. c. CA pancreas d. Crohn’s disease e. Giardiasis

Section 3

260. All are true about organoaxial gastric volvulus except: a. Borchardt’s triad is present (AIIMS GIS Dec 2006) b. Usually associated with diaphragmatic defect c. Endoscopy usually derotate d. Occurs in elderly

268. Ghrelin is responsible for: a. Stimulation of appetite c. Stimulation of sleep

345

Explanations H. PYLORI

1. Ans. d. Gastric leiomyoma (Ref: Harrison 20/e p2215, 19/e p915) Diseases Associated with H. pylori Antral Predominant Gastritis

Corpus Predominant Atrophic Gastritis

• Duodenal ulcerQ

Non-atrophic Pangastritis (Chronic Superficial gastritis)

• Gastric ulcerQ • Gastric adenocarcinomaQ

• MALT lymphomaQ



2. Ans. b. Metaplastic polyp



4. Ans. d. Warthin-Starry stain

3. Ans. a. Low grade MALT lymphoma



5. Ans. b. Type B gastritis (Ref: Bailey 27/e p1115; Harrison 20/e p2242, 19/e p372e-24t) Chronic Gastritis Type A

Type B

• Autoimmune etiologyQ • Circulating antibodies to the parietal cell results in the atrophy of the parietal cell mass, resulting in hypochlorhydria and ultimately achlorhydriaQ. • Associated with pernicious anemiaQ • Primarily involves body and fundusQ

• Associated with H. pylori infectionQ • Primarily involves antrumQ

• Type A: Autoimmune, Antibodies, Atrophy, Achlorhydria, Anemia • Type B: Bacteria

6. Ans. b. Gastric metaplasia

7. Ans. d. Hypertrophic gastritis



9 Ans. b. H. pylori (Ref: Sabiston 20/e p1200; Schwartz 10/e p1053; Bailey 27/e p1114)

8. Ans. a. MALTomas

PEPTIC ULCER ETIOLOGY AND CLINICAL FEATURES

10. Ans. d. Pernicious anemia (Ref: Sabiston 20/e p1918; Schwartz 10/e p1053-1073; Bailey 27/e p1116; Shackelford 8/e p673-676, 7/e p701-718) Specific Chronic Disorders Associated with PUD With strong associations • • • • • •

Systemic mastocytosisQ Chronic pulmonary diseaseQ Chronic renal failureQ CirrhosisQ NephrolithiasisQ Alpha1-antitrypsin deficiencyQ

With possible associations • • • •

HyperparathyroidismQ Coronary artery disease Polycythemia Vera Chronic pancreatitisQ



11. Ans. c. Plummer-Vinson syndrome

12. Ans. b. Gastric acid; c. Alcohol abuse; e. Smoking



13. Ans. a. 1st part of duodenum



14. Ans. d. Lesser curvature near incisura angularis (Ref: Sabiston 20/e p1207)

Gastric ulcers • • • • •

MC type: Type I gastric ulcer, is located near angularis incisura on the lesser curvature. NSAID ulcers (Type V) typically occur in the antrum but may be located anywhere in the stomach and may be multiple in origin. Type II and III: High acid secretionQ Type I and IV: Normal or low acid secretionQ Association: − Type I: Blood group ‘A’Q −  Type II, III, and IV: Blood group ‘O’Q

Stomach and Duodenum

347

15. Ans. d. None of the above (Ref: Harrison 20/e p2226, 19/e p1922)



16. Ans. b. 2, 3 and 4 only



17. Ans. c. Duodenal ulcer; d. Gastric ulcer (Ref: Harrison 20/e p2227, 19/e p1918)

Clinical Features of Peptic Ulcer

Section 3

• Documented eradication of H. pylori in patients with PUD is associated with a dramatic decrease in ulcer recurrence to 60 years of age due to increased use of NSAIDs in this group.



29. Ans. a. IV fluids; c. Immediate surgery; e. IV pantoprazole (Ref: Sabiston 20/e p1204; Schwartz 10/e p1068; Bailey 27/e p1126; Shackelford 8/e p695, 7/e p713-716)

Management of Perforated Peptic Ulcer • Perforated peptic ulcer usually presents as an acute abdomen. • Initially, a chemical peritonitis develops from the gastric and/or duodenal secretions, but within hours a bacterial peritonitis supervenes. • Fluid sequestration into the third space of the inflamed peritoneum can be impressive, and fluid resuscitation is mandatoryQ. • The patient is in obvious distress, and the abdominal examination shows peritoneal signs. • Usually, marked involuntary guarding and rebound tenderness is evoked by a gentle examination. Diagnosis • Upright chest x-ray shows free air in about 80%Q of patients.

Section 3

Treatment • Nasogastric aspiration, analgesia and antibiotics, resuscitated with isotonic fluid, antisecretary agents (PPI, H2 receptor antagonist) • Surgery is almost always indicatedQ. • Sometimes, the perforation has sealed spontaneously by the time of presentation, and surgery can be avoided if the patient is doing well. • Non-operative management in: −− Objective evidence that the leak has sealed (i.e., radiologic contrast study)Q −− Absence of clinical peritonitisQ.

30. Ans. b. 75% (Ref: Schwartz 10/e p1061) • Upright chest x-ray shows free air in about 80% of patients of perforated peptic ulcer.

Stomach and Duodenum 31. Ans. b. Hemorrhage

32. Ans. b. X-ray abdomen



33. Ans. c. Dilution of acid in the peritoneal cavity

34. Ans. a. Omental bursa



35. Ans. c. Basal pneumonia (Ref: Shackelford 8/e p695, 7/e p714)

36. Ans. a. Gastroduodenal artery



37. Ans. c. Proton pump inhibitors

PEPTIC ULCER DIAGNOSIS AND TREATMENT

38. Ans. a. Before breakfast (Ref: Sabiston 20/e p1201; Schwartz 10/e p971,972,979; Bailey 27/e p1119) PPIs are taken before breakfast.



39. Ans. a. Highly selective vagotomy (Ref: Sabiston 20/e p1205-1206; Schwartz 10/e p1093-1094; Bailey 27/e p1122; Shackelford 8/e p679, 7/e p720-729)



40. Ans. a. Nerves of Latarjet



41. Ans. a. Billroth-I operation (Ref: Sabiston 20/e p1208; Schwartz 10/e p1120; Bailey 27/e p1123; Shackelford 8/e p683, 7/e p733-737)



42. Ans. a. Finney’s Pyloroplasty (Ref: Sabiston 20/e p1206; Schwartz 10/e p1065; Shackelford 8/e p680, 7/e p895)

Section 3



349

Finney Pyloroplasty is performed in patients with a J-shaped stomach or extensive scarring and narrowing of a significant portion of the duodenal bulb.

Drainage Procedures in Peptic Ulcers • Heineke-Mikulicz Pyloroplasty: Most commonly performed drainage procedureQ. • Finney Pyloroplasty: Performed in patients with a J-shaped stomach or extensive scarring & narrowing of a significant portion of the duodenal bulbQ. • Jaboulay Gastroduodenostomy: It involves an anastomosis of distal end of stomach to the first & second portions of duodenum, done in severely scarred or deformed pylorus or duodenal bulbQ.

43. Ans. d. 80 (Ref: Sabiston 20/e p1197; Schwartz 10/e p1054; Harrison 20/e p2225, 19/e p1039)

Association of H pylori with Peptic ulcer • Sabiston: 75% GUs and 90% DUs



44. Ans. b. Carman’s meniscus sign



45. Ans. c. Vitamin B12 deficiency



46. Ans. a. Pouchet procedure; b. Kelling-Madlener operation; c. Csendes procedure (Ref: Sabiston 20/e p1208; Schwartz 9/e p913-916; Shackelford 8/e p692, 7/e p708-710)



47. Ans. a. Proximal two-thirds of stomach



48. Ans. c. Type IV ulcer most common type; e. 30% GU are associated with malignancy (Ref: Harrison 20/e p2226, 19/e p1915; Chandrasoma Taylor 3/e p582, 597) • MC type of gastric ulcer is Type I, is located near angularis incisura on the lesser curvatureQ. • H. pylori colonization is seen in 75-80% of patients with gastric ulcerQ. • A chronic duodenal ulcer never turns malignant, while less than 1% of chronic gastric ulcer may transform into carcinoma.



49. Ans. c. Anti-H. pylori drugs must be included in the treatment regime; d. H. pylori is known to increase incidence of gastric malignancies



50. Ans. d. Ipratropium (Ref: Sabiston 20/e p1194-1995, 1200-1201; Schwartz 10/e p1041-1044; Bailey 27/e p1119)

Drugs that Reduce Gastric Acid Secretion • Proton pump inhibitors • Muscarinic-3 receptors antagonistsQ • SomatostatinQ Q

• Histamine-2 receptor antagonistsQ • Prostaglandin E-1 receptor agonistsQ (Misoprostol)

• AlginatesQ are combined with antacids for use in reflux esophagitis because they are believed to increase adherence to esophageal mucosa.

Gastrointestinal Surgery

• Harrison: 30-60% GUs and 50-70% of DUs • Schwartz: 70-90% GUs and 90% DUs

350

Surgery Essence

51. Ans. d. Highly selective vagotomy 52. Ans. a. Truncal vagotomy and antrectomy



53. Ans. c. Truncal vagotomy and antrectomy



54. Ans. d. Duodenal erosion (Ref” Harrison 20/e p2228, 19/e p1881)

Gastrointestinal Surgery

Endoscopy • Fiber endoscopy is especially useful in visualizing postbulbar ulcers, giant duodenal ulcers, and stomal ulceration after partial gastrectomy, all of which can be missed by X-ray.

55. Ans. d. Typical periodicity



56. Ans. b. Truncal vagotomy and antrectomy



57. Ans. c. Vagotomy + repair of perforation (Ref: Sabiston 20/e p1202-1205; Schwartz 10/e p1068; Bailey 27/e p1123; Shackelford 8/e p691-692, 7/e p701-708) Complications



Treatment

Perforated

Type I: • Distal gastrectomy in stable patientsQ • Biopsy and patch closure in unstable patientsQ Type II and III: Patch closure in unstable patients (Truncal vagotomy with antrectomy in stable patients) Q

Obstruction

Truncal vagotomy with antrectomyQ (Rule out malignancy)

58. Ans. d. Duodenal ulcer (Ref: Shackelford 8/e p679, 7/e p704, 708; Schwartz 10/e p1068)

Operation for Peptic Ulcer • Hill and Baker procedure: Posterior truncal vagotomy with anterior HSVQ. • Taylor Procedure: Posterior truncal vagotomy with anterior lesser curve seromyotomyQ. The technique is very suitable for a laparoscopic approach.

59. Ans. d. Graham’s omentum patch repair



60. Ans. a. Proximal two-thirds of stomach



61. Ans. b. Vagotomy and antrectomy



62. Ans. d. Megaesophagus treatment by esophageal mucosal resection • Ménétrier’s disease is associated with hypochlorhydria (no use of HSV). • Giant gastric ulcers should be treated with truncal vagotomy + antrectomy. • Vagotomy, irrespective if truncal or highly selective, only has a temporary effect in stress ulceration, consisting of multiple, punctate, superficial erosions, confined initially to the proximal gastric mucosa.

Section 3

GASTRECTOMY AND COMPLICATIONS

63. Ans. a. Iron deficiency anemia (Ref: Sabiston 20/e p1212-1213; Schwartz 10/e p1090-1094; Bailey 27/e p1123; Shackelford 8/e p731, 7/e p757-765) MC metabolic complication after gastrectomy is iron deficiency anemia.



64. Ans. a. Vitamin B12 deficiency (Ref: Sabiston 20/e p1212; Schwartz 10/e p1094; Bailey 27/e p1124; Shackelford 8/e p731, 7/e p759-760)



65. Ans. c. Vitamin B12 deficiency



67. Ans. d. Osteomalacia



68. Ans. b. Presence of hypertonic content in small intestine (Ref: Sabiston 20/e p1212; Schwartz 10/e p1090-1092; Bailey 27/e p1123; Shackelford 8/e p719, 7/e p757-759)



69. Ans. c. Starch is beneficial



70. Ans. d. Surgery is usually indicated

66. Ans. c. HSV

• Surgery is rarely required in dumping or post-cibal syndrome, as most of the patients improve with time, dietary management and octreotideQ.

Stomach and Duodenum 71. Ans. b. 4th day (Ref: Shackelford 7/e p930, 944)



72. Ans. d. Fluid loss



73. Ans. b. Loss of storage capacity



74. Ans. d. Anastomotic hemorrhage



75. Ans. a. Billroth-II



76. Ans. c. Gastrojejunocolic fistula (Ref: Shackelford 6/e p1094, 1110; Bailey 26/e p1039)



77. Ans. c. Billroth-I gastrectomy



78. Ans. a. Clean contaminated wound (Ref: Sabiston 20/e p245; Schwartz 10/e p148)



79. Ans. a. Hernia surgery



80. Ans. c. Complication of partial gastrectomy



81. Ans. c. Somatostatin analogue (Ref: Schwartz 10/e p1093; Bailey 26/e p1039; Shackelford 8/e p811, 7/e p760)

Section 3



351

Postvagotomy Diarrhea • • • •

About 30% or more of patients suffer from diarrhea after gastric surgeryQ. For most patients, it is not severe and usually disappears within the first 3 to 4 monthsQ. Vagotomy is associated with alterations in stool frequencyQ. Truncal vagotomy results in increased frequency of daily bowel movements in 30-70% of patientsQ.

Treatment • Postvagotomy diarrhea resolves over time in most of the patientsQ. • If symptoms fail to resolve: −− CholestyramineQ (an anionic exchange resin that absorbs bile salts rendering them unabsorbable and inactive) −− OctreotideQ can be used for severe postvagotomy diarrhea

82. Ans. c. Billroth (Ref: Sabiston 20/e p1208; Schwartz 10/e p1120; Bailey 27/e p1120; Shackelford 8/e p683, 7/e p731-742)



83. Ans. b. Increased intestinal secretion

UPPER GI BLEED

84. Ans. b. Low urine output (Ref: Sabiston 20/e p1142; Schwartz 10/e p1060)



85. Ans. c. Adherent clot (Ref: Sabiston 20/e p1143; Shackelford 8/e p694, 7/e p711)



86. Ans. c. Can detect causes in all cases (Ref: Sabiston 20/e p1141-1142; Schwartz 10/e p1169; Bailey 27/e p1127)

Role of Endoscopy in Upper GI Bleed • Endoscopy: Foundation of diagnosis and management of patients with an upper GI bleedQ Early endoscopy (within 24 hours) results in • Reductions in blood transfusion requirementsQ • Decrease in the need for surgeryQ • Shorter length of hospital stayQ • In general, 20–35% of patients undergoing endoscopy will require a therapeutic endoscopic intervention, and 5-10% will eventually require surgeryQ. • Best tool for localization of the bleeding source is endoscopyQ • In 1–2% of patients with upper GI hemorrhage, the source cannot be identified because of excessive blood impairing visualization of the mucosal surface. • Aggressive lavage of the stomach with room temperature normal saline solution before the procedure can be helpfulQ.

Gastrointestinal Surgery



Surgery Essence

87. Ans. a. Endoscopy



88. Ans. b. Angiography can image bleeding at a rate of 0.05-0.1 ml/min or less (Ref: Sabiston 20/e p1152-1153) Selective angiography, using either the superior or inferior mesenteric arteries, can detect hemorrhage in the range of 0.5 to 1.0 mL/min.

Gastrointestinal Surgery

352

Mesenteric Angiography • Selective angiography, using either the superior or inferior mesenteric arteries, can detect hemorrhage in the range of 0.5 to 1.0 mL/minQ. • Only employed in the diagnosis of ongoing hemorrhageQ.

Section 3

• Particularly useful in identifying the vascular patterns of angiodysplasias. • It may also be used for localizing actively bleeding diverticulaQ. • Catheter-directed vasopressin infusion can provide temporary control of bleedingQ, permitting hemodynamic stabilization, although as many as 50% of patients experience rebleeding when the medication is discontinued. • It can also be employed for embolizationQ. • Complications: Hematomas, arterial thrombosis, contrast reactions, and acute renal failure.

Radionuclide Scanning • Radionuclide scanning with technetium-99m ( Tc)-labeled RBCs is the most sensitive but least accurate method for localization of GI bleedingQ. • With this technique, the patient’s own RBCs are labeled and reinjected. • The labeled blood is extravasated into the GI tract lumen, creating a focus that can be detected scintigraphicallyQ. • Initially, images are collected frequently and then at 4 hour intervals for up to 24 hours. 99m

Stomach and Duodenum

353

• The tagged RBC scan can detect bleeding as slow as 0.1 mL/min and is reported to be more than 90% sensitiveQ.



89. Ans. c. Endoscopy can best diagnose it; d. Peptic ulcer is the MC cause; e. ↑ed BUN (Ref: Sabiston 20/e p1202-1203; Schwartz 10/e p1064; Bailey 27/e p1127; Shackelford 8/e p694, 7/e p710-713)



90. Ans. b. Duodenal ulcer (Ref: Sabiston 20/e p1143) • Peptic ulcer is the most common cause of upper GI bleeding, present in one-half to two-third of patients with upper GI bleeding. Bleeding may be the initial presenting symptom in up to 10% of patients with peptic ulcer. Duodenal ulcer bleeding is more common than gastric ulcer bleeding. • Only 10–15% of ‘Heavy’ drinkers develop alcoholic cirrhosis (leading to esophageal varices).



91. Ans. b. Duodenal ulcer



92. Ans. b. Intravenous beta-blockers (RefL Sabiston 20/e p1149; Bailey 27/e p1127; Shackelford 8/e p1582, 7/e p710-713)

Section 3

• Unfortunately, the spatial resolution is lacking, and blood may move retrograde in the colon or distally in the small bowel. • Reported accuracy of localization is 40-60%, and it is particularly inaccurate in distinguishing right- from left-sided colonic bleedingQ.

• Pharmacotherapy in variceal bleeding consists of: −− Continuous IV infusion of somatostatin analogue octreotide −− Infusion of vasopressin or vasopressin plus nitroglycerine • Beta-blockers are used in secondary prevention of recurrent variceal bleed. • Beta-blockers have no role in the management of acute upper GI bleeding.

93. Ans. a. Sclerotherapy (Ref: Sabiston 20/e p1149; Shackelford 8/e p1582, 7/e p1597; Harrison 20/e p2196, 19/e p1890)

Variceal Bleeding • In addition to pharmacologic therapy endoscopy should be carried out as soon as possible • If varices are found they are treated with either endoscopic variceal ligation or sclerotherapyQ. • EVL is the treatment of choice for variceal bleedingQ. 94. Ans. a. Hepatic encephalopathy (Ref: Sabiston 20/e p1150; Schwartz 10/e p1223; Shackelford 8/e p1583, 7/e p1600-1601; Harrison 20/e p2196, 19/e p2064) Complications of Injection Sclerotherapy • Esophageal ulcerationsQ (May bleed or perforate) • MediastinitisQ

• Pleural effusionQ • Pulmonary edema • Late stricturesQ



95. Ans. c. Intravenous pantoprazole (Ref: Sabiston 20/e p1144; Schwartz 10/e p1143)



96. Ans. b. Repeat upper GI endoscopy (Ref: Sabiston 20/e p1144) • Sabiston says that “The cause of obscure-overt bleeding is often a common lesion that is missed on initial evaluation. Repeat upper and lower endoscopy is a valuable tool in identifying missed lesions because up to 35% patients have the bleeding source identified on second look endoscopy.”



97. Ans. b. Radiolabelled erythrocyte scanning (Ref: Sabiston 20/e p1152) • Radionuclide scanning with technetium-99m (99mTc)-labeled RBCs is the most sensitive but least accurate method for localization of GI bleedingQ.



98. Ans. b. FII a

99. Ans. d. Duodenal ulcer

100. Ans. b. Peptic ulcer

101. Ans. a. Vascular ectasia 102. Ans. d. Rockall scoring is used for risk stratification (Ref: Sabiston 20/e p1142; Schwartz 10/e p1060)

MALLORY-WEISS TEAR 103. Ans. a. Massive hemorrhage is MC manifestation (Ref: Sabiston 20/e p1145; Schwartz 10/e p1018-1020; Bailey 26/e p994; Shackelford 8/e p656, 7/e p768) Massive hemorrhage is rare in Mallory-Weiss syndrome.

Gastrointestinal Surgery



354

Surgery Essence 104. Ans. a. Mallory-Weiss syndrome 105. Ans. a. Gastric cardia

106. Ans. c. Mallory-Weiss syndrome

107. Ans. b. Left gastric artery (Ref: Harrison 20/e p2199, 19/e p277; Sabiston 20/e p1145; Schwartz 10/e p1145; Shackelford 8/e p656, 7/e p7/768) • “Mallory-Weiss tears are characterized by arterial bleeding. Most Mallory-Weiss tears stop bleeding spontaneously and supportive treatment is all that is required. If bleeding continues, infusion of vasoactive substances into the celiac artery or into the left gastric artery often obviates the need for operation.”

Gastrointestinal Surgery

DIEULAFOY’S LESION 108. Ans. a. Angiographic embolization is the preferred treatment (Ref: Sabiston 20/e p1146; Schwartz 10/e p1089; Bailey 27/e p1128; Shackelford 8/e p650, 7/e p769) • Initial attempts at endoscopic control are often successful. Application of thermal or sclerosant therapy is effective in 80100% of cases. In cases that fail endoscopic therapy, angiographic coil embolization can be successful. 109. Ans. a. Within 6 cm of GE junction

110. Ans. d. Aberrant vessel in the mucosa that bleeds form a mucosal defect

GASTRIC ANTRAL VASCULAR ECTASIA 111. Ans. c. Pain is most common clinical symptom (Ref: Sabiston 20/e p1146; Schwartz 10/e p1088; Shackelford 8/e p650, 7/e p653-654, 768-769) Most common clinical symptom is bleeding, not the pain. 112. Ans. b. Gastric antral vascular ectasia 113. Ans. b. Gastric antral vascular ectasia (Ref: Sabiston 20 /e p1146; Schwartz 10/e p1088)

MÉNÉTRIER’S DISEASE 114. Ans. d. Hyperchlorhydria (Ref: Sabiston 20/e p1231; Schwartz 10/e p1088; Bailey 27/e p1116) 115. Ans. d. Menetrier’s disease (Ref: Sabiston 20/e p1231; Schwartz 10/e p1088; Bailey 27/e p1116) • History of epigastric pain, anorexia, weight loss and pedal edema and low total protein and albumin levels with massive gastric folds in the stomach giving rise to cobblestone or cerebriform appearance, which is characteristically seen in Menetrier’s disease. 116. Ans. b. Hyperchlorhydria 117. Ans. a. Lymphoma; b. Ménétrier’s disease; c. Carcinoma; d. Eosinophilic gastritis (Ref: Chapman 4th/234; Harrison 19/e p1931) Causes of Thickened Gastric Folds Inflammatory • GastritisQ (Peptic, NSAIDs induced) • Infectious (CMVQ, histoplasmosis, syphilis) • GastrinomaQ • Acute pancreatitisQ • Crohn’s diseaseQ

• • • • • •

Infiltrative or Neoplastic LymphomaQ CarcinomaQ Pseudolymphoma Eosinophilic gastroenteritisQ AmyloidosisQ SarcoidosisQ

Others • Ménétrier’s diseaseQ • Gastric varices • Hypertrophic gastritisQ

118. Ans. a Giant folds in the pyloric antrum

GASTRIC POLYPS

Section 3

119. Ans. a. Adenoma (Ref: Sabiston 20/e p1215; Schwartz 10/e p1076-1077; Bailey 26/e p1045; Shackelford 8/e p764, 7/e p769)

Gastric Polyps • There are five types of gastric epithelial polyps inflammatory, hamartomatous, heterotopic, hyperplastic and adenoma. The first three types have negligible malignant potential. • MC gastric polyp is the hyperplasticQ or regenerative polyp, which frequently occurs in the setting of gastritis. • Polyps that are symptomatic, > 2 cm or adenomatous should be removed. • Among patients with FAP, gastric polyps (33–60%) are more common as compared to gastric adenomas (15%). • MC gastric polypQ: Hyperplastic • MC neplastic gastric polyp: Tubular

Stomach and Duodenum

355

120. Ans. a. Hyperplastic polyp 121. Ans. a. Most common gastric polyp, e. Surgery is done if they are symptomatic (Ref: Schwartz 10/e p1076-1077; Sabiston 20/e p1215)

122. Ans. a. Benign ulcer (Ref: Sabiston 20/e p1214; Schwartz 10/e p1074-1086; Bailey 27/e p1131; Shackelford 8/e p712, 7/e p774) 123. Ans. d. Hyperplastic polyps 124. Ans. a. Chronic atrophic gastritis, c. Smoking, d. H. pylori infection, e. Menetrier’s disease (Ref: Schwartz 10/e p1074-1086; Sabiston 20/e p1214; Bailey 27/e p1131) 125. Ans. a. A 126. Ans. a. Diffuse type of gastric cancer (Ref: Sabiston 20/e p1216; Schwartz 10/e p1216; Bailey 25/e p1068) 127. Ans. a. Pernicious anemia

128. Ans. d. Intestinal metaplasia type III

129. Ans. b. Hyperplastic polyp

130. Ans. a. Atrophic gastritis; c. Adenomatous polyp; d. Achlorhydria

131. Ans. c. Intestinal metaplasia

132. Ans. b. ‘O’ blood group

133. Ans. c. Atrophic gastritis

134. Ans. None of the above

Section 3

CARCINOMA STOMACH PREDISPOSING FACTORS

135. Ans. b. Most common at fundus 136. Ans. b. Intestinal metaplasia (Ref: Shackelford 8/e p712, 7/e p773-774; Schwartz 10/e p1074-1086) Precursor lesions of Carcinoma Stomach • Adenomatous gastric polyps • Chronic atrophic gastritis • Dysplasia

• Intestinal metaplasia • Ménétrier’s disease

Intestinal Metaplasia • Types: complete type I and incomplete types II and IIIQ • The risk for progression to gastric cancer is higher in type IIIQ metaplasia than in type I. • The types differ based on the patterns of mucin core protein (MUC) expression as well as cell type composition.

137. Ans. d. Gastric cancer

CA STOMACH CLINICAL FEATURES AND TREATMENT 138. Ans. a. Occurs at distal end (Sabiston 20/e p1216-1221; Schwartz 10/e p1074-1086; Bailey 27/e p1131; Shackelford 8/e p712, 7/e p774-778) 139. Ans. d. Type IV (Ref: Sabiston 19/e p1207; Schwartz 9/e p931; Bailey 27/e p1133) 140. Ans. b. More common in males (Ref: Sabiston 20/e p1216; Schwartz 10/e p1079) 141. Ans. b. Lobular carcinoma (Ref: Devita 10/e p338) 142. Ans. a. Protruding (Ref: Sabiston 20/e p1216; Schwartz 10/e p1079; Bailey 27/e p1133) 143. Ans. b. Siewert type II (Ref: Sabiston 20/e p1217) 144. Ans. a. Decreased E-cadherin

145. Ans. a. More common

147. Ans. c. Hematogenous spread

148. Ans. b. II

• T4a: Tumor invades serosa (visceral peritoneum)

146. Ans. a. Decreased E-cadherin

• Stage IIB: T1N3, T2N2, T3N1, T4aN0

149. Ans. c. p53 150. Ans. a. Suprapyloric (Ref: Sabiston 20/e p1219, 19/e p1210; Bailey 27/e p1135)

Lymph Node Stations 1. Right cardiac; 2. Left cardiac; 3. Lesser curvature; 4. Greater curvature; 5. Suprapyloric; 6. Infrapyloric 7. Left gastric; 8. Common hepatic; 9. Celiac; 10. Splenic hilus; 11. Splenic artery; 12. Hepatoduodenal ligament; 13. Retropancreatic; 14. Mesenteric root; 15. Transverse mesocolon; 16. Paraaortic

Gastrointestinal Surgery

• Type I: Presence of absorptive cells, paneth cells and goblet cells secreting sialomucins • Incomplete types: Presence of columnar and goblet cells secreting sialomucins, sulfomucins or both.

356

Surgery Essence 152. Ans. c. Rectum (Ref: Sabiston 20/e p1216; Schwartz 10/e p1069; Bailey 27/e p1135)

151. Ans. a. Celiac nodes

Gastrointestinal Surgery

Sister Mary Joseph nodule Gastrointestinal malignancies Gynecological malignancies • Gastric cancer (MC)Q  • Ovarian cancerQ  • Colonic cancerQ  • Uterine cancerQ • Pancreatic cancer (mostly body and tail)Q 153. Ans. b. Often associated with hypochlorhydria/achlorhydria 154. Ans. b. Gastric antrum • T4a: Tumor invades serosa (visceral peritoneum) • Stage IIIC: T4aN3, T4bN2, T4bN3

155. Ans. None

• N3a: Metastasis in 7-15 regional LNs

156. Ans. a. Enterogastric reflex is the cause (Ref: Bailey 26/e p1053) 157. Ans. b. D2 gastrectomy include total gastrectomty; c. Surgical non curative lesion should not be resected; e. Hematemesis present in majority of patients (Ref: Bailey 27/e p1136)

Carcinoma Stomach • • • • • •

H. pylori infection increases risk of gastric cancer by causing chronic gastritis, loss of gastric acidity and bacterial growth in stomach. Gastrectomy can be total or subtotal in D2 gastrectomy. Patient with incurable disease are not subjected to radical surgery. Treatment is palliative systemic chemotherapy. After gastrectomy, there is loss of parietal cell mass leads to vitamin B12 deficiency and replacement should be given routinely. MC symptom of carcinoma stomach is abdominal pain > weight loss. Hematemesis is very rare in carcinoma stomach.

158. Ans. a. Metastatic left supraclavicular lymphadenopathy

159. Ans. d. All of the above

161. Ans. b. Incidence is now decreasing in Japan and China

162. Ans. d. Fundus

160. Ans. a. Ca stomach

163. Ans. a. Superficial spreading type

Most Common site of Gastric Malignancies CA stomach

AntrumQ

CA stomach in pernicious anemia

FundusQ

Diifuse variety

FundusQ

Gastric lymphoma

AntrumQ

Burkitt’s lymphoma (by EBV)

Cardia or bodyQ

164. Ans. c. Involvement of mucosa, submucosa and muscularis (Ref: Sabiston 20/e p1222; Schwartz 10/e p1216; Bailey 27/e p1132)

Early Gastric Cancer • Adenocarcinoma limited to the mucosa and submucosa of the stomach, regardless of LN statusQ. • Approx. 10% of patients will have LN metastasisQ. • Cancer of the lesser curve is more common than cancer of the greater curvature. Treatment • Treatment options: Endoscopic mucosal resectionQ, limited surgical resection or gastrectomy • Overall curative rate with adequate gastric resection and lymphadenectomy is 95%Q • Best prognosisQ

Section 3

165. Ans. b. Staining with endoscopic biopsy (Ref: Sabiston 20/e p1218-1221; Schwartz 10/e p1217; Bailey 27/e p1135; Shackelford 8/e p712, 7/e p774-775)

Diagnosis of Carcinoma Stomach • Endoscopy is the best method to diagnose gastric cancer as it visualizes the gastric mucosa and allows biopsy for a histological diagnosis. • Chromoendoscopy helps identification of mucosal abnormalities through topical stainsQ. • Magnification endoscopy is used to magnify standard endoscopic field by 1.5-150 timesQ. • Narrow band imaging affords increased visualization of the microvasculatureQ. • Confocal laser endomicroscopy permits in vivo, three-dimensional microscopyQ including subsurface structures. • EUS is a tool for pre-operative staging and selection for neoadjuvant therapyQ.

Stomach and Duodenum

357

166. Ans. c. Partial gastrectomy + distal pancreatectomy (Ref: Sabiston 20/e p1221-1225; Schwartz 10/e p1221-1222; Bailey 27/e p1136; Shackelford 8/e p713-715, 7/e p775-778)

• Maintain a 5 cm margin proximally and distally to the primary lesion. • Because of the extensive lymphatic network of the stomach and the propensity for microscopic extension, the traditional surgical approach attempts to maintain a 5 cm margin proximally and distally to the primary lesion. • When the general oncologic goal of an R0 resection can be achieved by gastric preserving approach in mid and distal gastric cancer, partial gastrectomy is preferred over total gastrectomy. Treatment of Carcinoma Stomach according to site Proximal-third

• Extended gastrectomy, including the distal esophagusQ

Middle-third

• Total gastrectomy and D2 LN dissectionQ.

Distal-third

• Intestinal-type: Subtotal gastrectomy with D2 LN dissectionQ • Diffuse-type: Total gastrectomy with D2 LN dissectionQ

Section 3

Treatment of Carcinoma Stomach

• In subtotal gastrectomy the luminal extent of the resection comprises about 80% of the stomach. At the lesser curvature the resection should reach up to about 2 cm below the anatomic cardia. At the greater curvature the resection has to go beyond the right and left gastroepiploic arteries; the small remaining fundus is fed through the short gastric vessels from the splenic hilus. Indications for Adjuvant Chemoradiation in Carcinoma Stomach • T3, T4 or node positive cancersQ • Microscopically positive surgical marginsQ • Spleen preserving D2 resection is the recommended surgical approach for patients with potentially curable gastric cancer. • Splenectomy should be performed only in cases with intraoperative evidence of direct tumor extension into the spleen or when the primary tumor is located in the proximal stomach along the greater curvature or posterior wall of stomachQ. • Partial pancreatectomy should be performed only in cases of direct tumor extension to the pancreasQ.

168. Ans. b. Invasion of mucosa and submucosa irrespective to L.N. spread; c. Endoscopic removal of lesions; d. Conservative gastrectomy 169. Ans. a. Involvement of omental nodes; e. Krukenberg tumor (Ref: Schwartz 10/e p1226, 9/e p1068-1074; CSDT 11/e p1175)

Carcinoma Stomach • Bailey: “Involvement of other organ per se does not imply incurability, provided that it can be removed.” • Schwartz: “It should be strongly emphasized that many patients with positive lymph nodes are cured by adequate surgery. It should also be stressed that often lymph nodes that appear to be grossly involved with tumor turn out to be benign or reactive on pathologic examination. More than 15 resected lymph nodes are required for adequate staging. Therapeutic nihilism should be avoided and, in the low-risk patient, an aggressive attempt to resect all tumor should be made. The primary tumor may be resected en bloc with adjacent involved organs (e.g., distal pancreas, transverse colon, or spleen) during the course of curative gastrectomy.” • CSDT: “A solitary metastatic nodule in liver is also no indication against curable resection.” • Periumbilical node, Blumer’s shelf, and Krukenberg’s tumor represent incurable gastric cancer. 170. Ans. a. Carcinoma stomach (Ref: Robbins 9/e p772; Sabiston 20/e p1227) 171. Ans. c. Dysphagia 172. Ans. d. Weight loss 173. Ans. d. None of the above Subtotal gastrectomy should be done for resectable carcinoma stomach involving antrum. 174. Ans. c. Celiac 175. Ans. b. Stomach cancer 176. Ans. c. Testicular tumors (Ref: Sabiston 20/e p1216; Schwartz 10/e p1079; Bailey 27/e p1135) 177. Ans. b. Laparoscopy

Gastrointestinal Surgery

167. Ans. b. Subtotal gastrectomy

358

Surgery Essence 178. Ans. d. EUS (Ref: Schwartz 10/e p1080) • The best way to stage the tumor locally is via EUS, which gives fairly accurate (80%) information about the depth of tumor penetration into the gastric wall, and can usually show enlarged (> 5 mm) perigastric and celiac lymph nodes. 179. Ans. c. Stomach

Gastrointestinal Surgery

GASTROINTESTINAL STROMAL TUMOR 180. Ans. a Mesenchymal (Ref: Sabiston 20/e p1229-1230; Schwartz 10/e p1481-1485; Bailey 26/e p1054;Shackelford 8/e p951, 7/e p1028-1034) 181. Ans. a. More common in female (Ref: Sabiston 20/e p1229, 1230) “Gastric GISTs can manifest at any age, although most typically they manifest in patients older than 50 years. They generally have an equal male-to-female ratio or a slight male predominance.” (Sabiston 20/e p1229) “High-risk GISTs: Defined as >10 cm tumor, mitotic count >10/50 HPF, tumor >5 cm and mitotic count ≥ per 50 HPF, or tumor rupture.” (Sabiston 20/e p1230) 182. Ans. c. 3 years (Ref: Sabiston 20/e p1230) “The Scandinavian Sarcoma Group (SSG) XVIII trial compared an extended 36-month course of adjuvant imatinib versus a 12-month course after resection for high-risk GISTs (defined as >10 cm tumor, mitotic count >10/50 HPF, tumor >5 cm and mitotic count > per 50 HPF, or tumor rupture). Patients in the extended treatment arm had higher recurrence-free survival (65.6% versus 47.9%) and overall survival (92.0% versus 81.7%) at 5 years after surgery. The results of this trial have established a 3-year course as the standard of care after surgical resection of high-risk GIST.” (Sabiston 20/e p1230) 183. Ans. a. Segmental resection (Ref: Sabiston 20/e p1230; Schwartz 10/e p1084; Bailey 27/e p1140) “GIST: Wedge resection with clear margins is adequate surgical treatment.” (Schwartz 10/e p1084) 184. Ans. c. GIST

185. Ans. a. GIST 186.

Ans. b. Stomach

187. Ans. c. Commonly metastasize to lymph nodes

188. Ans. a. GIST 189.

Ans. b. GIST

190. Ans. b. CD-117

191. Ans. d. PET-CT

192. Ans. a. Most common in duodenum 193. Ans. a. Gastric carcinoma; b. Paraganglioma; c. Pulmonary Chordoma 194. Ans. d. Interstitial cells of Cajal 195. Ans. c. Leiomyosarcoma (Ref: www.ncbi.nlm.nih.gov/pubmed/3771120) Leiomyosarcomas that occur in the digestive tract can also cause gastrointestinal blockage or bleeding, which can manifest as blood in the stool. 196. Ans. c. Leiomyosarcoma • Gastrointestinal stromal tumours (GISTs) are the MC mesenchymal tumours of the GIT. • Formerly GISTs were commonly classified histologically as leiomyosarcomas; however, they are now known to arise from the interstitial cells of Cajal. • Majority of GISTs overexpress KIT and have characteristic mutations within the gene, which are the targets of drug treatment with tyrosine kinase inhibitors.

Section 3

• Leiomyosarcoma is a malignant tumour of smooth muscle differentiation and falls into a group of sarcomas that show complex karyotypic changes with no consistent recurrent genetic abnormality. • Upper GI bleeding is the MC clinical manifestation of GISTsQ, manifesting as hematemesis or melena in 40-65% of patients. Bleeding occurs because of an ulcer forming in the gastric mucosa overlying the tumor. • Bleeding is more commonly seen in GIST as compared to LeiomyosarcomaQ. 197. Ans. d. GIST • Upper GI bleeding is the most common clinical manifestation of GISTs, manifesting as hematemesis or melena in 40-65% of patients. • Bleeding occurs because of an ulcer forming in the gastric mucosa overlying the tumor. 198. Ans. d. ALK gene mutation is seen in most of the cases (Ref: Robbins 9/e p775-777; Sabiston 20/e p1229) ALK gene mutation is not seen in GIST.

Stomach and Duodenum

359

Anaplastic Lymphoma Kinase (ALK)

• The 2;5 chromosomal translocation is associated with approximately 60% anaplastic large-cell lymphomas (ALCLs). The translocation creates a fusion gene consisting of the ALK (anaplastic lymphoma kinase) gene and the nucleophosmin (NPM) gene: the 3’ half of ALK, derived from chromosome 2 and coding for the catalytic domain, is fused to the 5’ portion of NPM from chromosome 5. • The EML4-ALK fusion gene is responsible for approximately 3–5% of non-small-cell lung cancer (NSCLC). Also related to NeuroblastomasQ. • Germline mutations in the anaplastic lymphoma kinase (ALK) gene have recently been identified as a major cause of familial predisposition to neuroblastomaQ.

Section 3

• Anaplastic lymphoma kinase also known as ALK tyrosine kinase receptor or CD246 (cluster of differentiation 246) is an enzyme that in humans is encoded by the ALK gene.

GASTRIC LYMPHOMA 199. Ans. d. Most common site is fundus (Ref: Sabiston 20/e p1227-1228; Schwartz 10/e p1074-1084; Bailey 27/e p1140; Shackelford 8/e p960, 7/e p1035-1042 Most common site of gastric lymphoma is fundus is an incorrect statement. 200. Ans. b. Stomach 201. Ans. a. Chemotherapy; b. Radiotherapy; c. Surgery; d. Anti-H. pylori treatment (Ref: Sabiston 20/e p1228; Schwartz 10/e p1074, 1084; Bailey 27/e p1140; Shackelford 8/e p966, 7/e p1042) High-grade (aggressive) Stage I, II, III: Chemotherapy + RT Stage IV: Chemotherapy + RT Residual disease: Further chemotherapy or Surgery

• External beam radiotherapy: 30 Gy with 10 Gy boost • Chemotherapy regimens cyclophosphamide, doxorobucin, vincristine, prednisone (CHOP) ± rituximab • Rituximab is chimeric monoclonal antibody against CD-20, preferred for high grade MALT or DLBL. 202. Ans. d. Intractable pain (Ref: Sabiston 20/e p1228; Bailey 26/e p1054; Oxford Surgery 5/e p174) Indications of Surgery in Gastric Lymphoma • Failure of chemoradiationQ • HemorrhageQ

• ObstructionQ • PerforationQ

203. Ans. c. Total gastrectomy with adjuvant chemotherapy is treatment of choice 204. Ans. b. Gastric resection and chemotherapy (Ref: http://www.ncbi.nlm.nih.gov/pmc/articles/PMC2117016/) 205. Ans. d. Clinically can be easily differentiated from gastric adenocarcinoma by the presence of early satiety and prominent lymph node metastases

DUODENAL ATRESIA 206. Ans. a. Down’s syndrome (Ref: Sabiston 20/e p1870; Schwartz 10/e p1612,1615-1616; Bailey 27/e p1293; Shackelford 8/e p774, 7/e p811-813) 207. Ans. b. Duodenal atresia

208. Ans. b. Duodenal atresia

209. Ans. a. Duodenal atresia

210. Ans. a. Duodenoduodenostomy

211. Ans. a. Down’s syndrome (Ref: Sabiston 20/e p1870; Schwartz 10/e p1615; Bailey 27/e p133)

HYPERTROPHIC PYLORIC STENOSIS 212. Ans. a. Hypokalemia; b. Hypochloremic metabolic alkalosis (Ref: Sabiston 20/e p1869; Schwartz 10/e p1613-1614; Bailey 26/e p113-114; Shackelford 8/e p779, 7/e p813-816)

Gastrointestinal Surgery

Treatment of Gastric Lymphoma Low-grade MALT • Confined to gastric wall and no t(11:18) translocation: H.pylori • eradication therapy and re-evaluate at 12 months • Lymph node involvement and t(11:18) translocation: H.pylori eradication • therapy and re-evaluate at 3-6 months; if lymphoma persists: • −− Stage I: XRT −− Stage II: Chemotherapy + RT • Stage III or IV: H.pylori eradication therapy and Chemotherapy +/- RT

360

Surgery Essence 213. Ans. c. Correction of electrolyte disturbances

214. Ans. c. Ultrasound abdomen

215. Ans. c. Metabolic acidosis

216. Ans. a. Mass in epigastrium

217. Ans. a. Hypertrophic pyloric stenosis

Gastrointestinal Surgery

218. Ans. d. Hypertrophic pyloric stenosis (Ref: Sabiston 20/e p1869) Single bubble sign

• Congenital Hypertrophic Pyloric StenosisQ

Double bubble sign

• Duodenal atresiaQ, Annular pancreas

Triple bubble sign

• Jejunal atresiaQ

219. Ans. b. CHPS

220. Ans. b. The pyloric tumour is best felt during feeding

221. Ans. c. Metabolic alkalosis with paradoxical aciduria

222. Ans. b. Aciduria

223. Ans. d. Pyloric stenosis

224. Ans. c. 48 hours

225. Ans. d. High gastric residue

226. Ans. a. Congenital hypertrophic pyloric stenosis

227. Ans. e. Normal saline with potassium (Ref: Sabiston 20/e p1205; Schwartz 10/e p1069; Bailey 27/e p1130; Shackelford 8/e p779, 7/e p716-717) Fluid resuscitation requires replacement of the chloride and potassium deficiencies.

Gastric Outlet Obstruction • In cases of prolonged vomiting, patients may become dehydrated and develop a hypochloremic hypokalemic metabolic alkalosis secondary to loss of gastric juice rich in hydrogen, chloride, and potassium ionsQ. • In this setting, fluid resuscitation requires replacement of the chloride and potassium deficiencies in addition to nasogastric suction for relief of the obstructed stomachQ. 228. Ans. d. Visible peristalsis is always seen

229. Ans. d. Congenital hypertrophic pyloric stenosis

230. Ans. c. Epigastrium (Ref: Sabiston 20/e p1869; Schwartz 10/e p1614; Bailey 27/e p128) “Palpation of the pyloric “olive” tumor in the epigastrium by an experienced examiner is pathognomonic for HPS. If the olive is confirmed, no additional diagnostic testing is necessary.” (Sabiston 20/e p1869)

GASTRIC OUTLET OBSTRUCTION 231. Ans. c. Hypokalemic metabolic alkalosis; d. Paradoxical aciduria (Ref: Sabiston 20/e p1205; Schwartz 10/e p1069; Bailey 27/e p1130; Shackelford 8/e p779, 7/e p716-717) 232. Ans. d. First part of duodenum

233. Ans. a. Pyloric obstruction

234. Ans. b. Cancer of stomach

235. Ans. c. Resolves spontaneously without treatment

236. Ans. c. Truncal vagotomy with gastrojejunostomy

237. Ans. b. Gastric outlet obstruction

238. Ans. a. Acidosis

239. Ans. b. Pylorus (Ref: http://www.anaesthesiamcq.com/AcidBaseBook/ab7_2.php)

• Gastric alkalosis is most marked with vomiting due to pyloric stenosis or obstruction because the vomitus is acidic gastric juice onlyQ. • Vomiting in other conditions may involve a mixture of acid gastric loss and alkaline duodenal contentsQ and the acid-base situation that results is more variable. • Loss of alkaline small intestinal contents can even result in an acidosis if gastric acid secretion is suppressedQ.

Section 3

240. Ans. a. Alkaline urine 241. Ans. c. Surgery (Ref: Bailey 27/e p1142)

Acute Gastric Dilatation • • • • • •

This condition usually occurs in association with pyloroduodenal disorders or after surgery without nasogastric suctionQ. The stomach, which may also be atonic, dilates enormouslyQ. Often the patient is also dehydrated and has electrolyte disturbances. Failure to treat this condition can result in a sudden massive vomit with aspiration into the lungsQ. Treatment: Nasogastric suction with a large-bore tube, fluid replacement and treatment of the underlying conditionQ.

242. Ans. a. Dilatation of stomach seen on X-ray; b. Presents with vomiting; c. Aspiration; e. Atony of stomach

Stomach and Duodenum

361

BEZOARS Symptoms of Trichobezoar: Pain from gastric ulceration and fullness from gastric outlet obstruction with occasional gastric perforation and small bowel obstruction. 244. Ans. d. Malignancy 245. Ans. a. Trichobezoar 246. Ans. a. It is caused by Trichuris 247. Ans. b. Caused by ingestion…., c. In Rapunzel syndrome…., e. It increases… (Ref: Schwartz 10/e p1089; Sabiston 20/e p1233; Bailey 27/e p1142)

Section 3

243. Ans. c. GI obstruction (Ref: Sabiston 20/e p1233; Schwartz 10/e p1089; Bailey 27/e p1142; Shackelford 8/e p280, 7/e p805)

STRESS GASTRITIS 248. Ans. c. Surgery involves anterior gastrotomy with ligation of bleeding ulcers and superficial erosions (Ref: Sabiston 20/e p1211; Schwartz 10/e p1073-1074) 249. Ans. b. Stress ulcers in head injury (Ref: Schwartz 10/e p1090; Sabiston 20/e p1211; Bailey 27/e p1142) 250. Ans. c. Elective ventilation

251. Ans. d. Increased H. pylori infection

252. Ans. a. Fundus of stomach

253. Ans. d. Distal duodenum

254. Ans. b. Stomach

255. Ans. b. Fundus

256. Ans. a. Burn patients

• Cushing ulcer: Stress gastritis due to intracranial injury/increased ICPQ • Curling ulcer: After burn injury (> 35%); in the body and fundusQ; not in antrum and duodenum • Cameron ulcers or riding ulcers: Linear gastric erosions in hiatal herniasQ 257. Ans. b. Are solitary penetrating ulcer; d. Has also been described in children after head injury or craniotomy 258. Ans. c. Penicillin therapy

259. Ans. a. 1st part of duodenum

GASTRIC VOLVULUS 261. Ans. b. Acute gastric volvulus

262. Ans. b. Vomiting

STOMACH ANATOMY AND PHYSIOLOGY 263. Ans. c. Proximal branch of posterior vagus (Ref: Sabiston 20/e p1188-1189; Schwartz 10/e p1035-1040; Shackelford 8/e p675, 7/e p845)

Stomach Innervation • The extrinsic innervation of the stomach is both parasympathetic through the vagus and sympathetic through the celiac plexus. • The vagus nerve originates in the vagal nucleus in the floor of the fourth ventricle and traverses the neck in the carotid sheath to enter the mediastinum, where it divides into several branches around the esophagus. These branches coalesce above the esophageal hiatus to form the left and right vagus nerves. • At the GE junction, the left vagus is anterior, and the right vagus is posterior (LARP mnemonic)Q. Left vagus

Right vagus

• At the GE junction, left vagus is anteriorQ • Left vagus gives off the hepatic branch to the liverQ and then continues along the lesser curvature as the anterior nerve of LatarjetQ.

• At the GE junction, right vagus is posteriorQ • Criminal nerve of GrassiQ is the first branch of the right or posterior vagus nerve and is recognized as a potential etiology of recurrent ulcers when left undividedQ. • Right vagus also gives a branch off to the celiac plexusQ and then continues posteriorly along the lesser curvature.

Truncal vagotomy

• Performed above the celiac and hepatic branches of the vagiQ

Selective vagotomy

• Performed below the celiac and hepatic branches of the vagiQ

Highly selective vagotomy

• Performed by dividing the crow’s feet to the proximal stomach while preserving the innervation of the antral and pyloric parts of stomachQ.

Gastrointestinal Surgery

260. Ans. c. Endoscopy usually derotate (Ref: Sabiston 20/e p1232; Schwartz 10/e p1090; Shackelford 8/e p280, 7/e p876-878; Bailey 27/e p1142)

362

Surgery Essence 264. Ans. b. Body (Ref: Shackelford 8/e p262, 7/e p781) Shackelford “The gastric pacemaker, which is located in the body along the greater curvature, stimulates both the filling and mixing of food in the body and antrum.”

Gastrointestinal Surgery

Gastric Motility • Gastric pacemaker: Interstitial cells of Cajal (ICCs)Q • Location: In bodyQ along the greater curvature • ICCs are critical for the generation of sequential contractionsQ 265. Ans. a. Protects epithelium 266. Ans. c. Pyloric antrum (Ref: Sabiston 20/e p1190) Location

Cells

Gastric Body

MCD PIE (Mucus cells, Chief cells, D cells, Parietal cells, Interneurons and ECL cells)Q

Gastric antrum

MD GI (Mucus cells, D cells, G cells and Interneurons)Q

• Parietal cells secrete Ghrelin, Intrinsic factor, Leptin and Acid. (GILA)Q • Chief cells secrete pepsin and leptinQ • Stomach Histology (CMPE): Chief cells (44%) > Mucous cells (40%) > Parietal cells (13%) > Endocrine cells (3%)Q

Gastric Morphology • Muscularis mucosa is responsible for the rugaeQ that greatly increases surface area and also marks the microscopic boundary for invasive and non-invasive gastric carcinoma. • Submucosa is the strongest layerQ of the gastric wall. • Muscularis propria consists of three layersQ of smooth muscle. The middle layer is circular and is the only complete muscle layer of the stomach wall, this layer becomes progressively thicker toward the pylorus, where it becomes as a true anatomic sphincter.

Embryology of Stomach • The stomach arises as a dilatation in the tubular embryonic foregutQ • Assumes its normal asymmetric shape and position by the end of the 7th weekQ. • During the 6th to 10th week as the stomach enlarges it also rotates 90 degrees in a clockwise directionQ. 267. Ans. c. Duodenum (Ref: Sabiston 20/e p1245; Shackelford 8/e p820, 7/e p826) • Brunner’s Gland: Submucosal gland found in the duodenumQ 268. Ans. a. Stimulation of appetite (Ref: Sabiston 20/e p1192; Schwartz 10/e p22,1045,1047,1348; Shackelford 8/e p645, 7/e p645-646)

Ghrelin • Secreted by oxyntic cells in the fundus of the stomach • First gut peptide found to have orexigenic (appetite stimulating) propertiesQ. • Circulating levels of ghrelin are inversely related to BMI, adipose tissue mass and plasma insulin levelsQ.

Section 3

Q

Primary effects of Ghrelin • Motilin like effects on gastric motilityQ • Stimulates release of somatostatin and PPQ

269. Ans. c. Splenic artery (Ref: Sabiston 20/e p1188) • Posterior gastric artery is a branch of splenic arteryQ.

• Stimulates release of growth hormone from pituitaryQ

Stomach and Duodenum

363

GASTRIC DIVERTICULUM 271. Ans. d. Pain

MISCELLANEOUS 272. Ans. b. T.B., c. CA pancreas, d. Crohn’s disease: (Ref: Gastrointestinal Radiology 2ed/84) Causes of Duodenal Stricture • Chronic peptic ulcerQ disease • Crohn’s diseaseQ • TuberculosisQ

• • • •

CA head of pancreasQ Annular pancreasQ PancreatitisQ Cholecystitis

Section 3

270. Ans. b. Usually at cardiac end; c. Usually on posterior surface (Ref: Schwartz 10/e p1089)

273. Ans. a. Eagle-Barrett syndrome (Ref: Schwartz 10/e p1634)

Eagle-Barrett Syndrome or Prune-Belly Syndrome • Prune-belly syndrome describes the wrinkled appearance of the anterior abdominal wall that characterizes these patientsQ. • Also known as Eagle-Barrett syndrome and the triad syndrome because of its three major manifestationsQ. Characterized by • Extremely lax lower abdominal musculatureQ

• Dilated urinary tract including the bladderQ

• Bilateral undescended testesQ

• The incidence is significantly higher in malesQ. • Most significant comorbidity: Pulmonary hypoplasia (lead to death in the most severe cases) Q • Skeletal abnormalities include dislocation or dysplasia of the hip and pectus excavatumQ. Genitourinary Manifestation Major genitourinary manifestation: Ureteral dilatationQ Ureters are typically long and tortuous, and become more dilated distally. Ureteric obstruction is rarely presentQ. The dilatation is thought to be caused by decreased smooth muscle and increased collagen in the uretersQ. Approximately 80% of affected individuals have some degree of VUR, which can predispose to UTIQ.

• Most children have adequate renal parenchyma for growth and developmentQ. Treatment: • Ureteric surgery has no role unless an area of obstruction develops. • Bilateral orchidopexy can be performed in conjunction with abdominal wall reconstruction at 6-12 months of ageQ. 274. Ans. c. Pernicious anemia (Ref: Sabiston 20/e p1192) 275. Ans. d. Atrophic gastritis (Ref: Sabiston 20/e p1192) Causes of Hypergastrinemia

Ulcerogenic Causes (RAGS) • • • • •

Non-ulcerogenic Causes (PACH)

Retained excluded antrum Antral G-cell hyperplasia or hyperfunctionQ GastrinomaQ Gastric outlet obstructionQ Short-bowel syndromeQ Q

• • • • • •

Pernicious anemiaQ Antisecretory agents (PPIs)Q Atrophic gastritisQ Acid-reducing procedure (vagotomy) Chronic renal failureQ H. pylori infectionQ

276. Ans. b. Gastric ulcer

Hour-Glass Stomach • Hour-glass stomach is caused by cicatricial contraction of a saddle shaped ulcer at the lesser curvatureQ

Gastrointestinal Surgery

• • • • •

364

Surgery Essence Tea-Pot Stomach (Hand-bag Stomach) • Tea-pot stomach is caused by longitudinal shortening of gastric ulcer at the lesser curvature of stomach (stomach looks like tea-pot)Q 277. Ans. c. Ba enema

Gastrointestinal Surgery

278. Ans. a. Most common small bowel carcinoma; b. Type of periampullary carcinoma; c. Jaundice and anemia-most common symptom (Ref: Sabiston 20/e p1277; Schwartz 10/e p1160; Bailey 25/e p1076; Shackelford 8/e p804-806, 7/e p779) 279. Ans. d. Up to 50 cm (Ref: Narayan Reddy 20/e p431) • In adults, gastric lavage tube should be passed upto 50 cm markQ. 280. Ans. c. Amyloidosis 281. Ans. d. Pancreatic injury; e. Duodenal injury (Ref: Sabiston 20/e p1552-1553; Shackelford 8/e p1211, 7/e p688)

Management of Grade IV and V pancreatic injuries • Due to short of radical resection, other options designed to divert gastric, pancreatic, and biliary secretions away from the duodenum need to be considered for the management of patients with grade IV and V pancreatic injuriesQ. • These include duodenal diverticularization, pyloric exclusion or gastrojejunostomy, and triple-tube decompressionQ. • Duodenal diverticularization is accomplished by performing antrectomy and gastrojejunostomy to achieve gastric diversion, choledochostomy to divert bile if the ampulla is injured, tube duodenostomy for decompression of the duodenum, suture repair of any duodenal injuries, and extensive periduodenal and peripancreatic drainageQ. • When pancreatic injuries are associated with major duodenal injuries. Drainage or resection of the pancreas can be combined with suturing or stapling of the pylorus (pyloric exclusion procedure) to divert gastric flow from the duodenumQ. • Gastrointestinal continuity is then accomplished by gastrojejunostomy. • It is quite remarkable that gastroduodenal continuity is re-established 4 to 6 weeks after pyloric exclusion even when heavy nonabsorbable sutures or staples are used. • The pyloric exclusion procedure has largely replaced the duodenal diverticularization procedure, which entails antrectomy and gastrojejunostomy, as well as drainage and decompression of the duodenal injury and drainage of the pancreatic injury. 282. Ans. a. Gastric antrum is predominantly affected • Type A gastritis primarily involves body and fundusQ 283. Ans. b. > 10 cm (Ref: Shackelford 8/e p875, 7/e p895; Sabiston 19/e p1251) • Finney stricturoplasty is used for strictures > 10–15 cmQ. • Heineke-Mikulicz stricturoplasty is appropriate strictures < 10 cmQ in length. 284. Ans. b. 15 cm (Ref: Shackelford 8/e p773, 7/e p810; www.surgery.usc.edu/foregut/demeesterpub/235. Ans.pdf)

Hunt-Lawrence Pouch

Section 3

• Jejunal reservoir (Hunt-Lawrence pouch) is used for the treatment of microgastria. • Length of pouch: 12–15 cmQ  • Length of jejunal limb used: 35–40 cmQ 285. Ans. a. Carcinoid (Ref: Schwartz 10/e p1074) “Hyperplastic polyps which consist of elongated, branching, dilated glandular structures are the most common epithelial tumorsin the stomach (75–90 % of all gastric polyps). They are most likely associated with chronic gastritis and rarely transform into gastric cancer.” {Radiology Illustrated: Gastrointestinal Tract edited by Byung Ihn Choi (2014)/p126} “Carcinoid is the second most common epithelial tumor of the stomach and accounts for 11–41 % of all gastrointestinal carcinoids.” {Cytopathology in Oncology edited by Ritu Nayar (2013)/p128}

CHAPTER

11

Peritoneum

RETROPERITONEAL FIBROSIS Retroperitoneal Fibrosis • Characterized by proliferation of fibrous tissue in the retroperitoneumQ • Fibrosis is usually confined to central & paravertebral spaces between the renal arteries & sacrum and tends to encase the aorta, IVC & uretersQ. • Process usually begins at the level of aortic bifurcation & spreads cephaladQ upto renal artery generally. Etiology • Around 70% cases are primary or idiopathic (Ormond’s disease)Q • • • • •

Causes of Secondary (30%)Q Retroperitoneal Fibrosis Inflammatory conditions: CATHQ (Chronic pancreatitis, Actinomycosis, Tuberculosis, Histoplasmosis) Drugs: Methysergide (Most important), methyldopa, hydralazine, entacapone, beta-blockers, bromocriptine, phenacetin, amphetaminesQ (MAHE-BP) Malignancies: CA prostate, NHL, CA stomach, sarcoma & carcinoid tumor Autoimmune disorders: SLE, PAN & ankylosing spondylitis Radiation

Clinical Features • More common in males of 40-60 yearsQ. • Early symptoms are vague & non-specificQ (abdominal or flank pain, weight loss, malaise, & hypertension) • Obstructive uropathy (dysuria, frequency, fever due to secondary infection of hydroneprotic kidney) is the earliest and MC specific symptomQ. • Ureters are MC involved, MC site is lower third of ureter. • Partial or complete obstruction occurs in 75% patientsQ. Diagnosis: In absence of uremia, diagnosis is made by IVP. • IVP or RGP: −− Hydronephrosis with dilated tortuous upper ureter −− Medial pulling of ureters or pipestem uretersQ −− Extrinsic ureteral compressionQ • CT scan is IOC for retroperitoneal fibrosisQ. • MRI is IOC in cases of compromised renal function, because contrast cannot be given. Treatment • Primary, idiopathic retroperitoneal fibrosis: Ureteral stenting & immunosuppression (TAPS: Tamoxifen, Azathioprine, Penicillamine, Steroids)Q • Secondary retroperitoneal fibrosis: Midline transperitoneal ureterolysis with wrapping the ureter with omental flap or lateral retroperitoneal ureteral transpositionQ. TAPS: Tamoxifen, Azathioprine, Penicillamine, Steroids are used in primary retroperitoneal fibrosisQ.

SPONTANEOUS BACTERIAL PERITONITIS Spontaneous Bacterial Peritonitis • SBP is a common and severe complication of ascites characterized by spontaneous infection of the ascitic fluid without an intraabdominal sourceQ. • Prevalence of SBP is 10–30%Q in patients of ascites, with 20% in-hospital mortality rateQ. • MC organism in adults: E. coliQ >Klebsiella. • MC organism in children: Group A streptococciQ

366

Surgery Essence Mechanism

Gastrointestinal Surgery

• Bacterial translocation with gut flora traversing the intestine into mesenteric lymph nodes, leading to bacteremia & seeding of the ascitic fluidQ. • Predisposing Factors: Bowel preparation, metabolic alkalosis, dehydration and hypoproteinemiaQ • It seen more commonly in patients presenting with GI hemorrhageQ. Clinical Features • Patients with ascites may present with fever, altered mental status, elevated WBC countQ, and abdominal pain or discomfort, or they may present without any of these features. • High degree of clinical suspicion and peritoneal taps are important for making the diagnosis. Diagnosis • Presence of >250Q polymorphonuclear cells of ascitic fluid is consistent with SBP; with ascitic fluid culture growing single organism. • If more than two organisms are identified, secondary bacterial peritonitis due to a perforated viscus should be consideredQ. • Culture negative neutrocytic ascites is diagnosed, when an ascitic fluid PMN count of >250 is unaccompanied by a positive ascitic fluid cultureQ. • Culture negative neutrocytic ascites carries a similar prognosisQ to SBP and is managed similarly. Treatment • Treated with cefotaxime plus albuminQ • Norfloxacin decreases the incidence of SBPQ in patients with variceal bleeding; patients with low-protein ascites and patients with a prior history of SBP. • Repeat diagnostic paracentesis is indicated after 48 hours of appropriate antibiotic therapy only if there is a lack of clinical improvement or in cases of secondary bacterial peritonitisQ. Prognosis • Occurrence of SBP is an important landmark in natural history of cirrhosis with 1 and 2 year survival rate of 30% and 20%, respectivelyQ.

SECONDARY (ACUTE SUPPURATIVE) BACTERIAL PERITONITIS Secondary (Acute Suppurative) Bacterial Peritonitis • • • •

When bacteria contaminate the peritoneum as a result of spillage from an intra-abdominal viscusQ. Infection in secondary bacterial peritonitis is polymicrobialQ E. coli & Bacteroides are MC organismsQ. The species of organism isolated vary with the source of the initial process and the normal flora present at the site.

PERITONITIS ASSOCIATED WITH CHRONIC AMBULATORY PERITONEAL DIALYSIS (CAPD) Peritonitis Associated With Chronic Ambulatory Peritoneal Dialysis (CAPD)

Section 3

• Peritonitis is one of the MC complications of CAPDQ, occurring with an incidence of approximately one episode every 1 to 3 yearsQ. • Refractory or recurrent peritonitis is MC cause of technical failure of CAPDQ. • MC organism: Staphylococcus epidermidisQ (30–50%). Clinical Features • Patients present with abdominal pain, fever, and cloudy peritoneal dialysate containing >100 WBC/mm3, with >50% of the cells being neutrophilsQ. • Gram staining detects organisms only in 10–40% of casesQ. Treatment • CAPD associated peritonitis is treated by the intraperitoneal administration of antibiotics, usually a first-generation cephalosporinQ. • Overall, 75% of infections are cured by culture-directed antibiotic therapyQ. • Recurrent or persistent peritonitis requires removal of the dialysis catheter and resumption of hemodialysisQ.

Peritoneum

367

PNEUMOCOCCAL PERITONITIS

• Primary pneumococcal peritonitis may complicate nephrotic syndrome or cirrhosis in childrenQ. • Otherwise healthy children, particularly girls between 3–9 years of age, may also be affected, and it is likely that the route of infection is sometimes via the vagina and fallopian tubesQ. • At other times, and always in males, the infection is blood-borne and secondary to respiratory tract or middle ear diseaseQ. Clinical Features • • • •

Onset is sudden, earliest symptom is pain localized to the lower half of the abdomen. Temperature is raised to 39°C or more and there is usually frequent vomitingQ. After 24–48 hours, profuse diarrhea is characteristic and increased frequency of micturition (caused by severe pelvic peritonitis)Q On examination, abdominal rigidity is usually bilateral but is less than in most cases of acute appendicitis with peritonitis.Q

Section 3

Pneumococcal Peritonitis

Diagnosis • WBC count ≥ 30 000/μL with approximately 90% polymorphs suggests pneumococcal peritonitis rather than appendicitisQ. Treatment • Antibiotic therapy + Correcting dehydration and electrolyte imbalance + Early surgeryQ • Laparotomy or laparoscopy may be used. • If the exudate be odourless and sticky, the diagnosis of pneumococcal peritonitis is practically certainQ

INTRA-ABDOMINAL ABSCESS Intra-Abdominal Abscess • MC site of intra-peritoneal abscess: Pelvis • Right subhepatic space (lies between inferior surface of liver and hepatic flexure and transverse mesocolon) is the most dependent portion of the abdominal cavity in the recumbent positionQ. • Pelvic cavity is the most dependent area of the peritoneal cavity in the upright positionQ. Q

• High spiking fevers, chills, abdominal pain, anorexia, and delay of return of bowel functionQ in the postoperative patient are typical presenting signs and symptoms of intraperitoneal abscess. Diagnosis • CT scan: Investigation of choice for diagnosis of intra-abdominal abscessQ Treatment • Preferred treatment: CT guided percutaneous drainageQ • Operative drainage: If percutaneous drainage is not possible or contraindicated

Pelvic Abscess • Pelvis is the MC site of an intraperitoneal abscess because the vermiform appendix is often pelvic in position and the fallopian tubes are frequent sites of infectionQ. • A pelvic abscess can also occur as a sequel to any case of diffuse peritonitis and is common after anastomotic leakageQ following colorectal surgery Clinical Features • Most characteristic symptoms are diarrhea & passage of mucus in the stools. Q • Rectal examination reveals a bulging of the anterior rectal wallQ, which, when the abscess is ripe, becomes softly cystic. • Left to nature, a proportion of these abscesses burst into the rectum, after which the patient nearly always recovers rapidly. If this does not occur, the abscess should be drained deliberately. Diagnosis • If any uncertainty exists, the presence of pus should be confirmed by ultrasound or CT scanning with needle aspiration if indicatedQ. Treatment • In women, vaginal drainage through the posterior fornix (Posterior colpotomy) is often chosenQ. • In other cases, when the abscess is definitely pointing into the rectum, rectal drainage is employedQ.

Gastrointestinal Surgery

Clinical Features

368

Surgery Essence • Laparotomy is almost never necessaryQ. • Rectal drainage of a pelvic abscess is far preferable to suprapubic drainage, which risks exposing the general peritoneal cavity to infectionQ. • Drainage tubes can also be inserted percutaneously or via the vagina or rectum under ultrasound or CT guidanceQ.

Gastrointestinal Surgery

WOUND DEHISCENCE (BURST ABDOMEN) Wound Dehiscence (Burst Abdomen) • Serous or serosanguinous discharge from the wound is the first signQ of dehiscence • Most commonly observed between 6th and 8th post-operative dayQ (may occur at any time following wound closure) • Wound dehiscence is partial or total disruption of any or all layers of the operative wound. • Extrusion of abdominal viscera after rupture of all layers is known as eviscerationQ. Predisposing Factors for Wound Dehiscence Local Risk Factors

Systemic Risk Factors

• Inadequate closure (Most important) • Increased intra-abdominal pressure • Deficient wound healing due to: − InfectionsQ − SeromaQ −− HematomaQ − Presence of drainQ Q

• • • •

Old age ObesityQ ImmunosuppressionQ Systemic diseases: −− DiabetesQ −− Jaundice, SepsisQ Q

− UremiaQ − CancerQ

Management • Wound dehiscence without evisceration: Prompt elective closureQ of the wound • Wound dehiscence with evisceration: −− Wound is covered with moist towels −− Under GA, any exposed bowel or omentum is rinsed with RL containing antibiotics and then returned to abdomen −− Previous sutures are removed, wound is reclosed (Tension suturingQ)

DUODENAL STUMP BLOWOUT Duodenal Stump Blowout • Duodenal stump blowout is massive leakage from duodenal stump following Billroth-II gastrectomyQ. Clinical Features • It usually occurs on 4th to 7th post-operative dayQ. • Usually presents as sudden intense thoracoabdominal pain, sudden elevation in pulse and temperature or generalized deterioration of condition. Treatment

Section 3

• Adequate drainage must be instituted immediately, which is done by putting a catheter through an incision below the right costal marginQ. • TPN should be instituted and attention should be directed towards fluid and electrolyte therapyQ. • Fistula closure can be anticipated within 2–3 weeksQ.

MESENTERIC CYST Mesenteric Cyst • Mesenteric cyst is encountered most frequently in the 2nd decade of life • More common in women Types of Mesenteric cysts • Chylolymphatic (MC)Q • Simple (mesothelial) • Enterogenous

• Urogenital remnant • Dermoid (teratomatous cyst)

Peritoneum Enterogenous Cyst

• MC type, arises in congenitally misplaced lymphatic tissue that has no efferent communication with lymphatic systemQ • Arises most frequently in mesentery of ileumQ. • Thin wall of cyst, filled with clear lymph or chyle. • Occasionally, the cyst attains a great size. • Mostly unilocular and solitaryQ • Chylolymphatic cyst blood supply is independent from that of the adjacent intestineQ • Enucleation is possible without the need for resection of gutQ.

• Derived either from a diverticulum of the mesenteric border of intestine or from a duplication of intestineQ. • Thicker wall than a chylolymphatic cyst and it is lined by mucous membrane, sometimes ciliatedQ. • Content is mucinous and is either colorless or yellowish brown as a result of past hemorrhage. • Muscle in the wall of an enteric duplication cyst and adjacent bowel has a common blood supplyQ • Removal of the cyst always entails resection of the related portion of intestineQ.

Section 3

Chylolymphatic Cyst

369

Clinical Features • A painless abdominal swellingQ • Recurrent attacks of abdominal painQ with or without vomiting (temporary impaction of a food bolus in a segment of bowel narrowed by the cyst or possibly from torsion of the mesentery) • Acute abdominal pain may arises as a result of: Torsion, rupture, hemorrhage, infection. • Tillaux triad: Fluctuant swelling near the umbilicus + moves freely in a plane perpendicular to the attachment of the mesentery + zone of resonance around the cystQ. Diagnosis • CT scan: Investigation of choice for diagnosis of mesenteric cystQ • USG: Helpful in diagnosis Treatment • Chylolymphatic cysts: Enucleation is treatment of choiceQ • Enterogenous cyst: Resection & anastomosis is the treatment of choiceQ • Aspiration alone has a high rate of cyst recurrenceQ.

Pseudomyxoma Peritonei • Pseudomyxoma peritonei describes mucinous ascites arising from a ruptured appendiceal or ovarian adenocarcinomaQ. • MC site of primary: AppendixQ • Peritoneum becomes coated with a mucus-secreting tumor that fills the peritoneal cavity with tenacious semisolid mucus and large, loculated cystic massesQ. • Occurs most commonly in 40–50 years of age • Occurs with equal frequency in men and womenQ. Clinical Features • • • •

Patients are often asymptomaticQ until late in the course of their disease. On presentation, global deterioration in health long before the diagnosis is made Abdominal pain & distention and nonspecific complaints are common. Physical examination: A new hernia, ascites, distended abdomen with nonshifting dullnessQ and, occasionally, a palpable abdominal mass.

Diagnosis • CT (chest, abdomen & pelvis): Information regarding the diagnosis and the ability to resect the tumor completely or perform an adequate cytoreductionQ. • Preoperative colonoscopy: Differentiate a mucinous neoplasm of the appendix from that arising from the colonQ. • Often, the diagnosis is made at laparotomy (peritoneal cavity containing tenacious semisolid mucus and large, loculated cystic masses) Treatment • Cytoreduction (Resection of as much of the tumor as possible) + Intraperitoneal hyperthermic chemotherapy (IPHC)Q. • Operative management: Omentectomy, stripping of involved peritoneum, resection of involved organs and appendectomy with right hemicolectomyQ

Gastrointestinal Surgery

PSEUDOMYXOMA PERITONEI

370

Surgery Essence Prognosis • Adenomucinosis (Adenocarcinoma of appendix): Best survival rate (75% at 5 years) • Peritoneal mucinous carcinomatosis: Worst (14% at 5 years)

Gastrointestinal Surgery

ACUTE MESENTERIC LYMPHADENITIS Acute Mesenteric Lymphadenitis • Syndrome of acute right lower quadrant abdominal pain associated with mesenteric lymph node enlargement and a normal appendixQ. • Diagnosis is made upon exploration of the abdomen of a patient suspected of having acute appendicitis at which time a normal appendix and enlarged mesenteric lymph nodesQ are discovered. • Occurs most commonly in children and young adultsQ • Equal frequency in males and females. • Etiology often remains unknown, although some cases are associated with Yersinia infection of the ileum. • Yersinia enterocolitica has been associated with this syndrome in childrenQ.

MALIGNANT PERITONEAL MESOTHELIOMA Malignant Peritoneal Mesothelioma • • • •

MC primary malignant peritoneal neoplasm: Malignant mesotheliomaQ Results from malignant transformation of the simple squamoid epithelium covering the peritoneal cavity. More common in males, median age of presentation is 50 years. Most patients had exposure to asbestosQ.

Clinical Features Most patients present with abdominal pain and weight loss. Ascites is common and often intractable. Omentum may become diffusely involved with tumor, present as an epigastric mass. In contrast to pseudomyxoma peritonei, local invasion of intra-abdominal organs, such as the liver, intestine, bladder, and abdominal wall, can occur. • Encasement of bowel can create a malignant bowel obstruction. • • • •

Diagnosis • CT scan: Mesenteric thickening, peritoneal studding, hemorrhage within the tumor, and ascitesQ. Treatment • Complete surgical resection is usually not possible because of the extent of disease. Prognosis

Section 3

• Median survival is 30–60 months

Multiple Choice Questions a. Bile c. Gastric enzyme

RETROPERITONEAL FIBROSIS

1. Ormond’s disease is: (MHPGMCET 2009, 2007) a. Retractile testis b. Idiopathic retroperitoneal lymphadenopathy c. Idiopathic retroperitoneal fibrosis d. Idiopathic mediastinitis



2. Retroperitoneal fibrosis most commonly presents with: a. Pedal edema b. Ascites (JIPMER 2011) c. Ureteric obstruction d. Back pain



3. Most common organ involved in retroperitoneal fibrosis is:  (Recent Question 2014, AIIMS Nov 93) a. Aorta b. Ureter c. Inferior vena cava d. Sympathetic nerve plexus



4. Localized idiopathic fibrosis is seen in all of the following except: (UPSC 2001) a. Riedel’s struma b. Hypertrophic scar c. Sclerosing cholangitis d. Panniculitis



5. False about retroperitoneal fibrosis is:(Recent Question 2015) a. Ureter is most commonly involved b. More common in females c. Primary idiopathic form is called Ormond’s disease d. Corticosteroids are mainstay of treatment

PERITONITIS 6. All are true about SBP except:  a. In hospital mortality rate is 20% b. Neutrocytic ascites has better prognosis c. Norfloxacin is useful for prevention d. Cefotaxime is used for treatment



7. Emergency operation done in cases of:  (PGI Nov 2010) a. Volvulus b. Obstructed hernia c. Appendicular perforation with paralytic ileus d. Toxic megacolon e. Colonic perforation



8. Investigation for acute abdomen includes: (PGI May 2010) a. USG b. Multidetector CT c. Contrast enhanced CT d. X-ray abdomen e. Echocardiography



9. A post-op patient presents with peritonitis and massive contamination because of duodenal leak. Management of choice is: (AIIMS June 2001) a. Four quadrant peritoneal lavage b. Duodenostomy + Feeding jejunostomy + Peritoneal lavage c. Total parenteral nutrition d. Duodenojejunostomy



10. In which of the following condition, air under both sides of diaphragm is visualized? (PGI Dec 2001, June 2001) a. Perforated Meckel’s diverticulum b. Uterine rupture following illegal abortion c. Perforation of duodenal ulcer d. Liver abscess e. Appendicular perforation





12. A 30-year-old a male patient presented with abdominal pain, fever, nausea, vomiting and respiratory distress. His BP was found to be 80/40 mm Hg and pulse rate of 115/min. The following chest X-ray was recorded in emergency. What is the immediate management? (AIIMS May 2016) a. IV fluids and antibiotics followed by laparotomy 
 b. Immediate laparotomy 
 c. Intravenous fluids 
 d. Intravenous potassium



13. A 40 years old male was brought to emergency with severe abdominal pain. On examination, pulse rate was 112/minute and systolic BP was 80 mm Hg. Chest X-ray is given below. What is the most appropriate management?   (Recent Question 2019) a. Exploratory laparotomy b. Saline wash of stomach c. Intercostal tube drainage d. IV antibiotics



14. Most common cause of peritonitis in adult male is:  (Recent Question 2014, All India 93) a. Duodenal ulcer perforation b. Abdominal tuberculosis c. Enteric perforation d. Perforated appendix



15. Apart from Escherichia coli, the other most common organism implicated in acute suppurative bacterial peritonitis is: (Recent Question 2014, All India 2006) a. Bacteroides b. Klebsiella c. Peptostreptococcus d. Pseudomonas

(ILBS 2012)



11. Which of the following causes least irritation of the peritoneal cavity? (All India 99)

b. Blood d. Pancreatic enzyme

Gastrointestinal Surgery

372

Surgery Essence

16. Primary peritonitis with pneumococcus is associated with: a. Lymphomas b. Nephrotic syndrome c. Carcinoids d. None of the above



17. Generalized diffuse peritonitis has been compared to second and third degree burns of: a. 13% b. 30% c. 45% d. 60%



18. Early surgery is indicated in: a. Amoebiasis peritonitis b. Biliary peritonitis c. Typhoid peritonitis d. All





20. The commonest organism seen in peritonitis is: (TN 2001) a. Escherichia coli b. Clostridium welchii c. Staphylococci d. Klebsiella





21. Which of the following complications is not seen with peritonitis? (MAHE 2006) a. Renal failure b. Residual abscess c. Endotoxic shock d. Bone marrow suppression







22. A 25-year-old female presents with pyrexia for ten days, develops acute pain in periumbilical region spreading all over the abdomen. What would be the most likely cause?  (Recent Question 2014, UPSC 2007) a. Perforation peritonitis due to intestinal tuberculosis b. Generalized peritonitis due to appendicular perforation c. Typhoid enteric perforation and peritonitis d. Acute salpingo-oophoritis with peritonitis 23. Spontaneous peritonitis in cirrhosis patients; the polymorphonuclear cells are: (NEET 2013, UPPG 2008) a. More than 200 cells/cumm b. More than 300 cells/cumm c. More than 400 cells/cumm d. More than 500 cells/cumm 24. Which of the following is true regarding classical spontaneous bacterial peritonitis? (NEET 2013, COMEDK 2010) a. Ascitic fluid neutrophil count is 250/cumm b. Bowel perforation should be present c. Multiple organisms are isolated from ascetic fluid d. Board-like rigidity is present in abdomen



25. A 10-year-old female who used to use the swimming pool regularly, comes with a three day history of vomiting, fever and abdominal pain. On examination, abdominal tenderness and guarding are present. The liver dullness is not obliterated. Likely diagnosis is: (AIIMS 99) a. Gangrenous intussusceptions b. Perforation c. Spontaneous biliary peritonitis d. Primary peritonitis



26. Sonu, a 15-year-old girl, a regular swimmer presents with sudden onset of pain in abdomen, abdominal distension and fever of 39°C and obliteration of the liver dullness. Most probable diagnosis is:  (Recent Question 2014, AIIMS June 2001, Nov 99) a. Ruptured typhoid ulcer b. Primary bacterial peritonitis c. Ruptured ectopic pregnancy d. UTI with PID

27. All of the following regarding diagnosis of acute peritonitis are correct except: (MCI March 2007) a. Raised WBC count in peritoneal aspirate b. Moderately raised amylase levels are diagnostic of peritonitis c. CT scan may aid in diagnosis d. Upright films shows free air under the diaphragm

MESENTERIC CYST





Section 3

19. Most common cause of generalized peritonitis in a 40-yearold adult male is: (AIIMS 92) a. Enteric perforation b. Ruptured liver abscess c. Duodenal ulcer perforation d. Perforated CA stomach





28. Most common type of mesenteric cyst is:(MHSSMCET 2005) a. Enterogenous b. Chylolymphatic c. Urogenital d. Teratomatous 29. True about mesenteric cyst: (PGI Dec 2005) a. Moves perpendicular to the line of attachment b. Teratomatous is most common c. Chylolymphatic cyst has separate blood supply d. Surgical removal of bowel along cyst is treatment of choice in all the cyst 30. Mesenteric cyst whose removal entrails removals of part of gut: (TN 95) a. Chylolymphatic cyst b. Enterogenous cyst c. Dermoid d. All 31. All are mesenteric cyst except: (DNB 2007) a. Dermoid cyst b. Chylolymphatic cyst c. Gartner’s cyst d. Enterogenous cyst 32. A part of adjacent intestine will be removed in: (JIPMER 2012) a. Enterogenous cyst b. Chylolymphatic cyst c. Dermoid cyst d. Mesothelial cyst

ASCITES











33. Transudative ascites is/are associated with: (PGI May 2011) a. Myxedema b. Budd-Chiari syndrome c. Acute pancreatitis d. Portal vein thrombosis e. Congestive heart failure 34. Serum-ascites albumin gradient >1.1 g/dL is seen in: (COMEDK 2005) a. Nephrosis b. Cirrhosis c. Pancreatic ascites d. Neoplasm 35. Mucinous ascites is seen in:  (PGI June 2000) a. Stomach CA b. TB c. Nephrotic syndrome d. Cirrhosis 36. The following are true regarding ascites except: a. Only when the amount of fluid present exceeds 1500 ml. It can be recognized clinically (Kerala 2000) b. Shifting dullness is absent when there is a very large accumulation of fluid c. In cirrhosis there is obstruction to the venous outflow of the liver due to obliterative fibrosis of the intra hepatic venous bed d. A transudate has a protein content of greater than 30 gms of protein per litre e. In Meig’s syndrome it is associated with pleural effusion and solid fibroma of ovary 37. Pseudochylous ascites occurs in: a. Cirrhosis b. Hyperlipidemia c. Filariasis d. Malignant ascites 38. A patient came with ascites. Ascitic fluid analysis was done and found to have SAAG more >1.1. All of the following can be the cause except:  (AIIMS November 2017) a. Cirrhosis b. Liver failure c. Metastasis to liver d. Tubercular peritonitis

Peritoneum

PSEUDOMYXOMA PERITONEI



a. Greater sac b. Lesser sac c. Pouch of Morrison d. Omental bursa e. Right subphrenic 51. In a patient recovering from peritonitis, which of the following would be the most characteristic sign of pelvic abscess: a. Fever and abdominal pain (MHPGMCET 2008) b. Tachycardia c. Mucus in the stool for first time d. All of the above

40. In pseudomyxoma peritonei, mucinous cyst-adenocarcinoma of which following organ is involved:  (Orissa 2011) a. Pancreas  b. Ovary  c. Kidney  d. Abdominal testis



52. Treatment of pouch of Douglas abscess is: (Recent Question 2016) a. Laparotomy b. Posterior colpotomy c. Antibiotics d. Extraperitoneal drainage



41. Pseudomyxoma peritonei arises from: (PGI Dec 2008) a. Carcinoma ovary b. Ovarian cyst c. Ovarian dermoid d. Adenocarcinoma colon e. Mucocele of appendix





42. True about pseudomyxoma peritonei: a. Seen in male only b. Cytoreductive surgery needed c. Always appendectomy needed d. Radiation therapy given e. Locally malignant tumor

53. Most common site for intra abdominal abscess following laparotomy is: (AIIMS 92) a. Sub hepatic b. Subphrenic c. Pelvic d. Paracolic 54. Correct about subphrenic abscess is: a. Rarely chest symptoms b. Toxemia c. Rarely toxaemia d. No sign and symptoms 55. The most favored treatment for a pelvic abscess in cul-de-sac is: (DPG 96) a. Laparotomy b. Colpotomy c. External I and D d. Antibiotics 56. Colopotomy is done to treat: (APPG 98) a. Ischeorectal abscess b. Pelvic abscess c. Appendicular abscess d. Perianal abscess 57. Most pathognomic in pelvic abscess is: (UPPG 2007) a. Constipation b. Mucopurulent discharge c. Loose stool d. Bleeding 58. Most common site of intraperitoneal abscess is: (DPG 2008) a. Right superior intraperitoneal space b. Right inferior intraperitoneal space c. Left superior intraperitoneal space d. Left superior intraperitoneal space 59. Most common site of intra-peritoneal abscess: (APPG 2008) a. Morrison’s pouch b. Omental bursa c. Pelvic region d. Left subhepatic pouch 60. A posteriorly perforating ulcer in the pyloric antrum of the stomach is likely to produce initial localized peritonitis or abscess formation in the: (AIIMS Nov 2004) a. Greater sac b. Left subhepatic and hepatorenal spaces (Pouch of Morrison) c. Right subphrenic space d. Lesser sac





(PGI June 2004)

43. All are true about pseudomyxoma peritonei except: a. Common in male (UPPG 2007) b. Associated with ovary tumors c. Yellow jelly collection of fluid d. Appendiceal adenocarcinoma 44. All are true about pseudomyxoma peritonei except: a. Associated with ovarian tumors (DPG 2008) b. Appendix is most common site of origin c. Yellow jelly collection of fluid d. Common in male











45 False about pseudomyxoma peritonei is: a. b. c. d.

(JIPMER May 2018) Recurrence after surgery Refractory to drugs Hyperthermic intraperitoneal chemotherapy is treatment option Most commonly associated with appendiceal tumor





ABDOMINAL ABSCESS









46. Commonest site of intraperitoneal abscess is: (Orissa 2011) a. Lesser sac  b. Greater sac  c. Pelvis  d. Paracolic gutter  47. The part of peritoneal cavity that is most dependent in supine position:  (MHSSMCET 2009) a. Right subphrenic space b. Lesser sac c. Supra mesocolic space d. Right subhepatic space 48. Most common site of intra-abdominal abscess:  (MHPGMCET 2006) a. Pelvic b. Subphrenic space c. Mesenteric d. Paracolic gutters 49. Most common cause of infection and collection of fluid in the left subhepatic space:  (MHPGMCET 2009) a. Perforation at the lesser curvature of stomach b. Complicated acute pancreatitis c. Ruptured abscess of the left lobe of the liver d. Perforation of posterior duodenal wall ulcer 50. Posterior perforated ulcer on pyloric antrum cause abscess formation in:  (PGI June 2009)



61. A patient developed wound infection post laparotomy for pyoperitoneum, was treated conservatively. Now, granulation tissue is seen in the wound. Next step in management is: (Recent Question 2015) a. Daily dressing b. Mesh repair c. Incision and drainage d. Re-suturing with interrupted stitches

PNEUMOPERITONEUM

62. Treatment of pneumoperitoneum, as a result of colonoscopic perforation in a young patient is: (PGI June 98) a. Temporary colostomy b. Closure + lavage c. Permanent colostomy d. Symptomatic



63. Best investigation for air in peritoneal cavity is: (CMC 98) a. USG b. Laparotomy c. Laparoscopy d. X-ray abdomen-erect view

Gastrointestinal Surgery



Section 3

39. True statement regarding tubercular peritonitis and ascites:  (PGI November 2017) a. SAAG Hernia (10%) > Crohn’s disease (5%)Q • Metastatic or peritoneal carcinomatosis are the MC malignancies leading to SBOQ. • Primary colonic cancers (particularly those arising from the cecum & ascending colon) may present as a SBOQ. Pathophysiology • Early in the course of an obstruction, intestinal motility & contractile activity increase in an effort to propel luminal contentsQ past the obstructing point. • Increase in peristalsis early in the course of bowel obstruction is present both above and below the point of obstructionQ (diarrhea in partial or even complete small bowel obstruction in the early period) • Later in the course of obstruction, the intestine becomes fatigued & dilatesQ, with contractions becoming less frequent & less intense. • As the bowel dilates, water & electrolytes accumulate both intraluminally & in the bowel wallQ itself. • This massive third-space fluid loss accounts for the dehydration & hypovolemiaQ. • Metabolic effects of fluid loss depend on the site & duration of the obstruction. • As the intraluminal pressure increases in the bowel, a decrease in mucosal blood flow can occur. Clinical Features • Cardinal symptoms of intestinal obstruction: Colicky abdominal pain (1st symptom)Q, nausea, vomiting, abdominal distention, and a failure to pass flatus & feces (i.e., obstipation)Q. • Typical crampy abdominal pain occurs in paroxysms at 4- to 5-minute intervals & occurs less frequently with distal obstructionQ. • Nausea & vomiting are more common with proximal obstruction. • Cramping abdominal pain is the initial and most prominent symptom in distal obstructionQ • Abdominal distention is more common in distal obstructionQ • Abdominal distention occurs as the obstruction progresses, and the proximal intestine becomes increasingly dilatedQ. • As the obstruction becomes more complete with bacterial overgrowth, the vomitus becomes more feculent, indicating a late and established intestinal obstructionQ. • Patient may present with tachycardia & hypotensionQ, demonstrating the severe dehydration that is present. • Fever suggests the possibility of strangulationQ. • Abdominal distention is dependent on the level of obstructionQ. • Early in the course of bowel obstruction, peristaltic waves can be observed, particularly in thin patients, and auscultation of the abdomen may demonstrate hyperactive bowel sounds with audible rushes associated with vigorous peristalsisQ (i.e., borborygmi). • Late in the obstructive course, minimal or no bowel sounds are noted. • Localized tenderness, rebound, & guarding suggest peritonitis and the likelihood of strangulationQ. • Rectal examination: To assess for intraluminal masses and to examine the stool for occult bloodQ (an indication of malignancy, intussusception, or infarction) Diagnosis • X-ray Abdomen: Confirm the clinical suspicion and define more accurately the site of obstruction (60% diagnostic accuracy)Q • Supine radiographs: Dilated loops of small intestineQ without evidence of colonic distention, diagnose site & level of obstructionQ • Erect radiographs: Multiple air-fluid levels, which often layer in a stepwise patternQ (up to 3-5 air fluid levels Gastric (48 hours)Q > Colonic (3-5 days)Q • Post-operative ileus is most pronounced in colonQ • Characteristic sequence of return of normal motility: Small intestinal motility returning to normal within the first 24 hours, gastric motility within 48 hours and colonic motility returning to normal 3 to 5 daysQ. • Because small bowel motility is returned before colonic and gastric motility, listening for bowel sounds is not a reliable indicator that ileus has fully resolvedQ. Management of Post-operative Ileus • The essence of treatment is prevention, with the use of nasogastric suction and restriction of oral intake until bowel sounds and the passage of flatus returnQ. • Following general principles should be applied: −− The primary cause must be removedQ. −− Gastrointestinal distension must be relieved by decompressionQ. −− Close attention to fluid and electrolyte balanceQ is essential. −− There is no place for the routine use of peristaltic stimulants.

Gastrointestinal Surgery

Paralytic Ileus

384

Surgery Essence • Rarely, in resistant cases, medical therapy with an adrenergic blocking agent in association with cholinergic stimulation, e.g. neostigmine (Catchpole regimen)Q, may be used, provided that an intraperitoneal cause has been excluded. • If paralytic ileus is prolonged and threatens life, a laparotomy should be considered to exclude a hidden cause and facilitate bowel decompressionQ

Gastrointestinal Surgery

INTUSSUSCEPTION Intussusception • Telescoping of one portion of the intestine into the other. • Middle layer is isolated between two sharp bends and first to become gangrenousQ. • Apex is most prone to gangrene • Highest incidence between 4 and 10 monthsQ of age • Approx 80–90% of cases occur between 3 and 36 monthsQ • Mostly idiopathic in infants and toddlersQ (no clear etiology). • MC type: IleocolicQ > Ileo-ileocolic > Ileo-ileal > ColocolicQ Etiology and Predisposing Factors • Upper respiratory tract infections or gastroenteritisQ (adenovirus and rotavirus have been implicated) have been thought to be contributory to the development of “idiopathic” intussusception. Hypertrophy of Peyer’s patchesQ can be seen at surgery, but no single etiologic factor predominates. • Approximately 5–10% of cases have a true pathologic lead point. The older the toddler, the more likely there will be a lead pointQ • MC lead point is Meckel’s diverticulumQ > PolypQ • Other lead points include polyps submucosal lipomas, the appendix, intestinal duplication, foreign bodies, and tumors such as hamartomas associated with Peutz-Jeghers syndrome. • Henoch-Schönlein purpura: Submucosal hemorrhage acts as a lead pointQ • Cystic fibrosis: Risk for recurrent intussusceptionQ Clinical Features • Typical history: Sudden, short-duration, cyclic crampy abdominal painQ. • During these episodes the infant cries inconsolably with the knees drawn upQ. • Between episodes the infant is asymptomaticQ. • Vomiting is almost universalQ • Initially the passage of stools may be normal while later on blood mixed with mucus is evacuated- red currant jelly stool • • • •

An abdominal mass may be palpated- a sausage shaped abdominal massQ (increase in size & firmness during the paroxysm of pain) There may be an associated feeling of emptiness in the right iliac fossa (Sign of Dance)Q Occult or gross blood in 60–90% of cases on rectal examinationQ Apex may be palpable or even protrude from anus in extensive ileocolic or colocolic intussusceptionQ

Diagnosis • USG: Kidney-shaped mass in the longitudinal view or a target sign in the transverse view • Hydrostatic reduction by contrast agent or air enema (preferred) is the diagnostic & therapeutic procedure of choiceQ • Successful reduction is confirmed by reflux of airQ (or barium) into the small bowel

Section 3

Treatment • Hydrostatic reduction by contrast agent or air enema is the diagnostic & therapeutic procedure of choiceQ. • The success rate with air or barium reduction should exceed 70%Q. • Failure of reduction or the presence of peritonitis mandates operative interventionQ, which can be performed laparoscopically or by a standard approach • Definitive surgical procedure: Ileocolectomy with primary anastomosisQ Recurrence • Recurrence after successful hydrostatic reduction is 5–10%, recurrence rate after operative reduction is 1–4%. • Recurrence is usually managed by hydrostatic reductionQ. • Third recurrenceQ is an indication for operative intervention to look for a lead point.

Intestinal Obstruction

385

Radiological Investigations in Intussusception Barium Enema

• Features of small intestinal obstruction • Abdominal soft tissue density in some cases which may show: • Target signQ: Soft tissue mass with concentric area of lucency due to mesenteric fat. • Meniscus signQ: Crescent of gas within colonic lumen that outlines the apex of intussusception Q

• Claw sign : Rounded apex of intussusception protrudes into the contrast column • Coiled spring signQ: Edematous mucosal folds of returning limb of intussusception outlined by contrast material. Q

Ultrasound • Target sign or Bull’s eye sign on transverse scanQ • Pseudokidney sign on longitudinal scanQ

Section 3

Plain X-ray Film

MECONIUM SYNDROME Meconium Syndrome • The meconium syndromes of infancy represent a complex group of gastrointestinal conditions associated with cystic fibrosis (CF), with considerable overlap in clinical presentation and management. • The abnormal chloride transport in patients with CF results in tenacious, viscous secretions affecting a wide variety of organs, including the intestine, pancreas, lungs, salivary glands, reproductive organs, and biliary tractQ • The clinical presentation of the meconium syndromes ranges from a meconium plug to simple and complicated meconium ileusQ. Meconium Plug • Meconium plug syndrome is a frequent cause of neonatal intestinal obstruction • Associated with −− Hirschsprung’s diseaseQ, Maternal diabetesQ −− HypothyroidismQ, Cystic FibrosisQ Clinical Features

Diagnosis • Abdominal X-ray: Multiple dilated loopsQ of intestine. • Diagnostic and therapeutic procedure of choice is a water-soluble contrast enemaQ. Treatment • Diagnostic and therapeutic procedure of choice is a water-soluble contrast enemaQ. This often results in the passage of a plug of meconium and relief of the obstructionQ

MECONIUM ILEUS Meconium Ileus Simple Meconium Ileus • Meconium ileus in the newborn represents the earliest clinical manifestation of CF and affects roughly 15% of patients with this inherited disease. • In simple meconium ileus, the terminal ileum is dilated and filled with thick, tarlike, inspissated meconiumQ. • Smaller pellets of meconium are found in the more distal ileum, leading into a relatively small colonQ. Clinical Features • Present with signs and symptoms of distal intestinal obstructionQ. Diagnosis Abdominal X-ray in Meconium Ileus • Dilated, gas-filled loops of small bowelQ • Absence of air-fluid levelsQ • Mass of meconium within the right side of the abdomen mixed with gas to give a ground-glass or soap bubble appearanceQ.

Gastrointestinal Surgery

• Affected infants are often pretermQ • Present with signs and symptoms of distal intestinal obstructionQ • Abdominal distention is a prominent featureQ

386

Surgery Essence • Investigation of choice: Water-soluble (Gastrografin) contrast enemaQ. Treatment • Water-soluble (Gastrografin) contrast enema is successful in relieving the obstruction in up to 75% of casesQ, with a bowel perforation rate of less than 3%.

Gastrointestinal Surgery

• Contrast agents are hypertonic relative to serum, so infants should be well hydrated and electrolytes and vital signs carefully monitored following the procedure • • • •

Operative management is required when the obstruction cannot be relieved with contrast enemaQ. Simple evacuation of the luminal meconium without the need to create a stoma is all that is necessary in most cases. Irrigate the proximal and distal bowel with either warmed saline solution or 4% N-acetylcysteineQ. Bishop-Koop operation with irrigation stoma is rarely used in meconium ileus.

COMPLICATED MECONIUM ILEUS Complicated Meconium Ileus • Meconium ileus is considered complicated when perforation of the intestine has taken place. • This may occur in uteroQ or the early neonatal periodQ • Meconium peritonitis is an aseptic chemical peritonitisQ caused by spillage of meconium • Meconium within the peritoneal cavity results in severe peritonitis with a dense inflammatory response and calcification (Snow-storm sign on USGQ) • The presentation of complicated meconium ileus is variable and includes formation of a meconium pseudocyst, adhesive peritonitis with or without secondary bacterial infection, or ascitesQ. • Treatment: Most patients are managed non-operatively with enema or oral polyethylene glycol solution.

ACUTE MESENTERIC ISCHEMIA Acute Mesenteric Ischemia • Emboli (50%)Q: − Arrhythmia, Valvular diseaseQ, Myocardial infarction −− Cardiac aneurysm, Aortic atherosclerotic disease

− Hypokinetic ventricular wall

Thrombosis (25%): Atherosclerotic disease • Nonocclusive (5–15%): −− Pancreatitis, Heart failure, SepsisQ • Venous occlusion: −− Hypercoagulable state

− Cardiac bypass, Burns, Renal failureQ

− Sepsis Compression, Pregnancy, Portal hypertension

Acute Mesenteric Ischemia

Section 3

Clinical Features • • • • • •

Early diagnosis is the key to successful management of AMIQ. Most patients have nonspecific symptoms of abdominal painQ. Abdominal pain out of proportion to the findings on physical examination and persisting beyond 2 to 3 hours is the classic picture. Diarrhea, nausea, vomiting, and anorexia can also be part of the initial symptom complex. Melena or hematochezia in 15%, and occult fecal blood is found in half of the patients. Leukocytosis is common.

Diagnosis • IOC in AMI is mesenteric arteriographyQ • CT scan: Wall hyperdensity, absence of wall enhancement, wall thickening, bowel dilation, pneumatosis, gas in mesenteric vein branches and in portal vein branches. • Hemoconcentration, leukocytosis and metabolic acidosis is present. • Hyperkalemia and hyperphosphatemia in bowel infarction should be suspected.

Intestinal Obstruction

387

Treatment

• Catheter-directed papaverine to reverse the severe mesenteric vasospasmQ is initiated early after arteriography • Anticoagulation is given to prevent propagationQ of mesenteric thrombus • In addition to aggressively correcting the low cardiac output, terminating vasoconstrictor use, and discontinuing digitalis preparations, intra-arterial papaverine infusion is the treatment of choiceQ. • In the absence of peritonitis, supportive care with anticoagulation and continued papaverine infusion is recommended. • Evidence of peritonitis: Exploratory laparotomy, with conservative resection of necrotic bowel.

Section 3

• Effective management: Early diagnosis, aggressive resuscitation, early revascularization, and ongoing supportive care. • Mucosal layer is the most sensitive to ischemia, bacterial translocation should be anticipated and intravenous antibioticsQ used to treat the associated bacteremia.

Investigation of Choice Acute mesenteric ischemiaQ

AngiographyQ

Mesenteric venous thrombosis Chronic mesenteric ischemia

Q

Q

CECTQ AortographyQ

NON-OCCLUSIVE MESENTERIC ISCHEMIA Non-occlusive Mesenteric Ischemia • Accounts for 20% of all cases of AMI • Has manifestations similar to those of mesenteric arterial thrombosis, but it occurs with patent mesenteric arteries. • Splanchnic vasoconstriction is the underlying pathophysiologic process and is precipitated by hypoperfusion from medications, depressed cardiac output, or renal or hepatic diseaseQ. • BP in the bowel falls below 40 mm Hg, ischemia develops leading to infarction and bowel necrosis Increased Risk of Non-occlusive mesenteric ischemiaQ Acute myocardial infarctionQ CHF, dysrhythmiaQ Sepsis, hypovolemia Use of splanchnic vasoconstrictorsQ

• • • •

Hemodialysis patientsQ Recent history of cardiopulmonary bypass Major abdominal surgeryQ Pancreatitis, aortic dissection, or burnsQ.

• Nonocclusive mesenteric ischemia is typified by diffuse spasm of the SMA branches with intermittent areas of narrowing and dilation. • Arteriogram: Diffuse vasospasm with marked narrowing of the major branches of the SMA, often with the “string of sausages” appearanceQ with reflux of contrast into the aorta



• Perfusion is markedly compromised because the intense distal vasospasm causes high peripheral resistance, with frequent reflux of contrast into the aorta.

EMBOLIC OCCLUSION IN ACUTE MESENTERIC ISCHEMIA Embolic Occlusion in Acute Mesenteric Ischemia • Accounts for 40–50% of cases of AMI. • Most emboli originate in the heart and are secondary to myocardial infarctionQ, cardiac arrhythmia, endocarditis, cardiomyopathy, ventricular aneurysm, valvular disorders, or depressed left ventricular function as a result of ischemic heart disease. • Most mesenteric emboli lodge in the SMA (branches from the aorta at an oblique angle)Q • More than 50% of emboli lodge in the mid to distal segment of the SMAQ • Key to successful management: High index of suspicion leading to early diagnosis, aggressive resuscitation, and early mesenteric revascularizationQ. • Acute SMA embolism: Sudden and dramatic symptoms because of the absence of collateral circulation. • Severe abdominal pain contrast markedly with the absence of physical findingsQ. • Rectal examination is not helpful because the presence of occult blood is typically a late occurrence.

Gastrointestinal Surgery

• • • •

388

Surgery Essence MESENTERIC VENOUS THROMBOSIS Mesenteric Venous Thrombosis • MVT accounts for 5–15% of patients with mesenteric ischemia. • SMV is MC involved, frequently with extension of thrombus into portal veinQ. • IMV is most often sparedQ

Gastrointestinal Surgery

Clinical Features • • • • •

Most commonly, patients complain of midabdominal colicky painQ Nausea, vomiting, diarrhea, and anorexia frequently accompany their abdominal discomfort. Occult blood in the stool in half of the patients, gross bleeding such as hematemesis, hematochezia, or melena occurs in approximately 15%. Family history of venous thromboembolismQ in half of the patients Bowel infarction ultimately develops in 30–60%Q.

Diagnosis • Elevation of the WBC count in 50–65% of patients. • Serum amylase is usually normal, and serum lactate is elevated only in patients with advanced bowel ischemia and suggests necrosisQ. • CECT with IV contrast is the diagnostic test of choiceQ for patients with suspected acute MVT Treatment • • • •

Rapid initiation of systemic anticoagulationQ is important. Exploratory laparotomy in localized or diffuse peritoneal irritation, Acute thrombus in large veins: Thrombectomy followed by recombinant tissue plasminogen activator solution. The patient is treated with heparin intraoperatively and anticoagulation is continued postoperativelyQ.

Prognosis • MVT have a high risk of recurrence (35–70%)Q, most frequently within 30 days, thus emphasizing the need for early and persistent anticoagulation. • Mortality rate is high, up to 50%.

CHRONIC MESENTERIC ISCHEMIA Chronic Mesenteric Ischemia • • • •

CMI is most commonly the result of advanced atherosclerotic disease of multiple mesenteric arteriesQ. Symptomatic CMI is rare because of the good collateral circulatory network that exists between the mesenteric vessels Symptomatic disease is more common in femalesQ Risk factors: Positive family history, smoking, hypertension, and hypercholesterolemiaQ. Causes of Chronic Mesenteric Ischemia • Atherosclerotic diseaseQ

• Arterial hyperplasia/dysplasia

• Inflammatory diseaseQ

Clinical Features

Section 3

• Classical picture: Postprandial abdominal pain (intestinal angina or intestinal claudication) leading to an aversion to food and weight lossQ. • Pain is characteristically diffuse, midabdominal, midepigastric, and crampy in nature. • Pain develops within 15–45 minutes after eating (severity related to the size of the meal ingested) Diagnosis • Evaluation of the mesenteric arteries frequently begins with a duplex scan. • Aortography with AP and lateral views is diagnostic technique of choiceQ. Treatment • Revascularization by balloon angioplasty or stent placement for elderly patients (poor candidates for surgery). Restenosis and reintervention rates may be high (30–50%) • Definitive therapy: Surgery with transaortic endarterectomy or bypass grafting • Results of surgery are generally highly gratifying in properly selected patients, with rapid resolution of symptoms and return of weight • Long-term patency of the grafts is excellent, exceeding 90%.

Intestinal Obstruction

389

COLONIC PSEUDO-OBSTRUCTION

• Pseudo-obstruction of the colon describes the condition of distention of the colon, with signs and symptoms of colonic obstruction, in the absence of an actual physical cause of the obstructionQ • Acute colonic pseudo-obstruction is also known as Ogilvie’s syndromeQ • Two types: Primary and secondary • Secondary pseudo-obstruction is more commonQ Pathophysiology • Mechanism thought to play: Sympathetic overactivity overriding the parasympathetic systemQ • Indirect support for this theory has been derived from the success in treating the syndrome with neostigmine, a parasympathomimetic agentQ • Further support comes from reports of immediate resolution of the syndrome after administration of an epidural anesthetic that provides sympathetic blockade

Section 3

Colonic Pseudo-obstruction

Primary Pseudo-obstruction • It is a motility disorder: −− A familial visceral myopathy (hollow visceral myopathy syndrome) or −− A diffuse motility disorder involving the autonomic innervation of the intestinal wall. Causes of secondary Pseudo-obstruction • • • • •

Neuroleptic medicationsQ OpiatesQ Severe metabolic illness Myxedema (HypothyroidismQ) HyperparathyroidismQ

• • • • •

UremiaQ Lupus, scleroderma, dermatomyositisQ Parkinson’s diseaseQ Traumatic retroperitoneal hematomasQ Diabetes mellitusQ

Clinical Features • Acute form: Most commonly affects patients with chronic renal, respiratory, cerebral, or cardiovascular disease, involves only the colonQ. • Chronic form: Affects other parts of the gastrointestinal tract, usually presents as bouts of subacute and partial intestinal obstruction, and tends to recur periodically.

Diagnosis • Abdominal X-ray: Distended colon, with the right and transverse segmentsQ tending to be most dramatically affected (radiologic appearance of large bowel obstruction). Transition point is frequently present, usually at or near the splenic flexureQ. • Most useful investigation: Water-soluble contrast enemaQ • Contrast enema differentiate mechanical obstruction and pseudo-obstruction Treatment • Initial treatment: Nasogastric decompression, replacement of extracellular fluid deficits, and correction of electrolyte abnormalitiesQ. • All medications that inhibit bowel motility, such as opiates, should be discontinued. • Most patients improve with this regimenQ. • Treat this condition with neostigmineQ • Mechanical obstruction should be excluded (either by water-soluble contrast enema or colonoscopy) before the administration of neostigmineQ • Resolution is indicated by the passage of stool and flatus by the patient, within ten minutes of drug administration

LARGE BOWEL OBSTRUCTION Large Bowel Obstruction • Classified as dynamic (mechanical) or adynamic (pseudo-obstruction). • Mechanical obstruction is characterized by blockage of the large bowel (luminal, mural, or extramural), resulting in increased intestinal contractilityQ • Pseudo-obstruction is characterized by the absence of intestinal contractility, often associated with decreased or absent motility of the small bowel and stomachQ

Gastrointestinal Surgery

• Acute colonic pseudo-obstruction should be suspected when a medically ill patient suddenly develops abdominal distentionQ • The abdomen is tympanitic, usually nontender, and bowel sounds are usually present

390

Surgery Essence • MC cause of LBO: Colorectal cancerQ (CA Rectum > sigmoid) • Adhesions (MC cause of small bowel obstruction) are rarely a cause of colonic obstruction. Pathophysiology

Gastrointestinal Surgery

• Colon becomes distended as gas (about two thirds is swallowed air, the remainder includes the products of bacterial fermentation), stool, and liquid accumulate proximal to the site of blockage. • In obstructed hernia or volvulus, the blood supply can become compromised, or strangulated; initially, the venous return is blockedQ • Vascular compromise of the obstructed colon can occur due to excessive distentionQ • Closed-loop obstruction: When both the proximal and distal parts of the bowel are occludedQ (strangulated hernia or volvulus) • Closed-loop obstruction is seen when a cancer occludes the lumen of the colon in the presence of a competent ileocecal valveQ • Increasing colonic distention causes the pressure in the cecum to become so high that the vessels in the bowel wall are occluded, and necrosis and perforationQ can occur Clinical Features • Cancers of rectum or left colon are more likely to obstructQ than those arising in the more capacious proximal colon. • Failure to pass stool and flatus associated with increasing abdominal distention and cramping abdominal painQ. Diagnosis • Abdominal X-ray: Distended colonQ • CT scan: Helpful in revealing an inflammatory process such as diverticulitis. • Water-soluble contrast enema: For the diagnosis of suspected case of volvulus or distal sigmoid cancerQ Treatment • Virtually all patients with complete acute large bowel obstruction require prompt surgical interventionQ and should not undergo a trial of non-operative management. • Acute large bowel obstruction in patients with competent ileocecal valve is a true surgical emergency because of high chances of perforation (MC site: Cecum)Q. • Once diagnosis has been made, surgical exploration should be undertaken as soon as possible after appropriate resuscitationQ Treatment of Large Bowel Obstruction Site of Obstruction Procedure Right-sided colonic obstruction • Resection with ileo-transverse anastomosisQ (cancer or volvulus) Cancer of sigmoid colon • Hartmann’s operationQ (sigmoidectomy with descending colostomy & closure of the rectal stump), • Sigmoidectomy with primary colorectal anastomosisQ • Abdominal colectomy with ileorectal anastomosisQ Cancer of distal or mid rectum • Loop colostomy or defunctioning colostomyQ (to relieve obstruction) followed by neoadjuvant chemoradiationQ, (with the plan to resect the primary lesion at a later time) Small Bowel Obstruction Thin complete linesQ Seen in jejunumQ

Large Bowel Obstruction Absent

Haustra No of loops Distribution of loops Diameter of loops

Absent ManyQ CentralQ 3–5 cmQ

Thick incomplete bandsQ Few PeripheralQ >5 cmQ

Radius of curvature Solid feces

SmallQ Absent

LargeQ PresentQ

Section 3

Valvulae conniventes

SIGMOID VOLVULUS Sigmoid Volvulus • Volvulus can occur in any segment of large bowel attached to a long & floppy mesenteryQ or fixed to the retroperitoneum by a narrow base of originQ • MC site of volvulus: Sigmoid colon • More commonly anticlockwise (can be both clockwise or anticlockwise) • Equal frequency in both sexesQ

Intestinal Obstruction

391

Associated Predisposing Factors

Clinical Features • Present as acute or subacute intestinal obstruction • Sudden onset of severe abdominal pain, vomiting, & obstipation. • Abdomen is markedly distended and tympanitic, with the distention often more dramaticQ than would be associated with other causes of obstruction. • Severe abdominal pain, rebound tenderness, & tachycardia are ominous signsQ.

Section 3

• Age: 60–70 yearsQ. • Chronic constipationQ • Institutionalized or neurologically impaired or psychiatric patientsQ (their medication may decrease intestinal motility, or they may fail to pass stool regularly, leading to fecal loaded large bowel predisposing to volvulus) • Diet high in fibre and vegetablesQ (as in third world countries)

Radiological (X-ray) Characteristics • Markedly dilated sigmoid colon with the appearance of a bent inner tube or coffee bean appearanceQ. • Inferior convergence of the dilated loop points towards left side of pelvis. Whorl sign on CT scan. • Contrast enema demonstrates the point of obstruction with the pathognomic ‘birds beak’ or ‘bird of prey’ or ‘ace of spades’ signQ Management of Sigmoid Volvulus • Initial management: ResuscitationQ followed by endoscopic decompression and detorsionQ. • Decompression/detorsionQ can be achieved by placement of rectal tube through a proctoscope or the use of a colonoscope. • If detorsion/decompression cannot be achieved with either the rectal tube or colonoscope, laparotomy with resections of the sigmoid colonQ is done. • Even if detorsion of the sigmoid volvulus is successful, risk of recurrence is high (50%)Q • Sigmoid colectomy is indicated after the patient has stabilized

CECAL VOLVULUS Cecal Volvulus • Cecal volvulus is actually a cecocolic volvulus • Consists of an axial rotation of the terminal ileum, cecum, and ascending colonQ with concomitant twisting of the associated mesentery. • Cause: Lack of fixation of the cecum to the retroperitoneum.Q Q

• Cecal volvulus occurs in clockwise directionQ • More common in womenQ • Affects a younger age group as compared to sigmoid volvulus, in 5th decadeQ Predisposing Factors for Cecal volvulus • Previous surgeryQ • PregnancyQ

• MalrotationQ • Obstructing lesions of the left colonQ

Clinical Features • Sudden onset of abdominal pain & distention. • Presents with features of small bowel obstruction. • Asymmetric distention of the abdomen, with a tympanitic mass palpable in either the left upper quadrant or midabdomenQ. Diagnosis • • • •

Abdominal X-ray: Dilated cecum, displaced to the left side of the abdomen. Distended cecum assumes a gas-filled comma shape or kidney bean shapeQ, the concavity of which faces inferiorly and to the right. Haustral markings in the distended loop indicate that the dilated bowel is colon. Torsion results in small bowel obstructionQ (radiographic pattern of SBO)

Gastrointestinal Surgery

• Any evidence of bowel gangrene or perforation contraindicates non-operative decompression and an immediate surgical explorationQ is done. • Paul-Mikulicz operation: In cases of suspicion of impending gangrene

392

Surgery Essence • Contrast enema is used to confirm the diagnosis and to exclude a carcinoma of the distal bowel as a precipitating cause of the volvulusQ Treatment

Gastrointestinal Surgery

• Most cases require operation to correct the volvulus & prevent ischemiaQ. • If ischemia has already occurred, immediate operation is obviously required. • Right colectomy with primary anastomosis is the procedure of choiceQ • In frankly gangrenous bowel, resection of the gangrenous bowel with ileostomy is a safar approach • Recurrence rates are high with ceropoxy, and right colectomy remains the procedure of choice for cecal volvulus. Cecal bascule • A condition in which cecum folds in a cephalad direction anteriorly over a fixed ascending colonQ • Cecal bascule commonly causes intermittent bouts of abdominal pain • Mobile cecum permits intermittent episodes of isolated cecal obstruction that are spontaneously relievedQ as the cecum falls back into its normal position

• Sigmoid volvulus is more commonly anticlockwise (can be both clockwise or anticlockwise)Q • Cecal volvulus and small intestine volvulus are mostly clockwiseQ

MALROTATION: EMBRYOLOGY Malrotation: Embryology • Rapid growth plus elongation of the midgut starting in the fifth week leads to herniation of midgut with superior mesenteric vessels as its stalkQ. • The midgut undergoes 270-degree counterclockwise rotationQ around the superior mesenteric vessels. • This initial rotation results in the normal position of the DJ flexure in the left upper quadrant at the level of the gastric antrumQ. • The DJ flexure becomes fixed to the posterior abdominal wall by the ligament of Treitz. As a result of this rotation and fixation, the third portion of the duodenum lies posterior to the SMAQ. • In the 10th week the herniated intestinal loop begins to returnQ to the abdominal cavity. • The cecocolic loop undergoes another 270-degree counterclockwise rotationQ around the SMA, which leads to the normal position of the cecum in the right lower quadrant. • Subsequently, the ascending colon and descending colon become fixed to the posterior abdominal wall. • During the fourth and fifth weeks of gestation, the small intestine mesentery attaches itself to the posterior abdominal wall in a broad base extending diagonally from the DJ flexure to the cecumQ. • MC rotational abnormality: Non-rotationQ • MC type of malrotation: Incomplete rotationQ

TYPES OF MALROTATION Types of Malrotation

Section 3

Nonrotation • • • • •

MC rotational abnormality Failure of counterclockwise rotationQ after return of the midgut to the abdominal cavity. Duodenal ‘C’ loop, ligament of Treitz and small intestine is on right side of abdomen. Cecum is on the left side of abdomenQ Proximal jejunum & ascending colon are fused as one pedicle, through which blood supply to the entire midgut (SMA)Q is located.

Incomplete Rotation • Counterclockwise rotation is arrested at around 180 degreesQ. • Most commonQ forms of malrotation. • The small intestine lies on the right side with the DJ flexure to the right of the vertebral column, and the duodenum has a corkscrew configurationQ. • The large intestine lies on the left side with the cecum at abnormal locations, usually in the midlineQ.

Intestinal Obstruction

Reverse Rotation • In reverse rotation, part of the rotation occurs in a clockwise direction around the SMA. • Duodenum assumes an anterior position and the colon lies posterior to the duodenum and the SMA. Hyperrotation • If the counterclockwise rotation extends beyond 270 degrees, the cecum comes to rest in the left hypochondrium position.

Section 3

• Other forms of fixation anomalies may be due to failure of fixation of the ascending colon in the right hypochondrium. Associated with this abnormal fixation is a narrow intestinal mesentery and Ladd’s bands. • Ladd’s bands represent the retroperitoneal attachments that normally fix the cecum and ascending colon to the posterior abdominal wall. Because the right colon is more medial, the bands extend across the duodenum from the right upper quadrant to the cecum and ascending colonQ.

393

MALROTATION Malrotation • Patients have a 30–62% risk of having associated anomalies, and most involve GITQ. • Incidence of clinically symptomatic malrotation is 1 in 6000 live birthsQ. • Malrotation may initially be recognized at any age • Approximately in 90% of patients symptoms develop before 1 year of age, with 50% to 75% appearing within the first monthQ of life. Clinical Features • Malrotation can be totally asymptomatic and discovered only during work-up for an unrelated condition or during an autopsy examination. • Malrotation without volvulus may be manifested as chronic, vague abdominal pain, with or without intermittent bilious emesis, & failure to thriveQ. • Neonates typically have bilious emesis, which may be the only initial symptom of midgut volvulusQ. • Systemic signs such as increasing lethargy with poor perfusion, temperature instability, cardiopulmonary compromise, and low urine output in cases of delayed diagnosisQ.

Diagnosis Upper Gastrointestinal Contrast Study • Gold standard test for diagnosis is an upper gastrointestinal contrast studyQ. • Malrotation is diagnosed by an abnormal position of the ligament of TreitzQ. • Normal location is typically to the left of vertebral column & posterior to stomachQ • Volvulus can be diagnosed by contrast-enhanced upper gastrointestinal series showing a corkscrew configuration of upper portion of small intestine or a “bird’s beak” appearance at third portionQ of duodenum. Treatment • Acutely ill child with peritonitis

• Emergency surgeryQ without radiologic studies

• Symptomatic from volvulus

• Nasogastric decompression, intravenous fluid, and broad-spectrum antibiotics followed by urgent surgeryQ

• Chronic symptoms or symptom-free

• Elective correctionQ In Malrotation

• Plain abdominal radiographs are not helpful in ruling in or out midgut volvulusQ. • A contrast enema study is not part of the work-up for malrotation. The presence of a normally located cecum in the right lower quadrant does not rule out malrotationQ Operative Treatment of Malrotation • The standard approach via a right upper quadrant transverse incisionQ. • If volvulus is present, it should be reduced by counterclockwise rotationQ as necessary because volvulus usually occurs in a clockwise direction

Gastrointestinal Surgery

• Hematologic studies may show metabolic acidosis, thrombocytopenia, & leukopenia. • Acute onset of bilious vomiting in a neonate is a sign of malrotation until proved otherwise. It demands immediate radiologic evaluationQ.

394

Surgery Essence • Bowel with uncertain viability should be wrapped with warm moist gauze sponges for at least 15 minutes

Section 3

Gastrointestinal Surgery

• Frankly gangrenous bowel should be resected and a stoma or stomas fashioned. • Ladd’s bands, which represent the posterior peritoneal attachments of the right colon that cross over the duodenum, should be divided on the lateral aspect of the duodenumQ. • Widening of the mesenteric base is necessary, & duodenum is mobilized & straightened by dividing the abnormal ligament of Treitz and Ladd’s bands. • Incidental appendectomy should be performed to avoid diagnostic confusion in the future because the cecum will be placed in the left lower quadrantQ. • Duodenum and proximal jejunum are placed on the right sideQ • Terminal ileum and cecum are placed in the left hypochondriumQ

Multiple Choice Questions ACUTE INTESTINAL OBSTRUCTION

1. True about strangulation of intestine is: (MHPGMCET 2001) a. Arterial blood flow affected first b. Usually venous blood flow affected first c. Blood flow normal d. No gangrene



2. Most common cause of hyponatremia in surgical practice: a. Small intestinal obstruction (MHPGMCET 2008) b. Duodenal fistula c. Pancreatic fistula d. Intussusception



3. Best investigation for acute intestinal obstruction is: (Recent Question 2014) a. Barium studies b. X-ray c. USG d. ERCP 4. Early sign of intestinal strangulation: (PGI SS June 2001) a. Continuous pain b. Abdominal rigidity and shock c. Abdominal fluid d. Dilated bowel loops on USG





5. The most common cause of small intestinal obstruction is:  (Recent Question 2014) a. Intussusception b. Iatrogenic adhesions c. Trauma d. Carcinoma



6. Commonest cause of acute intestinal obstruction is: (Recent Question 2016) a. Adhesions b. Volvulus c. Inguinal hernias d. Internal hernias



7. A 40 years old male was brought to the emergency with the history of multiple episodes of colicky pain, bilious vomiting with no passage of feces and flatus. X-ray abdomen was done. On the basis of findings, what is the diagnosis?  (Recent Question 2016) a. Duodenal obstruction b. Jejunal obstruction c. Ileal obstruction d. Colonic obstruction



9. The commonest cause of intestinal obstruction in a 30 years old Indian female: (All India 93) a. TB stricture b. Crohn’s disease c. Postoperative adhesions d. Adenocarcinoma



10. Acute intestinal obstruction is characterized by:  (PGI Dec 2003) a. Vomiting is common in duodenal obstruction b. Pain after each attack of vomiting is characteristic of ileal obstruction c. In colonic obstruction distension is common than vomiting d. X-ray erect posture is diagnostic e. Colicky pain to steady pain indicates strangulation

11. Features of intestinal obstruction: clinically/investigation by: (PGI June 2006) a. Abdominal distension b. Vomiting c. Fluid level in X-ray >4 d. Localized tenderness e. Diarrhea

12. In intestinal obstruction, investigations needed are:  (PGI Dec 2001) a. Barium swallow b. Intestinal barium meal c. Stomach barium meal d. Erect X-ray abdomen e. Supine X-ray abdomen

13. A 30-years old lady presented with acute pain abdomen, constipation and vomiting suspecting acute intestinal obstruction. The investigation of choice for the patient is: a. X-ray abdomen erect posture (PGI June 2003) b. Ba enema c. USG d. CT scan

14. A women of 35-years, comes to emergency department with symptoms of pain in abdomen and bilious vomiting but no distension of bowel. Abdominal X-ray showed no air fluid level. Diagnosis is: (Recent Question 2014; AIIMS June 2009) a. CA rectum b. Duodenal obstruction c. Adynamic ileus d. Pseudo-obstruction

15. One of the following will always present with bilious vomiting: (All India 94) a. Pyloric stenosis b. Esophageal atresia c. Atresia of the 3rd part of the duodenum d. Malrotation of the gut



8. While doing emergency laparotomy for an intestinal obstruction, which organ would you first visualize to say whether it is small bowel or large bowed obstruction?  (AIIMS November 2018) a. Ileum b. Sigmoid colon c. Cecum d. Rectum

16. Distended abdomen in intestinal obstruction is mainly due to: (All India 95, PGI Dec 98) a. Diffusion of gas from blood b. Fermentation of residual food c. Bacterial action d. Swallowed air

17. In case of new born, the commonest cause of intestinal obstruction is: (AIIMS Nov 95) a. Annular pancreas b. Duodenal atresia c. Jejunal atresia d. Esophageal atresia

Section 3

Gastrointestinal Surgery

396

Surgery Essence

18. Regarding adhesive intestinal obstruction, true is:  (AIIMS Nov 94) a. Avoid surgery for initial 48–72 hours b. Never operate c. Operate after minimum 10 days of conservative treatment d. Immediate operation



19. Water loss is severe if intestinal obstruction occurs at:  (JIPMER 90) a. First part of duodenum b. Third part of duodenum c. Mid-jejunum d. Ileum



20. The first to appear in a cause of acute intestinal obstruction is: (Recent Quetions 2016) a. Constipation b. Colicky pain c. Vomiting d. Distension



21. Primary feature of small intestinal obstruction: (APPG 96) a. Fever b. High peristalsis with colic c. Abdominal distension d. Empty rectum



22. Best way to diagnose lower small intestinal obstruction: a. Pain abdomen (PGI 96) b. Abdominal distension c. Profuse vomiting d. Multiple air gas shadows on X-ray

23. For intestinal obstruction immediate operation should not be done in case of: (Kolkata 2000) a. Postop adhesion b. Appendix perforation c. Volvulus d. Obstructed hernia

24. Which of the following is most suggestive of neonatal small bowel obstruction? (All India 2003) a. Generalized abdominal distension b. Failure to pass meconium in the first 24 hours c. Bilious vomiting d. Refusal of feeds



25. What is the sure sign of intestinal obstruction?  (Recent Question 2013) a. Vomiting and distension b. Jelly like stool c. Diarrhoea d. Localized tenderness



26. Ileal obstruction due to round worm obstruction treatment is: (Recent Question 2013) a. Resection with end to end anastomosis b. Resection with side to side anastomosis c. Enterotomy, removal of worms and primary closure d. Diversion



27. Investigation of choice for intermittent GI obstruction:  (Recent Question 2017) a. X-ray b. USG c. Enteroclysis d. Barium meal follow-through



28. Step ladder pattern of gas shadow is seen in:  (Recent Question 2017) a. Duodenal obstruction b. Intestinal obstruction c. Gastric outlet obstruction d. Sigmoid volvulus



30. All statement about adult intussusception are true except:  (PGI Nov 2009) a. Idiopathic and more enteric rather than colonic b. Lead point present in majority of cases c. Resection of bowel is adequate for large bowel intussusception d. Hydrostatic reduction with barium or air are done if bowel is not gangrenous



31. A previously healthy infant presents with recurrent episode of abdominal pain. The mother says that the child has been passing altered stool after episodes of pain, but gives no history of vomiting or bleeding per rectum. Which of the following is the most likely diagnosis? (All India 2011) a. Rectal polyps b. Intussusception c. Meckel’s diverticulum d. Necrotizing enterocolitis



32. True about Intussusceptions in children: a. Most common variety is ileocolic b. Associated with pathological lead point c. May be seen after viral infection d. Can be relieved by barium enema e. Surgery is always indicated



33. True statement about treatment of intussusception: a. Air enema (PGI Nov 2010) b. Saline enema c. Ba enema d. Hydrostatic reduction e. Colonoscopy is always done to confirm diagnosis



34. A 10-month-old child was brought to your hospital having recurrent attacks of pain abdomen. On examination, sausage shaped mass was palpable in the right lumbar region. Barium enema was done. What is the diagnosis on the basis of barium enema findings? a. Colorectal polyp b. Rectal prolapse c. Intussusception d. Carcinoma colon



35. Intussusception usually begins from: (DNB 2007) a. Jejunum b. Terminal ileum c. Colon d. Rectum



36. The most common type of intussusception: (MCI June 2018, DNB 2009, 2005, 2001, 2000, MHPGMCET 2009) a. Ileocolic b. Colocolic c. Ileoileal d. Retrograde

INTUSSUSCEPTION 29. A neonate presents with colicky pain and vomiting with sausage-shaped lump in the abdomen, diagnosis is:  (UPPG 2009) a. Enterocolitis b. Perforation of the abdomen c. Intussusception d. Acute appendicitis

(PGI Nov 2010)

Intestinal Obstruction

38. A child was operated for small intestine mass with intussusception and after the operation the tumor was diagnosed in histological section. Which is the most likely tumor associated? (AIIMS Nov 2012) a. Carcinoid b. Villous adenoma c. Lymphoma d. Smooth muscle tumor



39. What is intussuscipiens? (Recent Question 2014) a. The entire complex of intussusception b. The entering layer c. The outer layer d. The process of reducing the intussusception 40. What is the name of sign in this barium enema done in the patient of intussusception? a. Meniscus sign b. Claw sign c. Coiled spring sign d. Target sign



46. Intussusception is frequently associated with:  (JIPMER 2014, 2012) a. Submucosal lipoma b. Intramural lipoma c. Subserusal lipoma d. Subfascial lipoma



47. Which of the following is true about intussusception? a. Common in neonates (DPG 2005) b. Fever always present c. Not associated with tumors of intestine d. Usually relieved by barium enema



48. The least common type of intussusception is:  (Recent Question 2016) a. Multiple b. Colocolic c. Ileoileal d. Ileoilecolic



49. Henoch-Schnolein purpura may rarely cause: a. Intussusception b. Volvulus c. Atrial fibrillation d. Hernia



50. Claw sign seen in: a. Intussusception c. Both



51. A young patient was brought to the emergency with colicky abdominal pain, bilious vomiting. CT findings are given below. What is the name of this sign? a. Claw sign b. Coiled spring sign c. Meniscus sign d. Target sign



44. Recurrent obstruction, mass per rectum and diarrhea in child: (Recent Question 2014) a. Intussusception b. Rectal prolapsed c. Internal hernia d. Hemorrhoids



45. Commonest cause of intussusception is: (WBPG 2015) a. Submucous lipoma b. Meckel’s diverticulum c. Hypertrophy of submucous Peyer’s patches d. Polyp

(APPG 2008)



52. Complete treatment of intussusception is indicated by:  (APPG 2008) a. Free passage of barium in the terminal ileum b. Passage of feces and flatus along with barium c. Improvement of clinical condition d. None



53. A 12-months old male child suddenly draws up his legs and screams with pain. This is repeated periodically throughout the night interspersed with periods of quiet sleep. When seen after 12 hours the child looks pale, has just vomited and passed thin blood-stained stool; there is a mass around umbilicus. What is the most likely diagnosis? (UPSC 97) a. Appendicitis b. Intussusception c. Gastroenteritis d. Roundworm obstruction



54. Sign of Dance is: (MAHE 2005) a. Empty right iliac fossa in intussusception b. Pincer shaped appearance in barium enema in intussusception c. Tenderness at the McBurney’s point d. Passing of large quantities of urine in hydronephrosis

42. Features of intussusceptions are: (PGI June 2001) a. Pincer sign b. Target sign c. Dove sign d. Coiled spring sign e. Dance sign

43. Recurrent pain abdomen with intestinal obstruction and mass passes per rectum goes in favor of: (PGI Dec 99) a. Internal herniation b. Stricture c. Strangulated hernia d. Intussusception

b. Volvulus d. None

Gastrointestinal Surgery

41. A 10-months old infant presents with acute intestinal obstruction. Contrast enema X-ray shows the intussusceptions. Likely cause is: (All India 2002) a. Payers patch hypertrophy b. Meckel’s diverticulum c. Mucosal polyp d. Duplication cyst



Section 3

37. All are true about intussusception except: (AIIMS GIS May 2011) a. Barium must be used in children after 48 hours b. X-ray shows paucity of colonic gas c. X-ray shows lump shadow d. USG show pseudo-kidney sign

397

398

Surgery Essence

Gastrointestinal Surgery

55. Which of the following is true about intussusception in children? (Recent Question 2017) a. Most common type is ileocecocolic b. Most common type is colocolic c. Entering tube is intussuscipiens d. Contrast enema is not useful in the management 56. A patient has acute abdominal pain with blood and mucus in stool with palpable mass per abdomen is due to:  (Recent Question 2014; AIIMS June 2000) a. Meckel’s diverticulum b. Volvulus c. Intussusception d. Hypertrophic pyloric stenosis



58. A new born child has not passed meconium for 48 hours. What is the diagnostic procedure of choice? (All India 2008) a. USG b. Contrast enema c. CT d. MRI

1-Paul Mikulicz, 2-Bishop-Koop, 3-Santulli 1-Paul Mikulicz, 2- Santulli, 3- Bishop-Koop 1- Santulli, 2-Bishop-Koop, 3- Paul Mikulicz 1- Bishop-Koop, 2- Paul Mikulicz, 3-Santulli

65. Fluid levels are not visible in: (PGI June 98) a. Meconium ileus b. Intussusception c. Colon pouch d. Duodenal obstruction

66. A new born girl not passed meconium for 48 hours, has abdominal distention and vomiting. Initial investigation of choice would be: (AIIMS Nov 2007) a. Manometry b. Genotyping for cystic fibrosis c. Lower GI contrast study d. Serum trypsin immunoblot

MECONIUM SYNDROME

67. Meconium peritonitis occurs: (PGI June 99) a. Just before birth b. Just after birth c. Before and after birth d. Due to birth trauma

MESENTERIC ISCHEMIA

68. Occlusion to superior mesenteric artery affects jejunum and: a. Pyloric antrum (UPPG 2010) b. Fundus of stomach c. Duodenum distal to the opening of CBD d. Greater curvature e. Descending colon



59. Snow storm ascites is seen in: (APPG 2005) a. Meconium ileus b. Hirschsprung’s disease c. Ileocaecal tuberculosis d. Pseudomyxoma peritonei





60. Extraluminal abdominal calcifications in the newborn may be seen in: (COMEDK 2010) a. Meconium aspiration b. Hirschsprung’s disease c. Meconium peritonitis d. Meconium plug syndrome

69. Most common cause of acute mesenteric ischemia is:  (AIIMS May 2011, Nov 2008) a. Arterial thrombosis b. Venous thrombosis c. Embolism d. Non occlusive disease





61. Intra-abdominal calcification in a plane X-ray abdomen is most often seen in: (Karnataka 95) a. Meconium ileus b. Meconium peritonitis c. Meconium plug syndrome d. Necrotising enterocolitis

70. All are causes of dynamic intestinal obstruction except: a. Gallstones b. Bands (JIPMER 2010) c. Intussusception d. Mesenteric vascular occlusion





62. Which one of the following statements regarding meconium peritonitis is not correct? (UPSC 2000) a. It is a septic peritonitis b. It develops in later intra-uterine life or during or just after delivery c. This condition should always be considered when a baby is born with tense abdomen d. Plain X-ray abdomen of this condition reveals calcification on liver and spleen

71. A young female complains of pain in umbilical region since few days, which is more especially after taking meals. What is the likely diagnosis? (MHSSMCET 2005) a. Peptic ulcer disease b. Meckel’s diverticulum c. Typhlitis d. Abdominal angina



72. If patient with superior mesenteric artery thrombosis doesn’t develop collateral circulation, then what is the best treatment?  (MHSSMCET 2005) a. Resection anastomosis b. Endarterectomy c. Arterial reconstruction d. Papaverine injection

63. Meconium ileus is associated with: (All India 2000, 98, 96) a. Fibrocystic disease of pancreas b. Liver aplasia c. Cirrhosis of liver d. Malnutrition

73. Which of the following is a common cause of abdominal angina? (MHSSMCET 2006) a. SMA thrombosis b. SMA embolism c. SMA aneurysm d. SMA rupture



Section 3

57. A 6 months old child woke up in night, crying with abdominal pain, which got relieved on passing red stool. What is the most likely diagnosis? (AIIMS November 2014) a. Meckel’s diverticulum b. Intussusception c. Malrotation d. Intestinal obstruction

a. b. c. d.

64. Which of the following statement is correct about the surgical management of meconium ileus?



74. Ischemia of which of the vessel would cause least damage? a. Renal artery b. SMA (AIIMS Nov 2011) c. IMA d. Celiac trunk 75. All are true about non-obstructive mesenteric ischemia except: (AIIMS GIS Dec 2006) a. Vasopressor treatment b. Cardiac shock c. Burns d. Hypercoagulable state 76. String of lakes appearance on angiography is seen in: a. Chronic mesenteric ischaemia  (Recent Question 2016) b. Non-occlusive mesenteric ischemia c. Mesenteric venous thrombosis d. Mesenteric arterial thrombosis

Intestinal Obstruction

78. All are true about mesenteric ischemia except: a. Due to embolism to SMA (AIIMS GIS 2003) b. Most common cause is AF c. Embolus gets lodged most commonly at branching of SMA from aorta d. Most common cause of small bowel syndrome in adults

79. Type of mesenteric ischemia best visualized by CECT:  (AIIMS GIS Dec 2006) a. Mesenteric ischemia by embolic occlusion b. Acute mesenteric artery thrombosis c. Non-occlusive mesenteric ischemia d. Acute mesenteric venous thrombosis



80. A 65-years old Ramdeen presents with abdominal pain and distension of abdomen. His stools were maroon colored and he gives a past history of cerebrovascular accident and myocardial infarction. What will be the probable diagnosis?  (AIIMS Nov 2000, Nov 97) a. Ulcerative colitis b. Acute mesenteric ischemia c. Irritable bowel syndrome d. Crohn’s disease 81. A man aged 60 years has history of IHD and atherosclerosis. He presents with abdominal pain and maroon stools. Most likely diagnosis: (All India 2001) a. Acute intestinal obstruction b. Acute mesenteric ischemia c. Peritonitis d. Appendicitis 82. True about mesenteric vein thrombosis: (PGI Dec 2003) a. Peritoneal signs always present b. Thrombectomy is always done c. Heparin is given d. Surgery can lead to short bowel syndrome



83. Identify the following vessel in this arteriography:  (Recent Question 2018) a. Superior mesenteric artery b. Inferior mesenteric artery c. Splenic artery d. Common hepatic artery



84. Post-operative ileus is most pronounced in:  (AIIMS Nov 2011, AIIMS GIS Dec 2010) a. Colon b. Stomach c. Ileum d. Duodenum



85. Paralytic ileus is seen in: a. Spinal cord injury c. Hypermagnesemia

(PGI June 99) b. Hypocalcemia d. Uremia

86. Prolonged postoperative ileus is best treated by:(PGI Dec 98) a. Long tube insertion b. Calcium pentonthenate c. Laparotomy and exploration d. Peristaltic stimulants

87. First to recover from post-operative ileus:  (Recent Question 2016, DNB 2014, 2008) a. Small intestine b. Stomach c. Colon d. None



88. Paralytic ileus is characterized by all except: (SGPGI 2005) a. No bowel sound on ausculatation b. No passage of flatus c. Gas filled loops of intestine with multiple fluid levels d. Loops of intestine are not seen due to loss of peristalsis

89. Routine management of paralytic ileus includes all of the following except: (MCI March 2005) a. Electrolyte correction b. Nasogastric aspiration c. Parasympathomimetics d. IV fluids 90. First to recover from paralytic ileus: (Recent Question 2017) a. Stomach b. Small intestine c. Rectum d. Colon

91. True about postoperative ileus is: (MHPGMCET 2003) a. No intestinal sounds heard b. Intestinal peristalsis never becomes normal again c. Is due to hypernatremia d. Begins 2-3 days postoperatively

92. Most common electrolyte imbalance that causes paralytic ileus is: (DNB 2014) a. Hyponatremia b. Hypernatremia c. Hypokalemia d. Hyperkalemia

LARGE BOWEL OBSTRUCTION

93. A patient has presented with left sided colonic malignancy with obstruction in emergency. What is the treatment of choice?  (MHSSMCET 2005) a. Hartman’s Procedure b. Total colectomy c. Left hemi-colectomy d. Defunctioning colostomy



94. Treatment of obstructing, resectable right colonic cancer: (AIIMS GIS Dec 2010) a. b. c. d.



Ileotransverse anastomosis after right hemicolectomy Diverting ileostomy Subtotal colectomy with ileorectal anastomosis Subtotal colectomy with exteriorization of the both ends

95. A patient of CA rectum presents with obstruction. Treatment: (PGI SS June 2006) a. Defunctioning colostomy b. Hartmann’s procedure c. Anterior resection d. Abdomino-perineal resection

Gastrointestinal Surgery



PARALYTIC ILEUS

Section 3



77. All are true about acute mesenteric ischemia except: a. Branch point of middle colic artery is most common location for embolism (AIIMS GIS 2003) b. Acute venous thrombosis is best judged on CT c. Non-obstructive mesenteric ischemia has very good prognosis d. Gold standard investigation is angiography

399

Gastrointestinal Surgery

400

Surgery Essence

96. What are the features of colonic obstruction? (PGI Dec 2000) a. No passage of gas absolutely (Obstipation) b. No passage of stools absolutely c. Distention of abdomen d. Mild fever initially e. Fecal vomitus

108. A 56-year‑old woman has not passed stools for the last 14 days. X-ray shows no air fluid levels, Probable diagnosis is: a. Paralytic ileus (Recent Question 2014; All India 2001) b. Aganglionosis of the colon c. Intestinal pseudo-obstruction d. Duodenal obstruction



97. Most common cause of colonic obstruction is:  (Recent Question 2016) a. Volvulus b. Hernia c. Adhesions d. Neoplasm

109. Acute pseudo-obstruction of the colon known as: (DNB 2012, UPPG 2007) a. Sjogren’s syndrome b. Gardener’s syndrome c. Ogilvie’s syndrome d. Peutz-Jegher’s syndrome



98. In obstruction of the large gut rupture occurs at the: a. Cecum b. Ascending colon c. Transverse colon d. Descending colon



99. Commonest cause of colonic obstruction in neonates is: a. Meconium ileus b. Aganglionic colon c. Ileal atresia d. Volvulus

100. Acute mechanical large bowel obstruction should be operated early because: (UPSC 95) a. Electrolyte imbalance due to third space loss b. Septicemia from absorption of bowel contents c. Early gangrene and perforation d. Respiratory embarrassment to massive abdominal distension 101. Acute mechanical large bowel obstruction should be operated early because of the risk of: (UPSC 97) a. Respiratory embarrassment due to abdominal distension b. Electrolyte imbalance from vomiting c. Septicemia from bowel contents d. Closed-loop obstruction and cecal perforation

PSEUDO-OBSTRUCTION 102. Ogilvie’s syndrome results from the denervation of the colon distal to the: (COMEDK 2006) a. Hepatic flexure b. Mid-transverse colon c. Splenic flexure d. Descending colon 103. True about Ogilvie’s syndrome are all except:  (Recent Question 2016, 2015; AIIMS Nov 2007) a. It is caused by mechanical obstruction of the colon b. It involves entire/part of the large colon c. It occurs after previous surgery d. It occurs commonly after narcotic use

VOLVULUS 110. Sigmoid volvulus: (PGI SS Dec 2009) a. Clockwise b. CECT is diagnostic c. Rigid sigmoidoscopy is the initial treatment d. Sigmoidectomy is contraindicated 111. Most common site of volvulus: (DNB 2012, GB PANT 2011) a. Sigmoid colon b. Caecum c. Transverse colon d. Stomach 112. False about volvulus: (AIIMS GIS May 2008) a. Sigmoid volvulus is most common b. In absence of ischemia, mesocolopexy is done c. Ogilvie’s syndrome refers to cecal volvulus d. Elective sigmoid resection after detorsion 113. False regarding cecal volvulus: (ILBS 2011) a. Present with small bowel obstruction b. Present with air fluid levels in right upper quadrant and convexity towards left c. Endoscopic derotation is not effective like sigmoid volvulus d. Cecopexy can be a form of treatment 114. This characteristic appearance is seen in: a. Gastric volvulus b. Small intestinal volvulus c. Cecal volvulus d. Sigmoid volvulus

Section 3

104. Investigation of choice for pseudo-obstruction: a. Water soluble contrast enema (AIIMS GIS Dec 2006) b. Barium enema c. CECT d. Colonoscopy 105. Colonic Pseudo-obstruction occurs in all, except:  (AIIMS June 94) a. Diabetes mellitus b. Dermatomyositis c. Scleroderma d. Hyperthyroidism 106. Ogilvie’s syndrome: (Recent Question 2017) a. Acute colonic pseudo-obstruction b. Chronic colonic pseudo-obstruction c. Fecal obstruction d. Intussusception 107. Most common cause of Ogilvie’s syndrome:  (Recent Question 2017) a. Head injury b. Electrolyte abnormalities c. Carcinoma d. Drugs

115. In cecal volvulus: (AIIMS GIS Dec 2006) a. Resolves with endoscopic treatment as frequently as sigmoid volvulus b. Right hemicolectomy is the treatment of choice c. Conservative management d. Colonoscopic decompression 116. Obstruction with multiple air fluid levels in a newborn, suggest diagnosis of: (MHSSMCET 2010) a. Duodenal atresia b. Ileal atresia c. Ladd’s band d. Midgut volvulus

Intestinal Obstruction

118. True about colonic volvulus: a. Most common in caecum b. Common in psychiatric patient c. Bird’s beak sign d. May present as intestinal obstruction

(PGI Nov 2010)

119. False about cecal volvulus: (AIIMS GIS 2003) a. Mostly resolve with colonoscopic reduction b. More common than cecal basecule c. Right hemicolectomy is TOC d. Truly is cecocolic volvulus 120. Which of the following statement about volvulus is false? a. More common in psychiatric patients (All India 2008) b. Sigmoid volvulus is more common than caecal volvulus c. Lower GI scopy is contraindicated in sigmoid volvulus d. Volvulus of caecum is managed by conservative methods 121. Coffee bean sign is usually seen in: (J and K 96) a. Volvulus b. Pyloric obstruction c. Intussusception d. Intestinal obstruction 122. This characteristic appearance is seen in: (Recent Question 2017) a. Gastric volvulus b. Small intestinal volvulus c. Cecal volvulus d. Sigmoid volvulus

128. All are predisposing factors for sigmoid volvulus except:  (DNB 2007) a. Hirschprung’s diseases b. Chagas diseases c. Chronic constipation d. Tuberculosis 129. Least common volvulus site in neonate is: (JIPMER 2013) a. Ilioileal b. Large bowel volvulus c. Small bowel volvulus d. Gastric volvulus 130. Midgut volvulus symptoms appear at: (Recent Question 2017) a. 1st week b. 2nd week c. 3rd week d. 4th week

MALROTATION 131. Child presents with recurrent abdominal pain and bilious vomiting. Condition was diagnosed by barium follow through. Surgery was done, mesenteric widening, appendectomy, cutting the Ladd’s band. What is the diagnosis? a. Recurrent cecal volvulus  (AIIMS Nov 2010) b. Malrotation c. Recurrent appendicitis d. Stricture TB 132. The cecum is found to be placed below the stomach and in the midline. Which of the following abnormality must have taken place while rotation of the gut? (AIIMS Nov 2010) a. Non-rotation b. Malrotation c. Reversed rotation d. Mixed rotation 133. Malrotation presents as: a. Mass abdomen c. Bilious vomiting

(PGI Dec 2002) b. Bleeding PR d. Hematemesis

124. Predisposing factors for sigmoid volvulus are: a. Band of adhesion b. Long pelvic mesocolon c. Narrow attachment of pelvic mesocolon d. Loaded pelvic colon e. All of the above 125. Rotation of sigmoid volvulus occurs: a. Clockwise b. Anticlockwise c. Initially clockwise later anticlockwise d. Either clockwise or anticlockwise

(Kerala 97)

(AMU 2005)

126. Sigmoid volvulus rotation occurs: (UPPG 2007, 2005) a. Clockwise b. Anticlockwise c. Both clockwise and anti clockwise d. Axial in direction 127. Which of the following statements abut sigmoid volvulus is incorrect? (DPG 2009 March) a. More common with laxative abuse b. Non-operative treatment has no role c. Recurrence rate around 40% d. Sigmoid resection is definitive treatment

134. Bowel can get strangulated in all of the following space except: (AIIMS Nov 2000) a. Rectouterine pouch b. Ileocolic recess c. Paraduodenal recess d. Omental recess 135. Spastic ileus is seen in: a. Porphyria c. Hypokalemia

(PGI Dec 99) b. Retroperitoneal abscess d. MI

136. True about visceral pain: (PGI Nov 2011) a. It is poorly localized b. Resembles “fast pain” produced by noxious stimulation of the skin c. Mediated by B fibers in the dorsal roots of the spinal nerves d. Causes relaxation of nearby skeletal muscles e. Shows relatively rapid adaptation 137. A young healthy male patient presented with abdominal pain and history of altered bowel habits from the last 6 months. On CT examination, there was dilated distal part of ileum, thickened ileocecal junction with thickened cecum with presence of sacculations on the antimesenteric border. The vascularity of adjoining mesentery is also increased and there is surrounding mesentery fat. Which of the following is not a differential diagnosis? (AIIMS Nov 2013) a. Ulcerative colitis b. Crohn’s disease c. Tuberculosis d. Ischemic bowel disease

Gastrointestinal Surgery

MISCELLANEOUS

123. Definitive treatment of sigmoid volvulus is: (PGI June 97) a. Surgical correction b. Colectomy c. Enema d. Endoscopic correction

Section 3

117. ‘Bird of prey’ sign is seen in the radiographic barium examination of: (COMEDK 2008) a. Gastric volvulus b. Intussusception c. Sigmoid volvulus d. Cecal volvulus

401

Explanations ACUTE INTESTINAL OBSTRUCTION

1. Ans. b. Usually venous blood flow affected first (Ref: Sabiston 20/e p1252; Schwartz 10/e p1147; Bailey 27/e p1281) • Venous return is compromised before arterial supply in strangulated intestinal obstructionQ.



2. Ans. a. Small intestinal obstruction (Ref: Bailey 24/e p56) Hyponatremia • The most frequent cause of sodium depletion in surgical practice is obstruction of the small intestineQ, with its rapid loss of gastric, biliary, pancreatic and intestinal secretions by antiperistalsis and ejection, whether by vomiting or aspiration. • Duodenal, total biliary, pancreatic and high intestinal external fistulae are all notorious for bringing about early and profound hyponatremiaQ.



3. Ans. b. X-ray

4. Ans. a. Continuous pain (Ref :Sabiston 20/e p1252; Schwartz 10/e p1147; Bailey 27/e p1281)



5. Ans. b. Iatrogenic adhesions

6. Ans. a. Adhesions



8. Ans. c. Cecum (Ref: Bailey 27/e p1294)

7. Ans. b. Jejunal obstruction

“After full resuscitation, the abdomen should be opened through a midline incision. Care should be taken to ensure that the loss of tamponade of the abdominal wall does not lead to increased caecal distension and rupture (this starts with splitting along the line of the taenia coli on the antimesenteric border). Distension of the caecum will confirm large bowel involvement. Identification of a collapsed distal segment of the large bowel and its sequential proximal assessment will readily lead to identification of the cause.” –Bailey 27/e p1294

9. Ans. c. Postoperative adhesions



10. Ans. a. Vomiting is common in duodenal obstruction, c. In colonic obstruction distension is common than vomiting, d. X-ray erect posture is diagnostic, e. Colicky pain to steady pain indicates strangulation (Ref: Sabiston 20/e p1249-1252; Schwartz 10/e p1147-1149; Bailey 27/e p1281)



11. Ans. a. Abdominal distension, b. Vomiting



13. Ans. a. X-ray abdomen erect posture



14. Ans. b. Duodenal obstruction (Ref: Sabiston 20/e p1249; Schwartz 10/e p1147; Bailey 27/e p1285-1286)

12. Ans. d. Erect X-ray abdomen, e. Supine X-ray abdomen

• Abdominal pain, bilious vomiting without abdominal distention is suggestive of proximal small intestinal obstruction, distal to ampulla of Vater (Duodenal obstruction). • Nausea and vomiting are more common with proximal obstructionQ • Abdominal distention is more common in distal obstructionQ

15. Ans. c. Atresia of the 3rd part of the duodenum



16. Ans. d. Swallowed air (Ref: Sabiston 20/e p1250; Schwartz 10/e p1147; Bailey 26/e p1186) Consideration of Laparoscopic Management in Patients with Gas Fluid • Swallowed airQ is the major source (NitrogenQ is not well • Enormous quantities of fluid from the extracellular Absorbed by intestinal mucosa) Space are lost into gut (third space loss) • Gases produced by bacterial fermentation (H2, CO2, CH4) • Net GI secretion is enhanced in obstruction



17. Ans. b. Duodenal atresia (Ref: Bailey 27/e p133, 1293) • MC site of intestinal atresia: DuodenumQ • MC cause of neonatal intestinal obstruction: Duodenal atresiaQ



18. Ans. a. Avoid surgery for initial 48–72 hours (Ref: Sabiston 20/e p1252; Schwartz 10/e p1149; Bailey 27/e p1293)

Treatment of Adhesive Obstruction • Initial management is based on intravenous rehydration and nasogastric decompression; occasionally, this treatment is curativeQ. • Although an initial conservative regimen is considered appropriate, regular assessment is mandatory to ensure that strangulation does not occurQ.

Intestinal Obstruction

403

• Conservative treatment should not be prolonged beyond 72 hoursQ

• When obstruction is caused by an area of multiple adhesions, the adhesions should be freed by sharp dissectionQ • To prevent recurrence, the bare area should be covered with Omental graftsQ. • Laparoscopic adhesiolysis may be considered in highly selected cases of chronic subacute obstructionQ

19. Ans. a. First part of duodenum



20. Ans. b. Colicky pain (Ref: Bailey 27/e p1285)

Section 3

• When, as is usual, laparotomy is required, although multiple adhesions may be found, only one may be causative. This should be divided and the remaining adhesions left in situQ unless severe angulation is present. Division of these adhesions will only cause further adhesion formation.

Symptoms of Intestinal Obstruction • Symptoms of intestinal obstruction: Pain, vomiting, distention and constipation • Pain is the first symptom encounteredQ; it occurs suddenly and is usually severe. Pain in Intestinal Obstruction • • • • •

It is colicky in nature and is usually centred on the umbilicus (small bowel) or lower abdomen (large bowel)Q The pain coincides with increased peristaltic activityQ With increasing distension, the colicky pain is replaced by a mild constant diffuse pain The development of severe pain is indicative of the presence of strangulationQ Pain may not be a significant feature in postoperative simple mechanical obstruction and does not usually occur in paralytic ileusQ



21. Ans. b. High peristalsis with colic

22. Ans. d. Multiple air gas shadows on X-ray



23. Ans. a. Postop adhesion

24. Ans. c. Bilious vomiting



25. Ans. a. Vomiting and distension



26. Ans. c. Enterotomy, removal of worms and primary closure (Ref: Schwartz 10/e p1149; Farquharson’s 8/e p470; Sabiston 20/e p1253)



27. Ans. c. Enteroclysis



28. Ans. b. Intestinal obstruction (Ref: Sabiston 20/e p1242) “Characteristic findings of small bowel obstruction on supine radiographs are dilated loops of small intestine, without evidence of colonic distention. Upright radiographs demonstrate multiple air-fluid levels, which often layer in a stepwise fashion.” –Sabiston 20/e p1242

INTUSSUSCEPTION

29. Ans. c. Intussusception (Ref: Sabiston 20/e p1879; Schwartz 10/e p1622; Bailey 27/e p1287; Shackelford 8/e p986, 7/e p1059-1061)



30. Ans. a. Idiopathic and more enteric rather than colonic (Ref: Bailey 27/e p1287)

Intussusception in Adults • Bailey says “Adult cases of intussusceptions are invariably associated with a lead point, which is usually a polyp (e.g. PeutzJegher’s syndrome), a submucosal lipoma or other tumorQ.” • “In adults, colocolicQ intussusception is common but in children, ileocolic is the commonest variety (77%).”

31. Ans. b. Intussusception



32. Ans. a. Most common variety is ileocolic, b. Associated with pathological lead point, c. May be seen after viral infection, d. Can be relieved by barium enema



33. Ans. a. Air enema, b. Saline enema, c. Ba enema, d. Hydrostatic reduction



34. Ans. c. Intussusception (Ref: Sabiston 20/e p1879; Schwartz 10/e p1622; Bailey 27/e p1289; Shackelford 8/e p986, 7/e p1060-1061)

Gastrointestinal Surgery

• Resection of small bowel is done when the affected bowel segment is of questionable viability. • Diversion is the first step in case of colonic obstruction, followed by resection and anastomosis of affected segment and then closure of diversion colostomy at a later date. • Intestinal luminal obstruction such as due to Bezoars or fecolihs of worm intestations are dealt with by enterotomy and removal followed by primary closure.

Gastrointestinal Surgery

404

Surgery Essence Radiological Investigations in Intussusception Plain X-ray Film Barium Enema • Features of small intestinal obstructionQ • Claw signQ: Rounded apex of intussusception protrudes into the • Abdominal soft tissue density in some cases which contrast column may show: Q • Coiled spring signQ: Edematous −− Target sign : Soft tissue mass with concentric area mucosal folds of returning limb of lucency due to mesenteric fat. of intussusception outlined by −− Meniscus signQ: Crescent of gas within colonic contrast material. lumen that outlines the apex of intussusception.

Ultrasound • Target sign or Bull’s eye sign on transverse scanQ • Pseudokidney sign on longitudinal scanQ



35. Ans. b. Terminal ileum



37. Ans. a. Barium must be used in children after 48 hours (Ref: Sabiston 20/e p1879; Schwartz 10/e p1622; Bailey 27/e p1293; Shackelford 8/e p986, 7/e p1060-1061)

36. Ans. a. Ileocolic



38. Ans. c. Lymphoma (Ref: Sabiston 20/e p1879; Schwartz 10/e p1622; Bailey 27/e p1284; Shackelford 8/e p986, 7/e p771)



39. Ans. c. The outer layer



42. Ans. b. Target sign, d. Coiled spring sign, e. Dance sign

43. Ans. d. Intussusception



44. Ans. a. Intussusception

45. Ans. c. Hypertrophy of submucous Peyer’s patches



46. Ans. a. Submucosal lipoma

47. Ans. d. Usually relieved by barium enema



48. Ans. a. Multiple (Ref: Bailey 27/e p1284)

40. Ans. c. Coiled spring sign

41. Ans. a. Payers patch hypertrophy

Types of Intussusception (in decreasing order) 1. Ileocolic (77%)Q 4. Colocolic (2%): MC in adults 2. Ileo-ileo-colic (12%) 5. Multiple (1%) 3. Ilioileal (5%) 6. Retrograde (0.2%)

49. Ans. a. Intussusception



51. Ans. d. Target sign (Ref: Sabiston 20/e p1879; Schwartz 10/e p1622; Bailey 27/e p1284)

50. Ans. a. Intussusception



52. Ans. a. Free passage of barium in the terminal ileum

• The given sign is target sign, seen in intussusception.

• In intussusception, successful reduction is confirmed by reflux of air (or barium)Q into the small bowel

53. Ans. b. Intussusception



55. Ans. a. Most common type is ileocecocolic (Ref: Sabiston 20/e p1879; Schwartz 10/e p1622; Bailey 27/e p1283)

54. Ans. a. Empty right iliac fossa in intussusceptions



56. Ans. c. Intussusception

57. Ans. b. Intussusception

MECONIUM SYNDROME



Section 3



58. Ans. b. Contrast enema (Ref: Sabiston 20/e p1876; Schwartz 10/e p1617-1619; Shackelford 8/e p979, 7/e p1053-1054) • Failure to pass meconium in the first 2 days of life (48 hours) is typically suggestive of a lower GI tract obstruction such as Hirschprung’s disease, meconium plug syndrome or anorectal malformation. A contrast enema is the most suitable primary investigation to make the diagnosis, amongst the options provided. 59. Ans. a. Meconium ileus (Ref: Sabiston 20/e p1875; Schwartz 10/e p1617-1619; Bailey 27/e p1294, 134; Shackelford 8/e p978, 7/e p1053-1054) 60. Ans. c. Meconium peritonitis (Ref: Sabiston 20/e p1876; Schwartz 10/e p1617-1619; Bailey 27/e p134; Shackelford 8/e p979, 7/e p1054) 61. Ans. b. Meconium peritonitis 62. Ans. a. It is a septic peritonitis • Meconium peritonitis is an aseptic chemical peritonitis caused by spillage of meconium 63. Ans. a. Fibrocystic disease of pancreas (Ref: Sabiston 20/e p1875; Schwartz 10/e p1617; Shackelford 8/e p978, 7/e p1053-1054) • Meconium ileus in the newborn represents the earliest clinical manifestation of CF and affects roughly 15% of patients with this inherited disease

Cystic Fibrosis Diagnosis • The diagnosis of CF is usually confirmed in the postoperative periodQ. • The pilocarpine iontophoresis sweat test revealing a chloride concentration >60 mEq/L is the most reliable and definitive method to confirm the diagnosis of CFQ. This test may not be reliable in infants and is usually performed later • A more immediate test includes detection of the mutated CFTR geneQ. This test, coupled with a careful family history and clinical presentation, permits confirmation of the diagnosis in most infantsQ.

Intestinal Obstruction

405

64. Ans. a. 1-Paul Mikulicz, 2-Bishop-Koop, 3-Santulli (Ref: Sabiston 20/e p; Schwartz 10/e p1619; Bailey 27/e p1294)



Surgical Management of Meconium Ileus Bishop-Koop’s Distal Chimney Ileostomy



65. Ans. a. Meconium ileus



67. Ans. a. Before and after birth

Santulli’s Proximal Chimney Ileostomy

Section 3

Paul Mikulicz Double Barrel Ileostomy

66. Ans. c. Lower GI contrast study

• Meconium peritonitis may occur in utero or the early neonatal period.

MESENTERIC ISCHEMIA 68. Ans. c. Duodenum distal to the opening of CBD (Ref: Bailey 27/e p1253)



Superior Mesenteric Artery • SMA supplies the midgut, from distal part of duodenum to proximal 2/3rd of transverse colonQ. 69. Ans. c. Embolism (Ref: Sabiston 20/e p1155; Schwartz 10/e p859-866; Bailey 27/e p1253; Shackelford 8/e p1028, 7/e p1075-1077)



70. Ans. d. Mesenteric vascular occlusion



71. Ans. d. Abdominal angina (Ref: Sabiston 20/e p1155; Schwartz 10/e p859-866; Bailey 27/e p1253; Shackelford 8/e p1033, 7/e p1084-1092)



72. Ans. a. Resection anastomosis

73. Ans. a. SMA thrombosis

74. Ans. c. IMA (Ref: Grays 40/e p1141)

• Grays Anatomy: Occlusion of IMA does not always result in irreversible ischemia of the descending and sigmoid colon, because the marginal artery of colon usually receives an adequate supply from the left branch of the middle colic arteryQ

75. Ans. d. Hypercoagulable state (Ref: Sabiston 20/e p1155; Schwartz 10/e p859-866; Bailey 27/e p1253; Shackelford 8/e p1031, 7/e p1081-1082)



76. b. Non-occlusive mesenteric ischemia 77. Ans. c. Non-obstructive mesenteric ischemia has very good prognosis



78. Ans c. Embolus gets lodged most commonly at branching of SMA from aorta (Ref: Sabiston 20/e p1155; Schwartz 10/e p859-866; Bailey 27/e p1253; Shackelford 8/e p1028, 7/e p1077-1078)



79. Ans. d. Acute mesenteric venous thrombosis (Ref: Sabiston 20/e p1155; Schwartz 10/e p918-927; Bailey 27/e p1253; Shackelford 8/e p1031, 7/e p1082-1084)



80. Ans. b. Acute mesenteric ischemia (Ref: Sabiston 20/e p1155; Schwartz 10/e p859-866; Bailey 27/e p1253; Shackelford 8/e p1028, 7/e p1077-1082)



81. Ans. b. Acute mesenteric ischemia



83. Ans. a. Superior mesenteric artery (Ref: Sabiston 20/e p1239; Schwartz 10/e p863)

• More than 50% of emboli lodge in the mid to distal segment of the SMA.

82. Ans. c. Heparin is given, d. Surgery can lead to short bowel syndrome

• Coronal maximum intensity projection of the superior mesenteric artery and its branches. This image was created on a workstation with CT data from a state-of-the-art 16-slice multidetector CT scanner. Note the fine detail that is visible of the end-organ arteries of the jejunum and ileum.

PARALYTIC ILEUS

84. Ans. a. Colon (Ref: Sabiston 20/e p306; Schwartz 10/e p1151-1152; Bailey 27/e p1297)



85. Ans. a. Spinal cord injury, c. Hypermagnesemia, d. Uremia (Ref: Sabiston 20/e p306; Schwartz 10/e p1151-1152; Bailey 27/e p1297)



86. Ans. c. Laparotomy and exploration (Ref: Bailey 27/e p1297) • If paralytic ileus is prolonged and threatens life, a laparotomy should be considered to exclude a hidden cause and facilitate bowel decompression.

Gastrointestinal Surgery



406

Surgery Essence

87. Ans. a. Small intestine



88. Ans. d. Loops of intestine are not seen due to loss of peristalsis



89. Ans. c. Parasympathomimetics (Ref: Bailey 25/e p1202)

Gastrointestinal Surgery

• Rarely, in resistant cases, medical therapy with an adrenergic blocking agent in association with cholinergic stimulation, e.g. neostigmine (the Catchpole regimenQ), may be used, provided that an intraperitoneal cause has been excluded.

90. Ans. b. Small intestine (Ref: Sabiston 20/e p306; Bailey 27/e p1296)



91. Ans. a. No intestinal sounds heard

92. Ans. c. Hypokalemia

LARGE BOWEL OBSTRUCTION

93 Ans. d. Defunctioning colostomy (Ref: Sabiston 20/e p1337; Bailey 27/e p1265, Naingot 11/e p501) • Most common site of colorectal cancer is rectum, in obstructing carcinoma rectum loop colostomy or defunctioning colostomy should be done to relieve obstruction followed by neoadjuvant chemoradiation, with the plan to resect the primary lesion at a later time.



94. Ans. a. Ileotransverse anastomosis after right hemicolectomy



96. Ans. a. No passage of gas absolutely (Obstipation), b. No passage of stools absolutely, c. Distention of abdomen



97. Ans. d. Neoplasm

98. Ans. a. Cecum

100. Ans. c. Early gangrene and perforation

95. Ans. a. Defunctioning colostomy 99. Ans. b. Aganglionic colon

101. Ans. d. Closed-loop obstruction and cecal perforation

• Virtually all patients with complete acute large bowel obstruction require prompt surgical intervention and should not undergo a trial of non-operative management • Acute large bowel obstruction in patients with competent ileocecal valve is a true surgical emergency because of high chances of perforation (MC site: Cecum)Q • Once diagnosis has been made, surgical exploration should be undertaken as soon as possible after appropriate resuscitationQ

PSEUDO-OBSTRUCTION 102. Ans. c. Splenic flexure (Ref: Sabiston 20/e p1336; Bailey 27/e p1297) • Ogilvie’s syndrome: Distended colon, with the right and transverse segments tending to be most dramatically affected. Transition point is frequently present, usually at or near the splenic flexure. 103. Ans. a. It is caused by mechanical obstruction of the colon 104. Ans. a. Water soluble contrast enema 105. Ans. d. Hyperthyroidism 106. Ans. a. Acute colonic pseudo-obstruction (Ref: Sabiston 20/e p1337; Schwartz 10/e p1221; Bailey 27/e p1297) “Colonic pseudo-obstruction: This may occur in an acute or a chronic form. The former, also known as Ogilvie’s syndrome, presents as acute large bowel obstruction.”-Bailey 27/e p1297 107. Ans. b. Electrolyte abnormalities (Ref: Sleisenger 10/e p1877; Sabiston 20/e p1337; Schwartz 10/e p1221; Bailey 27/e p1297) 108. Ans. c. Intestinal pseudo-obstruction 109. Ans. c. Ogilvie’s syndrome

Section 3

VOLVULUS 110. Ans. c. Rigid sigmoidoscopy is the initial treatment (Ref: Sabiston 20/e p1335; Schwartz 10/e p1219-1220; Bailey 27/e p1295; Shackelford 8/e p1809, 7/e p1850-1853) 111. Ans. a. Sigmoid colon 112. Ans. c. Ogilvie’s syndrome refers to cecal volvulus (Ref: Sabiston 20/e p1335; Schwartz 10/e p1219; Bailey 27/e p1297; Shackelford 8/e p1684, 7/e p1853-1854) 113. Ans. d. Cecopexy can be a form of treatment 114. Ans. c. Cecal volvulus (Ref: Sabiston 20/e p1335; Schwartz 10/e p1220; Bailey 27/e p1289) • Cecal volvulus: Distended cecum assumes a gas-filled comma shape or kidney bean shapeQ, the concavity of which faces inferiorly and to the right. 115. Ans. b. Right hemicolectomy is the treatment of choice 116. Ans. d. Midgut volvulus

Intestinal Obstruction

407

117. Ans. c. Sigmoid volvulus 118. Ans. b. Common in psychiatric patients, c. Bird’s beak sign, d. May present as intestinal obstruction 120. Ans. c. Lower GI scopy is contraindicated in sigmoid d. Volvulus of cecum is managed by conservative methods 121. Ans. a. Volvulus 122. Ans. d. Sigmoid volvulus (Ref: Sabiston 20/e p1335; Schwartz 10/e p1219; Bailey 27/e p1284) • Sigmoid volvulus: Markedly dilated sigmoid colon with the appearance of a bent inner tube or coffee bean appearanceQ. 123. Ans. b. Colectomy

124. Ans. d. Loaded pelvic colon

125. Ans. d. Either clockwise or anticlockwise

126. Ans. c. Both clockwise and anti-clockwise

Section 3

119. Ans. a. Mostly resolve with colonoscopic reduction

• Sigmoid volvulus is more commonly anticlockwise (can be both clockwise or anticlockwise)Q • Cecal volvulus and small intestine volvulus are mostly clockwiseQ 127. Ans. b. Non-operative treatment has no role

128. Ans. d. Tuberculosis (Ref: Schwartz 10/e p1221, 9/e p1055)

• Hirschsprung’s disease and Chagas disease can lead to megacolon, a risk factor for sigmoid volvulus. 129. Ans. b. Large bowel volvulus • Small bowel volvulus is most common form of volvulus in neonates. Colonic volvulus is very rare. 130. Ans. a. 1st week (Ref: Bailey 25/e p85) “Midgut volvulus can occur at any age, although it is seen most often in the first few weeks of life.”

MALROTATION 131. Ans. b. Malrotation (Ref: Sabiston 20/e p1871; Schwartz 10/e p1616-1617; Shackelford 8/e p970, 7/e p1046-1048; Bailey 27/e p134-135) 132. Ans. d. Mixed rotation (Ref: Sabiston 20/e p1871; Schwartz 10/e p1616) Mixed rotation of gut: • The intestine doesn’t rotate as it re-enters the abdomen after physiological hernia. • Caecum lies just inferior to the pylorus of the stomach. It may result in volvulus (twisting) of intestine, which leads to obstruction further

MISCELLANEOUS 134. Ans. a. Rectouterine pouch

135. Ans. a. Porphyria (Ref: Harrison 19/e p1982)

Spastic Ileus • Spastic ileus or dynamic ileus is rare, results from extreme and prolonged contraction of the intestine.



• Heavy metal poisoningQ • UremiaQ

Spastic ileus is seen in • PorphyriaQ • Extensive intestinal ulcerationQ

Gastrointestinal Surgery

133. Ans. b. Bleeding PR, c. Bilious vomiting

408

Surgery Essence 136. Ans. a. It is poorly localized (Ref: Guyton 13/e p627; Ganong 25/e p167)

Characteristic Features Visceral Pain • Visceral pain is poorly localized, often the pain is referred or radiatingQ

Section 3

Gastrointestinal Surgery

• Highly localized types of damage to the viscera seldom cause severe pain • Often accompanied by vomiting and hypotensionQ • Pain of hollow viscus is often felt as a colicQ (it comes and goes to reappear again). • There are no proprioceptors in viscera and few temperature and touch sense organs. Pain receptors are present, although they are more sparsely distributed than in somatic structures. • Afferent fibers from visceral structures reach the CNS via sympathetic and parasympathetic pathways. • Their cell bodies are located in the dorsal roots and the homologous cranial nerve ganglia. • Specifically, there are visceral afferents in the facial, glossopharyngeal, and vagus nerves; in the thoracic and upper lumbar dorsal roots; and in the sacral roots. • Essentially all visceral pain that originates in the thoracic and abdominal cavities is transmitted through small type C pain fibers and, therefore, can transmit only the chronic-aching-suffering type of pain. 137. Ans. d. Ischemic bowel disease (Ref: Sabiston 20/e p1155; Schwartz 10/e p1167; Bailey 27/e p1253; Schackelford 8/e p1023, 7/e p1077-1082) • In ischemic bowel disease or mesenteric ischemia, vascularity of adjoining mesentery is decreased. • History of abdominal pain and altered bowel habits with thickened cecum and ileocecal junction and increased vascularity of surrounding mesentery can be observed in Ulcerative colitis, Crohn’s disease and tuberculosis. Mesenteric Ischemia • Findings on CT scans associated with bowel ischemia include dilation of the bowel lumen, bowel wall thickening, abnormal bowel wall enhancement, arterial occlusion, venous thrombosis, and intramural or portal venous gasQ. • Dilation of an ischemic bowel segment suggests interruption of normal peristaltic activityQ. • Symmetrical bowel wall thickening greater than 3 mm in a distended segmentQ of bowel suggests ischemia. • Greater degrees of bowel wall thickening should raise suspicion of mesenteric venous thrombosis (MVT). • Intravenous contrast is useful in demonstrating the heterogeneity of the ischemic bowel wall (lack of bowel wall enhancement) and may show occlusion of mesenteric arteries if given by rapid bolus administrationQ.

CHAPTER

13

Small Intestine

MECKEL’S DIVERTICULUM Meckel’s Diverticulum • Most commonly encountered congenital anomaly of the small intestineQ • Occur 2% of the populationQ. • 2% prevalenceQ • 2 inch in lengthQ • • • •

Rule of two in Meckel’s diverticulum • Half of these who are symptomatic are Pancreatic mucosa (5%) >colonic mucosa (rarely)

Clinical Features • Most are entirely benign & incidentally discovered during autopsy, laparotomy, or barium studies • MC clinical presentation is GI bleeding (25-50%)Q • Hemorrhage: MC symptomatic presentation in children ≤2 yearsQ • Hemorrhage is manifested as painless bright red blood from the rectum, with intermittent episodesQ persisting without treatment. • Source of the bleeding is a chronic acid-induced ulcer in the ileum adjacent to a Meckel’s diverticulum that contains gastric mucosa. • Intestinal obstruction (31%): Due to volvulus, intussusception, or, rarely, incarceration of the diverticulum in an inguinal hernia (Littre’s hernia)Q. Complications of Meckel’s Diverticulum • MC complication in children & young adults: BleedingQ

•  MC complication in adults: Intestinal obstructionQ

• Diverticulitis (10-20%) is more common in adult patients. • Progression of the diverticulitis may lead to perforation and peritonitis. • When the appendix is found to be normal during exploration for suspected appendicitis, the distal ileum should be inspected for the presence of an inflamed Meckel’s diverticulumQ. Diagnosis • Most accurate diagnostic test in children: Scintigraphy with sodium 99mTc-pertechnetateQ. • 99mTc-pertechnetate is preferentially taken up by the mucus-secreting cells of gastric mucosa & ectopic gastric tissue in the diverticulumQ. (Sensitivity-85%, specificity-95% and an accuracy-90% in the pediatric age group) • Less accurate in adults because of the reduced prevalence of ectopic gastric mucosaQ • Sensitivity & specificity can be improved by pentagastrin and glucagon or H2-receptor antagonists (cimetidine)Q. • In adults with normal nuclear medicine findings, barium studies should be performed. Treatment • Symptomatic Meckel’s diverticulum: Diverticulectomy or resection of the segment of ileumQ bearing the diverticulum. • Segmental intestinal resection is required for bleeding because the bleeding site usually is in the ileum adjacent to the diverticulumQ. • Asymptomatic diverticula found in children during laparotomy should be resectedQ. • Incidentally found Meckel’s diverticulum should be removed at any age up to 80 years as long as no additional conditions (e.g., peritonitis) made removal hazardousQ.

410

Surgery Essence INTESTINAL ATRESIA Intestinal Atresia • MC site of intestinal atresia: DuodenumQ • MC cause of neonatal intestinal obstruction: Duodenal atresiaQ

Gastrointestinal Surgery

Jejunoileal Atresia • • • • •

Atresia & stenosis are among the MC causes of neonatal intestinal obstructionQ. Incidence of jejunoileal atresia is 1 in 300 to 1500Q live births. Gender ratio is equal. Jejunal atresia is slightly more common than ileal atresia. In 80-90% of cases the atresia is isolated. However, in up to 20% of cases atresias are multiple. • Cystic fibrosis is an important comorbid condition with reported incidence is 10-20%. White infants with jejunoileal atresia have more than 210 times the risk for cystic fibrosisQ.

Clinical Features • • • •

Infants with atresia or stenosis usually have bilious vomiting on the first day of life. The higher the obstruction, the earlier the vomiting. Abdominal distention is more pronounced with distal obstruction. More than 60% of these infants fail to pass meconium in the first day of life, may have grayish mucoid contents in the rectal vaultQ.

Diagnosis • Can be diagnosed by prenatal ultrasonographyQ. • Associated with maternal polyhydramniosQ • Abdominal radiographs show gas- and fluid-filled bowel loops with absence of gas distallyQ. Treatment • Management includes intravenous fluid, decompression of the stomach, withholding of enteral feeding, and antibiotics. • After resuscitation the infant is taken to the operating room for exploratory laparotomy. • The goals of the operation are to restore intestinal continuity after resection of the atretic segment while preserving intestinal length. Prognosis • The prognosis in these patients is excellent, with survival rates of 90%Q. • Risk of adhesive bowel obstruction & necrotizing enterocolitis (NEC)Q

SMALL INTESTINAL DUPLICATION Small Intestinal Duplication • • • •

Intestinal duplications are mucosa-lined structures that are in continuity with the GI tract. MC site of duplication: Ileum (within the leaves of mesentery)Q Located along mesenteric border, contain all the three layers of bowelQ Share a common wall & blood supply with adjacent bowelQ Types of Small Intestinal Duplication

Section 3

Cystic (75%)Q

• More common • Do not have communication with the lumenQ of normal small intestine • Manifested as partial small bowel obstructionQ

Tubular • Less common (25%) • Parallel to the normal bowel lumenQ • Higher incidence of communicationQ with the existing lumen of small intestine • Significant incidence of ectopic gastric mucosaQ. • BleedingQ is a common manifestation

Clinical Features • Recurrent abdominal pain, emesis from intestinal obstruction, & hematocheziaQ. • Bleeding from ulceration in the duplication or in the adjacent intestine if the duplication contains ectopic gastric mucosaQ.

Small Intestine

411

Diagnosis

Treatment • Short duplications: Resection of the cyst and adjacent intestine with end-to-end anastomosisQ • Long duplications: −− Multiple enterotomies and mucosal strippingQ in the duplicated segment, which will allow the walls to collapse and become adherent. −− Divide the common wall using a linear cutting staplerQ to form a common lumen. • Excellent prognosis in duplications who undergo complete excisionQ without compromise of the length of remaining intestine

Section 3

• CT, ultrasonography, and technetium pertechnetate scanning can be very helpful. • USG: Classical triple layer effectQ

SMALL INTESTINAL DIVERTICULA Duodenal Diverticula • Relatively common, representing the second MC site for diverticulum formation after the colon. • More common in women (rare in patients ileocecal region > colon (Can occur anywhere) • Equal incidence among males and femalesQ •  Most commonly occurs in the 4th-7th decadesQ of life. • Pneumatosis in neonates is usually associated with necrotizing enterocolitisQ. Pathology • On histologic section, honeycomb appearanceQ. • Cysts are thin walled & break easily.

•  Spontaneous rupture gives rise to pneumoperitoneum.

Predisposing Factors • COPDQ •  Inflammatory, obstructive, or infectious conditions of the intestineQ • Immunocompromised stateQ. •  Endoscopy and jejunostomy placement; ischemiaQ • DiabetesQ Clinical Features • Symptoms are nonspecific, most commonly diarrhea, abdominal pain, abdominal distention, nausea, vomiting, weight loss, and mucus in stoolsQ. • Pneumoperitoneum occurs usually in association with small bowel rather than large bowel pneumatosisQ. • Pneumatosis intestinalis represents one of the few cases of sterile pneumoperitoneum and should be considered in the patient with free abdominal air but no evidence of peritonitisQ. Diagnosis • On plain films, radiolucent areas within the bowel wall (appear as grapelike clusters or tiny bubbles)Q • Alternatively, barium contrast or CT studies can be used to confirm the diagnosisQ. Treatment • Management of the uncomplicated primary disease is conservative. • When symptoms demand treatment, the first line is intermittent high flow oxygen therapyQ, providing a concentration of 70% continuously for 5 days by nasal specula. • Cysts may resolve with antibiotics, particularly metronidazoleQ. • In resistant cases, maintenance treatment with sulfasalazineQ may be helpful.

RADIATION ENTERITIS Radiation Enteritis • Radiation therapy affect rapidly dividing cells in small intestine • Rapidly dividing cells in small intestinal epithelium may sustain severe, acute, and chronic deleterious effects. • Serious late complications are unusual if the total radiation dosage is 5000 cGyQ.

Section 3

Pathology • Radiation damage tends to be acute and self-limitingQ • Late effects of radiation injury: −− Damage to small submucosal blood vesselsQ −− Progressive obliterative arteritis & submucosal fibrosisQ −− Thrombosis & vascular insufficiencyQ Predisposing Factors • Previous abdominal operationsQ • Preexisting vascular disease, hypertension, diabetesQ • Adjuvant treatment: 5-FU, doxorubicin, dactinomycin and methotrexateQ Clinical Features • Radiation damage tends to be acute & self-limiting, with symptoms consisting mainly of diarrhea, abdominal pain, & malabsorptionQ.

Small Intestine

Prevention • By adjusting ports & dosages of radiation to deliver optimal treatment specifically to the tumor and not to surrounding tissues. • Exclude the small bowel from the irradiated field by reperitonealization, omental transposition, & placement of absorbable mesh slings. • Sucralfate is used to prevent diarrhea associated with abdominal radiation. • Superoxide dismutase reduces complications. • Most effective radioprotectant: AmifostineQ (WR-2721)

Section 3

• Late effects of radiation injury resulting eventually in thrombosis & vascular insufficiency leading to necrosis & perforation of the involved intestine, but more commonly stricture formation with symptoms of obstruction or small bowel fistulasQ.

413

Treatment • Acute radiation enteritis: Control symptoms with antispasmodics, analgesics antidiarrheal agentsQ. • Operative intervention: Obstruction (MC), fistula formation, perforation & bleedingQ • Operative procedures: Bypass or resection with reanastomosis.

GASTROINTESTINAL TUBERCULOSIS Gastrointestinal Tuberculosis • Mycobacterium tuberculosisQ is responsible for all the cases of GI tuberculosis • M. bovis has largely been eliminated by public health measures • More common in poor socioeconomic statusQ Pathogenesis Primary Intestinal Tuberculosis

• Arises from swallowed sputumQ containing tuberculous bacilli • Influenced by virulence & quantity of bacilli and host resistance of infectionQ

• When the intestines become infected by lymphatic spread from the mesenteric nodes, nodal disease is considered as the primary site & intestinal involvement is secondary. • Earliest intestinal lesions are found in submucosaQ, while the overlying mucosa is normal. Sites of Intestinal Involvement • MC site is terminal ileum & ileocecal junctionQ • Other regions in decreasing frequency are: colon, jejunum, rectum, anal canal, duodenum, stomach & esophagus • Site of predilection is dictated by the factors: abundance of lymphoid tissue, rate of absorption of intestinal contents, prolonged stasis & digestive activity of intestinal contentsQ Pathology Ulcerative Tuberculosis (60%) • Tuberculous intestinal ulcers are usually deep & transversely placedQ in the direction of lymphatics • Multiple ulcers may be seen, most often in terminal ileumQ • Disease progression is associated with the appearance of inflammatory mass around the bowel. • Diseased part of the GIT becomes thickened and serosal surface is studded with tubercles. • Marked increase in mesenteric fat with fat wrapping around the bowel loops • Regional lymph nodes become enlarged and may caseate, leading to mesenteric abscess formation • Bowel perforation is rare and is usually confined by perilesional inflammatory mass

Hyperplastic Tuberculosis (10%)

Sclerotic or Fibrotic Tuberculosis (30%)

• A fibroblastic reaction occurs in submucosa and subserosa resulting in marked thickening of the bowel wallQ • Involvement of adjacent mesentery, lymph nodes and omentum, results in formation of a mass lesion • Hyperplastic lesions are due to reduced bacterial virulence & increased host resistance

• Associated with strictures of intestine, typically described as “napkin-ring strictures” which may be single or multiple • When multiple, strictures may occur in short segment of bowel or over the entire length of intestine.

Gastrointestinal Surgery

• Ingestion of contaminated foodQ may cause primary tuberculosis (this route of infection has decreased in recent years)

Secondary Intestinal Tuberculosis

414

Surgery Essence Clinical Features

Gastrointestinal Surgery

• • • • •

Initial symptoms are vague & non-specificQ As the diseases progress, individual may develop fever (in two third), night sweats, malaise, weakness, anorexia & weight lossQ. MC symptom of GI tuberculosis is abdominal painQ. Diarrhea is another common symptom. Abdominal distention suggests presence of ascites or subacute intestinal obstruction Primary Small Bowel Disease

Colonic Tuberculosis

• Stools are large in amount, foul smelling and resemble those seen in patients with malabsorptionQ.

• Stools may be watery, small in amount and mixed with blood when disease affects predominantly the colonQ.

Complications • Intestinal obstruction & malabsorption are MC complicationsQ • Bowel perforation & GI hemorrhage are less common Diagnosis • Laboratory tests: MC abnormality is raised ESR (90% cases)Q Ascitic Fluid Showing • LymphocytosisQ (WBC >500/mm3) • High proteinQ content (>2.5 gm/dL)

• SAAG 4 cm and ≤7.0 cm and confined to the kidneyQ

N0: No regional lymph nodes metastasis N1: Metastasis in regional lymph node

T2a: Tumor >7 cm but ≤10 cm and confined to the kidneyQ T2b: Tumor >10 cm and confined to the kidneyQ T3a: Tumor grossly extends into renal veins or its segmental (muscle M: Distant metastases containing) branches, or tumor invades perirenal and/or renal sinus fat M0: No distant metastasis but not beyond Gerota’s fasciaQ M1: Distant metastasis present T3b: Tumor grossly extents into vena cava below diaphragmQ

T4: Tumor invades beyond Gerota’s fasciaQ (including contiguous extension into ipsilateral adrenal gland) Stage I T1N0M0

Stage II T2N0M0

Stage III T1-3 N1 M0 T3 N0 M0

Stage IV T4 anyN M0 AnyT anyN M1

Pediatric Tumors • MC malignant tumor of infancy • MC extracranial solid tumor in children • MC abdominal malignancy in children

• NeuroblastomaQ

• MC primary malignant renal tumor of childhood

• Wilms’ tumorQ

• MC renal tumor of infancy

• Congenital mesoblastic nephromaQ

• MC soft tissue tumor in infants & children

• RhabdomyosarcomaQ

• MC solid tumor of childhood

• Brain tumorQ

• MC cancer of childhood

• LeukemiaQ (30%) >Brain tumorsQ (22%)

Urology

T3c: Tumor extends into the vena cava above the diaphragm or invades the wall of vena cavaQ

580

Surgery Essence WILMS’ TUMOR Wilms’ Tumor • • • • •

Wilms’ tumor: MC primary renal tumor of childhood (2-5 years)Q. Wilms’ tumor: 2nd MC malignant abdominal tumor in children (MC is neuroblastoma). Arise from kidney, composed of three elements- Blastema, Epithelium & StromaQ. (BESt) MC presenting feature is asymptomatic abdominal mass or swellingQ. Mostly unilateral. • Characterized by triad of abdominal mass, fever & microscopic hematuriaQ.

• Fever typically resolve after tumor resection

Urology

Associated Malformations • WAGR SyndromeQ: It consists of aniridia, genital anomalies & mental retardation. The risk of Wilms’ tumor is increased by 33% in this syndromeQ. Associated with WT-1 gene deletion located on chromoseQ 11p 13 • Denys-Drash SyndromeQ: It consists of gonadal dysgenesis (Male pseudohermaphroditism), nephropathy leading to renal failure. Majority of patients with this syndrome have renal  failure. • Beckwith-Wiedmann SyndromeQ: It consists of enlargement of body organs, hemi-hypertrophy, renal medullary cysts and abnormal large cells in adrenal cortex, macroglossia, omphalocele, hepatoblastoma. Associated with WT-2 gene deletion located on chromosome 11p 15.5. Diagnosis • USG (first investigation)Q or CT abdomen for staging. • MRI is superior to other imaging modalities in delineating nephroblastomatosis elements. • Calcification tends to be more crescent shaped, discrete & peripheralQ in comparison of finely stippled calcification of neuroblastoma. Treatment • Surgical excision (transperitoneal radical nephrectomy) is treatment of choice. • Routine exploration of contralateral kidney is not necessary if imaging is satisfactory and doesn’t suggest bilateral process. • In unfavorable histology, Radiation therapy should be started within 10 daysQ after nephrectomy, Chemotherapy should be started 5 days after surgeryQ. • Chemotherapy: VCD (Vincristine + Cyclophosphamide + Doxorubicin or dactinomycin) • Whole lung irradiation is recommended for pulmonary metastasis. Preoperative Treatment should be Considered • Solitary kidneyQ • BilateralQ renal tumors • Tumor in horse shoe kidneyQ

• Tumor thrombus in IVC above the level of hepatic veinsQ • Respiratory distress due to metastaticQ disease

Prognosis • HistologyQ of Wilms’ tumor & tumor stage is identified as most important determinant of prognosisQ (Histology > Stage). The postchemotherapy based staging system is the ‘SIOP’ staging system developed by the International society of oncology. Two Staging Systems are currently being used for the staging of Wilm’s Tumor.

Section 4

Prechemotherapy Staging System • Developed by the National Wilms’ Tumor staging Group (NWTSG - Staging system) • This staging system is widely used in North America and Canada • ‘NWTSG’ approach involves employment of ‘primary surgery’. • Chemotherapy with or without Radiation therapy is given after surgery • Staging is done at time of surgery (Prechemotherapy)

Postchemotherapy Staging System • Developed by the International Society of Pediatric Oncology (SIOP - Staging system) • This staging system is widely used in Europe • ‘SIOP’ approach involves employment of preoperative chemotherapy without histological confirmation of Wilms’ tumor. • Primary chemotherapy for all patients regardless of extent • Staging is done at time of surgery (Postchemotherapy)

Kidney and Ureter

581

CARCINOMA RENAL PELVIS

• Transitional cell carcinoma accounts for 90% of upper urinary tract cancersQ. • Urothelial cancer often presents as a widespread urothelial abnormality: Patients with a single upper-tract carcinoma are at risk for developing bladder carcinoma (30-50%)Q and contralateral upper urinary tract carcinoma (2-4%). • More common in males Etiology • SmokingQ •  Industrial dyes or solventsQ • Excessive analgesic (Phenacetin) intakeQ •  Balkan’s nephropathyQ

Section 4

Carcinoma Renal Pelvis

Clinical Features • Painless gross hematuria (MC)Q, flank pain, irritative voiding symptoms Diagnosis • Ureteroscopic brush cytology for malignant cellsQ • IVP showing radioluscent intraluminal filling defectsQ • CT urography is IOC for evaluation of upper urinary tractQ • Ureteral tumors are often characterized by ureteral dilation below the site of the lesion, creating the appearance of a “goblet” (better appreciated on RGP)Q • Bergman signQ: A ureteral catheter passed upto ureter may coil distal to the ureter Treatment • Nephroureterectomy with a cuff of bladderQ. • Periodic follow up with cystourethroscopy as risk for developing bladder carcinoma (30-50%) and contralateral upper urinary tract carcinoma (2–4%)Q.

RENAL INJURIES Renal Injuries • • • • • •

Kidney is the most commonly injured part of urinary tractQ MC cause of blunt renal injury is motor vehicle accidentQ. Hematuria is the best indicatorQ of traumatic urinary system injury. More than 80% of patients sustaining penetrating renal injuries have other intra-abdominal injuriesQ. Blunt renal injuries are generally divided into minor and major injuries. Minor injuries account for approximately 85%Q of cases.

• Injuries involving the hilum are seldom repaired primarilyQ, and in most circumstances total nephrectomy is necessary. Imaging Studies Contrast enhanced CT is the IOC for Renal InjuriesQ Indications • Gross hematuriaQ • Microscopic hematuria with hypotensionQ anytime during initial resuscitation • IVP should be done to see the function of the opposite kidneyQ • Arteriography is used to define arterial injuries suspected on CT or to localize arterial bleeding that can be controlled by embolizationQ. Grade I II

Type

Description

Contusion

Microscopic (>3 RBCs/HPF)Q or gross hematuria, urological studies normal

Hematoma

Subcapsular, nonexpanding without parenchymal laceration.

Hematoma

Nonexpanding perirenal hematoma, confined to renal retroperitoneum

Laceration

1 cm parenchymal depth of renal cortex without collection system rupture or urinary extravasationQ

Urology

Penetrating wounds causing small parenchymal injuries are generally treated by débridement, primary repair, and drainageQ.

582

Surgery Essence Grade IV V

Type

Description

Laceration

Parenchymal laceration extending through collecting systemQ

Vascular

Main renal artery or vein injury with contained hemorrhage

Laceration

Completely “Shattered kidney”Q

Vascular

Avulsion of renal hilum, devascularising the kidney.

   Note: Advance one grade for bilateral injuries upto grade III.

Management

Urology

Nonoperative • Most (>95%) of renal injuries can be managed non-operativelyQ. • Significant renal injuries (Grade II–V) are found only in 5% of renal trauma. • A hemodynamically stable patient with an injury well staged by a CT scan can usually be managed without renal explorationQ. Hospital admission, bed rest, vital monitoring and repeated CT scan is required. Renal Exploration in Injuries • Renal exploration should be done by transabdominal approach in order to have a control on the renal vessels firstQ. Absolute Indications bleedingQ

Relative Indications

• Persistent renal • Expanding or pulsatile perirenal hematomaQ

• Urinary extravasation • Non viable tissue (>20% necrosis) • Segmental arterial injury

Indications of Nephrectomy • Hemodynamically unstable patient, with low body temperature and poor coagulation, with a normal contralateral kidneyQ. • Extensive renal injuriesQ when the patient’s life would be threatened by an attempt at renal repair. • Already poorly functioning hydronephrotic kidneyQ with continuous bleeding

COMPLICATIONS AFTER RENAL TRAUMA Complications after Renal Trauma 3–10%Q

• Complication rate after renal trauma is • Urinoma is the MC complicationQ after renal trauma • Delayed bleeding usually occurs within 1–2 weeksQ after injury Early Complications • Urinoma, delayed bleeding • Urinary fistula, abscess • Hypertension

Late Complications • Hydronephrosis, pyonephrosis • Stone formation, AV fistula • Delayed hypertension

RUPTURE OF THE URETER Rupture of the Ureter

Section 4

Ureteric Injury during Operation • MC cause of injury to the ureters is surgical trauma during hysterectomy or other pelvic surgeryQ • Preoperative catheterization of ureters makes them easier to protect during surgeryQ • Injuries discovered at the time of surgery should be repaired immediatelyQ Clinical Features • Unilateral Injuries • No symptomsQ: Secure ligation of a ureter may simply lead to silent atrophy of kidney. • Loin pain and feverQ: Possibly with pyonephrosis, occur with infection of the obstructed system. Urography shows no function, which will be permanent unless steps are taken quickly to relieve the obstruction by inserting a percutaneous nephrostomy. • A urinary fistulaQ develops through the abdominal or vaginal wound. NephrostomiesQ may be inserted and repair postponed until edema and inflammation have subsided.

Kidney and Ureter

583

Diagnosis

• Early repair is safe provided that the patient is fit for surgery Injury Recognized at the time of Operation • Ureterovesical continuity should be restored unless the patient’s condition is poorQ. • Deliberate ligation of the proximal ureter and temporary percutaneous nephrostomy is then the best course until the patient is well enough for a repairQ.

Section 4

• The diagnosis is rarely made until there is swelling in the loin or iliac fossa associated with a reduction of urine output. • Excretion urogram or CECT: Extravasation of contrast from the injured ureterQ. Treatment

Methods for Repairing a Damaged Ureter • Spatulation and end-to-end anastomosis without tensionQ

• No loss of length • Little loss of length

• Marked loss of length

• • • • • •

Mobilise kidney Psoas hitchQ of bladder Boari operationQ TransureteroureterostomyQ Interposition of isolated bowel loop or mobilised appendixQ NephrectomyQ

Bilateral Ureteral Injury • Ligation of both ureters leads to anuriaQ. • Ureteric catheters will not pass and urgent relief of obstruction by nephrostomy or immediate surgeryQ is essential. Boari Operation • A strip of bladder wall is fashioned into a tube to bridge the gapQ between the cut ureter and the bladderQ.

ADULT POLYCYSTIC KIDNEY DISEASE (AD) Adult Polycystic Kidney Disease (AD) • • • •

Inheritance is autosomal dominantQ with 100% gene penetrance, 50% offsprings are affected. Chromosome affected: 16 & 4Q; Protein abnormality: PolycystinQ Usually bilateralQ An important cause of renal failure, accounting for 10-15% of patients who receive hemodialysis.

Pathology Kidneys are grossly enlargedQ with multiple cysts Cyst are distributed uniformlyQ throughout cortex & medulla Cysts contain straw colored fluid that may become hemorrhagic Renal arteriolar thickening is a prominent finding in adults

Presentation • • • • • •

Usually occurs in 3rd or 4th decade MC clinical feature is hypertension (75% adults & 25% children)Q due to activation of rennin angiotensin systemQ. Pain due to infection (pyelonephritis)/obstruction/sudden hemorrhage. HematuriaQ, nocturia (due to impaired concentrating ability), nephrolithiasis (15–20%)Q Progressive decline in renal function leading CRF MC cause of death: Cardiovascular disordersQ ADPKD Extra-renal manifestations • Cysts: Liver (MC)Q, spleen, pancreas and ovaries • Berry aneurysms (10–40%)Q • Cyst in seminal vesicles (40%), Arachnoid membrane (8%)

• Colonic diverticulosisQ • Mitral valve prolapseQ

Diagnosis • USG: Enlarged kidney with uniformly increased medullary echogenecity

Urology

• • • •

584

Surgery Essence IVP in ADPKD • Stretching of the calyces by the cysts (spider leg or bell like deformity)Q

• Bubble appearanceQ (calyceal distortion) • Swiss cheese appearanceQ

• CT scan is IOCQ in ADPKD Management • Treatment is mainly aimed to control UTI, hypertension, calculi & general measures for uremia (low protein diet) • Pain relief by percutaneous aspiration with instillation of sclerosing agent or Rovsing’s operation (deroofing of the cyst)Q • Dialysis or renal transplantation (only definitive treatment) for renal failureQ.

INFANTILE POLYCYSTIC KIDNEY DISEASE (AR)

Urology

Infantile Polycystic Kidney Disease (AR) • • • •

Rare, autosomal recessiveQ, usually bilateral Always become apparent during childhood, rarely upto 20 years, most severe forms are seen earliest in lifeQ. If diagnosed at birth, child dies in 2 months due to uremia and pulmonary hypoplasiaQ Associated with hepatic fibrosis, leading to portal hypertension and hepatic failure; pulmonary fibrosisQ

Diagnosis • IVP: Delayed function with characteristic radial or medullary streaking (sunburst pattern)Q Treatment • No cure, only palliative supportQ (respiratory care, surgery for esophageal varices, hemodialysis and renal transplantation)

PELVIURETERIC JUNCTION (PUJ) OBSTRUCTION Pelviureteric Junction (PUJ) Obstruction • A blockage of the ureter at the junction with the renal pelvis resulting in restriction of urine flow • MC cause of fetal hydronephrosisQ • More common in boysQ, mainly left sided, bilateral in 10–15% cases Causes of PUJ Obstruction Congenital

Acquired

segmentQ

• Aperistaltic due to disorganization of smooth muscle or collagen deposition • Crossing aberrant renal vesselQ

• CalculusQ • InstrumentationQ • InfectionQ

Associated Abnormalities • PUJ Obstruction of opposite kidney (MC)Q in 40% • VURQ • VATER defectsQ (Vertebral anomalies, anorectal malformations, TE fistula, Radial and renal dysplasia) Clinical Presentation • Most infants are asymptomaticQ • Most infants are discovered by palpable abdominal mass or prenatal USGQ.

Section 4

Diagnosis • USG: Diagnoses hydronephrosis, but does not diagnose whether it is obstructive. • IVP: It was the primary radiological study to define PUJO but now replaced by DTPA scanQ. DTPA Scan • Investigation of choice for PUJO to establish that hydronephrosis is due to obstruction. • Pressure flow studies (Whitaker test)Q −− Invasive test, used only in cases of equivocal result of renal scanQ. −− Measures differential pressure between kidney and bladder −− A percutaneous puncture of kidney is made; contrast fluid is infused at a constant rate with monitoring of intra-pelvic pressure. An abnormal rise confirms obstruction. • Retrograde Pyelogram: Anatomic delineation more clear. Show the distal end of obstructionQ.

Kidney and Ureter

585

Treatment

Types of Pyeloplasty Open Endoscopic A. Anderson Hynes (Dismembered) pyeloplasty: • For small pelvisQ with PUJ dependent for good funnel drainage and intrarenal pelvisQ. • Gold standard for PUJ obstructionQ Q • More than 2 cm long area of stricture is a • For large redundant pelvis with high insertion of ureter . contraindicationQ. • Advantages: Broad applicability, including preservation of anomalous Q • Endoscopic procedures are: vessels and excision of the pathological UPJ and appropriate repositioningQ successful reduction pyeloplasty −− a. Endopyelotomy (percutaneous or retrograde) advised for associated renal B. Foley V-Y pyeloplasty: Best applied for high insertion of the ureter stone C. Flaps (Spiral and vertical) −− b. Balloon dilatations or Lasers

Section 4

• Conservative in children with good renal function without any complication • Pyeloplasty or nephrectomy

Indications for Nephrectomy in PUJO • Permanent severe loss of renal function (< 10% function)Q • Unmanageable complication in hydronephrotic kidney (Severe recalcitrant infection, Intractable pain, Hypertension)Q

Anderson-Hynes dismembered pyeloplasty

RENAL AGENESIS Renal Agenesis 1000Q,



Potter’s facies • Hypertelorism, prominent inner canthal foldQ • Blunted nose, recessive chin, broad and low set ears, limb deformitiesQ • Seen in bilateral renal agenesisQ

MEDULLARY SPONGE KIDNEY (AR) Medullary Sponge Kidney (AR) • Dilatation of distal portion of the collecting duct with numerous associated cysts giving kidneys an appearance of sponge • Autosomal recessive defect, usually bilateralQ, affecting all of the papillae, but it may be unilateral • Infection & calculi are occasionally seen as a result of urinary stasis in the tubules.

Urology

Incidence of renal agenesis is 1 in more common in malesQ and on left sideQ in unilateral renal agenesis. Ureter is absent on the side of the unformed kidney in 50%Q cases, in rest blind ureter is found. Bilateral is incompatible with life and rare. Associated anomalies: OligohydramniosQ, pulmonary hypoplasiaQ (due to defective proline synthesis), amnion nodosumQ, Potter’s faciesQ (seen in bilateral RA). • Unilateral renal agenesis is associated with unicornuate or bicornuate uterus and septate vagina in females. • Colonic shadow is placed more laterallyQ on X-ray KUB due to unilateral renal agenesis. • • • •

unilateralQ

586

Surgery Essence Associated with • HemihypertrophyQ of the body • HypercalcemiaQ • Stone formationQ (Calcium oxalate or calcium phosphate) • NephrocalcinosisQ (Calcium deposition in renal parenchyma) Clinical Features • Symptoms are due to infection & stone formationQ. • MC symptom: Renal colic > UTI > gross hematuria. Diagnosis • Made on the basis of excretory urogram or contrast-enhanced CT scan.

Urology

IVP (Excretory urogram): IOC to diagnose medullary sponge kidneyQ • “Bristles on brush”Q appearance due to dilated ducts • “Bouquet of flowers”Q appearance due to calcification in the ectatic ducts Treatment • There is no treatment for medullary sponge kidney. • Therapy is directed toward the complications (e.g. pyelonephritis and renal calculi).

RETROCAVAL URETER Retrocaval Ureter (Circumcaval Ureter) • An embryologically normal ureter becomes entrapped behind IVCQ • Because of abnormal persistence of right posterior subcardinalQ (as opposed to the supracardinal) vein. This forces the right ureter to encircle the vena cava from behind (Altered development of IVC)Q • Right ureter typically deviates medially behind the IVCQ, winding about and crossing in front of it from medial to lateral direction, to resume a normal course to the bladder. • More common in malesQ Clinical Features • Signs & Symptoms of ureteric obstructionQ Diagnosis • MRI is IOC to delineate anatomy clearly and non-invasivelyQ • IVP: “Reverse J”, “Fish Hook” or “Shepherd crook”Q deformity. • Retrograde ureterography Surgical Management • Ureteral division with relocation ureteroureterostomy in cases of obstruction.

HORSESHOE KIDNEY Horseshoe Kidney

Section 4

• MC renal fusion abnormalityQ with incidence of 1:400Q, more common in malesQ • Fusion at the lower polesQ by a parenchymatous or fibrous isthmus Etiopathogenesis • Fusion occurs before kidneys have rotated at their long axes • The axes of these masses are verticalQ whereas axes of normal kidneys are oblique to spine, because they lie along the edges of the psoas muscle • Pelvis and ureters are usually anteriorly placedQ or anteromedial, crossing anteriorly to isthmus • Calyces point posteriorlyQ. Lowermost calyx extends caudally or even mediallyQ • Migration is incomplete. Inferior mesenteric artery (IMA) prevents full ascentQ • Isthmus usually located adjacent to L3-L4 vertebra, just below the origin of IMA from aorta • Ureteral compression can occurs due to anteriorly displaced ureterQ or from obstruction by aberrant vessels leading to hydronephrosis and infection. Associated Abnormalities (Present in one-thirdQ cases) • CVS (VSD) and CNS involvement • Anorectal malformationsQ • Unicornuate or bicornuate uterusQ

• Renal: VUR and PUJ obstruction in one thirdQ cases • Hypospadias, undescended testisQ

Kidney and Ureter

587

Clinical Features

Diagnosis is usually made on IVP showing • Low lying kidneysQ, closed to vertebral column • Vertical axesQ through kidneys point towards the LS spine • Characteristic orientation of the calyces, directed posterior to each renal pelvis, with the lowermost calyx pointing caudally or even medially (Hand joining sign)Q • High insertion of the ureter appears to drape over a midline mass (Flower vase-like curves of ureters)Q

Section 4

• Most are asymptomaticQ, mostly it is an autopsy finding • MC symptom: vague abdominal painQ • Rovsing syndrome: abdominal pain, nausea and vomitingQ on hyperextension of spine

• Angiography is done before surgery as blood supply is unpredictableQ but not needed usually as helical CT is useful. Complications • Prone to ureteral obstruction due to high incidence of aberrant renal vesselsQ and the necessity for ureters to arch over the renal tissues • Hydronephrosis, stone, infectionQ • Large fused kidney occupying the concavity of sacrum may cause dystociaQ Treatment • Pyeloplasty is done only in symptomatic cases, isthmus is not dividedQ.

Aortic Aneurysm with Horseshoe Kidney essentialQ

• Preoperative angiography is for the proper evaluation of the renal arteries, as there are multiple aberrant renal arteries arising from aorta • Isthmus rarely needs to be divided • Left retroperitoneal approachQ is preferred.

BENIGN RENAL CYST Benign Renal Cyst • • • •

MC benign renal lesionsQ, represent > 70% of all asymptomaticQ renal masses. More common in men, and can be solitary or multipleQ Prevalence increases with age, can be found in > 50% of patients > 50 yearsQ Treatment: Percutaneous drainage and sclerosisQ with 95% alcohol. Bosniak’s Classification of Simple and Complex Cyst Simple benign cystQ with good through-transmission, no echoes within the cyst, sharply, marginated smooth wall; requires no surgery

Category II

Looks benign with septation, minimal calcification, and high density; requires no surgeryQ

Category II F

Calcification in wall thicker and more nodularQ than in category II, septa have minimal enhancement; requires no surgery

Category III

Complicated lesion cannot confidently be distinguished from malignancy, more calcification, more prominent septation; more likely to be benign than malignant; requires surgical explorationQ and/or removal

Category IV

Malignant lesionQ with large cystic components, irregular margins; solid vascular elements; requires surgical removalQ

URETEROCELE Ureterocele • Cystic dilation of terminal ureter Types • Intravesical (20%): Most often with single ureterQ • Ectopic (80%): Nearly always involve the upper pole of duplicated uretersQ. Clinical Features • MC presentation is UTI or urosepsisQ • Palpable abdominal mass (due to hydronephrosis)



More common in femalesQ

Urology

Category I

588

Surgery Essence • Prolapse through female urethra as a cyst • Calculi due to urinary stasis, mostly in distal ureter Diagnosis • USG: Hydroureteronephrosis, cyst in bladder • IVP: Typical Adder head or Cobra head or Spring onion appearanceQ is diagnostic of ureterocele • MCU: A smooth filling defectQ in the trigonal area • Cystoscopy: Enlarging & collapsing cystsQ as urine flows Treatment • Significant upper pole function: Endoscopic incision or cyst excision & reimplantationQ • Poor upper pole function: Upper pole nephrectomy and partial ureterectomy.

Urology

DUPLICATION OF URETER (AD) Duplication of Ureter (AD) • • • •

MC congenital anomaly of upper urinary tractQ Mode of inheritance is autosomal dominantQ More common in females and often bilateralQ “Yo-Yo” effectQ in fused ureter (incomplete duplication) is seen.

Types • Incomplete duplication: Both ureters join together and a single ureteric opening • Complete duplication: Both ureters open separately −− Weigert-Meyer’s ruleQ: In cases of complete duplication, the upper pole ureter and the lower pole ureter rotate on their long axes so that the upper pole ureteric orifice is medial & caudal to the lower pole orificeQ. −− Upper pole ureter becomes ectopic & obstructedQ, whereas the lower pole ureter end laterally and have a short intravesical tunnel leading to VURQ. Clinical Features • Many patients are asymptomaticQ • A common presentation is persistent or recurrent infectionsQ. • In females, the upper pole ureter may be ectopic, with an opening distal to the external sphincterQ or even outside the urinary tract. • Such patients have classic symptoms: incontinence characterized by constant dribbling with a normal pattern of voidingQ. • In males, because the mesonephric duct becomes the vas and seminal vesicles, the ectopic ureter is always proximal to the external sphincterQ, and associated incontinence does not occurQ. Diagnosis • IVP: Shows duplication in most of cases • MCU discloses VUR (in lower pole ureter) and demonstrate presence of ureterocele (in upper pole ureter). Treatment • Treatment of reflux alone is not influenced by duplication in most of the cases. • Lower grade reflux is treated medically and higher grade surgically • Surgery is reserved for upper pole obstruction or ectopyQ. If renal function in one segment is very poor, heminephrectomy is the most appropriate treatment. • MC congenital anomaly of upper urinary tract: Duplication of ureterQ • MC congenital anomaly of genitourinary tract: VURQ

Section 4

ECTOPIC URETERAL ORIFICE Ectopic Ureteral Orifice • Around 80% is associated with a duplication collecting systemQ. • More common in femalesQ Location • Males: ProstaticQ or posterior urethra (MC)Q, lateral in bladder • Females: Anterior urethra (MC)Q, vestibule, vagina Clinical Features • Females: Continuous incontinence with an otherwise normal voidingQ, persistent vaginal discharge (Ureter opening in vagina) Q • Males: MC presentation is UTI, no urinary incontinenceQ, recurrent epididymo-orchitis (Epididymitis in a prepubertal boy, ectopic ureter should be ruled out).

Kidney and Ureter

589

Complications

• IVP: Drooping lily signQ (Non visualized upper pole of a duplex system displaces the lower pole down, looking like a drooped down lily flower on IVP) Treatment • Mainly expectant, if there are no symptoms. • Ureteric reimplantation or upper pole nephrectomyQ depending on moiety function.

Section 4

• Ectopic ureter may be severely obstructed, causing massive hydronephrosis Diagnosis

VESICOURETERIC REFLUX (VUR) Vesicoureteric Reflux (VUR) • VUR is the most common inheritable diseaseQ of the genitourinary tract. • Autosomal dominant mode of transmission. • Overall incidence is >10%Q and in children with UTI is 30% • Majority of cases (75%) are asymptomaticQ. • Major cause of VUR is attenuation of trigoneQ and its contiguous intravesical ureteric musculatureQ. Types Primary

Secondary

• The length of submucosal ureter may be short • Deficiency of the longitudinal muscleQ of the intravesical ureter resulting in an inadequate valvular mechanism

• Caused by elevated pressures in the bladder • MC anatomical cause: Posterior urethral valves (50% have VUR)Q • Other causes: Neurogenic bladder or bladder dysfunction

Investigations • MCU is IOC for VURQ • Urine culture • DMSA scan: IOC for pyelonephritis and cortical renal scarringQ MCU Grading of VUR (International classification) Grade I

Reflux into non dilated ureterQ

Grade II

Reflux into pelvis & calycesQ without dilation

Grade III

Mild to moderate dilation of the ureter, renal pelvis & calycesQ with minimal blunting of the fornices

Grade IV

Dilation of the pelvis & calyces with bluntingQ.

Grade V

Gross dilation of the ureter, pelvis & calyces; loss of papillary impression and ureteral tortuosityQ.

• With bladder growth and maturation, most low-grade reflux resolves spontaneouslyQ. • Severe grades of reflux are less likely to resolveQ. • Mean age of reflux resolution is 6-7 yearsQ. • Resolution rates: Grades I & II: 80–84%, Grade III: 50%, Grade IV: 20-30%, Grade V: 0-5%Q • Younger children, especially the neonates, are more likely to have spontaneous resolutionQ • Reflux of infected urine cause pyelonephritis. Repeated such episodes lead to renal scarring and nephropathy resulting in hypertension and azotemiaQ. • If urine is kept sterile, significant nephropathy rarely occurs. Management • Medical management: Keep the urine sterileQ and wait for spontaneous resolution Medical Management Recommended as the Initial Management for • All prepubertal children with grade I-III refluxQ as most of the cases usually resolve. • Unilateral grade IV reflux, especially in young childrenQ. Drugs used in VUR • Age up to 6 weeks: Amoxicillin or AmpicillinQ. • Age after 6 weeks: The biliary system is mature enough to handle TMP-SMX (DOC for prophylaxis)Q. Usually nighttime doses are given. Other option is nitrofurantoin.

Urology

Natural History

590

Surgery Essence −− Periodic cultures every 3 monthsQ for evaluation of breakthrough infections. −− DMSA scan if recurrent bouts of pyelonephritisQ are suspected. Yearly radiographic studies for resolution. • Surgical management: Ureterovesicoplasty or ureteric reimplantationQ and STINGQ (Subureteric transurethral injection of teflon paste) are the treatment options. Methods of Ureteric Implantation • Lich-Gregoir techniqueQ by direct implantation of ureter • Leadbetter-Politano techniqueQ involves creation of a submucosal anti-reflux tunnel.

Urology

Indications of Surgical Management in VUR • • • • • •

Breakthrough UTIsQ despite prophylactic antibiotics Severe grades of reflux- grade V or bilateral grade IVQ New renal scars or deterioration of renal functionQ as on serial USG of DMSA scan. Reflux that persist in girls at full linear growth (at puberty)Q Reflux associated with congenital abnormalities (Bladder diverticula)Q. All secondary reflux, which persistQ after correction of the primary cause e.g. fulguration of posterior urethral valves or management of uninhibited detrusor.

RENAL ARTERY ANEURYSMS Renal Artery Aneurysms • Most are saccularQ, and 75% occur at the bifurcation of the primary or secondary branchesQ. • Medial fibroplasiaQ is the MC cause of true renal aneurysms, followed by degenerative atherosclerosis and polyarteritis nodosa. Spontaneous or traumatic dissection is the MC cause of false renal aneurysmsQ. Clinical Features • Usually asymptomaticQ or have associated renal artery occlusive disease and renovascular hypertension or ischemic nephropathy. • Rupture occurs in < 3%Q of cases, but when the aneurysm ruptures in a pregnant woman, the fetal mortality rate is 75%, and the maternal mortality rate is 50%Q. Diagnosis • Around 50% are diagnosed incidentallyQ, when renal arteriogram is performed for another reason or during workup for hypertension. • Plain film of abdomen may show intra-renal or extra-renal ring like calcificationQ. Treatment • Surgical repair Indications of Surgery • Any aneurysm >2 cmQ • Woman of childbearing ageQ

• EnlargingQ on serial X-rays • Poorly calcified or poor access to healthcareQ

• Patients who are followed for renal artery aneurysms at regular intervals should have thorough medical control of their blood pressure.

HEPATORENAL SYNDROME Hepatorenal Syndrome

Section 4

• HRS is a state of functional renal failure (reduced GFR) without renal pathology in patients with severe liver diseaseQ. • Low cardiac output and high plasma rennin predicts development of HRSQ • Occurs in about 10% of patients with advanced cirrhosis or acute liver failureQ. Pathophysiology of HRS • Marked disturbances in the arterial renal circulation: Increase in renal vascular resistance accompanied by a reduction in systemic vascular resistanceQ. • The reason for renal vasoconstriction is most likely multifactorial and is poorly understood. • Structurally or histologically kidneys are normal and recover function after successful liver transplantationQ. • Pathogenic hallmark of HRS is intense renal vasoconstriction with co-existent vasodilatationQ. • In HRS, urine sodium is typically < 10 mEq/L with hyperosmolar urine, oliguria (< 400 mL/24 hr), fractional excretion of sodium < 1; and urine creatinine-to-plasma creatinine ratio > 30:1.

Kidney and Ureter Type 1 HRS

591

Type 2 HRS • Characterized by a reduction in glomerular filtration rateQ with an elevation of serum creatinine level, but it is fairly stableQ • Associated with a better outcome

Clinical Features • HRS is often seen in patients with refractory ascites and requires exclusion of other causes of acute renal failure. Diagnosis • The diagnosis is made usually in the presence of a large amount of ascites in patients who have a stepwise progressive increase in creatinineQ.

Section 4

• Characterized by a progressive impairmentQ in renal function and a significant reduction in creatinine clearance within 1–2 weeks of presentationQ. • Poor outcomeQ

International Ascitic Club Criteria for HRS • Serum creatinine > 1.5 mg/dLQ • Low urine volume (< 500 mL/day) and low urine sodium (< 10 mEq/L)Q • Absence of shock, bacterial infection, nephrotoxic drugs, diarrhea or renal fluid lossesQ • No sustained improvement in renal function after diuretic withdrawal and expansion of plasma • Absence of significant proteinuria (< 500 mg/day)Q volume with 1.5 L of isotonic salineQ • No evidence of obstructive uropathyQ Treatment • • • •

Drug of choice is terlipressinQ (albumin improves the therapeutic response) Midodrine + octreotide + IV albumin may reverse renal failureQ in some patients with HRS Best therapy for HRS: Liver transplantation (recovery of renal function is typical in this setting)Q In patients with either type 1 or type 2 HRS, the prognosis is poor unless transplant can be achieved within a short period of time.

URINARY ASCITES Urinary Ascites Forty percent of neonatal ascites is caused by urinary conditionsQ. Urinary ascites occurs when high intraluminal pressure forces urine to extravasate from the kidneyQ, usually across a renal fornix. Urine then enters the retroperitoneum and travels across the peritoneum as a transudate. If aspirated from the peritoneal cavity, ascites or extravasated urine contains electrolyte & creatinine levels similar to serumQ. The urine within the peritoneum is subject to the large absorptive mesothelial surface that quickly normalizes these values, masking the identity of ascitic fluid as urine. • Diagnosis of urinary ascites may be difficult and may require definitive upper tract drainage in the form of nephrostomy tubes in order to establish the etiology of the ascites and allow its resolution. • Urinary ascites in the case of distal obstruction may serve to lower urinary pressures and offer some protection to the developing kidneysQ. • • • • •

Posterior urethral valvesQ Urethral strictureQ Urethral atresiaQ Bladder outlet obstructionQ Ectopic ureteroceleQ Neurogenic bladderQ

Causes of Urinary Ascites • Hydrocolpos • Sacrococcygeal teratomaQ • Bladder perforationQ during delivery • Urachal lacerations secondary to umbilical artery catheterization

PATENT URACHUS Patent Urachus • A patent urachus is suspected in the neonatal period by continuous or intermittent drainage of fluid from umbilicusQ • Most common organisms cultured from the umbilical drainage include Staphylococcus aureus, Escherichia coliQ • Additional presentations include an enlarged or edematous umbilicus & delayed healing of cord stumpQW Diagnosis • Diagnosis is confirmed by demonstration of the fluid-filled canal on longitudinal ultrasound or contrast filling on retrograde fistulogram or voiding cystourethrogram (VCUG)Q Management • Management of an infected urachus with abscess formation includes initial drainage under antibiotic coverageQ • Once the infection has subsided, complete excision of the patent urachus including a bladder cuff is required • Removing all anomalous tissue avoids: −− Recurrences or stone formationQ −− Prevents the rare event of later transformation into a malignant adenocarcinomaQ.

Urology

• • • • • •

Multiple Choice Questions c. Junction of ureter and renal pelvis d. At crossing of gonadal vessels and ureter

RENAL AND URETERIC CALCULI

1. Renal calculi associated with proteus infection: (All India 2011, 2009) a. Uric acid b. Triple phosphate c. Calcium oxalate d. Xanthine



2. Nephrolithiasis occurs with the toxicity of: (COMEDK 2005) a. Ritonavir b. Saquinavir c. Indinavir d. Nelfinavir



3. Not true about ‘Struvite Stones’ is: a. Better known as staghorn calculus b. These are triple phosphate stones c. Common in infected urine d. Usually seen in acidic urine

15. A patient was admitted with complaints of ureteric stone. He was on treatment with IV fluids and analgesics. Suddenly he developed radiating pain to the pubic area and medial aspect of the thigh. The stone is coming down. What is the most probable site of lodgment of the stone? (DPG 2011) a. At renal pelvis b. At pelvic brim c. At the level of gonadal vessels d. Intramural portion of the ureter



16. Triad of renal colic, swelling in loin which disappears after passing urine is called: (All India 96) a. Kocher’s triad b. Saint’s triad c. Dietl’s crisis d. Charcot’s triad



17. Ureteric colic due to stone is caused by: (UPPG 2010, All India 2008) a. Stretching of renal capsule due to back pressure b. Increased peristalsis of ureter to overcome the obstruction c. Irritation of intramural ureter d. Extravasation of urine



18. A patient with alkaline urine which is cloudy with plenty of pus cells is suffering from infection with: (Kerala 89) a. E. coli b. Proteus c. TB d. None



19. Referred pain from ureteric colic is felt in the groin due to involvement of the following nerve: (All India 2003) a. Subcostal b. Iliohypogastric c. Ilioinguinal d. Genitofemoral



20. Commonest presentation of bilateral ureteric stones: a. CRF b. UTI (AIIMS 91) c. Pain d. Hematuria



21. Most severe pain in ureteric stone is seen in cases of: a. Oxalate stones (UPPG 99) b. Triple phosphate c. Cystine stone d. Uric acid stone



22. Ureteric colic characterized by all except: a. Acute onset b. Stillness of the patient c. Responds to antispasmodics d. Radiates to the groin



23. Treatment of choice of ureteric colic is: (GB Pant 2010) a. Nitrites b. Pethidine c. Adrenaline d. Diclofenac

(AIIMS Nov 2001)

(TN 98) b. Urinary stones d. Bacterial infections



4. Randall’s plaques causes: a. Bile stones c. Premalignant lesions



5. Commonest stone in case of UTI: (AIIMS Nov 97) a. Phosphate b. Urate c. Cysteine d. Calcium oxalate



6. Oxalate stones are found in: (PGI June 2006) a. Ethylene glycol b. Ethanol c. Diethyl glycol d. Methyl alcohol



7. Staghorn calculus is made of: (Recent Question 2017, DNB 2012, UPSC 97) a. Oxalate b. Phosphate c. Uric acid d. Cystine 8. Renal stones which are laminated and irregular in outline are: (Recent Question 2013) a. Uric acid b. Calcium oxalate c. Struvite d. Cystine 9. Most common renal stone: (Recent Question 2017) a. Calcium oxalate stone b. Uric acid stone c. Staghorn calculi d. Cystine stone 10. Stone formed in alkaline urine is: (Recent Question 2017) a. Calcium oxalate b. Calcium phosphate c. Cysteine d. Uric acid 11. Chronic laxative abuse can result in the formation of which type of stone? (Recent Question 2018) a. Xanthine b. Cysteine c. Ammonium urate d. Struvite 12. Potent producer of urease is: (Recent Question 2017) a. E. coli b. Proteus c. Klebsiella d. Pseudomonas 13. Struvite stone is caused by which metal? (Recent Question 2018) a. Magnesium b. Calcium c. Sodium and potassium d. Both a and b

RENAL AND URETERIC CALCULI CLINICAL FEATURES



14. Locate the renal stone with pain radiating to medial side of thigh and perineum due to slipping of stone in males: a. At pelvic brim (AIIMS June 2010, All India 96) b. Intramural opening of ureter

(UPPG 2007)

RENAL AND URETERIC CALCULI DIAGNOSIS AND TREATMENT

24. A petient present with pain and tenderness in the left iliac fossa. USG shows a 3 cm stone in the renal pelvis without any hydro-nephrosis. Most appropriate management: a. PCNL (AIIMS May 2012) b. ESWL c. Diuretics d. Medical dissolution threapy with KCl

Kidney and Ureter 25. All are radioopaque except one: (AIIMS June 2000) a. Oxalate b. Uric acid c. Cystine d. Mixed



26. What is the name of this intervention performed frequently for renal stone removal? a. ESWL b. PCNL c. Ureteroscopy d. Laparoscopic stone surgery



27. What is the name of this intervention performed frequently for renal stone removal? a. ESWL b. PCNL c. Ureteroscopy d. Laparoscopic stone surgery

28. A child presents with complaints of abdominal colic and hematuria USG showed a renal stone 2.5 cm in diameter in renal pelvis the next step in management of this case:  (AIIMS Nov 2000) a. ESWL b. Pyelolithotomy c. Nephroureterostomy d. Conservative



29. Which of the following stones is hard to break by ESWL? a. Calcium oxalate monohydrate (All India 2010) b. Calcium oxalate dehydrate c. Uric acid d. Struvite



30. Ramesh, 30-year-old male presented with repeated attacks of renal colics. X-ray KUB was done. Findings are suggestive of: a. Calcium oxalate stone b. Uric acid stone c. Struvite stone d. Cystine stone



31. A 50 years old female is admitted with abdominal pain and anuria. Radiological studies revealed: bilateral impacted ureteric stones with hydronephrosis. Urine analysis showed RBCs with pus cells in urine. Serum creatinine level was 16 mg/dl and urea level was 200 mmol/l. Which of the following should be the immediate treatment? (All India 2010) a. Hemodialysis b. ‘j’ stent drainage c. Lithotripsy d. Ureteroscopic removal of stones



32. What complication should one expect when PCNL is done through 11th intercostals space? (All India 2010) a. Hydrothorax b. Hematuria c. Damage d. Remnants fragments

33. Which of the following is not a contraindication for extra corporeal shockwave lithotripsy (ESWL) for renal calculi? a. Uncorrected bleeding diathesis  (AIIMS June 2003) b. Pregnancy c. Ureteric stricture d. Stone in a calyceal diverticulum

34. A 10 mm calculus in the right lower ureter associated with proximal hydroureterone-phrosis is best treated with: a. ESWL (All India 2003) b. Antegrade percutaneous access c. Open ureterolithotomy d. Ureteroscopic retrieval



35. Treatment used for lower ureteric stone is: (AIIMS June 98) a. Endoscopic removal b. Diuretics c. Drug dissolution d. Laser



36. Correct order of stones on the basis of images of crystals: a. Calcium oxalate, struvite, uric acid, cystine b. Calcium oxalate, uric acid, struvite, cystine c. Uric acid, calcium oxalate, struvite, cystine d. Struvite, uric acid, calcium oxalate cystine

Urology



Section 4



593

594

Surgery Essence

38. Identify the crystals depicted in urine microscopy. (APPG 2015) a. Oxalate crystals b. Cystine crystals c. Struvite crystals d. Uric acid crystals

Urology



37. Steinstrasse is: (Recent Questions 2017, HPU 2001) a. Staining of stones b. Stones c. Failure of ESWL d. Ureteric obstruction due to fragments in ureter



39. Which of the following statements about the Holmium: YAG laser is incorrect? (AIIMS June 2004) a. It has a wavelength of 2100 nm b. Its use for uric acid stones has caused deaths due to generation of cyanide c. It is effective against the hardest urinary stones d. It can even cut the wire of stone baskets



40. All are indicated in a patient with cystinuria with multiple renal stones except: (AIIMS Nov 2012) a. Cysteamine b. Increase fluid intake c. Alkalinization of urine d. Penicillamine



41. Percentage of renal stones which are radio opaque: a. 10% b. 25 c. 37% d. 75% e. 90%



42. Dormia basket is used for removal of renal calculi in the: a. Pelvic ureteric junction b. Upper 1/3rd of ureter c. Middle 1/3rd of ureter d. Lower 1/3rd of ureter



43. Which is false regarding ureteric stones? (AIIMS 92) a. Urine is always infected b. Should be removed immediately c. Source is always the kidneys d. Pain in referred to tip of penis in intramural stones

Section 4



44. Identify this crystal found in urine analysis. a. Calcium carbonate stone (AIIMS November 2015) b. Ammonium phosphate stone c. Uric acid d. Calcium oxalate stone



45. Which one of the following is radiolucent stone?  (Recent Question 2017, 2015, GB Pant 2010, MCI Sept 2007) a. Calcium oxalate b. Cystine c. Uric acid d. Phosphate



46. Treatment of choice for 0.5 mm renal calyx stone is: a. ESWL b. PCNL (DNB 2007) c. Ureteroscopy d. Cystoscopy



47. All are risk factors for nephrolithiasis except: (DNB 2007) a. Renal tubular acidosis b. High protein intake c. High calcium intake d. Hypercalciuria



48. All of the following statement about renal calculi are true, except: (All India 93) a. Cystine stones form in acidic urine b. Struvite stones form in alkaline urine c. Oxalate stones are radiopaque d. Uric acid stones are resistant to ESWL



49. Staghorn calculus is made of: a. Oxalate b. Phosphate c. Uric acid d. Cystine



50. What is the name of this intervention?

(DNB 2000)

a. ESWL b. PCNL c. URS d. Electrohydraulic lithotripsy

51. Treatment used for lower ureteric stone is: (AIIMS June 98) a. Endoscopic removal b. Diuretics c. Drug dissolution d. Laser



52. Chandu, 45-year-old male shows calcification on the right side of his abdomen in an AP view. In lateral view the calcification is seen to overlie the spine. Most likely diagnosis is: (All India 2001) a. Gallstones b. Calcified mesenteric nodes c. Renal stones d. Calcified rib 53. Which of the following advises is not given to a 35 years old female patient with recurrent renal stone? (AIIMS Nov 2012) a. Increase water b. Restrict protein c. Restrict salt d. Restricted calcium intake



Kidney and Ureter

55. Which of the following statement is true about the given image? (Recent Question 2018)

a. Stent for bile duct obstruction caused by malignancy b. DJ stent after ESWL c. Stent for pancreatic duct obstruction caused by malignancy d. PCNL 56. Which of the following is not true regarding ureteric calculus? (MHPGMCET 2008) a. Severity of pain increases with size of the calculus b. Hematuria is invariably seen in all cases c. Pain may radiate to the anterior aspect of thigh d. Pain may radiate to tip of the penis





58. LASER used in treatment of ureteric calculi: (MHSSMCET 2008, 2006, All India 2003) a. Holmium b. Nd-Yag c. Argon d. CO2

59. Following is true about PCNL except: (MHSSMCET 2009) a. Bleeding is least concerned complication b. Involves the placement of a hollow needle into the renal colleting system through the soft tissue of the loin and the renal parenchyma c. Perforation of the colon or pleural cavity during placement of the percutaneous track d. Perforation of the collecting system may occur 60. A patient is passing stones recurrently in urine for past few years. All are to be restricted in diet except: (AIIMS Nov 2010) a. Protein restriction b. Calcium restriction c. Salt restricted d. Phosphate restriction

61. Management of 4 cm size renal staghorn calculus:  (AIIMS November 2017) a. ESWL b. PCNL c. Intra renal repair surgery d. Open pyelolithotomy

RENAL INFECTIONS

62. Most common cause of emphysematous pyelonephritis: (GB Pant 2011, COMEDK 2008) a. E. coli b. Proteus c. Klebsiella d. Pseudomonas

(AIIMS 92)



64. Following is true of pyonephrosis except: a. Commonly associated with renal calculi b. Always unilateral c. Is a complication of hydronephrosis d. Follows acute pyelonephritis



65. Subcapsular nephrectomy is indicated is: (PGI 93) a. Perinephric abscess b. Hydronephrosis c. Pyonephrosis d. Solitary adenocarcinoma



66. A boy is suffering from acute pyelonephritis. Most specific urinary finding will be: (AIIMS May 2012) a. WBC cast b. Leucocyte esterase test c. Nitrite test d. Bacteria in Gram stain

67. Xanthogranulomatous pyelonephritis is often associated with infection by: (DNB 2010, COMEDK 2009) a. Proteus b. E. coli c. H. influenza d. Klebsiella

68. Most common predisposing factor for chronic pyelonephritis is: (DNB 2004, 2000) a. Diabetes mellitus b. Renal stone c. Posterior urethral valve d. Vesicoureteric reflux

GENITOURINARY TUBERCULOSIS

69. Most common route of infection in kidney tuberculosis: (Recent Question 2015, All India 93) a. Ascending spread b. Hematogenous c. Lymphatic spread d. Direct invasion

70. The most sensitive imaging modality to detect early renal tuberculosis is : (Recent Questions 2017) a. Intravenous urography b. Computed tomography c. Ultrasound d. Magnetic resonance imaging

71. Earliest sign of renal tuberculosis on IVP: a. Caliectasis b. Moth eaten calyx c. Splaying of calyces d. Hydronephrosis

Urology



57. Commonest cause of ureteric obstruction:  (MHSSMCET 2006) a. Stone b. Clot c. Cast d. Carcinoma

63. Sheela, a middle aged diabetic female presented with flank pain and fever. On U/S the kidney was irregular and showed fat density lesion with calculi. The diagnosis is most probably: (AIIMS Nov 2001) a. TB kidney b. Xanthogranulomatous kidney c. Chronic pyelonephritis d. Renal abscess

Section 4

54. Which of the following are radiolucent renal stones? (MHPGMCET 2002, JIPMER 2012) a. Uric acid stones b. Cystine stones c. Mixed stones d. Calcium oxalate stones

595

Urology

596

Surgery Essence

72. In genitourinary TB, true is: (PGI Dec 98) a. Sterile pyuria is consistent finding b. AFB in early morning sample is always positive c. MC site is pelvis d. Commonest cause of pyelonephritis



73. Renal tuberculosis originates in the: (Recent Questions 2017) a. Renal papilla b. Renal medulla c. Afferent tubules d. Efferent arteriole of glomerulus



74. “Golf-hole” ureter is seen in: (WBPG 2015, 2014; Karnataka 94) a. Ureteric calculus b. Ureteral polyp c. Tuberculosis of ureter d. Retroperitoneal fibrosis



75. Earliest and often the only presentation of TB kidney is: (DNB 2005, 2001) a. Increased frequency b. Pain c. Hematuria d. Renal calculi



76. In a patient who was has acid-fast bacilli in the urine: a. Calcification of the bladder is common b. Bladder disease is associated with extensive renal disease c. Ureteric involvement causes shortening of the ureters d. Renal disease can produce changes identical to reflux nephropathy e. Ureteric calculi are commonly present



77. A 35 years old male came to hospital with dysuria and pus cells on examination. Urine culture came negative (sterile pyuria). After few hours patient deteriorated suddenly and died due to renal failure. Gross autopsy pictures of kidneys shown. Family history of renal disease was present. What is the diagnosis? (JIPMER November 2017)

HYDRONEPHROSIS

81. A lady present with unilateral hydronephrosis on USG. She is asymptomatic. Ureteropelvic drainage is normal. What is/ are treatment modality for her? (PGI Dec 2008) a. Pyloroplasty b. Analgesia SOS c. Under observation d. Follow up by USG



82. Unilateral hydronephrosis is due to: a. Bladder neck contracture b. Stricture urethra c. Carcinoma of prostate d. Ureterocele

(AMC 99)

83. A 60 years old male with poor stream of urine, post void residual urine is 400 mL, bilateral hydronephrosis and prostate weighing 70g. His urea is 120 and creatinine 3.5. Ideal “next immediate” step: (AIIMS Nov 2010) a. Catheterize with Foley catheter b. Bilateral PC nephrostomies c. CT to rule out carcinoma d. MRI pelvis

Section 4

a. Renal cell cancer c. Cystic renal disease

b. Renal tuberculosis d. Metastasis



78. Genitourinary TB in a male patient presents with: a. Painful and tender epididymis (JIPMER 2011) b. Bacteriuria without pyuria c. Renal cysts (unilateral) d. Microscopic hematuria



79. Sterile pyuria is characteristically seen in: (All India 2011) a. Renal tuberculosis b. Chronic Hydronephrosis c. Wilm’s Tumor d. Neuroblastoma

80. In a patient of genitourinary tuberculosis, IVP was done. What is the name of the sign seen in the image? a. Moth eaten calyx b. Pipe stem ureter c. Bladder calcification d. Thimble bladder

84. Hydronephrosis due to obstruction of ureter is best diagnosed by: (MHPGMCET 2008) a. IVU b. Radioisotope scan c. Retrograde pyelography d. Whitaker test

85. What is the name of radiological sign seen in a patient with history of renal stones? a. Cup sign b. Honeycombing c. Egg in cup appearance d. Rim sign

Kidney and Ureter DIAGNOSTIC AND LABORATORY INVESTIGATIONS





86. DTPA scan of hypertensive young lady is normal, USG shows small kidney on left side. Next investigation will be: a. CT Scan abdomen (AIIMS Nov 2000) b. Retrograde pyelogram c. Digital subtraction angiography d. DMSA 87. After a single episode of painless gross hematuria in a boy. Doctor performed an excretory urogram showing a filling defect towards the lower renal infundibulum 1.5 cm in size. What will be the next investigation to be done? (Karnataka 2003, AIIMS Nov 2000) a. Cystoscopy b. Urine cytology c. USG d. Retrograde pyelography 88. An absolute contraindication for IVP is: (All India 96) a. Allergy to the drug b. Multiple myeloma c. Blood urea > 200 mg d. Renal tumor

89. During investigation of hydronephrosis, isotope renogram is useful mainly in: (UPSC 2000) a. Detecting vesicoureteric reflux b. Anatomical definition c. Distinguishing between non-obstructed system d. Identifying ectopic kidney tissue

90. Pseudo kidney is: a. Thickened bowel loop on USG b. Hydronephrosis c. Unascended kidney d. Undescended testes

(J and K 2001)

91. The investigation of choice for renal scarring defect in kidney: (Recent Question 2016, All India 2012) a. DMSA scan b. DTPA scan c. Dexa scan d. MCU 92. The substance present in the gallbladder stones or the kidney stones can be best identified by the following techniques: a. Fluorescence spectroscopy (All India 2003) b. Electron microscopy c. Nuclear magnetic resonance d. X-ray diffraction



93. Reflux nephropathy is diagnosed mainly by: (MCI Sept 2005) a. X-ray KUB b. Micturating cystourethrogram c. CT scan d. MRI

94. A patient presents with hematuria of several days and dysmorphic RBC casts in urine. The site of origin is: a. Kidney b. Ureter(AIIMS Nov 2001) c. Bladder d. Urethra



95. Isotope Renogram: (Karnataka 94) a. Study of rennin mechanism b. Contrast study of kidneys, ureter and bladder c. Utilized in mapping the anatomy of kidneys d. Graphic representation of radioactivity of kidneys 96. ‘Rim’ and ‘ball’ nephrograms in intravenous urography are seen in: (COMEDK 2009) a. Normal kidneys b. Acute obstructive nephropathy c. Chronic obstructive nephropathy d. Chronic renal failure



98. Indications of percutaneous nephrostomy: a. Stone removal b. Ureteral obstruction c. Anterograde renography d. Renal tumor resection e. Ischemic renal failure



99. One of the following is characterized by RIM sign: (AIIMS Nov 98) a. Hydronephrosis b. Hypernephroma c. Chronic pyelonephritis d. Polycystic kidney

(PGI Nov 2010)

100. Percutaneous nephrostomy is indicated in: (DNB 2001) a. Polycystic kidney disease b. Solitary adenocarcinoma c. Simple hydronephrosis d. Pyonephrosis 101. Most useful investigation in a child, who recovered from the bout of pyelonephritis: (Recent Question 2017) a. DTPA scan b. DMSA scan c. MCU d. IVP

BENIGN RENAL TUMORS 102. Regarding angiomyolipoma of kidney, what is incorrect? a. Pain in the loin (AIIMS Nov 94) b. Presents with hypertension c. Bleeding is self limited d. Nephrectomy is the treatment of choice 103. A complex renal cyst was incidentally detected on ultrasound in a patient following which he underwent a CT for an insurance workup. What is the likely diagnosis?  (AIIMS May 2017)

Urology



97. Radiation exposure is the least in the following procedure: a. Micturating cystourethrogram (AIIMS Nov 2010) b. IVP c. Bilateral nephrostogram d. Spiral CT for stones

Section 4





597

a. Oncocytoma c. Perinephric cyst

b. Angiomyolipoma d. Renal cell carcinoma

104. Central stellate scar on CT scans are seen in: (COMEDK 2008) a. Renal hemangioma b. Renal oncocytoma c. Wilm’s tumour d. Papillomas

RENAL CELL CARCINOMA TYPES 105. Renal cell carcinoma histopathologicaly showing ‘perinuclear halo’ and “Plant like” structure in malignant cells is seen in: a. Clear cell tumor (PGI Dec 2000) b. Papillary carcinoma c. Collecting duct carcinoma d. Chromophobe cell carcinoma

598

Surgery Essence 106. Which of the following statements is true regarding kidney tumors? (PGI Dec 2001) a. Mutated VHL gene is associated with clear cell carcinoma b. Extreme hyperdiploidy occurs c. Extreme hypodiploidy occurs d. Renal papillary carcinoma has defect in chromosome 8 e. Oncocytoma has defect in chromosome 11

Urology

107. The most common histological variant of renal cell carcinoma is: (Bihar PG 2016) a. Clear cell type b. Chromophobe type c. Papillary type d. Tubular type 108. Chromophobe variant to renal cell carcinoma is associated with: (All India 2010) a. VHL gene mutations b. Trisomy of 7 and 17 (+7, +17) c. 3 p deletions (3 p-) d. Monosomy of 1 and Y ( -1, -Y) 109. Bilateral renal cell carcinoma is seen in: a. Eagle-Barett’s syndrome b. Beckwith-Weidman syndrome c. von-Hippel Lindau (VHL) syndrome d. Bilateral angiomyolipoma

(COMEDK 2008)

110. Most common site of origin of RCC: (MHSSMCET 2008) a. PCT b. DCT c. Collecting ducts d. Loop of Henle 111. Sickle cell anemia is associated with which type of renal cell carcinoma? (JIPMER November 2017) a. Medullary b. Papillary c. Chromophobe d. Colloid

RENAL CELL CARCINOMA CLINICAL FEATURES, PARANEOPLASTIC SYNDROMES 112. All can be seen in hypernephroma, except: (AIIMS Nov 93) a. Polycythemia b. Renal vein thrombosis c. Hypertension d. Hematuria 113. Not correct regarding renal cell carcinoma: a. May be associated with varicocele b. May invade renal vein (AIIMS Nov 94, June 95) c. More common in female d. Arises from proximal convoluted tubule 114. Most common site for secondary metastasis in a case of hypernephroma: (AIIMS Feb 97) a. Adrenal b. Lungs c. Brain d. Bones

Section 4

115. Painless gross hematuria occurs in: (All India 94) a. Renal cell carcinoma b. Polycystic kidney c. Stricture of urethra d. Wilm’s tumor 116. In renal cell carcinoma, which is true? (JIPMER 95) a. Hypercalcemia b. Polycythemia c. Cushing syndrome d. All 117. Cannon ball deposits seen in the lungs are characteristic of: a. Seminoma testis b. Carcinoid (DNB 2003) c. Hypernephroma d. Pheochromocytoma 118. Bilateral RCC may be seen in: a. Tuberous sclerosis b. von-Willebrand’s disease c. von-Hippel Lindau disease d. von-Recklinghausen disease

(DNB 2002)

119. All are features of hypernephroma except: (HPU 2005) a. Persistent pyrexia b. Hematuria c. Polycythemia d. Lower pole involvement 120. All are true about renal cell carcinoma except: a. Invasion of renal vein means inoperability b. Presents with abdominal pain, hematuria c. Arises from tubular epithelium d. More common in males

(DPG 2006)

121. Regarding RCC all are true except: (DPG 2005) a. Renal lump, pain abdomen, hematuria b. Associated with anemia and low ESR c. Propensity to invade IVC d. Invasion of renal vein is contraindication for surgery 122. Not correct regarding renal cell carcinoma: (Recent Questions 2017) a. May be associated with varicocele b. May invade renal vein c. More common in female d. Arises from proximal convoluted tubule 123. A 55-year-old male with 35 pack years presented with painless mass in left scrotal sac and microscopic hematuria. On laboratory investigation, Alpha-fetoprotein and lactate dehydrogenase was negative. What is the diagnosis? (AIIMS May 2013) a. Epididymitis b. Seminoma c. Renal cell carcinoma d. Carcinoma lung 124. All can be seen in hypernephroma, except: (AIIMS Nov 93) a. Polycythemia b. Renal vein thrombosis c. Hypertension d. Hematuria 125. Paraneoplastic syndrome associated with RCC are all of the following except: a. Polycythemia b. Hypercalcemia c. Malignant hypertension d. Cushing syndrome 126. Most common presentation of renal adenocarcinoma:  (COMEDK 2005) a. Hematuria b. Local pain c. Mass d. Fever 127. Commonest manifestation of Grawitz’s tumor in male: a. Secondary deposits (MHPGMCET 2009) b. Pathological fracture c. Hematuria d. Rapidly developing varicocele

RENAL CELL CARCINOMA DIAGNOSIS AND TREATMENT 128. A patient presented with renal cell carcinoma invading IVC and renal vein. False statement is: (AIIMS Nov 2001, June 2001) a. Pre operative biopsy is not necessary b. IVC involvement indicates inoperability c. Pre-op radiotherapy is not essential d. Chest X-ray should be done to rule out pulmonary metastasis 129. Most important prognostic indicator for renal cell carcinoma:  (AIIMS May 2009) a. Nuclear grade b. Histological type c. Size d. Pathological staging 130. A 60 years old male, who is a chronic smoker, presented with ‘too late’ triad. Chest X-ray film image is given below. What is the most probable diagnosis? (Recent Question 2016) a. Carcinoma bladder b. Carcinoma renal pelvis c. Choriocarcinoma d. Renal cell carcinoma

Kidney and Ureter

599

Section 4

a. Oncocytoma c. Perinephric cyst 131. The treatment of choice in renal cell carcinoma with the tumor lf less than 4 cm in size is: (AIIMS Nov 2004) a. Partial nephrectomy b. Radical nephrectomy c. Radical nephrectomy + post operative radiotherapy d. Radical nephrectomy + chemotherapy 132. False regarding hypernephroma is:  (Recent Question 2014, AIIMS Nov 93) a. Radiosensitive b. Arise from cortex usually from pre existing adenoma c. May present with rapidly developing varicocele d. Usually adenocarcinoma 133. A 30 years old male presents with pain on the right flank and hematuria. CECT abdomen reveals a large 8 x 8 cm sized solid mass in the right kidney and 3 x 3 cm solid mass occupying the upper pole of left kidney. The most appropriate surgical treatment for this patient is: (AIIMS Nov 2002) a. Bilateral radical nephrectomy b. Right radical nephrectomy and biopsy of the mass from opposite kidney c. Right radical nephrectomy and left partial nephrectomy d. Right radical nephrectomy only

135. Renal collar to prevent spread of malignancy from kidney is put around: (JIPMER 93) a. Aorta b. IVC c. Renal vein d. Renal artery 136. A 40 years old man presented with painless hematuria Bimanual examination revealed a ballotable mass over the right flank. Subsequently right nephrectomy was done and mass was seen to be composed of cells with clear cytoplasm. Areas of hemorrhage and necrosis were frequent. Cytogenic analysis of this mass is likely to reveal an abnormality of: (All India 2004) a. Chromosome 1 b. Chromosome 3 c. Chromosome 11 d. Chromosome 1 137. A 65 years old male banker came for an ultrasound to renew his medical insurance. In right kidney, a complex cyst was found and the picture of CT scan is given below. Most likely diagnosis is: (AIIMS May 2017)

138. A patient with a suspected renal tumor/mass is diagnosed by exfoliative urinary cytology. Which of the following histological types is most likely to be diagnosed on urinary exfoliative cytology? (All India 2012) a. Transitional cell carcinoma b. Adenocarcinoma c. Well differentiated carcinoma (Low grade) d. All three type can be easily detected on exfoliative cytology 139. The commonest systemic abnormality associated with renal cell carcinoma is: (COMEDK 2009) a. Hypertension b. Polycythemia c. Elevated ESR d. Pyrexia 140. A 40 years old patient with a single kidney presents with a solitary exophytic mass of 4 cm localized at its lower pole. Which amongst the following is the best recommended management option? a. Partial nephrectomy b. Radical nephrectomy c. Radical nephrectomy with dialysis d. Radical nephrectomy with immediate renal transplantation 141. Radical Nephrectomy include all of the following except: a. Early ligation of vessels (MHSSMCET 2008) b. Lymphadenectomy c. Keeping fascia back in place d. Removal of kidney including the Gerota’s fascia 142. In radical nephrectomy, following structures are removed except: (Recent Question 2017) a. Gerota’s fascia b. Ipsilateral adrenal gland c. Surrounding hilar nodes d. Para-aortic nodes 143. Best investigation for diagnosis and extension of IVC thrombus in renal cell carcinoma: (Recent Question 2016) a. MRI b. CT c. Venacavagraphy d. USG 144. Stage T3 renal cell carcinoma: (Recent Question 2017) a. Tumor invades beyond Gerota’s fascia b. Contiguous extension into the ipsilateral adrenal gland c. Tumor grossly extends into the inferior vena cava d. Confined to the kidney

WILMS’ TUMOR 145. Commonest presentation of Wilm’s tumour is: a. Hematuria c. Hydronephrosis

(Recent Questions 2017) b. Abdominal lump d. Pain in abdomen

Urology

134. Ideal approach for renal malignancy is: a. Transperitoneal b. Retroperitoneal c. Lumbar incision d. Abdominothoracic incision

b. Angiomyolipoma d. Renal cell carcinoma

600

Surgery Essence 146. True regarding Wilm’s tumour is: (AIIMS Nov 94) a. Bone metastasis b. Always unilateral c. Very commonly metastasize to liver d. Worst prognosis among infants 147. The most important determinant of prognosis in Wilm’s tumor is: (All India 2006) a. Stage of disease b. Loss of heterozygosity of chromosome 1p c. Histology d. Age less than 1 year at presentation

Urology

148. The ideal timing of radiotherapy for Wilm’s tumor after surgery is: (All India 2006) a. Within 10 days b. Within 2 weeks c. Within 2 months d. Anytime after surgery 149. Neuroblastoma differs from Wilm’s tumor radiologically by all except: (AIIMS June 2001) a. Calcification b. Aorta and IVC are not eroded but pushed aside c. Same location d. Intraspinal extension of tumor 150. Earliest symptom of Wilm’s tumour: (Recent Questions 2017) a. Hematuria b. Pyrexia c. Abdominal mass d. Metastases 151. The triad of Wilm’s tumour is: a. Hematuria b. Mass abdomen c. Pain d. Fever e. Weight loss 152. All are true about Wilm’s tumour except: a. Fever and weakness are clinical features b. Arises from primitive cells c. Hematuria almost always present d. It presents as abdominal mass

(All India 97)

153. Good prognosis in Wilm’s tumour is seen in: (Punjab 2008, Kerala 91) a. 2-5 years b. Less than 1 year c. Male child d. Female child

TUMORS OF RENAL PELVIS 160. Commonest type of cancer of the renal pelvis and upper ureter is: (Recent Questions 2017) a. Transitional cell carcinoma b. Adenocarcinoma c. Squamous cell carcinoma d. Nephroblastoma 161. Epidermoid carcinoma of renal pelvis is usually associated with: (Karnataka 94) a. Multiple papillomas b. Pelvic calculus c. Tuberculosis of kidney d. Filariasis 162. ‘Stipple sign’ in transitional cell carcinoma of the renal collecting system is best demonstrated by:(COMEDK 2009) a. Intravenous urography b. Retrograde pyeloureterography c. Radionuclide scan d. Ultrasound scan 163. Nephroureterectomy is indicated in: (DNB 2011) a. Renal cell carcinoma b. Chronic pyelonephritis c. Polycystic kidney disease d. Transitional carcinoma of the pelvis extending till ureter 164. Gold standard treatment of TCC involving renal pelvis:  (Recent Question 2016) a. Radical nephroureterectomy b. Pelviureterectomy c. Radical nephrectomy d. Conservative Nephrectomy 165. In a chronic smoker, who presented with hematuria, RGP was performed as patient was having carcinoma renal pelvis. What is the name of sign seen on this film? a. Bergman sign b. Stipple sign c. Goblet sign d. Both b and c

154. All are true regarding Wilm’s tumour except: (AIIMS 92) a. Preoperative use of Actinomycin D b. Postoperative radiotherapy c. Good prognosis in infants d. Neuroblastoma is the commonest differential diagnosis 155. Commonest site of metastasis of Wilm’s tumour is: a. Bones b. Lungs (AIIMS 94) c. Liver d. Brain

Section 4

156. Which of the following is the postchemotherapy based staging system in Wilm’s tumor? (All India 2009) a. National Wilm’s tumor staging system (NWTSG) b. International society of Pediatric Oncology (SIOP) c. AJCC TNM d. Chadwick 157. All are associated with Wilm’s tumor except: (AIIMS Feb 97) a. Aniridia b. Male pseudo hermaphrodite c. Arthogryposis multiplex congenita d. Hemihypertrophy 158. Which of the following is the treatment of choice for stage I Wilm’s tumor? (All India 2012) a. Laparoscopic nephrectomy b. Open nephroureterectomy c. Chemotherapy d. Observation 159. Wilm’s tumor chromosome is: a. 13 q b. 13 p 14 c. 11 p 13 d. 17

(JIPMER 2012)

RENAL TRAUMA 166. Which of the following is true about renal trauma? a. Urgent IVP is indicated (All India 95) b. Exploration of the kidney to be done in all cases c. Lumbar approach to kidney is preferred d. Renal artery aneurysm is common 167. All except one are correct regarding renal trauma: a. Observation is best (AIIMS June 95) b. IVP is indicated c. Exploration indicated in all cases d. Hematuria is a cardinal sign 168. Which does not happen in unilateral renal trauma? a. Hypertension b. Uremia (AIIMS 92) c. Clot formation d. Perinephric hematoma

Kidney and Ureter

170. Renal trauma is best treated by: a. Observation and supportive measures b. Early drainage of perirenal hematoma c. Heminephrectomy d. Nephrostomy

(UPSC 98)

171. Forty eight after sustaining a blunt abdominal injury, a 15-year-old by presents with hematuria and pain in the left side of abdomen. On examination, he has a pulse rate of 96/ minute with a BP of 110/70 mm Hg. His Hb is 10.8 gm% with a PCV of 31%. Abdominal examination revealed tenderness in left lumbar region but no palpable mass. The most appropriate investigation to diagnose and find the extent of renal injury would be: (UPSC 2005) a. Sonographic evaluation of abdomen b. Intravenous pyelography c. Contrast enhanced computed tomography d. MR urography 172. What percent of cases with injury to kidney require surgical exploration? (MAHE 2008) a. 20% b. 90% c. 50% d. 70% 173. During renal rupture the nephrectomy is not attempted until: (UPPG 2010) a. Fluid replacement b. Antibiotics covers c. Contralateral renal function is ascertained d. Renal angiogram

175. Number of grades of blunt trauma kidney by C.T. scan are: a. 3 b. 4 (Orissa 2011) c. 5 d. 6

URETERIC INJURY 179. Inadvertent surgical injury of the ureter leads to: a. Complete renal atrophy b. Hematuria c. Renal failure d. Hydronephrosis e. Hypertension 180. Commonest cause of ureteric injury during surgical operation is: (UPPG 2007, 2006) a. Abdominoperineal resection b. Hysterectomy c. Prostatectomy d. Colectomy

POLYCYSTIC KIDNEY DISEASE 181. All of the following are features of adult polycystic kidney disease except: (COMEDK 2005) a. Autosomal recessive trait b. Present as renal mass c. Haematuria d. Renal failure 182. Polycystic kidney disease is associated with all of the following except: (COMEDK 2010) a. Cerebral aneurysms b. Mitral valve prolapsed c. Renal cell carcinoma d. Hepatic cysts 183. Not true about polycystic kidney disease is: (AIIMS Nov 97) a. Autosomal dominant b. Proteinuria < 2 gm/day c. Leads to CRF d. Decompression of cyst leads to normal renal function 184. Image of kidney in a patient having polycystin 2 mutation is given below. What is the inheritance mode in this disease?  (Recent Question 2016) a. Autosomal dominant b. Autosomal recessive c. X-linked dominant d. X-linked recessive

176. Absolute indication for surgical exploration after renal trauma? (MHSSMCET 2008) a. Hematuria b. Pulsatile hematoma c. Cortical renal contusion d. Delayed arterial injury 177. A 25-year-old male presents to emergency with history of road traffic accident two hours ago. The patient is hemodynamically stable. Abdomen is soft. On catheterization of the bladder, hematuria is noticed. The next step in the management should be: (Recdent Question 2016, AIIMS Nov 2004) a. Immediate laparotomy b. Retrograde cystouretherography (RGU) c. Diagnostic peritoneal lavage (DPL) d. Contrast enhanced computed tomography (CECT) of abdomen

185. In adult polycystic kidney, all are true except: a. Hypertension is rare (AIIMS June 2001) b. Hematuria is a common symptom c. Cysts are seen in liver spleen and pancreas d. Autosomal dominant transmission is seen

Urology

174. In renal trauma, which statement is not correct? a. Exploration is indicated in 90% of cases (Orissa 2011) b. Hematuria is a cardinal sign c. Transperitoneal approach is preferred d. IVP is urgently indicated

178. Following a blunt trauma abdomen, a patient had renal laceration and urinoma. Even after 12 days, urinoma persisted, but patient was stable and there was no fever. Next step in management would be: (AIIMS November 2017) a. Percutaneous exploration and repair b. Wait and watch c. J-shaped urinary stent d. Percutaneous nephrostomy

Section 4

169. After RTA, a young male presented with non-pulsatile retroperitoneal hematoma. On table IVU was done. Right kidney was not visualized. Left kidney showed immediate excretion of dye. What is next stap in the management? a. Nephrectomy (AIIMS Nov 2011) b. Open Gerotas fascia and explore proximal renal vessels c. Perform retrograde pyelography d. Perform on table angiography

601

Urology

602

Surgery Essence 186. True of autosomal dominant polycystic kidney disease I and II respectively: (PGI June 2002) a. Chromosomes 16 and 5 b. 16 and 4 c. 11 and 5 d. 11 and 4 e. 4 and 5 187. Polycystic kidneys can be associated with: a. Cysts in liver b. Coarctation of aorta c. Berry aneurysms d. All 188. Polycystic kidney may be associated with cyst in all the sites except: (Bihar PG 2014, All India 91) a. Lung b. Liver c. Pancreas d. Brain 189. Treatment of choice in polycystic kidney is: (Kerala 91) a. Removal of cyst b. Nephrectomy c. Dialysis d. Renal transplant 190. The incidence of liver cysts in childhood polycystic kidney disease is: (All India 92) a. 5% b. 10% c. 18% d. 50% 191. What is the diagnosis on the basis of CECT findings? a. ADPKD b. Bilateral RCC with multiple liver metastasis c. Multiple secondaries with unknown primary d. HCC with multiple metastasis to kidneys

194. The typical appearance of “spider leg” on excretory urography is seen in: (Recent Question 2016, UPSC 2008) a. Hydronephrosis b. Polycystic kidney c. Medullary sponge kidney d. Renal cell carcinoma 195. All are true about polycystic kidney except: (UPPG 2009) a. Inherited as autosomal recessive b. Hypertension and hematuria are common symptoms c. Spider leg deformity d. Associated with cysts in the liver and spleen 196. True about adult polycystic kidney disease is all except: a. Autosomal dominant inheritance (AIIMS 2001) b. Hypertension is rare c. Can be associated with cysts in liver, lungs and pancreas d. Pyelonephritis is common 197. Which of the following is the common extrarenal involvement in autosomal dominant polycystic kidney disease? a. Mitral valve prolapse (AIIMS Nov 2004) b. Hepatic cysts c. Splenic cysts d. Colonic diverticulosis 198. Which one of the following statements is wrong regarding adult polycystic kidney disease? (AIIMS May 2004) a. Kidneys are enlarged in size b. The presentation is unilateral c. Intracranial aneurysms may be associated d. Typically manifests in the 3rd decade 199. All of the following are true about childhood polycystic kidney disease, except: a. Autosomal dominant b. Pulmonary hypoplasia may be seen c. Renal cysts are present at birth d. Congenital hepatic fibrosis may be seen

192. Polycystic disease of the kidney may have cysts in all the following organs except: (All India 2004) a. Lungs b. Liver c. Pancreas d. Spleen

Section 4

193. IVP was done in the patient of ADPKD. What is the name of this sign? (Recent Question 2016) a. Spider leg appearance b. Swiss cheese appearance c. Bubble appearance d. Bristles of brush appearance

200. All the following are features of polycystic disease of kidneys except: (MCI June 2018) a. Hematuria b. Hypertension c. Renal failure d. Erythrocytosis

PUJ OBSTRUCTION 201. Not true about congenital PUJ obstruction is: a. Can be associated with renal agenesis (AIIMS Nov 2001) b. Can be diagnosed antenatally c. Bilateral in 10–15% of cases d. Aberrant vessel is the most common cause 202. All are true in PUJO except: a. Commoner in boys b. Bilateral lesions occur in 10-40% c. Right sided lesions predominate d. Intrinsic lesions predominate 203. Most infants and children with PUJO present with: a. Pain (GB Pant 2008) b. Hematuria c. Painless abdominal mass d. Renal failure 204. Best management for a symptomatic 6 years male with PUJ obstruction: (GB Pant 2010) a. Endopyelotomy b. Foley V-Y pyeloplasty c. Dismembered pyeloplasty d. Wait and watch

Kidney and Ureter

206. This surgery is performed for which of the following condition? a. Carcinoma renal pelvis b. PUJ obstruction c. Ureteric strictures d. VUR

207. Investigation of choice for documentation of obstructive nature of pelvicalyceal system dilatation: (Recent Questions 2017) a. IVP b. DTPA scan c. Whittaker test d. Ultrasound 208. Not true about PUJ obstruction is: (AIIMS Nov 2001) a. Retrograde pyelography is useful to locate the site of obstruction b. Endoscopic pyelotomy is contraindicated c. Whittakar test is of clinical significance d. Dismembered pyeloplasty is the procedure of choice 209. Distention of abdomen with passage of large amount of urine is known as: (MHPGMCET 2001) a. Dietl’s crisis b. Anderson-Hynes crises c. Meteriorism d. Strangury

CONGENITAL ANOMALIES OF KIDNEY 210. Potter facies and oligohydramnios are pathognomic of: a. Bilateral renal agenesis b. Unilateral renal agenesis c. bilateral renal disease d. Unilateral cystic disease

212. A symptom of medullary sponge kidney disease is: a. Nocturia b. Anemia (All India 95) c. Azotemia d. UTI 213. IVP was done in a patient having hypercalcemia and nephrolithiasis. What is the diagnosis based on IVP findings? a. ADPKD b. ARPKD c. Medullary sponge kidney d. Congenital cystic nephroma

218. An absent kidney is found in: (AMU 2005) a. 1:200 individuals b. 1:700 individuals c. 1:1400 individuals d. 1:5000 individuals 219. Medullary cystic disease of the kidney is best diagnosed by: a. Ultrasound b. Nuclear scan c. Urography d. Biopsy

ABERRANT RENAL ARTERY 220. Aberrant renal artery, all true except: a. More common in women b. Usually towards left c. May cause hydronephrosis d. Usually divided to gain access to renal pelvis 221. All are true of aberrant renal artery except: (PGI 93) a. Bilateral b. Leads to hydronephrosis c. Common in females d. More common on left side

RETROCAVAL URETER 222. ‘Reverse J’ deformity on IVP is seen in: a. Congenital megaureter b. Ureterocele c. Retrocaval ureter d. VUR 223. Given radiological sign is seen in: a. PUJ obstruction b. Aberrant left renal artery c. Retrocaval ureter d. Retroperitoneal fibrosis

Urology

211. Potter’s facies is characterized by: a. Hyperteleorism b. Prominet inner canthus c. Recessive chin d. Low set ears e. All are true

214. Which of the following is the most common renal vascular anomaly? (All India 2010) a. Supernumerary renal arteries b. Supernumerary renal veins c. Double renal arteries d. Double renal veins 215. Renal collar which surrounds the aorta has its two limbs split by: (All India 99) a. Left renal vein b. Left renal artery c. Isthmus of horseshoe kidney d. All of the above 216. Persistent fetal lobulation of adult kidney is due to: a. Congenital renal defect  (AIIMS Nov 2007) b. Obstructive uropathy c. Intrauterine infections and scar d. Is a normal variant 217. Incidence of Renal ectopia is: (All India 92) a. 1:100000 b. 1:75000 c. 1:10,000 d. 1:1000

Section 4

205. Anderson-Hynes operation is performed for: a. Achalasia cardia (MCI June 2018) b. Pyloric stenosis c. Pseudopancreatic cyst d. Pelviureteric junction obstruction

603

604

Surgery Essence 224. Retrocaval ureter occurs due to persistence of: a. Azygous vein (Recent Question 2016) b. Hemiazygous c. Anterior cardinal vein d. Posterior cardinal vein

236. This characteristic appearance is seen on IVP in:  (Recent Question 2019, 2017) a. Tuberculosis b. VUR c. Ureterocele d. Ureteric stone

HORSESHOE KIDNEY

Urology

225. Which is false of Horseshoe kidney? a. Spider like appearance in IVP b. Ureteral obstruction common c. Lower calyx is reversed d. Heminephrectomy improves function

(AIIMS 92)

226. Isthmus of horses is located at what level? (Recent Questions 2017) a. L1-L2 vertebra b. L3-L4 vertebra c. L4-L5 vertebra d. L2-L3 vertebra 227. “Hand joining sign” and ‘Flower vase’ pattern of uteters is characteristic of: (Recent Questions 2017) a. Sigmoid kidney b. Horseshoe kidney c. Crossed ectopia d. L-shaped kidney 228. With regard to horse-shoe kidneys, true is: a. Usually symptomatic b. Most cases require surgery c. Division of isthmus is usually required to ensure adequate dependent drainage d. Isthmus may contain aberrant vessels

URETERIC ABNORMALITIES 237. Ectopic ureter opening is not located in: (MAHE 2005, AIIMS Nov 98) a. Bulbar urethra b. Prostatic urethra c. Seminal vesicle d. Bladder neck 238. What is the diagnosis based on IVP findings? a. VUR b. ADPKD c. Ectopic ureteric orifice d. Ureterocele

229. Horseshoe kidney ascent is prevented by: a. Superior mesenteric artery (Recent Question 2017) b. Superior mesenteric vein c. Inferior mesenteric artery d. Inferior mesenteric vein

RENAL CYST 230. Spider leg appearance in IVP is suggestive of: a. Renal cyst b. Renal carcinoma c. Renal Tb d. Hydronephrosis e. Chronic renal failure 231. Which of the following is the most common renal cystic disease in infants is? a. Polycystic kidney b. Simple renal cyst c. Unilateral renal dysplasia d. Calyceal cyst

Section 4

URETEROCELE 232. Cobra head appearance on excretory urography is suggestive of: (MCI March 2010) a. Horseshoe kidney b. Duplication of renal pelvis c. Simple cyst of kidney d. Ureterocele 233. Treatment of choice for ureterocele? (MHSSMCET 2009) a. DJ stent b. Laparoscopic repair c. LASER ablation d. Endoscopic diathermy 234. Adder head appearance on IVP is/are seen in: (PGI Nov 2011) a. Polycystic kidney b. Ureterocele c. Horseshoe kidney d. Hydronephrosis e. Ectopic ureter 235. A 3-year-old girl presents with recurrent UTI. On USG shows hydronephrosis with filling defect and negative shadow of bladder with no ectopic orifice: (UPPG 2004) a. Vesicoureteric reflux b. Hydronephrosis c. Ureterocele d. Sacrococcygeal teratoma

239. Most common congenital anomaly of the upper renal tract is: (Recent Questions 2017) a. Duplication of renal pelvis b. Duplication of ureter c. Ectopic ureteric orifice d. Congenital megaureter 240. On the basis of given IVP image, what is the most probable diagnosis? (Recent Question 2016) a. Bilateral duplication of ureter 
 b. Right bifid and left complete duplication 
of ureter c. Right bifid and left incomplete duplication of ureter d. Left bifid and right complete duplication of ureter

Kidney and Ureter

242. Ectopic ureter may be frequently associated with: a. Oliguria b. Dysuria c. Bilateral hydroureter d. Paradoxical incontinence 243. True statement about duplex draining system of urinary tract are all except: (Recent Question 2015, Punjab 2007) a. It is most common anomaly of the upper urinary tract b. Upper moiety drains lower in the bladder c. Lower pole moiety is more prone to obstruction and upper pole more prone to reflux d. Yo-Yo Reflux may occur if ureters get fused 244. Yo-Yo reflux is seen in: (Recent Question 2017) a. Duplication of ureter b. Polycystic kidney disease c. Medullary sponge kidney d. Ureterocele 245. Given diagram represent which rule? a. Weigert-Meyer’s rule b. Pascal’s rule c. Lambert’s rule d. Beer’s rule

250. In the female, the most common site of termination of the ectopic ureter is the: (Recent Question 2016) a. Vestibule b. Fallopian tube c. Ovary d. Uterus 251. Presentation and evaluation of ectopic ureter, all are true except: a. In males there is duplication in 80% of cases b. Can present as abdominal mass or HDN on antenatal USG c. IVP will usually show a functioning but dilated upper pole segment and identify the site of ureteral opening d. The lower pole is seen as the classic ‘drooping lily’ sign

Section 4

241. A 9 years old boy presented with abdominal pain and recurrent UTI, IVP reveals duplication of left ureter. The most likely site of ectopic opening would be: (All India 99) a. Prostatic urethra b. Ejaculatory duct c. Seminal vesicle d. Vas deference

605

252. Classic symptom of ectopic ureter in females: a. Painful defecation (Recent Question 2016) b. Urinary frequency c. Ureteral incontinence with otherwise normal voiding d. Labial swelling 253. MC presentation of ectopic ureter in males: a. Recurrent UTI b. Epididymitis c. Pelvic pain d. Seminal vesicle infections

VESICOURETERIC REFLUX 254. In case of vesicoureteric reflux which will be investigation of choice: (AIIMS Nov 98) a. Micturating cystourethrogram b. IVP c. Cystography d. Radionuclide study 255. In a patient suspected to be suffering from vesicoureteric reflex, which one of the following radiological investigations may confirm the diagnosis? (UPSC 2007) a. Intravenous urography b. Micturating cystourethrography c. Pelvic ultrasound d. Antegrade pyelography

247. In a patient with complete ureteral duplication, the opening of ureter draining the upper pole of the kidney is: a. Above and medial to the lower pole ureter b. Above and lateral to the lower pole ureter c. Below and medial to the lower pole ureter d. Below and lateral to the lower pole ureter e. Always lateral to lower pole ureter 248. According to Weigert-Meyer’s rule of duplication of ureter, the lower pole ureter in urinary bladder is:  (JIPMER November 2017) a. Lateral and cephalad to the upper pole ureter b. Lateral and caudal to the upper pole ureter c. Medial and cephalad to the upper pole ureter d. Medial and caudal to the upper pole ureter 249. In the male, the most common site of termination of the ectopic ureter is the: a. Posterior urethra b. Anterior urethra c. Seminal vesicles d. Vas deference

257. A 5-year-old child was brought with history of recurrent UTI, fever and abdominal pain. MCU was done. What is the diagnosis based on the given image? a. Grade II VUR b. Grade III VUR c. Grade IV VUR d. Grade V VUR

Urology

246. Weigert-Meyer’s rule applies to: a. Fusional anomalies b. Renal dysplasia c. Polycystic kidney disease d. Ureteral duplications

256. Reflux into pelvis and calyces without dilatation: a. I b. II c. IV d. IV

606

Surgery Essence 258. Treatment of choice for grade IV vesicoureteric reflux with recurrent UTI: (AIIMS June 2000) a. Cotrimoxazole b. Bilateral reimplantation of ureter c. Injection of collagen in the ureter d. Endoscopic resection of ureter

Urology

259. Which of the following statements is true of primary grade IV-V vesicoureteric reflux in young children? (AIIMS May 2006) a. Renal scarring usually begins in the midpolar regions b. Postnatal scarring may occur even in the absence of urinary tract infections c. Long-term outcome is comparable in patients treated with either antibiotic prophylaxis or surgery d. Oral amoxicillin is the choice antibiotic for prophylaxis 260. The most common cause of renal scarring in a 3 years old child is: a. Trauma b. Tuberculosis c. VUR induced pyelonephritis d. Interstitial nephritis

RENAL ARTERY ANEURYSM 261. The risk of rupture in renal artery aneurysms is: a. Less than 1% b. 5% c. 20% d. 75% e. None of the above

HEPATORENAL SYNDROME 262. Features of hepatorenal syndrome are: (PGI June 2006) a. Urine sodium < 10 meq/L b. Normal renal histology c. Renal functional abnormal even after liver become normal d. Proteinuria 263. Which of the following statements are incorrect with regard to hepatorenal syndrome in a patient with cirrhosis? a. Creatinine clearance 40 ml/min (All India 2003) b. Urinary sodium < 10 mEq/L c. Urine osmolality lower than plasma osmolality d. No sustained improvement in renal function after volume expansion 264. Which of the following statement is incorrect with regard to hepatorenal syndrome in a patient with cirrhosis? a. The creatinine clearance is > 40 ml/min b. The urinary sodium is < 10 mmol/L c. The urine osmolality is lower than the plasma osmolality d. There is poor response to volume expansion

Section 4

DIALYSIS 265. The following are the complications of hemodialysis except: a. Hypotension b. Peritonitis c. Hypertension d. Bleeding tendency

RENAL TRANSPLANT 266. First autologous renal transplantation was done: a. Hardy b. Kavosis (All India 2010) c. Higgins d. Studor 267. All of the following statements are correct about renal transplantation except: (AIIMS Nov 2004) a. Renal transplantation is heterotopic b. Cyclosporine is the mainstay of immuno-suppression

c. In India, organ harvesting from brain dead patients is not permitted by law d. Kidney after removal is flushed with cold perfusion solution 268. After renal transplant, the commonest malignancy is:  (AIIMS June 97) a. Lymphoma b. Renal cell carcinoma c. Skin cancer d. Adrenal cancer 269. Not true about right kidney is: (DNB 2003, AIIMS June 2001) a. Right kidney is preferred over the left for transplantation b. It is lower than the left kidney c. Right renal vein is shorter than the left d. Right kidney is related to the duodenum 270. After renal transplantation, which drug is given?  (PGI June 96) a. Cyclophosphamide b. Corticosteroids c. Interferon d. Cyclosporine 271. Urinary ascites is due to: (PGI Dec 98) a. Injury to bladder during birth b. Ureteric obstruction c. Congenital urethral atresia d. Urethral valves 272. A newborn presents with discharge of urine from the umbilicus for 3 days. Diagnosis is: a. Meckel’s diverticulum b. Mesenteric cysts c. Urachal fistula d. Omphalocele

KIDNEY AND URETER ANATOMY AND PHYSIOLOGY 273. Ureters are identified during surgery by: a. Peristalsis due to flow of urine b. Rich arterial plexus c. Relation to renal vein and artery d. Relation to lumbar plexus

(AIIMS 96)

274. All of the following structures cross the right ureter anatomically except: (All India 2012) a. Terminal ileum b. Vas deferens c. Genitofemoral nerve d. Right colic and ileocolic vessels 275. Ureteric construction is seen at all the following positions, except: (All India 2002) a. Ureteropelvic junction b. Ureterovesicle junction c. Crossing of iliac artery d. Ischial spine 276. The narrowest part of the ureter is at the: (AIIMS Nov 2005) a. Ureteropelvic junction b. Iliac vessel crossing c. Pelvic ureter d. Ureterovesicle junction 277. All the following are true regarding blood supply to the kidney, except: (All India 2002) a. Stellate veins drain superficial zone b. It is a type of portal-circulation c. The renal artery divides into five segmental arteries before entering the hilum d. Its segmental arteries are end-arteries 278. The commonest site of surgical ureterovaginal fistula is: a. Below infudibulopelvic ligament (PGI 99) b. Below uterine artery in the Mackenrodts ligament c. Vaginal angle d. Above uterine artery

Kidney and Ureter

280. What is column of Bertini in kidney? a. Renal tumour b. Tongue like papillary projection c. Calculus d. None

(APPG 2008)

281. Ureter is diagnosed during operation by: (All India 98) a. Venous plexus b. High arterial supply c. Peristaltic movements d. Circumference 282. The resting ureteric pressure: (PGI June 99) a. 5–7 cm of H2O b. 15–30 cm of H2O c. 7–10 cm of H2O d. 0–5 cm of H2O

283. Left loin nephrectomy, structure not cut is: (PGI Dec 98) a. Trapezius b. Serratus inferior posterior c. Latissimus Dorsi d. Internal oblique 284. Unilateral small smooth kidney is seen in: (Karnataka 2003) a. Reflux nephropathy b. Lobar infarction c. Renal artery stenosis d. Chronic glomerulonephritis 285. The neonatal kidney achieves concentrating ability equivalent to adult’s kidney by: a. One year of age b. Eighteen months of age c. Three to six months of age d. Just before puberty

MISCELLANEOUS 286. A 60 years old male presents with hematuria at onset of micturition, cause is: (AIIMS June 99) a. Urethral stone b. Bladder tumor c. Ureteric stone d. Prostatitis 287. A 23 years old male who is otherwise normal complains of mild pain in his right iliac fossa in a waveform pattern which increases during the right and he becomes exhausted and is admitted in the hospital. On examination there is mild hematuria. Urine examination reveals plenty of RBCs, 50WBCs/hpf. Urine pH is 5.5. Most likely diagnosis is: b. CA urinary bladder d. Cystitis

289. Normal capacity of the renal pelvis is: a. 7 mL b. 10 mL c. 15 mL d. 20 mL 290. Low and fixed specific gravity of urine is seen in: a. Diabetes mellitus (SGPGI 2005) b. Diabetes insipidus c. Chronic renal failure d. Acute glomerulonephritis 291. Auto nephrectomy is seen in: a. Sickle cell anemia b. Renal TB c. Sarcoidosis d. Lymphoma

(JIPMER 95)

292. Urine incontinence is seen in all except: a. Ureterovaginal fistula (MHPGMCET 2003) b. Vesicovaginal fistula c. Ectopic ureter d. Urerthrovaginal fistula 293. If due to a certain pathology there is marked loss of ureter, then what is best therapeutic options? (MHSSMCET 2005) a. Transureterostomy b. Psoas Hitch operation c. Intestinal segment substitute d. Boari’s flap operation 294. According to American Urology Association, definition of microscopic hematuria: (Recent Question 2016) a. >3 RBCs/hpf b. >20 RBCs/hpf c. >50 RBCs/hpf d. >100 RBCs/hpf 295. Sickle cell trait is associated with which type of RCC?  (JIPMER November 2017) a. Medullary b. Papillary c. Chromophobe d. Clear cell

Urology

a. Glomerulonephritis c. Ureteral calculus

288. What is oliguria? (MCI June 2018, Recent Question 2015) a. Excretion of less than 300 ml in 24 hours b. Excretion of less than 500 ml in 24 hours c. Excretion of less than 300 ml in 12 hours d. Excretion of less than 100 ml in 24 hours

Section 4

279. True about ureter is: (AIIMS 91) a. Gonadal vessels lie anterior to it b. It lies in front of great vessels c. About 50 cm long d. Nerve supply from T8-T10 e. Internal iliac artery

607

Explanations RENAL AND URETERIC CALCULI

1. Ans. b. Triple phosphate (Ref: Smith 18/e p255; Campbell 11/e p1182-1196; Bailey 27/e p1406)



2. Ans. c. Indinavir



4. Ans. b. Urinary stones (Ref: Campbell 11/e p1175)

3. Ans. d. Usually seen in acidic urine

Randall’s Plaques calcificationsQ

• Randall’s plaques are soft tissue found in the deep renal medulla skirting the surface of the epithelium of the papilla, where they act as nucleating elements for renal calculi or stonesQ.

6. Ans. a. Ethylene glycol (Ref: Smith 18/e p254)

5. Ans. a. Phosphate

Calcium Oxalate Crystals • Calcium oxalate crystals in the urine are the most common constituent of human kidney stones, and calcium oxalate crystal formation is also one of the toxic effects of ethylene glycol poisoningQ. • Excessive oxalate may occur secondary to the accidental or deliberate ingestion of ethylene glycol (partial oxidation to oxalateQ. This may result in diffuse and massive deposition of calcium oxalate crystals and may occasionally lead to renal failure.

7. Ans. b. Phosphate

8. Ans. b. Calcium oxalate

9. Ans. a. Calcium oxalate stone (Ref: Campbell 11/e p1209; Smith 18/e p252; Bailey 27/e p1406) 10. Ans. a. Calcium phosphate (Ref: Campbell 11/e p1237; Smith 18/e p251)

11. Ans. c. Ammonium urate (Ref: Campbell 11/e p1195, 1196)

“Conditions associated with ammonium acid urate crystallization include laxative abuse, recurrent urinary tract infection, recurrent uric acid stone formation, and inflammatory bowel disease.”-Campbell 11/e p1195 “The underlying pathophysiologic mechanism of ammonium acid urate stone formation due to laxative abuse has been postulated to be the result of dehydration due to gastro-intestinal fluid loss causing intracellular acidosis and enhanced ammonia excretion. Because urinary sodium is low in the setting of laxative use, urate complexes with abundant ammonia, thereby leading to urinary supersaturation of ammonium acid urate.”- Campbell 11/e p1196

12. Ans. b. Proteus (Ref: Campbell 11/e p1194; Smith 18/e p255; Bailey 27/e p1406)



13. Ans. d. Both a and b (Ref: Campbell 11/e p1213; Smith 18/e p255; Bailey 27/e p1406)

RENAL AND URETERIC CALCULI CLINICAL FEATURES

14. Ans. a. At pelvic brim (Ref: Smith 18/e p257; Campbell 1/e p1-2; Bailey 27/e p1407)



15. Ans. b. At pelvic brim

16. Ans. c. Dietl’s crisis (Ref: Bailey 27/e p1411)

Dietl’s Crisis • Intermittent hydronephrosis (Dietl’s crisis): A swelling in the loin is associated with acute renal pain. Some hours later the pain is relieved and the swelling disappears when a large volume of urine is passedQ.

17. Ans. b. Increased peristalsis of ureter to overcome the obstruction (Ref: Smiths 18/e p257) • The severity and colicky nature of ureteric colic pain are caused by the hyperperistalsis and spasm of smooth muscles of the ureter as it attempts to rid itself of a foreign body or to overcome obstruction.

Pain from Acute Obstruction of Ureter (Stone or Blood clot) • Ureteral pain is typically stimulated by acute obstruction (passage of a stone or a blood clot)Q. • Back pain from renal capsular distentionQ combined with severe colicky pain (due to renal pelvic and ureteral muscle spasmQ that radiates from the costovertebral angle down toward the lower anterior abdominal quadrant, along the course of the ureter. • The severity and colicky nature of this pain are caused by the hyperperistalsisQ and spasmQ of this smooth muscle organ as it attempts to rid itself of a foreign body or to overcome obstruction.

Kidney and Ureter

18. Ans. b. Proteus



21. Ans. a. Oxalate stones (Ref: Bailey 27/e p1406)

19. Ans. b. Iliohypogastric

20. Ans. c. Pain

• Oxalate stones are irregular in shape and covered with sharp projections, which tend to cause bleedingQ. • The surface of the calculus is discolored by altered bloodQ. • A calcium oxalate monohydrate stone is hard and radiodenseQ. 22. Ans. b. Stillness of the patient

23. Ans. d. Diclofenac

Section 4

Oxalate Stones



609

RENAL AND URETERIC CALCULI DIAGNOSIS AND TREATMENT

24. Ans. a. PCNL (Ref: Smith 18/e p272; Campbell 11/e p1236; Bailey 27/e p1409)



26. Ans. a. ESWL




28. Ans. a. ESWL (Ref: Campbell 11/e p1236, 1247, 1251; Bailey 27/e p1408)

25. Ans. b. Uric acid

27. Ans. b. PCNL

The best treatment in this situation is PCNL. Since PCNL is not mentioned in the option, the best option is ESWL despite of size 2.5 cm, as it is preferred over other three for the management of renal stones.

29. Ans. a. Calcium oxalate monohydrate (Ref: Smith 18/e p268; Campbell 11/e p1268; Bailey 27/e p1408) • “Calculi composed of cystine, callium oxalate monohydrate are known to be resistant to fragmentation (ESWL)”



30. Ans. c. Struvite stone (Ref: Smith 18/e p255; Campbell 11/e p1182-1196; Bailey 25/e p1295-1300)



31. Ans. b. ‘j’ stent drainage (Ref: ‘Acute Care Surgery: Principles and Practice’ (Springer) 2007/571) • Prompt drainage of hydronephrosis by ‘J’ stent drainage is the single best option to manage uremia in this patient with bilateral renal calculi, to allow recovery of renal function at the earliestQ. • Hemodialysis may be used afterwards if renal recovery is prolonged necessitating removal of waste productsQ • Prompt drainage of hydronephrosis by ‘J’ stent placement is the procedure of choice for hydronephrosis complicated with renal failure in the setting of urinary obstructionQ. • Prompt drainage of hydronephrosis is indicated of renal function is compromised or urinary infection (UTI) is suspected, to preserve/salvage renal function. • Prompt drainage can be achieved by placement of a ureteral sent or through percutaneous nephrostomyQ.



32. Ans. a. Hydrothorax (Ref: Smith 18/e p272; Campbell 11/e p1282; Bailey 27/e p1409) • PCNL done through the 11th intercostals space traverses the lower aspect of pleura and can result in significant hydrothorax from large amount of irrigative fluid.



33. Ans. d. Stone in a calyceal diverticulum (Ref: Smith 17/e p264-268; Campbell 11/e p1278, 10/e p1380-1381)



34. Ans. d. Ureteroscopic retrieval (Ref: Smith 18/e p272; Campbell 11e p1283; Bailey 27/e p1408)



35. Ans. a. Endoscopic removal (Ref: Campbell 11/e p1283; Bailey 27/e p1408)



36. Ans. b. Calcium oxalate, uric acid, struvite, cystine (Ref: Smith 18/e p 255; Campbell 11/e p1182-1196; Bailey 27/e p1406)



37. Ans. d. Ureteric obstruction due to fragments in ureter



38. Ans. c. Struvite crystals



39. Ans. b. Its use for uric acid stones has caused deaths due to generation of cyanide (Ref: Smith 18/e p167; Campbell 11/e p1262)



40. Ans. a. Cysteamine (Ref: Harrison 19/e p1871; Smith 18/e p256; Campbell 11/e p1229) • Patient with cystinuria with multiple renal stones should be treated with increase urine volume (high fluid intake), alkalinization of urine, Penicillamine and tiopronin. (a-Mercaptopropionyl glycine).

Urology

• The drainage procedure of choice in emergent situations is cystoscopy with placement of internalized double ‘J’ ureteral stent. This procedure has the advantage of being a completely internal drainage systemQ. • Percutaneous nephrostomy may be used to allow urinary drainage if the stone is too impacted to allow passage of a guide wire for sent placementQ

610

Surgery Essence

41. Ans. e. 90% (Ref: Smith 18/e p265)

Imaging of Ureteric Colic (Calculus) • A plain film of the abdomen and renal ultrasound examination will diagnose most stonesQ. • Spiral CT (Non-contrast CT)Q has become the study of choice in emergent situations, as the entire urinary tract can be scanned rapidly and without contrast injectionQ. • Calculi can be readily identified and distinguished from clot or tumorQ.

Urology

• About of 90% of calculi are radiopaqueQ (calcium, cystine). • Excretory urography is necessary to verify their location within the urinary tract and also affords a qualitative measure of renal function. • An acutely obstructed kidney may show only increasing density of renal shadow without significant radiopaque material in calices. • A non-opaque stone (uric acid) will be seen as a radiolucent defect in the opaque contrast media.

42. Ans. d. Lower 1/3rd of ureter



43. Ans. a. Urine is always infected, b. Should be removed immediately

44. Ans. d. Calcium oxalate stone (Ref: Smith 18/e p255; Campbell 11/e p1182-1196, 10/e p1296-1302; Bailey 25/e p1295-1300) In the given image, which shows enveloped or bipyramidal crystals are seen in calcium oxalate (dihydrate) stones. Calcium Oxalate

Calcium Oxalate Monohydrate

Brushite

Enveloped or bipyramidalQ

Dumbbell or hourglassQ

Needle shapedQ

Struvite

Section 4

Coffin

lidQ

Uric Acid

Cystine

Multifaceted, irregular plates or rosettesQ

Hexagonal or benzene ringQ



45. Ans. c. Uric acid



47. Ans. c. High calcium intake

46. Ans. a. ESWL



48. Ans. d. Uric acid stones are resistant to ESWL



49. Ans. b. Phosphate



50. Ans. c. URS (Ref: Sabiston 20/e p2071; Schwartz 10/e p1666; Bailey 27/e p1408-1409)



51. Ans. a. Endoscopic removal



52. Ans. c. Renal stones (Ref: Bailey 27/e p1388)

Kidney and Ureter

611

X-Ray KUB

Opacities on a plain X-ray that may be confused with renal calculus • • • •

Calcified mesenteric LN Gallstones or concretion in the appendix Tablets or foreign bodies in the alimentary canal (e.g. cyclopenthiazide) Ossified tip of the 12th rib

• Phleboliths: calcification in the walls of veins, especially in the pelvis • Calcified tuberculous lesion in the kidney • Calcified adrenal gland

Section 4

• Kidney stones should be looked opposite to second lumbar vertebraQ. • In a lateral X-ray of abdomen gallstones are anterior and renal and ureteric stones overlie the lumbar spineQ.

53. Ans. d. Restricted calcium intake (Ref: Harrison 19/e p1870; Smith 18/e p253) • A source of calcium at each meal may actually help prevent oxalate stones from forming as the calcium binds with oxalate in food and thus prevents the oxalate from being absorbed into the bodyQ. Dietary Modification in Stone Disease Increase Intake of • • • •

FluidQ Dietary calciumQ Potassium and phytatesQ Vitamin CQ

Decrease Intake of • • • • •

OxalateQ Animal proteinQ Sucrose Fructose Sodium



54. Ans. a. Uric acid stones



55. Ans. b. DJ stent after ESWL (Ref: Sabiston 20/e p2114; Schwartz 10/e p1666; Bailey 27/e p1408)



56. Ans. a. Severity of pain increases with size of the calculus (Ref: Bailey 27/e p1407) • The severity of pain is not related to the size of the stone.

Ureteric Colic



There is a pattern of severe exacerbation on a background of continuing pain Radiates to the groin, penis, scrotum or labium as the stone progresses down the ureter The severity of pain is not related to the size of the stoneQ The pain is almost invariably associated with haematuriaQ There may be few physical signs

57. a. Stone • MC cause of ureteric obstruction: StoneQ • MC cause of ureteric colic in hematuria: ClotQ



58. Ans. a. Holmium



60. Ans. b. Calcium restriction (Ref: Smith 18/e p253)

59. Ans. a. Bleeding is least concerned complication



61. Ans. b. PCNL (Ref: Campbell 11/e p1240; Smith 18/e p272; Bailey 27/e p1409)

RENAL INFECTIONS

62. Ans. a. E. coli (Ref: Smith 18/e p206; Campbell 11/e p279-280)



63. Ans. b. Xanthogranulomatous kidney (Ref: Smith 18/e p208; Campbell 11/e p287)



64. Ans. b. Always unilateral (Ref: Smith 18/e p209; Campbell 11/e p283; Bailey 27/e p1412) • For HDN, DJ stenting and percutaneous nephrostomy, both are having same results but DJ stenting is less invasive. • For pyonephrosis, percutaneous nephrostomy is better than DJ stenting.

Urology

• • • • •

612

Surgery Essence

65. Ans. c. Pyonephrosis (Ref: P Modi, G Kadam, R Goel - Journal of Endourology, 2007) • A subcapsular nephrectomy (SN) is sometimes needed to successfully remove the kidney because of the dense perinephric adhesions in pyonephrosis.



66. Ans. c. Nitrite test (Ref: Smith 18/e p200) Sensitivity and Specificity of Urinalysis in UTI

Urology

Tests

Sensitivity (%)

Specificity (%)

Esterase

83 (67–94)

78 (64–92)

Nitrite

53 (15–82)

98 (90–100)Q

Esterase or Nitrite

93 (90–100)

72 (58–91)

White blood cells

73 (32–100)

81 (45–98)

Bacteria

81 (16–99)

83 (11–100)

Any above

99.8 (99–100)

70 (60–92)



67. Ans. a. Proteus



68. Ans. d. Vesicoureteric reflux (Ref: Bailey 27/e p1404) • Chronic pyelonephritis is so often associated with vesicoureteric reflux that some feel that it is better named “Reflux nephropathy”. It is important cause of renal damage and death from end-stage renal failure.

GENITOURINARY TUBERCULOSIS

69. Ans. b. Hematogenous (Ref: Smith 18/e p223-225; Campbell 11/e p422; Bailey 27/e p1405)



70. Ans. a. intravenous urography (Campbell 11/e p425) • IVU is the gold standard for imaging early renal TB. Intravenous urography: The majority of cases will show positive findings on excretary urography, the most common findings being hydrocalycosis, hydronephrosis or Hydroureter due to stricture formation. Early signs include the moth-eaten appearance of calyceal erosion and papillary irregularity. These signs are best seen on early excretary films because they are often masked by increasing density of the contrast on later films of the IVU. Cavitory lesions communicating with the collecting system are characteristic of TB.



71. Ans. b. Moth eaten calyx (Ref: Smith 18/e p224; Campbell 11/e p425; Bailey 27/e p1405)



72. Ans. a. Sterile pyuria is consistent finding



73. Ans. a. Renal papilla (Ref: Smith 18/e p223; Campbell 11/e p422; Bailey 27/e p1405)

Pathogenesis of Genito-urinary Tuberculosis • Granulomas at renal pyramid → Enlarge (tubercular abscess) → Burst into PC system and pus discharge in urine (sterile pyuriaQ. • Vesical irritability is an early clinical manifestationQ.

Section 4



74 Ans. c. Tuberculosis of ureter

Cystoscopy in Tuberculosis • Earliest sign is pallorQ around ureteric orifice. • Other features are tubercular ulcer and golf hole ureteric orificeQ.

75. Ans. a. Increased frequency



76. Ans. b. Bladder disease is associated with extensive renal disease, c. Ureteric involvement causes shortening of the ureters, d. Renal disease can produce changes identical to reflux nephropathy



77. Ans. b. Renal tuberculosis (Ref: Campbell 11/e p425; Smith 18/e p224; Bailey 27/e p1405)



78. Ans. d. Microscopic hematuria



80. Ans. d. Thimble bladder

79. Ans. a. Renal tuberculosis

Kidney and Ureter

613

HYDRONEPHROSIS 81. Ans. b. Analgesia SOS, c. Under observation, d. Follow up by USG (Ref: Bailey 27/e p1411)



82. Ans. d. Ureterocele



83. Ans. a. Catheterize with Foley catheter (Ref: Bailey 27/e p1412, 1349; CSDT 13/e p922) • Catheterization is mandatory for acute urinary retentionQ. Spontaneous voiding may return, but a catheter should be left indwelling for 3 days while detrusor tone returns.



84. Ans. b. Radioisotope scan



85. Ans. d. Rim sign 


Section 4



Chronic Hydronephrosis • Rim sign: On IVP, rim sign is seen due to chronic hydronephrosis (Atrophic changes in kidney with the dilatation of pelvicalyceal system). The inner margin of hydronephrotic rim in concave towards the renal hilum.

DIAGNOSTIC AND LABORATORY INVESTIGATIONS

86. Ans. c. Digital subtraction angiography (Ref: Bailey 27/e p1389) • Hypertension in a young female is most likely due to renal artery stenosis, caused by fibromuscular dysplasia. DSA will show the stenosis.

Renal Artery Stenosis • Satisfactory imaging of the renal vessels can even be achieved by digital subtraction angiographyQ after intravenous injection of contrast medium. • Intra-arterial angiography is considered “gold standard” for diagnosis of large vessel diseaseQ, usually performed simultaneous with planned intervention.

87. Ans. c. USG • Causes of filling defect on IVP: Stone, mass, cystQ. • After excretory urogram, next best investigation will be USG for lesion characterizationQ.



88. Ans. a. Allergy to the drug (Bailey 27/e p1389)

Intravenous Urography (IVU)



89. Ans. c. Distinguishing between non-obstructed system (Ref: Bailey 27/e p1392) • A 99mTc-DTPA scan is particularly useful to prove that collecting system dilatation is caused by obstruction.

Radioisotope Scanning • Radioisotope scanning is used in particular to obtain information about function in individual renal unitsQ. • Diethyltriaminepentaacetic acid (DTPA) behaves in the kidney like inulin: it is filtered by the glomeruli and not absorbed by the tubules. Using a gamma camera, DTPA labelled with technetium-99m can be followed during its transit through individual kidneys to give a dynamic representation of renal function. • A 99mTc-DTPA scan is particularly useful to prove that collecting system dilatation is caused by obstruction. In obstruction, radioactivity will remain in the kidney even if urine flow is stimulated by administration of a diuretic like furosemideQ. • Other substances [dimercaptosuccinic acid (DMSA), mercaptoacetylglycine (MAG-3) and sodium orthoiodohippurate (Hippuran)] labelled with suitable radioactive isotopes have similarly been used to investigate renal functionQ. • Isotope bone scanning is fundamental to the staging of kidney and prostate cancers, which typically metastasise to the skeletonQ.

90. Ans. a. Thickened bowel loop on USG (Ref: Sutton 7/e p873) • Pseudokidney sign or target sign is USG finding of intussusceptionQ.

Urology

• Although IVU gives excellent images of the urinary tract, its use should be restricted because in a few patients the iodine in the contrast medium may provoke a potentially life-threatening anaphylactic reactionQ. • Patients with a history of allergy, atopy and eczema are particularly vulnerableQ, but severe reactions may occur without warning.

614

Surgery Essence

91. Ans. a. DMSA scan (Ref: Bailey 27/e p1392) • Renal scarring or structure of kidney is best demonstrated by a DMSA scan.



Investigation of Choice ADPKD (Retroperitoneal Fibrosis) Medullary Sponge Kidney VUR Retrocaval ureter

CT scanQ IVPQ MCUQ MRIQ

PUJ Obstruction Renal structure or surface

DTPA scanQ DMSA scanQ

92. Ans. d. X-ray diffraction (Ref: www.imaging.robarts.ca/.../2005pmb50)

Urology

X-ray Diffraction • The X-ray diffraction is dedicated to materials identification and characterize through single crystal and power X-ray diffraction analysis. • Monoenergetic X-ray diffraction (XRD) analysis is an established standard for the assessment of urinary stone compositionQ. • For the precise determination of true stone composition, x-ray diffraction analysis has often been the method of first choiceQ.

93. Ans. b. Micturating cystourethrogram



94. Ans. a. Kidney (Ref: Smith 18/e p528; Campbell 11/e p23) Clinical Significance of Different Casts • A normal constituentQ of urine and has no attached significanceQ • Tom Horsfall proteinQ is protein secreted by epithelial cells of loop of henle. This protein may be excreted as Hyaline castQ

1. Hyaline casts

2. RBC cast 3. WBC casts 4. Brood granular casts 5. Pigmented muddy brown granular casts

• • • • •

Are suggestive of glomerular injuryQ or acute glomerulonephritis Are suggestive of interstitial injury and may be seen in interstitial nephritisQ WBC cast with bacteria indicate pyelonephritisQ Are seen in CRFQ and suggests interstitial fibrosis and dilatation of tubules. Are suggestive of ischemic or nephrotoxic injuryQ (Tubular Necrosis)



95. Ans. d. Graphic representation of radioactivity of kidneys



96. Ans. c. Chronic obstructive nephropathy (Ref: Wolfgang 2/e p550) Cortical Rim Nephrogram is seen in • Acute total main renal artery occlusionQ • Renal vein thrombosisQ



• Rim and ball nephrogram on IVP is seen in chronic obstructive nephropathyQ. • Rim sign is seen in chronic hydronephrosisQ. 97. Ans. a. Micturating cystourethrogram Procedure

Section 4

• Acute tubular necrosisQ • Severe chronic urinary obstructionQ

Mean Effective Dose (mSv) Value

X-ray abdomen

0.7

Intravenous urogram (6 films)

2.5Q

MCU

1.2Q

Cystography

1.8

Lithotripsy

1.3

Nephrostomy

3.4Q

PCNL

4.5

Ureteric stenting

4.7

CECT abdomen

10Q

Renal angiogram

2 to 30

Kidney stent insertion

12.7

Kidney and Ureter

98. Ans. a. Stone removal, b. Ureteral obstruction, c. Anterograde renography, d. Renal tumor resection (Ref: Bailey 27/e p1412; Campbell 11/e p153)

• For HDN, DJ stenting and percutaneous nephrostomy, both are having same results but DJ stenting is less invasiveQ. • For pyonephrosis, percutaneous nephrostomy is better than DJ stentingQ. 99. Ans. a. Hydronephrosis

100. Ans. d. Pyonephrosis

Section 4

“Percutaneous nephrostomy is indicated to drain the upper urinary tract collecting system in cases of obstruction at an intrarenal location, at the ureteropelvic junction, or anywhere in the ureter”.



615

101. Ans. b. DMSA scan (Ref: Campbell 11/e p285, 2940; Smith 18/e p206; Bailey 27/e p1404)

BENIGN RENAL TUMORS 102. Ans. c. Bleeding is self limited, d. Nephrectomy is the treatment of choice, (Ref: Smith 18/e p331; Campbell 11/e p1309; Bailey 27/e p1416) 103. Ans. b. Angiomyolipoma (Ref: Campbell 11/e p1306, 10/e p1499; Bailey 27/e p1416) In the given CT image, the renal mass is containing fat content, which is suggestive of angiomyolipoma, which is a fat containing benign renal neoplasm. “Angiomyolipoma is the only benign renal tumor that is confidently diagnosed on cross-sectional imaging. The presence of fat (confirmed on non-enhanced thin-cut CT by a value of −20 Hounsfield Units [HU] or less) within a renal lesion is considered the diagnostic hallmark. Findings of more than 20 pixels with attenuation less than −20 HU and of more than 5 pixels with attenuation less than −30 HU have been shown to have a positive predictive value of 100%. Ultrasonography shows a well-circumscribed, highly echogenic lesion with shadowing. On angiography (or CT-angiography) aneurysmal dilation is found in 50% of angiomyolipomas. The size of the aneurysms has been reported to correlate with the risk of rupture. MRI can be used in difficult cases or in lieu of CT, with findings on fat-suppressed images being highly suggestive of the diagnosis.”-Campbell 11/e p1306 104. Ans. b. Renal oncocytoma (Ref: Smith 18/e p330; Campbell 11/e p1305) Central Stellate Scar is seen in • FNHQ • Fibrolamellar HCCQ

• Serous cystadenomaQ (pancreas) • Renal oncocytomaQ

RENAL CELL CARCINOMA: TYPES 106. Ans. a. Mutated VHL gene is associated with clear cell carcinoma, c. Extreme hypodiploidy occurs 107. Ans. a. Clear cell type 108. Ans. d. Monosomy of 1 and Y (-1, -Y) (Ref: Smith 18/e p333; Campbell 11/e p1329; Bailey 27/e p1417) • These tumors exhibit multiple chromosome loss and extreme hypodiploidyQ. • Loss of multiple chromosomes 1Q, 2Q, 6, 10, 13, 17, 21 and YQ. 109. Ans. c. von-Hippel Lindau (VHL) syndrome 110. Ans. a. PCT (Ref: Smith 18/e p333; Campbell 11/e p1329; Bailey 27/e p1417) 111. Ans. a. Medullary (Ref: Campbell 11/e p1333) • “Renal medullary carcinoma is a subtype of RCC that occurs almost exclusively in patients with the sickle cell trait. It is typically diagnosed in young African-Americans, often in the third decade of life, and many cases are both locally advanced and metastatic at the time of diagnosis.”-Campbell 11/e p1333

RENAL CELL CARCINOMA CLINICAL FEATURES, PARANEOPLASTIC SYNDROMES 112. Ans. None (Ref: Smith 18/e p355; Campbell 11/e p1334; Bailey 27/e p1417)

Urology

105. Ans. d. Chromophobe cell carcinoma (Ref: Smith 18/e p333; Campbell 11/e p1329; Bailey 27/e p1417)

616

Surgery Essence

Urology

Incidence of Systemic Syndromes Associated with RCC Syndrome

%

↑ ESR

55.6

Hpertension

37.5

Anemia

36.3

Calhexia, weight loss

34.5

Pyrexia

17.2

Abnormal liver function

14.4

Hypercalcemia

4.9

Polycythemia

3.5

Neuromyopathy

3.2

Amyloidosis

2.0

113. Ans. c. More common in female

114. Ans. b. Lungs

115. Ans. a. Renal cell carcinoma

116. Ans. d. All

117. Ans. a. Seminoma testis, c. Hypernephroma (Ref: Smith 18/e p333) • Cannon-Ball pulmonary metastases are characteristic feature of RCC and testicular carcinoma. As a rule, RCC produces spherical or round cannon-ball metastases. 118. Ans. c. von-Hippel Lindau disease

119. Ans. d. Lower pole involvement

120. Ans. a. Invasion of renal vein means inoperability 121. Ans. d. Invasion of renal vein is contraindication for surgery, b. Associated with anemia and low ESR 122. Ans. c. More common in female

123. Ans. c. Renal cell carcinoma

124. Ans. None • Cushing syndrome is the least common among the given options.

125. Ans. None > d. Cushing syndrome

126. Ans. a. Hematuria

127. Ans. c. Hematuria

RENAL CELL CARCINOMA DIAGNOSIS AND TREATMENT 128. Ans. b. IVC involvement indicates inoperability (Ref: Smith 18/e p340; Campbell 11/e p1355; Bailey 27/e p1419) • 45–70% of patients with RCC and IVC thrombus can be cured with an aggressive surgical approach including radical nephrectomy and IVC thrombectomy. 129. Ans. d. Pathological staging

130. Ans. d. Renal cell carcinoma (Ref: Campbell 11/e p1340; Harrison 19/e p578) Renal Cell Carcinoma

Section 4

• Cannon-Ball pulmonary metastases are characteristic feature of RCCQ. • As a rule, RCC produces spherical or round cannon-ball metastasesQ.

131. Ans. a. Partial nephrectomy

132. Ans. a. Radiosensitive

133. Ans. c. Right radical nephrectomy and left partial nephrectomy

134. Ans. a. Transperitoneal

Kidney and Ureter 135. Ans. c. Renal vein

617

136. Ans. b. Chromosome 3

“CT scanning is more sensitive than US or IVU for detection of renal masses. A typical finding of RCC on CT is a mass that becomes enhanced with the use of intravenous contrast media. In general, RCC exhibits an overall decreased density in Hounsfield units compared with nor- mal renal parenchyma but shows a heterogeneous pattern of enhancement or increased attenuation (slightly decreased from the surrounding parenchyma) when contrast is used. In addition to defining the primary lesion, CT scanning is also the method of choice in staging the patient by visualizing the renal hilum, perinephric space, renal vein and vena cava, adrenals, regional lymphatics, and adjacent organs.”-Smith 18/e p336

Section 4

137. Ans. d. Renal cell carcinoma (Ref: Smith 18/e p336; Campbell 11/e p1338; Bailey 27/e p1419) Most likely diagnosis for an asymptomatic complex cyst in a 65 years old male as given in CT image, which appears as exophytic mass is renal cell carcinoma.

138. Ans. a. Transitional cell carcinoma 139. Ans. c. Elevated ESR

140. Ans. a. Partial nephrectomy

141. Ans. c. Keeping fascia back in place (Ref: Smith 18/e p339; Campbell 11/e p1345) • The prototypical concept of Radical nephtrectomy encompasses the basic principles of early ligation of the renal artery and vein, removal of the kidney with primary dissection external to the Gerota’s fascia, excision of the ipsilateral adrenal gland, and performance of a complete regional lymphadenectomy from the crus of the diaphragm to the aortic bifurcationQ. • It has been well demonstrated that removal of the ipsilateral adrenal gland is not routinely necessary in the absence of radiographic adrenal enlargement unless the malignant lesion extensively involves the kidney, is locally advanced, or is located in the upper portion of the kidney immediately adjacent to the adrenal glandQ.

Radical Nephrectomy • Radical nephrectomy is the primary treatment for localized RCC. • Its goal is to achieve the removal of tumor and to take a wide margin of normal tissue. • Radical nephrectomy encompasses: −− Basic principles of early ligation of the renal artery and veinQ (Artery followed by vein) First vein is incircled then artry is ligated. −− Removal of the kidney with primary dissection external to the Gerota’s fasciaQ −− Excision of the ipsilateral adrenal glandQ −− Performance of a complete regional lymphadenectomy from the crus of the diaphragm to the aortic bifurcationQ. 142. Ans. d. Para-aortic nodes (Ref: Campbell 11/e p1345)

143. Ans. c. Venacavagraphy

144. Ans. c. Tumor grossly extends into the inferior vena cava (Ref: Campbell 11/e p1337; Smith 18/e p334)

WILMS’ TUMOR 145. Ans. b. Abdominal lump (Ref: Smith 18/e p342; Campbell 11/e p3572; Bailey 27/e p1421) 147. Ans. c. Histology

148. Ans. a. Within 10 days

149. Ans. c. Same location

150. Ans. c. Abdominal mass

151. Ans. a. Hematuria, b. Mass abdomen, d. Fever

152. Ans. c. Hematuria almost always present

153. Ans. b. Less than 1 year

154. Ans. a. Preoperative use of actinomycin D

155. Ans. b. Lungs

156. Ans. b. International society of Pediatric Oncology (SIOP) (Ref: Campbell 11/e p3575-3576, , Schwartz 10/e p1638-1639) The postchemotherapy based staging system is the ‘SIOP’ staging system developed by the International society of oncology. 157. Ans. c. Arthogryposis multiplex congenita 158. Ans. b. Open nephrourecterectomy (Ref: Campbell 11/e p3574) • The treatment of choice for satge I Wilm’s tumor is transperitoneal radical nephrectomy (radical nephroureterectomy)Q followed by chemotherapy with or without radiotherapy depending upon tumor histology. 159. Ans. c. 11 p13

TUMORS OF RENAL PELVIS 160. Ans. a. Transitional cell carcinoma (Ref: Smith 18/e p322; Campbell 11/e p1370) • Urothelial (transitional cell) carcinoma make up > 90% of upper urinary tract tumors

Urology

146. Ans. a. Bone metastasis

618

Surgery Essence 161. Ans. b. Pelvic calculus 162. Ans. b. Retrograde pyeloureterography (Ref: Smith 18/e p324; Campbell 11/e p1371) • Goblet sign and Stipple sign describe the appearance of ureteral dilation below the site of an intraluminal ureteral filling defect, best seen at retrograde pyelography (RGP)Q. • The Stipple sign refers to the pointillistic end-on appearanceQ on IVP or RGP of contrast material tracking into the interstices of a papillary lesion. • Because maturity of TCC have a papillary configuration, presence of this sign should raise the suspicion of TCC, while the Stipple sign is best seen in large papillary bladder tumorsQ, it can occur anywhere in urothelial tumor, which expresses papillary architecture. 163. Ans. d. Transitional carcinoma of pelvis extending till ureter

Urology

164. Ans. a. Radical nephroureterectomy 165. Ans. d. Both b and c (Ref: Smith 18/e p322; Campbell 11/e p1370; Bailey 27/e p1421)

RENAL TRAUMA 166. Ans. a. Urgent IVP is indicated (Ref: Smith 18/e p286; Campbell 11/e p1151; Bailey 27/e p1413) 167. Ans. c. Exploration indicated in all cases

168. Ans. b. Uremia

169. Ans. b. Open Gerota’s fascia and explore proximal renal vessels (Ref: Smith 18/e p286; Campbell 11/e p1151) • The correct option should be “take the control or explore proximal renal vessels before opening Gerota’s fascia”, to avoid excessive intra-operative bleeding. Excretory Urography (IVU or IVP)- Campbell 11/e p1151 • Historically, excretory urography was the most commonly used modality to evaluate genitourinary injuries. Largely replaced by CT, a limited role includes the intraoperative “single-shot” IVP. The indications are uncommon, but when the surgeon encounters an unexpected retroperitoneal hematoma surrounding a kidney during abdominal exploration, the study can provide essential informationQ. • The main purpose of the one-shot IVP is to assess the presence of a functioning contralateral kidney and to radiographically stage the injured sideQ. • If findings are not normal or near normal, the kidney should be explored to complete the staging of the injury and reconstruct any abnormality foundQ. • In the given problem, patient is having a non-pulsatile hematoma, which is not an indication for surgical exploration. • On table IVU was done to see the function of contralateral kidney, the function of opposite kidney should be ascertained before planning nephrectomy in any trauma patient. • Angiography is largely used to define arterial injuries suspected on CT or to localize and control arterial bleeding. Renal embolization has proved useful in the primary setting with persistent bleeding in a hemodynamically stable patient. • Take the control or explore proximal renal vessels before opening Gerota’s fascia”, to avoid excessive intra-operative bleeding. 170. Ans. a. Observation and supportive measures

171. Ans. c. Contrast enhanced computed tomography

Section 4

172. Ans. a. 20% (Ref: Smith 18/e p287) • Minor renal injuries from blunt trauma account for 85% of cases and do not usually require operationQ. • Bleeding stops spontaneously with bed rest and hydrationQ. • Cases in which operation is indicated include those associated with persistent retroperitoneal bleeding, urinary extravasation, evidence of nonviable renal parenchyma, and renal pedicle injuries (less than 5% of all renal injuries)Q. 173. Ans. c. Contralateral renal function is ascertained

174. Ans. a. Exploration is indicated in 90% of cases

175. Ans. c. 5

176. Ans. b. Pulsatile hematoma

177. Ans. d. CECT of abdomen 178. Ans. c. J-shaped urinary stent (Ref: Campbell 11/e p3736) Management of urinoma is by endoscopic intervention, with cystoscopy, retrograde pyelography, placement of a ureteral stent, urethral catheter drainage, and intravenous antibiotics.

Kidney and Ureter

619

URETERIC INJURY 180. Ans. b. Hysterectomy

POLYCYSTIC KIDNEY DISEASE 181. Ans. a. Autosomal recessive trait (Ref: Smith 18/e p537; Campbell 11/e p3017; Bailey 27/e p1402) 182. Ans. c. Renal cell carcinoma 183. Ans. d. Decompression of cyst leads to normal renal function

Section 4

179. Ans. a. Complete renal atrophy, d. Hydronephrosis (Ref: Smith 18/e p288; Bailey 27/e p1414)

184. Ans. a. Autosomal dominant (Ref: Campbell 11/e p3017; Smith 18/e p537) 185. Ans. a. Hypertension is rare 186. Ans. b. 16 and 4 187. Ans. a. Cysts in liver, c. Berry aneurysms 188. Ans. a. Lung 189. Ans. d. Renal transplant 190. Ans. d. 50% 191. Ans. a. ADPKD 192. Ans. a. Lungs 193. Ans. a. Spider leg appearance (Ref: Smith 18/e p537; Campbell 11/e p3017; Bailey 27/e p1402) IVP in ADPKD • Stretching of the calyces by the cysts (spider leg or bell like deformity)Q

• Bubble appearanceQ (calyceal distortion) • Swiss cheese appearanceQ

194. Ans. b. Polycystic kidney 195. Ans. a. Inherited as autosomal recessive

196. Ans. b. Hypertension is rare

197. Ans. b. Hepatic cyst

198. Ans. b. The presentation is unilateral

199. Ans. a. Autosomal dominant (Ref: Campbell 11/e p3017; Bailey 27/e p1402) 200. Ans. d. Erythrocytosis (Ref: Campbell 11/e p3017; Smith 18/e p537; Bailey 27/e p1402)

PUJ OBSTRUCTION 201. Ans. d. Aberrant vessel is the most common cause (Ref: Smith 18/;e p575; Campbell 11/e p1105; Bailey 26/e p1290-1292) 202. Ans. c. Right sided lesions predominate 203. Ans. c. Painless abdominal mass 204. Ans. c. Dismembered pyeloplasty 206. Ans. b. PUJ obstruction 207. Ans. b. DTPA scan 208. Ans. b. Endoscopic pyelotomy is contraindicated 209. Ans. a. Dietl’s crisis

CONGENITAL ANOMALIES OF KIDNEY 210. Ans. a. Bilateral renal agenesis (Ref: Smith 18/e p513; Campbell 11/ep3007, 2975; Bailey 27/e p1410) 211. Ans. e. All are true 212. Ans. d. UTI (Ref: Smith 18e p522; Campbell 11/e p3037) • MSK is usually a benign process, and it may remain asymptomatic and undetected for life. Clinical presentation usually occurs after age 20 years with most common presentation being renal colic (50-60%), followed by urinary tract infection (20-30%) and gross hematuria (10-18%) 213. Ans. c. Medullary sponge kidney (Ref: Smith 18/e p522; Campbell 11/e p3037) IVP (Excretory urogram): IOC to diagnose medullary sponge kidneyQ brush” Q

• “Bristles on appearance due to dilated ducts • “Bouquet of flowers” Q appearance due to calcification in the ectatic ducts 214. Ans. a. Supernumerary renal arteries (Ref: Campbell’s 10/e p26; Smith 18/e p522)

Urology

205. Ans. d. Pelviureteric junction obstruction (Ref: Campbell 11/e p1105; Smith 18/e p575)

620

Surgery Essence • Vascular anomalies involving the kidney are very common being present in 25% to 40% of kidneys. • Supernumerary renal arteries with two or more renal arteries supplying each kidney are the most common renal vascular anomaly.

Abnormalities of Renal vasculature MC renal vascular anomaly is the presence of supernumerary renal arteriesQ. Variations of the main renal artery and vein are common, present in 25% to 40% of kidneys. MC variation is occurrence of supernumerary renal arteries (two or more arteries to a single kidneyQ) MC sub-group of supernumerary renal arteries is a duplicated renal artery (double renal artery)Q involving a second dimunitive renal artery supplying each kidney • Supernumerary renal veins are also common, but occur about half as commonly as supernumerary renal arteries. • • • •

215. Ans. a. Left renal vein (Ref: Smith 18/e p522)

Urology

Renal Collar • The main renal vein divides and sends one limb anterior and another limb posterior to aorta to reach the IVCQ. • Formed on the left sideQ and represents persistence of the embryonic state. 216. Ans. d. Is a normal variant (Ref: www.ajronline.org/content/188/5/1380.full)

Persistent Fetal Lobulation • • • • •

Persistent fetal lobulation is a normal variant seen occasionally in adult kidneys. It occurs when there is incomplete fusion of the developing renal lobulesQ. Embryologically, the kidneys originate as distinct lobules that fuse as they develop and grow. It is often seen on ultrasound, CT or MRI as smooth indentations of the renal outline in between renal pyramidsQ. They should be distinguished from renal cortical scarring, which generally overlie the pyramidsQ.

217. Ans. d. 1:1000 (Ref: Campbell 10/e p3136, Smith 18/e p522)

Ectopic Kidney • The actual incidence among autopsy series varies from 1 in 500 to 1 in 1200 but the average occurrence is about 1 in 900Q. 218. Ans. c. 1:1400 individuals (Ref: Campbell 11/e p3007, 10/e p3128; Smith 18/e p513) • Most autopsy series suggest that unilateral renal agenesis occurs once in 1100 birthsQ. • In an historical survey of excretory urograms, the incidence ranged between 1 in 1500Q. 219. Ans. d. Biopsy (Ref: Campbell 11/e p3021; Smith 18/e p537)

Medullary Cystic Disease • • • •

Excretory urography and ultrasonography frequently fail to detect cysts because they are smallQ. Cysts may be seen on imaging studies if they are large enough, but, early in the disease, cysts are rarely visibleQ. It is best diagnosed by biopsyQ. Histologically, there is a characteristic triad: −− Irregular thickening and disintegration of the tubular basement membrane −− Marked tubular atrophy with cyst development −− Interstitial cell infiltration with fibrosis

Section 4

ABERRANT RENAL ARTERY 220. Ans. d. Usually divided to gain access to renal pelvis (Ref: Smith 18/e p522)

Aberrant Renal Artery • • • •

Arteries that originate from vessels other than aorta or the main renal artery UnilateralQ, more common on left sideQ, involving lower poleQ of kidney May cause hydronephrosis due to extrinsic compressionQ These are end arteriesQ, therefore any injury or division may lead to lower pole infarction. • The renal arteries are end-arteries, division leads to infarction of parenchymaQ. • Renal veins have extensive collaterals and an aberrant vein can be divided with impunityQ.

221. Ans. a. Bilateral

Kidney and Ureter

621

RETROCAVAL URETER

Section 4

222. Ans. c. Retrocaval ureter (Ref: Smith 18/e p575; Campbell 11/e p1125; Bailey 27/e p1402) 223. Ans. c. Retrocaval ureter (Ref: Smith 18/e p575; Campbell 11/e p1125; Bailey 27/e p1402) Retrocaval Ureter (Circumcaval ureter) Diagnosis • MRI is IOC to delineate anatomy clearly & non-invasivelyQ • IVP: “Reverse J”, “Fish Hook” or “Shepherd crook”Q deformity. 224. Ans. d. Posterior cardinal vein

HORSESHOE KIDNEY 225. Ans. a. Spider like appearance in IVP, d. Heminephrectomy improves function (Ref: Smith 18/e p520; Campbell 11/e p2996; Bailey 27/e p1399) 226. Ans. b. L3-L4 vertebra

227. Ans. b. Horseshoe kidney

228. Ans. d. Isthmus may contain aberrant vessels

229. Ans. c. Inferior mesenteric artery (Ref: Campbell 11/e p2293; Smith 18/e p520; Bailey 27/e p1399)

RENAL CYST 230. Ans. a. Renal cyst (Ref: Smith 18/e p515; Campbell 11/e p1300; Bailey 27/e p1402) 231. Ans. a. Polycystic kidney (Ref: Smith 18/e p514; Campbell 11/e p3017) • Incidence of polycystic kidney: 1 in 400Q (0.25%) • Simple renal cyst is MC cystic disease in human kidney (incidence is 0.22% from birth to 18 years)Q

URETEROCELE 232. Ans. d. Ureterocele (Ref: Smith 18/e p571; Campbell 11/e p3076; Bailey 27/e p1401) 233. Ans. d. Endoscopic diathermy

234. Ans. b. Ureterocele

235. Ans. c. Ureterocele

236. Ans. c. Ureterocele (Ref: Smith 18/e p571; Campbell 11/e p3076; Bailey 26/e p1286) • IVP: Typical Adder head or Cobra head or Spring onion appearanceQ is diagnostic of ureterocele

URETERIC ABNORMALITIES 237. Ans. a. Bulbar urethra (Ref: Smith 18/e p570; Campbell 11/e p3098; Bailey 27/e p1400) 238. Ans. c. Ectopic ureteric orifice

239. Ans. b. Duplication of ureter

240. Ans. b. Right bifid and left complete duplication 
of ureter (Ref: Campbell 11/e p3075; Smith 18/e p578) Right Incomplete (bifid) with Left Complete Duplication of Ureter Normal system

Duplex kidney

Urology

• MC congenital anomaly of upper urinary tract: Duplication of ureterQ • MC congenital anomaly of genitourinary tract: VURQ

622

Surgery Essence 241. Ans. a. Prostatic urethra (Ref: Smith 18/e p573; Campbell 11/e p3075; Bailey 27/e p1400)

242. Ans. d. Paradoxical incontinence

243. Ans. c. Lower pole moiety is more prone to obstruction and upper pole more prone to reflux 244. Ans. a. Weigert-Meyer’s rule (Ref: Smith 18/e p570; Campbell 11/e p3098; Bailey 27/e p1400) 245. Ans. a. Duplication of ureter (Ref: Campbell 11/e p3075)

246. Ans. d. Ureteral duplications

247. Ans. c. Below and medial to the lower pole ureter 248. Ans. a. Lateral and cephalad to the upper pole ureter (Ref: Campbell 11/e p3098; Smith 18/e p570; Bailey 27/e p1400) 249. Ans. a. Posterior urethra

250. Ans. a. Vestibule

251. Ans. a. In males, there is duplication in 80% cases • Ectopic ureteral orifice is associated with a duplication collecting systemQ in 80%, in femalesQ, not in males.

Urology

252. Ans. c. Ureteral incontinence with otherwise normal voiding

253. Ans. a. Recurrent UTI

VESICOURETERIC REFLUX 254. Ans. a. Micturating cystourethrogram (Ref: Smith 18/e p191; Campbell 11/e p3141) 255. Ans. b. Micturating cystourethrography

256. Ans. a. I

257. Ans. d. Grade V VUR (Ref: Smith 18/e p191; Campbell 11/e p3141)

258. Ans. a. Cotrimoxazole 259. Ans. b. Postnatal scarring may occur even in the absence of urinary tract infections • Although UTI is the most important cause for scarring and nephropathy in patients with VUR, scarring can occur even in the absence of UTI due to pressure effect of refluxQ on the renal tissue.

Section 4

260. Ans. c. VUR induced pyelonephritis

RENAL ARTERY ANEURYSM 261. Ans. b. 5% (Ref: Smith 18/e p523; Campbell 11/e p2999)

HEPATORENAL SYNDROME 262. Ans. a. Urine sodium < 10 mEq/L, b. Normal renal histology (Ref: Sabiston 20/e p1043; Blumgart 5/e p389; Schackelford 7/e p1449, 1527; Harrison 19/e p2066) 263. Ans. c. Urine osmolality lower than plasma osmolality • Hepatorenal syndrome is associated with urine osmolality greater than plasma osmolalityQ. 264. Ans. a. The creatinine clearance is > 40 mL/min, c. The urine osmolality is lower than the plasma osmolality

Kidney and Ureter

623

DIALYSIS Complications during Hemodialysis • Hypotension is the MC acute complication of hemodialysis, particularly among patients with diabetes mellitus. • Muscle cramps during dialysis are also a common complication of the procedure. • Anaphylactoid reactions to the dialyzer, particularly on its first use, have been reported most frequently with the bioincompatible cellulosic-containing membranes. • Cardiovascular disease constitutes the major cause of death in patients with ESRD. Cardiovascular mortality and event rates are higher in dialysis patients than in patients post-transplantation, although rates are extraordinarily high in both populations.

Section 4

265. Ans. b. Peritonitis, c. Hypertension (Ref: Harrison 19/e p1824)

RENAL TRANSPLANT 266. Ans. a. Hardy (Ref: Campbell 11/e p1087) First autologous renal transplantation was performed by ‘Hardy’ in 1963Q ‘In 1963, Hardy performed the first renal autotransplantation to resolve on extensive ureteral lesion’. 267. Ans. c. In India, organ harvesting from brain dead patients is not permitted by law (Ref: Bailey 27/e p1546-1547)

Renal Transplantation • Most of the organs used for transplantation are obtained from brainstem-dead, heart-beating deceased donorsQ and in the majority of cases multiple organs are procured. • In India, for organ harvesting from brain dead patientsQ, the relatives are formally asked to sign a prescribed form, in contrast to U.K., where the transplant co-ordinators and nurse just write and sign in the file about the consent given. • After removal from the donor, the kidney is flushed with chilled organ preservation solution and, if necessary, stored briefly on iceQ until transplanted into the recipient. • Calcineurin blockers are especially useful in renal transplant patients. These include cyclosporine and tacrolimusQ. 268. Ans. c. Skin cancer (Ref: Bailey 27/e p1542)

Malignancy after Transplantation

269. Ans. a. Right kidney is preferred over the left for transplantation (Ref: Bailey 27/e p1549) If the left kidney has a single renal artery (10% of kidneys have two or more renal arteries) it is usually chosen for transplantation because it has a longer renal vein, which simplifies the transplant operationQ.

Renal Transplantation • Before the donation it is essential to perform imaging (usually MR angiography or CT angiography) to delineate the anatomy of the arterial supply to the kidneysQ. • If the left kidney has a single renal artery (10% of kidneys have two or more renal arteries) it is usually chosen for transplantation because it has a longer renal vein, which simplifies the transplant operationQ. • The presence of multiple arteries does not necessarily preclude donation although implantation of living donor kidneys with multiple arteries may increase the chances of vascular complications developing after implantationQ. 270. Ans. b. Corticosteroids, d. Cyclosporine (Ref: Bailey 27/e p1540)

Immunosuppressive Drug Regimen in Renal Transplantation • Combination of glucocorticoid with cyclosporine or tacrolimus, azathioprine or mycophenolate mofetil and sometimes antilymphocyte antibody preparationQ.

Urology

• After transplantation there is an increased risk of developing most types of malignancy but the risk is particularly high for those types of tumour in which viral infection plays an etiological role. • The risk is particularly high for skin cancerQ and a condition called post-transplant lymphoproliferative disorder (PTLD)Q. • The risk of skin cancer after transplantation rises with age and with exposure to sunlight, and it has been predicted that 50% of transplant patients will develop a skin malignancy within 20 years of transplantationQ. • Transplant patients also have a 300-fold increased risk of developing Kaposi’s sarcomaQ, although this malignancy is still very uncommon after transplantation.

624

Surgery Essence 271. Ans. d. Urethral valves (Ref: Campbell’s 11/e p2891)

272. Ans. c. Urachal fistula (Ref: Campbell’s 11/e p3176)

KIDNEY AND URETER ANATOMY AND PHYSIOLOGY 273. Ans. a. Peristalsis due to flow of urine (Ref: Oxford Urology 6/e p2123) • Ureters are situated retroperitoneally, and are identified by peristalsisQ.

Urology

Kidney and Ureter: Anatomy and Physiology • • • • • •

Kidney receives 20% of cardiac output Renal vein is anterior, renal artery in middle, renal pelvis is posterior (VAP) Main renal artery divides into 4–5 segmental arteries. First and most consistent division is posterior branch. There are 2 or more LNs at the renal hilum (first site of metastasis)Q. From the left kidney, lymphatic trunk then drain into para‑aortic nodes. From the right kidney, lymphatics drain into hilar lymph node, inter-aortocaval and paracaval nodes.

274. Ans. c. Genitofemoral nerve (Ref: BDC 5/e p325, 326) • The genitofemoral nerve lies posterior to the ureterQ. The ureter lies on the genitofemoral nerve, which is posterior. 275. Ans. d. Ischial spine (Ref: Snell’s 7/e p284, 382) • At the level of ischial spine ureter changes its direction from down-backward to forward - medially but there is no constriction. • The normal anatomical narrowing of ureter are the potential sites of obstruction by ureteric (kidney) stone.

Normal Anatomical Narrowing of Ureter

Section 4

• • • • •

Uretero-pelvic junctionQ Where ureter cross the brim of pelvic inletQ (at site of crossing of iliac arteryQ) Juxtaposition of vas deferens or broad ligament Uretero-vesicle junctionQ Ureteric orifice

276. Ans. d. Ureterovesicle junction

277. Ans. b. It is a type of portal-circulation (Ref: Snell’s 7/e p283)

Renal Circulation Artery Vein • Right renal artery is longer than leftQ. • Left renal vein is longer than rightQ • Close to hilum, renal artery divides into 5 segmental arteries, • Stellate veins drain most superficial zone of renal cortexQ that are end-arteriesQ • Left kidney is preferred for transplantation due to longer • Renal artery → 5 segmental arteries → Arcuate artery renal veinQ →Interlobular artery → Afferent glomerular arterioleQ 278. Ans. b. Below uterine artery in the Mackenrodt’s ligament (Ref: Keith and Moore 4/e p374, 384)

Kidney and Ureter

625

• Maximum chance of ureteric injury is in hysterectomy, while ligating uterine vessels in the Mackenrodt’s ligamentQ. • Ureter is also vulnerable to injury when ovarian vessels are being ligated during oophorectomy.

• Gonadal vessels and colic vessels lie anterior to ureter; IVC is medial; gonadal vein and inferior mesenteric vein is on left sideQ. 280. Ans. b. Tongue like papillary projection

281. Ans. c. Peristaltic movements

282. Ans. d. 0–5 cm of H2O

• Resting ureteric pressure is ~0–5 cm H2OQ • Ureteric contraction pressures range from 20–80 cm H2OQ.

Section 4

279. Ans. a. Gonadal vessels lie anterior to it (Ref: Keith and Moore 4/e p281)

283. Ans. a. Trapezius (Ref: Smith 18/e p339)

Nephrectomy • Two common approaches: − Transperitoneal abdominalQ approach − Retroperitoneal loinQ approach • Retroperitoneal loin approach is preferred method except in malignant tumors and renal injuries (to exclude other injuries)Q

Nephrectomy (Loin Approach) • Following muscles are cut in loin approach: −− Lattisimus dorsiQ   − Serratus posterior-inferiorQ 284. Ans. c. Renal artery stenosis

− External and internal oblique, transversus abdominisQ

285. Ans. a. One year of age

• The neonatal kidney achieves concentrating ability equivalent to adult’s kidney by one year of ageQ • The neonatal kidney achieves adult’s GFR by two years of ageQ.

MISCELLANEOUS 286. Ans. a. Urethral stone Relation of hematuria to micturition

Site of bleeding UrethraQ

Blood appears at the end

VesicalQ

Blood is intimately mixed throughout the process

Prerenal, renal or vesicalQ

287. Ans. c. Ureteral calculus • Clinical picture is suggestive of ureteric colic. 288. Ans. a. Excretion of less than 300 mL in 24 hours

289. Ans. a. 7 mL

• The average capacity of the renal pelvis is 4–8 mLQ. 290. Ans. c. Chronic renal failure (Ref: Smith 18/e p49) • Low and fixed specific gravity of urine is seen in chronic renal failure. 291. Ans. b. Renal TB Autosplenectomy Autonephrectomy

Sickle cell anemiaQ Renal TBQ

292. Ans. d. Urerthrovaginal fistula (Ref: Bailey 27/e p1429-1430, 25/e p1320)

293. Ans. a. Transureterostomy

294. Ans. a. >3 RBCs/hpf 295. Ans. a. Medullary (Ref: Campbell 11/e p1333) “Renal medullary carcinoma is a subtype of RCC that occurs almost exclusively in patients with the sickle cell trait. It is typically diagnosed in young African-Americans, often in the third decade of life, and many cases are both locally advanced and metastatic at the time of diagnosis.”-Campbell 11/e p1333

Urology

Blood appears at the beginning

CHAPTER

22

Urinary Bladder

EXSTROPHY OF BLADDER (ECTOPIA VESICAE) Exstrophy of Bladder (Ectopia Vesicae) • Extrophy of bladder is complete ventral defect of Urogenital sinus and the overlying skeletal systemQ. • Defect in the infraumbilical part of anterior abdominal wall, associated with incomplete development of anterior wall of the bladderQ. Embryology • Basic defect is abnormal overdevelopment of the cloacal membrane and its rupture. • Timing of this rupture of this defective cloacal membrane determines the variant of the extrophy-epispadias complex that results. Clinical Features • Posterior wall of the bladder protrudes through the defect with mucosal edges fused with skin and urine spurts onto the abdominal wall from the ureteral orificesQ. • Rectus muscles which are inserted on the pubic rami are also widely separatedQ. • An umbilical herniaQ though usually small is present along with extrophic bladder. • In males, complete epispadias with a wide & shallow scrotum. Undescended testis & inguinal herniasQ are common. • Females also have epispadias with bifid clitoris and wide separation of the labiaQ. • Anus is dislocated anteriorly in both sexes and there may be rectal prolapseQ. Complications • Consequences of untreated bladder exstrophy are total urinary incontinenceQ and an increased incidence of bladder cancer, usually adenocarcinoma. • Many untreated exstrophy of bladder reveal fibrosis, derangement of muscularis mucosa and chronic infection leading to hydronephrosisQ. Treatment • EnterocystoplastyQ is the method of choice to augment bladder capacity and aid in reservoir function. • Urinary diversion with cystectomyQ is treatment of choice for small, fibrotic or inelastic bladder. • Complete reconstruction is achieved by: −− Bladder closure with sacral osteotomy and lengthening of penisQ (Posterior iliac osteotomyQ is done in ectopia vesicae) −− Antiureteral reflux procedure with bladder neck reconstructionQ −− Repair of epispadiac penisQ

BLADDER STONE Vesical Calculus Primary Bladder Calculi

Secondary Bladder Calculi

• Develop in absence of any known functional, anatomic or infectious factorsQ

• Develop in concert with bladder outlet obstructionQ, infection, impaired bladder emptying or a foreign body.

Migrant Bladder Calculi • • • •

Found in upper urinary tract, passed into bladder and retained Most stones migrate out of ureter into bladder are < 1 cm and easily passed into urethra Calculi that are retained, are associated with small bladder outlet or bladder outlet obstruction Retained upper tract stones may grow to large size in bladder

Urinary Bladder

627

Bladder Stone

• In older men with bladder stones composed of uric acid, the stone most likely formed in the bladderQ. • Stones composed of calcium oxalate are usually initially formed in the kidneyQ. • In adults, MC type of vesical stone (seen in >50% of cases) is composed of uric acidQ. • Less frequently, bladder calculi are composed of calcium oxalate, calcium phosphate, ammonium urate, cystine, or magnesium ammonium phosphate (when associated with infection).

Section 4

• Most renal stones that are small enough to pass through the ureters are also small enough to pass through a normally functioning bladder and an unobstructed urethraQ.

Endemic Bladder Calculi • In children, stones are composed mainly of ammonium acid urateQ, calcium oxalate, or an impure mixture of ammonium acid urate and calcium oxalate with calcium phosphate. • The common link among endemic areas relates to feeding infants human breast milk and polished riceQ. • These foods are low in phosphorus, ultimately leading to high ammonia excretionQ. • These children also usually have a high intake of oxalate-rich vegetables (increased oxalate crystalluria) and animal protein (low dietary citrate)Q. • Vesical calculi may be single or multiple, especially in the presence of bladder diverticula, and can be small or large enough to occupy the entire bladder. They range from soft to extremely hard, with surfaces ranging from smooth and faceted to jagged and spiculated (“jack” stones)Q. • Jack stones are composed of calcium oxalate dihydrateQ

PRIMARY BLADDER CALCULI (ENDEMIC BLADDER CALCULI) Primary Bladder Calculi (Endemic Bladder Calculi) • Mainly seen in underdeveloped countriesQ (North Africa, Thailand, Myanmar, Indonesia), in pediatric age group. • Most common in children 50 years), usually because of bladder outlet obstructionQ. • MC type: Uric acid (sterile urine) > Struvite stonesQ (Infected urine) • Bladder stones are usually solitaryQ, multiple in 25% patients. Etiology • Bladder outlet obstruction (MC cause)Q •  Foreign body (Foley’s catheter, forgotten DJ stents)Q • Neurogenic bladderQ •  Bladder diverticulaQ Clinical Features • Typical symptoms are intermittent, painful voiding and terminal hematuria with severe pain at the end of micturitionQ. • Pain may be referred to the tip of the penis or to the labia majoraQ.

Urology

• USG bladder: Identifies the stone with its characteristic shadowing and stone moves with changing body position. Treatment

628

Surgery Essence Diagnosis • A large percentage of bladder stones are radiolucent (uric acid)Q. • USG bladder: Identifies the stone with its characteristic shadowing and stone moves with changing body positionQ. Treatment • Small stones: Removed or crushed transurethrally (Cystolitholapexy)Q • Larger stones: Disintegrated by transurethral electrohydraulic lithotripsy or CystolithotomyQ Stones of Genitourinary Tract • MC renal stone: Calcium oxalateQ •  MC primary bladder stone: Ammonium urateQ Q • MC bladder stone: Uric acid >Struvite •  MC prostate stone: Calcium phosphateQ

Urology

MALACOPLAKIA Malacoplakia • Inflammatory disease of the bladder that can also affect ureters & kidneys • In the bladder, it manifests as plaques or nodules made of large histiocytes (von Hansemann cells)Q with laminar inclusion bodies (Michaelis-Gutmann bodies)Q form around bacterial fragments (E. coli) which acts as nidus. • Believed to result from the inadequate killing of bacteria by macrophages or monocytes that exhibit defective phagolysosomal activityQ. Risk Factors • Immunosuppression (Steroids, Transplantation)Q •  Diabetes, Lymphoma, Rheumatoid arthritisQ Clinical Features • More commonly affects women and associated with a history of UTI. • Patients with malacoplakia often have chronic illness or are immunosuppressed.Q • Irritative voiding symptoms (urgency and frequency) and hematuria are common. Diagnosis • USG or CT may demonstrate a mass in the bladder and evidence of obstruction if the disease extends to the ureter. • Culture of the lesion can yield bacteria, most commonly E. coli. Management • • • •

TMP-SMX and fluoroquinolonesQ are recommended in the treatment of malacoplakia. In malacoplakia limited to the lower urinary tract, antibiotic therapy alone is usually sufficient. Best chance of cure: Antibiotics + SurgeryQ The prognosis is poor and mortality rate is high in patients having bilateral renal involvement, regardless of treatment.

SCHISTOSOMIASIS (BILHARZIASIS) Schistosomiasis (Bilharziasis) • Schistosomiasis is an infection with Schistosoma haematobiumQ. • Endemic in Africa, Egypt & Middle East • More common in MalesQ S. hematobium

Affects bladderQ

S. japonicum

Affects liver & small intestineQ

S. mansoni

Affects large intestineQ

Section 4

Lifecycle • ManQ is the only definitive host & intermediate host is snailQ. • Life cycle begins with the passage of eggs into freshwater regions through urineQ. • When the eggs are hatched, miracidia are produced. These penetrate the snail and eventually form into cercariae. • Eggs → MiracidiumQ (penetrate the snail) → CercariaQ (penetrate skin of man) → Migrate to vesical venous plexusQ for reproduction → Eggs in urineQ • Fluke embryos (cercariae) penetrate the skin of man from infected water and migrate to the vesical venous plexus, where sexual reproduction takes placeQ. • Eggs are laid in the submucosal veins & penetrate the bladder wall to enter urineQ.

Urinary Bladder

629

Pathology

Complications • Bladder calcification: Schistosomiasis is the MC causeQ of bladder calcification worldwide. • Urolithiasis, ascending urinary tract infection, urethral & ureteric stricture with subsequent hydronephrosis, & renal failure. • Squamous cell carcinomaQ (Most serious complication) Symptoms

Section 4

• Schistosoma eggs are highly antigenic and induce intense granulomatous reaction in bladder resulting in bladder fibrosis with ureteric or urethral stricturesQ. • Calcification of dead eggs within bladder can produce a calcified bladder or bladder stonesQ.

• MC symptom of urinary schistosomiasis is urinary frequencyQ. • Swimmer’s itch is the first clinical sign due to local inflammatory response from cercarial penetration ( lung Diagnosis Cystoscopy • • • •

Diagnosis and initial staging is made by cystoscopy and transurethral resection (TUR) Q Urinary Cytology Cytologic examination of exfoliated cells are useful in detecting cancer in symptomatic patients and assessing response to treatmentQ Most useful for early diagnosis of recurrence in TCCQ Exfoliated Markers for Detection of Bladder Cancer • Newer urinary tumor markers like BTA test, urinary nuclear matrix protein (NMP22) can detect cancer specific proteins in urine (BTA/NMP22)Q • Hyaluronidase, Lewis-X antigen on exfoliated urothelial cells • Determination of telomerase activity in exfoliated cells • Have been used to detect new index tumors and recurrent tumorsQ • Expected to play important role in near future

• CT and MRI: used for stagingQ Management • Cystoscopy and TUR or biopsy; further management is based on stage, grade, size, multiplicity and recurrence patternQ • Systemic chemotherapy: MVAC (Methotrexate + Vinblastine + Adriamycin + Cisplatin) • Drugs used for Intravesical chemotherapy: BCG, mitomycin-c, epirubicin, thiotepa Prognosis • There is strong correlation between tumor grade and stage and tumor recurrence, progression and survival • In patients with organ confined disease, presence of lymph node metastases appears to be the most important prognostic factor • Presence of lymphovascular invasion is associated with poor prognosisQ

8th AJCC (2017) TNM Staging for CA Bladder

Section 4

T: Primary tumor

N: Regional lymph nodes

Ta: Non-invasive papillary carcinomaQ Tis: Carcinoma in situ “Flat tumor”Q

N1: Single regional LN in true pelvis (hypogastric, obturator, external iliac or presacral LN)Q

T1: Tumor invades subepithelial connective tissueQ

N2: Multiple regional LN in true pelvis (hypogastric, obturator, external iliac or presacral LN)Q

T2a: Superficial muscularis propria invasion (inner half)Q T2b: Deep muscularis propria invasion (outer half)Q

N3: LN metastasis to common iliac LNsQ

T3a: Microscopic extension into perivesical fatQ T3b: Macroscopic extension into perivesical fatQ T4a: Cancer invading pelvic viscera (e.g., prostatic stroma, vaginal wall, rectum, uterus)Q T4b: Extension to pelvic sidewalls, abdominal walls, or bony pelvisQ

M: Distant metastases M0: No distant metastasis M1: Distant metastasis present

Urinary Bladder

633

Treatment of CA Bladder Intravesical BCGQ

Ta (single, low to moderate grade, not recurrent)

Complete TURQ

Ta (large, multiple, high grade or recurrent) and T1

Complete TUR followed by intravesical chemo or immunotherapyQ

T2-T4

• • • •

Any T, N+,M+

Systemic chemotherapy followed by selective surgery or irradiation

Radical cystectomyQ Neoadjuvant chemotherapy followed by radical cystectomyQ Radical cystectomy followed by adjuvant chemotherapy Neoadjuvant chemotherapy followed by concomitant chemotherapy and irradiation

Section 4

Tis

BCG BCG • BCG is attenuated strain of Mycobacterium bovisQ • Mechanism of action: Immunologically mediated −− Exact mechanism is not knownQ −− Binds to fibronectin on bladder cells and elicits TH1 responseQ • Most effectiveQ intravesical chemotherapy Contraindications of BCG Absolute Relative ImmunosupressedQ patients • Deranged LFT • Previous history of Koch’s Gross hematureaQ • UTI Immediately after TURBTQ • Poor performance status Traumatic catheterizationQ

Side Effects

• • • • • Total urinary incontinenceQ • History of BCG sepsisQ

• MC is frequency, urgency and dysuriaQ • Patients with severe BCG sepsis (high fever, chills confusion, hypotension, jaundice) should be treated with ATTQ.

URINARY DIVERSION

• Urinary diversion can lead to stricture at anastomosis, reflux of urine, reabsorption of solutes leading to dyselectrolytemiaQ. • Gold standard conduit for urinary diversion: Ileal conduitQ • Ileal conduit is best for urinary diversion as it is simplest to performQ and associated with minimal intra-operative and immediate post-operative complications. • Ileal or colonic conduits (Ureterosigmoidostomy): −− Hyperchloremic, hypokalemic metabolic acidosisQ (DK Raised HCl in ureterosigmoidostomy: Decreased K+, raised H+ and Cl-) • Jejunal conduits: −− Hypochloremic, hyponatremic, hyperkalemic, metabolic acidosisQ (RH Khurana Decreases NaCl in jejunal conduit: Raised H+, K+; Decreased Na+, Cl–) • Stomach conduit: −− Hypochloremic, hypokalemic, metabolic alkalosisQ (Everything decreases in stomach conduit: Decreased H+, K+, Cl–)

Urology

Urinary Diversion

Multiple Choice Questions ECTOPIA VESICAE

1. In ectopia vesicae, bone divided is: (MHSSMCET 2011) a. Pubic bone b. Sacrum c. Coccyx d. Iliac bone



2. Ectopic vesicae includes all except: (COMEDK 2005) a. Hypospadias b. Extrophy of bladder c. Defective abdominal wall d. Bifid clitoris



3. What is the diagnosis based on the given image?  (JIPMER November 2017) a. Gastroschisis b. Omphalocele c. Umbilical hernia d. Ectopia vesicae



4. All of the following are features of exstrophy of the bladder except: (All India 97) a. Epispadias b. Cloacal membrane is present c. Posterior bladder wall protrudes through the defects d. Umbilical and inguinal hernia





9. Secondary vesical calculus refers to stones formed due to: a. Hypercalciuria (Karnataka 2006) b. Injury c. Infection d. Migrating from above



10. A 60 years old man presented with intermittent painful voiding of urine. What is the diagnosis based on clinical picture and the given image? (Recent Question 2018)

a. Bladder stone c. Prostate calcification

b. Ureteric stone d. Schistosomiasis



11. Not true about bladder stones is: (AIIMS Nov 2001) a. Rare in Indian children b. Primary stones are rare c. Small stones can be removed per urethra d. Maximum stones are radioopaque

5. Which is not seen in complete ectopic vesicae: a. Umbilical hernia  (Recent Quetsion 2016) b. Visible uretero vesical efflux c. Hypospadias d. Waddling gate



12. The commonest bladder stone is: a. Triple phosphate b. Xanthine c. Uric acid d. Cysteine



6. About ectopia vesicae, following is true except:(PGI June 98) a. CA bladder may occur b. Ventral curvature of penis c. Incontinence of urine d. Visible ureterovesical efflux



13. What is the composition of this stone?



7. For treatment of the ectopic-vesiae, which of the following bone is divided to reach the site? (UPPG 2004) a. Pubic rami b. Iliac bone c. Ischium bone d. Symphysis

URINARY BLADDER STONES

8. Regarding urinary bladder stone one is not true:  (AIIMS June 98) a. Common in pediatric patient in tropics than that of non tropical areas b. Uric acid stones are dropped from above c. Jack stone is due to urea splitting bacteria d. Commonly distal passage obstruction cause stone

(Recent Question 2016)

Urinary Bladder

635

SCHISTOSOMIASIS

a. b. c. d.

Calcium oxalate monohydrate Calcium oxalate dihydrate Calcium phosphate Calcium carbonate

14. The following is true about bladder stones: a. Girls more than boys b. Treatment is litholapexy c. Always forms in kidneys and passes down to bladder d. Usually asymptomatic



15. Jack stone calculi is seen in which anatomic part: a. Prostate b. Kidney  (MAHE 2006) c. Ureter d. Bladder



16. Which of the following is false regarding endemic bladder stones? (AIIMS Nov 2013) a. Always associated with recurrence b. High incidence in cereal based diet c. Peak incidence in 3 years old children in India d. Most common type is ammonium urate or calcium oxalate

22. A patient, Ramu presents with hematuria for many days. On investigations he is found to have renal calculi, calcifications in the wall of urinary bladder and small contracted bladder; most probable cause is: (AIIMS Nov 2001) a. Schistosomiasis b. Amyloidosis c. Tuberculosis d. CA urinary bladder

23. One of the following disease will show urinary bladder calcification radiologicaly which resemble fetal head in pelvis: (AIIMS June 2000) a. Tuberculosis b. Schistosomiasis c. Chronic cystitis d. Malignancy

24. Metrifonate is effective against: (COMEDK 2008) a. Amebiasis b. Leishmaniasis c. Schistosomiasis d. Giardiasis



25. In a patient of schistosomiasis, X-ray pelvis was done. What is the name of this sign?

Section 4



URINARY BLADDER: MALAKOPLAKIA 17. Malakoplakia of the urinary bladder is considered to be associated with: (COMEDK 2005) a. TB b. Urothelial carcinoma c. Schistosomiasis d. Defect in phagocytosis

Urology

18. True about malakoplakia is: (MHPGMCET 2002) a. Benign lesion of urinary bladder b. May turn into malignancy c. Michaelis-Gutmann bodies are characteristic feature d. May cause sever hematuria and lead to death



URINARY BLADDER: TUBERCULOSIS

19. Cystoscopic findings in TB bladder are all except: a. Cobblestone mucosa (PGI Dec 97) b. Thimble bladder c. Golf hole ureter d. Whitish efflux from the ureteric holes

20. Thimble bladder is seen in:  (Recent Question 2015, All India 91) a. Acute tuberculosis b. Chronic tuberculosis c. Neurogenic bladder d. Schistosomiasis



21. Treatment of ‘Thimble bladder’ is: (Recent Question 2015) a. Anti-tubercular treatment b. Corticosteroids c. Ileocystoplasty d. Anti-tubercular drugs + steroids

a. Fetal head appearance c. Linear calcification

b. Rim sign d. Goblet sign

CARCINOMA URINARY BLADDER: RISK FACTORS

26. Carcinoma common in dye industry workers: (MHPGMCET 2001) a. Skin b. Scrotum c. Urinary bladder d. Maxilla



27. Transitional cell carcinoma can be seen in:  (MHSSMCET 2006) a. Analgesic nephropathy b. Urate nephropathy c. Pulmonary infections d. Myocardial infarction

636

Surgery Essence

28. Transitional cell carcinoma of bladder is associated with:  (PGI June 2001) a. Schistosomiasis b. Naphthylamine c. Smoking d. Tuberculosis of bladder 29. Squamous cell tumor of urinary bladder is due to: (PGI June 97) a. Stone b. Schistosomiasis c. Chronic cystitis d. Diabetes mellitus 30. Squamous cell carcinoma of urinary bladder is predisposed to by: (PGI June 2002) a. Urolithiasis b. Persistent urachus c. Schistosomiasis d. Polyp e. Smoking

Urology



31. In a survey, many children are examined and were found to have urogenital abnormalities. Which congenital anomaly is associated with increased risk of bladder carcinoma?  (AIIMS Nov 97) a. Medullary sponge kidney b. Bladder exstrophy c. Unilateral renal agenesis d. Doubler ureter

32. True about transitional cell carcinoma of urinary bladder: a. Smoking predisposes (PGI Dec 2003) b. Schistosoma infection predisposes c. Aniline dye workers d. Radiation

33. All are precancerous for carcinoma bladder except:  (Recent Question 2015, All India 91) a. Tuberculosis bladder c. Schistosomiasis



b. Aniline dyes d. Chronic ulcer

34. ‘Kiss cancer’ of the urinary bladder is: a. Highly malignant b. Malignant c. Benign d. Pre-malignant

35. Associated with urinary bladder carcinoma are all of the following except:  (MCI Sept 2009) a. Smoking b. HPV infection c. Schistosomiasis d. Cyclophosphamide 36. A 63-year-old male from the middle east presented with hematuria. The urine showed RBCs but no RBC casts. BUN level was normal. cystoscopy revealed an irregular growth on the mucus of the bladder. A biopsy from this growth revealed features of squamous cell carcinoma. The etiology of this cogidition is most likely linked to: (COMEDK 2014)

 a. Cigarette smoking c. Parasitic infection

41. Most malignant carcinoma of the bladder is: a. Malignant villous tumor b. Solid tumor c. Carcinomatous ulcer d. Adenocarcinoma 42. It is true of carcinoma of the urinary bladder that: a. It usually occurs in childhood b. Occurs more often in aniline dye workers c. It is located most frequently in the trigone d. Papillary formation is rare 43. Bladder tumors mostly arises from: (All India 91) a. Mucosa b. Submucosa c. Muscularis mucosa d. Serosa 44. About transitional cell carcinoma of bladder following is correct: a. Most common site is fundus b. Prognosis is excellent if muscle layer is invaded c. Exposure to industrial carcinogens predisposes d. Most of carcinomas are flat, solid and deeply infiltrating

CARCINOMA URINARY BLADDER: CLINICAL FEATURES AND DIAGNOSIS

45. A 55-year-old smoker presents with history of five episodes of macroscopic hematuria each lasting for about 4–5 days in the past five years. Which of the following investigations should be performed to evaluate the suspected diagnosis? a. Urine microscopy and cytology (All India 2011) b. X-ray KUB c. Ultrasound KUB d. DTPA scan 46. CA urinary bladder commonly presents as: (PGI Dec 2003) a. Hematuria b. Frequency c. Dysuria d. Abdominal lump 47. Most constant and persistent feature of CA bladder is: a. Increased frequency b. Hematuria (PGI Dec 95) c. Recurrent UTI d. Pain abdomen

48. What is true about carcinoma bladder? (AIIMS June 94) a. Common in smokers b. Commoner in females than that in males c. Mostly adenocarcinoma d. Pain in suprapubic region is the first symptom



49. A 67 years old chronic heavy smoker presents with 2 weeks history of frank hematuria. Ultrasound pelvis shows a filling defect. Most probable diagnosis: (JIPMER May 2018) a. Bladder diverticula b. Adenocarcinoma of bladder c. Squamous cell carcinoma of bladder d. Transitional cell carcinoma of bladder



50. A 60-year-old smoker came with a history of painless gross hematuria for one day. Most logical investigation would be: a. Urine routine (All India 2007) b. Plain X-ray KUB c. USG KUB d. Urine microscopy for malignant cytology

37. Which of the following drug causes carcinoma bladder? (Recent Question 2017) a. Cyclophosphamide b. Cisplatin c. Taxane d. Tamoxifen

CARCINOMA URINARY BLADDER: TYPES

38. SCC of bladder is best treated by: a. Chemotherapy c. Radiotherapy



(GB Pant 2011)

b. Radical cystectomy d. TUR

39. Most common bladder tumor: a. TCC b. SCC c. Rhabdomyosarcoma d. Sarcoma

(GB Pant 2011)

40. Most common tumor of urinary bladder is: a. Squamous cell carcinoma (DNB 2008, PGI June 97) b. Adenocarcinoma c. Transitional carcinoma d. Stratified squamous carcinoma



b. HPV infection d. Chronic alcohol abuse



Section 4



Urinary Bladder 51. A 60-year-old smoker came with the history of painless gross hematuria for one day. The investigation of choice would be:  (AIIMS Nov 2006) a. Urine routine and microscopy b. Plain X-ray KUB c. USB KUB d. Urine for malignant cytology



52 False statement regarding urothelial bladder tumor is:  (PGI May 2018) a. Most common variety b. Schistosomiasis is not a risk factor c. Strongly related to smoking d. Pain is the most common presenting feature e. Most common site is trigone



53. An elderly male presents with one episode of gross heematuria. All of the following investigations are recommended for this patient except: (All India 2007) a. Cystoscopy b. Urine microscopy for malignant cells c. Urine tumor markers d. Intravenous pyelogram



54. Urinary cytology is a useful screening test for the diagnosis of:  (Recent Question 2016) a. Renal cell carcinoma b. Wilms’ tumour c. Urothelial carcinoma d. Carcinoma prostate



55. Which of the following is a tumor marker for bladder cancer? (Recent Question 2017) a. AFP b. CEA c. Bladder surface protein d. NMP-22



56. A 60 years old smoker male patient presents with painless gross hematuria for 1 day. IVU shows 1.2 cm filling defect at the lower pole of infundibulum. Which is the next best investigation to be done? (MCI November 2017) a. Cystoscopy b. Urine cytology c. USG abdomen d. DMSA scan

CARCINOMA URINARY BLADDER: TREATMENT 57. Treatment of choice for low grade superficial bladder carcinoma:  (JIPMER 2011) a. Local excision b. Radical cystectomy c. Intravesical BCG d. Chemotherapy

58. A lady who presented with hematuria was found to have Stage II Transitional Cell Carcinoma of bladder. Which of the following statements about management of her condition is true? a. Cystoscopic fulguration is the standard treatment b. 70% chance of requiring cystectomy in 5 years after TURP c. History of smoking is not a risk factor d. There is no role of chemotherapy



59. BCG is used in the treatment of: (MHPGMCET 2006) a. Carcinoma cervix b. Carcinoma colon c. Carcinoma of urinary bladder d. All 60. pT2, pT3 or CIS carcinoma bladder not responding to BCG is best treated by: (MHSSMCET 2008) a. Intravesical mitomycin-C and interferon b. Systemic chemotherapy c. Cystoscopic d. Radical cystectomy

61. Treatment of choice for bladder pTa: (MHSSMCET 2010) a. Endoscopic tumor resection b. Endoscopic tumor resection and intravesical chemotherapy c. Partial cystectomy with intravesical d. Radical cystectomy with or without radical radiotherapy BCG



62. Which of the following is the most effective intravesical therapy for superficial bladder cancer? (AIIMS Nov 2005) a. Mitomycin b. Adriamycin c. Thiotepa d. BCG 63. Which of the following is not an intravesical chemotherapeutic agent? (UPSC 2005) a. Mitomycin C b. BCG c. Epirubicin d. Thiotepa 64. A 65-year-old male smoker presents with gross total painless hematuria. The most likely diagnosis is:  (All India 2003) a. Carcinoma of urinary bladder b. Benign prostatic hyperplasia c. Carcinoma prostate d. Cystolithiasis 65. BCG is used in tumour therapy: (JIPMER 98) a. Bladder b. Stomach c. Esophagus d. Colon 66. A 60-year-old female presented with hematuria and diagnosed transitional cell carcinoma of bladder stage T1N1M0. Best treatment modalities is: (UPPG 2008) a. Transurethral resection b. Transurethral resection and intravesical chemoimmunotherapy c. Total cystectomy and pelvic lymphadenectomy d. Systemic chemotherapy 67. Identify the false statement regarding urothelial cell carcinoma of the bladder. (APPG 2016) a. Ileal conduit diversion is required after cystectomy b. Intravesical chemotherapy and immunotherapy are not found to be beneficial in Non Muscular Invasive Bladder Cancer (NMIBC) c. Radical cystectomy following chemotherapy has been shown to be of benefit in Muscle Invasive Bladder Cancer d. Strongly associated with smoking & Schistosoma hematobium

68. Laser used in carcinoma bladder: (Recent Question 2017) a. Carbon dioxide laser b. Nd-YAG laser c. Ho-YAG laser d. Argon laser



69. Chemotherapy used for metastatic bladder cancer: a. AC (Adriamycin and Cisplatin) (Recent Question 2016) b. Interferon c. MVAC (Methotrexate, Vinblastine, Adriamycin and Cisplatin) d. Cisplatin alone

URINARY BLADDER INJURY 70. A person after pelvic fracture could not pass urine. On examination bladder is not palpable. What is probable diagnosis? (PGI Dec 2008) a. Posterior urethra rupture, with-retention of urine b. Rectourethral injury c. Intraperitoneal rupture of bladder d. Extraperitoneal rupture of bladder

Urology





Section 4



637

Urology

638

Surgery Essence 71. True about extraperitoneal urinary bladder rupture is all except: a. Associated with fracture  (MHPGMCET 2002) b. More common than intraperitoneal bladder rupture pelvis in about 70% cases c. Commonly associated with anterior urethral rupture d. Can be managed conservatively without surgical intervention 72. Urine gests collected in which place in case of extraperitoneal rupture of bladder: (AIIMS Nov 95) a. Groin b. Below urogenital diaphragm c. Above urogenital diaphragm d. Perineal space 73. In extraperitoneal rupture of bladder, urine extravasates in: a. Groin  (AIIMS Nov 94, All India 93) b. Intraperitoneal region c. Extraperitoneal region d. Perivesical space 74. Tear-drop bladder is seen in: (PGI June 99) a. Tuberculosis b. Hunner’s ulcer c. Perivescial hemorrhage with rupture d. Perivesical hemorrhage without rupture



77. A 40-year-old male was brought to the casualty with history RTa. Cystography was done. What is the name of sign seen in the image? a. Pear sign b. Flame sign c. Tear drop bladder d. Rim sign





78. Intraperitoneal bladder rupture management: a. Requires laparotomy  (Recent Question 2017) b. Antegrade cystogram is needed c. Simple catheter drainage is the treatment d. Conservative management



79. Most reliable investigation in bladder rupture is: a. IVP b. Cystoscopy c. Retrograde cystogram d. Catheterization

75. A 40-year-old male was brought to the casualty with history RTA. Cystography was done. What is the name of sign seen in the image? a. Pear sign b. Flame sign c. Tear drop bladder d. Rim sign

URINARY DIVERSION

Section 4



76. A 40-year-old male was brought to the casualty with history RTa. Cystography was done. What is the name of sign seen in the image? a. Pear sign b. Flame sign c. Tear drop bladder d. Rim sign

(JIPMER 98)



80. In ureterosigmoidostomy all occur except: a. Hyponatremia b. Hyperkalemia c. Hyperchloremia d. Acidosis



81. Urinary diversion is indicated in the following except?  (MHSSMCET 2008) a. Ectopia vesicae b. Carcinoma bladder c. Neurogenic bladder d. Bladder hematoma

82. Which among the following will complicate as hyperchloremic acidosis? (AIIMS June 98) a. Ureterosigmoidostomy b. Diarrhea c. Vomiting d. Ileoplasty

URINARY INCONTINENCE 83. Urinary incontinence results from all except: a. Neurogenic bladder b. Vesico vaginal fistula c. Ectopic ureter d. Rectovesical fistula

Urinary Bladder 84. Postmicturition dribbling is due to: (AMU 2005) a. Detrusor b. Dribbling decreased in case of urethral stricture c. Collection of urine in ‘U’ shaped curve of bulb of penis d. Neurogenic bladder



85. To differentiate between stress incontinence and detrusor instability investigation done is: (AIIMS June 97) a. Cystosurethroscopy b. Urodynamic study c. MCU d. Retrograde urethroscopy





93. Marion’s disease is due to: (DNB 90) a. Muscular hypertrophy of internal sphincter of urinary bladder b. Fibrosis of the neck of bladder c. Vesicular diverticula d. Vesicular calculi



94. What is the name of this operation? a. Lich-Gregoir operation b. Anderson Hynes operation c. Boari’s operation d. Leadbetter-Politano operation



95. What is Boari’s FLAP surgery? (MHSSMCET 2005) a. Ureterostomy b. DJ stent in situ c. Bowel interposition d. Flap of the bladder wall fashioned into a tube replace lower ureter 96. In Boari operation: (GB Pant 2011) a. Ureteric retransplant b. Lower ureteric reconstruction c. Diversion d. Bowel interposition

(Recent Question 2016)

Section 4



639

86. In which case cystometric study is indicated?(AIIMS Nov 98) a. Neurogenic bladder b. Stress incontinence c. Fistula d. Urge incontinence

MISCELLANEOUS 87. Which is a normal finding in cystometry? (PGI 97) a. Absence of systolic detrussor contraction b. Residual volume of 75 ml c. Leakage on coughing d. First sensation of urination at 300 ml 88. Commonest cause for pulsion diverticulum of the urinary bladder is: a. Benign enlargement of prostate b. Fibrous prostate c. Contracture of bladder neck d. Stricture urethra 89. Catheterization of bladder done in: (PGI Dec 2006) a. CA prostate b. Postoperative retention c. Preoperative before taking the patient for appendicitis d. Stricture e. Rupture 90. A young lady presents with symptoms of urinary tract infection. All of the following findings on a midstream urine sample support the diagnosis of uncomplicated acute cystitis, except: (All India 2011) a. Positive nitrite test b. CFU count < 1000/ml c. Detection of one bacteria/field on Gramstain d. > 10 WBC/HPF 91. Normal intravesical pressure during voiding:   (MHPGMCET 2007) a. 20–35 cm H2O b. 35–50 cm H2O c. 50–65 cm H2O d. 65–80 cm H2O 92. What is Marion’s disease? (MHSSMCET 2005) a. Benign prostatic hypertrophy b. Superficial thrombophlebitis of breast c. Bladder outlet obstruction d. Interstitial cystitis





97. Interstitial cystitis is also known as: (DNB 2012) a. Eosinophilic cystitis b. Radiation cystitis c. Hunner’s cystitis d. Tubercular cystitis



98. All are seen in cystitis except:



99. After a surgery, the surgeon asked the intern to remove the Foley’s catheter but he could not do it. The surgeon himself tried to remove the Foley’s catheter but he was unsuccessful. What should be done next? (AIIMS May 2016) a. CT guided rupture of bulb of Foley’s b. Inject ether to dissolve the balloon and pull it out c. Inject water to over distend the balloon until it bursts and Foley’s can be removed d. Use ultrasound guidance to locate and prick the balloon and then remove the catheter

b. Hematuria d. Nocturia

(DNB 2014)

Urology

a. Fever c. Dysuria

Explanations ECTOPIA VESICAE

1. Ans. d. Iliac bone (Ref: Smith 18/e p583; Campbell’s 11/e p3194; Bailey 27/e p1424-1425)

Posterior iliac osteotomy is done in ectopia vesicae.

2. Ans. a. Hypospadias

3. Ans. d. Ectopia vesicae

4. Ans. b. Cloacal membrane is present



5. Ans. c. Hypospadias

6. Ans. b. Ventral curvature of penis

7. Ans. b. Iliac bone

URINARY BLADDER STONES

8. Ans. b. Uric acid stones are dropped from above (Ref: Smith 18/e p275; Campbell 11/e 1291; Bailey 27/e p1434) Vesical Calculus Primary Bladder Calculi

Secondary Bladder Calculi

• Develop in absence of any known functional, anatomic or infectious factorsQ

• Develop in concert with bladder outlet obstructionQ, infection, impaired bladder emptying or a foreign body.

9. Ans. c. Infection (Ref: Smith 18/e p275; Campbell 11/e p1292; Bailey 27/e p1434)



10. Ans. a. Bladder stone (Ref: Bailey 27/e p1435; Smith 18/e p276; Campbell 11/e p1291-1292)



11. Ans. d. Maximum stones are radio-opaque (Ref: Smith 18/e p275; Campbell 11/e p1292; Bailey 27/e p1435)



12. Ans. c. Uric acid



13. Ans. b. Calcium oxalate dihydrate (Ref: Radiology Review Manual By Wolfgang Dähnert 7/e p1006) Spiculated stone or Jackstone (Child’s toy jack) are composed of calcium oxalate dihydrate and more common in bladder. Radiographic Shape of Stone Spiculated stone or Jackstone (Child’s toy jack)

Composition: Calcium oxalate dihydrate Location: Urinary bladder >Kidney

Mulberry stone

Mamillated contour Composition: Calcium oxalate dihydrate

Seed calculi

Small stones of similar size with lapidary effect (like cut gems) Formed in small cavity (calyceal diverticulum, cyst, hydronephrosis)



14. Ans. b. Treatment is litholapexy

15. Ans. d. Bladder



16. Ans. a. Always associated with recurrence (Smith 18/e p275; Campbell 11/e p1292; Bailey 27/e p1434)

URINARY BLADDER: MALACOPLAKIA

17. Ans. d. Defect in phagocytosis (Ref: Smith 18/e p212; Campbell 11/e p289)



18. Ans. c. Michaelis-Gutmann bodies are characteristic feature

URINARY BLADDER: TUBERCULOSIS

19. Ans. d. Whitish efflux from the ureteric holes (Ref: Smith 18/e p227; Campbell 11/e p427; Bailey 27/e p1442)



20. Ans. b. Chronic tuberculosis

21. Ans. c. Ileocystoplasty

SCHISTOSOMIASIS

22. Ans. a. Schistosomiasis (Ref: Smith 18/e p230; Campbell 11/e p437; Bailey 27/e p1442)



23. Ans. b. Schistosomiasis



25. Ans. a. Fetal head appearance (Ref: Schwartz 10/e p1286; Bailey 27/e p1444)

24. Ans. c. Schistosomiasis

Urinary Bladder

641

Schistosomiasis

CARCINOMA URINARY BLADDER: RISK FACTORS

26. Ans. c. Urinary bladder (Ref: Smith 18/e p312; Campbell 11/e p2187-2188; Bailey 27/e p1446)



27. Ans. a. Analgesic nephropathy

28. Ans. a. Schistosomiasis; b. Naphthylamine; c. Smoking



29. Ans. a. Stone; b. Schistosomiasis; c. Chronic cystitis

30. Ans. a. Urolithiasis; c. Schistosomiasis



31. Ans. b. Bladder exstrophy



32. Ans. a. Smoking predisposes; b. Schistosoma infection predisposes; c. Aniline dye workers; d. Radiation



33. Ans. a. Tuberculosis bladder; d. Chronic ulcer

Section 4

• Calcification in the wall of the bladder (Fetal head appearance)Q • Calcifications of the distal ureters have a characteristic pattern of linear or parallel calcificationsQ on plain radiographs

34. Ans. c. Benign (Ref: Bailey 27/e p1445, 26/e p1330) Benign Papillary Tumour

• ‘Kiss’ cancer: Papillary tumour with daughter implantation • The papilloma consists of a single frond with a central vascular core with villi; it looks like a red sea anemone • Inverted papilloma is a condition in which the proliferative cells penetrate under normal mucosa so that the lesion is covered with smooth urothelium. • It is benign.

35. Ans. b. HPV infection 36. Ans. c. Parasitic infection 37. Ans. a. Cyclophosphamide (Ref: Campbell 11/e p2188; Smith 18/e p310; Bailey 27/e p1446)

CARCINOMA URINARY BLADDER: TYPES

38. Ans. b. Radical cystectomy (Ref: Smith 18/e p313; Campbell 11/e p2225; Bailey 27/e p1451)



39. Ans. a. TCC



41. Ans. b. Solid tumour (Ref: Bailey 27/e p1448)

40. Ans. c. Transitional carcinoma

Squamous Cell Tumours • Squamous cell tumours tend to be solid and are nearly always associated with muscle invasionQ. • This is the most prevalent form of bladder cancer in areas where bilharzia is endemic. • Squamous cell tumours may be associated with chronic irritation caused by stone disease in the bladder as a result of metaplasiaQ.



42. Ans. b. Occurs more often in aniline dye workers, c. It is located most frequently in the trigone



43. Ans. a. Mucosa

44. Ans. c. Exposure to industrial carcinogens predisposes

CARCINOMA URINARY BLADDER: CLINICAL FEATURES AND DIAGNOSIS

45. Ans. a. Urine microscopy and cytology (Ref: Smith 18/e p313; Campbell 11/e p2197; Bailey 27/e p1447)



46. Ans. a. Hematuria



49. Ans. d. Transitional cell carcinoma of bladder



50. Ans. d. Urine microscopy for malignant cytology



52. Ans. b. Schistosomiasis is not a risk factor, d. Pain is the most common presenting feature

47. Ans. b. Hematuria 51. Ans. d. Urine for malignant cytology

48. Ans. a. Common in smokers

Urology

• Muscle-invasive tumours are nearly always solid although there may be a low tufted surface. • These tumours are often large and broad basedQ, having an irregular, ulcerated, appearance within the bladder. • The incidence of metastases, whether from lymphatic invasion in the pelvis or blood-borne to the lung, liver or bones, is much more common and will cause the death of 30–50% of patientsQ.

642

Surgery Essence

53. Ans. c. Urine tumor markers



56. Ans. a. Cystoscopy

54. Ans. c. Urothelial carcinoma

55. Ans. d. NMP-22

CARCINOMA URINARY BLADDER: TREATMENT

57. Ans. a. Local excision (Ref: Smith 18/e p316; Campbell 11/e p2207; Bailey 27/e p1450-1451)



58. Ans. b. 70% chance of requiring cystectomy in 5 years after TURP

Urology

• Treatment of stage II bladder cancer is radical cystectomy or neoadjuvant chemotherapy followed by radical cystectomy. • Rests of the three options are wrong.

59. Ans. c. Carcinoma of urinary bladder



60. Ans. d. Radical cystectomy



61. Ans. a. Endoscopic tumor resection



62. Ans. d. BCG (Ref: Smith 18/e p318, 17/e p316; Campbell 11/e p2212-2216; Bailey 27/e p1450, 26/e p1334) Drugs used in Intravesical Chemotherapy • Mitomycin CQ

• EpirubicinQ

• BCG (most effective)Q

• ThiotepaQ



63. Ans. b. BCG (Ref: Smith 17/e p316-317; Campbell 10/e p2343-2345; Bailey 27/e p1450)



64. Ans. a. Carcinoma of urinary bladder 65. Ans. a. Bladder



66. Ans. c. Total cystectomy and pelvic lymphadenectomy



67. Ans. b. Intravesical chemotherapy and immunotherapy are not found to be beneficial in Non Muscular Invasive Bladder Cancer (NMIBC)



68. Ans. b. Nd-YAG laser (Ref: Campbell 11/e p2210) “Laser coagulation allows minimally invasive ablation of tumors up to 2.5 cm in size. The neodymium : yttrium-aluminum-garnet (Nd : YAG) laser has the best properties for use in bladder cancer.”-Campbell 11/e p2210



69. Ans. c. MVAC (Methotrexate, Vinblastine, Adriamycin and Cisplatin)

URINARY BLADDER INJURY

70. Ans. d. Extraperitoneal rupture of bladder (Ref: Smith 18/e p290; Campbell 11/e p2386; Bailey 27/e p1428)



71. Ans. c. Commonly associated with anterior urethral rupture (Ref: Smith 18/e p290; Campbell 11/e p2368; Bailey 27/e p1425) Bladder Rupture Extraperitoneal (80%)

Section 4

Intraperitoneal (20%)

• MC cause is pelvic fracture . • Classic triad: Suprapubic pain and tenderness + Difficulty in ability to pass urine + HematuriaQ • Diagnosed by cystogram or CT cystographyQ • Flame sign or pear signQ (pattern of contrast extravasation) is seen • Treated by simple catheter drainage (Typically 10 days of catheter drainage will provide adequate healing time) Q • Surgical repair is indicated in cases of repeated blockade of catheter due to bleeding, projecting bone fragment or tear extending to the bladder neckQ. Q

• Cause: Blow, kick or fall on fully distended bladderQ • Usually seen in males, MC site of rent is dome of bladderQ. • Apart from classic triad suggestive of bladder rupture (Suprapubic pain and tenderness + Difficulty in ability to pass urine + Hematuria) patients develop peritonism and abdominal distentionQ. • Diagnosis is made by retrograde cystography or CT cystographyQ. • X-ray abdomen shows ground glass appearanceQ (due to fluid in abdomen) • Laparotomy with peritoneal lavage and bladder repair with SPC should be doneQ.



72. Ans. c. Above urogenital diaphragm

73. Ans. d. Perivesical space



75. Ans. b. Flame sign (Ref: Smith 18/e p290; Campbell 11/e p2386; Bailey 27/e p1425)



76. Ans. a. Pear sign

74. Ans. c. Perivesical hemorrhage with rupture

77. Ans. c. Tear drop bladder

78. Ans. a. Requires laparotomy (Ref: Campbell 11/e p2387; Smith 18/e p291; Bailey 27/e p1425)

79. Ans. c. Retrograde cystogram

Urinary Bladder

643

URINARY DIVERSION 80. Ans. b. Hyperkalemia (Ref: Smith 18/e p394, 401; Campbell 11/e p2344-2345; Bailey 27/e p1452-1453)



81. Ans. d. Bladder hematoma (Ref: Bailey 27/e p1452)

Indications of External Urinary Diversion • • • •

To relieve distal obstructionQ When the bladder has been removed or has lost normal neurological controlQ Incurable fistulaQ Irremovable obstructionQ

Section 4



82. Ans. a. Ureterosigmoidostomy

URINARY INCONTINENCE

83. Ans. d. Rectovesical fistula (Ref: Bailey 27/e p1430)

Rectovesical fistula doesn’t cause urinary incontinence, as the level of fistula is above the sphincter mechanism. Causes of Incontinence Problems of social control

• Uninhibited detrusor hyperreflexia and impaired social perception in dementiaQ

Storage problems

• Small bladder capacity owing to fibrosis (tuberculosis, radiotherapy or interstitial cystitis) Q • Small functional capacityQ owing to severe detrusor instability, neurogenic dysfunction or infection

Impairment of emptying

• Chronic retentionQ or neurogenic bladder dysfunctionQ have small functional bladder capacities with detrusor overactivity causing incontinence, despite having large residual volumes of urine.

Weak sphincter

• This leads to genuine stress incontinenceQ

Fistulae

• Leakage from fistulae (vesicovaginal) Q or upper tract duplication with an ectopic ureterQ.



84. Ans. c. Collection of urine in ‘U’ shaped curve of bulb of penis



85. Ans. b. Urodynamic study (Ref: Bailey 27/e p1432)

Detrusor Instability

Genuine Stress Incontinence • This is defined as urinary leakage occurring during increased bladder pressure when this is solely due to increased abdominal pressureQ and not to increased true detrusor pressure. • It is caused by sphincter weaknessQ.

Uses of Urodynamic Testing • • • •

To distinguish GSI (due to sphincter weakness) from detrusor instability in womenQ For the classification of neurogenic bladder dysfunctionQ To distinguish bladder outflow obstruction from idiopathic detrusor instability in menQ To investigate incontinence or other lower urinary tract symptomsQ

Urology

• Phasic increases in pressure give rise to urgency and urge incontinence (detrusor instability)Q. • This abnormality is found in patients with neurogenic bladder dysfunction, such as in multiple sclerosis (MS) or Parkinson’s disease or following a stroke or spinal injury, when it is known as detrusor hyperreflexiaQ. • About 50% of men with bladder outflow obstruction have detrusor instability, and in about half of them the instability resolves after prostatectomyQ. • Idiopathic detrusor instability is common and must be distinguished from genuine stress incontinence (GSI) in women before performing bladder neck suspension procedures.

644

Surgery Essence

86. Ans. a. Neurogenic bladder (Ref: Bailey 27/e p1429)

Cystometry • Cystometic studies are urodynamic studies in which the pressure changes in the bladder is simultaneously measured with bladder filling and during micturitionQ. • It helps in accurate assessment of detrusor and sphincter activity especially if a neurogenic abnormality is suspectedQ.

MISCELLANEOUS

87. Ans. a. Absence of systolic detrusor contraction

Rest three options are false.

Urology



88. Ans. c. Contracture of bladder neck (Ref: Bailey 27/e p1437)

Pulsion diverticula: The usual cause is bladder outflow obstruction.

89. Ans. a. CA prostate; b. Postoperative retention (Ref: Bailey 27/e p1437) • Retention of urine can occur after major and lengthy operations of the anal canal and perineal regionQ. So, in these types of operations, it is usual to forestall a catheter before or at the conclusion of the operation. • Appendicitis is not a major operation, so catheterization is usually not recommended. • Prostatic cancer presents with bladder outlet obstruction. This is relieved by catheterizationQ. • If the catheter will not pass, it is usually due to poor technique, lack of anesthesia, traumatization of urethra or a urethral stricture. • If the catheter cannot be passed, suprapubic puncture and urethral instrumentation should be done.



90. Ans. a. Positive nitrite test (Ref: Smith 18/e p200) Sensitivity and Specificity of Urinalysis in UTI



Tests

Sensitivity (%)

Specificity (%)

Esterase

83 (67–94)

78 (64–92)

Nitrite

53 (15–82)

98 (90–100)Q

Esterase or Nitrite

93 (90–100)

72 (58–91)

White blood cells

73 (32–100)

81 (45–98)

Bacteria

81 (16–99)

83 (11–100)

Any above

99.8 (99–100)

70 (60–92)

91. Ans. b. 35–50 cm H2O (Ref: Bailey 27/e p1431) • The normal voiding pressure should not exceed 60 cm H2O in men and about 40 cm H2O in women, with a flow rate of between 20 and 25 mL/sec.

Urodynamic Testing The principle is to artificially simulate bladder filling and emptying while obtaining pressure measurementsQ. The patient attends with a full bladder and is allowed to void in private to measure the maximum urinary flow rateQ. After voiding, the residual urine is measured by ultrasoundQ. Radiographic screening may be carried out to assess bladder neck closure and urinary leakage during movement or coughing (stress incontinence) or during bouts of phasic detrusor pressure (detrusor instability) Q. • The normal bladder will accept approximately 400–550 ml when filled at room temperature at a rate of < 50 ml/min. • The pressure increase in the bladder should be < 15 cm H2OQ. In addition, phasic pressure increases should not be seen. • The normal voiding pressure should not exceed 60 cm H2O in menQ and about 40 cm H2O in women, with a flow rate of between 20 and 25 ml/secQ.

Section 4

• • • •



92. Ans. c. Bladder outlet obstruction (Ref: Bailey 27/e p1467)

Marion’s Disease • Congenital obstruction of the posterior urethra due to muscular hypertrophy and stenosis of the bladder neck

Urinary Bladder

93. Ans. a. Muscular hypertrophy of internal sphincter of urinary bladder



94. Ans. c. Boari’s operation (Ref: Bailey 27/e p1415)

• A strip of bladder wall is fashioned into a tube to bridge the gapQ between the cut ureter and the bladder. • Used for lower ureteric reconstruction • Boari flap provides the needed length to create a tension-free anastomosis between the ureter and the bladderQ

95. Ans. d. Flap of the bladder wall fashioned into a tube replace lower ureter (Ref: Bailey 27/e p1415)



96. Ans. b. Lower ureteric reconstruction



97. Ans. c. Hunner’s cystitis (Ref: Bailey 27/e p1443; Smith 18/e p585)



98. Ans. a. Fever (Ref: Smith 17/e p206)



99. Ans. d. Use ultrasound guidance to locate and prick the balloon and then remove the catheter (Ref: http://www.aafp.org/afp/2000/0915/ p1397.html)

Section 4

Boari operation

645

Best technique in this situation is ultrasound-guided rupture of balloon.

Urology

CHAPTER

23

Prostate and Seminal Vesicles

BENIGN PROSTATIC HYPERPLASIA (BPH) Benign Prostatic Hyperplasia (BPH) Incidence and Epidemiology • BPH originates in the transition zoneQ, incidence is age relatedQ • The prevalence of BPH is 20% in 41–50 years, 50% in 51–60 years, > 90% older than 80 years. Etiology • Seems to be multifactorial and endocrine controlledQ. • Prostate is composed of stromal & epithelial elementsQ, and each, either alone or in combination, can give rise to hyperplastic nodulesQ and symptoms associated with BPH. Pathology • BPH develops in the transition zone. It is truly a hyperplastic processQ. • Nodular growth pattern is composed of varying amounts of stroma & epithelium. • Stroma is composed of collagen & smooth muscleQ. • Alpha-blocker therapy result in excellent responsesQ in patients with BPH having significant component of smooth muscle, while those with BPH predominantly composed of epithelium might respond better to 5-alpha-reductase inhibitorsQ. • Effects starts early with alpha-blockers whereas effect starts after 1 month and may take 6 months for maximum effect with 5-alpha-reductase inhibitorsQ. • Patients with significant components of collagen in the stroma may not respond to either form of medical therapy. • As BPH nodules in the transition zone enlarge, compress the outer zones of the prostate, resulting in the formation of a surgical capsule separating the transition zone from the peripheral zone, and serves as a cleavage plane for open enucleationQ of the prostate. Pathophysiology • Prostatic size on DRE correlates poorly with symptoms because the median lobe is not readily palpableQ. • Prostatic stroma, composed of smooth muscle & collagen, is rich in adrenergic nerve supply. The level of autonomic stimulation thus sets a tone to the prostatic urethra. Use of alpha-blocker therapy decreases this tone, resulting in a decrease in outlet resistanceQ. • Irritative voiding complaints of BPH result from the secondary response of bladder to the increased outlet resistanceQ. • Bladder outlet obstruction leads to detrusor muscle hypertrophy, hyperplasia & collagen depositionQ (collagen deposition is most likely responsible for a decrease in bladder compliance). Symptoms • Obstructive symptoms include hesitancy, decreased force & caliber of stream, sensation of incomplete bladder emptying, double voiding (urinating a second time within 2 hours of the previous void), straining to urinate & post-void dribblingQ. • Irritative symptoms include frequency, urgency & nocturia (FUN) Q • IPSS (International Prostatic Symptom Score): score can range from 0–35. Mild: 0–7, moderate: 8–19, severe: 20–35. Signs • Size & consistency of the prostate is noted. • BPH usually results in a smooth, firm, elastic enlargement of the prostate. • Induration is suggestive of cancer and the need for further evaluation (PSA, TRUS and biopsy)Q. Imaging • IVP or ultrasound is recommended only in the presence of concomitant urinary tract disease or complications from BPH (hematuria, urinary tract infection, renal insufficiency, history of stone disease). • Uroflowmetry: (Qmax >15 ml/sec is normal, 10–15 ml/sec is equivocal and < 10 ml/sec is suggestive of obstruction)Q • Cystometrograms and urodynamic profiles are reserved to differentiate outflow obstruction from neurogenic bladder (voiding pressure > 80 cm H2O signifies outlet obstruction)Q

Prostate and Seminal Vesicles

647

Section 4

Anatomy of the prostate gland

MEDICAL THERAPY IN BPH Medical Therapy in BPH • Alpha-blockers: (Prazosin, terazosin, doxazosin, tamsulosin, alfuzosin, silodosinQ) −− Relaxes smooth muscle & decreases urethral resistanceQ. −− Side effects: orthostatic hypotension, dizziness, tiredness, retrograde ejaculation, rhinitis & headacheQ. • 5-Alpha-reductase inhibitors: (Finasteride, dutasteride, triptorelin pamoate) −− Blocks the conversion of testosterone to dihydrotestosterone, affecting the epithelial componentQ of the prostate, resulting in a reduction in the size of glandQ and improvement in symptoms. • Six months of therapy are required to see the maximum effects on prostate size (20% reduction)Q and symptomatic improvement. However, symptomatic improvement is seen only in men with enlarged prostates (> 40 cm3)Q. −− Side effects: decreased libido, decreased ejaculate volume and impotenceQ.



Urology

• Combination therapy −− The reduction in risk associated with combination therapy (66% risk reduction) is greater than that associated with doxazosin or finasteride aloneQ. −− Patients most likely to benefit from combination therapy: whom baseline risk of progression is very high, generally patients with larger glands and higher PSA valuesQ.

648

Surgery Essence Absolute Indications for Surgery in BPH 1. Refractory urinary retentionQ (failing at least one attempt at catheter removal) 2. Recurrent UTIQ 3. Recurrent gross hematuriaQ 4. Bladder stonesQ 5. Renal insufficiencyQ 6. Minimal improvement on medical treatmentQ

TREATMENT OF BPH Treatment of BPH

Urology

• MC indication for surgery is symptoms interfering with quality of lifeQ (bothersome symptoms & symptoms of BOO).

Conventional Surgical Therapy • TURP (Gold standard) : −− Cystoscopic removal of strips of prostatic tissue using diathermy loop. −− Two techniques: NESBIT technique (preferred) & Mauer Mayer technique −− Best irrigant fluid is 1.5% glycineQ (Electrolyte solutions like NaCl are not compatible with electrocautery, so not used). −− Glycine is composed of glycolic acid and ammonium, which can cause CNS (visual) toxicity.Q −− TURIS: TUR in saline using bipolar cauteryQ −− Chips of prostate are removed by Ellik’s evacuator. Q

• Verumontanum is the single most important anatomical landmark in TURPQ. • Verumontanum lies immediately proximal to external sphincter and serve as the distal landmark for prostate resection to prevent injury to the external sphincter. • Verumontanum: Distal landmark for prostate resectionQ. • Verumontanum: Landmark for proximal limit of external sphincterQ. −− Resection proceeds at 1 gm/minute for an hour. −− Risks of TURP: Retrograde ejaculation (75%)Q, impotence (5–10%)Q & incontinence (< 1%)Q. −− Complications: Bleeding, urethral stricture or bladder neck contractureQ, perforation of the prostate capsule with extravasation, and if severe, TUR syndrome. TUR syndrome (Dilutional hyponatremia or water intoxication) • TUR syndrome (Dilutional hyponatremia or water intoxication)Q resulting from a hypervolemic, hyponatremic state due to absorption of the hypotonic irrigating solution. • Clinical Features: Nausea, vomiting, confusion, hypertension, bradycardia, & visual disturbancesQ. • The risk increases with resection times > 90 minutes or gland size > 75 gm. • Treatment includes diuresis (furosemide) and in severe cases, hypertonic saline (3%) administration. Late Complications of TURP • Bladder neck stenosis (4%) > Urethral stricture (3.6%) • Bladder neck stenosis is seen more often with small (< 30 gm) fibrotic prostates.

Section 4

• Transurethral incision of the prostate (TUIP): −− For posterior commissure hyperplasia (elevated bladder neck), involves two incisions using the Collins knife at the 5- and 7-o’clock positions. −− The incisions are started just distal to the ureteral orifices and are extended outward to the verumontanum. • TUIP lowers the incidence of bladder neck contracture when compared to TURP, so TUIP should be strongly considered in patients with smaller gland in place of TURP. • TUIP is used for smaller (20 gm) prostate, young patients. • Decreased incidence of retrograde ejaculation as compared to TURP. • Open simple prostatectomy: Glands > 75 gm, concomitant bladder diverticulum or a bladder stone or if dorsal lithotomy positioning is not possible. −− Suprapubic prostatectomy: Performed transvesically (Frayer’s)Q and operation of choice in dealing with concomitant bladder pathology (Bladder stones or diverticulum)Q. −− Retropubic prostatectomy (Millin’s)Q: Transverse incision is made in surgical capsule of prostate and enucleation is done. −− Perineal prostatectomy (Youngs)Q: Abandoned now • Carcinoma prostate originates in peripheral zone of prostate, so prostatectomy for BPH confers no protection for subsequent cancerQ.

Prostate and Seminal Vesicles

649

Minimally Invasive Therapy in BPH 4. Transurethral needle ablation 5. High intensity focused ultrasound 6. Intraurethral stents

ACUTE BACTERIAL PROSTATITIS Acute Bacterial Prostatitis

Section 4

1. Laser therapy: Two main energy sources of lasers have been utilized- holmium:YAG (best)Q, Nd:YAG. 2. Transurethral electrovaporization 3. Hyperthermia

Acute inflammation of prostate associated with UTI Caused by ascending urethral infection or reflux of infected urineQ into prostatic ducts MC organism: E. coliQ Patients present with sudden onset high grade fever with chills and rigors, severe irritative symptoms and enlarged, tender and boggy prostateQ • Catheterization and prostatic massage is contraindicatedQ • MC used antibiotics are: TMP-SMX and Ciprofloxacin (Both are having better concentration in prostatic tissue) Q • Around 4-6 weeksQ of antibiotic therapy is used to avert chronic bacterial prostatitis. • • • •

CHRONIC BACTERIAL PROSTATITIS Chronic Bacterial Prostatitis • Due to persistent bacterial infectionQ of prostate • Insidious in onset, characterized by relapsing or recurrent UTIQ caused by persistence of pathogen in prostatic fluid despite of antibiotic therapy • Diagnosis is made by microscopic examination and culture of prostatic expressate and culture of urine obtained before and after prostatic massageQ • Treated by chronic antibiotic suppression (3–4 months).Q

PROSTATIC ABSCESS Prostatic Abscess Most cases result from complications of acute bacterial prostatitisQ Fluctuation is a very late signQ Predisposing factors: Diabetes, renal insufficiency, immunosuppression, urethral instrumentation, chronic indwelling catheterQ Diagnosis: TRUS or pelvic CT scan is crucial for diagnosis and treatment Treated by transurethral drainage and antibioticsQ

PROSTATIC CALCULI Prostatic Calculi • • • • • •

Thought to represent calcified corpora amylaceaQ, which is composed of calcium phosphateQ Usually phosphate, seen on TRUS in nearly all elderly malesQ Lie at the periphery of transition zone (posterior and posterolateral zones of prostate) Usually asymptomatic due to peripheral location, tend to occur in clustersQ Calculi do not predispose to infectionsQ But if infections occurs in the presence of calculi, it is almost impossible to eradicateQ the as the calculi may get infected and serve as a source of bacterial persistence and recurrent UTI.

Urology

• • • • •

650

Surgery Essence PROSTATE CANCER Prostate Cancer • MC cancer of males, MC cause of bone secondariesQ • African-American men have highest incidence, less common in AsiansQ • Best screening protocol for CA prostate: PSA + DREQ Risk Factors • Advancing age & increased fat intakeQ increase the risk • Lycopene, Vitamin A & E and selenium decrease the riskQ • MC genetic alteration in CA prostate is hypermethylation of glutathione transferase (GSTP-1)Q gene promoter located on chromosome 11Q.

Urology

Pathology • MC type is adenocarcinomaQ >TCC • Prostatic adenocarcinoma are often multifocal & heterogenousQ • It is often accompanied by premalignant lesion PIN (prostatic intraepithelial neoplasia) • Neoplastic glands are smaller, more crowded and lack branching & papillary infoldings. • Diagnosis of CA prostate based on absence of basal cell layerQ. • Basal cell layer is present in normal glands, BPH glands and the precursor lesions of CA prostateQ. • Site: Peripheral zone- 75%Q, Transition zone- 15%, Central zone- 10% Spread • Spread occurs by direct local invasion and through hematogenous and lymphatics • Local invasion most commonly involves seminal vesicles & base of bladderQ • Hematogenous spread occurs mostly to bone (axial skeleton is MC site with lumbar spineQ being most frequently implicated) forming osteoblastic secondariesQ • Visceral metastasis most commonly involve lungsQ > liver > adrenal glands • Lymphatic metastasis are most often identified in obturator nodesQ • CNS involvement is usually a result of direct extension from skull metastasisQ Clinical Features • Most patients with early-stage CA prostate are asymptomatic, being peripheralQ. • Presence of symptoms suggest locally advanced or metastatic disease • DRE: Hard, irregular, nodular prostate with median sulcus obliterationQ Laboratory Findings • Azotemia (bilateral ureteral obstruction), anemia in metastatic disease, raised ALP in bony metastasis • PSA velocity >0.75 ng/ml/yr indicates carcinomaQ Prostate Biopsy • TRUS guided biopsy is done in patients with abnormal DRE or elevated PSA or bothQ. • Differentiation of tumor is graded by Gleason scoreQ. A sum of 7 or more suggests an aggressive cancer.

Section 4

Imaging • TRUS is used for staging, most lesions are hypoechoicQ. Endorectal MRI in CA Prostate • Most optimal imaging to appreciate the prostate anatomyQ • Prostate cancer is associated with lower levels of citrate & higher levels of choline and creatineQ compared to BPH or normal tissues. • The combined information provided by MRI & MR spectroscopy (for detection of citrate) may allow for a more accurate assessment of cancer location & stageQQ Axial imaging in CA Prostate • CT scan is mainly used to detect LN metastasis • Intravenous administration of superparamagnetic nanoparticles, which gain access to lymph nodes by means of interstitial-lymphatic fluid transport, at the time of high resolution MRI, appears to improve visualization of small nodal metastasisQ Bone Scan • Patients with PSA ≥ 15 ng/ml or greater, locally advanced disease (T3b, T4) are at higher risk for bone metastasis, and should be considered for bone scanQ.

Prostate and Seminal Vesicles

651

Prostascint (Antibody imaging for prostate cancer)

• True capsule: Formed by condensation of the peripheral part of the glandQ • False capsule: Fomed by visceral layer of the pelvic fasicaQ. Batson Periprostatic Venous Plexus • • • •

Located between two (true & false) capsule In front this plexus recieves deep dorsal vein of plexus & ends in internal iliac veinQ Has valveless communication with vertebral plexus, leading to early metastasis to lumbar vertebraQ Via hematogenous spread, prostate cancer first involve Batson periprostatic venous plexus & then into vertbral plexus of veinsQ

Section 4

• Indium-111 capromab pendetide is used in Prostascint • Radiolabeled monoclonal antibody directed against prostate specific membrane antigen • Helpful in detecting soft tissue metastasis including LN metastasis in biopsy proven prostate cancer which appear to be localized Capsules of Prostate

GLEASON SCORE AND GRADING SYSTEM Gleason Score and Grading system • Gleason score is the MC used histological grading system for prostate cancerQ. • The two most predominant histological patterns of the prostate cancer are assigned a Gleason grade ranging from 1–5Q. • Primary grade is assigned to the pattern of cancer that is most commonly observed in the histological slides of the specimenQ. • Secondary grade is assigned to the second most commonly observed pattern in the specimen. • Gleason score is the sum of the two grades. Thus it is also known as Gleason sumQ. • If the entire specimen has only one pattern present, then both the primary and secondary grades are reported as the same grade. • The Gleason grade ranges from 1 to 5, with 5 having the worst prognosisQ. • The Gleason score ranges from 2 to 10Q. • The Gleason score is used to help evaluate the prognosis of men with prostate cancer. Together with other parameters, the Gleason score is incorporated into a strategy of prostate cancer staging which predicts prognosis and help guide therapy. • A point of importance is that the primary Gleason grade is most importantQ with respect to placing patients in prognostic groups. • For example in patients with a Gleason score 7, a Gleason 4+3 is a more aggressive cancer than a Gleason 3+4.

TUMOR MARKERS OF CA PROSTATE (APART FROM PSA)

• Prostatic acid phosphatase: −− PAP activity is 1000 fold greater in the prostateQ than any other tissue −− PAP is not prostate specificQ and detectable levels are noted after prostatectomy −− Increased in renal, liver and bony malignanciesQ • Alkaline phosphatase: −− Raised in liver involvement or bony metastasisQ • Alpha-methyl co-A racemaseQ • HepsinQ • DD3Q

PROSTATE SPECIFIC ANTIGEN (PSA) Prostate Specific Antigen (PSA) • It is a glycoprotein, serine protease.Q • Free: 10–40%;Q Complexed to antiprotease: 60–90%Q • Formed in prostateQ, secreted in seminal fluid • Causes liquefaction of seminal coagulumQ

Urology

Tumor Markers of CA prostate (apart from PSA)

652

Surgery Essence • • • •

Normal value: ≤ 4 ng/mlQ (in > 50 years); Value > 20 ng/ml is diagnostic of CA prostateQ PSA is the single test with highest positive predictive value for CA prostateQ. PSA is prostate specific, not the cancer specificQ Level of PSA is directly related to tumor burdenQ

• Its use without DRE is not recommended as 25% of men with CA prostate have PSA levels 5% of tissue in resection for benign disease has cancer, normal DREQ T1c: Tumor identified by needle biopsy (e.g., because of elevated PSA)Q

N1: Metastasis in a regional LNsQ (obturator, internal iliac, external iliac, presacral LNs)

T2a: Tumor involves one half of one lobe but not both lobesQ T2b: Tumor involves more than one half of one lobe or less T2c: Tumor involves both lobesQ T3a: Extracapsular extension on one or both sides including bladder neck involvementQ T3b: Seminal vesicle involvementQ

M: Distant metastases M1a: Distant metastasis in non-regional lymph nodesQ M1b: Distant metastasis to boneQ M1c: Distant metastasis to other sitesQ

T4: Tumor is fixed or invades adjacent structures other than seminal vesicles: external sphincter, rectum, levator muscles and/or pelvic wallQ

Section 4

Treatment of CA Prostate T1a

• Incidentally found tumors at TURP, by definition low volume (≤5%), usually well-differentiated associated with very slow growth rateQ. • Managed by watchful waiting (Regular follow up with DRE & PSA)Q

T1b, T1c and T2

• Management depends on patient’s age, life expectancy, performance status and patient’s preference. • In younger, fitter men (70 years) with life expectancy 20 million sperm/mLQ. Sperm motility and sperm cytology or morphologyQ is another measure of semen quality. Sperm morphology is a sensitive indicator of overall testicular healthQ, because these characteristics are determined during spermatogenesis. Abnormalities

Section 4

Semen Analysis

• Low ejaculate volume may indicate: − Retrograde ejaculationQ −  Ejaculatory duct obstructionQ −− Incomplete collectionQ −  Androgen deficiencyQ • Azoospermia: −− Testicular failureQ −  Obstruction of vas deferensQ • Absence of fructose: −− Seminal vesicle agenesis or obstructionQ Semen Analysis-Minimal Standards of Adequacy Ejaculate volume: 1.5–5.5 mLQ Sperm concentration: >20 × 106 sperm/mLQ Motility: >50%Q Forward progression: 2 (scale 1–4) Q Morphology: >30% WHO normal forms (> 4% Kruger normal forms) No agglutination (clumping), white cells, or increased viscosityQ. Transrectal Ultrasound • High-frequency (5–7) mHz transrectal ultrasound (TRUS) offers super imaging of the prostate, seminal vesicles, and ejaculatory ductsQ. • Due to both accuracy and convenience, TRUS has replaced surgical vasography in the diagnosis of obstructive lesions that cause infertilityQ.

SPERM ASPIRATION Sperm Aspiration

• In cases of sperm aspiration from the testicle and epididymis, IVF along with intracytoplasmic sperm injection (ICSI) is requiredQ. • An obvious prerequisite for these procedures is ongoing sperm productionQ. • Although evaluated indirectly by hormone levels and testis volume, the most direct way to verify sperm production is with a testis biopsyQ. Vasal Aspiration • Vasal aspiration provides the most mature or fertilizable spermQ, as they have already passed through the epididymisQ, where sperm maturation is completed.

Epididymal Aspiration • Epididymal sperm aspiration is performed when the vas is not present or is scarred and unusableQ. • Epididymal sperm are not as mature as vasal spermQ; as a consequence, epididymal sperm require ICSI to fertilize the eggQ.

Testis Sperm Retrieval • The most recently developed aspiration techniqueQ • Indicated for patients in whom there is an unreconstructable blockage in the epididymis, or in cases of severe testis failureQ, in which so few sperm are produced that they cannot reach the ejaculate.

Urology

• Sperm aspiration techniques are indicated for men in whom the transport of sperm is not possible because the ductal system is absent or surgically unreconstructableQ. • An example of this is vasal agenesis. Acquired forms of obstruction may also exist, the most common of which is failed vasectomy reversalQ. • Aspiration procedures can involve microsurgery to collect sperm from the sperm reservoirs within the genital tractQ. • At present, sperm are routinely aspirated from the vas deferens, epididymis, or testicleQ.

Multiple Choice Questions BENIGN PROSTATIC HYPERPLASIA



1. Benign prostatic hyperplasia first develops in the: a. Periurethral transition zone (COMEDK 2011) b. Peripheral zone c. Central zone d. Anterior fibromuscular stroma















12. In follow up of BPH, most important indication of surgery is: a. Prostate size > 75 gm (AIIMS Nov 2010) b. Single episode of UTI requiring 3 days of antibiotics c. Cannot use medication due to hypertension d. Bilateral hydronephrosis



13. What of the following is an absolute indication for surgery in cases of benign prostatic hyperplasia?  (MHSSMCET 2005, All India 2003) a. Bilateral hydroureteronephrosis b. Nocturnal frequency c. Recurrent urinary tract infection d. Voiding bladder pressures > 50 cm of H2O

2. In BPH most common lobe involved is: (Recent Question 2016, WBPG 2015, AIIMS June 2000) a. Lateral b. Posterior c. Median d. Anterior

33.. Most common site of BPH: a. Peripheral zone c. Transition 
 zone



(Recent Question 2017) b. Middle zone d. Central zone

4. Which is the earliest symptom of benign hypertrophy of prostate? (Karnataka 94, 96) a. Frequency b. Hematuria c. Incontinence d. Strangury 5. Which of the following is an absolute indication for surgery in cases of benign prostatic hyperplasia? (All India 2003) a. Bilateral hydroureteronephrosis b. Nocturnal frequency c. Recurrent urinary tract infection d. Voiding bladder pressures > 50 cm of water 6. Which of the following lasers is used for treatment of benign prostatic hyperplasia as well as urinary calculi?  (All India 2003) a. CO2 laser b. Excimer laser c. Ho: YAG laser d. Nd-YAG laser 7. A 60-years old diabetic and hypertensive with second grade prostatism admitted for prostatectomy developed myocardial infarction. Treatment now would be: (All India 99) a. Finasteride b. Terazocin c. Finasteride and terazocin d. Diethyl stilbestrol 8. The following statements regarding finasteride are true except: (All India 2005) a. It is used in the medical treatment of benign prostatic hypertrophy b. Impotence is well documented after its use c. It blocks the conversion of dihydrotestosterone to testosterone d. It is a 5-α reductase inhibitor 9. Indication for surgery in benign prostatic hypertrophy are all except: (Recent Question 2016) a. Prostatism b. Chronic retention c. Hemorrhage d. Enlarged prostate 10. Grade I benign prostate with outflow obstruction is best treated with: a. Retropubic prostatectomy b. Transurethral resection c. Transvesicle prostatectomy d. Androgen therapy 11. Assessment of patient with prostatism include all except: a. Rectal examination (APPG 2013, DPG 2009 March) b. Serum prostate specific antigen c. Pressure flow urodynamic studies d. Transrectal ultrasound scanning

14. The drug that has the fastest onset of action in benign prostatic hyperplasia is: (COMEDK 2008) a. Finasteride b. Tamsulosin c. Dutasteride d. Flutamide 15. In the management of symptomatic benign prostatic hyperplasia with finasteride the period of trial required for determining a satisfactory response is: (COMEDK 2009) a. 1 month b. 2 months c. 4 months d. 6 months

16. Mechanism of action of Silodosin: (Recent Question 2016) a. Alpha antagonist b. Beta antagonist c. Anticholinergic d. PDE5 inhibitor

17. Which of the following drug can decrease the size of prostate? (AIIMS November 2017) a. Tamsulosin b. Sildenafil c. Finasteride d. Prazosin

TURP AND COMPLICATIONS 18. The most important use of transrectal ultrasonography (TRUS) is for: (COMEDK 2007) a. Screening for CA prostate b. Distinguishing prostate cancer from BPH c. Systematic prostate biopsy in suspected prostate cancer d. Guiding transurethral resection of prostate cancer

19. The most common complication of transurethral resection of prostate (TURP): (COMEDK 2007, 2008, 2009) a. Erectile dysfunction b. Retrograde ejaculation c. Urinary incontinence d. Impotence



20. Delirium, mental confusion and nausea in patients who had undergone transurethral resection of prostate suggests:  (MCI Nov 2017, Sept 2009) a. Hypernatremia b. Sepsis c. Hepatic coma d. Water retention



21. Which of the following is the most common cause of delayed urinary tract obstructive symptoms after TURP? a. Stricture of the navicular fossa (All India 2011) b. Stricture of the membranous urethra c. Stricture of the bulb of urethra d. Bladder neck stenosis

Prostate and Seminal Vesicles



23. What is the reason for following set of symptoms after prostatic surgery- restlessness, vomiting and change in sensorium? (AIIMS June 99) a. Electrolyte imbalance b. Bladder neck obstruction c. Acute pyelonephritis d. Ureter stenosis 24. TURP was done in an old patient of BHP, after which he developed altered sensorium cause is? (MCI June 2018, AIIMS June 2001, AIIMS June 99) a. Hypernatremia b. Hypokalemia c. Hyponatremia d. Hypomagnesemia

(PGI November 2017) a. More morbidity than retropubic prostatectomy b. Can cause retrograde ejaculation c. Open prostatectomy is preferred in larger obstructive mass d. Less risk of bleeding in transurethral laser vaporization than in TURP e. Resectoscope is passed through the urethra and prostate resected into multiple pieces and removed

CARCINOMA PROSTATE

33. Most common site of development of carcinoma of prostate is: (Recent Question 2015, DNB 2012, Orissa 2011, MHPGMCET 2002, 2001) a. Peripheral zone  b. Central zone  c. Transitional zone  d. Fibromuscular stroma 



34. On MRI, origin of carcinoma prostrate is seen in: (Recent Question 2017) a. Peripheral zone b. Central zone c. Transition zone d. Periurethral zone



35. A 49-years old man suffering from carcinoma of prostate was x-rayed. He showed areas of sclerosis and collapse of T10 and T11 vertebrae in x-ray. The spread of this cancer to the above vertebrae in X-ray. The spread of this cancer to the above vertebrae was through: (AIIMS Nov 2002) a. Sacral canal b. Lymphatic vessels c. Internal vertebral plexus of veins d. Superior rectal vein



36. Mr. Chaturvedi, a 70 years old man comes to casualty with urinary retention and back pain. Which investigation should be performed? (AIIMS Nov 2000, Nov 99) a. Serum acid phosphatase b. Serum calcium c. Serum alkaline phosphates d. Serum electrophoresis



37. A 65 years old male was diagnosed with prostate cancer three years back and was treated by surgery and hormone therapy. Presently he has developed urinary symptoms and progressive backache. What is the tumor marker, which can be indicative of disease relapse? (AIIMS Nov 2003) a. CA-125 b. Beta-HCG c. CEA d. PSA

38. Specific marker for prostatic cancer is:  (PGI Dec 99) a. Alkaline phosphatase b. Prostate specific antigen c. Acid phosphatase d. CA-125



39. Screening of prostate CA commonly done by:  (PGI Nov 2010, May 2005) a. DRE (digital rectal exam) b. USG c. MRI d. PSA e. CT scan



40. A 50 years old male with positive family history of prostate cancer has come to you for a screening test. The most sensitive screening test to pickup prostate cancer is: a. DRE (All India 2007) b. PSA c. DRE+ PSA d. Endorectal coil MRI with T1W and T2W images

Urology

25. Commonest cause of periumbilical pain after 30 min. of TURP done under spinal anesthesia with Bupivacaine: a. Meteorism (AIIMS June 2000) b. Perforation of bladder c. Recovery from bupivacaine anaesthesia d. Mesentery artery ischemia 26. Which of the following substances is not used as an irrigant during transurethral resection of the prostate?  (AIIMS Nov 2003) a. Normal saline b. 1.5% glycine c. 5% dextrose d. Distilled water 27. A 70 years old patient benign prostatic hyperplasia underwent transurethral resection of prostate under spinal anaesthesia. One hour later, he developed vomiting and altered sensorium. The most probable cause is:  (MHSSMCET 2008, AIIMS June 2001, All India 2003) a. Over dosage of spinal anesthetic agent b. Rupture of bladder c. Hyperkalemia d. Water intoxication 28. All of the following can be seen after transurethral resection of prostate except: (AIIMS Nov 2000) a. Congestive cardiac failure b. Transient blindness c. Convulsions d. Hypernatremia 29. TUR (Transurethral resection) syndrome is due to: a. Hyponatremia b. Hypokalemia(UPSC 95) c. Hypovolemia d. Hypoxia 30. Consider the following conditions: 1. Urinary flow rate < 10 cc/second 2. Residual volume of urine > 100 cc 3. Serum level of prostatic specific antigen > 10 mmol/litre 4. Trabeculated urinary bladder Which of the above are indications of TURP for BHP? a. 1, 2 and 3 b. 2, 3 and 4 c. 1, 2 and 4 d. 1, 3 and 4 31. Which one of the following is used as an irrigation solution during transurethral resection of the prostate? a. 1.5% glycine(COMEDK 2014) b. Physiological Saline c. Ringer’s lactate d. 5% dextrose

32. True about transurethral resection of the prostate:

Section 4

22. During TURP, surgeon takes care to dissect above the verumontanum to prevent injury to: (All India 2011) a. External urethral sphincter b. Urethral crest c. Prostatic utricle d. Trigone of bladder

655

656

Surgery Essence 41. Transrectal ultrasonogram in evaluation of carcinoma prostate is most useful for: (All India 2008) a. Taking guided biopsy b. Identifying seminal vesicle invasion c. Nodal sampling d. Measuring the extent of invasion

Urology







42. In prostatic metastasis, the site most commonly involved is: a. Obturator nodes (PGI June 99) b. Perivesical nodes c. Pre-sacral nodes d. Para aortic nodes 43. CA prostate commonly metastasizes to the vertebrae:  (All India 2001) a. Because valveless communication exist with Batson’s periprostatic plexus b. Via drainage to sacral lymph node c. Of direct spread d. None of above 44. Secondary deposits form prostatic carcinoma is commonest in: (Recent Question 2015) a. Bone b. Kidney c. Liver d. Brain 45. In carcinoma prostate with metastasis which is raised?   (Recent Question 2016) a. ESR b. Alkaline phosphatase c. Acid phosphatase d. Bilirubin

Section 4

46. A patient presents with complains of sciatica. On radiological examination there was sclerotic lesions on his skull. Which of the following is most likely to be elevated in this patient? (AIIMS 2000) a. CEA b. Prostate specific antigen c. Alkaline phosphatase d. Alpha-1 antitrypsin

47. Serum acid phosphatase is raised in:  (MCI June 2018) a. Osteosarcoma b. Prostatic carcinoma c. Paget’s disease d. Hyperparathyroidism



48. Normal level of PSA in males is: a. < 4 ng/ml b. 4–10 ng/ml c. > 10 ng/ml d. PSA is not produced by normal males



49. Gleason scoring is done for: ( Recent Question 2017, DNB 2009) a. Prostatic cancer b. Lung cancer c. Bladder cancer d. Hodgkins lymphoma

(DNB 2011)

50. Which of the following is true about prostate cancer screening? (APPG 2008) a. Digital screening along with PSA is additive b. Prostate cancer is common among young males c. Tumor markers are diagnostic d. Bleeding per rectum is earliest manifestation of disease



51. True about prostate CA is: (DPG 2008) a. Arises in the periurethral zone b. Extremely radio sensitive c. Obturator nodes are most commonly involved d. PSA is not used in workup 52. Gleason score: all are true except: (AIIMS May 2011, Nov 2008) a. Used for grading prostate cancer b. Scores range from 1–10 c. Higher the score, poorer the prognosis d. Helps in planning management



53. Osteoblastic metastasis commonly arise from: (JIPMER 2014, 2013, AIIMS May 2013) a. Breasts b. Prostate c. Lung d. RCC



54. Best screening marker of prostate cancer is: (UPPG 2009) a. AFP b. Prostate specific antigen c. CA 19-20 d. CA 125 to 26

55. The most important use of transrectal ultrasonography (TRUS) is for: (COMEDK 2007) a. Screening for CA prostate b. Distinguishing prostate cancer from BPH c. Systematic prostate biopsy in suspected prostate cancer d. Guiding transurethral resection of prostate cancer

56. Treatment for metastatic CA Prostate: a. GnRH analogue b. Estrogen therapy c. Radiotherapy with chemotherapy d. Radiotherapy

(JIPMER 2011)

CARCINOMA PROSTATE TREATMENT

57. Which is not used in carcinoma prostate? (PGI Dec 97) a. Estrogen b. Progesterone c. Cyproterone acetate d. Flutamide

58. An 85‑years old man underwent transurethral resection of prostate. A histological examination of his specimen showed foci of adenocarcinoma management will be: (AIIMS Nov 2000) a. Endocrine therapy b. Radical surgery c. Hormone therapy d. No further treatment 59. A 75-years old frail elderly man underwent TURP. The biopsy revealed adenocarcinoma. What is the next line of management? (All India 94) a. Radiotherapy b. Surgery followed by hormonal replacement therapy c. Conservative treatment d. Surgery followed by radiotherapy 60. Which of the following is the most troublesome source of bleeding during a radical retropubic prostatectomy? (All India 2005) a. Dorsal venous complex b. Inferior vesical pedicle c. Superior vesical pedicle d. Seminal vesicular artery 61. A 70-years old man with CA prostate with osteoblastic secondaries in pelvis and lumbar vertebra showed well differentiated Adeno Carcinoma prostate on needle biopsy. He is ideally treated by: a. Radical prostatectomy b. TURP c. Radiation d. Hormonal manipulation 62. Management of Carcinoma prostate in a 50-years old man revealed after TURP: (MHSSMCET 2006) a. No treatment required b. Hormonal therapy c. Bilateral subcapsular orchidectomy d. Radical prostatectomy

63. A 70-year-old man with prostate cancer was given radiotherapy. The recurrence of the cancer is monitored biochemically by: (AIIMS Nov 2012) a. Androgens only b. Prostate specific antigen and carcinoembryonic antigen c. Prostate specific antigen only d. ALP and CEA

Prostate and Seminal Vesicles

65. Which of the following drugs is useful for treatment of advanced prostate cancer? (AIIMS November, May 2014) a. Goserelin b. Ganirelix c. Cetrorelix d. Abarelix 66. Regarding prostatectomy which one of the statements is false: (APPG 2015) a. Water intoxication and hyponatremia can give rise to CHF b. Perineal prostatectomy (Young) is a commonly done surgical procedure c. Retrograde ejaculation occurs in about 65% of men d. Intraurethral stents are helpful in the management of men who are grossly unfit (ASA grade 4)

67. Sipuleucel-T is a vaccine for: (Recent Question 2017) a. RCC b. Testicular tumor c. Carcinoma prostate d. Carcinoma bladder



68. Ketoconazole action in prostate carcinoma is: (Recent Question 2017) a. Pituitary inhibition b. Adrenal ablation c. Inhibits DHT formation d. Androgen antagonist

PROSTATITIS

69. Complication which commonly accompanies acute prostatitis: (Recent Question 2016) a. Epididymitis b. Orchitis c. Seminal vesiculitis d. Sterility



70. A 60 years old male presented with fever, chills and dysuria. Patient was hospitalized in emergency for 5 days. PSA level was 7.4. Next best step in this patient: (AIIMS Nov 2013) a. Repeat PSA c. TRUS guided biopsy

b. TURP d. Antibiotics and admit

a. Artificial insemination of donor (All India 99) b. Penile-prosthesis c. Microtesticular aspiration and intracytoplasmic injection d. None of the above 75. A 55-year-old diabetic patient presented with impotence with history of failure to get erection after papaverine intracavernous injection. Color Doppler shows no abnormality of arteries but shows mild venous run-off. Treatment of choice: (All India 99) a. Intracavernous injection of papaverine b. Penile prosthetic implants c. Vacuum constriction device d. Psychotherapy

76. The most important in assessing fertility potential is: a. Sperm count b. Sperm motility c. Sperm morphology d. Quantity of ejaculated semen e. None of the above



77. Which of the following is true about obstructive azoospermia? a. ↑ FSH and ↑ LH (All India 2011) b. Normal FSH and normal LH c. ↑ LH, normal FSH d. ↑ FSH, normal LH

78. In a couple for treatment of infertility from the last four years, female partner is normal. Male partner has 0.8 ml semen volume per ejaculate on two repeated samples and absent fructose, with no sperms on examination under microscope. What is the next line of management? (AIIMS Nov 2013) a. Per-rectal examination to check ejaculatory duct obstruction b. Give antioxidants c. Testicular biopsy d. Transrectal ultrasound to detect duct obstruction

(Recent Question 2017)

(Kerala 94)

72. Semen analysis of a young man who presented with primary infertility revealed low volume, fructose negative ejaculate with azoospermia. Which of the following is the most useful imaging modality to evaluate the cause of his infertility?  (All India 2003) a. Colour duplex ultrasonography of the scrotum b. Transrectal ultrasonography c. Retrograde urethrography d. Spermatic venography



80. Corpora amylaciae is seen in: a. Thymus b. Lymph node c. Spleen d. Prostate



81. Hot flush is not associated with: a. Medical castration b. Surgical castration c. Ketoconazole therapy d. Androgen receptor blockade e. Radical prostatectomy

(PGI Dec 2008)



82. Medical castration is effected by: (Kerala 90) a. Deithylstilbesterol b. LHRH analogues c. Gossypol d. Hanovan



83. Complimentary operation done at the time of prostatectomy is: a. Vasectomy b. Circumcision c. Hernia repair d. All of the above



84. Prostate calculi are usually composed of: a. Calcium oxalate b. Calcium phosphate c. Struvite d. Uric acid

73. Absence of fructose in semen indicates: a. Obstruction to seminal vesicles b. Obstruction at prostatic urethra c. Vas deferens obstruction d. Testicular failure

74. A 25 years old married male presents with infertility. He had undergone retroperitoneal lymph node dissection at age of 15 years for embryonal carcinoma of right testis. Semen analysis shows-quantity-0.5 ml, no. sperm, no fructose. Biopsy of testis shows normal spermatogenesis. Best treatment here would be:

(PGI Dec 2008)

Urology

Bilateral vas deferens obstruction Ejaculatory duct obstruction Testicular failure Prostatic urethral obstruction

PROSTATE ANATOMY AND PHYSIOLOGY

INFERTILITY



79. Fructose absence in semen analysis suggests: a. b. c. d.

71. Most common organism responsible for acute bacterial prostatitis: (Recent Question 2018) a. E. coli b. Peptostreptococci c. Enterococci d. Streptococci agalactiae



Section 4

64. Treatment of metastatic prostate carcinoma is: (JIPMER 2011) a. Radiotherapy b. Estrogen only c. GnRH analogs d. Radiotherapy with chemotherapy

657

Explanations BENIGN PROSTATIC HYPERPLASIA Ans. a. Periurethral transition zone (Ref: Smith 18/e p350; Campbell 11/e p2433; Bailey 27/e p1458) Ans. c. Median Ans. c. Transition 
zone (Ref: Campbell 11/e p2433; Smith 18/e p350; Bailey 27/e p1458) 4. Ans. a. Frequency Ans. a. Bilateral hydroureteronephrosis, c. Recurrent urinary tract infection (Ref: Campbell 11/e p2509; Bailey 27/e p1463)



1. 2. 3. 5.



6. Ans. c. Ho: YAG laser (Ref: Smith 18/e p356; Campbell 11/e p2526)



7. Ans. b. Terazocin (Ref: Smith 18/e p354; Campbell 11/e p2473; Bailey 27/e p1464)



8. Ans. c. It blocks the conversion of dihydrotestosterone to testosterone 9. Ans. d. Enlarged prostate

10. Ans. b. Transurethral resection (Ref: Smith 18/e p356; Campbell 11/e p2510; Bailey 27/e p1464) 11. Ans. d. Transrectal ultrasound scanning (Ref: Bailey 27/e p1462; Smith 18/e p218) In prostatitis, DRE examination reveals tender, enlarged glands that are irregular and warm. PSA levels are often elevated. TRUS is only indicated in patients who do not respond to conventional therapy.

Let us Consider Every Option • In prostatitis, DRE examination reveals tender, enlarged glands that are irregular and warmQ. • PSA levels are often elevatedQ. • TRUS is only indicated in patients who do not respond to conventional therapyQ.

12. Ans. d. Bilateral hydronephrosis

13. Ans. a. Bilateral hydroureteronephrosis, c. Recurrent urinary tract infection



14. Ans. b. Tamsulosin

15. Ans. d. 6 months

16. Ans. a. Alpha antagonist

17. Ans. c. Finasteride (Ref: Goodman Gilman 12/e p308, 1205; Katzung 13/e p720) “Finasteride has been reported to be moderately effective in reducing prostate size in men with benign prostatic hyperplasia and is approved for this use in the USA.”-Katzung 13/e p720

TURP AND COMPLICATIONS

18. Ans. c. Systematic prostate biopsy in suspected prostate cancer (Ref: Rumack’s Diagnostic Ultrasound 3/e p411) Uses of TRUS in CA prostate • To guide biopsy

•  To guide therapy



19. Ans. b. Retrograde ejaculation (Ref: Smith 18/e p354; Campbell 11/e p2515; Bailey 27/e p1466) 20. Ans. d. Water retention 21. Ans. d. Bladder neck stenosis 22. Ans. a. External urethral sphincter



23. Ans. a. Electrolyte imbalance



25. Ans. b. Perforation of bladder (Ref: Bailey 27/e p1466)

24. Ans. c. Hyponatremia

Complications of TURP • • • •

Perforation of the bladder or the prostatic capsule can occur at the time of transurethral surgery. This usually occurs from a combination of inexperience in association with a large prostate or heavy blood loss. A large perforation with marked extravasation may require the insertion of a small suprapubic drain. If not detected, it may present postoperatively after the effect of spinal anesthesia, as suprapubic pain.

26. Ans. a. Normal saline (Ref: Campbell 11/e p2510)

The use of an ionic solution (i.e., normal saline) leads to dissipation of the cutting current and poor cutting efficacy

27. Ans. d. Water intoxication

28.

Ans. d. Hypernatremia



29. Ans. a. Hyponatremia

30.

Ans. c. 1, 2 and 4



31. Ans. a. 1.5% glycine (Ref: Campbell 11/e p2510)



32. Ans. b. Can cause……, c. Open prostatectomy…, d. Less risk of bleeding…, e. Resectoscope is passed…

Prostate and Seminal Vesicles

659

CARCINOMA PROSTATE 33. Ans. a. Peripheral zone (Ref: Smith 18/e p357; Campbell 11/e p2594; Bailey 27/e p1469)



34. Ans. a. Peripheral zone (Ref: Campbell 11/e p2396; Smith 18/e p357; Bailey 27/e p1469)



35. Ans. c. Internal vertebral plexus of veins



36. Ans. a. Serum acid phosphatase (Ref: Smith 18/e p360; Campbell 11/e p2602-2605; Bailey 27/e p1469)



37. Ans. d. PSA (Ref: Smith 18/e p360; Campbell 11/e p2602; Bailey 27/e p1471)



38. Ans. b. Prostate specific antigen



39. Ans. a. DRE (Digital rectal exam), d. PSA



40. Ans. c. DRE+ PSA



41. Ans. a. Taking guided biopsy



42. Ans. a. Obturator nodes



43. Ans. a. Because valveless communication exist with Batson’s periprostatic plexus (Ref: Campbell 11/e p2594)



44. Ans. a. Bone



45. Ans. c. Acid phosphatase



46. Ans. b. Prostate specific antigen



47. Ans. b. Prostatic carcinoma



48. Ans. a. < 4 ng/ml



49. Ans. a. Prostate cancer



50. Ans. a. Digital screening along with PSA is additive



51. Ans. c. Obturator nodes are most commonly involved

Section 4



52. Ans. b. Scores range from 1 to 10 (Ref: Smith 18/e p362; Bailey 27/e p1472; Campbell 11/e p2595) Gleason score ranges from 2 to 10.

53. Ans. b. Prostate (Ref: Harrison 19/e p580; Devita 9/e p2512-2513; CSDT 12/e p1202)

Osteoblastic metastasis commonly arises from carcinoma prostate. • • • •

MC site of primary for bone metastasis: CA BreastQ MC cause of osteoblastic secondaries in males: CA ProstateQ MC cause of osteoblastic secondaries in females: CA BreastQ MC tumor metastasize to bone in females: CA BreastQ

54. Ans. b. Prostate specific antigen



55. Ans. c. Systematic prostate biopsy in suspected prostate cancer



56. Ans. a. GnRH analogue (Ref: Smith 18/e p372) • In metastatic CA prostate, Androgen ablation (Hormone therapy) is first line of treatment: Orchiectomy + Flutamide or LHRH + FlutamideQ. • Leuprolide and goserelin are GnRH analogues, which are used primarily for the treatment of hormone-responsive prostate cancerQ.

CARCINOMA PROSTATE TREATMENT

57. Ans. b. Progesterone (Ref: Smith 18/e p371; Campbell 11/e p2789; Bailey 27/e p1474)



58. Ans. d. No further treatment

59. Ans. c. Conservative treatment



60. Ans. a. Dorsal venous complex

61. Ans. d. Hormonal manipulation



62. Ans. d. Radical prostatectomy



63. Ans. c. Prostate specific antigen only (Ref: Smith 18/e p371; Campbell 11/e p2617; Bailey 27/e p1473; Harrison 19/e p581) Cancer control (in CA Prostate) after radiotherapy has been defined by various criterias including: (Ref: Harrison 19/ep581) • A decline in PSA to less than 0.5 or 1 ng/ml • “Non-rising” PSA values • Negative biopsy of the prostate 2 years after completion of the treatment The current standard definition of biochemical failure (the Phoenix definition) is a rise in PSA by ≥ 2 ng/ml higher than the lowest PSA achieved

Urology



660

Surgery Essence

64. Ans. c. GnRH analogs



65. Ans. a. Goserelin (Ref: Katzung12/e p972; Goodman and Gilman 12/e p1763-1764; Smith 18/e p372; Campbell 11/e p291)

Goserelin is useful for the treatment of advanced prostate cancer. “The treatment of choice for patients with advanced prostate cancer is elimination of testosterone production by the testes through either surgical or chemical castration. Bilateral orchiectomy or estrogen therapy in the form of diethylstibestrol was previously used as firstline therapy. Presently, the use of GnRH agonists-including leuprolide and goserelin, alone or in combination with anti-androgen (e.g. flutamide, bicalutamide or nilutamide) is the preferred approach.”- Katzung12/e p972





66. Ans. b. Perineal prostatectomy (Young) is a commonly done surgical procedure

67. Ans. c. Carcinoma prostate (Ref: Campbell 11/e p2813; Smith 18/e p373)

Urology

“In prostate cancer, several immunologic strategies have been under clinical development. The most important of these include the sipuleucel-T (Provenge) autologous prostatic acid phosphatase (PAP)-loaded dendritic cell vaccine, the GVAX allogeneic recombinant whole cell vaccine, and CTLA-4 inhibitory approaches.”- Campbell 11/e p2813

68. Ans. b. Adrenal ablation (Ref: Campbell 11/e p2787; Smith 18/e p372; Bailey 27/e p1474)

PROSTATITIS

69. Ans. c. Seminal vesiculitis (Ref: Smith 18/e p218; Campbell 11/e p310, 313; Bailey 27/e p1474)



70. Ans. d. Antibiotics and admit (Ref: Smith 18/e p218; Campbell 11/e p310, 313; Bailey 27/e p1475) • A 60 years old male presented with fever, chills and dysuria. Patient was hospitalized in emergency for 5 days. PSA level was 7.4. This patient is most probably suffering from acute bacterial prostatitis and treated by antibiotics.



71. Ans. a. E. coli (Ref: Campbell 11/e p305; Smith 18/e p218; Bailey 27/e p1474) “The most common cause of bacterial prostatitis is the Enterobacteriaceae family of gram- negative bacteria, which originate in the gastrointestinal flora. The most common organisms are strains of Escherichia coli, which are identified in 65% to 80% of infections.”- Campbell 11/e p305

INFERTILITY

72. Ans. b. Transrectal ultrasonography (Ref: Smith 18/e p699) • • • •



Sperms are produced in seminiferous tubules and then stored and matured within the epididymisQ. Vas deferens carries the sperms from epididymis to the urethra where they open by separate openings into prostatic urethraQ. Just before opening, each vas deferens is joined by ducts of seminal vesiclesQ. Vas deferens and seminal vesicle ducts join to form the ejaculatory ductQ

73. Ans. a. Obstruction to seminal vesicles (Ref: Smith 18/e p696)

Section 4

• Absence of fructose in the semen indicates seminal vesicle agenesis or obstruction of its duct or the ejaculatory ductQ.

74. Ans. c. Microtesticular aspiration and intracytoplasmic injection (Ref: Smith 18/e p713)



75. Ans. c. Vacuum constriction device (Ref: Campbell 11/e p738)

Vacuum Erection Device Therapy • In patients who do not respond to or decline oral or local vasoactive pharmacotherapeutic optionsQ, vacuum erection device therapy may be alternatively explored. • The principle of vacuum erection device therapy is to mechanically create negative pressure surrounding the penis in order to engorge it with blood and then restrain blood egress from the organ to maintain the erection-like effectQ. • Efficacy rates in achieving satisfactory erections of 67–90%, but satisfaction rates with the device are lower, ranging from 34 to 68%. • The device is more acceptable to older men in a steady relationship compared with young, single men.

Prostate and Seminal Vesicles

• Further, it may offer a means to preserve the elasticity of penile tissues after priapism or penile prosthesis explantation or after surgical correction of Peyronie’s diseaseQ. • It has been suggested to facilitate erection recovery after treatments for prostate cancerQ.

76. Ans. b. Sperm motility



77. Ans. b. Normal FSH and normal LH (Ref: Shaw’s 13/e p203; Oxford textbook of Medicine 4/e p283)

Obstructive azoospermia is associated with normal levels of FSH and LH.

Section 4

• Success is limited in patients with severe vascular abnormalities such as proximal venous leakage or arterial insufficiency or fibrosis secondary to priapism or prosthesis infectionQ.

661

Azoospermia



Obstructive Azoospermia

Hypogonadotrophic Azoospermia

Hypergonadotrophic Azoospermia

Azoospermia due to obstruction: • Normal FSH and LHQ • Normal testosteroneQ • Testicular volume is usually normalQ

Azoospermia due to hypothalamic or pituitary failure: • Low FSH and LHQ • Low testosteroneQ • Testicular volume is usually reducedQ

Azoospermia due to testicular (end organ) failure: • Persistently elevated FSHQ • Testicular volume is usually reducedQ

78. Ans. d. Transrectal ultrasound to detect duct obstruction (Ref: Smith 18/e p713)

• Absent fructose with no sperms in ejaculate is suggestive of obstruction of vas deferens with seminal vesicle agenesis or obstruction. Next line of management in this patient would be transrectal ultrasound to detect duct obstruction. • “High-frequency (5-7) mHz transrectal ultrasound (TRUS) offers superb imaging of the prostate, seminal vesicles, and ejaculatory ductsQ. • Due to both accuracy and convenience, TRUS has replaced surgical vasography in the diagnosis of obstructive lesions that cause infertilityQ.”

79. Ans. b. Ejaculatory duct obstruction (Ref: Campbell 11/e p599; Smith 18/e p709)

PROSTATE ANATOMY AND PHYSIOLOGY

80. Ans. d. Prostate (Ref: Smith 18/e p369; Campbell 11/e p305; Bailey 27/e p1456-1457)



81. Ans. e. Radical prostatectomy (Ref: Bailey 27/e p1473; Campbell 11/e p2794)

Hot Flashes



82. Ans. a. Diethylstilbestrol, b. LHRH analogues (Ref: Smith 18/e p372) Androgen Ablation Therapy for Prostate Cancer Level

Agent

Pituitary

• DiethylstilbestrolQ

•  GoserelinQ

Adrenal

• KetoconazoleQ

•  AminoglutethimideQ

Testicle

• Orchiectomy

Prostate cell

• BicalutamideQ

•  LeuprolideQ

Q

•  FlutamideQ     •  NilutamideQ



83. Ans. None



84. Ans. b. Calcium phosphate (Ref: Smith 18/e p277; Campbell 11/e p305; Bailey 27/e p1468)

Urology

• Hot flashes (also called hot flushes, vasomotor symptoms) have been recognized as a side effect of androgen ablation (medical or surgical)Q • Described as a subjective feeling of warmth in the upper torso and head followed by objective perspiration • Are among the most common side effects of androgen ablationQ, affecting between 50% and 80% of patients • Treatment of hot flashes should be reserved for those who find them bothersomeQ. • Drugs used for treatment: DES, oral progestogens, cyproterone acetate, clonidine, sertraline, venlafaxineQ

CHAPTER

24

Urethra and Penis

HYPOSPADIAS Hypospadias • Hypospadias results when fusion of urethral folds is incomplete, and urethral meatus opens on the underside of penis or perineum (ventral surface of penis)Q. • Occurs in 1:250 male births and multifactorialQ in inheritance. • Hypospadias is MC congenital malformation of urethraQ. • Estrogens & progestins given during pregnancy increase the risk. • Anterior forms are more commonQ then posterior because fusion of urethral folds is from posterior to anterior. • 70% cases are distal penile or coronalQ. • Circumcision is not done in patients with hypospadias, as the prepuce can later be used in surgical repairQ. • Glanular • Coronal • Subcoronal

Types of Hypospadias • Penile • Scrotal • Penoscrotal • Perineal

• Surgical pathology in addition to ventrally placed ectopic meatus, hypospadias has: • Chordee ventral curvature of penis due to contracture of fibrous cord which has replaced the distal urethra and corpus spongiosum. Severity of chordee is proportional to degree of hypospadiasQ. • Hooded prepuce deficient on ventral aspect and excess on dorsal aspectQ. • Stenosis of ectopic meatusQ • Multiple urethral orifices • Flattening of glansQ • Microphallus Associated Abnormalities • Undescended testisQ (10%) with or without sexual ambiguity • Inguinal herniaQ (10%) • Urinary tract abnormality (upper and lower) Associated Problems • Abnormal stream, painful erection (chordee) & infertility (in proximal or posterior types)Q Management • Treatment is not required in anterior variety. (repair is done for cosmetic reasons only) • Optimal time of repair is 6–12 monthsQ. • Meatal advancement or local skin flap advancementQ is the surgical procedure done along with removal of chordee. Named Procedures in Hypospadias • • • • •

Dennis-Brown technique (Two stage) Q MAGPIQ (Meatal advancement and glanuloplasty integrated) for coronal or subcoronal Mathiew procedureQ (Perimeatal based flap, one stage) for distal penile Asopa or DuckettQ technique using vascularized preputial island (one stage) Thiersch-Duplay or BrackaQ technique for proximal penile

Urethra and Penis

663

Complications of Surgery

• Incidence of hypospadias: 1 in 250Q • Incidence of horse-shoe kidney: 1 in 400 (0.25%)Q • Incidence of renal agenesis: 1 in 1000Q

EPISPADIAS

Section 4

• MC complications is urethral fistulaQ (10%) • Meatal stenosisQ due to devascularisation of distal neourethra • Urethral stricture and stenosis due to poor vascularity of flap, persistent chordeeQ

Epispadias • Urethra opens on the dorsum (upper aspect) of the penis in males, in females there is a fissure in the wallQ of the urethra which opens above the clitoris Associated Anomalies • Extrophy of bladderQ (ectopia vesicae) with pubic diastasis and waddling gait • Dorsal ChordeeQ • VUR in 40% casesQ Clinical Features • Females: Bifid clitoris & separation of the labia. Most are incontinentQ because of maldevelopment of the urinary sphincters. • Males: Patients with glandular epispadias seldom have urinary incontinence. However, incontinence in penopubic is 95% and penile epispadias is 75%Q. • Epispadias is a mild form of bladder exstrophy, and in severe cases, exstrophy and epispadias coexist. Management • Surgery is required to correct the incontinence, remove the chordee to straighten the penis, and extend the urethra out onto the glans penisQ. • Bladder augmentation combined with the artificial sphincter may be required in patients in whom incontinence cannot be corrected.

POSTERIOR URETHRAL VALVE Posterior Urethral Valve • Symmetrical folds of urothelium extending distally from prostatic urethra to external urinary sphincterQ.

• Acts as a flap valveQ (one way valve)- allows catheter, but balloons out during micturition and obstructs stream. Clinical Features • • • • •

Newborns may present with palpable abdominal masses (distended bladder, hydronephrotic kidneys & ascites)Q Infants with urinary infection & sepsisQ. Sometimes, the valves are incompleteQ and the patient remains without symptoms until adolescence or adulthood. Approximately 30% of patients experience end stage renal disease.Q Vesicoureteral reflux occurs in 50% of patientsQ.

Associated with • • • •

OligohydramniosQ Renal parenchymal dysplasia (most important factor in overall prognosis)Q Abnormal bladder function (25%)Q Pulmonary hypoplasia (MC cause of death)Q

Investigations • Investigation of choice: MCUQ (shows proximal urethral dilatation with distal stricture) • Cystoscopy: Shows dilation of urethra above valve. • Prenatal diagnosis by ultrasound, showing bilateral hydroureteronephrosis with enlarged thickened bladder as early as 28 weeksQ of gestation (Keyhole sign on prenatal USG).

Urology

• Exclusively an anomaly of male urethraQ. • Four types, Type 1 is MC (lies just distal to the verumontanum or at the verumontanum)Q

664

Surgery Essence Management • First a small polyethylene feeding tube is inserted in the bladder and left for several days. Then further management is done according to serum creatinine level. −− Normal serum creatinine - transurethral ablation (endoscopic fulgration)Q of the valves −− Increased serum creatinine and the worsening of condition – vesicostomy (Blockson’s technique is best) to bypass the obstruction and when normal creatinine levels are achieved, transurethral ablation (endoscopic fulgration)Q is done. Prognosis • Bladder dysfunction and renal hypoplasia is associated with poor prognosis and is major cause of progressive renal failureQ. • Pulmonary hypoplasia is MC cause of deathQ.

Urology

PHIMOSIS Phimosis • Phimosis is a condition in which the contracted foreskin cannot be retracted over the glansQ. • Chronic infection from poor local hygiene is its most common causeQ. • Most cases occur in uncircumcised males, although excessive skin left after circumcision can become stenotic and cause phimosis. Types • Congenital • Acquired: Usually presents late in life and associated with inflammation, balanitis xerotica obliterans, trauma or cancerQ. Clinical Features • • • •

Difficulty in micturitionQ is the main symptom. Ballooning of prepuce during micturitionQ is suggestive of phimosis. Edema, erythema, and tenderness of the prepuce and the presence of purulent discharge usually cause the patient to seek medical attention. Inability to retract the foreskin is a less common complaint.

Complications • Balanoposthitis, Hydronephrosis or hydroureter • Prepucial calculi, carcinoma under foreskinQ Treatment • Local steroid cream for 4–6 weeksQ. • Circumcision should be done if no response to steroids, recurrent balanitis or balanoposthitis, age > 16–18 yearsQ. • If phimosis is associated with considerable infection, it should be treated with broad-spectrum antimicrobial drugs. The dorsal slit of foreskinQ, if improved drainage is necessary. • Circumcision for phimosis should be avoided in children requiring general anesthesia; except in cases with recurrent infectionsQ. • The procedure should be postponed until the child reaches an age when local anesthesia can be used.

PARAPHIMOSIS

Section 4

Paraphimosis • Acquired condition in which the foreskin, once retracted over the glans, cannot be replaced in its normal position. • It is uncommon for the urethra to be compressed, so the micturition is normally not affectedQ. Pathology • Chronic inflammation under the redundant foreskin leads to contracture of preputial opening (phimosis) and formation of a tight ring of skin when the foreskin is retracted behind the glans. • The skin ring causes venous congestion leading to edema and enlargement of the glansQ. • As the condition progresses, arterial occlusion & necrosis of the glansQ may occur. Treatment • Ice bags, gentle manual compressionQ and injection of a solution of hyaluronidase in normal saline may help to reduce swelling. • Circumcision: If conservative method fails

Urethra and Penis

665

PRIAPISM

• Painful, persistent erectionQ not normally associated with sexual excitement or desire, which does not subside after sexual excitement or desire. • Most patients present with an erection of at least 24 hoursQ duration. • Priapism is an emergency, if therapy is delayed for 36–48 hours, then marked tissue damage (due to ischemia) is likely to occur with cavernosal fibrosis and impotenceQ. Types • High flow (non-ischemic) priapismQ: Occurs secondary to penile or perineal trauma, arterial sinusoidal shunt within corpus cavernosum. • Low flow (ischemic) priapismQ: Painful priapism, can lead to compartment syndrome, more common than high flow Priapism, caused by Sickle cell anemia, leukemia, spinal cord lesions, fat emboli, malignant penile inflammationQ, autonomic neuropathy, drugs (Trazadone)Q • Majority due to vasoactive intracorporeal injectionsQ

Section 4

Priapism

Clinical Presentation: Two peak ages • Children 5–10 years old: Most common due to sickle cell diseaseQ, attacks are usually nocturnal and the patient awakens with painful erection. • Adults 20–50 years: Mostly iatrogenicQ, priapism involves only corpora cavernosa. The spongiosum and glans are flaccid. Diagnosis • Diagnosis is mainly clinicalQ, only investigation useful to supplement clinical examination is doppler. Treatment • If present early, within 4–6 hours: KetamineQ (dissociative anesthesia) causes 50% detumescence. • Aspiration and saline irrigation till the aspirate is bright red, followed by injection of diluted phenylephrineQ. • Active treatment in high flow priapism as it represents a compartment syndromeQ. • Selective internal pudendal arteriography and selective embolization of the artery feeding the shunt for high flow non-ischemic priapismQ • Operative intervention: • Winter’s procedure (percutaneous cavernoglandular shunt)Q −− Corpora spongiosa shunt −− Corpora saphenous shunt Surgical Management of Ischemic Priapism (surgical creation of shunt to allow blood to drain from the corpora cavernosa) Corporo-glanular shunt

Corporo-spongiosal shunt • Shunts created between corpora cavernosa and corpora spongiosaQ • These are proximal shunts and are performed in rare circumstances Examples: • Quackel or Sacher shuntQ

Corporo-saphenous shunts • Shunts created between corpora cavernosa and saphenous veinQ • Rarely performed Examples: • Grayhack shuntQ

Corporo-dorsal vein shunt • Shunt between corpora cavernosa and superficial or deep dorsal vein of penisQ • Rarely performed Examples: • Barry shuntQ

PEYRONIE’S DISEASE Peyronie’s Disease • Peyronie’s disease (plastic induration of penis/ penile fibromatosisQ) usually seen over 40 years of age. • It is due to fibrous plaques in one or both corpus cavernosum of varying sizes involving tunica albugineaQ which may later calcify or ossify. • Cause remains obscure, the dense fibrous plaque is microscopically consistent with findings of severe vasculitis. • Palmar fibromatosis (Dupuytren’s contracture), plantar fibromatosis and penile fibromatosis (Peyronie’s disease) are components of the same pathological process called superficial fibromatosisQ. • Galezia’s Triad: (DPR) Dupuytren’s contracture + Peyronie’s disease + Retroperitoneal fibrosisQ

Urology

• Shunts created between glans penis and corpora cavernosaQ. • These are distal shunts and represent the first line surgical therapy. Examples: • Winter shuntQ • AI-Ghorab shuntQ • Ebbehoj and T shunt

666

Surgery Essence Clinical Features • Painful erection, curvature of penis and poor erection distal to involved areaQ. • No pain when the penis is in nonerect state. • Palpable induration or mass appears usually on the dorsolateral aspectQ of the penis. Treatment • Spontaneous remission occurs in about 50% casesQ, so observation & emotional support advised initially. • If the penile deformity is distressing, Nesbitt’s operationQ can be performed to straighten the penis. • Nesbitt operation: Straightening of penis by placing non-absorbable sutures in corpus cavernosum opposite to the plaque.

URETHRAL INJURY

Urology

Urethral Injury • Urethral injuries occur most often in men, usually associated with pelvic fractures or straddle type falls. • Urethra is separated in two anatomic divisions: − Posterior urethra: Prostatic urethra + Membranous urethraQ − Anterior urethra: Bulbous urethra + Penile urethraQ

INJURIES TO POSTERIOR URETHRA Injuries to Posterior Urethra

Etiology • The part of urethra most likely injured in pelvic fracture is membranous urethraQ. • Membranous urethra is sheared at the bulbomembranous or prostatomembranous junctionQ. • Bulbomembranous junction is more prone than prostato membranous junction during pelvic fracture (posterior urethra is densely adhered to pubis via urogenital diaphragm & puboprostatic ligaments) Clinical Features • Retention of urine + Blood at urethral meatus + Pelvic hematoma and High lying prostateQ • Presence of blood at external urethral meatus indicates that immediate urethrographyQ is necessary to establish the diagnosis. • Associated with deep extravasation of urine in pelvis & retroperitoneal tissuesQ. • Pie in sky appearanceQ on IVP in membranous urethral injury. • Superior displacement of prostate does not occur if the puboprostatic ligaments remain intact. Instrumental Examination • The only instrumentation involved should be for urethrographyQ. • Catheterization or urethroscopy should not be done in every case (as its associated with an increased risk of hematoma, infection, and conversion of partial urethral tear into complete transection of urethra)Q. • In suspected partial injury, gentle single attempt to catheterize the patient acts as a stent over which urethra healsQ. Complications • Bladder rupture may be associated with posterior urethral injuries in 20% of casesQ. • Stricture, impotence, and incontinenceQ are complications of prostatomembranous disruption. • Stricture following primary repair and anastomosis occurs in about 50% of cases, with delayed repair incidence of stricture can be reduced to about 5%Q. • The incidence of impotence after primary repair is 30–80% , can be reduced to 30–35% by delayed urethral reconstructionQ. Diagnosis

Section 4

• When blood is noticed at meatus, immediate RGU should be done to rule out urethral injury.

Treatment Immediate Management • No urethral instrumentation or manipulation and suprapubic cystostomy • In suspected partial injury, gentle single attempt to catheterize the patient acts as a stent over which urethra heals. • Incomplete laceration of the posterior urethra heals spontaneously, and the suprapubic cystostomy can be removed within 2–3 weeksQ. • SPC remains the gold standard for initial management, endoscopic alignment can be done over guidewire if patient presents within 7–10 daysQ. Delayed Urethral Reconstruction • Reconstruction of the urethra after prostatic disruption can be undertaken within 3 monthsQ.

Urethra and Penis

Section 4

Injury to the posterior (membranous) urethra

667

INJURIES TO ANTERIOR URETHRA Injuries to Anterior Urethra Etiology • Direct blow to the perineumQ is the mechanism of injury. • Straddle injuryQ may cause laceration or contusion of the urethra. • Self-instrumentation or iatrogenic instrumentation may cause partial disruptionQ. Pathology and pathogenesis: MC site is bulbar urethra. A. Contusion • Is a sign of crush injury without urethral disruption. Perineal hematoma usually resolves without complications. B. Laceration • A severe straddle injuryQ may result in laceration of part of the urethral wall, allowing extravasation of urine. • If the extravasation is unrecognized, it may extend into the scrotum, along the penile shaft, and up to the abdominal wall and limited only by Colles’ fascia and often results in sepsis, infection, and serious morbidityQ. Clinical Features • Retention of urine + Blood at urethral meatus + Perineal hematoma and Normal prostate on P/RQ • Superficial extravasation of urine, urine collects in scrotum, anterior perineum, beneath superficial fascia of penis and spreads under fascia scarpaQ. • Massive urinary extravasation and infection in the perineum and scrotum in delayed presentation Complications

Treatment • Urethral contusion: −− After urethrography, if the voiding occurs normally, without pain or bleeding, no additional treatment. • Urethral lacerations: −− Suprapubic cystostomyQ for complete urinary diversion while the urethral laceration heals. • Urethral laceration with extensive urinary extravasation: −− Suprapubic cystostomy for urinary diversion and antibiotic therapy for Infection and abscess formation Prognosis • Urethral stricture is a major complication but in most cases do not require surgical reconstructionQ.

STRICTURE URETHRA Stricture Urethra • Area of narrowing in the caliber of urethra due to formation of scar in the tissues surrounding the urethra • Male urethra is more prone to trauma & strictureQ formation. • Large catheters and instruments are more likely than small ones to cause ischemia, internal trauma leading to strictureQ. Etiology • Traumatic (MC)Q, straddle injuries for anterior and pelvic fracture for posterior urethra

Urology

• Heavy bleeding from corpus spongiosum injury and urethral meatusQ. • Sepsis and infectionQ due to urinary extravasation (Aggressive debridement and drainage for infection) • Stricture at the site of injuryQ

668

Surgery Essence

Urology

• Inflammatory or infectious (Infection from long-term catheter useQ is a major cause, gonococcal urethritis is seldom a cause of strictures today) • Ischemia • Malignant • Congenital (Fossa navicularis and membranous urethra is MC siteQ of congenital urethral stricture) Pathophysiology • Healing occurs by scar formation after injury to urothelium • Process of scar formation occurs in spongy erectile tissuesQ (corpus spongiosa) of the penis that surrounds urethra (spongiofibrosisQ • Scar tissue contracts, reduces the caliber of urethral lumen, causing resistance to antegrade flow of urine. Clinical Features Obstructive voiding symptoms • Decreased force of urinary stream, improving with pressure (MC)Q • Spraying or double streamQ • Incomplete emptying of the bladder, terminal dribbling and urinary intermittencyQ • Urinary retention, dilation of proximal urethra and prostatic ducts, urinary tract infections Complications • Prostatitis, cystitis, urinary diverticula and chronic UTI • Severe prolonged obstruction result in decompensation of UV junction resulting in reflux, hydronephrosis and renal failureQ • Urethral fistula and periurethral abscessQ commonly develop in association with chronic severe strictures Diagnosis • Location, length, depth and density of stricture should be evaluated for appropriate treatment. • Retrograde urethrogram or MCUQ to demonstrate location and extent of stricture RGU MCU and RGU, both are required for adequate assessment MCU for posterior urethra, RGU for anterior urethra Cystourethroscopy for visualization of stricture High frequency ultrasound for short bulbar strictures (more accurate in measuring stricture length than RGU and is helpful in determining whether to excise or graft) • MRI for defining the distorted pelvic anatomy associated with posterior urethral stricturesQ resulting from trauma. Management • • • •

• Periodic urethral dilations to stretch the scar without producing additional scarring. • Internal urethrotomy: Incising the stricture transurethrally using endoscopic equipment to release scar tissue. The incision is made under direct vision at 12 O’clock position with urethrotome, with curative success rate of 20–35% (Good success in short strictures without spongiofibrosisQ) Open Reconstruction • Excision & re-anastomosis for strictures ≤ 2 cmQ: Complete excision of the fibrotic segment with a widely spatulated tension-free reanastomosis, most dependable technique • Excision & tissue transfer for strictures > 2 cmQ: Full-thickness skin graft tissue is harvested from the desired non-hair bearing location, penile skin, bladder epithelium, or buccal mucosa (MC and best results)Q. • Urethral stents: Endoscopically placed, designed to be incorporated into wall of urethra and provide a patent lumen.

CARCINOMA PENIS Carcinoma Penis • Most commonly occur in 6th decade of life, but “40% patients are less than 40 years” • Most commonly associated etiologic factor is poor hygieneQ • PhimosisQ is commonly associated (50%)

Section 4

Q

• Neonatal circumcision confers immunityQ against CA penis, HIV or STDs, but not if done later. • Most important carcinogens are smegma and HPV infection (16, 18, 31, 33)Q. Premalignant Lesions • Buschke-Lowenstein tumourQ (Verrucous carcinoma): Tumour destroy adjacent tissue by compression, no metastasis usually. (Locally Malignant) • Balanitis Xerotica ObliteransQ: Whitish patch on glans, meatus and urethra, meatal stenosis. • LeukoplakiaQ (more common in diabetics) • Cutaneous horn • Long standing genital warts

Urethra and Penis

669

Carcinoma in Situ

Clinical Features • • • • • •

Squamous cell carcinoma (80%)Q is the MC type, most commonly originates from glansQ >prepuce>sulcus>shaft (GPS). Others are transitional cell carcinoma (15%), basal cell carcinoma, malignant melanoma, sarcoma. MC symptom is lesion itself associated with foul smelling discharge. Phimosis is associated in 50%. There is little or no pain. Lesions are typically confined to penis at the time of presentation.

Section 4

• Bowen’s diseaseQ: Intraepithelial skin neoplasm (solitary thickened, grey white plaque with ulceration and scabbing) with HPV association in 80% cases. Converts into infiltrating SCC in 10%. No high incidence of visceral malignancy. When it involves glans and prepuce, it is called Erythroplasia of QueyratQ • Erythroplasia of QueyratQ: Red velvety plaques over glans or prepuce, treated by 5% 5-FU cream or ND YAG laser.

• More than 50% patients of CA Penis presents with enlarged inguinal lymph nodesQ. • 50% of patients presenting with enlarged lymph inguinal lymph nodes are reactive (non-metastatic), used to subside after 4-6 weeks of antibioticsQ. • Priapism is the MC and earliest symptom of metastatic CA penisQ. • MC cause of death is bleeding caused by erosion of femoral artery by metastatic inguinal lymph nodesQ. • 2nd MC cause of death is sepsis. • HypercalcemiaQ is seen in absence of osseous metastasis in 20% of patients, appears to correlate with volume of diseaseQ. Patterns of Spread • Buck’s fasciaQ and tunica albuginea represents a barrier to corporal invasion and hematogenous spread. • Primary dissemination is to inguinal, femoral and iliac LNs. • • • •

Prepuce and shaft skin drain into the superficial inguinal LNsQ (Superficial to tensor fascia lata). Glans and corporal bodies drain to both superficial and deep inguinal LNsQ (deep to tensor fascia lata). Anterior urethra to inguinal LN and posterior urethra to internal iliac LNs. Penile drainage is bilateral because of multiple croos-connectionsQ.

• Penetration of buck’s fascia and tunica albiginea leads to invasion of vascular corpora and vascular dissemination (rare). • Distant metastases in < 10% cases, may involve lung, liver bone or brain. Diagnosis • Good incisional biopsyQ from the periphery of the lesion from its junction with the normal tissue for grade and depth of invasion is mandatory for diagnosis. • Sentinel lymph node biopsy (CABANA procedure) is done for inguinal LN status. Radiological Investigations • MRI is IOC for staging in CA penis. Treatment • Without any treatment of invasive carcinoma, death within 2 years. • Small non-invasive lesion involving prepuce: 5-FU cream, Nd-YAG laser, radiotherapy + close follow-up is mandatory/ Wide excision or circumcision • Lesions involving glans or distal shaft: Partial penectomy with 2 cm marginQ • Lesions involving proximal shaft or 2 cm margins are not achieved: Total penectomy with perineal urethrostomyQ. • Bilateral Ilioinguinal LN dissectionQ for metastatic lymph nodes. • Chemotherapy used are Bleomycin, 5-FU, Cisplatin, methotrexate. • Radiotherapy for selected superficial small lesions. Prognosis • Survival correlates with presence or absence of nodal diseaseQ.

Stage I Stage II Stage III Stage IV

Jackson (Extent of spread)Q Staging for CA Penis Confined to glans or prepuce Extension to shaft Operable inguinal LN metastasis Inoperable inguinal LN metastasis or local or advanced spread

Urology

• Assessment of depth by USG or MRIQ (CT is not effective)

Urology

670

Surgery Essence

8th AJCC (2017) TNM Staging for CA Penis T: Primary tumor

N: Regional lymph nodes

Tis: Carcinoma in-situQ Ta: Noninvasive verrucous carcinomaQ

N1: Palpable mobile unilateral inguinal LNQ

T1a: Invades subepithelial connective tissue without lymphovascular invasion and is not poorly differentiatedQ T1b: Invades subepithelial connective tissue with lymphovascular invasion or poorly differentiatedQ

N2: Palpable mobile multiple or bilateral inguinal LNsQ N3: Fixed inguinal nodal masses or pelvic lymphadenopathy unilateral or bilateralQ

T2: Invades corpus spongiosum with or without invasion of urethraQ T3: Invades corpus cavernosum with or without invasion of urethraQ T4: Invades other adjacent structures

M: Distant metastases M0: No distant metastasis M1: Distant metastasis present

Q

Stage 0

Stage I

Stage IIA

Stage IIB

Stage IIIA

Stage IIIB

Stage IV

Tis N0M0 Ta N0M0

T1a N0M0

T1b N0M0 T2 N0M0

T3 N0M0

T1-3 N1 M0

T1-3 N2 M0

T4 anyN M0 Any T N3 M0 Any T any N M1

CARCINOMA MALE URETHRA Carcinoma of Male Urethra • Chronic irritation and infection are the strongest risk factors • Incidence of urethral stricture in men with development of urethral cancer: 24–76%Q • HPV-16Q has a causative role in SCC of urethra • MC presenting symptom: Palpable mass associated with obstructive voiding symptomsQ Pathology Q

• MC site is bulbomembranous urethraQ> penile urethra> prostatic urethra

Section 4

• Overall, MC type is SCCQ >TCC > adenocarcinoma. • MC type of carcinoma prostatic urethra are TCCQ >SCC. • MC type of carcinoma penile urethra are SCCQ >TCC. • Lymphatics of the anterior urethra drain into the superficial and deep inguinal LNsQ • Lymphatics of the posterior urethra drain into external iliac, obturator and hypogastric LNsQ. Diagnosis • MRI with gadolinium is IOC for evaluating local soft tissue, LNs metastasisQ Treatment • Surgery is mainstay of treatmentQ. Radiotherapy is also used. • Ilioinguinal node dissection only in presence of palpable adenopathy.

Multiple Choice Questions HYPOSPADIAS 1. True about hypospadias: (JIPMER 2011) a. Associated with chordee b. 50% associated with undescended testis c. Due to failure of fusion of posterior wall of urethra d. Circumcision done immediately





2. True about hypospadias: a. Defect seen in ventral penis b. Always associated with chordee c. Associated with hooded prepuce d. Circumcision should be avoided



4. True about hypospadias is all except: a. Sex determination is not possible b. Prepuce is hooded c. Incidence is 1 in 3000 male births d. May not require surgical treatment

5. Most common congenital anomaly of urethra: (MHSSMCET 2008) a. Hypospadias b. Epispadias c. Meatal stenosis d. PU valve



6. Commonest hypospadias is:  (DNB 2011, 2001, AIIMS Dec 95) a. Penile b. Glandular c. Scrotal d. A or C

7. All are true about hypospadias, except: (AIIMS June 93) a. Circumcision in infancy is contraindicated b. Avoid surgery till puberty c. No treatment required in glandular variety d. If associated chordee is present, 2 stage operation is done 8. The best time for surgery of hypospadias is: (All India 2003) a. 1–4 months of age b. 6–10 months of age c. 12–18 months of age d. 2–4 years of age



13. In severe hypospadias the possibility of an intersex problem is settled by: a. Careful inspection of genitals b. Biopsy for gonadal tissue c. Karyotyping d. Hormone assay



14. All of the following are seen in hypospadias except:  (All India 96) a. Ectopia vesicae b. Hooded prepuce c. Chordee d. Infertility



15. Which of the following urethral anomaly is the most common? (TN 99) a. Hypospadias b. Pin hole meatus c. Epispadias d. Stricture urethra

(MHPGMCET 2003)





12. Penis is curved in downward direction in all types of hypospadias except: (Recent Question 2016) a. Glandular b. Coronal c. Penile d. Perineal

(PGI May 2010)

3. Circumcision is contraindicated in:  (NEET 2013; WBPG 2012; GB Pant 2010) a. Paraphimosis b. Meatal stenosis c. Hypospadias d. Phimosis





9. Features of hypospadias are all except: a. Chordee (WBPG 2014, All India 98) b. Hooded prepuce c. No-treatment required with glandular variety d. Cryptorchidism

16. Which of the following is true regarding hypospadias? (APPG 2015) a. It is attributed to failure of complete urethral tubularisation in the fetus b. Urethral opening is most commonly in the perineum c. Urethra opens proximally and dorsally d. It is seen in 1 in 1500 boys

17. Read the following statements and choose the appropriate answer:

Statement A (Assertion) - Ritual circumcision is contraindicated in infants with hypospadias Statement R (Reasoning) - In hypospadias, foreskin is deficient dorsally and there is a variable degree of chordee  (APPG 2016) a. Statements A & R are both Correct but R is not the Reasoning for A b. Statement A is wrong but Statement R is Correct c. Statements A & R are both Correct and R is the reasoning for A d. Statement A is Correct but Statement R is wrong

18. Which of the following developmental defects of the urogenital sinuses never occurs in the female? (APPG 2016) a. All these defects can occur b. Hypospadias c. Ectopia vesicae d. Epispadias



19. Best age for hypospadias repair: (Recent Question 2017) a. Before 1 year b. 2 years c. 4 years 
 d. 5 years



10. In hypospadias all are seen except:(BIHAR 2014; PGI Dec 99) a. Hooded penis b. Dorsal chordee c. Spatulated glans d. Meatal stenosis



20. Name of surgery in hypospadias: (Recent Question 2017) a. Dennis-Brown b. Ombridann’s c. 
Ladd and Gross d. Keetley-Torek



11. Which is not true of hypospadias? (AIIMS 92) a. Chordee is reversed after 5 years b. Glandular type needs no treatment c. Circumcision should not be done d. Surgical correction has good results in infancy



21. Order of correction of hypospadias: (JIPMER November 2017) a. Straightening of penis – Balanoplasty – Urethroplasty b. Urethroplasty – Balanoplasty – Straightening c. Straightening – Urethroplasty – Balanoplasty d. Balanoplasty – Urethroplasty – Straightening

672

Surgery Essence EPISPADIAS

22. Epispadias is associated with: (All India 2008) a. Bifid pubic symphysis b. Chordee c. Anal atresia d. Intestinal obstruction



23. Epispadias in relation to hypospadias: a. Is more common b. Less common c. Occurs with the same frequency d. Is difficult to treat



33. A five years old child presents with ballooning of prepuce after micturition. Examination of penis reveals prepucial adhesions. Which of the following the best treatment: a. Adhesiolysis and dilatation (All India 2011) b. Circumcision c. Dorsal slit d. Conservative management



34. Which of the following statement is false about the given conditions?

Section 4

Urology

POSTERIOR URETHRAL VALVE

24. All are true about posterior urethral valve except: a. Most common in boys (GB PANT 2011) b. Can be detected by prenatal USG c. Early catheterization should be done d. Diagnosed by early urethroscopy



25. A three years old boy presents with poor urinary stream. Most likely cause is:  (AIIMS June 2003) a. Stricture urethra b. Neurogenic bladder c. Urethral calculus d. Posterior urethral valve



26. A male child with recurrent UTI with dribbling of urine most likely cause is:  (Punjab 2011) a. VUR b. Posterior urethral valves c. Stricture urethra d. Neurogenic bladder

a. b. c. d.

Both 1 & 2 can be congenital Both 1 & 2 can be acquired Both 1 & 2 can be treated by circumcision None of the above

PARAPHIMOSIS



27. For posterior urethral valve-investigation of choice is:  (BIHAR PG 2014; AIIMS June 97, PGI Dec 2002,  Dec 2003, June 2003) a. Cystoscopy b. MCU c. Cystourethroscopy d. Retrograde urethroscopy



35. Not true about paraphimosis is: a. Iatrogenic b. Seen in diabetes mellitus c. Gangrene of glans d. Circumcision is the treatment



28. Posterior urethral valve are commonly observed in: a. Boys b. Girls c. Adult males d. Adult females





29. Most common uropathic obstruction in children is: a. Stricture b. Stones (UPPG 2009) c. Posterior urethral valve d. Anterior urethral valve

36. Which of the following are TRUE regarding the picture depicted here of a patient who underwent Foley’s catheterization?  (APPG 2016) (P) Obstruction of venous and lymphatic return from glans (Q) Commonly due to sickle cell anemia (R) The condition occurred due to the doctor forgetting to replace the retracted prepuce (S) Previous circumcision is the cause of this condition here



30. Most common location of posterior urethral valve: (PGI SS 2004, MHPGMCET 2008) a. Proximal to verumontanum b. Distal to verumontanum c. At the level of verumontanum d. At the bladder neck



31. Posterior urethral valve is usually seen: a. Above verumontanum (MHSSMCET 2005, 2006, 2008) b. Below verumontanum c. At the level of bladder neck d. At the level of verumontanum

PHIMOSIS 32. The recommended treatment for preputial adhesions producing ballooning of prepuce during micturition in a 2 years old boy is: (AIIMS June 2003) a. Wait and watch policy b. Circumcision c. Dorsal slit d. Preputial adhesions release and dilatation

a. PQRS are all true c. Only P and R are true

37. About paraphimosis true is: a. Catheter induced b. Circumcision is treatment c. Hyaluronidase injection d. All of the above

(AIIMS June 98)

b. Only R and S are true d. Only P and S are true (PGI June 96)

Urethra and Penis CIRCUMCISION 38. Circumcision is done in a child in which of the following conditions: (TN 91) a. Phimosis b. Recurrent balanitis c. Paraphimosis d. All of the above



39. Indications of circumcision are all except: (MHPGMCET 2002) a. Chronic balanoposthitis b. Jew religion c. Carcinoma penis d. Paraphimosis 40. All are true regarding circumcision except: (JIPMER November 2017) a. Hemorrhage due to bleeding from frenular artery b. Increases sexual drive c. Avoid correction of congenital anomaly d. Reduces sexually transmitted infections

PRIAPISM 41. The Grayhack shunt is established between: (All India 2010) a. Corpora cavernosa and corpora spongiosa b. Corpora cavernosa and saphenous vein c. Corpora cavernosa and dorsal vein d. Corpora cavernosa and glans







42. Persistent priapism is rarely seen as a consequence of: (COMEDK 2010) a. Sickle cell disease b. Leukemia c. Spinal cord disease d. Prolonged sexual activity 43. In Priapism, cavernous blood study will reveal: a. PO2 < 30%, PCO 2 >60%  (MHSSMCET 2009) b. PCO2 > 60% c. PCO2 > 60% d. PO2 < 60% 44. In children persistent priapism may result due to: a. Thrombosis of venous plexus b. Leukemia c. Wilm’s tumour d. Trauma

PEYRONIE’S DISEASE

46. Palpable fibrous plaque on dorsal penile shaft indicates:  (DPG 2005, Karnataka 95) a. Paget’s disease b. Potter’s syndrome c. Prehn’s sign d. Peyronie’s disease



47. This condition is associated with which of the following?



49. All are true about Peyronie’s disease except: a. Self limiting (UPPG 2007, 2006) b. Medical treatment is effective c. Association with Dupuytren’s contracture d. Calcified plaques



50. Peyronie’s disease is: (MHPGMCET 2007) a. Browning of penis b. Ectopic opening of urethra c. Curved deformity of penis due to fibrous plaque d. Absent glans penis



51. Nesbitt’s operation is done for: (MHPGMCET 2009, 2005) a. Ectopic testis b. Hypospadias c. Peyronie’s disease d. Any of the above



52. Peyronie’s contracture is seen in: (Recent Question 2014; 2013) a. Dupuytren’s disease b. Hypospadias c. Epispadias d. Exstrophy

URINARY TRACT INFECTION











53. What will be next investigation to be done in case of a 2 years old female child with 1st episode of UTI? (AIIMS June 98) a. Abdominal ultrasound b. DMSA scan c. 6 monthly urine culture d. Nothing actively needed 54. A child with recurrent urinary tract infection is most likely to show: (All India 2005) a. Posterior urethral valves b. Vesicoureteric reflux c. Neurogenic bladder d. Renal and ureteric calculi 55. Which fruit juice helps in preventing UTI? (AIIMS Nov 2011, Nov 2006) a. Grape b. Raspberry c. Cranberry d. Orange 56. Commonest organism giving rise to urinary tract infection: (Recent Question, 2017) a. E. coli b. Proteus c. Staphylococcus d. Streptococcus 57. Urinary tract infection exists when the bacterial count in 1 ml. midstream specimen of urine is: (Recent Question, 2016) a. 100 b. 1000 c. 104 d. 105 or over 58. The most reliable urine specimen is obtained by: (UPSC 2005) a. Urethral catheterization b. Catheter aspiration c. Midstream voiding d. Suprapubic aspiration

URINARY RETENTION

a. Retroperitoneal fibrosis c. Both of the above

b. Dupuytren’s contracture d. None of the above



59. Acute urinary retention in a male child may be due to:  (Recent Question 2016) a. Prostatic radiotherapy b. Urethral stricture c. Hysteria d. Meatal ulcer with scabbing



60. Acute onset of anuria in elderly men: a. Bilateral infraction of kidneys b. Obstructive urinary disease c. Acute tubular necrosis d. Acute cortical

Urology

45. Priapism in a polytrauma patient signify: a. Penile injury  (Recent Question 2017) b. Spinal cord injury c. Significant head injury d. Pelvic injury

48. The following statements are true about Peyronie’s disease except: (AIIMS Nov 2002) a. Patient presents with complaints of painful erection b. Condition affects adolescent males c. The condition can be associated with Dupuytren’s contracture of the tendon of the hand d. Spontaneous regression occurs in 50% of the cases

Section 4





673

674

Surgery Essence



61. Most frequent causes of acute retention of urine include all except:  (DPG 2009 March) a. Meatal ulcer with scabbing in children b. Haemorrhoidectomy c. Herniorrhaphy d. Fecal impaction 62. Urinary retention in child is most commonly caused by: a. Metal scab with ulceration (PGI Dec 2003) b. Posterior urethral valve c. Urethral stricture d. Epispadias e. Congenital short penis

URETHRAL INJURY 63. A 25 years old male presents to emergency department following a road traffic accident. On examination there is pelvic fracture and blood at urethral meatus. Following are true about patient except: (AIIMS Nov 2002) a. Anterior urethra is the most likely site of injury b. Retrograde urethrography should be done after the patient is stabilized c. Foley catheter may be carefully passed if the RGU is normal d. Rectal examination may reveal a large pelvic

Urology



64. Not true about urethral injuries is: (AIIMS Nov 2001) a. Catheterize the patient immediately b. Can be associated with fracture pelvis c. Bladder injury is associated with post urethral injuries d. Blood at the external urethral meatus is an imp feature

Section 4





65. All of the following can be done in a case of pelvic fracture with pelvic hematoma and had not passed urine since trauma except:  (AIIMS Nov 99) a. Pass indwelling urethral catheter b. IV fluid infusion c. IV pyelography d. Digital per rectal examination



66. Membranous urethral rupture causes collection of blood in: a. Ischiorectal fossa (Recent Question, 2014; AIIMS Nov 93) b. Deep perineal pouch c. Superficial inguinal region d. Pelvic diaphragm



67. All are true about bulbar urethral rupture, except: a. Perineal hematoma (DNB 2011, AIIMS June 93) b. Floating prostate on per rectal examination c. Collection of urine in perineum d. Bleeding per urethra 68. Following trauma, a patient presents with a drop of blood at the trip of urinary meatus. He complains of inability to pass urine. Next step should be:  (All India 2001) a. IVP should be done b. MCU should be done c. Catheterize, drain bladder, and remove the catheter thereafter d. Catheterize, drain bladder and retain in catheter thereafter



69. In case of pelvic fracture with urethral injury, the most important first step in management is: a. Repair in injured urethra b. Fixation of pelvic fracture c. Treatment of shock and hemorrhage d. Splinting urethra with catheters













70. Commonest late complication of traumatic rupture of urethra is:  (JIPMER 92) a. Diverticulum b. Retrograde ejaculation c. Stricture d. Chordee 71. Rupture of membranous urethra occurs more commonly due to: (AIIMS 92) a. Thin unsupported wall b. Fixity of urethra c. Angulation d. Proximity to bladder 72. Treatment of fracture pelvis with rupture urethra is: a. Suprapubic cystostomy (Kerala 95) b. Explore and correct the fracture, repair urethra c. Catheterization d. Urethrogram to assess injury 73. Urine extravasation occurs in the following in case of penile urethral rupture, except: (JIPMER 2003) a. Ischiorectal fossa b. Scrotum c. Abdomiadias d. Below superficial fascia of penis 74. A young man gets into a fight after taking beer and is kicked by the lower abdomen. There was pelvic fracture. Blood at meatus. Most likely cause is: (MCI Nov 2017) a. Rupture of membranous urethra b. Bulbar urethral injury c. Kidney laceration d. Ureteric injury 75. All the features of membranous urethral injury except: a. Blood of meatus b. Retention of urine c. Pelvic fracture d. None (MAHE 2007, 2008) 76. A patient was brought to the hospital with a history of RTA eight hours back. A few drops of blood were noted at the external urethral meatus. He had not passed urine and his bladder palpable per abdomen. The probable diagnosis is: a. Urethral injury (AIIMS Nov 2006) b. Rupture bladder c. Urethral injury with extravasation of urine in the retroperitoneum d. Anuria due to hypovolemia



77. With the knowledge of anatomy of the pelvis and perineum, which of the following is true regarding collection of urine in urethral rupture above deep perineal pouch?  (AIIMS Nov 2012) a. Medial aspect of thigh b. Scrotum c. True pelvis only d. Anterior abdominal wall



78. Following urethral rupture, immediate procedure to be done is:  (MCI Sept 2008, March 2009) a. Urinary catheterization b. Suprapubic cystostomy c. Referral to a urologist d. Observation



79. A 32-year-old man with pelvic fracture is in urinary retention with blood at the external urinary means. Retrograde urethrogram shows prostatomembrane disruption. The most appropriate immediate treatment is:  (COMEDK 2014) a. Urethral catheterization b. Exploration and repair of urethra c. Suprapublic cystostomy d. Perineal urethrostomy

URETHRAL STRICTURE

80. Which of the following is not an appropriate investigation for anterior urethral stricture?  (AIIMS June 97) a. Magnetic resonance imaging b. Retrograde urethrogram c. Micturating cystourethrogram d. High frequency ultrasound

Urethra and Penis



82. What is the location of stricture in the given RGU?

a. Membranous urethra c. Penile urethra

83. The following are complications of stricture urethra except:  (Karnataka 96) a. Periurethral abscess b. Inguinal hernia c. Hydronephrosis d. Papilloma of bladder



84. Commonest cause of urethral stricture in a young person is: a. Trauma b. Gonococcal (Karnataka 96) c. Syphilis d. Tuberculosis

85. The recent treatment of short bridle passable stricture of urethra in the penile and bulbous urethra is: a. Internal urethrotomy with Thompson-Walker’s urethrotome (MAHE 2005) b. Optical internal urethrotomy c. Syme’s operation d. Wheelhouse operation 86. Optical urethroplasty is done in: (UPPG 2007) a. Congenital stricture of urethra b. Hypospadias c. Epispadias d. Testicular tumors



87. On exertion urine stream increased in: (APPG 96) a. Prostate enlargement b. Marion’s disease c. Posterior urethral valves d. Urethral stricture



91. True about verrucous carcinoma is all except: (Punjab 2009) a. Locally aggressive form of condyloma acuminate b. Also known as Buschke-Lowenstein Tumor c. They frequently metastasize d. Wide excision is the treatment of choice



92. Buschke-lowenstein tumor is: (MHCET 2016) a. Malignant transformation in plantar wart b. Malignant transformation in anogenital wart c. Malignant transformation in common wart d. Malignant transformation in seborrheic wart



93. What is the name of this penile tumor?

88. Post gonococcal stricture urethra is most commonly situated in the: (Recent Questions 2015) a. Bulbar urethra b. Penoscrotal junction c. Distal part of spongy urethra d. Just distal to external meatus 89. Most common cause of urethral stricture is: (Recent Questions 2013) a. Trauma b. Infection c. Congenital d. Post endoscopy 90. A 40 years old patient of pelvic injury presents with stricture of bulbar urethra of 1.5 cm length. Best management: a. Urethral dilatation  (Recent Question 2017) b. Excision with end-to-end urethroplasty c. Partial graft urethroplasty d. Urethrotomy

a. b. c. d.



Bowen’s disease Erythroplasia of Queyrat Buschke-Lowenstein tumor None of the above

94. In CA penis, soft tissue planes are best delineated by: a. MRI b. CT scan(GB PANT 2011) c. X-ray d. USG 95. The most common cause of death in carcinoma penis:  (MHPGMCET 2008, AIIMS Nov 94) a. Uremia b. Urinary sepsis c. Lung metastases d. Erosion of femoral vessels



96. Cabana procedure is done in: (GB PANT 2010) a. CA testis b. RPLND c. Sentinel LN biopsy in penile carcinoma d. None



97. Sentinel lymph node of carcinoma penis: (MHSSMCET 2006) a. Cabana b. Virchow c. Delphian d. Darwins



98. Erythroplasia of Queyrat occurs in:  (MCI Sept 2008) a. Scrotum b. Testes c. Penis d. Bladder 99. Not true about carcinoma penis is: (AIIMS Nov 2001) a. Erythroplasia of Queyrat is a precancerous condition b. 40% of patients are under 40 years of age c. Circumcision if done any time before puberty provides 100% protection against carcinoma penis d. More than 50% patients have inguinal LN enlargement when they present



100. What is true about carcinoma penis? (AIIMS June 94) a. Metastasis is rare b. Occurs more commonly in circumcised male c. Arises from corona of glans d. Pain is frequent

Urology







b. Bulbar urethra d. Prostatic urethra





CARCINOMA PENIS

Section 4

81. The commonest cause of an obliterative stricture of the membranous urethra is: (All India 2003) a. Fall-astride injury b. Road-traffic accident with fracture pelvis and rupture urethra c. Prolonged catheterization d. Gonococcal infection

675

676

Surgery Essence 101. Circumcision is included in management of CA penis at: a. Glans (PGI Dec 98) b. Prepuce c. Glandulo prepucial d. Shaft of penis 102. Features of carcinoma penis are all except: (Recent Question 2017) a. Circumcision soon after birth provides total immunity b. Metastatic to inguinal nodes c. Surgery is treatment of choice d. Transitional cell carcinoma

Urology

103. Treatment of choice of small preputial penile carcinoma is:  (Recent Question 2016) a. Total penectomy b. Partial penectomy c. Emasculation d. Wide excision 104. Sentinel lymph node biopsy was first done in:  (Recent Question 2017, DNB 2012) a. Carcinoma breast b. Carcinoma colon c. Carcinoma penis d. Melanoma 105. All are true about carcinoma penis except: a. Most common type is verrucous (Recent Question 2013) b. Spreads by blood borne metastasis c. Leads to erosion of artery d. Slowly progressive

URETHRAL CARCINOMA 106. Most common site of urethral carcinoma in men is:   (Recent Question 2016, All India 2010) a. Bulbomembranous urethra b. Penile urethra c. Prostatic urethra d. Fossa navicularis

MISCELLANEOUS 107. True about congenital short urethra: (PGI Dec 2003) a. Urethra is short b. Opening is always ventral c. Prepuce deficient ventrally d. Splitting of the two secrotum in the midline e. Spatulated penis

Section 4

108. The posterior urethra is best visualized by: (AIIMS Nov 2005) a. Static cystogram b. Retrograde urethrogram c. Voiding cystogram d. CT cystogram 109. Length of the male urethra: a. 10 cm c. 20 cm

(Recent Question 2016) b. 15 cm d. 25 cm

110. Length of female urethra: a. 2 cm c. 6 cm

(Recent Question 2017) b. 4 cm d. 8 cm

111. Which part of the male urethra is the widest and most distensible?  (Recent Question 2016, 2015) a. Prostatic b. Membranous c. Bulbous d. Penile 112. The least dilatable part of the urethra: a. Prostatic b. Membranous c. Spongy d. All are equally dilatable 113. Bleeding penile ulcer is seen in all except:  (Kerala 94) a. Syphilis b. LGV c. Chancroid d. Granuloma inguinale 114. During urethral catheterization in male patients, resistance is encountered at the following sites except: (ICS 2005) a. Base of navicular fossa b. Mid-penile urethra c. Urogenital diaphragm d. Bulbomembranous junction 115. Smegma is secreted by: (DPG 2008) a. Tyson gland b. Brenner gland c. Cowper’s gland d. Bartholin’s gland 116. There is a high-risk of renal dysplasia in: (AIIMS June 2003) a. Posterior urethral valve b. Bladder exstrophy c. Anorectal malformation d. Neonatal sepsis 117. Which of the following catheter materials is most suited for long-term is used?  (All India 2005) a. Latex b. Silicone c. Rubber d. Polyurethane 118. All are indications for penile angiography except: a. Painful priapism (AIIMS May 2009) b. Peyronie’s disease c. Erectile dysfunction d. Arterio-venous malformation 119. Narrowest part of urethra:  (PGI SS June 2001) a. External urethral meatus b. Membranous urethra c. Prostatic urethra d. Bladder neck 120. Narrowest part of the urethra is: (MCI March 2009) a. Prostatic urethra b. Bulbar urethra c. Penile urethra d. Membranous urethra 121. Which of the following is the shortest urethra?  (Recent Question 2017) a. Bulbar b. Prostatic c. 
Penile d. Membranous 122. Chordee is not seen in: a. Hypospadias c. Priapism

(MHSSMCET 2005) b. Chronic urethritis d. Peyronie’s disease

123. In “three glass test” shreds are presents in first glass only. The most probable diagnosis is:  (DNB 2012) a. Urethritis b. Cystitis c. Prostatitis d. Renal pathology

Explanations HYPOSPADIAS

1. Ans. a. Associated with chordee (Ref: Smith’s 18/e p637; Campbell 11/e p3399-3401; Bailey 27/e p1478)



2. Ans. a. Defect seen in ventral penis; c. Associated with hooded prepuce; d. Circumcision should be avoided



3. Ans. c. Hypospadias

4. Ans. c. Incidence is 1 in 3000 male births



5. Ans. a. Hypospadias

6. Ans. b. Glandular



7. Ans. b. Avoid surgery till puberty; d. If associated chordee is present, 2 stage operation is done



8. Ans. b. 6–10 months of age

9. Ans. d. Cryptorchidism



10. Ans. b. Dorsal chordee

11. Ans. a. Chordee is reversed after 5 years



12. Ans. a. Glandular

13. Ans. c. Karyotyping



14. Ans. a. Ectopia vesicae

15. Ans. a. Hypospadias



16. Ans. a. It is attributed to failure of complete urethral tubularisation in the fetus



17. Ans. d. Statement A is Correct but Statement R is wrong



18. Ans. b. Hypospadias



19. Ans. a. Before 1 year (Ref: Campbell 11/e p3401; Smith 18/e p639; Bailey 27/e p1478)



20. Ans. a. Dennis-Brown (Ref: Campbell 11/e p3410)



21. Ans. c. Straightening – Urethroplasty – Balanoplasty

EPISPADIAS

22. Ans. b. Chordee (Ref: Smith 18/e p639; Campbell 11/e p3221; Bailey 27/e p1478)



23. Ans. b. Less common

POSTERIOR URETHRAL VALVE

24. Ans. d. Diagnosed by early urethroscopy (Ref: Smith 18/e p636; Campbell 11/e p2880; Bailey 27/e p1477)



25. Ans. d. Posterior urethral valve

26. Ans. b. Posterior urethral valves



28. Ans. a. Boys

29. Ans. c. Posterior urethral valve



30. Ans. b. Distal to verumontanum

31. Ans. b. Below verumontanum

PHIMOSIS

32. Ans. a. Wait and watch policy (Ref: Smith 18/e p640; Campbell 11/e p3370; Bailey 27/e p1486)



33. Ans. d. Conservative management



34. Ans. a. Both 1 & 2 can be congenital (Ref: Schwartz 10/e p1664; Bailey 27/e p1486, 1489) Phimosis (1) can be congenital and acquired but paraphimosis (2) is only acquired.

PARAPHIMOSIS

35. Ans. b. Seen in diabetes mellitus (Ref: Smith 18/e p641; Campbell 11/e p3364-3370; Bailey 27/e p1489)



36. Ans. c. Only P and R are true



37. Ans. d. All of the above

CIRCUMCISION

38. Ans. d. All of the above (Ref: Smith 18/e p641; Bailey 27/e p1487)

Circumcision is indicated in patients with infection, phimosis or paraphimosis.

27. Ans. b. MCU

678

Surgery Essence

39. Ans. c. Carcinoma penis (Ref: Smith 18/e p641) Indications of Circumcision • Phimosis • ParaphimosisQ • Recurrent UTIQ Q



• Religion (Jews and Muslims)Q • Balanitis or balanoposthitisQ • BXO (balanitis xerotica obliterans)

40. Ans. b. Increases sexual drive (Ref: Smith18/e p641; Bailey 27/e p1487)

PRIAPISM

41. Ans. b. Corpora cavernosa and saphenous vein (Ref: Smith 18/e p640; Campbell 11/e p684; Bailey 27/e p1491; Glenn’s Urologic Surgery 7/e p489, 490, 491)

Urology

The Grayhack shunt is a surgical shunt between corpora cavernosa and saphenous vein for the treatment of ischemic priapism.

42. Ans. c. Spinal cord disease (Ref: Bailey 27/e p1491; Campbell 11/e p671)

Priapism is rarely seen as a consequence of spinal cord disease.

Persistent Priapism The penis remains erect and becomes painful. This is a pathological erection and the glans penis and corpus spongiosum are not involvedQ. The condition is usually seen as a complication of a blood disorder such as sickle cell disease or leukaemiaQ. However, it can sometimes follow therapeutic injection of papaverine or even an abnormally prolonged bout of otherwise normal sexual activityQ. • A tiny proportion is caused by malignant disease in the corpora cavernosa or the pelvis. • Priapism is rarely seen as a consequence of spinal cord diseaseQ. • • • •



43. Ans. a. PO2 < 30%, PCO2 > 60% (Ref: Campbell 11/e p678) Blood Gas Values in Priapism Source



PO2 (mm Hg)

PCO2 (mm Hg)

pH

• Normal arterial blood • (room air)

> 90

< 40

7.40

• Normal mixed venous • blood (room air)

40

50

7.35

• Ischemic Priapism • (first corporal aspirate)

< 30Q

> 60Q

< 7.25Q

44. Ans. b. Leukemia

45. Ans. b. Spinal cord injury

PEYRONIE’S DISEASE

46. Ans. d. Peyronie’s disease (Ref: Smith’ 18/e p640; Campbell 11/e p722-725; Bailey 27/e p1490)



47. Ans. c. Both of the above (Ref: Sabiston 20/e p2019; Schwartz 10/e p1461; Bailey 27/e p1490)

Section 4



The given condition is called Peyronie’s disease. Galezia’s triad components are Peyronie’s disease + Retroperitoneal fibrosis + Dupuytren’s contracture.



48. Ans. b. Condition affects adolescent males



49. Ans. b. Medical treatment is effective



50. Ans. c. Curved deformity of penis due to fibrous plaque



51. Ans. c. Peyronie’s disease



52. Ans. a. Peyronie’s disease

URINARY TRACT INFECTION

53. Ans. a. Abdominal ultrasound (Ref: Smith 18/e p198; Campbell 11/e p253)

Urethra and Penis

679

This patient must be having anatomic genitourinary abnormalitis (VUR), and the next best investigation is USG.

65

Epidemiology of UTI by Age Group and Sex Incidence (%) Risk Factors Female Male 0.7 2.7 Foreskin, anatomic GU abnormalities 4.5 0.5 Anatomic genitourinary (GU) abnormalities 4.5 0.5 Functional GU abnormalities 20 0.5 Sexual intercourse, diaphragm use 35 20 Surgery, prostate obstruction, catheterization 40 35 Incontinence, catheterization, prostate obstruction

Section 4

Age (years)

Ultrasound • Ultrasound study is an important renal imaging technique because it is noninvasive, easy to perform, and rapid and offers no radiation or contrast agent risk to the patientQ.

54. Ans. b. Vesicoureteric reflux 55. Ans. c. Cranberry (Ref: Smith 18/e p204) • Alternatives to antibiotic therapy in the treatment of recurrent cystitis/UTI include intravaginal estriol, lactobacillus vaginal suppositories, and cranberry juice taken orallyQ. • Cranberry juice is traditionally used for prophylaxis and treatment of UTIQ.



56. Ans. a. E. coli (Ref: Smith 18/e p199)

Urinary Tract Infection • Most UTIs are caused by a single bacterial speciesQ. • At least 80% of the uncomplicated cystitis and pyelonephritis are due to E. coliQ, with most of pathogenic strains belonging to the O serogroups. • Other less common uropathogens include Klebsiella, Proteus, and Enterobacter spp. and enterococci. • In hospital acquired UTIs, a wider variety of causative organisms is found, including Pseudomonas and Staphylococcus sppQ. • UTIs caused by S. aureus often result from hematogenous disseminationQ. • Group B beta-hemolytic streptococci can cause UTIs in pregnant womenQ. • In children, Klebsiella and Enterobacter spp. are common causes of UTIQ. • Anaerobic bacteria, lactobacilli, corynebacteria, streptococci (not including enterococci) and S. epidermidis are found in normal periurethral flora. They do not commonly cause UTIs in healthy individuals and are considered common urinary contaminantsQ.

57. Ans. d. 105 or over (Ref: Smith 18/e p200)

Urine Culture in UTI • The gold standard for identification of UTI is the quantitative culture of urine for specific bacteriaQ. • The urine should be collected in a sterile container and cultured immediately after collection. When this is not possible, the urine can be stored in the refrigerator for up to 24 hours. • The sample is then diluted and spread on culture plates. Each bacterium will form a single colony on the plates. • The number of colonies is counted and adjusted per milliliter of urine (CFU/mL). • Traditionally, > 100,000 CFU/mL (> 105) Q is used to exclude contamination. 58. Ans. d. Suprapubic aspiration (Ref: Smith 18/e p200) Suprapubic aspiration avoids potential contamination.

Urinary Tract Infection • Most often, the urine is often obtained from a voided specimen. • In children who are not toilet trained, a urine collection device, such as a bag, is placed over the genitalia, and the urine is cultured from the bagged specimen. • Suprapubic aspiration avoids potential contaminationQ; however, due to its invasiveness, it is rarely used except in children and selected patients. • Urine obtained from a urinary catheter is less invasive than a suprapubic aspiration and is less likely to be contaminated than that from a voided specimenQ.

Urology

Traditionally, > 100,000 CFU/mL (> 105) is used to exclude contamination.

680

Surgery Essence URINARY RETENTION 59. Ans. d. Meatal ulcer with scabbing (Ref: Bailey 27/e p1426) Acute urinary retention in a male child may be due to local inflammatory causes like meatal ulcer with scabbing. Etiology of Urinary Retention in Children • • • • • •

Neurological processes (17%) Severe voiding dysfunction (15%) UTI (13%) Constipation (13%) Adverse drug effect (13%) Local inflammatory causes (7%)

• • • • •

Locally invading neoplasms (6%) Benign obstructing lesions (6%) Idiopathic (6%) Combined UTI and constipation (2%) Incarcerated inguinal hernia (2%)

60. Ans. b. Obstructive urinary disease (Ref: Bailey 27/e p1426)

Urology

Most Frequent Causes of Acute Urinary Retention Male • • • •

Both (Males and Females)

Bladder outlet obstruction (MC) Urethral stricture Acute urethritis or prostatitis Phimosis

Blood clot Urethral calculus Rupture of the urethra Neurogenic (injury or disease of the spinal cord) • Smooth muscle cell dysfunction associated with ageing • • • •

Female • Retroverted gravid uterus • Bladder neck obstruction (rare)

• Fecal impaction • Anal pain (haemorrhoidectomy) • Intensive post-operative analgesic treatment • Some drugs • Spinal anaesthesia

Urinary Retention • In acute urinary retention with significant bladder distention, rapid drainage might precipitate decompression induced hematurea (Ex-vacuo hematurea)Q • Catheter should be intermittently clamped and released to permit gradual bladder decompression over 30–60 minutesQ.

61. Ans. c. Herniorrhaphy

62. Ans. a. Meatal scab with ulceration

URETHRAL INJURY

63. Ans. a. Anterior urethra is the most likely site of injury (Ref: Smith 18/e p294; Campbell 11/e p2388; Bailey 27/e p1479)



64. Ans. a. Catheterize the patient immediately



66. Ans. b. Deep perineal pouch



67. Ans. b. Floating prostate on per rectal examination (Ref: Smith 18/e p294; Campbell 11/e p2391; Bailey 27/e p1481)



68. Ans. d. Catheterize, drain bladder and retain in catheter thereafter



69. Ans. c. Treatment of shock and hemorrhage

70. Ans. c. Stricture



71. Ans. b. Fixity of urethra

72. Ans. a. Suprapubic cystostomy



73. Ans. a. Ischiorectal fossa (Ref: Smith 18/e p292; Campbell 11/e p2391; Bailey 27/e p1479-1481)

65. Ans. a. Pass indwelling urethral catheter

Section 4

Bulbar Urethral Injury

Membranous Urethral Injury

Incidence

• More commonQ

Mechanism of injury

• Direct blow to the perineum (Straddle injury)Q

• Blunt pelvic trauma with fracture pelvisQ

Signs and symptoms

• • • •

• • • •

Urine extravasation

• Superficial extravasationQ

• Less common

Retention of urine Blood at urethral meatusQ Perineal hematomaQ Normal prostateQ Q

Retention of urineQ Blood at urethral meatusQ Pelvic hematomaQ High lying prostateQ

• Deep extravasationQ



74. Ans. a. Rupture of membranous urethra

75. Ans. d. None



76. Ans. a. Urethral injury

77. Ans. c. True pelvis only



78. Ans. b. Suprapubic cystostomy

79. Ans. c. Suprapubic cystostomy

Urethra and Penis

681

URETHRAL STRICTURE 80. Ans. a. Magnetic resonance imaging (Ref: Smith 18/e p642; Campbell 11/e p918-919)

Stricture Urethra • Use of MRI for routine strictures or pelvic fracture urethral distraction defect is not routinely beneficial. 81. Ans. b. Road-traffic accident with fracture pelvis and rupture urethra 82. Ans. b. Bulbar urethra (Ref: Sabiston 20/e p2093; Schwartz 10/e p1665; Bailey 27/e p1483)

Section 4



Retrograde Urethrogram



83. Ans. d. Papilloma of bladder

84. Ans. a. Trauma



85. Ans. b. Optical internal urethrotomy (Ref: Campbell 11/e p921, 10/e p971-972) Campbell says “The data show that strictures at the bulbous urethra that are < 1.5 cm in length and not associated with dense, deep spongiofibrosis (i.e. straddle injuries) can be managed with internal urethrotomy, with a 74% moderately long-term success rate.”

Internal Urethrotomy



86. Ans. a. Congenital stricture of urethra (Ref: www.ncbi.nlm.nih.gov › Postgrad Med J › v.82(970); Aug 2006) • Short bulbar strictures as a result of trauma or otherwise congenital are best treated by anastomotic urethroplasty Q.



87. Ans. d. Urethral stricture



88. Ans. a. Bulbar Urethra

89. Ans. a. Trauma

90. Ans. b. Excision with end-to-end urethroplasty (Ref: Campbell 11/e p921) “Excision with primary anastomosis has proved to be the gold standard form of repair for anterior urethral strictures. In years past, excision with primary anastomosis was thought to be a relatively limited procedure and applicable only for strictures less than 1.5 to 2.0 cm. However, with better understanding of the anatomy, longer and longer strictures have been successfully addressed with excision and primary anastomosis.” –Campbell 11/e p921

Urology

• Internal urethrotomy refers to any procedure that opens the stricture by incising it transurethrally Q. • Internal urethrotomy is done for short, soft, passable, bulbar stricture. • The urethrotomy procedure involves incision through the scar to healthy tissue to allow the scar to expand (release of scar contracture)Q and the lumen to heal enlarged. • MC complication of internal urethrotomy is recurrence of stricture. • The data show that strictures at the bulbous urethra that are < 1.5 cm in length and not associated with dense, deep spongiofibrosisQ (i.e. straddle injuries) can be managed with internal urethrotomy, with a 74% moderately long-term success rate Q.

682

Surgery Essence CARCINOMA PENIS

91. Ans. c. They frequently metastasize (Ref: Smith 18/e p389; Campbell 11/e p849; Bailey 27/e p1492)



92. Ans. b. Malignant transformation in anogenital wart (Ref: Campbell 11/e p848)

93. Ans. c. Buschke-Lowenstein tumor (Ref: Schwartz 10/e p1218; Bailey 27/e p1493)

Urology

“Buschke–Löwenstein tumour is uncommon. It has the histological pattern of a verrucous carcinoma. It is locally destructive and invasive but appears not to spread to lymph nodes or to metastasise. Treatment is by surgical excision.” –Bailey 27/e p1493

94. Ans. a. MRI (Ref: Campbell 11/e p851)



95. Ans. d. Erosion of femoral vessels (Ref: Campbell 11/e p849)



96. Ans. c. Sentinel LN biopsy in penile carcinoma (Ref: Campbell 11/e p850)



97. Ans. a. Cabana (Ref: Campbell 11/e p850)



98. Ans. c. Penis (Ref: Campbell 11/e p848)



99. Ans. c. Circumcision if done any time before puberty provides 100% protection against carcinoma penis (Ref: Campbell 11/e p847)

100. Ans. c. Arises from corona of glans

101. Ans. b. Prepuce

102. Ans. d. Transitional cell carcinoma

103. Ans. d. Wide excision

104. Ans. c. Carcinoma penis (Ref: Mastery of Surgery 5/e p1531) • “The historic contribution by Cabana in 1977 (Cancer 1977; 39:456) established the importance, lymphatic histology with sentinel lymph node mapping of patients with penile carcinoma. This approach identified the sentinel lymph node as the first site of residual nodal metastasis and is predictive of the nodal status of the remaining node basin.” 105. Ans. a. Most common type is verrucous

URETHRAL CARCINOMA 106. Ans. a. Bulbomembranous urethra (Ref: Smith 18/e p322; Campbell 11/e p882)

MISCELLANEOUS 107. Ans. a. Urethra is short 108. Ans. c. Voiding cystogram (Ref: Sutton Radiology 7/e p898-899) • Voiding cystourethrography is the best method to visualize the posterior urethra Q. • Anterograde techniques are best for visualization of posterior urethra Q. • Reterograde techniques (contrast is injected through tip of urethra) is best for visualization of anterior (penile) urethra Q. 109. Ans. c. 20 cm

110. Ans. b. 4 cm

111. Ans. a. Prostatic 112. Ans. b. Membranous

113. Ans. a. Syphilis

114. Ans. b. Mid-penile urethra

Section 4

115. Ans. a. Tyson gland • Tyson’s Glands ultimately produce an oily substance, which, when mixed with shed skin cells, constitute smegmaQ. 116. Ans. a. Posterior urethral valve (Ref: Cambell 11/e p2880) • Renal parenchymal dysplasia is the most important factor in overall prognosisQ in posterior urethral valve. 117. Ans. b. Silicone (Ref: www.nursing-standard..co.uk) • All silicone catheters are suitable for use in patients with latex allergy and can remain in situ for 12 weeksQ or according to manufacturer’s instruction.

Urethra and Penis

683

Catheter

Long-Term (12 weeks) • All-siliconeQ: 12 weeks • Silicone elastomer coated: 12 weeks • Hydrogel coated: 12 weeks

118. Ans. a. Painful priapism (Ref: Campbell 11/e p 678) • Penile angiography is indicated in high flow, non-ischemic priapism, which is painlessQ. • It is not indicated in painful (low-flow) priapismQ.

Section 4

Short-Term (14-28 days) • PVCQ: 14 days • LatexQ: 14 days • TeflonQ: 28 days

119. Ans. a. External urethral meatus 120. Ans. d. Membranous urethra 121. Ans. d. Membranous (Ref: Campbell 11/e p1635; Smith 18/e p14; Bailey 27/e p1477) 122. Ans. c. Priapism 123. Ans. c. Prostatitis (Ref: Bailey 27/e p1475, 26/e p1357)

Three Glass Urine Test • The three glass urine test is valuable in diagnosis of chronic prostatitis • If the first glass with the initial voided sample shows urine containing prostatic threads, prostatitis is present.

Urology

CHAPTER

25

Testis and Scrotum

UNDESCENDED TESTIS Undescended Testis • UDT affects 3% of full-termQ newborns. • Incidence by 1 year of age is 1%. • Approximately 70% to 77% of UDT will spontaneously descend, usually by 3 months Q of age. • Birth weightQ may be the principal determinant of UDT at birth and at 1 year of life, independent of the length of gestation. • In UDT, 80% are palpable and 20% are nonpalpableQ. • MC location for an ectopic UDT is within the superficial pouchQ. Pathology • Germ cell histology of both the testes is abnormal. The histopathologic hallmarks of UDT are evident between 1-2 yearsQ of age: • Decreased numbers of Leydig cells (Earliest abnormality)Q • Degeneration of Sertoli cellsQ • Delayed disappearance of gonocytesQ • Delayed appearance of adult dark spermatogoniaQ • Failure of primary spermatocytes to developQ • Reduced total germ cell countsQ • Hypoplasia of the Leydig cells, observed from the 1st monthQ of life, is the earliest postnatal histologic abnormalityQ in UDT. • Adverse effects on contralateral testes: Autoantibodies against UDT causes degenerative changes in contralateral testis, germ cell histology is abnormal and risk of carcinomaQ is also increased. Associated Anomalies • Epididymal anomalies and patent processus vaginalis up to 90%Q cases of UDT. • Renal Anomalies in 10% cases (Renal hypoplasia, agenesis, horse shoe kidney, PUJ obstruction) • Hypospadias  Hazards: (SATHI- Sterility, Atrophy, Trauma, Tumor, Torsion, Hernia, Inflammation) Neoplasia • • • • • • • • •

Relative risk of testicular tumor is increased 17 times. MC tumor that develops is seminomaQ. Higher the testis, greater the riskQ (Abdominal testis has higher risk than inguinal) Orchiopexy does not decrease the risk, it helps in early detection onlyQ.

Infertility Histopathological changes start at 1 yearQ. At 6-8 years, spermatogenesis is absent Q. Endocrine functions are retained as the Leydig cells are less sensitive to temperatureQ. Surgical repositioning before the onset of histopathological changes decreases the risk of subfertility. Paternity is significantly compromised in men with bilateral, but not unilateral UDT.

• Hernia: Patent processus vaginalisQ is seen in 90% cases of UDT. • Torsion: Increased susceptibility Diagnosis • Inguinal exploration is IOC for UDTQ.

Testis and Scrotum

685

Diagnostic Laparoscopy

Management: Orchiopexy, Ideal time: 6-12 monthsQ of age. (Best time is 6 monthsQ)

Section 4

• IOC for ‘non-palpable’ UDTQ. • MC application of laparoscopy in children is for UDT. • Accuracy of transperitoneal laparoscopy in locating a non-palpable testis is 100%Q and it subsequently defines the management options. • Vas and testicular artery is traced in pelvis. • Blind ended vas doesn’t conclude the absence of testisQ, whereas blind ended testicular artery is a definitive investigation for an absence of testisQ. • Laparoscopy is also useful in lap. assisted orchiopexy.

Types of Orchiopexy 1. Fowler-Stephens orchiopexyQ 2. Microvascular testicular autotransplantation (Best results)Q 3. Ladd and Gross orchiopexyQ

4. Ombridann’s orchiopexyQ 5. Placing testis in Dartos pouchQ 6. Keetley-Torek orchiopexyQ

ECTOPIC TESTIS

• An ectopic testicle descends normally through the inguinal canal but then moves into an abnormal position in the groin areaQ. • An ectopic testis is usuallyQ. Fully developedQ. The main hazard is liability to injuryQ. Locations of Ectopic Testis • • • • •

Superficial inguinal pouch (MC location)Q Femoral canalQ (the inner portion of the thigh near the groin) PerineumQ (below the scrotum) Suprapubic regionQ (above the penis) Contralateral scrotum (Least common)Q

Embryology • Ectopic testis are likely related to abnormalities of the gubernaculumQ, which is a fibrous, cord-like membrane that runs through the inguinal canal from the abdomen to the scrotum. • The gubernaculum helps to guide the descent of the testiclesQ and has branches that attach to these other locations. • Ectopic testicles usually will not descend into the normal position in the scrotum on their own. • Most ectopic testicles can be felt (are palpable)Q. Treatment • Surgical treatment to place an ectopic testicle in its normal position any time after about age 6 months but no later than 2 years of ageQ.

Urology

Ectopic Testis

686

Surgery Essence RETRACTILE TESTIS Retractile Testis • Retractile testis: Testis has completed the process of descent but found in groin because of an overactive cremasteric reflex or increased cremasteric muscle toneQ. Clinical Features • MC age of presentation: 2-6 yearsQ • Testis can be manipulated down in scrotumQ & remain in this location temporarily • Development of scrotum is normalQ (underdeveloped scrotum in UDT) Treatment

Urology

• Retractile testes require no treatmentQ.

TESTICULAR TORSION Testicular Torsion • Twisting of testis on the spermatic cord, resulting in strangulation of the blood supply and infarction of testis. • Types of testicular torsion: Intravaginal and Extravaginal.

Intravaginal Testicular Torsion • Torsion occurs within the space of tunica vaginalis, which is highly investedQ, resulting in lack of normal fixation of testis & epididymis to the fascial & muscular coverings (scrotal parietal wall) • MC age group affected is 10-25 years, with peak in prepubertal ageQ • Cremaster fibers have a spiral attachment over the cord, it favors rotation when cremaster reflex is strongQ. Predisposing Factors • Inversion of the testis (testis lies transversely or upside down) is MC predisposing factorQ • High investment of tunica vaginalis causes the testis to hang within the tunica like a clapper in a bellQ • Separation of the epididymis from the body of testisQ permit torsion of testis without involving cord Clinical Features • Sudden agonizing scrotal pain with nausea or vomitingQ • Dysuria or other bladder symptoms are usually absentQ • Affected testis high-riding in scrotum, may have abnormal transverse orientationQ • Cord is usually thickenedQ • Absent cremasteric refluxQ is highly suggestive of torsion testis (present in epididymitis) • After several hours massive scrotal edema may obliterate all the findings • Prehn’s sign is negativeQ (On elevation of testis, pain relieved in epididymoorchitis but not in torsion testis) • Deming sign: Affected testis at higher level because of twisting of cordQ • Angel sign: Opposite testis lies horizontally because of present of mesorchiumQ

Section 4

Imaging • Color Doppler detects the decreased blood flow to the testis in torsion and is investigation of choiceQ to exclude torsion from epididymoorchitis. • Tc99 pertechnate scan demonstrate poor radionuclide tracer uptakeQ Treatment • Testicular torsion is urological emergency as ischemic injury occurs as soon as 4 hoursQ after occlusion of the cord. • Immediate surgical explorationQ is indicated if testicular torsion is suspected because if treated within first 4 hours, the chances of testicular salvage are high. • Even if manual detorsion is done, it may not totally correct the rotation and prompt exploration is still indicated. • Viable as well as testis of marginal viability are preserved, and orchiopexy is done either by placing into dartos pouch without suture fixation or suture fixation of tunica albuginea with the parietal wallQ.

Testis and Scrotum

• Risk of autoimmunization against own sperms is low in children 2 cm but 2 cm but not 5 cmQ

pT3: Invades the spermatic cord with or without vascular/lymphatic invasionQ pT4: Invades the scrotum with or without vascular/lymphatic invasion

M: Distant metastases M1: Non-regional nodal or pulmonary metastasesQ M2: Non-pulmonary visceral massesQ

Q

Serum Tumor Markers (S)

Section 4

LDH

hCG (mIU/mL)

AFP (ng/mL)

S0

≤N

≤N

≤N

S1

10,000

8th AJCC (2017) Staging for Testicular Cancer Stage Grouping T N M Stage 0 pTis N0 M0 Stage Ia T1 N0 M0 Stage Ib T2-T4 N0 M0 Stage Is Any T N0 M0 Stage IIa Any T N1 M0 Stage IIb Any T N2 M0 Stage IIc Any T N3 M0 Stage IIIa Any T Any N M1 Stage IIIb Any T Any N M0-M1 Stage IIIc Any T Any N M0-M1a Any T Any N M1b

S S0 S0 S0 S1-S3 S0-S1 S0-S1 S0-S1 S0-S1 S2 S3 Any S

Staging and Treatment Stage

Extent of disease

Seminoma

Nonseminoma

IA

Testis only, without vascular or lymphatic invasion (T1)

IB

Testis with vascular or lymphatic invasion (T2), or ex- Radiation therapyQ tension through tunica albuginea (T2), or involvement of spermatic cord (T3), or scrotum (T4)

RPLNDQ

IIA

Nodes ≤ 2 cm (N1), S0/S1

RPLND or chemotherapy followed by RPLNDQ

Radiation therapyQ

Radiation therapy

RPLND or observationQ

Testis and Scrotum IIB

Nodes >2-5 cm (N2), S0/S1

Radiation therapyQ

RPLND ± adjuvant chemotherapy chemotherapy followed by RPLNDQ

IIC

Nodes > 5 cm (N3), S0/S1

ChemotherapyQ

Chemotherapy followed by RPLNDQ

III

Distant metastasis

Chemotherapy

Chemotherapy followed by surgery (biopsy or resection) Q

Is

Only serum tumor markers are raised (S1 to S3) Q

ChemotherapyQ

or

ChemotherapyQ

INTRATUBULAR GERM CELL NEOPLASIA (ITGCN) OR TESTICULAR CARCINOMA IN SITU (CIS)

Section 4

Q

693

Intratubular Germ Cell Neoplasia (ITGCN) or Testicular Carcinoma in Situ (CIS) • Preinvasive precursor of all testicular GCTs except: −− Spermatocytic seminomaQ −− Yolk sac tumorQ −− Teratoma in childrenQ • Presently, there is no established tumor marker for CIS, and testicular ultrasound has been shown to be unreliable with respect to diagnosing CIS. • Therefore, testicular biopsy remains the “gold standard” for diagnosing CISQ. Risk Factors for ITGCN • • • •

History of testicular carcinoma Extragonadal GCTQ CryptorchidismQ Contralateral testis with unilateral testicular cancerQ Q

• Atrophic contralateral testisQ with unilateral testicular cancer • Somatosexual ambiguityQ • InfertilityQ

Pathology • ITGCN cells are located on the basement membrane of the seminiferous tubule and possess morphologic features of malignancy: Large irregular nucleus, coarse chromatin, and abundant cytoplasmQ. • Typically only one layer of ITGCN cells is seen, but occasionally the whole tubule is filled with ITGCN cells, which may represent an early stage in progression to GCTQ. Clinical Features • Testis may be atrophic but is usually normal to palpation • Testis biopsy reliably diagnoses ITGCN because the ITGCN is almost always found throughout the testisQ • Progression of ITGCN to invasive disease may take 15 yearsQ.

Urology

Treatment • Treatment options include observation, radiation therapy, and orchiectomyQ • Chemotherapy so far appears to be ineffective against ITGCN.

Tumour Markers Oncofetal substances

Cellular Enzymes

1. α-FP: (Increases in YET) Q −− Produced by trophoblastic cells −− Increases in Yolk sac tumour, embryonal carcinoma and terato carcinoma. (YET) −− Does not increase in pure choriocarcinoma and pure seminomaQ −− Metabolic half life: 5-7 daysQ

1. LDH: −− Not a specific tumour marker −− Most useful as a marker for “bulk’ disease −− Raised serum LDH has poor prognosis

2. β-hCG: (Increases in CESMINOMA) −− Produced by syncytiotrophoblastic cellsQ −− Increases in all choriocarcinomas, 50% embryonal carcinoma and 5-10% of pure seminoma (as they contain syncytiotrophoblast like giant cells). −− Serum half life: 24-36 hoursQ

2. PLAP (Placental alkaline phosphatase): −− Elevated levels present in as many as 40% patients with advanced disease −− Most useful as a marker for “bulk’ disease −− Elevated in seminomaQ 3. GGT (Gamma glutamyl transpeptidase): −− Marker of seminoma testisQ −− Marker for “bulk” disease

694

Surgery Essence YOLK SAC TUMOR Yolk Sac Tumor • MC testicular tumor of infants and childrenQ. • The terms endodermal sinus tumor, adenocarcinoma of the infantile testis, juvenile embryonal carcinoma, and orchioblastoma are all used synonymouslyQ. Pathology • In its pure form, the lesion has a homogeneous, yellowish, mucinous appearanceQ. • Embryoid bodies, a common finding in yolk sac tumors, resemble 1- to 2-week-old embryos. Three most common microscopic patterns

Urology

1. Microcystic- honeycomb appearance, with hyaline globulesQ 2. Endodermal sinus- perivascular formations known as Schiller-Duval bodiesQ 3. Solid- small polygonal cells, clear cytoplasm, frequent mitoses • The pattern of metastatic disease of yolk sac tumors in childhood differs from the pattern in adult germ cell tumors, owing to a higher incidence of hematogenous spread. Clinical Features • MC testis tumor in prepubertal boys, presents as slow-growing scrotal mass in a young boyQ • Hydrocele is present in 25% of casesQ. Tumor Marker • AFP is elevated in >90%Q.

SEMINOMA Seminoma • Three subtypes: Classic, anaplastic and spermatocytic. • Typical or classic seminoma: −− Accounts for 80-85%Q of all seminomas and occurs most commonly in 30 years of age −− Histologically, it is composed of islands or sheets of relatively large cells with clear cytoplasm and densely staining nucleiQ. • Syncytiotrophoblastic elements occur in 10-15%Q and lymphocytic infiltration occurs in 20%Q. The incidence of syncytiotrophoblastic elements corresponds to the frequency of β-hCG productionQ. −− The slower growth rate of seminomasQ may be inferred from the observation that treatment failures may become evident 2-10 years after apparently adequate irradiation of metastatic sites. • Anaplastic seminoma: −− Accounts for 5% to 10% of all seminomas −− Despite its rarity, up to 30% of patients dying with seminoma have an anaplastic morphologyQ. • More aggressive and potentially more lethalQ variant leading to less favorable resultsQ. • These characteristics include greater mitotic activity, higher rate of local invasion, increased rate of metastatic spread and higher rate of tumor marker (β-hCG) production.

Section 4

−− No difference from classic seminoma when patients are treated appropriately and compared stage for stageQ. −− Inguinal orchiectomy plus radiation therapy is equally effective in controlling both anaplastic and classic seminomaQ. • Spermatocytic seminoma: −− Accounts for 2-12% of all seminomas −− It is distinctive tumor both clinically and radiologically as compared to seminomaQ • Variants of germ cell tumor that do not arise from an intratubular germ cell neoplasiaQ (the other being teratomas of childrenQ). −− Composed of cells that vary in size and have deeply pigmented cytoplasm and rounded nuclei containing characteristic filamentous chromatinQ. −− Uncommon tumor representing 1-2% of all testicular neoplasmsQ − Only orchidectomy is required for treatment. − Affected individuals are >65 yearsQ − Prognosis is excellentQ. −− Slow growing tumor that rarely if ever produces metastasesQ

Testis and Scrotum

695

TERATOMA

• Most common testicular tumor in prepubertal adults: TeratomaQ • Tumor is composed of two or more embryonic germ cell layersQ that may be both mature and immature. • Tumor is very heterogeneous with both solid and cysticQ components. Teratoma is divided into three subsets 1. Mature: Well-differentiated ectodermal, mesodermal, or endodermal tissues 2. Immature: Incompletely differentiated tissues 3. Teratoma with areas of malignant transformation: sarcoma, squamous carcinoma, adenocarcinoma

Section 4

Teratoma

• Teratomas are potentially malignantQ Clinical Features • Age range: first, second, and third decades. • Mature and immature forms have metastatic potential in adults but in children are uniformly benignQ. • The primary tumor generally presents as an enlarged testis with both solid and cystic componentsQ. • The teratoma component of metastatic GCT is resistant to chemotherapy and radiotherapyQ. Tumor Markers • AFP is raised in 20-25%Q

TESTICULAR LYMPHOMA Testicular Lymphoma • MC testicular tumor in a patient >50 years • MC secondary neoplasm of the testis, accounting for 5% of all testicular tumors. • It may be seen as late manifestation of widespread lymphoma; initial presentation of clinically occult disease; and primary extranodal diseaseQ. Q

Pathology • Grossly, bulging, gray or pink lesion with ill-defined margins. Hemorrhage and necrosisQ are common. • Diffuse large B.cell lymphoma > Burkitt lymphoma Clinical Findings

Treatment • FNAC should be considered in a known or suspected diagnosis of lymphomaQ • Radical orchiectomy is reserved for suspected primary lymphomaQ of the testicle.

LEYDIG CELL TUMORS Leydig Cell Tumors • • • •

MC non-germ cell tumors of testis and account for 1-3% of all testicular tumors. Bimodal age distribution: 5-9 years and 25-35 years 25% tumors occur in childhood, bilateral in 5-10%. Cause: unknown; unlike germ cell tumors (no association with cryptorchidism)Q

Pathology • Small, yellow, well-circumscribed lesion devoid of hemorrhage or necrosis. • Microscopically, hexagonally shaped cells with granular, eosinophilic cytoplasm containing lipid vacuoles are seen. • Reinke crystalsQ are fusiform-shaped cytoplasmic inclusions are pathognomonic for Leydig cells.

Urology

• Painless enlargementQ of the testis is common. • Generalized constitutional symptoms occur in one-fourth of patients. • Bilateral testis involvement occurs in 50% of patients, usually asynchronously.

696

Surgery Essence Clinical Features • • • •

Prepubertal children present with virilization, and tumors are benignQ. Adults are usually asymptomatic, although gynecomastia may be present in 20–25%Q. 10% of tumors in adults are malignant. Laboratory findings include elevated serum and urinary 17-ketosteroids and estrogensQ.

Treatment • Initial treatment: Radical orchiectomyQ • RPLND for malignant lesionsQ. Prognosis

Urology

• Prognosis is excellent for benign lesions, while it remains poor for disseminated disease.

CARCINOMA SCROTUM (CHIMNEY SWEEP’S CANCER) Carcinoma Scrotum (Chimney Sweep’s Cancer) • Squamous cell carcinoma of scrotumQ, most commonly resulted from exposure to environmental carcinogens including chimney soot, tars, paraffin and petroleum productsQ. • Superficial inguinal lymph nodes are the first lymph nodes involved. Q Risk Factors • Most cases results from poor hygiene and chronic inflammationQ. Diagnosis • Diagnosis is established by biopsy of scrotal skinQ. Treatment • Wide excision with 2 cm marginsQ should be performed for malignant tumors. • Prognosis correlates with presence or absence of nodal involvementQ. • MC common benign lesion of scrotum: Sebaceous cystQ • MC common malignanat tumor of scrotum: Squamous cell carcinomaQ

IMPORTANT TOPICS Important Topics • Length of one seminiferous tubule: 1 meterQ • Length of epididymis: 4 metersQ

•  Total length of seminiferous tubules: 250 metersQ

Section 4

Most Common Lymph Nodes Involved CA Penis

Inguinal LNQ

CA Testis

On right: Inter-aortocavalQ LN On left: ParaaorticQ LN

CA Bladder

ObturatorQ LN

CA Prostate

ObturatorQ LN

Multiple Choice Questions UNDESCENDED TESTIS

1. Best time for surgery of undescended testis is: (Recent Question 2014, 2015; All India 2010) a. Just after birth b. 6 months of age c. 12 months of age d. 24 months of age



2. Surgery for undescended testis is recommended at what age? (COMEDK 2014, All India 2011) a. 6 months b. 12 months c. 24 months d. 36 months



3. Incidence of undescended testis in normal new born: a. 3% b. 6% (DNB 2007) c. 9% d. 12%



4. Most common tumors in undescended testis: a. Seminoma c. Embryonal carcinoma

5. Stephen Fowler surgery is done for: (GB PANT 2010) a. Ectopic testis b. Undescended testis c. Hypospadias d. Epispadias



6. All can be prevented by orchiopexy in cryptorchidism except: (AIIMS Nov 99) a. Testicular tumor b. Epididymoorchitis c. Torsion of testis d. Sexual ambiguity













7. Which of the following investigation is used to confirm anorchia? (AIIMS Nov 2013) a. PET b. MRI c. Laparoscopy d. USG 8. Best investigation for undescended testis in 1-year-old child is:  (Recent Question 2017) a. USG abdomen b. CT c. MRI d. Laparoscopy 9. Orchidopexy is done in cases of undescended tests at the age of: (AIIMS June 2006) a. Infancy b. 1-2 years c. 5 years d. Puberty 10. What is not seen in undescended testis? (AIIMS June 95) a. Hydrocele b. Hernia c. Teratoma d. Seminoma 11. Incompletely descended testis is commonest on: a. Right side b. Left side c. Both side d. Right sided only 12. A 5-year-old male child has been brought with a complaint that there is only one testis in the scrotum. On examination, it is found that the testis on the opposite side is felt in the inguinal canal. The patients should be advised: (UPSC 96) a. Orchiopexy b. To wait till puberty c. Orchidectomy d. Administration of androgens 13. Which one of the following statement is true of undescended testis? (UPSC 97) a. Usually descends spontaneously at puberty b. Orchidopexy to be done if no descent by puberty c. Has higher incidence of malignancy d. Maintains normal sperm production

(JIPMER 2012)



15. Testis does not descend beyond: a. 2 months b. 4 months c. 6 months d. 8 months



16. In cryptorchidism, hallmark histological changes appear in testis at: (Recent Question 2016) a. 4 months b. 6 months c. 8 months d. 1 year



17. Fowler-Stephen surgery is done in: (Recent Question 2017) a. Epispadias b. Hypospadias c. Exstrophy of bladder d. Cryptorchidism

(DNB 2005, Punjab 2009) b. Teratoma d. None





14. True about incompletely descended testis are all of the following except: (MCI March 2008) a. Early repositioning can preserve function b. It may lead to sterility, if bilateral c. Poorly developed secondary sexual characters d. May be associated with indirect inguinal. hernia

ECTOPIC TESTIS

18. Most common site of ectopic testis: (GB PANT 2010) a. Superficial inguinal pouch b. Root of penis c. Femoral triangle d. Perineum



19. Ectopic testis is found in all location except: (Recent Questions 2016) a. Lumbar b. Perineal c. Intra abdominal d. Inguinal



20. Complication of ectopic testis is: a. Seminoma b. Atrophy c. Torsion d. All

(Kerala 94)

TESTICULAR TORSION

21. True about torsion of testis is all except: (AIIMS Nov 2001) a. Presents with sudden pain in testis b. Commonly associated with pyuria c. Doppler U/S shows decreased blood flow to the testis d. Simultaneous orchiopexy of the other side should also be done

22. A 30-year-old male patient presents with sudden onset swelling and pain over the right hemiscrotum. On examination the scrotum is reddened and tender. Which of the following statement about the affecting condition is not true? a. Probable diagnosis is torsion (All India 2008) b. The right testis is likely to ride high in the scrotal compartment c. If torsion confirmed, treat with antibiotics and analgesics and perform corrective surgery immediately d. If torsion confirmed, treat with antibiotics and analgesics and perform corrective surgery after 14 days

23. All of the following abnormalities are predisposing causes for torsion of the testis except: (COMEDK 2006) a. Inversion of testis b. Low investment of tunica vaginalis c. Between 10 to 25 years of age d. Separation of the epididymis

698

Surgery Essence

Urology



24. All are true regarding torsion of the testis, except: (Orissa 2011) a. Common in adolescents and young adults b. Inversion of testis is the most common predisposing cause c. Elevation of testis reduces the pain d. If diagnosis is doubtful, prompt exploration is the rule 25. A 40-years old man in suffering from fever and pain in scrotum which is not relieved by elevation of testis, 3 days before had sexual contact with a young female: (All India 97) a. Testicular tumor b. Acute epididymitis c. Torsion of testis d. Acute orchitis (DNB 2007)



26. Torsion of testis has to be treated within: a. Immediately b. 6 hours c. 12 hours d. 15 hours



27. In testicular torsion, surgery within how much time can save viability of testis? (Recent Question 2013) a. 6 hours b. 12 hours c. 24 hours d. 1 weeks

28. All the following statements are true regarding torsion of testis except: (APPG 2015) a. Most common between 10 and 25 years of age b. Prompt exploration and twisting & fixation is the only way to save the torted testis c. Anatomical abnormality is unilateral and contalateral testis should not be fixed d. Inversion of testis is the most common predisposing cause

33. Which is not true regarding varicocele? a. Testicular veins involved b. More common on the right side c. May be the first feature of a renal tumour d. Feels like a bag of worms



34. A 58-years old male presenting with acute onset of varicocele on left side most probable cause: (CMC 98) a. CA testes b. Epididymitis c. Inguinal lymph nodes d. CA kidney

35. Varicocele of pampiniform plexus of veins has all the following characteristics except: (MPPG 97) a. Negative transillumination test b. Reducible c. Cough impulse is present d. Frequently on right side 36. Most common cause of surgically treatable male infertility is: (MAHE 98) a. Varicocele b. Cryptorchidism c. Stricture urethra d. Epididymitis

37. Which of the following is true about varicocele except? a. Incompetent valves of testicular vein are responsible for varicocele (MAHE 2006) b. 90% are on the left side c. Asymptomatic cases require surgery d. Femoral catheterization with spermatic vein ablation is done in recurrence



38. With reference to varicocele, which one of the following is not true of it? (UPSC 2007) a. Varicosity of cremastric veins b. Left side is affected usually c. Feel like a bag of worms d. May lead to infertility



39. After varicocele surgery, venous drainage occurs by: (MHSSMCET 2006) a. Cremasteric veins b. Penile veins c. Ectopic in the iliac fossa d. Present at the usual location



40. True about varicocele is: a. More common on right side b. Can cause oligospermia c. No effect on valsalva d. Lies anterior to testis

VARICOCELE





29. Not true about varicocele is: (AIIMS Nov 2001) a. Common on the right side b. Can present as a later sign of renal cell carcinoma c. Has bag of worm like feeling d. Can lead to infertility 30. Varicocele is common on left testis because: (All India 98) a. Left testicular vein drains into IVC which has high pressure b. Left testicular vein drains into left renal vein which has high pressure c. Left testis is lower situated d. Compression of testicular vein by rectum

(All India 88)



31. What is the grade of given varicocele?

(DNB 2007)

Section 4

41. A young adolescent male came with painless swelling in scrotum since 2-3 months. On palpation, it feels like a bag of worms. What is the possible diagnosis? (MHSSMCET 2010) a. Varicocele b. Testicular abscess c. Epididymo-orchitis d. Hydrocele

a. Subclinical c. Grade II

b. Grade I d. Grade III

32. In the treatment of varicocele, testicular vein ligation is done at the level of: (DPG 96) a. Above inguinal ligament b. Below inguinal ligament c. Neck of the sac d. Scrotum



42. True about varicocele due to renal cell carcinoma is: a. More common on right side (MHPGMCET 2003) b. Temperature induced damage to testes occurs c. Cough impulse is positive d. Does not decompress in supine position



43. Operative managements of varicocele are indicated in which of the following conditions(s)? (PGI Dec 2008) a. Ipsilateral testis small size b. Oligopsermia on semen analysis c. Grade- 3 varicocele (large size) d. Signs or symptoms present e. Subclinical presentation

Testis and Scrotum

Section 4

44. Bag of worm like sensation is felt in: (Recent Question 2015) a. Varicocele b. Hydrocele c. Torsion of testis d. Congenital hernia

SPERMATOCELE

45. Regarding spermatocele which is correct?  a. Occurs in head of epididymus b. Barely water fluid in appearance c. Tender d. Contain spermatozoa



46. Chinese lantern on transillumination seen in: a. Spermatocele c. Hydrocele of cord

(PGI 88)

(Recent Question 2014) b. Epididymal cyst d. Secondary Hydrocele



47. Regarding spermatocele all are correct except: a. Occurs in the head of epididymis b. Barley water fluid c. Tender d. Contain spermatozoa



48. Spermatoceles are most commonly found at: (Recent Question 2017) a. Head of epididymis b. Testes c. Prostate d. Seminal vesicle



56. Lords plication is done for: (All India 2010) a. Inguinal hernia b. Testicular cancer c. Hydrocele d. Testicular varices



57. Classical treatment of hydrocoele: a. Aspiration b. Aspiration and sclerosant agent c. Surgery d. Tapping

HYDROCELE



49. Hydrocele is labeled ‘vaginal’ when it is: a. Limited to scrotum b. Upto inguinal canal c. Communicating into coelomic cavity d. Upto deep inguinal ring

(AIIMS 96)

51. Cause of hydrocele in infants: (CMC 98) a. Patent processus vaginalis b. Patent gubernaculums c. Impaired drainage d. Epididymal cyst e. Infection



52. Congenital hydrocele is best treated by:



(DNB 2009, 2008, 2005, 2001, Punjab 2011, AIIMS June 2001) b. Excision of sac d. Herniotomy

53. Which is false about hydrocele? a. Almost always fluid is transudate b. Get above the swelling c. Testis is separate from swelling d. Obscures inguinal hernia



58. Most common cause of acute epididymitis in males: (COMEDK 2010, GB PANT 2011) a. E. coli b. Proteus c. Chlamydia trachomatis d. N. gonorrhoea



59. Positive Prehn’s sign is: (DNB 2010) a. Elevation of testis increases pain of epididymitis b. Elevation of testis reduces pain of epididymitis c. Depression of testis increases pain of epididymitis d. Depression of testis reduces pain of epididymitis

60. T.B. testis first affects: a. Vas c. Body of testis

61. True about acute epididymitis is: (AIIMS Feb 97, All India 96) a. Associated with urinary infection b. Painless c. Scrotum size is reduced d. Does not mimic with torsion of testes



62. A 16-year-old boy presents with acute onset pain in the left testis. The following statements about his management are true except: (AIIMS Nov 2005) a. The patient should be prescribed antibiotics and asked to come after a week b. Colour flow Doppler will be very useful in diagnosis c. Scrotal exploration should be done without delay if Doppler is not available d. If left testis is not viable on exploration, patient should undergo left Orchidectomy and right orchidopexy

(APPG 2008)



54. Lord’s and Jaboulay’s operation is done for: (AMU 2005) a. Rectal prolapsed b. Fistula in ano c. Inguinal hernia d. Hydrocele



55. What do these images depict? a. Jaboulay’s operation b. Hernia repair c. Surgery for Fournier’s gangrene d. Lord’s plication for hydrocele

(APPG 2016)

(DPG 95) b. Epididymis d. Tunica vaginalis

Urology



a. Eversion of sac c. Lords procedure

(DPG 2008)

EPIDIDYMO‑ORCHITIS

50. Treatment of a large hydrocele in an infant: (Kerala 97) a. Repeated aspirations b. Ligation of sac at the opening of inguinal canal c. Herniotomy d. Eversion of sac



699

700

Surgery Essence



Urology





63. Orchitis without epididymitis is seen in: (All India 92) a. Gonorrhoea b. Tuberculosis c. Syphilis d. Chlamydia infection 64. In differential diagnosis of epididymo-orchitis and torsion it is important that: (AIIMS 95) a. Elevation of testis in torsion relieves pain b. Elevation of testis in epididymoorchitis relieves pain c. Tenderness is characteristic of torsion of testis d. Fever is characteristic of epididymoorchitis 65. Acute orchitis all are seen except: a. Increased local temperature b. Decreased blood flow c. Etythematous scrotum d. Raised TLC 66. Prehn sign is seen in: a. Acute orchitis c. Testicular torsion

TESTICULAR CARCINOMA

74. Most radiosensitive testicular tumour is: (MCI March 2008) a. Seminoma b. Teratoma c. Interstitial tumours d. Lymphoma



75. Most common testicular tumour in 4th decade: (MCI Sept 2008) a. Teratoma b. Dermoid c. Seminoma d. All of the above



76. Which of the following is false about testicular tumor? a. Pure seminoma is more aggressive than non-seminoma b. Seminoma is radiosensitive (AIIMS May 2009) c. Seminoma can be treated by orchidectomy with radiotherapy d. Seminoma spreads commonly through lymphatics



77. A patient presented with a hard swelling in this right testis. All are true statements except: (AIIMS Nov 2001) a. Trans scrotal biopsy is needed b. Inguinal exploration is done c. High inguinal exploration should be done d. Scrotal USG is done



78. A 20 years old male presents with hard painless testicular swelling. On investigation, AFP is 3080 ng/ml. No para‑aortic or iliac nodes as well as no mediastinal lymph nodes found. Ultrasound shows uniform echotexture and small areas of necrosis. Surrounding structures are normal. What is the next best step? a. FNAC (AIIMS May 2018) b. Trucut biopsy c. High inguinal orchidectomy d. PET-CT



79. Germ cell tumors of testis are: (PGI June 2001) a. Seminoma b. Teratoma c. Leydig cell tumour d. Gynandroblastoma e. Rhabdomyoma



80. Most common testicular tumor in prepubertal adults is:   (AIIMS May 2008) a. Yolk sac tumor b. Embryonal cell Ca c. Seminoma d. Teratoma



81. Intratubular germ cell tumor found adjacent to: (PGI May, Dec 2006) a. Spermatocytic seminoma b. Dysgerminoma c. Yolk sac tumor d. Embryonal carcinoma e. Choriocarcinoma



82. Which of the following is not seen in testicular carcinoma? a. Abdominal lump (APPG 2006) b. Epidydimoorchitis c. Inguinal lymphadenopathy d. Hydrocele



83. All of the following clinicopathologic features are seen more often in seminomas as compared to non-seminomatous germ cell tumors of the testis except: (AIIMS May 2005, Nov 2004) a. Tumors remain localized to testis for a long time b. They are radiosensitive c. They metastasize predominantly by lymphatics d. They are often associated with raised levels of serum AFP and HCG

(Recent Question 2013)

(Recent Question 2015; 2013) b. Chronic orchitis d. None

67. Which of the following statements is true regarding acute epididymitis? (APPG 2016) a. All the statements are true b. Mostly bilateral c. Absence of blood flow on doppler Examination d. Occurs in young sexually active men and is commonly due to C. trachomatis

FOURNIER’S GANGRENE (JIPMER 90)



68. Fournier’s gangrene occurs in the: a. Toes b. Scrotum c. Fingers d. Muscles



69. All are features of Fournier’s gangrene except: (MAHE 2007) a. Testicles are involved b. Obliterative arteritis seen c. Hemolytic streptococci, isolated d. Necrotizing fascitis



70. All are features of Fournier’s gangrene except: (MAHE 2008) a. Testicles are involved b. Obliterative arteritis seen c. Hemolytic streptococci d. Necrotising fasciitis e. E. coli, staphylococci, Cl. welchii can be isolated



71. Fournier’s gangrene is seen in: a. Scrotum b. Shaft of penis c. Base of penis d. Glans penis

(MCI Sept 2008)

Section 4

TESTICULAR CARCINOMA PREDISPOSING FACTORS



72. Predisposing factor of Testicular germ cell tumor: a. Cryptorchidism (PGI Dec 2003) b. Testicular feminization syndrome c. Klinefelter’s syndrome d. Radiation e. Trauma 73. Testicular cancer is common in: (All India 91) a. Ectopic testis b. Undescended abdominal testis c. Atrophic testis d. Anteverted testis

84. Following statements are true about germ cell tumors of testis except: (AIIMS Nov 2002) a. They constitute 90–95% of pall primary testicular tumors b. Seminoma is the most common tumor developing in the patients with cryptorchid testis c. AFP is markedly raised in all germ cell tumors d. High inguinal orchidectomy is the initial surgical procedure

Testis and Scrotum 85. Testicular tumor with best prognosis is: (PGI Dec 96) a. Teratoma b. Seminoma c. Choriocarcinoma d. All of the above



86. Marker for testicular tumor is: (AIIMS June 97, PGI June 95) a. Beta-hCG b. Acid phosphatase c. Alkaline phosphatase d. Alpha-fetoprotein



87. A testicular tumor in a man aged 60-years is; most likely to be: (All India 2001) a. Germ cell tumor b. Sertoli cell tumor c. Teratocarcinoma d. Lymphoma

88. Shyam, a 50-years old male presents with a hard scrotal swelling. All of the following can be done except: 

a. Testicular biopsy c. Inguinal exploration

(AIIMS June 2001) b. Chest X-ray d. CT abdomen

89. Regarding testicular tumour, the following are false except: (AIIMS June 2003) a. They are commonest malignancy in older man b. Seminomas are radiosensitive c. Only 25% of stage I teratomas are cured by surgery alone d. Chemotherapy rarely produces a cure in those with metastatic disease



90. Placental alkaline phosphatase is marker of: a. Theca cell tumour b. Teratoma c. Choriocarcinoma d. Seminoma



91. Tumor marker for seminoma: a. PLAP b. LDH c. AFP d. hCG



92. What % of testicular carcinoma is associated with cryptorchidism? a. 10% b. 30% (PGI Dec 98) c. 70% d. 90%



93. Which of the following statements is true regarding testicular tumors? (All India 2006) a. Are embryonal cell carcinomas in 95% of cases b. Bilateral in upto 10% cases c. Teratomas are more common than seminomas d. Usually present after 50-years of age



(PGI Dec 99)

(AIIMS Nov 2013)

94. Which of the following is true about seminoma?  (PGI Dec 2005) a. It is radiosensitive b. It arises only from cryptorchid testis c. AFP is increased d. It is chemosensitive (DNB 2011)



95. Testicular teratoma in adults is: a. Benign b. Malignant c. Locally aggressive d. Border line



96. It is true about seminoma testis that: a. It commonly occurs in the 6th decade b. An undescended tests is more liable to develop this tumour c. Histologically it resembles dysgerminoma of the ovary d. It is radioresistant



97. Seminoma testis is seen: a. Always in undescended testis b. Always bilateral c. Occurs in younger d. Occurs in elderly

(Assam 96)

98. Testis tumor is associated with secondary hydrocele in: (Karnataka 95) a. 1% of cases b. 10% of cases c. 20% of cases d. 30% of cases



99. Most common testicular tumor in prepubertal adults is: (AIIMS May 2008) a. Yolk sac tumor b. Embryonal cell Ca c. Seminoma d. Teratoma

100. Carcinoma testis, the lymphatic metastasis goes to the first site seen in: (UPPG 2010) a. Para-aortic lymph nodes b. Superficial inguinal nodes c. Deep inguinal nodes d. Internal iliac nodes 101. A 25-year-old man presents with hydrocele on the left side. Associated condition could be a: a. Nephroma b. Hepatic malignancy c. Testicular tumour d. Penile malignancy 102. All are true regarding seminoma except: a. Common in age between 35 and 45 years b. Metastasis to lymphatics c. Radioresistant d. Not seen before puberty

(UPPG 2009)

103. Seminoma of testis has all the following tumour markers except: (COMEDK 2006) a. AFP b. hCG c. LDH d. Alkaline phosphate 104. Most malignant testicular tumour is: (DNB 2004) a. Seminoma b. Teratoma c. Choriocarcinoma d. Embryonal carcinoma 105. A 20-year-old male presents with scrotal mass. The first investigation to be done is: (Recent Question 2014; JIPMER 2011) a. Clinical evaluation (Palpation and transillumination) b. USG c. Biopsy d. AFP 106. Most radiosensitive testicular tumor is: (UPSC 2005, MHPGMCET 2002) a. Seminoma b. Teratoma c. Lymphoma d. Sertoli cell tumor 107. Testicular tumor, that is rare in childhood: (MHPGMCET 2006) a. Seminoma b. Teratoma c. Interstitial cell tumor d. None 108. Testicular tumor can simulate: a. Hydrocele b. Hematocele c. Acute epididymoorchitis d. Chronic orchitis

(PGI SS June 2004)

109. Most common testicular tumor in children:  (Recent Question 2017) a. Yolk sac tumor b. Leydig cell tumor c. 
Seminoma d. Choriocarcinoma 110. A 12-year-old boy presents with serotal mass. The next best things to do in this patent is: (JIPMER 2011) a. Clinical evaluation b. USG c. Biopsy d. Immenate surgery

Urology





Section 4



701

702

Surgery Essence TESTICULAR CARCINOMA STAGING 111. High inguinal orchidectomy specimen showed tumor testis with involvement of epididymis without vascular invasion; stage is: (MAHE 2007) a. T1 b. T2 c. T3 d. T4 112. High inguinal orchiectomy specimen showed teratoma testis with involvement of epididymis; stage is:  (DNB 2011, MAHE 2008) a. T1 b. T2 c. T3 d. T4b

Urology

TESTICULAR CARCINOMA TREATMENT 113. Stage I seminoma testis, treatment of choice is: a. High inguinal orchidectomy (AIIMS Nov 2001) b. High inguinal orchidectomy and radiotherapy c. Radiotherapy and chemotherapy d. Trans-scrotal orchidectomy 114. The treatment of metastatic testicular carcinoma is: a. Bleomycin, Etoposide, Cisplatin (PGI June 99) b. Vinablastine, Etoposide, Cisplatin c. Doxorubicin, 5-FU, mercaptopurine d. Methotrexate, 5-FU, Vincristine 115. A 27-year-old man presents with a left testicular tumor with a 10 cm retroperitoneal lymph node mass. The treatment of choice is: (All India 2003) a. Radiotherapy b. Immunotherapy with interferon and interleukins c. Left high inguinal orchiectomy plus chemotherapy d. Chemotherapy alone 116. Treatment of extragonadal germ cell tumour is: (All India 99) a. Chemotherapy b. Radiotherapy c. Surgery d. Immunotherapy

Section 4

117. Which one of the following is the treatment of choice for a 4 cm retroperitoneal lymph node mass in a patient with non seminomatous germ cell tumor of the testis? (AIIMS Nov 2004) a. Radical radiotherapy alone b. High orchidectomy + RPLND c. RPLND alone d. High orchidectomy alone 118. A patient comes with stage III non seminomatous testicular tumor, treatment of choice is: (AIIMS June 97) a. Radiotherapy b. Chemotherapy c. Hormonal therapy d. Surgery 119. Disseminated seminoma is treated by: a. Chemotherapy or radiotherapy and ochidectomy b. Only radiotherapy c. Only chemotherapy d. RPLND 120. Stage-II testicular teratoma is treated by: a. Orchidectomy + RPLND (DNB 2008, 2005, AMU 05) b. Orchidectomy + Chemotherapy c. Orchidectomy d. Radiotherapy

(MCI Sept 2008)

121. Treatment of stage I teratoma is: a. Chemotherapy b. Radiotherapy c. Chemotherapy plus Radiotherapy d. Observation /RPLND

122. Which of the following is a known complication of modified RPLND (Retroperitoneal lymph node dissection) done for nonseminomatous germ cell tumor of testis? (MHSSMCET 2005) a. Impotence b. Bladder atony c. Dry ejaculation d. Retrograde ejaculation 123. Which of the following testicular tumor does not require RPLND? (MHSSMCET 2005) a. Germ cell tumor b. Embryonal cell tumor c. Seminoma d. Teratoma 124. Treatment of Non-seminomatous germ cell tumor of testis with more than 4 cm RPLN includes: (PGI May 2011) a. RPLND b. Inguinal orchidectomy c. Chemotherapy d. Radiotherapy e. Orchidectomy through mid testicular incision

SEX CORD/GONADAL STROMAL TUMORS 125. Not true of sertoli cell tumour: a. Poor response to radiotherapy b. Prominent lymphocytes in section c. Common in adults d. Can be malignant in 10–20% of cases

(Punjab 2009)

CARCINOMA SCROTUM 126. The lymph nodes first involved in cancer of the skin of the scrotum are: (Karnataka 96) a. Superficial inguinal b. External iliac c. Para aortic d. Gland of Cloquet

ORCHIDECTOMY 127. Subcapsular orchiectomy is done for cancer of: (DNB 2008, APPG 96) a. Tests b. Prostate c. Penis d. Urethra 128. Ligation of cord in orchidectomy for treatment of testicular tumor is done at: (PGI 96) a. External ring b. Internal ring c. Base of scrotum d. Just above epididimis 129. Subcapsular orchidectomy is done for cancer of: (DPG 2005) a. Testis b. Prostate c. Penis d. Male breast cancer 130. Orchidectomy is not done in: a. Prostate cancer b. Seminoma testes c. Filarial epididemo-orchitis d. Male breast cancer

(AIIMS June 2001)

Testis and Scrotum (AIIMS Nov 99)

MISCELLANEOUS 132. Dermoid arises from: a. Pluripotent cell c. Ectoderm

(PGI June 97) b. Totipotent cell d. Mesoderm

133. Differential diagnosis of acute funiculitis with a small inguinal swelling is: (TN 91) a. Undescended testes b. Acute orchitis c. Lymphadenitis d. Small strangulated inguinal hernia 134. Which of the following closely mimics testicular malignancy? a. Hydrocele b. Hematocele c. Spermatocele d. Cyst of epididymis

135. The life of preserved semen for artificial insemination is: a. One year b. Two years c. Five years d. Ten years e. Fifty years 136. Best indication for testicular biopsy in a male is: (Bihar PG 2014; All India 97) a. Polyspermia b. Oligospermia c. Necrospermia d. Azoospermia 137. Which of the following structure in the spermatic cord is not damaged during vasectomy? (AIIMS Nov 2012) a. Testicular artery b. Ilioinguinal nerve c. Autonomic nerves d. Pampiniform plexus

Section 4

131. Orchiectomy is not indicated in: a. Seminoma testis b. Prostatic carcinoma c. Tubercular epididymitis d. Male breast cancer

703

138. Young male with history of trauma having left sided testis swollen and erythematous. Other side normal. Diagnosis:  (Recent Question 2013) a. Torsion b. Carcinoma c. Hematoma d. Hernia

Urology

Explanations UNDESCENDED TESTIS

1. Ans. b. 6 months of age (Ref: Smith 18/e p380; Campbell 11/e p3443; Bailey 27/e p1498)

If spontaneous testicular descent does not occur, surgical treatment after 6 months of (corrected gestational) age is indicated.

2. Ans. a. 6 months (Ref: Sabiston 20/e p1886; Abdominal wall Hernias- Principle and Management: Springer 2001/176)

Timing of Orchiopexy in UDT Sabiston 20/e p1886, 18/e p2071 • In most pediatric centers, orchiopexy for unilateral UDT is done when patient have reached about 6 months of ageQ. This early intervention may permit post-natal germ cell development to proceed normallyQ. Abdominal wall Hernias- Principle and Management: Springer 2001/176 • In infants with congenital UDT, still undescended after 12 weeks of age, orchiopexy is recommended at 6 months of age before germ cell development becomes derangedQ. • Achieve scrotal placement ideally by 6 months of age and by 1 year of age at the latestQ.

3. Ans. a. 3% (Campbell 11/e p3434)

4. Ans. a. Seminoma (Campbell 11/e p3451)



5. Ans. b. Undescended testis

6. Ans. a. Testicular tumor



7. Ans. c. Laparoscopy (Ref: Smith 18/e p140; Campbell 11/e p3440; Bailey 27/e p1499)

Diagnostic laparoscopy is a definitive investigation for an absence of testis (anorchia). • “Impalable undescended testes are either absent or located in the abdomen or inguinal canal. There is no benefit from imaging and these are best managed with a laparoscopy and usually a staged approach.” –Bailey 26/e p111

8. Ans. d. Laparoscopy (Ref: Campbell 11/e p3441; Bailey 27/e p125) • “Laparoscopy is the procedure of choice to confirm or exclude the presence of a viable or remnant abdominal testis, unless a prominent scrotal nubbin is palpable with other clinical signs of monorchism.” –Campbell 11/e p3441



9. Ans. a. Infancy

10. Ans. a. Hydrocele



11. Ans. a. Right side (Campbell 11/e p3434)

12. Ans. a. Orchiopexy



13. Ans. c. Has higher incidence of malignancy

14. Ans. c. Poorly developed secondary sexual characters



15. Ans. b. 4 months

16. Ans. d. 1 year



17. Ans. d. Cryptorchidism (Ref: Campbell 11/e p3447)

ECTOPIC TESTIS

18. Ans. a. Superficial inguinal pouch (Ref: Smith 18/e p25; Bailey 27/e p1498)



19. Ans. a. Lumbar, c. Intra abdominal

20. Ans. d. All

TESTICULAR TORSION

21. Ans. b. Commonly associated with pyuria (Ref: Smith 18/e p707; Campbell 11/e p3391; Bailey 27/e p1500)



22. Ans. d. If torsion confirmed, treat with antibiotics and analgesics and perform corrective surgery after 14 days



23. Ans. b. Low investment of tunica vaginalis

24. Ans. c. Elevation of testis reduces the pain



25. Ans. c. Torsion of testis

26. Ans. a. Immediately (Ref: Campbell 11/e p3391)



27. Ans. a. 6 hours (Ref: Campbell 11/e p3391)



28. Ans. c. Anatomical abnormality is unilateral and contralateral testis should not be fixed (Ref: Campbell 11/e p3391)

VARICOCELE

29. Ans. a. Common on the right side (Ref: Smith 18/e p707; Campbell 11/e p3393; Bailey 27/e p1501)

Testis and Scrotum 30. Ans. b. Left testicular vein drains into left renal vein which has high pressure (Ref: Campbell 11/e p3393)



31. Ans. d. Grade III (Ref: Sabiston 20/e p2081; Schwartz 10/e p1653; Bailey 27/e p1501) The given varicocele is of grade III (visible and palpable at rest). Varicocele Classification Subclinical

Not palpable or visible at rest or during Valsalva maneuver, but demonstrable by special testsQ (reflux found on Doppler examination)

Grade 1

Palpable during ValsalvaQ maneuver, but not otherwise

Grade 2

Palpable at restQ, but not visible

Grade 3

VisibleQ & palpable at rest



32. Ans. a. Above inguinal ligament

33. Ans. b. More common on the right side



34. Ans. d. CA kidney

35. Ans. d. Frequently on right side



36. Ans. a. Varicocele



37. Ans. c. Asymptomatic cases require surgery (Ref: Campbell 11/e p3393)



38. Ans. a. Varicosity of cremasteric veins

39. Ans. a. Cremasteric veins



40. Ans. b. Can cause oligospermia

41. Ans. a. Varicocele (Ref: Campbell 11/e p3393)



42. Ans. d. Does not decompress on lying down position (Ref: Campbell 11/e p3393)



43. Ans. a. Ipsilateral testis small size, b. Oligopsermia on semen analysis, d. Signs or symptoms present (Ref: Campbell 11/e p3393)



44. Ans. a.Varicocele (Ref: Campbell 11/e p3393)

Section 4



705

SPERMATOCELE

45. Ans. a. Occurs in head of epididymis, b. Barely water fluid in appearance, d. Contain spermatozoa (Ref: Bailey 27/e p1504)



46. Ans. b. Epididymal cyst

47. Ans. c. Tender

• Chinese-Lantern pattern on transillumination is more commonly seen in epididymal cyst.

48. Ans. a. Head of epididymis

HYDROCELE 49. Ans. a. Limited to scrotum (Ref: Campbell 11/e p3384; Bailey 27/e p1503)



50. Ans. c. Herniotomy

51. Ans. a. Patent processus vaginalis (Ref: Campbell 11/e p3384)



52. Ans. d. Herniotomy

53. Ans. c. Testis is separate from swelling



54. Ans. d. Hydrocele (Ref: Campbell 11/e p3384; Bailey 27/e p1503)



55. Ans. a. Jaboulay’s operation (Ref: Campbell 11/e p3384; Bailey 26/e p1382) Treatment of Vaginal hydrocele • Small hydrocele: Lord’s procedure (Plication of sac)Q • Medium hydrocele: Jaboulay’s procedure (Eversion of sac)Q • Large hydrocele: Excision of sacQ



56. Ans. c. Hydrocele

57. Ans. c. Surgery (Ref: Campbell 11/e p3386)

EPIDIDYMO‑ORCHITIS

58. Ans. c. Chlamydia trachomatis (Ref: Smith 18/ep241; Campbell 10/e p3117-3118; Bailey 27/e p1505)



59. Ans. b. Elevation of testes reduces pain of epididymitis



60. Ans. b. Epididymis



61. Ans. a. Associated with urinary infection



62. Ans. a. The patient should be prescribed antibiotics and asked to come after a week



63. Ans. c. Syphilis



64. Ans. b. Elevation of testis in epididymoorchitis relieves pain (Ref: Smith 18/e p241; Campbell 11/ep 2913; Bailey 27/e p1505)

Urology



706

Surgery Essence Testicular Torsion

Epididymoorchitis

• Seen in prepubertal age group (10-25) • Urine culture is sterile • No fever, sudden agonizing painQ with affected testis high-ridingQ in scrotum • Cremasteric reflex is absentQ • Prehn’s sign is negativeQ • Color Doppler: Decreased blood flowQ • Treatment: Immediate surgical explorationQ

Urology

Q

• Adults, age group 20-30Q • Evidence of UTI • Fever, swollen, red and tender scrotum, thickened cord with reactive hydroceleQ • Cremasteric reflex is presentQ • Prehn’s sign is positiveQ • Scrotal USG: Enlarged epididymis with increased blood flow with reactive hydroceleQ. • Treatment: Antibiotics, rest , scrotal elevation and NSAIDsQ.



65. Ans. b. Decreased blood flow

66. Ans. a. Acute orchitis



67. Ans. d. Occurs in young sexually active men and is commonly due to C. trachomatis

FOURNIER’S GANGRENE

68. Ans. b. Scrotum (Ref: Campbell 11/e p403; Bailey 27/e p1509)



69. Ans. a. Testicles are involved

70. Ans. a. Testicles are involved

71. Ans. a. Scrotum

TESTICULAR CARCINOMA PREDISPOSING FACTORS

72. Ans. a. Cryptorchidism, b. Testicular feminization syndrome, c. Klinefelter’s syndrome (Ref: Smith 18/e p380; Campbell 11/e p784; Bailey 27/e p1506)



73. Ans. b. Undescended abdominal testis

TESTICULAR CARCINOMA

74. Ans. a. Seminoma (Ref: Smith 18/e p381; Campbell 11/e p784; Bailey 27/e p1506; CSDT 11/e p1071)



75. Ans. c. Seminoma



76. Ans. a. Pure seminoma is more aggressive than non-seminoma (Ref: Smith 18/e p381; Campbell 11/e p788; Bailey 27/e p1506)



77. Ans. a. Trans scrotal biopsy is needed



78. Ans. c. High inguinal orchidectomy



79. Ans. a. Seminoma, b. Teratoma (Ref: Smith 18/e p381; Campbell 11/ep786; CSDT 11/e p1071)



80. Ans. d. Teratoma (Ref: Smith 18/e p381; Campbell 11/e p788)

Section 4

• MC testicular tumour in prepubertal adults: TeratomaQ • MC testicular tumor of infants and children: Yolk sac tumorQ

81. Ans. b. Dysgerminoma, d. Embryonal carcinoma, e. Choriocarcinoma (Ref: Smith 18/ep382; Campbell 11/e p797)



82. Ans. c. Inguinal lymphadenopathy



83. Ans. d. They are often associated with raised levels of serum AFP and hCG (Ref: Smith 18/e p383; Campbell 11/e p790)



84. Ans. c. AFP is markedly raised in all germ cell tumors



85. Ans. b. Seminoma (Ref: Smith 18/e p381; Campbell 11/e p787; Bailey 27/e p1506)



86. Ans. a. Beta‑hCG, d. Alpha fetoprotein



87. Ans. d. Lymphoma (Ref: Smith 18/e p388; Campbell 11/e p812)



88. Ans. a. Testicular biopsy



91. Ans. a. PLAP (Ref: Smith 18/e p383; Campbell 11/ep 788)

89. Ans. b. Seminomas are radiosensitive

90. Ans. d. Seminoma

• Tumor marker for seminoma is PLAP (Placental alkaline phosphatase). Though beta-hCG is also raised in 5-10% of pure seminoma, as they contain syncytiotrophoblast like giant cells.

92. Ans. a. 10% (Ref: Smith 18/e p380) The strongest association has been with the cryptorchid testis. Approximately 7-10% of testicular tumors develop in patients who have a history of cryptorchidism; seminoma is the most common form of tumor these patients have.

Testis and Scrotum

707

Testicular Tumors: Epidemiology and Risk Factors

• Of primary testicular tumors, 1-2% are bilateralQ, and about 50% of these tumors occur in men with a history of unilateral or bilateral cryptorchidism. • Seminoma is the most common germ cell tumor in bilateral primary testicular tumorsQ, while malignant lymphoma is the most common bilateral tumor of the testisQ.

Section 4

• Of all primary testicular tumors, 90-95% are germ cell tumorsQ (seminoma and nonseminoma) • More common in whites and individuals of higher socioeconomic classQ • Slightly more common on the right side than on the left, which parallels the increased incidence of cryptorchidism on the right sideQ.

• The strongest association has been with the cryptorchid testisQ. Approximately 7-10%Q of testicular tumors develop in patients who have a history of cryptorchidism; seminoma is the most common form of tumor these patients have. • However, 5-10% of testicular tumors occur in the contralateral, normally descended testisQ. • The relative risk of malignancy is highest for the intraabdominal testisQ (1 in 20) and is significantly lower for the inguinal testis (1 in 80). • Orchiopexy does not alter the malignant potential of the cryptorchid testis; however, it does facilitate examination and tumor detectionQ.

93. Ans. None

94. Ans. a. It is radiosensitive

95. Ans. b. Malignant



96. Ans. b. An undescended testis is more liable to develop this tumour, c. Histologically it resembles dysgerminoma of the ovary



97. Ans. c. Occurs in younger



98. Ans. b. 10% of cases (Ref: Smith 18/e p791)

• Approximately 5-10% of testicular tumors may be associated with hydrocelesQ.

99. Ans. d. Teratoma

100. Ans. a. Para-aortic lymph nodes

101. Ans. c. Testicular tumour

102. Ans. c. Radioresistant

104. Ans. c. Choriocarcinoma

105. Ans. b. USG

106. Ans. a. Seminoma

107. Ans. a. Seminoma

109. Ans. a.Yolk sac tumor (Ref: Campbell 11/e p3594)

110. Ans. a. Clinical evaluation

103. Ans. a. AFP 108. Ans. b. Hematocele

TESTICULAR CARCINOMA STAGING 111. Ans. a. T1 (Ref: Campbell 11/e p791)

112. Ans. a. T1

113. Ans. b. High inguinal orchidectomy and radiotherapy

114. Ans. a. Bleomycin, Etoposide, Cisplatin

115. Ans. c. Left high inguinal orchiectomy plus chemotherapy (Ref: Smith 18/e p384; Campbell 11/e p796; Bailey 27/e p1508) 116. Ans. a. Chemotherapy

117. Ans. b. High Orchidectomy + RPLND

118. Ans. b. Chemotherapy

119. Ans. c. Only chemotherapy

120. Ans. a. Orchidectomy + RPLND (Ref: Smith 18/e p385; Campbell 11/e p796; Bailey 27/e p1508) 121. Ans. d. Observation /RPLND

122. Ans. d. Retrograde ejaculation (Ref: Smith 18/e p385, 17/e p380)

Retroperitoneal Lymph Node Dissection (RPLND) • RPLND has been the preferred treatment of low-stage NSGCTsQ • A thoracoabdominal or midline Transabdominal approach may be used • All nodal tissue between the ureters from the renal vessels to the bifurcation of the common iliac vessels is removedQ. • RPLND is associated with significant morbidity, especially with respect to fertility in young menQ. • With a standard RPLND, sympathetic nerve fibers are disrupted, resulting in loss of seminal emissionQ.

123. Ans. c. Seminoma

124. Ans. a. RPLND, b. Inguinal orchidectomy, c. chemotherapy

Urology

TESTICULAR CARCINOMA TREATMENT

708

Surgery Essence SEX CORD/GONADAL STROMAL TUMORS 125. Ans. b. Prominent lymphocytes in section (Ref: Smith 18/e p387; Campbell 11/e p811)

CARCINOMA SCROTUM 126. Ans. a. Superficial inguinal (Ref: Smith 18/e p391)

ORCHIDECTOMY 127. Ans. b. Prostate (Ref: Bailey 25/e p1378; Smith 18/e p372)

Urology

Bilateral orchidectomy, whether total or subcapsular, will eliminate the major source of testosterone production in patients of carcinoma prostate.

Carcinoma Prostate • Orchidectomy is performed to carry out androgen ablation in the treatment of locally advanced (T3 or T4) disease or of metastatic diseaseQ. • In 1941, prostate cancer was shown to be responsive to such treatment by Charles Huggins, the only urologist to win a Nobel Prize. • Bilateral orchidectomy, whether total or subcapsularQ, will eliminate the major source of testosterone production. 128. Ans. b. Internal ring (Ref: Bailey 27/e p1508)

High Inguinal Orchidectomy • The cord must be ligated as close as possible to the internal ring to facilitate complete removal of cord tissueQ in case a later retroperitoneal lymph node dissection is required. 129. Ans. b. Prostate

130. Ans. c. Filarial epididemo-orchitis

131. Ans. c. Tubercular epididymitis (Ref: Bailey 27/e p1474, 1508)

Indications of Orchidectomy • Clotted hydroceleQ        •  Testicular tumorsQ

•  Prostate cancerQ     •  Male breast cancerQ

MISCELLANEOUS 132. Ans. b. Totipotent cell

133. Ans. d. Small strangulated inguinal hernia

134. Ans. b. Hematocele

135. Ans. d. Ten years

136. Ans. d. Azoospermia (Ref: Smith 18/e p699)

Testicular Biopsy in Male Infertility

Section 4

• The testis biopsy provides direct information regarding the state of spermatogenesisQ. • Abnormalities of seminiferous tubule architecture and cellular composition are then categorized into several patternsQ. • This procedure is most useful in the azoospermic patientQ, in which it is often difficult to distinguish between a failure of sperm production and obstruction within the reproductive tract ducts. • A testis biopsy allows definitive delineation between these 2 conditions and can guide further treatment options in azoospermic menQ. • Testis biopsies may also be indicated to identify patients at high risk for intratubular germ cell neoplasia. This premalignant condition exists in 5% of men with a contralateral germ cell tumor of the testis and is more prevalent in infertile than fertile menQ. 137. Ans. b. Ilioinguinal nerve (Ref: Grays 40/e p1262) Ilioinguinal nerve is not a constituent of spermatic cord, hence, it is not damaged during vasectomy. 138. Ans. c. Hematoma (Ref: Schwartz 9/e p1467)

SECTION

5

Cardiothoracic Vascular Surgery CH A PT ERS ˆˆ ˆˆ ˆˆ ˆˆ

Arterial Disorders Venous Disorders Lymphatic System Thorax and Lung

CHAPTER

26

Arterial Disorders

ARTERIAL OCCLUSION Causes of Peripheral Arterial Occlusive Disease

Common

Rare

Atherosclerosis (MC)Q Buerger’s diseaseQ Takayasu arteritis SLE Post-traumatic Radiation injury

External compression Popliteal entrapment Thoracic outlet syndrome Retroperitoneal fibrosis Coarctation of aorta Cystic medial necrosis Fibromuscular dysplasia Causes of Arterial Occlusive Disease

Atherosclerosis

Buerger’s Disease

• Seen in patients >40 yearsQ • MC in 6th or 7th decadeQ • Causes occlusion of largeQ & mediumQ sized vessels (Abdominal aorta, iliac, femoral, Popliteal, tibial & peroneal arteries) • Symptoms of peripheral arterial insufficiency is predominantly seen in lower limbsQ

• Seen in young males Mural thrombus following MIQ • Less common sources: Aneurysms & thrombi formed on atheromatous plaques Leg Brain Retina Mesenteric vessels Spleen Kidneys

Emboli cause Ischemic Symptoms • Pain, pallor, paresis, pulselessness & paraesthesiaQ • TIA or strokeQ • Amaurosis fugax • Possible gangrene of corresponding loop of intestineQ • Local pain • Loin pain and hematuria

Cardiothoracic Vascular Surgery

712

Surgery Essence Clinical Features • Embolic arterial occlusion is an emergency that requires immediate treatmentQ. • The leg is often affected, with pain, pallor, paresis, loss of pulsation & paraesthesia (or anesthesia). Diagnosis can be made clinicallyQ in a patient who has no history of claudication and has a source of emboli, who suddenly develops severe pain or numbness of the limb, which becomes cold & mottledQ. • Movement becomes progressively more difficult and sensation is lostQ. • Pulses are absent distallyQ but the femoral pulse may be palpable. Treatment • Because of the ensuing stasis, a thrombus can extend distally and proximallyQ to the embolus. • Immediate administration of 5000 U of heparin IV can reduce this extension and maintain patency of the surrounding (particularly the distal) vessels until the embolus can be treatedQ. • The relief of pain is essential because it is severe & constant. • Embolectomy & thrombolysisQ are the treatments available for limb emboli. • Dextran-40 or 70 to reduce plasma viscosity can be given in the management of the ischemic limb for temporary improvementQ. • Low molecular weight dextrans are used during acute attack of thromboangitis. They cause hemodilution, decrease viscosity of blood & improve microcirculation. Intra-arterial injection is said to be more effective than intravenousQ.

BUERGER’S DISEASE (THROMBOANGITIS OBLITERANS) Buerger’s Disease (Thromboangitis Obliterans) • Segmental inflammatory diseaseQ, affecting small & medium sizedQ arteries in upper & lowerQ extremities • Inflammatory process involves neighboring veins & nervesQ • Definite relationship with smokingQ Histopathology • Sharply segmental acute & chronic vasculitis of small & medium vessels with thrombosis of lumen which may undergo organization & recanalizationQ • Thrombus contains microabscesses • Inflammatory process extends to involve neighboring veins & nervesQ • With time, all three structures (artery, vein & nerve) become incased in fibrous tissueQ Clinical Features (RIM) • Characterized by triad of intermittent claudication, Raynaud’s phenomenon & migratory superficial vein thrombophlebitisQ • Typically seen in young (1.2 1.0-1.2 0.5-0.9 0.1-0.4

Interpretation Noncompressible, severely calcified vessel (in DM & ESRD)Q Normal vesselsQ Intermittent claudicationQ (mild to moderate ischemia) Critical limb ischemiaQ (Ischemic ulceration, gangrene)

Fontaine Classification of Limb Ischemia Stage I

AsymptomaticQ

Stage IIa

Mild claudicationQ

Stage IIb

Moderate to severe claudicationQ

Stage III

Ischemic rest painQ

Stage IV

Ulceration or gangreneQ

Cardiothoracic Vascular Surgery

• • • •

Q

714

Surgery Essence

Cardiothoracic Vascular Surgery

ARTERIAL ULCER Arterial Ulcer • Arterial insufficiency ulcers ischemic ulcers are mostly located on the lateral surface of the ankle or the distal digitsQ. • Most common on distal ends of limbsQ. Etiology • Caused by lack of blood flow to the capillary beds of lower extremities. • Most often endothelial dysfunction is causative factor in diabetic microangiopathy & macroangiopathyQ Characteristic Features • Punched-out appearanceQ •  Intensely painfulQ Q • Pulses are not palpable • Associated skin changes (thin shiny skin, absence of hair, brittle nailsQ) Diagnosis • The lesion can be easily identified clinically. • Arterial doppler & pulse volume recordings for baseline assessment of blood flow. • Radiographs may be necessary to rule outosteomyelitis. Treatment • Vascular surgery to revascularize the area. • In infection: Antibiotics + DebridementQ  • Ischemic time for digits is upto 8 hoursQ. • Ischemic time for extremities is 4-6 hoursQ. • Organ containing bag should be placed in a solution of saline with iceQ.

ARTERIOVENOUS FISTULA (AVF) Arteriovenous Fistula (AVF) • AVF (communication between an artery & vein) may be congenital or acquired (penetrating trauma or surgically created for hemodialysis) • MC type of AVF: CongenitalQ • MC cause of acquired AVF: Penetrating traumaQ • Structural effects on veins: Veins are arterialized (become dilated, tortuous & thick walledQ) Physiological Effects of AVF • Increased pulse pressureQ (Increased systolic & decreased diastolic) • Increased venous return leading to increased HR & increased COQ • Left ventricular enlargementQ and later cardiac failureQ may occur • A congenital fistula in the young patient may cause overgrowth of the limbQ • In the leg indolent ulcers may result from relative ischemia below the short circuitQ Clinical Signs

Section 5

• A pulsatile swellingQ      • Thrill on palpationQ     • Continuous bruit on auscultationQ • Nicoladoni’s or Branham’s sign: Pressure on artery proximal to fistula causes the swelling to diminish in size, a thrill or bruit to cease, the pulse rate to fall & the pulse pressure returns to normalQ. Diagnosis • Duplex scan and/or angiography confirm the diagnosisQ. Treatment • Treatment is by embolizationQ.

 •  Excisional surgery (rarely) for severe deformity or recurrent hemorrhageQ. Types of Surgically Created Fistula

Brescia-Cimino Fistula

• Radial artery & cephalic veinQ

Snuffbox Fistula

• Posterior branch of Radial artery & cephalic vein

Feinberg Fistula

• Radial artery & basilic veinQ

Arterial Disorders

715

THORACIC OUTLET COMPRESSION SYNDROME (TOS)

• TOS refers to compression of subclavian vessels & nerves of brachial plexus in the region of thoracic inletQ. • Divided into: Vascular forms (Arterial and /or Venous) & Neurogenic forms • Compression resulting from TOS is dynamic & best evaluated clinically by mechanical provocative maneuversQ • Symptoms most commonly develop secondary to neural compromiseQ • Middle-aged womenQ are most commonly affected

Section 5

Thoracic Outlet Compression Syndrome (TOS)

Neurovascular structures of the upper extremity may be compressed by Cervical ribQ Long transverse process of C7Q Abnormal first ribQ Osteoarthritis

• • • •

• • • •

Trauma (neck hematoma, bone dislocationQ) Fibrous bandsQ (congenital and acquired) NeoplasmsQ Scalenes muscleQ

Clinical Features • Symptoms vary depending on the anatomic structure that is compressedQ. • In > 90% of cases, neurogenic manifestations are reportedQ. • Ulnar nerve (C8-T1) involvement is most commonQ.

• Symptoms of subclavian artery compression: Fatigue, weakness, coldness, ischemic pain, & paresthesia. Thrombosis with distal embolization rarely can occur, producing vasomotor symptoms (Raynaud’s phenomenon) in the hand or ischemic changesQ. • Venous compression: Edema, venous distention, collateral formation, & cyanosis of the affected limbQ Diagnosis • Compression resulting from TOS is dynamic and best evaluated clinically by mechanical provocative maneuversQ • Specific investigations (CT scan, MRI, Angiography, X-ray) are used to exclude other conditions and to establish the associated diagnosisQ. Treatment • Approx. 50-90% of patients can be successfully treated by improvements in postural sitting, standing & sleeping positions, behavior modification at work and muscle stretching & strengthening exercisesQ. Indications for Surgical Intervention • • • • •

Failure of conservative managementQ Progression of sensory or motor symptomsQ Presence of excessively prolonged ulnar or median nerve conduction velocitiesQ Narrowing or occlusion of the subclavian arteryQ Thrombosis of the axillary or subclavian veinQ

• Operation for TOS: Complete removal of the first rib, with division of scalenus anticus & mediusQ. • Large aneurysms or thrombosis of the subclavian artery: Graft reconstructionQ • Subclavian vein thrombosis: Thrombolytic & anticoagulant therapy and simultaneous surgical decompressionQ.

Provocative Clinical Tests to establishing the diagnosis of Thoracic Outlet Syndrome (TOS) Provocative Test Adson’s Test (Scalene Test) Q

Instruction Patient is instructed to: • Take a deep breath and hold it • Extend the neck fully • Turn face towards the side

Inference • Maneuver tightens the anterior & middle scalene muscles, thus decreasing the interscalene space & magnifying any preexisting compression. • Obliteration or diminution of radial pulse suggests the diagnosis

Cardiothoracic Vascular Surgery

• It is associated with: −− Motor weakness & atrophy of the hypothenar & interosseous musclesQ −− Pain & paresthesia along the medial aspect of the arm, hand, 5th finger & medial aspect of 4th fingerQ.

Cardiothoracic Vascular Surgery

716

Surgery Essence Costoclavicular TestQ (Military Position or Halsted Test)

Patient is instructed to: • Draw shoulders downwards and backwards

• Maneuver narrows the costoclavicular space by approximating the clavicle to the first rib thus tending to compress the neurovascular bundle • Obliteration of radial pulse or reproduction of symptoms indicates compression

Hyperabduction TestQ (Wright Test)

Patient is instructed to: • Hyperabduct (Raise) the arm to 180°

• Maneuver causes the neurovascular structures to be pulled around the pectoralis minor tendon, coracoid process and head of humerus • Obliteration or diminution of radial pulse suggests the diagnosis

Roos TestQ (Arm Claudication Test)

Patient is instructed to: • Numbness or pain in the hands with exercise suggests the • Draw shoulders backwards diagnosis • Rise arms to horizontal position with elbows flexed to 90° • Exercise the hands

RAYNAUD’S PHENOMENON Raynaud’s Phenomenon • Raynaud’s phenomenon is characterized by episodic digital ischemia on exposure to cold or emotional stressQ • Manifested by the sequential development of digital blanching, cyanosis & rubor (redness) of fingers or toesQ • BCR: Blanching, Cyanosis & Rubor (redness) Triphasic Color Response Include Three Stages BlanchingQ (stage of local syncope)

• With exposure to cold, digital arterioles goes into spasm • Decreased flow is evidenced by pallor or blanching • Digits may appear white

CyanosisQ (stage of local asphyxia)

• Capillaries and venules dilate • Cyanosis results from deoxygenated blood present in these vessels

Red EngorgementQ (stage of recovery)

• • • •

With rewarming or passing of attack, the digital vasospasm resolves Blood flow into dilated arteries and capillaries increases dramatically Reactive hyperemia imparts bright red color to the digits In addition to rubor and warmth, patient often experiences a throbbing, painful sensation during the hyperemic phase

• Raynaud’s phenomenon is divided into: −− Primary or idiopathic (Raynaud’s diseaseQ) −− Secondary (associated with other diseases) Associations

Section 5

• Raynaud’s phenomenon occurs frequently in patients who also have migraine or variants anginaQ. These associations suggest a common predisposing cause for vasospasmQ. • Occupational groups that use vibrating tools are more predisposed • SclerodactylyQ (thickening and tightening of digital subcutaneous tissue) may develop in few patients.

RAYNAUD’S DISEASE Raynaud’s Disease • Diagnosis of Raynaud’s disease is made when the secondary causes of Raynaud’s phenomenon are ruled outQ. • Majority (70-90%) of patients are young womenQ 2.6 cm in females • MC vessel involved in aneurysm: Circle of WillisQ • MC location of extra-cranial aneurysm: Aorta >Iliac >Popliteal >Femoral (AIPF)Q • MC site of extra-cranial arterial aneurysm is infrarenal aortaQ • MC site of peripheral aneurysm: Popliteal aneurysmQ • Degenerative aneurysms (caused by atherosclerosis) are MC AAA (90%)Q • Width of aneurysm is most important predicting factor of ruptureQ. • Juan ParodiQ introduced endovascular aortic aneurysm repair (EVAR). Classification • • • • •

True (all three layers of vessel are involved), false (do not have all three layers of vessel) Infected (mycotic) aneurysm are false aneurysm Dissecting aneurysm (dissection with aneurysmal dilatation of false lumen) Fusiform (symmetrical enlargement involving whole circumference of artery) Saccular (affect only part of the arterial circumference) have higher risk of ruptureQ

ABDOMINAL AORTIC ANEURYSM Abdominal Aortic Aneurysm • MC site of aortic aneurysm is infrarenal aorta • Risk Factors: Age, male gender, white race, smoking and family historyQ Q

Clinical Presentation • Natural history of AAAs is continuous expansionQ

Section 5

• Rupture is MC & most lethal complicationQ • Most rupture occurs in retroperitoneal space, others in abdominal cavity, IVC, iliac vein or duodenum (4th part-MC) • AAA rupture most commonly in left retroperitoneumQ • Growth rate of AAAs vary with aneurysm size, more rapid growth seen in aneurysms 5 cm or larger • MC symptom: Chronic vague abdominal or back painQ • Triad of aortic rupture: Sudden onset midabdominal or flank pain + shock + pulsatile abdominal mass; present in one third cases onlyQ • Acutely expanding AAA produce severe deep back pain or abdominal pain radiating to back, associated with tenderness to palpation of aneurysm (this presentation signifies impending rupture and urgent evaluation & treatment is required)Q • In aortocaval or aortoiliac fistula: Unilateral or bilateral lower extremity edema, high output CHF & continuous abdominal bruit or palpable thrill is present

Arterial Disorders

719

• CCBs such as diltiazem and nifedipine are the drugs of choiceQ. • AAA may rupture into GIT, MC site is 4th part of duodenumQ, producing primary aortoenteric fistula, shock and massive GI bleeding. • Occasionally, microembolization can occur, resulting in small patchy areas of ischemia, usually on the plantar aspect of the foot, referred to as trash footQ.

Management of Abdominal Aortic Aneurysm Diagnosis • Plain X-ray detects AAA in up to 70% cases by characteristic “eggshell” pattern of calcification • Negative abdominal radiograph doesn’t exclude the diagnosis.

Section 5

• Gross hematuria from intravesicular venous hypertension is one of characteristic sign of aortocaval fistula

• CT is IOC for diagnosis & planning repair in AAAQ • MRI is IOC for diagnosis and with MR angiography planning repair in AAA with renal insufficiencyQ • Advantage of Percutaneous arteriography over CT or MRI is its ability to measure pressure gradient across occlusive lesions if present, and potentially to direct treatment Screening for Abdominal Aortic Aneurysm • AAAs remain asymptomatic for several years, death from rupture occurs in one third of untreated cases • Ultrasound is preferred method of screeningQ Pre-operative Evaluation • Patients with major clinical predictors of cardiac risk are considered for pre-operative angiography

• Prompt operative intervention is indicated in cases of ruptureQ Medical Management • NSAIDs & tetracycline may have potential to reduce aneurysmal growth by inhibiting MMP. Indications of repair of AAA • Diameter 5.5 cm or more in menQ

• Rate of expansion >1 cm/yearQ

• Symptomatic aneurysmQ

• Atypical aneurysmsQ (dissecting, pseudoaneurysm, mycotic, saccular and penetrating ulcer) regardless of size

• For women and patients with greater than average rupture risk, AAA diameter 4.5 to 5.0 cm Treatment • Open repair: Transperitoneal & Retroperitoneal approach

Indications for Retroperitoneal Approach • • • •

History of multiple prior operationsQ Hostile abdomenQ Radiation treatmentQ Suprarenal aneurysm extensionQ

• Horse-shoe kidneyQ • Peritoneal dialysis • Inflammatory aneurysm or ascitesQ

• Advantage: Reduced GI and pulmonary complications, reduced length of ICU & hospital stayQ • Disadvantage: Poor accessibility to distal right arteries and right renal artery Results of Open Repair • Mortality rate of elective open infrarenal AAA repair is AortaQ          • MC organisms: Staphylococcus >SalmonellaQ

ILIAC ARTERY ANEURYSM Iliac Artery Aneurysm • Occur in conjunction with aortic aneurysm in 40% cases • Most isolated iliac aneurysm involve common iliac artery (70%) & internal iliac artery (20%) • Multiple iliac aneurysms occur in most patients and are bilateral in 33% cases Etiology • Occur in association with atherosclerosis • Can also occur in pregnancy in absence of atherosclerosis as well as in Marfan and Ehlers-Danlos syndromes, Kawasaki disease, Takayasu’s arteritis, cystic medial necrosis & arterial dissection Clinical Features • More common in males, right & left sides are equally involvedQ • Symptoms are caused by compression of adjacent pelvic structuresQ (bladder, colon, ureter, rectum, lumbosacral nerves and pelvic vein) • Most common iliac aneurysms can be palpated on abdominal exam whereas internal iliac artery aneurysm are more readily palpated on rectal examinationQ Treatment

Section 5

• Operative mortality rate in patients with ruptured iliac aneurysm is 40%. • Iliac aneurysms >3.5Q cm are repaired if possible.

Minimum Size for Surgery (AIPF: All India Police Force) Abdominal Aortic Aneurysm (5.5 cm) = Iliac aneurysm (3.5 cm) + Popliteal/Femoral aneurysm (2.0 cm) Indications of Surgery in Aneurysms on the basis of Size (diameter) • Descending thoracic aorta

• ≥6.5 cmQ

• Ascending thoracic aorta

• ≥5.5 cmQ

• Abdominal aorta

• ≥5.5 cmQ

• Iliac artery

• ≥3.5 cmQ

• Femoral & Popliteal artery

• ≥2.0 cmQ

Arterial Disorders

721

POPLITEAL ARTERY ANEURYSM • MC peripheral aneurysm and account for 70% casesQ • Most patients are male with bilateral disease in 53% cases • Amputation rate in acute thromboembolism is up to 30% Indications for treatment • Symptomatic patientsQ • Aneurysm >2 cmQ

• Thrombus in aneurysmQ • Angiographic evidence of distal embolizationQ

Diagnosis

Section 5

Popliteal Artery Aneurysm

• Diagnosed by physical examination and duplex scanQ

FEMORAL ARTERY ANEURYSM Femoral Artery Aneurysm • Bilateral in >50% cases and 92% have a concomitant aortoiliac aneurysmQ • True FAA are almost always degenerative atherosclerotic aneurysmsQ Etiology

• Diagnosis of atherosclerotic FAA is usually made by physical examination & confirmed by USGQ. Treatment • In patients who present with femoral pseudoaneurysms after catheterization, USG-guided compression with or without thrombin injectionQ, can also be used to treat the disease • All true FAA >2 cm should be repaired because of risk of thrombo-embolic complications or increased risk of ruptureQ • High risk of rupture in cases of large, false or aneurysm involving profunda femorisQ • Treatment involves resection & replacement with prosthetic interposition graft with attempt of revascularization of profunda femoris. Results of Open Repair • Asymptomatic patients do well but amputation rate of 10% in symptomatic patients. • Highest amputation rates in drug addicts requiring treatment of infected FAAQ. • Autologus repair with a vein graft and immediate coverage with sartorius muscle flap decreases rate of reinfection and recurrent bleedingQ.

SUBCLAVIAN ARTERY STENOSIS Subclavian Artery Stenosis • MC cause of subclavian artery stenosis: Atherosclerotic disease • LeftQ subclavian artery stenosis is significantly more common than right • MC site of stenosis: First partQ of subclavian artery • Stenosis typically occurs just distal to origin of subclavian artery & lies proximal to origin of vertebral arteryQ. • Stenosis of first part of subclavian artery may give rise to subclavian steal syndromeQ • Subclavian steal syndrome is characterized by reversed flow in vertebral artery to compensate for a proximal stenosis in the ipsilateral subclavian artery there by stealing blood from the ‘brain’ to feed the ‘arm’Q.

Cardiothoracic Vascular Surgery

• True FAA are almost always degenerative atherosclerotic aneurysms • False FAA may develop as a result of disruption of graft-artery anastomosis following surgical revascularization with aortofemoral or femoropopliteal bypass • MC organisms causing mycotic aneurysms are Staphylococcus aureus, E. coli, SalmonellaQ. Diagnosis

Cardiothoracic Vascular Surgery

722

Surgery Essence

SUBCLAVIAN STEAL SYNDROME Subclavian Steal Syndrome • Occlusion of either the innominate (brachiocephalic) or the subclavian artery before the origin of vertebral artery reverses the direction of blood flow in the ipsilateral vertebral arteryQ. • This reversal of flow often is asymptomatic but may cause ischemia in the posterior circulationQ. • Neurological features are weakness, vertigo, visual complaints, & syncopeQ. • Classically symptoms occur when arm exercise increase the steal of blood flow from the brainstemQ. • The exercise reduces peripheral resistance in the affected arm, lowering blood pressure distal to occlusion. This in turn results in increased retrograde flow from the vertebral arteryQ. • If the contralateral vertebral artery cannot keep up with the demand, the arm may steal blood from the basilar artery, lowering the pressure in the posterior cerebral circulationQ. • The result may be transient vertebrobasilar ischemiaQ.

VASCULAR GRAFT Vascular Graft

Section 5

Bioprosthetic • • • •

AutograftQ Homograft (allograft) Heterograft (Xenograft) Tissue engineered

Synthetic • Textile: −− DacronQ • Non-textile: −− ePTFEQ   −− PolyurethaneQ

Best natural vascular graft: Reversed saphenous veinQ Best synthetic vascular graft: DacronQ Best vascular graft for suprainguinal bypass: DacronQ Best vascular graft for infrainguinal bypass: Saphenous veinQ Best vascular graft for aorta: DacronQ Most preferred graft for CABG: LIMA (left internal mammary artery) > Saphenous veinQ MC used graft for CABG: Saphenous veinQ

Arterial Disorders

723

TRAUMATIC AORTIC RUPTURE

• Traumatic aortic rupture is a cause of sudden death after an automobile collision or fall from a great heightQ. • Vessel is relatively fixed distal to the ligamentum arteriosum, just distal to the origin of left subclavian arteryQ. • Causes: Trauma to the chest, Rapid deceleration injuryQ • MC site: Adjacent to ligamentum arteriosumQ Clinical Presentation • Traumatic aortic rupture is lethal in 75-90% of patientsQ and only 15-20% arrive at the hospital aliveQ. • Approx. 90% of those who arrive alive at the hospital have an injury in the region of the aortic isthmus >Injuries to the ascending aortaQ. • Usually associated with other injuries like solid organ. Diagnosis • X-ray chest: Widened mediastinumQ • Aortography: Gold standard for diagnosisQ • Contrast enhanced CT, Trans-esophageal Echocardiography (TEE) are useful in making diagnosis. Treatment • Control of systolic BP to less than 100 mm Hg • Endovascular intra-aortic stentingQ • Direct repair or excision & grafting using a Dacron graftQ.

Section 5

Traumatic Aortic Rupture

TAKAYASU’S ARTERITIS (AORTOARTERITIS OR PULSELESS DISEASE) Takayasu’s Arteritis (Aortoarteritis or Pulseless Disease)

• Pathologic changes produce stenosis, dilation, aneurysm formation & occlusionQ. Clinical Features • Occurs predominantly in adolescent girls and young womenQ, age of 10-40 yearsQ • More common in AsiaQ • Subclavian artery is MC involved vessel leading to loss or weakening of pulses (Pulseless Disease)Q. • Characteristic clinical features: Hypertension reflecting renal artery stenosis, retinopathy, cerebrovascular symptoms, angina and congestive heart failure, abdominal pain or GI bleeding or extremity claudication. Diagnosis • Laboratory data: Raised ESR, CRP & WBC countQ • IOC for diagnosis: CT angiographyQ • Gold standard for diagnosis: AngiographyQ Treatment • Steroid therapy initially, with cytotoxic agents in patients who do not achieve remissionQ. • Surgical treatment is performed only in advanced stages, and bypass needs to be delayed during active phases of inflammation.

GIANT CELL ARTERITIS (TEMPORAL ARTERITIS) Giant cell Arteritis (Temporal Arteritis) • Predominantly afflicts patients older than 50 years of ageQ, with a slight (2 : 1) female preponderance. • Incidence increases for each decade over age 50 years. • Superficial temporalQ, vertebral, & major aortic arch branches may be involved. Clinical Features • Ischemic symptoms are common, including claudication of facial or extremity muscles & retinal ischemiaQ. • Headache is a common symptom. • Blindness, usually irreversible, is a dreaded complicationQ. Treatment • When the clinical diagnosis is suspected, treatment must be prompt and consists of high-dose corticosteroid therapyQ. • Surgery is rarely indicated except in cases of major aortic branch involvement with ischemic symptoms.

Cardiothoracic Vascular Surgery

• Rare but well-recognized chronic inflammatory arteritis affecting large vessels, predominantly the aorta & its main branchesQ. • Chronic vessel inflammation leads to wall thickening, fibrosis, stenosis, & thrombus formationQ. • Symptoms are related to end-organ ischemia.

724

Surgery Essence

Section 5

Cardiothoracic Vascular Surgery

CELIAC PLEXUS BLOCK Celiac Plexus Block • • • • •

Celiac plexus also known as the solar plexus is located behind stomach & omental bursa, and in front of the crura of the diaphragm Location is at the level of the first lumbar vertebra. Formed (in part) by the greater & lesser splanchnic nerves of both sides, and also parts of the right vagus nerve. Supplies upper GI organs, lower esophagus, liver & pancreas Done usually bilaterally using alchol or phenol

Indications: • Intractable pain from cancers (pancreatic cancer)      • Intractable pain related to chronic pancreatitis Side-Effects: • Most common side effects include hypotension & diarrhea. −− Hypotension: Because of the sympathetic blockade of splanchnic vasculature −− Diarrhea: Unopposed parasympathetic activity following celiac plexus block

Multiple Choice Questions

ARTERIAL OCCLUSION

1. Acute vascular ischemia manifests as: (PGI Dec 2008) a. Pulselessness b. Paralysis c. Flushing d. Anesthesia e. Coolness



12. In a subclavian artery block at outer border of 1st rib, all of the following arteries help in maintaining the circulation to upper limb except: (AIIMS May 2011) a. Subscapular artery b. Superior thoracic artery c. Thyrocervical trunk d. Suprascapular artery 13. Both arterial and venous thrombosis occur in: a. Antiphospholipid antibodies  (PGI Nov 2011) b. Antithrombin III deficiency c. Hyperhomocysteinemia d. Protein C deficiency e. Mutation in factor V gene 14. Maximum tourniquet time for the upper limb is: a. 1/2 hour b. 1 hour c. 1.5 hours d. 2 hours e. 2.5 hours



2. Clinical feature of acute arterial embolism is: (PGI Nov 2017) a. Pulselessness b. Pain c. Erythema of distal part d. Numbness e. Sensory loss



3. The most common cause of peripheral limb ischemia in India is: (AIIMS Nov 2005) a. Trauma b. Atherosclerosis c. Buerger’s disease d. Takayasu disease





4. Which among the following is not a feature of peripheral arterial occlusion? (Recent Question 2016) a. Shock b. Pallor c. Pain d. Pulselessness





5. Not a feature of acute arterial occlusion: (DNB 2010, AIIMS Nov 98) a. Cyanosis b. Pallor c. Paralysis d. Paraesthesia

15. Intermittent claudication at the level of the hip indicates:  a. Popliteal artery occlusion (Recent Question 2016) b. Bilateral iliac artery occlusion c. Common femoral occlusion d. Superficial femoral artery occlusion



16. Management of a case of iliac artery embolism requires: a. Embolectomy b. Injection c. Hypotensive therapy d. Sympathectomy



6. What will be the diagnosis of Ramu, who is 45-years old male with history of chronic smoking and pain in lower limb due to blockage of femoral artery? (AIIMS Feb 97) a. Thromboangitis obliterans b. Atherosclerosis c. Embolism d. Arteritis



17. Intermittent claudication is caused by: a. Venous occlusion b. Arterial insufficiency c. Neural compression d. Muscular dystrophy 18. Treatment of acute femoral embolus is: (AIIMS 91) a. Warfarin b. Heparin c. Immediate embolectomy d. Embolectomy after 5 days bed rest 19. A useful though temporary improvement in a patient’s ischemic foot can be attained by giving intravenously: a. 10% Mannitol b. 10% Dextrose c. Dextran-40 d. Dextran-100 20. All are true about embolic arterial occlusion except:  (JIPMER 95) a. No previous history b. Muscles are unaffected c. Pulse is absent d. Anesthesia is present 21. Tourniquet time of upper limb: (MHSSMCET 2010) a. 1 hour b. 2 hours c. 3 hours d. 4 hours 22. Which of the following is the most common symptom of aortoiliac occlusive disease? (AIIMS November 2016) a. Calf claudication b. Gluteal claudication c. Impotence d. Symptomless











7. Fogarty’s catheter is used for: (Recent Question 2015; UPSC 2007) a. Drainage of urinary bladder b. Parenteral hyperalimentation c. Removal of embolus form blood vessels d. Ureteric catheterization 8. Fogarty’s catheter is used for? (AIIMS Nov 2010) a. Urethral catheterization b. Removal of blood clots from the arteries c. Bladder drainage d. TPN 9. Which one of the following is not a symptom of atherosclerotic occlusive disease at the bifurcation of aorta (Leriche syndrome)? (UPSC 2008) a. Claudication of buttock and thigh b. Claudication of the calf c. Sexual impotence d. Gangrene localized to the feet 10. Intermittent claudication is defined as:  a. Pain in muscle at rest only b. Pain in muscle on first step c. Pain in muscle on exercise only d. Pain in muscle on last step











(All India 2009)

11. Pseudoclaudication is caused by:  (All India 2009) a. Femoral artery stenosis b. Popliteal artery stenosis c. Lumbar canal stenosis d. Radial artery stenosis



23. Which of the following is true about Leriche syndrome? a. Caused by aortoiliac occlusion  (Recent Question 2017) b. Erection or impotence problems c. Gluteal claudication is seen d. All of the above 24. Fontaine and Rutherford classification of peripheral arterial disease is based on: (Recent Question 2017) a. Clinical b. Arterial stenosis on imaging c. Both clinical and arterial stenosis on imaging d. All of the above

726

Surgery Essence

Cardiothoracic Vascular Surgery

BUERGER’S DISEASE

25. Not included in treatment of Buerger’s disease: (PGI May 2011) a. Lumbar sympathectomy b. Endovascular stent c. Rheostatic agent d. Extra-anatomical bypass



26. All are true about intermittent claudication except: a. Most common in calf muscle  (PGI May 2010) b. Pain in positional c. Atherosclerosis is important predisposing factor d. Relieved by rest



27. A 45-years old male having a long history of cigarette smoking presented with gangrene of left foot. An amputation of the left foot was done. Representative sections from the specimen revealed presence of arterial thrombus with neutrophilic infiltrate in the arterial wall. The inflammation also extended into the neighboring veins and nerves. The most probable diagnosis is: (AIIMS May 2006) a. Takayasu arteritis b. Giant cell arteritis c. Hypersensitivity angitis d. Thromboangitis obliterans



28. True statement of Buerger’s disease is/are: (PGI June 2004) a. Small and medium sized vessels involved b. Commonly involves upper limb than lower limbs c. Common in male d. Common in female



29. Drug used for Buerger’s disease: (MAHE 2005) a. Xanthinol nicotinate b. Propranolol c. GTN d. All of the above



30. In a patient of vascular disease, angiography was performed and the image is given below. What is the most probable diagnosis?

34. True about ischemic rest pain: a. More in night b. MC in calf muscle c. Increase upon elevation of limbs d. Relieved by dependent position e. Often associated with trophic changes



35. What is the diagnosis based on the given image?  (Recent Question 2016) a. Dry gangrene b. Raynaud’s disease c. Wet gangrene d. Gas gangrene



36. Most common cause of gangrene of foot of 30-year-old farmer who is a chronic smoker: (AIIMS Nov 99) a. Raynaud’s disease b. Myocardial infarction c. Atherosclerosis d. Thromboangitis obliterans 37. Ramu, a 40-year-old male, a chronic smoker presents with claudication and a medial leg ulcer. For the past following procedures would not relieve his rest pain: (AIIMS June 2001) a. Lumbar sympathectomy b. Omentoplasty c. Conservative amputation d. Femoropopliteal bypass 38. Which one is not true regarding Buerger’s disease? a. Men are usually involved  (AIIMS June 97) b. Occurs below 50 years of age c. Smoking is predisposing factor d. Veins and nerves are never involved

a.

Popliteal artery aneurysm b. Leriche syndrome c. Buerger disease d. Aortic stenosis



Section 5



31. Buerger’s disease usually affects all of the following except:  (Recent Question 2014; MCI Sept 2009, 2010) a. Small sized arteries b. Medium sized arteries c. Large arteries d. Deep veins



32. Superficial thrombophlebitis is seen in: (MCI March 2005) a. AV fistula b. Raynaud’s disease c. Buerger’s disease d. Aneurysm



33. Which of the following is true about Buerger’s disease? a. Atherosclerotic (AIIMS Nov 2012) b. Neural involvement present c. Ulnar artery and peroneal arteries involved d. Only arteriole is involved

(PGI May 2010)





39. All of the following are the clinical feature of thromboangitis obliterans except: (All India 2002) a. Raynaud’s phenomenon b. Claudication of extremities c. Absence of popliteal pulse d. Migratory superficial thrombophlebitis



40. Buerger’s disease affects all except: a. Lymphatics c. Nerves



(Recent Question 2015; DNB 2009) b. Small vessels d. Veins

41. Most common cause of death in patients with Buerger’s disease is:  (AIIMS 87) a. Gangrene b. Pulmonary embolism c. Myocardial infarction d. Carcinoma lung

Arterial Disorders



LUMBAR SYMPATHECTOMY





48. Which of the following is spared in lumbar sympathectomy? a. L1 b. L2 (Recent Question 2015) c. L3 d. L4 (JIPMER Nov 2017)



49. In all of the following, sympathectomy is effective except one: (AIIMS Sept 96) a. Intermittent claudication b. Hyperhydrosis c. Raynaud’s disease d. Causalgia 50. Lumbar sympathectomy is not indicated in: (AIIMS June 97) a. Healing of ulcer over great toe b. Claudication c. Rest pain d. Buerger’s disease









51. Which of the following best responds to sympathectomy?  (Recent Question 2016) a. Buerger’s disease b. Hyperhydrosis c. Raynaud’s disease d. Acrocyanosis 52. In extraperitoneal approach, to left sympathectomy the following may be injured: (Recent Question 2016) a. Ureter b. Gonadal vessels c. A+B d. IVC 53. In a lumbar sympathectomy the sympathetic chain in its usual position is likely to be confused with the: a. Psoas minor b. Genitofemoral nerve c. Ilioinguinal nerve d. Lymphatics

54. Removal of L1 ganglion in sympathectomy results in: (DNB 2006, JIPMER 91) a. Impotence b. Retention of urine c. Sterility d. Causalgia

CRITICAL LIMB ISCHEMIA

55. Which of the following statement is not true? a. Ankle brachial index 1 is normal  (AIIMS Nov 2011) d. Smoking is more specific for peripheral vascular disease than coronary artery disease



56. A patient with critical lower limb ischemia presents with: a. Intermittent claudication b. Intermittent claudication and gangrene c. Rest pain and ischemic ulcers (All India 2009) d. Intermittent claudication and ischemic ulcers



57. An adult patient with leg pain and gangrene of toe. His ankle to brachial arterial pressure ratio would be less than: a. 1 b. 0.3 (DNB 2011) c. 0.5 d. 0.8



58. Definition of critical limb ischemia includes: a. Rest/Nisht pain (COMEDK 2014) b. Ankle blood pressure > 50 mm Hg c. Intermittent clandication d. Well preserved tissues



59. Normal value of ankle brachial index is: (Recent Question 2014, 2013) a. 0.8 b. 1 c. 1.2 d. 1.4



60. ABPI in imminent necrosis: (Recent Question 2015) a. < 0.3 b. < 0.6 c. < 0.9 d. > 1.2



61. False elevation of ABPI is seen in: a. DVT b. Acute limb ischemia c. Chronic venous insufficiency d. Calcified vessel walls

(Recent Question 2018)



62. ABPI increases artificially in: a. Arteriosclerosis calcified arteries b. Ischemic ulcers c. Intermittent claudication d. DVT

(Recent Question 2018)

ARTERIAL ULCER

63. One of the following is not indicated for arterial leg ulcer: a. Debridement b. Elevation of limb (PGI 96) c. Head end of bed is raised d. Low dose aspirin



64. Foot ulcers secondary to arterial insufficiency are successfully treated by all of the following techniques except: a. Debridement of devitalized tissue (COMEDK 2004) b. Elevation of the affected extremity c. Antibiotic administration d. Bed rest

Cardiothoracic Vascular Surgery



45. Indications for sympathectomy are all except: (MHPGMCET 2003) a. Intermittent claudication b. Ischemic pains c. Rest pains d. Buerger’s disease 46. Lumbar sympathectomy is of value in the management of: a. Intermittent claudication (All India 2009, 2005) b. Distal ischemia affecting the skin of the toes c. Arteriovenous fistula d. Back pain 47. Sympathectomy is indicated in all following conditions except: (All India 2009, 2003, Punjab 2007) a. Ischemic ulcers b. Intermittent claudication c. Anhidrosis d. Acrocyanosis



Section 5



42. Commonest site of thromboangitis obliterans is:  (All India 90) a. Femoral artery b. Popliteal artery c. Iliac artery d. Pelvic vessels 43. Following are used in treatment of Buerger’s disease except: a. Trental b. Anticoagulation c. Sympathectomy d. Antiplatelets (All India 93) 44. A 40-year-old male, who is a chronic smoker presented with long history of intermittent claudication and blackish discoloration of toes. What is the preferred treatment? a. Abstinence from smoking only  (Recent Question 2017) b. Vasodilators c. Lumbar sympathectomy d. Amputation

727

728

Surgery Essence

Section 5

Cardiothoracic Vascular Surgery



65. True regarding leg ulcers & their location is/are: (PGI May 2018) a. Arterial insufficiency – tip of the toes b. Arterial insufficiency – medial side of leg c. Venous insufficiency – above medial malleolus d. Diabetic neuropathic ulcer – planter aspect of metatarsal head e. Pressure ulcer – heel

AMPUTATION





66. Re-implantation time for lower limb is: (Kerala 97) a. 6 hours b. 4 hours c. 8 hours d. 10 hours 67. Stump pain is relieved by: (Kerala 97) a. Continuous tapping over the stump b. Warming up the stump c. Using steroids d. Using analgesics 68. For reimplantation surgery, the detached digit or limb is best preserved in cold: (Recent Question 2014; UPSC 2000) a. Glycerol b. Distilled water c. Hypertonic saline d. Isotonic saline 69. Phantom limb is based upon:  (DNB 2009) a. Law of projection b. Webers law c. Munro-Kellie doctrine d. Renshaw cell inhibition 70. Amputated finger is transported in: (Recent Question 2017) a. Plastic bag with wet ice b. Plastic bag with dry ice c. 
Plastic bag with cold saline d. Plastic bag with cold water



76. The most common cause of acquired arteriovenous fistula is:  (All India 2006) a. Bacterial infection b. Fungal infection c. Blunt trauma d. Penetrating trauma



77. Commonest cause of A-V fistulae is: (Recent Question 2013, DNB 2000) a. Congenital b. Traumatic c. Surgical creation d. Tumor erosion



78. Complications arising out of A-V fistula done for renal failure include the following except: (JIPMER 2003) a. Infection b. Thrombosis c. High output cardiac failure d. Necrosis of the distal part 79. Arteriovenous fistula can safely be ligated if the following is positive: (COMEDK 2004) a. Allen’s test b. Henle-Coenen sign c. Trendelenberg test d. Schwartz test



THORACIC OUTLET SYNDROME



ARTERIOVENOUS FISTULA

71. Nicoladoni sign is also known as: (AIIMS Nov 2008) a. Murray sign b. Frei sign c. Darrier sign d. Branham sign



72. All are true about arteriovenous fistula except: (PGI November 2017) a. Trauma is most common cause of acquired fistula b. Congenital fistula is easier to repair by surgery than acquired fistula c. Artificial fistula is surgically created fistulas for hemodialysis access d. High blood pressure from artery results in arterialization of vein e. Large A-V fistula may cause thrombocytopenia



73. All of the following are correct regarding AV fistula except: a. Arterialisation of the veins b. Proximal compression causes increases in heart rate c. Localized gigantism (MHSSMCET 2005, All India 2001) d. Causes LV enlargement and cardiac failure 74. A patient presented with local gigantism of the leg and increased pulsations of the lower limb veins. Most probable diagnosis is: (AIIMS Nov 2001) a. Tumor b. AV fistula c. Varicose veins d. Incompetence of the saphenofemoral junction 75. Nicoladoni Branham sign is:  (PGI Dec 98) a. Compression cause bradycardia b. Compression cause tachycardia c. Hypotension d. Systolic filling







80. Thoracic outlet syndrome is primarily diagnosed by: a. Clinical evaluation b. CT scan (All India 2009) c. MRI d. Angiography 81. Which of the following is not a complication of surgery for thoracic outlet syndrome?  (AIIMS May 2007) a. Pneumothorax b. Brachial plexus injury c. Lymphocutaneous fistula d. Long thoracic nerve injury 82. Adson’s test is positive in: (MHSSMCET 2005) a. Cervical spondylosis b. Fracture ribs c. Cervical rib d. All of the above 83. This is the X-ray of a patient of thoracic outlet syndrome. What is the best management for this patient?

a. Physiotherapy b. Conservative management c. Surgical intervention d. None of the above

84. Which is not true about thoracic outlet syndrome? (Recent Question 2016, AIIMS Nov 98) a. Radial nerve is commonly affected  b. Neurological features are most common c. Resection of 1st rib relieves symptom d. Positive Adson’s test

Arterial Disorders







86. Commonest symptom associated with thoracic outlet syndrome is: (Recent Question 2016) a. Intermittent claudication b. Pain on radial distribution c. Pain in ulnar distribution d. Gangrene 87. All are seen in thoracic outlet syndrome except: a. Mass in the neck b. Wasting of forearm muscles c. Adson’s test positive d. Pallor



b. 25–45 years d. > 65 years







97. True regarding management of diabetic foot: ( PGI May 2010) a. Strict diabetic control b. Venous system is commonly involved c. Topical antibiotics are used d. Early amputation should done e. Diabetic ulcers are trophic ulcers



98. Site of diabetic foot ulcer: a. Medial malleolus c. Heel e. Head of toes



99. Diabetic gangrene is due to: a. Ischemia b. Increased blood glucose c. Altered defense by host and neuropathy d. All of the above

91. All of the following are true regarding Raynaud’s phenomenon except:  (AIIMS Nov 2012) a. It involves acral parts of fingers b. Migratory thrombophlebitis is seen only in Raynaud’s phenomenon c. Drugs acting by inhibiting the beta receptors in blood vessels also play a role d. Emotional stress may also precipitate Raynaud’s phenomenon 92. True statement about Raynaud’s phenomena:(PGI Dec 2006) a. Lower limb more commonly involved than upper limb b. More common in female c. Superficial thrombophlebitis d. Associated with migraine 93. Raynaud’s syndrome occurs in all of the following except: a. SLE (DNB 2009, MCI Sept 2007) b. Rheumatoid arthritis c. Osteoarthritis d. Cryoglobulinemia 94. Sequence of colour changes observed in Raynaud’s disease:  (MCI Sept 2009) a. Red, blue, white b. White, blue, red c. Blue, red, white d. White, red, blue



95. If a patient with Raynaud’s disease immersed his hand in cold water, the hand will: (All India 2003) a. Become red b. Remain unchanged c. Turn white d. Become blue



96. All are true about Raynaud’s phenomena except: (Kerala 95) a. Exposure to cold aggravates b. Spasm of vessels c. More common in females d. Atherosclerosis of vessels

(PGI June 2005) b. Lateral malleolus d. Head of metatarsal (Kerala 94)

100. Diabetic gangrene is due to all except: (TN 86) a. Vasospasm b. Atherosclerosis c. Peripheral neuritis d. Increased sugar in blood 101. Etiopathogenesis of diabetic foot include the following except: (UPSC 2007) a. Myelopathy b. Osteoarthropathy c. Microangiopathy d. Infection

AORTIC DISSECTION 102. The most common site of acute aortic dissection is: a. Right lateral wall of ascending aorta b. Arch of aorta (DNB 2013, COMEDK 2010) c. Suprarenal abdominal aorta d. Infrarenal abdominal aorta 103. A 50-year-old male patient, an alcoholic and smoker presents with a 3 hours of severe retrosternal chest pain and increasing shortness of breath. He started having this pain while eating, which was constant and radiated to the back and intersapular region. He was a known hypertensive. On examination, he was cold and clammy with a heart rate of 130/min, and a BP of 80/40 mmHg. JVP was normal. All peripheral pulses were present and equal. Breath sounds were decreased at the left lung base and chest X-ray showed left pleural effusion. What is the most likely diagnosis? a. Acute aortic dissection b. Acute myocardial infarction c. Rupture of the esophagus d. Acute pulmonary embolism 104. Dissection of which artery is seen in pregnancy? a. Carotid artery b. Aorta (PGI June 2000) c. Coronary artery d. Femoral artery

AORTIC ANEURYSM 105. Most common cause of abdominal aortic aneurysm is: a. Atherosclerosis b. Trauma (All India 2010) c. Syphilis d. Vasculitis 106. False statement about abdominal artery aneurysm (AAA): a. Surgery indicated when size AAA >6 cm (PGI Nov 2011) b. 90% of AAA is present below renal artery c. Blue toe syndrome may be associated d. Mortality rate after surgery is >25% e. Commonly causes colon ischemia

Cardiothoracic Vascular Surgery

90. All of the following are predisposing factors for thoracic outlet syndrome except: (Recent Question 2017) a. Scalene muscle b. Long transverse process of C7 c. 
Spondylosis d. Cervical rib

RAYNAUD’S DISEASE



(PGI 97)

88. Adson’s test is used for determining vascular insufficiency. It is useful in: (Recent Question 2013) a. Peripheral vascular disease b. Varicose veins c. Cervical rib d. AV fistula 89. Most common age group affected in thoracic outlet obstruction syndrome is: (DNB 2014)

a. 10–25 years c. 45–65 years

DIABETIC FOOT

Section 5



85. Which is not true regarding thoracic inlet syndrome? a. Most commonly radial nerve  (AIIMS Sept 96) b. Resection of 1st rib is effective treatment c. Physiotherapy and position exercises relieves symptom d. Neurological signs and symptoms are common

729

Cardiothoracic Vascular Surgery

730

Surgery Essence 107. The most common site of rupture of abdominal aortic aneurysm is: (All India 2009) a. Laterally into the left retroperitoneum b. Laterally into the right retroperitoneum c. Posteriorly into the posterior retroperitoneum d. Anteriorly into the peritoneum (Intraperitoneal) 108. Most common complication of descending aortic aneurysm surgery:  (MHSSMCET 2006) a. Renal failure b. Distal emboli c. Pulmonary infections d. Myocardial infarction 109. The size at which elective surgery is indicated in abdominal aortic aneurysm: (MHSSMCET 2008) a. 5 cm b. 5.5 cm c. 6 cm d. 6.5 cm 110. Abdominal aortic aneurysm is operated when the size is more than: (Recent Question 2018) a. 35 mm b. 45 mm c. 55 mm d. 65 mm 111. Abdominal aneurysm is characterized by all except:  (PGI June 2000) a. Elective surgery complication should be 0.5

• CLI results from severe arterial occlusive diseaseQ in whom resting blood flow cannot provide for basal nutritional needs of tissues • Ischemic symptoms are present even during rest (rest pain)Q • Pain is worse when legs are horizontal and may improve when legs are kept in a dependent positionQ. • Ischemic ulcers, lesions on the foot and gangreneQ may be seen • ABI d (Ref: Analgesics www.stoppain.org/pain_medicine/content/chronicpain/phantom.asp)

Stump Pain • Stump pain is located at the end of an amputated limb’s stump.

Treatment • No one treatment has been shown to be effective for stump painQ. • Because it is a pain due to an injured peripheral nerve, drugs used for nerve pain may be helpful. −− Other approaches also are tried in selected cases, including: −− Nerve blocksQ −− Transcutaneous electrical nerve stimulationQ −− Surgical revision of the stump or removal of the neuromaQ −− Cognitive therapies

68. Ans. d. Isotonic saline

69. Ans. a. Law of projection (Ref: Ganong 25/e p171)

• A phantom limb is the sensation that an amputated or missing limb (even an organ appendix) is still attached to the body and is moving appropriately with other body parts based upon “Law of Projection”. • It states that no matter where a sensory pathwayis stimulated alongits coure, the sensation produced is referred back to site of receptor.

70. Ans. a. Plastic bag with wet ice (Ref: Sabiston 20/e p1996; Schwartz 10/e p1800)

“If it is anticipated that the amputated part will be considered for replantation, it is critical to transport the patient and the part in an appropriate manner. The amputated part is placed in a clean, dry, plastic bag, which is sealed and placed on top of ice in a Styrofoam container. This keeps the part sufficiently cool at 4° C to 10° C without freezing. The amputated part is wrapped in a lightly moistened saline gauze to prevent tissue drying.”- Sabiston 20/e p1996 “In preparation for replantation, the amputated part and proximal stump should be appropriately treated. The amputated part should be wrapped in moistened gauze and placed in a sealed plastic bag. This bag should then be placed in an ice water bath. Do not use dry ice, and do not allow the part to contact ice directly; frostbite can occur in the amputated part, which will decrease its chance of survival after replantation. Bleeding should be controlled in the proximal stump by as minimal a means necessary, and the stump should be dressed with a non-adherent gauze and bulky dressing.”-Schwartz 10/e p1800

ARTERIOVENOUS FISTULA

71. Ans. d. Branham sign (Ref: Sabiston 19/e p1785-1786; Bailey 26/e p899)



72. b. Congenital fistula is easier to repair by surgery than acquired fistula

Cardiothoracic Vascular Surgery

• It typically is described as a “sharp,” “burning,” “electric-like,” or “skin-sensitive” pain. • Due to a damaged nerve in the stump region.

Cardiothoracic Vascular Surgery

738

Surgery Essence

73. Ans. b. Proximal compression causes increases in heart rate

74. Ans. b. AV fistula

75. Ans. a. Compression cause bradycardia



76. Ans. d. Penetrating trauma

77. Ans. a. Congenital



79. Ans. a. Allen’s test (Ref: Sabiston 20/e p1979; Bailey 27/e p443, 444, 890)

78. Ans. d. Necrosis of the distal part

Allen’s Test • Tests the adequacy of the blood supply to the hand from the radial and ulnar arteries and the arcade between themQ • Allen’s test is used to know the integrity of palmar arch (patency of radial and ulnar arteries)Q. If these are patent, AVF can be safely ligated. Method of Allen’s Test • Elevate the hand and apply digital pressure on the radial and ulnar arteries to occlude them. • Ask the patient to make a fist several times. • The tips of the finger should go pale. Release each artery in turn and observe the return of colour

THORACIC OUTLET SYNDROME

80. Ans. a. Clinical evaluation (Ref: Sabiston 20/e p1603-1604; Schwartz 10/e p829, 928; Bailey 27/e p991) • Thoracic outlet syndrome is diagnosed primarily by clinical evaluation and the diagnosis is based on reproducibility of symptoms (resulting from compression of neurovascular bundle at the thoracic outlet) during mechanical provocative maneuvers (Adson’s test or costoclavicular test or Hyperabduction test or Roos Arm Claudication test) • Specific investigations (CT scan, MRI, Angiography, X-ray) are used to exclude other conditions and to establish the associated diagnosis.



81. Ans. c. Lymphocutaneous fistula (Ref: Sabiston 20/e p1604; Bailey 26/e p872)

Brachial plexus injuries, vascular injuries, pleural effusion, winged scapula, and infection are complications that may arise secondary to first rib removal. • • • •

Complications of Surgical Treatment of TOS Brachial plexus injuriesQ • Pleural effusionQ Winged scapula due to long thoracic nerve injuryQ • PnemothoraxQ Horner’s syndromeQ • Air embolism Vascular injuries (subclavian vesselsQ) • Infection



82. Ans. c. Cervical rib



83. Ans. c. Surgical intervention (Ref: Sabiston 20/e p1603-1604; Schwartz 10/e p829; Bailey 27/e p991-992)



Cervical rib is responsible for the thoracic outlet syndrome and the preferred treatment for cervical rib responsible for thoracic outlet syndrome is surgical resection.



84. Ans. a. Radial nerve is commonly affected 85.  Ans. a. Most commonly radial nerve



86. Ans. c. Pain in ulnar distribution 87. Ans. b. Wasting of forearm muscles (Ref: Sabiston 20/e p1603, 19/e p1594-1595; Schwartz 9/e p704, 790; Bailey 25/e p895)



88. Ans. c. Cervical rib



90. Ans. c. Spondylosis (Ref: Sabiston 20/e p1603; Schwartz 10/e p829; Bailey 27/e p991)

89. Ans. b. 25–45 years

Section 5

RAYNAUD’S DISEASE

91. Ans. b. Migratory thrombophlebitis is seen only in Raynaud’s phenomenon (Ref: Harrison 20/e p1928, 19/e p1647; Sabiston 20/e p16031604; Schwartz 10/e p1823; Bailey 27/e p967)



92. Ans. b. More common in female, d. Associated with migraine (Ref: Harrison 20/e p1928, 19/e p1648; Sabiston 20/e p1779-1780; Schwartz 10/e p904; Bailey 27/e p967)



93. Ans. c. Osteoarthritis



96. Ans. d. Atherosclerosis of vessels

94. Ans. b. White, blue, red

95. Ans. c. Turn white

DIABETIC FOOT

97. Ans. a. Strict diabetic control, c. Topical antibiotics are used, e. Diabetic ulcers are trophic ulcers (Ref: Harrison 20/e p2882, 19/e p1769; Sabiston 20/e p1767; Schwartz 10/e p1877, 1879; Bailey 27/e p532, 533, 559, 953)

Arterial Disorders

98. Ans. c. Heel, d. Head of metatarsal

99. Ans. d. All of the above 101. Ans. a. Myelopathy

AORTIC DISSECTION 102. Ans. a. Right lateral wall of ascending aorta (Ref: Harrison 20/e p1919, 19/e p1640; Sabiston 20/e p1746; Schwartz 10/e p806-816; Bailey 27/e p909-911) 104. Ans. b. Aorta

AORTIC ANEURYSM 105. Ans. a. Atherosclerosis (Ref: Harrison 20/e p1919, 19/e p1637; Sabiston 20/e p1722; Schwartz 10/e p850-859; Bailey 27/e p961) • “90% all abdominal aortic aneurysms are related to atherosclerotic disease and most of these aneurysms are below the level of renal arteries.”

Section 5

100. Ans. a. Vasospasm

103. Ans. a. Acute aortic dissection

739

106. Ans. a. Surgery indicated when size AAA > 6 cm (Ref: Harrison 20/e p1919, 19/e p1640; Sabiston 20/e p1725; Schwartz 10/e p850-859; Bailey 27/e p961) • Surgery indicated when size of AAA >5.5 cmQ (Not the 6 cm). • After rupture, mortality rate of emergent operation is 45-50%Q. • Most serious gastrointestinal complication is ischemia of the left colon and rectum. Post-operative hypotension and hemodynamic instability are contributory factorsQ. • Occasionally, microembolization can occur, resulting in small patchy areas of ischemia, usually on the plantar aspect of the foot, referred to as trash footQ. 107. Ans. a. Laterally into the left retroperitoneum 108. Ans. d. Myocardial infarction (Ref: Harrison 20/e p1919, 19/e p1640; Sabiston 20/e p1741; Schwartz 10/e p850-859; Bailey 27/e p965) • MC complication of descending aortic aneurysm surgery is non-fatal MI >Renal failure. 110. Ans. c. 55 mm (Ref: Sabiston 20/e p1725; Schwartz 10/e p852; Bailey 27/e p961)

“Surgical treatment is generally recommended for aneurysms more than 5.5 cm in maximal diameter, those demonstrating more than 5 mm of growth in 6 months or more than 1 cm in a year, and aneurysms with a saccular rather than the typical fusiform anatomy.”-Sabiston 20/e p1725 111. Ans. b. Emergency surgery complication Splenic arteryQ • MC site of splanchnic artery aneurysm: Splenic arteryQ 117. Ans. a. Splenic artery is most commonly involved 118. Ans. c. Discontinuation of arteria radicularis magna (Ref: Sabiston 20/e p1781)

Arteria Radicularis Magna • Arteria radicularis magna (artery of Adamkiewicz) is the main source of blood supply to the anterior spinal artery in the thoracolumbar segment of the spinal cordQ. • It arises from either one of the lower posterior intercostal arteries (T9-T11) or of the subcostal artery (T12), or less frequently of the upper lumbar arteries (L1 and L2)Q. • Occlusion or interruption is one of the major reasons for spinal cord ischemia (leading to paraparesis /paraplegia) during surgery for thoracoabdominal aneurysmsQ. 119. Ans. a. Saccular aneurism involves whole circumference 120. Ans. b. MRI 121. Ans. b. Aortic arch aneurysm (Ref: Cardiac Surgery by John Norman/p388)

Cardiothoracic Vascular Surgery

109. Ans. b. 5.5 cm

740

Surgery Essence

Cardiothoracic Vascular Surgery

• “Oliver’s sign: Downward displacement of cricoid cartilage in time with ventricular contraction, in the presence of an aortic arch aneurysm.”– Cardiac Surgery by John Norman/p388

FEMORAL ARTERY ANEURYSM 122. Ans. a. Ultrasound guided compression of the neck of aneurysm, b. Thrombin injection, c. Bypass graft repair (Ref: Sabiston 20/e p1782; Bailey 27/e p966) 123. Ans. a. Radial, b. Brachial, c. Femoral (Ref: Rutherford Vascular Surgery 6/e pvol I/456) • “IV drug abusers can lead to number of specific vascular complications including septic thrombophlebitis, Aortic dissection, AV fistula and necrotizing fasciitis with gangrene of extremity. The most frequent vascular complication in drug addicts is infected pseudoaneurysm of the femoralQ, brachialQ or radial arteryQ.”

POPLITEAL ARTERY ANEURYSM 124. Ans. c. Popliteal artery (Ref: Sabiston 20/e p1782; Bailey 27/e p966) 125. Ans. d. Uncommon among peripheral aneurysm Minimum Size for Surgery (AIPF: All India Police Force) Abdominal Aortic Aneurysm (5.5 cm) = Iliac aneurysm (3.5 cm) + Popliteal/Femoral aneurysm (2.0 cm)

PSEUDOANEURYSM 126. Ans. b. Trauma • MC cause of pseudoaneurysm: Trauma (Penetrating trauma or iatrogenic by catheterization)Q 127. Ans. a. Ligation of involved vessel and wide surgical debridement (Ref: Rutherford Vascular Surgery 6/e pvol II/248-249, 813, 1055-1056)

Treatment of Pseudoaneurysm • USG guided compression is the first therapeutic maneuverQ in non-invasive vascular technology. Although it is safe but efficacy is modest and the procedure is uncomfortable for patient and provider. • Injection of thrombin into the pseudoaneurysm under duplex ultrasound guidance has not replaced compression as treatment for pseudoaneurysm. This technique is safe and effectiveQ. It reduces physical effort and time requirements for the technologist and is significantly less uncomfortable for the patient. • As spontaneous resolution of pseudoaneurysm has a low incidence, surgical repair is the main therapy and gold standard therapyQ. Indications of Surgery

Section 5

Absolute • • • • •

Relative

Failure of other treatment modalitiesQ Suspected secondary infectionQ Evidence of vascular compromiseQ Ongoing or imminent hemorrhageQ Skin erosion and necrosis due to false aneurysm expansionQ

• Femoral neuropathy • Continuous anticoagulation • Concomitant AV fistula

Surgical Intervention • Operative repair can involve simple stitch or replacement of the entire vessel with graftQ. • Mainstays of surgical treatment: Proximal control (above inguinal ligament of needed, for extensive groin pseudoaneurysm), use of monofilament suture for vascular repair and debridement of devitalized tissueQ. • In the presence of any infection or when large residual tissue defect persists, muscle coverage with either sartorius or rectus abdominis flap over the repaired vessel must be usedQ.

Arterial Disorders

741

Treatment of Pseudoaneurysm in IV Drug Abusers

SUBCLAVIAN STEAL SYNDROME 128. Ans. a. First part (Ref: Grainger Diagnostic Radiology 4/e p 773; Bailey 27/e p952) MC site of subclavian artery stenosis is the first part of the subclavian artery. 129. a. Reversal of blood flow in the ipsilateral vertebral artery (Ref: Bailey 27/e p952)

Section 5

• Ligation is optional treatment for infected pseudoaneurysm because it is easy, safe and cost effective (in IV drug abuser there is high chance of infection)Q • Reconstruction is not recommended because of extension of infection at location of pseudoaneurysm and at artificial graft siteQ.

VASCULAR GRAFT 130. Ans. a. Autologous vein 131. Ans. a. Saphenous vein graft (upside-down) (Ref: Bailey 27/e p890, 949; Washington Manual of surgery 5/e p322)

Infra-inguinal Occlusive Disease (Femoral, Popliteal, Tibial Occlusive Disease) • Autologous vein is the conduit of choice for infra-inguinal bypass surgeryQ • Great saphenous vein is the vein of choice (Lesser saphenous vein or arm veins may be used)Q • These grafts may be used either ‘in situ’ after valve destruction or reversed conduit When Autologous vein grafts are not available PTFE grafts may be used. For above knee grafts, patency rates of PTFE grafts approach those achieved by autologous venous graftsQ • For below knee/distal grafts, patency rates of PTFE grafts are substantially lowerQ.

Most preferred graft for CABG is left internal mammary artery and most commonly used is long saphenous vein. “The conduit with the highest patency rate (98% at 5 years and 85%–90% at 10 years) is the internal thoracic artery which is most commonly left attached proximally to the subclavian artery (although occasionally used as a free graft) and anastomosed distally to the target coronary artery.”- Schwartz 10/e p743 133. Ans. c. Smooth muscle cells (Ref: Vascular.surgery.duke.edu/files/.../Vascular_Grafts_2-27-09.pdf; Schwartz 9/e p762-764)

Vascular Graft Failure • Smooth muscle cells in the middle layer (media) of the vessel wall become activated, divide, proliferate and migrate into the inner layer (intima)Q. • The resulting abnormal neointimal cells express pro-inflammatory molecules, including cytokines, chemokines, and adhesion molecules that further trigger a cascade of events that lead to occlusive neointimal hyperplasia and eventually graft failureQ. 134. Ans. b. Textile synthetic (Ref: Sabiston 20/e p234; Schwartz 10/e p4; Bailey 27/e p949, 950, 951) 135. Ans. c. Saphenous vein 136. Ans. a. Dacron (Ref: Schwartz 10/e p4) • Dacron is the favoured material for aortoiliac work, it gives excellent results. 137. Ans. a. Dacron 138. Ans. d. Can be easily incised and the opening resutured 139. Ans. d. PVC

140. Ans. a. LIMA

VASCULAR TRAUMA 141. Ans. c. The pulse is weakened, d. There is no pulse 142. Ans. a. Most commonly associated with deceleration injury, b. High morality, c. Surgery definitive treatment, d. Aortography gold standard (Ref: Sabiston 20/e p1810, 1818; Schwartz 10/e p214-215; Bailey 27/e p369) 143. Ans. a. Mediastinal widening, b. Abnormal aortic contour, d. Right paratracheal stripe thickening, e. Left apical pleural cap (Ref: Chapman 4/e p163-164; CSDT 11/e p257-259)

Cardiothoracic Vascular Surgery

132. Ans. c. Internal mammary artery (Ref: Sabiston 20/e p1672-1673; Schwartz 10/e p743; Bailey 27/e p890)

742

Surgery Essence

Cardiothoracic Vascular Surgery

Traumatic Aortic Rupture

         

Clinical Features

Radiographic Features

History of high-speed decelerationQ injury Flail chestQ Fractured sternumQ SVC syndrome Multiple or 1st or 2nd rib fractures Upper extremity hypertension or pulse deficitsQ • Hematoma in the carotid sheaths • Interscapular bruitsQ • Hoarseness with normal larynx • • • • • •

• • • • • • • • • •

Widening of mediastinumQ (70%) Esophageal deviation to the rightQ Tracheal deviation to rightQ LeftQ apical cap (65%) Downward displacement of left main stem bronchusQ Right paratracheal stripe thickeningQ Deviation of NG tube to right Left hemothorax Displaced left paraspinal stripeQ Displaced right paraspinal stripe

144. Ans. b. Left medial visceral rotation (Ref: Sabiston 20/e p1546, 1815) • Left medial visceral rotation exposes entire length of abdominal aorta, celiac axis, proximal part of mesenteric arteries and proximal left renal arteryQ. 145. Ans. b. Mechanical compression Adequate manual compression of the subclavian artery to obtain hemostasis is often not possible because of the interposing subcutaneous tissue and bony structure, as well as lack of structural support around subclavian artery. 146. Ans. d. Cattell-Braasch maneuver (Ref: Sabiston 20/e p1546) Maneuvers for Retroperitoneal Exposure Kocher’s Maneuver • Surgical maneuver to expose structures in the retroperitoneumQ behind duodenum & pancreasQ • Used for mobilization of duodenumQ

Extended Kocher’s Maneuver • Right sided medial visceral rotationQ • Right colon and duodenum is reflected mediallyQ • Exposes IVC, Infrarenal aorta, right renal artery and iliac vesselsQ • Recommended for drainage of infra-mesocolic hematomaQ

• • •



Cattell-Braasch Mattox Maneuver Maneuver Left sided medial visceral rotationQ • For extensive retroperitoneal Left sided viscera (Left kidney, left exposureQ colon, spleen and pancreas) are brought to midlineQ • Right colon is fully mobilized and Exposes entire length of abdominal reflected mediallyQ aorta, celiac axis, proximal part of mesenteric arteries and proximal • Good option for left renal arteryQ exposure of the infrapancreatic Recommended for drainage of segmentQ central supramesocolic hematomaQ

147. Ans. b. Kocher’s maneuver

ANGIOGRAPHY AND COMPLICATIONS 148. Ans. b. Arteriography (Ref: Sabiston 20/e p1762; Schwartz 10/e p832-833, 918; Bailey 26/e p190) • Seldinger needle is used for angiography (arteriography).

Arteriography

Section 5

• Aortic and lower extremity arteriograms are generally performed by needle puncture of the femoralQ or brachial arteriesQ followed by guidewire placement and catheter insertion using the Seldinger technique. 149. Ans. d. Intracerebral hemorrhage (Ref: Sabiston 20/e p1762; Schwartz 10/e p832-833; Bailey 26/e p190) Complications of contrast Arteriography

• • • • •

Puncture Site or Catheter Related Hemorrhage, hematomaQ PseudoaneurysmQ Arteriovenous fistula AtheroembolizationQ Local thrombosisQ

• • • •

Contrast Agent Related Anaphylactoid or sensitivity reaction Vasodilation, hypotensionQ NephrotoxicityQ HypervolemiaQ (osmotic load)

Arterial Disorders

743

150. Ans. a. Radial artery (Ref: Lee Anesthesia 12/e p25)

151. Ans. b. Single wall puncture is indicated in those with normal coagulation profile (Ref: Mastery of Surgery 5/e p218) • Single wall puncture is indicated in those with coagulopathy as there is more risk of bleeding in double wall puncture technique.

Angiography

Section 5

• Arterial puncture and cannulation is performed to measure PaO2, PaCO2, SpO2 and pH to clarify the acid-base and electrolyte status. • Any artery that can be compressed after puncture may be used (but not end arteries), usually the radialQ (preferred), brachial or femoral.

• Arterial access is obtained for hemodynamic monitoring and angiography and interventions. • Femoral artery is most frequently cannulated artery in the bodyQ. • Femoral artery is a large caliber vessel appropriate for angioplasty and stenting of peripheral vesselsQ. • Radial artery is most frequently used site of arterial cannulation for hemodynamic monitoringQ. • Femoral artery is identified in the inguinal region by its pulsation. • If fluoroscopy is being used, the femoral artery is typically located over the medial third of the femoral headQ. • Femoral artery cannulation can be done using Seldinger technique. • The Seldinger technique is a medical procedure to obtain safe access to blood vessels (both arteries and veins) and other hollow organsQ. • Single wall puncture is indicated in those with coagulopathy or if thrombolysis is planned because of increased risk of bleeding in double wall techniqueQ.

153. Ans. b. Subclavian artery

MISCELLANEOUS 154. Ans. c. Integrity of palmar arch 155. Ans. b. Superficial temporal

Cirsoid Aneurysm • A cirsoid aneurysm is the dilation of a group of blood vessels due to congenital malformations with AV (arteriovenous) shuntingQ. • Cirsoid means resembling a varix. • Most commonly occurs over the head usually the superficial temporal artery. • Superficial temporal artery is the most commonly involved arteryQ. 156. Ans. c. Palpable pulses (Ref: www.ncbi.nlm.nih.gov Br Med J v.1(3927); Apr 11, 1936)

Erythrocyanosis • A condition caused by exposure to coldQ • Characterized by swelling of the limbs and the appearance of irregular red-blue patches on the skinQ • Occurring especially in girls and womenQ 157. Ans. a. Aneurysm Endovascular Hunterian Ligation

Hunterian Ligation • Hunterian ligation refers to one of the oldest successful interventions for arterial aneurysms: Ligation of the femoral artery to treat a popliteal aneurysmQ by John Hunter in 1785. 158. Ans. c. It is not superficial and does not cross prominent bone • The popliteal pulse is difficult to feel because it is not superficial and does not cross a prominent boneQ.

Cardiothoracic Vascular Surgery

TAKAYASU ARTERITIS 152. Ans. a. Aortoarteritis (Ref: Harrison 19/e p2189; Sabiston 20/e p1780; Schwartz 10/e p788, 901; Bailey 27/e p967, 1416)

744

Surgery Essence

Cardiothoracic Vascular Surgery

159. Ans. c. Myocardial infarction (Ref: CSDT 11/e p822; Miller’s Anesthesia 6/e p2053) • Miller says “Myocardial performance is the single most important determinant of outcome following a major vascular operationQ.” • Non-fatal and fatal MIs are the most important and specific outcomes that determine perioperative cardiac morbidity in patients for vascular surgeryQ. • CSDT says “MI and stroke are most common causes of death in vascular surgeriesQ.” 160. Ans. d. Bifurcation of pulmonary artery (Ref: Harrison 20/e p1922, 18/e p2171)

Saddle Thrombus • A large thrombus lodged at an arterial bifurcation, where blood flows from a large-bore vessel to a smaller one. • The ‘classic’ saddle embolus, which occurs at the bifurcation of the pulmonary arteries in fatal pulmonary embolism secondary to a centrally migrating venous embolus, is distinctly uncommonQ 161. Ans. c. Salmonella 162. Ans. d. All of the above (Ref: Harrison 20/e p845, 19/e p296e-1; Schwartz 10/e p431-432, 836)

Stent Thrombosis • Clotting suppressant agents and anti-clotting agents should be continued after drug eluting stents to prevent stent thrombosisQ. • Endothelialization is a hallmark of vascular healing and is important for the prevention of thrombus formationQ. • For drug-eluting stents (which, by design, delay formation of a new endothelium cover over the stent), the incidence of clot formation within the stent may persist for a longer period of timeQ. • Drug eluting stents have been associated with delayed arterial healing and the prevalence of latent thrombus after five years, suggesting that patients may continue to be risk for stent thrombosis for an extended period of time. • Treatment with the antiplatelet drugs appears to be the most important factor reducing this risk of thrombosis, and early cessation of these drugs after drug-eluting stenting markedly increases the risk of stent thrombosis and myocardial infarctionQ. 163. Ans. d. Accidental injury to major vessels in thigh or groin (Ref: Bailey 18/e p69, 147)

Butcher’s Thigh • Butchers thigh is penetrating wound of femoral triangle due to knife slipping while boning meat. • Penetrating wound involving main veins in the thigh or groin are potentially fatal, as exsanguination may follow the first aid dressing which has apparently controlled the bleeding. 164. Ans. a. Direct pressure (Ref: Advanced Assessment and Treatment of Trauma by Americans (2010)/71)

External Bleeding Control of external hemorrhage during the early phase (circulation) of resuscitation is imperative. External bleeding is best controlled by direct digital pressure. Direct pressure assists in the process of coagulation by slowing the flow of blood out of the vessels and giving clot time to form. In most cases of external bleeding, if pressure is applied quickly to the area of hemorrhage (direct pressure) or to the blood vessel supplying the bleed (indirect pressure), the volume of blood escaping will be greatly reduced. • To be effective, direct pressure must be at least equal to the pressure of the blood attempting to escape. • Arterial bleeding is often difficult to control and may require upto 5 minutes of firm direct pressure to be successful.

Section 5

• • • •

165. Ans. a. Giant cell arteritis (Ref: Harrison 20/e p2583, 19/e p2188) Temporal arteritis is also known as Giant cell arteritis. 166. Ans. d. Most common side effect is diarrhea and hypotension (Ref: Anesthesiology Keyword Review by Raj K. Modak 2013/page 130; http:// www.ncbi.nlm.nih.gov/pubmed/7818115) 167. Ans. b. Carotid angioplasty and stenting 168. Ans. d. Onyx (Ref: Sabiston 20/e p1733; Embolization by Pascal Chabrot (20130/p3) “Temporary embolic agents include absorbable gelatin sponge, Gelfoam sponge powder, starch microspheres, oxidized cellulose, and collagen fiber preparations.” “The FDA approved Onyx (Ethylene-Vinyl Alcohol Copolymer) in 2005 for the treatment of arteriovenous malformations. It is classified as a liquid, nonadhesive, nonabsorbable, permanent embolic agent that can be used off-label for small and large vessels.”

CHAPTER

27

Venous Disorders

RISK FACTORS FOR HYPERCOAGULABLE STATES/THROMBOSIS Risk Factors for Hypercoagulable States/Thrombosis Inherited

Acquired

• Defective inhibition of coagulation factors: −− Factor V LeidenQ (resistant to inhibition by activated protein C) −− Antithrombin III deficiencyQ −− Protein C or S deficiencyQ −− Prothrombin gene mutationQ (G20210A) • Impaired clot lysis: −− Dysfibrinogenemia −− Plasminogen deficiencyQ −− t-PA deficiencyQ −− PAI-I excessQ • Uncertain mechanism: −− HomocystinuriaQ −− High homocysteine levels due to MTHFR mutationQ

• Diseases or syndromes: −− Lupus anticoagulant/anticardiolipin syndromeQ −− MalignancyQ, recent MIQ, infectionQ −− Myeloproliferative disorder −− Thrombotic thrombocytopenic purpuraQ −− EstrogenQ treatment −− Hyperlipidemia, Diabetes mellitusQ −− Hyperviscocity, polycythemiaQ −− Nephrotic syndromeQ −− Paroxysmal nocturnal hemoglobinuriaQ −− Inflammatory bowel diseaseQ −− Behcet’s syndromeQ • Physiological states: −− Pregnancy (especially post-partumQ) −− ObesityQ, ImmobilizationQ, Old ageQ −− Post-operative stateQ

• MC genetic cause for thrombophilia: Factor V Leiden • MC congenital cause of venous thrombosis: Factor V Leiden • MC genetic hereditary blood coagulation disorder: Factor V Leiden Risk of DVT in Relation to Age and Duration of Surgery

High Risk • General urological surgery in patient >40 yearsQ • Extensive pelvic or abdominal surgeryQ • Major orthopaedic surgeryQ of lower limbs

Moderate Risk

Low Risk

• General surgery in patients ≥40 yearsQ • Surgery lasting for ≥30 minutesQ • General surgery in patients 3 months without an obvious causeQ. • Enlargement is due to follicular hyperplasiaQ.

38. Ans. b. Kikuchi disease (Ref: Nelson 20/e p2414, 18/e p2094)



39. Ans. b. Extra nodal NK cell/T cell lymphoma nasal type

Cardiothoracic Vascular Surgery

• Investigation of choice in detecting small para-aortic lymph node: CT scanQ

CHAPTER

29

Thorax and Lung

MEDIASTINUM Mediastinum Mediastinum is situated between the lungs in the center of the thorax. Mediastinum is divided into 3 compartments Middle or Posterior or Visceral compartment Paravertebral sulci Lies within pericardial cavityQ Lies posterior to posterior pericardium including trachea and tracheaQ.

Anterior or Anterosuperior Lies in front of anterior pericardium and tracheaQ

• • • • • •

Anterior Mediastinum ThymomaQ (MC in anterior mediastinum) LymphomaQ Germ cell tumorsQ Thyroid and parathyroid massesQ Bronchogenic cystQ AneurysmQ



• • • •

Locations of the Common Mediastinal Masses Middle Mediastinum Cysts (MC in middle mediastinum): • −− Pericardial (MC)Q • −− BronchogenicQ • −− Enterogenous • −− NeuroentericQ • Q Vascular masses (aneurysm ) • LN enlargement and lymphomaQ • Mesenchymal tumors • Pheochromocytoma

Tumor Type Neurogenic tumorsQ CystsQ ThymomasQ

Mediastinal Tumors in Adults Percentage of Total 21 20 19

Posterior Mediastinum Neurogenic tumors (MC overallQ) MeningocelesQ Mesenchymal tumors Pheochromocytoma LymphomaQ Bochdalek herniaQ Bronchogenic cystQ Enterogenous cystQ

Location Posterior All Anterior

Thorax and Lung Tumor Type

Percentage of Total

Location

13

Anterior/middle

Germ cell tumors

11

Anterior

Mesenchymal tumors

7

All

Endocrine tumors

6

Anterior/middle

Mediastinal Tumors in Children Percentage of Total

Location

Neurogenic tumors

40Q

PosteriorQ

Lymphomas

18Q

Anterior/middle

Cysts

18

All

Germ cell tumors

11

Anterior

Mesenchymal tumors

9

All

Thymomas

Rare

Anterior

Section 5

LymphomasQ

Tumor Type

771

Mediastinal Masses (MM) MC anterior MM: ThymomaQ MC middle MM: CystQ (Pericardial cyst is MCQ) MC posterior MM: Neurogenic tumorsQ MC MM (overall): Neurogenic tumorsQ MM seen in all three compartments of mediastinum: Lymphoma, bronchogenic cyst & mesenchymal tumorsQ IOC for diagnosis of MM (except neurogenic tumors): CTQ IOC for diagnosis of neurogenic tumors: MRIQ

THYMOMA Thymoma • • • •

MC neoplasm of thymus; MC site: Anterior mediastinumQ Most frequently seen in 40-60 years of age Most thymomas are completely surrounded by a fibrous capsuleQ On the basis of cell types, divided into: −− Lymphocytic (25%) −− Epithelial (25%) −− Lymphoepithelial (50%)Q

Clinical Features • Mostly asymptomaticQ, detected incidentally on chest X-ray • May cause dysphagia, dyspnea, SVC syndrome and paraneoplastic syndromesQ Paraneoplastic Syndromes Autoimmune • • • •

SLEQ Rheumatoid arthritis Polymyositis Sarcoidosis

Hematological • • • •

CytopeniasQ Red cell aplasiaQ HypogammaglobulinemiaQ ErythrocytosisQ

Neuromuscular • • • •

Myasthenia gravisQ (MC) Neuromuscular disorders Myotonic dystrophy Myositis

Diagnosis • CT: Investigation of choice for diagnosis of thymomaQ. • Definitive diagnosis is made on histological studyQ. • CytokeratinQ is the marker that best distinguishes thymomas from lymphomas. • CT scan: Most lymphomas are associated with marked lymphadenopathy and thymomas most frequently appear as a solitary encapsulated massQ.

Cardiothoracic Vascular Surgery

• • • • • • •

772

Surgery Essence Staging

Cardiothoracic Vascular Surgery

• Masaoka stagingQ system is used. Treatment • Treatment of choice: Total thymectomy performed through median sternotomyQ • Large thymoma (>5 cm) with evidence of invasion: Thymectomy + ChemotherapyQ • Myasthenia gravis is treated with thymectomy and anticholinesterase drugsQ.

BRONCHOGENIC CYST Bronchogenic Cyst • Bronchogenic cysts originate as sequestrations from the ventral foregutQ, the antecedent of the tracheobronchial tree. • Cyst wall is composed pathognomonic inner layer of ciliated respiratory epitheliumQ. • MC location: Middle mediastinumQ (65-90%) Bronchogenic Cyst Mediastinal

Parenchymal (intrapulmonary)

• Sub-carinal, right paratracheal and hilar locations are most common • Does not communicate with the tracheobronchial treeQ

• Typically perihilar with predilection for lower lobesQ • Communicate with the tracheobronchial treeQ

Clinical Features • • • •

Two thirds are asymptomaticQ May produce symptoms that depend on their anatomic location. Paratracheal region: Airway compression and respiratory distress Lung parenchyma: Become infected, present with fever and cough.

Diagnosis • Chest X-ray: Shows a dense mass • CT scan or MRI: Delineates the precise anatomic location Treatment • Surgical excision is recommended in all patients to provide definitive histologic diagnosisQ, alleviate symptoms, and prevent the development of associated complications.

PULMONARY SEQUESTRATION Pulmonary Sequestration • Malformations of the lung without bronchial communication with an aberrant systemic blood supplyQ. • MC site: Posterobasal segment of left lower lobeQ • Presentation is with recurrent chest infectionQ.

Section 5

Pulmonary Sequestration

Intralobar • • • • •

Reside within lung Infrequently associated with other anomaliesQ Found within the medial or posterior segments of the lower lobes Two thirds occurring on left sideQ. In 85% of cases supplied by an anomalous systemic vessel arising from the infradiaphragmatic aortaQ, located within the inferior pulmonary ligament. • Venous drainage through the inferior pulmonary vein. • Because of the risk for infection and bleeding, usually removed, either by segmentectomy or lobectomyQ. parenchymaQ

Extralobar • • • •

Surrounded by a separate pleural coveringQ Occur predominantly in malesQ More common on the left sideQ. In 40%, multiple other anomaliesQ are encountered. • Usually asymptomaticQ • Because there is usually no bronchial communication, the risk for infection is low. • Many of these malformations may be observedQ.

Thorax and Lung

773

Diagnosis

• Historically, angiography was considered an important preoperative study before embarking on resection of a sequestration. • More recently, CT and MRI have replaced the need for angiography and provide excellent mapping of the blood supplyQ.

PECTUS EXCAVATUM

Section 5

• CT is preferred modality for diagnosis of pulmonary sequestrationQ.

Pectus Excavatum • • • •

Pectus excavatum (also called funnel chest) is the MC chest wall deformityQ Incidence: 1 of 400 children. More common in males Arises from excessive growth of the lower costal cartilages, causing posterior sternal depressionQ.

Clinical Features • Most patients are asymptomatic at the time of presentation • Decrease in respiratory reserveQ or pain along the costal cartilages with exercise. Diagnosis

• In severe cases, decreased stroke volume and cardiac output with a restrictive pattern (decreased maximal breathing capacity) on PFTQ. • Chest CT: Haller indexQ (Ratio of width of chest wall to depth of sternum to vertebral body Treatment • Early repair with best results between 2-8 years of age. Indications for operative intervention • • Psychosocial factorsQ CosmesisQ

• Presence of respiratory or cardiovascular insufficiencyQ

• Ravitch procedureQ: Open surgical procedure for pectus excavatum • Nuss procedureQ: Minimal invasive surgery for pectus excavatum

PECTUS CARINATUM Pectus Carinatum • Also known as pigeon chestQ • Sternum is elevated above the level of chestQ • MC type: Anterior displacement of mid & lower sternum with adjacent costal cartilagesQ • More common in males • Associated with mitral valve disease & coarctation of aortaQ Clinical Features • Most patients are asymptomaticQ • Treatment is offered for cosmetic reasons Treatment • Surgery involves mobilizing sternum & costal cartilage close to its anatomical position

Cardiothoracic Vascular Surgery

• PFT: Evaluation of baseline pulmonary functionQ • ECHO: Cardiovascular assessment

774

Surgery Essence

Cardiothoracic Vascular Surgery

EVENTRATION OF THE DIAPHRAGM Eventration of the Diaphragm • Abnormally elevated position of diaphragm from paralysis or atrophy of the muscle fibers. • May be congenital anomalyQ caused by failure of muscularizationQ of the dome of hemidiaphragm, or it may be acquired as a result of dysfunction of the phrenic nerve related to the neuromuscular disease or trauma including operative trauma. • Birth trauma: MC cause of acquired diaphragmatic eventration, usually related to breech presentation. • Most commonly diagnosed in pediatric patientsQ • More common in males and affect left hemidiaphragm more frequently. Treatment • Diaphragmatic plication is preferred over resectionQ. • Surgery to fix the diaphragm in inspirationQ so the paradoxical movement and mediastinal shift are minimized.

PLEURAL EFFUSION Pleural Effusion • • • •

About 300 mLQ of fluid is required for the development of costophrenic angle blunting seen on an upright chest X-ray. At least 500 mLQ of effusion is necessary for detection on clinical examination. Chest tube insertion is carried out in such a way (angled chest tube, low insertion site) that drainage is as complete as possible. Chest tube insertion is done in 5th ICS in midaxillary line. • Ideally chest tube insertion should be done through triangle of safety (4th to 6th intercostal space, anterior to mid-axillary lineQ)

Boundaries of Triangle of Safety • Lateral border of pectoralis majorQ   • Anterior border of Lattisimus dorsiQ   • Superior border of 5th or 6th ribQ Pleural Effusion Exudative

Transudative

• Local that influence the formation and absorption of pleural fluid are altered. • Laboratory Features • Total serum protein >3 gm/dlQ • Pleural fluid protein/serum protein >0.5Q • Pleural fluid LDH/serum LDH >0.6Q • Pleural fluid LDH >2/3rd upper limit of serumQ factorsQ

• Systemic factorsQ that influence the formation and absorption of pleural fluid are altered. • Laboratory Features • Total serum protein 1500 ml (blunt injury)Q

• Selected descending torn aorta or great vessel injuryQ

• Ongoing tube thoracostomy drainage of >200 ml/hr for 3 consecutive hoursQ in non-coagulopathic patients

• Cardiac herniationQ

• Caked hemothoraxQ despite of placement of two chest tubes • Tracheo-bronchial injuryQ

• Pericardial tamponadeQ • Massive air leakQ from chest tube with inadequate ventilation • Open pneumothoraxQ • Esophageal perforationQ

Thorax and Lung

777

LUNG ABSCESS

• Lung abscess refers to a microbial infection of the lung that results in necrosis of the pulmonary parenchyma. • MC cause of primary lung abscess: Anaerobic bacteriaQ • Etiology of anaerobic lung abscess: AspirationQ Routes of Infection • Aspiration of organisms that colonize oropharynx (MC)Q • Inhalation of infection or aerosols • Hematogenous dissemination from extrapulmonary site • Direct inoculation (as in tracheal intubation or stab wounds) • Contiguous spread from an adjacent site of infection

Section 5

Lung Abscess

Clinical Features • Classic presentation: An indolent infection that evolves over several days or weeks, usually in a host who has a predisposition to aspirationQ. • A common feature is periodontal infection with pyorrhea or gingivitis. • Symptoms: Fatigue, cough, sputum production, and feverQ Chills are uncommon. Diagnosis • Lung abscess can be detected by chest X-ray and CT • CT scan: Investigation of choice for lung abscessQ Treatment • Treatment depends on the presumed or established etiology. • Infections caused by anaerobic bacteria: ClindamycinQ

• Lung abscess due to S. aureus: VancomycinQ • Indications for surgery: Failure to respond to medical management, suspected neoplasm, and hemorrhageQ. Causes of Failures of Medical Management • Failure to drain pleural collectionsQ • Inappropriate antimicrobial therapyQ • Obstructed bronchusQ

• Giant abscessQ • Resistant pathogenQ • Refractory lesionsQ

CHYLOTHORAX Chylothorax • A chylothorax occurs when the thoracic duct is disrupted and chyle accumulates in the pleural space. • MC cause of chylothorax: Trauma (most frequently thoracic surgeryQ) • More common on right sideQ Clinical Features • Patients with chylothorax present with dyspnea, chest pain, fatigue • Thoracentesis reveals milky fluidQ • TG level >1.2 mmol/L (110 mg/dLQ). Diagnosis • Chest X-ray: Large pleural effusionQ • Patients with chylothorax and no obvious trauma should have a lymphangiogram and a mediastinal CT scan to assess the mediastinum for lymph nodesQ. Treatment • Treatment of choice: Chest tube insertion + Administration of octreotide and Medium chain triglyceridesQ. • If these modalities fail, a pleuroperitoneal shunt should be placed unless the patient has chylous ascites. • An alternative treatment is ligation of the thoracic ductQ. • Patients with chylothoraxes should not undergo prolonged tube thoracostomy with chest tube drainage because this will lead to malnutrition and immunologic incompetenceQ.

Cardiothoracic Vascular Surgery

• Persistence of fever beyond 5–7 days or progression of the infiltrate suggests failure of therapy and a need to exclude factors such as obstruction, complicating empyema, and involvement of antibiotic-resistant bacteriaQ.

778

Surgery Essence

Cardiothoracic Vascular Surgery

CHYLURIA Chyluria • Filariasis is the MC cause, with chyluria occurring in 1–2% of cases 10–20 years after initial infestation. • • • •

Clinical Features

Causes of Chyluria • TuberculosisQ FilariasisQ Ascariasis • PregnancyQ Malaria • ChildbirthQ Tumour

• It usually presents as painless passage of milky white urine, particularly after a fatty meal. • The chyle may clot, leading to renal colic, and hypoproteinaemia may result. Diagnosis • Intravenous urography and/or lymphangiography will often demonstrate the lymphourinary fistula. Treatment • Treatment includes a low-fat and high protein diet, increased oral fluids to prevent clot colic, and laparotomy and ligation of the dilated lymphatics. • Attempts have also been made to sclerose the lymphatics either directly or via instrumentation of the bladder, ureter and renal pelvis.

THORACIC DUCT INJURY Thoracic Duct Injury traumaQ

• Causes: Iatrogenic (MC), penetrating and blunt trauma • Iatrogenic chylothorax is a well-known complication of thoracic operationsQ Clinical presentation • Most of the patient presents with chylothorax • In iatrogenic injuries, chylous fluid coming from drain site Complications of Thoracic Duct Injury • DehydrationQ • Electrolyte lossesQ • Loss of fat and fat-soluble vitaminsQ

• Protein lossQ • Impaired immunityQ secondary to loss of circulating lymphocytes (lymphopeniaQ)

Management • Conservative management: Low-fat diet with medium chain triglyceridesQ, TPN, correction of electrolyte imbalance and adequate drainage by chest tube or neck drainQ • Octreotide and recently, Etilefrine (an adrenergic agent that acts by causing smooth muscle contraction of the thoracic duct) are effectiveQ. • Iatrogenic thoracic duct injury shows higher rate of spontaneous closure with conservative treatmentQ than traumatic injury. • Surgery: Surgery is indicated after 5 days of conservative treatment, if chyle loss exceeds 1500 ml or if leak persist after 2 weeksQ.

Section 5

• VATS guided thoracic duct ligation is surgical treatment of choiceQ. Poirier’s Triangle • Thoracic duct is readily identified within the confines of Poirier’s triangleQ via left posterolateral thoracotomy. • Boundaries: Arch of aorta, Left subclavian artery and Vertebral column • The thoracic duct traverses the triangle on esophagus that forms the floor of the triangle.

POSTOPERATIVE LUNG COLLAPSE (ATELECTASIS) Postoperative Lung Collapse (Atelectasis) • MC postoperative respiratory complication: AtelectasisQ • As a result of the anesthetic, abdominal incision, and postoperative narcotics, the alveoli in the periphery collapse and a pulmonary shunt may occur. • Aggressive pulmonary toilet to prevent buildup of secretions and secondary infection High risk in heavy smokers, obese patientsQ

Thorax and Lung

779

Clinical Features

Management • Prevention of atelectasis: Pain control, deep breaths (spirometry) & coughQ • Rarely, intermittent positive pressure breathing and chest physiotherapy may be required. • Encouraging the patient to breathe deeply and cough is the single most valuable management approach in preventing and resolving atelectasis and pneumoniaQ. • Pneumonia: Managed with aggressive pulmonary toilet, induced sputum for culture and sensitivity testing, IV antibiotic therapy.

Section 5

• MC cause of a postoperative fever in the first 48 hours: AtelectasisQ • Symptoms: Low-grade fever, malaise and diminished breath sounds in the lower lung fields.

ACUTE RESPIRATORY DISTRESS SYNDROME Acute Respiratory Distress Syndrome • ARDS is a clinical syndrome of severe dyspnea of rapid onset, hypoxemia, and diffuse pulmonary infiltrates leading to respiratory failureQ. • ARDS is caused by diffuse lung injury from many underlying medical and surgical disorders. • Diffuse alveolar damage is the most characteristic feature of ARDSQ. Clinical Disorders Associated with ARDS

• • • • •

Indirect Lung Injury

PneumoniaQ

SepsisQ

• • Severe trauma: Multiple bone fractures, Flail chest, Head trauma, BurnsQ • Multiple transfusions, Drug overdose • PancreatitisQ • Postcardiopulmonary bypassQ

Aspiration of gastric contentsQ Pulmonary contusionQ Near-drowningQ Toxic inhalation injuryQ

Clinical Course and Pathophysiology

Exudative Phase • Alveolar capillary endothelial cells and type I pneumocytes are injuredQ • Edema rich in protein accumulates in interstitial & alveolar spacesQ • Neutrophils traffic into the pulmonary interstitium & alveoliQ • Condensed plasma proteins aggregate in the air spacesQ with cellular debris & dysfunctional pulmonary surfactant to form hyaline membrane whorls. • Alveolar edema in dependent portions of lung • Diminished aeration &

atelectasisQ

• Intrapulmonary shunting & hypoxemiaQ develop • Tachypnea & increased work of breathing frequently result in respiratory fatigue and respiratory failureQ

Proliferative Phase • Lasts from day 7 to day

21Q

• Most patients recover rapidly and are liberated from mechanical ventilation during this phaseQ. • Some patients develop progressive lung injury and early changes of pulmonary fibrosisQ. • First signs of resolution are evident with initiation of lung repair, organization of alveolar exudates, and lymphocytepredominant pulmonary infiltrateQ. • Proliferation of type II pneumocytes along alveolar basement membranesQ. • Presence of alveolar type III procollagen peptide, a marker of pulmonary fibrosis, is associated with a protracted clinical course and increased mortality from ARDSQ

Fibrotic Phase • Require long-term support on mechanical ventilators and/or supplemental oxygen. • Alveolar edema and inflammatory exudates converted to extensive alveolar duct and interstitial fibrosisQ. • Acinar architecture is markedly disrupted, leading to emphysema-like changes with large bullaeQ. • Progressive vascular occlusion & pulmonary hypertension. • Increased risk of pneumothorax, reductions in lung compliance, and increased pulmonary dead spaceQ.

Cardiothoracic Vascular Surgery

Direct Lung Injury

780

Surgery Essence

Cardiothoracic Vascular Surgery

Management of ARDS • A large-scale, randomized controlled trial sponsored by the National Institutes of Health and conducted by the ARDS Network compared low VT (6 mL/kgQ predicted body weight) ventilation to conventional VT (12 mL/kg predicted body weight) ventilation. • Mortality was significantly lower in the low VT patients (31%) compared to the conventional VT patients (40%). • This improvement in survival represents the most substantial benefit in ARDS mortalityQ demonstrated for any therapeutic intervention in ARDS to date.

PULMONARY EMBOLISM Pulmonary Embolism • • • •

Risk factors for pulmonary embolism are the risk factors for thrombi formation within venous circulation. Calf venous thrombosis: Low risk for embolism MC form of thromboembolic disease Thrombosis of larger veins: High risk for embolism (due to loosely attached thrombus to venous wall) • MC site for DVT: Calf veinsQ • MC source for pulmonary emboli: Proximal vein of lower extremityQ (femoro-popliteal and iliac vein) Risk Factors for Pulmonary Thromboembolism • • • • • • • •

age)Q

Age (Increasing ObesityQ Immobility (bed rest >4 days)Q PregnancyQ and PuerperiumQ High dose estrogen therapyQ Surgery/trauma (especially of pelvis, hip or lower limb)Q Malignancy (especially pelvis, abdominal, metastatic) Heart failure/Recent MIQ

• • • • • • • •

Nephrotic syndromeQ Inflammatory bowel diseaseQ PolycythemiaQ PNHQ or Lupus anticoagulant Behcet’s syndromeQ HomocystinuriaQ Paralysis of lower limb Varicose veins, Infection

Clinical Features • • • • • •

Most (60–80%) are clinically silent beause they are small and there is dual circulation in lungs. Symptoms: Dyspnea (MC)Q, chest pain, hemoptysis and cough Signs: Tachypnea (MC)Q, fever, unilateral leg swelling, wheeze, pleural friction rub More than 60% obstruction occurs in pulmonary circulation leading to sudden death, COR pulmonale or cardiovascular collapse. Multiple emboli over time may cause pulmonary hypertension and right ventricular failure. Paradoxical embolus can pass through an inter-atrial or inter ventricular defect. There by entering the systemic circulation. • Any patient with high likelihood of pulmonary embolism on clinical evaluation straightaway undergoes imaging tests, while a patient with low clinical likelihood should first undergo D-dimer test. Factors for Clinical Assessment of Pulmonary Embolism

Section 5

• Clinical signs and symptoms of DVTQ • An alternative diagnosis is less likely than pulmonary embolism • Heart rate > 100/minQ • HemoptysisQ

• Immobilization or previous surgery in 4 weeksQ • Previous DVT/PEQ • MalignancyQ (on treatment, treatment in past 6 months)

ECG Changes in Pulmonary Embolism (Sinus tachycardia: MC and non-specific finding on ECGQ) Features of Acute Right Heart Strain • • • • •

Acute right axis deviation P pulmonale Right bundle branch block Inverted T waves ST segment change

Highly predictive of PE • S1Q3T3 Seen in 10 micronQ in diameter occur in most adults after close fracture of long bonesQ and histological traces of fat can be found in the lungs and other internal organs. Clinical Presentation • Usually manifests itself within 24-48 hoursQ. • Early warning signs (within 72 hours of injury): Slight rise in temperature (pyrexia) and pulse rate (tachycardia)Q

Diagnosis • In addition to the classic clinical features, signs of retinal artery emboli (Striate hemorrhage and exudatesQ) may be present. • Sputum and urine: Presence of fat globulesQ. • Chest X-ray: Patchy pulmonary infiltration (Snow storm appearance)Q Laboratory Tests No characteristic laboratory test, suggestive findings are: • ThrombocytopeniaQ  •  (platelets hemoptysis. Slightly more common on right side, more frequently occurs in upper lobesQ Major source of hemoptysis are bronchial arteriesQ Endobronchial growth of central tumors cause cough, stridor, wheeze and dyspnoea Peripheral tumors present as pain due to pleural or chest wall involvement Symptoms due to regional spread • Tracheal obstruction, esophageal compression, RLN paralysisQ • Pancoast syndromeQ (involvement of C8T1 nerves by pancoast tumor causing pain in ipsilateral shoulder and armQ)

• Horner’s syndrome • SVC syndrome (MC cause is small cell carcinoma >SCC)Q • Malignant pleural effusionQ

Section 5

• • • • •

Metastases • MC Site of metastasis: BrainQ > BoneQ > LiverQ > AdrenalQ > LungQ (BBLAL) • CA lung is MC primary for metastasis to Kidney, Esophagus, Pancreas, Adrenal, Brain & Skin (KEPABS). Paraneoplastic syndromes • CVS: Thrombophlebitis, non-bacterial thrombotic • GIT: Carcinoid syndrome endocarditis • Erythrocytosis • Metabolic: • Neuromuscular: −− Inappropriate ACTH and ADH secretion (small cell)Q −− Dementia, optic neuritis, retinopathy, limbic encephalitis −− Hypercalcemia (SCC)Q −− Autonomic neuropathy (small cell)Q • Acanthosis nigricans (adenocarcinoma), −− Lambert-Eaton syndromeQ (small cell) dermatomyositis, icthyosis, erythema gyretum repens −− Polymyositis, cerebellar degenerationQ Diagnosis and Staging

8TH AJCC TNM CLASSIFICATION OF LUNG CANCER 8th AJCC (2017) TNM Classification of Lung Cancer Tis: Carcinoma in situ T1a: Tumor ≤1 cm in greatest dimensionQ T1b: Tumor >1 cm but ≤2 cm in greatest dimensionQ T1c: Tumor >2 cm but ≤3 cm in greatest dimensionQ T2: Tumor >3 cm but ≤5cm or tumor with any of the following features: Involves main bronchus, regardless of distance to the carina but without involvement of carinaQ Invades visceral pleuraQ Associated with atelectasis or obstructive pneumonitis that extends to the hilar region either involving part of or the entire lungQ T2a: Tumor >3 cm but ≤4 cm in greatest dimensionQ T2b: Tumor >4 cm but ≤5 cm in greatest dimensionQ T3: Tumor >5 cm but ≤7 cm in greatest dimension or one that directly invades any of the following: parietal pleura, chest wall (including superior sulcus tumors), phrenic nerve, parietal pericardium; or separate tumor nodule(s) in the same lobe as the primaryQ T4: Tumor >7 cm or of any size that invades any of the following: diaphragm, mediastinum, heart, great vessels, trachea, recurrent laryngeal nerve, esophagus, vertebral body, carina; or Separate tumor nodule(s) in a different ipsilateral lobe to that of primaryQ N1: Metastasis in ipsilateral peribronchial and/or ipsilateral hilar lymph nodes and intrapulmonary nodes, including involvement by direct extensionQ N2: Metastasis in ipsilateral mediastinal and/or subcarinal lymph node(s)Q N3: Metastasis in contralateral mediastinal, contralateral hilar, ipsilateral or contralateral scalene, or supraclavicular lymph node(s)Q

Cardiothoracic Vascular Surgery

• Tissue diagnosis: Tumor tissue can be obtained by bronchial or transbronchial biopsyQ through fiberoptic bronchoscopyQ; by percutaneous biopsy of enlarge node • Integrated PET-CT scan is the best imaging modalityQ for diagnosis and staging

Cardiothoracic Vascular Surgery

786

Surgery Essence M1a: Separate tumor nodule(s) in a contralateral lobe; tumor with pleural or pericardial nodules or malignant pleural (or pericardial) effusionQ M1b: Single extra-thoracic metastasis in a single or multiple organsQ M1c: Multiple extra-thoracic metastasis in single or multiple organsQ 8th AJCC (2017) TNM Stage Groupings Stage T N M Occult cancer TX N0 M0 0 Tis N0 M0 IA T1 N0 M0 IA1 T1mi-T1a N0 M0 IA2 T1b N0 M0 IA3 T1c N0 M0 IB T2a N0 M0 IIA T2b N0 M0 IIB T1a-c, T2a-b N1 M0 T3 N0 M0 IIIA T1a-c, T2a-b N2 M0 T3 N1 M0 T4 N0-1 M0 IIIB T1a-c, T2a-b N3 M0 T3, T4 N2 M0 IIIC T3, T4 N3 M0 IVA Any T Any N M1a/b IVB Any T Any N M1c Treatment of Operable NSCCL • Stage IA, IB, IIA, IIB: Surgical resectionQ • Adjuvant chemotherapy is given in stage IIQ • Stage IIIA with minimal N2 involvement: Neoadjuvant chemotherapy followed by surgical resection with complete mediastinal LN dissectionQ • Postoperative radiotherapy for patients found to have N2 diseaseQ

Section 5

WHO Classification of Carcinoma Lung

Adenocarcinoma

Squamous Cell Carcinoma

• MC histological typeQ • MC in non-smokers, young patients, femalesQ • Located peripherallyQ • Slow growth and propensity to metastasize to opposite lungQ • Metastasize more frequently to CNSQ • Most cells contain mucinQ • Noguchi classificationQ is used for adenocarcinoma

• MC in smokersQ • MC type in India • MC variety associated with hypercalcemia (produces PTH-rp)Q • CentralQ in distribution • Prone to undergo central necrosis & cavitationQ • Pancoast tumor is histologically SCCQ • Associated with best prognosisQ

Small Cell Carcinoma • Most malignant, centralQ in distribution, strongly related to smokingQ • Cells are small with little cytoplasm called “oat cell”Q • Associated with massive hilar or mediastinal lymphadenopathy, mediastinal invasion and perihilar massQ • MC variety associated with paraneoplastic syndrome, hypokalemia and SVC syndromeQ • Most responsive to chemotherapy (cisplatin + etoposide) • Shows response to radiotherapyQ • Hormones produced by small cell carcinoma: ACTH, AVP (vasopressin), calcitonin, ANF, gastrin releasing peptideQ

Large Cell Carcinoma • Highly undifferentiated with cavitating nature • Metastasize early with poor prognosis

Thorax and Lung

787

BRONCHIOLOALVEOLAR CARCINOMA (BAC)

• BAC occurs in the pulmonary parenchyma in the terminal bronchioloalveolar regionsQ. Histologically • Characterized by a pure bronchioloalveolar growth pattern with no evidence of stromal, vascular, or pleural invasionQ. • Key feature: Growth along preexisting structuresQ without destruction of alveolar architecture. This growth pattern has been termed “lepidic”, and allusion to the neoplastic cells resembling butterflies sitting on a fence. Two subtypes

Section 5

Bronchioloalveolar Carcinoma (BAC)

• Nonmucinous: Columnar, peg-shaped, or cuboidal cells • Mucinous: Distinctive, tall, columnar cells with cytoplasmic and intra-alveolar mucin, growing along the alveolar septa. Ultrastructurally • BAC are a heterogeneous group, consisting of mucin-screting bronchiolar cellsQ, Clara cellsQ, or, rarely, type II pneumocytesQ

PANCOAST TUMOR (SUPERIOR SULCUS TUMOR) Pancoast Tumor (Superior Sulcus Tumor)

Treatment • Preoperative RT followed by En bloc resection of lung and chest wall with consideration of postoperative RT or intra-operative brachytherapyQ.

MYXOMA Myxoma • • • •

MC type of primary cardiac tumor in all age groups: MyxomaQ Occur at all ages, most commonly in 3rd to 6th decades, with a female predilection. Approximately 90% are sporadic Remainder are familial with autosomal dominant transmission (NAME and LAMB syndrome)

Pathology • Myxomas are gelatinous structures, consist of myxoma cells embedded in a stroma rich in glycosaminoglycans. • Most are solitary, are located in the atria (particularly the left atrium, where they usually arise from the interatrial septum in the vicinity of the fossa ovalis), and are often pedunculated on a fibrovascular stalk. • In contrast to sporadic tumors, familial or syndromic tumors tend to occur in younger individuals, are often multiple, may be ventricular in location, and are more likely to recur after initial resection. Clinical Features • MC clinical presentation mimics that of mitral valve disease: either stenosis owing to tumor prolapse into the mitral orifice or regurgitation resulting from tumor-induced valvular trauma. • A characteristic low-pitched sound, a “tumor plop,” may be appreciated on auscultation during early or mid-diastole • Constitutional signs and symptoms: Fever, weight loss, cachexia, malaise Diagnosis • IOC for diagnosis: 2D transthoracic or transesophageal echocardiographyQ • CT and MRI: Information regarding size, shape, composition and surface characteristics • Cardiac catheterization and angiography are no longer considered mandatory

Cardiothoracic Vascular Surgery

• Pancoast’s (or superior sulcus tumor) syndrome results from local extension of a tumor growing in the apex of the lung with involvement of eighth cervical and 1st and 2nd thoracic nerves, with shoulder pain characteristically radiates in the ulnar distribution of the arm, often with radiologic destruction of 1st and 2nd ribsQ. • Often Horner’s syndrome and Pancoast’s syndrome co-exist • IOC for diagnosis: MRIQ

788

Surgery Essence

Section 5

Cardiothoracic Vascular Surgery

Treatment • Surgical excision utilizing cardiopulmonary bypass is curativeQ. • Recur in 12–22% of familial cases but in only 1–2% of sporadic cases. • Tumor recurrence most likely is due to multifocal lesions in the former and inadequate resection in the latterQ.

CARDIAC TUMORS Cardiac Tumors • • • • • •

MyxomaQ

MC primary cardiac tumor: MC malignant tumor of heart in adults: AngiosarcomaQ MC benign tumor of heart in children: RhabdomyomaQ MC malignant tumor of heart in children: RhabdomyosarcomaQ MC tumor of cardiac valves: Papillary fibroelastomaQ MC site of involvement in cardiac metastasis: PericardiumQ

CHEST WALL TUMORS Chest Wall Tumors • MC chest wall tumor: MetastasisQ • MC benign primary chest wall neoplasm: OsteochondromaQ • MC malignant primary chest wall neoplasm: ChondrosarcomaQ

Multiple Choice Questions a. Hypogammaglobulinemia b. Hyperalbuminemia c. Red cell aplasia d. Myasthenia gravis

MEDIASTINAL TUMORS

1. Middle mediastinal masses include all the following except: a. Bronchogenic cyst (COMEDK 2004) b. Ascending aortic aneurysm c. Pericardial cyst d. Ganglioneuroma

2. Which of the following is not an anterior mediastinal mass?  (Recent Question 2018) a. Thymoma b. Neurogenic tumor c. Thyroid mass d. Lymphoma





3. Common location of thoracic pheochromocytoma:  (Recent Question 2017) a. Anterior mediastinum b. Posterior mediastinum c. 
Middle mediastinum d. Superior mediastinum 4. Middle mediastinal masses include all the following except:  (COMEDK 2005) a. Bronchogenic cyst b. Lymphoma c. Pericardial cyst d. Ganglioneuroma 5. Following are the tumors of posterior mediastinum except:  (COMEDK 2006) a. Neuroblastoma b. Ganglioneuroma c. Paravertebral abscess d. Thymic tumor



6. The commonest anterior mediastinal tumors is:  (MCI June 2018, Recent Question 2017, WBPG 2015, Recent Question 2015, COMEDK 2008) a. Aneurysm of descending aorta b. Neurogenic tumour c. Thymoma d. Bronchogenic cyst







7. During exploration, a patient is found to have a tumor in the thymus that is invading the pericardium and surrounding the left and right phrenic nerves. The pathologist says that appears on frozen section to be a benign thymoma. The surgeon now should: (Recent Question 2018) a. Repeat frozen section b. Attempt as complete a resection as possible c. Close the chest and plan irradiation therapy d. Close the chest and await permanent sections

9. D/D of anterior mediastinal mass includes (PGI May 2011) a. Teratoma b. Thymoma c. Lymphoma d. Neurogenic tumor e. Parathyroid carcinoma



10. Lymphoma most commonly affects which compartment of the mediastinum?  (MHPGMCET 2006) a. Anterior b. Middle c. Posterior d. Inferior



11. The most common mediastinal tumor:  (MHCET 2016, MHSSMCET 2008) a. Thymoma b. Lymphoma c. Neurofibroma d. Bronchogenic cyst 12. In thymoma, all are seen except: 

(AIIMS June 2001)

16. Which tumor among the following is not found in anterior mediastinum?  (AIIMS Nov 95) a. Retrosternal goitre b. Thymoma c. Teratomatous mass d. Neurogenic tumour

17. Most common tumor in the posterior mediastinum is: (Recent Question 2016, DNB 2005, 2000, All India 2008, DPG 2008) a. Neurofibroma b. Teratoma c. Lymphoma d. Bronchogenic cyst 18. Thymectomy causes: (TN 98) a. Failure of rejection of transplanted organs b. Myasthenia gravis c. Autoimmune disorders d. None of the above 19. The most common primary tumor of mediastinum: a. Lymphoma b. Teratoma c. Neurogenic tumor d. Thymoma 20. Tumors of anterior mediastinum include the following except:  (Kerala PG 2015, UPSC 2007) a. Thymoma b. Lymphoma c. Germ cell tumour d. Schwannoma

8. Common tumor of posterior mediastinum are:  (PGI Nov 2009) a. Lymphoma b. Neuroblastoma c. Neurogenic tumors d. Thymoma e. Bronchogenic cyst







13. Not a posterior mediastinal tumor:  (AIIMS Nov 98) a. Neurofibroma b. Lymphoma c. Thymoma d. Gastroenteric cyst 14. Posterior mediastinal tumors:  (PGI June 2003) a. Neuroblastoma b. Bronchogenic cyst c. Neuroenteric cyst d. Lymphoma e. Anterior thoracic meningioma 15. Majority of lung cysts occur in: (AIIMS Nov 94) a. Mediastinum b. Near carina c. Base of the lung d. Peribronchial tissue

21. Which of the following is commonly present in middle mediastinum? (PGI November 2017) a. Schwannoma b. Thymoma c. Bronchogenic cyst d. Teratoma e. Substernal thyroid

PLEURAL EFFUSION

22. Most common site for putting chest drain in case of pleural effusion: (Bihar PG 2014, AIIMS June 2000, All India 2002) a. 2nd intercostal space mid-clavicular line b. 7th intercostal space mid-axillary line c. 5th intercostal space mid clavicular line d. 5th intercostal space just lateral to vertebral column



23. Meig’s syndrome consist of the following except: (Karnataka 94) a. Ascites b. Hydrothorax c. Benign ovarian tumor d. Malignant ovarian tumor



24. Pseudochylous pleural effusion is most often seen in: a. Tb b. Lymphoma c. CA lung d. Filariasis

790

Surgery Essence

Section 5

Cardiothoracic Vascular Surgery



25. A rapidly filling hemorrhagic pleural effusion is suggestive of:  (COMEDK 2004) a. Pneumococcal infection b. Tuberculosis c. Bronchiectasis d. Bronchogenic carcinoma

PNEUMOTHORAX

26. Spontaneous pneumothorax is commonly seen in:  (PGI June 2002) a. Smokers b. Young females c. Old age d. Short statured men



27. Which of the following is a cause of unilateral hyperluscent lung on chest radiography? (COMEDK 2009) a. Poland syndrome b. Asthma c. Acute bronchiolitis d. Pleural effusion 28. Pneumothorax of what size generally needs operative treatment? (MHSSMCET 2010) a. >10% b. >20% c. >30% d. >40 29. In a patient with one episode of spontaneous pneumothorax, which is advised? (Jharkhand 2003) a. Stop diving b. Stop smoking c. Stop flying d. All 30. For open pneumothorax, which of the following is treatment of choice?  (AIIMS June 97) a. IPPV b. ICD with underwater seal c. Thoracostomy and close the rent d. Wait and watch 31. A case of spontaneous pneumothorax comes to you. What will be earliest treatment of choice? (AIIMS June 97) a. IPPV b. Needle aspiration c. ICD d. Wait and watch











32. Spontaneous pneumothorax exceeding ….. % of chest cavity should have a chest tube inserted: a. 10 b. 25 c. 45 d. 60 33. In pneumothorax due to blunt injury, treatment of choice is:  (AIIMS 92) a. Observation b. Pneumonectomy c. Thoracotomy d. Intercostal drainage 34. A patient presents with sudden onset of breath lessness after subclavian vein cannulation. On examination, breath sounds are absent while the chest is hyper-resonant on percussion on one side. Most likely cause is:  (All India 2012) a. Iatrogenic pnumothorax b. Subclavian vein air embolus c. Malposition of cannula d. Cardiac arrhythmia 35. While inserting a central venous catheter, a patient develops respiratory distress. The most likely cause is:   (Bihar PG 2014, All India 2002) a. Hemothorax b. Pneumothorax c. Pleural effusion d. Hypovolemia 36. All of the following are characteristic features of primary spontaneous pneumothorax except: (AIIMS 92) a. Male gender b. Old age c. Tall stature d. History smoking 37. Spontaneous pneumothorax is commonly seen in:  (PGI June 2002) a. Smokers b. Young females c. Old age d. Short statured men



38. Which of the following statements about pneumothorax is true? (AIIMS Dec 94) a. Breath sounds are increased b. Percussion note is decreased c. Always needs chest tube insertion d. Often needs chest tube insertion 39. In left sided massive pneumothorax, ECG shows all except: a. Left axis deviation (AIIMS 94) b. Absent R wave c. Peaked P wave d. Precordial T wave inversion



40. Intrapleural pressure greater than atmospheric pressure is diagnostic of:  (MHPGMCET 2001) a. Valvular pneumothorax b. Closed pneumothorax c. Open pneumothorax d. All

41. How will you check the functioning of the ICD tube?  (AIIMS November 2017) a. By observing for continuous air bubbles coming out of the underwater drain b. By observing the movement of air water column in the tube during respiration c. By taking X-ray chest repeatedly d. By auscultation

TENSION PNEUMOTHORAX











42. A 30-year-old female comes acute breathlessness, neck vein distention, and absent breath sounds and mediastinal shift. Which of the following should be done immediately? a. HRCT is the investigation of choice (PGI June 2008) b. ABG analysis should be done c. CXR d. Large bore needle puncture of pleura 43. Tension pneumothorax due to fracture rib is treated by: a. Strapping (DPG 2011, PGI 96) b. Tube drainage c. IPPV d. Internal fixation with open reduction 44. Treatment of choice for tension pneumothorax is: a. Immediate IC tube drainage (SGPGI 2005) b. Continuous aspiration by needle c. Intermittent aspiration by needle d. Thoracotomy with repair of leakage 45. True regarding management of traumatic pneumothorax is a. Immediate ICD tube insertion (SGPGI 2005) b. CT-scan should be done to confirm pulmonary leak c. Intermittent needle aspiration d. Sealed 46. A patient after road traffic accident presented with tension pneumothorax. What is the first line of management? a. Insert wide bore needle in 2nd intercostal space b. Immediate chest X-ray (AIIMS Nov 2013) c. CT scan d. Emergency thoracotomy 47. Condition which builds within hemithorax resulting in collapsed lung, flattened diaphragm, contralateral mediastinal shift and compromised venous return to right side of heart is known as: (MCI Sept 2007) a. Open pneumothorax b. Flail chest c. Massive pulmonary hemorrhage d. Tension pneumothorax

Thorax and Lung 48. Lung injury with bad prognosis is:



49. What is the 1st thing to be done to a patient with tension pneumothorax?  (UPPG 96) a. Insertion of wide bore needle in the inter costal space b. Water seal drainage c. Leave the patient at rest for air to be absorbed d. None



50. Tensions pneumothorax results in:  (ICS 98) a. Alkalosis b. Increased cardiac output c. Decreased venous return d. All of the above



51. What is the emergent management of tension pneumothorax? (AIIMS November 2014) a. Chest X-ray b. Emergency room thoracotomy in unstable patients c. Insert needle in 2nd intercostal space d. Tube thoracostomy in 5th intercostal space



53. This patient came with acute dyspnea. Which of the following statements is TRUE? (APPG 2016) a. It is due to old tuberculosis and fibrosis of left lung b. X-ray shows Westermark’s sign on right side c. A needle should be inserted into the chest wall in the second right intercostal space as an emergency d. He needs emergency bronchoscopy to remove possible FB in left side

HEMOTHORAX

54. Decision regarding surgery in a case of hemothorax due to blunt trauma chest should be based on: (All India 2008) a. Chest symptoms b. Hemodynamic status c. Nature of chest tube output d. X-ray finding



56. About hemothorax:  (PGI Dec 2002) a. Seen in choriocarcinoma b. Supine posture is better than erect posture c. Needle aspiration may be needed for diagnosis d. Thoracotomy is always done



57. Excessive bleeding during hemothorax is caused usually by:  (AIIMS June 94) a. Vena cava b. Internal mammary artery c. Heart d. Major artery



58. Which of the following vessel is injured in hemothorax patient? Recent Question 2017) a. Pulmonary artery b. Pulmonary vein c. Bronchial artery d. Intercostal arteries

LUNG ABSCESS

59. A 80-year-old male presented with lung abscess in left upper zone. Best treatment modality is: (UPPG 2008) a. Antibiotics according to organisms b. Surgical drainage c. Tube thoracostomy d. Wait and Watch

60. Management of a lung abscess refractory to prolonged antibiotic treatment includes all of the following except: a. Lobectomy (COMEDK 2004) b. Open drainage c. Tube drainage d. Intracavitary antibiotic instillation (PGI Dec 99)



61. Complication of empyema are all except: a. Empyema necessitans b. Bronchopleural fistula c. Osteomyelitis d. Pneumonia



62. Commonest cause of lung abscess: a. Aspiration b. Hematogenous spread from distant site c. Direct contact d. Lymphatic spread



63. Least common site of lung abscess is: (PGI June 99) a. Left upper lobe b. Left lower lobe c. Right upper lobe d. Right lower lobe



64. Most common cause of amoebic lung abscess is: (AIIMS Nov 94) a. Direct extension from liver b. Hematogenous spread c. Lymphatic spread d. By inhalation

(AIIMS Nov 96)

65. Failure of adequate drainage in an empyema with a bronchopleural fistula is indicated by: a. Drainage less than 100 cc per day b. Hemorrhagic drainage less than 100 cc per day c. The development of haemoptysis d. Continued productive cough with purulent material e. All of the above

66. Most common cause of cold abscess of chest wall is: (TN 95) a. Pott’s spine b. TB abscesses of chest wall c. TB of ribs d. Intercostal lymphadenitis

Cardiothoracic Vascular Surgery



52. A person met with road traffic accident and came to casualty with contusion on anterior chest wall with Pulse rate-90/ minute, BP-120/80 mm Hg, respiratory rate-16/minute. Normal heart sounds are heard but breath sounds were decreased on the left side and trachea was deviated towards right. Which of the following is the first line management?  (AIIMS May 2017) a. Needle thoracostomy b. Pericardiocentesis c. Chest tube insertion and drainage d. Immediate exploratory thoracotomy

55. The ideal treatment for hemothorax of blood loss greater than 500 ml/hour:  (PGI June 99) a. Wait and watch b. Needle aspiration c. Intercostal tube d. Open thoracotomy with ligation of vessel

Section 5

a. Open pneumothorax c. Tension pneumothorax

(DNB 2009) b. Closed pneumothorax d. All have same prognosis

791

792

Surgery Essence

Section 5

Cardiothoracic Vascular Surgery





67. Empyema necessitans is defined as so when: (UPSC 2002) a. Plural empyema is under pressure b. Pleural empyema has ruptured into bronchus c. Pleural empyema has ruptured into the pericardium d. Pleural empyema is showing extension to the subcutaneous tissue 68. Empyema can be caused by the following parasites except:  (MHSSMCET 2008) a. E. granulosus b. Entamoeba coli c. Paragonimus westermani d. Strongyloides stercoralis

PLEURAL COLLECTIONS

69. True about chylothorax: (DPG 2007) a. Left side more common b. Clear fluid c. Immediate thoracotomy should be done d. TOC is excision and ligation of thoracic duct



70. Chyluria is caused by all except: (MCI Sept 2009) a. Pregnancy b. Childbirth c. Filariasis d. Bile duct stones

SEQUESTRATION OF LUNGS 71. Intralobar sequestration of lungs takes its blood supply from: (Recent Question 2014, AIIMS Nov 94) a. Internal mammary artery b. Descending abdominal aorta c. Pulmonary artery d. None of the above

TRACHEOBRONCHIAL FOREIGN BODY 79. In erect posture, commonest site of foreign body in bronchus: (AIIMS June 99) a. Right posterior basal b. Right anterior basal c. Lateral basal d. Medial basal 80. Foreign body aspiration in supine position causes which of the following parts of the lung commonly to be affected? a. Apical left lobe (Recent Question 2014, AIIMS June 2002) b. Apical lobe of right lung c. Apical part of the lower lobe d. Posterobasal segment of left lung

VATS

82. In Video assisted thoracoscooic surgery for better vision, the space in the operative field is created by:  (AIIMS June 2002) a. Self retaining retractor b. CO2 insufflations c. Collapse of ipsilateral lung d. Rib spacing



83. VATS refers to: a. Vacuum assisted thoracic surgery b. Video assisted thoracoscopic surgery c. Video assisted transplant surgery d. None of the above

72. Lung sequestration occurs most commonly seen in which lobe? (AIIMS June 93) a. Apical b. Left posterior basal c. Left porterosuperior d. Right lateral basal

73. All of the following statements regarding bronchial cysts are true except: (MAHE 2005) a. Seen in mediastinum b. 50-70% occur in lung c. Are commonly infected d. Multilocular



74. True about bronchogenic cyst:  (Punjab 2007) a. Most of them are located at base of lung b. They arise from anomalous development of foregut c. They usually communicate with lung d. They are lined by pseudostratified epithelium

81. A foreign body completely obstructing the right main bronchus causes: (PGI June 99) a. Decreased ventilation perfusion ratio b. Increased ventilation in left lung c. Perfusion doubles in right lung d. Increased VP ratio in right lung

(Orissa 2011)

THORACOTOMY 84. The following are indications for performing thoracotomy after blunt injury of the chest, except: (UPSC 2008) a. 1000 ml drainage after placing an intercoastal tube b. Continuous bleeding through intercoastal tube of more than 200 ml/hour for three or more hours c. Cardiac tamponade d. Rib fracture

85. Thoracotomy is indicated in all the following except: a. Penetrating chest injuries (MHPGMCET 2003) b. Rapidly accumulating hemothorax c. Massive air leak d. Pulmonary contusion



76. Extralobar bronchogenic cysts may communicate with the following except: a. Esophagus b. Stomach c. Bronchus d. None of the above

86. Surgical indication in the treatment of hemoptysis: a. Profuse uncontrolled bleeding (PGI Dec 2006) b. Bronchiectasis c. Bronchial adenoma d. Bronchial fistula



77. Sequestrated lung is supplied most commonly by: (Kerala 98) a. Bronchial arteries b. Descending aorta c. Subclavian artery d. Intercostal arteries

87. Muscle not cut in posterolateral thoracotomy is:  (PGI Dec 98) a. Serratus anterior b. Latissimus dorsi c. Rhomboides major d. Pectoralis major



88. Which is not an indication of thoracotomy? (AIIMS Nov 98) a. Massive pneumothorax b. Pulmonary contusion c. Bleeding more than 200 ml/hour in thoracotomy tube d. Esophageal rupture

75. Intralobar sequestration of lung is commonest in the: a. Apical segment of upper lobe (Recent Question 2016) b. Medial segment of middle lobe c. Lateral basal segment of lower lobe d. Posterior basal segment of lower lobe





78. Diagnosis of lung sequestration by: (JIPMER 2000) a. CT b. Angiography c. MRI d. X-ray

Thorax and Lung 89. During emergency thoracotomy, the incision is made > 1 cm lateral to sternal margin to preserve: (All India 2012) a. Intercoastal artery b. Superior epigastric artery c. Internal mammary artery d. Intercostal vein



90. Emergency thoracotomy is indicated following chest trauma in the following conditions except: (APPG 2016) a. Scapular and sternal fractures b. Esophageal perforation c. Penetrating injury with cardiac tamponade d. Massive hemothorax

BENIGN LUNG TUMORS



91. The most common benign tumor of the lung is:  (COMEDK 2008) a. Hamartoma b. Alveolar adenoma c. Teratoma d. Fibroma 92. Image of CT chest is given below. Which of the following statement is incorrect about this condition?  (Recent Question 2016) a. Most common benign tumor of lung b. More common in females c. Usually peripheral d. Grow slowly

99. In a patient of mesothelioma, one often finds: a. Hypoglycemia b. An association with asbestosis c. Hemorrhagic pleural effusion d. Clubbing of fingers e. All of the above 100. Which of the following is true about Mesothelioma? a. Pleural effusion is exudative (Recent Question 2017, 2016) b. Butchart staging is used c. Manganese exposure is a predisposing factor d. Cough and dyspnoea are common late features

SQUAMOUS CELL CARCINOMA 101. Cavity formation in bronchogenic carcinoma occurs in:  (Recent Question 2016) a. Oat cell carcinoma b. Squamous cell carcinoma c. Adenocarcinoma d. Bronchoalveolar

ADENOCARCINOMA

93. Most common symptom of bronchial adenoma is: a. Chest pain b. Cough (All India 96) c. Recurrent hemoptysis d. Weight loss



94. True about bronchial adenoma: (All India 98) a. 10-15% of all lung tumour b. Mostly malignant c. Recurrent hemoptysis d. Peripherally located



95. The following is true about bronchial carcinoids: a. Highly radiosensitive (JIPMER 2011) b. Metastasis common c. Carcinoid syndrome does not manifest d. Commonly arises from terminal bronchioles



96. Blood stained sputum may be the only symptom in:  (Kerala 90) a. Bronchiectasis b. Carcinoma bronchus c. Adenoma bronchus d. Pulmonary T.b.

MESOTHELIOMA

97. All are true regarding mesothelioma except: a. Bilaterally symmetrical (AIIMS May 2011) b. Associated with asbestos exposure c. Histopathology shows biphasic pattern d. Occurs in late middle age



98. Pleural mesothelioma is associated with: (PGI Dec 2005) a. Asbestosis b. Berylliosis c. Silicosis d. Baggasosis

104. What is true regarding adenocarcinoma lung? (AIIMS Dec 94) a. Causes 50% of lung cancers b. Unlikely histological variant in young patients c. Associated with subcutaneous angiomyolipoma d. Peripheral location 105. Commonest type of lung carcinoma in nonsmokers is:  (Kerala PG 2015, AIIMS Dec 94) a. Squamous cell carcinoma b. Adenocarcinoma c. Alveolar cell carcinoma d. Small cell carcinoma 106. Lung to lung metastasis is seen in: a. Adenocarcinoma of lung b. Squamous cell carcinoma c. Small cell carcinoma d. Neuroendocrine tumor of lung

SMALL CELL CARCINOMA 107. Which of the following statements about small cell carcinoma is true? (All India 2009) a. Bone metastasis is uncommon b. Peripheral in location c. Chemosensitive tumor d. Paraneoplastic syndrome with PTH is common 108. Poorest prognosis in lung cancer is associated with: (COMEDK 2005) a. Small cell carcinoma b. Adenocarcinoma c. Squamous cell carcinoma d. Adenosquamous cancer

Cardiothoracic Vascular Surgery

102. True statement about adenocarcinoma lung are: a. Common in females (PGI June 2005) b. Not associated with smoking c. Central cavitation is a characteristic feature d. Peripheral involvement is common e. Upper lobe involvement is common 103. A patient presented with 1cm coin lesion over right upper lobe of lung on X-ray not suggestive of metastasis. FNAC revealed adenocarcinoma, no lymphadenopathy. Treatment is: a. Surgery  (JIPMER 2010) b. Surgery + chemotherapy c. Surgery + Radiotherapy d. Surgery + chemoradio therapy



Section 5



793

Section 5

Cardiothoracic Vascular Surgery

794

Surgery Essence 109. Marker of small cell cancer of lung is: (DNB 2011) a. Synaptobrevin b. Chromogranin c. Cytokeratin d. Vimentin 110. All are elaborated by small cell carcinoma lung, except: a. ADH b. ACTH (PGI June 2000) c. 5-HT d. Noradrenaline 111. The lung tumour responding best to radiotherapy:  (UPPG 96) a. Small cell anaplastic b. Squamous cell carcinoma c. Adenocarcinoma d. All respond equally well 112. In small cell carcinoma of the lung, one of the following is not seen:  (Kerala 97) a. Hypercalcemia b. Hyponatremia c. Watery diarrhea d. Hypokalemia 113. All of the following statements about small cell carcinomas are true, except: (PGI June 2006) a. Commonest malignancy of lung b. Associated with paraneoplastic syndrome c. Cause SVC obstruction d. Chemosensitive e. Commonly metastasizes to brain 114. Which of the following statements about small cell carcinoma is true? (All India 2009) a. Bone metastasis is uncommon b. Peripheral in location c. Chemosensitive tumor d. Paraneoplastic syndrome with increase PTH is common 115. In a chronic smoker, a highly malignant aggressive and metastatic lung carcinoma is:  (AIIMS May 2001) a. Squamous cell carcinoma b. Small cell carcinoma c. Adenocarcinoma d. Large cell carcinoma 116. Following hormonal levels are increased in small cell carcinoma of lung except: (All India 97) a. ACTH b. Growth hormone c. ANF d. AVP 117. Carcinoma lung responding best to chemotherapy: a. Squamous cell carcinoma b. Oat cell type c. Adenocarcinoma d. All respond equally 118. Clubbing is least common in: (AIIMS Dec 97) a. Squamous cell carcinoma b. Adenocarcinoma c. Small cell carcinoma of lung d. Mesothelioma 119. Most cases of paraneoplastic syndrome are associated with which type of lung carcinoma:  (JIPMER 2013) a. Small cell carcinoma b. Bronchogenic carcinoma c. Bronchoalveolar carcinoma d. Adenocarcinoma

CARCINOMA LUNG 120. Which of the following statements about lung carcinoma is true? (All India 2010) a. Squamous cell variant accounts for 70% of all lung cancers b. Oat cell variant typically present with cavitation c. Oat cell variant is typically associated with hilar adeno­pathy d. Adenocarcinoma variant is typically central in location

121. All of the following statements about non-small cell carcinoma of lung (NSCCL) are true except: (All India 2012) a. Contralateral mediastinal nodes are contraindication to surgical resection b. Single agent chemotherapy is preferred for patients >70 years with advanced disease c. Squamous cell carcinoma is the most common NSCCL among Asian population d. Geftinib is most effective for female smokers with adenocarcinoma histology 122. Ramesh, 40-year-male patient presenting with polyuria, pain abdomen, nausea, vomiting, altered sensorium was found to have bronchogenic carcinoma. The electrolyte abnormality seen in him would be: (AIIMS May 2002) a. Hypokalemia b. Hyperkalemia c. Hypocalcaemia d. Hypercalcemia 123. A 60-year-old male presented to the emergency with breathlessness, facial swelling and dilated veins on the chest wall. The most common cause is: (All India 2003) a. Thymoma b. Lung cancer c. Hodgkin’s lymphoma d. Superior vena caval obstruction 124. Which of the following tumor is most commonly associated with superior vena cava syndrome? (All India 2011) a. Lymphoma b. Small cell carcinoma c. Non small cell carcinoma d. Metastasis 125. In a 56-year-old chronic smoker there was a mass in bronchus which is successfully resected. This mass is most likely to be positive for: (AIIMS Nov 2009) a. Cytokeratin b. Vimentin c. Epithelial membrane cadherin d. Leukocyte 126. A 60-year-old male presents to the clinic with complaints of abdominal pain, constipation and coin pain radiating to the groin of over a week duration. He has 25-pack year smoking history and is currently being evaluated for a hicar mass picked UP on chest radiography. As part of this laboratory work-up, the serving calcium was found to be elevated. The elevated serum calcium level is most likely linked to: a. Paratharoid adenoma (COMEDK 2014) b. Parathoroid hyperplasia c. Vitamin D intoxication d. Secretion of PTH-related peptic 127. Which of the following has no infectious etiology? a. Nasopharyngeal carcinoma (AIIMS Nov 2009) b. Hepatocellular carcinoma c. Non-small cell lung carcinoma d. Gastric carcinoma 128. Most common site of metastasis in lung carcinoma: a. Liver b. Adrenal (AIIMS May 2007) c. Bone d. Brain 129. True about lung carcinoma: (PGI June 2009) a. Adenocarcinoma most common b. Squamous cell carcinoma is most common c. Originates for type-II pneumocytes d. Oat cell (Neuroendocrine cells) e. Squamous cell carcinoma peripherally located

Thorax and Lung

131. Superior sulcus tumor of the lungs characteristically present with: (JIPMER 2011) a. Horner syndrome b. Breathlessness c. Hemoptysis d. Pancoast syndrome 132. Superior vena cava syndrome is caused most commonly by:  (MCI Sept 2009, AIIMS Nov 95) a. Adenocarcinoma b. Squamous cell carcinoma c. Small cell carcinoma d. Large cell carcinoma 133. A 60-year-old male was diagnosed as carcinoma right lung. On CECT chest there was tumour of 5 × 5 cm in upper lobe and another 2 × 2 cm size tumour nodule in middle lobe. The primary modality of treatment is: (DPG 2010, All India 2004) a. Radiotherapy b. Chemotherapy c. Surgery d. Supportive treatment

135. Hoarseness secondary to bronchogenic carcinoma is usually due to extension of the tumor into: (UPSC 2002) a. Vocal cord b. Superior laryngeal nerve c. Left recurrent laryngeal nerve d. Right vagus nerve 136. All seen in pancoast syndrome except: a. Brachial plexus involvement b. Dyspnoea c. Clubbing d. Myasthenia gravis

(HPU 2005)

137. Most common site of metastasis of carcinoma bronchi: a. Liver + Bones b. Prostate (HPU 2005) c. Kidney d. Breast 138. Clinical manifestations of bronchogenic carcinoma include of the following except: (ICS 2005) a. Hoarseness of voice due to involvement of left recurrent laryngeal nerve b. Horner’s syndrome c. Diaphragmatic palsy due to infiltration of phrenic nerve d. Gastroparesis due to vagal involvement 139. The site of temporal bone metastasis is most commonly seen with: (UPPG 2010) a. Carcinoma breast b. Carcinoma bronchus c. Carcinoma kidney d. Carcinoma prostate 140. Carcinoma responding maximally to radiotherapy is: (MCI Sept 2006) a. Squamous cell carcinoma b. Adenocarcinoma c. Small cell carcinoma d. Large cell carcinoma 141. Most common type of carcinoma lung is: (AIIMS May 93) a. Small cell carcinoma b. Adenocarcinoma c. Squamous cell carcinoma d. Large cell carcinoma

143. In pancoast tumor, following is seen except: (PGI June 98) a. Horner’s syndrome b. Rib erosion c. Hemoptysis d. Pain in shoulder and arm 144. Pancoast’s syndrome is due to: (Recent Question 2017) a. C4-5 invasion b. C8-T1 invasion c. Lower trunk involvement d. C5-6 invasion 145. In case of CA lung, which among the following will be contraindication for surgical resection? (AIIMS Nov 2000) a. Malignant pleural effusion b. Hilar lymphadenopathy c. Consolidation of one lobe d. Involvement of visceral pleura 146. A 50-year-old-smoker male presents with pain along the left arm and ptosis. His chest radiograph shows a soft tissue opacity at the left lung apex with destruction of adjacent ribs. The picture is suggestive: (AIIMS Nov 2003) a. Adenocarcinoma lung b. Bronchial carcinoid c. Pancoast tumour d. Bronchoalveolar carcinoma 147. The first step when doing a penumononectomy for cancer of the bronchus is to: (UPPG 97) a. Ligate the pulmonary vein b. Ligate pulmonary artery c. Divide the bronchus d. Perform lymph node clearance 148. A 60 years old man presents with non productive cough and haemoptysis for 4 weeks; He has grade III clubbing, and a lesion in the apical lobe on X-ray most likely diagnosis here is: (All India 2001, AIIMS June 2000) a. Small cell carcinoma b. Non-small cell carcinoma c. Fungal infection d. Tuberculosis 149. A patient presents with secondaries to the adrenals. The most common site of primary is: (WBPG 2012, All India 2000) a. Lung b. Kidney c. Breast d. Stomach 150. Most common symptom of lung carcinoma: a. Cough b. Dyspnea c. Weight loss d. Chest pain

(AIIMS 90)

151. The commonest intrabronchial cause of hemoptysis is:  (AIIMS May 95) a. Carcinoma lung b. Adenoma lung c. Emphysema d. Bronchiectasis 152. A 60-year-old chronic smoker presents with complaints of hemoptysis. Her chest X-ray appears to be normal. What is the next best investigation? (All India 2001) a. Bronchoscopy b. High resolution CT c. Sputum cytology d. Pulmonary function test 153. A 60-year-old man is suspected of having bronchogenic carcinoma. TB has been ruled out in this patient. What should be the next investigation?  (All India 2001) a. CT guided FNAC b. Bronchoscopy and biopsy c. Sputum cytology d. X-ray chest

Cardiothoracic Vascular Surgery

134. A 65-year-old miner has lost 7 kg weight within two months, has presented with cough, and blood streaked sputum. He was treated for pulmonary tuberculosis 10 years ago. He also has drooping of his left eyelid for one month. On physical examination, there is ptosis of the left eye and papillary miosis. Chest X-ray revealed round opacification in the left upper apical lobe. What is the most probable diagnosis?  (AIIMS May 2006) a. Secondary tuberculosis b. Adenocarcinoma c. Squamous cell carcinoma d. Asbestosis

142. All of the following are true regarding oat cell carcinoma of lung, except: (AIIMS June 99) a. Variant of large cell anaplastic carcinoma b. Chemotherapy is effective c. Paraneoplastic syndrome may be present d. Causes SIADH

Section 5

130. Histological variants of bronchoalveolar carcinoma includes:  (PGI Nov 2011) a. Clara cells b. Adenosquamous c. Mucin secreting cells d. Type II pneumocytes e. Neuroendocrine cell

795

Cardiothoracic Vascular Surgery

796

Surgery Essence 154. A patient presents with a cavitatory lesion in right upper lobe of lung. The best investigation is: (All India 2000) a. Bronchoscopy, lavage and brushing b. CT scan c. X-ray d. FNAC

PULMONARY EMBOLISM 155. All of the following conditions may predispose to pulmonary embolism except: (All India 2003) a. Protein S deficiency b. Malignancy c. Obesity d. Progesterone therapy 156. A 23-year-old boy, a badminton player, sustained injury of left ankle. He was immobilized for 3 months, the cast was removed and patient was able to walk normally. Later he complained of pain and swelling in the left calf, left ankle and foot. His mother massaged him for 30 minutes. After a while he developed acute onset of breathlessness and was brought to emergency and died. Most likely cause of death is: (AIIMS May 2017) a. Pulmonary thromboembolism b. Congestive cardiac failure c. Massive stroke d. Hypovolemic shock 157. A patient had a femur fracture for which internal fixation was done. Two days later, the patient developed sudden onset shortness of breath with low-grade fever. What is the likely cause? (AIIMS May 2017) a. Pneumothorax b. Fat embolism c. Pleural effusion d. Congestive heart failure 158. A patient with history of trauma, presented with bilateral femoral fracture, respiratory distress & red urine. For evaluation of patient all the following are included in major criteria of Gurd’s criteria except: (PGI May 2018) a. Unexplained decrease in platelets b. Tachycardia c. Petechiae d. CNS depression e. Pulmonary edema 159. In acute pulmonary embolism, the most frequent ECG finding is: (AIIMS May 2006) a. S1Q3T3 pattern c. Sinus tachycardia

b. ‘P’ pulmonale d. Right axis deviation

Section 5

160. Most common symptom in pulmonary embolism: (MCI Sept 2007) a. Dyspnea b. Pleuritic chest pain c. Cyanosis d. Hemoptysis

163. Early and reliable indication of air embolisms during anesthesia can be obtained by continuous monitoring of:  (COMEDK 2008) a. ECG b. Blood pressure c. End-tidal CO2 d. Oxygen saturation

164. A mill-wheel type of murmur during laparoscopy suggests: a. Tension pneumothorax  (COMEDK 2009) b. Intra-abdominal bleeding c. Gas embolism d. Pre-existing valvular disease 165. Radiographic features of pulmonary embolism include all except: (COMEDK 2011) a. Westermarck’s sign b. The Fleischner sign c. Hampton’s hump d. Virchow sign 166. CT angiographic finding of acute pulmonary thromboembolism includes: (PGI Nov 2011) a. Filling defect of main pulmonary artery b. Pleural fibrosis c. Lobar and segmental oligemia d. Pleural effusion e. Peripheral, wedge-shaped consolidations

167. Patient admitted with fracture shaft of femur in a few days developed respiratory distress, ↓SPO2 and petechial rashes. Diagnosis: (JIPMER 2011) a. Fat embolism b. Pulmonary embolism c. Hemolytic anemia d. Crush syndrome 168. Investigation of choice in pulmonary embolism: a. Ventilation perfusion scan (JIPMER 2010) b. MRI c. CECT d. X-ray 169. A patient with fracture pelvis is admitted in ICU after surgery. Postoperatively he develops sudden dyspnea and chest. The likely cause is: (AIIMS Nov 99) a. Pulmonary thromboembolism b. Shock c. Respiratory infection d. ARDS 170. All are true about pulmonary embolism, except:  (AIIMS May 94) a. Chest pain is the most common symptom b. Most commonly presents within 2 weeks c. More is the survival time, more is the chance of recovery d. Arises from leg veins 171. The most common source of pulmonary embolism is: a. Amniotic fluid embolism (AIIMS May 95) b. Calf vein thrombi c. Large veins of leg d. Cardiothoracic surgery

161. Ventilation perfusion imaging is most useful for the diagnosis of: (COMEDK 2004) a. Pulmonary thromboembolism b. Asthma c. Interstitial lung disease d. Hypersensitivity pneumonitis

172. A 55-year-old man who has been on bed rest for the past 10 days complains of breathlessness and chest investigation should be: (All India 2004, 2003) a. Lung ventilation-perfusion scan b. Pulmonary arteriography c. Pulmonary venous angiography d. Echocardigraphy

162. Gold standard to diagnose pulmonary embolism is: a. Chest X-ray (MHCET 2016) b. Pulmonary angiography c. Ventilation purfusion scintiscan d. CT chest

173. The most definitive method of diagnosing pulmonary embolism is: (AIIMS Nov 2005) a. Pulmonary arteriography b. Radioistope perfusion pulmonary scintigraphy c. EKG d. Venography

Thorax and Lung

175. In pulmonary embolism, fibrinolytic therapy is responsible for: (PGI Dec 97) a. Risk of hemorrhage b. Prognosis good c. Massive emboli d. All of the above 176. IVC filter is used in following except: a. Massive emboli b. Negligible size of emboli c. Repeated emboli d. None

(PGI Dec 97)

177. D-Dimer is the most sensitive diagnostic test for: a. Pulmonary embolism (DPG 2011) b. Acute pulmonary edema c. Cardiac tamponade d. Acute myocardial infarction

179. The commonest site of lodgment of pulmonary embolus is in the territory of: (UPSC 95) a. Right lower lobe b. Right upper lobe c. Left Lower lobe d. Left upper lobe 180. A young male presented with dyspnea, bleeding and petchial hemorrhage in the chest after 2 days following fracture shaft of the femur right side. Most likely cause is: (UPPG 2008) a. Air embolism b. Fat embolism c. Pulmonary thromboembolism d. Amniotic fluid embolism

THORACIC INJURY 181. True regarding presentation(s) of thoracic duct injury:  (PGI June 2009) a. Electrolyte imbalance b. Lymphopenia c. Dehydration d. Lymphedema e. Chylothorax may be present 182. True about chest trauma: (PGI June 2008) a. ECG done in all cases associated with sternal fracture b. Under water seal drainage if associated with pneumo­ thorax c. X-ray chest investigation of choice d. Urgent surgery needed in all cases 183. Interstitial emphysema may be found in the following conditions: (Kerala 98) a. Chest injury b. Tracheostomy c. Surgical wound d. All

184. Most common abnormality associated with ARDS: a. Hypoxemia (JIPMER 2011) b. Hypercapnea c. Diffuse alveolar damage d. Bilateral alveolar infiltrates 185. The ideal tidal volume in a patient ventilated for ARDS is:  (COMEDK 2010) a. 6 mL/kg b. 10 mL/kg c. 14 mL/kg d. 20 mL/kg 186. Adult respiratory distress syndrome is defined by all except: a. PCWP >18 mm Hg (COMEDK 2005) b. PaO2 /FiO2 1L, Normal DLCO (DNB 2006) b. FEV1 > 1L, Decreased DLCO c. FEV1 > 2L, Normal DLCO d. FEV1 > 2L, Decreased DLCO 219. Which of the following is false about hemoptysis?  (AIIMS Nov 2013) a. Massive hemoptysis is bleeding >600 ml in 24 hours b. In 90% cases, bleeding from bronchial arteries c. CT chest is the first investigation done d. In an unstable patient, rigid bronchoscopy is done to identify the lesion 220. Hamman’s sign is seen in: (APPG 2016) a. Pneumomediastinum b. Diaphragmatic paralysis c. Empyema thoracis d. Subphrenic abscess

221. A 44 years old male underwent a VATS thymectomy for myasthenia graves. During surgery, the pleura was accidentally injured. The surgeon decided to put a drain in the pleural cavity. Which of these statements is correct about the timing of removal of intercostal chest tube?  (AIIMS November 2016) a. After partial expansion of lungs and 1500 ml (blunt injury)Q • Ongoing tube thoracostomy drainage of >200 ml/hr for 3 consecutive hoursQ in non-coagulopathic patients • Caked hemothoraxQ despite of placement of two chest tubes • Tracheo-bronchial injuryQ

806

Surgery Essence BENIGN LUNG TUMORS

Cardiothoracic Vascular Surgery

91. Ans. a. Hamartoma (Ref: Schwartz 10/e p622, 9/e p526; Bailey 27/e p934) 92. Ans. b. More common in females (Ref: Schwartz 10/e p622; Bailey 27/e p934) Popcorn calcification is diagnostic of pulmonary hamartoma, which is more common in males.

93. Ans. c. Recurrent hemoptysis (Ref: Harrison 18/e p753)



95. Ans. c. Carcinoid syndrome does not manifest (Ref: Robbins 9/e p719; Harrison 20/e p602, 19/e p563)



96. Ans. c. Adenoma bronchus

94. Ans. c. Recurrent hemoptysis

MESOTHELIOMA

97. Ans. a. Bilaterally symmetrical (Ref: Sabiston 20/e p1607; Schwartz 10/e p688; Bailey 27/e p146, 922)



98. Ans. a. Asbestosis

99. Ans. e. All of the above

100. Ans. b. Butchart staging is used

SQUAMOUS CELL CARCINOMA 101. Ans. b. Squamous cell carcinoma

ADENOCARCINOMA 102. Ans. a. Common in females, d. Peripheral involvement is common (Ref: Harrison 20/e p538, 19/e p511; Sabiston 20/e p1583; Schwartz 10/e p679,1018,1607; Bailey 27/e p925) 103. Ans. a. Surgery (Ref: Harrison 20/e p547, 19/e p516; Sabiston 20/e p1590) 104. Ans. d. Peripheral location

105. Ans. b. Adenocarcinoma

106. Ans. a. Adenocarcinoma of lung

SMALL CELL CARCINOMA 107. Ans. c. Chemosensitive tumor (Ref: Harrison 20/e p538, 19/e p522) Small cell carcinomas are highly chemosensitive with an overall 90% regression rate with chemotherapy. Property

Small cell carcinoma

Non small cell carcinoma

Location

• Central locationQ

• Peripheral locationQ

Metastasis

• Highly metastatic lesion with widespread metastasis at time of diagnosis. • Common site of metastasis include brain, bone, liver and adrenalsQ

• Less metastatic than small cell carcinoma

Paraneoplastic syndrome

•  AVP (Vasopression)Q • ACTHQ • CalcitoninQ •  ANFQ • Gastrin Releasing peptide

• PTH-rpQ

Response to chemotherapy

• Superior responseQ • Overall regression rate 90%Q • Rate complete regression in 30%

• Inferior response • Objective shrinkage in 30-50% • Complete response: uncommon

108. Ans. a. Small cell carcinoma

109. Ans. b. Chromagranin

110. Ans. d. Noradrenaline

Section 5

111. Ans. a. Small cell anaplastic    112. Ans. a. Hypercalcemia, c. Watery diarrhea 113. Ans. a. Commonest malignancy of lung    114. Ans. c. Chemosensitive tumor    115. Ans. b. Small cell carcinoma 116. Ans. b. Growth hormone

117. Ans. b. Oat cell type

118. Ans. c. Small cell carcinoma of lung 119. Ans. a. Small cell carcinoma

CARCINOMA LUNG 120. Ans. c. Oat cell variant is typically associated with hilar adenopathy (Ref: Sabiston 20/e p1583; Schwartz 10/e p623-645; Bailey 27/e p925; Harrison 20/e p538, 19/e p511) • Small cell carcinoma or oat cell variant is associated with massive hilar or mediastinal lymphadenopathy, mediastinal invasion and perihilar mass

Thorax and Lung

807

121. Ans. d. Geftinib is most effective for female smokers with adenocarcinoma histology (Ref: Harrison 19/e p521; Davidson 21/e p703)

Contraindication to Surgical Resection in NSCCL (Severe CID FM) • Severe or unstable cardiac or other medical conditionsQ • Contralateral mediastinal nodes (N3)Q • Invasion of central mediastinal structures including heart, great vessels, trachea and esophagus (T4)Q

• Distant metastasis (M1)Q • FEV1 70 years).

122. Ans. d. Hypercalcemia         123. Ans. d. Superior vena caval obstruction (Ref: Harrison 20/e p541, 19/e p1787) 124. Ans. b. Small cell carcinoma      125. Ans. a. Cytokeratin (Ref: Fishman Pulmonary Disease 4/e p848) • Cytokeratin positivity is seen in almost all NSCCL (Non small cell carcinoma lung) due to their epithelial origin. • Chromagranin and synaptophysin positivity is seen in small cell carcinoma. 126. Ans. d. Secretion of PTH-related peptic   127. Ans. c. Non-small cell lung carcinoma 128. Ans. b. Adrenal               129. Ans. a. Adenocarcinoma most common, d. Oat cell (Neuroendocrine cells) 130. Ans. a. Clara cells, c. Mucin secreting cells, d. Type II pneumocytes (Ref: Robbins 9/e p716-717, 8/e p725) 131. Ans. d. Pancoast syndrome (Ref: Harrison 19/e p519; Sabiston 20/e p583; Schwartz 10/e p623, 641-642; Bailey 27/e p926) 132. Ans. c. Small cell carcinoma Best treatment is induction chemoradiotherapy followed by surgery. 134. Ans. c. Squamous cell carcinoma Squamous Cell Carcinoma • MC in smokers, MC type in IndiaQ • MC variety associated with hypercalcemiaQ (produces PTH-rp) • Pancoast tumor is histologically SCCQ

• Central in distribution and prone to undergo central necrosis and cavitationQ • Associated with best prognosisQ

135. Ans. c. Left recurrent laryngeal nerve 136. Ans. d. Myasthenia gravis 137. Ans. a. Liver + Bones 138. Ans. d. Gastroparesis due to vagal involvement 139. Ans. b. Carcinoma bronchus (Ref: www.ncbi.nlm.nih.gov v.63(Suppl 1); Jul 2011)

Temporal Bone Metastasis • Metastatic tumors to the temporal bone are uncommon • Usually seeded by the hematogenous route • MC metastatic lesion in the temporal bone: CA BreastQ • Lung, prostate and renal carcinomas are all well documented for their metastatic potential to the temporal boneQ. 140. Ans. c. Small cell carcinoma

141. Ans. b. Adenocarcinoma

Adenocarcinoma • MC histological type, MC in non-smokers, young patients, femalesQ • Located peripherally with slow growth and propensity to metastasize to opposite lungQ

• Metastasize more frequently to CNS • Most cells contain mucinQ • Noguchi classification is used for adenocarcinomaQ

142. Ans. a. Variant of large cell anaplastic carcinoma 143. Ans. c. Hemoptysis 144. Ans. b. C8-T1 invasion (Ref: Sabiston 20/e p355; Schwartz 10/e p623) 145. Ans. a. Malignant pleural effusion 147. Ans. b. Ligate pulmonary artery (Ref: Bailey 27/e p932-933)

146. Ans. c. Pancoast tumour

Cardiothoracic Vascular Surgery

133. Ans. c. Surgery (Ref: Sabiston 20/e p1591)

808

Surgery Essence

Cardiothoracic Vascular Surgery

Pneumonectomy • • • •

Pneumonectomy is anatomically more straightforward than lobectomy (in carcinoma bronchus): The pulmonary artery is first dissected, divided and suturedQ. The pulmonary veins are then isolated, divided and sutured. The main bronchus is divided so that no blind stump remains. The technique of stump closure is important if a bronchopleural fistula is to be avoided. The tissues are carefully handled and the stump is usually stapled.



148. Ans. b. Non-small cell carcinoma

149. Ans. a. Lung

150. Ans. a. Cough



151. Ans. a. Carcinoma lung

152. Ans. a. Bronchoscopy

153. Ans. b. Bronchoscopy and biopsy



154. Ans. a. Bronchoscopy, lavage and brushing (Ref: Bailey 27/e p923) Uses of Bronchoscopy Diagnostic

Confirmation of disease: • Carcinoma of the bronchusQ • Inflammatory and Infective process

Investigative

• Tissue biopsyQ

Preoperative assessment

• Before lung resectionQ • Before esophageal resection • Persistent hemoptysisQ

Therapeutic

• Removal of secretions • Removal of foreign bodiesQ • Stent placement, endobronchial resection

PULMONARY EMBOLISM 155. Ans. d. Progesterone therapy (Ref: Harrison 20/e p1910, 19/e p1631; Sabiston 20/e p294-296; Schwartz 10/e p924-925; Bailey 27/e p987) • Estrogen, not the progesterone therapy predisposes to thrombosis and pulmonary embolism. 156. Ans. a. Pulmonary thromboembolism (Ref: Harrisons 20/e p1910, 19/e p1632) 157. Ans. b. Fat embolism (Ref: Harrison 20/e p1910, 19/e p1632; Robbins 9/e p128; Apley’s 9/e p681; Rockwood 6/e p553) 158. Ans. a. Unexplained decrease in platelets, b. Tachycardia (Ref: Textbook of Orthopedics (2018)/p237) Gurd’s Criteria for Diagnosing Fat Embolism Major Criteria

Minor Criteria

• Petechial rash • Respiratory symptoms (Tachypnea, dyspnea, bilateral inspiratory crepitations, hemoptysis, bilateral diffuse patchy shadowing on chest X-ray) • Neurological sign (confusion, drowsiness, coma) 159. Ans. c. Sinus tachycardia

• • • • •

Tachycardia (>120/min) Pyrexia (>34.90C) Retinal changes (fat or petechiae) Jaundice Renal changes (anuria or oliguria)

Laboratory Features • Thrombocytopenia (>50% decrease on admission value) • Sudden decrease in hemoglobin level >20% of admission value • Raised ESR • Fat macroglobulinemia

160. Ans. a. Dyspnea        161. Ans. a. Pulmonary thromboembolism

162. Ans. b. Pulmonary angiography     163. Ans. c. End-tidal CO2

Section 5

Capnography • It is the continuous measurement of end tidal CO2 and its waveforms. • It works on the principle that infrared light is absorbed by CO2. • Useful in diagnosing pulmonary embolism by air, fat or thrombus (sudden fall in End-tidal CO2 occurs). It may become zero if the embolus is large enough to block pulmonary circulationQ. 164. Ans. c. Gas embolism 165. Ans. d. Virchow sign 166. Ans. a. Filling defect of main pulmonary artery, c. Lobar and segmental oligemia, d. Pleural effusion, e. Peripheral, wedge-shaped consolidations (Ref: Harrison 20/e p1913-1914, 19/e p1632-1633; Danhert Radiology 5/e p51; Bhadury radiology 2/e p32) 167. Ans. a. Fat embolism (Ref: Apley’s 8/e p535-536, Rockwood 6/e p553) 168. Ans. c. CECT 169. Ans. a. Pulmonary thromboembolism

Thorax and Lung

809

170. Ans. a. Chest pain is the most common symptom 171. Ans. c. Large veins of leg 173. Ans. a. Pulmonary arteriography

Section 5

172. Ans. a. Lung ventilation-perfusion scan

174. Ans. a. Thrombolytic therapy (Ref: Harrison 19/e p1634; Sabiston 20/e p296; Schwartz 10/e p924-925; Bailey 27/e p990)

Treatment of Pulmonary Embolus • Most pulmonary emboli can be treated by anticoagulation and observation • Severe right heart strain and shortness of breath indicates the need for fibrinolytic treatmentQ. • Thrombolysis (or embolectomy) is treatment of choice in massive PE or high risk (hypotension ± Right ventricular dysfunction)Q Prophylaxis Against Pulmonary Embolism • In patients who are considered at high risk of embolism or when anticoagulants are contraindicated, a vena cava filter may be inserted to prevent the onward passage of any emboli. • Filters can also be placed in patients who continue to have pulmonary emboli despite adequate anticoagulationQ. • Greenfield filter: Most commonly usedQ 175. Ans. a. Risk of hemorrhage 176. Ans. b. Negligible size of emboli 177. Ans. a. Pulmonary embolism 178. Ans. c. Dyspnea, pain, hemoptysis 179. Ans. d. Left upper lobe 180. Ans. b. Fat embolism

THORACIC INJURY 181. Ans. a. Electrolyte imbalance, b. Lymphopenia, c. Dehydration, e. Chylothorax may be present (Ref: Sabiston 20/e p428; Schwartz 10/e p177-179, 200-203; Bailey 27/e p756)

Chest Trauma • Sternal fracture also constitute a marker for serious associated injuries, including myocardial contusion, myocardial rupture, esophageal perforation, airway injuries and thoracic aortic rupture. • In blunt cardiac injuries, ECG is the first diagnostic testQ. • Routine investigation in the emergency department of injury to the chest is based on clinical examination, supplemented by chest radiography. • In the unstable patient, chest radiography is the investigation of first choiceQ, provided that it does not interfere with resuscitation. 183. Ans. d. All

ADULT RESPIRATORY DISTRESS SYNDROME 184. Ans. a. Hypoxemia (Ref: Harrison 20/e p2031, 19/e p1736; Sabiston 20/e p1599) 185. Ans. a. 6 mL/Kg (Ref: Harrison 20/e p2033, 19/e p1736)

Management of ARDS • A large-scale, randomized controlled trial sponsored by the National Institutes of Health and conducted by the ARDS Network compared low VT (6 mL/kgQ predicted body weight) ventilation to conventional VT (12 mL/kg predicted body weight) ventilation. • Mortality was significantly lower in the low VT patients (31%) compared to the conventional VT patients (40%). • This improvement in survival represents the most substantial benefit in ARDS mortalityQ demonstrated for any therapeutic intervention in ARDS to date. 186. Ans. a. PCWP >18 mm Hg (Ref: Harrison 19/e p1736, 18/e p2205) Diagnostic Criteria PaO2/FiO2 Onset Chest X-ray

Acute Lung Injury (ALI) • ≤300 mm HgQ • AcuteQ • Bilateral alveolar or interstitial infiltratesQ

ARDS • ≤200 mm HgQ • AcuteQ • Bilateral alveolar or interstitial infiltratesQ Contd…

Cardiothoracic Vascular Surgery

182. Ans. a. ECG done in all cases associated with sternal fracture, b. Under water seal drainage if associated with pneumothorax, c. X-ray chest investigation of choice (Ref: Bailey 27/e p366, 938)

810

Surgery Essence

Cardiothoracic Vascular Surgery

Contd… Absence of Left Atrial Hypertension

• PCWP ≤18 mm HgQ • No clinical evidence of increased left atrial pressureQ

• PCWP ≤18 mm HgQ • No clinical evidence of increased left atrial pressureQ

187. Ans. a. Diffuse alveolar damage 188. Ans. d. PaO2/FiO2 Long saphenous veinQ 218. Ans. c. FEV1 >2L, Normal DLCO (Ref: Bailey 27/e p916-918)

Cardiothoracic Vascular Surgery

• If generalized it is almost never considered for surgical resectionQ.

812

Surgery Essence

Section 5

Cardiothoracic Vascular Surgery

Pulmonary Function Test • • • •

Pulmonary function studies are routinely performed when any resection greater than a wedge resection will be performed. Of all the measurements available, the two most valuable are FEV1 and DLCO. For every 10% decline in patients DLCO, the risk of any pulmonary complications (As estimated by odd ratio) increased by 42%. Patients with FEV1 >2.0 L can tolerate pneumonectomy, and those with an FEV1 of >1.5 L can tolerate lobectomy.

219. Ans. c. CT chest is the first investigation done (Ref: Harrison 20/e p517, 19/e p246; Sabiston 20/e p1598) • “Massive hemoptysis may be defined as greater than 500 to 600 mL of blood loss from the lungs in 24 hours.”-Sabiston 19/e p1589 • “Massive hemoptysis is one of the most dreaded of all respiratory emergencies and can have a variety of underlying causes. In 90% of cases, the source of massive hemoptysis is the bronchial circulation.” • “For most patients, the next step in evaluation of hemoptysis should be a standard chest radiograph. If a source of bleeding is not identified on plain film, a CT of the chest should be obtained.” • “If all of these studies are unrevealing, bronchoscopy should be considered. Rigid bronchoscopy with an 8.5-mm or larger bronchoscope is needed.” • “As most large-volume hemoptysis arises from an airway lesion, it is ideal if the site of the bleeding can be identified either by chest imaging or bronchoscopy (more commonly rigid than flexible).” 220. Ans. a. Pneumomediastinum 221. Ans. c. After complete expansion of lungs and 15%Q TBSA • Regimen of fluid resuscitation follows the fluid loss, which is at its maximum in first 8 hours and slows such that by 2-36 hours the patient can be maintained on her/his normal daily requirement. Fluids used in resuscitation

• Ringer Lactate is most commonly usedQ. • Some centers use human albumin, FFP or hypertonic salineQ

Section 6

Fluid Resuscitation

• If oral resuscitation is to be commenced, it is important that the water given is not salt free. Hyponatremia and water intoxication can be fatalQ.

• In children, maintenance fluid must be given, usually dextrose-saline.Q • Simplest and most widely used formula: Parkland formulaQ • • • •

Hypertonic saline has been effective in treating burn shockQ. It produces hyperosmolality and hypernatremia. This reduces the shift of intracellular water to the extracellular space. Advantage includes less tissue edema and a resultant decrease in escharotomies and intubationQ.

• Protein should be given after the first 12 hours of burn. • The commonest colloid based formula is Muir and Barclay formulaQ. Monitoring of Resuscitation • The key to monitoring is urine outputQ. • Urine output should be 0.5-1.0 mL/kg/hourQ (i.e. 30-60 ml per hourQ). • Other measures for monitoring: −− Acid base balance and Hematocrit −− In cardiac dysfunction: Transesophageal USG and Central lineQ Venous Access for Infusion • In adults: Ideal sites are veins in hand, antecubital fossa or neck. • Saphenous vein cut down is useful in patient with difficult access and is used in preference to central venous cannulation. • CVP line is used for CVP monitoring, helps in estimating fluid overloadQ.

Parkland Brooke

Q

Q

GalvestonQ (pediatric)

Crystalloid Volume

Colloid Volume

Free water

4 mL/kg per % TBSA burn

None

None

1.5 mL/kg per % TBSA burn

0.5 mL/kg per % TBSA burn

2.0 L

5000 mL/m2 burned area + 1500 mL/m2 total area

None

None

Topical Antimicrobials used in Burn

Silver sulphadiazine cream (1%)

Silver nitrate solution (0.5%)

Mafenide acetate cream (5%)

Silver sulphadiazine and cerium nitrate

• This gives broad-spectrum prophylaxisQ against bacterial colonization • Particularly effective against Pseudomonas and MRSAQ • MC used

• Highly effective as a prophylaxis against Pseudomonas colonizationQ • Not as active as silver sulphadiazine cream against some of the Gram -ve aerobes. • It needs to be changed or the wounds resoaked every 2-4 hoursQ. • Produces black staining of all the furniture surrounding the patient.

• Popular in the USA • Painful to applyQ • Associated with metabolic acidosisQ.

• Useful for full-thickness burns. • Induces a hard effect on the burned skinQ • In elderly patients, to reduce cell-mediated immuno­ suppressionQ • Cerium nitrate forms a sterile eschar and boost cell mediated immunity

Plastic Surgery

Resuscitation Formulas Formula

820

Surgery Essence • • • • •

MC cause of death at the site of burn: Asphyxia MC cause of death in burns: SepticemiaQ MC cause of early death in burns: Hypovolemic shockQ MC cause of late death in burns: SepticemiaQ MC organism responsible for sepsis in burns: PseudomonasQ

POST BURN NECK CONTRACTURE

Plastic Surgery

Post Burn Neck Contracture • Cervical contractures are major problems in burns involving the chest, neck & faceQ. • Severe neck flexion contracture in the acute phase often require early reconstructionQ to aid in airway management • Neck contractures should usually be dealt with prior to facial burn reconstructionQ as the extrinsic contractile forces from the neck can cause facial deformities and can adversely affect the maturation of scars on the face. • When split-thickness skin grafting is unsuccessful because of recurrent contracture or does not provide a satisfactory aesthetic result, local flap reconstruction of anterior neck is an excellent techniqueQ. • Perioral Deformities: Microstomia, macrostomia, irreversible damage to dentition & loss of jawline definitionQ • Anterior neck contractures in the acute period are best prevented by aggressive splinting and incisional releases & graftingQ when indicated. • Deep second & third degree burns do not heal in timely fashion without autograftingQ • Escharotomies is done in case of deep second & third degree buns wounds to decrease constriction of escharQ

LOW-TENSION INJURIES Low-tension injuries • Low-tension or domestic appliance injuries do not have enough energy to cause destruction to significant amounts of subcutaneous tissuesQ when the current passes through the body. • Resistance is too great. • Entry & exit points, normally in the fingers, suffer small deep burns; these may cause underlying tendon and nerve damage, but there will be little damage between. • The alternating current creates a tetany within the muscles, and thus patients often describe how they were unable to release the device until the power was turned off. • A common finding in patients with electric burns is myoglobinuria manifested as highly concentrated and pigmented urine.

Section 6

• Main danger with these injuries is from the alternating current interfering with normal cardiac pacing. This can cause cardiac arrestQ. • The electricity itself does not usually cause significant underlying myocardial damage, so resuscitation, if successful, should be lasting.

Multiple Choice Questions BURNS: % BSA

1. According to “rule of nines”, burns involving perineum are: a. 1% b. 9% (MCI March 2009) c. 18% d. 27% 2. A five years old child presents to the emergency department with burns. The burn area corresponding to the size of his palm is equal to: (All India 2011) a. 1% BSA b. 5% BSA c. 10% BSA d. 20% 3. In a 6 years old child with burns involving the whole of head and trunk, estimated body surface area of burns is: a. 44% b. 52% (comedK 2008) c. 55% d. 58%







4. Rule of nine of estimate surface area of a burnt patient was introduced by: (Recent Question 2016) a. Mortix Kaposi b. Wallace c. Joseph Lister d. Thomas Barclay











5. Best method to assess burns in 5 years old child caused by boiling water: (AIIMS May 2013) a. Palm method b. Rule of 9 c. Lund and Browder chart d. Rule of one 6. A child has circumferential burn of both of thighs and buttocks, face and scalp with singeing of hairs. Calculate the percentage of burns: (JIPMER 2014, AIIMS May 2013) a. 24 b. 27 c. 37 d. 45 7. Head and face burn in infant is: (Recent Question 2014, 2013) a. 15% b. 18% c. 12% d. 32% 8. Percentage of burn in children is best assessed by: a. Rule of 9 b. Rule of palm = 1% c. Lund and Browder chart d. Wallace rule (DNB 2014) 9. Percentage of body surface area involved in the burns involving scalp and face in an adult using Berkow formula: (Recent Question 2017) a. 7% b. 8% c. 9% d. 10%

BURNS





10. A burn patient is referred when: (PGI June 2004) a. 10% superficial burn in child b. Scald in face c. 25% superficial burn in adult d. 25% deep burn in adult e. Burn in palm 11. In burns heat loss is by/due to: a. Dilatation of veins b. Shock c. Exposed area by evaporation d. None of the above 12. Metabolic derangements in severe burns are all except:  (PGI June 2000) a. ↑corticosteroid secretion b. Hyperglycemia c. ↑secretion of HCl d. Neutrophil dysfunction



13. Pus in burns form in: a. 2-3 days c. 2-3 weeks



14. Fever in burnt patient is caused by: a. Septicemia (PGI Nov 2009, June 2009) b. Due to hypermetabolism c. Decreased sweating d. Release of pyrogens from dead product e. Dehydration



15. True about thermal burn injury: (PGI June 2009) a. Outermost layer is zone of stasis b. Middle layer is zone of hyperemia c. Inner layer is zone of coagulation d. Hyperemia is due to vasodilatation e. Zone of stasis is associated with vascular damage



16. Undue restlessness in a patient during the immediate post burn period is often a manifestation of: (Karnataka 95) a. Hypoxia b. Hypovolemia c. Hyperkalemia d. Anxiety



17. All require hospitalization except: (DNB 2002, All India 91) a. 5% burns in children b. 10% scalds in children c. Electrocution d. 15% deep burns in adults



18. True about burns: (PGI 2000) a. Hyperglycemia is seen in early burns b. Child with burns should have damp dressing c. Chemical powder burns should be kept dry d. 3rd degree burns are painfull

b. 3-5 days d. 4 weeks

BURNS DEPTH 19. True regarding burns: (PGI Dec 2007) a. Only 2nd and 3rd degree is considered in the classification b. 2nd degree-Epidermis + papillary dermis c. Blisters-2nd degree d. Curling ulcer can occur e. Classified according to depth of invasion 20. In a patient with the burn wound extending into the superficial epidermis without involving the dermis would present all of the following except: (SGPGI 2005) a. Healing of the wound spontaneously without scar formation b. Anesthesia at the site of burn c. Blister formation d. Painful 21. A third degree circumferential burn in the arm and forearm region, which of the following is most important for monitoring? (UPPG 2004) a. Blood gases b. Carboxy-oxygen level c. Macroglobinuria cryoglobinuria d. Peripheral pulse and circulation 22. True statement regarding 2nd degree deep burn: (PGI Dec 2008) a. Blanch on pressure b. Erythema c. Dry white colour d. Painless e. Predispose to hypothermia

822

Surgery Essence

Plastic Surgery

23. Degree of burns in a patient with prominent vessels with decreased needle prick sensation and dryness: a. Superficial partial thickness burns b. Deep partial thickness burns  (MHSSMCET 2006) c. Electric burns d. Full thickness burns 24. Burn involving epidermis and full thickness of dermis: a. First degree burns (MHSSMCET 2008) b. Partial-thickness second degree burns c. Full-thickness second degree burns d. Third degree burns

25. True about burn is: (UPPG 2010) a. Full thickness burn feels, leathery, painless b. Electric burn are superficial c. IV fluid formula used Curreri and Brooke d. Skin grafting done after 48 hours e. 1 year of age in head and neck region covers 18%



26. Which of the following is not seen in 3rd degree burns?  (MCI March 2009) a. Loss of skin appendages b. No vesicles c. Red color d. Extremely painful 27. In second degree burns, re-epithelialisation occurs around: a. 1 week b. 2 weeks (MCI Sept 2009) c. 3 weeks d. 4 weeks







28. Which of the following is false regarding deep 2nd degree burns? (MCI Sept 2009) a. Heal by scar deposition b. Painless c. Damage to deeper dermis d. Less blanching

Section 6



31. Blisters are seen in which type of burns? (DNB 2009) a. Superficial first degree b. Superficial second degree c. Third degree d. Deep first degree



32. 2nd degree burns indicate involvement of:  (JIPMER 2013) a. Epidermis b. Dermis c. Subcutaneous tissue d. Deep fascia



33. False regarding deep second-degree burns: (MCI June 2018) a. Heal by scar deposition b. Painless c. Damage to deeper dermis d. Less blanching



34. Which layer is involved in blister formation in a superficial partial thickness burn? (AIIMS November 2017) a. Epidermis b. Dermis c. Papillary dermis d. Reticular dermis

TREATMENT OF BURNS







35. Parkland formula is:  (JIPMER 2010) a. Percentage of burns × weight (kg) × 4 = volume in ml b. Percentage of burns × weight (kg)/2 = 1 volume in ml c. Percentage of burns × weight (kg) × 9 = volume in ml d. 500 ml/m2 BSA + 1500 ml/m2 = volume in ml 36. In a 50 kg adult, how much of fluid should be given in first 8 hours in burns of 40%? (Recent Question 2018, MCI Nov 2017) a. 2 litres b. 4 litres c. 8 litres d. 6 litres

37. A woman was brought to the casualty 8 hours after sustaining burns on the abdomen, both the limbs and back. What will be the best formula to calculate amount of fluid to be replenished? (AIIMS May 2017) a. 2 mL/kg × %TBSA b. 4 mL/kg × %TBSA c. 8 mL/kg × %TBSA d. 4 mL/kg × % TBSA in first 8 hours followed by 2 mL/kg/ hour × % TBSA 38. A 1.5-years-old child was brought to emergency with history of burn by hot water on both hands and palms. The lesion was pink, oozing and painful to air and touch. Which of the following is the best management for this patient? a. Paraffin gauze and dressing  (AIIMS May 2017) b. Collagen dressing c. Excision and grafting d. Apply 1% silver sulfasalazine ointment and keep the wound open 39. During fluid resuscitation in a burns patient using Parkland’s formula, volume of fluid given in first 8 hours: a. 25% b. 50% (Recent Question 2017) c. 75% d. 100%



40. IV formula for burn is: (Recent Question 2015, UPPG 2009) a. Total % body surface area x weight x 4 = volume in ml b. Total % body surface area x weight x 5 = volume in ml c. Total % body surface area x weight x 6 = volume in ml d. Total % body surface area x weight x 7 = volume in ml



41. Which of the following formula for fluid administration in a patient with burns is not correct? (MHSSMCET 2008) a. Parkland: 4 ml kg/% TBSA burn of RL b. Brooks: 1.5 ml kg/% TBSA burn of RL + 0.5 ml kg% burn + 2000 ml D5W c. Shrine: 5000 ml m2 TBSA burn + 2000 ml m2 TBSA d. Evans: 8 ml kg/% TBSA burn of RL 42. Treatment of burns includes: (PGI June 2008) a. No bandage to head and neck b. Immediate application of ice cold water c. Superficial burns without blister-no need of dressing d. Escharotomy done for peripheral circumscribed lesions 43. What should be the ideal temperature of the cool water to be applied over burns? (MHPGMCET 2006) a. Ice cold b. 3-4°C c. 8-10°C d. 14-15°C 44. Safest strategy of treatment for a patient of inhalational burn injury who has presented within 4-5 hours:  (MCI Nov 2017, MHPGMCET 2007) a. Binasal catheter O2 inhalation b. O2 therapy with well-fitting face mask c. Elective cricothyroidotomy d. Elective endotracheal intubation

29. Superficial burns; true is/are: (PGI June 2001) a. Always requires skin grafting b. Dry and inelastic c. Blister formation d. Painless e. Can be healed within 7-10 days

30. Burns with vesiculation, destruction of the epidermis and upper dermis is: (PGI June 99) a. 1st degree b. 2nd degree c. 3rd degree d. 4th degree













45. In burns management, which of the following is the fluid of choice? (Recent Question 2014, DNB 2012, 2005) a. Dextrose 5% b. Normal saline c. Ringer lactate d. Isolyte-M

46. A 50 kg female has second degree deep burn involving 45% total body surface area (TBSA). Regarding her management which of the following statement (s) is/are true: (PGI Dec 2008) a. Give rapid normal saline infusion b. Half of the calculated fluid should be given in initial 8 hours c. 9 liters of Ringer’s lactate should be given in first 24 hours d. Urine output should be maintained at 25–30 ml/hour e. CVP line should be inserted

Burns

48. What is the most important aspect of management of burn injury in the first 24 hours? (UPSC 2007) a. Fluid resuscitation b. Dressing c. Escharotomy d. Antibiotics



59. True regarding opsite dressing is: (PGI May 2018) a. Wound can be seen b. Vapor permeable c. Impermeable to bacteria d. Water permeable e. Increased chances of maceration



60. In children with burns, maintenance IV fluid normally given is: (MHCET 2006) a. Ringer lactate b. 5% dextrose c. Normal saline d. Dextrose saline

COMPLICATIONS OF BURNS 61. Late deaths in burns is due to: (PGI Dec 99) a. Sepsis b. Hypovolemia c. Contractures d. Neurogenic



62. Most common cause of death due to burns in early period is: a. Sepsis b. Hypovolemic shock c. Both d. None (APPG 2008)



63. Most common carcinoma after burns is: (DPG 2008) a. Squamous cell carcinoma b. Adenocarcinoma c. Melanoma d. Mucoid carcinoma

51. Which of the following is true about burns? (PGI Dec 2005) a. 3rd generation cephalosporin is drug of choice b. S. aureus is most common infection of burn c. Toxic shock syndrome is most common in burns patients d. Pseudomonas is most common infection in dry wound e. Moist dressing is done



64. Most common cause of death in burns is: (Punjab 2008) a. Primary shock b. Secondary shock c. Hemorrhagic shock d. Septicemic shock





52. Which of the following is true about burn management? a. Intravenous access fluid is done and antibiotics is not given in children (PGI Dec 2005) b. Escharotomy should be done for peripheral circumscribed lesion c. Moist dressing is done d. Parkland formula is used with 8 ml/kg body weight e. Prognosis depend on the time of resuscitation of the patient



65. Burns shock is: (Punjab 2011) a. Hypovolemic b. Neurogenic c. Endotoxic d. Cardiogenic 66. Which of the following statement(s) is/are true about postburn neck contracture? (PGI June 2009) a. Occur because of conservative management of deep burn b. Treated by flaps c. Obliteration of cervicomental angle d. Dental abnormalities may be present e. Never develop in deep dermal burn



53. Exposure treatment is done for burns of the: a. Upper limb b. Lower limbs c. Thorax d. Abdomen e. Head and neck





54. Deep skin burn is treated with:  (AIIMS 91) a. Split thickness graft b. Full thickness graft c. Amniotic membrane d. Synthetic skin derivatives

49. In excessive burns, least useful is: (AIIMS June 94) a. Blood b. Dextran c. Ringer lactate d. Nasogastric intubation



50. True statement about burn resuscitation: (PGI Dec 2003) a. Colloid preferred in initial 24 hours b. Colloid preferred if burnt area is >15% of total BSA c. Half of the calculated fluid given in initial 8 hours d. Urine output should be maintained at 50-60 mL/hour e. Diuretics should be given to all patients of electric burn





55. The cold water treatment of burns has the disadvantage that it increase the chances of: a. Pain b. Exudation c. Infection d. None of the above



56. The best guide to adequate tissue perfusion in the fluid management of a patient with burns, is to ensure a minimum hourly urine output of: (Karnataka 2004) a. 10-30 ml b. 30-50 ml c. 50-70 ml d. 70-100 ml

57. Burns in which part of body are nursed without occlusive dressing? (DPG 2005) a. Hands b. Legs c. Head and Neck d. Chest

58. Which of the following is effected against Pseudomonas and is used in burns patients? (DNB 2009) a. Silver sulphadiazine b. Silver sulphazine c. Sulphamethoxazole d. Sulphadoxine

67. Death from burns in first 10 days is due to all except: a. Shock b. Infection (DNB 2005) c. Renal failure d. Respiratory distress

MISCELLANEOUS

68. Domestic low-voltage electric supply can cause all the following except: (MHPGMCET 2007) a. Contact wound b. Cardiac arrest c. Cardiac fibrillation d. Deep subcutaneous tissue damage

69. Main danger with low tension (Domestic) electric AC current: (MHPGMCET 2009) a. Renal injury (ARF) b. Cardiac arrest c. Muscle necrosis d. Paralysis

70. Operation theatre fire is most commonly due to: (DNB 2010) a. Argon beam coagulators b. Lasers c. Fibre optic illumination d. Electrosurgical equipment



71. Myoglobinuria is seen in which type of burn? (DNB 2012) a. Flame burn b. Scald burn c. Electric burn d. Contact burn

Plastic Surgery





Section 6



47. All of the following are true regarding fluid resuscitation in burn patients except: (MCI March 2008) a. Consider intravenous resuscitation in children with burns greater than 15% TBSA b. Oral fluids must contain salts c. Most preferred fluid is Ringer’s lactate d. Half of the calculated volume of fluid should be given in first 8 hours

823

Explanations BURNS: % BSA

1. Ans. a. 1% (Ref: Sabiston 20/e p507; Schwartz 10/e p227-236, 1820-1822; Bailey 27/e p621)



2. Ans. a. 1% BSA

3. Ans. a. 44% (Ref: Sabiston 20/e p508) Berkow Formula to Estimate Burn Size (%) Body Part 0-1 year 1-4 years Head 19Q 17Q Neck 2 2 Anterior trunk 13 13 Posterior trunk 13 13



5-9 years 13Q 2 13 13

5. Ans. c. Lund and Browder chart (Ref: Schwartz 19/e p199-200)

4. Ans. b. Wallace

“In children younger than 3 years old, the head accounts for a larger relative surface area and should be taken into account when estimating burn size. Diagrams such as the Lund and Browder chart give a more accurate accounting of the true burn size in children.”- Schwartz 10/e p199 “For children and infants, the Lund-Browder chart is used to assess the burned body surface area. Different percentages are used because the ratio of the combined surface area of the head and neck to the surface area of the limbs is typically larger in children than that of an adult.” http://en.wikipedia.org/wiki/Total_body_surface_area

6. Ans. c. 37 (Ref: Sabiston 20/e p507)



7. Ans. b. 18% (Ref: Sabiston 20/e p508)

A Child Has • Circumferential burn of both of thighs = 6.5 + 6.5 = 13 • Buttocks = 2.5 + 2.5 = 5 • Face and scalp with singeing of hairs = 17 Total burn = 13 + 5 + 17 = 35% Body Part Head Neck Anterior trunk Posterior trunk Right buttock

2.5 1 4

2.5 1 4

2.5 1 4

2.5 1 4

2.5 1 4

2.5 1 4

Left upper arm

4

4

4

4

4

4

Right lower arm

3

3

3

3

3

3

Left lower arm

3

3

3

3

3

3

Right hand

2.5

2.5

2.5

2.5

2.5

2.5

Left hand

2.5

2.5

2.5

2.5

2.5

2.5

Right thigh

5.5

6.5

8

8.5

9

9.5

Left thigh

5.5

6.5

8

8.5

9

9.5

Right leg

5

5

5.5

6

6.5

7

Left leg

5

5

5.5

6

6.5

7

Right foot

3.5

3.5

3.5

3.5

3.5

3.5

Left foot

3.5

3.5

3.5

3.5

3.5

3.5

Left buttock Genitalia Right upper arm



Berkow Diagram to Estimate Burn Size (%) Based on Area of Burn in an Isolated Body Part 0-1 yr 1-4 yr 5-9 yr 10-14 yr 15-18 yr ADULT 19 17 13 11 9 7 2 2 2 2 2 2 13 13 13 13 13 13 13 13 13 13 13 13 2.5 2.5 2.5 2.5 2.5 2.5

8. Ans. c. Lund and Browder chart

9. Ans. a. 7% (Ref: Sabiston 20/e p508)

Burns

825

BURNS 10. Ans. a. 10% superficial burn in child; b. Scald in face; c. 25% superficial burn in adult; d. 25% deep burn in adult; e. Burn in palm (Ref: Sabiston 19/e p521-522; Schwartz 10/e p227-236; Bailey 27/e p620)



11. Ans. c. Exposed area by evaporation (Ref: Sabiston 20/e p508; Bailey 26/e p386-387)



12. Ans. c. ↑secretion of HCl



14. Ans. a. Septicemia; b. Due to hypermetabolism (Ref: Total Burn Care by David N. Herndon 3/e p158; Sabiston 20/e p509; Bailey 27/e p627)



15. Ans. c. Inner layer is zone of coagulation; d. Hyperemia is due to vasodilatation; e. Zone of stasis is associated with vascular damage: (Ref: Sabiston 20/e p506; Schwartz 10/e p197-199; Bailey 26/e p386-387)



16. Ans. d. Anxiety



18. Ans. a. Hyperglycemia is seen in early burns

13. Ans. b. 3-5 days

17. Ans. a. 5% burns in children

Section 6



BURNS DEPTH

19. Ans. b. 2nd degree-Epidermis + papillary dermis; c. Blisters-2nd degree; d. Curling ulcer can occur; e. Classified according to depth of invasion (Ref: Sabiston 20/e p506-57; Schwartz 10/e p229; Bailey 26/e p389-390)



20. Ans. b. Anesthesia at the site of burn; c. Blister formation

21. Ans. d. Peripheral pulse and circulation (Ref: Sabiston 20e p515; Schwartz 10/e p234, 1820; Bailey 27/e p624)

Escharotomies • When deep second- and third-degree burn wounds encompass the circumference of an extremity, peripheral circulation to the limb can be compromisedQ. • Development of generalized edema beneath a non-yielding eschar impedes venous outflow and affects arterial inflow to the distal beds. • This can be recognized by numbness and tingling in the limb and increased pain in digitsQ. • Arterial flow can be assessed by determination of Doppler signals in the digital arteries and the palmar and plantar arches in affected extremities. • Capillary refill can also be assessed. • Extremities at risk are identified either on clinical examination or by measurement of tissue pressures >40 mm HgQ. • These extremities require escharotomies (release of the burn eschar by incising the lateral and medial aspects of the extremity)Q

23. Ans. b. Deep partial thickness burns

24. Ans. d. Third degree burns



25. Ans. a. Full thickness burn feels, leathery, painless; e. 1 year of age in head and neck region covers 18% (Ref: Bailey 27/e p622, 631; CSDT 11/e p/273) • Electric burns are both superficial and deep, depending upon the thickness involved. • Curreri, Sutherland and Davies formulas are feeding formulas of burn patientsQ.



26. Ans. d. Extremely painful

27. Ans. b. 2 weeks



28. Ans. d. Less blanching

29. Ans. c. Blister formation; e. Can be healed within 7-10 days



30. Ans. b. 2nd degree

31. Ans. b. Superficial second degree



32. Ans. b. Dermis

33. Ans. b. Painless



34. Ans. c. Papillary dermis (Ref: Sabiston 20/e p506; Schwartz 10/e p229; Bailey 27/e p622)

TREATMENT OF BURNS

35. Ans. a. Percentage of burns × weight (kg) × 4 = volume in ml (Ref: Sabiston 20/e p514; Schwartz 10/e p232; Bailey 27/e p624) • Half of fluid is given in first 8 hours and other half in next 16 hoursQ



36. Ans. b. 4 litres (Ref: Sabiston 20/e p514; Schwartz 10/e p230; Bailey 27/e p624)



37. Ans. b. 4 mL/kg × %TBSA (Ref: Sabiston 20/e p514; Schwartz 10/e p230; Bailey 27/e p624)



38. Ans. a. Paraffin gauze and dressing (Ref: Kirk General Surgical Operations 6/e p548)



39. Ans. b. 50% (Ref: Sabiston 20/e p514; Schwartz 10/e p230; Bailey 27/e p624)

Plastic Surgery

22. Ans. c. Dry white colour; d. Painless; e. Predispose to hypothermia

826

Surgery Essence

40. Ans. a. Total % body surface area x weight x 4 = volume in ml



41. Ans. c. Shrine: 5000 ml m2 TBSA burn + 2000 ml m2 TBSA; d. Evans: 8ml Kg/% TBSA burn of RL



42. Ans. a. No bandage to head and neck; c. Superficial burns without blister-no need of dressing; d. Escharotomy done for peripheral circumscribed lesions (Ref: Sabiston 20/e p513; Schwartz 10/e p200-204; Bailey 27/e p624)

Plastic Surgery

Management of Burn Wound

Exposure Method

Closed Method

• No dressings are applied over woundQ after application of the agent to the wound 2-3 times a day • Used for face and headQ • Disadvantages: −− Increased pain, heat lossQ −− Risk of cross-contaminationQ

• Occlusive dressing is applied over the agent and changed twice dailyQ • Generally closed method is preferredQ • Advantages: −− Less pain, less heat lossQ −− Less risk of crosscontamina­tionQ • Disadvantages: −− Potential increase in bacterial growthQ if dressing is not changed twice daily.



43. Ans. d. 14-15°C



44. Ans. d. Elective endotracheal intubation (Ref: Bailey 27/e p620)

Initial Management of the Burned Airway • Early elective intubation is safestQ • Delay can make intubation very difficult because of swellingQ • Be ready to perform an emergency cricothyroidotomy if intubation is delayedQ

45. Ans. c. Ringer lactate



46. Ans. b. Half of the calculated fluid should be given in initial 8 hours; c. 9 liters of Ringer’s lactate should be given in first 24 hours; e. CVP line should be inserted



47. Ans. a. Consider intravenous resuscitation in children with burns greater than 15% TBSA



48. Ans. a. Fluid resuscitation



49. Ans. a. Blood (Ref: Sabiston 20/e p514; Schwartz 10/e p230; Bailey 27/e p624)

• IV fluid resuscitation: In children with burn >10% TBSA and adult with burn >15% TBSA

Section 6

• RL is preferred agent for resuscitation for the initial 24 hoursQ. • Nasogastric intubation is done to decrease the risk of emesis and possible aspiration (as paralytic ileus develops in the patients of burn). • Dextran is a colloid and can be used after 24 hours however, albumin is the preferred and most widely used colloidQ.

50. Ans. c. Half of the calculated fluid given in initial 8 hours; d. Urine output should be maintained at 50-60 mL/hour



51. Ans. d. Pseudomonas is most common infection in dry wound (Ref: Sabiston 20/e p516; Schwartz 10/e p202; Bailey 27/e p625) • In burn management, topical antimicrobials are used. • Pseudomonas is the most common infectionQ in burn patients. It has replaced streptococci and staphylococci because of availability of good antibiotics. • Toxic shock syndrome is most commonly associated with tampon use in menstruating females. It is caused by Staphylococcus infection producing TSST-1. • Damp dressing should not be used.



52. Ans. b. Escharotomy should be done for peripheral circumscribed lesion; e. Prognosis depend on the time of resuscitation of the patient



53. Ans. e. Head and neck

54. Ans. a. Split thickness graft



55. Ans. c. Infection (Ref: CSDT 11/e p1272)

56. Ans. b. 30-50 ml



57. Ans. c. Head and Neck

58. Ans. a. Silver sulphadiazine



59. Ans. a. Wound can be seen, b. Vapor permeable, c. Impermeable to bacteria (Ref: Sabiston 20/e p517)

Burns

827

“OpSite Dressing: Provides a moisture barrier; inexpensive; decreased wound pain; use complicated by transudate and exudate requiring removal; no antimicrobial properties”- Sabiston 20/e p517 60. Ans. d. Dextrose saline

COMPLICATIONS OF BURNS

61. Ans. a. Sepsis (Ref: Sabiston 20/e p516; Schwartz 10/e p233; Bailey 26/e p394-395) • Following successful resuscitation, most acute morbidity and virtually all mortality in severely burned patients are related to infectionQ.

Section 6



• This is because thermal injury causes profound immunosuppression that is proportional to the TBSA of the burnQ. • Inspite of burn patients at significant risk to infection, prophylactic systemic antibiotics are not part of modern care, as they do not reduce septic complications and only lead to increased bacterial resistanceQ. • • • •

62. Ans. b. Hypovolemic shock



64. Ans. d. Septicemic shock

MC cause of death at the site of burn: AsphyxiaQ MC cause of death in burns: SepticemiaQ MC cause of early death in burns: Hypovolemic shockQ MC cause of late death in burns: SepticemiaQ 63. Ans. a. Squamous cell carcinoma

Carcinoma in Burns • Squamous cell carcinoma is MC carcinoma in burnsQ. • SCC commonly occurs in long standing (Marjolin’s ulcer), old scar or keloidQ. • Both Marjolin’s ulcer and keloid are complications that arise after burnsQ.

65. Ans. a. Hypovolemic (Ref: Sabiston 20e p514; Schwartz 10/e p204; Bailey 27/e p618-619) • Proper fluid management is critical to survivalQ in burn patient. • The hypovolemic shockQ in burn patient is special in the sense that total body water remains unchanged in a burn patient. • The thermal injury leads to a massive shift from the intravascular compartment to the extravascular compartment leading to edema formationQ. 66. Ans. a. Occur because of conservative management of deep burn; b. Treated by flaps; c. Obliteration of cervicomental angle: d. Dental abnormalities may be present (Ref: Total Burn Care by David N. Herndon 3/e p714-715; Sabiston 19/e p534-535; Schwartz 10/e p182; Bailey 27/e p628)



67. Ans. c. Renal failure

MISCELLANEOUS

68. Ans. d. Deep subcutaneous tissue damage (Ref: Bailey 27/e p631)



69. Ans. b. Cardiac arrest



70. Ans. d. Electrosurgical equipment (Ref: British Journal of Anesthesia, vol 50, Issue 7, Page 659-664)

Operation Theatre Fire • • • •

The two most common source of operation theatre fire is electrosurgical unit (ESU) and lasers. ECRI’s analysis of case reports show that the most common ignition sources are electrosurgical instruments (68%) and lasers (13%). Most common fire location is airway (35%), head or face (28%), and elsewhere on or inside the patient (38%). An oxygen-enriched atmosphere was a contribution factor in 74% of all cases.

71. Ans. c. Electric burn

Plastic Surgery



CHAPTER

31

Plastic Surgery and Skin Lesions

GRAFT TAKE Graft Take • Skin graft take occurs in three phases, imbibition, inosculation, & revascularization. Plasma Imbibition Inosculation Revascularization • Graft survives up to first 48 hoursQ • Donor and recipient capillaries are • After 5 daysQ, revascularization because of plasma imbibition aligned during inosculationQ occurs • Involves free absorption of nutrients • Inosculation completes by 4–5 • Graft demonstrates both into the graft daysQ arterial and venous outflowQ • During these initial few days the graft is most susceptible to deleterious factors such as infection, mechanical shear forces and hematoma or seromaQ.

SKIN GRAFT Partial Thickness (Thiersch) or Split Skin Graft • • • •

Consist of epidermis & variable portion of dermisQ Large size of graft can be takenQ Site: Thigh (MC)Q, upper arm, flexor aspect of forearm and abdominal wall Grafts are hairless and do not sweatQ (these structures are not transferred) • Skin graft must be applied to a well-vascularized recipient wound bed. It will not adhere to exposed bone, cartilage, or tendon devoid of periosteum, perichondrium, or peritenon, respectively, or devoid of its vascularized perimembranous envelope.

• MC causes of skin graft failure: Hematoma (or seroma), infection, & movement (shear). • Pie crusting: Stab incisions in the graft preemptively to create small outlets for fluid to drain from beneath the graft • Beta hemolytic Streptococci can destroy split skin grafts completely, presence of this organism is a contraindication to graftingQ.

• Graft immobilization is critical to the graft take and can be accomplished with bolster dressing, light compression wraps or a vacuum assisted closure device.

Skin Grafts Partial Thickness (Thiersch) Graft • It includes all epidermis & part of dermisQ. • Partial thickness grafts are thin, uptake of graft is easy (easy survival)Q. • Large grafts could be taken as the donor site is left with a part of dermis which will cause easy regeneration of epidermisQ. • Contract upto 40%, not useful for cosmetic surgeriesQ. • Donor site will heal wellQ without any contraction, and is reusable.

Full Thickness (Wolfes) Graft It includes all epidermis & dermisQ. Uptake is difficult because of thickness Less chances of survival Small grafts could be takenQ as the donor site does not have epidermal or dermal remnants to allow epithelialization • Very minimal contraction making it suitable for cosmetic surgeries on faceQ. • Donor site will have to be closed primarily or left open to granulate and contractQ. • • • •

Contraction of Graft • Primary : Occurs when the graft is harvested, depends upon amount of dermis present, more in full thickness graft • Secondary : Occurs after the surgery, more in partial thickness graft

Plastic Surgery and Skin Lesions

829

MESHED SKIN GRAFTS

• Split grafts may be meshed to expand the surface area that can be coveredQ. • This technique is particularly useful when a large area must be resurfaced, as in major burns. • Meshed grafts usually also have enhanced reliability of engraftment, because the fenestrations allow for egress of wound fluid and excellent contour matching of the wound bed by the graftQ. • Fenestrations in meshed grafts re-epithelialize by secondary intention from the surrounding graft skin. • Major drawbacks of meshed grafts are poor cosmetic appearance and high secondary contractionQ.

Section 6

Meshed Skin Grafts

• Meshing ratios used usually range from 1:1.5 to 1:6, with higher ratios associated with magnified drawbacks.

PEDICEL GRAFT OR FLAP Pedicel Graft or Flap • Flap: Partially or completely isolated segment of tissue with its own blood supplyQ Absolute Indications for Flaps • Exposed bone • Radiated vesselQ • BrainQ Q



• Open joint or non-biologicalQ implant materialsQ • Pressure sores at bony prominences

For critical and small areas such as an eyelid, a full thickness graft is selected, so that contraction of the grafted material is minimum. Type of Flaps on the basis of source of Vascular Supply Axial • Axial flap is based on a named blood vesselQ • Provide a reproducible and stable skin or skinmuscle (myocutaneous) flapQ. • Can be used to provide much needed length and bulk • Axial flap that remains attached to its proximal blood supply and transposed to a defect is known as a pedicled flapQ.

Free • Autogenous transplantationQ of vascularized tissues. • Complete detachment of the flap, with devascularization, from the donor siteQ • Revascularization of the flap with anastomoses to blood vessels in the recipient siteQ

MARJOLIN’S ULCER Marjolin’s Ulcer • Low grade SCC , which develops on a chronic benign ulcer or a long standing scar tissue. • Arises from the edgeQ of the ulcer Q

Marjolin’s ulcer may develop in • Post burn scarQ • Chronically discharging osteomyelitis sinusQ • Long standing venous ulcerQ Q • Post-radiation ulcer • Chronic ulcer due to trauma Characteristic Features • • • •

Slow growingQ as scar tissue is relatively avascular •  Painless as there no nerves in the scar tissueQ No secondary depositsQ in regional lymph node, as there are no lymphatic vessels in scar tissue If the ulcer invades the normal tissue, then only lymph node may be involved by lymphatic spread RadioresistantQ due to avascularity

Diagnosis • IOC for diagnosis: BiopsyQ Treatment • Wide local excision followed by flap coverQ

•  Radiotherapy is avoidedQ

Plastic Surgery

Random • Random flaps rely on the low perfusion pressures found in subdermal plexusQ to sustain the flap • Used to reconstruct relatively small, fullthickness defects that are not amenable to skin grafting.

830

Surgery Essence BOWEN’S DISEASE Bowen’s disease • • • •

This is an SCC in situQ, of which 3–11% progress to SCC. Etiological agents: Chronic solar damage, inorganic arsenic and HPV16Q This is rare, slow-growing intraepidermal SCC that often mimics a chronic dermatosisQ. It should now be considered as a form of AIN III (Anal intraepithelial neoplasia).

Clinical Features

Plastic Surgery

• It usually presents with pruritusQ and on examination looks like psoriasis or senile keratosis. • Presents as a slowly enlarging, erythematous, scaly patch or plaqueQ. Treatment • Topical therapy with 5-fluorouracil or imiquimod is an effective treatmentQ. • Alternatives: Surgical excision with a 4-mm margin or Mohs’ micrographic surgeryQ for larger or recurrent lesions.

BASAL CELL CARCINOMA (RODENT ULCER) Basal Cell Carcinoma (Rodent Ulcer) • • • •

Locally invasive carcinoma, arises from the basal layerQ of the epidermis MC type of skin cancerQ 90% of BCC are seen in the faceQ, above a line from the corner of mouth to lobule of ear. MC site: Nose >Inner canthusQ of the eye, also known as Tear cancerQ.

Types of BCC • Nodular: MC type of BCCQ, characterized by small slow growing pearly nodules, often with telangiectatic vessels on its surface. Central depression with umbilicationQ is a classic sign. • Pigmented: Mimic malignant melanoma • Cystic • Superficial Characteristic Features of BCC • Low grade malignancyQ • More common in fair and dry skinned people • Nuclear palisadingQ on histology

• Exposure to sunlightQ is an important etiological factor • Has been seen following prolonged administration of ArsenicQ

Spread • • • •

BCC usually spreads by local invasionQ, rarely metastasizes Rodent ulcer: It gradually destroys the tissues, it comes in contact with. Lymphatic spread is not seenQ (Regional lymph nodes are not enlarged) Blood spread is extremely rare.

Diagnosis • Diagnostic procedure for BCC is wedge biopsyQ. Treatment

Section 6

• Non-aggressive tumor on trunk or extremities: Excision or electrodissection and curettageQ • Large, aggressive, located at vital areas or recurrent: Moh’s micrographic surgeryQ

SQUAMOUS CELL CARCINOMA (EPITHELIOMA OR EPIDERMOID CARCINOMA) Squamous Cell Carcinoma (Epithelioma or Epidermoid Carcinoma) • It is a carcinoma of the cells of epidermis that usually migrate outwards to the surface. • Originate from prickle cell layerQ; Seen in > 40 years of age • • • •

MC skin cancer in darkly pigmented racesQ 2nd MC skin cancer in light skinned racesQ MC causative factor: SunlightQ MC site: Ears, cheeks, lower lip and back of handsQ

Plastic Surgery and Skin Lesions

831

Predisposing Factors for SCC Senile or actinic keratosis Chronic skin lesions (lupus vulgarisQ, cutaneous TB) Sunlight or irradiationQ Chronic irritation; HIV, HPV-16Q

• • • •

Contact with tars and hydrocarbonsQ Erythroplasia of QueyratQ ImmunosuppressionQ Psoralens, Arsenic exposureQ

Pathology • Microscopically mass of keratin is surrounded by normal looking squamous cells, presenting with characteristic prickle cell appearanceQ, which are arranged in concentric manner as seen in ‘onion skin’. This whole appearance is called “cell nest or epithelial pearlQ”. • MC type of SCC: Ulcerative typeQ

Section 6

• • • •

Q

Clinical Features • MC symptom: Nodule or ulcerQ • Edge of ulcer: Raised and everted with indurated base (pathognomonic)Q Diagnosis • Diagnosis is made by wedge biopsyQ (taken from edge of ulcer) Treatment • Small (< 1 cm) or non-invasive SCC: Excision with 1 cm marginQ • Large, aggressive, located at vital areas or recurrent: Moh’s micrographic surgeryQ

MOH’S MICROGRAPHIC SURGERY FOR SCC AND BCC Moh’s Micrographic Surgery for SCC and BCC • Mohs’ technique uses serial excision in small increments coupled with immediate microscopic analysis to ensure tumor removal, yet limit resection of aesthetically valuable tissueQ. • Advantage: All specimen margins are evaluatedQ. • Major benefit: Ability to remove a tumor with minimal sacrifice of uninvolved tissueQ. • Particular value in managing tumors of the eyelid, nose, or cheekQ • Indicated in large, aggressive tumors located at vital areas or recurrent tumorsQ • Major drawback: Procedure length (Total lesion excision may require multiple attempts at resection, and many procedures may be carried out over several days)

MALIGNANT MELANOMA Malignant Melanoma • Melanoma is neoplastic disorder produced by malignant transformation of normal melanocytesQ. • Site most commonly associated with melanocytic transformation in the skin, where melanocytes reside at the dermo-epidermal junction (Junctional melanocytes)Q. • MC site of MM in men: Back & trunkQ • MC site of MM in women: Lower extremityQ • Most susceptible individuals: Fair complexions, red or blonde hair, blue eyes and freckles and who tan poorly and sunburn easily. • MM is positive for S-100, HMB-45, vimentin but negative for cytokeratin-20Q. Risk factors for Malignant Melanoma • • • •

Xeroderma pigmentosumQ Actinic damage (UVR)Q Family history of melanomaQ Presence of dysplastic naevusQ

• Giant congenital melanocytic nevus • Increased number of ordinary melanocytic naevi • History of sunburnQ

Plastic Surgery

• Recurrence & metastases rates are comparable to those of wide local excisionQ.

832

Surgery Essence

Plastic Surgery

Types of Malignant Melanoma: (In order of decreasing frequency) Superficial spreading

• MC type of MMQ • MC site: Torso

Nodular

• Most malignantQ • MC site: Head, neck and trunk • Vertical growth phase only

Lentigo maligna

• Least malignantQ • MC site: Face

Acral lentiginous

• Least common, worst prognosisQ, • MC site: Sole, under great toe nail

Characteristic Features • • • •

Classic appearance of melanoma: ABCD (Asymmetry, Border irregularity, Color variation, Diameter >6 mm)Q MC route of metastasis: Through LymphaticsQ MC site of systemic metastasis: LiverQ Other common visceral sites of metastasis: Lung, brain, GIT (small intestine), bone, adrenal. • Microsatellites: Discrete tumor nests > 0.05 mm in diameter, separated from main body of tumor by normal dermal collagen or subcutaneous fatQ. • Microsatellites are associated with increased risk of regional LN metastasisQ.

Diagnosis • Confirmed by ‘full thickness excisional biopsy’Q • Incisional biopsy for large lesions and lesions in proximity to important structures (eye, nose, ear) Treatment • Treatment: Surgical excisionQ with sentinel LN biopsy (Margin: 1 cm for < 1 mm thickness, 2 cm for 1–4 mm thickness, 2–3 cm for > 4 mm thickness) • LN dissection if LN is palpable or positive on sentinel LN biopsy • MM is radioresistant tumor; Chemotherapy: IFN-alpha 2b Clark’s levelsQ (on the basis of depth of invasion): EPIRS I

• Melanoma restricting to Epidermis and appendagesQ

II

• Invading Papillary dermis without filling itQ

III

• Reach Interface of papillary and reticular dermisQ

IV

• Invading reticular dermisQ

V

• Invading subcutaneous tissueQ

• MM is sub-classified into 5 Clark levels, to indicate their depth of invasion and prognosisQ. • Breslow’s depth of invasion: Actual measurement of the deepest invasion from the granular layerQ.

Section 6

Breslow’s Thickness Stage I

• < 0.75 mmQ

Stage II

• 0.75–1.5 mmQ

Stage III

• 1.6–4.0 mmQ

Stage IV

• > 4.0 mmQ

Prognostic Factors (Depends most importantly on stagingQ) • Depth of invasion (most important prognostic factor)Q • UlcerationQ (presence of ulceration carries worst prognosis)

• Lymph node statusQ • Satellite lesionQ • Distant metastasisQ

Plastic Surgery and Skin Lesions

833

VASCULAR ANOMALIES

Port-wine Stain

Strawberry Angiomas

• A vascular malformation • Present at birthQ • Grows along with the childQ • Do not regressQ • Face involvement in areas supplied by 5th cranial nerveQ

Salmon Patch

Type of capillary hemangioma Baby is normal at birthQ Appears at the age of 1–3 weeksQ Grows with the child upto 1 year of age and then cease to growQ • By the age of 9 years, 90% demonstrate complete involutionQ • Emptying signQ is demonstrable • • • •

• Also known as Macular stain or stork biteQ • Present at birthQ • Seen over forehead in the midline and over the occiputQ • Disappears by the age 1 yearQ

Section 6

Vascular Anomalies

Hemangioma • Benign, abnormally dense collections of dilated small blood vessels • Occurs in the skin or internal organs • Hemangioma is a compressible swelling • Calcification is seen in congenital hemangioma and GLUT-1 is negativeQ Clinical Types of Hemangioma

According to Depth of Invasion • Capillary hemangioma: −− Best cosmetic results obtained by laser ablationQ • Cavernous hemangioma

According to Rate of Involution • Rapidly Involuting Congenital Hemangioma (RICH): −− Grow at a rapid rate for 4–6 months, then growth ceasesQ −− Spontaneous involution begin and completed by 5–7 years of ageQ • Non-Involuting Congenital Hemangioma (NICH): −− Mostly present at birthQ −− Undergo rapid growth during first 4–6 monthsQ −− Grow in proportion to the growth of the childQ

• Treatment of hemangiomas: Observation with reassuranceQ of parents that regression and involution will occur. • Local wound care, topical application of lidocaine for pain and laser cauterization may be beneficial treatment modalities. • Laser therapy has been effective in lightening affected skinQ.

EPIDERMOID CYST (SEBACEOUS OR EPIDERMAL CYST) Epidermoid Cyst (Sebaceous or Epidermal Cyst) • Epidermoid cyst results from proliferation of epidermal cells within a circumscribed space of dermis (which had got implanted within the dermisQ) • Sebaceous cyst is a misnomer as the cysts are not of sebaceous origin and the white creamy material filled within is not sebum, but is keratin (desquamated epithelial cellsQ) • Type of retention cyst (secretions are pent up in a gland owing to blockage of the duct) Pathology • Cyst wall consist of a layer of epidermis oriented with the basal layer superficial and more matured layers are deep. • Desquamated cells (keratin) collect in the centre and form creamy substance of the cyst. Clinical Features • Usually asymptomaticQ, unless get infected or inflamed and become painful • Firm, round, flesh colored to yellow or white subcutaneous nodules of variable size.

Plastic Surgery

Treatment

834

Surgery Essence • Central punctumQ may teether the cyst to the overlying epidermis, from which the white creamy material can be expressed. • Rarely malignanciesQ (BCC, SCC) can develop in epidermoid cyst. • No punctum in scrotal and scalp sebaceous cystQ. Treatment • Excision with the wall is treatment of choiceQ. • Infected cyst: Incision and drainage (After resolution of the abscess, cyst wall must be excised to prevent recurrence)Q

Plastic Surgery

LIPOMA Lipoma • MC subcutaneous neoplasm: LipomasQ •  Also known as universal tumorQ • MC site: TrunkQ (may appear anywhere) •  Can be encapsulated or diffuse • Other sites: Subcutaneous, Subfascial, subsynovial, intra-articular, intramuscular, subserous, submucous, CNS, intraglandular and reteroperitoneal Pathology • Lobulated tumor composed of normal fat cells Clinical Features • Typically soft and fleshy on palpation • May grow to a large size and become substantially deforming. • Bracket calcification is seen in lipoma of corpus callosumQ Treatment • Surgical excisionQ is required for tumor removal

ADIPOSIS DOLOROSA (DERCUM’S DISEASE) Adiposis Dolorosa (Dercum’s Disease) • A rare condition characterized by multiple, painful lipomasQ. • These lipomas mainly occur on the trunk, upper arms and upper legs. • Mostly occur between 35–50 years, more common in women Clinical Features • Multiple lipomas and neuropathic pain are the cardinal symptomsQ Treatment • Treatment is usually targeted towards pain reliefQ rather than lipoma removal.

RETROPERITONEAL LIPOMA

Section 6

Retroperitoneal Lipoma • These swellings sometimes reach an immense sizeQ. • Retroperitoneal lipoma is often malignant (liposarcoma) and may increase rapidly in sizeQ. • A retroperitoneal lipoma sometimes undergoes myxomatous degenerationQ • A complication that does not occur in a lipoma in any other part of the bodyQ Clinical Features • Swelling or indefinite abdominal pain • More common in women Diagnosis • Diagnosis is usually by ultrasound and CT scanning.

Plastic Surgery and Skin Lesions

835

HIDRADENITIS SUPPURATIVA • Hidradenitis suppurativa is a defect of the terminal follicular epitheliumQ • Follicular defect results in apocrine gland blockage, obstructed infection leads to abscess formation throughout affected axillaryQ, inguinalQ and perianal regionsQ. • Following spontaneous rupture of these localized collections, foul-smelling sinuses form and repeated infections create a wide area of inflamed, painful tissueQ. Treatment • Acute infections: Application of warm compresses, antibiotics and open drainageQ • Chronic Hidradenitis: Wide excision and skin graftingQ

Section 6

Hidradenitis Suppurativa

KERATOACANTHOMA Keratoacanthoma • Keratoacanthoma is self healing nodular lesion with central ulcerationQ • More common in men, usually found on the face of 50–70 years old. • Lesions can grow to 1–3 cm over 6 weeks and typically resolve spontaneously over the subsequent 6 monthsQ. Etiopathogenesis • Classically a cup-shaped growth that exhibits symmetry about its middle. • Central crater is filled with a plug of keratinQ. • Unclear etiology (may be caused by HPV infecting a hair follicle) • Associated with smoking and chemical carcinogen exposure. Treatment • Removal of central keratin plug may speed resolutionQ. • Excision is recommended for persistent lesionsQ • Excision scar is often better than that which remains after resolution. Cocks Peculiar Tumour Potts Puffy Tumour

Cylindroma

Infected or ulcerated sebaceous cyst of the scalpQ Resembles fungating epitheliomaQ Osteomyelitis of the Frontal bone of skullQ Associated with subperiosteal swelling & edemaQ Also known as Calcifying epithelioma of MalherbeQ Benign hair follicle derived tumorQ A malignant epithelial tumour also known as Turban tumorQ Known as cylindroma because of histological appearance

Plastic Surgery

Pilomatrixoma

• • • • • • • •

Multiple Choice Questions SKIN GRAFTING 1.

2.

3.

4.

5.

6.

7.

8.

‘Take in’ of split skin graft occurs when? (PGI Dec 2008) a. Tight dressing is applied b. Excessive discharge from wound c. β-hemolytic streptococcus infection is present d. Wound bed not vascularised Thiersch graft is which type of graft?  (JIPMER 2012, MHPGMCET 2008, 2001, DPG 2005) a. Partial thickness b. Full thickness c. Pedicle d. Patch Which of the following statements about mesh skin grafts is not correct? (UPSC 2006) a. They permit coverage of large areas b. They allow egress of fluid collections under the graft c. They contract to the same degree as a grafted sheet of skin d. They “take” satisfactorily on a granulating bed Split skin grafts in young children should be harvested from: (UPSC 2007) a. Buttocks b. Thigh c. Trunk d. Upper limb Which of the following is not a wound closure technique? (UPSC 2008) a. Partial thickness skin graft b. Composite graft c. Vascular graft d. Musculocutaneous graft Who said: “Skin is the best dressing”? (Karnataka 2004) a. Joseph Lister b. John Hunter c. James Paget d. Mc Neill Love Within 48 hours of transplantation, skin graft survives due to: (AIIMS Nov 2000, AIIMS Nov 99) a. Amount of saline in graft b. Plasma imbibition c. New vessels growing from the donor tissue d. Connection between donor and recipient capillaries Ideal graft for leg injury with 10 × 10 cm exposed bone: (AIIMS Nov 99) a. Amniotic membrane graft b. Pedicle graft c. Full thickness graft d. Split thickness skin graft

9.

Wolfe Graft is: (APPG 2015) a. Thin split thickness graft b. Thick split thickness skin graft c. Medium thickness split thickness skin graft d. Full thickness skin graft

10.

Skin grafting is absolutely contraindicated in which skin infection? (AIIMS June 97) a. Staphylococcus b. Pseudomonas c. Streptococcus d. Proteus What does “Take in” means in case of skin grafting? a. Revascularization of the graft (AIIMS June 97) b. Return of the sensation c. When the graft becomes adherent to recipient site d. Non adherent graft is shed off

11.

12.

13.

14.

All can take split thickness graft except:  (MCI March 2005, AIIMS Sept 96) a. Fat b. Muscle c. Skull bone d. Deep fascia All are true about skin grafting, except: (All India 2000) a. Partial thickness graft involves epidermis and part of dermis b. Full thickness graft includes epidermis, dermis, without subcutaneous tissue c. For large areas, full thickness graft is used d. Full thickness graft has cosmetic value The given instrument is used for harvesting the graft from healthy area in split skin thickness graft:

a. Dermatome c. Catlin amputating knife 15.

16.

17.

b. Silver’s knife d. Humby knife

Split skin graft can be applied over: (PGI June 99) a. Muscle b. Bone c. Cartilage d. Eyelid Skin graft for facial wounds is taken from: (AIIMS 92) a. Medial aspect of thigh b. Cubital fossa c. Groin d. Postauricular region Full thickness skin graft can be taken from the following sites except: (AIIMS 87) a. Elbow b. Back to neck c. Supraclavicular area d. Upper eyelids (DNB 2009)

18.

Skin graft stored at 4°C can survive up to: a. 1 week b. 2 weeks c. 3 weeks d. 4 weeks

19.

The best skin graft for open wounds is: a. Isograft b. Homograft c. Allograft d. Autograft

20.

All are advantages of split thickness skin grafting except: a. Good uptake (Recent Question 2013) b. Reusable donor site c. Less contraction d. Large grafts can be harvested Identify the correct statement regarding skin grafts & flaps:  (APPG 2016) a. Grafts are tissues that are transferred without their blood supply and they revascularise at the new site b. In pedicle flaps, various tissues, often with bone or muscle, are transferred c. Full thickness grafts are useful for rebuilding missing elements of nose etc. d. Split thickness grafts have limited durability and are called Wolfe grafts

21.

(All India 93)

Plastic Surgery and Skin Lesions FLAPS True statement for axial flap is: (All India 97) a. Carries its own vessels within it b. Kept in limb c. Transverse flap d. Carries its own nerve in it

23.

Best procedure to be done after an injury to leg associated with exposure of underlying bone and skin loss:  (AIIMS Nov 98) a. Pedicle flap b. Split skin grafting c. Full thickness grafting d. Skin flap

24.

Skin flap is used in all except: a. Bone b. Tendon c. Burn wound d. Cartilage

25.

The subdermal plexus forms the vascular basis for: (JIPMER 2002) a. Randomised flaps b. Axial flaps c. Mucocutaneous flaps d. Fasciocutaneous flaps (DPG 2007)

26.

Myocutaneous flap includes which tissues? a. Muscle only b. Muscle and vascular pedicle c. Muscle and skin d. Skin, muscle and vascular pedicle

27.

Best heal flap is: a. Medial plantar artery flap b. Lateral plantar artery flap c. Reversed sural artery flap d. Anterolateral thigh flap

(Recent Question 2017)

28.

A 3-6 cm scalp defect is closed by: a. Primary simple closure b. Split skin grafting c. Secondary closure d. Local flaps

(Recent Question 2017)

29.

33.

A tumour arising in a burns scar is likely to be: (Recent Question 2014, COMEDK 2009, PGI June 2006, June 97) a. Basal cell carcinoma b. Squamous cell carcinoma c. Malignant melanoma d. Fibrosarcoma

34.

Which of the following is true about Marjolin’s ulcer?  (Orissa 2011, PGI Dec 97) a. Ulcer over scar b. Rapid growth c. Rodent ulcer d. Painful

PREMALIGNANT LESIONS OF SKIN Bowen’s disease is: a. Mimics chronic dermatosis b. Premalignant condition c. Presents with pruritus d. All of the above

36.

All the following are premalignant conditions except:  (MHSSMCET 2008) a. Actinic Keratosis b. Steatoma multiplex c. Erythroplakia of Queyrat d. Keratosis of lip

SQUAMOUS CELL CARCINOMA Squamous cell carcinoma can arise from: a. Long standing venous ulcers b. Chronic lupus vulgaris c. Rodent ulcer d. All of the above

38.

Margins of squamous cell carcinoma is: (Recent Question 2016) a. Inverted b. Everted c. Rolled d. Undermined

39.

Buschke-Lowenstein tumor is: a. Molluscum contagiosum b. Condyloma lata c. Giant condyloma accuminata d. Metastasis

40.

Moh’s micrographic surgery is done for: a. Cutaneous melanoma (Recent Question 2016) b. Dermatofibrosarcoma protuberans c. Squamous cell carcinoma d. None of the above

BASAL CELL CARCINOMA 41.

The commonest clinical pattern of basal cell carcinoma is:  (COMEDK 2008, MCI March 2005) a. Nodular b. Morpheaform c. Superficial d. Keratotic

42.

Most common site of basal cell carcinoma is:  (All India 94, MHPGMCET 2001) a. Face b. Trunk c. Neck d. Extremities

MARJOLIN’S ULCER 31.

The most common malignancy found in Marjolin’s ulcer is: a. Basal cell carcinoma (MCI June 2018, DPG 2009 Feb) b. Squamous cell carcinoma c. Malignant fibrous histiocytoma d. Neutrophic malignant melanoma

(TN 2003)

Plastic Surgery

37.

Transversely oriented rectus abdominis muscle flap Vertically oriented rectus abdominis muscle flap Pectoralis major myocutaneous flap Serratus anterior muscle flap

Split thickness skin graft is not taken up by: (MCI June 2018) a. Fat b. Muscle c. Deep fascia d. Skull bone

(DPG 2005)

35.

In the patient as shown below, chest wall closure has been achieved by using which flap? (AIIMS May 2017)

a. b. c. d. 30.

True about Marjolin’s ulcer: (PGI June 2007) a. The associated cancer is squamous cell carcinoma b. May occur due to chronic venous insufficiency c. Progress to basal cell carcinoma d. Arise from base of the ulcer

Section 6

22.

32.

837

838

Surgery Essence 43.

What is the most probable diagnosis based on the given image? (AIIMS November 2017)

MALIGNANT MELANOMA 54.

Plastic Surgery

55.

44.

45.

46.

Section 6

47.

a. Basal cell carcinoma b. Malignant melanoma c. Squamous cell carcinoma d. Marjolin’s ulcer A 48 years old sports photographer has noticed a small nodule over the upper lip from four months. The nodule is pearly white with central necrosis, telangiectasia. The most likely diagnosis would be: a. Basal cell carcinoma b. Squamous cell carcinoma c. Atypical melanoma d. Kaposi sarcoma Which of the following cutaneous malignancies do not metastasize through the lymphatics? (All India 94) a. Squamous cell carcinoma b. Basal cell carcinoma c. Melanoma d. Kaposi’s sarcoma Diagnostic procedure for basal cell carcinoma: a. Wedge biopsy b. Shave c. Incisional biopsy d. Punch biopsy All of the following are true about basal cell carcinoma except: (DNB 2009) a. Most common site is upper eyelid b. Locally invasive c. Rarely metastasizes d. Associated with exposure to sun

48.

In pigmented basal cell carcinoma, treatment of choice is: a. Chemotherapy b. Radiotherapy (PGI 98) c. Cryosurgery d. Excision

49.

Moh’s micrographic excision for basal cell carcinoma is used for all of the following except: (Karnataka 2006) a. Recurrent Tumor b. Tumor less than 2 cm in diameter c. Tumors with aggressive histology d. Tumors with perineural invasion

50.

Basal cell carcinoma spreads by: (MAHE 2007) a. Lymphatics b. Hematogenous c. Direct spread d. None of the above

51.

About basal cell carcinoma, false is: a. Spreads to local lymph nodes b. Seen on face c. Seen on exposure of sunlight to skin d. Responds well to radiation

52.

Characteristic feature of basal cell carcinoma is: (AIIMS May 2012) a. Keratin pearls b. Foam cells c. Nuclear palisading d. Psammoma bodies

53.

(DPG 2006)

Reconstruction of tip of nose after excision of basal cell carcinoma is done by: (DNB 2014)

a. Bipedicled flap b. Bilobed flap c. Full thickness skin graft d. Split skin graft

56.

57. 58. 59.

60.

In the Clarke’s level of tumor invasion for malignant melanoma level 3 refers to: (COMEDK 2006) a. All tumor cells above basement membrane b. Invasion into reticular dermis c. Invasion into loose connective tissue of papillary dermis d. Tumors cell at junction of papillary and reticular dermis True about malignant melanoma: (PGI June 2008) a. Lymphatic spread b. Lymph node biopsy is always done c. Biopsy to be done when sentinel node is involved d. Microsatellitism Common features of melanoma, which of the following is not the part of mnemonic ABCDE? (MHSSMCET 2009) a. Elevation b. Asymmetric outline c. Variation in color d. Diameter < 6 mm In malignant melanoma, change seen is all except: a. Ulceration b. Bleeding (DPG 2006) c. Satellite lesions d. Hair in mole Treatment of choice for melanoma is: (DPG 2006) a. Chemotherapy b. Surgical excision c. Radiotherapy d. Surgery and chemotherapy Most common origin of melanoma is from:  (Bihar PG 2014, AIIMS Nov 2001) a. Junctional melanocytes b. Epidermal cells c. Basal cells d. Follicular cells Most common type of malignant melanoma is: a. Superficial spreading (AIIMS Nov 2001, UPPG 2009) b. Lentigo maligna melanoma (JIPMER 2014, 2012) c. Nodular d. Acral lentiginous

61.

Most common site of lentigo maligna melanoma is:  (DNB 2013, AIIMS Nov 2001) a. Face b. Legs c. Trunks d. Soles

62.

The most malignant form of malignant melanoma is: a. Nodular (PGI June 99) b. Hutchinson’s melanotic freckle c. Acral lentiginous type d. Superficial spreading Prognosis of melanoma depends on: (PGI June 98) a. Stage b. Depth of melanoma of biopsy c. Duration of growth d. Site All of the following statements about malignant melanoma are true except: (All India 97) a. Prognosis is better in female than in male b. Acral lentiginous melanoma carries a good prognosis c. Stage IIa shows statelite deposits d. Most common type is superficial spreading melanoma Biopsy from a mole on the foot shows cytologic atypia of melancytes and diffuse epidermal infiltration by anaplastic cells, which are also present in the papillary and reticular dermis. The most likely diagnosis is: (All India 2004) a. Melanoma, Clark level IV b. Congential melanocytic nevus c. Dysplastic nevus d. Melanoma, Clark level III Which of the following is true about melanoma? a. Amelanotic melanoma is associated with worst prognosis b. Complete excisional biopsy is the management c. Thinner melanoma has good prognosis d. Back in MC site is females (PGI Dec 2005) e. Congenital giant nevus is associated with minimal risk of malignancy

63.

64.

65.

66.

Plastic Surgery and Skin Lesions Melanoma should be excised with a margin of: (UPSC 88) a. 2 cm b. 5 cm c. 7 cm d. 10 cm

68.

Most severe form of malignant melanoma is: a. Superficially spreading b. Nodular infiltrating type c. Those arising in lower type d. Those in choroid

True about congenital hemangioma: a. Congenital variety stops growing after birth b. Fully mature at birth (PGI June 2007, Dec 2007) c. NICH variety persists d. RICH variety involutes e. Calcification can occur

81.

Following is regressive tumor: (MHPGMCET 2007) a. Venous angioma b. Strawberry angioma c. Port-wine stain d. Juvenile angioma

82.

Which of the following statement is correct?

(Kerala 94)

69.

Melanoma staging is based on which classification? a. Breslow b. Clark’s (DNB 2009) c. Both d. Bethesda

70.

Worst prognosis in melanoma is seen in the subtype: a. Superficial spreading (Kerala 2001) b. Nodular melanoma c. Lentigo maligna melanoma d. Amelanotic melanoma

71.

Least common site for spread of melanomas: a. GIT b. Lungs c. Liver d. Renal

72.

Which one of the following is not included in the treatment of malignant melanoma? (UPSC 2005) a. Radiation b. Surgical excision c. Chemotherapy d. Immunotherapy

73.

A 40 years old man presented with al flat 1 cm × 1 cm scaly, itchy black mole on the front of thigh. Examination did not reveal any inguinal lymphadenopathy. The best course of management would be: (UPSC 2007) a. FNAC of the lesion b. Incision biopsy c. Excisional biopsy d. Wide excision with inguinal lymphadenectomy

74.

Inguinal lymph node enlargement is seen in: (MPPG 97) a. Seminoma testis b. Malignant melanoma foot c. CA cervix d. None

75.

Risk factor for malignant melanoma all the following are risk factors foe malignant melanoma except: (DNB 2014) a. Giant congenital nevi b. Family history melanoma c. Exposure to UV light d. HPV infection

76.

All of the following are marker of melanoma except:  (Recent Question 2016) a. S-100 b. Cytokeratin-20 c. HMB-45 d. Vimentin

77.

Immunotherapy is effective in: (Recent Question 2016) a. SCC b. BCC c. Malignant melanoma d. None

78.

Pigmented lesion suspicious of melanoma of size 1 cm with ulceration and features of ABCD. What is the next step?  (Recent Question 2017) a. Wide local excision with 1 cm margin b. Excision biopsy with 1-2 mm margin c. Punch biopsy at the edge of the lesion d. Incisional biopsy

(DNB 2012)

Which of the following is a regressing tumor? (DPG 2011) a. Portwine stain b. Strawberry angioma c. Venous angioma d. Plexiform angioma

a. 1-Strawberry angioma, 2-Portwine stain, 3-Salmon’s patch b. 1-Strawberry angioma, 2- Salmon’s patch, 3- Portwine stain c. 1- Salmon’s patch, 2- Strawberry angioma, 3-Salmon’s patch d. 1- Portwine stain, 2- Strawberry angioma, 3-Salmon’s patch 83.

Treatment for strawberry angioma: (MHSSMCET 2006, JIPMER 95) a. Steroids b. Local excision c. Masterly inactivity d. Antibiotic coverage

84.

The best results in treatment of capillary nevus have been achieved by: (AIIMS 84) a. Full thickness skin graft b. Dermabrasion c. Tatooing d. Argon laser treatment

85.

Spontaneous regression is seen in all except: (All India 93) a. Salmon patch b. Small cavernous hemangioma c. Portwine stain d. Strawberry angioma

Plastic Surgery

SKIN PATCH/STAIN/HEMANGIOMA 79.

80.

Section 6

67.

839

Plastic Surgery

840

Surgery Essence 86.

Salmon patch usually disappears by age:  (Recent Question 2016) a. One month b. One year c. Puberty d. None of the above

87.

Eleven months old child presents with erythematous lesion with central clearing which has been decreasing in size:  (All India 97) a. Strawberry angioma b. Nevus c. Portwine stain d. Cavernous hemangioma

88.

Regarding hemangiomas following are true: a. Salmon patch disappears after the age of one b. Port wine stain present throughout life c. Salmon patch-on forehead midline and over occiput d. All are correct

89.

The best cosmetic results for large capillary (port wine) hemangiomas are achieved by: (UPSC 2005) a. Excision and split-thickness skin b. Laser ablation c. Chemotherapy d. Immunotherapy

90.

Best method to treat a large port-wine hemangiomas: a. Radiotherapy (DNB 2010) b. Tattooing c. Excision with skin grafting d. Pulsed eye laser

91.

Which is not true about Sturge Weber syndrome?  (AIIMS Sept 96) a. Portwine stain b. Calcification in brain c. Cortical atrophy d. Intracranial hamartoma

92.

93.

Which of these does not change or remains same throughout life? (AIIMS Nov 2001) a. Salmon patch b. Strawberry angiomas c. Portwine stain d. Capillary hemangiomas Spontaneous regression is seen in: (Recent Question 2016) a. Portwine hemangioma b. Strawberry hemangioma c. Cavernous hemangioma d. Arterial angioma

SEBACEOUS CYST 94.

Sebaceous cyst does not occur in the: a. Scalp b. Scrotum c. Back d. Sole

95.

True about epidermoid cyst: (PGI Dec 2005) a. Punctum is present b. Keratin is present c. Sebaceous material present d. Autosomal inheritance e. May turn malignant

96.

What is the most probable diagnosis based on the given image? (Recent Question 2017)

97.

Cystic lesion over scalp as shown in the image:   (Recent Question 2019)

a. Sebaceous cyst c. Neural tumor

b. Dermoid cyst d. Meningioma

98.

Sebaceous cyst is: a. Distention cyst c. Implantation dermoid

b. Retention cyst d. Mucus cyst

99.

Cock’s peculiar tumor is: a. Infected sebaceous cyst c. Cyst in the skull

(DNB 2009, MCI Sept 2007) b. Osteomyelitis of skull d. Tumor of the skull

(DNB 2004)

100. Cock’s peculiar tumor is: (MCI June 2018, Recent Question 2014, AIIMS Nov 2010) a. Basal cell carcinoma b. Squamous cell carcinoma c. Ulcerated sebaceous cyst d. Cylindroma

LIPOMA 101. The term universal tumor refers to: a. Adenoma b. Papilloma c. Fibroma d. Lipoma 102. Dercum’s disease is commonest in the: (Recent Question 2016) a. Face b. Arm c. Back d. Thigh 103. Lipoma becomes malignant commonly at which site: a. Subcutaneous b. Retro-pertioneal c. Sub-aponeurotic d. Intermuscular 104. Dercum’s disease is characterized by: a. Lipodermatosclerosis b. Tender subcutaneous lipoma c. Morbid obesity d. None

(DNB 2008)

105. Myxomatous degeneration is lipoma is seen in those occurring in: (MHSSMCET 2006) a. Breast b. Pancreas c. Intramuscular d. Retroperitoneum

HIDRADENITIS SUPPURATIVA

Section 6

106. Hidradenitis suppurativa is found to occur in:  (Recent Question 2016) a. Axilla b. Circumanal c. Scalp d. Groin

KERATOCANTHOMA

a. c.

Dermoid cyst Lipoma

b. d.

Sebaceous cyst Hemangioma

107. Keratocanthoma is: a. A type of basal cell carcinoma b. Infected sebaceous cyst c. Self healing nodular lesion with central ulceration d. Pre-malignant disease

Plastic Surgery and Skin Lesions

117. Treatment for pyoderma gangrenosum is: (Jharkhand 2003) a. Steroids b. I.V. antibiotics c. Surgery + antibiotics d. Surgery alone

MISCELLANEOUS 109. Cause of persistence of a sinus or fistulae includes: a. Foreign body b. Non dependent drainage c. Unrelieved obstruction d. Presence of malignancy e. All of the above 110. The best dressing is: a. Opsitie c. Tulle grass

118. Which of the following materials for implants will evoke least inflammatory tissue response? a. Polypropylene b. Bovine collagen c. Polyaglactin d. Cotton 119. Cylindroma is: a. Appendage tumor c. Pleomorphic adenoma

b. Amnion d. Skin

111. Hydrocele is a type of ……cyst: a. Retention b. Distension c. Exudation d. Traumatic 112. Calcifying epithelioma is seen in: a. Dermatofibroma b. Adenoma sebaceum c. Pyogenic granuloma d. Pilomatrixoma

116. Which is true regarding frostbite injury? (PGI November 2017) a. In first and second degree frostbite, affected part shows redness & edema b. Spontaneous recovery without any treatment is rule c. Extensive involvement in frostbite is called chilblain d. Initial treatment is rewarming e. Autoamputation may occur in severe cases

(JIPMER 95)

Section 6

108. True about keratocanthoma: (PGI 2000) a. Benign tumor b. Malignant skin tumor like squamous cell carcinoma c. Treatment same as for squamous cell carcinoma d. Easy to differentiate from squamous cell carcinoma histologically e. Treatment is masterly inactivity

841

(DPG 2007) b. Acinic cell carcinoma d. Warthin’s tumour

120. Pyogenic granuloma, true statements is/are: (PGI June 2007) a. Vascular pathology b. Bleeds rarely c. Increased in pregnancy d. Local excision e. Recurrent and malignant 121. Lines of Blaschko represent: (All India 2011) a. Lines along lymphatics b. Lines along blood vessels c. Lines along nerves d. Lines of development 122. Bedsore is an example of: a. Tropical ulcer c. Venous ulcer

113. Which of the following statement is correct?

123. Ainhum is seen in: a. Base of great toe c. Base of toe

(All India 99) b. Trophic ulcer d. Post thrombotic ulcer (Recent Question 2014, All India 99) b. Base of fingers tips d. Ankle

124. The given condition is most commonly seen in:

114. Boil can occur at all sites except: a. Pinna b. Skin c. Scalp d. Palm 115. Frost bite is treated by: a. Rapid rewarming c. IV pentoxyphyllin

(TN 95)

(AMC 2000) b. Slow rewarming d. Amputation

a. Great toe c. Third toe

b. Little toe d. Fourth toe

125. Trophic ulcers are caused by: (PGI June 2002) a. Leprosy b. Buerger’s disease c. Syringomyelia d. DVT e. Varicose veins 126. Which of the following is true? a. Viral warts spontaneously resolve b. Plantar warts should not be excised c. Callosity are formed occupationally d. Corns are viral in etiology

(PGI Dec 2005)

127. Ulcer with undermined edges is seen in: (MHPGMET 2005) a. Malignant ulcer b. Tubercular ulcer c. Venous ulcer d. Trophic ulcer

Plastic Surgery

a. 1-Cock’s peculiar tumor, 2-Pott’s puffy tumor, 3-Pilomatrixoma, 4-Cylindroma b. 1-Cock’s peculiar tumor, 2-Pott’s puffy tumor, 3Cylindroma, 4- Pilomatrixoma c. 1- Pilomatrixoma, 2-Pott’s puffy tumor, 3- Cock’s peculiar tumor, 4-Cylindroma d. 1- Cylindroma, 2-Pott’s puffy tumor, 3-Pilomatrixoma, 4Cock’s peculiar tumor

Plastic Surgery

842

Surgery Essence 128. Which of the following is not a true cyst? (MHPGMCET 2006) a. Sebaceous cyst b. Dermoid cyst c. Bone cyst d. Apoplectic cyst

134. Which of the following is a compressible swelling: (DNB 2013, 2010) a. Lipoma b. Hernia c. Hemangioma d. Sebaceous cyst

129. What is the most probable diagnosis based on the given image? (Recent Question 2017)

135. Which of the following flap is used for eye lid surgery?  (MHSSMCET 2010) a. Bilope flap b. Rhomboid flap c. Bipedicle flap d. Transposition flap 136. Bilobed graft is used in: a. Nose c. Cheek

a. c.

Dermoid cyst Lipoma

b. d.

Sebaceous cyst Hemangioma

130. Pilomatrixoma is: (MHPGMCET 2006) a. A fleshy skin mass b. A type of skin tag c. A benign epithelial tumor d. A malignant skin neoplasm 131. Sinus is lined by: (MHPGMCET 2007) a. Simple squamous epithelium b. Columnar epithelium c. Granulation tissue d. Fibrous tissue 132. A swelling which is variable in consistency with diffuse margins is likely to be: (MHSSMCET 2005) a. Inflammatory b. Benign c. Malignant d. Non-specific

Section 6

133. Zadek’s procedure is: (MHSSMCET 2005) a. Resection of part of nail with nail bed b. Resection of complete nail with part of nail bed c. Injection phenol at base of toe nail d. Wide excision of nail

(Recent Question 2017) b. Eyelid d. Fingertips

137. Radiotherapy is the treatment of choice of which one of the following tumors? (UPSC 2008) a. Verrucous carcinoma b. Malignant melanoma c. Marjolin’s ulcer d. Rodent ulcer 138. Pott’s Puffy tumor refers to: a. Osteomyelitis of the frontal bone b. Tuberculosis of the spine c. Actionomycosis of maxilla d. Osteonecrotic tumor of Jaw

(COMEDK 2014)

139. Potts puffy tumor is: (Recent Question 2019, 2018) a. Subperiosteal abscess of ethmoid bone b. Subperiosteal abscess of frontal bone c. Mucocele of ethmoid bone d. Mucocele of frontal bone 140. Manchot, Salmon, Taylor names are related to:  (Recent Question 2017) a. Free-flap b. Arterial supply to the skin c. Nerve supply to the skin d. Arterial supply to muscle

Explanations SKIN GRAFTING 1. Ans. None (Ref: Sabiston 20/e p1939; Schwartz 10/e p264-265, 266; Bailey 27/e p639-640)



Light compression wraps favor ‘take in’ of split skin graft, not the tight dressing.

2. Ans. a. Partial thickness (Ref: Sabiston 20/e p1939; Schwartz 10/e p264-265, 266; Bailey 27/e p639)



3. Ans. c. They contract to the same degree as a grafted sheet of skin (Ref: Sabiston 20/e p1939; Schwartz 10/e p264-265; Bailey 27/e p635,636,639,640)



4. Ans. b. Thigh



6. Ans. a. Joseph Lister

5. Ans. c. Vascular graft

• Joseph Lister said “Skin is the best dressing”.

7. Ans. b. Plasma imbibition



8. Ans. b. Pedicle graft (Ref: Sabiston 20/e p1939; Schwartz 9/e p1651; Bailey 27/e p635, 637)

• Skin graft must be applied to a well-vascularized recipient wound bed. It will not adhere to exposed boneQ, cartilageQ, or tendonQ devoid of periosteum, perichondrium, or peritenon, respectively, or devoid of its vascularized perimembranous envelope. • Radiation damaged tissues are poor recipient sites. • So an exposed bone surface is covered by a graft which has its own blood supply. Such grafts are known as flaps or pedicle grafts.



9. Ans. d. Full thickness skin graft

10. Ans. c. Streptococcus



11. Ans. a. Revascularization of the graft



13. Ans. c. For large areas, full thickness graft is used

12. Ans. c. Skull bone



14. Ans. d. Humby knife



15. Ans. a. Muscle (Ref: CSDT 12/e p1211) For critical and small areas such as an eyelid, a full thickness graft is selected, so that contraction of the grafted material is minimum.



16. Ans. d. Postauricular region (Ref: CSDT 12/e p1211) Donor Sites for Full Thickness Grafts • • • •

Eyelids Postauricular skinQ Supraclavicular skinQ Antecubital skinQ Q

• • • •

Inguinal areaQ Genital areaQ Submammary skinQ Subgluteal skinQ



17. Ans. b. Back to neck



18. Ans. b. 2 weeks (Ref: Facial Plastic and Reconstructive Surgery by Ira D. Papel/44) Excess split-skin autografts harvested and meshed during a surgical session are often stored at short-term for later burn surgery or graft failure. The current procedure in skin storage involves wrapping the meshed autograft on a piece of ringer lactate or normal saline moistened gauze, transferring it into a sterile container and storing it in a 40 C for 2 weeks. The graft should never be totally immersed in saline because it will become macerated. After 14 days of storage the respiratory activity of skin graft reduced by 50%.



19. Ans. d. Autograft



21. Ans. a. Grafts are tissues that are transferred without their blood supply and they revascularise at the new site

20. Ans. c. Less contraction

FLAPS

22. Ans. a. Carries its own vessels within it (Ref: Sabiston 19/e p1917-1919; Schwartz 9/e p1651-1654; Bailey 27/e p637)



23. Ans. a. Pedicle flap

24. Ans. c. Burn wound

844

Surgery Essence

25. Ans. a. Randomised flaps

26. Ans. d. Skin, muscle and vascular pedicle



27. Ans. c. Reversed sural artery flap (Ref: Sabiston 20/e p1969; Schwartz 10/e p1877)



28. Ans. d. Local flaps (Ref: Sabiston 20/e p1950; Schwartz 10/e p1860)

“Scalp flaps are elevated at the subgaleal level, and many possible designs exist. In theory, defects as large as 30% of the scalp can be closed with scalp flaps elevated on major vessels.”-Sabiston 20/e p1950

Section 6

Plastic Surgery



29. Ans. c. Pectoralis major myocutaneous flap (Ref: Sabiston 20/e p1953; Schwartz 10/e p1873; Grabb and Smith Plastic Surgery 7/e p921) In the patient as shown in the question, chest wall closure has been achieved by using pectoralis major myocutaneous flap. Pectoralis major myocutaneous flap is one of the most commonly used flaps in chest wall reconstruction. Flaps frequently used for chest wall reconstruction include one or both the pectoralis major muscles, latissimus dorsi muscles, and rectus abdominis muscles, as well as the greater omentum. “Reconstruction of the Chest Wall: The choice of muscle depends on the location of the defect; options include the latissimus dorsi, serratus anterior, and pectoralis major muscle flaps. Other muscle flaps with limited but specific uses are the trapezius and superiorly based rectus abdominis. The greater omentum can be transposed on the right gastroepiploic artery as a pedicle flap to provide well-vascularized tissue with the bulk and pliability to obliterate dead space, but it is a secondary choice because of the risks of intraabdominal complications.” -Sabiston 20/e p1953 “The pectoralis major muscle is the workhorse pedicled flap for coverage of the sternum, upper chest, and neck.” -Schwartz 10/e p1873

Uses

Type of Flap Pectoralis Major Myocutaneous (PMMC) flap

Head & neck reconstructionQ Sternal & chest wall reconstruction, especially anterior & superior central chestQ

Transverse Rectus Abdominis Muscle (TRAM) flap

Breast reconstructionQ

Vertical Rectus Abdominis Muscle (VRAM) flap

Used to cover defects of sternum, chest wall, pelvis & perineal areasQ

Latissimus dorsi flap

Breast reconstructionQ Chest wall reconstruction especially lateral or anterolateral defects of upper 1/3rd of chest wallQ

Serratus anterior flap

Head & neck reconstructionQ Non-sternal & lower chest wall defects
Q

Mathes & Nahai Classification of muscles flap Type I II III

Mathes & Nahai Classification of Muscles Flap (According to vascular supply) Description Example Single dominant pedicleQ • Tensor fascia lata Single dominant pedicle with minor pediclesQ • Gracilis • Trapezius Two dominant pediclesQ • Gluteus maximum • Rectus abdominis • Serratus anterior

Plastic Surgery and Skin Lesions Multiple segmental pedicles without a single dominant pedicleQ

V

Single dominant pedicle with secondary segmental pediclesQ

• • • • •

Sartorius External Oblique Pectoralis major Latissimus dorsi Internal oblique

30. Ans. d. Skull bone

MARJOLIN’S ULCER

31. Ans. b. Squamous cell carcinoma (Ref: Schwartz 10/e p259, 1817; Bailey 27/e p605)



32. Ans. a. The associated cancer is squamous cell carcinoma, b. May occur due to chronic venous insufficiency



33. Ans. b. Squamous cell carcinoma

Section 6



IV

845

34. Ans. a. Ulcer over scar

PREMALIGNANT LESIONS OF SKIN

35. Ans. d. All of the above (Ref: Sabiston 20/e p748; Schwartz 10/e p847, 1217-1218; Bailey 27/e p606, 607)



36. Ans. b. Steatoma multiplex (Ref: Sabiston 20/e p747; Schwartz 10/e p847, 1217, 1218; Bailey 27/e p606-607) Premalignant Skin Lesions • Senile or actinic keratosisQ • Bowen’s diseaseQ • Erythroplasia of QueyratQ

• Chronic scarsQ • RadiodermatitisQ • ProkeratosisQ

SQUAMOUS CELL CARCINOMA

37. Ans. a. Long standing venous ulcers, b. Chronic lupus vulgaris (Ref: Sabiston 20/e p747; Schwartz 10/e p1218; Bailey 27/e p605)



38. Ans. b. Everted



39. Ans. c. Giant condyloma accuminata (Ref: Bailey 27/e p1368, 1493)

Giant Condyloma Acuminatum



40. Ans. c. Squamous cell carcinoma

BASAL CELL CARCINOMA

41. Ans. a. Nodular (Ref: Sabiston 20/e p748; Schwartz 10/e p486-487; Bailey 27/e p604-605)



42. Ans. a. Face



43. Ans. a. Basal cell carcinoma (Ref: Harrison 19/e p500; Robbins 9/e p1157) Lesion near inner canthus with raised, pearly borders and central crust and with telangiectasia on surface of the lesion is highly suggestive of basal cell carcinoma.



44. Ans. a. Basal cell carcinoma



45. Ans. b. Basal cell carcinoma

46. Ans. a. Wedge biopsy



47. Ans. a. Most common site is upper eyelid

48. Ans. d. Excision



49. Ans. b. Tumor less than 2 cm in diameter (Ref: Sabiston 20/e p735; Schwartz 10/e p486-487; Bailey 27/e p605)



50. Ans. c. Direct spread

51. Ans. a. Spreads to local lymph nodes



52. Ans. c. Nuclear Palisading

53. Ans. b. Bilobed flap (Ref: Bailey 27/e p641)

Bilobed flap is used to cover a convex defect as on tip of nose. The bilobed flap is widely used for small nasal defects because it allows one to distribute tensions further from he primary defect, thus controlling the degree of tension along the alar margin

Plastic Surgery

• Giant condyloma acuminatum (also known as a Buschke-Löwenstein tumor)Q • Rare cutaneous condition characterized by an aggressive, wart-like growth that is a verrucous carcinomaQ. • It is attributed to HPVQ.

846

Surgery Essence

Plastic Surgery

MALIGNANT MELANOMA

54. Ans. d. Tumors cell at junction of papillary and reticular dermis (Ref: Sabiston 20/e p728; Schwartz 10/e p488-492; Bailey 27/e p608)



55. Ans. a. Lymphatic spread, b. Lymph node biopsy is always done, c. Biopsy to be done when sentinel node is involved, d. Microsatellitism



56. Ans. d. Diameter < 6 mm

57. Ans. d. Hair in mole



58. Ans. b. Surgical excision

59. Ans. a. Junctional melanocytes



60. Ans. a. Superficial spreading

61. Ans. a. Face



62. Ans. a. Nodular

63. Ans. a. Stage, b. Depth of melanoma of biopsy, d. Site



64. Ans. b. Acral lentiginous melanoma carries a good prognosis, c. Stage IIa shows statelite deposits

According to latest staging, presence of satellites is included in stage III.

65. Ans. a. Melanoma, Clark level IV



66. Ans. a. Amelanotic melanoma is associated with worst prognosis, b. Complete excisional biopsy is the management, c. Thinner melanoma has good prognosis (Ref: Bailey 27/e p610)

Amelanotic Melanoma • Amelanotic melanoma is a type of skin cancer in which the cells do not make melanin. • They can be pink, red, purple or of normal skin color, hence difficult to recognizeQ. • It has an asymmetrical shape and an irregular faintly pigmented borderQ. • Atypical appearance leads to delay in diagnosis, the prognosis is badQ. • Recurrence rate is highQ.

Desmoplastic Melanoma • Desmoplastic melanoma is mostly found on the head and neck regionQ. • It has a propensity for perineural infiltrationQ and often recurs locallyQ if not widely excised. • It may be amelanotic clinicallyQ.

67. Ans. a. 2 cm

68. Ans. b. Nodular infiltrating type



69. Ans. c. Both

70. Ans. d. Amelanotic melanoma



72. Ans. a. Radiation

73. Ans. c. Excisional biopsy



74. Ans. b. Malignant melanoma foot

75. Ans. d. HPV infection



77. Ans. c. Malignant melanoma



78. Ans. b. Excision biopsy with 1-2 mm margin (Ref: Devita 10/e p1354; Sabiston 20/e p734; Schwartz 10/e p490)

71. Ans. d. Renal

76. Ans. b. Cytokeratin-20

Section 6

SKIN PATCH/STAIN/HEMANGIOMA

79. Ans. b. Strawberry angioma (Ref: Schwartz 10/e p485; Bailey 27/e p613)



80. Ans. a. Congenital variety stops growing after birth, b. Fully mature at birth, c. NICH variety persists, d. RICH variety involutes, e. Calcification can occur (Ref: Schwartz 10/e p485; Bailey 26/e p598-599)



81. Ans. b. Strawberry angioma



82. Ans. a. 1-Strawberry angioma, 2-Portwine stain, 3-Salmon’s patch (Ref: Schwartz 10/e p1849; Bailey 27/e p613) Strawberry Angiomas Type of capillary hemangioma Baby is normal at birthQ Appears at the age of 1-3 weeksQ Grows with the child upto 1 year of age and then cease to growQ • By the age of 9 years, 90% demonstrate complete involutionQ • Emptying signQ is demonstrable • • • •

Port-wine Stain • • • • •

A vascular malformation Present at birthQ Grows along with the childQ Do not regressQ Face involvement in areas supplied by 5th cranial nerveQ

Salmon Patch • Also known as Macular stain or stork biteQ • Present at birthQ • Seen over forehead in the midline & over the occiputQ • Disappears by the age 1 yearQ

Plastic Surgery and Skin Lesions

847

Section 6



83. Ans. c. Masterly inactivity

84. Ans. d. Argon laser treatment



85. Ans. c. Portwine stain

86. Ans. b. One year



87. Ans. a. Strawberry angioma

88. Ans. d. All are correct



89. Ans. b. Laser ablation

90. Ans. d. Pulsed eye laser (Ref: (Roxburgh 17/e p194, 205)

Selective photothermolysis or pulsed eye laser is the treatment of choice for portwine hemangioma. Excellent results have been obtained with careful and time-consuming treatment with a 585-nm flash lamp-pumped pulsed eye laser. Treatment sessions can begin in babies and anesthesia is not always necessary.

91. Ans. d. Intracranial hamartoma (Ref: Bailey 26/e p599)

Sturge-Weber Syndrome / Encephalotrigeminal Syndrome • Usually sporadic, characterized by: −− Large unilateral cutaneous angiomaQ (port-wine stain) −− Angiomas in brain involving ipsilateral cerebral hemisphere and meninges −− Focal seizuresQ typically occurs opposite to the side of lesionQ −− Adrenal pheochromocytoma −− Cerebral angiomas lead to cortical atrophyQ • Angiomas are visible radiologically as Tram-track or rail track calcification mainly in occipital regionQ

92. Ans. c. Portwine stain

93. Ans. b. Strawberry hemangioma



94. Ans. d. Sole (Ref: Schwartz 10/e p1218, 9/e p411; Bailey 27/e p598-599)



95. Ans. a. Punctum is present, b. Keratin is present, e. May turn malignant



96. Ans. b. Sebaceous cyst (Ref: Sabiston 20/e p1860; Schwartz 10/e p486; Bailey 27/e p598)



97. Ans. a. Sebaceous cyst



98. Ans. b. Retention cyst

99. Ans. a. Infected sebaceous cyst

100. Ans. c. Ulcerated sebaceous cyst

LIPOMA 101. Ans. d. Lipoma (Ref: Sabiston 20/e p2011; Schwartz 10/e p486; Bailey 27/e p544, 936) 102. Ans. c. Back 103. Ans. b. Retro-pertioneal (Ref: Sabiston 20/e p2011-2012; Schwartz 9/e p413; Bailey 27/e p1065) 104. Ans. b. Tender subcutaneous lipomas

105. Ans. d. Retroperitoneum

HIDRADENITIS SUPPURATIVA 106. a. Axilla, b. Circumanal, d. Groin (Ref: Sabiston 20/e p1410; Schwartz 10/e p476,506,1233; Bailey 27/e p595-596,1367-1368)

KERATOACANTHOMA 107. Ans. c. Self healing nodular lesion with central ulceration (Ref: Bailey 27/e p606) 108. Ans. a. Benign tumor, e. Treatment is masterly inactivity

Plastic Surgery

SEBACEOUS CYST

848

Surgery Essence MISCELLANEOUS 109. Ans. e. All of the above

110. Ans. d. Skin

111. Ans. c. Exudation

112. Ans. d. Pilomatrixoma

Plastic Surgery

113. Ans. a. 1-Cock’s peculiar tumor, 2-Pott’s puffy tumor, 3-Pilomatrixoma, 4-Cylindroma (Ref: Bailey 27/e p657, 602, 603) Cocks Peculiar Tumour

Potts Puffy Tumour

• Infected or ulcerated sebaceous cyst of the scalpQ • Resembles fungating epitheliomaQ

• Osteomyelitis of the frontal bone of skull • Associated with subperiosteal swelling & edemaQ

Q

Pilomatrixoma

Cylindroma

• Also known as Calcifying epithelioma of MalherbeQ • Benign hair follicle derived tumorQ

• A malignant epithelial tumour also known as Turban tumorQ • Known as cylindroma because of its appearance

114. Ans. d. Palm 115. Ans. b. Slow rewarming (Ref: Bailey 27/e p953, 422)

Frostbite • • • • •

Frostbite injuries affect the peripheries in cold climates. The initial treatment is with slow rewarmingQ in a bath at 42 °C. The cold injury produces delayed microvascular damage. Level of damage is difficult to assess. Surgery usually does not play a role in its management, until there is absolute demarcation of the level of injury.

116. Ans. a. In first and second….., d. Initial treatment…., e. Autoamputation may occur…. 117. Ans. a. Steroids (Ref: Bailey 27/e p596)

Pyoderma Gangrenosum • Relatively uncommon destructive cutaneous lesion. • Clinically, a rapidly enlarging, necrotic lesion with undermined border and surrounding erythema characterize this disease. • Commonly associated with IBD, rheumatoid arthritis, hematologic malignancy and monoclonal immunoglobulin A gammapathy. Treatment • First-line therapy: Systemic treatment by corticosteroids and cyclosporineQ. • If ineffective, alternative therapeutic procedures include systemic treatment with corticosteroids and mycophenolate mofetil; mycophenolate mofetil and cyclosporine. 118. Ans. a. Polypropylene

Section 6

119. Ans. a. Appendage tumor 120. Ans. a. Vascular pathology, c. Increased in pregnancy, d. Local excision (Ref: Bailey 27/e p614; Roxburgh 17/e p197)

Pyogenic Granuloma • Relatively common vascular lesion of skin and mucosa • The name is misnomer (it is neither a granuloma, nor pyogenic in origin) • Most are small (0.5–1.5 cm), raised, pedunculated, soft, red nodular lesions showing superficial ulceration and a tendency to bleed after trivial traumaQ. • Often arise in pregnancy particularly on gingivaQ or elsewhere in oral mucosa Treatment • Local excision with a minimal margin.

Plastic Surgery and Skin Lesions

849

121. Ans. d. Lines of development (Ref: Neurocutaneous Disorders by Ruggieri (2008)/364) Lines of Blaschko represents random line of development of skin. 123. Ans. c. Base of toe (Ref: Bailey 25/e p914)

Ainhum • • • • •

Ainhum is a disease of unknown etiology Usually affects black menQ (and occasionally women) who have run barefoot in childhoodQ. It is recorded in central Africa, central America and the Orient. A fissure appears at the level of the interphalangeal joint of a toe, usually of the little toeQ. Fissure is followed by a fibrous band that encircles the digit and causes necrosisQ.

Section 6

122. Ans. b. Trophic ulcer

Treatment • Early stage: Z-plastyQ; Later stage: AmputationQ 124. Ans. b. Little toe 125. Ans. a. Leprosy, c. Syringomyelia (Ref: Sabiston 19/e p1943-1945; Bailey 27/e p69-70) 126. Ans. a. Viral warts spontaneously resolve, c. Callosity are formed occupationally (Ref: Sabiston 19/e p1402; Schwartz 10/e p485, 1233; Bailey 27/e p599,1368,1486,1496) Corn and callosities are hyperkeratosis due to chronic excessive pressure or friction on the skin.

Warts • • • •

Warts are epidermal growths associated with HPV infectionQ Histologically characterized by: Hyperkeratosis, acanthosis, papillomatosis and koilocytes Recurrences are common Some warts are risk factors for SCCQ.

Treatment • Removed by number of chemical including formalin, podophyllin etcQ. • Surgical excision or curettage with electrodissectionQ can also be done. • Treatment of extensive areas of skin requires surgical excision under GA.

128. Ans. d. Apoplectic cyst (Ref: Bailey 24/e p209)

Cyst • Cyst is a sac that is filled with a fluid or semi-fluid material. • Two of the most common types of cyst that occur under the skin surface are sebaceous cyst and dermoid cyst. These are true cyst, line by epithelium. • Pseudocyst of pancreas and apoplectic cyst are not lined by epithelium, and are not true cystQ. 129. Ans. a. Dermoid cyst (Ref: Sabiston 20/e p1860; Schwartz 10/e p1737; Bailey 27/e p667) 130. Ans. c. A benign epithelial tumor 131. Ans. c. Granulation tissue (Ref: Bailey 27/e p616) • A sinus is a blind-ending tract that connects a cavity lined with granulation tissue (often an abscess cavity) with an epithelial surface. 132. Ans. c. Malignant 133. Ans. b. Resection of complete nail with part of nail bed (Ref: Orthopedics and Fractures by T. Duckworth, C. M. Blundell (2010)/215) Zadek’s procedure is resection of complete nail with part of nail bed in ingrowing toe-nail.

Zadek’s Procedure • For recurrent problems in ingrowing toe-nail, the toenail and its growing point removed so that the nail does not ever re-grow (Zadek’s procedure)Q.

Plastic Surgery

127. Ans. b. Tubercular ulcer

850

Surgery Essence 134. Ans. c. Hemangioma 135. Ans. c. Bipedicle flap (Ref: Bailey 27/e p641) 136. Ans. a. Nose (Ref: Sabiston 20/e p836)

Local Flaps • A local flap is raised next to a tissue defect in order to reconstruct it.

Section 6

Plastic Surgery

Transposition flap Z-plasty Rhomboid flap Rotation flap Advancement flap V-to-Y advancement Bilobed flap Bipedicle flap

Basic Patterns of Local Flaps • Most basic design, leaving a graftable donor site • For lengthening scars or tissues • For cheek, temple, back and flat surface defects • For convex surfaces • For flexor surfaces; may need triangles excised at the base to make it work (commonly called Burrow’s triangles) • Commonly used for fingertips and extremities • For convex surfaces, especially the noseQ • For eyelids, rarely elsewhereQ.

137. Ans. d. Rodent ulcer (Ref: http://emedicine.medscape.com/article/276624-treatment#aw2aab6b6b4) • Among the given options, only rodent ulcer (BCC) is radiosensitive, rest all are not responsive to radiotherapy. 138. Ans. a. Osteomyelitis of the frontal bone 139. Ans. b. Subperiosteal abscess of frontal bone 140. Ans. b. Arterial supply to the skin (Ref: Grabb & Smith /p29)

CHAPTER

32

Wound Healing, Tissue Repair and Scar

WOUND Wound • Wound: Breach in continuity of skin or surface epithelium • Simple wound: Only skin & subcutaneous tissue is involvedQ • Complex wound: Involves underlying nerves, vessels, tendons with devitalized tissueQ

WOUND HEALING Wound Healing • Mechanism by which body attempts to restore the integrity of injured part Phases of Wound Healing Inflammatory Phase • Begins immediately after woundingQ • Last for 2-3 daysQ • Bleeding → Vasoconstriction & thrombus formation to limit blood loss → Platelets stick to damaged endothelial liningsQ

Proliferative Phase

Remodeling Phase

• Last from 3rd day to 3rd weekQ • Consist of fibroblast activity with production of collagen, glycosaminoglycans & proteoglycansQ • AngioneogenesisQ: Growth of new vessels as capillary loops • Re-epithelializationQ of wound surface • Increase in tensile strength of wound due to increased type III collagen, deposited in random fashionQ • Wound contractionQ: Reduces the surface area of wound

• Characterized by maturation of collagenQ • Type I collagen replacing type III until a ratio of 4:1 is achievedQ • Realignment of collagen fibers along the line of tensionQ • Decreased wound vascularityQ • Wound contractionQ • Maturation of collagen: Increased tensile strength of woundQ • Wound strength is maximum at 12th week post injuryQ (represents approximately 80% of uninjured skin strengthQ)

FACTORS ADVERSELY AFFECTING WOUND HEALING Factors that Inhibit Wound Healing Local Factors • • • • • • •

InfectionQ IschemiaQ Foreign bodyQ HematomaQ Movement Mechanical stress Necrotic tissue

Systemic Factors • • • • • • •

Diabetes mellitusQ Ionizing radiationQ, temperatureQ Advanced ageQ, MalnutritionQ Vitamin C and A deficiencyQ Mineral (Zinc and IronQ) deficiencies Drugs (SteroidsQ, Doxorubicin) JaundiceQ, UremiaQ, MalignancyQ

852

Surgery Essence CLASSIFICATION OF SURGICAL WOUNDS Classification of Durgical Wounds Wounds Class

Definition

I: Clean

• Uninfected operative wound without inflammationQ • Respiratory, alimentary, genital or infected urinary tract is not enteredQ • Wounds are closed primarilyQ, if necessary drained with closed drainageQ Examples of Clean Wound • Inguinal hernia • Joint replacementQ • ThyroidectomyQ

• MastectomyQ • Abdominal aortic aneurysm (AAA) repairQ

Plastic Surgery

Q

II: Clean contaminated

• Operative wound in which respiratory tract, GIT or genitourinary tract is entered under controlled condition without unusual contaminationQ Examples of Clean Contaminated Wound • CholecystectomyQ • CBD explorationQ

III: Contaminated

• • • •

• Elective GI surgeries (elective colonic resectionQ, gastrectomyQ)

Open, fresh accidental woundsQ Operations with major break in sterile techniquesQ Gross spillage from GITQ Incision in which acute non-purulent inflammation is encounteredQ Examples of Contaminated Wound • • • •

IV: Dirty

• • • •

Spill during elective GI surgeryQ Perforated gastric ulcerQ Appendicular perforationQ Penetrating abdominal traumaQ

• Enterotomy during bowel obstructionQ • Human biteQ • Open fractureQ

Old traumatic wound with retained devitalized tissueQ Wound with clinical infection or perforated viscera with high degree of contaminationQ Organism causing post-op infection is already presentQ in the wound before operation Associated with severe inflammationQ Examples of Dirty Wound • Perforated diverticulitis • Frank pusQ

Q

Section 6

Wounds Class

Risk of Infection

• Fecal peritonitisQ • Necrotizing soft tissue infectionQ Need for Prophylaxis

Clean

5%

• Usually not requiredQ

Contaminated

10%

Clean-contaminated

20–30%Q

• RequiredQ

Dirty

30–40%

• Treatment requiredQ (not the prophylaxis)

Q

• Usually requiredQ

Q

Q

CLASSIFICATION OF WOUND CLOSURE & HEALING Classification of Wound Closure & Healing Healing by Primary Intention

Healing by Secondary Intention

Healing by Tertiary Intention

• Also known as healing by first intentionQ • Occurs when there is: −− Apposition of wound edgesQ −− Minimal surrounding tissue trauma & least inflammationQ −− Associated with best scarQ

• Occurs in the wounds that are: −− Left openQ −− Allowed to heal by granulation, contraction & epithelializationQ

• Also known as “delayed primary intention healing”Q • Occurs when wound edges are not opposed immediately, in contaminated or untidy woundsQ • Delayed closure of woundQ is done when inflammatory & proliferative phase of healing is well established • Results in less satisfactory scarQ as compared to healing by primary intention

Wound Healing, Tissue Repair and Scar

853

CHRONIC WOUND

• Wounds that do not heal within 3 months • Delay in healing can occur at any phase but most often occur in inflammatory phaseQ Treatment • Surgical treatment is only indicated if non-operative treatment has failed or patient suffers from intractable painQ

DEGLOVING INJURY

Section 6

Chronic Wound Q

Degloving • Skin & subcutaneous fat are stripped by avulsion from the underlying fasciaQ • Leaving neurovascular structures, tendon or bone exposedQ

COMPARTMENT SYNDROME Compartment Syndrome • Typically occur in closed lower limb injuries Clinical Features

Q

• Characterized by severe painQ; Pain on passive movement of affected compartment muscleQ • Distal sensory disturbanceQ • Finally by absence of pulses distallyQ (a late signQ) Diagnosis • Compartment pressure is measured by using a pressure monitor & catheter placed in muscle compartmentQ Treatment • Fasciotomy if pressure is constantly > 30 mm Hg or presence of clinical signs of compartment syndromeQ • In fasciotomy: Longitudinal incision is given over skin, subcutaneous fat & fascia; muscle should bulge through fasciotomy openingQ

PRESSURE SORE

Also known as Bed sores/Pressure ulcer/Decubitus ulcer/Pressure sore/Trophic ulcer/Penetrating ulcerQ Definition: Tissue necrosis with ulceration due to prolonged pressureQ Preventable Incidence: 5% of hospitalized patients; Higher incidence in: ParaplegicQ patients, elderly & severely illQ patients MC site: IschiumQ > Greater trochanterQ > SacrumQ > HeelQ > MalleolusQ > OcciputQ (Indira Gandhi Stadium inauguration by HM Office) Mechanism: External pressure exceeds the capillary occlusive pressure (>30 mm Hg) → Blood flow to the skin stops → Tissue hypoxia, necrosis & ulcerationQ • Other mechanisms: Due to impaired nutrition, defective blood supply, neurological deficitQ • • • • • •

Neurological Causes (SPL DPT) • Syringomyelia, Spina bifida, Spinal injuryQ • Diabetic neuropathyQ • Peripheral neuritis, Peripheral nerve injuryQ • ParaplegiaQ Q • Leprosy • Tabes dorsalisQ Clinical Features • Painless punched out ulcerQ; Base formed by boneQ Stage 1 2 3 4

• • • •

Staging of Pressure Sore Description Non-blanchable erythema without a breach in epidermis (early superficial ulcerQ) Partial thickness skin loss involving epidermis & dermis (Late superficial ulcerQ) Full thickness skin loss extending into subcutaneous tissue but not through underlying fascia (Early deep ulcerQ) Full thickness skin loss through fascia with extensive tissue destruction, maybe involving muscle, bone, tendon or joint (Late deep ulcerQ)

Plastic Surgery

Pressure Sore

854

Surgery Essence Management • Prevention is the best treatmentQ with: Good skin care; special pressure dispersion cushions or foams; Use of low air-loss & airfluidized bedsQ; Urinary or fecal diversion in selected case • Bed bound patients: Turned at least every 2 hoursQ • Wheel chair bound patients: Lift themselves off their seat for 10 second every 10 minutesQ • Surgical treatment: Reserved for the patient with no improvement after conservative management; includes adequate debridement, vacuum assisted closure or Flap closure

Plastic Surgery

• Large skin flaps with muscle & intact sensory innervations is preferredQ (Example: Extensor fascia lata with lateral cutaneous nerve of thighQ)

VACCUM ASSISTED CLOSURE/ NEGATIVE PRESSURE WOUND THERAPY Vacuum Assisted Closure/Negative Pressure Wound Therapy (NPWT) • • • • •

NPWT promotes wound healing by applying a vacuum through a special sealed dressing. Continued vacuum draws out the fluid from wound & increases blood flow to the area. Vacuum may be applied continuously or intermittently, depending upon the types of wound being treated & clinical objectives. Negative pressure of –125 mm HgQ is used. Dressing should be changed 2-3 times/weekQ Primary Effects of NPWT on Wound Healing MacrodeformationQ

Drawing the wound edges together leading to contraction

Stabilization of wound environment

Wound protected from outside micro-organisms in a warm & moist environment

Reduced edemaQ

With removal of soft tissue exudates

Q

Microdeformation

Q

Leading to cellular proliferation on the wound surface Contraindications for NPWT Use

• Malignancy in the woundQ • Untreated osteomyelitisQ

• Non-enteric & unexplored fistulaQ • Necrotic tissue with escharQ

SCAR Scar • Maturation phase of wound healing leads to formation of scarQ • Immature scar (Pink, raised, hard & itchyQ) → As the collagen matures & becomes denser, scar becomes almost acellular, as fibroblast & blood vessels reduce → Scar becomes paler, flattens, softer & itching diminishesQ → Tensile strength of scar increases; maximum at 12th weekQ (after 3 months) post-injury; represent approx. 80% of uninjured skin strengthQ • Types of scar: Atrophic scar, hypertrophic scar & keloid

ATROPHIC SCAR

Section 6

Atrophic Scar • Pale, flat & stretched in appearance; Appear on the back & in areas of tensionQ • Easily traumatized as epidermis & dermis are thinned; Excision & resuturing rarely improves such scarQ

HYPERTROPHIC SCAR Hypertrophic Scar • Excessive scar tissue that does not extend beyond the boundaries of original incision or woundQ • Results from prolonged inflammatory phaseQ of wound healing and/or unfavorable scar citingQ (across the lines of skin tension)

Wound Healing, Tissue Repair and Scar

855

Histology

Section 6

• Excess collagen & hypervascularityQ (more marked in keloid with more type III collagen) • Contains well organized type III collagen; Improves spontaneously with timeQ Treatment • Linear hypertrophic scar: Treated with pressure therapy or silicone gel sheet applicationQ • Ongoing hypertrophy: Treated with intralesional steroids (TriamcinoloneQ) • Scars persist after 1 year: Surgical excision + primary closure of woundQ

KELOID Keloid • • • •

Excessive scar tissue that extends beyond the boundaries of original incision or woundQ Etiology: Unknown; Genetic predisposition, more common in blacksQ Associated with: Elevated levels of growth factors; deeply pigmented skinQ Common in certain areas of body: Above clavicleQ, upper extremitiesQ, on the trunkQ, faceQ (Especially seen in triangle whose boundaries are xiphisternum & each shoulder tipQ)

Histology • Excess collagen & hypervascularity; Contain disorganized type I & III collagenQ • Thicker collagen bundlesQ form acellular node like structures Treatment • • • •

Keloids rarely regress with time, often refractory to medical & surgical interventionQ First line treatment: Silicones in combination with pressure therapy & intralesional corticosteroid injectionQ Refractory cases (after 12 months of therapy): Excision + Post-op radiotherapyQ (external beam or brachytherapy) New treatment modalities: Internal cryotherapyQ & 5% ImiquimodQ Feature

Hypertrophic Scar

Keloid

Genetic

Not familial

May be familialQ

Race

Not race related

Sex

Female = male

FemaleQ >male

Age

Children

10-30 yearsQ

Border

Remains within woundQ

Natural history

Subsides with time

Rarely subsides

Site

Flexor surfacesQ

Sternum (MCQ), shoulder, face

Etiology

Related to tensionQ

Unknown

Develop

Within 4 weeks

3 months to year after trauma

Symptoms

Raised, some pruritus Respect wound confines

Pain, pruritus, hyperesthesia Growth beyond wound margins

Histology

Parallel orientation of collagen fibers

Thick wavy collagen fibers in random orientation

Q Q

Q

Contracture • A tight web restricting the range of movement at the jointQ • When the scar crosses joints or flexion creases is known as contractureQ Clinical Features • It can cause hyperextension or hyperflexion deformityQ • In neck, may interfere with head extensionQ Treatment • Multiple Z-plastiesQ; Inset of grafts or flapsQ • Splintage & intensive physiotherapy in post-op periodQ

Outgrows wound area

Plastic Surgery

CONTRACTURE

BlackQ >white

Multiple Choice Questions WOUND HEALING























1. Wound healing is affected by: (PGI Dec 2007) a. Age b. Nutrition c. Dryness or wetness of wound d. Drugs e. Temperature 2. Prevention of wound infection is done by: (PGI June 2005) a. Pre-op shaving b. Pre-op antibiotic therapy c. Monofilament suture d. Wound apposition 3. True about wound healing: (PGI June 2009) a. Infected wound heal by primary intention b. Deep dermal wound heal by scar formation c. Wound contraction is found in healing by secondary intention d. More intense inflammatory response in primary intention 4. True about chronic wound: (PGI Nov 2009) a. Found in DM b. Always require surgical treatment c. May be associated with vascular compromise d. Monofilament sutures prevent infection e. Any wound that does not heal within 3 month 5. Cell not involved in healing of clean wound: a. Macrophages (PGI Nov 2011) b. Platelet c. Fibroblasts d. Polymorphonuclear leukocytes e. Myofibroblasts 6. Management of an open wound seen 12 hours after the injury: (DPG 2011) a. Suturing b. Debridement and suture c. Secondary suturing d. Heal by granulation 7. The vitamin which has inhibitory effect on wound healing is: (MAHE 2005) a. Vitamin A b. Vitamin E c. Vitamin C d. Vitamin B-complex 8. The tensile strength of the wound starts and increases after: a. Immediate suture of the wound (WBPG 2012, MAHE 2005) b. 3–4 days c. 7–10 days d. 6 months 9. In a sutured surgical wound, the process of epithelialization is completed within: (UPSC 2007) a. 24 hours b. 48 hours c. 72 hours d. 96 hours 10. Factors that may adversely affect the healing of wounds include all the following except: a. Exposure to UV light b. Exposure to radiation c. Obstructive jaundice d. Advanced neoplasia 11. Primary closure of incised wounds must be done within: a. 2 hours b. 4 hours  c. 6 hours d. 12 hours e. 16 hours 12. The tensile strength of wound reaches that of normal tissue by: a. 6 weeks b. 2 months c. 4 months d. 6 months 13. Following are required for wound healing except: a. Zinc b. Copper  (All India 93) c. Vitamin C d. Calcium



14. Patient has lacerated untidy wound of the leg and attended the casualty after 2 hours. His wound should be: a. Sutured immediately (Recent Question 2016) b. Debrided and sutured immediately c. Debrided and sutured secondarily d. Cleaned and dressed



15. When is the maximum collagen content of wound tissue?  a. Between 3rd to 5th day b. Between 6th to 17th day c. Between 17th to 21st day d. None of the above 16. A patient with grossly contaminated wound presents 12 hours after an accident, his wound should be managed by: a. Thorough cleaning and primary repair (UPSC 96) b. Thorough cleaning with debridement of all dead and devitalized tissue without primary closure c. Primary closure over a drain d. Covering the defect with split skin graft after cleaning 17. Delayed wound healing is seen in all except: (APPG 96) a. Malignancy b. Hypertension c. Diabetes d. Infection 18. In the healing of clean wound the maximum immediate strength of the wound is reached by: a. 2–3 days b. 4–7 days c. 10–12 days d. 13–18 days 19. A clean incised wound heals by: (DPG 92) a. Primary intention b. Secondary intention c. Excessive scaring d. None of the above 20. Which one of the following surgical procedures is considered to a have a clean-contaminated wound? a. Elective open cholecystectomy for cholelithiasis b. Herniorrhaphy with mesh repair c. Lumpectomy with axillary node dissection d. Appendectomy with walled off abscess 21. Fibroblasts in healing wound are derived from:  (UPSC 2008, PGI 98) a. Local mesenchyme b. Epithelium c. Endothelium d. Vascular fibrosis



22. Tensile strength of wound becomes normal after: (Recent Question 2013) a. 6 weeks b. Never c. 4 months d. 6 months 23. In classification of contaminated wound, which of the following are included? (PGI May 2018) a. Resection of unprepared bowel b. Perforated appendix resection c. Resection of intestinal fistula d. Inguinal hernia repair e. Hysterectomy

KELOID AND HYPERTROPHIC SCAR

24. True statement(s) regarding hypertrophic scar: a. Grow beyond wound margin (PGI Dec 2008) b. More common in female c. Not familial d. Rarely subsides e. Not race related



25. True statement regarding hypertrophic scar: a. Usually occurs across the flexural areas b. Does not improve with time c. Overgrows its boundaries d. Develops months after surgery (JIPMER May 2018)

Wound Healing, Tissue Repair and Scar





29. The following statement about keloid is true:  (Recent Question 2018) a. Elevated levels of growth factor is not seen b. Extended excision is the treatment of choice c. It do not extend beyond the wound d. It will have more collagen and vascularity

30. Most common site of hypertrophic keloid is: a. Face b. Leg (AIIMS Nov 93) c. Presternal area d. Arm 31. First line treatment for keloid is: a. Intralesional injection of keloid b. Local steroid c. Radiotherapy d. Wide excision (Recent Question 2015, AIIMS Dec 94)



32. Drug used for intralesional injection keloid is:  (AIIMS June 95) a. Prednisolone b. Triamcinolone c. Androgen d. Hydrocortisone



33. Keloid scar is made up: a. Dense collagen c. Granulomatous tissue



34. What is true about keloids? a. It appears immediately after surgery b. It appears a few days after surgery c. It is limited in its distribution d. It is common in old people 35. Keloid is best treated: a. Intralesional injection of trimacinolone b. Wide excision and grafting c. Wide excision and suturing d. Deep X-ray therapy





(Recent Question 2016) b. Loose fibrous tissue d. Loose areolar tissue (JIPMER 95)

(UPSC 95, 2001)

36. The following statement about keloid is true: (Recent Question 2016) a. They do not extend into normal skin b. Local recurrence is common after excision c. They often undergo malignant change d. They are more common in whites than in blacks

37. The worst position for scars is: a. Back b. Shoulder c. Sternum d. Abdomen



38. Keloid formation is not seen over: (Recent Question 2016) a. Ear b. Face c. Eyelids d. Neck

VACUUM ASSISTED CLOSURE

39. Vacuum assisted closure is contraindicated in: a. Chronic osteomyelitis (Recent Question 2017) b. Large amount of necrotic tissue with eschar c. Abdominal wound d. Surgical wound dehiscence

40. All of the following are principles of negative pressure wound therapy except: (Recent Question 2017) a. Stabilization of wound environment b. Clearance of infection c. Macrodeformation of the wound d. Decreased edema 41. Negative pressure wound therapy (NPWT) is used in:  (Recent Question 2018) a. Bedsore in sacrum after debridement b. After amputation negative suction c. Osteomyelitis d. Unexplored fistulas

MISCELLANEOUS

42. The best scars are seen in: a. Infants b. Children c. Adults d. Very old people 43. If suture marks are to be avoided, skin sutures should be removed by: (JIPMER 81, AMC 89) a. 72 hours b. 1 week c. 2 weeks d. 3 weeks 44. Degloving injury is: (Kerala 2000) a. Surgeon made wound b. Lacerated wound c. Blunt injury d. Avulsion injury e. Abrasive wound 45. In treatment of hand injuries, the greatest priority is:  (All India 96) a. Repair of tendons b. Restoration of skin cover c. Repair of nerves d. Repair of blood vessels 46. ‘Limb salvage’ primarily depends on: (AIIMS 97) a. Vascular injury b. Skin cover c. Bone injury d. Nerve injury 47. In an open injury during toileting and debridement, muscle viability is detected by: (PGI June 2003) a. Colour of the muscle b. Muscle size c. Muscle function d. Muscle contractility e. Punctate bleeding spots on cut edge



48. Criteria for viability of muscle are all except:  (Recent Question 2014) a. Colour b. Intact fascia c. Contractibility d. Bleeding on cutting 49. Degloving injury: (Recent Question 2019, 2018) a. Separation of skin only b. Separation of skin + subcutaneous tissue c. Separation of skin + subcutaneous tissue + fascia exposing tendons d. Separation of tendon exposing the bone

Plastic Surgery





Section 6



26. Which of the following is true about keloid?  (Recent Question 2018, 2017) a. Wide local excision is treatment of choice b. Collagen is same but arranged haphazardly c. Will not spread beyond wound site d. More amount of growth factors 27. All are true about keloid except: (PGI Dec 2007) a. Grows beyond would margin b. Excess collagen deposition c. Precancerous leading to cancer d. More common in female e. Whites are at high risk 28. Which one of the following statements is true regarding the picture depicted here? (APPG 2016) a. Commonly seen on eyelids, genitalia, palms and soles b. Tend to occur 3 months to one year after initial insult c. All these statements are true d. Commonly extend into underlying subcutaneous tissue

857

Explanations WOUND HEALING

1. Ans. a. Age; b. Nutrition; d. Drugs; e. Temperature (Ref: Sabiston 20/e p130-134, 19/e p151-164; Schwartz 10/e p241-268; Bailey 27/e p24)



2. Ans. b. Pre-op antibiotic therapy; c. Monofilament suture; d. Wound apposition (Ref: Bailey 27/e p52) • Bailey says “Preoperative shaving should be avoided except for aesthetic reasons or to prevent adherence of dressingsQ. If it is to be undertaken, it should be undertaken immediately before surgery as the SSI rate after clean wound surgery may be doubled if it is performed the night before, because minor skin injury enhances superficial bacterial colonizationQ. Cream depilation is messy and hair clipping is best, with the lowest rate of infectionQ.” Avoiding Surgical Site Infections • • • • • • • • • •

Staff should always wash their hands between patientsQ Length of patient stay should be kept to a minimumQ Preoperative shaving should be avoided if possibleQ Antiseptic skin preparationQ should be standardized Bowel preparation for intra-abdominal surgeriesQ Pre-operative antibiotics given IV at the time of inductionQ Attention to theatre technique and discipline Avoid hypothermia perioperatively and ensure supplemental oxygenation in recoveryQ Monofilament sutures are used over polyfilament sutures to prevent infectionQ Proper apposition of the wound and prevention of any dead space and hematomaQ



3. Ans. b. Deep dermal wound heal by scar formation; c. Wound contraction is found in healing by secondary intention (Ref: Sabiston 20/e p130-134; Schwartz 10/e p234, 1820; Bailey 27/e p24-25; Robbins 9/e p106)



4. Ans. a. Found in DM; c. May be associated with vascular compromise; d. Monofilament sutures prevent infection; e. Any wound that does not heal within 3 month (Ref: Bailey 27/e p29)



5. Ans. b. Platelet; e. Myofibroblasts (Ref: Sabiston 20/e p130-134; Schwartz 10/e p234,1820; Bailey 27/e p24-25)

Cutaneous Wound Healing • Wound contraction: Most clearly differentiates primary from secondary healingQ • Permanent wound contraction requires the action of myofibroblasts, altered fibroblasts that have the ultrastructural characteristics of smooth muscle cellsQ. • Contraction of these cells at the wound site decreases the gap between the dermal edges of the wound.

6. Ans. b. Debridement and suture (Ref: Sabiston 19/e p245; Schwartz 10/e p234,1820; Bailey 27/e p24-26, 89; Robbins 9/e p109) • If the blood supply to the wound is adequate and bacterial invasion is absent, wound can be safely closed anytime following proper debridement and irrigationQ. • If there is established infection and tissue is of doubtful viability has been left in-situ, then the wound is left open and re-explored after 48 hoursQ. • If there is infection, and the doubtful viable tissue is now healthy, the deep tissues can be repaired and the wound is closedQ. • If however there is further necrosis and infection, the wound is again debrided and left openQ.



7. Ans. b. Vitamin E



9. Ans. b. 48 hours



8. Ans. b. 3–4 days 10. Ans. a. Exposure to UV light

11. Ans. c. 6 hours (Ref: Sabiston 19/e p245; Schwartz 9/e p219; Bailey 27/e p24-25,89; Robbins 9/e p109)

Wound Healing, Tissue Repair and Scar

859

Wounds can be Closed by



12. Ans. None (Ref: Sabiston 20/e p141; Schwartz 10/e p245; Robbins 9/e p109)

Section 6

• Primary suture: −− Clean woundsQ −− Selected contaminated wounds after thorough wound toileting and debridementQ • Delayed primary suture: −− Heavily contaminated woundsQ −− Wounds in which wound toileting has been delayed for 6–8 hoursQ • Left open to heal by secondary closure

The tensile strength of wound never equals that of unwounded skin.

Wound Strength • At the end of the 1st week, wound strength is approximately 10% of that unwounded skinQ. • Strength increases rapidly over the next 4 weeksQ. • This rate of increase then slows at approximately the third month after the original incision, and reaches a plateau at about 70–80% of the tensile strengthQ of unwounded skin, a condition that may persist for life. • The recovery of tensile strength results from the excess of collagen synthesis over collagen degradation during the first two months of healing and later from structural modification of collagen fibresQ (cross linking, increased fiber size) after collagen synthesis ceases.

13. Ans. None



15. Ans. c. Between 17th to 21st day

14. Ans. b. Debrided and sutured immediately

• Over the first three weeks, strength and collagen content both increases but after 21 days collagen content remain static and only wound strength increasesQ. 16. Ans. b. Thorough cleaning with debridement of all dead and devitalized tissue without primary closure



17. Ans. b. Hypertension

18. Ans. d. 13–18 days



19. Ans. a. Primary intention

20. Ans. a. Elective open cholecystectomy for cholelithiasis



21. Ans. a. Local mesenchyme



22. Ans. b. Never



23. Ans. a. Resection of unprepared bowel, b. Perforated appendix resection

KELOID AND HYPERTROPHIC SCAR

24. Ans. c. Not familial; e. Not race related (Ref: Sabiston 20/e p142-143; Schwartz 10/e p261-263; Bailey 27/e p31; Robbins 9/e p109)



25. Ans. a. Usually occurs across the flexural areas



26. Ans. d. More amount of growth factors (Ref: Bailey 27/e p31) “A keloid scar is defined as excessive scar tissue that extends beyond the boundaries of the original incision or wound. Its aetiology is unknown, but it is associated with elevated levels of growth factor, deeply pigmented skin, an inherited tendency and certain areas of the body (e.g. a triangle whose points are the xiphisternum and each shoulder tip). The histology of both hypertrophic and keloid scars shows excess collagen with hypervascularity, but this is more marked in keloids where there is more type III collagen.”Bailey 27/e p31



27. Ans. c. Precancerous leading to cancer; e. Whites are at high risk (Ref: Sabiston 20/e p143; Schwartz 10/e p261-263; Bailey 27/e p31; Robbins 9/e p109)



28. Ans. b. Tend to occur 3 months to one year after initial insult (Ref: Sabiston 20/e p143; Schwartz 10/e p261; Bailey 27/e p31)



29. Ans. d. It will have more collagen and vascularity



30. Ans. c. Presternal area

Plastic Surgery



860

Surgery Essence

31. Ans. a. Intralesional injection of keloid (Ref: Bailey 27/e p31)



32. Ans. b. Triamcinolone

33. Ans. a. Dense collagen



34. Ans. b. It appears a few days after surgery

35. Ans. a. Intralesional injection of triamcinolone



36. Ans. b. Local recurrence is common after excision 38. Ans. c. Eyelids

37. Ans. c. Sternum

VACUUM ASSISTED CLOSURE

Plastic Surgery

39. Ans. b. Large amount of necrotic tissue with eschar (Ref: Long, Mary Arnold; Blevins, Anne (2009). “Options in negative pressure wound therapy”. Journal of wound, ostomy and continence nursing 36 (2): 202-11)

40. Ans. b. Clearance of infection



41. Ans. a. Bedsore in sacrum after debridement

MISCELLANEOUS

42. Ans. d. Very old people



43. Ans. b. 1 week (Ref: Bailey 27/e p31-32) • • • • •



Suture marks may be minimised by using monofilament sutures that are removed early (3–5 days)Q. Sutures inserted under tension will leave marksQ. The wound can be strengthened post suture removal by the use of sticky strips. Fine sutures (6/0 or smaller) placed close to the wound margins tend to leave less scarringQ. Subcuticular suturing avoids suture marks either side of the wound or incisionQ.

44. Ans. d. Avulsion injury (Ref: Bailey 27/e p27)

45. Ans. d. Repair of blood vessels

46. Ans. a. Vascular injury



47. Ans. a. Colour of the muscle; d. Muscle contractility; e. Punctate bleeding spots on cut edge (Ref: Bailey 25/e p354-356)

Muscle Viability is Detected by ‘4C’

Section 6

• • • •

ColourQ: Dead muscle has dark unhealthy colour, has lost its sheen ContractilityQ: Dead muscles do not twitch when held by forceps. ConsistencyQ: Dead muscle has lost its turgor and is mushy in consistency Capillary bleedingQ: Dead muscle does not bleed at cut ends.



48. Ans. c. Intact fascia



49. Ans. b. Separation of skin + subcutaneous tissue (Ref: Bailey 27/e p27)

SECTION

7

Neurosurgery CHAPTERS ˆˆ ˆˆ

Cerebrovascular Diseases CNS Tumors

CHAPTER

33

Cerebrovascular Diseases

ARNOLD-CHIARI MALFORMATION Arnold-Chiari malformation Type I Chiari malformation • Displacment of cerebellar tonsil into cervical canalQ • Associated with syringomyelia of cervical canal • Typically produces symptoms during adolescence or adult lifeQ • Not associated with hydrocephalus.Q • Patients complain of recurrent headache, neck pain, urinary frequency, and progressive lower extremity spasticityQ.

Type II Chiari malformation • Lesion represents an anomaly of the hindbrain • Characterized by elongation of the 4th ventricle and kinking of the brainstem, with displacement of the inferior vermis, pons, and medulla into the cervical canalQ. • Type II Chiari malformation is characterized by pro­gressive hydrocephalus with a myelomeningoceleQ. • Plain skull radiographs show a small posterior fossa and a widened cervical canalQ. • CT scanning with contrast and MRI display the cerebellar tonsils protruding downward into the cervical canal and the hindbrain abnormalities. • The anomaly is treated by surgical decompressionQ.

VERTEBRAL BODY ANOMALIES Vertebral Body Anomalies Spina Bifida Occulta

• Meningocele

Spina Bifida Aperta • Myelomeningocele

• Myeloschisis

MENINGOCELE Meningocele • Herniation of meninges through a defect in the posterior vertebral archesQ. • Spinal cord is usually normal and assumes a normal position in the spinal canal • There may be tethering, syringomyelia, or diastematomyelia. Clinical Features • A fluctuant midline mass, that may transilluminate occurs along the vertebral column, in the lower backQ. • Most meningoceles are well-covered with skin and pose no threat to the patient. Diagnosis • Plain roentgenograms demonstrate a defectQ. Treatment • Asymptomatic children with normal neurologic findings and full-thickness skin covering the meningocele may have surgery delayedQ. • Patients with leaking CSF or a thin skin covering should undergo immediate surgical treatment to prevent meningitisQ.

MYELOMENINGOCELE Myelomeningocele • Most severe form of dysraphism involving the vertebral column • Incidence: 1/4,000 live birthsQ • MC site of myelomeningocele: Lumbosacral region (75%)Q

864

Surgery Essence Etiology • Genetic predisposition exists; the risk of recurrence after one affected child increases to 3-4% and increases to 10% with two previous abnormal pregnancies. • Nutritional and environmental factors: FolateQ is intricately involved in the prevention and etiology of NTDs. • Maternal peri-conceptional use of folic acid supplementation reduces the incidence of neural tube defects in pregnancies at risk by at least 50%Q. • To be effective, folic acid supplementation should be initiated before conception and continued until at least the 12th week of gestation when neurulation is completeQ.

Neurosurgery

Prevention • All women of childbearing age and who are capable of becoming pregnant take 0.4 mg of folic acid daily. Drugs increasing the risk of myelomeningocele • TrimethoprimQ • CarbamazepineQ • PhenytoinQ

• PhenobarbitalQ • PrimidoneQ • Valproic acidQ

Clinical Features • Produces dysfunction of skeleton, skin, gastrointestinal and genitourinary tracts, in addition to the peripheral nervous system and CNSQ. • Extent and degree of the neurologic deficit depend on the location of the myelomeningocele, as well as the associated lesions. Location

Manifestation

Low sacral region

Bowel and bladder incontinence associated with anesthesia in the perineal area but with no impairment of motor function.

Midlumbar region

Flaccid paralysis of lower extremity, absence of deep tendon reflexes, lack of response to touch and pain

Thoracic region

Increasing neurologic deficit as the myelomeningocele extends higher into the thoracic region.

Upper thoracic and cervical region

Very minimal neurological deficit and no hydrocephalus

DANDY-WALKER MALFORMATION Dandy-Walker Malformation • • • •

MC posterior fossa malformationQ Prevalence: 1 per 30,000 live births Consists of a cystic expansion of 4th ventricle in the posterior fossa & midline cerebellar hypoplasiaQ Results from a developmental failure of the roof of the 4th ventricleQ during embryogenesis. Dandy-Walker Malformation is characterized by the triad of • Hypoplasia of vermisQ • Cephalad rotation of the vermian remnant & cystic dilatation of 4th ventricle extending posteriorlyQ  • Enlarged posterior fossa with torcular-lambdoid inversionQ

Section 7

Clinical Features • • • •

MC manifestation: MacrocephalyQ (80% cases); Associated with hydrocephalusQ (90% cases) Infants present with a rapid increase in head size & prominent occiputQ. Transillumination of the skull may be positive. Most children have evidence of long-tract signs, cerebellar ataxia, and delayed motor & cognitive milestones, probably due to the associated structural anomaliesQ.

Associations • CNS abnormalities are present in 70% cases (i.e, Agenesis of the posterior cerebellar vermis & corpus callosumQ. cortical dysplasia, polymicrogyria) Diagnosis • IOC for diagnosis: MRIQ

Cerebrovascular Diseases

865

Treatment Prognosis • High (~70%) mortality rate due to associated abnormalities • Poorer prognosis if diagnosed prior to 21 weeks of gestation

CAUSES OF CEREBROVASCULAR ACCIDENTS Causes of Cerebrovascular Accidents

Ischemic (85%) • Embolic (75%) • Thrombotic (25%)

Hemorrhage (15%) • • • •

Q

Parenchymal (10%)Q Subarachnoid (1-2%) Epidural (1 cm or any symptomatic SDH should be surgically drained. • Chronic SDH typically consists of a viscous fluid, with a texture and dark brown color reminiscent of motor oil. • A simple burr hole can effectively drain most chronic SDHs. • The procedure is converted to open craniotomy if the SDH is too congealed for irrigation drainage, the complex of membranes prevents effective drainage, or persistent hemorrhage occurs that cannot be reached with bipolar cautery through the burr holeQ.

Section 7

• Follow-up head CT scans: Approximately 1 month later to document resolution. • Harrison says “Epidural hematomas occur in upto 10% of severe head injury cases and are less often associated with underlying cortical damage than subdural hematomas.”Q

SUBARACHNOID HEMORRHAGE Subarachnoid Hemorrhage • MC cause: Trauma >Spontaneous rupture of Berry aneurysmQ Clinical Features • Sudden transient loss of consciousnessQ (occurs in nearly half of the patients) • Excruciating severe headacheQ: presenting complaint in 45% of cases (worst headache of patients life) more common upon regaining consciousness when loss of consciousness is associated

Cerebrovascular Diseases

• Associated prodromal symptoms (suggest location of progressively enlarging unruptured aneurysm): −− Third cranial nerve palsyQ: Aneurysm at junction of PCA and ICA −− Sixth nerve palsyQ: Aneurysm in cavernous sinus −− Occipital and posterior cervical pain: Inferior cerebellar artery aneurysm −− Pain in or behind the eyeQ: MCA aneurysm Diagnosis • Noncontrast CT scan: Investigation of choice (Lumbar puncture is not indicated prior to an imaging procedure) • CSF picture: Hallmark of aneurysmal rupture is blood in CSF (Xanthochromic spinal fluidQ)

Section 7

• Neck stiffness and vomitingQ: are common associations • Focal neurological deficit: uncommon. • Sudden headache in the absence of focal neurological deficit is the hallmark of aneurysmal rupture.Q

867

• Lumbar puncture should be performed, if the CT scan fails to establish the diagnosis of SAH and no mass lesion or obstructive hydrocephalus is found to establish the presence of subarachnoid blood.Q Treatment • Traumatic subarachnoid hemorrhage is managed conservativelyQ.

DIFFUSE AXONAL INJURIES (DAI) Diffuse Axonal Injuries (DAI) • DAI represents the presence of widespread axonal damage (white matter) in both hemispheres secondary to severe head injuryQ. • Results from application of severe acceleration/deceleration or angular strain to the brain (injuries to axons by shearing forceQ) • MC location: Lobar white matter at the junction of grey and white matterQ > Corpus callosum > Brain stem Pathology • Hemorrhagic or non-hemorrhagic white matter tears in both hemispheres. Clinical Features • • • •

Clinical presentation vary from concussion to coma Loss of consciousness is a common findingQ DAI: MC cause of post-traumatic vegetative stateQ Raised ICT may or may not be associatedQ

• MRI is IOC for DAIQ (better than CT scan). Prognosis • DAI carries an extremely poor prognosisQ. Brain Injury Primary Brain Injury • Primary brain injury occurs at the time of impact • Includes injuries such as: −− Brainstem and hemispheric contusionsQ −− Diffuse axonal injuryQ −− Cortical lacerationsQ

Secondary Brain Injury • Secondary brain injury occurs at some time after the moment of impactQ • PreventableQ • Principle causes: Hypoxia, hypotension, raised ICP, reduced cerebral perfusion pressure and pyrexiaQ

CEREBRAL CONTUSIONS Cerebral Contusions • Cerebral contusions result from the brain being damaged by: −− Impacting against the skull either at the point of impact (the ‘coup’) or on the other side of the head (‘contre-coup’)Q −− As the brain slides forwards and backwards over the ridged cranial fossa floor (most often affecting the inferior frontal lobes and temporal poles) Q

Neurosurgery

Diagnosis

868

Surgery Essence Diagnosis • CT scan: Heterogeneous with mixed areas of high and low densityQ. There may be an associated mass effect. Contusion appears uniformly hyperdense. Treatment • Cerebral contusions rarely require immediate surgical treatment. • Patient with cerebral contusions must be admitted for observation as these lesions will tend to mature and expand for 48–72 hours following injuryQ.

Neurosurgery

• Small proportion of cerebral contusions will require delayed surgical evacuation to reduce the mass effectQ.

SKULL BASE FRACTURES Skull Base Anterior Cranial Fossa • Formed by frontal boneQ

Middle Cranial Fossa • Formed by temporal & sphenoid boneQ

Posterior Cranial Fossa • Formed by occipital boneQ

Cribriform plate: Sieve like structure between anterior cranial fossa & nasal cavity Anterior Cranial Fossa Fracture

Middle Cranial Fossa Fracture

Posterior Cranial Fossa Fracture

• MC type of skull base fractureQ • Caused by fracture of cribriform plateQ • Clinical features: −− Subconjunctival hematomaQ −− CSF rhinorrheaQ −− EpistaxisQ −− AnosmiaQ −− Periorbital hematoma or “Raccoon eyes”Q −− Carotico cavernous fistulaQ −− Frontal lobe contusionQ

• Caused by fracture of petrous part of temporal boneQ • Clinical features: −− CSF otorrheaQ −− Paradoxical rhinorrheaQ −− HemotympanumQ −− Battle signQ: Bruising or ecchymosis behind ear −− Ossicular disruptionQ
 −− VII & VIII cranial nerve palsiesQ −− Temporal lobe contusionQ

• Caused by fracture of occipital boneQ • Clinical features: −− Visual disturbancesQ −− VI cranial nerve injuryQ −− Jugular foramen syndrome (Vernet syndromeQ): Paresis of IX, X, XI cranial nervesQ −− Basilar artery injuryQ −− Occipital contusionQ

STEPWISE APPROACH TO TREATMENT OF ELEVATED INTRACRANIAL PRESSURE Stepwise Approach to Treatment of Elevated Intracranial Pressure

Section 7

• Insert ICP monitor—ventriculostomyQ versus parenchymal device • General goals: maintain ICP 20–25 mm Hg for >5 min: −− Drain CSF via ventriculostomyQ (if in place) −− Elevate head of the bedQ; midline head position −− Osmotherapy—mannitolQ 25–100 g q4h as needed (maintain serum osmolality 320 mOsmol/kg (ComedK 2010) b. Arterial PaCO2 100 mm Hg d. Arterial pH 7.4 Which of the following is not correct about head injury?  (Recent Question 2018) a. MRI needed to assess hemorrhage b. GCS assessment helps in prognosis c. Hematoma must be operated d. All of the above (Punjab 2011)

54.

Cerebral edema is not caused by: a. Lead toxicity b. Craniosynostosis c. Corticosteroids administration d. Vitamin A intoxication

55.

Not a primary brain injury: a. Diffuse axonal injury c. Concussion

56.

Which among the following is a not a primary brain injury? (JIPMER 2010) a. Cortical lacerations b. Brainstem herniation c. Diffuse axonal injury d. Brainstem contusion

57.

About cranial trauma false is: (AIIMS Nov 2010) a. Raccoon eyes seen in subgaleal hemorrhage b. Depressed skull fracture is associated with brain injury at the immediate area of impact c. Caroticocavernous fistula occur in base skull fracture d. Post traumatic epilepsy seen in 15%

58.

Which is an ominous sign in case of severe head injury? a. Development of diabetes insipidus (PGI Nov 2010) b. Anisocoria c. New focal deficit d. Depressed skull fracture e. Decorticate posturing

59.

A 25 years old male of head injury was brought to emergency in unconscious state. What is the name of sign seen in this image?

(Punjab 2011) b. Contusion d. Intracerebral hematoma

a. Duret hemorrhage b. Battle sign c. Kernohan’s phenomenon d. Raccoon sign 60.

Battle’s sign is present in:  (MHSSMCET 2006, 2005) a. Anterior cranial fossa fracture b. Middle cranial fossa fracture c. Posterior cranial fossa fracture d. Fracture lesser wing of sphenoid

Cerebrovascular Diseases 61.

63.

71.

The earliest manifestations of increased intracranial pressure following head injury is: (All India 2005) a. Ipsilateral papillary dilatation b. Contralateral papillary dilatation c. Altered mental status d. Hemiparesis

72.

A 65 years old male was brought to casualty with history of RTA. NCCT was done. What is the diagnosis?  (Recent Question 2017) a. EDH b. SDH c. Subarachnoid hemorrhage d. None of the above

Section 7

62.

In a vehicular accident, extensive contusions of brain due to acceleration and deceleration injury indicate what kind of injury? (MHSSMCET 2006) a. Penetrating injury b. Coup-Countercoup injury c. Second impact syndrome d. Crush injury The most common neurologic abnormality that occurs with head injury is: (Karnataka 2005) a. Hemiplegia b. Ocular nerve palsy c. Altered consciousness d. Convulsion Cushing reflex is: (UPPG 2007) a. ↑Mean arterial pressure with increased intracranial pressure b. ↑Mean arterial pressure with decreased intracranial pressure c. ↓Mean arterial pressure with increased intracranial pressure d. ↓Mean arterial pressure with decreased intracranial

873

pressure

64.

65.

66.

67.

68.

70.

True regarding epidural hematoma is/are: (PGI May 2018) a. Arterial bleed b. On CT scan it gives biconvex lenticular hyperdense appearance c. Located on lateral side of hemisphere d. Common after injury at pterion

73.

False statement regarding subdural hematoma:  (JIPMER May 2018) a. Occurs on both sides b. Not visible on X-ray c. Surgery can be done d. Unilateral surgery

74.

Transtentorial uncal herniation causes all except: (AIIMS May 2001) a. Ipsilateral dilated pupils b. Ipsilateral hemiplegia c. Cheyne stokes respiration d. Decorticate rigidity

75.

A patient present with unilateral painful ophthalmoplegia. Imaging revealed an enlargement of cavernous sinus on the affected side. The likely diagnosis is: (AIIMS May 2008) a. Gradenigo syndrome b. Cavernous sinus thrombosis c. Tolosa-Hunt Syndrome d. Orbital Pseudotumor

76.

Non-noxious stimuli perceived as pain is termed as:  (AIIMS May 2008) a. Allodynia b. Hyperalgesia c. Hyperesthesia d. Hyperpathia

77.

Spontaneous CSF leaks may be associated with all except: a. Increased Intracranial Tension  (AIIMS Nov 2008) b. Pseudotumor cerebri c. Empty Sella Syndrome d. Encephalocele

78.

All of the following statements about Diffuse Axonal Injury (DAI) are true except: (All India 2008) a. Caused by shearing force b. Predominant white matter hemorrhages, in basal ganglion and corpus callosum c. Increased intracranial tension is seen in all cases d. Most common at junction of grey and white matter

79.

Retraction ball is seen in: a. Burns c. Diffuse axonal injury

80.

Neurosurgery is indicated for all except: (Recent Question 2013) a. SDH b. EDH c. Depressed fracture d. Diffuse axonal injury

(Recent Question 2015) b. Acute pancreatitis d. Tracheobronchial injury

Neurosurgery

69.

Raised intracranial pressure will cause: (MCI March 2007) a. Tachycardia b. Hypotension c. Papilloedema d. Normal looking anterior fontenalle in infants True statement regarding fracture base of the skull are all of the following except: a. Prophylactic antibiotics are usually not required b. Associated with 8th cranial nerve palsy c. Early surgery is indicated for optimal outcome d. May present with CSF otorrhoea The cause of systemic secondary insult to injured brain include all of the following except:  (AIIMS May 2006) a. Hypercapnia b. Hypoxemia c. Hypotension d. Hypothermia In a patient with head injury damage in the brain is aggravated by: (All India 2010) a. Hyperglycemia b. Hypothermia c. Hypocapnia d. Serum osmolality Best prognostic factor for head injury is: (All India 2007) a. Glasgow coma scale b. Age c. Mode of injury d. CT A 25 years old male was brought to casualty with history of RTA. NCCT was done. What is the diagnosis? a. EDH (Recent Question 2017) b. SDH c. Subarachnoid hemorrhage d. None of the above

874

Surgery Essence 81.

Neurosurgery

82.

83.

84.

85.

86.

Patient with a history of fall presents weeks later with headache and progressive neurological deterioration. The diagnosis is: (Recent Question 2016) a. Acute subdural hemorrhage b. Extradural hemorrhage c. Chronic subdural hemorrhage d. Fracture skull

87.

The treatment of post traumatic epilepsy is: a. Mannitol infusion b. Immediate corticosteroids c. Long term anticonvulsants d. Long term corticosteroids Following are the features of raised intracranial tension except: a. Altered sensorium b. Papilloedma c. Convulsions d. Tachycardia Facial nerve palsy is seen in the following fracture: a. Anterior cranial fossa b. Middle cranial fossa c. Cranial vault d. Posterior cranial fossa The most important clinical finding in a case of head injury is: (JIPMER 91) a. Pupillary dilatation b. Level of consciousness c. Focal neurological deficit d. Fracture skull

88.

89.

90.

91.

Section 7

A 24-year-old man falls on the ground when he is struck in the right temple by a baseball. While being driven to the hospital, he lapses into coma. He is unresponsive with the dilated right pupil when be reaches the emergency department. The most important step in initial management is: (BIHAR PG 2014; All India 2002) a. Craniotomy b. CT scan of the head c. X-ray of the skull and cervical spine d. Doppler ultrasound examination of the neck All of the following lower intracranial pressure except: a. Mannitol b. Furosemide (All India 98) c. Corticosteroids d. Hyperventilation In skull fracture, the condition in which an operation is not done immediately is: (All India 96) a. Depressed fracture b. Compound fracture c. CSF leak d. Increased size of head Surgery is not useful in: a. Cerebral edema b. Depressed fracture c. Extradural hemorrhage d. Subdural hemorrhage In a patient with head injury black eye associated with subconjunctival hemorrhage occurs when there is: a. Fracture of floor of anterior cranial fossa b. Bleeding between the skin and galea aponeurotica c. Hemorrhage between galea aponeurotica and pericranium d. Fracture of greater wing of sphenoid bone

92.

Signs of cerebral compression are all except:  (Recent Question 2016) a. Bradycardia b. Hypotension c. Papilloedma d. Vomiting In patient of head injury with rapidly increasing intracranial tension without hematoma, the drug of choice for initial management would be: (UPSC 2000) a. Lasix b. Steroids c. 20% Mannitol d. Glycine

93. All of the following are indications of CT scan in head injured patient except: (DNB 2014) a. GCS < 13 b. Vomiting 1 episode c. Focal neurological deficit d. Mild head injury in patient Age > 65 years

94.

95.

All of the following are the components of Cushing triad except: (Recent Question 2017) a. Bradycardia b. Hypertension c. Pupillary dilatation d. Respiratory irregularity Management of raised ICP are all except: a. Hypothermia (Recent Question 2017) b. Hypercapnia c. Decompressive craniectomy d. Barbiturate

GLASGOW COMA SCALE True about Glasgow coma scale: (JIPMER 2011) a. Includes verbal response b. Includes papillary reflex c. High score means poor prognosis d. Includes measurement of intracranial pressure 97. A patient who had traumatic head injury has spontaneous eye opening, tries to remove examiners hands on painful stimuli and irrelevant talks/stances. What is the GCS score?  (AIIMS November 2017) a. 11 b. 13 c. 3 d. 1 98. A patient of motor vehicle accident was admitted to the casualty. He does not speak but moans every now and then, eyes are closed but opens to pain, the right limb is not moving but the left limb shows movement to pain. Both the legs are in extended posture. What will be the GCS score?  (AIIMS May 2017) a. 5 b. 7 c. 9 d. 11 99. Glasgow outcome score of vegetative state:  (Recent Question 2017) a. 2 b. 3 c. 5 d. 8 100. Best predictor in the GCS:  (Recent Question 2017) a. Eye opening b. Motor response c. Verbal response d. All 101. A patient with head injury opens eyes to painful stimulus, uses inappropriate words, and localizes pain, what is his GCS score? (MHCET 2016, All India 2011) a. 8 b. 10 c. 12 d. 14 96.

102. Minimal Glasgow coma scale is: (Recent Question 2015, DNB 2012, UPPG 2010, MCI March 2007) a. 0 b. 1 c. 2 d. 3 e. 4 103. Mild head injury is having Glasgow coma scale of: a. 3-5 b. 5-8 (SGPGI 2005) c. 8-10 d. 13-15 104. Which of the following is not a component of Glasgow coma scale? (DNB 2009, All India 2006) a. Eye opening b. Motor response c. Pupil size d. Verbal response 105. Total score in Glasgow coma scale of a conscious person is: a. 8 b. 3 (All India 2006) c. 15 d. 10 106. Regarding Glasgow coma scale, which is not true? a. Ranges from 6-12 (AIIMS Nov 94) b. Low score indicates deteriorating brain function c. Based on eye opening, verbal response and motor response d. Score below 5 shows poor prognosis

Cerebrovascular Diseases

108. A person with inappropriate words evaluated by GCS will have a verbal score of: (All India 2012) a. 4 b. 3 c. 2 d. 1

118. Commonest site of meningocele is:  (BIHAR PG 2014, DNB 2005, 2001, All India 89) a. Lumbosacral b. Occipital c. Frontal d. Thoracic 119. Which of the following statement is correct?

109. Prognostic factor in head injury: (Recent Question 2013) a. Age of patient b. Glasgow coma scale c. Mode of injury d. Presence of facial trauma 110. What are the minimum and maximum possible values of Glasgow Coma Score? (AIIMS November 2015) a. Minimum = 3, Maximum = 15 b. Minimum = 0, Maximum = 13 c. Minimum = 0, Maximum = 15 d. Minimum = 3, Maximum = 18 111. Glasgow coma scale of a patient with head injury, who is confused, able to localize on right side and does flexion on left side and opens eye for painful stimuli on sternum:  (Recent Question 2018) a. 6 b. 11 c. 12 d. 7

BRAIN ABSCESS 112. Management of epidural abscess is: a. Immediate surgical evaluation b. Conservative management c. Antibiotics d. Aggressive debridement

(DNB 2011)

114. Brain abscess may be due to the following: a. Chronic suppurative otitis media b. Chronic lung abscess c. Trauma d. Any of the above 115. Subdural collection of pus in a head injury patients after 3 days, the responsible organism is: (UPPG 2009) a. Staph aureus b. B hemolytic streptococcus c. H. influenza d. Pneumococcus 116. A young female patient with long history of sinusitis presented with frequent fever along with personality changes and headache of recent origin. The fundus examination revealed papilledema. The most likely diagnosis is:  (AIIMS Nov 2004) a. Frontal lobe abscess b. Meningitis c. Encephalitis d. Frontal bone ostemomyelitis

CNS CONGENITAL ANOMALIES 117. Meningomyelocele patient after being operated developed hydrocephalus due to: (PGI Dec 98) a. Arnold Chiari malformation b. Injury to absorptive surface c. Central canal injury d. Arachnoidal block

a. 1-Meningocele, 2-Meningomyelocele b. 1-Meningocele, 2-Encephalocele c. 1- Encephalocele, 2-Meningomyelocele d. 1- Encephalocele, 2- Meningocele 120. Not true regarding Dandy-Walker cyst: (AIIMS June 98) a. Cerebellar vermis hypoplasia b. Hydrocephalus c. Arachnoid cyst d. Posterior fossa cyst 121. What will be the diagnosis of the child with pulsatile swelling on medial side of the nose? (AIIMS June 98) a. Teratoma b. Meningocele c. Dermoid cyst d. Carcinoma of ethmoid bone 122. A new born presents with swelling in base of the spine in which meninges herniates through bony defect cause is? a. Defect in pedicle  (UPPG 2009) b. Defect in body c. Defect in fusion of vertebral arches d. Defect is transverse process 123. A new born with meningomyelocele has been posted for surgery. The defect should be immediately covered with:  (AIIMS May 2013, All India 2012) a. Normal saline guaze b. Povidone iodine guaze c. Tincture benzoin guaze d. Methylene blue guaze

HYDROCEPHALUS 124. In normal pressure hydrocephalus, all are seen except:  (PGI Dec 97) a. Convulsion b. Ataxia c. Dementia d. Incontinence 125. Most commonly performed shunt for hydrocephalus is: (AIIMS May 2015) a. Ventriculoperitoneal b. Ventriculopericardial c. Ventriculopleural d. Lumboperitoneal

NERVE COMPRESSION SYNDROME 126. Carpal tunnel syndrome is due to the compression of: a. Median nerve (JIPMER 2011, ComedK 2006) b. Anterior interosseous nerve c. Radial nerve d. Ulnar nerve 127. Carpel tunnel syndrome is caused by all except:  (Recent Question 2016, AIIMS May 2011) a. Amyloidosis b. Hypothyroidism c. Addison’s disease d. Diabetes mellitus

Neurosurgery

113. Brain abscess in cyanotic heart disease is commonly located in: (All India 2006) a. Cerebellar hemisphere b. Thalamus c. Temporal lobe d. Parietal lobe

Section 7

107. All are true about Glasgow coma scale, except: a. Score between 3 and 15 (AIIMS June 94) b. Obeying motor command is given maximum score c. Consists of eye opening, motor and verbal response d. Increased score indicates poor prognosis

875

876

Surgery Essence 128. In causalgia, the nerve most commonly affected are: a. Radial and ulnar b. Median and sciatic c. Radial and peroneal d. Ilioinguinal and sural 129. Commonest cause of carpal tunnel syndrome is: a. Malunited Colle’s fracture  (All India 95) b. Rheumatoid arthritis involving flexor retinaculum c. Myxedema d. Pregnancy

Neurosurgery

NERVE INJURIES 130. Tinnel’s sign indicates: a. Atrophy of nerves c. Injury to nerve

(Kerala 91)

b. Neuroma d. Regeneration of nerves

131. Bilateral phrenic nerve palsy is caused by: a. Carcinoma bronchus b. Polio c. Medullary carcinoma thyroid d. Paget’s disease 132. After an open injury, the optimum time for nerve suture is: a. Immediately b. Within one moth c. 1-2 month d. 2-4 month e. When wound is free from infection 133. In Erb-Duchene paralysis, the injury is limited to the: a. 2nd and 3rd cervical nerves  (COMED-K 2008) b. 3rd and 4th cervical nerves c. 4th and 5th cervical nerves d. 5th and 6th cervical nerves 134. Peripheral nerves can withstand ischemia up to: (JIPMER 93) a. 30 minutes b. 1 hour c. 2 hours d. 4 hours

MISCELLANEOUS 135. Which of the following is the most common location of intracranial neurocysticercosis? (AIIMS Nov 2005) a. Brain parenchyma b. Subarachnoid space c. Spinal cord d. Orbit 136. The nerve of Kuntz is an important landmark in: a. Lumbar sympathectomy b. Cervicodorsal sympathectomy c. Obturator neurectomy d. Splanchnicectomy e. Herniorraphy

Section 7

137. A dome shaped skull with a high forehead in the infant with slight hydrocephalus (Olympian brow) is seen in:  (Recent Question 2016) a. Marasmus b. Congenital syphilis c. Rickets d. Arnold Chiari syndrome 138. The parasitic infection capable of producing spinal cord compression is/are: a. Leishmaniasis b. Wuchereriasis c. Echinococcosis d. Amoebiasis 139. The following are CNS findings of CO2 narcosis: (PGI 90) a. Excitement b. Increased pH of CSF c. Decreased pH of CSF d. Papilledema

140. All of the following conditions are known to cause diabetes insipidus except: (AIIMS 2004) a. Multiple sclerosis b. Head injury c. Histiocytosis d. Viral encephalitis 141. Cells from the neural crest are involved in all except: a. Hirschprung’s disease  (AIIMS June 2003) b. Neuroblastoma c. Primitive neuroectodermal tumor d. Wilm’s tumor 142. All can commonly occur in a patient who suffered decelera­ ting injury in which pituitary stalk was damaged. except one: a. Diabetes mellitus (AIIMS Nov 2000) b. Thyroid insufficiency c. Adrenocortical insufficiency d. Diabetes insipidus 143. The defective migration of neural crest cells results in: a. Congenital megacolon b. Albinism  (PGI June 2006) c. Adrenogenital hypoplasia d. Dentinogenesis imperfect 144. Premature filling of veins is a manifestation in cerebral angiography of: a. Trauma b. Brain tumor c. Arteriovenous malformation d. Arterial occlusion 145. Neurosurgical treatment of epilepsy usually involves, removal of epileptic focus from which lobe: (Karnataka 2003) a. Frontal lobe b. Temporal lobe c. Occipital lobe d. Parietal lobe 146. Blow out fracture refers to: a. Fracture of orbit c. Fracture base of skull

(JIPMER 2011) b. Fracture of nasal septum d. Fracture of mandible

147. The ‘Phenomenon of Kernohan’s notch’ is associated with:  (MHSSMCET 2006) a. Third nerve palsy with contralateral hemiplegia b. Subfalacine herniation c. Transtentorial herniation d. Foramen magnum fracture 148. Signs of base of skull fracture are following except: a. Raccoon eyes (MHSSMCET 2011) b. Battle’s sign c. Constricted pupil d. Hemotympanum 149. A newborn present with congestive heart failure. On examination has bulging anterior fontenalle with a bruit on auscultation. Trans-fontenallar USG shows a hypoechoic midline mass with dilated lateral ventricles. Most likely diagnosis: (AIIMS Nov 2011, May 2010, Nov 2006) a. Medullloblastoma b. Encephalocele c. Vein of Galen malformation d. Arachnoid cyst

Explanations BERRY ANEURYSM

1. Ans. a. Junction of anterior communication artery with anterior cerebral artery (Ref: Harrison 20/e p1904; Sabiston 20/e p1904; Schwartz 9/e p1534; Bailey 26/e p311)



2. Ans. b. Vertebral artery



3. Ans. c. Wall contains smooth muscle fibroblasts (Ref: Harrison 19/e p01784)



4. Ans. d. Intracranial hemorrhage

5. Ans. b. Vertebral artery

CEREBROVASCULAR ACCIDENTS

6. Ans d. Aplastic anemia

7. Ans. a. Putamen (Ref: Harrison 20/e p3092, 19/e p2582)



8. Ans. a. Embolism (Ref: Harrison 20/e p3069, 19/e p2560)

9. Ans. a. Cortical vein thrombosis (Ref: Harrison 19/e; p2566)

• Young women on OCPs are predisposed to stroke due to venous thrombosis of lateral saggital sinus or small cortical veins (cortical vein thrombosis)Q.

10. Ans. b. Intracerebral hemorrhage

11. Ans. c. Hypertension

12. Ans. c. Putamen/external capsule



13. Ans. a. Rupture of circle of Willis aneurysm

14. Ans. b. Vasospasm (Ref: Harrison 20/e p2086, 19/e p1785)

Vasospasm • Narrowing of the arteries at the base of the brain following SAH occurs regularly. • This vasospasm causes symptomatic ischemia and infarction in approximately 30% patients and is the major cause of delayed morbidity or deathQ. • Sign of ischemia appear 4-14 days after the hemorrhage, most frequently at about 7 daysQ. Major Causes of Delayed Neurological Deficit after CVA Re-ruptureQ HydrocephalusQ

15. Ans. b. Ischemic stroke

VasospasmQ HyponatremiaQ 16. Ans. a. Brain (Ref: Robbins 9/e p1255)

Duret Hemorrhage • In case of increased ICP down ward herniation of brainstem occur, which cause stretching of perforators of basilar artery and may results in bleed (Duret hemorrhage)Q. • Duret hemorrhage is small area of bleeding in ventral and paramedian part of upper brainstem (midbrain and pons)Q. • It usually indicates a fatal outcome, however survival has been reported. • Diagnosis is made on CT or MRI.

17. Ans. a. Blunt cerebrovascular trauma (Ref: Sabiston 20/e p424; Schwartz 10/e p198) “Digital subtraction angiography subsequently confirmed blunt cerebrovascular injuries (BCVI) in 30% of this high-risk cohort. Commonly referred to as the Denver criteria, these risk factors are used to screen patients and to prompt further evaluation.”-Sabiston 20/e p424

EDH AND SDH

18. Ans. a. Cortical vessels (Ref: Sabiston 20/e p1916; Schwartz 10/e p1719; Bailey 27/e p334)



19. Ans. d. Extradural hematoma

20. Ans. d. Epidural hemorrhage



21. Ans. b. Temporoparietal

22. Ans. c. Subacute (Ref: Bailey 27/e p334,335; Schwartz 10/e p1719-1720)

Classification of SDH • Acute SDH: 21 daysQ

878

Surgery Essence

23. Ans. c. 15 days



26. Ans. d. 21 days (Ref: Harrison 19/e p457e-3; Sabiston 19/e p439-441; Schwartz 10/e p1719-1720; Bailey 27/e p335)

24. Ans. d. Surgical evacuation

25. Ans. b. Only P, R, S are correct



27. Ans. b. 10% (Ref: Harrison 19/e p441e-3)

Neurosurgery

• Harrison says “Epidural hematomas occur in upto 10% of severe head injury cases and are less often associated with underlying cortical damage than subdural hematomas.”Q

28. Ans. d. Extradural

29. Ans. b. Immediate evacuation

30. Ans. a. Severe hypotension



31. Ans. c. Chronic subdural hematoma

32. Ans. b. Rupture of superior cerebral vein



33. Ans. a. Subarachnoid hemorrhage (Ref: Harrison 20/e p2084, 19/e p1784-1785; Sabiston 20/e p1916; Schwartz 10/e p1730-1731; Bailey 27/e p335,658,661)



34. Ans. c. Injury to cortical bridging veins



35. Ans. b. Subdural hematoma (Ref: www.ncbi.nlm.nih.gov/pubmed/11915757 by M Guénot - 2001)

Chronic Subdural Hematoma • Virchow, in 1857, denied a traumatic origin, and gave the name of “pachymeningitis hemorrhagica interna”Q to this pathology which he explained by inflammatory processes. • The traumatic etiology of chronic subdural hematoma was recognized in the 20th century, especially by Trotter in 1914.

36. Ans. a. Extradural hemorrhage



38. Ans. d. Left chronic subdural hematoma

37. Ans. d. Left chronic subdural hematoma



39. Ans. c. In the left temporal region if no localizing sign is found (Ref: Ramamurthi and PN Tandon’s Textbook of Neurosurgery 3/e Vol-1/ p442)

Burr hole to emergently relieve the intracranial pressure should be done in the left temporal region if no localizing sign is found, to evaluate and decompress the dominant hemisphere.

Choice of Side for Initial Burr Hole Start with a temporal burr hole on the side: • Ipsilateral to a blown pupil: This will be on the correct side in >85% of epidural hemorrhages and other extra- axial mass lesions. • If both pupils are dilated, use the side of the first dilating pupil (If known). • If pupils are equal, or it is not known which side dilated first, place on side of obvious external trauma. • If no localization clues, place hole on left side (to evaluate and decompress the dominant hemisphere).

SUBARACHNOID AND INTRACRANIAL HEMORRHAGE

40. Ans. c. Aneurysm

41. Ans. a. CT brain (Ref: Harrison 20/e p2084, 19/e p1785, 1786)



42. Ans. c. Berry aneurysm

43. Ans. b. Berry aneurysm rupture



44. Ans. b. Hess and Hunt scale (Ref: Harrison 20/e p2085, 19/e p1785; Sabiston 20/e p1906)

• Hess and Hunt Scale: Most widely used scale to grade the severity of subarachnoid hemorrhage, predict the prognosis / outcome of hemorrhage and thereby plan further interventionQ. Grading Scale for Subarachnoid Hemorrhage

Section 7

Grade

Hunt – Hess Scale

1

Mild headache, normal mental status, no cranial nerve or motor findings

2

Severe headache, normal mental status, may have cranial nerve deficit

3

Somnolent, confused, may have cranial nerve or mild motor deficit

4

Stupor, moderate to severe motor deficit, may have intermittent reflex posturing

5

Coma, reflex posturing or flaccid

Good grade: Grade 1, 2 and 3; Poor grade: Grade 4 and 5

45. Ans. c. Subarachnoid hemorrhage



46. Ans. b. Ruptured aneurysm

47. Ans. c. Aneurysmal subarachnoid hemorrhage



48. Ans. d. Subarachnoid hemorrhage

49. Ans. a. Subarachnoid hemorrhage (SAH)



50. Ans. b. Aneurysmal subarachnoid hemorrhage

Cerebrovascular Diseases

51. Ans. d. Hypothermia (Ref: Comprehensive Board Review in Neurology by Mark K. Borsody (Thieme) p63)

• Triple H therapy of subarachnoid hemorrhage used to ameliorate cerebral perfusion, consists of: 1. Hypervolemia 2. Hypertension 3. Haemodilution

HEAD INJURY 52. Ans. a. Serum osmolality >320 mOsmol/kg (Ref: Harrison 20/e p2077, 19/e p1779-1780; Schwartz 10/e p195-197, 575-578, 1715-1721; Bailey 26/e p318)

Section 7

Triple H Therapy



879

Mannitol • Mannitol is widely used to reduce ICPQ • Commonly used preparation: 20% solution, 0.25-1 gm/kg is given IV as bolusQ • Serum osmolality should not be allowed to go >320 mOsm/L, to avoid systemic acidosis and renal failureQ. Uses of Mannitol • To reduce increased ICT or intraocular tensionQ • To maintain GFR and urine flow in impending renal failureQ • Forced diuresis in hypnotic or other poisoningQ • To counteract low osmolality of plasma/ECFQ due to rapid hemodialysis or peritoneal dialysis Contraindication of Mannitol Acute tubular necrosisQ AnuriaQ pulmonary edemaQ

Acute left ventricular failureQ CHFQ Cerebral hemorrhageQ



53. Ans. c. Hematoma must be operated



54. Ans. c. Corticosteroids administration

Steroids are used in treatment of cerebral edema

55. Ans. d. Intracerebral hematoma (Ref: Bailey 27/e p331-332)



56. Ans. b. Brainstem herniation



57. Ans. a. Raccoon eyes seen in subgaleal hemorrhage (Ref: Schwartz 10/e p174)



58. Ans. b. Anisocoria, e. Decorticate posturing (Ref: Harrison 19/e p1774)

Head Injury • A sudden enlargement (dilation) of one pupil (anisocoria) is an ominous signQ. • Abnormal posturing (decorticate posturing) a characteristic positioning of the limbs caused by severe diffuse injury or high ICP, is an ominous signQ.

59. Ans. b. Battle sign (Ref: Schwartz 10/e p174; Bailey 27/e p332)



60. Ans. c. Posterior cranial fossa fracture



62. Ans. c. Altered consciousness



63. Ans. a. ↑Mean arterial pressure with increased intracranial pressure (Ref: Schwartz 10/e p1580-1583; Bailey 27/e p328-329)

“Battle’s sign: A skull base fracture may be associated with bruising over the mastoid process.”-Bailey 27/e p332 61. Ans. b. Coup-Countercoup injury (Ref: Bailey 27/e p336, 26/e p317)

Cushing Reflex • The Cushing reflex classically presents as an increase in systolic blood pressure, reduction of the heart rate (bradycardia), and irregular respirationQ. • It is caused by increased intracranial pressureQ.  • These symptoms can be indicative of insufficient blood flow to the brain (ischemia) as well as compression of arteriolesQ.

64. Ans. c. Papilloedema

Neurosurgery

• Otorrhea, rhinorrhea, raccoon eyes and Battle’s sign suggest a basilar skull fractureQ.

880

Surgery Essence 65. Ans. c. Early surgery is indicated for optimal outcome (Ref: Bailey 27/e p333)

Base of Skull Fractures • Base of skull fractures may be associated with 7th or 8th nerve palsiesQ. • CSF otorrhoea or rhinorrhoea often resolves spontaneouslyQ. • Antibiotics are not required prophylactically unless for concomitant facial fracturesQ. • A delayed craniotomyQ and anterior fossa dural repair is occasionally requiredQ for persistent CSF leak to prevent meningitis.

66. Ans. d. Hypothermia (Ref: Bailey 25/e p299-300; Harrison 19/e p457e-1)

Neurosurgery

Causes of Secondary Brain Injury Hypoxia : PO2 20 mm Hg Low cerebral perfusion pressureQ: CPP one episode Indications for CT Imaging within 8 Hours 1. Age > 65 years 3. Dangerous mechanism of injury (CT within 8 hours) 2. Coagulopathy (e.g. on warfarin) 4. Retrograde amnesia > 30 min

94. Ans. c. Pupillary dilatation (Ref: Schwartz 10/e p1713; Bailey 27/e p328)

“Cushing’s triad is the classic presentation of intracranial hypertension, bradycardia, and irregular respirations.”- Schwartz 10/e p1713

95. Ans. b. Hypercapnia (Ref: Sabiston 20/e p1919; Bailey 27/e p336)

GLASGOW COMA SCALE

96. Ans. a. Includes verbal response (Ref: Harrison 20/e p3183, 19/e p1777; Sabiston 20/e p1918; Schwartz 10/e p168,1711; Bailey 27/e p331, 26/e p312)

Revised Glasgow Coma Scale 2014

Eye Opening (E) Spontaneous 4

Revised GCS (2014) Verbal Response (V) Best Motor Response (M) Oriented 5 Obeying commands

To Speech To PressureQ

3 2

Confused WordsQ

4 3

1

SoundsQ None

2 1

Q

• • •

5 4

Abnormal flexion 3 Extension 2 None 1 GCS specifically recommends avoiding sternal rubsQ as it causes bruising & responses can be difficult to interpret. They also do not recommend routine use of retromandibular pressureQ. Revised GCS (2014) changes are highlighted in the above table. Maximum score-15Q, minimum score-3Q. Best predictor of outcome: Motor responseQ None



Localizing Normal flexion (withdrawal)

6

Section 7

• Reporting of Non-testable Score Aspects: In cases of a non-testable aspect, the new GCS should only be noted in its components. Any element that cannot be tested should be marked as NT, for “not testable”. • For intubated patients or patients with tracheostomy, VNT is used. It is no longer recommend to assign 1 point to nontestable elements, therefore a combined score should not be used. GCS-P • GCS-P is calculated by subtracting the Pupil Reactivity Score (PRS) from the Glasgow Coma Scale (GCS) total score: GCS-P = GCS – PRSQ • Pupil reactivity score represents the number of nonreactive pupils (0, 1, or 2)Q. • This number is subtracted from the GCS score (3–15), resulting in the GCS-P (1-15)Q. Pupils Unreactive to Light

PRS

Both pupils 2 One pupil 1 Neither pupil 0 Note: Higher score is assigned to non-reactive pupils.

GCS-PA CT • GCS-PA CT: GCS, Pupils, Age & CT findingsQ • Probability of mortality 6 months after head injury based on the patient’s admission GCS-P and age with no CT abnormality (A), exactly 1 CT abnormality (B), and 2 or more CT abnormalities (C). • Potential CT abnormalities include intracranial hematoma, absent cisterns & SAHQ.

Cerebrovascular Diseases 97. Ans. b. 13 (Ref: Sabiston 20/e p1918; Schwartz 10/e p1712; Bailey 27/e p331)



98. Ans. c. 9 (Ref: Sabiston 20/e p1918; Schwartz 10/e p1712; Bailey 27/e p331)



99. Ans. b. 3 (Ref: Sabiston 20/e p1918; Schwartz 10/e p1712; Bailey 27/e p331)

100. Ans. b. Motor response (Ref: Sabiston 20/e p1918; Schwartz 10/e p1712; Bailey 27/e p331) 101. Ans. b. 10

102. Ans. d. 3

103. Ans. d. 13-15

104. Ans. c. Pupil size

105. Ans. c. 15

106. Ans. a. Ranges from 6-12

107. Ans. d. Increased score indicates poor prognosis 109. Ans. b. Glasgow coma scale

108. Ans. b. 3

110. Ans. a. Minimum = 3, Maximum = 15

111. Ans. b. 11

Section 7



883

BRAIN ABSCESS 112. Ans. a. Immediate surgical evacuation (Ref: Bailey 27/e p656-657) 113. Ans. d. Parietal lobe (Ref: Harrison 20/e p1014, 19/e p900; Sabiston 20/e p1934-1935; Schwartz 10/e p1745; Bailey 26/e p609-610) Brain abscess in congenital heart diseases occur due to hematogenous seeding of blood borne bacteria. These blood borne bacteria bypass the capillary bed due to right to left shunt. They commonly infect parietal and frontal lobes (territory of middle cerebral artery). 114. Ans. d. Any of the above

115. Ans. a. Staph aureus

116. Ans. a. Frontal lobe abscess

CNS CONGENITAL ANOMALIES 117. Ans. a. Arnold Chiari malformation (Ref: Sabiston 20/e p1932; Schwartz 10/e p1745; Bailey 27/e p484, 668) Type II Chiari malformation is characterized by progressive hydrocephalus with a myelomeningocele. 118. Ans. a. Lumbosacral (Ref: Sabiston 20/e p1932; Schwartz 10/e p1750; Bailey 27/e p484, 667) 119. Ans. a. 1-Meningocele, 2-Meningomyelocele (Ref: Sabiston 20/e p1931-1932; Schwartz 10/e p1750; Bailey 27/e p667) 120. Ans. c. Arachnoid cyst (Ref: Sabiston 20/e p1932; Schwartz 9/e p1553) 121. Ans. b. Meningocele

122. Ans. c. Defect in fusion of vertebral arches

123. Ans. a. Normal saline guaze (Ref: Sabiston 20/e p1931-1932; Schwartz 10/e p1645; Bailey 27/e p484, 667) Meningomyelocele should be covered with a non-sticking sterile saline soaked guaze and plastic shield wrap to maintain moisture.

HYDROCEPHALUS 125. Ans. a. Ventriculoperitoneal (Ref: Bailey 27/e p653-656; Nelson 19/e p2008-2011) Most common shunt used for hydrocephalus is ventriculoperitoneal shunt. “Therapy for hydrocephalus depends on the cause. Medical management, including the use of acetazolamide and furosemide, can provide temporary relief by reducing the rate of CSF production, but long-term results have been disappointing. Most cases of hydrocephalus require extracranial shunts, particularly a ventriculoperitoneal shunt. Endoscopic third ventriculostomy (ETV) has evolved as a viable approach and criteria have been developed for its use, but the procedure might need to be repeated to be effective.” - Nelson 19/e p2011

NERVE COMPRESSION SYNDROME 126. Ans a. Median nerve (Ref: Harrison 20/e p3444, 19/e p2221; Sabiston 20/e p2006; Schwartz 10/e p1791; Bailey 27/e p 508)

Carpal Tunnel Syndrome • Carpal tunnel syndrome is an entrapment median neuropathyQ • Causing paresthesia, pain, numbness in the distribution of the median nerve due to its compression at the wrist in the carpal tunnel. Q Risk Factors for Carpal Tunnel Syndrome PregnancyQ Rheumatoid arthritisQ DiabetesQ  Colle’s fracturesQ ObesityQ AmyloidosisQ HypothyroidismQ AcromegalyQ Q Heavy manual work or work with vibrating tools Use of steroids and estrogensQ

Neurosurgery

124. Ans. a. Convulsion (Ref: Harrison 20/e p3112, 19/e p2606; Sabiston 20/e p1930)

884

Surgery Essence Clinical Features • Main symptom: Intermittent  numbness of the thumb, index, long and radial half of the ring finger. Numbness often occurs at nightQ.  • Long-standing CTS leads to permanent nerve damage with constant numbness, atrophy of some of the muscles of the thenar eminence and weakness of palmar abduction. Q • Pain is primarily numbness that is so intense that it wakes one from sleepQ.

Neurosurgery

Diagnosis • Specific clinical tests include: −− Tinel’sQ percussion over the carpal tunnel −− Phalen’s testQ (reproduction of paraesthesia with full wrist flexion) −− Carpal tunnel compression with full wrist flexion • Rarely does electrophysiological testing add to the clinical tests, but it is a good tool for tracking changes. Treatment • Conservative treatments: Use of night splints and corticosteroid injectionQ. • Disease modifying treatment: Surgery to cut the transverse carpal ligamentQ (creating space for the nerve) 127. Ans. c. Addison’s disease

128. Ans. b. Median and sciatic

129. Ans. d. Pregnancy

NERVE INJURIES 130. Ans. d. Regeneration of nerves (Ref: Schwartz 10/e p1798-1799,1805; Bailey 27/e p385) Nerve Injury Neuropraxia • Reversible physiological nerve conduction blockQ • Seen in crutch palsy, tourniquet palsy, and Saturday night palsyQ

Axonotemesis • Loss of conduction due to axonal interruptionQ but the nerve is in continuity and the neural tubes are intact • Seen in closed fractures and dislocations

Neurotemesis • There is complete division of nerveQ (epineurium, perineurium, endoneurium and axon all lost their continuity) • Seen in open wounds

Nerve Repair • A clean cut nerve is best repaired as soon as this can be done safelyQ. • The higher (proximal) the lesion, the worse the prognosisQ. • Pure motor or pure sensory nerves recover better than mixed, because there is less chances of axonal confusion. • Tinel’s sign indicate regeneration of nerveQ. • Rate of regeneration of nerve is 1 mm/dayQ. Prognosis after Nerve Suturing

Section 7

• Radial nerve (best)Q >Median nerve >Ulnar nerve >Peroneal nerve >Sciatic and femoral nerve (worst prognosis)Q 131. Ans. b. Polio

132. Ans. e. When wound is free from infection

133. Ans. d. 5th and 6th cervical nerves

134. Ans. d. 4 hours

MISCELLANEOUS 135. Ans. a. Brain parenchyma 136. Ans. b. Cervicodorsal sympathectomy (Ref: http://www.ncbi.nlm.nih.gov/pmc/articles/PMC1468385/)

Nerve of Kuntz • Nerve of Kuntz: Thoracic origin of a sympathetic supply to the upper limbQ • An inconstant intrathoracic ramus which joined the 2nd intercostal nerve to the ventral ramus of the 1st thoracic nerve, proximal to the point where the latter gave a large branch to the brachial plexus, has become known as the ‘nerve of Kuntz’ • These variant sympathetic pathways may be responsible for the recurrence of symptoms after sympathectomy surgeryQ.

Cerebrovascular Diseases 137. Ans. b. Congenital syphilis

138. Ans. c. Echinococcosis

139. Ans. c. Decreased pH of CSF

140. Ans. a. Multiple sclerosis

141. Ans. d. Wilm’s tumor

142. Ans. a. Diabetes mellitus

efective migration of neural crest results in Hirschprung’s disease (congenital megacolon), albinism, melanoma and oroD pharyngeal teratoma. 144. Ans. c. Arteriovenous malformation • Diagnosis of AVM is made by conventional catheter cerebral angiographyQ • Diagnosis is based on demonstration of arteries and veins on the same conventional angiographic image, proving the high-flow shunting of blood (leading to early filling of veinsQ) through the nidus network or fistulous vessels.

Section 7

143. Ans. a. Congenital megacolon, b. Albinism

885

• In the typical AVM, there is a cloudlike nidus, or network of smaller vessels, well seen on angiography. 145. Ans. b. Temporal lobe (Ref: Harrison 20/e p3065, 19/e p2556; Sabiston 20/e p1925; Schwartz 10/e p1746-1747)

Neurosurgical Treatment of Epilepsy • The most common surgical procedures performed for epilepsy are anterior temporal lobectomyQ, focal cortical resection, multiple subpial transection, hemispherectomy, and corpus callosotomy. 146. Ans. a. Fracture of orbit

147. Ans. c. Transtentorial herniation (Ref: Harrison 19/e p1772; Bailey 25/e p624)

Kernohan’s Notch Phenomenon • Kernohan’s notch is a cerebral peduncle indentation associated with some forms of transtentorial herniation (uncal herniation)Q. • Compression of the contralateral cerebral peduncle against the free edge of the tentorium (Kernohan’s notch) causes an ipsilateral hemiparesis with ipsilateral 3rd nerve palsyQ.

Kernohan-Woltman Sign • Lateral displacement of the midbrain may compress the opposite cerebral peduncle, producing a Babinski’s sign and hemiparesis contralateral to the original hemiparesis (the Kernohan-Woltman signQ). 148. Ans. c. Constricted pupil 149. Ans. c. Vein of Galen malformation (Ref: Sabiston 19/e p1876)

• Vein of Galen is formed by the confluence of the two internal cerebral veins and receives the entire deep venous drainage of the cerebrum. • Vein of Galen then joins the inferior saggital sinus and empties the venous drainage into the straight sinus.

Vein of Galen Malformation • Vein of Galen malformation is characterized by aneurysmal dilatation and arteriovenous malformation of GalenQ. • Most commonly presents in the neonatal periodQ Clinical Features • • • •

High output failure, bounding carotid pulse, hydrocephalusQ Increased intra-cranial pressure, intraventricular hemorrhage from rupture. Cerebral ischemia from intracranial ‘steal’ phenomenon and CHFQ. Marked continuous cranial bruitQ

Diagnosis • Cranial ultrasonography: Initial investigation of choiceQ • MRI and Angiography: Used to define the lesion better

Neurosurgery

Diagnosis of Vein of Galen malformation should be suspected in any newborn presenting with unexplained congestive heart failure and hydrocephalus.

CHAPTER

34

CNS Tumors

WHO CLASSIFICATION OF BRAIN TUMORS WHO Classification of Brain Tumors 1. Neuroepithelial tumours: −− Glioma: Astrocytomas, Oligodendrogliomas, Ependymoma
, Choroid plexus tumour −− Pineal tumours −− Neuronal tumours: Ganglioglioma, Gangliocytoma, Neuroblastoma −− Medulloblastoma 2. Nerve sheath tumours: Vestibular schwannoma 3. Meningeal tumours: Meningioma 4. Pituitary tumours 5. Germ cell tumours: Germinoma, Teratoma 6. Lymphomas
 7. Tumour-like malformations: Craniopharyngioma, Epidermoid tumours, Dermoid tumour, Colloid cyst 8. Metastatic tumours
 9. Contiguous extension from regional tumours: Glomus tumour

BRAIN TUMOR • MC primary brain tumor

• MeningiomaQ (35%) > glial tumorsQ (30%)

• MC brain tumor

• MetastasisQ

• MC malignant BT of childhood • Most radiosensitive BT

• MedulloblastomaQ

• BT associated with calcification (COM)

• CraniopharyngiomaQ (most) > ODGQ (90%) > MeningiomaQ (20–25%)

Brain Tumor • Most brain tumors occur sporadicallyQ • Radiation exposure & genetic abnormalities are the risk factorsQ Genetic abnormalities associated with brain tumors (RL not in MTV GT) • • • •

RetinoblastomaQ Li-FraumeniQ NF-1 & 2Q MEN 1Q

• • • •

Turcot’s syndromeQ VHL syndromeQ Gorlin syndromeQ Tuberous sclerosisQ

Clinical Features • • • •

Three cardinal symptoms: Seizures, Raised ICT & focal neurological deficitQ (FND) Raised ICT leads to headache (worse in morning & strainingQ, associated with nausea & vomiting) FND: Progressive over time, characteristic of locationQ Pituitary adenoma may also present with endocrine abnormalitiesQ

Diagnosis • IOC for diagnosis: MRIQ

CNS Tumors

887

Treatment:

ASTROCYTOMA

Section 7

Dexamethasone: Reduces peritumoral edemaQ Anti-epileptics: For tumors close to sensorimotor stripQ Mannitol: Administered before dural opening & operative resectionQ Surgery: Primary goals of surgery includes histologic diagnosis & reduction of mass effect by removal of as much as tumor with preservation of neurological functionQ • Radiotherapy: In cases of positive margins & tumor infiltrating surrounding brainQ • Craniospinal irradiation: For tumors associated with CSF spread • • • •

Astrocytoma • • • • • •

Astrocytomas arise from astrocytesQ Mostly supratentorial in adults & infratentorial in childrenQ Astrocytoma is MC posterior fossa tumor in childrenQ Majority of astrocytoma are low grade in children & high grade in adultsQ MC astrocytoma in children: Pilocytic astrocytomaQ MC astrocytoma in adults: Glioblastoma multiformeQ (GBM)

Pathology • Majority of astrocytomas infiltrate adjacent brain. Juvenile pilocytic astrocytomas & pleomorphic xanthoastrocytomas are exceptionsQ
 • Histologic features associated with higher grade tumors: Hypercellularity, nuclear atypia & endovascular hyperplasiaQ. • NecrosisQ is present only with GBMs; it is required for the diagnosis. WHO Classification of Astrocytoma Low-grade, or grade II Astrocytomas

Grade III or Anaplastic

• Discrete appearing, contrast enhancing and often cystic with a mural noduleQ. • Mean age: First two decades of life. • Curable by radical resectionQ (no infiltration of surrounding brain) • Radiation therapy and chemotherapy have no role • Median survival time: 8-10 years.

• Occur in children and young adultsQ. • Most patients present with seizuresQ. • Typically demonstrate nuclear atypia; have a low degree of cellularity • Treatment: Observation and follow-up, radiation with or without chemotherapy, and surgery. • Surgery is not curative because most of these tumors are infiltrative with no clear marginsQ. • The median survival time is 7–8 years.

• Irregular enhancement on MRI • Treatment: Cytoreductive surgery followed by EBRTQ. • Median survival time for anaplastic astrocytoma: 2–3 years

Grade IV or Glioblastoma Multiforme • Endothelial proliferation or necrosisQ on histology makes the tumor grade IV. • Know as butterfly tumor as it crosses midlineQ • Seen in older patients (>50 years). • GBMs: Ring enhancement with central necrosis on MRIQ. • Treatment: Cytoreductive surgery followed by EBRTQ. • The extent of tumor resection has a significant effect on time to tumor progression and median survivalQ. • Carmustine and cisplatin have been the primary agents used against malignant gliomasQ. • TemozolomideQ has shown some promise in the management of newly diagnosed and recurrent GBM, with an overall survival time of 13.6 months. • Median survival time for GBM is 1pQ. These alterations are usually associated with a better prognosis. • Characterized by classic histologic feature of “fried egg” cytoplasm, “chicken wire” vasculature, and microscopic calcificationsQ.

Neurosurgery

Grade I or Pilocytic Astrocytoma

888

Surgery Essence Clinical Feature • This tumor frequently presents with seizures Diagnosis • Calcifications and hemorrhage on CT or MRI suggest the diagnosisQ. • Calcifications is seen in 28-60% in ODGs on plain radiographs, and on 90% of CTQ. Treatment • Primary modality of treatment: Surgical resection + ChemotherapyQ • Respond to procarbazine, lomustine (CCNU), vincristine (PCV) chemotherapy. • Chromosomal deletion, 1p and 19q, has been associated with robust response to temozolomideQ.

Neurosurgery

Prognosis • Median survival time ranges from 3 to 5 years

EPENDYMOMA Ependymoma • Arise from ependymal lining of cerebral hemispheres & remnants of central canal of spinal cord. • Manifest predominantly in children (within the fourth ventricle) and young adults. • MC histologic type in adults: Myxopapillary ependymomaQ, which typically arises from filum terminaleQ of spinal cord and appears in lumbosacral regionQ. Diagnosis • On CT or MRI, ependymomas typically appear as diffusely enhancing massesQ relatively well demarcated from adjacent neural tissue. • MRI findings include a well-circumscribed lesion with varying degrees of enhancement. Ventricular or brainstem displacement and hydrocephalus are frequent features. Treatment • Optimal treatment includes maximal possible resection without causing neurological deficits followed by EBRT. • Ependymomas have the potential to spread through the neuraxis by seeding of CSF; craniospinal radiationQ is recommended in this case.

MEDULLOBLASTOMA Medulloblastoma • • • • •

MC malignant brain tumor of childhood: Medulloblastoma Turcot syndrome (A variant of FAP) is associated with increased incidence of medulloblastoma Highly malignant tumor found in cerebellumQ and infratentorial location Occur predominantly in childrenQ (peak incidence at 3-4 yearsQ) Medulloblastoma is most radiosensitive brain tumorQ

Section 7

• MC site: Vermis (75%)

• MC site in adults: Lateral cerebellar hemisphere

Clinical Characteristics • Child usually presents with features of increased intracranial tensionQ. • Adults present with ataxia and unilateral dysmetria as lateral origin is more commonQ. Metastasis • Dissemination through CSF is common leading to drop metastasisQ. • Metastasis outside CNSQ affects bone, lymph node and liver. • Tumor dissemination is most important prognostic factorQ. Treatment • Despite of extreme radiosensitivity, it should be surgically excisedQ. • Surgical excision should be followed by radiotherapy and chemotherapyQ. • Carmustine (BCNU) and vincristine are primarily used for recurrences, in poor-risk patients, and in children 75%.

CNS Tumors

889

MENINGIOMA

Section 7

Meningioma • MC primary brain tumor: Meningioma (35%) >Glial tumors (30%) • Meningiomas are the MC intracranial, extra-axial dural-based neoplasmQ. • Predominantly benign tumors of adultsQ, more common in womenQ. • Derived from meningomesothelial cells of arachnoidQ • Mostly occur along the superior sagittal sinusQ • Most are slow growing & encapsulatedQ Pathology: • Round encapsulated mass showing characteristic enplaque pattern of growthQ • Tumor may range from firm to fibrous to finely gritty and may show extreme calcification & psammoma bodiesQ Clinical Presentation: • Motor deficit in 90% (spasticity & lower limb weakness), sensory deficit in 60% or sphincter dysfunction of bladderQ. Radiological findings in Meningioma • Nearly all meningiomas enhance intenselyQ following contrast administration. • Abnormal vascular markingsQ • Enlarged foramen spinosumQ on the side of lesion • Dural tail • Calcification (20-25%) in the tumorQ

• Invasion of bone cause localized bony hyperostosis/ mixed osteoblastic & osteolytic response less commonly. It may show ‘sun ray spicules’ & local bone expansion with pneumatization so called ‘blistering’ • Signs of increased intracranial tensionQ

Treatment: • • • •

Surgery is the treatment of choice for symptomatic meningiomas. Extent of resection is the most important factor in the prevention of recurrence. Simpson grading system is used for meningioma resectionQ Recurrence after gross total resection occurs in 11-15% of cases. CNS Tumors Intra-axial (LANe)

Extra-axial (PSM) • PituitaryQ • SchwannomaQ • MeningiomaQ

PITUITARY ADENOMA Pituitary Adenoma Pituitary adenomas arise primarily from anterior pituitary glandQ MC cause of hyperpituitarism: Pituitary adenomaQ Classified as either functional (secreting) or nonfunctional (nonsecreting) tumors Former presenting earlier with symptoms caused by physiologic effects and the latter presenting when of sufficient size to cause neurologic deficits by mass effect on the chiasm with consequent bitemporal hemianopsiaQ. • Incidence is increased in MEN-1Q

• • • •

• Pituitary adenoma can be differentiated from hyperplasia by reticulin stainQ • Absence of reticulin stain in pituitary adenomaQ Clinical Features • Occur commonly in third & fourth decades and affect both sexes equallyQ. • MC functional tumor is prolactinomaQ, which causes amenorrhea & galactorrhea in womenQ. Diagnosis • MRI is IOC for pituitary tumorsQ • Typically, the pituitary gland enhances rapidly owing to lack of blood-brain barrier.

Neurosurgery

• Neuronal • Astrocytoma (Glioma)Q • Lymphoma

890

Surgery Essence • Microadenoma may appear as a nonenhancing area within the gland. • Diagnostic workup includes a full endocrinologic profile and a formal visual fields testQ. Treatment • Dopamine agonist, bromocriptineQ, can shrink prolactinomas in 75% of patients with macroadenomas in 6-8 weeks, but only as long as therapy is maintained. • Bromocriptine may also work on GH-secreting tumors with tumor shrinkage in 70% of cases.

Section 7

Diagnosis • IOC: MRI with gadolinium enhancementQ • Lesions are at the gray-matter and white-matter junction, well circumscribed, surrounded by edemaQ. Treatment • Surgery is recommended for accessible lesions (up to 3) causing mass effect followed by whole-brain radiation therapy (WBRT) to eradicate micrometastasesQ. • Stereotactic radiosurgery followed by WBRT has also been shown to be as effective as surgery in the management of metastatic brain tumors (< 3 cm). • Chemotherapy is not useful in most brain metastases except small cell lung cancer and seminomas. Prognosis • Median survival time with optimal treatment: 7–12 months

Multiple Choice Questions CNS TUMORS PREDISPOSING FACTORS



1. All of the following hereditary conditions predispose to CNS tumors, except: (DPG 2010, AIIMS 2005) a. Neurofibromatosis 1 and 2 b. Tuberous sclerosis c. Von-Hippel-Lindau syndrome d. Xeroderma pigmentosum 2. All of the following statements about Neurofibromatosis are true, except: (All India 2009) a. Autosomal recessive inheritance b. Cutaneous neurofibromas c. Cataract d. Scoliosis



3. Which of the following is the most common tumor associated with type-1 neurofibromatosis? (AIIMS Nov 2007, May 2003) a. Optic nerve glioma b. Meningioma c. Acoustic schwannoma d. Low grade astrocytoma



4. Neurofibromatosis type-2 is associated with: (PGI Dec 2000) a. Bilateral acoustic schwannoma b. Multiple care-au-lait spots c. Chromosome-22 d. Lisch nodule e. Posterior subcapsular lenticular cataract 5. Widened neural foramina is frequently seen in: (All India 2012) a. Neurofibromatosis b. Tuberous sclerosis c. Sturge-Weber syndrome d. Klippel-Fiel syndrome





6. All of the following may be associated with Von-Hippel Lindau syndrome, except: (All India 2009) a. Retinal and cerebella hemangioblastomas b. Gastric carcinoma c. Pheochromocytoma d. Renal cell carcinoma



7. In Von-Hippel Lindau syndrome, the retinal vascular tumours are often associated with intracranial hemangioblastoma. Which one of the following regions is associated with such vascular abnormalities in this syndrome? (All India 2005) a. Optic radiation b. Optic tract c. Cerebellum d. Pulvinar



8. Which of the following statement about VHL syndrome is true? (All India 2012) a. Multiple tumors are rarely seen b. Craniospinal hemangioblastoma are common c. Supratentorial tumors are common d. Tumors of Schwann cells are common



9. Neurofibromatosis-2 is/are associated with: (PGI Nov 2011) a. Meningioma b. Schwannoma c. Glioma d. Lisch nodule e. Hearing loss



10. A child presents to the clinic with history of seizures and mental retardation. Clinical examination reveals multiple hypopigmented macules. What is the likely diagnosis? a. Tuberous Sclerosis (All India 2010) b. Neurofibromatosis c. Sturge Weber Syndrome d. Linear epidermal nevus syndrome



11. The diagnosis of a patient presenting with seizures, mental retardation and sebaceous adenoma is: (All India 95) a. Hypothyroidism b. Tuberous sclerosis c. Toxoplasmosis d. Down syndrome

12. Triad of tuberous sclerosis includes all, except: (All India 2009) a. Epilepsy b. Adenoma sebacium c. Low intelligence d. Hydrocephalus

13. Adenoma sebacium is a feature of: (AIIMS 2005) a. Neurofibromatosis b. Tuberous sclerosis c. Xanthomatosis d. Incontinentia pigmenti



14. CNS tumor seen in Von Hippel Lindau syndrome is: a. Meningioma (PGI Dec 99) b. Cerebellar hemangioblastoma c. CNS lymphoma d. Glioma



15. Neurofibromatosis is associated with: (PGI Dec 98) a. Papillary carcinoma b. Islet cell tumor c. Pheochromocytoma d. Glucagonoma



16. Plexiform neurofibromatosis commonly affects: a. Facial nerve b. Trigeminal nerve c. Peripheral nerve d. Glossopharyngeal nerve



17. Musculoskeletal abnormality in neurofibromatosis is: a. Hypertrophy of limb b. Scoliosis c. Café au lait spots d. Pseudo arthrosis e. All



18. Neurofibromatosis presents as all of the following except: a. Elephantiasis neuromatosa (UPSC 2001) b. Plexiform neuroma c. Von Recklinghausen’s disease d. Lymphadenovarix



19. Brain tumor is associated with all except: a. Tuberous sclerosis b. Von Hippel landau syndrome c. Neurofibromatosis d. Sturge-Weber syndrome



20. What is not a feature of Sturge-Weber syndrome? a. Rail track appearance b. Hemiatrophy of the brain c. Convulsion d. Empty sella

(PGI Dec 99)

CNS TUMORS: CLINICAL FEATURES AND TREATMENT 21. A 55-years old female presents with grade I Ependymoma extending from C7-T1 with no neural defect. Surgery is done, next management is: (DPG 2008) a. Post-op chemotherapy b. Post-op chemoradiation c. Imaging, regular follow-up, chemotherapy if required d. Imaging, regular follow-up, radiotherapy if required

22. All of the following are features of brain tumor except:  (Recent Question 2017) a. Pin point pupil b. Seizures c. Headache d. Focal neurological deficit

894

Surgery Essence

23. Most common brain tumour:

Neurosurgery

a. Meningioma c. Metastasis

(Recent Question 2017) b. Glioma d. Astrocytoma



24. Psychiatrics symptoms, true except: (PGI 2000) a. More common with supra than infra tentorial tumors b. More common with slow growing c. More with temporal than frontal lobe tumours d. More with brain stem lesions



25. Which of the following tumor is not known to increase in pregnancy? (All India 2006) a. Glioma b. Pituitary adenoma c. Meningioma d. Neurofibroma



26. Which one of the following tumors shows calcification on CT scan? (All India 2005) a. Ependymoma b. Medulloblastoma c. Meningioma d. CNS lymphoma



27. A 35 years old patient presented to hospital with headache, ataxia and imbalance. On laboratory evaluation, RBC count was very high. MRI brain was performed and the image is given below. What is the most probable diagnosis? a. Medulloblastoma b. Pilocytic astrocytoma c. Hemangioblastoma d. Craniopharyngioma



34. Lowest incidence of cerebral tumours is seen in: a. Occipital b. Frontal c. Temporal d. parietal



35. Cerebellar hemangioblastoma and retinal tumours are seen in: (JIMPER 2012) a. VHL syndrome b. NF-1 c. Tuberous selerosis d. NF-2

BRAIN METASTASIS 36. Which of the following carcinoma most frequently metastasizes to brain? (MCI June 2018; AIIMS 2005) a. Small cell carcinoma lung b. Prostate cancer c. Rectal carcinoma d. Endometrial cancer

37. Most common site of brain metastasis: a. Brainstem b. Cerebellum c. Cerebral cortex d. Thalamous

(DNB 2011)

ASTROCYTOMA 38. Which of the following is the most common type of glial tumors? (All India 2006) a. Astrocytomas b. Medulloblastomas c. Neurofibromas d. Ependymomas

39 In children most common posterior fossa tumour is: (AIIMS Dec 95) a. Meningiomas b. Astrocytoma c. Medulloblastoma d. Glioblastoma multiforme

40. A 25-year-old male presented with morning headache, projectile vomiting and seizures. MRI was done. What is the diagnosis on the basis of radiological findings? a. Meningioma b. Glioblastoma c. Medulloblastoma d. Oligodendroglioma



28. Stereotactic radiosurgery is done for: a. Glioblastoma multiforme b. Medulloblastoma spinal cord c. Ependymoma d. AV malformation of brain

(JIPMER 2002)

29. Which of the following brain tumors doesn’t spread via CSF? (DPG 2011, All India 2004) a. Germ cell tumor b. Medulloblastoma c. CNS Lymphoma d. Craniopharyngioma

Section 7





30. The characteristic feature of a frontal lobe tumor is: a. Abnormal gait b. Aphasia  (All India 94) c. Distractibility d. Antisocial behavior 31. Prophylactic craniospinal irradiation is recommended in: a. Gemistocytic astrocytoma (PGI 2007) b. Posterior fossa ependymoma c. Meningioma d. Medulloblastoma



32. All of the following tumors may be malignant except: a. Glioma b. Astrocytoma (All India 97) c. Hemangioblastoma d. Ependymoma



33. The CNS tumor present with calcification: (PGI June 99) a. Oligodendroglioma b. Astrocytoma c. Medulloblastoma d. pheochromocytoma

41. Glioblastoma multiforme may occur in the following except: (DPG 93) a. Cerebrum of adult b. Brain stem of child c. Spinal cord of adult d. Adrenal medulla of child

42. Which of the following brain tumors is highly vascular in nature? (AIIMS May 2006) a. Glioblastoma b. Meningiomas c. CP angle epidermoid d. Pituitary adenomas



43. Most common intracranial neoplasm in adults is:



a. Meningioma c. Posterior fossa tumor

(AIIMS May 93, May 94) b. Astrocytoma d. Ganglioneuroma

CNS Tumors



50. A child present with raised ICT. On CT scan, a lesion is seen around foramen of Monroe and multiple periventricular calcific foci. What is the most probable diagnosis? a. Central neurocytoma (AIIMS Nov 2011) b. Ependymoma c. Subependymal giant cell astrocytoma d. Ganglioganglioma

MEDULLOBLASTOMA

45. Which of the following statements about cerebellar astrocytomas in pediatric age group is false? (All India 2008) a. These are usually low grade tumors b. These are more commonly seen in the 1st and 2nd decades c. These tumors have a good prognosis d. These tumors are more common in females 46. Most common site of sub ependymal astrocytoma (Giant cell): (AIIMS Nov 2007) a. Trigone of lateral ventricle b. Foramen of Monro c. Temporal horn of lateral ventricle d. 4th ventricle 47. Glioblastoma multiforme is a variant of: (COMEDK 2005) a. Medulloblastoma b. Meningioma c. Astrocytoma d. Neuroblastoma



48. A 10 years old child was brought to the hospital by mother with history of headache, nausea, vomiting, irritability, difficulty to coordinate movements and visual complaints. MRI was performed and the image is given below. What is the most probable diagnosis? (Recent Question 2016) a. Medulloblastoma b. Pilocytic astrocytoma c. Hemangioblastoma d. Craniopharyngioma

51. Which of the following is true about medulloblastoma? a. Radiosensitive tumor (PGI Dec 2005) b. Spreads through CSF c. Surgical treatment is not done d. Occurs in young age group e. It is a supratentorial tumor



52. Chang staging is used for? a. Retinoblastoma c. Ewing’s sarcoma



53. Long term effect of craniospinal irradiation for medulloblastoma is: (JIPMER 2011) a. Secondary malignancy b. Neuro endocrine abnormalities c. Neurocognitive effects d. Hearing loss



54. Medulloblastoma exclusively occurs in the: a. Medulla b. Cerebellum c. Cerebral hemisphere d. Spinal cord



55. True about medulloblastoma is: a. Highly radiosensitive b. Surgery is the only treatment c. Occurs in adult age group d. Chemotherapy is useful

(AIIMS May 2010) b. Medulloblastoma d. Rhabdomyosarcoma

(AIIMS Nov 95)

MENINGIOMA



56. A 45-years old female complains of progressive lower limb weakness, spasticity, urinary hesitancy. MRI shows intradural enhancing mass lesion. Most likely diagnosis is:  (AIIMS Nov 2011, Nov 2006, All India 2007) a. Dermoid cyst b. Intradural lipoma c. Neuroepithelial cyst d. Meningioma 57. Radiological features of meningioma: (PGI 2009) a. Calcification b. Erosion c. Sutural diastasis d. Osteosclerosis e. Vascular erosion

58. A 40-year-old female presented with weakness of lower limbs with bladder dysfunction. On the basis of MRI findings, what is the diagnosis? a. Astrocytoma b. Meningioma c. Pituitary adenoma d. Oligodendroglioma



49. All are true regarding pilocytic astrocytoma, except: a. Seen in elderly above 80 years (AIIMS May 2009) b. Seen in posterior fossa c. Good prognosis d. Most common primary brain tumor in children

Neurosurgery





Section 7

44. What is the most probable diagnosis based on the given MRI image? (Recent Question 2016) a. Pilocytic astrocytoma b. Glioblastoma multiforme c. Oligodendroglioma d. Meningioma

895

896

Surgery Essence

Neurosurgery

59. A 48-years old woman comes with bilateral progressive weakness of both lower limbs, spasticity and mild impairment of respiratory movements. MRI shows an intradural mid-dorsal midline enhancing lesion. What is the diagnosis?  (AIIMS May 2010) a. Intradural lipoma b. Meningioma c. Neuroenteric cyst d. Dermoid cyst

60. Best prognosis among following is seen in: (DNB 2007) a. Astrocytoma b. Oligodendroglioma c. Meningioma d. Medulloblastoma



61. Extra-axial intracranial lesion showing contrast enhancement on MRI: (All India 2012) a. Meningioma b. Ependymoma c. Arachnoid cyst d. Astrocytoma



62. A lady had meningioma with inflammatory edematous lesion. She was planned for surgery. Junior resident mistake in writing pre-op notes is: (AIIMS May 2012) a. Stop steroids b. Wash head with shampoo c. Antibiotic sensitivity d. Continue antiepileptics

63. All of the following tumors usually show psammoma bodies except: (MHCET 2016) a. Papillary carcinoma of thyroid b. Meningioma c. Serous cystadenoma of ovary d. Hepatocellular carcinoma

CRANIOPHARYNGIOMA 64. Suprasellar calcification with polyuria seen in: (PGI Dec 2002) a. Langerhan cell histocytosis b. Medulloblastoma c. Pinealoma d. Craniopharyngioma e. Astrocytoma 65. A 6-years old boy has been complaining of headache, ignoring to see the objects on the sides for four months. On examination, he is not mentally retarded, his grades at school are good, and visual acuity is diminished in both the eyes. Visual charting showed significant field defect. CT scan of the head showed suprasellar mass with calcification. Which of the following is the most probable diagnosis? a. Astrocytoma c. Pituitary adenoma

Section 7





67. A six year old child managed by complete surgical removal of craniopharyngioma developed multiple endocrinopathies. Which of following hormones should be replaced first?  (All India 2011) a. Hydrocortisone b. Growth Hormone c. Thyroxine d. Prolactin



68. Which of the following is the most common cause of a mixed cystic and solid suprasellar mass seen on cranial MR scan of a 10 years old child? (AIIMS 2005) a. Pituitary adenoma b. Craniopharyngioma c. Optic chiasmal glioma d. Germinoma



69. All the following are true of craniopharyngioma except: a. Derived from Rathke’s pouch (All India 94) b. Contains epithelial cells c. Present in sella or infra-sellar location d. Causes visual disturbances

PITUITARY ADENOMA

71. The most preferred approach for pituitary surgery at the present time is: (JIPMER November 2017; All India 2006) a. Transcranial b. Transethmoidal c. Transphenoidal d. Transcallosal 72. A 35 years old female with amenorrhea, galactorrhea has bitemporal hemianopia. The most probable diagnosis is: a. GnRH adenoma (JIPMER 2010) b. Sellar chordoma c. Craniopharyngioma d. Prolactin secreting pituitary microadenoma

73. A 30 years old male complains of loss of erection; he has low testosterone and high prolactin level in blood; what is the likely diagnosis? (All India 2001) a. Pituitary adenoma b. Testicular failure c. Craniopharyngioma d. Cushing’s syndrome



74. Most common cause of hypersecreting pituitary tumour is: a. Pituitary adenoma (DNB 2009) b. Pituitary carcinoma c. Autoimmue disease of pituitary d. Transection of stalk

(AIIMS Nov 2004, 2005) b. Craniopharyngioma d. Meningioma

66. A 15 years old boy was brought to the hospital with the chief complaints of headache, vomiting and visual complaints. On examination, there were signs of growth retardation. MRI image is given below. What is the most probable diagnosis? a. Medulloblastoma b. Pilocytic astrocytoma c. Hemangioblastoma d. Craniopharyngioma

70. Which of the following is true about pituitary adenoma? a. Accounts for 10% of brain tumors (PGI Dec 2005) b. Erodes the sellar and extends into surrounding area c. Prolactinoma is least common d. It is differentiated by reticulin stain

SPINAL TUMORS

75. The commonest extradural spinal tumor is: a. Neurofibroma b. Glioma c. Meningioma d. Metastasis



76. Commonest spinal tumour is: (SCTIMS 98) a. Meningioma b. Ependymoma c. Neurofibroma d. Neuroblastomas



77. Most common location of spinal tumors: (AIIMS Nov 2007) a. Intramedullary b. Intradural extramedullary c. Extradural d. Equally distributed

CNS LYMPHOMA

78. True about primary CNS lymphoma: (PGI 2009) a. Reticulin staining done b. Essentially B-cell type c. Associated with EBV d. Indolent disease with good prognosis e. Chemotherapy highly effective

CNS Tumors



88. Imaging modality of choice for detecting radiation induced cerebral necrosis: (AIIMS Nov 2009, 2005) a. PET scan b. Biopsy c. MRI d. CT



89. Not a neuroglial tumor: a. Shwannoma b. Astrocytoma c. Medulloblastoma d. Ependymoma



90. Investigation of choice for leptomeningeal carcinomatosis:  (AIIMS Nov 2011) a. PET Scan b. SPECT c. Gd enhanced MRI d. CT scan

SCHWANNOMA AND NEUROFIBROMA

80. Dumbbell tumor is seen in: (GB Pant 2011) a. Meningioma b. Neurofibroma c. Ependymoma d. Thymoma



81. Vestibular schwannoma arises most frequently from: (All India 2011) a. Superior vestibular nerve b. Inferior vestibular nerve c. Cochlear nerve d. Facial nerve

MISCELLANEOUS

82. Commonest orbital tumour causing exophthalmos is: (Recent Question 2016) a. Glioma b. Meningioma c. Hemangioma d. Neuroblastoma



83. Which of the following is primary neurogenic tumour? (JIPMER 90) a. Meningioma b. Glioblastoma c. Acoustic neuroma d. Neuroblastoma



84. Witzelsucht syndrome (i.e. “Pathological Joking”) is seen in: a. Frontal lobe tumours c. Temporal lobe tumours

(All Inida 90) b. Parietal lobe tumours d. Intra Ventricular tumours

85. MRI is the investigation of choice in all of the following except: (COMEDK 2007, 2004) a. Syringomyelia b. Brain stem tumors c. Skull bone tumors d. Multiple sclerosis

86. All of the following are neuronal tumors, except:



(All India 2011) b. Ganglioglioma d. Ependymoma

87. A 20-year female patient with 6th cranial nerve palsy on T2 weighted MRI shows a hyperintense lesion in cavernous sinus which shows homogenous contrast enhancement. Most probable diagnosis is: (AIIMS Nov 2010) a. Schwannoma b. Meningioma c. Cavernous sinus hemangioma d. Astrocytoma

91. A patient was diagnosed with intracranial cavernous angioma on MRI. MRI finding chracteristic of this lesion is: (AIIMS Nov 2011) a. Well defined nidus b. Definite arterial feeders c. Phlebectasis d. Popcorn like lesion 92. All of the following are true about long terms sequel of cranio spinal radiothreapy for children with CNS tumors except: a. Neurocognitive dysfunction  (All India 2012) b. Endocrinologic dysfunction c. Musculoskeletan hypoplasia d. Neuropscychological sequel are independent of radiation dose 93. Enlargement of pituitary tumour after adrenalectomy is called as: (DNB 2009) a. Nelson syndrome b. Steel-Richardson syndrome c. Hamman-Rich syndrome d. Job’s syndrome

94. Stereotactic surgery is used for treatment of:  (AIIMS Nov 2012) a. Brain tumor b. Lungs carcinoma c. Cervix cancer d. Renal carcinoma



95. Highly vascular tumor of brain and spinal cord in adults:  (AIIMS May 2013) a. Metastasis b. Pilocytic astrocytoma c. Hemangioblastoma d. Cavernous malformation

Neurosurgery

a. Gangliocytoma c. Neurocytoma

(Kerala 95)

Section 7

79. All are true regarding Primary CNS lymphoma except: a. Radiotherapy and chemotherapy is of no value b. Occurs in AIDS patient (AIIMS Feb 97) c. Commonly occurs in immune-compromised persons d. EBV may be a cause

897

Explanations CNS TUMORS PREDISPOSING FACTORS

1. Ans. d. Xeroderma pigmentosum (Ref: Harrison 20/e p645, 19/e po599; Bailey 27/e p145)



2. Ans. a. Autosomal recessive inheritance (Ref: Harrison 20/e p649, 19/e p604; Sabiston 20/e p754; Schwartz 10/e p677; Bailey 27/e p145) Neurofibromatosis Neurofibromatosis-1

Neurofibromatosis-2

• Also known as peripheral neurofibromatosis or vonRecklinghausen’s syndromeQ • Most prevalent type (90%Q) • NF-1 gene: Chromosome 17Q • Autosomal dominantQ • Diagnostic Criteria for NF-1 (Diagnosed when any two of the following are present): 1.  ≥6 café-au-lait maculesQ >5 mm in greatest diameter in prepubertal individuals and >15mm in greatest diameter in post-pubertal individuals. 2. Axillary or inguinal frecklingQ 3. ≥2 iris Lisch nodulesQ 4.  ≥2 neurofibromas or one plexiform neurofibromaQ 5. Sphenoid dysplasia or cortical thinning of long bone, with or without pseudoarthrosis 6.  Optic gliomasQ 7. A first degree relative with NF-1 whose diagnosis was based on the aforementioned criteria.

• Also known as central neurofibromatosis or bilateral acoustic neurofibromatosisQ • Less prevalent (10%) • NF-2 gene: Chromosome 22Q • Autosomal dominantQ • Diagnostic Criteria for NF-2Q (Diagnosed when any one of the following is present): 1.  Bilateral 8th nerve masses consistent with acoustic neuromasQ 2. A parent, sibling, or child with NF-2 and either 3.  Unilateral 8th nerve mass or any two of the following: −− NeurofibromaQ −− MeningiomaQ −− GliomaQ −− SchwannomaQ • Bilateral acoustic neuromas are the most distinctive tumors in patients with NF-2Q.



3. Ans. a. Optic nerve glioma



4. Ans. a. Bilateral acoustic schwannoma, c. Chromosome -22, e. Posterior subcapsular lenticular cataract



5. Ans. a. Neurofibromatosis (Ref: Differential Diagnosis in Conventional Radiology (Thieme) 2007/260)



6. Ans. b. Gastric carcinoma (Ref: Harrison 20/e p649, 19/e p599; Sabiston 20/e p693; Bailey 27/e p145) Von Hippel Lindau Syndrome (AD) Characteristic Tumors/Cysts

Other Tumors/Cysts

• Hemangioblastomas: −− Cerebellar hemangioblastomaQ −− Retinal hemangioblastomaQ −− Spinal hemangioblastomaQ

• • • • •

RCCQ PheochromocytomaQ Pancreatic endocrine tumors Adrenal carcinomas Benign cysts in kidney, epididymis, liver or pancreasQ

• Polycythemia is a characteristic feature in VHL due to erythropoietin production by hemangioblastoma and/or RCCQ.

7. Ans. c. Cerebellum



9. Ans. a. Meningioma, b. Schwannoma, c. Glioma, e. Hearing loss

8. Ans. b. Craniospinal hemangioblastoma are common



10. Ans. a. Tuberous Sclerosis (Ref: Roxburgh’s 17/e p201; Harrison 20/e p649, 19/e p604) Tuberous Sclerosis/Epiloia/Bourneville’s Disease Skin Lesion • Adenoma sebaciumQ (facial angiofibroma) • Ash leaf shaped hypopigmentd maculesQ • Shagreen PatchQ- yellow thickening of lumbosacral skin • Depigmented nevi

Neurological • • • •

SeizureQ Mental retardationQ Subependymal nodulesQ which may calcify HydrocephalusQ

Benign Neoplasm • RhabdomyomaQ • AngiomyomaQ of liver, kidney, pancreas etc. • EpendymomaQ • AstrocytomaQ

CNS Tumors 11. Ans. b. Tuberous sclerosis



13. Ans. b. Tuberous sclerosis



14. Ans. b. Cerebellar hemangioblastoma



15. Ans. c. Pheochromocytoma



16. Ans. b. Trigeminal nerve



17. Ans. e. All

12. Ans. d. Hydrocephalus

18. Ans. d. Lymphadenovarix

• Elephantiasis neuromatosa is the most impressive manifestation of NF-1.

19. Ans. d. Sturge-Weber syndrome



20. Ans. d. Empty sella (Ref: Bailey 26/e p599)

Section 7



899

Sturge-Weber Syndrome / Encephalotrigeminal Syndrome • Usually sporadic, characterized by: −− Large unilateral cutaneous angiomaQ (port-wine stain) −− Angiomas in brain involving ipsilateral cerebral hemisphere and meninges −− Focal seizuresQ typically occurs opposite to the side of lesionQ −− Adrenal pheochromocytoma −− Cerebral angiomas lead to cortical atrophyQ • Angiomas are visible radiologically as Tram-track or rail track calcification mainly in occipital regionQ

CNS TUMORS: CLINICAL FEATURES AND TREATMENT 21. Ans. d. Imaging, regular follow-up, radiotherapy if required (Ref: Harrison 20/e p643, 19/e p601; Sabiston 20/e p1912; Schwartz 10/e p1734, 1738; Bailey 27/e p664) • Harrison says “Following the gross total excision of an Ependymoma, the prognosis is excellent. The five year disease free survival is >80%. However, many ependymoma cannot be totally excised, and postoperative focal external beam radiation or stereotactic radiosurgery is used.” 22. Ans. a. Pin point pupil (Ref: Sabiston 20/e p1909; Schwartz 10/e p1732; Bailey 27/e p662-663)



23. Ans. c. Metastasis (Ref: Sabiston 20/e p1915; Schwartz 10/e p1732; Bailey 27/e p663)



24. Ans. b. More common with slow growing, c. More with temporal than frontal lobe tumours (Ref: Kaplan and Sadock’s Concise Textbook of Clinical Psychiatry (2008)/74)

Psychiatric Symptoms in Brain Tumors • Mental changes are likely to occur in patients with supratentorial tumors, and more commonly among patients with tumors of the frontal and temporal lobes. • Psychiatric symptoms are more common in frontal lobe tumorsQ. • Depression: More common in frontal lobe tumorsQ • Psychosis: More common in temporal lobe tumorsQ • Left sided frontal tumors are more commonly associated with akinesia and depression, while right sided lesions are more often associated with euphoria. • Delirium is most often a component of rapidly growingQ, large, or metastatic tumors.

25. Ans. a. Glioma (Ref: CGDT 9/e p429) • Although brain tumors are not specifically related to gestation, meningiomas, angiomas, and neurofibromas are thought to grow more rapidly with pregnancyQ.



26. Ans. c. Meningioma (Ref: Sutton Radiology 7/e p1739) • Meningioma range from firm and fibrous to finely gritty or they may be extremely calcified with Psammoma bodiesQ. • Calcification is also seen in ependymoma, but more common in meningiomaQ.

Neurosurgery



900

Surgery Essence

27. Ans. c. Hemangioblastoma (Ref: Harrison 20/e p2754, 19/e p578; Sabiston 20/e p1913) Hemangioblastoma • • • •

MC primary intra-axial tumor in the adult posterior fossaQ Occur almost exclusively in the posterior fossa (cerebellumQ) Solid or cystic with a mural noduleQ May occur sporadically, and 20% of cases may be associated with von Hippel-Lindau disease (hemangioblastomas, retinal angiomas, RCC, pheochromocytoma, renal and pancreatic cysts) Q.

Neurosurgery

Pathology • Histologically benignQ, and may be associated with erythrocytosisQ. • Appear as cystic tumors with an enhancing tumor on the cyst wall known as the mural nodule • Pathology reveals abundant thin-walled vascular channelsQ Clinical Features • Most patients present with headache, hydrocephalus, symptoms of raised ICT and cerebellar dysfunctionQ • Polycythemia due to increased erythropoietin production occurs in ~20% casesQ Diagnosis • Sharply demarcated homogeneous masses composed of cyst with non-enhancing walls, except for a mural nodule which vividly enhancesQ Treatment • Surgical resection is curative for sporadic (non-VHL associated) tumorsQ. • En-bloc resection of the mural nodule alone, leaving the cyst wall, is sufficientQ.

28. Ans. d. AV malformation of brain (Ref: Sabiston 20/e p1923; Schwartz 10/e p1749)



29. Ans. d. Craniopharyngioma (Ref: Harrison 20/e p648, 19/e p0599-601)



30. Ans. d. Antisocial behavior

Frontal Lobe Tumors • Personality changes are common symptom of frontal lobe tumor. • Frontal lobe controls behavior and judgmentQ. • Patients may be socially disinhibited, display antisocial behavior, shows severe impairment of judgment, insight and foresightQ. • Witzelsucht syndrome: Self amusement from poor jokes and puns, also known as pathological joking, is seen in frontal lobe tumorsQ

31. Ans. d. Medulloblastoma (Ref: Harrison 20/e p647, 19/e p601, 603)

Prophylactic Craniospinal Irradiation • Prophylactic craniospinal irradiation is useful in CNS malignancy which disseminate via CSF or any malignancy with high risk of CNS spreadQ.

Section 7

Common Manifestations



• • • •

MedulloblastomaQ GlioblastomaQ GerminomaQ Small cell Ca of lungQ

Less Common Manifestation • ALLQ • Non hodgkin’s lymphomaQ • Leptomeningeal RhabdomyosarcomaQ



32. Ans. c. Hemangioblastoma (Ref: Harrison 20/e p2754, 19/e p578; Sabiston 20/e p1913; Schwartz 10/e p1735-1736)



33. Ans. a. Oligodendroglioma (Ref: Harrison 20/e p646, 19/e p600; Sabiston 20/e p1911; Schwartz 10/e p1733; Bailey 27/e p664)

Tumors with Calicification: (COM) Craniopharyngioma (Most) > Oligodenoroglioma (90%) > Meningioma (20–25%)

34. Ans. a. Occipital



35. Ans. a. VHL syndrome

BRAIN METASTASIS

36. Ans. a. Small cell carcinoma lung (Ref: Harrison 20/e p649, 19/e p604)

37. Ans. c. Cerebral cortex

CNS Tumors

901

ASTROCYTOMA 38. Ans. a. Astrocytomas (Ref: Harrison 20/e p644, 19/ep599; Sabiston 20/e p1911; Schwartz 10/e p1733, 1738-1739; Bailey 27/e p664)



39. Ans. b. Astrocytoma



40. Ans. b. Glioblastoma
 • Glioblastoma multiforme: Ring enhancement with central necrosis on MRIQ. 41. Ans. d. Adrenal medulla of child



42. Ans. a. Glioblastoma (Ref: Harrison 20/e p645, 18/e p3384-3386; Osborn Neuroradiology (1994)/541, 591) • Osborn says “Glioblastoma is highly vascular, sometimes so vascular that it resembles an AV malformation on angiography.”



43. Ans. a. Meningioma (Ref: Harrison 20/e p643, 19/e p602)

Harrison says “meningiomas are diagnosed with increasing frequently as more people undergo neuroimaging for various indications. They are now the most common primary brain tumor, accounting for approximately 35% of total.

Section 7



• MC primary brain tumor: Meningioma (35%) > glialtumors (30%) • MC malignant brain tumor of childhood: Medulloblastoma • MC brain tumor: Metastasis

44. Ans. b. Glioblastoma multiforme (Ref: Sabiston 20/e p1911; Harrison 20/e p645, 19/e p91e-4; Robbins 9/e p1307)



45. Ans. d. These tumors are more common in females (Ref: Nelsons 20/e p2455)

Cerebellar astrocytomas do not show any clear gender predilection and are equally common in both males and females.

46. Ans. b. Foramen of Monro (Ref: Neurology in Clinical Practice 4/e p428; Sutton Radiology 7/e p1735)

Subependymal Giant Cell Astrocytoma • Most common site of subependymal giant cell astrocytoma is the ependymal wall of lateral ventricle near the foramen of MonroQ. • Causes obstruction at the foramen of Monro leading to ventricular enlargement and raised ICT. • Presence of multiple periventricular calcific foci (calcified subependymal nodules) suggest the diagnosis of Tuberous sclerosis with subependymal giant cell astrocytomaQ

47. Ans. c. Astrocytoma



48. Ans. b. Pilocytic astrocytoma (Ref: Sabiston 20/e p1911; Harrison 20/e p644, 19/e p441e-39f; Robbins 9/e p1340)



49. Ans. a. Seen in elderly above 80 years

50. Ans. c. Subependymal giant cell astrocytoma



51. Ans. a. Radiosensitive tumor, b. Spreads through CSF, d. Occurs in young age group (Ref: Harrison 20/e p647, 19/e p602; Sabiston 20/ep,; Schwartz 10/e p1734)



52. Ans. b. Medulloblastoma



53. Ans. c. Neurocognitive effect (Ref: www.ncbi.nlm.nih.gov/pubmed/9121399)

Craniospinal Irradiation (CSI) • Hypothyroidism: One of the earliest late side effects of CSI and 2nd MC (after GH disturbanceQ) • Prevalence of hypothyroidism is 40–80% after CSIQ • Significantly increased risk of development of benign thyroid nodules and papillary carcinoma of the thyroid many years laterQ.

54. Ans. b. Cerebellum

55. Ans. a. Highly radiosensitive

MENINGIOMA

56. Ans. d. Meningioma (Ref: Harrison 20/e p648, 19/e p602; Chapman 4/e p 431; Sabiston 20/e p1913; Schwartz 10/e p1735,1738; Bailey 27/e p665)



57. Ans. a. Calcification, d. Osteosclerosis, e. Vascular erosion



58. Ans. b. Meningioma (Ref: Chapman 4/e p431)



59. Ans. b. Meningioma



60. Ans. c. Meningioma • Meningioma is slow growing and encapsulated tumor having best prognosis among the given options.

Neurosurgery

MEDULLOBLASTOMA

902

Surgery Essence

61. Ans. a. Meningioma

62. Ans. a. Stop steroids

• Sudden withdrawal of steroids may lead to adrenal insufficiency, particularly during period of stress such as surgery.

63. Ans. d. Hepatocellular carcinoma

Neurosurgery

CRANIOPHARYNGIOMA

64. Ans. d. Craniopharyngioma (Ref: Harrison 20/e p648, 19/ep 603, 18/e p3389; Schwartz 10/e p1736-1737)



65. Ans. b. Craniopharyngioma



66. Ans. d. Craniopharyngioma (Ref: Harrison 20/e p648, 19/e p2262)



67. Ans. a. Hydrocortisone



68. Ans. b. Craniopharyngioma

69. Ans. c. Present in sella or infra-sellar location

PITUITARY ADENOMA

70. Ans. a. Accounts for 10% of brain tumors, b. Erodes the sellar and extends into surrounding area, d. It is differentiated by reticulin stain (Ref: Harrison 20/e p2673, 19/e p603; Sabiston 20/e p1914; Schwartz 10/e p1735; Bailey 27/e p665-666,841,842)

Cushing Disease • Pituitary dependent cause of Cushing syndrome (pituitary adenoma secreting excessive ACTH)

71. Ans. c. Transphenoidal

72. Ans. d. Prolactin secreting pituitary microadenoma



73. Ans. a. Pituitary adenoma

74. Ans. a. Pituitary adenoma

SPINAL TUMORS

75. Ans. d. Metastasis (Ref: Scott Atlas, MRI of the Brain and Spine 3rd/1742; Sabiston 20/e p1913; Schwartz 10/e p1737-1739)



76. Ans. c. Neurofibroma

77. Ans. c. Extradural

CNS LYMPHOMA

78. Ans. a. Reticulin staining done, b. Essentially B-cell type, c. Associated with EBV (Ref: Harrison 20/e p647, 19/e p601; Sabiston 20/e p1914; Schwartz 10/e p1736; Bailey 25/e p630)



79. Ans. a. Radiotherapy and chemotherapy is of no value

SCHWANNOMA AND NEUROFIBROMA

80. Ans. b. Neurofibroma



81. Ans. b. Inferior vestibular nerve (Ref: Harrison 20/e p648, 19/e p220; Sabiston 19/e p1889; Schwartz 9/e p1541; Bailey 26/e p616)

• The term “dumbbell” lesion is used to describe neurofibroma.

Section 7

MISCELLANEOUS

82. Ans. a. Glioma



84. Ans. a. Frontal lobe tumours



85. Ans. c. Skull bone tumors (Ref: Bailey 27/e p197-198,199)

83. Ans. d. Neuroblastoma

Magnetic Resonance Imaging • MRI was discovered by LauterbeurQ in 1973.       • MRI is best for soft tissuesQ.



86. Ans. d. Ependymoma (Ref: Robbins 9/e p601) • Ependymomas are glial tumors, derived from the ependymal cells that line the ventricular surface. These tumors do not originate from neuronsQ. 87. Ans. a. Schwannoma (Ref: Osborn Radiology/501) • Schwannoma is hypointense on T1 and hyperintense on T2.

CNS Tumors 88. Ans. a. PET scan



90. Ans. c. Gd enhanced MRI

89. Ans. c. Medulloblastoma



91. Ans. d. Popcorn like lesion (Ref: Osborn Neuroradiology/313) MRI appearance of cavernous angioma is highly characteristic, showing “popcorn like lesion” Q.



92. Ans. d. Neuropsychological sequelae are independent of radiation dose (Ref: Tumors of Pediatric CNS (Thieme) 2001/141) • Total radiation dose is the strongest factor determining the magnitude of white matter changes as well as neuropsychological effectsQ.



Section 7



903

93. Ans. a. Nelson syndrome (Ref: Harrison 20/e p2682, 19/e p2273)

Nelson Syndrome • Nelson syndrome refers to a spectrum of symptoms and signs arising from an adrenocorticotropin (ACTH)–secreting pituitary macroadenoma after a therapeutic bilateral adrenalectomy. • The spectrum of clinical features observed relates to the local effects of the tumor on surrounding structures, the secondary loss of other pituitary hormones, and the effects of the high serum concentrations of ACTH on the skin.



94. Ans. a. Brain tumor (Ref: Sabiston 20/e p1923; Schwartz 10/e p1749) Stereotactic surgery is used for treatment of brain tumors 95. Ans. c. Hemangioblastoma (Ref: Harrison 20/e p2754, 19/e p578; Sabiston 20/e p1913; Schwartz 10/e p1735-1736) • Highly vascular tumor of brain and spinal cord in adults Hemangioblastoma.

Neurosurgery

SECTION

8

Head and Neck CHAPTERS ˆˆ ˆˆ ˆˆ ˆˆ

Oral Cavity Salivary Glands Neck Facial Injuries and Abnormalities

CHAPTER

35

Oral Cavity

RISK FACTORS FOR CANCER OF ORAL CAVITY Risk Factors for Cancer of Oral Cavity • • • •

TobaccoQ AlcoholQ Areca nut/pan masalaQ Sharp or jagged toothQ

• Ill-fitting denturesQ • Syphilitic glossitisQ • Human papilloma virusQ

• Epstein-Barr virusQ • Plummer-Vinson syndromeQ • Poor nutrition

Conditions Associated with Malignant Transformation

High-risk Lesions • ErythroplakiaQ • Speckled ErythroplakiaQ • Chronic hyperplastic candidiasisQ

Medium-risk Lesions • Oral submucous fibrosisQ • Syphilitic glossitisQ • Sideropenic dysphagiaQ (Paterson-Kelly syndrome)

Low-risk or Equivocal-risk Lesions • Oral lichen planus • Discoid lupus erythematosus • Discoid keratosis congenita

ORAL SUBMUCOUS FIBROSIS Oral Submucous Fibrosis • Oral submucous fibrosis is a progressive disease in which fibrous bands form beneath the oral mucosaQ. • Almost entirely confined to the Asian populationQ • Risk factor for oral cavity malignancies (squamous cell carcinomaQ) Risk Factors • Research strongly indicates that oral submucous fibrosis is significantly associated with the use of pan masala areca nut, with or without concurrent alcohol useQ. • Tobacco smoking alone is not associated with oral submucous fibrosis. Pathology • Characterised by epithelial fibrosis with associated atrophy & hyperplasia of overlying epitheliumQ. • Epithelium shows changes of epithelial dysplasiaQ. Clinical Features • Scarring produces contracture, resulting in limited mouth opening & restricted tongue movementQ. Treatment • Restricted mouth opening can be treated with either intralesional steroids or surgical excision & skin graftsQ.

IMPORTANT POINTS ABOUT CARCINOMA ORAL CAVITY • • • • • • •

MC gene mutated in CA oral cavity: p53Q MC site of CA oral cavity: Tongue >LipQ MC histological type of CA oral cavity: Squamous cell carcinomaQ MC type of cancer in India: CA oral cavityQ MC site of CA oral cavity in India: Buccal mucosaQ (38%) > Anterior tongue (16%) > Lower alveolus (15.7%) MC pattern of spread: Local extension & regional lymphatic spreadQ LN metastasis is most common in: CA tongueQ > Floor of mouth > Lower alveolus > Buccal mucosa > Upper alveolus > Hard palate > LipQ.

908

Surgery Essence • • • • •

Bilateral lymphatic spread is common in: Lower lipQ, supraglottisQ & soft palateQ. MC site of metastasis: LungQ Edge biopsy is recommended for diagnosis of oral cavity malignanciesQ. MRI: IOC for staging of head & neck malignanciesQ. MC flap used for reconstruction of head & neck malignancies: PMMC (pectoralis major myocutaneous) flap based on pectoral branch of Thoracoacromial vesselsQ

8th AJCC (2017) TNM Classification of Carcinoma Lip & Oral Cavity

Head and Neck

Primary Tumor (T)

Regional Lymph Nodes (N)

Tis

Carcinoma in situ.

N1

Metastasis in a single ipsilateral LN, ≤3 cm in greatest dimension without extranodal extensionQ

T1

Tumor ≤2 cm in greatest dimension & ≤5 mm depth of invasionQ

N2a

Metastasis in single ipsilateral LN, >3 cm but ≤6 cm in greatest dimension without extranodal extensionQ

T2

Tumor ≤2 cm in greatest dimension & >5 mm but ≤10 mm depth of invasion or tumor >2 cm but ≤4 cm in greatest dimension & depth of invasion ≤10 mmQ

N2b

Metastases in multiple ipsilateral LN, none >6 cm in greatest dimension without extranodal extensionQ

T3

Tumor >4 cm in greatest dimension >10 mm depth of invasionQ

N2c

Metastases in bilateral or contralateral LN, none >6 cm in greatest dimension without extranodal extensionQ

T4a

Lip: Tumor invades through cortical bone, inferior alveolar nerve, floor of mouth, or skin (of chin or nose)Q

N3a

Metastasis in a LN >6 cm in greatest dimension without extranodal extensionQ

N3b

Metastasis in a single or multiple LNs with clinical extranodal extensionQ

Oral cavity: Tumor invades through cortical bone mandible or maxillary sinus or invades the skin of faceQ. T4b

Tumor invades masticator space, pterygoid plates, or skull base and/or encases internal carotid arteryQ.

Distant Metastasis M0

No distant metastasis.

M1

Distant metastasis.

Stage Grouping 0 Tis N0M0

I T1 N0M0

II T2 N0M0

III

IVA

T3 N0M0 T1-3 N1 M0

T4a N0-1 M0 T1-4a N2 M0

IVB Any T N3 M0 T4b Any N M0

CARCINOMA LIP Carcinoma Lip

Section 8

• • • • • •

MC site of CA lip: Vermillion of lower lipQ Typically seen in males of 40-70 yearsQ Definite correlation between CA lip and exposure to sunlight (UV radiationsQ) MC presentation: Non-healing ulcer or growthQ LN metastasis is rare and develops late, mainly to submental and submandibular LNsQ. Bilateral lymphatic spread is seen in CA lower lipQ. Treatment of Carcinoma Lip

• T1 and T2 • Surgery is TOCQ • If 1/3rd or less of lip is involved: ‘V’ or ‘W’ shaped full thickness excision with lateral margin of 5 mm + Primary closureQ • If more than 1/3rd of lip is involved: Flap reconstruction (Abbe-Estlander flap)Q • T3 and T4 • Combined radiation and surgeryQ (vermilonectomy or lip shave) Prognosis • CA lip has the best prognosisQ in CA oral cavity.

IVC Any N Any T M1

Oral Cavity

909

Lip Reconstruction Circumoral Advancement Flaps • Karapandzic flapQ: Uses a sensate, neuromuscular flap based on labial arteryQ. • Webster-Bernard repairQ: Use lateral nasolabial flap with buccal advancement

CARCINOMA BUCCAL MUCOSA (CHEEK) Carcinoma Buccal Mucosa (Cheek) • MC site of CA oral cavity in India: Buccal mucosaQ • Related to chewing a combination of tobacco mixed with betel leaves, areca nut and lime shellQ • Most malignant tumors are low grade SCCQ • Frequently appearing on background of leukoplakia • Lymphatic spread is first to level I and II LNsQ. Clinical Feature

Section 8

Cross-lip Flaps • Lip-Switch (Abbe-Estlander) flapQ used to repair defects of either upper or lower lip, based on superior labial arteryQ

• Pain is minimal, obstruction of Stenson’s duct can lead to parotid enlargement. Treatment • T1: Excision with primary closureQ • T2: Surgery ± RadiotherapyQ • T3 and T4: Surgery + Radiotherapy or chemoradiationQ

CARCINOMA TONGUE Carcinoma Tongue • MC site is middle of lateral borderQ or ventral aspect of the tongue. • MC histological type is squamous cell carcinomaQ. • MC associated risk factors are tobacco and alcoholQ. • MC variety is ulcerativeQ. • 30% patients presents with cervical node metastasisQ.

Carcinoma oral tongue • • • • • •

The intrinsic tongue musculature provide little restriction to tumour growth, thus it may enlarge considerably before producing symptoms. Presents as painless mass or ulcer that fails to heal after minor traumaQ MC complaint: Mid-irritation of tongueQ. MC site: Lateral border of the junction of middle & posterior thirdQ. Primary basin for cervical metastasis is superior deep jugular nodes (Level II) Q. For diagnosis, wedge biopsy is taken from the edge of ulcer but in proliferative growth, punch biopsy is takenQ. T1 T2 T3 T4

• • • • •

Treatment of Carcinoma Oral Tongue Partial glossectomy with primary closureQ Hemiglossectomy for small well-circumscribed and well differentiated lesionQ Radiotherapy for large, poorly differentiated lesionQ Total glossectomy followed by radiationQ Surgery (Total glossectomy, mandibulectomy, MRND, laryngectomy) + Postoperative radiationQ

Management of Recurrence • Most recurrences occur within 2 years. • Radiation failure is managed by glossectomyQ. • Surgical failure is managed by radiationQ. • If recurrence is limited to mucosa, it is best managed by surgery • If recurrence is in the soft tissue of the neck, palliation is indicated.

Head and Neck

CARCINOMA ORAL TONGUE

910

Surgery Essence CANCER OF HARD PALATE Cancer of Hard Palate • SCC of hard palate is rare, Associated with reverse smoking • Minor salivary gland tumors occur in the hard palate. • Most cancers are well differentiated and of ulcerative varietyQ. Clinical Features • Presents as painless massQ in the roof of the mouth • Lymphatic metastasis is uncommon, mainly to level I and II.

Head and Neck

Treatment • Smaller tumors: Excision with underlying bone. • Larger tumors: Maxillectomy • Radiotherapy is used in advanced lesions.

CHEMOTHERAPY IN CANCERS OF ORAL CAVITY, HEAD AND NECK Chemotherapy in Cancers of Oral Cavity, Head and Neck • Adjuvant chemotherapy has been reported to improve the rate of organ preservation with no change in overall survivalQ. • Chemotherapy is often employed in palliative setting in patients with recurrent, unresectable or distant metastasesQ. • Drugs used: CisplatinQ, Methotrexate, 5-FU, Docetaxel and Paclitaxel • Cisplatin is the cornerstone drug in the modern management of head & neck cancerQ. • Most beneficial is concurrent chemotherapyQ. • The addition of concurrent chemotherapy (cisplatin) to conventional radiation significantly improved survival over radiation aloneQ. • Concurrent chemoradiation protocols have improved locoregional control and reduce the development of distant diseaseQ.

CARCINOMA OF POSTERIOR THIRD OR BASE OF TONGUE Carcinoma of Posterior Third or Base of Tongue • Remains asymptomatic for long time and patient present with metastasis in cervical nodesQ. • First node involved is superior deep jugular nodes (Level II), spread is then along the jugular chain to the mid-jugular (Level III) and lower jugular (Level IV) Q. Clinical Features • Early symptoms: Sore throat, feeling of lump in throat, and slight discomfort on swallowing • Because many lesions are silent, level II neck mass is often the first signQ.

RADIOTHERAPY IN CANCERS OF ORAL CAVITY Radiotherapy in Cancers of Oral Cavity

Section 8

• • • •

SCC is vascularized and well-oxygenated tends to be most radiosensitiveQ. Deep invasion of muscle or bone tends to decrease the response to radiotherapy Large cervical metastatic nodes are best managed by a combination of surgery and radiation therapy rather than by radiation aloneQ. Mostly given as EBRT (External Beam Radiotherapy), 60 Gray over 6 weeksQ. Complications of Radiotherapy 1. Xerostomia (MC)Q 2. MucositisQ 3. Temporary or permanent dysgeusia 4. Osteoradionecrosis (ORN)Q: −− Related to carries tooth in the radiation fieldQ −− Results from decreased production of saliva and damage to microvasculature of mandible and maxillaQ −− Best managed with prophylactic dental careQ −− ORN may require daily hyperbaric oxygen treatments for 4-6 weeks, either alone or in conjunction with surgical interventionQ

Oral Cavity

911

MALIGNANT NEOPLASMS OF PARANASAL SINUSES

• Most frequently involved: Maxillary sinusesQ > Ethmoids > Frontal > Sphenoid. • Ethmoidal tumors mainly spread to jugulodiagastric and subdiagastric nodesQ. • Maxillary tumors mainly spread to mandibular nodesQ. • People working in hardwood furniture industry, nickel refining, leather work and manufacture of mustard gas have shown higher incidence of sinonasal cancerQ. • Cancer of the maxillary sinus is common in Bantus of South Africa where locally made snuff is used, which is found rich in nickel and chromiumQ.

Section 8

Malignant Neoplasms of Paranasal Sinuses

• Workers of furniture industry develop adenocarcinoma of the ethmoids and upper nasal cavityQ • Those engaged in nickel refining develop SCC and anaplastic carcinomaQ. • More than 80% of the malignant tumors are of SCC varietyQ.

CARCINOMA OF MAXILLARY SINUS Carcinoma of Maxillary Sinus • Common in 40-60 years of age, more common in malesQ • Systemic metastasis are rare, may be seen in lungs (MC)Q and occasionally in bone. Clinical Features • Early features of maxillary sinus malignancy are nasal stuffiness, blood stained discharge, facial paraesthesia, or pain and epiphoraQ. • Nodal metastasis is uncommon and occurs only in the late stage of disease. • Maxillary tumors mainly spread to mandibular nodesQ. Diagnosis • CT scan: Best non-invasive method to find the extent of disease. Treatment • For SCC, combination of radiotherapy and surgeryQ gives better results than either alone. • Radiotherapy can be given before or after surgery.

MANDIBULECTOMY

Marginal Mandibulectomy • Conservative mandibulectomyQ • Refers to partial excision of the superior portion of mandible in vertical phaseQ • Inner cortical surface and a portion of underlying medullary cavity is excisedQ • Preserve mandibular continuityQ • Indicated when tumor lies within 1 cm of the mandible or abuts the periosteum without evidence of direct bony invasionQ

• • • •

Segmental Mandibulectomy Entire through and through segment of mandible is resected. Results in mandibular discontinuityQ Requires major reconstructive procedure for cosmetic and functional purposesQ Indications: 1. Invasion of medullary space of mandibleQ 2. Tumor fixation to occlusal surface of mandible in edentulous patientQ 3. Invasion of tumor into the mandible via mandibular or mental foramenQ 4. Tumor fixed to the mandibleQ

TRISMUS Trismus • • • •

Restriction to mouth opening, including restrictions caused by trauma, surgery or radiationQ. Implications: Reduced nutrition due to impaired mastication, difficulty in speaking, and compromised oral hygieneQ. Often observed in persons who have received radiation to the head and neck, in conjunction with difficulty in swallowingQ. Limited jaw mobility can result from trauma, surgery, radiation treatment, or even TMJ problemsQ. • RadiationQ that affects the temporomandibular joint, the pterygoid muscles, or the masseter muscle, is most likely to result in trismus. • Some patients who have not received radiation treatment may develop trismus secondary to scarring and edema after surgeryQ.

Head and Neck

Mandibulectomy

912

Surgery Essence CANCRUM ORIS (NOMA DISEASE OR GANGRENOUS STOMATITIS) Cancrum Oris (Noma Disease or Gangrenous Stomatitis) • Cancrum oris  or  gangrenous stomatitis, is a  gangrenous  disease   leading to  tissue destruction of the face, especially the  mouth  and cheekQ • Rapidly progressive, polymicrobial, opportunistic infection that occurs during periods of compromised immune functionQ.  • Main organisms implicated: Fusobacterium, Prevotella and  Borrelia vincentiiQ

Head and Neck

Predisposing Factors • • • •

Malnutrition or dehydrationQ Poor oral hygieneQ Poor sanitationQ Unsafe drinking water

• Recent illnessQ • MalignancyQ • Immunodeficiency disorder (AIDS)Q

Clinical Features • • • • •

Mainly affects children 10 mm • Appearance similar to minor ulcerations • More painful • Take >1 month to heal and leave a scarQ. • Typically develop after puberty with frequent recurrencesQ. Q

Herpetiform Ulcerations • Characterized by small, numerous, 1-3 mm lesions that form clusters. • Most severe formQ • Occurs more frequently in females • Onset is often in adulthood. • Typically heal in 3 cm

CARCINOMA LIP

21. Abbe-Estlander flap is used for: (All India 2008) a. Lip b. Tongue c. Eyelid d. Ears 22. Abbey Estlander flap is based on: (AIIMS May 2008) a. Lingual artery b. Facial artery c. Labial artery d. Internal maxillary artery 23. Carcinoma of lip is characterized by the following except that: (UPSC 2007) a. 90% of the lip cancers occurs on the lower lip b. The most common site of origin is the vermillion border c. 2 cm x 2 cm cell carcinomas can be treated by V-shaped excision and primary closure d. Since lymph node metastases are common after a radical dissection of neck is mandatory

914

Surgery Essence

24. Stain used to diagnose premalignant lesions of lip is: (DNB 2011, 2006) a. Crystal violet b. H and E c. Toluidine blue d. Giemsa



25. Treatment of choice for carcinoma of lip of less than 1 cm is: a. Radiation (All India 90) b. Chemotherapy c. Excision d. Radiation and chemotherapy

Head and Neck



27. True statement (s) about oral cancer is/are: (PGI June 2004) a. Most common in buccal mucosa b. Metastasis uncommon c. Respond to radiotherapy d. Surgery done e. Syphilis and dental irritation predisposes

28. Metastasis of CA buccal mucosa goes to: (AIIMS Nov 96, All India 97) a. Regional lymph node b. Liver c. Heart d. Brain







29. In carcinoma cheek, what is the best drug for single drug chemotherapy? (AIIMS June 93) a. Vincristine b. Cyclophosphamide c. Cisplatin d. Daunorubicin 30. An old man who is edentulous developed squamous cell carcinoma in buccal mucosa that has infiltrated to the alveolus. Following is not indicted in treatment: a. Radiotherapy (All India 2002) b. Segmental mandibulectomy c. Marginal mandibulectomy involving removal of the outer table only d. Marginal mandibulectomy involving removal of upper half of mandible



36. A patient with CA tongue is found to have lymph nodes in the lower neck. The treatment of choice for the lymph nodes is: a. Lower cervical neck dissection (All India 2005) b. Suprahyoid neck dissection c. Tele radiotherapy d. Radical neck dissection



37. Carcinoma of tongue most commonly occur at: a. Dorsum (Recent Question 2013 MCI March 2009) b. Lateral border of anterior 2/3rd c. Lateral border of posterior 1/3rd d. Tip



38. Tongue ulcer with everted edges is: (MHPGMCET 2005, 2001) a. Aphthous ulcer b. Tubercular c. Malignant d. Dental 39. A 60-year-old man presents with an ulcer on lateral margin of tongue also complains of ear pain, most probable diagnosis is: (PGI 96) a. Dental ulcer b. Carcinomatous ulcer c. Tuberculosis ulcer d. Syphilitic ulcer 40. Carcinoma of the tongue: a. Occurs most commonly on the lateral border of the middle third of tongue b. Metastasize readily to cervical lymph nodes c. Is usually radiosensitive d. Treated surgically should include homolateral neck dissection except for very small lesion e. All of the above







31. A patient with cheek cancer has a tumour of 2.5 cm located close to and involving the lower alveolus. A single mobile ipsilateral lymph node measuring 6 cm is palpable. The TNM stage is: (Recent Question 2013, COMEDK 2008) a. T1N1M0 b. T2N2M0 c. T2N1M0 d. T4N2M0 32. In the reconstruction following excision of previously irradiated cheek cancer, the flap will be: a. Local tongue b. Cervical c. Forehead d. Pectoralis major myocutaneous

CARCINOMA PALATE

Section 8

35. A patient presented with a 1 X 1.5 cms growth on the lateral border of the tongue. The treatment indicated would be: a. Laser ablation (Recent Question 2014, AIIMS June 2002) b. Interstitial brachytherapy c. External beam radiotherapy d. Chemotherapy

26. Neuromuscular preserving flap in lip: (Recent Question 2017) a. Abbe flap b. Webster flap c. Karpandzic flap d. Johansen flap

CARCINOMA BUCCAL MUCOSA AND CHEEK



CARCINOMA MAXILLA

42. Treatment for stage T3N1 of carcinoma maxilla is: (DNB 2012, AIIMS June 96) a. Radiation therapy only b. Chemotherapy only c. Surgery and radiation d. Chemotherapy and radiation



43. The lymph not to be involved first in maxillary carcinoma: a. Superior deep cervical nodes (DNB 2007) b. Jugulodigastric nodes c. Submandibular d. Subdigastric nodes

33. All are true about carcinoma palate, except: (AIIMS June 94, Nov 93) a. Slow growing b. Bilateral lymphatic spread c. Adenocarcinoma d. Presents with pain

CARCINOMA TONGUE

34. A patient has carcinoma of tongue in the right lateral aspect with lymph node of 4 cm size in level 3 on the left side of neck, what is the stage? (AIIMS Nov 2006) a. N0 b. N1 c. N2 d. N3

41. The commando operation is: (Recent Question 2016) a. Abdomino-perineal resection of the rectum for carcinoma b. Disarticulation of the hip for gas gangrene of the leg c. Extended radical mastectomy d. Excision of carcinoma of the jaw and lymph nodes enbloc

MANDIBLE AND MANDIBULECTOMY

44. A 80-year-old patient presents with a midline tumor of the lower jaw, involving the alveolar margin. He is edentulous. Treatment of choice is: (All India 2001) a. Hemimandibulectomy b. Commando operation c. Segmental mandibulectomy d. Marginal mandibulectomy

Oral Cavity EPULIS

TRISMUS 45. Trismus in oral cancer patients is severe in those treated with: (Karnataka 99) a. Surgery and Radiotherapy b. Chemotherapy alone c. Surgery alone d. Not related to treatment



CANCRUM ORIS

46. All are true about cancrum oris except: (PGI Dec 97) a. Associated with malnutrition and vitamin deficiency b. Follows chronic infection c. Involves jaw d. Treatment is excision and skin grafting with tubed pedicle graft



56. Epulis arises from: a. Enamel c. Gingiva 57. Epulis is: a. Benign c. Reactive process

(Recent Question 2013, PGI Dec 99) b. Root of teeth d. Pulp (DNB 2014) b. Malignant d. Precancerous

58. Which of the following statement is incorrect about the given condition?

Section 8



915

LUDWIG’S ANGINA

47. Which of the following statements best represent Ludwig’s angina? (AIIMS May 2005) a. A type of coronary artery spasm b. An infection of the cellular tissues around submandibular salivary gland c. Esophageal spasm d. Retropharyngeal infection

a. Arise from gingiva b. Benign c. Types are fibromatous, ossifying and acanthomatous d. None of the above

DENTAL CYST AND ABNORMALITIES

MISCELLANEOUS





54. What are Rushton bodies? a. Hyaline bodies of odontogenic cyst b. Refractile bodies of radicular cyst c. Bodies seen in ameloblastoma d. Hyaline bodies seen in dentigerous cyst













(DNB 2012)

55. Which of the following is correct about ameloblastoma? a. Highly malignant (Recent Question 2016) b. Occurs in children 2 cm b. Metastasis in lymph nodes >5 cm c. Metastasis in a lymph node >6 cm d. None

Head and Neck

48. The most frequent tooth to be impacted is: (UPSC 2007, Karnataka 98) a. Lower third molar b. Lower canine c. Upper third molar d. Upper premolar 49. Impacted wisdom teeth may produce referred pain via: a. Lingual nerve (Orissa 99) b. Facial nerve c. Branch of the auriculotemporal nerve d. None of the above 50. Radiographic finding of floating teeth can be seen in:  (Karnataka 2002) a. Ectodermal dysplasis b. Cleidocranial dysplasia c. Osteopetrosis d. Histiocytosis-X 51. The most common cyst of the oral region is: (DPG 2008) a. Dentigerous cyst b. Keratosis cyst c. Dermoid cyst d. Periapical cyst 52. Dentigerous cyst arises from: (DPG 2009 Feb, MHPGMCET 2003) a. The root of a caries tooth b. The periosteum of the fractured mandible c. An unerupted permanent tooth d. The sequestrum of osteomyelitis of mandible 53. Which jaw cyst is premalignant? (AIIMS May 2012) a. Nasopalatine cyst b. Radicular cyst c. Odontogenic keratocyst d. Dentigerous cyst

Explanations CARCINOMA ORAL CAVITY PREDISPOSING FACTORS

1. Ans. c. Erythroplakia has a higher risk for malignancy (Ref: Bailey 27/e p761-762; Devita 9/e p564; Cancer of the Head and Neck by Suen and Myer 4/e p284-285) • Oral submucous fibrosis is almost entirely confined to the Asian population and is characterized pathologically by epithelial fibrosis with associated atrophy and hyperplasia of the overlying epithelium. Leukoplakia

Erythroplakia

• White keratotic plaque or patch that cannot be rubbed off and cannot be given another diagnostic name • Risk factors: Smoking, alcohol, ill-fitting dentures, jagged toothQ • Key pathologic features: Hyperkeratosis, parakeratosis, acanthosisQ • In most cases, lesions regress spontaneously after stopping alcohol or tobacco consumption or correction of underlying causeQ. • Baseline biopsy should be doneQ • Lesions with moderate to severe dysplasia should be excisedQ. • Oral leukoplakia has low potential for malignancyQ. Q

• Red mucosal plaque, most commonly found on the soft palate and tonsillar pillarsQ • Does not arise from an obvious mechanical or inflammatory cause • Cannot be ascribed to another clinical or pathological condition • Key pathologic features: severe cellular dysplasiaQ • Because of increased malignant potential, all erythoplakic lesions must be biopsiedQ. • Higher risk (17 times) of malignant transformation than leukoplakiaQ.



2. Ans. c. Discoid lupus erythematosus (Ref: Sabiston 20/e p789; Schwartz 10/e p580; Bailey 27/e p761)



3. Ans. b. Erythroplakia

4. Ans. d. Repositioning of ill fitting dentures

Leukoplakia • In most cases, lesions regress spontaneously after stopping alcohol or tobacco consumption or correction of underlying causeQ.

5. Ans. a. Excision

6. Ans. d. Lichen planus



7. Ans. a. Submucous oral fibrosis (Ref: Bailey 27/e p763; Devita 9/e p729)



8. Ans. a. EBV, c. HPV

9. Ans. a. Lichen planus of mouth



11. Ans. a. Leukoplakia

12. Ans. c. Ascertaining that denture is fitted properly



13. Ans. c. Betel nut chewing

10. Ans. d. Erythroleukoplakia

CARCINOMA ORAL CAVITY

14. Ans. a. Oral cavity (Ref: Second primary malignancies in patients with head and neck cancers by Sandeep Samant, Head and Neck 2005; 27; 1042) • “The site of index cancer influences the most likely site of a second primary malignancy, as an example, in a series of 1257 patients with SCC of the head and neck, patients with an index malignancy arising in the larynx were more like to develop a second primary cancer in the lung, while those arising in the oral cavity were more likely to develop a second primary in the head and neck or esophagus. This relationship has been observed in other studies as well.” • Patients with head and neck SCC are at increased risk for the development of second primary malignancies compared with general populationQ. • These second primary malignancies typically develop in the aerodigestive tract (lung, head and neck, esophagus)Q. • MC second primary malignancy: Lung cancerQ • Highest relative increase in risk is for a second head and neck cancerQ.



15. Ans. c. Alveobuccal complex (Ref: Bailey 26/e p709-710; Devita 9/e p729; Cancer of the Head and Neck by Suen and Myer 4/e p297-304)



16. Ans. c. Tongue

17. Ans. d. 2% toluidine blue (Ref: www.headandneckoncology.org/content/1/1/5)

Toluidine Blue • Toluidine blue is a basic metachromatic dye with high affinity for acidic tissue components, thereby staining tissues rich in DNA and RNAQ. • Wide applications both as vital staining in living tissues and as a special stain used in vivo to identify dysplasia and carcinoma of the oral cavityQ.

Oral Cavity

917

19. Ans. b. Tongue (Ref: Sabiston 20/e p796; Schwartz 10/e p582; Bailey 27/e p765)



18. Ans. a. I, II, III.



20. Ans. d. Lymph node >3 cm (Ref: Devita 10/e p430)



21. Ans. a Lip: (Ref: Bailey 27/e p769; Devita 9/e p744-745; Cancer of the Head and Neck by Suen and Myer 4/e p301-302)



22. Ans. c. Labial artery (Ref: Bailey 27/e p769; Devita 9/e p744; Cancer of the Head and Neck by Suen and Myer 4/e p301-302) Lip Reconstruction Cross-lip Flaps

Circumoral Advancement Flaps

• Lip-Switch (Abbe-Estlander) flapQ used to repair defects of either upper or lower lip, based on superior labial arteryQ

Section 8

CARCINOMA LIP

• Karapandzic flapQ: Uses a sensate, neuromuscular flap based on labial arteryQ. • Webster-Bernard repairQ: Use lateral nasolabial flap with buccal advancement



23. Ans. d. Since lymph node metastases are common after a radical dissection of neck is mandatory



24. Ans. c. Toluidine blue (Ref: Indian Journal od Dental Research 2007; vol-18; Issue 3; p103-105) • Toluidine blue detects efficiently and rapidly mitotic figures in sections of paraffin embedded human tissues especially in oral cavity.

25. Ans. c. Excision

26. Ans. c. Karpandzic flap (Ref: Grabb & Smith /p378)

CARCINOMA BUCCAL MUCOSA AND CHEEK 27. Ans. b. Metastasis uncommon, c. Respond to radiotherapy, d. Surgery done, e. Syphilis and dental irritation predisposes (Ref: Bailey 27/e p772; Devita 9/e p749; Cancer of the Head and Neck by Suen and Myer 4/e p302-305)

28. Ans. a. Regional lymph node



29. Ans. c. Cisplatin (Ref: Bailey 27/e p155; Devita 9/e p749; Cancer of the Head and Neck by Suen and Myer 4/e p291-292)



30. Ans. c. Marginal mandibulectomy involving removal of the outer table only • Whenever SCC of oral cavity involve the mandible (or within 1 cm of mandible), mandibulectomy becomes necessaryQ. • In marginal mandibulectomy, inner cortical surface and a portion of underlying medullary cavity is excised (not only the outer table) Q. 31. Ans. d. T4N2M0



32. Ans. d. Pectoralis major myocutaneous (Ref: Bailey 26/e p716; Devita 9/e p749; Cancer of the Head and Neck by Suen and Myer 4/e p302-305)

Reconstruction of Cheek • For cheek reconstruction, mucosal flaps are used. • PMMC (pectoralis major myocutaneous) flap: Most widely used flap for head and neck reconstructionQ. Males • Forehead flap based on anterior branch of superficial temporal artery can be usedQ.

Females • Deltopectoral flap based on perforating branch of internal mammary artery is usedQ.

CARCINOMA PALATE

33. Ans. d. Presents with pain (Ref: Bailey 27/e p774; Devita 9/e p750; Cancer of the Head and Neck by Suen and Myer 4/e p311-313)

CARCINOMA TONGUE

34. Ans. c. N2



35. Ans. b. Interstitial brachytherapy: (Ref: Bailey 27/e p769; Devita 9/e p747-749, 752-754; Cancer of the Head and Neck by Suen and Myer 4/e p297-301) • Suen and Myer says “Radiation therapy may be curative in early cancer (T1 and some T2) and may preserve maximal normal anatomy and function. Brachytherapy allows delivery of a large radiation boost to the primary tumor bed.”

Head and Neck



918

Surgery Essence 36. Ans. d. Radical neck dissection (Ref: Bailey 27/e p769; Devita 9/e p748; Cancer of the Head and Neck by Suen and Myer 4/e p299)



Lymph Node Metastasis in CA Tongue • Elective or therapeutic treatment of the cervical lymphatics is recommended for virtually all patients with cancer of the oral tongue. • It is recommended that patients with bulky metastatic deposits undergo standard radical dissection or MRNDQ.

38. Ans. c. Malignant (Ref: Bailey 27/e p616)

37. Ans. b. Lateral border of anterior 2/3rd

Head and Neck

Type of Ulcer

Edge

• Septic ulcer

• Sloping edgesQ

• Tuberculous ulcer

• Undermined edgesQ

• Carcinomatous ulcer

• Everted hard edgesQ

• Rodent ulcer

• Barely visible pearly edgesQ

• Syphilitic ulcer

• Punched-out appearance with raised indurated edgesQ



39. Ans. b. Carcinomatous ulcer



41. Ans. d. Excision of carcinoma of the jaw and lymph nodes en-bloc (Ref: Cancer of the Head and Neck by Suen and Myer 4/e p291)

40. Ans. e. All of the above

Commando’s Operation (Combined Mandibulectomy and Neck Dissection Operation) • Commando’s operation: Total glossectomy hemimandibulectomy + Removal of floor of mouth + Radical lymph node dissectionQ • Indicated when carcinoma is fixed to mandible with infiltration of floor of mouthQ.

CARCINOMA MAXILLA

42. Ans. c. Surgery and radiation: (Bailey 27/e p774; Devita 9/e p768-771; Cancer of the Head and Neck by Suen and Myer 4/e p179)



43. Ans. c. Submandibular nodes (Ref: Grays 39/e p577)

MANDIBLE AND MANDIBULECTOMY

44. Ans. c. Segmental mandibulectomy (Ref: Devita 9/e p746; Cancer of the Head and Neck by Suen and Myer 4/e p293-294)

TRISMUS

45. Ans. a. Surgery and Radiotherapy: (Ref: Bailey 25/e p750)

CANCRUM ORIS

46. Ans. b. Follows chronic infection (Ref: www.ncbi.nlm.nih.gov)

LUDWIG’S ANGINA

47. Ans. b. An infection of the cellular tissues around submandibular salivary gland: (Ref: Bailey 27/e p756)

Ludwig’s Angina

Section 8

• Ludwig’s angina is infection of submandibular spaceQ. • Bacteriology: Mixed infectionsQ involving both aerobes and anaerobesQ are common. Etiology: • Dental infections are responsible for 80% of cases Q. Clinical Features: • Marked difficulty in swallowing (odynophagia)Q, with varying degree of trismus. Treatment: • Systemic antibioticsQ with incision and drainage of abscessQ.

DENTAL CYST AND ABNORMALITIES

48. Ans. a. Lower third molar (Ref: Scott-Brown’s Otorhinolaryngology 7/e p1924-1925)

Oral Cavity

919

Impacted Tooth



49. Ans. c. Branch of the auriculotemporal nerve: (Ref: Scott-Brown’s Otorhinolaryngology 7/e p1924-1925) • Unerupted  wisdom teeth, erupting teeth, and malocclusion can cause ear  pain secondary to direct impingement of the auriculotemporal nerveQ.



Section 8

• Tooth that has failed to erupt completely or partially to its correct position in the dental arch and its eruption potential has been lost. • MC affected tooth: Lower 3rd molarQ >Upper 3rd molar >Upper canine

50. Ans. d. Histiocytosis-X (Ref: Sutton 7/e p1542)

Floating Tooth Sign • On radiographic examination of mandible, erosion of the bony alveoli around the teeth, so that they seem to be floating in spaceQ • Seen in histiocytosis XQ

51. Ans. d. Periapical cyst (Ref: Scott-Brown’s Otorhinolaryngology 7/e p1924-1925)

Periapical Cyst (Radicular Cyst) • • • • • •

MC type of jaw cyst Periapical cyst is inflammatory in originQ. Extremely common lesions found at the apex of teethQ. Develop as a result of long-standing pulpitis, caused by advanced carious lesions or by trauma to the tooth. Periapical inflammatory lesions persist as a result of the continued presence of bacteria or other offensive agents in the area. Treatment: Complete removal of offending material and appropriate restoration of the tooth or extraction. Q

52. Ans. c. An unerupted permanent tooth (Ref: Scott-Brown’s Otorhinolaryngology 7/e p1924-1925) • Dentigerous cyst arises from an unerupted permanent tooth. Dental Abnormalities



Dentigerous Cyst

Ameloblastoma

• Develops at the apices of caries toothQ with necrotic pulp • More common in upper jawQ

• Arises from unerupted teeth • MC in lower 3rd molars • Cyst also contain unerupted teethQ

X-rays shows: • Circular radioluscent areaQ in relation to root of normally erupted tooth • Margins may be scleroticQ

X-ray shows: • Unilocular cyst or soap bubble appearanceQ

Q

• Benign neoplasm arising from ameloblast (odontogenic epithelium)Q • MC odontogenic tumourQ • Most ameloblastoma occur in mandible near angle and ramus of mandibleQ

53. Ans. c. Odontogenic keratocyst (Ref: Scott-Brown’s Otorhinolaryngology 7/e p1924, 1925)

Odontogenic Keratocyst • • • •

Odontogenic keratocyst is locally aggressive and has a high rate of recurrenceQ. Most often diagnosed in patients between 10-40 years. Occur most commonly in males within the posterior mandible, particularly in the region of 3rd molar toothQ. Radiographically present as well-defined unilocular or multilocular radiolucencies. • Odontogenic keratocysts are characterized by an epithelial lining that is parakeratinized & stratified. • It is characterized by basal layer of neatly arranged, palisaded, columnar and cuboidal cells above which are several layers of squamous epithelium. This lining has a high mitotic rate and rarely may become dysplastic and develop into squamous cell carcinomaQ.

Treatment: • Aggressive and complete removal of the lesion • Recurrence rates for inadequately removed lesions can reach 60%Q.

Head and Neck

Dental Cyst

920

Surgery Essence

54. Ans. a. Hyaline bodies of odontogenic cyst (Ref: Shafers 6/e p268)

Rushton Bodies • Rushton bodies or hyaline bodies of odontogenic cysts feature as eosinophilic, straight or curved, irregular or rounded structure within the epithelial lining of odontogenic cyst. • Rushton bodies occur almost exclusively within odontogenic cyst.

55. Ans. c. Most common odontogenic tumour

Head and Neck

EPULIS

56. Ans. c. Gingiva

Epulis • Epulis is any benign lesion situated on the gingiva.     •  Three types: fibromatous, ossifying and acanthomatous.

57. Ans. a. Benign



58. Ans. d. None of the above) Epulis arises from gingiva, is benign and types are fibromatous, ossifying and acanthomatous.

MISCELLANEOUS

59. Ans. a. Watchful expectancy (Ref: Bailey 25/e p716, 717)

Asymptomatic hemangioma on ventral surface of the tongue in 10 years old boy is treated by watchful expectancy.

Hemangioma • Mucosal hemangiomas can occur in oral cavity or oropharynxQ. • Mostly seen in childrenQ • When hemangiomas are present at birth or in young children, they should be only observed for some period as spontaneous regression can occurQ.

60. Ans. a. Chemotherapy (Ref: http://www.ncbi.nlm.nih.gov/pmc/articles/PMC2640019/)

Pharyngocutaneous Fistula • Pharyngocutaneous fistula (PCF) is the MC complication after total laryngectomyQ. • Reported incidence: 3-65% • Appears in the early post-operative period after total laryngectomyQ (3rd- 8th post-operative day). • Risk Factors: − Wound closure under tensionQ − Concurrent neck dissectionQ − Prior radiation therapyQ

61. Asn. a. T3 N2 M0, b. T2 N2 M0, c. T4 N2 M0, d. T3 N3 M0



63. Ans. d. Carcinomatous ulcers

62. Ans. b. Aphthous ulcer (Ref: Robbins 9/e p728)

Section 8

Aphthous ulcers, tubercular and herpetic ulcers are painful.

Carcinomatous Ulcer • Carcinomatous ulcers are painless but may become painful in advanced stages, with extension into surrounding tissues.

64. Ans. b. Syphilis (Ref: Robbins 9/e p379) • Syphilis chancres can occur in the mouth and they are painless.



65. Ans. d. Curettage (Ref: medind.nic.in/ibn/t06/i4/ibnt06i4p677) • Reparative granuloma of Jaw is treated by curettage.

Oral Cavity

921

Giant Cell Reparative Granuloma

Clinical Features • It is a disease of the young presenting as a painless swelling in the anterior jaw and • Radiographically appearing as a lytic expansile lesion with a characteristic tendency of resorbing the root tips of adjacent unerupted teeth.

Section 8

• Giant cell reparative granuloma is an apparently reactive intraosseous lesion of the mandible and maxilla following trauma induced intraosseous hemorrhageQ and containing prominent giant cells. • Also known as Central giant cell granulomaQ • MC site: Anterior part of mandibleQ (2/3rd of cases) between the 2nd premolar and 2nd molarQ with extension across the midline. • 2nd MC site: Small bones of hands and feetQ

Treatment • Curettage or local excisionQ • Recurrence rate: 22-50% • Lesion eradication typically does not require >2 excisions. • Chemical cautery, electrocautery, cryotherapy, calcitonin, Interferon alpha and intralesional steroids are used for more aggressive and recurrent lesions.

66. Ans. d. Surgery and radiotherapy (Ref: Mastery of Surgery 5/e p308; Bailey 27/e p767)



67. Ans. c. Metastasis in a lymph node >6 cm

Head and Neck

CHAPTER

36

Salivary Glands

ETIOLOGY OF SALIVARY GLAND TUMORS Etiology of Salivary Gland Tumors • • • • • •

Radiotherapy to head and neck (for mucoepidermoid carcinoma)Q EBV infection (for lymphoepithelial carcinoma)Q Exposure to silica dust, nitrosaminesQ Increased risk in females with early menarche and nulliparityQ Trisomy 5 in primary mucoepidermoid carcinoma of minor salivary glands Polysomy of 3 and 17 especially in adenoid cystic carcinoma • • • • • • • • •

Translocation involving chromosome 11 in mucoepidermoid carcinoma MC neoplasm of salivary gland: Pleomorphic adenomaQ MC malignant tumor of salivary gland: Mucoepidermoid carcinomaQ MC neoplasm of salivary gland in children: HemangiomaQ MC malignant tumor of salivary gland in children: Mucoepidermoid carcinomaQ MC malignant tumor of minor salivary glands: Adenoid cystic carcinomaQ Best diagnostic modality for parotid swelling: FNACQ Open incisional biopsy is contraindicatedQ due to tumor cell implantation and formation of parotid fistulaQ. Best imaging investigation for salivary gland neoplasms: MRIQ

IN SALIVARY GLAND TUMORS In Salivary Gland Tumors • MC site of minor salivary gland tumors are oral cavity (hard palate)Q • There are no minor salivary glands in the anterior half of the palate, so tumors arise on posterolateral hard palate and all of the soft palateQ • Malignancy varies inversely with the size of glandQ (most of minor salivary gland tumors are malignant) • Parotid gland: 25%Q malignant, Submandibular and Sublingual gland: 50%Q malignant, Minor salivary glands: 75%Q malignant • Open surgical biopsy is contraindicatedQ, as it can cause tumor seeding of the track • Most salivary gland tumors are radioresistantQ • Neutron therapy has been used in the management of unresectable salivary gland tumorsQ • Name of incision for parotidectomy: Sistrunk incisionQ Indications of Radiotherapy in Salivary Gland Tumors • High grade tumorsQ • Bone invasionQ • Large primary lesionsQ • Cervical LN metastasisQ Q • Perineural invasion • Positive surgical marginsQ

PLEOMORPHIC ADENOMA Pleomorphic Adenoma • It is MC benign salivary gland tumor and MC tumor of major salivary glandsQ. • MC site is parotid tail (superficial lobe)Q • Less common in the submandibular glands and sublingual glands, relatively rare in minor salivary glands.

Salivary Glands

• Pleomorphic adenoma is unicentric but recurrences are multicentricQ • Usually not involve the facial nerve. Clinical Features • Presents as painless swelling without any appreciable change in size, with typical site at below, in front and behind the ear lobuleQ. • Slow growing lobular tumor affecting women around 40 years. • Pleomorphic adenoma involving deep lobe may push the tonsil and pillars of fauces towards midline and known as dumbbell tumorQ with component both in neck and oral cavity.

Section 8

• Known as mixed tumorQ as it is composed of both epithelial & mesenchymal components • Encapsulated but sends pseudopodia (finger-like projections)Q into surrounding glands, enucleation is not done to avoid recurrence.

923

Diagnosis • FNAC is diagnosticQ Treatment • Superficial parotidectomy (Patey’s operation)Q • Excision of whole gland in cases of pleomorphic adenoma of submandibular gland • Name of incision for parotidectomy: Lazy ‘S‘, modified Blairs or Sistrunk incision Complications • Malignant change (3-5%) −− Known as carcinoma ex pleomorphic adenoma or malignant mixed tumor −− Rapid growth, pain, paraesthesia, enlarged cervical LN and restriction of jaw movements, facial weakness or skin invasion and fixation of mastoid tip is suggestive of malignant transformation −− Histological findings suggestive of malignant change are microscopic foci of necrosis, hemorrhage, calcification and excessive hyalinizationQ −− Prognosis for invasive carcinoma depends on degree of local infiltration • RecurrenceQ, particularly after enucleation

WARTHIN’S TUMOR (PAPILLARY CYSTADENOMA LYMPHOMATOSUM)

• Second MC benign tumor of the parotid glandQ • Derived from salivary tissues inclusion in lymph nodesQ (so can arise from cervical nodes) • Occurs exclusively in parotid glandQ and almost always occur in the lower portion of parotidQ overlying the angle of mandible. • Consists of both epithelial & lymphoid elementsQ thus known as adenolymphoma (probably arises from remnants of parotid tissue trapped in lymph nodes within the parotid gland) • More common in malesQ, in 5th to 7th decadeQ. • Associated with smokingQ, bilateral in 10%Q cases, never involves facial nerve. • It is well encapsulated, cystic, extremely slow growing tumor, never turns malignantQ. • Peculiar feature of Warthin’s tumor: ‘hot’ spot in 99mTc-pertechnate scanQ. (Other tumors of the parotid show ‘cold’ spot) Histopathology • Papillary cystic pattern lined with columnar oncocytes and cuboidal cells with marked lymphoid componentQ • Lined by a double layer of neoplastic epithelial cellsQ resting on a dense lymphoid stroma sometimes bearing germinal centers. • The double layer of lining cells distinctiveQ, with a surface palisade of columnar cells resting on a layer of cuboidal to polygonal cells Diagnosis • FNAC is best diagnostic modality Treatment • Superficial parotidectomyQ

Head and Neck

Warthin’s Tumor (Papillary Cystadenoma Lymphomatosum)

924

Surgery Essence MUCOEPIDERMOID CARCINOMA Mucoepidermoid Carcinoma • MC malignant tumor of parotid, MC radiation induced neoplasmQ of parotid • MC malignant salivary gland tumor in childrenQ

Head and Neck

• Consist of admixture of squamous cells, mucous secreting cells, intermediate cells and clear or hydropic cellsQ • Include two major elements- mucin producing cells and epithelial cellsQ of epidermoid variety • Greater the epidermoid content, more malignant is the behavior • Usually not causes facial paralysisQ • Of two types: Low grade and high grade Low-grade Type • Well circumscribed mass having cystic mucinousQ material • Mucin producingQ cells predominate • Well differentiated • More common in children • TOC: Superficial or total parotidectomyQ

High-grade Type • Grossly infiltrative and has less tendency to cyst formation (hard tumor) • Squamous cells predominate • Poorly differentiatedQ • Less common • TOC: Total parotidectomy +/- radical neck dissection

ACINIC CELL CARCINOMMA Acinic Cell Carcinoma • Rare tumors composed of cells resembling the normal serous acinar cellsQ of salivary glands • Occur almost exclusively in parotid glandsQ • Low grade malignancy, mostly affecting women • Present as round or ovoid solitary encapsulated tumor • Histologically tumor is characterized by highly cellular structure with relative absence of stroma • Tumor tends to involve the regional lymph nodes • Treatment: Radical excisionQ

ADENOID CYSTIC CARCINOMA Adenoid Cystic Carcinoma • Second MC malignant tumor after mucoepidermoid carcinomaQ • MC malignant tumor in submandibular, sublingual and minor salivary glandsQ

Section 8

• MC site of origin is in minor salivary glands located in oral cavity (hard palate) followed by sinonasal tractQ • MC type is Cribiform patternQ, and is characterized by "Swiss-Cheese" appearanceQ • It has neurotropic properties, MC involved nerves are facial nerveQ, mandibular (V3) and maxillary branches of trigeminal nerve. • Skip lesionsQ along nerves are common, leading to treatment failure, because of difficulty in treating full extent of invasion. • May grow along haversian system of bone without showing bone destruction • It is tracherous tumor as it appears benign even when it is malignant Clinical Features • Characterized by its tendency to invade perineural tissues and lymphatics, thus causes pain (which may be prominent and early symptom) and facial nerve paralysisQ • High incidence of distant metastasis but indolent growthQ • Incidence of distant metastasis is correlated with stage of disease (size of primary tumor and status of LNs) • MC site of metastasis is lung, lung metastasis are usually multiple and prolonged survival without treatment is not unusualQ

Salivary Glands

925

Diagnosis

Treatments • Radical excision (irrespective of benign appearance) with largest cuff of normal tissues around the boundaries of tumor (poorly incapsulated with infiltrating nature)Q. • Post-op radiotherapy should be given if margins are positiveQ.

SIALOLITHIASIS

Section 8

• Best diagnostic modality is FNACQ • MRI is radiological IOC as it detects early perineural spread and intracranial extensionQ

Sialolithiasis • 80% of all salivary gland stones occur in submandibular glandQ, 10% occur in parotid, 7% in sublingual and the remainder in minor salivary glands. • MC site is Wharton's ductQ > submandibular gland substance • Composition of stone: Calcium and magnesium phosphate or carbonateQ • Due to deposition of calcium salts, 80% stones are radio opaqueQ Submandibular Salivary Gland Calculi are More Common than Parotid Because • Wharton's duct has long, curved and upward course and is hooked by lingual nerve leading to inadequate drainageQ • Secretion is more viscidQ than parotid gland secretion Clinical Features • Pain and swellingQ of submandibular region, aggregated by food, classically by sucking a lemon • Stone impacted in the duct may produce the referred pain in the tongue due to irritation of lingual nerve, as it hooks around the submandibular ductQ Diagnosis • IOC for diagnosis of sialolithiasis: NCCTQ Treatments • Stone in the duct is removed by giving incision directly over the stoneQ in long axis • Excision of submandibular glandQ when stone is in the gland substance.

Complications of Parotidectomy • Facial nerve paresis or paralysis •  Sensory abnormalities associated with sacrifice of greater auricular nerveQ Q • Frey’s syndrome •  Salivary fistulaQ Q

FREY’S SYNDROME OR AURICULOTEMPORAL SYNDROME (GUSTATORY SWEATING) Frey’s Syndrome or Auriculotemporal Syndrome (Gustatory Sweating) • It results from damage of auriculotemporalQ nerve during dissection in parotidectomy • Aberrant cross innervations between secretomotor parasympathetic fibers of parotid gland and sympathetic fibers supplying the sweat glandQ Clinical Feature • Sweating and erythema over the region of parotid glands as a consequence of autonomic stimulation of salivation by smell or taste of food. Diagnosis • Minor’s starch iodine testQ Treatment • Antiperspirant (aluminium chloride) application • Botulinum toxin treatment is used for symptomatic Frey’s syndrome • Surgical interruption of secretary fibers by tympanic neurectomy, in non-responding cases

Head and Neck

COMPLICATIONS OF PAROTIDECTOMY

926

Surgery Essence PAROTID FISTULA Parotid Fistula • Internal fistula opens inside the mouth and doesn’t give rise to symptoms • External fistula: gland fistula or duct fistula Causes • Rupture of parotid abscessQ •  Inadvertent incision during drainage of parotid abscessQ • Penetrating injury •  After superficial parotidectomyQ

Head and Neck

Clinical Presentation • When the external fistula is connected to the gland, external opening is pinpoint, though discharge is present for several months, usually closes spontaneously. • When external fistula is connected with major duct, there is outpouring of parotid secretions onto cheek during meals with excoriation of surrounding skin Diagnosis • Sialograply or sialogram Treatment • Newman and Seabrock’s operationQ (in cases of fistula connected with main duct, this operation reconstructs the duct)

RANULA Ranula • A cystic swelling in the floor of mouth that resembles a frog bellyQ. • Term ranula should only be applied to a mucous extravasation cyst that arises from the sublingual glandQ Etiology • Commonly the lesion is induced by local trauma and duct rupture, followed by mucin spillage into the surrounding soft tissues (mucous extravasation phenomenon)Q • Uncommonly, it is due to obstruction, probably caused by mucous plug or a sialolith Histopathology • Mucin accumulation surrounded by granulation and fibrous tissue (mucous extravasation phenomenon)Q • A cyst cavity, filled with mucin and lining by the ductal epithelium (mucous retention cyst)Q • Chronic inflammation of the cyst wall is present. Infiltration by numerous neutrophils, histiocytes, and plasma cells are commonQ. Clinical Features Exclusively present on the floor of mouthQ Usually unilateral, lateral to midlineQ Smooth, dome shape, fluctuating and painless swellingQ The color is usually bluish, but deep lesions may have a normal color. The size varies from a few to several centimeters in diameter. Very large lesions which may occupy the floor of mouth can also occur. Diagnosis • • • • •

• Diagnosis is usually made clinicallyQ.

Section 8

Treatment • Surgical removal or marsupializationQ. • MC structure injured during ranula surgery: Submandibular ductQ

PLUNGING RANULA Plunging ranula • • • •

Intraoral ranula with cervical prolongationQ. Plunging ranula extends from the floor of the mouth below the mylohyoid into the neck. This is nearly always an extravasation pseudocyst. Presents as soft painless, ballot table mass with cross fluctationQ.

Salivary Glands

927

SJÖGREN SYNDROME

• Chronic disease characterized by dry eyes (keratoconjunctivitis sicca) and dry mouth (xerostomia) resulting from immunologically mediated destruction of the lacrimal and salivary glandsQ. • It occurs as an isolated disorder (primary form), also known as the sicca syndrome or more often in association with another autoimmune disease (secondary form)Q. • Rheumatoid arthritis is the MC associated disorderQ Pathology

Section 8

Sjögren Syndrome

• Characterized by lymphocytic infiltration and fibrosis of the lacrimal and salivary glandsQ. • Earliest histologic finding in both the major and minor salivary glands is periductal and perivascular lymphocytic infiltrationQ. • Most important antibodies: Directed against SS-A (Ro) and SS-B (La)Q, which can be detected in 90% of patients (serologic markersQ of the disease) • Patients with high titers of antibodies to SS-A are more likely to have early disease onset, longer disease duration, and extraglandular manifestationsQ. Clinical Features • • • •

Occurs most commonly in womenQ between the ages of 50 and 60. Characteristic symptoms: Keratoconjunctivitis and xerostomiaQ Parotid gland enlargement is present in half of the patientsQ Extraglandular disease are seen in one third of patients

Diagnosis • Biopsy of the lip (to examine minor salivary glands) is essential for the diagnosis of Sjögren syndromeQ.

Head and Neck

Multiple Choice Questions SALIVARY GLAND TUMORS











1. Which among the following is most common neoplasm of salivary gland? (Recent Question 2014,  WBPG 2012, AIIMS June 98, All India 2002) a. Pleomorphic adenoma b. Adenoid cystic carcinoma c. Mucoepidermoid carcinoma d. Mixed tumor 2. Most common tumor of parotid gland is:  (DNB 2008, 2000, AIIMS June 93) a. Squamous cell carcinoma b. Pleomorphic adenoma c. Adenolymphoma d. None of the above 3. Best diagnostic modality for parotid swelling is:  (AIIMS Nov 94) a. Enucleation b. FNAC c. Superficial parotidectomy d. Excisional biopsy 4. Swelling of deep lobe of parotid gland presents as swelling in: (DNB 2001) a. Parapharyngeal space b. Cheek c. Temporal region d. Below the ear 5. What is the name of this incision given for parotidectomy? a. Battle incision b. Modified Blairs incision c. Maylard incision d. Cherney incision

6. True about parotid tumor: (PGI Nov 2010) a. Facial nerve involvement indicates malignancy b. Pleomorphic adenoma is MC variety c. Malignant disease is MC variety d. Superficial parotidectomy is the treatment of choice



7. About 50% the tumors are benign and even malignant tumors of this salivary gland are slow growing: a. Parotid gland b. Sublingual gland c. Submandibular gland d. All of the above



8. Most common site of minor salivary gland tumor: a. Cheek b. Palate (MHSSMCET 2005) c. Sub-lingual gland d. Tongue



9. Open biopsy is done for salivary gland tumor unless they are arising from: (Recent Question 2015) a. Palate b. Buccal c. Sublingual d. Parotid



10. Surgical treatment of parotid tumor involving the deep lobe is: (Recent Question 2016) a. Total parotidectomy with facial nerve preservation b. Total parotidectomy with facial nerve sacrifice c. Subtotal parotidectomy d. Subtotal parotidectomy with facial nerve sacrifice

11. Treatment of pleomorphic adenoma without facial nerve infiltration and limited to superficial lobe: a. Superficial parotidectomy  (Recent Question 2016) b. Total parotidectomy c. Parotidectomy followed by radiotherapy d. Observation

PLEOMORPHIC ADENOMA

12. Regarding pleomorphic adenoma of salivary gland true statement(s) is/are:  (PGI Dec 2008) a. Parotid gland is most commonly involved b. Malignant transformation does not occur c. Also called mixed tumor d. More commonly found in men than women e. Superficial parotidectomy is treatment of choice

13. A patient presented with gradually progressive painless mass since 10 years. It is firm to nodular & variable in consistency at each site. Most probable diagnosis is:  (Recent Question 2019)

a. Dermoid cyst c. Pleomorphic adenoma

b. Sebaceous cyst d. Malignancy

14. True regarding benign mixed parotid tumour is: (DNB 2005) a. Slow growing and lobular b. Firm and capsulated c. 50% of parotid tumour d. All of the above 15. Most common tumor of parotid gland:  (MCI June 2018, MHPGMCET 2007) a. Warthin’s tumor b. Pleomorphic adenoma c. Adenocarcinoma d. Hemangioma

Salivary Glands



17. Ramavati, a 40-year-old female presented with a progressively increasing lump in the parotid region. On oral examination, the tonsil was pushed medially, Biopsy showed it to be pleomorphic adenoma. The appropriate treatment is: a. Superficial parotidectomy (AIIMS June 2001) b. Lumpectomy c. Conservative total parotidectomy d. Enucleation



18. All are true for pleomorphic adenoma except: a. Arises from parotid b. May turn into malignant c. Minor salivary glands involved d. None



19. Mixed tumors of the salivary glands are: a. Most common in submandibular gland b. Usually malignant c. Most common in parotid gland d. Associated with calculi







MUCOEPIDERMOID CARCINOMA

29. Most common malignant tumour of parotid is: (Recent Question 2016, DNB 2011, 2010, DPG 2008) a. Epidermoid carcinoma b. Mucoepidermoid carcinoma c. Squamous cell carcinoma d. Adenocarcinoma



30. Mucoepidermoid carcinoma of parotid arises from:  (PGI June 99) a. Secretory cells b. Excretory cells c. Myoepithelial cells d. Myofibril



31. Mucoepidermoid carcinoma is seen in: (MHSSMCET 2005) a. Sebaceous gland b. Pancreas c. Parotid gland d. All

(PGI Dec 99)

(All India 2006)

20. Which of the following is an indication of radiotherapy in pleomorphic adenoma of parotid? (All India 2004) a. Involvement of deep lobe b. 2nd histologically benign recurrence c. Microscopically positive margins d. Malignant transformation



21. Mixed parotid tumour arises from: a. Epithelium b. Epithelium + Mesenchymal c. Mesenchymal d. None of the above

32. The most common tumour of the minor salivary gland is:  (DNB 2013,WBPG 2012, COMEDK 2008) a. Mucoepidermoid carcinoma b. Acinic cell carcinoma c. Adenoid cystic carcinoma d. Pleomorphic adenocarcinoma



33. Tumor with perineural invasion:  (Recent Question 2015, DNB 2009, AIIMS Nov 2010,MHSSMCET 2007) a. Adenocarcinoma b. Adenoid cystic carcinoma c. Basal cell carcinoma d. Squamous cell carcinoma



23. True statement regarding Warthin’s tumor: a. Common in females (Recent Question 2016, JIPMER 2010) b. Most malignant c. Hot spots on Tc-99 scan d. Most common tumor of minor salivary gland

34. All the following tumors can spread perineurally except:  (MHSSMCET 2007) a. Adenoid cystic carcinoma of salivary gland b. Carcinoma gallbladder c. Hilar cholangiocarcinoma d. None of the above



24. Exclusively found in parotid gland: (PGI May 2011) a. Warthin’s tumor b. Acinic cell c. Pleomorphic d. Adenocarcinoma e. Mucoepidermoid

35. Which among the following parotid tumor spreads through neural sheath? (Karnataka 2013, NEET Pattern, DNB 2013, AIIMS June 97, 96) a. Mixed parotid tumor b. Adenoid cystic carcinoma c. Squamous Cell carcinoma d. Oxyphilic lymphoma



36. Swiss cheese pattern is seen in: a. Warthin’s tumor b. Adenoid cystic carcinoma c. Pleomorphic adenoma d. Mucoepidermoid carcinoma



37. Adenoid cystic carcinoma of parotid gland all are true except:  (UPPG 2010) a. Most common malignant tumour of minor salivary gland b. Most common submandibular gland tumor c. Spreads perineurally d. Local recurrence is common e. Radiotherapy is the treatment of choice

22. All are true about pleomorphic adenoma except: (DNB 2014) a. Malignant transformation risk is high b. Mixed tumor c. Benign tumour d. Encapsulated

ADENOID CYSTIC CARCINOMA

WARTHIN’S TUMOR







27. Hot spot on Tc-99 is seen in which parotid tumour? (Recent Question 2016, JIPMER 2014, 2010; AIIMS May 2013) a. Adenolymphoma b. Adenoid cystic carcinoma c. Acinic cell tumour d. Adenocarcinoma 28. Cystic spaces lined by double layer of neoplastic epithelial cells resting on dense lymphoid tissue is a feature of:  (Recent Question 2016, APPG 2015) a. Dermoid cyst b. Warthin tumor c. Aneurysmal bone cyst d. Hashimoto’s thyroiditis

25. Warthin’s tumour is:(DNB 2012, AIIMS May 2005, June 2003) a. An adenolymphoma of parotid gland b. A pleomorphic adenoma of parotid c. A carcinoma of the parotid d. A carcinoma of submandibular salivary gland 26. Treatment of choice for Warthin’s tumour: (AIIMS Nov 2001, All India 98, 96) a. Superficial parotidectomy b. Enucleation c. Radiotherapy d. Injection of a sclerosant agent



(DPG 2007)

Head and Neck





Section 8

16. Treatment of choice for pleomorphic adenoma: (Recent Question 2015, 2014, DNB 2008, DPG 2008, MCI Sept 2010, 2007, AIIMS Nov 2001, Nov 95, All India 97) a. Superficial parotidectomy b. Radical parotidectomy c. Enucleation d. Radiotherapy

929

930

Surgery Essence

Head and Neck

ACINIC CELL CARCINOMA

38. Acinic cell tumor is tumor of: (MHPGMCET 2002) a. Parotid gland b. Breast c. Parathyroid d. Thyroid



39. Acinic cell carcinomas of the salivary gland arise most often in the: (All India 2006) a. Parotid gland b. Minor salivary glands c. Submandibular gland d. Sublingual gland

CARCINOMA PAROTID

40. Which of the following is false about salivary gland tumors?  (DPG 2008, 2007) a. Pleomorphic adenoma is MC tumor of parotid b. Adenoid cystic carcinoma MC occurs in minor salivary glands c. Warthin’s tumor is MC malignant tumor of salivary glands d. Perineural invasion is seen is adenoid cystic carcinoma



41. True statement(s) about salivary gland tumors:  (PGI June 2004) a. Pleomorphic adenoma can arise in submandibular gland b. Warthin’s tumour arises from submandibular gland c. Pleomorphic adenoma is most common tumour of submandibular gland d. Acinic cell carcinoma is most malignant e. Frey’s syndrome is due to injury of auriculotemporal nerve



42. All of the following statements about lymphoepithelioma of the parotid gland are true, except:  (All India 2009) a. Parotid gland is the most common site of lymphoepithelioma in the head and neck region b. It is associated with EBV infection c. It is highly radiosensitive d. It is a type of squamous cell carcinoma



43. Most of the parotid tumor are managed by: (All India 97) a. Total parotidectomy b. Radical parotidectomy c. Superficial parotidectomy d. Radical parotidectomy and neck dissection



44. All of the following are true regarding malignant salivary gland tumours except: (DNB 2010) a. Painful b. Present with skin ulceration c. Cervical lymphadenopathy d. Simple enucleation is treatment of choice

Section 8

SALIVARY GLAND STONES

45. Commonest salivary gland to get stones: (APPG 2015, Recent Question 2014, NEET 2013, DNB 2011, 2003, DPG 2006, MCI March 2005, 2007, AIIMS Nov 99, June 99) a. Parotid b. Submandibular c. Minor salivary gland d. Sublingual



46. In which one of the following conditions the sialography is contraindicated? (All India 2005) a. Ductal calculus b. Chronic parotitis c. Acute parotitis d. Recurrent sialadenitis



47. Treatment of submandibular salivary gland duct calculi is:  (TN 90) a. Excision of submandibular gland b. Opening the duct at the frenulum c. Opening the duct and removal of calculus d. Excision of gland and duct

48. All of the following statement regarding stones in the submandibular gland are true except: (MCI March 2007) a. 80% of stones occur in the submandibular gland b. Majority of submandibular stones are radiolucent c. Stones are the most common cause of obstruction within the submandibular gland d. Patient presents with acute swelling in the region of the submandibular gland

49. Investigation using dye to find out stone in salivary gland:  (Recent Question 2013) a. Sialography b. Mammography c. MR angiography d. USG

50. What percent of submandibular salivary gland stones are radiopaque? (MHCET 2016) a. 10% b. 70% c. 80% d. 90%

51. What are the three cranial nerves that are at risk during the removal of submandibular salivary gland? (APPG 2016) a. Marginal mandibular branch of facial nerve, glossopharyngeal nerve and spinal accessory nerve b. Lingual nerve, marginal mandibular branch of facial nerve & spinal accessory nerve c. Hypoglossal nerve, facial nerve & glossopharyngeal nerve d. Marginal mandibular branch of facial nerve, lingual nerve & hypoglossal nerve

PAROTIDECTOMY AND COMPLICATIONS

52. After removal of parotid gland, patient is having sweating on the cheeks while eating. In this complication seen after parotidectomy, the auriculotemporal nerve which contains parasympathetic secretomotor fibers to parotid gland is fused with which nerve? (AIIMS May 2012) a. Greater petrosal nerve b. Facial nerve c. Greater auricular nerve d. Buccal nerve

53. Frey’s syndrome occurs due to aberrant misdirection of fibers from salivary glands to sweat glands. These fibers come from which of the following?  (Recent Question 2019) a. Facial nerve b. Trigeminal nerve c. Auriculotemporal nerve d. Glossopharyngeal nerve 54. All of the following statements are true about Frey’s syndrome except:  (Recent Question 2019) a. Gustatory sweating b. Aberrant misdirection of sympathetic fibers of auriculotemporal nerve c. Botulinum toxin is one of the treatments suggested d. Less chances with enucleation than parotidectomy

55. Management of Frey’s syndrome include following except:  (MHSSMCET 2010, 2008) a. Botulinum toxin b. Temporal fascial graft c. Aluminum chloride d. Antiperspirants

Salivary Glands 

57. Which of the following is not true regarding radical parotidectomy? (MHSSMCET 2009) a. In radical parotidectomy the facial nerve is preserved b. Anesthesia of the ear lobe due to sectioning of the great auricular nerve can occur c. Gustatory sweating (Frey’s syndrome) can occur d. None

58. Which of the following group constitute Frey’s syndrome? a. Hyperhidrosis, enophthalmos and miosis (Karnataka 94) b. Anhidrosis, enophthalmos and miosis c. Redness and sweating over the auriculotemporal region during meal d. Pain over the distribution of the auriculotemporal nerve during meal



59. The nerve sacrificed in parotid surgery: (DNB 2013, APPG 98) a. Auriculotemporal b. Facial c. Buccal d. Cervico facial



60. Incision for superficial parotidectomy: a. L-shaped b. Y-shaped c. S-shaped d. Z-shaped

(WBPG 2015)



65. What is ranula? (DNB 2007, 2005) a. Retention cyst of sublingual gland b. Retention cyst of submandibular gland c. Extravasation cyst of sublingual glands d. Extravasation cyst of submandibular glands



66. Which of the following is not true about ranula?  (MHSSMCET 2010) a. Arises from sublingual salivary glands b. Pseudocyst c. Appearance is like “Frog’s belly” d. None



67. Excision of ranula is associated with injury to: (DNB 2010, MHSSMCET 2007, PGI 96) a. Lingual nerve b. Lingual artery c. Parotid gland d. Submandibular duct

68. Which of the following statements about ‘ranula’ is most correct? (All India 2011) a. It is type of epulis b. It is a cystic swelling in the floor of mouth c. It is a type of thyroglossal cyst d. It is a type of mucus retention cyst

PAROTID FISTULA



61. Newman and Seabrook’s operation is used for: a. Repair of parotid fistula (Recent Question 2016) b. For parotid calculi c. For carcinoma of tongue d. For treatment of recurrent chronic parotitis



63. Which of the following best represents ‘ranula’? a. A type of epulis (AIIMS May 2005) b. A thyroglossal cyst c. Cystic swelling in the floor of mouth d. Forked uvula



64. What is the most probable diagnosis based on the given image?

a. Pleomorphic adenoma b. Ranula c. Warthin’s tumor d. Adenoid cystic carcinoma

70. True about salivary gland: (PGI Dec 2006) a. There are more than 400 minor glands b. 90% of all parotid tumors are malignant c. 90% of minor salivary tumors are malignant d. Superficial parotidectomy done in parotid tumors e. Warthin’s tumor common in parotid

71. Which of the following is not a landmark for facial nerve during parotid surgery?  (All India 2008) a. Digastric muscle b. Inferior belly of omohyoid c. Tragal pointer d. Retrograde dissection of distal branch

72. The parotid duct is known as: a. Wharton’s duct b. Stenson’s duct c. Duct of Santorini d. Duct of Wirsung



73. Nerve which lies in association to Wharton’s duct is: a. Hypoglossal b. Lingual c. Facial d. Spinal accessory



74. Inflammatory enlargement of deep lobe of parotid gland is seen in: a. Posterior pharyngeal wall b. Supratonsillar area c. Anterior tonsillar pillar d. Tonsillar fossa/bed



75. Which of the following nerves lies closest of the Whartson’s duct? (COMEDK 2007) a. Hypoglossal b. Lingual c. Chorda tympani d. Facial



76. Structure exposed after parotid surgery is: a. Internal jugular vein b. Facial nerve c. Muscles d. Facial artery

(DPG 2008)

Head and Neck



69. Plunging ranula is: (Recent Question 2015) a. Cystic growth of sublingual gland b. Lymph node c. A tumor in floor of mouth d. None

SALIVARY GLANDS ANATOMY AND PHYSIOLOGY

62. Seabrook’s operation is done for: (MHSSMCET 2006) a. Parotid duct fistula b. Thyroglossal fistula c. Thyroglossal cyst d. Branchial fistula

RANULA

Section 8

56. The ‘Starch iodine test’ is useful to diagnose: (MHSSMCET 2011) a. Wegener’s granulomatosis b. Cat scratch disease c. Sarcoidosis d. Frey’s syndrome

931

932

Surgery Essence

Head and Neck



77. All of the following are anatomical markers for localization of facial nerve during parotid surgery, except: (DPG 2008) a. Posterior belly of digastric b. Mastoid process c. Inferior belly of omohyoid d. Bony external auditory meatus 78. In submandibular gland surgery, the nerve least likely to be injured is: (DPG 2011, JIPMER 93) a. Inferior alveolar nerve b. Hypoglossal nerve c. Lingual nerve d. Mandibular branch of facial nerve

79. Most common location of ectopic submandibular salivary gland tissue is: (MCI Sept 2009, UPPG 2002) a. Cheek b. Palate c. Angle of mandible d. Tongue 80. In surgery of submandibular salivary gland, nerve often involved: (PGI June 97) a. Hypoglossal b. Glossopharyngeal c. Facial d. Lingual

SJOGREN’S SYNDROME



81. True regarding Sjogren’s syndrome are all of the following except: (MCI Sept 2009) a. Autoimmune condition b. Males are commonly affected c. Progressive destruction of lacrimal and salivary gland d. No single laboratory investigation is pathognomonic 82. Sjogren’s syndrome refers to disease of: a. Parotid glands b. Thyroid disease c. Parathyroid glands d. Multiple endocrine neoplasia

Section 8

83. Biopsy of the parotid gland in a patient with Sjogren’s disease shows:  (JIPMER 2011) a. Neutrophils b. Lymphocytes c. Eosinophils d. Basophils

MISCELLANEOUS 84. Bacterial pyogenic parotitis affecting the parotid gland is most common after: (MCI March 2008) a. Uveo-parotid fever b. Mumps c. Debilitation after major surgery d. After administration of iodine 85. Bilateral parotid enlargement does not occur in:  (AIIMS Nov 96) a. Sjogren’s syndrome b. Sarcoidosis c. SLE d. Chronic pancreatitis 86. A bacterial pyogenic parotitis is found most commonly in which of the following? (Orissa 99) a. Mumps b. Debilitation after major surgery c. Drug reaction (iodine mumps) d. Uveoparotid fever 87. Sialosis refers to: (Karnataka 2006) a. Bilateral parotitis b. Sjogren’s syndrome c. Non-inflammatory parotid enlargement d. Bilateral salivary duct ectasia 88. Ackerman’s tumour is: (DPG 2008) a. Mucoepidermoid carcinoma b. Epidermoid carcinoma c. Squamous cell carcinoma d. Adenocarcinoma (DNB 2008)



89. Bilateral parotid enlargement is seen in: a. Wegner’s granulomatosis b. Sjogren’s syndrome c. Kimura’s disease d. All of the above



90. True regarding acute sialadenitis is/are: (PGI May 2018) a. Most common in submandibular glands b. Most common type is viral c. Can present with stasis of saliva d. There may be tender pre-auricular nodes e. Stone removal may be done by probing through oral route

Explanations SALIVARY GLAND TUMORS

1. Ans. a. Pleomorphic adenoma (Ref: Bailey 27/e p787; Devita 9/e p774; Cancer of the Head and Neck by Suen and Myer 4/e p480-490) • • • • •

MC neoplasm of salivary gland: Pleomorphic adenomaQ MC malignant tumor of salivary gland: Mucoepidermoid carcinomaQ MC neoplasm of salivary gland in children: HemangiomaQ MC malignant tumor of salivary gland in children: Mucoepidermoid carcinomaQ MC malignant tumor of minor salivary glands: Adenoid cystic carcinomaQ

All salivary gland tumors are most common in parotid except • Adenoid cystic carcinoma: MC malignant tumor of minor salivary glandsQ • Squamous cell carcinoma: Mostly seen in submandibular glandQ

2. Ans. b. Pleomorphic adenoma



5. Ans. b. Modified Blairs incision

3. Ans. b. FNAC

4.  Ans. a. Parapharyngeal space



6. Ans. a. Facial nerve involvement indicates malignancy, b. Pleomorphic adenoma is MC variety, d. Superficial parotidectomy is the treatment of choice

In parotid tumors, rapid growth, pain, paraesthesia, enlarged cervical LN and restriction of jaw movements, facial weakness or skin invasion and fixation of mastoid tip is suggestive of malignant transformation.

7. Ans. b. Sublingual gland, c. Submandibular gland (Ref: Bailey 27/e p783; Devita 9/e p774-778; Cancer of the Head and Neck by Suen and Myer 4/e p480-490)



8. Ans. b. Palate (Ref: Bailey 27/e p776; Devita 9/e p777-778; Cancer of the Head and Neck by Suen and Myer 4/e p487-489) • MC malignant tumor of minor salivary glands: Adenoid cystic carcinomaQ • MC site of origin is in minor salivary glands located in oral cavity (hard palate) followed by sinonasal tractQ



9. Ans. d. Parotid



11. Ans. a. Superficial parotidectomy

10. Ans. a. Total parotidectomy with facial nerve preservation

PLEOMORPHIC ADENOMA

12. Ans. a. Parotid gland is most commonly involved, c. Also called mixed tumour, e. Superficial parotidectomy is treatment of choice (Ref: Bailey 27/e p787; Devita 9/e p774; Cancer of the Head and Neck by Suen and Myer 4/e p414)



13. Ans. c. Pleomorphic adenoma (Ref: Bailey 27/e p787)



14. Ans. d. All of the above

15. Ans. b. Pleomorphic adenoma



16. Ans. a. Superficial parotidectomy

17. Ans. c. Conservative total parotidectomy



18. Ans. d. None (Ref: Robbins 9/e p787, 8/e p758) • Pleomorphic adenoma arises most commonly from the parotid glandQ. • Pleomorphic adenoma is less common in the submandibular gland and sublingual glands; relatively rare in the minor glandsQ.



19. Ans. c. Most common in parotid gland



20. Ans. c. Microscopically positive margins (Ref: Devita 9/e p776; Cancer of the Head and Neck by Suen and Myer 4/e p499-501)



21. Ans. b. Epithelium + Mesenchymal



22. Ans. a. Malignant transformation risk is high

WARTHIN’S TUMOR

23. Ans. c. Hot spots on Tc-99 scan (Ref: Bailey 27/e p789; Devita 9/e p774; Cancer of the Head and Neck by Suen and Myer 4/e p414)



24. Ans. a. Warthin’s tumor, b. Acinic cell

25. Ans. a. An adenolymphoma of parotid gland

934

Surgery Essence

26. Ans. a. Superficial parotidectomy



27. Ans. a. Adenolymphoma



28. Ans. b. Warthin tumor

MUCOEPIDERMOID CARCINOMA

29. Ans. b. Mucoepidermoid carcinoma (Ref: Bailey 27/e p788; Devita 9/e p774-777; Cancer of the Head and Neck by Suen and Myer 4/e p489)



30. Ans. a. Secretory cells



31. Ans. c. Parotid gland

Head and Neck

ADENOID CYSTIC CARCINOMA

32. Ans. c. Adenoid cystic carcinoma (Ref: Bailey 27/e p727,778; Devita 9/e p777-778; Cancer of the Head and Neck by Suen and Myer 4/e p487-489)



33. Ans. b. Adenoid cystic carcinoma



34. Ans. d. None of the above (Ref: Bailey 25/e p752; Devita 9/e p778, 1021, 1037; Cancer of the Head and Neck by Suen and Myer 4/e p488) Perineural Spread is seen in • Adenoid cystic carcinomaQ • CA GBQ

• Ductal adenocarcinoma of pancreasQ • CholangiocarcinomaQ



35. Ans. b. Adenoid cystic carcinoma



37. Ans. b. Most common submandibular gland tumour

36. Ans. b. Adenoid cystic carcinoma

ACINIC CELL CARCINOMA

38. Ans. a. Parotid gland



39. Ans. a. Parotid gland (Ref: Devita 9/e p774; Cancer of the Head and Neck by Suen and Myer 4/e p489-490)

CARCINOMA PAROTID

40. Ans. c. Warthin’s tumor is MC malignant tumor of salivary glands



41. Ans. a. Pleomorphic adenoma can arise in submandibular gland, c. Pleomorphic adenoma is most common tumour of sub mandibular gland, e. Frey’s syndrome is due to injury of auriculotemporal nerve



42. Ans. a. Parotid gland is the most common site of Lymphoepethelioma in the Head and Neck region (Ref: Devita 9/e p729, 752, 774) • The most common site of lymphoepethelioma is the nasopharynxQ. Lymphoepethelioma occurs rarely in the parotid and submandibular glands.

Lymphoepethelioma • • • •

Lymphoepethelioma: Undifferentiated carcinoma of the nasopharyngeal type Lymphoepethelioma is a variant of squamous cell carcinomaQ that arises in lymphoid bearing areas Found most commonly in the nasopharynxQ Rarely occur in parotid and submandibular glandsQ Common sites of Lymphoepethelioma in Head and Neck

Section 8

• Nasopharynx (MC site)Q   •  Faucial tonsils    •  Lingual tonsils (base of tongue) • Histologically the squamous component is highly undifferentiated while the lymphoid component is essentially benign (nonneoplastic lymphocytes) • EBV is commonly linked when this tumor is located in the nasopharynxQ • High tendency to metastasize and is exquisitely radiosensitiveQ Important characteristic features • High tendency to metastasizeQ       •  Extreme radiosensitivityQ

43. Ans. c. Superficial parotidectomy (Ref: Bailey 27/e p789; Devita 9/e p775-777; Cancer of the Head and Neck by Suen and Myer 4/e p494-501)

Treatment of Choice for Pleomorphic Adenoma • In parotid gland: superficial parotidectomyQ

•  In other salivary glands: excision of the affected glandQ

Salivary Glands

935

Treatment of Salivary Gland Tumors

Parotid malignancies

• Superficial parotidectomyQ with preservation of facial nerve • Total parotidectomyQ with nerve preservation if deep lobe is involved

• Other salivary glands

• En-bloc excisionQ of tumor (involved gland)

• Neck dissection for lymph nodes: In clinically palpable nodes and High grade malignancies • Radiotherapy

Section 8

• Surgery −− Principal treatment of cancer of salivary glands is surgical excision, used either as a single modality or in most cases, in conjunction with adjuvant radiation therapy.

Indications of radiotherapy in Salivary Gland Tumors • High grade tumorsQ • Large primary lesionsQ • Perineural invasionQ

• Bone invasionQ • Cervical LN metastasisQ • Positive surgical marginsQ

• No role of chemotherapy in salivary gland tumors due to incomplete and short lived response without any survival advantage.

44. Ans. d. Simple enucleation is treatment of choice

SALIVARY GLAND STONES 45. Ans. b. Submandibular (Ref: Bailey 27/e p780) 46. Ans. c. Acute parotitis (Ref: Bailey 27/e p786)

Sialograply (Ptyalography) • Contrast X-ray examination of salivary glands and duct Indications

Contraindications

• Salivary duct stones and stricturesQ • Chronic sialedenitisQ • Tumors of salivary glands

• Contrast allergyQ • Acute sialedenitisQ

47. Ans. c. Opening the duct and removal of calculus 48. Ans. b. Majority of submandibular stones are radiolucent 49. Ans. a. Sialography (Ref: Sutton’s radiology 7/e p535, Bailey & Love 25/e p760)



50. Ans. c. 80%

51. Ans. d. Marginal mandibular branch of facial nerve, lingual nerve & hypoglossal nerve

PAROTIDECTOMY AND COMPLICATIONS

52. Ans. c. Greater auricular nerve (Ref: Grays 40/e p436; Bailey 27/e p792) • “Cross innervation between somatic sensory supply (greater auricular) and parasympathetic secretomotor fibers to the parotid is considered to be part of the anatomical basis for the phenomenon of gustatory sweating (Frey’s syndrome) seen after parotid surgery, when the nerve is at risk of injury.”

Complications of Parotidectomy • Facial nerve paresis or paralysis •  Sensory abnormalities associated with sacrifice of greater auricular nerveQ • Frey’s syndromeQ •  Salivary fistulaQ Q



53. Ans. c. Auriculotemporal nerve (Ref: Bailey 27/e p792)



54. Ans. b. Aberrant misdirection of sympathetic fibers of auriculotemporal nerve (Ref: Bailey 27/e p792)



55. Ans. b. Temporal fascial graft (Ref: Bailey 27/e p795; Cancer of the Head and Neck by Suen and Myer 4/e p498)



56. Ans. d. Frey’s syndrome



57. Ans. a. In radical parotidectomy the facial nerve is preserved (Ref: Bailey 27/e p792; Cancer of the Head and Neck by Suen and Myer 4/e p495) • Radical parotidectomy: Facial nerve is sacrificed, particularly if there is a reasonable prospect of cure.

Radical Parotidectomy • Radical parotidectomy envolves resection of all parotid gland tissue and elective sectioning of facial nerve usually through the main trunk.

Head and Neck



936

Surgery Essence

58. Ans. c. Redness and sweating over the auriculotemporal during meal

59. Ans. b. Facial

60. Ans. c. S-shaped

PAROTID FISTULA

61. Ans. a. Repair of parotid fistula

62. Ans. a. Parotid duct fistula (Ref: Cancer of the Head and Neck by Suen and Myer 4/e p498)

Head and Neck

RANULA

63. Ans. c. Cystic swelling in the floor of mouth (Ref: Bailey 27/e p779)



64. Ans. b. Ranula (Ref: Bailey 27/e p779)



65. Ans. c. Extravasation cyst of sublingual gland



67. Ans. d. Submandibular duct (Ref: Clinical Surgery by Rob’s and Smith vol-9/56)

66. Ans. d. None

The treatment of ranula constitutes a problem, owing to technical difficulty of complete excision without damage to adjacent structures such as submandibular duct.

68. Ans. b. It is a cystic swelling in the floor of mouth

69. Ans. a. Cystic growth of sublingual gland

SALIVARY GLANDS ANATOMY AND PHYSIOLOGY

70. Ans. a. There are more than 400 minor glands, c. 90% of minor salivary glands are malignant, d. Superficial parotidectomy done in parotid tumors, e. Warthin’s tumor common in parotid (Ref: Bailey 27/e p776)

Minor salivary glands • The mucosa of the oral cavity contains approximately 750 minor salivary glandsQ. • They are distributed in the mucosa of the lips, cheeks, palate, floor of the mouth and retromolar areaQ. • These minor salivary glands also appear in other areas of the upper aerodigestive tractQ including the oropharynx, larynx and trachea as well as the sinuses. • Overall, they contribute to 10% of the total salivary volume • Their secretion is mainly mucousQ in nature and has many functions such as coating the oral cavity with saliva.

71. Ans. b. Inferior belly of omohyoid (Ref: Bailey 27/e p790)

Major Surgical Landmarks to the Facial Nerve • Tympanomastoid suture line (most constant landmark)Q • Tragal pointerQ •  Retromandibular veinQ • Posterior belly of digastricQ •  Styloid processQ

72. Ans. b. Stenson’s duct (Ref: Bailey 27/e p778,780,785; Cancer of the Head and Neck by Suen and Myer 4/e p495)

Salivary Glands Anatomical Features • Parotid gland: Serous acini • Submandibular gland: Mucus + Serous aciniQ

Q

•  Sublingual + minor salivary glands: Mucinous aciniQ

Section 8

• Parotid gland: Stenson’s ductQ (opens at upper 2nd molar toothQ) • Submandibular gland: Wharton’s ductQ (opens into the papilla just lateral to the frenulumQ) • Sublingual gland: Ducts of RivinusQ, Bartholin ductQ • Sublingual gland secretes via tiny openings, ducts of Rivinus directly into the floor of mouth or via several ducts which unite to form the common sublingual duct of Bartholin, which then merges with Wharton’s duct. 73. Ans. b. Lingual (Ref: Bailey 27/e p780) Lingual nerve lies in a close association with submandibular duct and most prone to damage during submandibular gland surgery.

Submandibular Gland • The deep part of the gland lies on the hyoglossus muscle closely related to the lingual nerve and inferior to the hypoglossal nerveQ. • Submandibular duct lies between the lingual and hypoglossal nerves on hyoglossus, but, at the anterior border of the muscle, it is crossed laterally by the lingual nerveQ, terminal branches of which ascend on its medial side.

Salivary Glands

937

Management of Sialolithiasis



74. Ans. d. Tonsillar fossa/bed (Ref: BDC 4/e pvol III/136, 217)



75. Ans. b. Lingual



76. Ans. b. Facial nerve (Ref: Bailey 27/e p789)

Section 8

• If the stone is lying within the submandibular duct in the floor of the mouth anterior to the point at which the duct crosses the lingual nerve (second molar region), the stone can be removed by incising longitudinally over the ductQ. • Once the stone has been delivered, the wall of the duct should be left open to promote free drainage of saliva. • Suturing the duct will lead to stricture formation and the recurrence of obstructive symptoms. • Where the stone is proximal to the lingual nerve, i.e. at the hilum of the gland, stone retrieval via an intraoral approach should be avoided as there is a high risk of damage to the lingual nerveQ during exploration in the posterior lingual gutter. Treatment is by simultaneous submandibular gland excision and removal of the stone and ligation of the submandibular duct under direct visionQ.

• Parotid gland is related to lateral pharyngeal wall. Tonsillar fossa is present in lateral pharyngeal wall.

Structure exposed after parotid surgery is facial nerve. • Superficial parotidectomy: The dissection is made in the Patey's facio-venous plane and commenced at the postero-inferior border of the parotid gland, where the main trunk of the facial nerve is found outQ. • Radical parotidectomy: Facial nerve is sacrificed, particularly if there is a reasonable prospect of cureQ.

77. Ans. c. Inferior belly of Omohyoid



78. Ans. a. Inferior alveolar nerve (Ref: Bailey 27/e p780) Important anatomical relationships of the submandibular glands • Lingual nerveQ • Facial arteryQ Q • Hypoglossal nerve • Marginal mandibular branch of the facial nerve • Anterior facial veinQ



79. Ans. c. Angle of mandible (Ref: Bailey 27/e p780)

Ectopic/Aberrant Salivary Gland Tissue



80. Ans. a. Hypoglossal, c. Facial, d. Lingual

SJOGREN'S SYNDROME

81. Ans. b. Males are commonly affected (Ref: Robbins 9/e p227; Bailey 27/e p796; Harrison 20/e p2561, 19/e p2166)



82. Ans. a. Parotid glands

83. Ans. b. Lymphocytes

MISCELLANEOUS

84. Ans. c. Debilitation after major surgery (Ref: Bailey 27/e p786)

Acute Suppurative Parotitis • Characterized by presence of pus and seen in debilitated/ dehydrated/ or in patients with poor oral hygieneQ. • MC organism (responsible for): Staph. aureusQ Treatment • Initial treatment is proper hydration/ antibiotics/ improving oral hygieneQ. • If abscess develops then it is drainedQ by giving a J shaped incision.

Head and Neck

• MC ectopic salivary tissue is the Stafne bone cyst. • This presents as an asymptomatic, clearly demarcated radiolucency of the angle of the mandible, characteristically below the inferior dental neurovascular bundleQ. • It is formed by invagination into the bone on the lingual aspect of the mandible of an ectopic lobe of the juxtaposed submandibular glandQ. • No treatment is requiredQ.

938

Surgery Essence

85. Ans. c. SLE (Ref: Harrison 20/e p2561, 224) Causes of Bilateral Parotid Enlargement

Head and Neck

• Viral infections: −− MumpsQ −− Influenza − Epstein-Barr virusQ • SarcoidosisQ • AmyloidosisQ • Kimura’s disease

− Coxsackie virus AQ − CytomegalovirusQ − HIVQ

• • • •

Wagner’s Granulomatosis Sjögren's syndromeQ Metabolic: − Diabetes mellitusQ − Chronic pancreatitisQ Endocrine: − AcromegalyQ



86. Ans. b. Debilitation after major surgery



87. Ans. c. Non-inflammatory parotid enlargement (Ref: Bailey 27/e p795)

− Hyperlipoproteinemias − Hepatic cirrhosisQ − Gonadal hypofunction

Sialosis • Sialosis is an uncommon non-neoplastic and non-inflammatory disorder causing bilateral non-painful enlargement of the major salivary glandsQ.

88. Ans. c. Squamous cell carcinoma (Ref: Oxford textbook of oncology, Vol. I/1995/1046)

Ackerman's tumour is a distinct variant of well-differentiated squamous cell carcinoma.

Ackerman's tumour • Ackerman's tumour is a distinct variant of well-differentiated SCC that may develop in any mucosal surface in the upper respiratory and digestive tractQ. • It is also called as verrucous carcinomaQ. • Most verrucous carcinoma are reported within the oral cavityQ but they make up 1-2% of malignant laryngeal tumours within the larynx. • Verrucous carcinomas occur mainly in the elderly patientsQ. • Verrucous carcinomas may be multifocal and warty in their appearanceQ. • The pushing infiltrating margins are characteristic of verrucous carcinomas. • Lymphatic spread is rare. Risk Factors for Verrucous Carcinoma • Frequent use of tobacco (MC)Q+              • HPV infectionQ Histology • Well-differentiated, keratinising epithelium, with few mitotic figures and no cellular atypia. Treatment

Section 8

• Surgery is the treatment of choice.



89. Ans. d. All of the above



90. Ans. b. Most common type is viral, c. Can present with stasis of saliva, e. Stone removal may be done by probing through oral route…. (Ref: Bailey 27/e p796)

CHAPTER

37

Neck

CAROTID BODY TUMOR Carotid Body Tumor (Chemodectoma) • Arises from chemoreceptor cellsQ on the medial side of carotid bulb • Histologically it is a non-chromaffin paragangliomaQ • Usually benign, unifocal and nonhereditary, Schamblin classification is used for carotid body tumour • Associated with pheochromocytomaQ • Higher incidence in areas where people live at high altitudes because of chronic hypoxia leading to carotid body hyperplasia. Clinical features • • • • •

Present most commonly in the 5th decadeQ Approximately 10% have family historyQ. Patient presents with a long history of several years of a slowly enlarging painless lump at the carotid bifurcation. Mass is firm, rubbery, pulsatile and is mobile from side to side but not up and down A bruitQ may also be present

Diagnosis • Doppler study • Carotid angiogram: Lyre signQ (splaying of internal & external carotid arteries) • FNAC & biopsy are contraindicatedQ because of their highly vascular nature Treatment • Because these tumors rarely metastasizeQ and their overall rate of growth is slow, the need for surgical removal must be considered carefully as complication of surgery are potentially serious. • Operation is best avoided in elderly patientsQ. • Preoperative embolization is performed for tumors >3 cm. • Tumors >5 cm are associated with a need for concurrent carotid artery replacement. Complications • Most frequent sequela from resection: Cranial nerve injury (MC-superior laryngeal nerveQ) • First-bite syndromeQ: Pain with the initiation of mastication • Excision of bilateral carotid body tumors may lead to baroreceptor failure, with wide fluctuations in BP.

CYSTIC HYGROMA Cystic Hygroma • • • • •

Cystic hygromas are multiloculated cystic spacesQ lined by endothelial cells It results due to sequestration of a portion of jugular lymph sac from the lymphatic systemQ. Cysts are filled with clear lymph and are lined by endotheliumQ. Turner’s syndrome is associated with cystic hygromaQ. Most cystic hygromas involve the lymphatic jugular sacs

940

Surgery Essence • • • •

MC site: Posterior neck regionQ (Posterior triangleQ) Other common sites: Axilla, mediastinum, inguinal & retroperitoneal regionsQ Approximately 50% of them present at birthQ It may show spontaneous regressionQ

Clinical Features

Head and Neck

• • • •

Usually present as soft cystic masses that distort the surrounding anatomy, can result in acute airway obstruction. Usually manifests in the neonates or in early infancyQ (50% present at birth). Prone to infection & hemorrhage within the mass. Swelling is soft & partially compressible and invariably increases in size when the child coughs or cries. • Characteristic features: Brilliantly translucentQ

Diagnosis • IOC for diagnosis: MRIQ • MRI play a crucial role in preoperative planningQ Treatment • Complete surgical excision is the preferred treatmentQ. • Injection of sclerosing agentsQ such as bleomycin or OK-432 (Picibanil), derived from Streptococcus pyogenes may eradicate the cystic hygroma.

BRANCHIAL CLEFT REMNANTS Branchial Cleft Remnants • Branchial cleft remnants typically present as a lateral neck massQ on a toddler. • Structures of the head & neck are derived from 6 pairs of branchial arches, their intervening clefts & pouches. • Congenital cysts, sinuses, or fistulas result from failure of these structures to regress, persisting in an aberrant location. • All branchial remnants are present at the time of birth; however, they are often not recognized until later in life. • These lesions may present as sinuses, fistulas, or cartilaginous rests in infantsQ. • They occur more commonly as cysts in older children and adolescentsQ. • Clinical presentation may range from a continuous mucoid drainage from a fistula or sinus to the development of a cystic massQ that may become infected. • Branchial remnants may also be palpable as cartilaginous lumps or cordsQ corresponding with a fistulous tract. Dermal pits or skin tags may also be evident.

Branchial Cleft Remnants

Section 8

First Branchial Cleft Remnants • Typically located in the front or back of the ear or in the upper neck near the mandibleQ. • Fistulas typically course through the parotid gland, deep or through branches of the facial nerve, and end in the external auditory canalQ.

Second Branchial Cleft Remnants

Third Branchial Cleft Remnants

• Most commonQ • External ostium is located along the anterior border of the SCM muscle, in the vicinity of the upper half to lower third of the muscleQ. • Stepladder counterincisions are often necessary to excise the fistula completelyQ. • Typically, the fistula penetrates platysma, ascends along the carotid sheath to the level of hyoid bone, and turns medially to extend between carotid artery bifurcationQ. • The fistula then courses behind the posterior belly of digastric and stylohyoid muscles to end in the tonsillar fossaQ.

• Third branchial cleft remnants usually do not have associated sinuses or fistulas • Located in the suprasternal notch or clavicular regionQ. • Most often contain cartilageQ • Present clinically as a firm mass or subcutaneous abscessQ.

Neck

941

BRANCHIAL ABNORMALITIES

Branchial Cyst • Develops from vestigial remnants of 2nd branchial cleftQ • Lined by squamous epitheliumQ • Contains thick, turbid fluid full of cholesterol crystalsQ. • Cyst usually presents in the upper neck in early or middle adulthoodQ • Found at the junction of upper third & middle third of the SCM muscle at its anterior borderQ. • Fluctuant swelling that may transilluminate and is often soft in its early stagesQ. • USG and FNAC aid diagnosis • Treatment is by complete excisionQ

• • • • • • • •

Branchial Fistula Branchial fistula may be unilateral or bilateralQ Thought to represent a persistent 2nd branchial cleftQ. Tract is lined by ciliated columnar epitheliumQ There may be a small amount of recurrent mucous or mucopurulent discharge onto the neckQ. External orifice is nearly always situated in the lower third of the neck near anterior border of SCMQ Internal orifice is located on the anterior aspect of posterior faucial pillar just behind the tonsilQ. Internal aspect of the tract may end blindly at or close to the lateral pharyngeal wall, constituting a sinus rather than a fistula. The tract follows the same path as a branchial cyst and requires complete excision, often by more than one transverse incision in the neckQ.

Section 8

Branchial Abnormalities

CERVICAL RIB

• Rib arising from 7th cervical vertebra • MC on right sideQ

Head and Neck

Cervical Rib Q

942

Surgery Essence Types: 1. Complete: Reaches up to 1st thoracic ribQ 2. Bulbous end: Has a bulbous end 3. Tapering end: Rib tapers 4. Fibrous band: Rib is represented by thick fibrous band Clinical Features

Head and Neck

• Cervical rib with local symptoms: Lump & tenderness in supraclavicular fossaQ • Cervical rib with vascular symptoms: Pain, pallor & pulselessnessQ • Cervical rib with nerve pressure symptoms: Pain & paraesthesia along medial aspect of forearm & handQ Diagnosis • Diagnosed by X-ray of neckQ Treatment • Mild cases: Sling exercise • In severe cases: Scalenotomy (resection of scalenus anterior muscles) • In troublesome cases: Removal of cervical ribQ

CERVICAL LYMPH NODES Cervical Lymph Nodes • Cervical lymphatic nodal basins contain 50-70 lymph nodes per side • Virchow or left supraclavicular nodes are included in level IVQ. • Divided into seven levels Level

Lymph Node

IA IB

• SubmentalQ • SubmandibularQ

II

• UpperQ jugular

III

• MiddleQ jugular

IV

• LowerQ jugular

V

• Posterior triangularQ

VI

• Anterior compartmental or centralQ

VII

• Superior mediastinalQ

Section 8

Cervical lymph node levels

Neck

943

NECK DISSECTION

Comprehensive Neck Dissection

Selective Neck Dissection

• Radical Neck Dissection: Removal of lymph nodes I-V + spinal accessory nerve + internal jugular vein + sternocleidomastoid muscleQ • Extended Radical Neck Dissection: Radical Neck Dissection + removal of one or more groups of lymph nodes, non-lymphatic structures or both • Bilateral Radical Neck Dissection • Modified Radical Neck Dissection: Removal of level I-V lymph nodes with(Mnemonic: SISm) −− Type I: preserves only spinal accessory nerveQ −− Type II: preserves both spinal accessory nerve and internal jugular veinQ −− Type III: preserves spinal accessory nerve, internal jugular vein and sternocleidomastoid muscleQ (Functional neck dissectionQ)

• Supraomohyoid Neck Dissection: Removal of level I-III LNsQ • Extended supraomohyoid Neck Dissection: Removal of level I-IV LNsQ • Posterolateral Neck Dissection: Removal of level II-V LNs + suboccipital LNs + retroauricular LNs • Lateral Neck Dissection: Removal of level II-V LNs + internal jugular vein • Central Compartment Neck Dissection: Removal of level VI LNs

Section 8

Neck Dissection

CARCINOMA LARYNX Carcinoma Larynx • • • • •

More common in malesQ, seen in 40-70 yearsQ TobaccoQ and alcoholQ are well established risk factor Cigarette smoke: Benzopyrene and hydrocarbons are carcinogenic in man. About 90-95% of laryngeal malignancies are squamous cell carcinomaQ with various grades of differentiation. Cordal lesions are often well-differentiatedQ while supraglottic ones are Anaplastic. Carcinoma Larynx Glottic Cancer

Subglottic Cancer

• Supraglottic cancer is less frequent than glottic cancerQ. • Nodal metastases occur earlyQ, upper and middle jugular nodes are often involved. • Bilateral metastases may be seen in cases of epiglottic cancer. • Pain on swallowing is the most frequent initial symptomQ. • Mass in the neck may be the first sign. • Hoarseness is a late symptom. • Pain may be referred to ear by way of vagus nerve and auricular nerve of ArnoldQ.

• MC site of CA LarynxQ. • Mostly originates from free edge and upper surface of anterior 1/3rd of true vocal cordQ followed by middle third 1/3rd. • Spread locally to anterior commissure than to opposite cord (conus elasticus initially acts as barrier for subglottic spreadQ). • Fixation of vocal cord indicates spread of disease to thyroaretenoid muscle and is a bad prognostic signQ. • As vocal cord is free of lymphatics, nodal metastases is never seen in cordal cancerQ, unless the disease spread beyond membranous cord. • Hoarseness is MC and the earliest symptomQ because of this glottic cancer is detected early.

• Least common siteQ. • Subglottic region extends from glottic area to lower border of cricoid cartilage. • Spreads locally around anterior wall to opposite side or downwards to trachea, upward spread to vocal cord is late and hoarseness is not an early symptomQ. • Lymphatic metastases to prelaryngeal, pretracheal (Delphian nodeQ) and lower jugular nodes. • Earliest and most prominent symptom is stridorQ, but it appears only in advanced stage. • Hoarseness is late feature.

Treatment of CA Larynx Stage

Site

Treatment

T1

All site

• External beam radiotherapyQ

T2

Glottic and Subglottic lesion Supraglottic lesion

• RadiotherapyQ • Supraglottic laryngectomyQ

All sites

• Total laryngectomy with neck dissectionQ for clinically positive nodes with postoperative radiotherapy if nodes are not palpable

T3 and T4

Head and Neck

Supraglottic Cancer

944

Surgery Essence NASOPHARYNGEAL CARCINOMA Nasopharyngeal Carcinoma • • • •

Nasopharyngeal cancer is most common in China particularly in southern states and TaiwanQ. People in Southern China, Taiwan and Indonesia are more prone to this cancerQ. MC tumor to produce cervical LN metastasisQ MC tumor responsible for secondaries in the neck with no obvious primary malignancyQ

Head and Neck

• Burning of incense or wood (Polycyclic hydrocarbon), use of preserved salted fish (Nitosamines) along with vitamin C deficient diet (vitamin C blocks nitrosification of amines and is thus protective) may be other factors operative in ChinaQ. Etiology • Exact etiology is not known. The factors responsible are: −− Genetic: Chinese −− Viral: Epstein-Barr virusQ −− Environmental: Air pollution, smoking of tobacco and opium, nitrosamines from dry salted fish, smoke from burning of incense and wood Pathology • Squamous cell carcinomaQ in various grades of its differentiation or its variants as transitional cell carcinoma and lymphoepithelioma, is the most common. • MC site of origin: Fossa of Rosenmuller in the lateral wall of NasopharynxQ. • LN involvement is common because of rich lymphatic network in the nasopharynxQ. Clinical Features • Age: It is mostly seen in fifth to seventh decades but may involve younger age groups. • Sex: Males are three times more prone than females. • Cervical lymphadenopathy is MC presenting symptom (60-90%)Q. • Nasal: Nasal obstruction, nasal discharge, denasal speech (rhinolalia clausa) and epistaxis.Q • Otologic: Due to obstruction of Eustachian tube , there is conductive hearing loss, serous or suppurative otitis media.Q • Presence of unilateral serous otitis media in an adult should raise suspicion of nasopharyngeal growth. • Involvement of IX , X and XI cranial nerves may occur, constituting jugular foramen syndrome. • Can cause conductive deafness (Eustachian tube blockage), ipsilateral temporoparietal neuralgia (involvement of CN V) and palatal paralysis (CN X)-collectively called Trotter’s triad. • Cervical nodal metastases may be the only manifestation of nasopharyngeal cancerQ. • Nodal metastases are seen in 75% of the patients, when first seen, about half of them with bilateral nodes. • Distant metastases involve bone, lung, liver and other sites. • Jaccods’s triad: Ipsilateral ophthalmoplegia + Amaurosis + Ipsilateral neuralgiaQ Diagnosis • CT scan: Demonstrate erosion of bone at the base of skull and the extent of tumor. • Biopsy is essential to show the exact histology of the malignancy.

Section 8

• In absence of nasopharyngeal lesion but with strong suspicion of malignancy, nasopharynx is exposed by transpalatal approach and a strip of mucosa and submucosa from the region of fossa of Rosenmuller should be taken and subjected to histology. Treatment • IrradiationQ is treatment of choice. • Chemotherapy for stage III and IV cancers

Multiple Choice Questions CAROTID BODY TUMOR

1. True about carotid body tumour is:  (UPPG 2010) a. Origin from non chromaffin tissue b. Most commonly is seen with people live at high altitude c. Family history positive d. FNAC is diagnostic e. Painful non mobile lump in the neck



2. A 40-years old patient is suffering from carotid body tumor. Which of the following is the best choice of treatment for him? (AIIMS Nov 2004) a. Excision of tumor b. Radiotherapy c. Chemotherapy d. Carotid artery ligation both proximal and distal to the tumor



3. A 60-year-old male presented with painless, compressible swelling in right side of neck. What is the diagnosis on the basis of given image findings? a. Branchial cyst b. Cystic hygroma c. Chemodectoma d. Branchial fistula

CYSTIC HYGROMA











4. True statement about carotid body tumor is: (PGI June 2002) a. Non-chromaffin paraganglioma b. Good prognosis c. Rarely metastasizes d. Similar to mixed parotid tumor 5. True about carotid body tumour is: (DPG 2006) a. Arises from Schwann cell b. Causes hypertension c. Arises from endothelial cell d. None of the above



6. Main problem associated with carotid body tumor operation is: (MHPGMCET 2001) a. The tumor blends with jugular vein b. The tumor blends with bifurcation of carotid artery c. Recurrence d. Vasovagal shock



7. Which one is not true regarding carotid body tumour? a. Unilateral  (Recent Question 2016, AIIMS June 97) b. Surgical resection is the treatment c. Non-chromaffin paraganglioma d. Middle age group is affected 8. Carotid body tumor most commonly presents at: (Recent Question 2015) a. 20-30 years b. 40-50 years c. 60-70 years d. Early childhood









9. Cystic hygroma may be associated with: (MCI March 2005) a. Turner’s syndrome b. Klinefelter’s syndrome c. Down’s syndrome d. All of the above 10. Brilliantly translucent swelling in the neck region in a 2 years child diagnosis is: (UPPG 2009) a. Lipoma b. Teratoma c. Cystic hygroma d. Thyroglossal cyst 11. This is the image of a newborn baby. The swelling is: a. Transilluminant b. Brilliantly transilluminant c. Translucent d. Brilliantly translucent

12. Which is incorrect about cystic hygroma? (APPG 2008) a. Brilliantly translucent b. Radiotherapy c. Sclerotherapy with bleomycin d. Sclerotherapy with actinomycin 13. All are true about cystic hygroma except: a. Pulsatile (AMU 95, DPG 2010) b. May cause respiratory obstruction c. Common in neck d. Present in birth 14. True about cystic hygroma:  (DPG 2008) a. Present in anterior triangle of neck b. Sclerosing agents are not useful c. Pre-operative MRI is crucial d. Surgery is always indicated 15. Treatment of cystic hygroma includes: (PGI May 2011) a. Complete excision b. Marsupialization c. Repeated aspiration d. Injection of sclerosing agents e. Observation or regular follow up 16. Cystic hygroma is known to occur in all except:  (Karnataka 2005, MHPGMCET 2002) a. Calf b. Neck c. Axilla d. Mediastinum



17. Treatment of choice for cystic hygroma: a. Percutaneous aspiration (DNB 2013, MHPGMCET 2007) b. Intralesional sclerosant injection c. En-bloc resection d. Surgical excision



18. Which of the following is false regarding cystic hygroma? a. Brilliantly translucent lesion (MHPGMCET 2008) b. Surgical excision is treatment of choice c. Recurrence is common with percutaneous Picibanil therapy d. A cystic lesion containing blood filled spaces

Head and Neck

946

Surgery Essence

19. True regarding cystic hygroma is: (AIIMS Nov 93) a. Non transilluminant b. Lined by columnar epithelium c. Lined by stratified squamous epithelium d. Develops from jugular lymphatic sequestration



20. All are true about cystic hygroma except: a. Aspiration is diagnostic (Punjab 2009, PGI Dec 99) b. 50% present at birth c. Presents as posterior cervical swelling d. Sequestration of lymphatic tissue



21. Cystic compressible, translucent swelling in the posterior triangle of neck: (DNB 2008, All India 89) a. Cystic hygroma b. Branchial cyst c. Thyroglossal cyst d. Dermoid cyst



22. The following are examples of Retention cysts due to blockade of excretory duct except one which is an example of Distension cyst due to exudation. Which is the one?  (APPG 2016) a. Cystic hygroma b. Sebaceous cyst c. Bartholin’s cyst d. Ranula



27. True about branchial cyst: (PGI June 2007) a. Seen deep to lower 1/3rd of sternocleidomastoid b. Wall consists of lymphoid tissue c. Filled with straw colored fluid with cholesterol crystals d. Presents at birth



28. What is true about branchial cyst? a. Present in anterior triangle of neck b. Cauterization is done c. Arises from 2nd cleft d. Present in lower 3rd of neck



29. Branchial cyst arises from which branchial cleft? a. First b. Second (MCI Sept 2009) c. Third d. Fourth



30. The commonest site of branchial cysts is:  (MCI June 2018, All India 94) a. Upper 1/3rd of the SCM b. Lower 1/3rd of the SCM c. Upper 2/3rd of the SCM d. Lower 2/3rd of the SCM



31. The most common site of the internal opening of a branchial fistula is at the: (UPSC 95) a. Lateral nasopharyngeal wall b. Fossa of Rosenmuller c. Gingivo-labial sulcus d. Tonsillar fossa



32. Commonest treatment of branchial cyst: (HPU 2005) a. Cystectomy b. Aspiration c. Excision d. Nothing done



33. Brachial cyst is lined by: (Recent Question 2017) a. Columnar epithelium b. Cuboidal epithelium c. Squamous epithelium d. Ciliated columnar epithelium



34. Branchiogenic carcinoma: (Recent Question 2017) a. Branchial cyst cancer b. Carcinoma arising from bronchus c. Type of carcinoma lung d. Commonly seen in young adults

BRANCHIAL CYST AND FISTULA

23. True about branchial cyst: (PGI Nov 2011) a. Arise from lower third of sternocleidomastoid b. Peak age of presentation is 3rd decade c. Cyst wall consists of lymphoid tissue d. Fluid contains cholesterol crystal e. Lined by squamous epithelium



24. Most frequent site of branchial cyst is at: ( MHSSMCET 2005) a. Upper third of posterior border of Sternocleidoma-stoid b. Lower third of anterior border of Sternocleidoma-stoid c. Upper third of anteromedial border of Sternocleidomastoid d. Supraclavicular fossa



25. True about branchial anomaly: (AIIMS Nov 2006) a. Cysts are more common than sinuses b. For sinuses surgery is not always indicated c. Cysts present with dysphagia and hoarseness of voice d. Most commonly due to 2nd branchial remnant 26. The given swelling develops from the remnant of:

Section 8



(DPG 2007)

THYROGLOSSAL CYST AND FISTULA

35. Excision of the hyoid bone is done in: (HPU 2005) a. Branchial cyst b. Branchial fistula c. Thyroglossal cyst d. Sublingual dermoids



36. Which is never a cause of thyroglossal fistula? a. Infection of thyroglossal cyst b. Inadequate removal of thyroglossal cyst c. Congenital d. None of the above



37. Thyroglossal fistula develops due to: a. Developmental anomaly b. Injury c. Incomplete removal of thyroglossal cyst d. Inflammatory disorder

(Kerala 91)

CERVICAL RIB

a. 1st branchial cleft c. 3rd branchial cleft

b. 2nd branchial cleft d. 4th branchial cleft



38. Regarding cervical rib, which statement is correct? a. It always connects to the scalene tubercle by a fibrous band b. It passes through the apex of the supraclavicular triangle c. It causes pressure on the ulnar nerve d. Pain is often located in the forearm



39. Adson’s test is positive in: a. Cervical rib c. Cervical fracture

(Kerala 89) b. Cervical spondylosis d. Cervical dislocation

Neck NECK DISSECTION



40. Structures not removed in radical neck dissection: a. X nerve (PGI June 2007) b. XI nerve c. Tail of parotid d. Parotid and post-auricular nerve 41. Structures preserved in radical neck dissection is:  (Recent Question 2017, All India 2000) a. Vagus nerve b. Submandibular gland c. Sternocleidomastoid d. Internal Jugular Vein



42. Which structure is preserved during modified radical neck dissection?  (DNB 2004) a. Phrenic nerve b. Submandibular gland c. Sternocleidomastoid d. Thoracic duct



43. In post radical neck dissection shoulder syndrome, all are seen except: (AIIMS Nov 2008) a. Restricted range of movement b. Pain c. Shoulder drooping d. Normal electromyographic finding



44. Modified radical dissection of neck all structures are preserved except: (UPPG 2010) a. Sternomastoid b. External jugular vein c. Internal jugular d. Spinal accessory 45. Level V cervical nodes includes: (MCI Sept 2007) a. Upper jugular nodes b. Middle jugular nodes c. Lower jugular nodes d. Posterior triangle nodes



46. In radical neck dissection, which structure is not removed? a. Cervical group of lymph nodes  (MCI March 2005) b. Sternocleidomastoid muscle c. Internal jugular vein d. None of the above 47. Kallu, 60 years old male presented with carcinoma stomach and palpable LN in left supraclavicular region. What is the level of this lymph node? a. III b. IV c. V d. VI





51. Structures preserved in modified radical neck dissection: a. Accessory nerve (PGI May 2011) b. Sternocleidomastoid muscle c. Submandibular gland d. Internal jugular vein e. Omohyoid muscle



52. Structures not removed in functional neck dissection is: a. Carotid artery, vagus nerve  (AIIMS Nov 93) b. Sternomastoid muscle, internal jugular vein c. Spinal accessory nerve, submandibular salivary gland d. Neck nodes



53. Structures preserved in functional radical dissection of the neck: (Recent Question 2017) a. Internal jugular vein b. Sternomastoid c. Lymph nodes d. Accessory nerve



54. Radical dissection of neck includes all except:  (Recent Question 2017) a. Cervical lymph nodes b. Sternocleidomastoid c. Phrenic nerves d. Internal jugular vein



55. A nerve injured in radical neck dissection leads to loss of sensation in medial side of the arm, nerve injured is: a. Long thoracic nerve (DNB 2014) b. Thoracodorsal nerve c. Dorsal scapular nerve d. Medial cutaneous nerve of arm



56. In MRND type-II, structures preserved are: (MHCET 2016) a. Spinal accessory nerve + SCM b. Spinal accessory nerve + internal jugular vein c. SCM + internal Jugular vein d. Level I-V LN + SCM



57. Removal of level I, II, III, IV lymph node in neck is called: (Recent Question 2016) a. Extended supraomohyoid dissection b. Supraomohyoid dissection c. Anterolateral dissection d. Posterolateral dissection

CA LARYNX

58. Precancerous lesion of the larynx include: a. Keratosis laryngis b. Pachydermia laryngis c. Laryngis sicca d. Sclerma

CA NASOPHARYNX



48. Radical neck dissection included all except:  (MHSSMCET 2005) a. Sternocleidomastoid b. Accessory nerve c. Level III lymph nodes d. Jugular vein



49. Posterior triangle LN is what level? (MHSSMCET 2009) a. Level I b. Level II c. Level V d. Level IV

(Recent Question 2017)



59. Trotters triad is seen in: a. Angiofibroma b. Nasopharyngeal carcinoma c. Laryngeal carcinoma d. Growth in fossa of Rosenmuller



60. Which of the following is the most common tumour to produce metastasis to cervical lymph nodes? (UPSC 2008) a. Glottic carcinoma b. Nasopharyngeal carcinoma c. Carcinoma base of tongue d. Carcinoma lip

Head and Neck



50. In extended supraomohyoid neck dissection, lymph lode dissection is done up to:  (Recent Question 2016, MHSSMCET 2010) a. 2 b. 3 c. 4 d. 5

Section 8





947

948

Surgery Essence

61. Secondaries in the neck with no obvious primary malignancy is most often due to: (JIPMER 93) a. CA Stomach b. CA Larynx c. CA Nasopharynx d. CA Thyroid

Head and Neck

MISCELLANEOUS 62. True about superior sulcus tumor: a. Anhidrosis in thoracic region b. Pain in upper aspect of arm c. Flexor atrophy d. 2nd and 3rd rib erosion



63. A 30‑year‑old lady presented with cystic neck swelling as shown in the picture below. What could be the possible diagnosis? (Recent Question 2019)

a. Dermoid cyst c. Branchial cyst

Section 8

(DPG 2007)



b. Sebaceous cyst d. Secondaries neck



64. In which of the following head and neck cancers, is lymph node metastasis least common? (AIIMS May 2008) a. Tongue b. Buccal mucosa c. Hard palate d. Lower alveolus



65. Blow out carotid is characteristically seen with:  (AIIMS Nov 98) a. Thyroidectomy b. Radical neck dissection c. Flap necrosis d. Sistrunk operation



66. Which of the following does not move on deglutition?  (All India 91) a. Subligual dermoid b. Thyroid nodule c. Pretracheal lymph node d. Thyroglossal cyst



67. An elderly male presents with 4 × 5 cm lump in right neck. FNAC revealed it to be squamous cell carcinoma. No primary was found. A diagnosis of unknown primary was made. According to AJCC system or classification, the TNM staging of tumour would be: (JIPMER 2014, AIIMS May 2013) a. T1N2M0 b. T0N2aM1 c. T1N2cM0 d. T0N2aMx

Explanations CAROTID BODY TUMOR

1. b. Most commonly is seen with people live at high altitude; c. Family history positive (Ref: Sabiston 20/e p810, 19/e p814-815; Schwartz 10/e 678, 849; Bailey 27/e p757; Devita 9/e p772-773)



2. Ans. a. Excision of tumor



3. Ans. c. Chemodectoma (Ref: Sabiston 20/e p810; Schwartz 10/e p678; Bailey 27/e p757)



4. Ans. a. Non-chromaffin paraganglioma; b. Good prognosis; c. Rarely metastasizes



5. Ans. d. None of the above



6. Ans. b. The tumor blends with bifurcation of carotid artery



7. Ans. None



8. Ans. b. 40-50 years

CYSTIC HYGROMA

9. Ans. a. Turner’s syndrome (Ref: Sabiston 20/e p1861, 19/e p1819; Schwartz 10/e 598, 1852; Bailey 27/e p754)

10. Ans. c. Cystic hygroma

11. Ans. b. Brilliantly transilluminant



12. Ans. b. Radiotherapy

13. Ans. a. Pulsatile



14. Ans. c. Pre-operative MRI is crucial

15. Ans. a. Complete excision, d. Injection of sclerosing agents



16. Ans. a. Calf

17. Ans. d. Surgical excision



18. Ans. d. A cystic lesion containing blood filled spaces

19. Ans. d. Develops from jugular lymphatic sequestration



20. Ans. a. Aspiration is diagnostic

21. Ans. a. Cystic hygroma



22. Ans. a. Cystic hygroma

BRANCHIAL CYST AND FISTULA

23. Ans. b. Peak age of presentation is 3rd decade; c. Cyst wall consists of lymphoid tissue; d. Fluid contains cholesterol crystal; e. Lined by squamous epithelium (Ref: Sabiston 20/e p1862, 19/e p834; Schwartz 10/e p598,1602; Bailey 27/e p753-754)



24. Ans. c. Upper third of anteromedial border of Sternocleidomastoid



25. Ans. d. Most commonly due to 2nd branchial remnant (Ref: Sabiston 20/e p1862, 19/e p834; Schwartz 10/e p598,1602; Bailey 27/e p753-754)



26. Ans. b. 2nd branchial cleft (Ref: Sabiston 20/e p1862; Schwartz 10/e p598; Bailey 27/e p753) “A branchial cyst, thought to develop from the vestigial remnants of the second branchial cleft, is usually lined by squamous epithelium, and contains thick, turbid fluid full of cholesterol crystals. The cyst usually presents in the upper neck in early or middle adulthood and is found at the junction of the upper third and middle third of the sternomastoid muscle at its anterior border. It is a fluctuant swelling that may transilluminate and is often soft in its early stages so that it may be difficult to palpate.” -Bailey 27/e p753



27. Ans. b. Wall consists of lymphoid tissue; c. Filled with straw colored fluid with cholesterol crystals



28. Ans. c. Arises from 2nd cleft



29. Ans. b. Second



30. Ans. a. Upper 1/3rd of the SCM



31. Ans. d. Tonsillar fossa



32. Ans. c. Excision



33. Ans. c. Squamous epithelium (Ref: Sabiston 20/e p1862; Schwartz 10/e p598; Bailey 27/e p753)



34. Ans. a. Branchial cyst cancer “Branchiogenic carcinoma, which is squamous cell carcinoma arising in a branchial cyst, is extremely rare and a highly contentious clinicopathologic entity.” 

950

Surgery Essence



THYROGLOSSAL CYST AND FISTULA 35. Ans. c. Thyroglossal cyst 36. Ans. c. Congenital (Ref: Sabiston 20/e p1861, 19/e p814; Schwartz 10/e p598,1521-1522,1602; Bailey 27/e p755)

Thyroglossal Fistula • A thyroglossal fistula usually presents as discharging sinus in the midlineQ of the neck in the line of thyroid descent. • It is never congenital but follows infection or inadequate removal of a thyroglossal cystQ • Acquired conditionQ

Head and Neck

Clinical Features • Presentation is with a fistulous opening near to the midline of the neckQ • Fistula may become infected & discharge pusQ Treatment • Fistula should be excised along with the thyroglossal tract upto the base of the tongue. • This requires removing the central (middle one third) of the hyoid boneQ.

37. Ans. c. Incomplete removal of thyroglossal cyst

CERVICAL RIB

38. Ans. c. It causes pressure on the ulnar nerve; d. Pain is often located in the forearm (Ref: Sabiston 20/e p1604, 19/e p1595; Bailey 27/e p939, 952, 992)



39. Ans. a. Cervical rib

NECK DISSECTION

40. Ans. a. X nerve; c. Tail of parotid; d. Parotid and post-auricular nerve (Ref: Sabiston 20/e p794; Schwartz 10/e p595; Bailey 27/e p758-759; Cancer of the Head and Neck by Suen and Myer 4/e p416-418) Radical Neck Dissection

Modified Radical Neck Dissection

• Removal of lymph nodes I-V + spinal accessory nerve + internal jugular vein + sternocleidomastoid muscleQ

• Removal of level I-V lymph nodes with (Mnemonic: SISm): −− Type I: preserves only spinal accessory nerveQ −− Type II: preserves both spinal accessory nerve and internal jugular veinQ −− Type III: preserves spinal accessory nerve, internal jugular vein and sternocleidomastoid muscleQ (Functional neck dissectionQ)



41. Ans. a. Vagus nerve

42. Ans. c. Sternocleidomastoid



43. Ans. d. Normal electromyographic finding (Ref: Surgical Management of Neck Metastasis by Jack L Gluckman, Jonas T Johnson/53)

Shoulder Syndrome • In radical neck dissection, the most crippling complication is the “Shoulder syndrome” arising from denervation and atrophy of the trapezius muscle due to sacrifice of the spinal accessory nerveQ. Shoulder syndrome is characterized by • Inability to abduct the shoulder beyond 900 cephaladQ • Long standing pain in the shoulderQ

Section 8

• Deformity of the shoulder girdle (drooping of the shoulder with abduction and external rotation)Q

44. Ans. b. External jugular vein



45. Ans. d. Posterior triangle nodes (Ref: Sabiston 20/e p792; Schwartz 10/e p595-597; Bailey 27/e p728,729,764,800,801)



46. Ans. d. None of the above

47. Ans. b. IV



48. Ans. None

49. Ans. c. Level V



50. Ans. c. 4 (Ref: Sabiston 20/e p794; Schwartz 10/e p595; Bailey 27/e p758-759; Cancer of the Head and Neck by Suen and Myer 4/e p416-418)



51. Ans. a. Accessory nerve; b. Sternocleidomastoid muscle; d. Internal jugular vein



52. Ans. a. Carotid artery, vagus nerve; b. Sternomastoid muscle, internal jugular vein



53. Ans. a. Internal jugular vein; b. Sternomastoid; d. Accessory nerve

Neck 54. Ans. c. Phrenic nerves



55. Ans. d. Medial cutaneous nerve of arm (Ref: Bailey 25/e p733)



56. Ans. b. Spinal accessory nerve + internal jugular vein



57. Ans. a. Extended supraomohyoid dissection

CA LARYNX

58. Ans. a. Keratosis laryngis; b. Pachydermia laryngis (Ref: Schwartz 10/e p589-591; Scott-Brown 5/e p106) • Localized area of thickening of the epithelium which appear as single or multiple chalky white elevations on the upper surface and edge of one or both the cords usually with involvement of the membranous portion are not uncommon.

Section 8



951

• These have been designated as keratosis, hyperkeratosis, leukoplakia, pachydermia laryngis or by other terms by laryngologist and pathologist. • Laryngeal keratosis or pachydermia laryngis is a precursor of laryngeal cancer that bears great similarity to oral leukoplakiaQ.

CA NASOPHARYNX

59. Ans. b. Nasopharyngeal carcinoma (Ref: Sabiston 20/e p804-805; Schwartz 10/e p580, 593-594; Bailey 27/e p733-735; Devita 9/e p764-766)



60. Ans. b. Nasopharyngeal carcinoma

61. Ans. c. CA Nasopharynx

MISCELLANEOUS

62. Ans. d. 2nd and 3rd rib erosion (Ref: Sabiston 20/e p1591, 1592; Schwartz 10/e p623, 641-642)



63. Ans. a. Dermoid cyst (Ref: Sabiston 20/e p1860)



64. Ans. c. Hard palate (Ref: Bailey 27/e p764-765; Devita 9/e p750; Cancer of the Head and Neck by Suen and Myer 4/e p288-289) • LN metastasis is most common in: CA tongueQ >Floor of mouth >Lower alveolus >Buccal mucosa >Upper alveolus >Hard palate > LipQ.



65. Ans. b. Radical neck dissection (Ref: www.ajnr.org/content/28/1/181)

History of radiation exposure followed by radical neck dissection increases the risk of carotid blowout in head and neck cancers.

• Carotid blowout refers to rupture of the carotid and its branches • It is one of the most devastating complications associated with therapy for head and neck cancersQ • Carotid blowout tends to occur in head and neck cancerQ, radiation induced necrosisQ, recurrent tumorsQ or pharynocutaneous fistulasQ. • The clinical signs and symptoms related to rupture of carotid artery have been referred as carotid blowout syndrome • Reported morbidity and mortality rates are 40% and 60% respectively.

66. Ans. a. Sublingual dermoid (Ref: Schwartz 9/e p1344-1345) Structures Moving with Deglutition • Thyroid gland • Thyroglossal cystQ Q



67. Ans. d. T0N2aMx

• Pre and paratracheal nodesQ • Sub-hyoid bursaQ

Head and Neck

Carotid Blowout

CHAPTER

38

Facial Injuries and Abnormalities

CLEFT LIP AND CLEFT PALATE Cleft Lip and Palate • Clefts of the lip, alveolus & hard and soft palate are the MC congenital abnormalities of the orofacial structuresQ. • Frequently occur as isolated deformities but can be associated with other medical conditions, particularly congenital heart diseaseQ. • Incomplete clefts affect only a portion of the lip and contain a bridge of tissue connecting the central & lateral lip elements, referred to as Simonart’s bandQ. • Cleft lip is due to non-fusion of maxillary process with medial nasal processQ. • Unilateral cleft lip is associated with posterior displacement of alar cartilageQ Incidence • Highest incidence reported for cleft lip & palate occurs in the Indian tribes of Montana, USA (1:276). • Cleft lip/palate predominates in malesQ • Cleft palate alone appears to be more common in femalesQ. • Incidence of cleft lip & palate is 1:600 live birthsQ

•  Incidence of isolated cleft palate is 1:1000 live birthsQ.

Distribution • In unilateral cleft lip the deformity affects the left sideQ in 60% of cases. Typical Distribution of Cleft Types • Cleft lip alone: 15%

• Cleft lip & palate: 45% (MC)Q

•  Isolated cleft palate: 40%

Etiology of cleft lip and palate • Etiology of cleft lip & palate: Genetic predisposition & a contributory environmental componentQ. • Environmental factors: Maternal epilepsyQ & drugs (steroids, diazepam & phenytoinQ). Associated syndromes • Although most clefts of the lip and palate occur as an isolated deformity, Pierre Robin sequence remains the most common syndromeQ. • Other associated syndromes: Stickler’s (ophthalmic and musculoskeletal abnormalities), Shprintzen’s (cardiac anomalies), Down’s, Apert’s and Treacher-Collins’ syndromes. Types of Cleft Lip

Types of Cleft Palate

• Unilateral cleft lip • Bilateral cleft lip

• Incomplete: Cleft of the hard palate remains attached to the nasal septum and vomerQ • Complete: Nasal septum and vomer are completely separated from the palatine processesQ

Antenatal diagnosis • All but isolated cleft palate can be diagnosed by ultrasound after 18 weeksQ gestation Problems immediately after birth • Some babies are able to feed normally but some will need assistance • Breathing problems in Pierre Robin sequence may be life threatening

Facial Injuries and Abnormalities

953

Management • Cleft lip • Cleft palate

• Repaired between 3 and 6 months of ageQ • Repaired between 6 and 18 months of ageQ

Principles of Surgery • Cleft lip surgery attaches and reconnects the muscles around the oral sphincterQ • Cleft palate surgery aims to bring together mucosa and muscles with minimal scarringQ • Two-stage procedures attempt to minimize dissectionQ Secondary Management

Section 8

• Surgical techniques are aimed at restoring normal anatomy.

• Following primary surgery, regular review by a multidisciplinary team is essentialQ. • Many aspects of cleft care require long-term review: Hearing, speech, dental development, facial growthQ.

LAHSAL SYSTEM OF CLASSIFICATION

• • • • • •

LAHSAL system is a diagrammatic classification of cleft lip & cleft palate. In this classification system, mouth is divided into 6 parts. i.e. LAHSAL Right Lip; right Alveolus; Hard palate; Soft palate; left Alveolus; left Lip First character is for patients right lip & last character for patients left lip Complete cleft is indicated with a capital letter & an incomplete cleft with small letter. No cleft is represented with a dot. Examples of LAHSAL System of Classification Bilateral complete cleft lip & palate

LAHSAL

Left complete cleft lip

…..L

Right incomplete cleft lip & alveolus

la….

Incomplete hard palate, complete soft palate defect

..hS..

CLEFT LIP REPAIR TECHNIQUES Cleft lip Repair Techniques • Millard Rotation Advancement Technique: Most widely usedQ • Le MuserierQ



•  ThompsonQ •  Tennison-RendallQ

Head and Neck

LAHSAL System of Classification

954

Surgery Essence TIMING OF PROCEDURES FOR CLEFT LIP AND CLEFT PALATE Timing of Primary Cleft Lip and Palate Procedures (After Delaire)          Cleft lip alone

Head and Neck

• Unilateral (one side): One operation at 5-6 months • Bilateral (both sides): One operation at 4-5 months

Cleft palate alone

Cleft lip and palate

• Soft palate only: One operation at 6 monthsQ • Soft and hard palate: Two operations −− Soft palate at 6 monthsQ −− Hard palate at 15-18 months

• Unilateral: Two operations • Cleft lip and soft palate at 5-6 months • Hard palate and gum pad with or without lip revision at 15-18 months • Bilateral: Two operations −− Cleft lip and soft palate at 4-5 months −− Hard palate and gum pad with or without lip revision at 15-18 months

MANDIBULAR FRACTURE Fractures of the Mandible • Condylar neckQ is the weakest part of the mandible and MC site of fractureQ • Mandible may fracture directly at the point of the blowQ • Indirectly where the force from the blow is transmitted and the mandible fractures at a point of weakness distant from the original blow, known as ‘guardsman’ fractureQ. • ‘Butterfly’ fracture of the mandible: A segment of mandible is detached from the rest of the mandible in the canine regionsQ. Diagnosis • Recommended radiographic evaluation of a mandible fracture: Panoramic radiograph (Panorex) and Towne’s view X-rayQ. Treatment • As in midface fractures, restoration of dental occlusion forms the foundation for fracture managementQ. • Intermaxillary fixation before fracture exposure and plating is necessaryQ. • Condylar and subcondylar mandible fractures are most often treated by IMF aloneQ. • Medical management of mandibular fractures involves a purée-type diet, interdental fixation for several weeks, 1% chlorhexidine mouth rinses, and antibioticsQ.

MIDFACE FRACTURE Midface Fractures

Section 8

• Midface fractures involving the maxilla can be classified by fracture patterns know as Le Fort I, II, and III. Le Fort I

• Fracture line runs above and parallel to palateQ • Effectively separates alveolus and palate from the facial skeleton aboveQ

Le Fort II

• Pyramidal in shapeQ • Passes through the root of nose, lacrimal bone, floor of orbit, upper part of maxillary sinus and pterygoid plateQ • Orbital floor is always involvedQ

Le Fort III

• Complete disjunction of the facial skeleton from the skull baseQ • Fracture line runs high through the nasal bridge, septum and ethmoids, and through the bones of orbit to the frontozygomatic sutureQ.

Management • Interdental or intermaxillary fixation is necessary to reestablish the proper dentoskeletal relationships, immobilize the fractured bones, and ensure normal postoperative occlusionQ.

Facial Injuries and Abnormalities

955

ZYGOMATIC BONE FRACTURE

• MC fracture of the middle third of the face: Nose > zygomatic boneQ • Also known as Tripod fractureQ, because the zygoma is fractured at its 3 processes: 1. Zygomatico-frontal fractureQ 2. Zygomatico-temporal fractureQ 3. Infraorbital fractureQ Clinical Features of Zygomatic Bone Fracture • • • •

Flattening of malar prominenceQ Step-deformity of infraorbital margin EpistaxisQ Restricted ocular movements due to entrapment of inferior rectus muscleQ (may lead to diplopiaQ)

• Anesthesia in the distribution of infraorbital nerveQ • Oblique palpebral fissure, due to entrapment of lateral palpebral ligament • Periorbital emphysema due to escape of air from the maxillary sinus

Section 8

Zygomatic Bone Fracture

Diagnosis • X-ray Water’s viewQ • CT scan: Best for diagnosis of zygomatic bone fractureQ Treatment • Only displaced fractures require treatmentQ • Treatment of choice: Open reduction and internal fixationQ

BLOW OUT FRACTURE OF ORBIT Blow-out Fractures of the Orbit • • • •

Direct trauma to the globe of the eye may push it back within the orbit. Occur when a blunt object strikes the globeQ. Weakest plate of bone, most commonly the orbital floor, fractures, and the orbital contents herniate down into the maxillary antrumQ. Tear-drop signQ is seen

Treatment • Significant delay in treatment may be associated with less success than early diagnosis and planned treatment. • Orbital floor exploration allows for release of displaced or entrapped soft tissue, correcting any extra-ocular motility disturbancesQ.

Head and Neck

• Soft-tissue herniation lead to muscular dysfunction, particularly the inferior oblique and inferior rectus, leading to failure of the eye to rotate upwards. • Enophthalmos and diplopia can followQ • Paraesthesia in the distribution of the infraorbital nerve is an important clue to the blow-out fractureQ.

Multiple Choice Questions









(PGI 2001)



13. Cleft lip is due to non fusion of: a. Maxillary process with lateral nasal process b. Maxillary process with medial nasal process c. Maxillary process with mandibular process d. All of the above



2. Surgical correction in cleft palate primarily aims at all of the following except: (MCI March 2010) a. Control of regurgitation b. To promote normal dentition and facial growth c. To get a normal speech d. Normal appearance of lips, nose and face

14. Unilateral cleft lip is associated with: (PGI 99) a. Posterior displacement of alar cartilage b. Columella elongated c. Always cleft palate d. Defective sucking



3. Ideal time for cleft lip repair surgery:  (MHPGMCET 2006, JIPMER 97) a. 3-6 weeks b. 6-12 weeks c. 1-1.5 years d. 3-4 years

15. Most common congenital anomaly of the face is: a. Cleft lip alone (MCI March 2008) b. Isolated cleft palate c. Cleft lip and cleft palate d. All have equal incidence



16. Millards ‘Rule of Ten’ includes all except: (AMU 95) a. 10 lbs b. 10 weeks of age c. 10 gm% hemoglobin d. 10 months of age

CLEFT LIP AND PALATE 1. True about cleft palate: a. Surgery should be done at 1 year b. 50% recover speech after operation c. Associated with hearing loss d. Associated with cleft lip in 45%

(PGI Nov 2010)

4. Ideal time for surgery in case of unilateral cleft lip:

a. 12 months

17. In LAHSHAL terminology for cleft lip and cleft palate, LAHSHAL denotes: (Recent Question 2013) a. Bilateral cleft palate only b. Bilateral cleft lip only c. Bilateral cleft lip and palate d. No cleft



5. Unilateral cleft lip is best repaired at: (Recent Question 2017) a. 4-5 months b. 5-6 months c. 6-9 months d. 9-12 months



6. All are do about submucosal cleft palate except: (DNB 2012) a. Bifid uvula b. Notched hard palate c. Lip pits d. Zona pellucida

18. Which of the following is the ideal time for the repair of cleft palate? (Recent Question 2014, AIIMS November 2014) a. 9-12 months b. 18-24 months c. 2-3 years d. 5-6 years



7. Which is the appropriate age for repair of cleft palate?



19. Pierre Robbin’s sequence includes: (PGI Dec 2008) a. Glossoptosis b. Airway obstruction c. Cleft lip d. Micrognathia e. Heart anomaly



20. Which one of the following is the primary defect in Pierre Robbin’s syndrome? (UPSC 2006) a. Micrognathia b. Glossoptosis c. High arched palate d. Cleft palate



21. Pierre Robbin’s syndrome is: a. Cleft palate with syndactyly b. Cleft palate with mandibular hypoplasia and respiratory obstruction c. Cleft lip with mandibular hypoplasia d. Cleft lip



22. Hynes pharyngoplasty is used to improve a child’s: a. Appearance b. Teething c. Speech d. Feeding

a. 6 months to 1 year c. At puberty

(All India 98, 94, AIIMS June 98) b. 12-15 months d. Just after birth



8. With respect to repair of cleft palate, the soft palate is first repaired, ideal time for which is?  (Recent Question 2016) a. 12 months b. 9 months c. 6 months d. 3 months



9. Commonest type of cleft lip is: a. Bilateral b. Midline c. Combined with cleft palate d. Unilateral

(AIIMS 91)



10. In cleft lip operation all the stitches are removed on: (Recent Question 2016) a. 2nd day b. 4th day c. 10th day d. 14th day



11. Unilateral clefts are most common on: a. Left side b. Right side c. Median d. None of the above



12. The following is the method for operating cleft lip except: (Recent Question 2016) a. Le Muserier’s method b. Tennison’s method c. Millard’s method d. Wardill’s method

23. Rhinoplasty is usually done at the age (years) of until the nose is fully grown: a. 6 years b. 12 years c. 16 years d. 25 years 24. Rhtidectomy operation involves: (JIPMER 92) a. Correction of nasal defects b. Removal of wrinkles in forehead c. Straightening of curved penis d. Correction of protruding lips

Facial Injuries and Abnormalities

957

MAXILLOFACIAL INJURY 25. Fracture mandible with edentulous jaw is best treated with: a. External fixator (UPPG 2004) b. Minerva-plaster c. Interdental wiring d. Intermaxillary elastic traction



26. Most common site of mandible fracture: (Recent Question 2017) a. Condyle b. Angle c. Ramus d. Body

27. Best view for mandible is: a. Antero-posterior c. Oblique





(UPPG 2007) b. Lateral d. Orthopentomogram

28. A man sustained injury and presented with fluid coming out through nose. What could be the possible fracture? (MCI March 2007) a. Fracture base of skull b. Fracture of mandible c. Fracture of maxilla d. None of the above 29. A 20-year-old man is hit on the eye with a ball. On examination there is restriction of lateral and upward gaze and diplopia. There is no obvious visible sign of enophthalmos, the likely diagnosis is: a. Zygoma fracture b. Maxillary fracture c. Blow out fracture of the orbit d. Injury to lateral rectus 30. Best treatment of above condition will be: a. Do nothing and assurance b. Explore the orbit c. Ophthalmic exercise to correct diplopia d. Reinsertion of lateral rectus muscle 31. Clinical feature of fracture of zygomatic bone include all of the following except: (All India 97) a. Diplopia b. Trismus c. Bleeding d. CSF rhinorrhea



32. What is the type of fracture? a. Le Fort I b. Le Fort II c. Le Fort III d. Le Fort IV

(Recent Question 2016)



33. Mandible is commonly fractured: a. At the neck of the condyle b. Through the angle c. Through the cannine fossa d. At the middle



34. Le Forte II facial fracture implies: (Recent Question 2016) a. Fracture running through alveolar ridge b. Fracture running through midline of the palate and zygomatico maxillary suture c. Fracture running through zygomatic process of the maxilla, floor of orbit, root of nose on one side only d. Similar to C but on both sides



35. Le-Forte fracture is for: a. Facial skeleton c. Spinal injury

(Recent Question 2017) b. Lower limb bone d. Pelvis fracture



36. Tripod fracture is seen in: a. Zygomatic bone c. Maxilla

(DNB 2010) b. Temporomandibular joint d. Frontal bone

37. Costen’s syndrome refers to neurological pain associated with: (DNB 2010) a. Sphenopalatine ganglion b. Temporomandibular joint c. Glossopharyngeal nerve d. Lingual nerve

38. In Kernahan Striped Classification, the main reference point is: (Recent Question 2018) a. Incisive foramen b. Soft palate c. Hard palate d. Alveolus

Head and Neck



Section 8



Explanations CLEFT LIP AND PALATE

1. Ans. a. Surgery should be done at 1 year, c. Associated with hearing loss, d. Associated with cleft lip in 45% (Ref. Sabiston 20/e p19461947; Schwartz 10/e p1840-1844; Bailey 27/e p688-700)



2. Ans. d. Normal appearance of lips, nose and face (Ref. Sabiston 20/e p1947; Schwartz 10/e p1840-1844; Bailey 27/e p692, 25/e p662-668)

Objectives of the Cleft Palate Repair • To produce anatomical closure of the defectQ. • To create an apparatus for development and production of normal speechQ. • To minimize the maxillary growth disturbances and dento-alveolar deformitiesQ.

3. Ans. b. 6-12 weeks



5. Ans. b. 5-6 months (Ref: Sabiston 20/e p1946; Schwartz 10/e p1844; Bailey 27/e p692)

4. Ans. b. 3 to 6 months



6. Ans. c. Lip pits (Ref. Cleft Palate and Craniofacial abnormalities by Ann W. Kummer/51)

Submucosal Cleft Palate • A congenital defect that affects the underlying structure of the palate, while the oral surface mucosa is intact • Most children with submucosal cleft palate are asymptomatic and this is often not diagnosed until later • Identification of submucosal cleft palate requires intraoral examination for: 1. Bifid uvula 2. Zona pellucida (submucosal absence of muscularis uvulae) 3. Notching of posterior border of hard palate 4. Nasopharyngeal regurgitation during feeding (only finding of occult submucosal cleft palate)

7. Ans. a. 6 months to 1 year



8. Ans. c. 6 months (Ref. Bailey 27/e p692)



9. Ans. c. Combined with cleft palate

10. Ans. b. 4th day (Ref. Sabiston 18/e p2134)

Guidelines for Day of Suture Removal by Area Body Regions

Removal

Body Regions

Removal

Eyelid

3-4

Chest, abdomen

8-10

Eyebrow

3-5

Ear

10-14

Nose

3-5

Back

12-14

Lip

3-4Q

Extremities

12-14

Face (other)

3-4

Hand

10-14

Scalp

6-8 days

Foot, sole

12-14

Q



11. Ans. a. Left side



12. Ans. d. Wardill’s method (Ref. Sabiston 20/e p1947; Schwartz 10/e p1840-1844; Bailey 27/e p692)



13. Ans. b. Maxillary process with medial nasal process



15. Ans. c. Cleft lip and cleft palate



16. Ans. d. 10 months of age

14. Ans. a. Posterior displacement of alar cartilage

“Rule of 10s” by Surgeons Wilhelmmesen and Musgrave Surgery is done in cleft lip in a child when • Age ≥10 weeksQ



•  Weight ≥10 poundsQ



•  Hemoglobin ≥10 gmQ

Facial Injuries and Abnormalities 17. Ans. c. Bilateral cleft lip and palate (Ref: Bailey 26/e p637)



18. Ans. a. 9-12 months



19. Ans. a. Glossoptosis, b. Airway obstruction, d. Micrognathia (Ref. Schwartz 10/e p1848; Bailey 27/e p689, 691, 701)

Pierre Robin Sequence • Pierre Robin sequence remains the most common syndrome in clefts of the lip and palate • Pierre Robin sequence is characterized by three pathognomonic findings: 1. Microretrognathia (primary defect)Q 2. GlossoptosisQ 3. Respiratory distressQ

Section 8



959

• Pierre Robin sequence may or may not be associated with a palatal cleftQ. • Micrognathia prevents the natural caudal migration of the tongue resulting in the deformityQ. • Functional consequences include intermittent respiratory obstruction and obstructive sleep apnea that may affect feeding, growth, and safety of the airwayQ. • Posteriorly displaced tongue (glossoptosis), which is associated with early respiratory and feeding difficultiesQ. • Treatment: Beverly-Douglas procedure (Tongue is fixed anteriorlyQ)



20. Ans. a. Micrognathia



21. Ans. b. Cleft palate with mandibular hypoplasia and respiratory obstruction



22. Ans. c. Speech (Ref. Bailey 25/e p663)

Hynes Pharyngoplasty • Used for the treatment of Velopharyngeal incompetence leading to speech problemsQ

23. Ans. c. 16 years (Ref. Bailey 27/e p699, 701, 702)

• Orthognathic surgery  is to correct conditions of the jaw and face related to structure, growth • Orthognathic surgery is usually performed when facial growth is complete (16 years in female patients, 19 years in male patientsQ). • It’s suggested to get rhinoplasty (correcting and reconstructing the form, restoring the functions, and aesthetically enhancing the nose) done after 16 yearsQ of age, when natural bone structure is “settled”.



24. Ans. b. Removal of wrinkles in forehead (Ref. Dorland’s Dictionary 28/e p1463)

Rhytidectomy • A facelift operation by surgical removal of wrinklesQ • Type of cosmetic surgery procedure used to give a more youthful facial appearance.

MAXILLOFACIAL INJURY

25. Ans. a. External fixator (Ref. Sabiston 20/e p422; Schwartz 10/e p197; Bailey 27/e p358)



26. Ans. a. Condyle (Ref: Sabiston 20/e p1949; Schwartz 10/e p1853; Bailey 27/e p358)



27. Ans. d. Orthopentomogram



28. Ans. a. Fracture base of skull (Ref. Sabiston 20/e p420; Schwartz 10/e p576; Bailey 27/e p333)

Head and Neck

Orthognathic Surgery

960

Surgery Essence Basilar Skull Fracture • Fracture of the base of the skull, typically involving the temporal bone, occipital bone, sphenoid bone, and/or ethmoid boneQ. • Such fractures can cause tears in the meninges, with resultant leakage of the CSFQ.

Head and Neck

• Leaking fluid may accumulate in the middle ear space, and dribble out through a perforated eardrum (CSF otorrheaQ) or into the nasopharynx via the eustachian tube, causing a salty taste. • CSF may also drip from the nose (CSF rhinorrheaQ) in fractures of the anterior skull base, yielding a halo signQ. • These signs are pathognomonic for basilar skull fractureQ.

29. Ans. c. Blow out fracture of the orbit (Ref. Sabiston 20/e p421; Schwartz 10/e p577; Bailey 27/e p677)



30. Ans. b. Explore the orbit



31. Ans. d. CSF rhinorrhea (Ref. Sabiston 20/e p421; Schwartz 10/e p577, 1854-1855; Bailey 27/e p359)



32. Ans. b. Le Fort II (Ref: Sabiston 20/e p810; Schwartz 10/e p678; Bailey 27/e p757)



33. Ans. a. At the neck of the condyle



34. Ans. d. Similar to C but on both sides (Ref. Sabiston 20/e p422; Schwartz 27/e p577; Bailey 27/e p360)



35. Ans. a. Facial skeleton (Ref: Schwartz 10/e p577; Bailey 27/e p360)



36. Ans. a. Zygomatic bone



37. Ans. b. Temporomandibular joint (Ref. Dhingra 4/e p400)

Costen’s Syndrome • Abnormality of temporomandibular joint due to defective bite • Characterized by otalgia, feeling of blocked ear, tinnitus and sometimes vertigo • Pain also radiates to frontal, parietal and occipital region

38. Ans. a. Incisive foramen (Ref: Classification in Facial Plastic Surgery By Paul James (2009)/p49 Kernahan Striped Y Classification • Incisive foramen is taken as reference • The ‘Y’ is divided into three sections: Lip, alveolus & palateQ

Section 8

Q

SECTION

9

Oncology CHAPTERS ˆˆ ˆˆ

Oncology Sarcoma

CHAPTER

39

Oncology

SCREENING IN MALIGNANCY • Colorectal cancerQ

Well-established Benefit of Screening in • CA cervixQ • CA oral cavityQ

• CA breastQ

American Cancer Society Recommendations for Early Detection of Cancer in Average-Risk, Asymptomatic Individuals Cancer Site Population Test or Procedure Frequency • Breast Women aged • Breast self-examination • Monthly, starting at age 20 ≥ 20 years • Clinical breast examination • Every 3 years, ages 20–39; Annual, starting at age 40Q • Mammography • Annual, starting at age 40Q • Colorectal Men and women • Fecal occult blood test (FOBT) or fecal • Annual, starting at age 50Q aged ≥ 50 years immunochemical test (FIT) • Flexible sigmoidoscopyQ • Every 5 years, starting at age 50 • FOBT and flexible sigmoidoscopyQ • Annual FOBT (or FIT) and flexible sigmoidoscopy every 5 years, starting at age 50Q • Double-contrast barium enema (DCBE)Q • DCBE every 5 years, starting at age 50Q • ColonoscopyQ • Colonoscopy every 10 yearsQ, starting at age 50 • Prostate Men aged ≥50 • Digital rectal examination (DRE) and • Offer PSA test and DRE annually, starting at age 50, for years prostate-specific antigen (PSA) testQ men who have life expectancy of at least 10 years • Cervix Women aged • Pap testQ • Cervical cancer screening beginning 3 years after first ≥18 years vaginal intercourse, but no later than age 21 years • Endometrial Women at • At the time of menopause, women at average risk menopause — should be informed about the risks and symptoms of endometrial cancer

SCREENING IMMUNOHISTOCHEMISTRY Screening Immunohistochemistry • • • • •

Epithelial Markers: Cytokeratin (positive in carcinomas)Q Lymphoid Markers: CD-45 (positive in lymphoma)Q Melanocytic Markers: S-100 (positive in melanoma)Q Mesenchymal Markers: Vimentin (positive in sarcoma)Q Neuroendocrine Markers: Chromagranin and neuron specific enolaseQ

TUMOR MARKERS Tumor Markers • • • •

Tumor markers are indicators of cellular, biochemical, molecular, or genetic alterations by which neoplasia can be recognizedQ. These surrogate measures of the biology of the cancer provide insight into the clinical behavior of the tumorQ. This is particularly useful when the cancer is not clinically detectableQ. The information provided may: −− Be diagnostic and distinguish benign from malignant diseaseQ −− Correlate with the amount of tumor present (so-called tumor burdenQ) −− Allow subtype classification to more accurately stage patientsQ −− Be prognostic, either by the presence or absence of the marker or by its concentrationQ −− Guide choice of therapy and predict response to therapyQ

964

Surgery Essence Markers

Associated Cancers

Non-neoplastic Conditions

Hormones • Human chorionic gonadotropin • Calcitonin • Catecholamines

• Trophoblastic tumorsQ, nonseminomatous testicular tumors • Medullary carcinomaQ of thyroid • PheochromocytomaQ

• Alpha-Fetoprotein

• Liver cell cancer, nonseminomatousQ germ cell tumor of testis, lungQ cancer • Adenocarcinoma of the colonQ, pancreasQ, lungQ, breastQ, ovaryQ, prostateQ

• Pregnancy

Oncofetal Antigens

• CEA

• Cirrhosis, hepatitis

Q

• Pancreatitis, hepatitis, inflammatory bowel disease, smoking

Oncology

Isoenzymes • Prostatic acid phosphatase • Neuron-specific enolase • Lactate dehydrogenase

• Prostate cancer • Small cell cancer of lungQ, NeuroblastomaQ • Lymphoma, Ewing sarcoma

• Prostatitis, prostatic hypertrophy

• Hepatitis, hemolytic anemia, many others Specific Proteins

• Immunoglobulins • PSA and prostate specific membrane antigen

• Multiple myeloma and other gammopathies • Prostate cancerQ

• Infection, MGUS

Q

• Prostatitis, prostatic hypertrophyQ

Mucins and Other Glycoproteins • CA-125 • CA-19-9 • CD30 • CD25

• Cancer of ovaryQ, fallopian tube, endometriumQ, cervix, breastQ, lungQ, pancreasQ and colonQ • ColonQ cancer, pancreaticQ cancer • Hodgkin’s diseaseQ, anaplastic large cell lymphoma • Hairy cell leukemia, adult T cell leukemia/ lymphomaQ

• PregnancyQ, endometriosisQ, PIDQ, uterine fibroidsQ • Pancreatitis, Ulcerative colitis

SENTINEL LYMPH NODE BIOPSY Sentinel Lymph Node Biopsy Sentinel LN: First LN which receives lymph directly from tumorQ Cabana demonstrated the concept of SLN first in carcinoma penisQ SLN biopsy in carcinoma penis is known as Cabana procedureQ SLN biopsy is usually done in: CA breastQ, CA penisQ & Malignant melanomaQ SLN biopsy is also applied successfully in cancers of head & neckQ and vulvaQ No special OT is requiredQ Indication of SLN biopsy in breast cancer: Clinically non-palpable axillary LNQ SLN biopsy is usually done intra-operatively by using isosulphan blue dyeQ (1% lymphazurin) or radioactive (Tc-99 labeled sulphurQ) colloid. Accuracy of detection of SLN biopsy is best when both of the methods are combinedQ. • When radioactive colloid is used, the SLN is detected by gamma-cameraQ • Blue dye colors the afferent lymphatics & SLN, hence aids in the identificationQ • Most of the times >1 SLN in carcinoma breastQ

Section 9

• • • • • • • •

Contraindication of SLN Biopsy in CA Breast • Palpable lymphadenopathyQ

• Prior axillary surgery, chemotherapy or radiotherapyQ

• Skin tattooingQ (MC) • NecrosisQ

• Urine discoloration • Anaphylaxis

• Multifocal breast cancerQ

Complications of SLN Biopsy in CA Breast • Intercostobrachial nerve palsyQ (MC injured nerve in SLN biopsy)

Oncology

965

LYMPH NODE METASTASIS

Section 9

Important Lymph Nodes • Interpectoral nodes (CA breast)Q • Retropharyngeal nodes (CA Nasopharynx)Q • Pre-cricoid/Pre-tracheal/Pre-laryngeal lymph nodesQ • Nodes in left axilla (CA stomach)Q • Periumbilical metastatic cutaneous nodules • Left supraclavicular nodeQ • Femoral canal nodeQ • Cystic lymph nodeQ • Jugular fossa lymph nodeQ

Rotter’s nodesQ Rouvier nodesQ Delphian nodesQ Irish nodesQ Sister Mary Joseph nodesQ Virchow nodesQ Cloquet nodeQ LN of LundQ Krouse lymph node

BONE METASTASIS • • • • • •

MC site of primary for bone metastasis: CA Breast > CA prostate > RCC > CA lung > CA thyroid > CA bladder MC cause of osteoblastic secondaries in males: CA ProstateQ MC cause of osteoblastic secondaries in females: CA BreastQ MC tumor metastasize to bone in females: CA BreastQ Lytic expansile metastasis is seen in: RCC follicular carcinoma thyroid MC site of bone metastasis: Dorsal spine (Thoracic vertebraQ)

Bone Metastasis • Metastatic tumors of bone are more common than primary bone tumorsQ. • Tumors usually spread to bone hematogenously, axial skeleton is seeded more than appendicular skeleton partly due to persistence of red marrowQ. • In order of decreasing frequency, the sites most often involved are vertebrae (most common)Q >proximal femur >pelvis >ribs > sternum >proximal humerus >skull. • Extremities distal to elbow and knee are least commonly involved sitesQ, but if distal extremity is involved there is high probability of myelomaQ. • Metastasis to small bones originate from: Lung, kidney or colonQ • Bone is a common site of metastasis for carcinoma of the prostate, breast, lung, kidney, bladder, thyroid, lymphomas and sarcomasQ. • Bateson’s vertebral plexus allow cells to enter the vertebral circulation without first passing through the lungs and is responsible for high rate of prostate cancer metastasis to boneQ. • Bone scan is investigation of choice for bone metastasisQ. • Purely osteolytic lesions are best detected by plain radiography, but they are not apparent until they are >1 cm and have destroyed 30-50% of boneQ. • These are associated with hypercalcemia and with the excretion of hydroxyproline containing peptidesQ. Treatment • Treatment options: Bisphosphonates, corticosteroids, radiotherapy (EBRT) and radionucleotides. • EBRT is given in symptomatic bony metastasisQ. • Samarium-153, is a beta emitter, very effective in relieving pain of bone metastasisQ.

Bone Metastasis Purely osteolytic • Kidney (expansile)Q

Mostly osteolytic • Thyroid • Lung • GI tumors

Mostly osteoblastic • • • • • •

BreastQ Seminoma Carcinoid Neurogenic tumors Uterus and ovary Urinary bladder

Purely osteoblastic • ProstateQ

Oncology

Diagnosis

966

Surgery Essence ONCOLOGICAL EMERGENCY Oncologic Emergencies Structural-obstructive Oncologic Emergencies (Space Occupying Lesion)

Oncology

(PSM HAS Obstruction IN Urine) • • • • • • • • • • •

Pericardial tamponadeQ SVC syndromeQ Malignant biliary obstruction Hemoptysis Airway obstruction Spinal cord compressionQ Intestinal obstruction Increased intracranial pressure Intracerebral leukocytostasis Neoplastic meningitis Urinary obstruction

Metabolic or Hormonal Emergencies

Treatment Related Emergencies

(Paraneoplastic Syndromes)

(HT PNH)

(HAS) • HypercalcemiaQ • Adrenal insufficiency • SIADH

• • • • • • •

Hemolytic-Uremic syndrome Human antibody infusion reactions Tumour lysis syndromeQ TyphlitisQ Pulmonary infiltrate Neutropenia and infection Hemorrhagic cystitis

TUMOR LYSIS SYNDROME Tumor Lysis Syndrome • Caused by destruction of large number of rapidly proliferating neoplastic cellsQ • Frequently, acute renal failure develops as a result of the syndromeQ. • Most frequently associated with the treatment of Burkitt’s lymphoma, ALL and other high grade lymphomasQ, chronic leukemias and rarely with solid tumors. Pathophysiology • Hyperuricemia: Due to destruction of malignant cells and rapid turnover of nucleic acid • Hyperkalemia: Due to release of intracellular K leading to arrhythmia. • Hyperphosphatemia and Hypocalcemia: Due to release of intracellular phosphate, which combines with calcium into bone, calcium phosphate gets deposited in renal tubules causing renal failureQ. • Lactic acidosis: Due to deranged oxidative metabolismQ Characteristic Abnormalities of Tumor Lysis Syndrome • HyperuricemiaQ • HyperkalemiaQ • HyperphosphatemiaQ

• Lactic acidosisQ • HypocalcemiaQ

Treatment • Hydration, NaHCO3, Allopurinol, Rasburicase (recombinant urate oxidase), HemodialysisQ

Section 9

HYPERCALCEMIA OF MALIGNANCY Hypercalcemia of Malignancy • Main factor leading to hypercalcemia is either increased release of calcium form bone or increased calcium reabsorption from DCTQ. • Mostly underlying cause is secretion of PTH-rpQ. Treatment • Mainstay of therapy: Rehydration with a 0.9% saline and diuresis with furosemideQ • Other drugs used to lower serum calcium levels: −− Bisphosphonates (Zoledronic acid is DOC), CalcitoninQ −− Mithramycin (plicamycin), Gallium nitrateQ −− Glucocorticoids (Hydrocortisone) Q

Oncology

967

TYPHLITIS (NEUTROPENIC ENTEROCOLITIS)

• • • •

Also referred to as necrotizing colitis, ileocecal syndrome and cecitisQ Classically seen in neutropenic patients after chemotherapyQ with cytotoxic drugs. More common among childrenQ than among adults More common among patients with acute myelocytic leukemia (AML) or ALLQ

Clinical Features • Clinical syndrome of fever and right-lower-quadrant tenderness in an immunosuppressed hostQ. • Associated diarrhea (often bloody) is common

Section 9

Typhlitis (Neutropenic Colitis)

Diagnosis • Diagnosis can be confirmed by the finding of a thickened cecal wall on CT or USGQ. Treatment • Most cases resolve with medical therapy aloneQ. • Surgical intervention: If there is no improvement by 24 hours after start of antibiotic treatment and in perforationQ

SUPERIOR VENA CAVA SYNDROME Superior Vena Cava (SVC) Syndrome • Clinical manifestation of SVC obstruction, with severe reduction in venous return from head, neck and upper extremities. • MC cause is Lung cancer (small cell and squamous cell carcinoma)Q, alongwith lymphoma and metastatic tumors responsible for more than 90% of all SVC syndrome. • In young adults, malignant lymphoma is the leading cause of SVC syndromeQ. Clinical Features • Patients present with neck and facial swelling (especially around the eyes), dyspnoea, and coughQ. • Other symptoms include hoarseness, tongue swelling, headache, nasal congestion, epistaxis, dysphagia, pain, dizziness, syncope. • Characteristic physical findings are dilated neck veins, increased number of collateral veins covering the anterior chest wall, cyanosis, and edema of the face, arms and chestQ. Diagnosis • Most significant chest radiographic finding is widening of the superior mediastinum (MC right side)Q • CT scan: Investigation of choiceQ. • Potentially life threatening complication of superior mediastinal mass is tracheal obstructionQ. • Diuretics with low salt diet, head elevation and oxygen may produce temporary symptomatic relief. Treatment

Underlying cause

Radiation Therapy

Non-small cell lung cancer, Metastatic solid tumors

Chemotherapy

Small cell carcinoma or lymphoma

Q

Q

SurgeryQ

All other cases

RADIOSENSITIVITY OF TUMORS Most radiosensitive ovarian tumor

• DysgerminomaQ

Most radiosensitive brain tumor

• MedulloblastomaQ

Most radiosensitive testicular tumor

• SeminomaQ

Most radiosensitive lung tumor

• Small cell CAQ

Most radiosensitive kidney tumor

• Wilms tumorQ

Most radiosensitive bone tumor

• Ewing’s SarcomaQ and Multiple myelomaQ

Oncology

Treatment

968

Surgery Essence Radiosensitivity of Tumors

Highly sensitive • • • • •

`

Wilms tumorQ SeminomaQ Myeloma LymphomaQ Ewings sarcomaQ

Moderately sensitive • • • • • •

Small cell lung cancer Breast cancer Basal cell carcinoma Medulloblastoma Teratoma Ovarian cancer

Relatively resistant • • • • • •

SCC of lung RCC Rectal carcinoma CA Bladder Soft tissue sarcoma CA CervixQ

Highly resistant (MOP) • MelanomaQ • OsteosarcomaQ • Pancreatic carcinomaQ

Oncology

In Radiotherapy • • • • • • • • • •

Most radiosensitive tissue of body: Bone marrow Least radiosensitive tissue of body: Nervous tissue / BrainQ Most radiosensitive blood cell: LymphocyteQ (That’s why Lymphocytic predominant Hodgkins lymphoma has best prognosis) Least radiosensitive blood cell: PlateletQ Most common organ to be affected by radiation: SkinQ (Erythema earliest change, layer most commonly affected stratum basalis) Sebaceous gland function does not recover after radiotherapy. Pinna and axillae are common sites of radionecrosis i.e. for skin doses. Most radio resistant organ: Vagina Most common mucosa to be affected by radiation: Intestinal mucosaQ (Earliest symptom is diarrhea) Most sensitive abdominal organ: Kidney Q

IONIZING RADIATIONS Ionizing Radiation Particulate • • • •

Electromagnetic

Electron Proton Neutron Alpha particle (maximum ionization)

• X-rays • Gamma rays (maximum penetration)

SYSTEMIC RADIONUCLIDES Systemic Radionuclides • Systemic radionuclides are non-sealed radionuclides which are administered orally, intravenously or intracavitary. • Before administering it, pregnancy should be ruled out • Breastfeeding should be discontinued for 1–2 weeks.

Section 9

Types

T½ (Days)

Decay Particles

Use

Sodium iodide (I131) Q

8Q

Gamma, betaQ

Hyperthyroidism (diffuse toxic goiter, toxic multinodular goiter, or solitary toxic thyroid nodule), thyroid carcinomaQ

Sodium phosphate (P32) Colloidal chromic Phosphate

14.3

BetaQ

Myeloproliferative disorders (Polycythemia and thrombocytosisQ) Intra-cavitary therapy of malignant ascites, malignant pleural effusion and brain cystQ

Samarium-153 (Sm) chloride

1.9

BetaQ

Painful bone metastasesQ

Strontium-89 (Sr) chloride

50.5

Beta Never gamma

Painful bone metastasesQ

Rhenium (Re)

3.8

Beta and Gamma

Painful bone metastasesQ

Q

Oncology

969

RADIOTHERAPY

• • • • • • •

X-rays and gamma rays are the most common radiations used to treat cancers. X-rays are generated by linear accelerators Gamma rays are generated from decay of atomic nuclei in radio-isotopes like cobalt. Cobalt-60 is a synthetic radioactive isotope of cobalt with a half-life of 5.27 years Cobalt-60 is used only in teletherapy Radiation energy is absorbed by tissue causing ionization or excitationQ, which are responsible for various biological effects. Susceptibility of various phases of cell cycle to radiation: G2MQ >G2 >M >G1 >Early S >Late S PhaseQ.

Phase of Cell Cycle

Section 9

Radiotherapy

Comment

G2M >G2

• Most sensitiveQ to radiation

End of S phase

• Most resistantQ to radiation

G1

• Radiation exposure leads to chromosomal aberration

G2

• Radiation exposure leads to chromatid aberration Techniques to Reduce the uncertainty due to Respiratory Motion in Radiotherapy

Respiratory gating

• Patient nose is clamped • Patient breathes through an ABC apparatus, which simultaneously control radiation dose.

Deep inspiration breath holding technique • Patient is asked to hold the breath in deep inspiration. • Breath hold minimizes tumor motion • It expands the lung to its maximum volume putting the healthy lung tissue out of the radiation field

Radiotherapy Teletherapy

Brachytherapy

Systemic Therapy

• Beams of radiation generated at a distance and aimed at the tumor within the patient • Most commonly used form of radiation therapy

• Encapsulated sources of radiation implanted directly into or adjacent to tumor tissues

• Radionuclides targeted in some fashion to a site of tumor

BRACHYTHERAPY Brachytherapy • Radiation therapy with encapsulated source of radiation implanted directly into or adjacent to tumor tissue. • It is delivered in two ways (1) Intracavitary implantsQ (2) Interstitial implants • Interstitial implantation is of two types: 1. Permanent implants (PGI)Q: Pd-103Q, Gold (Au)-198Q, I-125Q 2. Temporary implants (ICT)Q: Ir-192Q, Cs-137Q (Temporary) • Normal tissues are spared from radiation injuryQ.

Oncology

• Patient respiration during radiotherapy can cause significant motion of the tumor volume, which can be mitigated by gating the accelerator beam to the patient respiration.Q • Respiratory gating is one of the latest techniques in radiation therapy and involves matching radiation treatment to a patient own respiratory pattern.Q • This approach decreases possible complications and side effects, while using higher doses and getting better outcomes.

Active breathing control

970

Surgery Essence Interstitial implants Permanent Interstitial Implants (PGI) • Performed when the tumor to be treated is inaccessible making the removal of radioisotope impossible or impractical. • These implants have usually short half lives. • Isotopes used (PGI): −− Palladium (Pd) 103Q −− Gold (Au) 198Q −− Iodine 125Q • Used in deep seated lesion in pelvis, abdomen, lung, colorectum

Temporary Interstitial Implants (ICT) • Temporary removable implants are used in anatomic areas where there is no body cavity or orifice to accept radioactive sources. • Isotopes used (ICT): −− Iridium 192 (MC)Q −− Cesium 137Q (ICT: Iridium cesium temporary) • Used in breast and chest wall irradiation, anterior, lateral and posterior wall of vagina

Oncology

INTENSITY MODULATED RADIOTHERAPY Intensity Modulated Radiation Therapy (IMRT) • The radiation dose is designed to conform to the three dimensional (3-D) shape of the tumour by modulation or controlling the intensity of the radiation beams to focus a higher radiation dose to the tumour while minimizing radiation exposure to surrounding normal tissuesQ. • Prostate cancerQ • Pancreatic tumors • Head and neck cancers

Indications of IMRT • Primary and metastatic brain tumors • Liver tumors (HCC and metastasis)

RADIOSENSITIZER AND RADIATION PROTECTOR Radiotherapy

• • • • • • • •

Radiosensitizer OxygenQ (most effective Radiosensitizer) MetronidazoleQ, misonidazole, tinidazole 5-FUQ (non-hypoxic cell sensitizer) HydroxyureaQ (non-hypoxic cell sensitizer) BUDR and IUDRQ (non-hypoxic cell sensitizer) CisplatinQ, paclitaxel, gemcitabine MitomycinQ, topotecan, vinorelbine Dactinomycin (Actinomycin D)Q

• Hypoxic cells are resistant to radiotherapyQ.

Radiation Protectors • Amifostine • IL-1 • GM-CSF

•  Augmentation of oxygen is the basis of radiosensitizationQ.

CHEMOTHERAPY

Section 9

Chemotherapy

• • • •

Highly Chemosensitive Tumors Hodgkin’s lymphoma • Teratoma (testis) Wilm’s tumor • Embryonal Rhabdomyosarcoma Ewing’s sarcoma • Choriocarcinoma ALL

• • • •

Chemoresistant Tumors Melanoma SCC of lung HCC Thyroid carcinoma

Chemotherapy in Cancers of Oral Cavity, Head and Neck • Adjuvant chemotherapy has been reported to improve the rate of organ preservation with no change in overall survivalQ. • Chemotherapy is often employed in palliative setting in patients with recurrent, unresectable or distant metastasesQ. • Drugs used: CisplatinQ, Methotrexate, 5-FU, Docetaxel and Paclitaxel • Cisplatin is the cornerstone drug in the modern management of head and neck cancerQ. • Most beneficial is concurrent chemotherapyQ. • The addition of concurrent chemotherapy (cisplatin) to conventional radiation significantly improved survival over radiation aloneQ. • Concurrent chemoradiation protocols have improved locoregional control and reduce the development of distant diseaseQ.

Multiple Choice Questions TUMOR MARKERS

1. The following is a marker of Paget’s disease of the mammary gland: (All India 2007) a. S-100 b. HMB-45 c. CEA d. Neuron specific enolase



2. In which of the following tumors alpha-feto protein is elevated? (AIIMS Nov 2005) a. Choriocarcinoma b. Neuroblastoma c. Hepatocellular carcinoma d. Seminoma











3. Which of the following tumor secretes erythropoietin?  (PGI June 2010) a. Pheochromocytoma b. Hepatoma c. RCC d. Adrenal adenoma e. Breast cancer



13. All are recognized tumor markers except: (PGI Dec 99) a. Beta hCG b. Beta-2 microglobulin c. Alpha fetoprotein d. Acid phosphatase



14. CEA is increased in all except:  (AIIMS May 2007) a. Lung cancer b. Breast cancer c. Colon cancer d. Osteogenic sarcoma



15. Regarding CEA-false is: (AIIMS 97) a. Prognostic indicator b. Glycoprotein c. Elevated in colorectal carcinoma d. Elevated only when there is hepatic metastasis

4. Uses of tumor marker are: (PGI June 2001) a. Screening of a cancer b. Follow-up a cancer patient, especially for knowing about recurrence c. Confirmation of a diagnosed cancer d. For monitoring the treatment of a cancer 5. Erythropoietin secreting tumor: a. Cerebellar hemangioblastoma b. Hepatoma c. Renal cell carcinoma d. Adrenal adenoma e. Fibromyoma of uterus

(PGI June 2001)

6. CA-125 is associated with:  a. Colon carcinoma c. Ovarian carcinoma e. Pancreatic carcinoma

(PGI June 2002) b. Breast carcinoma d. Bronchogenic carcinoma

7. CA-125 is associated with:  a. Pregnancy c. TB e. Endometriosis

(PGI Dec 2007) b. Breast carcinoma d. Endometrial carcinoma



8. CEA is associated with: (PGI June 2002) a. Adenocarcinoma of colon b. Pancreatic carcinoma c. Neuroblastoma d. Ovarian carcinoma e. Prostatic carcinoma



9. AFP is raised in: a. CA Prostate c. CA Lung e. CA Colon

b. HCC d. CA Breast

(PGI Dec 2003)

10. Which one of the following is frequent cause of serum alpha-fetoprotein level greater than 10 times the normal upper limit? (UPSC 2004) a. Seminoma b. Metastatic carcinoma of liver c. Cirrhosis of liver d. Oat cell tumor of lung

12. α-fetoprotein increase in all of the following except:  (All India 94) a. Hepatocellular carcinoma b. Seminoma of the testes c. GI neoplasms d. Embryonal cell carcinoma

11. Which of the following is marker for carcinoma? a. Cytokeratin b. Vimentin(All India 2012) c. Calcitonin d. CD-45

SCREENING IN MALIGNANCY 16. In which of the following diseases, the overall survival is increased by screening procedure? (All India 2005) a. Prostate cancer b. Lung cancer c. Colon cancer d. Ovarian cancer 17. Screening increase life span in which of the following carcinoma? (PGI June 2007) a. Breast b. Colon c. Prostate d. Lung (PGI Nov 2011) b. Carcinoma Breast d. Carcinoma Cervix



18. Screening is useful for: a. Carcinoma Lung c. Carcinoma Skin e. Carcinoma Ovary



19. Least amenable to screening is:  a. Breast b. Cervix c. Oral cavity d. Lung

(AIIMS June 94)

LYMPH NODE METASTASIS 20. Which one of the following is the most common tumor to produce metastasis to cervical lymph nodes? a. Glottic carcinoma (UPSC 2008, AIIMS June 2002) b. Nasopharyngeal carcinoma c. Carcinoma base of tongue d. Carcinoma lip

21. In which of the following head and neck cancers, is lymph node metastasis least common? (AIIMS May 2008) a. Tongue b. Buccal mucosa c. Hard palate d. Lower alveolus



22. Delphian nodes are:  a. Pretracheal c. Supraclavicular

(COMEDK 2008) b. Paratracheal d. Posterior triangle

972

Surgery Essence 23. A 55-year-old chronic smoker presents with complaints of hoarseness of voice, and single enlarged painless lymph node in left supraclavicular region. Next step to be done:  (AIIMS Nov 2000) a. CT Scan of chest b. Sputum examination for AFB c. Laryngoscopy and chest X-ray d. Excision biopsy of the node

Oncology





24. A patient comes with stony hard, painless lymph node in left supraclavicular fossa. A biopsy report states squamous cell carcinoma. What is the diagnosis?   (AIIMS Nov 99) a. Stomach carcinoma b. Breast carcinoma c. Lung carcinoma d. Pancreatic carcinoma 25. A 65-year-old smoker presents with hoarseness, hemoptysis and a hard painless lump in the left supraclavicular fossa. Which of the following is the most appropriate diagnostic step? (AIIMS June 2004) a. Undertake an open biopsy of the neck lump b. Undertake a radical neck dissection c. Do fine needle aspiration cytology d. Give a trial of Anti tuberculous therapy

26. Which carcinoma most commonly metastasizes to cervical lymph nodes? (AIIMS June 93) a. Maxillary sinus b. Posterior tongue c. Cheek d. Hard palate 27. Lymph node metastasis is a common feature with the following variant of soft tissue sarcoma: (All India 97) a. Fibrosarcoma b. Angiosarcoma c. Liposarcoma d. Neurofibrosarcoma



36. All of the following produce osteoblastic secondaries except:  (DNB 2012, All India 94) a. CA Prostate b. Carcinoid tumors c. CA Breast d. Multiple myeloma

37. Which of the following is rare site for metastasis? (SGPGI 2004) a. Vertebrae b. Skull c. Pelvis d. Forearm and leg bones



38. Pulsating tumors include all except: a. Bone sarcoma b. Osteoclastoma c. Secondaries from hypernephromas d. Secondary from prostate



39. Most common primary of metastatic bone tumor in a male is: (DNB 2009) a. Lung b. Liver c. Bone d. Brain

28. Most common cause of skeletal metastasis is: (UPPG 2009) a. Kidney b. Prostate c. Breast d. Thyroid



29. Treatment of bony metastasis is by: (JIPMER 2011) a. Samarium-153 b. I-131 with tositumumab c. P-32 d. Yttrium

Section 9



30. Best investigation for bone metastasis is:  (All India 2012, 2011) a. MRI b. CT c. Bone scan d. X-ray



31. Bony metastasis is common with all of the following except:  (All India 98) a. CA breast b. CA lung c. CA testis d. CA prostate



32. Not true about bone metastasis:  (AIIMS June 98) a. Uncommon distal to elbow and knee b. Breast secondary may be osteoblastic c. Renal cell carcinoma secondary are expansile d. Soft tissue sarcoma causes bony metastasis



33. Secondaries of all following cause osteolytic lesions except: a. Prostate b. Kidney  (All India 95) c. Bronchus d. Thyroid

34. Expansile lytic osseous metastases are characteristic of primary malignancy of: a. Kidney b. Bronchus c. Breast d. Prostate

(PGI 88)

ONCOLOGICAL EMERGENCIES

40. Features of tumor lysis syndrome:  (PGI May 2011, PGI Dec 2006) a. Hyperuricemia b. Hypercalcemia c. Hypophosphatemia d. Hyperphosphatemia e. Hyperkalemia



41. Tumor lysis syndrome is associated with all of the following laboratory feature except: (DNB 2012, AIIMS Nov 2003) a. Hyperkalemia b. Hypercalcemia c. Hyperuricemia d. Hyperphosphatemia



42. Tumor lysis syndrome is characterized by all except: (AIIMS November 2017) a. Hyperuricemia b. Hypercalcemia c. Hyperkalemia d. Hyperphosphatemia



43. Hypercalcemia associated with malignancy is most often mediated by: (All India 2005) a. PTH b. PTH-rp c. IL-6 d. Calcitonin



44. Hypercalcemia of malignancy treatment consist of all except:  (PGI May 2011) a. Dexamethasone b. Saline infusion c. Pamidronate d. Furosemide e. Phosphate

BONE METASTASIS

35. A malignant tumor of childhood, that metastasizes to bones most often is: (All India 2006) a. Wilm’s tumor b. Neuroblastoma c. Adrenal gland tumors d. Granulosa cell tumor of ovary

45. A patient with leukemia on chemotherapy develops acute lower abdominal pain associated with anemia, thrombocytopenia and leucopenia. Which of the following is clinical diagnosis? (All India 2006) a. Appendicitis b. Leukemic colitis c. Perforation peritonitis d. Neutropenic colitis

46. Which of the following tumor is most commonly associated with superior vena cava syndrome?  (Recent Question 2014, WBPG 2012, All India 2011) a. Lymphoma b. Small cell carcinoma c. Non small cell carcinoma d. Metastasis

Oncology 47. A 53 years old patient was admitted complains of dyspnea. On examination he has puffy face with engorged veins over the chest. SVC obstruction is suspected. Chest X-ray shows mediastinal enlargement. What is the next step? (AIIMS November 2017) a. Total blood count with peripheral smear b. CT thorax c. Start cyclophosphamide d. Urgent referral to RT



48. Which of the following is not an oncological emergency? a. Spinal cord compression (AIIMS June 2003) b. Superior vena cava syndrome c. Tumor lysis syndrome d. CA cervix stage IIIb with pyometra



a. b. c. d.

LYMPHOMA 49. The commonest site of lymphoma in the gastrointestinal system is: (COMEDK 2007) a. Small bowel b. Stomach c. Large intestine d. Esophagus

50. In neuroblastoma the most common presentation is: a. Lytic lesion in skull with suture diastasis (All India 98) b. Lung metastasis c. Renal invasion d. Secondaries in brain



51. Commonest tumor of lumbar region in children is:  (AIIMS June 98) a. Dermoid cyst b. Neuroblastoma c. Wilm’s tumour d. Appendix

SENTINEL LYMPH NODE BIOPSY

53. The given image shows methylene blue being injected in the peritumoral region. Which of the given procedure is being performed?  (AIIMS May 2018)

Brachytherapy USG Sentinel lymph node biopsy Lateral pectoral nerve block



55. True about sentinel lymph node biopsy: (PGI June 2004) a. Special OT is required b. Blue dyes injected c. Contraindicated if axillary LN is involved biopsy d. It is done to avoid inadvertent axillary LN biopsy e. Radioactive dye is used



56. Sentinel lymph node biopsy is most useful for: (AIIMS November 2018) a. Carcinoma cervix b. Carcinoma endometrium c. Carcinoma vulva d. Carcinoma vagina



57. Sentinel lymph node biopsy is done in all except: a. CA breast b. CA penis (DNB 2012) c. Malignant melanoma d. CA colon

GI MALIGNANCY

58. Upper GI endoscopy and biopsy from lower esophagus in a 48-year-old lady with chromic heart burn shows presence of columnar epithelium with goblet cells. The feature is most likely consistent with: (AIIMS June 2003) a. Dysplasia b. Hyperplasia c. Carcinoma in-situ d. Metaplasia

59. By mucosal resection which carcinoma can be diagnosed early? (AIIMS June 98) a. Esophageal carcinoma b. Anal carcinoma c. Colon carcinoma d. Pancreatic carcinoma

60. In which case immunoguided surgery is done?  (AIIMS June 98) a. CA colon b. CA pancreas c. CA jejunum d. CA anal canal

SPONTANEOUS REGRESSION 61. In which case spontaneous regression is not seen?  (AIIMS Sept 96, All India 98) a. Malignant melanoma b. Osteosarcoma c. Neuroblastoma d. Choriocarcinoma a. b. c. d.

Sentinel lymph node biopsy Tumor painting Breast tattooing Peritumor marking with dye

62. Spontaneously regressing tumors are: (PGI June 2006) a. Malignant melanoma b. Neuroblastoma c. Ewing’s sarcoma d. Wilm’s tumour

63. Spontaneous regression of malignant tumor is feature of:  (AIIMS June 93) a. Neuroblastoma b. Renal cell carcinoma c. Burkitt’s lymphoma d. Wilm’s tumor

Oncology

52. Sentinel lymph node biopsy is an important part of the management of which of the following conditions?  (All India 2002) a. Carcinoma prostate b. Carcinoma breast c. Carcinoma lung d. Carcinoma nasopharynx

54. Which of the following technique has been depicted in the image?  (AIIMS November 2018)

Section 9



973

974

Surgery Essence

64. Tumor known to regress is: a. Neuroblastoma c. Adenocarcinoma

(PGI Dec 97) b. Retinoblastoma d. CA breast

Oncology

RADIOTHERAPY

65. Most radio resistant phase in cell cycle:  (Recent Question 2014, JIPMER 2011) a. G1 b. Early S c. Late S d. G2



66. All of the following are pure beta emitters except:  (AIIMS May 2011) a. Yttrium-90 b. Phosphorus-32 c. Strontium-90 d. Samarium-153

67. All of the following radioisotopes are used as systemic radionuclide, except: (All India 2006) a. Phosphorus-32 b. Strontioum-89 c. Iridium-192 d. Samarium

68. Phosphorus-32 emits:  a. Beta particles c. Neutrons

(All India 2006) b. Alpha particles d. X- rays

69. Which one of the following has the maximum ionization potential? (All India 2006) a. Electron b. Proton c. Helium ion d. Gamma photon

70. Which of the following malignant tumors is radioresistant?  (All India 2006; AIIMS 2007) a. Ewing’s sarcoma b. Retinoblastoma c. Osteosarcoma d. Neuroblastoma

71. The most radiosensitive tumor among the following is:  (Recent Question 2014, All India 2006) a. Bronchogenic carcinoma b. Carcinoma parotid c. Dysgerminoma d. Osteogenic sarcoma



78. Amifostine protects all of the following except:  (All India 2009) a. CNS b. Salivary glands c. Kidneys d. GIT



79. Which of the following is the most radiosensitive tumor? a. Ewing’s sarcoma (Recent Question 2014, AIIMS Nov 2005) b. Hodgkin’s disease c. Carcinoma cervix d. Malignant fibrous histiocytoma



80. Which of the following radioactive isotopes in not used in brachytherapy? (AIIMS 2005) a. Iodine-125 b. Iodine-131 c. Cobalt-60 d. Iridium-192



81. High energy linear accelerators use:  a. X-rays b. Gamma-rays c. Alpha-rays d. Infrared-rays e. β-rays



82. Ionizing radiation cause maximum damage in: (PGI 2005) a. Hypoxic cells b. Cells in S phase c. Cells in G2M phase d. Dividing cells e. Neurons



83. Which radionuclide is commonly used in teletherapy units?  (Orissa 2011) a. Radium-226 b. Cobalt-60 c. Caesium-137 d. Iridium-192



84. The half life of radioactive Cobalt-60 is: a. 2.26 years b. 3.26 years c. 5.26 years d. 7.26 years



85. Amifostine is: a. Radiosensitiser c. Radiomodifier



86. Which of the following is radioprotective agent? a. Cisplatin (Recent Question 2016, All India 2012) b. Amifostine c. Methotrexate d. Colony stimulating factor

72. Which of the following imaging techniques gives maximum radiation exposure to the patient? (All India 2006) a. Chest X-ray b. MRI c. CT scan e. Bone scan

73. The technique employed in radiotherapy to counteract the effect of tumor motion due to breathing is known as: a. Arc technique b. Modulation (All India 2005) c. Gating d. Shunting

Section 9

74. Which one of the following radioisotope is not used as permanent implant?  (All India 2005) a. Iodine-125 b. Palladium-103 c. Gold-198 d. Cesium-137

75. Which of the following elements is obsolete in radiotherapy?  (AIIMS 2009) a. Cesium-137 b. Cobalt-60 c. Radium-226 d. Iridium-192



76. Craniospinal irradiation is employed in the treatment of: a. Oligodendroglioma (KGMC 2011, PGI 2009) b. Pilocytic astrocytoma c. Mixed oligoastrocytoma d. Medulloblastoma e. Glioblastoma



77. Which of the following is the most radiosensitive phase of the cell cycle? (MHCET 2016, AIIMS Nov 2012,  All India 2008, PGI 2009, 2008) a. G2M b. G2 c. S d. G1

(PGI 2006)

(Orissa 2011)

(AIIMS May 2012) b. Radioprotector d. Radiomimetic

87. For which malignancy, intensity modulated radiotherapy (IMRT) is the most suitable? (AIIMS Nov 2005) a. Lung b. Prostate c. Leukemia d. Stomach

MISCELLANEOUS 88. Which of the following malignant disease of children has the best prognosis? (AIIMS Nov 2003) a. Wilm’s tumor b. Neuroblastoma c. Rhabdomyosarcoma d. Primitive neuroectodermal tumor

89. Neoadjuvant chemotherapy is not used in: (AIIMS Feb 97) a. CA thyroid b. CA breast c. CA esophagus d. CA lung



90. Which of the following is the most beneficial technique of using chemotherapy with a course of radiotherapy in head and neck malignancies? (AIIMS Nov 2004) a. Neo adjuvant chemotherapy b. Adjuvant chemotherapy c. Concurrent chemotherapy d. Alternating chemotherapy and radiotherapy

Oncology

91. What is the name of the given instrument?

b. Chemoport d. Pacemaker



92. Drug of choice for chemotherapy induced vomiting: (Recent Question 2019) a. Granisetron b. Prazosin c. Clonidine d. Dimenhydrinate



93. Migratory thrombophlebitis is associated with the following malignancies except: (AIIMS Nov 2004) a. Lung cancer b. Prostate cancer c. Pancreas cancer d. Gastro-intestinal cancer

94. Small deposits of neuroendocrine cell hyperplasia in scarred lungs are known as: (JIPMER 2014) a. Teratoma b. Tumor let c. Carcinoid d. Hamartoma 95. The most common malignant tumor of adult males in India is: (All India 2004) a. Oropharyngeal carcinoma b. Gastric carcinoma c. Colo-rectal carcinoma d. Lung cancer



96. Most common cancer in India: (All India 97) a. CA cervix b. CA breast c. CA lung d. CA oral cavity



97. Trousseau’s sign is seen in all the following except: a. CA lung b. CA Stomach (All India 94) c. CA Pancreas d. Liposarcoma



98. Pseudolymphoma is seen in:  a. Sjogren’s syndrome b. SLE c. Mixed connective tissue disease d. Behect’s syndrome



99. Pancoast tumor is seen with cancer of: (PGI Dec 99) a. Apical lobe of lung b. Lingual lobe c. Thyroid d. Pyriform fossa

(PGI Dec 97)

100. Erythema chronicum migrans is seen in: (PGI June 99) a. Lyme’s disease b. Glucagonoma c. Gastrinoma d. Pheochromocytoma 101. All of the following are examples of round cell tumor, except: (AIIMS Nov 2005) a. Neuroblastoma b. Ewing Sarcoma c. Non hodgkin’s lymphoma d. Osteosarcoma 102. RET proto-oncogene mutation is a hallmark of which of the following tumors? a. Medullary carcinoma thyroid b. Astrocytoma c. Paraganglioma d. Hurthle cell tumor thyroid

107. Glomus tumor is seen in:  (Recent Question 2014, AIIMS Nov 2008) a. Liver b. Adrenals c. Pituitary d. Finger 108. BRCA-1 gene is located on:  (Recent Question 2014, AIIMS May 2011) a. Chromosome 13 b. Chromosome 11 c. Chromosome 17 d. Chromosome 22 109. A 24-year-old man presented with retroperitoneal, necrotic, heterogenous enhancing mass on CT near the hilum of the left kidney. What is the most probable diagnosis? a. Metastatic germ cell tumor (AIIMS Nov 2010) b. Metastatic melanoma c. Lymphoma d. Metastatic transitional cell tumor 110. Octreotide is used in all except: (AIIMS May 2011) a. Insulinoma b. Glucagonoma c. Glioma d. Carcinoids 111. Feature(s) of hamartoma is/are:  (PGI Nov 2011) a. Benign b. Malignant c. Malformation d. Mostly Asymptomatic 112. Most common site of carcinoma in India: (MHPGMCET 2001) a. Lung b. Oral cavity c. Breast d. Uterus 113. National Cancer Control Programme (NCCP) launched in India in the year: (Orissa 2011) a. 1975 b. 1982 c. 1985 d. 1992 114. Acanthosis nigricans is seen in: (DNB 2009) a. GI malignancy b. Lung cancer c. Breast cancer d. All of the above 115. Smoking is a risk factor for all cancer except: (DNB 2007) a. Esophagus b. Urinary bladder c. Pancreas d. Gallbladder 116. Most common Non-Hodgkin’s lymphoma of orbit: a. B cell b. T cell (AIIMS May 2013) c. NK cell d. Plasma cell 117. Most common malignant chest wall tumor: (Recent Question 2017) a. Chondrosarcoma b. Osteosarcoma c. Synovial sarcoma d. Rhabdomyosarcoma 118. Reddish firm swelling which bleeds on touch in a 20 years old female is: (Recent Question 2018) a. Hemangioma’ b. Fibroadenoma c. Pyogenic granuloma d. Lipoma

Oncology



Section 9

a. Gamma camera c. IVC filter

103. All are correctly matched except:  (PGI June 2006) a. BRCA-1: Lung b. BCL-2: Apoptosis c. Chromosome 16: Philadelphia chromosome d. APC gene: Colon 104. Adjuvant chemotherapy is of definite value in:  (AIIMS Nov 2006) a. CA colon b. CA pancreas c. CA gallbladder d. CA esophagus 105. Which of the following tumors are surgically curative? a. Pheochromocytoma b. Insulinoma c. Glucagonoma d. Appendicular carcinoid e. All 106. Which is not having underlying malignancy? (APPG 2008) a. Paget’s disease of bone b. Paget’s disease of nipple c. Paget’s disease of vulva d. Paget’s disease of anal region

975

Explanations TUMOR MARKERS

1. Ans. c. CEA (Ref: Harrison 20/e p532, 19/e p473; Schwartz 10/e p301-302; Sabiston 20/e p698-702)

Paget’s disease of nipple is differentiated by superficial spreading melanoma by CEA positivityQ.

2. Ans. c. Hepatocellular carcinoma



3. Ans. b. Hepatoma, c. RCC (Ref: Harrison 20/e p666, 19/e p400) Erythropoietin Secreting Tumors • Renal cell carcinoma • Hepatocellular carcinomaQ

• Cerebellar hemangioblastomaQ • Massive uterine leiomyomaQ

Q



4. Ans. a. Screening of a cancer, b. Follow up a cancer patient, especially for knowing about recurrence, d. For monitoring the treatment of a cancer (Ref: Harrison 20/e p447, 19/e p473; Sabiston 20/e p697-702)



5. Ans. a. Cerebellar hemangioblastoma, b. Hepatoma, c. Renal cell carcinoma, e. Fibromyoma of uterus



6. Ans. a. Colon carcinoma, b. Breast carcinoma, c. Ovarian carcinoma, d. Bronchogenic carcinoma, e. Pancreatic carcinoma



7. Ans. a. Pregnancy, b. Breast carcinoma, d. Endometrial carcinoma, e. Endometriosis



8. Ans. a. Adenocarcinoma of colon, b. Pancreatic carcinoma, d. Ovarian carcinoma, e. Prostatic carcinoma



9. Ans. b. HCC, c. CA Lung



11. Ans. a. Cytokeratin (Ref: Robbins 9/e p334)



12. Ans. b. Seminoma of the testes

13. Ans. None



14. Ans. d. Osteogenic sarcoma

15. Ans. d. Elevated only when there is hepatic metastasis

10. Ans. b. Metastatic carcinoma of liver

SCREENING IN MALIGNANCY

16. Ans. c. Colon cancer (Ref: Harrison 20/e p449, 19/e p481; Schwartz 10/e p298, 9/e p252; Bailey 27/e p146-147)

Schwartz says “Because the majority of colorectal cancers are thought to arise from adenomatous polyps, preventive measures focus upon identification and removal of these premalignant lesions. In addition, many cancers are asymptomatic and screening may detect these tumors at an early and curable stage.

17. Ans. b Colon



19. Ans. d. Lung

18. Ans. b Carcinoma Breast, d. Carcinoma Cervix

LYMPH NODE METASTASIS

20. Ans. b. Nasopharyngeal carcinoma (Ref: Sabiston 20/e p803-805; Schwartz 10/e 580, 593-594; Bailey 27/e p733-735; Devita 9/e p764-766)

Nasopharyngeal Carcinoma • MC tumor to produce cervical LN metastasisQ • MC tumor responsible for secondaries in the neck with no obvious primary malignancyQ

21. Ans. c. Hard palate (Ref: Bailey 27/e p764-765; Devita 9/e p750; Cancer of the Head and Neck by Suen and Myer 4/e p288-289) • LN metastasis is most common in: CA tongueQ >Floor of mouth >Lower alveolus >Buccal mucosa >Upper alveolus >Hard palate >LipQ.



22. Ans. a. Pretracheal (Ref: Bailey 27/e p800, 801)

Oncology



23. Ans. d. Excision biopsy of the node (Ref: Harrison 20/e p412, 19/e p409)



• Harrison says “In cases of lymphadenopathy, if the patient history and physical findings are suggestive of malignancy, then a prompt lymph node biopsy should be done. FNAC is not of much use, as it does not provide enough tissue to reach a diagnosis.”

24. Ans. c. Lung carcinoma (Ref: Harrison 20/e p538, 18/e p738; Sabiston 20/e p1583; Schwartz 10/e 623-645; Devita 9/e p799-812)

• SCC is a variant of lung cancer, rest three options are most commonly adenocarcinoma.

25. Ans. a. Undertake an open biopsy of the neck lump



27. Ans. b. Angiosarcoma (Ref: Harrison 20/e p654, 19/e p119e-2; Sabiston 20/e p766)

26. Ans. b. Posterior tongue

Section 9



977

BONE METASTASIS

28. Ans. c. Breast (Ref: Harrison 20/e p471-472, 19/e p119e-4; Devita 9/e p2512-2513; CSDT 12/e p1202; Apley 9/e p216)



29. Ans. a. Samarium-153

30. Ans. c. Bone scan (Ref: Sutton 7/e p1251)



31. Ans. c. CA testis

32. Ans. d. Soft tissue sarcoma causes bony metastasis



33. Ans. a. Prostate

34. Ans. a. Kidney

Pulsating Secondaries • Follicular carcinoma thyroidQ



•  RCCQ



35. Ans. b. Neuroblastoma (Ref: Schwartz 10/e p678,1639-1640; Sabiston 20/e p1887-1888; Bailey 27/e p847; Harrison 20/e p454, 19/e p618)

Neuroblastoma • • • • • •

Metastasis is present in 60–70% of patients at the time of diagnosisQ Common sites of metastasis: Long bones (MC)Q, Liver, Lymph nodes and Skin Lung metastasis is rare in neuroblastomaQ Neuroblastoma is the MC extracranial solid tumor in childhoodQ Neuroblastoma is the 2nd MC solid malignancy of childhood after brain tumorsQ MC intra abdominal solid tumor in childhood: NeuroblastomaQ

36. Ans. d. Multiple myeloma



38. Ans. d. Secondary from prostate



39. Ans. a. Lung (Ref: Bailey 24/e p1330)

37. Ans. d. Forearm and leg bones

• “Prostate, breast and lung primaries account for 80% of all bone metastasis.”

ONCOLOGICAL EMERGENCIES

40. Ans. a. Hyperuricemia, d. Hyperphosphatemia, e. Hyperkalemia

41. Ans. b. Hypercalcemia



42. Ans. b. Hypercalcemia (Ref: Harrison 20/e p519, 19/e p1795; Sabiston 20/e p90; Schwartz 10/e p81) “ Tumor lysis syndrome (TLS) is characterized by hyperuricemia, hyperkalemia, hyperphosphatemia, and hypocalcemia and is caused by the destruction of a large number of rapidly proliferating neoplastic cells. Acidosis may also develop. Acute renal failure occurs frequently.”-Harrison 20/e p519



43. Ans. b. PTH-rp (Ref: Harrison 20/e p663, 19/e p313, 609, 717, 2476, 2477)



45. Ans. d. Neutropenic colitis (Ref: Harrison 20/e p521, 19/e p488; Schwartz 10/e 1236, 1241; Bailey 27/e p1307)



46. Ans. b. Small cell carcinoma (Ref:Harrison 20/e p511, 19/e p1787)



47. Ans. b. CT thorax (Ref: Harrison 20/e p511, 19/e p1787-1788)

44. Ans. None

“ Computed tomography (CT) provides the most reliable view of the mediastinal anatomy. The diagnosis of SVCS requires diminished or absent opacification of central venous structures with prominent collateral venous circulation. Magnetic resonance imaging (MRI) has no advantages over CT.”- Harrison 20/e p511, 19/e p1788

48. Ans. d. CA cervix stage IIIb with pyometra (Ref: Harrison 20/e p511, 19/e p1787)

Oncology



978

Surgery Essence LYMPHOMA

49. Ans. b. Stomach (Ref: Sabiston 20/e p1278, 19/e p1218-1219; Schwartz 10/e 1259; Bailey 27/e p1140-1141)

GI Lymphoma • MC site for lymphomaQ in the GIT: Stomach >Ileum • MC site of gastric lymphoma: AntrumQ • MC type of gastric lymphoma: Diffuse large B-cell lymphomaQ (55%) > MALToma (40%) • DLBL is MC type of NHL, extranodal lymphoma and GI lymphoma.



Oncology



50. Ans. a. Lytic lesion in skull with suture diastasis

51. Ans. b. Neuroblastoma

SENTINEL LYMPH NODE BIOPSY

52. Ans. b. Carcinoma breast (Ref: Harrison 20/e p559, 19/e p497, 103e-3; Schwartz 10/e 305-306, 545-547; Sabiston 20/e p849-851)



53. Ans. a. Sentinel lymph node biopsy (Ref: Harrison 20/e p559; Sabiston 20/e p849-851; Schwartz 10/e p305-306)



54. Ans. c. Sentinel lymph node biopsy (Ref: Harrison 20/e p559; Sabiston 20/e p849-851; Schwartz 10/e p305-306)



55. Ans. b. Blue dyes injected



56. Ans. c. Carcinoma vulva (Ref: Harrison 20/e p559; Sabiston 20/e p849-851; Schwartz 10/e p306-306)

57. Ans. d. CA colon

GI MALIGNANCY

58. Ans. d. Metaplasia (Ref: Sabiston 20/e p1050; Schwartz 10/e 1017-1018; Bailey 27/e p1081)

Barrett’s Esophagus • Metaplasia of esophageal squamous epithelium into columnar in distalQ esophagus • It is consequence of severe reflux esophagitisQ • MC type of columnar epithelium is intestinal epithelium (Intestinal metaplasiaQ)

59. Ans. a. Esophageal carcinoma (Ref: Schwartz 10/e 1008-1009; Sabiston 20/e p1034-1035)

Endoscopic Mucosal Resection (in CA Esophagus) • EMR provides essential staging information that guides treatmentQ. • It may also be used as a therapeutic modality for premalignant and early malignant conditionsQ.

60. Ans. a. CA colon (Ref: www.ncbi.nlm.nih.gov/pubmed/11775180)

Radio-Immunoguided Surgery for Colorectal Cancer

Section 9

• The intra-operative detection of metastatic disease in colorectal cancer depends on tumor-associated antigen and antibodies as well as detection technology (A hand-held gamma detecting probe) Q.

SPONTANEOUS REGRESSION

61. Ans. b. Osteosarcoma (Ref: Robbins 9/e p477, 1041, 955, 1339, 1149) Tumors with Spontaneous Regression (NCR MR) • Neuroblastoma • ChoriocarcinomaQ • Renal cell carcinomaQ Q



62. Ans. a. Malignant melanoma, b. Neuroblastoma



64. Ans. a. Neuroblastoma, b. Retinoblastoma

• Malignant melanomaQ • RetinoblastomaQ 63. Ans. a. Neuroblastoma, b. Renal cell carcinoma

Oncology

979

RADIOTHERAPY 65. Ans. c. Late S (Ref: Harrison 19/e p103e-4; Schwartz 10/e 313-314)



66. Ans. d. Samarium-153 (Ref: Harrison 20/e p657, 19/e p103e-4)

Section 9



Pure Beta Emitters • Strontium (Sr)-90Q • Yttrium (Y)-90Q

• H-3 (Tritium) Q • Phosphorus (P)-32Q



67. Ans. c. Iridium-192 (Ref: Principle and Practice of Radiation Oncology (Lippincott) 4/e p637; Harrison 19/e p103e-4)



68. Ans. a. Beta particles



69. Ans. c. Helium ion (Ref: Harrison 19/e p103e-4)

Alpha particles (helium ion) and low energy neutrons are densely ionizing; X-rays and gamma rays are sparsely ionizing.

70. Ans. c. Osteosarcoma (Ref: Essentials of Radiology by Bhaduri/502)



71. Ans. c. Dysgerminoma Diagnostic procedure

72. Ans. c. CT scan (Ref: Bailey 25/e p130) Typical effective dose (mSv)

Equivalent no. of chest radiographs

< 0.01

< 0.5

Radiographic examinations Limbs and joints (except hip) Chest (single PA film)

0.02

1

Skull

0.07

3.5

Thoracic spine

0.7

35

Lumbar spine

1.3

65

Hip

0.3

15

Pelvis

0.7

35

Abdomen

1.0

50

Intravenous urography

2.5

125

Barium swallow

1.5

75

Barium meal

3

150

Barium follow-through

3

150

Barium enema

7

350

2.3

115

CT chest

8

400

CT abdomen or pelvis

10

500

Lung ventilation (133Xe)

0.3

15

Lung perfusion (99mTc)

1

50

Kidney (99mTc)

1

50

Thyroid (99mTc)

1

50

Bone (99mTc)

4

200

Dynamic cardiac (99mTc)

6

300

PET head (18F-FDG)

5

250

Radionuclide studies



73. Ans. c. Gating (Ref: Radiation oncology by Leibe Phillips 2/e p192-192)

The technique employed in radiotherapy to counteract the effect of tumor motion due to breathing is known as Gating.

74. Ans. d. Cesium-137 (Ref: Washington and Lever Principle of Radiotherapy 2/e p326; Harrison 19/e p103e-4, 18/e p691-692) • Cs-137 is not used as permanent implant, it is used as temporary interstitial implant.



75. Ans. c. Radium-226 (Ref: Text Book of Radiation Oncology by Leibel Philips 2nd/231)

Radium-226 • Radium-226 is not used anymore because of: −− Longest half life among all isotopes −− Emits alpha and gamma rays, which is the most dangerous combination −− Produces radon, a radioactive inert gas, which is difficult to remove

Oncology

CT head

980

Surgery Essence

76. Ans. d. Medulloblastoma, e. Glioblastoma (Ref: Harrison 20/e p648, 19/e p599-602)



77. Ans. a. G2M

78. Ans. a. CNS (Ref: Radiation Oncology 8/e p41; Harrison 19/e p839-840)

Amifostine • Amifostine offers no protection to CNS, as it doesn’t cross blood brain barrier • Amifostine is a radiation protector • Amifostine provide protection against hematologic and non-hematologic toxicity of cisplatin also Mechanism of action: • Amifostine scavenge free radicals produced by ionizing radiations Tissue Protected:

Oncology

• Gut lining, hematopoietic system and salivary glands

79. Ans. a. Ewing’s sarcoma



80. Ans. c. Cobalt-60 (Ref: Text Book of Radiation Oncology by Leibel Philips 2nd/231)



81. Ans. a. X-rays (Ref: Harrison 19/e p103e-4)



82. Ans. c. Cells in G2M phase, d. Dividing cells

83. Ans. b. Cobalt-60



84. Ans. c. 5.26 years

85. Ans. b. Radioprotector (Ref: Radiation Oncology 8/e p41)



86. Ans. b. Amifostine

87. Ans. b. Prostate (Ref: Text Book of Radiation Oncology by Leibel Philips 2/e p315, 334)

MISCELLANEOUS

88. Ans. a. Wilm’s tumor (Ref: CSDT 11/e p1345; CPDT 16/e p807-809) • 5-year survival in localized Wilm’s tumor of favorable histology: >97%



89. Ans. a. CA thyroid (Ref: Harrison 20/e p2716, 19/e p2303) • Thyroid carcinoma is poorly responsive to chemotherapy.



90. Ans. c. Concurrent chemotherapy (Ref: Bailey 27/e p151, 155; Devita 9/e p749; Cancer of the Head and Neck by Suen and Myer 4/e p291-292)



91. Ans. b. Chemoport “Chemoports are totally implantable venous access devices used to facilitate chemotherapy administration. Internal jugular veins and subclavian veins are the commonly used venous access for port placement. These devices are usually retained over a period of 1 to 2 years or more after which the device is explanted. Some of the long-term complications include catheter embolism, catheter or port occlusion, catheter breakage, device rotation, and vascular thrombosis.”



92. Ans. a. Granisetron (Ref: Harrison 20/e p256) “5-HT3 antagonists like ondansetron and granisetron prevent postoperative vomiting, radiation therapy–induced symptoms, and cancer chemotherapy– induced emesis, but also are used for other causes of emesis.”-Harrison 20/e p256

Section 9



93. Ans. b. Prostate cancer (Ref: Sabiston 20/e p1845, 19/e p1816-1817; Schwartz 10/e p927; Bailey 27/e p990) Malignancies associated with Migratory Thrombophlebitis • CA pancreas (MC)Q • Prostate cancerQ Q • CA lung • Ovarian cancerQ • GI malignanciesQ • LymphomaQ • Trousseau’s syndrome: Migratory thrombophlebitisQ • Trousseau’s sign: Carpopedal spasm in hypocalcemiaQ • Troisier’s sign: Palpable left supraclavicular LN (Virchow’s node)Q



94. Ans. b. Tumor let



96. Ans. d. CA oral cavity



97. Ans. d. Liposarcoma

95. Ans. a. Oropharyngeal carcinoma (Ref: Bailey 27/e p740; Devita 9/e p729)

Oncology

98. Ans. a. Sjögren’s syndrome (Ref:Harrison 20/e p340, 352, 19/e p353,365)

• Group of disorders having a benign course but exhibiting clinical and histological features suggestive of malignant lymphoma. • Characterized by benign infiltration of lymphoid cells or histiocytes which microscopically resembles a malignant lymphoma. Pseudolymphoma is seen in Autoimmune Disorders

Drug-induced

• Sjogren syndrome • Dysgammaglobulinemia

• Phenytoin and phenobarbital • Primidone



99. Ans. a. Apical lobe of lung



100. Ans. a. Lyme’s disease (Ref: Harrison 20/e p1008, 19/e p1150) Condition

Section 9

Pseudolymphoma



981

Seen in

Necrolytic erythema migrans

• Glucagonoma

Erythema chronicum migrans

• Lyme’s disease

Erythema infectiosum (fifth disease)

• Parvovirus B19

Erythema marginatum

• Acute rheumatic fever

101. Ans. d. Osteosarcoma (Ref: Robbins 9/e p476-479) Small Round Blue Cell Tumors (WEL PNR) • • • • •

Wilm’s tumor Ewing’s sarcomaQ LymphomaQ MedulloblastomaQ Small cell variant of osteosarcomaQ

• • • • •

Q

Primitive neuroectodermal tumorQ NeuroblastomaQ RhabdomyosarcomaQ Askin tumorQ Desmoplastic small cell tumorQ



102. Ans. a. Medullary carcinoma thyroid (Ref: Sabiston 20/e p691)



103. Ans. a. BRCA-1: Lung, c. Chromosome 16: Philadelphia chromosome



104. Ans. a. CA colon (Ref: Harrison 20/e p585, 19/e p543)



105. Ans. a. Pheochromocytoma, b. Insulinoma, d. Appendicular carcinoid (Ref: Sabiston 20/e p957; Schwartz 10/e 1574,1585-1588; Bailey 27/e p846,850,1315)

Glucagonoma are mostly malignant and metastatic at the time of presentation.

106. Ans. a. Paget’s disease of bone



107. Ans. d. Finger (Ref: Bailey 27/e p614, 711, 712)

Glomus Tumour • These arise from subcutaneous arteriovenous shunts (Sucquet–Hoyer canals) especially in the corium of the nail bed. • Typically, they are small, purple nodules measuring a few millimetres in size, which are disproportionately painful in response to insignificant stimuli (including cold exposure). • Subungual varieties may be invisible causing paroxysmal digital pain.

108. Ans. c. Chromosome 17



109. Ans. a. Metastatic germ cell tumor (Ref: Harrison 19/e p588; Computed Body Tomography by Lee Sagel 4/e p1207)

The necrotic retroperitoneal mass represents necrotic lymph nodes. Necrotic lymph nodes usually suggest malignant metastasis, most likely due to testicular (germ cell) tumor.

Oncology

• Harrison says “Chemotherapy can be administered as an adjuvant (i.e. in addition to surgery or radiation) after all clinical apparent disease has been removed. This use of chemotherapy may have curative potential in breast and colorectal neoplasms, as it attempts to eliminate clinically unapparaent tumor that may have already disseminated.”

982

Surgery Essence Testicular Tumors • Testicular tumors tend to metastasize via the lymphatic system. • In general, the testicular lymphatics which follow the course of the testicular vessels, drain directly into the lymph nodes in or near the renal hilus. • After involvement of these sentinel nodes, the lumbar paraaortic nodes become involved (unilaterally or bilaterally), followed by spread to the mediastinal and supraclavicular nodes or hematogenous dissemination to lungs, liver and brain. • Seminoma is c-kit positive tumorQ

110. Ans. c. Glioma (Ref: KDT 6/e p577)

Oncology

• Somatostatin is a ‘universal switch off’. Somatostatin analogue octreotide decreases secretion of various hormones. Uses of Octreotide • Pancreatic neuroendocrine tumors (insulinoma, • AcromegalyQ glucagonoma, VIPoma) Q • Bleeding varicesQ • Carcinoid tumors and syndromeQ • Enterocutaneous fistulaQ

111. Ans. a. Benign, c. Malformation, d. Mostly Asymptomatic (Ref: Robbins 9/e p267)

Hamartoma • • • • •

Hamartoma refers to an excessive, focal overgrowth of cells and tissues native to the organ in which it occursQ. Cellular elements are mature and identical to those found in the remainder of the organQ Do not reproduce the normal architecture of the surrounding tissueQ. Benign natureQ Mostly asymptomatic, rarely presents with life-threatening clinical problemsQ

Heterotopia (or Choristoma) • Choristoma is applied to microscopically normal cells or tissues that are present in abnormal locationsQ. • Examples: Rest of pancreatic tissue found in the wall of the stomach or small intestine, or a small mass of adrenal cells found in the kidney, lungs, ovaries, or elsewhereQ.



112. Ans. b. Oral cavity



113. Ans. a. 1975 (Ref: www.nihfw.org/.../NationalHealthProgramme)

National Cancer Control Programme

Section 9

• To control the problems associated with cancer the Govt. of India has launched a National Cancer Control Programme in 1975 stressing on primary prevention and early detection of cancerQ.

Males



Female

• MC cancer in males (PLC): Prostate >Lung >ColorectalQ

• MC cancer in females (BLC): Breast >Lung >ColorectalQ

• Cancer deaths in males (LPC): Lung >Prostate >ColorectalQ

• Cancer deaths in females (LBC): Lung >Breast >ColorectalQ

114. Ans. d. All of the above

“Acanthosis nigrican can be a reflection of an internal malignancy, most commonly the adenocarcinoma of GIT, lung, uterus and breast.”

115. Ans. d. Gallbladder



116. Ans. a. B cell (Ref: http://www.mdanderson.org/patient-and-cancer-information/cancer-information/cancer-types/eye-cancer/orbit.html)

Oncology

983

Orbital Lymphoma

Clinical Features • It may show up as a nodule in the eyelid or around the eye, or it may cause the eye to be pushed outQ. • This type of eye cancer usually does not cause painQ.   Diagnosis • First step in diagnosis of orbital lymphoma may be a CT scan of the orbit followed by a surgical biopsyQ. • Making the correct diagnosis of the biopsy is very important.  

Section 9

• MC type of cancer of the orbit in adultsQ • Usually a form of B-cell non-Hodgkin’s lymphomaQ.

Treatment • Radiation therapy, monoclonal antibody therapy, chemotherapy or a combination of these, depending on type of lymphoma the stage of the tumor.

117. Ans. a. Chondrosarcoma (Ref: Sabiston 20/e p1602; Schwartz 10/e p667) “Chondrosarcomas are the most common primary chest wall malignancy. As with chondromas, they usually arise anteriorly from the costochondral arches. CT scan shows a radiolucent lesion often with stippled calcifications pathognomonic for chondrosarcomas.”-Schwartz 10/e p667



118. Ans. c. Pyogenic granuloma (Ref: Sabiston 20/e p2012; Bailey 27/e p614) “Pyogenic granuloma is a misnomer for an exuberant outburst of highly vascular granulation tissue at the site of previous relatively trivial trauma. These lesions are friable, bleed easily, and may grow rapidly. They respond to curettage or simple excision. They usually occur on the fingertips.”-Sabiston 20/e p2012

Oncology

CHAPTER

40

Sarcoma

SOFT TISSUE SARCOMA Soft Tissue Sarcoma • Rare unusual neoplasm of soft tissues • MC site: ExtremityQ (lower >upper) > Trunk > Retroperitoneum >Head & Neck • MC type: Liposarcoma >Leiomyosarcoma >Synovial sarcoma >Malignant peripheral nerve sheath tumor >Malignant fibrous histiocytomaQ > GIST • MC pediatric soft tissue sarcoma: RhabdomyosarcomaQ • Hematogenous spread is typical of sarcomasQ Histopathological Type of STS is Site Dependent • Extremity • Malignant fibrous histiocytomaQ >Liposarcoma • Retroperitoneum • LiposarcomaQ • Viscera • GISTQ Pathology • • • •

STS tends to grow along fascial planesQ, with the surrounding soft tissue compressed to form a pseudocapsuleQ. Clinical behavior of STS is determined by: Anatomic location (depth), grade & sizeQ MC route of spread in soft tissue sarcoma: HematogenousQ MC site of metastasis: LungQ; Lymphatic metastasis is rareQ

Clinical Features • MC symptom of STS: Painless massQ • Size at presentation is dependent on the location of tumorQ. • Smaller tumors are located in distal extremitiesQ • Larger tumors are detected in proximal extremity and retroperitoneumQ. • Retroperitoneal STS almost always present as large asymptomatic massQ Diagnosis of Soft Tissue Sarcoma • • • • •

Core-cut or true-cut biopsy (CT or USG guided) is diagnosticQ Incisional biopsy is done if core-cut biopsy is non-diagnosticQ FNAC: To confirm or rule out presence of metastatic focus or local recurrenceQ MRI: IOC for assessing extremity STSQ CECT: IOC for assessing retroperitoneal sarcomaQ

Treatment • Adequate excision + adjuvant radiotherapy with or without adjuvant chemotherapyQ. • Two most active chemotherapy agents against STS: Doxorubicin & ifosfamideQ Prognosis • Best prognostic factor of soft tissue sarcoma: Grade • Best prognosis is seen in: Extremity STSQ • Most important predictor of metastasis in STS: GradeQ • MC cause of death in STS: MetastasisQ; 5-year survival rate for STS (all stages): 50–60%

Sarcoma

985

SARCOMAS WITH LN METASTASIS

Sarcomas with Lymph Node Metastasis (MARCES) • Malignant fibrous histiocytomaQ • AngiosarcomaQ • RhabdomyosarcomaQ

• Clear cell sarcomaQ • Epithelial sarcomaQ • Synovial sarcomaQ

Section 9

• MC site of metastasis in sarcomas of extremity: LungsQ • MC site of metastasis in retroperitoneal sarcomas: LiverQ > LungsQ • LN metastasis is uncommon in soft tissue sarcomaQ.

RHABDOMYOSARCOMA Rhabdomyosarcoma • • • •

Rhabdomyosarcoma arises from mesenchymal tissues. MC sites of origin: Head & neckQ (parameningealQ)>Extremities >Genitourinary tract >Trunk MC pediatric soft tissue sarcoma: RhabdomyosarcomaQ Associated with: NF, Beckwith-Weidman syndrome, Li-fraumeni and Fetal alcohol syndrome

Pathology MC histological type: Embryonal rhabdomyosarcomaQ; MC type in adults: Pleomorphic variantQ Diagnostic cell: RhabdomyoblastQ May contain tadpole cells or strap cellsQ Embryonal type consist of spindle cell variant and sarcoma botryoidesQ (tumor cells resemble tennis racketQ and tumor cells form submucosal zone of hypercellularity known as cambium layerQ) • Best & most special marker for RMS: MyogeninQ > MYOD–1Q > DesminQ • • • •

Clinical Features • • • •

MC presenting symptom: MassQ (may or may not be painful) Bimodal, first peak between 2–5 years, second peak between 15–19 years Extremity RMS are more common in lower extremityQ MC site of metastasis: LungQ

• Diagnosis is confirmed by biopsyQ • MRI: IOC for diagnosing extent of diseaseQ • CT: Used to rule out lung metastasisQ Treatment • Wide-local excisionQ of tumor with surrounding involved tissue • Tumor not amenable to primary excision: Neoadjuvant chemotherapy, after the tumor has decreased in size, resection of gross residual disease • Radiation therapy: When microscopic or gross residual disease exists after initial treatment. Prognosis • Prognosis is related to the site of origin, resectability, presence of metastases, number of metastatic sites, and histopathologic featuresQ. • Embryonal variant is a favorableQ and alveolar type has an unfavorable prognosisQ. Favorable Primary Sites • • • •

Orbit Nonparameningeal head & neckQ ParatestisQ VaginaQ Q

Unfavorable Primary Sites • ExtremityQ • ParameningealQ

Oncology

Diagnosis

986

Surgery Essence LEIOMYOSARCOMA Leiomyosarcoma • Leiomyosarcomas are malignant tumors composed of cells showing smooth muscle features. • MC site: UterusQ • Desmin and actin are the MC positive stainsQ. • Grading of leiomyosarcomas is difficult, and mitotic activityQ appears to be the best indicator of subsequent prognosis when combined with location and size.

Oncology

• Common major vascular sites: Pulmonary artery and IVCQ • Large tumor size and high mitotic rate/high grade are factors in poorer outcome. • Complete excision remains the primary therapeutic choiceQ.

ANGIOSARCOMA Angiosarcoma (Lymphangiosarcoma) • Rare tumor that develops as a complication of long-standing (>10 years) lymphoedemaQ. • Stewart and Treves described lymphangiosarcoma of the upper extremity in women with ipsilateral lymphedema after radical mastectomy. (Stewart-Treves Syndrome)Q Clinical Features • Acute worsening of edemaQ • Appearance of subcutaneous nodules with propensity towards hemorrhage and ulcerationQ Treatment • Preoperative chemotherapy and radiotherapy followed by surgical excision (radical amputation)Q • Associated with poor prognosis

SYNOVIAL SARCOMA Synovial Sarcoma • • • • •

Synovial sarcoma usually occurs in young adultsQ. Typically found in the para-articular areas of tendon sheaths & jointsQ. At least 50% of cases are in the lower limbsQ (especially the kneeQ), and most of the remainder are seen in the upper limbs. It generally does not originate from synovial tissueQ. Composed of two morphologically distinct types of cells that form a characteristic biphasic patternQ. • Characteristic chromosomal translocation, t(X;18)(p11.2;q11.2)Q • These hallmark translocations have become the gold standard in diagnosing synovial sarcomaQ • 100% of biphasic and 96% of monophasic synovial sarcomas possess the specific t(X;18)(p11.2;q11.2) translocationQ.

Treatment • Adequate excision + adjuvant radiotherapy with or without adjuvant chemotherapyQ.

Section 9

KAPOSI SARCOMA Kaposi’s Sarcoma • Kaposi’s sarcoma appears as rubbery bluish nodules that occur primarily on the extremitiesQ but may appear anywhere on the skin and visceraQ. • Classically, KS is seen in people of Eastern Europe or sub-Saharan AfricaQ.

AIDS-related Kaposi’s Sarcoma • AIDS-related KS occurs primarily in male homosexuals and not in IV drug abusers or hemophiliacsQ • Lesions spread rapidly to the nodes and GI and respiratory tract often are involvedQ. • Development of AIDS-related KS is associated with concurrent infection with a herpes-like virus (HHV-8)

Sarcoma

987

Pathology

• Treatment for all types of KS consists of radiationQ to the lesions. • Combination chemotherapy is effective in controlling the disease, although most patients develop an opportunistic infection during or shortly after treatmentQ. • Surgical treatment is reserved for lesions that interfere with vital functions:, such as −− Bowel obstruction and Airway compromiseQ • Most common malignancy in HIV positive individuals: NHL > Kaposi sarcomaQ

Section 9

• Usually multifocalQ rather than metastatic. • Histologically, the lesions are composed of capillaries lined by atypical endothelial cellsQ. • Early lesions may resemble hemangiomas, while older lesions contain more spindle cells and resemble sarcomas. • Lesions are locally aggressive but undergo periods of remissionQ. Treatment

DERMATOFIBROSARCOMA PROTUBERANS Dermatofibrosarcoma Protuberans (DFSP) • DFSP is a low-grade sarcoma because it may recur locally but rarely metastasizesQ. • Monomorphous, mononuclear, spindle cell lesion involving both dermis & subcutisQ. • MC site: TrunkQ (50%) >Extremities (30%) >Head & neck (20%) Pathology • Large lesions often are associated with satellite nodules; Positive for CD34Q • Have unpredictable radial extensionsQ of tumor permeating through the subcutaneous tissue large distances from the primary nodule. • More than 75% of DFSP have a ring chromosomeQ, composed of translocated portions of chromosomes 17 & 22Q Clinical Features • Typically presents in early or mid-adult life, beginning as a nodular cutaneous massQ. • Pattern of growth: Slow and persistentQ • Lesion enlarges over many years, it becomes protuberantQ Diagnosis

Oncology

• IOC for diagnosis: Incisional biopsy Treatment • Aggressive resection (2-4 cm margin) with removal of underlying fascia with special attention to radial margins (local recurrence rate 5 cm c. FNAC is diagnostic d. TNM staging done e. Radiosensitive



22. Which one of the following statements is true regarding soft tissue sarcoma? (APPG 2016) a. Most common location is retroperitoneal b. Fibrosarcoma is the commonest histological variety of soft tissue sarcoma c. Death is mostly due to lung metastases d. Account for Leiomyosarcoma >Malignant fibrous histiocytoma • MC retroperitoneal tumor: LiposarcomaQ

9. Ans. b. Enlarged size, c. Pseudoencapsulated, d. Spread though musculoaponeurotic plane (Ref: Devita 10/e p1255; Sabiston 20/e p760764; Schwartz 10/e 666-669)



10. Ans. a. Liposarcoma is MC retroperitoneal sarcoma, b. Incisional biopsy is needed when size > 5 cm, d. TNM staging done



11. Ans. d. Tumor grade

12. Ans. b. Sarcoma

13. Ans. b. Extremity



14. Ans. a. Orbit

15. Ans. b. Liposarcoma

16. Ans. a. Embryonal rhabdomyosarcoma



17. Ans. b. Sarcoma

Hematogenous spread is typical of sarcomas and lymphatic spread is typical of carcinoma.



18. Ans. d. All of the above (Ref: Robbins 8/e p1318)

Retroperitoneal Tumors • Deep seated mass in abdomen • When the tumor is very large do symptoms of pain or functional disturbances occur • Retroperitoneal tumors may present themselves with signs of weight loss, emaciation and abdominal pain • These tumors may also compress the kidney or ureter leading to renal failure



19. Ans. b. Desmin (Ref: Devita 9/e p1780-1784; Sabiston 20/e p1890; Schwartz 10/e 666-669,1465-1487))

Desmin can be used for the diagnosis of rhabdomyosarcoma.



20. Ans. a. Wide excision

21. Ans. a. Soft tissue sarcoma



22. Ans. c. Death is mostly due to lung metastases

23. Ans. a. Lung

DERMATOFIBROSARCOMA PROTRUBERANS

24. Ans. d. Dermatofibrosarcoma protuberans (Ref: Sabiston 20/e p750, 766))

KAPOSI SARCOMA

25. Ans. c. Vascular (Ref: Harrison 20/e p1448, 19/e p1242; Devita 9/e p2101-2104; Sabiston 20/e p750; Schwartz 10/e 485)



26. Ans. a. Kaposi sarcoma (Ref: Devita 9/e p2100)

• Most common malignancy in HIV positive individuals: NHL>Kaposi sarcomaQ 27. Ans. d. Occurs in AIDS patients only

28. Ans. b. Has increasing incidence among AIDS patients



30. Ans. b. HHV 8

29. Ans. b. Lower limbs

SECTION

10 Others

CHAPTERS ˆˆ ˆˆ ˆˆ ˆˆ ˆˆ ˆˆ ˆˆ ˆˆ ˆˆ ˆˆ ˆˆ

Pediatric Surgery Trauma Transplantation Anesthesia and Perioperative Complications Robotics, Laparoscopy and Bariatric Surgery Sutures and Anastomoses Sterilization and Infection Fluid, Electrolyte and Nutrition Blood Transfusion Shock Miscellaneous

CHAPTER

41

Pediatric Surgery Multiple Choice Questions





1. A six years old female presents with constipation and urinary retention. On examination on a presacral mass is noted. Most probable diagnosis is: (AIIMS May 2008) a. Pelvic neuroblastoma b. Rectal duplication cyst c. Sacrococcygeal teratoma d. Anterior sacral meningocele 2. First meconium is said to be formed during the month of fetal life: a. Second b. Fourth c. Seventh d. Ninth







3. The given tumor is embryological remnant of:





5. An one month old female child has swelling over the back in the sacral region. There is no cough impulse in the swelling. X-ray examination shows erosion of the coccyx. The most likely clinical diagnosis would be: (UPSC 95) a. Meningocele b. Lipoma c. Sacrococcygeal teratoma d. Neurofibroma 6. Most common solid malignant tumor of infancy: a. Neuroblastoma b. Nephroblastoma c. Germ cell tumor d. Rhabdomyosarcoma 7. Most common posterior mediastinal mass in children is: a. Hodgkin’s disease (Recent Question 2016) b. Neuroblastoma c. Esophageal duplication cyst d. Bronchogenic cyst

8. Malignant tumor of childhood that metastasizes to bone most often is: (Recent Question 2016) a. Neuroblastoma b. Nephroblastoma c. Adrenal gland tumors d. Ovarian granulose cell tumor 9. Which of the following is the most common tumor of newborn? (All India 2012) a. Neuroblastoma b. Wilm’s tumors c. Leukemia d. Sacrococcygeal teratoma 10. Primitive streaks remnants give rise to: (DNB 2012) a. Neuroblastoma b. Wilm’s tumour c. Sacrococcygeal teratoma d. Hepatoblastoma 11. Currarino triad includes: (Recent Question 2016) a. Pre-sacral meningocele + Sacral defect + Tethered cord b. Ectopia vesicae +
Anorectal malformation + Sacrococcygeal osseous defect c. Anorectal malformations + Sacrococcygeal osseous defect + Presacral mass d. Tethered cord + Anorectal malformations + Ectopia a. Neural tube vesicae b. Allantois 12. Most common renal tumor in children: (Recent Question 2017) c. Notochord a. Renal cyst d. Primitive streak b. Congenital mesoblastic nephroma 4. Sacrococcygeal teratoma is embryological remnant of: c. Neuroblastoma d. Nephroblastoma  (MHSSMCET 2007) 13. Most common intra-abdominal tumor in infant: a. Neural tube  (Recent Question 2017) b. Allantois a. Neuroblastoma b. Wilms tumor c. Notochord c. HCC d. Hypernephroma d. Primitive streak

Explanations

1. Ans. d. Anterior sacral meningocele (Ref: Sabiston 20/e p1932; Schwartz 10/e p1750, 9/e p1553)

Anterior Sacral Meningocele • Defect of anterior aspect of sacrumQ involving one or more segments with herniation of meningeal sac into extra-peritoneal region. • More common in femalesQ Clinical Features • Symptoms usually occur in 2nd to 3rd decadeQ • Symptoms are primarily owing to the mechanical effectsQ of a pelvic tumor. • Abdominal pain, constipation, urinary retentionQ or incontinence Diagnosis • X-ray: Unilateral sickle shaped distortion of sacral bone (Scimitar sacrumQ) is pathognomonic. This sign is not present in all cases.

2. Ans. a. Second (Ref: Bailey 25/e p85)

By end of 3 months upper small intestine has become filled with meconium. So the answer should be less than 3 months.

Meconium • Meconium is a sterile mixture of epithelial cells, mucin and bile, formed as the fetus starts to swallow amniotic fluid. • By end of 3 months upper small intestine has become filled with meconiumQ.

3. Ans. d. Primitive streak (Ref: Sabiston 20/e p1893; Schwartz 10/e p1641)



4. Ans. d. Primitive streak (Ref: Sabiston 20/e p1893-1894; Schwartz 10/e p1641)

Sacrococcygeal teratoma is thought to be a derivative of the primitive streak

Sacrococcygeal Teratoma • Teratomas occur most frequently in the neonatal period, sacrococcygeal region is the MC siteQ. • More common in femalesQ • Thought to be a derivative of the primitive streakQ • Most often an obvious external presacral massQ • Most of the tumor is usually external, with a minimal intrapelvic presacral componentQ • These lesions should be carefully followed with serial USG until delivery because the blood supply to the tumor may grow to the point of stealing a significant proportion of placental blood flow to the fetusQ. • The development of hydrops or placentomegaly is associated with a poor prognosisQ. • Most neonatal SCTs are benignQ. • Incidence of malignancy is related to age at time of diagnosis and is most frequently represented as yolk sac tumors or embryonal carcinomasQ. Treatment • Complete surgical excisionQ through a chevron-shaped buttock incision. • Resection of the coccyx is criticalQ because failure to remove this structure results in significantly higher local recurrence rates.

5. Ans. c. Sacrococcygeal teratoma



6. Ans. a. Neuroblastoma (Ref: Sabiston 20/e p1887-1888; Schwartz 10/e p678,1639-1640; Bailey 27/e p138)



7. Ans. b. Neuroblastoma (Ref: Schwartz 10/e p678)

Most common posterior mediastinal mass in children is neurogenic tumor (Neuroblastoma among the given options).

8. Ans. a. Neuroblastoma

Pediatric Surgery

9. Ans. d. Sacrococcygeal teratoma (Ref: Surgery of Childhood Tumors (Springer) 2008/49)

10. Ans. c. Sacrococcygeal teratoma 11. Ans. c. Anorectal malformations + Sacrococcygeal osseous defect + Presacral mass

• Currarino triad: Anorectal malformations + Sacrococcygeal osseous defect + Presacral mass

12. Ans. b. Congenital mesoblastic nephroma (Ref: Campbell 11/e p2885)

“Although congenital mesoblastic nephroma (CMN) is a rare benign congenital renal tumor it is the most common solid renal tumor in the neonatal period.”-Campbell 11/e p2885

Section 10

• Sacrococcygeal teratoma is the predominant teratoma as well as the most common neoplasm in the fetus and newbornQ with an estimated incidence of 1:20,000 to 1:40,000 live births and a female predominanceQ ranging from 2:1 to 4:1.



995

13. Ans. a. Neuroblastoma (Ref: Sabiston 20/e p1887; Schwartz 10/e p1639; Bailey 27/e p847)

Others

CHAPTER

42

Trauma

TRIMODAL MORTALITY MODEL FOR TRAUMA Immediate Death (Within minutes of injury) • Declared dead at scene or die shortly after arrival to hospital • Causes: −− Irreversible brain injuryQ −− Hemorrhage from injuries of heart, aorta, liver, lungs & pelvic fractureQ

• • • • •

Trimodal Mortality Model for Trauma Early Death (Death within hours of arrival to hospital) Intracranial hemorrhageQ Internal hemorrhage involving respiratory system & abdominal organsQ Multiple injuries leading to severe blood lossQ Tension pneumothoraxQ Cardiac tamponadeQ

Late Death (Days to weeks after injury) • SepsisQ • Multiple organ failureQ

TRIAGE Triage • Triage means to “sort”, especially used for mass-casualtiesQ • Involves prioritizing victims into categories based on: Severity of injuryQ, likelihood of survivalQ & urgency of careQ • Triage tags: Colour codes are used to identify the patients Categorization of Triage Tags Red (immediate)

• • • Yellow (delayed) • • Green (ambulatory) • Black (Expectant) •

Most critically injuredQ Includes patients with major head injury, injuries to thorax or abdomenQ Immediate care is requiredQ Less critically injuredQ Require in hospital treatmentQ No life or limb threatening injuriesQ Dead or moribund patientsQ

PRIMARY, SECONDARY AND TERTIARY SURVEY Primary Survey

Secondary Survey

Tertiary Survey

• Aimed at detecting & simultaneously treating immediately life threatening injuriesQ • Identified by the mnemonic ‘ABCDE’ • A: Airway maintenance with cervical spine protectionQ • B: Breathing (ventilation & oxygenation)Q • C: Circulation with hemorrhage controlQ • D: Disability (Brief neurological examination)Q • E: Exposure /Environmental controlQ

• Consists of head to toe systematic assessment of abdominal, pelvic & thoracic areasQ • Complete inspection of body surface to find all injuries & neurological examinationQ • Patients & surrogates should be queried to obtain an AMPLE history • A: AllergiesQ • M: MedicationQ • P: Past illness or pregnancyQ • L: Last mealQ • E: Events related to injuryQ

• Comprehensive patient evaluation after initial resuscitation periodQ • Usually performed about 24 hours after admissionQ • Include a thorough physical examination combined with targeted radiographic imaging (X-ray usage or CT) based on examination findingQ • Decreases the delay in diagnosis of potentially life-threatening injuriesQ

Trauma

997

PRIMARY SURVEY

Cervical spine injury should be suspected in all the patientsQ All trauma patients should have cervical spine immobilization & protected through outQ First priority: Cervical spine followed by airwayQ Asses the patency of airway: Elicit the verbal responseQ (simplest way: Ask the patients name; ability to speak indicates adequate airway protectionQ) • In patients, who cannot speak, suspect mental status depression or airway obstruction. Both are indication for airway managementQ • Other indications: Noisy breathingQ, facial traumaQ & GCS ≤ 8Q • • • •

Section 10

A: Airway Maintenance with Cervical Spine Protection

Compromised Airway Require Stepwise Progression • • • • •

First clearing the airway by suctioning secretions or blood followed by jaw thrust or chin liftQ Insertion of oropharyngeal or nasopharyngeal airwayQ Definitive airway of choice for must injured patients: Oral endotracheal intubation with cuffed endotracheal tubeQ ATLS updates (2018) recommended use of video laryngoscope for intubationQ In severe maxillofacial injuries: Emergency airway (Needle cricothyroidotomyQ ) & Definitive airway (TracheostomyQ ) In Severe Maxillofacial Injuries Emergency Airway • • • • •

Definitive Airway

Emergency airway: Needle cricothyroidotomyQ Performed quickly; High flow O2 is given via 4-6 mm tubeQ Advantage: Provides time for definitive airwayQ Disadvantage: CO2 retention occurs within 20-30 minutesQ Avoided in children 100/minQ, BP StomachQ (17%) >SIQ (12.9%) MC injured organ in gunshot wound: Small intestineQ (SI) MC injured bowel in blunt trauma abdomen: JejunumQ MC injured structure in seat belt injury: MesenteryQ MC injured site in deceleration injury: Duodenojejunal junctionQ First investigation done in blunt trauma abdomen: FASTQ Gold standard investigation in stable patients of blunt trauma abdomen: CECTQ

Others

Trauma Scoring System Revised Trauma Score

1000

Surgery Essence FAST FAST (Focused Assessment with Sonography for Trauma) • • • •

FAST: Emergency USG done very fast, performed within 2-4 minutesQ Rapid diagnostic examination to assess patients with potential thoracoabdominal injuriesQ FAST is the first investigation done in blunt trauma abdomen; FAST has replaced DPLQ 4 ‘P’s are evaluated in the sequence: Pericardial sac → Perihepatic region → Perisplenic region → PelvisQ

Traditional Four Views of FAST

Others

Subxiphoid transverse viewQ

• Assess pericardial sacQ

Right upper quadrant (RUQ) • Assess perihepatic longitudinal viewQ regionQ Left upper quadrant (LUQ) • Assess perisplenic longitudinal viewQ regionQ Suprapubic longitudinal & transverse viewQ

• Assess pelvisQ

• e-FAST (extended FAST) has two additional views, right & left thoracic views to rule out pneumothorax or hemothoraxQ . • Stratosphere sign or barcode sign is seen on e-FAST in pneumothoraxQ .

DIAGNOSTIC PERITONEAL LAVAGE Diagnostic Peritoneal Lavage • Performed for blunt trauma abdomen patients • DPL is performed through vertical infra-umbilical midline incision unless the patient has pelvic fracture or is pregnantQ • Linea alba is sharply incised & catheter is directed towards pelvisQ

Section 10

• Abdominal contents should initially be aspirated using a 10-mL syringeQ. • Through the catheter, one liter of saline or RL is infused into peritoneal cavityQ. • Lavage fluid is sent for assessment Positive DPL • >10 mL of gross blood is aspirated directly from peritoneal cavityQ

• Returned effluent contains: −− RBCs >1 lac/mm3Q −− Demonstrable bacteria or bileQ

-

• Sensitivity of DPL for detecting significant intra-abdominal injury is 82-96% & specificity 87-99%.

WBCs >500/mm3Q Amylase >174 IU/dLQ

Trauma

1001

MANAGEMENT OF BLUNT TRAUMA ABDOMEN

Section 10

PENETRATING TRAUMA Penetrating Abdominal Injuries • Gunshot Abdominal Wounds: −− Chances of internal injury is very high in gunshot wounds, thus little preoperative evaluation is required and laparotomy is mandatoryQ. • Stab Wounds to Abdomen: −− Exploratory laparotomy is indicated in patients with isolated penetrating abdominal wound if hypotensive or in shock or showing peritoneal signsQ.

Others

Anterior Stab Wounds Flank and Back Wounds • Local wound exploration can be performed to determine if there is any • Risk of injury to colon, kidney and penetration of the peritoneal cavityQ. ureterQ • If the tract terminates without entering the peritoneum, the injury can be • Triple contrast CTQ is advised to detect managed as a deep lacerationQ and laparotomy is not needed. colon and retroperitoneal injuries and the need for laparo­tomy. • Otherwise, penetration of the peritoneum is assumed and significant injury must be excluded by further diagnostic evaluationsQ (FAST, CECT, DPL or laparoscopy)

1002

Surgery Essence SPLENIC TRAUMA Splenic Injuries • MC organ injured in blunt trauma abdomen: SpleenQ Pathophysiology • Direct compressionQ of the organ in the left upper quadrant of the abdomen • Deceleration mechanism that tears the splenic capsule or parenchymaQ, mainly at areas fixed or tethered to the retroperitoneum

Others

Diagnosis • Identification of splenic injuries may occur during laparotomy in unstable patients taken emergently to the operating roomQ. • Unstable patients with intra-abdominal fluid on FAST require exploration, with the spleen commonly being the bleeding intraabdominal organQ. • In stable patients, abdominal CT performed with IV contrast is the mainstay for diagnosing & characterizing splenic injuriesQ. • Images are typically obtained with the contrast in the portal venous phase to enhance the splenic parenchyma maximally while still being able to visualize the vasculature. • Splenic injuries appear as disruptions in the normal splenic parenchyma, frequently with surrounding hematoma & free intraabdominal bloodQ. • Occasionally, active extravasation of contrast, identified as a high-density blush, can be identified, contained within a pseudoaneurysm or bleeding into the peritoneal spaceQ.  • Angiography has been used for injuries that demonstrate active extravasation by CTQ. • Angiography can identify specific sites of bleeding from the splenic parenchyma & underlying segmental or trabecular vessels; however, it cannot characterize the splenic parenchymal injury but can be complementary to CT. • Advantage of angiography: Potential to obstruct sites of bleeding endovascularly using angioembolizationQ. • Patients who are candidates for nonoperative management of their splenic injury but demonstrate a blush by CT, indicating active extravasation, may benefit from angiography with embolization to eliminate the splenic pseudoaneurysmQ.  • Angiographic embolization is considered only in hemodynamically stable patientsQ. Management • With appropriate patient selection, many patients with blunt splenic trauma can be managed without splenectomyQ. • No bleeding patient should go without splenectomy or splenic repair, especially in an attempt to push the figurative nonoperative envelopeQ. • Hemodynamic stability is a prerequisite for nonoperative management and must be present without ongoing intravascular volume supportQ. • Hemodynamic stability is indicated by a normal blood pressure and lack of tachycardia, no physical examination findings indicating shock, and absence of metabolic acidosisQ. • Nonoperative management is reserved for grades I, II and isolated grade III injuriesQ. Indications of Operative Management of Splenic Trauma • Instability at admission

•  Exact location of bleeding is unknownQ  •  Failed nonoperative managementQ

Q  

• Best approach: Midline incision with packing of all four quadrants in cases of hemodynamic instabilityQ. • Drains should not be placed unless there is concern that the tail of the pancreas was also injuredQ.

Section 10

Splenic Injury Secondary to Penetrating Abdominal Trauma • Splenic injury secondary to penetrating abdominal trauma is usually identified during laparotomy and should be addressed based on the presence or absence of ongoing bleeding. Q • Splenectomy is performed in cases of ongoing bleedingQ. American Association for the Surgery of Trauma: Spleen Organ Injury Scale Grade I II

III

Type

Description of Injury

Hematoma

Subcapsular tear 25% of spleen)

V

Hematoma

Completely shattered spleen

Laceration

Hilar vascular injury devascularizes spleen

Hepatic Injury • MC organ injured in blunt abdominal trauma: Spleen > liverQ • MC injured organ in penetrating trauma: Liver >Stomach >Small Intestine • Mechanisms of blunt hepatic trauma: Compression with direct parenchymal damage and shearing forces, which tear hepatic tissue and disrupt vascular and ligamentous attachments. • Most liver injuries (>85%) involve segments 6, 7 and 8 of the liverQ. • Most liver injury bleeding is venous Q; and therefore low pressure, tamponade is readily performed Diagnosis • Liver injuries are often first diagnosed on entering the abdomen in the unstable patient explored for the finding of free fluid on FAST examinationQ. • Stable patients with suspected hepatic trauma should undergo CECT abdomenQ. • Current CT modalities are excellent at providing significant anatomic detail that allows highly accurate characterization of injuries. • Contrast extravasation visualized as a high-density blush is identified indicating the presence of a pseudoaneurysm or active bleeding external to the liver capsuleQ. • Beer claw laceration: Multiple Linear laceration of liver on CECT • Liver injury grading involves the extent of parenchymal involvement and presence of vascular injuryQ

Others

LIVER TRAUMA

Section 10

Grade

1004

Surgery Essence Management

Others

• Unstable patients: Immediate laparotomyQ Conservative criteria for non-operative management require − Hemodynamically stable patientQ − No peritoneal signs on examinationQ Q −− Absence of other major injuries • Most treatment failures occur within the first 24 hours of admissionQ. • Failure of nonoperative management is defined as the development of hemodynamic instability or of liver-related multiple transfusions despite angiographic embolization, signs of peritonitis, or abdominal compartment syndromeQ. Deep Liver Laceration

• Opening the liver wound and directly approaching the bleeding vessel, a procedure known as tractotomyQ.

Penetrating Liver Tracts

• Tractotomy or tamponade using a balloon catheterQ

Injuries in the vicinity of retrohepatic IVC

• Packing alone, without operative exploration Q

Retrohepatic IVC Injury

• Atriocaval shunt (Shrock shunt)Q

• Liver parenchymal necrosis is the MC complication of severe liver injury in patients who undergo operationQ. • Rebleeding is the MC complication of nonoperative managementQ.

Section 10

Classification of Liver Injury (Moore) Grade

Types

Operative or CT Scan findings

I

Hematoma Laceration

Subcapsular, 75% of the hepatic lobe or >3 Couinauds segments within a single lobe Juxtahepatic venous injuries, i.e. retrohepatic vena cava/ central major hepatic veins

VI

Vascular

Hepatic avulsion

Trauma

1005

DUODENAL TRAUMA

• Usually associated with pancreatic injuries Diagnosis

• IOC for diagnosis: CECTQ (Only sign is gas or fluid collection in retroperitoneum & leakage of oral contrastQ) Management • Injury to 1st, 3rd & 4th part: Repaired like small bowel with suturesQ • Injury to 2nd part: Managed by damage control surgeries & triple tube ostomy is performed (Decompressive gastrostomy, decompressive duodenostomy & feeding jejunostomyQ)

Section 10

Duodenal Trauma Q

SEAT BELT INJURY Seat Belt Injury • • • • • •

Use of seat belt is associated with injury profile known as seat belt syndrome MC injured structure: MesenteryQ Classic seat belt sign: Skin abrasion of neck, chest & abdomenQ Classic seat belt sign is associated with high chances of internal organ injuryQ Caused by sudden deceleration → Mesentery tearQ May compress pancreas over vertebral column → Pancreaticoduodenal injuryQ Longitudinal Tear • Repair the tear as the bowel is perfused by adjacent tissuesQ

Transverse Tear • Leads to devascularization of bowelQ • Resection & anastomosis is doneQ

TRAUMA TRIAD OF DEATH Trauma Triad of Death • Trauma triad of death: Hypothermia + Coagulopathy + Metabolic acidosisQ • Coagulopathy is most commonly responsible for deathQ

Others

Trauma Triad of Death

DAMAGE CONTROL SURGERY Damage Control Surgery (DCS) • DCS centers on coordinating staged operative interventions with periods of aggressive resuscitation to salvage trauma patients sustaining major injuriesQ. • Damage control includes an abbreviated laparotomy, temporary packing, & closure of the abdomen in an effort to blunt the physiologic response to prolonged shock & massive hemorrhageQ.

1006

Surgery Essence • These patients are often at limits of their physiological reserve when they present to operating room and persistent operative efforts result in exacerbation of their underlying hypothermia, coagulopathy & acidosis, initiating a vicious cycle that culminates in deathQ. • In these situations, abrupt termination of the procedure after control of surgical hemorrhage & contamination, followed by ICU resuscitation & staged reconstruction, can be life savingQ.

Phases of Damage Control Surgery

Others

Phase I (Initial Exploration) • T  his phase consists of an initial operative exploration to attain rapid control of active hemorrhage & contaminationQ • Abdomen is entered via a midline incision and if exsanguinating hemorrhage is encountered four quadrant packingQ should be performed • Any violations of GI tract should be treated with suture closure or segmental stapled resectionQ • External drains are placed to control any major pancreatic or biliary injuries

Stage I Stage II Stage III Stage IV Stage V

Phase II (Secondary Resuscitation) • F  ollowing completion of the initial exploration, the critically ill patient is transferred to the ICUQ. • Invasive monitoring & complete ventilator supportQ are often needed. • This phase focuses on secondary resuscitation to correct hypothermia, coagulopathy & acidosisQ

• • • • •

Phase III (Definitive Operation) • It consists of planned re-exploration & definitive repairQ of injuries • This phase typically occurs 48 & 72 hours following initial and after successful secondary resuscitationQ • Abdomen should be closed primarily if possible • Risky GI anastomoses or complex reconstruction should be avoidedQ

Stages of DCS Patient selectionQ Operative control of hemorrhage & contaminationQ ICU resuscitationQ Definitive surgeryQ Abdominal closureQ

ABDOMINAL COMPARTMENT SYNDROME Abdominal Compartment Syndrome • ACS is defined as increased intra-abdominal pressure (IAP >20 mm Hg) resulting in compression of abdominal structuresQ, producing fatal complications due to pulmonary failure and mesenteric vascular compromise. • Normal IAP = 5-7 mm Hg; Intra-abdominal hypertension IAP ≥ 12 mm Hg • ACS occurs predominantly in: −− Patients in profound shockQ −− Patients requiring large amounts of resuscitation fluids & bloodQ −− Those with major visceral or vascular abdominal injuriesQ

Section 10

• ACS is characterized by a sudden increase in IAP, increased peak inspiratory pressure, decreased urinary output, hypoxia, hypercapnia, & hypotension secondary to decreased venous return to the heartQ. Physiologic Consequences of Increased Intra-abdominal Pressure Decreased • • • • •

Cardiac outputQ Central venous returnQ Visceral blood flowQ Renal blood flowQ Glomerular filtration

Increased • • • • • •

Cardiac rateQ Pulmonary capillary wedge pressureQ Peak inspiratory pressureQ Central venous pressureQ Intrapleural pressure Systemic vascular resistanceQ

Trauma

1007

Diagnosis

Grade I II III IV

Bladder Pressure (mm Hg) 12–15 16–20 21–25 >25

Abdominal Compartment Syndrome Grading System Clinical Features None OliguriaQ, splanchnic hypoperfusion Anuria, increased ventilation pressure Anuria, increased ventilation pressure & decreased PO2Q

Treatment Normovolemic resuscitation Hypovolemic resuscitation Decompression Emergency re-exploration

Section 10

• Diagnosis is confirmed by measuring bladder pressure, which ultimately represents IAP. • A urinary bladder catheter is the gold standard indirect method used to measure IAP.

Treatment • Treatment includes rapid decompression of elevated IAP by opening the abdominal wound & performing a temporary closure of abdominal wall with mesh or a plastic bag (Bogota bag)Q.

RETROPERITONEAL INJURIES



The retroperitoneum is usually divided into three parts based on site of injury. Some authors also describe a 4th zone, i.e. the portal and retrohepatic area. Zone 1 (central) extends from the esophageal hiatus to the sacral promontory. Zone 2 (lateral) extends from the lateral diaphragm to the iliac crest. Zone 3 (pelvic) is confined to the retroperitoneal space of the pelvic bowl. Zones of Retroperitoneal Hematoma Extent Management • Extends from esophageal hiatus to sacral promontoryQ • Central hematoma should always be explored with proximal and distal vascular • Hematoma or hemorrhage in the midline associated with injuries to controlQ abdominal aorta, IVC, their proximal branches and tributariesQ. • Zone 1 is divided into supramesocolic and inframesocolic zone depending on origin of vascular structure in relation to transverse mesocolonQ. Supramesocolic Zone 1

Inframesocolic Zone 1

• Suprarenal aorta, IVC, coeliac • Infrarenal aorta & axis, proximal SMA and vein, and IVC including 
their proximal renal arteries and veinsQ. bifurcationsQ. 
 Zone 2 (lateral)

• Extends from lateral diaphragm to iliac crestQ • Include distal renal arteries and veinsQ

Zone 3 (pelvic)

• Confined to retroperitoneal space of pelvic bowlQ • Include iliac arteries and veinsQ

Zone 4

• Include portal and retrohepatic areaQ

• Lateral hematomas are usually renal in origin and 
can be managed nonoperativelyQ, sometimes with angioembolization. 
 • Pelvic hematomas are exceptionally difficult to control and should, whenever possible, not be openedQ • Should be controlled with packing (intra- or extrapelvic) and angioembolizationQ 


Others

Zone Zone 1 (central)

1008

Surgery Essence NECK INJURIES Neck Injuries • Most severe neck injuries are caused by penetrating wounds and may present an immediate threat to life as a result of airway compromise or hemorrhageQ. • Major vascular & aerodigestive structures in the neck are located in anterior triangle, & all are deep to the platysmaQ. • Platysma & SCM are useful anatomic boundariesQ.

Others

• Injuries that do not penetrate the platysma can be considered superficial, and no further investigation is needed. Wounds that penetrate the platysma must be further evaluated. • Injuries that are anterior to SCM present a high likelihood of significant injury, whereas those that track posterior to SCM are unlikely to involve major vascular or aerodigestive structures. • Penetrating injuries to the posterior triangle should raise concern about trauma to cervical spine & spinal cordQ. Zone I

Zone II Zone III

Neck is divided into Three Horizontal Zones on craniocaudal location • At thoracic inletQ • Extends from sternal notch to cricoid cartilageQ • Injuries in this zone carry the highest mortality because of the presence of great vesselsQ & difficult surgical approach. • Midportion of the neckQ • Extends from cricoid cartilage to angle of mandibleQ • Extends from angle of mandible to base of skullQ

Zones of the Neck

THORACIC INJURIES Thoracic Injuries • MC cause of mortality in blunt thoracic trauma: Tracheobronchial injuriesQ • MC cause of mortality in penetrating thoracic trauma: HemothoraxQ (secondary to pulmonary laceration) • MC thoracic injuries: Chest wall injuriesQ

RIB FRACTURE

Section 10

Rib Fracture • Occurs secondary to compression of thoracic cage in anteroposterior direction or lateral direction • Uncommon to have fracture of floating ribs (11th & 12th ribs) & first rib fractureQ • High velocity impact can cause fracture of 1st rib & 10th-12th ribs. • MC rib fractured during CPR: 4th-6th ribQ • In cases of 1st rib fracture, suspect injury of subclavian vessels, brachial plexus & apex of lungQ • In cases of 10th-12th rib fracture, on right side suspect injury to liverQ & on left side suspect injury to spleenQ (long axis of spleen is related to 10th rib) Management • Most rib fractures are managed conservatively with adequate analgesia. Strapping should not be doneQ

Trauma

1009

STERNAL FRACTURE

• Uncommon, typically transverse • Located in upper & mid portions of body of sternumQ Clinical Features • Tenderness, swelling & deformity at site of fractureQ • Sternal fracture is associated with severe intrathoracic injuries • Specific injury to myocardium is common Diagnosis • Diagnosed by chest X-rayQ (lateral view) Treatment

Section 10

Sternal Fracture Q

• Management is similar to rib fracture • Adequate analgesia & good pulmonary hygieneQ • Surgical intervention is recommended for displaced fractureQ

PERICARDIAL TAMPONADE Pericardial Tamponade • Rapid accumulation of blood in pericardial space • Quantity of fluid: Even 200 mL fluid which collets rapidly can lead to pericardial tamponadeQ • Caused by penetrating traumaQ Clinical Features Q

• Characterized by Beck’s triad (MDH): Muffled heart sounds + Distended neck veins + HypotensionQ Diagnosis • IOC for diagnosis: EchocardiographyQ • Chest X-ray: Enlarged heart shadowQ Treatment • Emergency treatment: Needle pericardiocentesisQ • TOC: Surgical pericardiotomyQ

FLAIL CHEST

• A flail chest occurs when a segment of the chest wall does not have bony continuity with the rest of the thoracic cageQ. • This condition usually results from blunt trauma associated with multiple rib fractures, i.e. two or more consequetive ribs fractured in two or more placesQ. • The blunt force required to disrupt the integrity of the thoracic cage typically produces an underlying pulmonary contusion as well. Clinical Features • On inspiration the loose segment of the chest wall is displaced inwards and less air therefore moves into the lungs (Paradoxical respiration)Q • To confirm the diagnosis the chest wall can be observed for paradoxical motion of a chest wall segmentQ for several respiratory cycles and during coughing.

• Voluntary splinting as a result of pain, mechanically impaired chest wall movement and the associated lung contusion are all causes of the hypoxia & respiratory failureQ. • The patient is also at high risk of developing a pneumothorax or hemothoraxQ.

Others

Flail Chest

1010

Surgery Essence Diagnosis • Diagnosis is made clinicallyQ, not by radiography. Treatment • Chest strapping or splinting should be avoidedQ. • Currently, treatment consists of oxygen administration, adequate analgesia (including opiates) or epidural analgesia & physiotherapyQ. • IPPV (Intermittent positive pressure ventilation) is reserved for cases developing respiratory failure despite adequate analgesia & oxygenQ.

DIAPHRAGMATIC INJURIES

Others

Diaphragmatic Injury • Diaphragmatic injuries are often caused by penetrating injuriesQ. • Patients sustaining penetrating injuries below the nipples and above the costal margins should be investigated to rule out diaphragmatic injuryQ. Etiology • Penetrating trauma (knife, bullet, repair of hiatus hernia) • Blunt trauma (motor vehicle accident, fall from height, bout of hyperemesis): −− Caused by compressive force applied to the pelvis and abdomen. −− Rupture is usually large, with herniation of abdominal content into chest Clinical Features • Most diaphragmatic injuries are silent and the presenting features are those of injury to the surrounding organsQ. • Late complication: Herniation of abdominal contents in to the chestQ. • Herniation of organ: StomachQ >Colon >Small intestine >Omentum >Spleen >Kidney and pancreas. Diagnosis • There is no single standard investigation to diagnose diaphragmatic injuriesQ. • Chest X-ray after placement of a nasogastric tune may be helpful (as this may show the stomach herniated into the chest) • Contrast study of upper or lower GIT, CT scan and diagnostic peritoneal lavage all lack positive or negative predictive value. • Most accurate evaluation is by video assisted thoracoscopy (VATS) or laparoscopyQ, offering the advantage of allowing the surgeon to proceed to repair and additional evaluation of the abdominal organs. Treatment • Operative repairQ is recommended in all cases. • All penetrating diaphragmatic injury must be repaired via the abdomen and not the chest, to rule out penetrating hollow viscus injury. Bergvist Triad: Rib fracture + Fracture of spine/pelvis + Traumatic rupture of diaphragm

BLAST INJURIES Blast Injuries

Section 10

• Primary blast injuries result from the rapid overpressure or shock waves produced by an explosion • These injuries result from the dramatic changes in barometric pressure projected from the point of detonation • Primary blast injuries predominantly cause damage to air filled hollow organs of the body from rapid pressure change (barotraumas). • Damage to air filled organs includes middle ear, lungs & GIT.Q • Most sensitive & most frequently injured hollow organ: Tympanic membraneQ > Lungs • Blast damage to the lungs is the MC cause of life threatening injuryQ following an explosion. Most Severely Affected Organs

Most Commonly Affected Organs

Air Blast: Lungs Underwater: GITQ

Air: Tympanic membraneQ Underwater (Fully submerged): TMQ Underwater (Head is out): GITQ

Q

Trauma

1011

CHEST TUBE INSERTION

• ICD insertion is done in triangle of safety in 5th intercostal space in anterior axially lineQ. • Insertion is done over upper border of lower ribQ & all eyes of drain should be inside pleural spaceQ

Section 10

Intercostal Drain (ICD) Insertion

Boundaries of Triangle of Safety Anteriorly

• Lateral border of pectoralis majorQ

Laterally

• Lateral border of latissimus dorsiQ

Inferiorly

• Line of 5th intercostal spaceQ

Superiorly

• Base of axillaQ

• Used to provide evacuation of air or fluid from pleural spaceQ • Under water seal chamber: Most important element in pleural drainage; Acts as one-way valve for evacuation of pleural contents; When intra-pleural pressure rise, free contents of pleural space are forced out through chest tube into under water sealQ. • Re-entry of air into pleural space is blocked by underwater sealQ • Water in the tube is known as water column. The water column should be movingQ. • Tip of tube should be 2-3 cm below the surface of waterQ. Follow-up • Patency of chest tube is assessed by observing oscillations in water seal chamber with respiratory movementsQ. • Position of chest tube & resolution of intrapleural air or liquid is checked by chest X-rayQ • Column movement stops in blocked tube or displaced tubeQ. • Excessive bubbling in tube suggest bronchopleural fistulaQ Removal • When lung expands & air or fluid drainage is 15 cm of H2O b. Decreased pulmonary venous pressure c. IAP Measured using Foley’s catheterization of bladder d. With >25-30 mm Hg IAP, life threatening hypoxia and ARDS can occur 75. Abdominal compartment syndrome is characterized by the following except: (UPSC 2007) a. Hypercarbia and respiratory acidosis b. Hypoxia due to increased peak inspiratory pressure c. Hypotension due to decrease in venous return d. Oliguria due to ureter obstruction 76. Increased intra abdominal pressure is/are associated with: a. ↑ Pulmonary capillary wedge pressure (PGI June 2004) b. ↑ Venous return c. ↑ Pulmonary inspiratory pressure d. ↑ Renal blood flow e. ↑ Cardiac output 77. True about abdominal compartment syndrome: a. ↓ Cardiac output (PGI Dec 2008) b. ↓ Urine output c. ↓ Venous return d. ↓ Systemic vascular resistance 78. Which one of the following statements is true regarding Abdominal Compartment Syndrome? (APPG 2016) a. Reduction in visceral perfusion and increase in intracranial pressure b. diagnosis can be reliably made by physical examination c. The intraadbominal pressure in grade I ACS is 21-35 cm H320 d. Definitive surgery is immediately done followed by closure of abdomen 79.

Which of the following is best treatment for Grade II abdominal hypertension? (Recent Question 2016) a. Laparotomy b. Immediate decompression c. Hypovolemic resuscitation d. Normovolemic resuscitation

PENETRATING INJURIES 80.

Organ most commonly damaged in penetrating injury of abdomen is: (WBPG 2014, AIIMS Nov 94, Nov 95) a. Liver b. Small intestine c. Large intestine d. Duodenum

Others

71.

Damage control surgery is: (JIPMER 2014, AIIMS May 2013) a. Minimal intervention done to stabilize the patient and do the definitive surgery later b. Maximum possible surgical intervention is done immediately c. Done during triage procedure d. Done to control damage during surgery Where is the second step of damage control resuscitation carried out? (AIIMS May 2018) a. In emergency b. In ICU c. In OT d. Prehospital resuscitation

Section 10

64.

All the following are True regarding fast–except: a. It is a focused abdominal sonar for trauma  (APPG 2015) b. It is accurate in detecting < 50 ml of free blood c. It cannot reliably exclude injury in penetrating trauma d. It detects free fluid in the abdomen or pericardium

1015

1016

Surgery Essence 81.

Treatment of choice for stab injury caecum: a. Cecostomy b. Ileo-transverse anastomosis c. Transverse colostomy d. Sigmoid colostomy

(All India 89)

A man comes to emergency with stab injury to left flank. He has stable vitals. What would be the next step in manage­ ment: (AIIMS Nov 2008) a. CECT b. Diagnostic peritoneal lavage c. Laparotomy d. Laparoscopy

83.

A patient with stab injury to anterior abdomen presents with a tag of omentum protruding through the abdominal wall near the umbilicus. On evaluation he is hemodynamically stable and shows no signs of peritonitis. Initial management of patient should involve: (All India 2011) a. FAST b. Exploratory Laparotomy c. Local Wound Exploration and Suturing d. CECT Abdomen

Others

82.

84.

NECK INJURIES

Section 10

91.

Which of the following is used to define penetrating neck injury? (AIIMS May 2009, All India 2008) a. 2 cm depth of wound b. Injury to vital structures c. Breach of platysma d. Through and through wound

87.

The probable cause of sudden death in a case superficial injury to neck is: (DNB 2005) a. Injury to phrenic nerve b. Air embolism through external jugular vein c. Bleeding from subclavian artery d. Injury to trachea

88.

Zone of neck involving great vessels at thoracic inlet: a. I b. II (Recent Question 2016) c. III d. IV

BLAST INJURIES 89.

In a blast injury, which of the following organ is least vulnerable to the blast wave? (AIIMS June 2003) a. GI tract b. Lungs c. Liver d. Ear drum

90.

Most common organ injured in underwater explosion:  (MHSSMCET 2009) a. TM b. GIT c. Lungs d. Heart

A 17-year-old boy is admitted to the hospital after a road traffic accident. Per abdomen examination is normal. After adequate resuscitation, his pulse rate is 80/min and BP is 110/70 mm Hg. Abdominal CT reveals 1 cm deep laceration in the left lobe of the liver extending from the done more than half way through the parenchyma. Appropriate management at this time would be: (DPG 2011, UPSC 2005) a. Conservative treatment b. Abdominal exploration and packing of hepatic wounds c. Abdominal exploration and ligation of left hepatic artery d. Left hepatectomy

SPLENIC INJURIES 92.

A 30-year-old gentleman after sustaining road traffic accident present in emergency with BP 100/60 mmHg, Pulse 120 min and CT scan shows splenic laceration at inferior border after 2 units of blood transfusion, patients conditions are: BP 120/70 mmHg and pulse 84/min; the next line of management is: (PGI June 2003) a. Laparotomy b. Splenorrhaphy c. Continue the conservative treatment and take subsequent measures on monitoring the patient d. Splenectomy e. X-ray abdomen and aspiration

93.

A child presents in causality in stable condition after a blunt abdominal trauma associated with splenic trauma. Treatment of choice is:  (Recent Question 2016, AIIMS Nov 2000) a. Observation b. Splenectomy c. Arterial embolisation d. Splenorrhaphy

94.

True about blunt abdominal trauma with splenic rupture:  (PGI June 2008) a. Kehr’s sign-discoloration around umbilicus b. Spleen is most common organ to be involved c. Splenectomy is treatment of choice for splenic rupture d. Cullen’s sign seen A 30-year-old person met with a roadside accident. On admission his pulse rate was 120/minute, BP was 100/60 mmHg. USG examination revealed laceration of the lower pole of spleen and hemoperitoneum. He was resuscitated with blood and fluid. Two hours later, his pulse was 84/ minute and BP was 120/70 mm Hg. The most appropriate course of management in this case would be: (DPG 2011) a. Exploring the patient followed by splenectomy b. Exploring the patient followed by excision of the lower pole of spleen c. Splenorrhaphy d. Continuation of conservative treatment under close monitoring system and subsequent surgery if further indicated About trauma spleen false is: (DPG 2006) a. Partial splenectomy cannot be done b. Post splenectomy infection common c. Can cause late onset shock d. Mostly managed conservatively Trauma to spleen in a stable patient is best diagnosed by: a. X-ray abdomen (MCI Sept 2005, March 2008) b. USG c. CT scan d. Diagnostic peritoneal lavage

Which of the following is not done in case of puncture wound of left colon? (Recent Question 2015) a. Primary suture b. Hemicolectomy c. Externalization d. Resection and anastomosis

85. In the patient of penetrating injury to the abdomen with shock, next best step is: (Recent Question 2017) a. USG b. FAST c. CT d. Laparotomy

86.

HEPATIC INJURIES

95.

96.

97.

Trauma In a RTA patient sustained trauma to left side of chest and abdomen. Fluid in the peritoneum and sign of hypotension was found on physical examination. Most probable diag­ nosis is: (DNB 2014) a. Splenic injury b. Diaphragmatic injury c. Rib fracture d. Renal injury

99.

A 27 years old patient presented with left sided abdominal pain 6 hours after RTA. He was hemodynamically stable and FAST positive. CT scan showed grade III splenic injury. What will be appropriate treatment? (Recent Question 2015) a. Splenectomy b. Splenorrhaphy c. Splenic artery embolization d. Conservative management

STOMACH, DUODENUM AND PANCREATIC INJURIES 100. Which of the following statements related to gastric injury is not true? (All India 2007) a. Mostly related to penetrating trauma b. Treatment is simple debridement and suturing c. Blood in stomach is always related to gastric injury d. Heals well and fast 101. A young patient presents with a massive injury to proximal duodenum, head of pancreas and distal common bile duce. The procedure of choice in this patient should be: a. Roux-en-Y anastomosis (All India 2008) b. Pancreaticoduodenectomy (Whipple’s operation) c. Lateral tube jejunostomy d. Retrograde jejunostomy

CHEST TRAUMA 102. A patient died after a blunt trauma to chest. Most common cause of death in blunt trauma to chest is: (Recent Question 2016) a. Esophageal rupture b. Tracheo-bronchial rupture c. Pulmonary laceration d. Pneumothorax

104. A 40-year-old man brought to the emergency room with a stab injury to the chest. On examination patient is found to be hemodynamically stable. The neck veins are engorged and the heart sounds are muffled. The following statements are true for this patient except: (AIIMS Nov 2002) a. Cardiac tamponade is likely to be present b. Immediate emergency room thoracotomy should be done c. Echocardiogram should be done to confirm pericardial blood d. The entry wound should be sealed with an occlusive dressing 105. Following a major trauma a patient presented 54 hours later with raised JVP and CVP of 16 mm of Hg and persistent hypotension. Most probable diagnosis is:  (PGI Dec 2000, Dec 2003) a. Tension pneumothorax b. Cardiac tamponade c. Head injury d. Splenic trauma e. Air embolism

107. Sitaram a 40-year-old man, met with an accident and comes to emergency department with engorged neck veins, pallor, rapid pulse and chest pain. Diagnosis is: (AIIMS June 99) a. Pulmonary laceration b. Cardiac tamponade c. Hemothorax d. Splenic rupture 108. A patient is brought to casualty with severe hypotension following a road traffic accident. No external injury is evident. The cause of hypotension is: (AIIMS June 2001) a. Fracture rib b. Intrathoracic and abdominal bleed c. Iatrogenic shock d. Intracranial bleed 109. Which of the following is most common cause of hypotension in fracture ribs (T10 -T12)? (AIIMS Nov 99, June 99) a. Abdominal solid visceral organ injury b. Injury to aorta c. Inter costal artery damage d. Pulmonary contusion 110. Treatment of acutely developing massive left sided hemothorax in a young male after an accident is: a. Strapping of chest  (AIIMS Nov 93) b. Tube thoracostomy c. Endotracheal intubation + IPPV + pleural fluid aspiration d. Conservative, wait and watch 111. Treatment of rib fracture: a. Immediate thoracotomy b. IPPV c. Strapping d. ICWSD

(PGI Dec 2002)

112. Treatment of simple rib fracture include all of the following except: (Recent Question 2015) a. Analgesic b. Physiotherapy c. Strapping d. Early ambulation 113. In case of blunt injury thorax, most common complication is:  (AIIMS June 95) a. Pneumothorax b. Rib fracture c. Hemopneumothorax d. Aortic rupture 114. Best approach in thoracic trauma is: (Recent Question 2013) a. Midline sternotomy b. Parasternal thoracotomy c. Anterolateral thoracotomy d. Posterolateral thoracotomy

FLAIL CHEST 115. True about flail chest: (PGI June 2004) a. Fracture of 3 or 4 ribs b. Chest wall moves inwards during inspiration c. Mechanical Ventilation always needed d. Mediastinal shift e. Ultimately leads to respiratory failure 116. What is the treatment of choice in severe fail chest? a. IPPV b. Strapping (UPSC 2008) c. Wiring d. Nasal Oxygen

Others

103. Commonest cause of death in penetrating injury of chest: a. Tracheobronchial injury  (AIIMS Sept 96, June 2000) b. Esophageal rupture c. Pulmonary laceration d. Chylothorax

106. A male came to the emergency room after car accident. He had dyspnea and chest pain with ecchymosis on anterior chest wall. On examination, pulse rate was 120/min, BP was 80/50 mmHg, the breaths sounds were decreased on left side, JVP was raised and tympanic note was present on percussion. Pelvis and extremities were normal. What is your diagnosis?  (AIIMS May 2018) a. Tension pneumothorax b. Massive hemothorax c. Cardiac tamponade d. Hydropneumothorax

Section 10

98.

1017

1018

Surgery Essence 117. Steering wheel injury on chest of a young man reveals multiple fractures of ribs and paradoxical movement with severe respiratory distress. X-ray shows pulmonary contusion on right side without pneumothorax. What is the initial treatment of choice? (UPSC 2007) a. Immediate internal fixation b. Endotracheal intubation and mechanical ventilation c. Thoracic epidural analgesia and O2 therapy d. Stabilization with towel clips

Others

118. Management of flail chest with respiratory failure is: a. Chest tube drainage (DNB 2008, MCI Sept 2006) b. Oxygen administration c. IPPV d. Internal operative fixation of the fractures segments 119. Simple rib fracture should be treated with all except: (Recent Question 2014, MHPGMCET 2007) a. Analgesics b. Physiotherapy c. Early ambulation d. Strapping of chest 120. A man presented with fractures of 4th to 10th ribs and respiratory distress after RTa. He is diagnosed to have flail chest and a PaO2 of 10 secondsQ. • For patients with ongoing CPR and an advanced airway in place, a simplified ventilation rate of 1 breath every 6 secondsQ (10 breaths per minuteQ) is recommended. • Routine use of impedance threshold device (ITD) as an adjunct to conventional CPR is not recommendedQ. • In ACLS (Advanced Cardiac Life Support), vasopressin does not offer an advantage over the use of epinephrine aloneQ. Therefore, vasopressin has been removed from the Adult Cardiac Arrest Algorithm-2015 Update. In 2010 update, atropine was removedQ (earlier vasopressin was recommended as an alternative to epinephrine). • Low end-tidal carbon dioxide (ETCO2) in intubated patients after 20 minutes of CPR is associated with a very low likelihood of resuscitationQ.

Trauma

• During adult CPR tidal volume of 600 ml (6-7 ml/kg) should be adequate to cause the chest to riseQ. • During airway management in an unconscious trauma patient with possible cervical injury, neck hyperextension should be avoided. • The most widely used waveform in the automated electrical defibrillators (AEDs) now is the biphasic truncated exponential (BTE) waveform. • The following drugs may be given through the endotracheal tube during CPR: LignocaineQ, EpinephrineQ, VasopressinQ, AtropineQ, NaloxoneQ (LEVAN); (Amiodarone & sodium bicarbonate are not given endotracheallyQ) • Atropine is not recommended for routine use in the management of Pulseless Electrical Activity/asystole and has been removed from the ACLS Cardiac Arrest AlgorithmQ (Epinephrine, vasopressin & amiodarone are usedQ) Adult (>12 years)



Child (1-12 years)

Compression Depth

At least 2 inchesQ (5 cm, but not About 2 inchesQ (5 cm; >2.4 inches/6 cmQ) 1/3rd of chest depthQ)

Compression: Ventilation ratio

30:2Q (one or two rescuer CPR)

Compression rate

100-120/minQ

Section 10

• Emergency coronary angiography is recommended for all patients with ST elevation and for hemodynamically or electrically unstable patients without ST elevation for whom a cardiovascular lesion is suspectedQ.

1025

Infant (HLA-B >HLA-A). • Are the MC cause of graft rejectionQ • Their physiological function is to act as antigen recognition units • Are highly polymorphic (amino acid sequence differs widely between individuals) • Anti-HLA antibodies may cause hyperacute rejectionQ • In the case of liver transplants, HLA matching does not confer an advantageQ • Although it is beneficial in cardiac transplantation, it is not practicable because of the relatively small size of the recipient pool and the short permissible cold ischemic timeQ.

1036

Surgery Essence TYPES OF GRAFT Graft Autograft

Isograft

• Tissue transplanted from one site to another on the same patientQ

• Transplant from a geneti­ cally identical donor, such as an identical twinQ

Homograft (Allograft) • Transplant from individual of same speciesQ

Heterograft (Xenograft) • Transplant from another speciesQ

Xenograft

Others

Concordant Xenograft • • • •

Discordant Xenograft

Transplant between closely related speciesQ Example: For humans, old world monkeys & apes Advantage: Hyperacute rejections is not a threatQ Disadvantages: Zoonotic transfer of disease (Particularly retroviral transmission)

• Transplant between distant related or divergent speciesQ • Example: For humans, new-world monkeys & other mammals • For physiologic concern (organ size & availability), pigs are preferred animal donorQ • Disadvantages: High risk of hyperacute rejectionQ

COMPLICATIONS OF IMMUNOSUPPRESSION Complications of Immunosuppression Infection

Malignancy

• High risk of opportunistic infection by virusesQ • Recipient derived infectionsQ are more common than donor derived infections • Risk of bacterial infection is highest during first monthQ of transplantation • Risk of viral infection is highest during first 6 monthsQ of transplantation; MC problem is CMVQ Infection • Viral infection may result from reactivation of latent virus or from primary infection • Chemoprophylaxis is important in high risk patients • Pre-transplant vaccination against community acquired infection should be considered

• MC malignancy in transplant recipient: Skin cancerQ (SCC) • Increased risk of PTLD & Kaposi sarcoma

Non-Immune Side Effects • Hypertension & chronic allograft nephropathy caused by calcineurin inhibitorsQ • New onset diabetes after transplant is associated with tacrolimus or steroidsQ • Hyperlipidemia, anemia & accelerated cardiovascular diseaseQ • Cardiovascular disease is the leading cause of death in transplant survivorsQ

Common Infections After Solid Organ Transplantation, by Site of Infection Period after Transplantation

Section 10

Infected Site

Early (6 Months)

Donor organ

Bacterial and fungal infections of CMV infectionQ the graft, anastomotic site, and surgical wound

EBV infectionQ (may present in allograft organ)

Systemic

Bacteremia and candidemia (of­ ten resulting from central venous catheter colonization)

CMV infectionQ (fever, bone marrow suppression)

CMV infectionQ, especially in patients given early posttransplantation prophylaxis; EBV proliferative syndromes (may occur in donor organs)

Lung

Bacterial aspiration pneumonia with prevalent nosocomial organ­ isms associated with intubation and sedation (highest risk in lung transplantation)

Pneumocystis infectionQ; CMV pneumoniaQ (highest risk in lung transplantation); Aspergillus infectionQ (highest risk in lung transplantation)

Pneumocystis infectionQ; granuloma­ tous lung diseases (nocardiae, reactivat­ ed fungal and mycobacterial diseases)

Kidney

Bacterial and fungal (Candida) infections (cystitis, pyelonephritis) associated with urinary tract catheters (highest risk in kidney transplantation)

Renal transplantation: BK virus infec- Renal transplantation: Bacteria (late tion (associated with nephropathyQ); urinary tract infections, usually not as­ JC virus infection sociated with bacteremia); BK virus (nephropathyQ, graft failure, generalized vasculopathy)

Liver and biliary tract

Cholangitis

CMV hepatitisQ

CMV hepatitisQ Contd…

Transplantation

1037

Contd… Early (6 Months)



Toxoplasma gondii infectionQ (highest risk in heart transplantation)

Toxoplasma gondii infectionQ (highest risk in heart transplantation)

Gastrointestinal tract

Peritonitis, especially after liver transplantation

Colitis secondary to Clostridium difficileQ Colitis secondary to C. difficile infection infection (risk can persist) (risk can persist)

Central Nervous System



Listeria (meningitis); T. gondii infection

Listeria meningitisQ; Cryptococcus meningitis; Nocardia abscess; JC virus-associated PML

Section 10

Infected Site Heart

CMV INFECTION CMV Infection • CMV is most important pathogen in clinical transplantationQ. • CMV infections usually occur after 30-50 daysQ after transplantation. Clinical Features • Fever, malaise, arthralgia, leukopenia and thrombocytopenia, hepatitis, interstitial pneumonitisQ, enterocolitis and disseminated disease. Diagnosis • Invasive CMV infection with histologic evidence or a positive CMV culture from deep tissue specimens is confirmatoryQ • Chest X-ray: Bilateral diffuse interstitial pneumoniaQ Treatment • IV ganciclovirQ is the mainstay of treatment and is safe and effective for prophylaxis and treatment.

BK VIRUS INFECTION BK Virus

POST-TRANSPLANT LYMPHOPROLIFERATIVE DISORDER Post-Transplant Lymphoproliferative Disorder (PTLD) • PTLD is associated with replication of EBV in B cells induced by enhanced immunosuppression, primarily observed in patients who have received more than one course of polyclonal antilymphocyte globulin (ALG) or monoclonal OKT3Q. Clinical Features • Clinical presentation of PTLD includes fever, malaise and lymphadenopathyQ Diagnosis • The diagnosis is made by tissue biopsyQ. Treatment • Polyclonal PTLD: Discontinuation of immunosuppression and antiviral therapyQ. • Monoclonal PTLD: Radiation, chemotherapy and occasionally surgical resection. Antibody against CD20Q represents a novel approach in treating monoclonal PTLD with favorable outcome.

Others

• BK virus is member of family Polyoma virus, associated with nephropathy, typically after 1-4 moths after transplant. • High levels of BK virus replication detected by PCR in urine and blood are predictive of pathology, especially in setting of renal transplantation. • Urinary excretion of BK virus and BK viremia are associated with the development of ureteric strictures, Polyoma virus associated nephropathy (1-10% of renal transplant recipients), and (less commonly) generalized vasculopathy. • Timely detection and early reduction of immunosuppression are critical and can reduce rates of graft loss related to Polyoma virus associated nephropathy from 90% to 10-30%.

1038

Surgery Essence CLINICAL TESTING OF BRAINSTEM DEATH Clinical Testing for Brainstem Death

Others

• Absence of cranial nerve reflexes: (PCO) −− Pupillary ref lexQ −− Pharyngeal (Gag) & Tracheal (Cough) ref lexQ −− Corneal ref lexQ −− Oculovestibular (Caloric) reflexQ • Absence of motor response: −− Absence of motor response to painful stimuli applied to head & face plusQ −− Absence of motor response within the cranial nerve distribution to adequate stimulationQ • Absence of spontaneous respirationQ

DONORS Organ Donors • Organ donors are of two types: Dead or deceased donors & living donors Dead or Deceased Donors

Living Donors

• Brain dead donors also known as heart beating donors or donation after brain deathQ (DBD) • Cardiac or circulatory dead also known as non-heart beating donors or donation after circulatory deathQ (DCD)

• Limited to donation of: −− KidneyQ −− LiverQ −− Lung lobeQ

Donation after Brain Death (DBD) Donors

Donation after Circulatory Death (DCD) Donors

• Most deceased donors organs are obtained from patients with brain-stem deathQ. • Brain death occurs when severe brain injury causes irreversible loss of capacity of consciousness combined with irreversible loss of capacity for breathingQ.

• Due to rising demand for organ transplantation,

increase in the use of organs from DCD donors • DCD donors are grouped according to Maastricht classificationQ

Maastricht Classification for Donation after Circulatory Death (DCD) Donors Category

Description (DRACUla)

1

Dead on arrival at hospital

2

Resuscitation attempted without successQ

3

Awaiting cardiac arrest after withdrawal of supportQ (Most DCD donors from Category 3Q)

4

Cardiac arrest while brain deadQ

5

Cardiac arrest & Unsuccessful resuscitation in hospitalQ Uncontrolled Donors • Includes category 1, 2 & 5 • Warm ischemic time is longer & less predictableQ

Section 10

Q

Q

Controlled Donors • Includes category 3 & 4Q • Death results from planned withdrawal of life-sustaining cardiorespiratory supportQ • Most DCD donors are from controlled donorsQ

EXTENDED CRITERIA DONORS (ECD) FOR KIDNEY & LIVER TRANSPLANTATION Extended Criteria Donors Kidney Transplant Liver Transplant • Donor >60 years of ageQ • Mild to moderate steatosisQ • Donor age 50-59 years with at least two of the following: • Hepatitis C positiveQ Q −− Cerebrovascular accident as cause of death • Hepatitis B—core antibody positiveQ −− Pre-existing hypertensionQ −− Terminal serum creatinine >1.5 mg/dLQ

Transplantation

1039

ORGAN PRESERVATION SOLUTION

• Liver, pancreas and kidney can be successfully preserved for up to 2 days by flushing the organ with University of Wisconsin solution and storing them at hypothermia (0-5°C)Q.

University of Wisconsin Solution • UW solution: Cationic composition (high potassium and low sodium) mimics intracellular levelsQ to minimize diffusion down electrochemical gradients. • UW solution (marketed as Viaspan) contains high level of potassium and adenosineQ.

Section 10

Organ Preservation Solution

Special composition of UW solution Lactobionate and raffinose

• Minimizes cell swellingQ

Hydroxyethyl starch

• Prevention of the extracellular space expansionQ

Glutathione

• Anti-oxidantQ

Allopurinol

• Free radical scavengerQ

Adenosine

• Precursor for energy metabolismQ

MAXIMUM & OPTIMAL COLD STORAGE TIME Organ Kidney Liver Pancreas Small intestine Heart Lung

Maximum and Optimal Cold Storage Times Optimal storage time (hours) Safe maximum storage time (hours) Necrotizing enterocolitisQ • MC indication of SBT in adults: Mesenteric ischemiaQ >Crohn’s diseaseQ > TraumaQ Q

Indications of Small Bowel Transplantation Short bowel syndrome patients who experience: • IFALD (Intestinal Failure Associated Liver Disease) • Parenteral nutrition failure • Recurrent catheter related infections • Thrombosis of 2 of 6 major central access veins

• Alteration in growth & development of children • Severe dehydration & refractory electrolyte changes • Impending liver failure • Established liver disease with cirrhosis & portal hypertension

Donor Selection • Ideal donor should have body weight of 50-75% of the recipientQ • Viral serologic testing of cadaveric donor for EBV & CMV is important (Two MC viral infections after SBTQ) Types of Intestinal Transplantation • Small bowel with or without a portion of colon

• Combined liver & small bowel grafts

• Multi-visceral graft

1044

Surgery Essence Procedure (Isolated Intestinal Transplant) • • • • • • •

Graft: Isolated intestine allograft include entire jejunum & ileum with associated vasculature (SMA & SMVQ) Donor jejunum is divided just distal to ligament of Treitz & ileum is transected proximal to ileocecal valveQ SMA & SMV are divided at mesenteric root at inferior border of pancreasQ In recipients operation, donor SMV is anastomosed to recipient’s portal vein & donor SMA is anastomosed to infra-renal aortaQ. Bowel continuity is established proximally & distally by standard techniques of enteric anastomosis Graft jejunum is anastomosed to recipients duodenum & distal ileum is brought as an ileostomyQ Distal ileostomy is created for routine monitoring of graftQ

COMPLICATIONS OF SMALL BOWEL TRANSPLANTATION

Others

Complications of Small Bowel Transplantation • • • •

Technical complications: Bowel anastomotic leaks, intestine perforations & wound Complications MC complication following intestinal transplant: Bacterial infection (E. coli & KlebsiellaQ) > Viral Infection (CMV InfectionQ) Incidence of PTLD: SBT recipientQ (10-20%) > Lung-Heart recipient (5-10%) > Liver recipient (2-5%) > Kidney recipient (1-2%) GVHD: Incidence in SBT 0-14% (Because of large amount of lymphoid tissue in small intestine) • MC cause of graft loss in SBT: Sepsis >RejectionQ • MC cause of death after SBT: SepsisQ

HEART TRANSPLANTATION Heart Transplantation • First HT was performed by Christian BarnardQ in 1967. Indications of Heart Transplantation • Ischemic dilated cardiomyopathy secondary • Valvular heart diseaseQ to CADQ (MC) • MyocarditisQ • Idiopathic dilated cardiomyopathyQ • Congenital heart diseaseQ • Heart transplantation is considered in the patients with end-stage heart disease that: −− Failed to respond to all other conventional therapyQ −− Predicted survival without transplantation is only 6-12 monthsQ −− Patient 60 years Irreversible pulmonary hypertensionQ • Significant pulmonary vascular disease MalignancyQ • Active duodenal ulceration Other life-threatening illnessQ • Creatinine clearance Acute rejectionQ > Accelerated CADQ

Transplantation

1045

LUNG TRANSPLANTATION

• First lung transplantation was performed by Fritz DeromQ in1968 Indications of Lung Transplantation (P-CAB) • Pulmonary fibrosisQ • Primary pulmonary hypertensionQ • Cystic fibrosisQ

• COPDQ • Alpha-1 antitrypsin deficiencyQ • BronchiectasisQ

Types of Lung Transplantation Single Lung Transplantation (SLT) • Benefits two recipients from single donor • Advantages: Shorter extubation time; Less need for cardiopulmonary bypass; Shorter hospitalization • Disadvantages: −− Quality of life & success rate is lesser as compared to DLTQ −− Lung hyperinflation is a common complication following SLT. −− Graft compression by hyperinflated native lung can cause mediastinal shift & respiratory failureQ

Section 10

Lung Transplantation

Double Lung Transplantation (DLT) • Preferred surgical procedure for patients with cystic fibrosis & bronchiectasisQ (Because infection remains in the native lung, if both lungs are not transplanted) • Preferred over SLT due to better quality of life & higher success rateQ

Procedure • SLT: Performed through posterolateral thoracotomyQ image on left side • DLT: Performed through bilateral thoracotomy or median sternotomyQ • Sequence of anastomoses in lung transplantation (PV By BA PA): Pulmonary Vein → Bronchial Anastomosis → Pulmonary ArteryQ • Cardiopulmonary bypass is usually required if pulmonary hypertension is present Complications of Lung Transplantation • Dehiscence of airway anastomosisQ • Late airway stenosis at bronchial anastomosis due to ischemia: Treated by dilatationQ

Others

Multiple Choice Questions FLUIDS USED IN TRANSPLANTATION 1. Amputated digits are preserved: (AIIMS GIS Dec 2011, All India 92) a. Cold saline b. Cold Ringer Lactate c. Plastic bag in ice d. Deep freezer 2. Allopurinol is used in organ preservation as: a. Antioxidant (AIIMS May 2009) b. Preservative c. Free radical scavenger d. Precursor for energy metabolism

GRAFT

3. Kidney transplantation is an: (DNB 2009, COMEDK 2006) a. Allograft b. Isograft c. Xenograft d. Synergic graft



4. If mother is donating the kidney to her son, this is an example of:  (Recent Question 2019, MCI June 2018) a. Autograft b. Allograft c. Isograft d. Xenograft



5. Transplantation between genetically different members of the same species is termed as: (COMEDK 2009) a. Autograft b. Isograft c. Allograft d. Xenograft



6. A kidney transplant between identical twins is an example of: (Recent Question 2017, COMEDK 2011) a. Isograft b. Allograft c. Autograft d. Xenograft



7. Graft from sister to brother is: (JIPMER 90) a. Isograft b. Allograft c. Autograft d. Heterograft 8. An isograft indicates transfer of tissues between: (All India 93) a. Unrelated donors b. Related donors c. Mynozygotic twins d. From the same individual









9. Skin grafting done on wound following major skin taken from twin brother: (Recent Question 2015, 2013) a. Isograft b. Allograft c. Autograft d. Xenograft 10. Concordant xenograft is: (Recent Question 2017) a. Between closely related different species b. Between same species of different races c. Between same species d. Between non-identical twins 11. Least chance of recipient failure in transplant is seen in: (Recent Question 2018) a. Allograft b. Isograft c. Xenograft d. Heterotopic graft

MATCHING AND GRAFT REJECTION

12. Acute cellular rejection following solid organ transplantation occurs: (COMEDK 2011) a. Within minutes to hours of transplantation b. Within 48 hours of transplantation c. Between 5 to 30 days of transplantation d. Beyond 30 days after transplantation



13. Hyperacute rejection is due to: (Recent Question 2016, AIIMS Nov 2012) a. Preformed antibodies b. Cytotoxic T-lymphocyte medicated injury c. Circulating macrophage mediated injury d. Endothelitis caused by donor antibodies



14. Most important HLA for organ transplantation and tissue typing: (Recent Question 2016, MAHE 98) a. HLA-A b. HLA-B c. HLA-C d. HLA-D



15. HLA matching is not necessary in which of the following organ transplantation? (JIPMER 2002) a. Liver b. Bone marrow c. Pancreas d. Kidney 16. Immunological rejection is mediated by recipients: a. Eosinophils b. Lymphocytes c. Neutrophils d. Plasma cells 17. Transplantation of which one of the following organs is most often associated with hyper-acute rejection? a. Heart b. Kidney (UPSC 2006) c. Lungs d. Liver 18. Hyperacute rejection of graft is seen in? (MHSSMCET 2006, 2008, All India 2003) a. Lung b. Liver c. Kidney d. Pancreas

KIDNEY TRANSPLANTATION



19. Renal transplantation is most commonly done in: a. Chronic glomerulonephritis (PGI Dec 97) b. Bilateral staghorn calculus c. Horse shoe kidney d. Oxalosis 20. Commonest malignancy in renal transplant recipient is: (AIIMS Nov 95) a. Skin cancer b. Renal cell carcinoma c. Non-Hodgkin’s lymphoma d. Hodgkin’s lymphoma



21. Most common malignancy in post-transplant individuals: a. PTLD (Recent Question 2017) b. Squamous cell carcinoma of skin c. Kaposi sarcoma d. CNS Lymphoma



22. Highest chance of success in renal transplant is seen when the donor is the? a. Identical twin b. Father c. Mother d. Sister e. Husband



23. Principal cause of death in renal transplant patients: (Recent Question 2016) a. Uremia b. Malignancy c. Rejection d. Infection

Transplantation



25. Infection in renal transplant patient is usually caused by: a. CMV b. HIV (Recent Question 2016) c. Herpes d. Salmonella



26. Most common type of renal transplantation in India is: a. Allograft b. Autograft (All India 99) c. Isograft d. Xenograft 27. A patient had undergone a renal transplantation 2 months back and now presented with difficulty breath. X-ray showed bilateral diffuse interstitial pneumonia. The probable etiologic agent would be: (AIIMS June 2002) a. CMV b. Histoplasma c. Candida d. Pneumocystis carinii 28. Most common disease caused by CMV in a post renal transplant patients: (JIPMER 2011) a. Pyelonephritis b. Meningitis c. Pneumonia d. GI ulceration













29. An elderly male presents 2 months after renal transplantation with nephropathy. Which of the following can be a viral etiological agent? (AIIMS May 2014) a. Polymoa virus BK b. Human herpes virus type 6 c. Hepatitis C d. Human papilloma virus, high risk types 30. In renal transplant, graft is placed in? (Recent Question 2014) a. Upper retroperitoneal space b. Iliac fossa c. Normal anatomical site d. None 31. All of the following are absolute contraindications for renal transplantation except: (Recent Question 2017) a. Active infection b. Active malignancy c. Active drug abuse d. Reduced life expectancy 32. In renal transplant, transplanted kidney is placed in: (Recent Question 2018) a. Iliac fossa b. Subcostal area c. Renal fossa d. Loin 33. Immediately after kidney donation, what happens to the creatinine level in the donors?  (Recent Question 2018) a. Remains same b. Increases c. Decreases d. Level is independent of the donation



34. Which of the following is expanded criteria donor (ECD) for kidney transplantation? (Recent Question 2018) a. Donors with extremes of age b. Donors with excess alcohol intake c. Donors having cerebrovascular accident d. All of the above





35. Long-term complication of live kidney donors:  (Recent Question 2018) a. Hypertension b. HPV infection c. Renal carcinoma d. Pyelonephritis 36. Left kidney is preferred for transplantation because:  (Recent Question 2018) a. Longer renal vein b. Higher location c. Ease of surgery due to anatomical relations d. To prevent damage to liver



























37. Expanded criteria for liver donation include all of the following except: (All India 2012) a. Taken from diseased individual after brain death b. Hepatitis B serology positive c. Age of donor may be >70 years d. Can be taken from individual with mild hepatic steatosis 38. Extended criteria for liver donation include all of the following except: (All India 2012, AIIMS GIS Dec 2011) a. Donor age > 70 years b. HBsAg positive donor c. Mild hepatic steatosis d. Donor after cardiac death 39. Most common indication of liver transplantation in children: a. Biliary atresia c. Hemochromatosis

(Recent Question 2019) b. Wilson’s disease d. Primary biliary cirrhosis

40. Indications of Liver transplantation are all, except: a. Biliary (PGI June 2005) b. Sclerosing cholangitis c. Hepatitis A d. Cirrhosis e. Fulminant hepatic failure 41. Auxiliary orthotopic liver transplant is indicated for: a. Metabolic liver disease (AIIMS May 2008) b. As a standby procedure until finding a suitable donor c. Drug induced hepatic failure d. Acute fulminant liver failure for any cause 42. Reduced liver transplants: (GB Pant 2011) a. Given to two recipients after dividing into two parts b. Left lateral lobe divided and given to child c. Left lateral segment divided from segment 2 and given to child d. Part of liver segment transplanted into recipient depending upon requirement 43. All are scoring system used in liver transplant except:  (Recent Question 2016) a. CTP b. PELD c. MELD d. MPI 44. Liver transplantation was first done by: (Recent Question 2014) a. Starzl b. Huggins c. Carrel d. Christian Bernard 45. All are marginal liver donor except: a. Older donor b. HBV core antibody positive donors c. Moderate steatosis d. Severe hepatitis 46. A 65-years old male with cirrhosis would be unsuitable for liver transplantation in the presence of: a. CTP ‘B’ b. HCC 65 years e. Active alcohol abuse 47. Which of the following is not an indication for liver transplantation? (DNB 2002) a. Fatty liver b. HIV c. Willson’s disease d. Primary hyperoxaluria 48. Most common indication for liver transplantation is: a. HCV induced cirrhosis (Recent Question 2017) b. HBV induced cirrhosis c. Primary sclerosing cholangitis d. HCC 49. Most common indication for pediatric liver transplantation is: (Recent Question 2017) a. Biliary atresia b. Metabolic diseases c. Alagille’s syndrome d. HCV

Others



LIVER TRANSPLANTATION

Section 10

24. Investigation of choice in the early phase of renal transplant: a. IVP (Kerala 97) b. Retrograde cystourethrogram c. Ultrasonogram d. CT scan

1047

1048

Surgery Essence PANCREAS TRANSPLANTATION

Others



50. The advantage of bladder drainage over enteric drainage after pancreatic transplantation is better monitoring of:  (All India 2009) a. HBA1C levels b. Amylase levels c. Glucose levels d. Electrolyte levels



59. Indications of lung transplantation: a. COPD b. Alpha-1 antitrypsin deficiency c. Cystic fibrosis and brochiectasis d. All of the above



60. Order of anastomosis in lung transplant:

51. Site of transplantation in islet cell transplant for diabetes mellitus: (Recent Question 2016) a. Forearm muscles b. Pelvis c. Thigh d. Injected into the portal vein



SMALL INTESTINE TRANSPLANTATION



LUNG TRANSPLANTATION

52. All are true about intestinal transplant except:  (JIPMER GIS 2011) a. Principal barrier to widespread application is vigorous rejection reactions b. Severe form of GVHD occurs when T cells of graft respond to foreign HLA cells c. Uniquely dangerous complication is loss of protective mucosal barrier, bacterial translocation and severe sepsis d. Majority of intestinal grafts are multivisceral grafts 53. Most common cause of 
 death in intestinal transplant:  (Recent Question 2017) a. Sepsis b. Acute rejection c. 
PTLD d. GVHD

POST-TRANSPLANT INFECTIONS

61. Post-transplant lymphoma is most commonly associated with: (AIIMS May 2012) a. EBV b. CMV c. Herpes simplex d. HHV-6

MISCELLANEOUS

62. In which of the following year, the transplantation of human organs act was passed by Government of India? a. 1994 b. 1996 c. 2000 d. 2002



63. Commonest complication of immunosuppression is:  (Recent Question 2016) a. Malignancy b. Graft rejection c. Infection d. Thrombocytopenia



64. Following drugs are known immunosuppressive agent except: a. Prednisolone b. Cephalosporin c. Azathioprine d. Cyclosporine-A



65. Steroids are used in transplantation: (TN 2003) a. To prevent graft rejection b. To prevent infection c. To speed up recovery d. To enhance immunity



66. Which of the following organs/tissues are presently not being used for organ/tissue transplantation? (All India 2011) a. Blood vessels b. Lung c. Liver d. Urinary bladder



67. Cold ischemic time of the kidney should be ideally below: a. 2 hours b. 6 hours (DNB 2010) c. 12 hours d. 24 hours 68. Length of time for which an organ can be cold stored before transplantation is maximum with: (MHCET 2016) a. Liver b. Pancreas c. Kidney d. Small intestine 69. Best temperature to store the procured organ for transplantation is: (Recent Question 2017) a. –2°C b. 0°C c. 4°C 
 d. 6°C 70. All of the following nerves are commonly used for grafting except: (Recent Question 2018) a. Medial antebrachial cutaneous nerve b. Dorsal sensory branch of vagal nerve c. Musculocutaneous nerve d. Sural nerve

HEART TRANSPLANTATION

Section 10

(Recent Question 2017) a. Pulmonary artery, pulmonary vein, bronchus b. Pulmonary vein, bronchus, pulmonary artery c. 
Pulmonary vein, pulmonary artery, bronchus d. Pulmonary artery, bronchus, pulmonary vein



54. Dr. Christian Bernard performed the 1st heart transplant in the year: (Kerala 97) a. 1962 b. 1965 c. 1969 d. 1967



55. Human heart transplant was first done by: a. Christian Bernard (Recent Question 2017) b. Roy Calne c. Sutherland d. Reitz and Norman Shumway



56. Which of the following is category III for cardiac donation? a. Dead on arrival (Recent Question 2018) b. Unsuccessful resuscitation c. Awaiting cardiac arrest d. Cardiac arrest while brain dead





57. Dr. Christian Bernard is associated with: (DNB 2009) a. Heart transplant b. Renal transplant c. Liver transplant d. Hair transplant





58. All of the following is true about heart transplantation except: (Recent Question 2018) a. Immunosuppression is started preoperatively b. It is only orthotopic and not heterotopic c. A beating heart cadaver/donor is needed d. High pulmonary arterial resistance is a contraindication



Explanations FLUIDS USED IN TRANSPLANTATION

1. Ans. c. Plastic bag in ice (Ref: Sabiston 20/e p1996; Schwartz 10/e p 1800) • The amputated digits are cleansed under saline solution, wrapped in saline moistened gauze, and placed in a plastic bagQ. • The plastic bag containing the part is then placed on (not packed in) a bed of ice in a suitable containerQ. • The amputated part should never be immersed in nonphysiological solution such as antiseptics or alcohol.



2. Ans. c. Free radical scavenger (Ref: Schwartz 10/e p 332; Bailey 27/e p1546)

GRAFT

3. Ans. a. Allograft (Ref: Schwartz 10/e p 266; Bailey 27/e p 1533)



4. Ans. b. Allograft



5.

Ans. c. Allograft

6. Ans. a. Isograft



7. Ans. b. Allograft

8.

Ans. c. Mynozygotic twins

9. Ans. a. Isograft



10. Ans. a. Between closely related different species (Ref: Sabiston 20/e p631) “Concordant Xenografts: Concordant xenografts refer to transplants between closely related species; for humans, these include Old World monkeys and apes. The critical element defining an animal as concordant is the assembly of carbohydrate antigens on the cell surface. Similar to humans, concordant species lack galactosyl transferase, and as a result, their carbohydrates are the typical blood group antigens and they lack the N-linked disaccharide galactose-α(1- 3)-galactose (α-Gal). Thus, the natural antibodies present in the circulation of potential human recipients can be predicted by straightforward blood group typing, thereby avoiding the problem of hyperacute rejection.”-Sabiston 20/e p631



11. Ans. b. Isograft

MATCHING AND GRAFT REJECTION

12. Ans. c. Between 5 to 30 days of transplantation (Ref: Schwartz 9/e p274-275; Bailey 27/e p 1533-1537)



13. Ans. a. Preformed antibodies

14. Ans. d. HLA-D (Ref: Bailey 27/e p 1534-1538)

15. Ans. a. Liver

16. Ans. b. Lymphocytes (Ref: Bailey 27/e p1534)



• The cellular effectors of graft rejection include cytotoxic CD8 T cellsQ, which recognize donor HLA class I antigens expressed by the graft and cause target cell death by releasing lytic molecules such as perforin and granzyme.

17. Ans. b. Kidney

18. Ans. c. Kidney

KIDNEY TRANSPLANTATION

19. Ans. a. Chronic glomerulonephritis (Ref: Campbell 10/e p1226-1227) Most Common Cause of End-stage Renal Disease • Diabetes mellitus >Hypertension >GlomerulonephritisQ



20. Ans. a. Skin cancer (Ref: Bailey 27/e p1542)



21. Ans. b. Squamous cell carcinoma of skin (Ref: Sabiston 20/e p629)



22. Ans. a. Identical twin



23. Ans. d. Infection (Ref: Campbell 10/e p1251-1253)

Most Common Cause of Death in Renal Transplant Patients • Heart disease >Infection >Stroke Q



Q

24. Ans. c. Ultrasonogram: (Ref: Bailey 27/e p1550)

Vascular Complications after Kidney Transplantation • Vascular complications after renal transplantation are low, presents during the first week after transplantation with sudden pain and swelling at the site of the graft. • Diagnosis is confirmed by Doppler ultrasonographyQ. • Urgent surgical exploration is indicated and, in most cases, transplant nephrectomyQ is required.

1050

Surgery Essence

25. Ans. a. CMV



27. Ans. a. CMV (Ref: Sabiston 20/e p657; Bailey 27/e p1541; Harrison 20/e p1033, 19/e p1192) 28. Ans. c. Pneumonia

26. Ans. a. Allograft



29. Ans. a. Polyoma virus BK (Ref: Harrison 20/e p508, 19/e p926)

Polyoma virus BK can be an etiological agent in an elderly male, who presents 2 months after renal transplantation with nephropathy.

30. Ans. b. Iliac fossa



31. Ans. d. Reduced life expectancy (Ref: Sabiston 20/e p650)



32. Ans. a. Iliac fossa



33. Ans. b. Increases

Others

After kidney donation, both the serum creatinine and creatinine clearance increase.

34. Ans. d. All of the above (Ref: Sabiston 20/e p651)



36. Ans. a. Longer renal vein

35. Ans. a. Hypertension

The left kidney is preferred because of implantation advantage associated with a longer left renal vein making anastomosis easier.

LIVER TRANSPLANTATION

37. Ans. a. Taken from diseased individual after brain death (Ref: Sabiston 20/e p644)



38. Ans. b. HBsAg positive donor



39. Ans. a. Biliary atresia



40. Ans. c. Hepatitis A (Ref: Harrison 19/e p2068)



41. Ans. a Metabolic liver disease, d. Acute fulminant liver failure for any cause (Ref: Blumgart 5/e p1689-1693)



42. Ans. d. Part of liver segment transplanted into recipient depending upon requirement



43. Ans. d. MPI



45. Ans. d. Severe hepatitis



46. Ans. e. Active alcohol abuse

44. Ans. a. Starzl

Discussed in chapter no. 4 Liver.

47. Ans. b. HIV

48. Ans. a. HCV induced cirrhosis (Ref: Sabiston 20/e p638)

“Chronic hepatitis C virus (HCV) infection is the most common indication for transplantation in the West at present.”-Sabiston 20/e p638

49. Ans. a. Biliary atresia (Ref: Sabiston 20/e p639) “Biliary atresia is the most common indication for liver transplantation in the pediatric patient and is a major concern in the infant with persistent jaundice after birth.”-Sabiston 20/e p639

PANCREAS TRANSPLANTATION

50. Ans. b. Amylase levels (Ref: Sabiston 20/e p660; Schwartz 10/e p340-344; Bailey 27/e p1552) • Bailey: ‘Urinary drainage of the pancreas has the advantage that urinary amylase levels can be used to monitor graft rejection’Q

Section 10



51. Ans. d. Injected into the portal vein (Ref: Bailey 27/e p1552) • The islets are then purified from the dispersed tissue by density-gradient centrifugation and can be delivered into the recipient liver (the preferred site for transplantation) by injection into the portal veinQ.

SMALL INTESTINE TRANSPLANTATION

52. Ans. d. Majority of intestinal grafts are multivisceral grafts (Ref: Sabiston 20/e p668-669; Schwartz 10/e p352-354; Bailey 27/e p1555-1556)



53. Ans. a. Sepsis (Ref: Sabiston 20/e p672; Bailey 27/e p1556) “The most common reason for mortality after intestine transplantation is infection with sepsis, accounting for 50% of deaths.”Sabiston 20/e p672

Transplantation

1051

HEART TRANSPLANTATION 54. Ans. d. 1967 (Ref: Bailey 27/e 1556) • Barnard performed the first human heart transplant in Cape Town, South Africa, in 1967Q.

55. Ans. a. Christian Bernard (Ref: Bailey 27/e p1556)



56. Ans. c. Awaiting cardiac arrest (Ref: Bailey 27/e p1418)



57. Ans. a. Heart transplant

58. Ans. b. It is only orthotopic and not heterotopic

LUNG TRANSPLANTATION

59. Ans. d. All of the above (Ref: Sabiston 20/e p1598)



60. Ans. b. Pulmonary vein, bronchus, pulmonary artery (Ref: Bailey 27/e p1557)

Section 10



“Single-lung transplantation is performed through a posterolateral thoracotomy and double-lung transplantation through a bilateral thoracotomy or median sternotomy. During lung transplantation, the donor pulmonary veins on a left atrial cuff are anastomosed to the recipient left atrium. Next, the bronchial anastomosis and the pulmonary arterial anastomosis are completed.”-Bailey 27/e p1557

POST-TRANSPLANT INFECTIONS

61. Ans. a. EBV (Ref: Sabiston 20/e p672,27/e p1542)

MISCELLANEOUS

62. Ans. a. 1994 (Ref: www.medindia.net › Health Acts in India) • Transplantation of Human Organ Act was passed by Government of India in 1994Q.



63. Ans. c. Infection



66. Ans. d. Urinary bladder (Ref: Essentials of General Surgery by Lawrence 4/e p475)

65. Ans. a. To prevent graft rejection

Organs and Tissues that Can be transplanted at Present KidneyQ • Middle ear LungQ • Skin LiverQ • Bone/tendons Pancreas and islet cells of LangerhansQ • Bone marrow Heart and heart valvesQ • Blood vesselsQ (most common saphenous veinQ) Q Cornea



67. Ans. None >C (12 hours) (Ref: Bailey 27/e p1546, 26/e p1421) • Optimal storage time for kidney should be 5 cm above right atrium is a risk factor for VAEQ. • More common in posterior fossa surgeriesQ Management • Once VAE is suspected, any central line procedure in progress should be terminated immediatelyQ. • Administer 100% oxygenQ • Promptly place the patient in Trendelenberg (Head down) position and rotate towards the left lateral decubitus positionQ. This maneuver helps trap air in the apex of the ventricle, prevents its ejection into pulmonary arterial system and maintains right ventricular output. • Consider transfer to hyperbaric chamberQ.

12. Ans. a. Thrombocytopenia (Ref: Schwartz 10/e, 90-91,1428)



13. Ans. a. Halothane (Ref: en.wikipedia.org/wiki/Halothane)

Physical properties of Halothane • • • •

No analgesiaQ and least pungent (non-irritant)Q Pleasant to smell, so excellent for induction in childrenQ Halothane has highest fat/blood coefficientQ 51 (can get deposited in adipose tissues after prolonged exposure) Trifluoloroacetic acid is a metabolite and found in urineQ

14. Ans. a. IV antibiotics for 72 hours (Ref: Sabiston 20/e p223; Bailey 27/e p1266) Enhanced Recovery after Surgery (ERAS) Protocols (Fast Track ProgramsQ) • ERAS refers to patient-centered, evidence-based, multidisciplinary team developed pathways for a surgical specialty and facility culture to reduce the patient’s surgical stress response, optimize their physiologic function & facilitate recovery. • These care pathways form an integrated continuum, as the patient moves from home through the pre-hospital / preadmission, preoperative, intraoperative & postoperative phases of surgery and home again.

Section 10

Key Elements of an ERAS Program • • • • • • • • •

Pre-admission counseling 
 Avoidance of mechanical bowel preparationQ 
 Preoperative carbohydrate loadingQ 
 Avoidance of preoperative dehydrationQ 
 No nasogastric tubesQ 
 Short, transverse incisionsQ (or laparoscopic procedure) 
 Short-acting anaesthetic drugsQ Avoidance of perioperative fluid/salt overloadQ Thoracic epiduralsQ

• • • • • • •

Avoidance of opiate analgesiaQ 
 Maintenance of perioperative temperature 
 Prevention of postoperative nausea and vomiting 
 Early mobilizationQ
 Early introduction of oral fluids/diets/ supplementsQ
 Early removal of urinary cathetersQ 
 Continual audit of outcomes 


CHAPTER

45

Robotics, Laparoscopy and Bariatric Surgery

LAPAROSCOPY Laparoscopy • • • • •

Needle used for pneumoperitoneum: Veress needleQ Most commonly used gas: CO2Q Flow of gas: 1L/minQ Intra-abdominal pressure maintained during laparoscopy: 12-15 mm HgQ Trocar is inserted at or just below the umbilicusQ penetrating skin, superficial & deep fascia, fascia transversalis & parietal peritoneum.Q • Post-laparoscopy shoulder pain is due to CO2 retention causing irritation of diaphragm & referred pain to the shoulder through phrenic nerveQ.

PNEUMOPERITONEUM Pneumoperitoneum • Pneumoperitoneum can be created by closed or open method. Closed Method • Uses veress needle • Low risk of bowel injuryQ due to presence of safety valve at tip Q

Open Method • Uses Hasson cannulaQ • Low risk of major vessel injuryQ

GASES USED IN LAPAROSCOPY Gases Used in Pneumoperitoneum First pneumoperitoneum was created by filtered room airQ. CO2 & N2O are now preferred because of increased risk of gas embolism with room airQ. CO2: 200 times more diffusible than O2, rapidly cleared from the body & lungs, does not support combustionQ N2O: 68% as rapidly absorbed in blood as CO2, have mild analgesic effect, used for short operative procedures like sterilization or drillingQ. • For prolonged laparoscopic procedures, N2O should not be preferred because it supports combustion better than airQ. • • • •

PHYSIOLOGICAL EFFECTS OF LAPAROSCOPY Physiological Effects of Laparoscopy Cardiovascular

• ↑ Intra-abdominal pressure leads to ↑ CVP, ↑ PCWP, ↑SVR and ↑ MAP which further ↓ Preload and ↑ Afterload, ultimately decreasing cardiac outputQ.

Pulmonary

• Cephalad shift of diaphragm decreases FRC, chest wall compliance & tidal volume increasing the work of breathingQ. • Hypercapnia leading to increase in respiratory rate further adds to this.

Renal

• Increased IAP decreases renal flow, decreasing GFR & reduced urine output. • Raised pCO2 leads to RAAS stimulation. No long term change in GFR/UO.

1056

Surgery Essence Physiological Effects of Laparoscopy • Gastrointestinal

• Decreased perfusion to intestines & stomach (as a result of increase IAP) decreases pH • Decreased portal and hepatic flow leads to elevation of LFTs

• Peripheral

• Incidence of DVT, PE is generally lower post-laparoscopic procedures probably secondary to improved prophylaxis • Risk is increased with longer procedures and reverse Trendelenburg position.

vascular

GAS EMBOLISM

Others

Gas Embolism • Most commonly seen during induction of pneumoperitoneum at the time of insufflations of gas from unintended insufflations of gas directly into an open veinQ. • The more soluble a gas in the blood, the lower chances are for gas embolism. • CO2 is preferred for pneumoperitoneum as it is highly soluble in blood and is rapidly eliminatedQ. • CO2 Embolism: An initial rise in ET-CO2 due to pulmonary excretion of absorbed CO2 is followed by a sudden decrease due to fall in cardiac outputQ.

DAY CARE SURGERY Day Care Surgery or Ambulatory Surgery • Surgical procedures suitable for ambulatory surgery should be accompanied by minimal postoperative physiologic disturbances and an uncomplicated recoveryQ. • The primary predictors of prolonged stay or unanticipated admission after day-care surgery are related to the type of surgical procedure and associated complications (e.g. blood loss, incision pain, postoperative nausea & vomiting)Q • For superficial procedures (e.g. mastectomy,) reductions in both cost & per-operative complications have been observed when these procedures are performed on an outpatient basisQ. Specialty Dental Dermatology General Gynecology Ophthalmology Orthopedics Otolaryngology Pain clinic Plastic surgery

Section 10

Urology

Types of Surgical Procedures • Extraction, restoration, facial fractures • Excision of skin lesions • Biopsy, endoscopy, excision of masses, hemorrhoidectomy, herniorrhaphy, laparoscopic cholecystectomy, adrenalectomy, splenectomy, varicose vein surgeryQ • Cone biopsy, dilatation & curettage, hysteroscopy, diagnostic laparoscopy, laparoscopic tubal ligationQ, uterine polypectomy, vaginal hysterectomy • Contract extraction, chlazion excision, nasolacrimal duct probing, strabismus repair, tonometry • Anterior cruciate repair, knee arthroscopyQ, shoulder reconstructions, bunionectomy, carpal tunnel release, closed reduction, hardware removal, manipulation under anesthesia and minimally invasive hip replacements • Adenoidectomy, laryngoscopy, mastoidectomy, myringotomy polypectomy, rhinoplasty, tonsillectomy, tympanoplasty • Chemical sympathectomy, epidural injection, nerve blocksQ • Basal cell cancer excision, cleft lip repair, liposuction, mammoplastyQ (reductions & agumentations), otoplasty, scar revision, septorhinoplastyQ, skin graft • Bladder surgery, circumcision, cystoscopy, lithotripsy, orchidectomyQ, prostate biopsy, vasovasostomy, laparoscopic nephrectomy and prostatectomy

FAST TRACK SURGERY Fast Track Surgery • Coordinated perioperative approach aimed at reducing surgical stress & facilitating postoperative recovery • Fast track surgery comprises: −− Preoperative informed consent −− Fasting 6 hours for solids & 2 hours for liquids −− Atraumatic surgical technique (avoid drains) −− Reduction of stress

Robotics, Laparoscopy and Bariatric Surgery Elimination of pain by regional anesthesia (thoracic epidural anesthesia) Optimized fluid & temperature management Early enteral diet Prevention of gastrointestinal atony & postoperative nausea & vomiting Rapid postoperative mobilization

MINIMAL ACCESS SURGERY Minimal Access Surgery (MAS) includes • LaparoscopyQ • Perivisceral endoscopyQ • ThoracoscopyQ • Arthroscopy & intra-articular joint surgeryQ • Endoluminal endoscopyQ

Section 10

−− −− −− −− −−

1057

Advantages of Minimal Access Surgery (MAS) • Decrease in wound sizeQ • Improved mobilityQ • Reduction in wound infection, dehiscence, bleeding, herniation & • Decreased wound trauma nerve entrapmentQ • Decreased heat lossQ • Decrease in wound painQ • Improved visionQ • Faster recovery & shorter hospital stay

OBESITY & BMI Category Underweight Normal Overweight Obesity (Class I) Severe obesity (Class II) Morbid obesity (Class III) Superobesity

BMI 50Q

PATHOLOGIC CONSEQUENCES OF OBESITY System Health Endocrine Reproductive Cardiovascular Pulmonary

• • • • •

Heaptobiliary



Bone, joint, and cutaneous disease

• Osteoarthritis, acanthosis nigricans, friability of skin, venous stasis & ulcersQ

Neurologic

• Carpal tunnel syndrome, pseudotumor cerebri, stroke Increased Cancer Risk in Obese Patients (PEEL CP GO KBC) • • • •

ProstateQ EndometrialQ EsophagusQ LiverQ

• • • •

CervixQ PancreasQ GallbladderQ OvarianQ

BARIATRIC SURGERY Bariatric Surgery • Indication for Bariatric Surgery −− Patients that have a BMI of 35 Kg/m2 or more with comorbidityQ −− Those with a BMI of 40 kg/m2 or greater regardless of comorbidityQ

• • • •

KidneyQ Bile ductQ BreastQ Colon & rectumQ

Others

Pathologic Consequences of Obesity Pathology Increase in mortalityQ Insulin resistance and type 2 diabetes mellitusQ Male hypogonadism, gynecomastia, menstrual abnormalities, polycystic ovarian syndrome Coronary disease, congestive heart failure Obstructive sleep apnea, “obesity hypoventilation syndrome”, pulmonary hypertension, DVT & pulmonary embolismQ Nonalcoholic fatty liver disease, symptomatic gallstones

1058

Surgery Essence Bariatric Operation

Mechanism of Action

• Vertical banded gastroplasty • Laparoscopic adjustable gastric banding (Safest & reversible)

• RestrictiveQ

• Roux-en-Y gastric bypass (RYGB): MC performed procedure now-a-days • Largely RestrictiveQ/Mildly Malabsorptive • Bilopancreatic diversion • Duodenal switch

• Largely MalabsorptiveQ/Mildly Restrictive

Vertical Banded Gastroplasty

Others

• Restrictive operation: Restricts or decrease food intakeQ • Upper stomach near esophagus is stapled vertically to create a small pouch along the lesser curvature of stomachQ

Vertical banded gastroplasty

Laparoscopic adjustable gastric banding Roux-en-Y gastric bypass

Laparoscopic Adjustable Gastric Banding • Least invasive, safest & reversible • Involves placement of adjustable silicon band around the top part of stomach & creates a small stomach pouchQ Q

Roux-en-Y Gastric Bypass

Section 10

• Most commonly performed bariatric surgery worldwideQ • Upper part of stomach is separated from lower part & connected to Roux-limb of jejunumQ • Content of gastric pouch: 30-50 mLQ; Length of Roux-limb of jejunum: 70-100 cmQ

Duodenal Switch

Biliopancreatic Diversion

Robotics, Laparoscopy and Bariatric Surgery

1059

Biliopancreatic Diversion

Duodenal Switch Operation

Section 10

• When the bile is diverted from the intestinal tract so that only the distal 50 to 100 cm of ileum is used for bile reabsorption, the procedure is termed a biliopancreatic diversion. • Most effective bariatric surgeryQ, especially valuable in patients with severe morbid obesity or in those who have failed to maintain weight loss following gastric bypass surgery or restrictive proceduresQ. • Main side effect with BPD: Patients usually have an increase of 2-4 bowel movements/day, which in general are more malodorous, suggesting fat malabsorption.

• It involves a greater curvature sleeve gastrectomy with maintenance of the continuity of the antrum, pylorus and first portion of the duodenum. • This allows for a lower marginal ulcer rate and a lower incidence of dumping syndrome. Components of Duodenal Switch Operation • Sleeve gastrectomyQ • DuodenoileostomyQ • Jejunoileal bypassQ

• CholecystectomyQ • AppendectomyQ

PERIOPERATIVE MORTALITY IN BARIATRIC SURGERY Perioperative Mortality in Bariatric Surgery • MC cause of death within 30 days of bariatric surgery: Pulmonary embolismQ (36%) > Cardiac complications (24%) > Anastomotic leaks (20%) • MC cause of death in immediate postoperative period: Peritonitis secondary to anastomotic leakQ (leak most commonly occurs at the gastrojejunostomy)

NEW GUIDELINES OF BARIATRIC SURGERY FOR ASIA New Guidelines for Asia

NOTES Natural Orifice Transluminal Endoscopic Surgery (NOTES) • NOTES is a technique whereby the peritoneal cavity is entered endoscopically, via a natural orifice (mouth, rectum, vagina) and the surgery is carried out using specialised endoscopic technology and techniques. • NOTES cholecystectomy & appendicectomy have been successfully carried out in humans. • Additional procedures performed with NOTES: Staging of intra-abdominal malignancy, segmental colectomy, gastrojejunostomy

POEM Per Oral Endoscopic Myotomy (POEM) • POEM is the application of esophageal myotomy to the concept of NOTES by submucosal tunneling methodQ.  • Used for treatment of achalasiaQ • Involves creation of long esophageal myotomy using flexible endoscopeQ

Others

• Overweight if the BMI is 23 kg/m2 or moreQ (International Standard is 25). • Obese if the BMI is 25 kg/m2 or moreQ (I.S. is 30). • An Indian qualifies for bariatric surgery for obesity if the BMI is 32.5 kg/m2 (I.S. 35) with comorbidityQ or 37.5 kg/m2 without comorbidity (I.S. 40)Q.

1060

Surgery Essence ROBOTIC SURGERY Robotic Surgery • • • • •

In 1985 a robot, The PUMA 560, was used to place a needle for a brain biopsy using CT guidance. In 1987 robotics was used in the first Laparoscopic surgery, a cholecystectomy. In 1988, The PROBOT, developed at Imperial College London, was used to perform prostatic surgery. The ROBODOC from Integrated Surgical Systems was introduced in 1992 to mill out precise fittings in the femur for hip replacement. Further development of robotic systems was carried out by Computer Motion with the AESOP and ZEUS Robotic Surgical Systems and Intuitive Surgical with the introduction of The Da Vinci Surgical System.

Section 10

Others

Da Vinci Robot • It works on “master-slave” principleQ. • The surgeon inserts his hands into a “master” that translates motions of his hands into motions of the robotic arms and hand-like instruments. The surgeon acts as the “master” and the robot as the “slave” in this telerobotic “master-slave” system. • It is commonly used for prostatectomies, cardiac valve repair and gynecological Surgical proceduresQ

Multiple Choice Questions

LAPAROSCOPY





1. The intra-abdominal pressure during laparoscopy should be set between: (Recent Question 2017, DNB 2011, AIIMS Nov 2003) a. 5-8 mm of Hg b. 10-15 mm of Hg c. 20-25 mm of Hg d. 30-35 mm of Hg 2. Gases for pneumoperitoneum: a. CO2 b. N2 c. Room air d. N2O

11. During laparoscopy, the intra-abdominal pressure is:  (Recent Question 2014, MHSSMCET 2008) a. 5-10 mm Hg b. 12-15 mm Hg c. 15-20 mm Hg d. 20-25 mm Hg



12. Instrument used to create artificial pneumoperitoneum in laparoscopy: (MHSSMCET 2008) a. Maryland forceps b. Veress needle c. Trocar d. All of the above



13. Complications of laparoscopy: (MHSSMCET 2009) a. Diaphragmatic rupture b. Vascular injury c. Pneumothorax d. All of the above

(PGI June 2007)

3. Which gas is used in laparoscopy?  (Recent Question 2018, 2015, DNB 2012, AIIMS June 94) a. CO2 b. N2O c. O2 d. N2



4. Layers which are penetrated with trocar and cannula in production of pneumoperitoneum are: (PGI June 2005) a. Skin and superficial fascia b. Deep fascia c. Rectus abdomnis d. Transversus abdomnis e. Rectus sheath



5. Shoulder pain post laparoscopy is due to:  (Recent Question 2014, AIIMS Nov 2007) a. Subphrenic abscess b. CO2 retention c. Positioning of the patient d. Compression of the lung



6. A lady presented in the emergency department with a stab injury to the left side of the abdomen. She was hemodynamically stable and a contrast enhanced CT scan revealed a laceration in spleen. Laparoscopy was planned, however, the patients PO2 suddenly dropped as soon as the pneumoperitoneum was created. What is the most likely cause? (All India 2010) a. Gaseous embolism through splenic vessels b. Injury to the left lobe of the diaphragm c. Inferior vena cava compression d. Injury to colon



7. Day care surgery can be done in: (PGI Nov 2010) a. Lateral sphincterotomy b. Rhinoplasty c. Orchidectomy d. Total thyroidectomy e. Subcutaneous mastectomy



8. Advantage of minimal access surgery: (Recent Question 2016) a. ↑ Heat loss b. Better Hemostasis control c. Improved vision d. ↓ in wound pain



9. Advantage of minimally invasive surgery over open surgery are all except: (Recent Question 2015) a. Wide/better field of vision b. Less operative time c. Lest post-operative time d. Less post-operative morbidity





10. Minimal invasive surgery includes all except:  (MHPGMCET 2002) a. FESS b. Lap cholecystectomy c. Endoscopic sclerotherapy d. PCNL

NOTES

14. In surgical procedure NOTES, entry point is through: (Recent Question 2015) a. Abdomen b. Umbilicus c. Mouth d. Axilla



15. Which procedure is being performed in the given image? a. Laparoscopic cholecystectomy b. NOTES c. Percutaneous cholecystectomy d. Robotic cholecystectomy



16. POEM is used for:  (Recent Question 2018) a. Achalasia cardia b. Cancer esophagus c. Diffuse esophageal spasm d. Nutcracker esophagus

BARIATRIC SURGERY

17. Cancers associated with excess fat intake are/is:  (PGI Dec 2000) a. Breast b. Colon c. Prostate d. Lung e. Thyroid

18. Physiological changes seen in laparoscopy include all except: (AIIMS May 2015) a. Increased ICP b. Decreased FRC c. Increased CVP d. Increased pH

19. Bariatric surgical procedures include all except:  (WBPG 2015, AIIMS Nov 2008) a. Gastric banding b. Gastric bypass c. Biliopancreatic diversion d. Ileal transposition

Others

1062

Surgery Essence

20. What is the name of this bariatric procedure? a. LAGB b. VBG c. RYGB d. Biliopancreatic diversion



21. All of the following are primarily restrictive operations for morbid obesity, except: (All India 2010) a. Vertical band gastroplasty b. Duodenal switch operation c. Roux-en-Y operation d. Laparoscopic adjustable gastric banding



22. Complication (s) of obesity is/are:  (PGI Nov 2010) a. Venous ulcer b. Pulmonary embolism c. ↑Mortality d. Prostate cancer e. Pulmonary hypertension



23. Morbid obesity is BMI greater than: a. 25 b. 30 c. 40 d. 45

Section 10

29. What is the name of this surgery? a. Billroth I b. Billroth II c. Subtotal gastrectomy d. Sleeve gastrectomy



30. Vertical banded gastroplasty also known as stomach stapling is done for: (DNB 2010) a. Gastric carcinoma b. Achalasia cardia c. Perforated gastric ulcer d. Morbid obesity 31. What is the name of this bariatric procedure? a. LAGB: Purely restrictive b. LAGB: Restrictive + Malabsorptive c. VBG: Purely restrictive d. VBG: Restrictive + Malabsorptive



(MHSSMCET 2006)

24. Irrespective of comorbidity, bariatric surgeries should be done when BMI is greater than: (MHSSMCET 2008) a. 35 b. 40 c. 45 d. 50 25. What is the name of this bariatric procedure? a. Biliopancreatic diversion: Largely malabsorptive, mildly restrictive b. Duodenal switch: Largely malabsorptive, mildly restrictive c. Biliopancreatic diversion: Mildly malabsorptive, largely restrictive d. Duodenal switch: Mildly malabsorptive, largely restrictive



26. The most effective bariatric surgery with treatment in the form of weight loss for morbid obesity is: (MHSSMCET 2008) a. Roux-en-Y surgery b. Biliopancreatic diversion c. Vertical banded gastroplasty d. Any of the above



27. In Duodenal switch operation, which of the following is not done?  (MHSSMCET 2010) a. Cholecystectomy b. Appendectomy c. Jejunoileal anastomosis d. Distal gastrectomy 28. Peterson hernia: (MHSSMCET 2010) a. An internal hernia occurring behind Roux-en-Y limb b. An internal hernia occurring through window in the transverse mesocolon c. Cervical hernia d. None





32. Bariatric surgery which results in maximum weight loss:  (MHCET 2016) a. Biliopancreatic diversion b. Gastric sleeve c. Gastric banding d. Gastric bypass 33. Most commonly performed and acceptable method of bariatric surgery is: (AIIMS May 2015) a. Biliopancreatic diversion b. Biliopancreatic diversion with ileostomy c. Laparoscopic gastric banding d. Roux-en-Y gastric bypass 34. Bariatric surgery with maximum benefits and comorbidity reduction: (Recent Question 2017) a. Roux-en-Y gastric bypass b. Laparoscopic sleeve gastrectomy c. Biliopancreatic diversion d. Laparoscopic adjustable gastric banding

ROBOTIC SURGERY

35. What is the name of most commonly used robot used for the given surgery? a. PUMA 560 b. DaVinci robot c. PROBOT d. ROBODOC

Explanations LAPAROSCOPY

1. Ans. b. 10-15 mm of Hg (Ref: Sabiston 20/e p394-396; Schwartz 10/e p417)



2. Ans. a. CO2, c. Room air, d. N2O (Ref: Sabiston 20/e p396; Schwartz 10/e p417-419)



3. Ans. a. CO2



4. Ans. a. Skin and superficial fascia



5. Ans. b. CO2 retention



6. Ans. a. Gaseous embolism through splenic vessels (Ref: Laparoscopic Surgery by Garcia & Jacobs/25) • Sudden drop in pO2 immediately during induction of pneumoperitoneum suggest the possibility of gas embolism as a result of entry of insufflating gas into circulation through the tear in splenic vessels. • Gas embolism may also have resulted from inadvertent insertion of the trocar or veress needle into a vessel or abdominal organ.



7. Ans. a. Lateral sphincterotomy, b. Rhinoplasty, c. Orchidectomy, e. Subcutaneous mastectomy (Ref: Bailey 27/e p301)



8. Ans. c. Improved vision, d. ↓ in wound pain (Ref: Bailey 27/e p105-108)



9. Ans. b. Less operative time



10. Ans. d. PCNL (Ref: Bailey 27/e p105-106)



11. Ans. b. 12-15 mm Hg

12. Ans. b. Veress needle

13. Ans. d. All of the above

NOTES

14. Ans. c. Mouth (Ref: Bailey 27/e p117)



15. Ans. b. NOTES (Ref: Sabiston 20/e p397; Bailey 27/e p117)



16. Ans. a. Achalasia cardia

BARIATRIC SURGERY

17. Ans. a. Breast, b. Colon, c. Prostate (Ref: Sabiston 20/e p1162; Schwartz 10/e p1099-1131)



18. Ans. d. Increased pH: (Ref: Bailey 26/e p94; http://www.laparoscopyhospital.com/physiological-changes-laparoscopy.html)

Metabolic acidosis (decrease pH) from CO2 absorption is the primary derangement with laparoscopy.

19. Ans. d. Ileal transposition (Ref: Sabiston 20/e p1168; Schwartz 10/e p1099-1131)



20. 
Ans. c. RYGB



21. Ans. b. Duodenal switch operation



22. Ans. a. Venous ulcer, b. Pulmonary embolism, c. ↑ Mortality, d. Prostate cancer, e. Pulmonary hypertension (Ref: Sabiston 20/e p1162; Schwartz 9/e p951)



23. Ans. c. 40 (Ref: Schwartz 10/e p421-422)



24. Ans. b. 40



25. Ans. a. Biliopancreatic diversion: Largely malabsorptive, mildly restrictive



26. Ans. b. Biliopancreatic diversion (Ref: Sabiston 20/e p1171-1172; Schwartz 10/e p1103)



27. Ans. d. Distal gastrectomy (Ref: Sabiston 20/e p1172; Schwartz 10/e p1119-1121)



28. Ans. a. An internal hernia occurring behind Roux-en-Y limb

• Sleeve gastrectomy is done in duodenal switch operation not the distal gastrectomy.

Gibbon’s hernia

• Hernia with hydroceleQ

Berger’s hernia

• Hernia into pouch of DouglasQ

Beclard’s hernia

• Femoral hernia through opening of saphenous veinQ

Amyand’s hernia

• Inguinal hernia containing appendixQ

1064

Surgery Essence Ogilive’s hernia

• Hernia through the defect in conjoint tendon just lateral to where it inserts with the rectus sheathQ

Stammer’s hernia

• Internal hernia occurring through window in the transverse mesocolon after retrocolic gastrojejunostomyQ

Peterson hernia

• Hernia under Roux limb after Roux-en-Y gastric bypassQ



29. Ans. d. Sleeve gastrectomy



30. Ans. d. Morbid obesity



31. Ans. a. LAGB: Purely restrictive




32. Ans. a. Biliopancreatic diversion



33. Ans. d. Roux-en-Y gastric bypass (Sabiston 20/e p1169; Schwartz 10/e p1102-1103; Harrison 20/e p2849, 19/e p2398)

Others

Most commonly performed and acceptable method of bariatric surgery is Roux-en-Y gastric bypass “The three restrictive-malabsorptive bypass procedures combine the elements of gastric restriction and selective malabsorption. These procedures are Roux-en-Y gastric bypass, biliopancreatic diversion, and biliopancreatic diversion with duodenal switch. Rouxen-Y is the most commonly undertaken and most accepted bypass procedure. It may be performed with an open incision or by laparoscopy.”- Harrison 20/e p2849, 19/e p2398

34. Ans. a. Roux-en-Y gastric bypass (Ref: Sabiston 20/e p1049, 1178) “In appropriately selected patients, laparoscopic Roux-en-Y gastric bypass is the most durable method of weight loss and control of obesity-related comorbidities, including GERD.”-Sabiston 20/e p1049

ROBOTIC SURGERY

Section 10



35. Ans. b. DaVinci robot (Ref: Sabiston 20/e p399)

CHAPTER

46

Sutures and Anastomoses

TYPES OF SUTURES Suture Materials Absorbable • These sutures get absorbed in the tissues either by enzymatic digestion or by phagocytosisQ. 1. Natural absorbable: −− Plain & chromic catgutQ 2. Synthetic absorbable: (PVD) −− Polydioxanone (PDS)Q, Polyglycaprone, −− Polyglactin (Vicryl)Q −− Polyglycollic acid (Dexon)Q

Non-absorbable • These sutures remain in the tissues for indefinite period. 1. Natural non-absorbable: − LinenQ      −  SilkQ 2. Synthetic non-absorbable: (PEN) −− Polypropylene (Prolene)Q −− PolyesterQ (ethibond) −− Monofilament polyamide (Ethilon)Q −− NylonQ

Depending upon Number of Strands • • • • •

Monofilament Consist of single strandQ of fiber Sutures are smooth & strongQ Chances of bacterial contamination is lessQ Knot tied may become looseQ Prolene, Ethilon, nylon

Polyfilament • Consist of multiple strandsQ braided together • Easier to handle and knot tied does not slipQ • Bacteria may lodge in the crevices of the suture, so not suitable in presence of infectionQ • Silk, linen, polyglycollic acid (SLIP)

SUTURES: IMPORTANT POINTS Suture Silk Catgut

Types Braided or twisted multifilament; Coated (with wax or silicone) or uncoated Plain

Catgut

Chromic

Polyglactin (Vicryl)

Braided multifilament

Polyglyconate

Monofilament Dyed or undyed

Polyglycaprone (Monocryl) Polyglycolic acid (Dexon)

Monofilament

Polydioxanone (PDS)

Braided multifilament Dyed or undyed Coated or Uncoated Monofilament dyed or undyed

Raw material Natural protein Raw silk from silkworm

Tensile strength Loses 20% when wet; 80-100% lost by 6 months Collagen derived from Lost within healthy sheep or catle 7-10 days Tanned with chromium salts Lost within to improve handling and 21-28 days resist degradation in tissueQ Copolymer of lactide & Approx, 60% remains at glycolideQ in a ratio of 90:10, 2 weeks; 30% remains at coated with polyglactin & 3 weeks calcium stearate Copolymer of glycolic Approx, 70% remains at acid and trimethylene 2 weeks; 55% remains at carbonateQ 3 weeks Coplymer of glycolide 21 days maximum & caprolactoneQ Polymer of polyglycolic Approx, 40% remains at acidQ 1 week; 20% remains at 3 weeks Polyester polymerQ

Absorption rate Fibrous encapsulation in body at 2-3 weeks ; Absorbed slowly over 1-2 yearQ Phagocytosis and enzymatic degradation within 7-10 daysQ Phagocytosis and enzymatic degradation within 90 days Hydrolysis minimal until 5-6 weeks; Complete absorption 60-90 daysQ Hydrolysis minimal until 8-9 weeks; Complete absorption 180 daysQ 90-120 daysQ HydrolysisQ minimal at 2 weeks; significant at 4 weeks; Complete absorption 60-90 daysQ

Approx, 70% remains at Hydrolysis minimal at 90 days; 2 weeks; 50% remains at Complete absorption 180 daysQ 4 weeks; 14% remains at 8 weeks

1066

Surgery Essence PRINCIPLES OF SUTURING & ANASTOMOSES Wound Closure & Anastomoses • As a general rule, each suture should be separated by a gap that is twice the thickness of the skinQ. • When knots are cut short, the free ends or ‘ears’ should be left at least 1-2 mm longQ. This is particularly important with monofilament non-absorbablesQ. • It has been suggested by Jenkins that a suture length to wound length ratio of 4:1 indicates the optimum size of tissue bites and of suture spacingQ. • Anastomosis of vessels was pioneered by CarrelQ. • Elliptical incisions must be at least 3 times of the width for the wound to heal without tension. • Length to width ratio: 3:1

Others

TYPES OF SURGICAL KNOTS Square (Reef) Knot • Consist of two throws • Crossing is done in each throw • Secured knot

Surgeons Knot • Consist of two throws • Two wraps in first throw • Crossing occurs in each throw

Granny (Pseudo-square) Knot • Consist of two throws • Crossing does not occur in any throw • Not a secured knot

NEEDLES: IMPORTANT POINTS Needles • Parts of Needle: Swaged end (eyeless needle end for suture attachment) or eye, point & body • Best site for holding the needle with needle holder: 1/3rd from swaged end & 2/3rd from pointed endQ

Section 10

Types of Needle Round Body • Taper point & blunt point • Round in cross section

Cutting/Reverse Cutting • Cutting: Triangular in cross section pointing inwardsQ • Reverse Cutting: Triangular in cross section pointing outwardsQ

Sutures and Anastomoses

1067

SUTURING TECHNIQUES

1. Simple Interrupted Suture: • Needle is inserted at right angle, to the incision • Pass through the both aspects of suture line, & exit again at right angles • Each successive suture should be placed at twice the distance from edge of the woundQ. 2. Continuous Suture: • First suture is inserted in an identical manner to an interrupted suture • Rest of the sutures is inserted in a continuous manner, until the far end of wound is reached. • At the far end of the wound suture line should be secured with Aberdeen knotQ or by tying free end to the loop of cast suture.

Section 10

Suturing Techniques

Aberdeen Knot • Free end of suture is partially pulled through the final loop, several times before being pulled through a final time, completely, prior to cutting. 3. Mattress Sutures: • Used to produce eversion or inversion of wound edgesQ • Useful in producing accurate approximation of wound edges, when the edges are irregular in depth or dispositionQ Horizontal Mattress • Initial suture is inserted as for an interrupted suture but then needle moves horizontally & traverses both edges of the wound once againQ

Vertical Mattress • Initial suture is inserted as for an interrupted suture but then needle moves vertically & traverses both edges of wound once againQ.

Others

4. Subcuticular Suture: • Used in skin, where cosmetic appearance is importantQ & skin edges are approximated easily. • MC used suture is monocrylQ (polyglycaproneQ)

5. Purse String Suture: • Continuous stitch parallel to a circular wound is appliedQ • Used for hernia sac & appendectomyQ

1068

Surgery Essence BOWEL ANASTOMOSES Bowel Anastomoses Lembert described seromuscular suture technique for bowel anastomosis in 1826Q. Senn advocated a two-layer technique for closureQ Halsted favoured a one-layer extramucosal closureQ. Connell used a single layer of interrupted sutures incorporating all layers of the bowelQ. Kocher’s method, a two layer anastomosis, first a continuous all-layer suture using catgut, then an inverting continuous (or interrupted) seromuscular layer suture using silk, became the standard. There is evidence that inversion is safest in bowel (least likely to leak), although end-to-end staplers give an everted anastomosis without complication. • The single-layer extramucosal anastomosis, advocated by MathesonQ, causes the least tissue necrosis or luminal narrowing. • The Cheatle split (making a cut into the anti-mesenteric border) may help to enlarge the lumen of distal, collapsed bowelQ. • Bowel anastomotic leaks are generally occur on day 7Q.

Others

• • • • •

VASCULAR ANASTOMOSES Vascular Anastomoses • Vascular anastomoses require more precision than bowel anastomoses as they must be immediately watertightQ at the end of the operation when the clamps are removed. • Suture size depends on vessel calibre: −− 2/0 for aortaQ −  6/0 for popliteal to distal arteriesQ −− 4/0 for femoral arteryQ −  Microvascular anastomoses are made using a loupe and an interrupted suture down to 10/0 size • Polypropylene-like sutures with indefinite integrity give the best resultsQ • Intimal suture line must be smoothQ



•  Knots must be secureQ •  Needle must pass from within outwardsQ

TISSUE GLUE Tissue Glue Tissue glue is also available based upon a solution of n-butyl- 2-cyanoacrylate monomerQ. When it is applied to a wound, it polymerizes to form a firm adhesive bondQ Wound does need to be clean, dry, with near perfect hemostasis and under no tensionQ. Specific uses: Closing a laceration on the forehead of a fractious child in Accident and Emergency thus dispensing with local anaesthetic and sutures. • Relatively expensive, it is quick to use, does not delay wound healing and is associated with an allegedly low infection rate. • • • •

MESH Mesh in hernia repair • Term ‘mesh’ refers to prosthetic material used to strengthen a hernia repair. Mesh can be used: • To bridge a defect : Mesh is simply fixed over the defect as a tension-free patch • To plug a defectQ: A plug of mesh is pushed into the defect • To augment a repairQ: Defect is closed with sutures & mesh added for reinforcement.

Section 10

Q

• A well-placed mesh should have good overlap around all margins of defect, at least 2 cmQ but up to 5 cmQ if possible. • Suturing a mesh edge-to-edge into the defect (inlay), with no overlap, is not recommendedQ. Types of Mesh Net meshes

Flat sheets

• Net meshes are woven or knittedQ • Net meshes allow fibrous tissue in growth between strands and becoming adherent & integrated into host tissuesQ within a few months. • Initial fixation of mesh is by glue, sutures or staples.

• Flat sheets are not porousQ but can be perforated with multiple holes. • ‘Sheet’ meshes do not allow host tissue in growth but become encapsulated by fibrous tissueQ. • Always require strong, non-absorbable fixationQ to prevent mesh migration.

Sutures and Anastomoses

Synthetic mesh

Synthetic polymers of polypropylene, polyester or polytetrafluoroethylene (PTFE). Non-absorbable & provoke little tissue reaction. Polypropylene

Strong monofilament meshQ; Does not have any antibacterial propertiesQ Hydrophobic nature & monofilament microstructure impede bacterial in-growthQ.

Polyester

Braided filament mesh leading to increased risk of infectionQ Hydrophilic property allows rapid vascular & cellular infiltration within the fibrils providing a stronger host–tissue interfaceQ.

PTFE

PTFE meshes are flat sheetsQ ; Do not allow any tissue in-growthQ Used as a non-adhesive barrier between tissue layersQ.

Section 10

Types of Mesh

1069

Synthetic meshes are very strong. All meshes provoke a fibrous reaction. More dense or heavyweight meshes provoke a greater reaction leading to collagen contraction & stiffeningQ. Mesh shrinkage: Progressive decrease in size of a mesh over time due to natural contraction of fibrous tissue embedded in the mesh, reducing the area of mesh itself, leading to tissue tension & pain. It can lead to hernia recurrence if mesh no longer covers the defect. Meshes can shrink by up to 50%Q. Meshes with thinner strands & larger spaces between them, ‘lightweight, large-pore meshes’, are preferred as they have better tissue integration, less shrinkage, more flexibility & improved comfortQ. Light weight: weight 80 g/m2Q

Biological mesh

Sheets of sterilised, decellularised, non-immunogenic connective tissueQ. Derived from human or animal dermis, bovine pericardium or porcine intestinal submucosaQ. Provide a ‘scaffold’ to encourage neovascular in-growth & new collagen depositionQ. Host enzymes eventually break down the biological implant, which is replaced & remodelled with ‘normal’ host fibrous tissueQ.

Absorbable meshes

Made from polyglycolic acid fibreQ. Used in temporary abdominal wall closure & to buttress sutured repairsQ. No current role in hernia repair as they absorb & induce minimal collagen deposition.

Tissueseparating meshes

Designed for intraperitoneal useQ with two different surfaces, one being sticky & one being slipperyQ. Good adherence & host–tissue in-growth is required on parietal (muscle) side of mesh but the opposite (bowel) side needs to prevent adhesions to bowelQ. One side of mesh is coated by material which prevents adhesions, such as polycellulose, collagen, PTFEQ. Limitations to the use of mesh

Others

• Presence of infection limits the use of mesh, particularly heavyweight typesQ. • If a mesh becomes infected then it often needs to be removedQ.

Multiple Choice Questions

SUTURES

1. Which of the following is a non-absorbable suture?  (Recent Question 2016, All India 2008) a. Polypropylene b. Vicryl c. Catgut d. Polydioxanone

2. Absorbable sutures are: a. Catgut c. Polypropylene e. Vicryl



(PGI June 2004) b. Silk d. Polyglycolic acid

3. Surgically used suture material polydioxanone (PDS):  (COMEDK 2014) a. A non-absorbable and remains encapsulated b. Undergoes hydrolysis and complete absorption c. Undergoes phagocytosis and enzymatic degradation d. Is specifically used for heart valves of synthetic grafts 4. Catgut is prepared from submucosal layer of the intestine of: (Bihar PG 2014, DNB 2005, 2000) a. Cat b. Sheep c. Human being d. Rabbit

5. Vicryl, the commonly used suture material is a: (UPSC 2000) a. Homopolymer of polydiozanone b. Co-polymer of glycolide and lactide c. Homopolymer of glycolide d. Homopolymer of lactide 6. PDS is absorbed within: (WBPG 2012, MAHE 2001) a. 7 days b. 21 days c. 100 days d. 225 days 7. Surgically used suture material polydioxanone (PDS):  (WBPG 2012, COMEDK 2005) a. Is non-absorbable and remains encapsulated b. Undergoes hydrolysis and complete absorption c. Undergoes phagocytosis and enzymatic degradation e. Is specifically used for heart valves or synthetic grafts 8. The surgeon who introduce catgut in surgery was: (MAHE 2005) a. Astley Cooper b. Lord Lister c. John Hunter d. Syme

9. Which of the following is a delayed absorbable synthetic suture material? (DPG 2009 Feb) a. Chromic catgut b. Vicryl c. Silk d. Nylon



10. Which of the following is not absorbable suture?  (DNB 2011, APPG 2008) a. Catgut b. Polyamide c. Polyglactin d. Polyester



used as preservative for (AIIMS Nov 2002) Colloidal iodine Hydrogen peroxide time to removal of scalp (MHSSMCET 2009) 5 days 10 days

11. Which one of the following is packing catgut suture? a. Isopropyl alcohol b. c. Glutaraldehyde d. 12. Which of the following is ideal suture?  a. 3 days b. c. 7 days d.

13. Suture material used for laparoscopic choledochotomy repair: a. Silk b. Catgut c. Polyethylene d. Vicryl (MHSSMCET 2010)

14. Catgut is preserved in: (Recent Question 2013) a. Glutaraldehyde b. Isopropyl alcohol c. Iodine d. Cetrimide 15. Raw material used in nylon suture is: (APPG 2015) a. Polyethylene terephthalate b. Polyamide polymer c. Polybutylene terephthalate d. Polyester polymer 16. Which of the following suture has maximum tensile strength and minimum tissue reaction? (Recent Question 2015) a. Polyglycaprone b. Polypropylene c. Polyglactine d. Polydioxanone







17. After a midline laparotomy, you have been asked to suture the incision. What length of suture will you choose? (AIIMS November 2016) a. 2x incision length b. 4x incision length c. 6x incision length d. 8x incision length 18. A woman presents with complete wound dehiscence 4 days after a laparotomy. After prescribing antibiotics for the infection, the surgeon decides to suture the wound. Which of these suture materials should he use? (AIIMS November 2016) a. Vicryl b. Mersilk c. Catgut d. Ethilon

19. Which of the following is the preferred suture material for vascular anastomosis? (Recent Question 2017) a. Non-absorbable, elastic b. Non-absorbable, non-elastic c. Absorbable, elastic d. Absorbable, non-elastic 20. Maximum tissue reaction is seen with:(Recent Question 2018) a. Plain catgut b. Polydioxanone c. Silk d. Chromic catgut 21. Which of the following suture is absorbed in 180 days? (Recent Question 2018) a. Polydioxanone b. Catgut c. Chromic catgut d. Nylon 22. Which of the following is not true about Polydioxanone suture? a. It is a polymer of ether-ester units b. It is biodegradable suture c. It is completely absorbed in 9 months d. Sterilized by ethylene oxide 23. What type of suture is this? (APPG 2015) a. Purse string suture b. Halsted’s suture c. Pare suture d. Mattress interrupted sutures

Sutures and Anastomoses 24. Which of the following statement is correct about the given suture? (Recent Question 2017) a. Natural absorbable b. Synthetic absorbable c. Natural non-absorbable d. Synthetic non-absorbable



25. Which of the following statement is incorrect about the given suture? a. Smooth and strong b. Increased risk of bacterial infection c. Tied knot may become loose d. Consist of single strand



28. A Surgical attending has completed a modified radical mastectomy for a carcinoma breast patient. You have to suture the wound using subcuticular sutures. Which of these sutures will you choose? (AIIMS May 2016)

a.

Section 10



1071

b.

c.

26. What are the copolymers of given suture?  (Recent Question 2016) a. Glycolic acid and trimethylene carbonate b. Glycolide and caprolactone c. Polyglycolic acid and lactide d. Glycolide and lactide



27. What is the preservative used for the given suture? a. 2% glutaraldehyde b. Peracetic acid c. Isopropyl alcohol d. Beta-propiolactone

d. 29. Which the following statement is true about the given suture? (Recent Question 2018)

Others



a. b. c. d.



It is less reactive Derived from cat gut mucosa It is absorbed by phagocytosis and enzymatic dehydration Made from rabbit gut

30. True regarding 10-0 sutures is? (PGI May 2018) a. Thicker than 1-0 sutures b. Synthetic sutures c. Diameter is 0.9 mm d. Stronger than 1-0 e. All of the above 31. Trilene is degraded by:  (Recent Question 2018) a. Enzymatic degradation b. Non-enzymatic degradation c. Chemical degradation d. None

1072

Surgery Essence ANASTOMOSIS



Others











Section 10





32. Disparity of the bowel ends during end to end anastomosis is corrected by: (Karnataka 2004) a. Cheatle’s maneuver b. Connell suture c. Lambert suture d. Czerny technique 33. In abdominal surgery Lembert suture refers to: (Karnataka 2006) a. Single layer suturing b. Sero-muscular sutures c. All coat intestinal suturing d. Skin suturing 34. Colonic anastomosis is most likely to rupture on which postoperative day?  (MHSSMCET 2005) a. 1-2 days b. 3-4 days c. After 7 days d. After 14 day 35. Regarding vascular surgery distal to popliteal artery, which of the following is true? (MHSSMCET 2006) a. Suture with polypropylene b. 6-0 suture used c. Needle pass from within outwards d. All the above 36. When knots are cut short, the free ends of ‘ears’ should be left at least _____ mm long? (MHSSMCET 2008) a. 1-2 mm b. 3-4 mm c. 5-6 mm d. 7-8 mm 37. In elliptical incisions, length to width ratio: (Recent Question 2017) a. 4:1 b. 3:1 c. 2:1 d. 1:1 38. Length of suture required closing the incision to the wound length ratio: (Recent Question 2017) a. 4:1 b. 3:1 c. 2:1 d. 1:1 39. The single layer extramucosal anastomosis was popularized by: (MHSSMCET 2008) a. Carrel b. Antoine Lambert c. Norman Matheson d. Emil Theodar Kocher 40. On table colonic lavage was used for first time in 1968 by:  (MHSSMCET 2008) a. Muir b. Carrel c. Connell d. Lembert 41. Carrel’s triangle is used in: a. Vascular anastomosis b. Bowel anastomosis c. Tendon repair d. Nerve repair



42. Intestinal anastomosis strength is provided by: (JIPMER 2015) a. Mucosa b. Sub mucosa c. Serosa d. Muscularis mucosa 43. Stapler used for MIPH: (Recent Question 2013) a. Linear cutting stapler b. Circular cutting stapler c. Linear stapler d. Circular stapler



44. Tissue suturing glue contains: a. Cyanoacrylate b. Ethanolamine oleate c. Methacrylate d. Polychloroprene

MESH

47. Which of the following statement is incorrect about mesh?  (Recent Question 2017) a. Mesh can shrink upto 50% b. Weight of light-weight mesh is < 40 gm/m2 c. Weight of heavy-weight mesh is > 80 gm/m2 d. Flat sheet meshes does not require fixation



48. What is the weight of low-weight mesh? (Recent Question 2017) a. Surgical Site Infection (15–20%)Q



97. Ans. a. E. coli (Ref: Bailey 27/e p1441) “UTI: Escherichia coli is the most common organism followed by Proteus mirabilis, Staphylococcus epidermidis and Streptococcus faecalis.” Bailey 27/e p1441

Others

Causes of Secondary Psoas Abscess • Crohn’s disease • Diverticulitis • AppendicitisQ • Colon cancer • Ulcerative colitis • Vertebral osteomyelitisQ

1096

Surgery Essence HILTON'S METHOD

98. Ans. a. Protects vital structure (Ref: lessons4medicos.blogspot.com/.../hiltons-method-to-drain-abscesses.)

99. Ans. b. Axillary abscess 100. Ans. b. To prevent injury to vital structure

MISCELLANEOUS 101. Ans. a. Microaerophilic streptococci (Ref: Dorland's Medical Dictionary 28/e p1770, 1771; Bailey 27/e p5, 419, 597) 102. Ans. b. Fungal infection

103. Ans. a. 4

104. Ans. b. Tetanus

105. Ans. c. Rugby

Others

Herpes Gladiatorum • Herpes Gladiatorum is one of the most infectious of herpes-caused diseases transmitted by skin-to-skin contactQ. • Strongly associated with contact sports, also known as herpes rugbiorum or "scrumpox" (after rugby football), "wrestler's herpes" or "mat pox" (after wrestling)Q. 106. Ans. a. Tuberculosis, c. LGV (Ref: Bailey 25/e p1262)

107. Ans. d. None

Syphilitic Balanitis of Follmann • Syphilitic balanitis of Follmann is a very rare manifestation of primary syphilis infectionQ • Develop after the appearance of the primary chancre 108. Ans. c. Malignant change

109. Ans. b. Fifth post operation day (Ref: Bailey 25/e p265) Causes of Post-operative Fever

Day

Cause

• 2–5 days

• Atelectasis of the lung

• 3–5 days

• Superficial and deep wound infectionQ

• 5 days

• Chest infection including viral respiratory tract infection, UTI and thrombophlebitisQ

• >5 days

• Wound infection, anastomotic leakage, intracavitary collections and abscessesQ

Q

110. Ans. b. Complete resection

Antibioma • Antibiotic induced swelling • When an abscess occur in the breast and antibiotic was given, without even draining the abscess, the abscess cavity next will become fibrous and it result in firm to large lump in the breast. • Antibioma can be confused for malignancy, excision is done

Section 10

111. Ans. a. No antibiotics required in clean surgery, c. Spillage of stomach content converts a clean/contaminated case to a contaminated case, d. In clean/contaminated wounds infection rate is 10% (Ref: Sabiston 20/e p224, 251; Schwartz 10/e p 147) • Prophylactic systemic antibiotics are not indicated for patients undergoing low-risk straightforward clean surgical operations in which no obvious bacterial contamination or insertion if a foreign body has occurredQ. • SSI risk has traditionally been correlated to wound class. The accepted range of infection rates has been 1-5 % for clean, 3-11% for clean contaminatedQ, and >27% for dirty wounds • Contaminated wounds: Open accidental wounds encountered early after injury, those with extensive introduction of bacteria into a normally sterile area of the body due to major breaks in sterile technique (e.g. open cardiac massage), gross spillage of viscus contents such as from the intestine, or incision through inflamed, albeit nonpurulent tissueQ. • Hernia repair is clean woundQ. 112. Ans. d. All of the above 113. Ans. a. Southampton grading scale (Ref: Bailey’s 27/e p48) Southampton grading is used for surgical site infections. 114. Ans. b. Large volume of hemoserous discharge (Ref: Bailey 27/e p48)

CHAPTER

48

Fluid, Electrolyte and Nutrition

ADJUVANT NUTRITIONAL SUPPORT Adjuvant Nutritional Support • Methods of adjuvant nutritional support: Enteral nutrition & total parenteral nutrition

ENTERAL NUTRITION Enteral Nutrition • Enteral feeding means delivery of nutrients into the GITQ. • The alimentary tract should be used whenever possibleQ. • This can be achieved with oral supplements (sip feeding) or with a variety of tube-feeding techniques delivering food into the stomach, duodenum or jejunumQ. Advantages of Enteral route over Parenteral Route • Maintains integrity of gastrointestinal tractQ • Reduces the levels of proinflammatory cytokinesQ generated by the gut that • Reduces translocation of gut bacteriaQ that may contribute to hypermetabolism. lead to infection.

ROUTES OF ENTERAL NUTRITION Routes of Enteral Nutrition • Oral supplements by mouth • Nasogastric tube/ Nasojejunal tube: Nasojejunal tube is better due to low risk of aspirationQ • Feeding gastrostomy •  Feeding jejunostomy Gastrostomy Jejunostomy • Surgical technique of placing tube into stomach • Usually preferred over gastrostomy due to lower risk of aspirationQ • Common techniques are: StammQ, WitzelQ & JanewayQ Q • Especially useful in severe • Now-a-days percutaneous insertion under endoscopic control known as PEG pancreatitisQ, in whom a degree of (Percutaneous endoscopic gastrostomy) is preferred gastric outlet obstruction may present • Three methods are used for PEG: Push technique, pull technique & introducer techniqueQ due to edematous head of pancreasQ

ENTERAL NUTRITION: INDICATIONS & CONTRAINDICATIONS Enteral Nutrition

Indications • • • • • •

Protein-energy malnutrition with inadequate oral intakeQ Dysphagia except for fluidsQ Major trauma (or surgeryQ) when return to required dietary intake is prolonged Inflammatory bowel diseaseQ Distal, low-output ( Internal jugular veinQ > Femoral veinQ • Most ICU physicians and anesthetists favor cannulation of internal or external jugular veins as these vessels are easily accessible. Disadvantage: Exit site is situated inconveniently on the side of the neck, where repeated movements result in disruption of the dressing with the attendant risk of sepsisQ • Infraclavicular subclavianQ approach is more suitable for feeding as the catheter then lies flat on the chest wall, which optimizes nursing care. • For longer term TPN Hickman linesQ are preferable (minimize line dislodgement and reduce the possibility of line sepsis). • Post-insertion chest X-ray is essential before feeding is commenced to confirm the absence of pneumothorax and that the catheter tip lies in the distal SVC to minimize the risk of central venous or cardiac thrombosisQ.

INDICATIONS OF TPN

Section 10

Indications for Parenteral Nutrition

Primary Therapy Gastrointestinal cutaneous fistulasQ Renal failureQ (acute tubular necrosis) Short-bowel syndromeQ Acute burnsQ Hepatic failure (acute decompensation superimposed on cirrhosis)Q • Crohn’s diseaseQ • Anorexia nervosaQ • • • • •

Supportive Therapy • Acute radiation enteritisQ • Acute chemotherapy toxicityQ • Prolonged ileusQ • Weight lossQ preliminary to major surgery

Areas Under Intensive Study • Patients with cancer • Patients with sepsis

Fluid, Electrolyte and Nutrition

1099

TPN FORMULATIONS

Section 10

TPN Formulations Solution with Lipids (3-in-1) Solution without Lipids (2-in-1) (60/20/20)Q • Calories from dextrose: 55-60% • Calories from amino acids: 20-25% • Calories from lipids: 20%

(75/25)Q • Calories from dextrose: 75-80% • Calories from amino acids: 20-25%

COMPLICATIONS OF TPN Complications of TPN • MC complication of central venous catheterization (CVC): Catheter related sepsisQ • Most dangerous complication following CVC: PneumothoraxQ Complications of Parenteral Nutrition Related to nutrient deficiency • HypoglycemiaQ, hypocalcemiaQ, hypophosphatemia, hypomagnesemia (refeeding syndrome) • Chronic deficiency syndromes (essential fatty acidsQ, zincQ, mineral and trace elements)

Related to overfeeding • Excess glucose: hyperglycemiaQ, hyperosmolar dehydrationQ, hepatic steatosisQ, hypercapnia, increased sympathetic activity, fluid retention, electrolyte abnormalitiesQ • Excess fat: hypercholesterolemiaQ and formation of lipoprotein X, hypertriglyceridemiaQ, hypersensitivity reactions • Excess amino acids: hyperchloremic metabolic acidosis, hypercalcemiaQ, aminoacidemia, uremiaQ

Related to sepsis

Related to line

• Catheter-related sepsisQ • Increased risk of systemic sepsisQ

• On insertion: pneumothoraxQ, damage to adjacent arteryQ, air embolism,Q thoracic duct damageQ, cardiac perforation or tamponade, pleural effusion, hydromediastinum • Long-term use: occlusion, venous thrombosisQ

Electrolyte Abnormalities in TPN Hyponatremia and hypernatremiaQ Hypokalemia and HyperkalemiaQ Hypophosphatemia and hyperphosphatemiaQ Hypomagnesemia and HypermagnesemiaQ

• • • •

Hypocalcemia and hypercalcemiaQ High zinc and low zincQ High copper and low copperQ Hyperchloremic metabolic acidosisQ

REFEEDING SYNDROME Refeeding Syndrome • Potentially lethal condition • Occur with rapid & excessive feeding of patients with severe underlying malnutrition due to: Starvation, alcoholism, delayed nutritional support, anorexia nervosa & massive weight loss in obese patientsQ Pathophysiology • With refeeding, shift in metabolism from fat to carbohydrate substrate → Increased insulin release → cellular uptake of electrolyte particularly: −− Phosphate → HypophosphatemiaQ −− Magnesium → HypomagnesemiaQ −− Potassium → HypokalemiaQ −− Calcium → HypocalcemiaQ

• Increased risk of arrhythmia, confusion, respiratory failure & death due to dyselectrolytemiaQ

Others

• • • •

1100

Surgery Essence • Occur with both enteral & TPN; More common with TPNQ • Rate of feeding should begin slowly to prevent metabolic changesQ Treatment • • • •

Treatment involves matching intake with requirementsQ Avoid over feeding; Calorie delivery should be increased slowlyQ Vitamin administration, especially thiamine before initiation of feedingQ Hypophosphatemia & hypomagnesemia requires treatmentQ

NON-ANION GAP ACIDOSIS

Others

Causes of Non-Anion-Gap Acidosis

Tube-related • External pancreatic or small-bowel drainageQ • Ureterosigmoidostomy, jejunal loop, ileal loopQ • Drugs: −− Calcium chloride (acidifying agent) −− Magnesium sulphate (diarrhea) −− Cholestyramine (bile acid diarrhea)

Gastrointestinal

Metabolic

• Hypokalemia −− Proximal RTA (type 2)Q −− Distal (classic) RTA (type 1)Q • Hyperkalemia −− Type 4 RTAQ −− Mineralocorticoid deficiencyQ −− Mineralocorticoid resistanceQ −− Na delivery to distal Nephron −− Tubulointerstitial disease −− Ammonium excretion defect

Infective

• Potassium sparing diuretics (amiloride, triamterene, spironolactone) • TrimethoprimQ • PentamidineQ • ACE inhibitors and AT-II receptor blockersQ • NSAIDsQ • CyclosporineQ

• Acid loads (ammonium chloride, hyperalimentation) • Loss of potential bicarbonate: ketosis with ketone excretion • Expansion acidosis (rapid saline administration) • Hippurate • Cation exchange resins

COMPOSITION OF CRYSTALLOIDS & COLLOIDS Composition of crystalloid and colloid solutions (mM/L) Solution

Na+

K+

Ca2+

Cl-

Lactate

Hartmann’s (RL)

130

4

< 2.7

109

28

Normal saline (0.9% NaCl)

154

154

Dextrose saline (4% dextrose in 0.18% saline)

30

Gelofusine

150

Hemacel

145

5.1

30

121

35

90%



74. Insensible daily water loss is:  (Recent Question 2016) a. 500-600 ml b. 800-1000 ml c. 1000-1500 ml d. 2000 ml



75. Critical pH in Mendelson syndrome:  (Orissa 2004) a. 2.5 b. 3.0 c. 3.5 d. 4.0 76. The ideal colloidal solution is: (MHSSMCET 2005, MHPGMCET 2006) a. Dextran b. Plasma c. Albumin d. Hydroxyethyl starch 77. Skin fold thickness (for assessment of nutritional status) can be measured at all the following except: (MHSSMCET 2006) a. Biceps b. Triceps c. Suprailiac region d. None





78. In surgical patient malnutrition is best assessed by: a. Serum albumin (Recent Question 2013) b. Hb level c. Mid arm circumference d. Triceps skin fold thickness

Others



59. A postoperative patient with pH 7.25, MAP (mean arterial pressure) 60 mm Hg treated with:  (PGI 2003) a. IV sodium bicarbonate b. Only normal saline c. Fluid therapy with CVP monitoring d. Fluid restriction 60. Most common cause of water intoxication in surgical patient is due to:  (COMEDK 2005) a. Colorectal wash with plain water b. Syndrome of inappropriate secretion of ADH c. Irrigation during transurethral resection of prostate d. Excessive infusion of 5% glucose 61. A young man weighing 65 kg was admitted to the hospital with severe burns in a severe catabolic state. An individual in this state requires 40 kcal per kg body weight per day 1 gm of protein/kg body weight/day. This young man was given a solution containing 20% glucose and 4.25% protein. If 3000 ml of solution in infused per day: (AIIMS Nov 2003) a. The patient would not be getting sufficient protein b. The calories supplied would be inadequate c. Both protein and calories would be adequate d. Too much protein is being infused 62. After 30% loss of blood volume in road traffic accident. What is the next management? a. IV fluids only b. IV fluids with cardiac stimulant c. Dopamine d. Vasopressor drug



Section 10



1103

Explanations ENTERAL NUTRITION

1. Ans. b. Ear lobe to the umbilicus • Feeding tube length is measured by following the normal route for the tube (Nasal ala → To ear lobe → To epigastrium)Q • The distance between the nasal ala and ear lobe is almost equal to the distance between the epigastrium and umbilicus, the length can be measured from ear lobe to umbilicusQ.



2. Ans. a. Jejunostomy tube feeding (Ref: Sabiston 20/e p112; Bailey 27/e p284-286)

Unless the GI tract is nonfunctional, its use for nutritional support is preferable as compared to TPN. In a patient who is comatose either NG feeding or feeding through a gastrostomy tube may lead to vomiting and aspiration. This can be avoided by using a nasoenteric tube with the tip placed in jejunum under fluoroscopic guidance or endoscopic control. Alternatively, a catheter may be placed directly into the proximal jejunum through a small upper abdominal incision.

3. Ans. a. Intestinal obstruction, b. Severe pancreatitis, c. Severe diarrhea, e. Intestinal fistula



4. Ans. c. Feeding jejunostomy (Ref: Essentials of General Surgery by Lawrence 4/e p80)

• A feeding jejunostomy is the preferred procedure to provide supplementary nutrition in this patient as it provides protection to the pancreatic anastomosis and cause minimal stimulation of pancreatic secretion thereby giving rest to the pancreasQ. • After pancreatic surgery, TPN should or only be used when there is intolerance to enteral nutrition or enteral nutrition is contraindicatedQ. • Pancreatic surgery does not usually interfere with the function of the gastrointestinal tract and enteral route remains the preferred mode for providing nutrition. Operations where Early Oral Feeding is not Recommended • Major Hepatic SurgeryQ • Major Pancreatic SurgeryQ

• Major surgery involving upper GI tract: −− Esophageal resectionQ −− Gastric resectionQ

5. Ans. a. Constipation, b. Diarrhea, c. Aspiration pneumonia (Ref: Sabiston 20/e p117; Bailey 27/e p286)



6. Ans. c. IBD



7. Ans. c. Retraction method (Ref: Sabiston 20/e p115)

TOTAL PARENTERAL NUTRITION

8. Ans. e. Pancreatitis (Ref: Sabiston 20/e p118; Bailey 27/e p286)



9. Ans. a. Folic acid, b. Iron, c. Vitamin B12, d. Copper, e. Fatty acids (Ref: Sabiston 20/e p121; Bailey 27/e p288)



10. Ans. d. Fibers (Ref: Bailey 27/e p286) Total Parenteral Nutrition Energy Sources • CarbohydratesQ • LipidsQ • Amino acidsQ

Non-energy Essential sources • • • •

ElectrolytesQ Trace elementsQ VitaminsQ Water



11. Ans. a. Hypoglycemia, b. Hyperglycemia, c. Hypercalcemia, d. Hypercapnia, e. Hypophosphatemia



12. Ans. a. Subclavian vein (Ref: Sabiston 20/e p118; Bailey 27/e p287) • Preferred site for central vein infusion: SVCQ • Preferred access site for TPN: SubclavianQ > Jugular > Femoral vein

Fluid, Electrolyte and Nutrition

13. Ans. c. Chronic liver disease



15. Ans. d. Lipids form 20% of energy source (Ref: Sabiston 20/e p119)



16. Ans. d. Hyperphosphatemia



18. Ans. None (Ref: CSDT 12/e p161)



19. Ans. a. Parenteral



21. Ans. a. Catheter related complications

14. Ans. c. Sepsis

Section 10

17. Ans. c. Congestive cardiac failure 20. Ans. c. Hypotriglyceridemia

• MC complication of central venous catheterization (CVC): Catheter related sepsisQ • Most dangerous complication following CVC: PneumothoraxQ

1105

22. Ans. a. Hypomagnesemia (Ref: Harrison 19/e p88-97) Clinical Features Hypomagnesemia

Hyperkalemia

• Weakness, muscle cramps and tremorsQ • Marked neuromuscular and CNS hyperirritability with jerking and nystagmusQ

• Interferes with neuromuscular function to produce muscle weakness which may progress to flaccid paralysisQ and hypoventilation if respiratory muscles are involved

Hypercalcemia • Fatigue, depression, mental confusion, lethargyQ • Anorexia, nausea, vomiting, constipation and polyuriaQ



23. Ans. b. Fats, c. Amino acids, d. Dextrose

24. Ans. e. All of the above

25.  Ans. a. It is costly



26. Ans. b. 7 days

27. Ans. d. 60 gm/day

28. Ans. None



29. Ans. a. Up to 7 days

30. Ans. c. Hypercalcemia

31. Ans. c. Metabolic acidosis

ELECTROLYTE ABNORMALITIES

32. Ans. d. Metabolic acidosis (Ref: Sabiston 20/e p1869, 1205; Schwartz 10/e p73-75, 1599; Bailey 25/e p1065) • Chronic vomiting leads to Hypochloremic, hypokalemic, metabolic alkalosis.



33. Ans. b. Vomiting (Ref: Harrison 19/e p320)

Causes of High Anion–Gap Metabolic Acidosis



Lactic acidosisQ KetoacidosisQ (Diabetic, Alcoholic, Starvation) Toxins (Ethylene glycol, Methanol, Salicylates, Propylene glycol, Pyroglutamic acid)Q Renal failureQ (acute and chronic)

34. Ans. d. Pyloric stenosis

Others

• • • •

35. Ans. c. Duodenal (Ref: Sabiston 20/e p1287)

Enterocutaneous Fistulas • Enterocutaneous fistulas are classified according to their location and volume of daily output. • These factors dictate both treatment and morbidity and mortality ratesQ. • In general, the more proximal the fistula in the ‘intestine’ (but not the stomach), the more serious the problem, with greater fluid and electrolyte lossQ. • Maximum fluid and electrolyte imbalance occur in duodenal fistulasQ.

36. Ans. d. Colon (Ref: Schwartz 10/e p69; Bailey 27/e p281) • Maximum K+ concentration: ColonQ (30 mEq/L) >Saliva (25 mEq/L) Composition of GI Secretions Type of Secretion

Volume (mL/day)

Na (mEq/L)

K (mEq/L)

Cl (mEq/L)

HCO3– (mEq/L)

Stomach

1000–2000Q

60–90

10–30

100–130Q

0

Small intestine

2000–3000

120–140

5–10

90–120

30–40

Q

1106

Surgery Essence Colon



60

30Q

40

0

Pancreas

600–800Q

135–145Q

5–10

70–90

95–115Q

Bile

300–800Q

135–145Q

5–10

90–110

30–40



37. Ans. b. Pyloric stenosis



38. Ans. d. Pseudo (Ref: Harrison 19/e p714)

Pseudohyponatremia

Others

• Patients with multiple myeloma also have a decreased anion gap [i.e., Na+ – (Cl– + HCO3–)] because the M component is cationic, resulting in retention of chlorideQ. • This is often accompanied by hyponatremia that is felt to be artificial (Pseudohyponatremia) because each volume of serum has less water as a result of the increased proteinQ.

39. Ans. b. Decreased K+ in urine

40. Ans. a. Hyperchloremic acidosis



42. Ans. c. Excessive sweating

43. Ans. c. Hypokalemia

41. Ans. c. Hyperkalemic acidosis

IV FLUIDS

44. Ans. d. Darrow’s solution (Ref: Bailey 27/e p281; www.idruginfo.com/?cat=drug...Darrow’s%20Solution)



45. Ans. c. 4.5

46. Ans. b. 150 g



47. Ans. a. Metabolic alkalosis

48.  Ans. b. Degraded gelatin



49. Ans. d. Excreted excessively • In immediate post-operative period due to increased adrenocortical activity, there is Na+ retention and K+ excretionQ.



50. Ans. d. Thrombocytopenia • Dextran interferes with platelet functionQ.



51. Ans. a. Tissue perfusion (Ref: Sabiston 20/e p555, 52; Bailey 27/e p17) • Urine output is best clinical guide of tissue perfusionQ.



52. Ans. c. Hypertonic



55. Ans. a. GH levels decreased

53. Ans. c. Cardiopulmonary problem 56.  Ans. c. 130

54.  Ans. b. Increase in urine output



57. None

58.  Ans. b. 154 mEq



59. Ans. c. Fluid therapy with CVP monitoring

60.  Ans. d. Excessive infusion of 5% glucose



61. Ans. c. Both protein and calories would be adequate (Ref: Sabiston 20/e p119; Bailey 27/e p282) • Calories are calculated by catabolism of glucose (not proteins).

Section 10

Glucose: • Amount of glucose in 20% glucose in 3000 ml of solution: 3000 × 20/100 = 600 gms • 1 gm glucose on catabolism produces: 4.2 kcal • 600 gms of glucose would produce: 600 x 4.2 = 2520 kcal Protein: • Percentage of protein in fluid: 4.25% • Percentage of protein in 3000 ml of fluid: 3000 × 4.25/100 = 127.5 gms • Calories required for the patient: 40x65 = 2600 kcal • Proteins required for the patient: 2 x65 = 130 gms

Calories Proteins

Required Amount 2600 kcal 130 gms

Supplied by Solution 2520 kcal 127.5 gm



62. Ans. a. IV fluids only (Ref: Sabiston 20/e p553)



63. Ans. a. Urine output



64. Ans. d. 3% normal saline (Ref: Fluids and Electrolytes by Lippincott Williams and Wilkins/55)

Fluid, Electrolyte and Nutrition Isotonic

Hypertonic 5% dextrose in half normal saline

0.9% normal saline

5% dextrose in normal saline

Ringer lactate

Dextrose 10% in water



65. Ans. d. Hyponatremia



66. Ans. b. Suction bottles (Ref: www.ncbi.nlm.nih.gov/pubmed/16756621)

Hypotonic 0.45 normal saline

Measurement of Blood Loss during Surgery (Gravimetric Method) • Blood loss during operation is measured by: Weighing the swabs after use and subtracting the dry weight and fluid used + volume of blood collected in suction bottles (after subtracting irrigating fluid)Q

67. Ans. c. 130

Section 10

Dextrose 5% in water

1107

68. Ans. c. Ringer lactate (Ref: Schwartz 10/e p76)

`

MISCELLANEOUS

69. Ans. a. Hypertonic dehydration, c. Dumping syndrome



70. Ans. c. Mid-arm circumference (Ref:www.ncbi.nlm.nih.gov/pubmed/1492750)

Anthropometric Techniques for Nutritional Assessment • Anthropometric techniques incorporating measurements of skinfold thicknesses and mid-arm circumference permit estimations of body fat and muscle mass, and these are indirect measures of energy and protein stores. • Skinfold thickness is measured at ulnar, triceps, subscapular and suprailiac regionQ.

71. Ans. d. 3 days (Ref: adc.bmj.com/content/50/7/555.full.pdf) • The normal rise in cortisol secretion after surgery lasts about 3 days.



72. Ans. b. Fat woman



73. Ans. b. 75-80% (Ref: Sabiston 20/e p48; Bailey 27/e p280)



74. Ans. b. 800-1000 mL (Ref: Bailey 25/e p226)



75. Ans. a. 2.5 (Ref: Sabiston 19/e p293)

Mendelson’s Syndrome



76. Ans. c. Albumin



78. Ans. c. Mid arm circumference (Ref: Sabiston 20/e p562)

77. Ans. d. None

Others

• Critical pH value: 2.5 • Gastric pH of 2.5 or less with a gastric contents volume greater than 25 ml are critical valuesQ. • Maximum pulmonary damage is achieved at an aspirate pH value of 1.5. Q

CHAPTER

49

Blood Transfusion

TRANSFUSION PROTOCOL Transfusion protocols • BT should commence within 30 minutes or removing blood bag from refrigerators because of increased risk of bacterial contaminationQ • Whole blood or packed RBC transfusion must be completed within 4 hoursQ • Platelet and FFP transfusion should be completed within 20 minutesQ • Transfusion set should have standard filter of 170 μm sizeQ • Usual transfusion needle size should be of 18-19 gaugeQ

CHARACTERISTICS OF SELECTED BLOOD COMPONENETS Characteristics of Selected Blood Components Component

Volume (mL)

Content

Clinical Response

Whole Blood

450 ml ± 45

• No elements removed • Not for routine use • Contains RBCs, WBCs, plasma and platelets • Used for acute massive bleeding, open heart (WBCs and platelets may be non-functionalQ) surgery and neonatal total exchange

Packed RBCs

180–200

• RBCs with variable leukocyte content and small amount of plasma

• Increase Hb 1 gm/dL and hematocrit 3%Q

Platelets

50–70

• 5.5 x 1010/RD unit

• Increase platelet count 5000–10,000/µLQ

FFP

200–250

• Plasma proteins: Coagulation factors, proteins C and S, antithrombinQ

• Increases coagulation factors about 2%

Cryoprecipitate

10–15

• Cold-insoluble plasma proteins, fibrinogen, factor VIII, vWFQ

• Topical fibrin glue, also 80 IU factor VIIIQ

ANTICOAGULANTS Whole Blood Anticoagulant used ACD/CPD/CP2D

Maximum storage 21 daysQ

CPDA-1

35 daysQ

(citrate phosphate dextrose adenine) SAGM (saline adenine glucose mannitol

42 daysQ

Actions of Ingredients of anticoagulant solution Glucose

• ATP generation by glycolysisQ

Adenine

• Synthesis of ATPQ • Increases shelf life of RBC to 42 daysQ

Citrate

• Prevents coagulation by chelating calciumQ

Sodium diphosphate

• Maintain optimum pHQ

Blood Transfusion

1109

MASSIVE BLOOD TRANSFUSION

Effects due to Dilution of Sample

Effects due to Acidic Nature of Heparin

• Decreased PaCO2Q • Decreased HCO3Q

• Decreased pHQ

Metabolic Complications of Massive Transfusion General • Fluid overloadQ • HypothermiaQ • Impaired oxygen delivery capacity of HbQ (decreased 2, 3-DPG)

• • • • •

Section 10

Heparin Excess in Blood gas Syringe

Electrolyte HyperkalemiaQ HypocalcemiaQ HypomagnesemiaQ Metabolic alkalosisQ Metabolic acidosis (rare)Q

COMPLICATIONS OF BLOOD TRANSFUSION Complications of Blood Transfusion Reactions • Febrile non-hemolytic transfusion reaction (FNHTR): MCQ • Allergic • Delayed hemolyticQ • Transfusion-related acute lung injury (TRALI)Q • Acute hemolyticQ • Fatal hemolytic • Anaphylactic

• • • • • • • • •

Infections Hepatitis B and CQ Hepatitis GQ HIV-1 and -2Q HTLV-I and -IIQ MalariaQ West Nile virusQ Parvovirus B-19Q HHV-8Q CMVQ

Other Complications  • RBC allosensitizationQ • HLA allosensitizationQ • Graft-versus-host disease

RED BLOOD CELLS

• RBCs are stored at 1-6°CQ; Mean life of transfused RBCs is 35 daysQ. Anticoagulant used

Maximum storage

ACD/CPD/CP2D

21 daysQ

CPDA-1

35 daysQ

PLATELETS Platelet Concentrates • Volume: 50 mlQ • Platelets are the only blood products which are stored at room temperature, 20-240 CQ (survival is 4-5 days)Q. • 1 unit of platelet increases the count by 5000-10000Q. • The threshold for prophylactic platelet transfusion is 10,000/µLQ. • For invasive procedures, 50,000/µL platelets is the usual target level. • Platelet count should be 1,00,000/µL before accepting the patient for surgery.

Others

Red Blood Cells

1110

Surgery Essence • Transfused platelets generally survive for 2-7 days following transfusion. • ABO compatibility is desirable but not necessary. • Blood platelets in stored blood are non-functional after 24 hoursQ.

FRESH FROZEN PLASMA Fresh-frozen Plasma (FFP) • FFP is produced from the separation of plasma from donated bloodQ. • Stored at -18°C and has a shelf life of 1 yearQ.

Others

• Each unit contains 400 mg of fibrinogen and 1 unit activity of each of the clotting factorsQ. • Most labile clotting factors (V and VIII) may be diminishedQ proportional to shelf life. • FFP contains stable coagulation factors and plasma proteins: fibrinogen, antithrombin, albumin, proteins C and SQ. Indications for FFP • Correction of coagulopathies: −− Rapid reversal of warfarinQ −− Supplying deficient plasma proteinsQ

• Treatment of thrombotic thrombocytopenic purpuraQ

• Patients who are IgA-deficient and require plasma support should receive FFP from IgA-deficient donors to prevent anaphylaxis. • FFP should not be routinely used to expand blood volumeQ. • FFP: An acellular component and does not transmit intracellular infections, e.g., CMV.

CRYOPRECIPITATE Cryoprecipitate • Cryoprecipitate is a source of fibrinogen , factor VIII and von Willebrand factor (vWF)Q. • It is ideal for supplying fibrinogen to the volume-sensitive patient. Q

Q

• Stored at ≤-18°C • 1 unit of cryoprecipitate contains 80-145 units of Factor VIII and 250 mg of fibrinogenQ. • Cryoprecipitate is pooled from many donors, so there are maximum chances of disease transmission among all blood productsQ. • Cryoprecipitate may also supply vWF to patients with dysfunctional (type II) or absent (type III) von Willebrand disease.

Section 10

DEXTRAN Dextran • It is a polysaccharide polymer of varying molecular weight producing an osmotic pressure similar to the plasma • Disadvantages: −− It induces rouleaux of RBCs and this interferes with blood grouping and cross matchingQ procedures, hence need for a blood sample beforehand. −− It interferes with platelet function, hence it is recommended that total volume of dextran should not exceed 1000 mL. • LMW dextran (short acting) prevents sludging of RBCs in vessels and renal shut down in severe hypotension and it is less likely to induce rouleaux formation than HMW dextran (long acting).

Multiple Choice Questions BLOOD TRANSFUSION

1. MC blood transfusion reaction is: (All India 2008) a. Febrile non-hemolytic transfusion reaction b. Hemolysis c. Transmission of infections d. Electrolyte imbalance



2. All of the following infections may be transmitted via blood transfusion, except: (AIIMS May 2009, All India 2002) a. Parvo B-19 b. Hepatitis G c. Dengue virus d. Cytomegalovirus



3. Which of the following is the least likely complication after massive blood transfusion? (AIIMS May 2009) a. Hyperkalemia b. Citrate toxicity c. Hypothermia d. Metabolic acidosis



4. Fresh hold blood transfusion is done with in how much time of collection? (DNB 2006) a. Immediately b. 1 hours c. 4 hours d. 24 hours



5. Which of the following investigations should be done immediately to best confirm a non matched blood transfusion reaction? (All India 2010) a. Indirect Coomb’s test b. Direct Coomb’s test c. Antibody in patient’s serum d. Antibody in donor serum



6. Blood components products are: (PGI Dec 2005) a. Whole blood b. Platelets c. Fresh frozen plasma d. Leukocyte reduced RBC e. All of the above



7. A man is rushed to casualty, nearly dying after a massive blood loss in an accident. There is not much time to match blood groups, so the physician decides to order for one of the following blood groups. Which one of the following blood groups should the physician decide: (AIIMS June 2004) a. O negative b. O positive c. AB positive d. AB negative







8. One unit of fresh blood arises the Hb% concentration by:  (All India 2003, DNB 2012) a. 0.1 gm% b. 1 gm% c. 2 gm% d. 2.2 gm% 9. Which of the following statements about acute hemolytic blood transfusion reaction is true? (PGI June 2004) a. Complement mediated hemolysis is seen b. Type III hypersensitivity is responsible for most cases c. Rarely life threatening d. Renal blood flow is always maintained e. No need for stopping transfusion 10. True about blood transfusions: (PGI June 98) a. Antigen ‘D” determines Rh positivity b. Febrile reaction is due to HLA antigens c. Anti-d is naturally occurring antibody d. Cryoprecipitate contains all coagulation factors



11. Which of the following is better indicator of need for transfusion? (Recent Question 2016) a. Urine output b. Hematocrit c. Colour of skin d. Clinical examination



12. Massive blood transfusion is defined as: (PGI 95) a. 350 ml in 5 min b. 500 ml in 5 min c. 1 litre in 5 min d. Whole blood volume



13. How long can blood stored with CPDA? a. 12 days b. 21 days c. 28 days d. 48 days



14. Storage period of 35 days for blood is seen with: (AIIMS November 2017) a. CPD b. CPDA-1 c. ACD
 d. CP2D

(JIPMER 2003)

15. Massive transfusion in previous healthy adult male can cause hemorrhage due to: (PGI 98) a. Increased t-PA b. Dilutional thrombocytopenia c. Vitamin K deficiency d. Decreased fibrinogen

16. Arterial blood gas analysis in a bottle containing heparin causes a decrease in value of: a. pCO2 b. HCO3 c. pH d. All of the above



17. Massive blood transfusion is defined as: a. Whole blood volume in 24 hours (Recent Question 2013) b. Half blood volume in 24 hours c. 40% blood volume in 24 hours d. 60% blood volume in 24 hours

BLOOD TRANSFUSION COMPLICATIONS 18. After blood transfusion the febrile non-hemolytic transfusion reaction (FNHTR) occurs due to? a. Alloimmunization b. Antibodies against donor leukocytes and HLA Ag c. Allergic reaction d. Anaphylaxis

19. Blood grouping and cross-matching is must prior to infusion of:  (MHPGMCET 2007) a. Gelatin b. Dextran c. Albumin d. FFP



20. Blood grouping and cross matching is must prior to infusion of: (MHPGMCET 2008) a. Gelatin b. Albumin c. Dextran d. Hemaceal



21. Collection of blood for cross matching and grouping is done before administration of which plasma expander?  (MHSSMCET 2007) a. Hydroxyl ethyl starch b. Dextran c. Mannitol d. Hemacele

1112

Surgery Essence

22. Mismatched blood transfusion in anesthetic patient presents is: (PGI June 2000) a. Hyperthermia and hypertension b. Hypotension and bleeding from site of wound c. Bradycardia and hypertension d. Tachycardia and hypertension



31. Stored plasma is deficient in: (Recent Question 2016) a. Factors 7 and 8 b. Factors 2 and 5 c. Factors 5 and 8 d. Factors 7 and 9



32. With reference to fresh frozen plasma (FFP), which one of the following statement is not correct? (UPSC 2008) a. It is used as volume expander b. It is stored at – 40°C to – 50°C c. It is a source of coagulation factors d. It is given in a dose of 12-15 ml/kg body weight



33. In cholecystectomy, fresh frozen plasma should be given: a. Just before operation  (UPPG 2008) b. At the time of operation c. 6 hours before operation d. 12 hours after operation



34. Half life of factor VIII is: a. 4 hours c. 34 hours



35. Rosenthal’s syndrome is seen in deficiency of factor: a. II b. V (DNB 91) c. IX d. XI

23. All of the following are major complications of massive transfusion except: (All India 2006) a. Hypokalemia b. Hypothermia c. Hypomagnesaemia d. Hypocalcaemia

Others



24. Massive transfusions results in: (Recent Question 2016) a. DIC b. Hypothermia c. Hypercalcemia d. Thrombocytopenia

RED BLOOD CELLS

25. The maximum life of a transfused RBC is:  (Recent Question 2016) a. One hour b. One day c. 15 days d. 50 days

PLATELETS

26. Platelets can be stored at: a. 20-24°C for 5 days c. 4-8°C for 5 days

(AIIMS Nov 2005) b. 20-24°C for 8 days d. 4-8°C for 8 days

27. Blood platelets in stored blood do not remain functional after: (Recent Question 2016) a. 24 hours b. 48 hours c. 72 hours d. 96 hours 28. In a patient with thrombocytopenia, what is the target platelet count after transfusion to perform an invasive procedure? (AIIMS May 2015) a. 30,000 b. 40,000 c. 50,000 d. 60,000

PLASMA

Section 10



29. Indication of fresh frozen plasma is/are: a. Hypovolemia b. Nutritional supplement c. Coagulation factor deficiency d. Warfarin toxicity e. Hypoalbuminemia

(PGI Nov 2011)

30. True about FFP (Fresh frozen plasma) is the following except: (MHPGMCET 2009) a. Good source of all coagulation factors b. Prepared from single unit of blood c. Coagulation factor levels are equal to Plasma d. None of the above

(Recent Question 2016) b. 8 hours d. 48 hours

36. Best blood product to be given in a patient of multiple clotting factor deficiency with active bleeding: a. Whole blood (AIIMS May 2015) b. Packed RBCs c. Fresh frozen plasma d. Cryoprecipitate

CRYOPRECIPITATE (MCI March 2009) b. Factor V d. Factor IX



37. Cryoprecipitate contains: a. Factor II c. Factor VIII



38. Cryoprecipitate is a rich source of: (Recent Question 2016) a. Thromboplastin b. Factor VIII c. Factor X d. Factor VII

39. Which one of the following blood fractions is stored at -40°C? (UPSC 2006) a. Cryoprecipitate b. Human albumin c. Platelet concentrate d. Packed red cells 

40. Cryoprecipitate contains all except: a. Factor VIII c. Fibrinogen

(MHCET 2016, AIIMS Nov 2007) b. Factor IX d. VWF

Explanations BLOOD TRANSFUSION

1. Ans. a. Febrile non-hemolytic transfusion reaction (Ref: Harrison 19/e p138e-3, 18/e p954-956)



2. Ans. c. Dengue virus



4. Ans. d. 24 hours



5. Ans. b. Direct Coomb’s test (Ref: Harrison 19/e p138e-4, 18/e p954)



3. Ans. d. Metabolic acidosis (Ref: Harrison 19/e p138e-3, 18/e p954-956)

6. Ans. e. All of the above (Ref: Harrison 19/e p138e-2, 18/e p952-954)



7. Ans. a. O negative (Ref: Harrison, 19/e p138 e-1, 18/e p951; Bailey 27/e p21-22, 26/e p21-22, 25/e p21-22)



8. Ans. b. 1 gm%



9. Ans. a. Complement mediated hemolysis is seen (Ref: Harrison 19/e p138e-3, 18/e p954)



10. Ans. a. Antigen ‘D” determines Rh positivity; b. Febrile reaction is due to HLA antigens (Ref: Harrison 19/e p138e-1, 138e-4, 18/e p954)



11. Ans. b. Hematocrit



14. Ans. b. CPDA-1 (Ref: Wintrobe’s 13/e p1278)



16. Ans. d. All of the above (Ref: Clinical Laboratory Medicine 6/e p396)



17. Ans. a. i.e., Whole blood volume in 24 hours

12. Ans. d. Whole blood volume

13. Ans. c. 28 days (Ref: Sabiston 19/e p588)

15. Ans. b. Dilutional thrombocytopenia

BLOOD TRANSFUSION COMPLICATIONS

18. Ans. b. Antibodies against donor leukocytes and HLA Ag



21. Ans. b. Dextran



22. Ans. b. Hypotension and bleeding from site of wound (Ref: Schwartz 10/e p119,122,171-172, 9/e p83)



23. Ans. a. Hypokalemia



19. Ans. b. Dextran

20. Ans. c. Dextran

24. Ans. a. DIC; b. Hypothermia; d. Thrombocytopenia

RED BLOOD CELLS 25. Ans. d. 50 days (Ref: Schwartz 10/e p1914-1915, 9/e p78; Bailey 27/e p21, 26/e p21, 25/e p21)

PLATELETS

26. Ans. a. 20-24° C for 5 days (Ref: Harrison 18/e p953; Sabiston 19/e p588; Schwartz 10/e p85, 9/e p79; Bailey 27/e p21, 26/e p21)



27. Ans. a. 24 hours 28. Ans. c. (50,000) (Bailey 26/e p23; Nelson 20/e p2374)

PLASMA

29. Ans. c. Coagulation factor deficiency; d. Warfarin toxicity (Ref: Harrison 19/e p138e-3, 18/e p953; Sabiston 19/e p588)



30. Ans. a. Good source of all coagulation factors

31. Ans. c. Factors 5 and 8



32. Ans. a. It is used as volume expander

33. Ans. a. Just before operation



34. Ans. b. 8 hours



35. Ans. d. XI (Ref: http://en.wikipedia.org/wiki/Haemophilia_C)



36. Ans. c. Fresh frozen plasma (Harrison 19/e p138 e-3, 18/e p953; Sabiston 20/e p568, 19th/e p588)

CRYOPRECIPITATE

37. Ans. c. Factor VIII (Ref: Harrison 18/e p953; Sabiston 19/e p588; Schwartz 10/e p73-75,1599, 9/e p82; Bailey 26/e p21, 25/e p21)



38. Ans. b. Factor VIII



39. Ans. a. Cryoprecipitate



40. Ans. b. Factor IX

CHAPTER

50

Shock

SHOCK Shock • Shock is a clinical syndrome resulting from inadequate tissue perfusion • Shock is MC cause of death among surgical patientsQ Classification of Shock 1. HypovolemicQ (MC type) 2. Traumatic 3. Cardiogenic

4. Septic: HyperdynamicQ (early) & HypodynamicQ (late) 5. Neurogenic 6. Hypoadrenal Monitoring in Shock

1. The best management of shock is done by putting pulmonary catheter. PCWP is considered better guide than CVP for fluid titrationsQ as it can also determine left ventricular preload. 2. Invasive arterial pressure is mandatoryQ. 3. Blood gas analysisQ. There is metabolic acidosis in shock. 4. Mixed venous oxygen saturation is considered as best guide for tissue perfusion (i.e. cardiac output) 5. Urine output is best clinical guide of tissue perfusionQ.

HYPOVOLEMIC SHOCK Hypovolemic Shock • MC type of shockQ • Causes of hypovolemic shock: −− Blood lossQ (Trauma, bleeding) −− Loss of plasma due to extravascular fluid seqestration in burnsQ −− Loss of body sodium & water (diarrhea & vomitingQ) Pathophysiology of Hypovolemic Shock • ↓ Intravascular volume → ↓ Venous return → ↓ Ventricular filling → ↓ Stroke volume → ↓ Cardiac output → Inadequate tissue perfusion & compensatory mechanism activationQ • Inadequate tissue perfusion → Cold clammy skin; Oliguria → anuria; Drowsiness & confusionQ • Inadequate tissue perfusion → Lactic acidosisQ → Metabolic acidosisQ → TachypneaQ • Compensatory mechanism activationQ → Sympathetic stimulationQ (↑Adrenaline & ↑Noradrenaline) Sympathetic stimulation (↑ Adrenaline & ↑ Noradrenaline) • TachycardiaQ (↑Pulse rate) • Weak or thready pulse (↑Total peripheral resistance)

• SweatingQ • Initially maintained BP followed by hypotension

Shock

IV >40Q Confused or lethargic >140 DecreasedQ DecreasedQ >35/minQ NegligibleQ Crystalloid + bloodQ –10 mEq/L or less

Section 10

Parameter Blood loss (%) CNS Pulse (beats/min) Blood pressure Pulse pressure Respiratory rate Urine (mL/hr) Fluid Base deficit

Four Classes of Hemorrhagic Shock (According to the ATLS course) Class I II III 0-15Q 15-30Q 30-40Q Slightly anxious Mildly anxious Anxious or confused 100 >120 Normal Normal DecreasedQ Normal Decreased DecreasedQ 14-20/min 20-30/min 30-40/minQ >30 20-30 5-15 CrystalloidQ CrystalloidQ Crystalloid + bloodQ 0 to –2 mEq/L –2 to –6 mEq/L –6 to –10 mEq/L

1115

Clinical Features • History of trauma, bleeding, burns, diarrhea & vomiting Class I • Compensatory mechanism maintains cardiac output • Compensated hypovolemic shockQ

Class II

Class III

• HypoxemiaQ • HypotensionQ • Generalized vasoconstriction • Urine output: 20-30 mL/hour

• • • • • •

Decompensated shockQ (↓ CO, ↓ SBP) HypotensionQ TachycardiaQ (PR >120/min) TachypneaQ Urine output: 5-15 mL/hourQ Patient is confusedQ

Class IV • • • • •

Refractory stageQ Marked hypotensionQ Tachycardia & tachypneaQ No urine outputQ Patient is comatoseQ

CARDIOGENIC SHOCK Cardiogenic Shock • Impaired ability of the heart to pump blood Causes of Cardiogenic Shock • Myocardial infarctionQ • Myocardial depressants: Beta-blockers, calcium channel blockers, anti-arrhythmic

Diastolic dysfunction

• Ventricular hypertrophyQ; Restrictive cardiomyopathyQ; Cardiac tamponadeQ

Increased after load

• Aortic stenosesQ; Malignant hypertensionQ

Valvular structural abnormalities

• Papillary muscle ruptureQ • Aortic & mitral regurgitationQ

ArrhythmiasQ

• Ventricular tachyarrhythmia Pathophysiology of Cardiogenic Shock

• Impaired pumping ability of left ventricle → ↓ Stroke volume & inadequate systolic emptying • ↓ Stroke volume → ↓ Cardiac output → ↓ BP → ↓ Tissue perfusionQ • Inadequate systolic emptying → ↑ Left ventricular filling pressure (↑ preload) → ↑ Left atrial pressure → ↑ Pulmonary artery & capillary pressure → Pulmonary edemaQ Clinical Features • Clinical features vary depending on the cause: −− Signs of myocardial failure: ↑ JVP, reduced pulse volume, hypotension, tachycardia, basal coarse cracklesQ −− Obstructive (cardiac tamponade): Muffled heart sounds, pulsus paradoxusQ −− Pulmonary embolism: Sudden onset dyspnea, tachypnea, tachycardia, hypotension, localized pleural rubQ, severe central chest pain in massive emboli −− Other signs: Cold clammy peripheries with pallor, peripheral & central cyanosisQ

Others

Systolic dysfunction

1116

Surgery Essence SEPTIC SHOCK Septic Shock • Septic shock refers to sepsis accompanied by hypotension that cannot be corrected by infusion of fluidsQ Septic Shock Definition

• A subset of sepsis in which underlying circulatory and cellular/metabolic abnormalities lead to substantially increased mortality riskQ

Criteria in 2016

• Suspected (or documented) infection plus vasopressor therapy needed to maintain mean arterial pressure at ≥65 mm HgQ & serum lactate >2.0 mmol/L despite adequate fluid resuscitationQ

• Refractory septic shock: Septic shock that lasts for >1 hour & does not respond to fluid or pressure administration Pathophysiology of Septic Shock

Others

• Initiated by gram-negative (MC) or gram-positive bacteria, fungi or virus • Cell wall of organisms contain endotoxins & exotoxins • Endotoxins → Release of inflammatory mediators → Vasodilatation & ↑ capillary permeability → Altered peripheral circulation & massive dilation → ShockQ • Infection → Local inflammatory reaction → Release of inflammatory mediators → Systemic inflammatory response → Diffuse endothelial injury, vasodilatation & ↑ capillary permeability → Progressive vasodilatation & maldistribution of blood flow → Organ hypoperfusion → Multiple organ dysfunction syndromeQ Clinical Features • Fever with chills & rigor, warm peripheries due to vasodilatationQ • Bounding pulse, rapid capillary refilling & hypotensionQ • Evidence of infection at local siteQ Treatment • First line of treatment: Aggressive volume expansion with crystalloids & restoration of arterial oxygenation with inspired oxygen & frequently with mechanical ventilation are the highest prioritiesQ. 
 • Second line: Ionotropic support with dopamine, norepinephrine, or vasopressin in presence of hypotension or dobutamine if arterial pressure is normalQ. 
 • High dose activated protein CQ (APC) provides a survival benefit in patients with severe sepsis & septic shock 
 • Plasma expanders are useful as septic shock is associated with peripheral vasodilatation causing reactive hypovolemiaQ 
 • Antibiotics & surgical debridement or drainage to control infectionQ


NEUROGENIC SHOCK Neurogenic Shock • Result from loss or suppression of sympathetic tone → Massive vasodilatation in venous vasculature →↓Venous return → ↓ Cardiac output → Impaired tissue perfusion & cellular metabolismQ

Section 10

Causes of Neurogenic Shock • High cervical spinal cord injuryQ • Inadvertent cephalad migration of spinal anesthesiaQ

• Deep general anesthesiaQ (depress vasomotor tone) • Devastating head injuryQ

Clinical Features • Paralysis below the level of lesion; HypotensionQ • Bradycardia due to loss of sympathetic toneQ → Arterial & venous vasodilatationQ → Warm & dry skinQ • HypothermiaQ Treatment • Treatment involves a simultaneous approach to relative hypovolemia & to loss of vasomotor tone. 
 • Excessive volumes of fluidQ may be required to restore normal hemodynamics if given alone. 
 • Once hemorrhage has been ruled out, norepinephrine or a pure alpha-adrenergic agentQ (phenylephrine) may be necessary 


Shock

1117

HYPOADRENAL SHOCK

• In the stress or illness, surgery or trauma, adrenal secretes increased amount of cortisolQ • Unrecognized adrenal insufficiency complicates the host response to stress induced by acute illness or major surgery → Hypoadrenal shockQ Causes of Hypoadrenal Shock • Chronic administration of high doses of exogenous glucocorticoidsQ • Adrenal insufficiency secondary to: Idiopathic atrophyQ; Use of etomidateQ for intubation; TuberculosisQ, Metastatic diseaseQ; Bilateral adrenal hemorrhageQ; AmyloidosisQ

Section 10

Hypoadrenal Shock

CHARACTERISTIC FEATURES OF VARIOUS TYPES OF SHOCK Physiologic Characteristics of the Various Forms of Shock Type of Shock

CVP and PCWP

Cardiac Output

SVR

Venous O2 Saturation

Hypovolemic





­↑



Cardiogenic

­↑



­↑



Hyperdynamic

↑↓

­↓

­↓

­↑

Hypodynamic

Septic ↑↓



­↑

↑↓

Traumatic



↑↓

↑↓



Neurogenic





­↓



Hypoadrenal

↑↓



=↓ ­



PARAMETERS FOR SHOCK Shock Index (SI)

Modified Shock Index (MSI) • MSI is defined as heart rate divided by mean arterial pressureQ. • High MSI indicates a value of stroke volume & low systemic vascular resistance, a sign of hypodynamic circulationQ. • Low MSI indicates a hyperdynamic stateQ. • MSI has been considered a better marker than SI for mortality rate predictionQ.

Pulse Rate Over Pressure Evaluation (ROPE) • ROPE = Pulse Rate/Pulse pressure = PR/(SBP–DBP) • ROPE is useful in the assessment of compensated hemorrhagic shock.

DRUG OF CHOICE IN SHOCK Drug of Choice in Shock Anaphylactic shock

• AdrenalineQ

Cardiogenic shock

• Noradrenaline or dopamineQ

Distributive shock

• Noradrenaline or phenylephrineQ

Hypovolemic shock

• CrystalloidsQ

Shock with oliguria

• DopamineQ

Hypoadrenal shock

• CorticosteroidsQ

Septic shock

• Broad-spectrum antibioticsQ

Others

• SI is defined as heart rate divided by systolic BP . • Better marker for assessing severity of shock than heart rate & BP aloneQ. • Utility in trauma patients, sepsis, obstetrics, myocardial infarction, stroke & other acute critical illnessesQ. • Correlated with need for interventions such as blood transfusion & invasive procedures including operations. • SI is known as a hemodynamic stability indicatorQ. • SI does not take into account the diastolic BP Q

Multiple Choice Questions SHOCK

12.

1.

Shock is clinically best assessed by: (Recent Question 2016) a. Urine output b. CVP c. BP d. Hydration

2.

Which of the following is the best parameter to assess fluid intake in a poly-trauma patient? (All India 94) a. Urine output b. BP c. Pulse d. Pulse oximetry Following is the most important factor in the management of shock: (AIIMS 84) a. Blood pressure b. Cardiac output c. CVP to 8 cm of water d. Deficiency of effective circulation Which of the following is true for shock? (MCI Sept 2005) a. Hypotension b. Hypoperfusion to tissues c. Hypoxia d. All of the above

3.

4.

5.

6.

7.

8.

9.

Best guide for the management of resuscitation is: a. CVP (AIIMS November 2017) b. Urine output c. Blood pressure d. Saturation of oxygen Modified shock index formula is: (AIIMS November 2017) a. Heart rate / Systolic BP b. Heart rate / Diastolic BP c. Heart rate/ Mean arterial pressure d. Pulse rate/ Systolic BP First line of therapy in shock in the patients of trauma:  (Recent Question 2017) a. Crystalloids b. Colloids c. 
Inotropes d. Blood transfusion Optimum urine output in post-operative patient:  (Recent Question 2017) a. 1 mL/min b. 2 mL/min c. 3 mL/min d. 4 mL/min A patient came with profuse diarrhea and dehydration reaches OPD. For examination flow of fluids which cannula can be inserted: (AIIMS November 2018) a. Green b. Blue c. Grey d. Violet

NEUROGENIC SHOCK (AIIMS May 2014)

10.

Neurogenic shock is characterized by: a. Hypertension and tachycardia b. Hypertension and bradycardia c. Hypotension and tachycardia d. Hypotension and bradycardia

11.

A patient with spine, chest and abdominal injury in road traffic accident developed hypotension and bradycardia. Most likely reason is:  (AIIMS Nov 2013) a. Hypovolemic shock b. Hypovolemic + neurogenic shock c. Hypovolemic + septicemic shock d. Neurogenic shock

Which of the following are true about neurogenic shock?  a. Tachycardia (Recent Question 2016) b. Cold and moist extremity c. Due to parasympathetic blockade d. Diagnosis of exclusion

HEMORRHAGIC SHOCK 13.

14.



Immediate management of a patient with multiple fracture and fluid loss includes the infusion:  (All India 94) a. Blood b. Dextran c. Normal saline d. Ringer lactate Hemorrhage leads to: (MCI Sept 2005) a. Septic shock b. Neurogenic shock c. Hypovolemic shock d. Cardiogenic shock

15.

In traumatic cases, shock is most likely due to: (Recent Question 2016, DNB 2011, MCI Sept 2007) a. Injury to intra abdominal solid organ b. Head injury c. Septicemia d. Cardiac failure

16.

Which of the following is ideal in moderate hemorrhagic shock? (Karnataka 2012, MCI Sept 2007) a. Dextrose b. Ringer lactate c. Blood d. Dextran Compensatory mechanism in a patient with hypovolemic shock: (JIPMER 2011) a. Increased renal blood flow b. Decrease in cortisol c. Decrease in vasopressin d. Decreased cutaneous blood flow Features of hypovolemic shock are all except: (NIMHANS 86) a. Oliguria b. Bradycardia c. Low BP d. Acidosis One of the following is earliest indication of concealed acute bleeding: (All India 95) a. Tachycardia b. Postural HT c. Oliguria d. Cold clammy fingers

17.

18. 19.

20. Blood loss in class II hemorrhagic shock is:  (Recent Questions 2013) a. < 15% b. 15-30% c. 30-40% d. >40% 21. Most common type of shock in emergency room is: (Recent Question 2013) a. Cardiogenic b. Hypovolemic shock c. Obstructive d. Neurogenic 22. Most common type of shock in surgical practice: (DNB 2014) a. Cardiogenic b. Hypovolemic c. Neurogenic d. Septic shock 23. Most common feature of polytrauma in pediatric age group is: (Recent Question 2015) a. Hypothermia b. Hypovolemic shock c. Hypotension d. Hypoxemia 24. Which of the following is false about hemorrhagic shock?  (Recent Question 2017) a. Low heart rate b. Cold extremity c. Due to fluid losses d. Vascular resistance high

Shock 25.

Class III hemorrhagic shock refers to: a. Blood loss less than 15% b. Blood loss between 15%–30% c. Blood loss between 30%–40% d. Blood loss more than 40%

(AIIMS May 2018)

SEPTIC SHOCK 27.

28.

29.

The most important cause of the death in septic shock is: a. DIC b. Respiratory failure c. Renal d. Cardiac 35-years old Mona developed feature of septicemia. Shock in form of hypotension and low urine output. She was being treated for colonic necrosis. What will be the management? a. IV fluids + dopamine  (AIIMS June 99) b. IV fluids only c. Only dopamine d. Antibiotic in high dose

Plasma expanders are used in:  (Recent Question 2013, DNB 2012) a. Septic shock b. Vasovagal shock c. Neurogenic shock d. Cardiogenic shock 30. All of the following are true about distributive shock except: a. Decreased venous return (Recent Question 2017) b. Decreased cardiac output c. Decreased vascular resistance d. High mixed venous saturation

MISCELLANEOUS 31.

Blood clot the size of a clenched fist is roughly equal to: (DNB 2010) a. 250 mL b. 350 mL c. 500 mL d. 600 mL

32.

What is normal pulmonary capillary wedge pressure?   (MHSSMCET 2005) a. 4–8 mm of Hg b. 8–12 mm of Hg c. 12–16 mm of Hg d. 15–25 mm of Hg

33.

Patient is on shock. IV cannulation not possible, intraosseous line for IVF should be done within: (WBPG 2014) a. 1 minute b. 1.5 minutes c. 2 minutes

Section 10

26.

In traumatic cases, shock is most likely due to:  (MCI June 2018) a. Injury to intra-abdominal solid organ b. Head injury c. Septicemia d. Cardiac failure

1119

d. 2.5 minutes 34.

Green coloured IV cannula, the size is: (Recent Question 2015) a. 18 Gauge b. 20 Gauge c. 22 Gauge d. 24 Gauge

35.

In a patient with dehydration, which the following color intravenous cannula will you place for rapid fluid resuscitation? (AIIMS May 2016) a. Grey c. Blue b. Pink d. Green

36.

22 Gauge IV cannula color is: (AIIMS November 2017) a. Green b. Grey c. Blue d. Pink

Others

Explanations SHOCK

1. Ans a. Urine output (Sabiston 20/e p554; Schwartz 10/e p109-131; Bailey 27/e p17)

Shock • Shock: Inadequate delivery of oxygen and nutrients due to poor tissue perfusionQ to maintain normal tissue and cellular function • Mean arterial pressure Depression (9%) >Dementia (3%) >Functional psychosis (2%).

39. Ans. d. End expiratory carbon dioxide content (Ref: Schwartz 9/e p787-789)



40. Ans. b. Aneurysm of arch of aorta • Fiberoptic endoscopy is contraindicated in aneurysm of arch of aorta.

Others



1130

Surgery Essence

41. Ans. d. Acromegaly (Ref: Harrison 20/e p1667, 2578, 19/e p1126)



Leprosy, Syphilis and Thalassemia causes depressed bridge of nose. 42. Ans. a. Sequestration crisis (Ref: Harrison 20/e p693, 19/e p634)



43. Ans. c. Sir William Osler • Sir William Osler: “To study the phenomenon of disease without books is to sail an uncharted sea, while to study books without patients is not to go to sea at all”Q



44. Ans. b Noise (Thrill/bruit)







45. Ans. c. Dense cortical bone



46. Ans. c. Vasomotor rhinitis (Ref: www.ncbi.nlm.nih.gov/pubmed/16686388) • Vidian neurectomy is indicated in the cases of vasomotor rhinitis with profuse secretion refractory to conservative treatmentQ.



47. Ans. a. Carbon monoxide poisoning (Ref: www.biomedsearch.com/searchlist.html?p=3101...txt=oxygen...)

Others

• Severe carbon monoxide poisoning treated by hyperbaric oxygen therapyQ.

48. Ans. b. Tendinitis of common extensor origin (Ref: Bailey 27/e p448)

Tennis Elbow (Lateral Epicondylitis) • This is the most common cause of elbow pain excluding traumatic conditions • Usually occurs in patients of 30-50 years; Etiology is unknownQ in most of cases • Strenuous or overactivity may precede symptoms • Anterodistal lateral epicondyle tendernessQ Diagnosis • Resisted wrist extension is a reliable diagnostic testQ • Local anesthetic injection is diagnostically helpful Treatment • Vast majority improve with supervised conservative managementQ • Open or arthroscopic release yields good results in recalcitrant cases • Arthroscopic release also identifies associated pathology

49. Ans. d. Brunschwig operation. (Ref: Medifactsonline blogspot.com/2011/.../1000-eponyms-in-surgery)

Brunschwig Operation • Pelvic exentration : Surgery to remove the lower colon, rectum and bladder, and create permanent stoma. Q



50. Ans. a. Always associated with breech extraction (Ref: Bailey 27/e p582)

Torticollis • In torticollis the head is tilted toward and rotated away from the tight sternocleidomastoid muscle. • Congenital torticollis is usually secondary to intrauterine moulding but may present with fixed sternocleidomastoid contracture or with a palpable mass in the muscle. • Most cases resolve with stretching but, occasionally, surgical release of the sternocleidomastoid at one or both ends is needed.

51. Ans. d. Methylene blue

Section 10

Chromoendoscopy • Chromoendoscopy: Dyes are instilled into the GIT at the time of visualization with fibre-optic endoscopyQ. • Chiefly enhance the characterization of tissuesQ • Detail achieved can often allow for identification of the tissue type or pathologyQ Stains used • Absorptive stains have an affinity for particular mucosal elements, and include Lugol’s iodine, methylene blue and gentian violetQ. Lugol’s iodine

• Specifically stains non-keratinized squamous epitheliumQ • Useful for identifying squamous tissue, squamous dysplasia & squamous cell carcinomasQ.

Methylene blue

• Stains absorptive epitheliumQ • Useful for identifying abnormality in small intestine, colon & Barrett’s esophagusQ (intestinal metaplasia)

Miscellaneous

• Chief use of Indigo carmine: Identification of dysplastic cells in individuals with chronic UCQ. • Reactive stains undergo an observable change due to a chemical process related to the function of the gastrointestinal tract. Congo red is used as a test for achlorhydria in the stomachQ, as it changes colour from red to black at a pH less than 3. Uses of Chromoendoscopy • • • • •

Identification of squamous cell carcinomas or dysplasia of the esophagusQ Identification of Barrett’s esophagus & dysplasiaQ identification of early gastric cancerQ Characterization of colonic polyps & colorectal cancerQ In screening for dysplasia in individuals with ulcerative colitisQ.

Section 10

• Contrast stains are not absorbed but rather provide contrast by permeating between irregularities in the mucosa to highlight irregularities. The primary contrast stain is indigo carmineQ.

1131

52. Ans. c. T2 N0 M0 glottic cancer (Ref: Harrison 20/e p535, 19/e p503, 18/e p735) • Concomitant chemoradiotherapy is indicated in advanced cancers of head and neck. Q • T2 N0 M0 glottic cancer represents stage II cancer which is defined as localized disease, which is not an indication for concomitant chemoradiotherapy.



53. Ans. None (Ref: Harrison 20/e p3293, 19/e p2730) Smoking may be associated with all of the above cancers. Smoking Associated Cancers • LungQ • Nasopharynx, oropharynx hypopharynx and LarynxQ • Nasal cavity and paranasal sinusesQ

• • • • •

Oral cavityQ EsophagusQ StomachQ PancreasQ LiverQ

• • • •

KidneyQ Ureter and Urinary BladderQ Uterine CervixQ Acute Myeloid Leukemia

• Smoking is not associated with postmenopausal Breast cancer and endometrial cancer.

54. Ans. b. Neuroblastoma

Neuroblastoma



55. Ans. d. Trousseau’s sign



56. Ans. b. Ranula (Ref: Bailey 27/e p779)



57. Ans. a. Enzyme, b. Intraoperative histopathological examination, c. Fat, e. To check surgical margin in tumor surgery (Ref: Bailey 25/e p169-170)

Frozen Section Biopsy • Biopsy technique in pathology laboratories for making urgent on-table diagnosisQ • Frozen section biopsy is a procedure done in a pathology setup existing adjacent to the operation theatreQ. • Surgeons are the main users of this service. Procedure • An unfixed fresh tissue is frozen (using CO2 to -25°C) and section are made and stained. Uses of Frozen Section Biopsy • It is quick and surgeon can decide the further steps of procedure in the same sitting like nodal clearance/type of resection to be done. • During surgery after resection of the tumor to look for (on table) the clearance in the margin and depth, also to study the lymph nodes for their positivityQ. • Used for demonstration of certain constituents which are lost in processing with alcohol or xylene, e.g., fat, enzymesQ.

Others

• Hutchinson and Pepper syndrome is skull metastasis seen in neuroblastomaQ.

1132

Surgery Essence

58. Ans. a. Pancreas cancer, c. Acute pancreatitis, d. Pancreatic divisum (Ref: Harrison 20/e p352, 19/e p366)

Panniculitis

Others

• Inflammatory lesions of the subcutaneous fatQ • Divided into the distinct categories: −− Septal panniculitis: Inflammation is confined the interlobular septa of the subcutis −− Lobular panniculitis: Inflammation involves the entire fat lobule and often the septa as well −− Panniculitis secondary to vasculitis: Involve large vessels in the subcutis, in which the inflammation is usually restricted to the immediate vicinity of the involved vessel. • Pancreatic panniculitis: −− Manifests as painful or asymptomatic subcutaneous nodules or indurated plaques on the thighs, buttocks, lower trunk or distal extremities usually the lower. −− Lesions are associated with acute pancreatitisQ or less commonly, pancreatic carcinomaQ either of which may be asymptomatic −− It has also been associated with low grade pancreatitis in a patient with pancreas divisumQ.

59. Ans. a. Modified sweat gland, c. Present in axilla and groin, d. Hidradenitis suppurativa is infection of apocrine gland (Ref: Bailey 27/e p593)

Apocrine Glands • An apocrine sweat gland is a modified sweat gland • In humans, apocrine sweat glands are found only in certain locations of the body: Axilla, areola and nipples of the breast, perianal region, and some parts of the external genitalia. • Hidradenitis suppurative is chronic suppurative condition of apocrine glands bearing skin.

60. Ans. d. Carcinoma breast (Ref: Breast Pathology By Frances P O Malley (2006)/76-78)

Causes of Fat Necrosis • • • •



After surgery/surgical trauma: Wide local excision, Reduction mammoplastyQ Following radiotherapy including iridium implantsQ Following traumaQ Autologous fat injectionQ using the liposuction technique to fill in irregular contours and small soft tissue defects in the breast may lead to fat necrosis secondary to poor blood supply in the injected fat.

61. Ans. a. Formaldehyde (Ref: Surgical Pathology by Rosai and Ackermann 9/e p27)

Formalin • Formaldehyde as a buffered 10% aqueous solution (formalinQ) is the fixative most commonly used in histologyQ • In routine clinical diagnostics it offers the best possible compromise between a simple and a reliable method as well as extremely good structural preservationQ. • The strong cross-linking action of formaldehyde is essential, to protect the tissue from the aggressive effect of concentrated solvents in the course of fixation and embedding in paraffinQ. • Fixation of tissue arrests the autolysis and putrefaction and stabilizes the cellular and tissue contentsQ

Section 10



62. Ans. a. Aspergillus flavus (Ref: Ananthnarayan 7/e p625) • Primary Aflatoxin Producing Fungi: Aspergillus flavusQ and Aspergillus parasiticusQ 63. Ans. d. Floor (Ref: BDC 4/e pI/58)

Axillary Abscess • An axillary abscess is incised through the floorQ of the axilla, midway between the anterior and posterior axillary folds, and nearer to the medial wall in order to avoid injury to the main vessels running along the anterior, posterior and lateral walls.

64. Ans. b. Tracheal stenosis (Ref: Dhillon 3/e p67) • Topical Mitomycin C is the drug of choice used to aid the treatment of laryngeal stenosisQ. • Topical Mitomycin C can inhibit fibroblast activity and restenosisQ.



65. Ans. a. 10% buffered neutral formalin

Miscellaneous

66. Ans. a. Sarcoidosis (Ref: Essentials of Chest Radiology by Janette Collins/165)

• Sarcoidosis is a systemic disease characterized by non-caseating granulomas in multiple organs • In 90% of cases, symmetrical massive bilateral hilar lymphadenopathy occur • The cardiac border (Potato nodes) or lung involvement is present and can be revealed by chest X-ray or transbronchial biopsy



67. Ans. a. Carcinoma • “In normal persons, a click is felt when larynx is moved from side to side over vertebral column, this is called laryngeal click (post cricoid crepitus) It is absent in post cricoid carcinoma”— Moure’s sign.



68. Ans. b. Phenacetin - lung



69. Ans. c. Diameter of catheter (Ref: The ICU Book By Paul L. Marino 3/e p108)



Section 10

Sarcoidosis



1133

“The size of vascular catheters is expressed in terms of outside diameter of the catheter. Two units of measurements are used to describe catheter size: a metric-based French size and a wire-based gauge size. The French size is a series of whole numbers that increases from zero in increments of 0.33 millimeters (e.g., a size 5 French catheter will have an outside diameter of 5 × 0.33 = 1.65 mm). The gauge size was introduced for solid wires and is an expression of how many wires can be placed side-by-side in a given space. The gauge size varies inversely with the diameter of the wire (or catheter).”- The ICU Book By Paul L. Marino 3/e p108 70. Ans. b. Doctor’s signature (Ref: Sabiston 20/e p232; Schwartz 10/e p1969) Elements of the Surgical Safety Checklist Sign In



Time-Out 
 • Before skin incision, the entire team (nurses, surgeons, anesthesia professionals, and any others participating in the care of the patient) or specific members state aloud the following: 
 • Team confirms that all team members have been introduced by name 
& role. • Team confirms the patient’s identity, surgical site & procedure. 
 • Team reviews the anticipated critical events. • Surgeon reviews critical and unexpected steps, operative duration & anticipated blood loss. 
 • Anesthesia professionals review concerns specific to patient. 
 • Nurses review confirmation of sterility, equipment availability, and other 
concerns. • Team confirms that prophylactic antibiotics have been administered ≤60 
minutes before incision is made or that antibiotics are not indicated. 
 • Team confirms that all essential imaging results for correct patient are 
displayed in operating room. 


Sign Out 
 • Before the patient leaves the operating room, the following are done: 
 • Nurse reviews the following aloud with the team: • Name of procedure, as recorded 
 • That needle, sponge, & instrument counts are complete (or not 
applicable) 
 • That specimen (if any) is correctly labeled, including patient’s name • Whether there are any issues with equipment that need to be addressed 
 • The surgeon, nurse & anesthesia professional review aloud the key concerns for the recovery and care of the patient. 


71. Ans. a. Tip of nose to ear to xiphisternum (Ref: Practical Medical Procedures at a Glance By Rachel K. Thomas (2015)/p70) The commonly used method to measure nasogastric tube length is the NEX method. NEX: Nose–Ear–Xiphisternum.

“To measure the required length of tube, measure from the tip of the patient›s nose, to their ear, and then down to the xiphisternum.” -Practical Medical Procedures at a Glance By Rachel K. Thomas (2015)/p70

72. Ans. d. Relaxed tension lines in skin (Ref: Grabb and Smith’s Plastic Surgery 7/e p1-3; Gray’s 41/e p157)

“Kraissl’s lines are essentially exaggerated wrinkle lines obtained by studying the loose skin of elderly faces whilst contracting the muscles of facial expression. These lines for the most part correspond to Relaxed skin tension lines (RSTLs), but slight variation exists on the face, especially on the lateral side of the nose, the lateral aspect of the orbit, and the chin.”-Gray’s 41/e p157

73. Ans. d. Skin, subcutaneous tissue, superficial fascia and deep fascia (Ref: Bailey 27/e p28)

“Fasciotomy involves incising the deep muscle fascia and is best carried out via longitudinal incisions of skin, fat and fascia. The muscle will be then seen bulging out through the fasciotomy opening.”-Bailey 27/e p28

Others

• Before induction of anesthesia, members of the team (at least the nurse and an anesthesia professional) state that the following have been done: • Patient has verified his or her identity, surgical site & procedure and 
consent. 
 • The surgical site is marked or site marking is not applicable. • Pulse oximeter is on the patient & functioning. 
 • All members of the team are aware of whether the patient has a known 
allergy. 
 • Patient’s airway & risk of aspiration have been evaluated, and appropriate equipment & assistance are available. 
 • If there is a risk of blood loss of at least 500 mL (or 7 mL/kg body weight 
in children), appropriate access and fluids are available. 


Section 10

Others

1134

Surgery Essence

74. Ans. c. 1-Kocher incision; 2-Midline incision; 3-McBurney incision; 4-Battle incision; 5-Lanz incision; 6-Paramedian incision; 7-Transverse incision; 8-Rutherford Morrison incision; 9-Pfannenstiel incision



75. Ans. b. This test helps to detect ascites



76. Ans. d. Sim’s position



77. Ans. a. A (Ref: Bailey 25/e p339-340, Oxford Textbook of Vascular Surgery/ p202)



78. Ans. a. 0.33 mm, d. 0.013 inch



79. Ans. a. Prophylaxis against thromboembolism